You are on page 1of 624

rankershalt.

com

41 Years’
CHAPTERWISE TOPICWISE
SOLVED PAPERS
2019-1979

IITJEE
(JEE Main & Advanced)

Mathematics

Amit M Agarwal

Arihant Prakashan (Series), Meerut


rankershalt.com

Arihant Prakashan (Series), Meerut


All Rights Reserved

© Author
No part of this publication may be re-produced, stored in a retrieval system or distributed
in any form or by any means, electronic, mechanical, photocopying, recording, scanning,
web or otherwise without the written permission of the publisher. Arihant has obtained
all the information in this book from the sources believed to be reliable and true. However,
Arihant or its editors or authors or illustrators don’t take any responsibility for the absolute
accuracy of any information published and the damages or loss suffered there upon.

All disputes subject to Meerut (UP) jurisdiction only.

Administrative & Production Offices


Regd. Office
‘Ramchhaya’ 4577/15, Agarwal Road, Darya Ganj, New Delhi -110002
Tele: 011- 47630600, 43518550; Fax: 011- 23280316
Head Office
Kalindi, TP Nagar, Meerut (UP) - 250002
Tele: 0121-2401479, 2512970, 4004199; Fax: 0121-2401648

Sales & Support Offices


Agra, Ahmedabad, Bengaluru, Bareilly, Chennai, Delhi, Guwahati,
Hyderabad, Jaipur, Jhansi, Kolkata, Lucknow, Meerut, Nagpur & Pune

ISBN : 978-93-13196-97-6

For further information about the books published by Arihant


log on to www.arihantbooks.com or email to info@arihantbooks.com

/arihantpub /@arihantpub Arihant Publications /arihantpub


rankershalt.com

CONTENTS

1. Complex Numbers 1-26

2. Theory of Equations 27-47

3. Sequences and Series 48-71

4. Permutations and Combinations 72-83

5. Binomial Theorem 84-96

6. Probability 97-126

7. Matrices and Determinants 127-159

8. Functions 160-175

9. Limit, Continuity and Differentiability 176-224

10. Application of Derivatives 225-262

11. Indefinite Integration 263-277

12. Definite Integration 278-310

13. Area 311-334

14. Differential Equations 335-357

15. Straight Line and Pair of Straight Lines 358-382


rankershalt.com

16. Circle 383-416

17. Parabola 417-434

18. Ellipse 435-449

19. Hyperbola 450-460

20. Trigonometrical Ratios and Identities 461-473

21. Trigonometrical Equations 474-487

22. Inverse Circular Functions 488-497

23. Properties of Triangles 498-518

24. Vectors 519-548

25. 3D Geometry 549-572

26. Miscellaneous 573-598

Ÿ JEE Advanced Solved Paper 2019 1-18


rankershalt.com

SYLLABUS
JEE MAIN
UNIT I Sets, Relations and Functions selection, Meaning of P(n,r) and C (n,r), simple
Sets and their representation, Union, applications.
intersection and complement of sets and their
UNIT V Mathematical Induction
algebraic properties, Power set, Relation, Types
Principle of Mathematical Induction and its
of relations, equivalence relations, functions,
simple applications.
one-one, into and onto functions, composition
of functions. UNIT VI Binomial Theorem and its
UNIT II Complex Numbers and Simple Applications
Binomial theorem for a positive integral index,
Quadratic Equations
general term and middle term, properties of
Complex numbers as ordered pairs of reals,
Binomial coefficients and simple applications.
Representation of complex numbers in the form
a+ib and their representation in a plane, Argand UNIT VII Sequences and Series
diagram, algebra of complex numbers, modulus Arithmetic and Geometric progressions,
and argument (or amplitude) of a complex insertion of arithmetic, geometric means
number, square root of a complex number, between two given numbers. Relation between
triangle inequality, Quadratic equations in real AM and GM Sum upto n terms of special series:
and complex number system and their ∑ n, ∑ n2, ∑n3. Arithmetico - Geometric
solutions. Relation between roots and co- progression.
efficients, nature of roots, formation of quadratic
equations with given roots. UNIT VIII Limit, Continuity and
UNIT III Matrices and Determinants Differentiability
Real valued functions, algebra of functions,
Matrices, algebra of matrices, types of matrices,
polynomials, rational, trigonometric, logarithmic
determinants and matrices of order two and
and exponential functions, inverse functions.
three. Properties of determinants, evaluation of
Graphs of simple functions. Limits, continuity
deter-minants, area of triangles using
and differenti-ability. Differentiation of the sum,
determinants. Adjoint and evaluation of inverse
difference, product and quotient of two
of a square matrix using determinants and
functions. Differentiation of trigonometric,
elementary transformations, Test of
inverse trigonometric, logarithmic,
consistency and solution of simultaneous linear
exponential, composite and implicit functions;
equations in two or three variables using
derivatives of order upto two. Rolle's and
determinants and matrices.
Lagrange's Mean Value Theorems. Applications
UNIT IV Permutations and of derivatives: Rate of change of quantities,
Combinations monotonic - increasing and decreasing
Fundamental principle of counting, permutation functions, Maxima and minima of functions of
as an arrangement and combination as one variable, tangents and normals.
rankershalt.com

UNIT IX Integral Calculus form of the equation of a circle, its radius and
Integral as an anti - derivative. Fundamental centre, equation of a circle when the end points
integrals involving algebraic, trigonometric, of a diameter are given, points of intersection of
exponential and logarithmic functions. a line and a circle with the centre at the origin
Integration by substitution, by parts and by and condition for a line to be tangent to a circle,
partial fractions. Integration using equation of the tangent. Sections of cones,
trigonometric identities. Evaluation of simple equations of conic sections (parabola, ellipse
integrals of the type and hyperbola) in standard forms, condition for
y=mx + c to be a tangent and point (s) of
dx , dx , dx , dx , tangency.
x2 ± a2 x2 ± a2 a2 – x2 a2 – x2
dx , dx , (px + q) dx , UNIT XII Three Dimensional Geometry
2
ax + bx + c 2
ax + bx + c 2
ax + bx + c Coordinates of a point in space, distance
between two points, section formula, direction
(px + q) dx ratios and direction cosines, angle between two
,
a 2 ± x 2 dx and x 2 – a 2 dx
ax 2 + bx + c intersecting lines. Skew lines, the shortest
distance between them and its equation.
Integral as limit of a sum. Fundamental Theorem Equations of a line and a plane in different
of Calculus. Properties of definite integrals. forms, intersection of a line and a plane,
Evaluation of definite integrals, determining coplanar lines.
areas of the regions bounded by simple curves
in standard form. UNIT XIII Vector Algebra
Vectors and scalars, addition of vectors,
UNIT X Differential Equations components of a vector in two dimensions and
Ordinary differential equations, their order and three dimensional space, scalar and vector
degree. Formation of differential equations. products, scalar and vector triple product.
Solution of differential equations by the method
of separation of variables, solution of UNIT XIV Statistics and Probability
homogeneous and linear differential equations Measures of Dispersion: Calculation of mean,
of the type dy + p(x)y = q(x) median, mode of grouped and ungrouped data.
dx
Calculation of standard deviation, variance and
UNIT XI Coordinate Geometry mean deviation for grouped and ungrouped
Cartesian system of rectangular coordinates in a data.
plane, distance formula, section formula, locus Probability: Probability of an event, addition
and its equation, translation of axes, slope of a and multiplication theorems of probability,
line, parallel and perpendicular lines, intercepts Baye's theorem, probability distribution of a
of a line on the coordinate axes. random variate, Bernoulli trials and Binomial
Straight lines distribution.
Various forms of equations of a line, intersection
of lines, angles between two lines, conditions UNIT XV Trigonometry
for concurrence of three lines, distance of a Trigonometrical identities and equations.
point from a line, equations of internal and Trigonometrical functions. Inverse
external bisectors of angles between two trigonometrical functions and their
lines, coordinates of centroid, orthocentre and properties. Heights and Distances.
circumcentre of a triangle, equation of family of
UNIT XVI Mathematical Reasoning
lines passing through the point of intersection
Statements, logical operations And, or, implies,
of two lines.
implied by, if and only if. Understanding of
Circles, Conic sections tautology, contradiction, converse and contra
Standard form of equation of a circle, general positive.
rankershalt.com

JEE ADVANCED
Algebra
Algebra of complex numbers, addition, multiplication, conjugation, polar representation, properties
of modulus and principal argument, triangle inequality, cube roots of unity, geometric
interpretations.
Quadratic equations with real coefficients, relations between roots and coefficients, formation of
quadratic equations with given roots, symmetric functions of roots.
Arithmetic, geometric and harmonic progressions, arithmetic, geometric and harmonic means, sums
of finite arithmetic and geometric progressions, infinite geometric series, sums of squares and cubes
of the first n natural numbers.

Logarithms and their Properties


Permutations and combinations, Binomial theorem for a positive integral index, properties of
binomial coefficients.
Matrices as a rectangular array of real numbers, equality of matrices, addition, multiplication by a
scalar and product of matrices, transpose of a matrix, determinant of a square matrix of order up to
three, inverse of a square matrix of order up to three, properties of these matrix operations,
diagonal, symmetric and skew-symmetric matrices and their properties, solutions of simultaneous
linear equations in two or three variables.
Addition and multiplication rules of probability, conditional probability, independence of events,
computation of probability of events using permutations and combinations.

Trigonometry
Trigonometric functions, their periodicity and graphs, addition and subtraction formulae, formulae
involving multiple and sub-multiple angles, general solution of trigonometric equations.
Relations between sides and angles of a triangle, sine rule, cosine rule, half-angle formula and the
area of a triangle, inverse trigonometric functions (principal value only).

Analytical Geometry
Two Dimensions Cartesian oordinates, distance between two points, section formulae, shift of
origin.
Equation of a straight line in various forms, angle between two lines, distance of a point from a line.
Lines through the point of intersection of two given lines, equation of the bisector of the angle
between two lines, concurrency of lines, centroid, orthocentre, incentre and circumcentre of a
triangle.
rankershalt.com

Equation of a circle in various forms, equations of tangent, normal and chord.


Parametric equations of a circle, intersection of a circle with a straight line or a circle, equation of
a circle through the points of intersection of two circles and those of a circle and a straight line.
Equations of a parabola, ellipse and hyperbola in standard form, their foci, directrices and
eccentricity, parametric equations, equations of tangent and normal.

Locus Problems
Three Dimensions Direction cosines and direction ratios, equation of a straight line in space,
equation of a plane, distance of a point from a plane.

Differential Calculus
Real valued functions of a real variable, into, onto and one-to-one functions, sum, difference,
product and quotient of two functions, composite functions, absolute value, polynomial,
rational, trigonometric, exponential and logarithmic functions.
Limit and continuity of a function, limit and continuity of the sum, difference, product and
quotient of two functions, l'Hospital rule of evaluation of limits of functions.
Even and odd functions, inverse of a function, continuity of composite functions, intermediate
value property of continuous functions.
Derivative of a function, derivative of the sum, difference, product and quotient of two functions,
chain rule, derivatives of polynomial, rational, trigonometric, inverse trigonometric, exponential
and logarithmic functions.
Derivatives of implicit functions, derivatives up to order two, geometrical interpretation of the
derivative, tangents and normals, increasing and decreasing functions, maximum and minimum
values of a function, applications of Rolle's Theorem and Lagrange's Mean Value Theorem.

Integral Calculus
Integration as the inverse process of differentiation, indefinite integrals of standard functions,
definite integrals and their properties, application of the Fundamental Theorem of Integral
Calculus.
Integration by parts, integration by the methods of substitution and partial fractions, application
of definite integrals to the determination of areas involving simple curves.
Formation of ordinary differential equations, solution of homogeneous differential equations,
variables separable method, linear first order differential equations.

Vectors
Addition of vectors, scalar multiplication, scalar products, dot and cross products, scalar triple
products and their geometrical interpretations.
rankershalt.com

1
Complex Numbers
Topic 1 Complex Number in Iota Form
Objective Questions I (Only one correct option) 2 + 3i sin θ
6. A value of θ for which is purely imaginary, is
2z − n 1 − 2i sin θ (2016 Main)
1 Let z ∈ C with Im (z ) = 10 and it satisfies = 2i − 1
 3
2z + n π π −1  1 
(a) (b) (c) sin −1   (d) sin  
for some natural number n, then (2019 Main, 12 April II) 3 6  4   3
(a) n = 20 and Re(z ) = − 10 (b) n = 40 and Re(z ) = 10 6 i –3 i 1
(c) n = 40 and Re(z ) = − 10 (d) n = 20 and Re(z ) = 10 7. If 4 3i –1 = x + iy, then (1998, 2M)
α + i  20 3 i
2 All the points in the set S =  : α ∈ R (i = −1 ) lie
 α − i  (a) x = 3, y = 1 (b) x = 1, y = 1 (c) x = 0, y = 3 (d) x = 0, y = 0
on a (2019 Main, 9 April I) 13
(a) circle whose radius is 2. 8. The value of sum ∑ (i n + i n + 1 ), where i = −1, equals
(b) straight line whose slope is −1. n =1
(1998, 2M)
(c) circle whose radius is 1. (a) i (b) i − 1 (c) − i (d) 0
n
(d) straight line whose slope is 1.  1 + i
5 + 3z 9. The smallest positive integer n for which   = 1, is
3 Let z ∈ C be such that|z|< 1. If ω = , then 1 − i
5(1 − z ) (a) 8 (b) 16 (1980, 2M)
(2019 Main, 9 April II) (c) 12 (d) None of these
(a) 4 Im(ω) > 5 (b) 5 Re (ω) > 1
(c) 5 Im (ω) < 1 (d) 5 Re(ω) > 4 Objective Question II
3
x + iy (One or more than one correct option)
4 Let  −2 − i =
1
(i = −1 ), where x and y are real
 3  27 10. Let a , b, x and y be real numbers such that a − b = 1 and
numbers, then y − x equals (2019 Main, 11 Jan I) y ≠ 0. If the complex number z = x + iy satisfies
(a) 91 (b) 85 (c) – 85 (d) – 91  az + b
Im   = y, then which of the following is(are)
 π  3 + 2i sin θ   z+1
5. Let A = θ ∈  − , π : is purely imaginary 
  2  1 − 2i sin θ  possible value(s) of x? (2017 Adv.)

Then, the sum of the elements in A is (2019 Main, 9 Jan I) (a) 1 − 1 + y 2


(b) − 1 − 1 − y 2

3π 5π 2π
(a) (b) (c) π (d) (c) 1 + 1 + y 2
(d) − 1 + 1 − y2
4 6 3

Topic 2 Conjugate and Modulus of a Complex Number


Objective Questions I (Only one correct option) (1 + i )2 2
2 If a > 0 and z = , has magnitude , then z is
1 The equation|z − i| = |z − 1|, i = −1, represents a−i 5
equal to (2019 Main, 10 April I)
1 (2019 Main, 12 April I)
(a) a circle of radius 1 3 1 3
2 (a) − i (b) − − i
5 5 5 5
(b) the line passing through the origin with slope 1 1 3 3 1
(c) a circle of radius 1 (c) − + i (d) − − i
5 5 5 5
(d) the line passing through the origin with slope − 1
rankershalt.com

2 Complex Numbers

3 Let z1 and z2 be two complex numbers satisfying| z1 | = 9 12. If w = α + iβ, where β ≠ 0 and z ≠ 1, satisfies the
and | z2 − 3 − 4i | = 4. Then, the minimum value of  w − wz 
condition that   is purely real, then the set of
| z1 − z2|is (2019 Main, 12 Jan II)  1−z 
(a) 1 (b) 2 (c) 2 (d) 0 values of z is (2006, 3M)
z −α (a)| z | = 1, z ≠ 2 (b)| z | = 1 and z ≠ 1
4 If (α ∈ R) is a purely imaginary number and
z+α (c) z = z (d) None of these
|z| = 2, then a value of α is (2019 Main, 12 Jan I) z −1
13. If|z| = 1 and w = (where, z ≠ − 1), then Re (w) is
(a) 2 (b)
1
(c) 1 (d) 2 z+1 (2003, 1M)
2 1  1  1 2
(a) 0 (b) (c) ⋅ (d)
5 Let z be a complex number such that | z | + z = 3 + i |z + 1|2 z + 
1 |z + 1
| 2
|z + 1|2
(where i = − 1).
14. For all complex numbers z1 , z2 satisfying |z1| = 12 and
Then,| z |is equal to (2019 Main, 11 Jan II)
|z2 − 3 − 4i| = 5, the minimum value of|z1 − z2|is
34 5 41 5
(a) (b) (c) (d) (a) 0 (b) 2 (2002, 1M)
3 3 4 4
(c) 7 (d) 17
6. A complex number z is said to be unimodular, if z ≠ 1. 15. If z1 , z2 and z3 are complex numbers such that
z1 – 2z2
If z1 and z2 are complex numbers such that is 1 1 1
2 – z1z2 |z1| = |z2| = |z3| =  + + = 1, then |z1 + z2 + z3|is
1
z z 2 z3
unimodular and z2 is not unimodular.
(a) equal to 1 (b) less than 1 (2000, 2M)
Then, the point z 1 lies on a (2015 Main)
(c) greater than 3 (d) equal to 3
(a) straight line parallel to X-axis
16. For positive integers n1 , n2 the value of expression
(b) straight line parallel toY -axis
(1 + i )n 1 + (1 + i3 )n1 + (1 + i5 )n 2 + (1 + i7 )n 2 , here
(c) circle of radius 2
i = −1 is a real number, if and only if (1996, 2M)
(d) circle of radius 2
(a) n1 = n2 + 1 (b) n1 = n2 − 1
7. If z is a complex number such that |z| ≥ 2, then the (c) n1 = n2 (d) n1 > 0, n2 > 0
1 17. The sin x + i cos 2x
minimum value of z + complex numbers and
2 (2014 Main) cos x − i sin 2x are conjugate to each other, for
(a) is equal to 5/2 (a) x = nπ (b) x = 0 (1988, 2M)
(b) lies in the interval (1, 2) (c) x = (n + 1/2) π (d) no value of x
(c) is strictly greater than 5/2 18. The points z1 , z2, z3 and z4 in the complex plane are the
(d) is strictly greater than 3/2 but less than 5/2 vertices of a parallelogram taken in order, if and only if
8. Let complex numbers α and 1 /α lies on circles (a) z1 + z4 = z2 + z3 (b) z1 + z3 = z2 + z4 (1983, 1M)
(x − x0 )2 + ( y − y0 )2 = r 2 and (x − x0 )2 + ( y − y0 )2 = 4r 2, (c) z1 + z2 = z3 + z4 (d) None of these
respectively. 19. If z = x + iy and w = (1 − iz ) / (z − i ), then |w| = 1 implies
If z0 = x0 + iy0 satisfies the equation 2|z0|2 = r 2 + 2, then
that, in the complex plane (1983, 1M)
|α |is equal to (2013 Adv.)
1 1 1 1 (a) z lies on the imaginary axis (b) z lies on the real axis
(a) (b) (c) (d) (c) z lies on the unit circle (d) None of these
2 2 7 3
9. Let z be a complex number such that the imaginary part 20. The inequality |z − 4| < |z − 2| represents the region
of z is non-zero and a = z + z + 1 is real. Then, a cannot
2 given by (1982, 2M)
take the value (2012) (a) Re (z ) ≥ 0 (b) Re (z ) < 0
1 1 3 (c) Re (z ) > 0 (d) None of these
(a) − 1 (b) (c) (d)
5 5
3 2 4  3 i  3 i
10. Let z = x + iy be a complex number where, x and y are 21. If z =  +  + −  , then
 2 2  2 2 (1982, 2M)
integers. Then, the area of the rectangle whose vertices
are the root of the equation zz3 + zz3 = 350, is (2009)
(a) Re (z ) = 0 (b) Im (z ) = 0
(c) Re (z ) > 0, Im (z ) > 0 (d) Re (z ) > 0, Im (z ) < 0
(a) 48 (b) 32 (c) 40 (d) 80
z 22. The complex numbers z = x + iy which satisfy the
11. If|z|= 1 and z ≠ ± 1, then all the values of lie on z − 5i 
1 − z2 equation  = 1, lie on
(a) a line not passing through the origin (2007, 3M) z + 5i  (1981, 2M)
(b)|z|= 2 (a) the X-axis
(c) the X-axis (b) the straight line y = 5
(d) the Y-axis (c) a circle passing through the origin
(d) None of the above
rankershalt.com

Complex Numbers 3

Objective Questions II Passage II


(One or more than one correct option) Let S = S1 ∩ S 2 ∩ S3 , where
 z − 1 + 3 i  
23. Let s, t, r be non-zero complex numbers and L be the set of S1 = { z ∈ C :|z | < 4}, S 2 = z ∈ C : lm   > 0
solutions z = x + iy (x, y ∈ R, i = − 1 ) of the equation   1− 3i  
sz + tz + r = 0, where z = x − iy. Then, which of the and S3 : { z ∈ C : Re z > 0} (2008)
following statement(s) is (are) TRUE? (2018 Adv.)
29. Let z be any point in A ∩ B ∩ C.
(a) If L has exactly one element, then| s|≠ |t |
The|z + 1 − i|2 + |z − 5 − i|2 lies between
(b) If|s|=|t |, then L has infinitely many elements
(a) 25 and 29 (b) 30 and 34
(c) The number of elements in L ∩ {z :| z − 1 + i| = 5} is at most (c) 35 and 39 (d) 40 and 44
2
30. The number of elements in the set A ∩ B ∩ C is
(d) If L has more than one element, then L has infinitely many
elements (a) 0 (b) 1
(c) 2 (d) ∞
24. Let z1 and z2 be complex numbers such that z1 ≠ z2 and
|z1| = |z2|. If z1 has positive real part and z2 has negative Match the Columns
z + z2
imaginary part, then 1 may be (1986, 2M) 31. Match the statements of Column I with those of
z1 − z2 Column II.
(a) zero
(b) real and positive Here, z takes values in the complex plane and Im (z )
(c) real and negative and Re (z ) denote respectively, the imaginary part and
(d) purely imaginary
the real part of z (2010)

25. If z1 = a + ib and z2 = c + id are complex numbers such Column I Column II


that |z1| = |z2| = 1 and Re (z1z2) = 0, then the pair of A. The set of points z satisfying p. an ellipse with
complex numbers w1 = a + ic and w2 = b + id satisfies | z − i| z|| = | z + i | z|| is eccentricity 4/5
(a)|w1| = 1 (b)|w2| = 1 (1985, 2M) contained in or equal to
(c) Re (w1 w2 ) = 0 (d) None of these B. The set of points z satisfying q. the set of points z
| z + 4| + | z − 4| = 0 is satisfying Im ( z) = 0
Passage Based Problems contained in or equal to
C. If| w| = 2 , then the set of r. the set of points z
Read the following passages and answer the questions 1
points z = w − is contained satisfying|Im( z) |≤ 1
that follow. w
Passage I in or equal to
Let A, B, C be three sets of complex number as defined D. If| w| = 1, then the set of points s. the set of points
below 1
z = w + is contained in or satisfying|Re( z)|≤ 2
t.
A = { z : lm (z ) ≥ 1} w the set of points z
B = { z :|z − 2 − i| = 3} equal to satisfying| z| ≤ 3
C = { z : Re((1 − i )z ) = 2 } (2008, 12M)
26. min|1 − 3i − z|is equal to Fill in the Blanks
z ∈s
2− 3 2+ 3 32. If α , β, γ are the cube roots of p, p < 0, then for any x, y
(a) (b) xα + yβ + zγ
2 2 and z then = ... .
3− 3 3+ 3 xβ + yγ + zα (1990, 2M)
(c) (d)
2 2 33. For any two complex numbers z1 , z2 and any real
27. Area of S is equal to numbers a and b,|az1 − bz2|2+ |bz1 + az2|2 = K .
10 π 20 π 16 π 32 π (1988, 2M)
(a) (b) (c) (d)
3 3 3 3   x  x 
sin  2 + cos  2 − i tan (x)
28. Let z be any point in A ∩ B ∩ C and let w be any point 34. If the expression  
  x 
satisfying|w − 2 − i| < 3. Then,|z | − |w| + 3 lies between 1 + 2 i sin  2 
 
(a) − 6 and 3 (b) − 3 and 6
is real, then the set of all possible values of x is… .
(c) − 6 and 6 (d) − 3 and 9 (1987, 2M)
rankershalt.com

4 Complex Numbers

True/False 41. For complex numbers z and w, prove that


35. If three complex numbers are in AP. Then, they lie on a | z |2 w − |w|2 z = z − w, if and only if z = w or z w = 1.
(1999, 10M)
circle in the complex plane (1985 M)
42. Find all non-zero complex numbers z satisfying
36. If the complex numbers, z1 , z2 and z3 represent the
z = iz 2. (1996, 2M)
vertices of an equilateral triangle such that
| z1 | = | z2| = | z3 |, then z1 + z2 + z3 = 0. (1984, 1M) 43. If iz + z − z + i = 0,
3 2
then show that |z| = 1.
(1995, 5M)
37. For complex numbers z1 = x1 + iy1 and z2 = x2 + iy2, we
write z1 ∩ z2, if x1 ≤ x2 and y1 ≤ y2. Then, for all complex
44. A relation R on the set of complex numbers is defined
z1 − z2
1−z by z1 R z2, if and only if is real.
numbers z with 1 ∩ z, we have ∩ 0. (1981, 2M) z1 + z2
1+ z
Show that R is an equivalence relation. (1982, 2M)
Analytical & Descriptive Questions 45. Find the real values of x and y for which the following
38. Find the centre and radius of the circle formed by all the equation is satisfied
points represented by z = x + iy satisfying the relation (1 + i ) x − 2i (2 − 3i ) y + i
+ = i. ( 1980, 2M)
3+ i 3−i
z − α 
  = k (k ≠ 1), where α and β are the constant
z −β 1
46. Express in the form A + iB.
complex numbers given by α = α 1 + iα 2, β = β1 + iβ 2. (1 − cos θ ) + 2i sin θ (1979, 3M)
(2004, 2M)
a + ib a 2 + b2
39. Prove that there exists no complex number z such that 47. If x + iy = , prove that (x2 + y2)2 = 2
n c + id c + d2
|z| < 1 /3 and ∑ a rz r = 1, where|a r|< 2. (1978, 2M)
r =1 (2003, 2M) Integer Answer Type Question
40. If z1 and z2 are two complex numbers such that 48. If z is any complex number satisfying | z − 3 − 2i | ≤ 2,
1 − z1z2 then the maximum value of|2z − 6 + 5i |is …… (2011)
|z1| < 1 < |z2|, then prove that < 1. (2003, 2M)
z1 − z2

Topic 3 Argument of a Complex Number


Objective Questions I (Only one correct option) 5. Let z and w be two non-zero complex numbers
such that |z| = |w| and arg (z ) + arg (w) = π, then z
1. Let z1 and z2 be any two non-zero complex numbers such
equals (1995, 2M)
3z1 2z2
that 3|z1| = 4|z2|. If z = + , then (a) w (b) −w (c) w (d) −w
2z2 3z1 (2019 Main, 10 Jan I)
6. If z1 and z2 are two non-zero complex numbers such
1 17
(a) |z| = (b) Im(z ) = 0 that|z1 + z2| = |z1| + |z2|, then arg (z1 ) − arg (z2) is equal
2 2 to (1987, 2M)
(c) Re(z) = 0 (d) |z| =
5 π π
(a) − π (b) − (c) 0 (d)
2 2 2
2. If z is a complex number of unit modulus and argument 7. If a , b, c and u , v, w are the complex numbers
 1 + z representing the vertices of two triangles such that
θ, then arg   is equal to (2013 Main)
1 + z c = (1 − r ) a + rb and w = (1 − r ) u + rv, where r is a
complex number, then the two triangles (1985, 2M)
π
(a) − θ (b) −θ (c) θ (d) π − θ (a) have the same area (b) are similar
2
(c) are congruent (d) None of these
3. If arg (z ) < 0 , then arg (−z ) − arg (z ) equals (2000, 2M)
(a) π (b) − π Objective Questions II
(c) − π/2 (d) π /2 (One or more than one correct option)
4. Let z and w be two complex numbers such that |z| ≤ 1, 8. For a non-zero complex number z, let arg(z ) denote the
|w| ≤ 1 and|z + i w|= |z − iw| = 2 , then z equals principal argument with − π < arg(z ) ≤ π . Then, which of
(1995, 2M) the following statement(s) is (are) FALSE ? (2018 Adv.)
(a) 1 or i (b) i or −i π
(c) 1 or −1 (d) i or −1 (a) arg (−1 − i ) = , where i = −1
4
rankershalt.com

Complex Numbers 5

(b) The function f : R → (− π, π], defined by Match the Columns


f (t ) = arg (−1 + it ) for all t ∈ R, is continuous at all
points of R, where i = −1. 10. Match the conditions/expressions in Column I with
statement in Column II (z ≠ 0 is a complex number)
(c) For any two non-zero complex numbers z1 and z2,
z  Column I Column II
arg  1  − arg (z1 ) + arg (z2 ) is an integer multiple of
 z2  A. Re ( z) = 0 p. Re ( z2 ) = 0
2π. B. π q. Im ( z2 ) = 0
arg ( z) =
(d) For any three given distinct complex numbers z1 , z2 and 4
z3 , the locus of the point z satisfying the condition r. Re ( z2 ) = Im ( z2 )
 (z − z1 ) (z2 − z3 ) 
arg   = π, lies on a straight line.
 (z − z3 ) (z2 − z1 )  Analytical & Descriptive Questions
9. Let z1 and z2 be two distinct complex numbers and let 11. |z| ≤ 1,|w|≤ 1, then show that
z = (1 − t ) z1 + tz2 for some real number t with 0 < t < 1. If |z − w|2 ≤ (|z| − |w|)2 + (arg z − arg w)2 (1995, 5M)
arg (w) denotes the principal argument of a non-zero 12. Let z1 = 10 + 6i and z2 = 4 + 6i. If z is any complex
complex number w, then (2010)
number such that the argument of (z − z1 ) / (z − z2) is
(a) |z − z1| + |z − z2|= |z1 − z2|(b) arg (z − z1 ) = arg (z − z2 )
π /4, then prove that|z − 7 − 9i| = 3 2. (1991, 4M)
z − z1 z − z1
(c) =0 (d) arg (z − z1 ) = arg (z2 − z1 )
z2 − z1 z2 − z1

Topic 4 Rotation of a Complex Number


Objective Questions I (Only one correct option) π
(d)|z − 1|< 2,| arg (z + 1)|>
5 5 2
 3 i  3 i π
1. Let z =  +  + −  . If R(z ) and I (z ) 5. If 0 < α < is a fixed angle. If P = (cos θ ,sin θ ) and
 2 2  2 2 2
Q = {cos(α − θ ),sin(α − θ )}, then Q is obtained from P by
respectively denote the real and imaginary parts of z,
(2002, 2M)
then (2019 Main, 10 Jan II)
(a) R (z ) > 0 and I (z ) > 0 (b) I (z ) = 0 (a) clockwise rotation around origin through an angle α
(c) R (z ) < 0 and I (z ) > 0 (d) R (z ) = − 3 (b) anti-clockwise rotation around origin through an angleα
(c) reflection in the line through origin with slope tan α
2. A particle P starts from the point z0 = 1 + 2 i, where α
(d) reflection in the line through origin with slope tan
i = −1. It moves first horizontally away from origin by 2
5 units and then vertically away from origin by 3 units
6. The complex numbers z1 , z2 and z3 satisfying
to reach a point z1. From z1 the particle moves 2 units
z1 − z3 1 − i 3
in the direction of the vector i$ + $j and then it moves = are the vertices of a triangle which is
π z2 − z3 2 (2001, 1M)
through an angle in anti-clockwise direction on a
2 (a) of area zero
circle with centre at origin, to reach a point z2. The point (b) right angled isosceles
z2 is given by (2008, 3M) (c) equilateral
(a) 6 + 7i (b) −7 + 6i (c) 7 + 6i (d) − 6 + 7i (d) obtuse angled isosceles

3. A man walks a distance of 3 units from the origin Objective Questions II


towards the North-East (N 45° E) direction. From there,
he walks a distance of 4 units towards the North-West (One or more than one correct option)
(N 45° W) direction to reach a point P. Then, the 7. Let a , b ∈ R and a 2 + b2 ≠ 0.
position of P in the Argand plane is (2007, 3M)
 1 
(a) 3ei π/ 4 + 4 i (b) (3 − 4 i ) ei π / 4 (c) (4 + 3 i )ei π / 4 (d) Suppose S = z ∈ C : z = , t ∈ R, t ≠ 0, where
(3 + 4 i ) ei π / 4  a + i bt 
i = − 1. If z = x + iy and z ∈ S, then (x, y) lies on
4. The shaded region, where P = (−1, 0), Q = (−1 + 2 , 2 )
(2016 Adv.)
R = (−1 + 2 , − 2 ), S = (1, 0) is represented by (2005, 1M)
and centre 
1 1 
Y (a) the circle with radius , 0 for
π 2a  2a 
(a)|z + 1|> 2,| arg (z + 1)|< Q
4 a > 0, b ≠ 0
π
and centre  −
(b)|z + 1|< 2,| arg (z + 1)|< 1 1 
X′ X (b) the circle with radius − , 0 for a <
2 P O S 2a  2a 
π
(c)|z + 1|> 2,| arg (z + 1)|> 0, b ≠ 0
4 R
(c) the X-axis for a ≠ 0, b = 0
Y′
rankershalt.com

6 Complex Numbers

(d) the Y-axis for a = 0, b ≠ 0 13. Let bz + bz = c, b ≠ 0, be a line in the complex plane,
where b is the complex conjugate of b. If a point z1 is the
3+i reflection of the point z2 through the line, then show
8. Let W = and P = {W n: n = 1, 2, 3,... }.
2 that c = z1b + z2b. (1997C, 5M)
 1
14. Let z1 and z2 be the roots of the equation z 2 + pz + q = 0,
Further H 1 = z ∈ C : Re (z ) > 
 2
where the coefficients p and q may be complex numbers.
 −1  Let A and B represent z1 and z2 in the complex plane. If
and H 2 = z ∈ C : Re (z ) < , where C is the set of all
 2  ∠ AOB = α ≠ 0 and OA = OB, where O is the origin prove
complex numbers. If z1 ∈ P ∩ H 1, z2 ∈ P ∩ H 2 and O α
that p2 = 4q cos 2  .
represents the origin, then ∠ z1Oz2 is equal to  2 (1997, 5M)
(2013 JEE Adv.)
π 15. Complex numbers z1 , z2, z3 are the vertices A , B, C
(a)
2 respectively of an isosceles right angled triangle with
π right angle at C. Show that
(b)
6 (z1 − z2)2 = 2(z1 − z3 ) (z3 − z2). (1986, 2 1 M)
2

(c)
3 16. Show that the area of the triangle on the argand
5π diagram formed by the complex number z , iz and z + iz
(d) 1
6 is |z|2.
2 (1986, 2 1 M)
2
Fill in the Blanks 17. Prove that the complex numbers z1 , z2 and the origin
9. Suppose z1 , z2, z3 are the vertices of an equilateral form an equilateral triangle only if z12 + z22 − z1z2 = 0.
triangle inscribed in the circle |z| = 2. If z1 = 1 + i 3, (1983, 2M)
then z2 = K, z3 = … . (1994, 2M)
18. Let the complex numbers z1 , z2 and z3 be the vertices of
an equilateral triangle. Let z0 be the circumcentre of the
10. ABCD is a rhombus. Its diagonals AC and BD intersect triangle. Then, prove that z12 + z22 + z32 = 3z02. (1981, 4M)
at the point M and satisfy BD = 2 AC. If the points D and
M represent the complex numbers 1 + i and 2 − i
respectively, then A represents the complex number
Integer Answer Type Question
kπ   kπ 
…or… (1993, 2M)
19. For any integer k, let α k = cos  + i sin   , where
 7  7
11. If a and b are real numbers between 0 and 1 such that
the points z1 = a + i , z2 = 1 + bi and z3 = 0 form an i = −1. The value of the expression
equilateral triangle, then a = K and b = K . (1990, 2M) 12

∑|α k + 1 − α k|
k =1
Analytical & Descriptive Questions 3
is

12. If one of the vertices of the square circumscribing the ∑|α 4k − 1 − α 4k − 2|


k =1 (2016 Adv.)
circle|z − 1| = 2 is 2 + 3i. Find the other vertices of
square. (2005, 4M)

Topic 5 De-Moivre’s Theorem, Cube Roots and nth Roots of Unity


Objective Questions I (Only one correct option) 3. Let z0 be a root of the quadratic equation, x2 + x + 1 = 0,
1. If z and w are two complex numbers such that| zw| = 1 If z = 3 + 6iz081 − 3iz093 , then arg z is equal to
π (2019 Main, 9 Jan II)
and arg(z ) − arg(w) = , then (2019 Main, 10 April II) π π π
2 (a) (b) (c) 0 (d)
4 6 3
1− i
(a) zw = − i (b) zw = 15
2 4. Let z = cos θ + i sin θ . Then, the value of ∑ Im (z2m −1 ) at
− 1+ i m =1
(c) zw = i (d) zw =
2 θ = 2° is (2009)
1 1
3 i (a) (b)
2. If z = + (i = −1 ), then (1 + iz + z5 + iz 8 )9 is equal sin 2° 3 sin 2°
2 2 1 1
to (2019 Main, 8 April II) (c) (d)
(a) 1 (b) (−1 + 2i )9 (c) −1 (d) 0 2 sin 2° 4 sin 2°
rankershalt.com

Complex Numbers 7

5. The minimum value of |a + bω + cω 2|, where a, b and c Codes


are all not equal integers and ω (≠ 1) is a cube root of P Q R S
unity, is (a) (i) (ii) (iv) (iii)
(2005, 1M) (b) (ii) (i) (iii) (iv)
1 (c) (i) (ii) (iii) (iv)
(a) 3 (b) (c) 1 (d) 0 (d) (ii) (i) (iv) (iii)
2
6. If ω (≠ 1) be a cube root of unity and (1 + ω 2)n = (1 + ω 4 )n, Fill in the Blanks
then the least positive value of n is (2004, 1M)
2π 2π
(a) 2 (b) 3 (c) 5 (d) 6 14. Let ω be the complex number cos + i sin . Then
3 3
1 3 the number of distinct complex number z satisfying
7. Let ω = − +i , then value of the determinant
2 2 z+1 ω ω2
1 1 1
ω z+ω 2
1 = 0 is equal to ... .
1 −1 − ω ω 2 is
2
(2002, 1M)
ω2 1 z+ω
1 ω2 ω (2010)
(a) 3 ω (b) 3 ω (ω − 1) (c) 3 ω 2
(d) 3 ω (1 − ω) 15. The value of the expression
8. Let z1 and z2 be nth roots of unity which subtend a right 1 ( 2 − ω ) (2 − ω 2) + 2(3 − ω ) (3 − ω 2) + ...
+ (n − 1) ⋅ (n − ω ) (n − ω 2) ,
angled at the origin, then n must be of the form (where, k
is an integer) (2001, 1M) where, ω is an imaginary cube root of unity, is….
(1996, 2M)
(a) 4k + 1 (b) 4k + 2 (c) 4k + 3 (d) 4k
334 365
 1 i 3  1 i 3 True/False
9. If i = −1, then 4 + 5  − +  + 3 − +  is
 2 2   2 2  16. The cube roots of unity when represented on Argand
equal to (1999, 2M) diagram form the vertices of an equilateral triangle.
(1988, 1M)
(a) 1 − i 3 (b) −1 + i 3 (c) i 3 (d) −i 3
10. If ω is an imaginary cube root of unity, then (1 + ω − ω ) 27
Analytical & Descriptive Questions
is equal to (1998, 2M)
17. Let a complex number α , α ≠ 1, be a root of the equation
(a) 128 ω (b) −128 ω (c) 128 ω2 (d) −128 ω2
z p + q − z p − zq + 1 = 0
11. If ω (≠ 1) is a cube root of unity and (1 + ω )7 = A + Bω,
then A and B are respectively where, p and q are distinct primes. Show that either
(1995, 2M)
1 + α + α 2 + ... + α p − 1 = 0
(a) 0, 1 (b) 1, 1
or 1 + α + α 2 + ... + α q − 1 = 0
(c) 1, 0 (d) –1, 1
6 but not both together. (2002, 5M)
 2 πk 2 πk 
12. The value of ∑ sin 7
– i cos
7 
 is (1998, 2M)
18. If 1, a1 , a 2, ... , a n − 1 are the n roots of unity, then show
k =1

(a) – 1 (b) 0 (c) – i (d) i that (1 − a1 ) (1 − a 2) (1 − a3 ) K (1 − a n − 1 ) = n


(1984, 2M)
Match the Columns 19. It is given that n is an odd integer greater than 3, but n
2 kπ   2 kπ  is not a multiple of 3. Prove that x3 + x2 + x is a factor
13. Let zk = cos   + i sin   ; k = 1, 2, …9. of (x + 1)n − xn − 1 .
 10   10 
(1980, 3M)
Column I Column II 20. If x = a + b, y = aα + bβ, z = aβ + bα , where α , β are
P. For each zk, there exists a z j such that (i) True complex cube roots of unity, then show that
zk ⋅ z j = 1 xyz = a3 + b3 . (1979, 3M)

Q. There exists a k ∈ { 1, 2, … , 9 } such that (ii) False


z1 ⋅ z = zk has no solution z in the set of Integer Answer Type Question
complex numbers 21. Let ω = eiπ /3 and a , b, c, x, y, z be non-zero complex
R. |1 − z1||1 − z2| … |1 − z9| (iii) 1 numbers such that a + b + c = x, a + bω + cω 2 = y,
10
equal a + bω 2 + cω = z.
| x|2 + | y|2 + | z |2
2 kπ 
1 − ∑ cos 
9
S. (iv) 2 Then, the value of is …… (2011)
 equals | a |2 + | b|2 + | c|2
k =1  10 

(2011)
rankershalt.com

8 Complex Numbers

Answers
Topic 1 1 cot (θ / 2 )
46. A + iB = −i 48. 5
1. (c) 2. (c) 3. (b) 4. (a)  θ 1 + 3 cos2 (θ / 2 )
2 1 + 3 cos2 
5. (d) 6. (d) 7. (d) 8. (b)  2
9. (d) 10. (b, d) Topic 3
Topic 2 1. (*) 2. (c) 3. (a)
1. (b) 2. (b) 3. (d) 4. (d) 4. (c) 5. (d) 6. (c) 7. (b)
5. (b) 6. (c) 7. (b) 8. (c) 8. (a, b, d) 9. (a, c, d) 10. A → q ; B → p
9. (d) 10. (a) 11. (d) 12. (b) Topic 4
13. (a) 14. (b) 15. (a) 16. (d) 1. (b) 2. (d) 3. (d) 4. (a)
17. (d) 18. (b) 19. (b) 20. (d) 5. (d) 6. (c) 7. (d) 8. (c, d)
21. (b) 22. (a) 23. (a, c, d) 24. (a,d) i 3i
9. z 2 = − 2, z 3 = 1 − i 3 10. 3 − or 1 −
25. (a, b, c) 26. (c) 27. (b) 28. (d) 2 2
29. (c) 30. (b) 11. a = b = 2 ± 3
31. A → q, r ; B → p; C → p, s, t ; D → q, r, s, t 32. ω 2
12. z 2 = − 3 i , z 3 = (1 − 3 ) + i and z 4 = (1 + 3 ) − i
33. (a + b )(| z1| + | z 2| )
2 2 2 2
19. (4)
−1
34. x = 2nπ + 2α , α = tan k, where k ∈(1, 2 ) or x = 2nπ Topic 5
35. False 36. True 37. True 1. (a) 2. (c) 3. (a)
α − k 2β k (α − β ) 4. (d) 5. (c) 6. (b)
38. Centre = , Radius =  
1 − k2  1 −k 
2
7. (b) 8. (d) 9. (c) 10. (d)
 3 i 11. (b) 12. (d) 13. (c) 14. (1)
42. z = i , ± –  2
 n (n + 1 )
 2 2 15.   −n 16. True 21. (3)
 2 
45. (x = 3 and y = −1)

Hints & Solutions


Topic 1 Complex Number in Iota Form  α 2 − 1  2α 
2 2
Now, x2 + y2 =  2  + 2 
1. Let z = x + 10i, as Im (z ) = 10 (given).  α + 1  α + 1
Since z satisfies,
2z − n α 4 + 1 − 2α 2 + 4α 2 (α 2 + 1)2
= 2i − 1, n ∈ N , = = 2 =1
2z + n (α 2 + 1)2 (α + 1)2

∴ (2x + 20i − n ) = (2i − 1) (2x + 20i + n ) ⇒ x2 + y2 = 1


⇒ (2x − n ) + 20i = (− 2x − n − 40) + (4x + 2n − 20)i Which is an equation of circle with centre (0, 0) and
On comparing real and imaginary parts, we get radius 1 unit.
2x − n = − 2x − n − 40 and 20 = 4x + 2n − 20 α + i 
So, S =  ; α ∈ R lies on a circle with radius 1.
⇒ 4x = − 40 and 4x + 2n = 40  α − i 
⇒ x = − 10 and − 40 + 2n = 40 ⇒ n = 40 3. Given complex number
So, n = 40 and x = Re (z ) = − 10 5 + 3z
ω=
α+i 5(1 − z )
2. Let x + iy =
α −i ⇒ 5 ω − 5 ω z = 5 + 3z
(α + i )2 (α 2 − 1) + (2α )i α 2 − 1  2α  ⇒ (3 + 5 ω )z = 5 ω − 5
⇒ x + iy = 2 = = 2 + i
α +1 α2 + 1 α + 1  α 2 + 1 ⇒ |3 + 5 ω||z| = |5 ω − 5| …(i)
On comparing real and imaginary parts, we get [applying modulus both sides and |z1z2| = |z1||z2|]
Q |z| < 1
α2 −1 2α
x= and y = 2 ∴ |3 + 5 ω| > |5 ω − 5| [from Eq. (i)]
α2 + 1 α +1
rankershalt.com

Complex Numbers 9

ω + 3 > |ω − 1| (2 + 3i sin θ) (1 + 2i sin θ )


⇒ =
 5 (1 − 2i sin θ ) (1 + 2i sin θ)
2
 3 2 + 4i sin θ + 3i sin θ + 6i 2 sin 2 θ
Let ω = x + iy, then  x +  + y2 > (x − 1)2 + y2 =
 5 12 − (2i sin θ) 2
9 6
⇒ x +2
+ x > x + 1 − 2x
2
2 + 7i sin θ − 6 sin 2 θ
25 5 =
16x 16 1 1 + 4 sin 2 θ
⇒ > ⇒ x > ⇒ 5x > 1
5 25 5 2 − 6 sin 2 θ 7 sin θ
= +i
⇒ 5 Re( ω ) > 1 1 + 4 sin 2 θ 1 + 4 sin 2 θ
x + iy  1   –1
3

3 Q Re(z ) = 0
4. We have, =  − 2 − i = (6 + i ) 2 − 6 sin 2 θ
27  3   3 
∴ = 0 ⇒ 2 = 6 sin 2 θ
x + iy 1 1 + 4 sin 2 θ
⇒ =− (216 + 108i + 18i 2 + i3 )
27 27 1
⇒ sin 2 θ =
1 3
=− (198 + 107i )
27 1
⇒ sin θ = ±
[Q (a + b)3 = a3 + b3 + 3a 2b + 3ab2, i 2 = − 1, i3 = − i] 3
On equating real and imaginary part, we get −1  1 −1  1 
⇒ θ = sin  ±  = ± sin  
x = − 198 and y = − 107  3  3
⇒ y − x = − 107 + 198 = 91  6i −3 i 1
7. Given, 4 3i −1  = x + iy
 3 + 2i sin θ   1 + 2 i sin θ 
5. Let z =   ×  20 3 i
 1 − 2i sin θ   1 + 2 i sin θ 
 6i 1 1
(rationalising the denominator) ⇒ −3 i  4 −1 −1  = x + i y
3 − 4 sin 2 θ + 8i sin θ  20 i i
=
1 + 4 sin 2 θ ⇒ x + iy = 0 [Q C 2 and C3 are identical]
[Q a 2 − b2 = (a + b)(a − b) and i 2 = − 1] ⇒ x = 0, y = 0
 3 − 4 sin θ   8 sin θ 
2
13 13 13
=  + i
 1 + 4 sin 2 θ   1 + 4 sin 2 θ  8. ∑ (i n + i n + 1 ) = ∑ i n (1 + i ) = (1 + i ) ∑ in
n=1 n=1 n =1
As z is purely imaginary, so real part of z = 0  i − (1 − i13 ) 
3 − 4 sin 2 θ = (1 + i ) (i + i 2 + i3 + K + i13 ) = (1 + i )  
∴ = 0 ⇒ 3 − 4 sin 2 θ = 0  1−i 
1 + 4 sin 2 θ  i (1 − i ) 
3 3 = (1 + i )   = (1 + i ) i = i − 1
⇒ sin 2 θ = ⇒ sin θ = ±  1−i 
4 2
Y Alternate Solution
1 y=sin θ Since, sum of any four consecutive powers of iota is zero.
√3/2 13
∴ ∑ (i n + i n + 1 ) = (i + i 2 + K + i13 )
–π/2 –π/3 n=1
X′ π X
O π/3 2π/3 + (i 2 + i3 + K + i14 ) = i + i 2 = i − 1
n n
 1 + i 1 + i 1 + i
–√3/2 9. Since,   =1 ⇒  ×  =1
−1 1 − i 1 − i 1 + i
Y′ n
 2i 
⇒   =1
 π π 2π   2
⇒ θ ∈ − , , 
 3 3 3  ⇒ in = 1
2π The smallest positive integer n for which i n = 1 is 4.
Sum of values of θ = .
3 ∴ n =4
2 + 3i sin θ az + b ax + b + aiy (ax + b + aiy)((x + 1) − iy)
6. Let z = is purely imaginary. Then, we have 10. = =
1 − 2i sin θ z+1 (x + 1) + iy (x + 1)2 + y2
Re(z ) = 0
 az + b − (ax + b) y + ay(x + 1)
2 + 3i sin θ ∴ Im  =
Now, consider z =  z+1 (x + 1)2 + y2
1 − 2i sin θ
rankershalt.com

10 Complex Numbers

(a − b) y z −α
⇒ =y 4. Since, the complex number (α ∈ R) is purely
(x + 1)2 + y2 z+α
Q a − b =1 imaginary number, therefore
z −α z −α
∴ (x + 1) + y2 = 1
2
+ =0 [Qα ∈ R]
z+α z+α
∴ x = − 1 ± 1 − y2
⇒ zz − αz + αz − α 2 + zz − αz + αz − α 2 = 0
2 2
⇒ 2 z − 2 α2 = 0 [Qzz = z ]
Topic 2 Conjugate and Modulus of 2
Complex Number ⇒ α2 = z =4 [| z | = 2 given]
1. Let the complex number z = x + iy ⇒ α=±2
Also given,| z − i | = | z − 1| 5. We have,|z | + z = 3 + i
⇒| x + iy − i | = | x + iy − 1| Let z = x + iy
⇒ x + ( y − 1) = (x − 1) + y
2 2 2 2 ∴ x2 + y2 + x + iy = 3 + i

[Q| z | = (Re(z ))2 + (Im(z ))2 ] ⇒ (x + x2 + y2 ) + iy = 3 + i

On squaring both sides, we get ⇒ x + x2 + y2 = 3 and y = 1


x2 + y2 − 2 y + 1 = x2 + y2 − 2x + 1 Now, x2 + 1 = 3 − x
⇒ y = x, which represents a line through the origin with ⇒ x + 1 = 9 − 6x + x2
2

slope 1. 4
⇒ 6x = 8 ⇒ x =
(1 + i ) 2 3
2. The given complex number z = 4
a−i ∴ z= +i
(1 − 1 + 2i ) (a + i ) 3
= [Q i 2 = − 1]
a2 + 1 ⇒ |z | =
16
+1=
25
⇒ |z | =
5
2i (a + i ) −2 + 2ai 9 9 3
= = …(i)
a2 + 1 a2 + 1 6. PLAN If z is unimodular, then| z| = 1. Also, use property of modulus
i.e. z z =| z|2
Q z = 2 /5 [given]
Given, z2 is not unimodular i.e.|z2|≠ 1
4 + 4a 2 2 2 2 z − 2 z2
⇒ = ⇒ = and 1 is unimodular.
(a 2 + 1)2 5 1+ a 2 5 2 − z1z2
4 2 z1 − 2z2
⇒ = ⇒ a 2 + 1 = 10 ⇒ = 1 ⇒ |z1 − 2z2|2 =|2 − z1z2|2
1 + a2 5 2 − z1z2
⇒ a2 = 9 ⇒ a = 3 [Qa > 0] ⇒ (z1 − 2z2)(z1 − 2z2) = (2 − z1z2) (2 − z1z2) [zz = |z|2 ]
–2 + 6i ⇒ |z1| +4|z2| −2z1z2 − 2z1z2
2 2
∴ z= [From Eq. (i)]
10 = 4+|z1|2|z2|2−2z1z2 − 2z1z2 ⇒ (|z2|2−1)(|z1|2−4) = 0
 −2 + 6 i   1 3  1 3 Q |z2|≠ 1
So, z =   =  − + i ⇒ z = − − i
 10   5 5  5 5 ∴ |z1|= 2
Let z1 = x + iy ⇒ x2 + y2 = (2)2
[Qif z = x + iy, then z = x − iy] ∴ Point z1 lies on a circle of radius 2.
3. Clearly|z1|= 9, represents a circle having centre C1 (0, 0) 7. |z| ≥ 2 is the region on or outside circle whose centre is
and radius r1 = 9. (0, 0) and radius is 2.
and |z2 − 3 − 4i|= 4 represents a circle having centre 1
Minimum z + is distance of z, which lie on circle
C 2(3, 4) and radius r2 = 4. 2
The minimum value of |z1 − z2| is equals to minimum | z | = 2 from (−1 / 2, 0).
distance between circles|z1|= 9 and|z2 − 3 − 4i|= 4. 1  1 
∴ Minimum z + = Distance of  − , 0 from (−2, 0)
2  2 
QC1C 2 = (3 − 0)2 + (4 − 0)2 = 9 + 16 = 25 = 5
2 2
and|r1 − r2|=|9 − 4|= 5 ⇒ C1C 2 =|r1 − r2|  1 3  −1  3
=  −2 +  + 0 = =  + 2 + 0 =
∴ Circles touches each other internally.  2 2  2  2
Hence, |z1 − z2|min = 0
rankershalt.com

Complex Numbers 11

Y  − r 2
⇒ (|α|2 − 1) ⋅   = r 2(1 − 4|α|2 )
 2 
⇒ |α|2 − 1 = − 2 + 8|α|2
X′ D A X ⇒ 7|α|2 = 1
(–2,0) 1 , (0,0) (2,0)
(—
– ) ∴ |α| = 1 / 7
2 0
9. PLAN If ax + bx + c = 0 has roots α, β, then
2

− b ± b 2 − 4ac
α, β =
Y′ 2a
1 For roots to be real b 2 − 4ac ≥ 0.
Geometrically Min z + = AD
2 Description of Situation As imaginary part of
1 z = x + iy is non-zero.
Hence, minimum value of z + lies in the interval
(1, 2). 2 ⇒ y ≠0
Method I Let z = x + iy
8. PLAN Intersection of circles, the basic concept is to solve the
∴ a = (x + iy)2 + (x + iy) + 1
equations simultaneously and using properties of modulus of ⇒ (x2 − y2 + x + 1 − a ) + i (2xy + y) = 0
complex numbers.
⇒ (x2 − y2 + x + 1 − a ) + iy (2x + 1) = 0, …(i)
Formula used |z|2 = z ⋅ z
It is purely real, if y (2x + 1) = 0
and |z1 − z2|2 = (z1 − z2) (z1 − z2)
but imaginary part of z, i.e. y is non-zero.
= |z1|2 − z1z2 − z2z1 + |z2|2
⇒ 2x + 1 = 0 or x = − 1 / 2
Here, (x − x0 )2 + ( y − y0 )2 = r 2
1 1
From Eq. (i), − y2 − + 1 − a = 0
and (x − x0 )2 + ( y − y0 )2 = 4r 2 can be written as, 4 2
|z − z0|2 = r 2 and |z − z0|2 = 4r 2 3 3
⇒ a = − y2 + ⇒ a<
1 4 4
Since, α and lies on first and second respectively.
α Method II Here, z 2 + z + (1 − a ) = 0
2
1 − 1 ± 1 − 4 (1 − a )
∴ |α − z0|2 = r 2 and − z0 = 4 r 2 ∴ z=
α 2 ×1
⇒ (α − z0 ) (α − z0 ) = r 2 − 1 ± 4a − 3
⇒ z=
⇒ |α|2 − z0α − z0α + |z0|2 = r 2 …(i) 2
3
1
2 For z do not have real roots, 4 a − 3 < 0 ⇒ a<
and − z0 = 4 r 2 4
α
10. Since, zz (z 2 + z 2) = 350
1  1  ⇒ 2 (x + y2) (x2 − y2) = 350
2
⇒  − z0   − z0  = 4 r
2
α  α 
⇒ (x2 + y2) (x2 − y2) = 175
1 z z Since, x, y ∈ I, the only possible case which gives
⇒ − 0 − 0 + |z0|2 = 4r 2
|α|2 α α integral solution, is
Since, |α|2 = α ⋅ α x2 + y2 = 25 ... (i)
1 z ⋅α z x2 − y2 = 7 ... (ii)
⇒ − 0 − 0 ⋅ α + |z0|2 = 4r 2
|α|2 |α|2 |α|2 From Eqs. (i) and (ii),
⇒ 1 − z0α − z0α + |α|2|z0|2 = 4r 2|α|2 …(ii) x2 = 16, y2 = 9 ⇒ x = ± 4, y = ± 3
On subtracting Eqs. (i) and (ii), we get ∴ Area of rectangle = 8 × 6 = 48
(|α| − 1) + |z0| (1 − |α| ) = r (1 − 4|α| )
2 2 2 2 2 11. Let z = cos θ + i sin θ
z cos θ + i sin θ
⇒ (|α| − 1) (1 − |z0| ) = r (1 − 4|α| )
2 2 2 2
⇒ =
1 − z 2 1 − (cos 2 θ + i sin 2 θ )
 r 2 + 2 cos θ + i sin θ
⇒ (|α|2 − 1) 1 −  = r 2(1 − 4|α|2 ) =
 2  2 sin 2 θ − 2i sin θ cos θ
r2 + 2 cos θ + i sin θ i
Given, |z0|2 = = =
2 − 2i sin θ (cos θ + i sin θ ) 2 sin θ
z
Hence, lies on the imaginary axis i.e. Y-axis.
1 − z2
rankershalt.com

12 Complex Numbers

Alternate Solution 16. (1 + i )n1 + (1 − i )n1 + (1 + i )n2 + (1 − i )n2


z z 1
Let E = = = which is an imaginary. = [n1 C 0 + n1
C1i + n1
C 2i 2 + n1
C3 i3 + K ]
1 − z 2 zz − z 2 z − z
w − wz + [ C0 −
n1 n1
C1 i + C 2i 2 − n1 C3 i3 + ... ]
n1

12. Let z1 = be purely real ⇒ z1 = z1 + [n2 C 0 + n2


C1i + n2C 2i 2 + n2C3 i3 + K ]
1−z
w − wz w − wz + [n2 C 0 – n2
C1 i + n2C 2i 2 – n2C3 i3 + .. ]
∴ =
1− z 1−z
= 2 [n1 C 0 + n1
C2 i2 + n1
C 4i 4 + K ]
⇒ w − wz − wz + wz ⋅ z = w − zw − wz + wz ⋅ z
+ 2 [ C0 +n2 n2
C 2i 2 + n2
C 4i 4 + K ]
⇒ (w − w ) + (w − w)| z |2 = 0
= 2 [n1 C 0 − n1
C2 + n1
C 4 − K ] + 2 [n2 C 0 − n2
C2
⇒ (w − w ) (1 − | z |2 ) = 0
+ n2
C 4 −... ]
⇒ | z |2 = 1 [as w − w ≠ 0, since β ≠ 0]
This is a real number irrespective of the values of n1 and
⇒ | z | = 1 and z ≠ 1 n2.
z −1
13. Since,|z| = 1 and w = Alternate Solution
z+1 {(1 + i )n1 + (1 − i )n1 } + {(1 + i )n2 + (1 − i )n2 }
1+ w |1 + w|
⇒ z − 1 = wz + w ⇒ z = ⇒ |z| = ⇒ A real number for all n1 and n2 ∈ R.
1−w |1 − w|
[Q z + z = 2 Re (z ) ⇒ (1 + i )n1 + (1 − i )n1 is real number
⇒ |1 − w| = |1 + w| [ Q|z| = 1] for all n ∈ R]
On squaring both sides, we get 17. Since, (sin x + i cos 2x) = cos x − i sin 2x
1 + |w| − 2|w| Re (w) = 1 + |w| + 2|w| Re (w)
2 2
⇒ sin x − i cos 2x = cos x − i sin 2x
[using|z1 ± z2|2 = |z1|2 + |z2|2 ± 2|z1||z2| Re (z1z2)] ⇒ sin x = cos x and cos 2x = sin 2x
⇒ 4|w|Re|w| = 0 ⇒ tan x = 1 and tan 2x = 1
⇒ Re (w) = 0
⇒ x = π / 4 and x = π / 8 which is not possible at same
14. We know, |z1 − z2| = |z1 − (z2 − 3 − 4i ) − (3 + 4i )| time.
≥ |z1| − |z2 − 3 − 4i | − |3 + 4i| Hence, no solution exists.
≥ 12 − 5 − 5 [using|z1 − z2| ≥ |z1| − |z2|] 18. Since, z1 , z2, z3 , z4 are the vertices of parallelogram.
∴ |z1 − z2| ≥ 2
D(z4) C(z3)

Alternate Solution
Clearly from the figure|z1 − z2|is minimum when z1 , z2
lie along the diameter.
Y
B
A Z1 A(z1) B(z2)
,4
) Z2
(3
C1 ∴ Mid-point of AC = mid-point of BD
X′
12
X z1 + z3 z2 + z4
C2 ⇒ =
2 2
⇒ z1 + z3 = z2 + z4
 1 − iz 
Y′ 19. Since,|w| = 1 ⇒  = 1
z−i 
∴ |z1 − z2| ≥ C 2B − C 2A ≥ 12 − 10 = 2
⇒ |z − i| = |1 − iz|
15. Given, |z1| = |z2| = |z3| = 1 ⇒ |z − i | = |z + i | [Q |1 − iz | = | − i || z + i | = | z + i |]
Now, |z1| = 1 ∴It is a perpendicular bisector of (0, 1) and (0, − 1)
⇒ |z1|2 = 1 ⇒ z1z1 = 1 i.e. X-axis. Thus, z lies on the real axis.
Similarly, z2z2 = 1, z3 z3 = 1 20. Given,|z − 4| < |z − 2|
1 + 1 + 1 
 = 1 Since, |z − z1| > |z − z2| represents the region on right
Again now,
z1 z2 z3 side of perpendicular bisector of z1 and z2.
∴ |z − 2| > |z − 4|
⇒ | z1 + z2 + z3 |= 1 ⇒ |z1 + z2 + z3|= 1
⇒ Re (z ) > 3 and Im (z ) ∈ R
⇒ |z1 + z2 + z3| = 1
rankershalt.com

Complex Numbers 13

Y
It is false.
(c) If elements of set L represents line, then this line
and given circle intersect at maximum two point.
X′ X Hence, it is true.
O (2, 0) (3, 0) (4, 0)
(d) In this case locus of z is a line, so L has infinite
elements. Hence, it is true.
Y′ 24. Given,|z1| = |z2|
5 5
 3 i  3 i z1 + z2 z1 − z2 z1z1 − z1z2 + z2z1 − z2 z2
21. Given, z =  +  + −  Now, × =
 2 2  2 2 z1 − z2 z1 − z2 |z1 − z2|2
 −1 + i 3 −1 − i 3  |z1|2 + (z2 z1 − z1 z2) − |z2|2
Q ω = and ω 2 =  =
 2 2  |z1 − z2|2
3+i  −1 + i 3  z2z1 − z1z2
Now, = −i   = − iω = [Q|z1|2 = |z2|2 ]
2  2  |z1 − z2|2
3−i  −1 − i 3 
and =i  = iω 2
2  2  As, we know z − z = 2i Im (z )
∴ z = (− iω )5 + (iω 2)5 = − iω 2 + iω
∴ z2z1 − z1z2 = 2i Im (z2z1 )
= i(ω − ω 2) = i (i 3 ) = − 3
z1 + z2 2i Im (z2 z1 )
⇒ Re(z ) < 0 and lm (z ) = 0 ∴ =
z1 − z2 |z1 − z2|2
Alternate Solution
We know that, z + z = 2 Re(z ) which is purely imaginary or zero.
5 5
 3 i  3 i 25. Since, z1 = a + ib and z2 = c + id
If z= +  + −  , then
 2 2  2 2
⇒ |z1| = a + b = 1 and |z2|2 = c2 + d 2 = 1
2 2 2
…(i)
z is purely real. i.e. Im (z ) = 0
[Q|z1|=|z2| = 1]
z − 5i 
 Also, Re (z1z2) = 0 ⇒ ac + bd = 0
22. Given,  = 1 ⇒ |z − 5i| = |z + 5i|
z + 5i  a d
⇒ =− =λ [say]…(ii)
[Q if|z − z1| = |z − z2|, then it is a perpendicular b c
bisector of z1 and z2 ] From Eqs. (i) and (ii), b2λ2 + b2 = c2 + λ2c2
Y ⇒ b2 = c2 and a 2 = d 2
(0, 5) Also, given w1 = a + ic and w2 = b + id
O Now, |w1| = a 2 + c2 = a 2 + b2 = 1
X′ X

(0, –5) |w2| = b2 + d 2 = a 2 + b2 = 1

Y′ and Re(w1 w2) = ab + cd = (bλ )b + c(− λc) [from Eq. (i)]


= λ (b2 − c2) = 0
∴ Perpendicular bisector of (0, 5) and (0, – 5) is X-axis.
26. min|1 − 3 i − z|= perpendicular distance of point (1, − 3)
23. We have, Z ∈S
sz + tz + r = 0 …(i) | 3 − 3| 3 − 3
On taking conjugate from the line 3x + y = 0 ⇒ =
3+1 2
sz + tz + r = 0 …(ii) 27. Since, S = S1 ∩ S 2 ∩ S3
On solving Eqs. (i) and (ii), we get
Y
rt − rs
z= 2
|s| − |t|2
(a) For unique solutions of z
150°
|s|2 − |t|2 ≠ 0 ⇒ |s| ≠ |t| X′ X
O (4, 0)
It is true
(b) If|s| = |t|, then rt − rs may or may not be zero.
So, z may have no solutions. y = –3√x
∴ L may be an empty set. Y′
rankershalt.com

14 Complex Numbers

Clearly, the shaded region represents the area of sector 1


= 2 (cos θ + i sin θ ) − (cos θ − i sin θ )
1 2 1 5π 20π 2
∴ S= r θ = × 42 × = 3 5
2 2 6 3 = cos θ + i sin θ
2 2
28. Since, |w − (2 + i )| < 3 ⇒ |w| − |2 + i| < 3
Let z = x + iy
⇒ −3 + 5 < |w| < 3 + 5 3 5
⇒ −3 − 5 < − |w| < 3 − 5 …(i) ⇒ x = cos θ and y = sin θ
2 2
Also, |z − (2 + i )| = 3  2x
2
 2 y
2
⇒   +   =1
⇒ −3 + 5 ≤ |z| ≤ 3 + 5 …(ii)  3 5
∴ −3 < |z| − |w| + 3 < 9 x2 y2
⇒ + =1
29. |z + 1 − i| + |z − 5 − i|
2 2
9 / 4 25 / 4
= (x + 1)2 + ( y − 1)2 + (x − 5)2 + ( y − 1)2 9 /4 4
∴ e= 1− =
= 2(x2 + y2 − 4x − 2 y) + 28 25 / 4 5
= 2(4) + 28 = 36 [Q x2 + y2 − 4x − 2 y = 4] D. Let w = cos θ + i sin θ
30. Let z = x + iy 1
Then, z = x + iy = cos θ + i sin θ +
cos θ + i sin θ
Set A corresponds to the region y ≥ 1 …(i)
= 2 cos θ
Set B consists of points lying on the circle, centre at
(2, 1) and radius 3. ⇒ x = 2 cos θ , y = 0
i.e. x2 + y2 − 4x − 2 y = 4 …(ii) xα + yβ + zγ x( p)1/3 + y( p)1/3 ω + z ( p)1/3 ω 2
32. =
Set C consists of points lying on the x + y = 2 …(iii) xβ + yγ + zα x( p)1/3 ω 2 + y( p)1/3 ω3 + z ( p)1/3 ω
ω 2 (x + yω + z ω 2)
Y
ω 2 (xω + yω 2 + z )
P ω 2 (x + yω + zω 2)
= = ω2
(0,√2) x + yω + zω 2
(2,1)
y=1
33. |az1 − bz2|2 + |bz1 + az2|2
X′ X
(√2,0) = [a 2|z1|2 + b2|z2|2 − 2ab Re (z1z2)]
+ [b2|z1|2 + a 2|z2|2 + 2ab Re (z1z2)]
= (a 2 + b2) (|z1|2 + |z2|2 )
Y′
 x x
Clearly, there is only one point of intersection of the line sin + cos  − i tan x
 2 2
x + y = 2 and circle x2 + y2 − 4x − 2 y = 4. 34. ∈R
x
1 + 2 i sin
31. A. Let z = x + iy 2
⇒ we get y x + y =0
2 2  x x  x
sin + cos − i tan x 1 − 2i sin 
 2 2   2
⇒ y=0 =
x
⇒ Im (z ) = 0 1 + 4 sin 2
2
B. We have Since, it is real, so imaginary part will be zero.
2ae = 8, 2a = 10 x x x
∴ −2 sin sin + cos  − tan x = 0
⇒ 10e = 8 2 2 2
4 x x x x x
⇒ e= ⇒ 2 sin sin + cos  cos x + 2 sin cos = 0
5 2  2 2  2 2
 16 x  x  x
⇒ b2 = 25 1 −  = 9 x 2x 2 x
 25 ⇒ sin sin + cos   cos − sin  + cos  = 0
2  2 2  2 2 2
x2 y2
∴ + =1 ∴ sin
x
=0
25 9 2
C. Let w = 2 (cos θ + i sin θ ) ⇒ x = 2 nπ ... (i)
1
∴ z = 2 (cos θ + i sin θ ) −  x x  2x 2 x x
2 (cos θ + i sin θ ) or sin + cos   cos − sin  + cos = 0
 2 2  2 2 2
rankershalt.com

Complex Numbers 15

x
On dividing by cos3 , we get + (|α|2 − k2|β|2 ) = 0
2 (α − k β )
2
(α − β k ) 2
|α|2 − k2|β|2
⇒ |z|2 − z − z + = 0 …(i)
 x   2 x  2 x (1 − k2) (1 − k2) (1 − k2)
 tan + 1 1 − tan  + 1 + tan  = 0
 2   2  2
On comparing with equation of circle,
⇒ tan3
x x
− tan − 2 = 0 |z|2 + az + az + b = 0
2 2 whose centre is (− a ) and radius = |a|2 − b
x
Let tan = t α − k2β
2 ∴ Centre for Eq. (i) =
1 − k2
and f (t ) = t3 − t − 2
 α − k2β  α − k2β  αα − k2ββ
Then, f (1) = − 2 < 0 and radius =     −
and f (2) = 4 > 0  1 − k2   1 − k2  1 − k2
Thus, f (t ) changes sign from negative to positive in the k(α − β ) 
= 
1−k 
interval (1, 2). 2

∴ Let t = k be the root for which


f (k) = 0 and k ∈(1, 2)
39. Given, a1z + a 2z 2 + K + a nz n = 1
x 1
∴ t = k or tan = k = tan α and |z| < …(i)
2 3
⇒ x/2 = nπ + α ∴ |a1z + a 2z 2 + a3 z3 + K + a nz n| = 1
x = 2nπ + 2α , α = tan −1 k, where k ∈ (1, 2)
⇒  ⇒ |a1z| + |a 2z 2| + |a3 z3| + K + |a nz n| ≥ 1
 or x = 2nπ
[using|z1 + z2| ≤ |z1| + |z2|]
35. Since, z1 , z2, z3 are in AP.
⇒ 2{(|z| + |z| + |z| + K + |z|n } > 1
2 3
[using|a r| < 2]
⇒ 2z2 = z1 + z3 2|z|(1 − |z| ) n

i.e. points are collinear, thus do not lie on circle. Hence, ⇒ >1 [using sum of n terms of GP]
1 − |z|
it is a false statement.
⇒ 2|z| − 2|z|n + 1 > 1 − |z|
36. Since, z1 , z2, z3 are vertices of equilateral triangle and
⇒ 3|z| > 1 + 2|z|n + 1
|z1| = |z2| = |z3| 1 2
⇒ z1 , z2, z3 lie on a circle with centre at origin. ⇒ |z| > + |z|n + 1
3 3
⇒ Circumcentre = Centroid ⇒
1
|z| > , which contradicts …(ii)
z + z2 + z3 3
⇒ 0= 1
3 ∴ There exists no complex number z such that
∴ z1 + z2 + z3 = 0 n
|z| < 1 / 3 and ∑ a rz r = 1
37. Let z = x + iy ⇒ 1 ∩ z gives 1 ∩ x + iy r =1

or 1 ≤ x and 0 ≤ y …(i) 40. Given,|z1| < 1 and |z2| > 1 …(i)


1−z 1 − x − iy Then, to prove
Given, ∩0 ⇒ ∩0
1+ z 1 + x + iy  
1 − z1z2
 < 1 
z1
 |z1|
using z = |z |

(1 − x − iy) (1 + x − iy)
∩ 0 + 0i  z1 − z2    2 2 
(1 + x + iy) (1 + x − iy) ⇒ |1 − z1z2| < |z1 − z2| …(ii)
1−x − y
2 2
2iy On squaring both sides, we get,
⇒ − ∩ 0 + 0i
(1 + x)2 + y2 (1 + x)2 + y2 (1 − z1z2)(1 − z1z2) < (z1 − z2)(z1 − z2) [using|z|2 = zz ]
⇒ x2 + y2 ≥ 1 ⇒ 1 − z1z2 − z1z2 + z1z1z2z2 < z1z1 − z1z2 − z2z1 + z2z2
and −2 y ≤ 0 ⇒ 1 + |z1|2|z2|2 <|z1|2 + |z2|2
or x2 + y2 ≥ 1 and y ≥ 0 which is true by Eq. (i). ⇒ 1 − |z1|2 − |z2|2+ | z1|2|z2|2 < 0
38. As we know,|z|2 = z ⋅ z ⇒ (1 − |z1|2 )(1 − |z2|2 ) < 0 …(iii)
which is true by Eq. (i) as|z1| < 1 and|z2| > 1
|z − α|2
Given, = k2 ∴ (1 − |z1|2 ) > 0 and (1 − |z2|2 ) < 0
|z − β|2
∴ Eq. (iii) is true whenever Eq. (ii) is true.
⇒ (z − α )(z − α ) = k2(z − β ) (z − β )
⇒ |z|2 − αz − αz + |α|2 = k2(|z|2 − βz − β z + |β|2 ) ⇒ 1 − z1z2
 < 1 Hence proved.
 z1 − z2 
⇒ |z| (1 − k ) − (α − k β )z − (α − β k ) z
2 2 2 2
rankershalt.com

16 Complex Numbers

41. Given,|z|2 w − |w|2 z = z − w Again, |z|2 w − |w|2 z = z − w


⇔ z ⋅ zw − w ⋅ wz = z − w
⇒ zz w − ww z = z − w [Q |z|2 = zz ] …(i)
⇔ z (zw − 1) − w (zw − 1) = 0
Taking modulus of both sides, we get
⇔ (z − w)(zw − 1) = 0 [from Eq. (i)]
|zw||z − w| = |z − w| ⇔ z = w or zw = 1
⇒ |zw||z − w| = |z − w| [∴ |z| = |z| ] Therefore, |z|2 w − |w|2 z = z − w if and only if z = w or
⇒ | zw|| z − w | = |z − w | zw = 1.
⇒ |z − w|(|zw| − 1) = 0 42. Let z = x + iy.
⇒ |z − w| = 0 or |zw| − 1 = 0 Given, z = iz 2
⇒ |z − w| = 0 or |zw| = 1 ⇒ (x + iy) = i (x + i y)2
⇒ z − w=0 or |z w|= 1 ⇒ x − iy = i (x2 − y2 + 2i xy)
⇒ z=w or |zw| = 1 ⇒ x − iy = − 2xy + i (x2 − y2)
Now, suppose z ≠ w NOTE It is a compound equation, therefore we can generate
from it more than one primary equations.
Then,|zw| = 1 or|z||w| = 1
On equating real and imaginary parts, we get
1
⇒ |z| = =r [say] x = − 2xy and − y = x2 − y2
|w|
1 iφ ⇒ x + 2xy = 0 and x2 − y2 + y = 0
Let z = reiθ and w = e
r ⇒ x(1 + 2 y) = 0
On putting these values in Eq. (i), we get ⇒ x = 0 or y = − 1 / 2
1  1 1 When x = 0, x2 − y2 + y = 0 ⇒ 0 − y2 + y = 0
r 2 ei φ  − 2 (reiθ ) = reiθ − eiφ
r  r r ⇒ y(1 − y) = 0 ⇒ y = 0 or y = 1
1 1 When, y = − 1 / 2 , x2 − y2 + y = 0
⇒ reiφ − eiθ = reiθ − eiφ
r r 1 1 3
 1  iφ  1  iθ ⇒ x2 − − = 0 ⇒ x2 =
⇒ r +  e = r +  e 4 2 4
 r  r 3
⇒ eiφ = eiθ ⇒ φ = θ ⇒ x=±
2
1
Therefore, z = reiθ and w = eiθ Therefore, z = 0 + i 0 , 0 + i ; ±
3 i

r 2 2
1 3 i
⇒ zw = reiθ . e−iθ = 1 ⇒ z = i, ± − [Q z ≠ 0]
r 2 2
NOTE ‘If and only if ’ means we have to prove the relation in 43. Given, iz3 + z 2 − z + i = 0
both directions.
Conversely ⇒ iz3 − i 2z 2 − z + i = 0 [Q i 2 = − 1]
Assuming that z = w or z w = 1 ⇒ iz (z − i ) − 1(z − i ) = 0
2

If z = w, then ⇒ (iz 2 − 1)(z − i ) = 0


LHS = zz w − w wz = |z| ⋅z − |w| ⋅z
2 2
⇒ z − i = 0 or iz 2 − 1 = 0
= |z| ⋅z − |z| ⋅z = 0
2 2
1
⇒ z = i or z 2 = = − i
and RHS = z − w = 0 i
If zw = 1, then zw = 1 and If z = i, then|z| = |i| = 1
LHS = zz w − ww z = z ⋅ 1 − w ⋅ 1 If z 2 = − i, then |z 2| = |− i| = 1
= z − w = z − w = 0 = RHS ⇒ |z|2 = 1 ⇒ |z| = 1
z − z2
Hence proved. 44. Here, z1Rz2 ⇔ 1 is real
z1 + z2
Alternate Solution
z − z1
We have, |z|2 w − |w|2 z = z − w (i) Reflexive z1Rz1 ⇔ 1 =0 [purely real]
⇔ |z| w − |w|2 z − z + w = 0
2 z1 + z2
⇔ (|z|2 + 1)w − (|w|2 + 1)z = 0 ∴ z1Rz1 is reflexive.
z − z2
⇔ (|z| + 1)w = (|w| + 1)z
2 2 (ii) Symmetric z1Rz2 ⇔ 1 is real
z1 + z2
z |z|2 + 1
⇔ = − (z2 − z1 )
w |w|2 + 1 ⇒ is real ⇒ z2Rz1

z
is purely real. z1 + z2
w
z z ∴ z1Rz2 ⇒ z2Rz1
⇔ = ⇒ zw = zw …(i)
w w Therefore, it is symmetric.
rankershalt.com

Complex Numbers 17

(iii) Transitive z1Rz2 a + ib


47. Since, (x + iy)2 =
z1 − z2 c + id
⇒ is real
z1 + z2 |a + ib|  z1 |z1|
⇒ |x + iy|2 = Q z = |z |
and z2Rz3 |c + id|  2 2
z2 − z3
⇒ is real a 2 + b2
z2 + z3 ⇒ (x2 + y2) =
c2 + d 2
Here, let z1 = x1 + iy1 , z2 = x2 + iy2 and z3 = x3 + iy3
a 2 + b2
z − z2 (x − x2) + i ( y1 − y2) ⇒ (x2 + y2)2 =
∴ 1 is real ⇒ 1 is real c2 + d 2 Hence proved.
z1 + z2 (x1 + x2) + i ( y1 + y2)
{(x1 − x2) + i ( y1 − y2)}{(x1 + x2) − i ( y1 + y2)} 48. Given,|z − 3 − 2 i| ≤ 2 …(i)

(x1 + x2)2 + ( y1 + y2)2 To find minimum of|2z − 6 + 5 i|
5
⇒ ( y1 − y2) (x1 + x2) − (x1 − x2) ( y1 + y2) = 0 or 2 z − 3 + i , using triangle inequality
2
⇒ 2 x2y1 − 2 y2x1 = 0
x1 x2 i.e. ||z1| − |z2|| ≤ |z1 + z2|
⇒ = ... (i) 5 5
y1 y2 ∴ z −3 + i = z −3 −2i + 2i + i
2 2
Similarly, z2Rz3
9

x2 x3
= ... (ii) = (z − 3 − 2 i ) + i
y2 y3 2
9 9 5
From Eqs. (i) and (ii), we have
x1 x3
= ⇒ z1Rz3 ≥ |z − 3 − 2 i| − ≥ 2− ≥
y1 y3 2 2 2
5 5
Thus, z1Rz2 and z2Rz3 ⇒ z1Rz3 . [transitive] ⇒ z −3 + i ≥ or|2z − 6 + 5 i| ≥ 5
2 2
Hence, R is an equivalence relation.

(1 + i ) x − 2i (2 − 3i ) y + i Topic 3 Argument of a Complex Number


45. + =i |z1| 4
3+ i 3−i 1. (*) Given, 3|z1| = 4|z2|⇒ = [Q z2 ≠ 0 ⇒|z2| ≠ 0]
|z2| 3
⇒ (1 + i ) (3 − i ) x − 2i (3 − i ) + (3 + i ) (2 − 3i ) y
+ i (3 + i ) = 10i z1 z1 iθ z z
∴ = e and 2 = 2 e−iθ
⇒ 4x + 2ix − 6i − 2 + 9 y − 7iy + 3i − 1 = 10i z2 z2 z1 z1
⇒ 4x + 9 y − 3 = 0 and 2x − 7 y − 3 = 10 [Q z =|z|(cos θ + i sin θ) = |z| eiθ ]
z1 4 z 3
⇒ x = 3 and y = − 1 ⇒ = eiθ and 2 = e−iθ
1 1 z2 3 z1 4
46. Now, =
(1 − cos θ ) + 2i sin θ 2 sin 2 θ + 4i sin θ cos θ ⇒
3 z1
= 2eiθ and
2 z2 1 −iθ
= e
2 2 2 2 z2 3 z1 2
θ θ On adding these two, we get
sin − 2 i cos
1 2 2 3 z1 2 z2 1
= × z= + = 2eiθ + e−iθ
θ θ θ  θ θ 2 z2 3 z1 2
2 sin sin + 2 i cos  sin − 2 i cos 
2  2 2   2 2
θ θ 1 1
sin − 2 i cos = 2 cos θ + 2i sin θ + cos θ − i sin θ
= 2 2 2 2
θ θ θ [Q e± iθ = (cos θ ± i sin θ)]
2 sin sin 2 + 4 cos 2 
2 2 2 5 3
= cos θ + i sin θ
θ θ 2 2
sin − 2 i cos
= 2 2  5
2
 3 34
2
17
θ θ ⇒ |z| =   +   = =
2 sin 1 + 3 cos 2   2  2 4 2
2 2
Note that z is neither purely imaginary and nor purely
θ real.
cot
1 2
⇒ A + iB = −i ‘*’ None of the options is correct.
 θ θ
2 1 + 3 cos 2  1 + 3 cos 2
 2 2
rankershalt.com

18 Complex Numbers

2. Given,|z| = 1 , arg z = θ∴z = eiθ Similarly, when |z − i w| = 2 , then


z
is purely
1 w
∴ z = e–iθ ⇒ z = imaginary
z
Now, given relation
 
 1 + z  1 + z |z + iw| = |z − iw| = 2
∴ arg   = arg   = arg (z ) = θ Put w = i, we get
1 + z 1 + 1
 z |z + i 2| = |z + i 2| = 2
⇒ |z − 1| = 2
3. Since, arg (z ) < 0 ⇒ arg (z ) = − θ
⇒ z = −1 [Q|z| ≤ 1]
Y
Put w = − i , we get
(–z) |z − i 2| = |z − i 2| = 2
r π−θ
⇒ |z + 1| = 2 ⇒ z = 1 [Q|z| ≤ 1]
X′ X
O –θ ∴ z = 1 or − 1 is the correct option.
r
5. Since,|z|=|w|and arg (z ) = π − arg (w)
(z)
Y′ Let w = re iθ , then w = re–iθ
⇒ z = r cos (−θ ) + i sin (−θ ) ∴ z = rei ( π − θ ) = re iπ ⋅ e−iθ = − re−iθ = − w
= r (cos θ − i sin θ ) 6. Given,|z1 + z2| = |z1| + |z2|
and − z = − r [cos θ − i sin θ ] On squaring both sides, we get
= r [cos (π − θ ) + i sin (π − θ )] |z1|2 + |z2|2 + 2|z1||z2| cos (arg z1 − arg z2)
∴ arg (− z ) = π − θ = |z1|2 + |z2|2 + 2|z1||z2|
Thus, arg (− z ) − arg (z ) ⇒ 2|z1||z2|cos (arg z1 − arg z2) = 2|z1||z2|
= π − θ − (− θ ) = π ⇒ cos (arg z1 − arg z2) = 1
Alternate Solution ⇒ arg (z1 ) − arg (z2) = 0
 − z
Reason arg (− z ) − arg z = arg   = arg (− 1) = π 7. Since a , b, c and u , v, w are the vertices of two triangles.
 z 
Also, c = (1 − r ) a + rb
 z 
and also arg z − arg (− z ) = arg   = arg (− 1) = π and w = (1 − r ) u + rv …(i)
 − z
 a u 1
4. Given, |z + iw| = |z − iw |= 2 Consider  b v 1 
c w 1 
⇒ |z − (− iw)| = |z − (iw )| = 2
Applying R3 → R3 – {(1 − r ) R1 + rR2}
⇒ |z − (− iw)| = |z − (− iw )| a u 1
∴ z lies on the perpendicular bisector of the line joining = b v 1
− iw and − iw. Since, − iw is the mirror image of − iw in c − (1 − r ) a − rb w − (1 − r ) u − rv 1 − (1 − r ) − r
the X-axis, the locus of z is the X-axis.
a u 1
Let z = x + iy and y = 0. = b v 1 = 0 [from Eq. (i)]
Now, |z | ≤ 1 ⇒ x2 + 02 ≤ 1 ⇒ −1 ≤ x ≤ 1.
0 0 0 
∴ z may take values given in option (c).
8. (a) Let z = − 1 − i and arg(z) = θ
Alternate Solution
|z + i w| ≤ |z| + |iw|= |z| + |w| Now, tan θ =  =  − 1  = 1
 im (z ) 
 Re(z )   −1 
≤1 +1 =2
π
∴ |z + iw| ≤ 2 ⇒ θ=
4
⇒ |z + i w| = 2 holds when
Since, x < 0, y < 0
arg z − arg i w = 0  π 3π
∴ arg (z ) = −  π −  = −
⇒ arg
z
=0  4 4
iw
(b) We have, f (t ) = arg (−1 + it )
z  π − tan −1 t , t ≥ 0
⇒ is purely real.
iw arg (−1 + it ) =  −1
z − (π + tan t ), t < 0
⇒ is purely imaginary. This function is discontinuous at t = 0.
w
rankershalt.com

Complex Numbers 19

(c) We have, 10. Let z = a + ib.


z  Given, Re(z ) = 0 ⇒ a = 0
arg  1  − arg (z1 ) + arg (z2)
 z2  Then, z = ib ⇒ z 2 = − b2 or lm (z 2) = 0
z  Therefore, A → q
Now, arg  1  = arg (z1 ) − arg (z2) + 2nπ π
 z2  Also, given, arg (z ) = .
4
z   π π
∴ arg  1  − arg (z1 ) + arg (z2) Let z = r  cos + i sin 
 z2   4 4

= arg (z1 ) − arg (z2) + 2nπ − arg (z1 ) + arg (z2) π π π π



= 2nπ Then, z 2 = r 2 cos 2 − sin 2  + 2 ir 2 cos sin
 4 4 4 4
So, given expression is multiple of 2π. = ir 2 sin π /2 = ir 2
 (z − z1 ) (z2 − z3 ) Therefore, Re (z 2) = 0 ⇒ B → p.
(d) We have, arg   =π
 (z − z3 ) (z2 − z1 ) ⇒ a = b =2− 3 [Q a , b ← (0, 1)]
 z − z1   z2 − z3  11. Let z = r1 (cos θ1 + i sin θ1 ) and w = r2(cos θ 2 + i sin θ 2)
⇒   is purely real
 z − z3   z2 − z1  We have,|z| = r1 ,|w| = r2, arg (z ) = θ1 and arg (w) = θ 2
Given,|z| ≤ 1,|w| < 1
Thus, the points A (z1 ), B(z2), C (z3 ) and D (z ) taken in
⇒ r1 ≤ 1 and r2 ≤ 1
order would be concyclic if purely real.
Hence, it is a circle.
Now,
C(z3)
z − w = (r1 cos θ1 − r2 cos θ 2) + i (r1 sin θ1 − r2 sin θ 2)
⇒|z − w|2 = (r1 cos θ1 − r2 cos θ 2)2
+ (r1 sin θ1 − r2 sin θ 2)2
D(z)
B(z2) = r12 cos 2 θ1 + r22 cos 2 θ 2 − 2r1r2 cos θ1 cos θ 2
+ r12 sin 2 θ1 + r22 sin 2 θ 2 − 2r1r2 sin θ1 sin θ 2
= r12(cos 2 θ1 + sin 2 θ1 ) + r22(cos 2 θ 2 + sin 2 θ 2)
A(z1)
−2r1r2(cos θ1 cos θ 2 + sin θ1 sin θ 2)
∴(a), (b), (d) are false statement.
(1 − t ) z1 + t z2 = r12 + r22 − 2r1r2 cos (θ1 − θ 2)
9. Given, z = = (r1 − r2)2 + 2r1r2[1 − cos (θ1 − θ 2)]
(1 − t ) + t
A P B  θ − θ 2
= (r1 − r2)2 + 4r1r2 sin 2  1 
z1 z z2  2 
t : (1 - t)
2
  θ − θ 2 

Clearly, z divides z1 and z2 in the ratio of t : (1 − t ), ≤ |r1 − r2|2 + 4sin  1  [Q r1 , r2 ≤ 1]
0 < t <1   2 
⇒ AP + BP = AB i.e. |z − z1|+ |z − z2|=|z1 − z2| and |sin θ| ≤ |θ|, ∀ θ ∈ R 2
θ − θ 2
⇒ Option (a) is true. Therefore, |z − w |2 ≤ |r1 − r2|2 + 4 1
and arg (z − z1 ) = arg (z2 − z )  2 
= arg (z2 − z1 ) ≤ |r1 − r2|2 + |θ1 − θ 2|2
⇒ Option (b) is false and option (d) is true. ⇒ |z − w|2 ≤ (|z| − |w|)2 + (arg z − arg w)2
Also, arg (z − z1 ) = arg (z2 − z1 ) Alternate Solution
 z − z1  |z − w|2 = |z|2 + |w|2 − 2|z||w|cos (arg z − arg w)
⇒ arg   =0
 z2 − z1  = |z|2 + |w|2 − 2|z||w| + 2|z||w|
z − z1
∴ is purely real. − 2|z||w|cos (arg z − arg w)
z2 − z1
 arg z − arg w
z − z1 z − z1 = (|z| − |w|) + 2|z||w|⋅ 2 sin 2
2
 …(i)
⇒ =  2 
z2 − z1 z2 − z1 2
 arg z − arg w
z − z1 z − z1 ∴ |z − w|2 ≤ (|z| − |w|)2 + 4 ⋅ 1⋅ 1  
or =0  2 
z2 − z1 z2 − z1
[Q sin θ ≤ θ ]
Option (c) is correct.
⇒ |z − w| ≤ (|z| − |w|) + (arg z − arg w)
2 2 2
rankershalt.com

20 Complex Numbers

12. Since, z1 = 10 + 6i , z2 = 4 + 6i 3. Let OA = 3,so that the complex P Y

 z − z1  π number associated with A is


and arg   = represents locus of z is a circle 3e iπ / 4 . If z is the complex 4
 z − z2  4 3e iπ/4
number associated with P, A
shown as from the figure whose centre is (7, y) and then
∠ AOB = 90°, clearly OC = 9 ⇒ OD = 6 + 3 = 9 3
z − 3eiπ / 4 4 − iπ / 2 4i
∴ Centre = (7, 9) and radius =
6
=3 2 = e =− π/4
2 0 − 3eiπ / 4 3 3 X′ X
O
z ⇒ 3z − 9eiπ / 4 = 12 ieiπ / 4
C Y′
z = (3 + 4 i ) eiπ / 4
Y

°
45

O
(7, y) 4. Since, |PQ | = |PS | = |PR| = 2
6 (4, 6) A B (10, 6) ∴ Shaded part represents the external part of circle
z1 z2
having centre (−1, 0) and radius 2.
As we know equation of circle having centre z0 and
X radius r, is|z − z0| = r
D (7, 0) (10, 0)
(4, 0)
∴ |z − (−1 + 0i )| > 2
⇒ |z + 1| > 2
⇒ Equation of circle is| z − (7 + 9i )| = 3 2 Also, argument of z + 1 with respect to positive direction
of X-axis is π/4.
Topic 4 Rotation of a Complex Number π
5 5 ∴ arg (z + 1) ≤ …(i)
 3 i  3 i 4
1. Given, z =  +  + − 
 2 2  2 2 and argument of z + 1 in anticlockwise direction is −π /4.
Q Euler’s form of ∴ − π / 4 ≤ arg (z + 1) …(ii)
3 i  π π i( π / 6 )
From Eqs. (i) and (ii),
+ =  cos + i sin  =e |arg (z + 1)|≤ π / 4
2 2  6 6
3 i  −π   π 5. In the Argand plane, P is represented by e i0 and Q is
and − = cos   + i sin  −  = e−iπ / 6 represented by e i( α − θ )
2 2  6  6
5π 5π Now, rotation about a line with angle α is given by
−i
So, z = (eiπ / 6 )5 + (e−iπ / 6 )5 = e 6 + e 6
i
e θ → e (α − θ ). Therefore, Q is obtained from P by reflection
in the line making an angle α /2.
 5π 5π   5π 5π 
=  cos + i sin  +  cos − i sin 
 6 6  6 6 z1 − z3 1 − i 3 (1 − i 3 )(1 + i 3 ) z2
6. = =
[Q eiθ = cos θ + i sin θ] z2 − z3 2 2 (1 + i 3 )

= 2 cos 1 − i 23
6 =
π 2 (1 + i 3 ) π/3
∴ I (z ) = 0 and R(z ) = −2 cos = − 3 <0 4 z3 z1
6 =
 5π  π π 2 (1 + i 3 )
Q cos 6 = cos  π − 6  = − cos 6  =
2
 
(1 + i 3 )
2. Imaginary axis z2 − z3 1 + i 3 π π
⇒ = = cos + i sin
z1 − z3 2 3 3
z
2 − z3  z − z  π
z2 z'2 (7,6) ⇒ = 1 and arg  2 3
 =
1
z − z3  z1 − z3  3
) 1
,2 Hence, the triangle is an equilateral.
(1
90° z 0 3 1 Alternate Solution
(6,2)
5 z1 − z3 1 − i 3
Real axis ∴ =
z2 − z3 2
z2 − z3 2 1 +i 3 π π
z2′ = (6 + 2 cos 45° , 5 + 2 sin 45° ) = (7, 6) = 7 + 6i ⇒ = = = cos + i sin
By rotation about (0, 0), z1 − z3 1 − i 3 2 3 3
 iπ   z2 − z3  π z2 − z3
z2
= ei π/ 2 ⇒ z2 = z2′  e 2  ⇒ arg   = and also =1
z2′    z1 − z3  3 z1 − z3
 
 π π Therefore, triangle is equilateral.
= (7 + 6i )  cos + i sin  = (7 + 6i ) (i ) = − 6 + 7i
 2 2
rankershalt.com

Complex Numbers 21

1 a − ibt 5π 5π cos 7π
7. Here, x + iy = × ∴ z2 = cos π + i sin π , cos + i sin ,
a + ibt a − ibt 6 6 6
a − ibt 7π
∴ x + iy = 2 + i sin
a + b2t 2 6
− 3 i − 3 i
Let a ≠ 0, b ≠ 0 ⇒ z2 = − 1 , + , −
a − bt 2 2 2 2
∴ x= 2 and y = 2 2π 5π
a +bt 2 2
a + b2t 2 Thus, ∠z1Oz2 = , ,π
3 6
y − bt ay
⇒ = ⇒t = 9. z1 = 1 + i 3 = r (cos θ + i sin θ ) [let]
x a bx
a ⇒ r cos θ = 1, r sin θ = 3
On putting x = 2 , we get
a + b2t 2 ⇒ r = 2 and θ = π /3
So, z1 = 2 (cos π / 3 + sin π / 3)
 a 2 y2 
x  a 2 + b2 ⋅ 2 2  = a ⇒ a 2(x2 + y2) = ax Since, |z2| = |z3| = 2 [given]
 bx 
Y-axis
x
or x2 + y2 − =0 … (i)
a
2 z1
 1 1
x −  + y =
2
or
 2a  4a 2 z2
P (2, 0)
∴Option (a) is correct. X-axis
P (–1, 0) O
For a ≠ 0 and b = 0,
1 1
x + iy = ⇒ x = , y = 0
a a
z3
⇒ z lies on X-axis.
Now, the triangle z1 , z2 and z3 being an equilateral and
∴ Option (c) is correct.
the sides z1z2 and z1z3 make an angle 2π / 3 at the centre.
1 1
For a = 0 and b ≠ 0, x + iy = ⇒ x = 0, y = − π 2π
ibt bt Therefore, ∠POz2 = + =π
3 3
⇒ z lies on Y-axis. π 2π 2π 5π
and ∠POz3 = + + =
∴ Option (d) is correct. 3 3 3 3
8. PLAN It is the simple representation of points on Argand plane and Therefore, z2 = 2 (cos π + i sin π ) = 2 (− 1 + 0) = − 2
to find the angle between the points.  5π 5π  1 3
 π π nπ
n
nπ and z3 = 2  cos + i sin  =2  − i  =1 − i 3
Here, P = W =  cos + i sin  = cos
n
+ i sin  3 3   2 2 
 6 6 6 6
 1
H 1 = z ∈ C : Re(z ) >  Alternate Solution
 2 Whenever vertices of an equilateral triangle having
nπ centroid is given its vertices are of the form z , zω , zω 2.
∴ P ∩ H 1 represents those points for which cos is + ve.
6 ∴ If one of the vertex is z1 = 1 + i 3 , then other two
Hence, it belongs to I or IV quadrant. vertices are (z1ω ), (z1ω 2).
π π 11π 11π (−1 + i 3 ) (−1 − i 3 )
⇒ z1 = P ∩ H 1 = cos + i sin or cos + i sin ⇒ (1 + i 3 ) , (1 + i 3 )
6 6 6 6 2 2
− (1 + 3) (1 + i 2( 3 )2 + 2i 3 )
∴ z1 =
3 i
+ or
3 i
− …(i) ⇒ ,−
2 2 2 2 2 2
(−2 + 2i 3 )
Similarly, z2 = P ∩ H 2 i.e. those points for which ⇒ −2 , − =1 − i 3
2

cos <0 ∴ z2 = − 2 and z3 = 1 − i 3
6
–√3 , 1
10. Given, D = (1 + i ), M = (2 − i )
—– — Z2 √3 , 1
Z1 —–
2 2 — and diagonals of a rhombus bisect each other.
2 2
Let B ≡ (a + ib), therefore
(–1, 0) Z2
a+1 b+1
O = 2, = −1
–√3 , –1 Z √3 –1 2 2
Z1 —– , —
—– — 2
2 2 2 2 ⇒ a + 1 = 4, b + 1 = − 2 ⇒ a = 3, b = − 3
⇒ B ≡ (3 − 3i )
rankershalt.com

22 Complex Numbers

A D (1+i ) 1 1
⇒ z3 = (2 − i ) ± i (2i − 1) = (2 − i ) ± (−2 − i )
2 2
M (4 − 2i − 2 − i ) 4 − 2i + 2 + i 3 i
= , = 1 − i, 3 −
(2−i ) 2 2 2 2
 3   i
∴ A is either 1 − i or 3 −  .
 2   2
B C

Again, DM = (2 − 1)2 + (− 1 − 1)2 = 1 + 4 = 5


11. Since, z1 , z2 and z3 form an equilateral triangle.
But BD = 2DM ⇒ BD = 2 5 ⇒ z12 + z22 + z32 = z1z2 + z2z3 + z3 z1
and 2 AC = BD ⇒ 2 AC = 2 5 ⇒ (a + i ) + (1 + ib)2 + (0)2 = (a + i ) (1 + ib) + 0 + 0
2

⇒ AC = 5 and AC = 2 AM ⇒ a 2 − 1 + 2ia + 1 − b2 + 2ib = a + i (ab + 1) − b


5 ⇒ (a 2 − b2) + 2i (a + b) = (a − b) + i (ab + 1)
⇒ 5 = 2 AM ⇒ AM =
2 ⇒ a 2 − b2 = a − b
Now, let coordinate of A be (x + iy).
and 2 (a + b) = ab + 1
But in a rhombus AD = AB, therefore we have
⇒ (a = b or a + b = 1)
AD 2 = AB2
⇒ (x − 1)2 + ( y − 1)2 = (x − 3)2 + ( y + 3)2 and 2 (a + b) = ab + 1
⇒ x2 + 1 − 2x + y2 + 1 − 2 y = x2 + 9 − 6x+ y2 + 9 + 6 y If a = b , 2 (2a ) = a 2 + 1
⇒ 4x − 8 y = 16 ⇒ a − 4a + 1 = 0
2

⇒ x − 2y = 4 4 ± 16 − 4
⇒ a= =2 ± 3
⇒ x = 2y + 4 …(i) 2
5 5 If a + b = 1, 2 = a (1 − a ) + 1 ⇒ a − a + 1 = 0
2
Again, AM = ⇒ AM 2 =
2 4 1 ± 1 −4
5 ⇒a= , but a and b ∈ R
⇒ (x − 2)2 + ( y + 1)2 = 2
4 ∴ Only solution when a=b
⇒ (2 y + 2) + ( y + 1) =
2 2 5
[from Eq. (i)] ⇒ a = b =2± 3
4 ⇒ a = b =2− 3 [Q a , b ∈ (0, 1)]
5 12. Here, centre of circle is (1, 0) is also the mid-point of
⇒ 5 y2 + 10 y + 5 =
4 diagonals of square
⇒ 20 y2 + 40 y + 15 = 0 Y
⇒ 4 y2 + 8 y + 3 = 0
⇒ (2 y + 1) (2 y + 3) = 0 z3
⇒ 2 y + 1 = 0,2 y + 3 = 0 z1(2, 3)
1 3
⇒ y=− , y=−
2 2 (1, 0)
X
On putting these values in Eq. (i), we get O z
0
 1  3
x = 2  −  + 4, x = 2  −  + 4
 2  2 z2 z4
⇒ x = 3, x = 1
 i  3i 
Therefore, A is either 3 −  or 1 −  .
 2   2 z1 + z2
⇒ = z0
Alternate Solution 2
Since, M is the centre of rhombus. ⇒ z2 = − 3 i [where, z0 = 1 + 0 i ]
∴ By rotating D about M through an angle of ± π /2 , we z3 − 1
get possible position of A. and = e± iπ/ 2
z1 − 1
C B
 π π
⇒ z3 = 1 + (1 + 3i ) ⋅  cos ± i sin  [Q z1 = 2 + 3i ]
 2 2
= 1 ± i (1 + 3i ) = (1 + 3 ) ± i = (1 − 3 ) + i
M
(2– i) z2 and z4 = (1 + 3 ) − i

D z1(1+i ) A (z3) 13. Let Q be z2 and its reflection be the point P (z1 ) in the
given line. If O (z ) be any point on the given line then by
z3 − (2 − i ) 1 z3 − (2 − i ) 1 definition OR is right bisector of QP.
⇒ = (± i ) ⇒ = (± i )
−1 + 2 i 2 −1 + 2 i 2 ∴ OP = OQ or |z − z1| = |z − z2|
rankershalt.com

Complex Numbers 23

⇒ |z − z1|2 = |z − z2|2 ⇒ (z1 − z2)2 = 2{(z1z3 − z32 ) + (z2z3 − z1z2)}


⇒ (z − z1 ) (z − z1 ) = (z − z2) (z − z2) ⇒ (z1 − z2)2 = 2(z1 − z3 )(z3 − z2)
⇒ z (z1 − z2) + z (z1 − z2) = z1z1 − z2z2 16. We have, iz = zei π/2. This implies that iz is the vector
Comparing with given line zb + zb = c obtained by rotating vector z in anti-clockwise direction
z1 − z2 z1 − z2 z1z1 − z2z2 through 90°. Therefore, OA ⊥ AB. So,
= = = λ, [say]
b b c Y
B
z1 − z2 z − z2 z z − z2z2

z + iz
= b, 1 = b, 1 1 =c …(i) iz
λ λ λ
 z − z2   z − z2  A
∴ z1b + z2b = z1  1  + z2  1  z
 λ   λ  X′
O
X

zz − z2z2
= 1 =c [from Eq. (i)]
λ
Y′
14. Since, z1 + z2 = − p and z1z2 = q B (z2)
1 1 1
z1 |z1| Area of ∆ OAB = OA × OB = |z ||iz | = |z |2
Now, = (cos α + i sin α ) 2 2 2
z2 |z2|
z1 cos α + i sin α
⇒ = A (z1) 17. Since, z1 , z2 and origin form an equilateral triangle.
z2 1 O
[Q|z1|=|z2|] Q if z1 , z2, z3 from an equilateral triangle, then 
 
Applying componendo and dividendo, we get z12 + z22 + z32 = z1z2 + z2z3 + z3 z1 
z1 + z2 cos α + i sin α + 1
= ⇒ z12 + z22 + 02 = z1z2 + z2 ⋅ 0 + 0 ⋅ z1
z1 − z2 cos α + i sin α − 1
2 cos 2(α /2) + 2i sin (α /2) cos (α /2) ⇒ z12 + z22 = z1z2
=
−2 sin 2(α /2) + 2i sin (α /2) cos (α /2) ⇒ z12 + z22 − z1z2 = 0
2 cos (α /2) [cos (α /2) + i sin (α /2)] 18. Since, z1 , z2, z3 are the vertices of an equilateral
=
2i sin (α /2)[cos (α /2) + i sin (α /2)] triangle.
cot (α /2) −p  z + z2 + z3 
= = − i cot α /2 ⇒ = − i cot (α /2) ∴ Circumcentre (z0 ) = Centroid  1  ...(i)
i z1 − z2  3 
p2 Also, for equilateral triangle
On squaring both sides, we get = − cot2(α /2)
(z1 − z2)2 z12 + z22 + z32 = z1z2 + z2z3 + z3 z1 ... (ii)
2 On squaring Eq. (i), we get
p
⇒ = − cot2(α /2)
(z1 + z2)2 − 4z1z2 9z02 = z12 + z22 + z32 + 2 (z1z2 + z2z3 + z3 z1 )
p2 ⇒ 9z02 = z12 + z22 + z32 + 2 (z12 + z22 + z32 ) [from Eq. (ii)]
⇒ = − cot2(α /2)
p − 4q
2
⇒ 3z02 = z12 + z22 + z32
⇒ p2 = − p2 cot2(α /2) + 4q cot2(α /2) kπ   kπ 
⇒ p (1 + cot α /2) = 4q cot2(α /2)
2 2 19. Given, α k = cos   + i sin  
 7  7
⇒ p2 cosec2(α /2) = 4q cot2(α /2)
 2 kπ   2 kπ 
⇒ p2 = 4q cos 2 α /2 = cos   + i sin  
 14   14 
15. Since, triangle is a right angled isosceles triangle. ∴ α k are vertices of regular polygon having 14 sides.
∴ Rotating z2 about z3 in anti-clockwise direction
Let the side length of regular polygon be a.
through an angle of π / 2 , we get
z2 − z 3 | z 2 − z 3 | iπ / 2 A (z1) ∴ α k + 1 − α k = length of a side of the regular polygon
= e
z1 − z3 | z1 − z 3 | =a …(i)
where,|z2 − z3|= |z1 − z3| and α 4k−1 − α 4k− 2 = length of a side of the regular
⇒ (z2 − z3 ) = i (z1 − z3 ) B (z3) C (z2) polygon
=a …(ii)
12
On squaring both sides, we get
(z2 − z3 ) = − (z1 − z3 )
2 2 ∑ αk+1 −αk
k =1 12 (a )
∴ = =4
⇒ z22 + z32 − 2z2z3 = − z12 − z32 + 2z1z3 3
3 (a )
⇒ z12 + z22 − 2z1z2 = 2z1z3 + 2z2z3 − 2z32 − 2z1z2 ∑ α 4k−1 − α 4k− 2
k =1
rankershalt.com

24 Complex Numbers

Topic 5 De-Moivre’s Theorem, Cube Roots 3. Given, x2 + x + 1 = 0


and nth Roots of Unity − 1 ± 3i
⇒ x=
1. It is given that, there are two complex numbers z and w, 2
such that| z w| = 1 and arg (z ) − arg (w) = π / 2 [Q Roots of quadratic equation ax2 + bx + c = 0
∴ |z || w| = 1 [Q| z1 z2| = | z1 || z2|] −b± b2 − 4ac
π are given by x = ]
and arg (z ) = + arg (w) 2a
2 −1 + 3 i
1 ⇒ z0 = ω , ω 2 [where ω = and
Let| z | = r, then| w| = …(i) 2
r −1 − 3 i
π ω2 =
and let arg(w) = θ, then arg(z ) = + θ …(ii) 2
2 are the cube roots of unity and ω, ω 2 ≠ 1)
So, we can assume
Now consider z = 3 + 6i z081 − 3i z093
z = rei ( π / 2 + θ ) …(iii)
= 3 + 6i − 3i (Qω3 n = (ω 2)3 n = 1)
[Q if z = x + iy is a complex number, then it can be
written as z = reiθ where, r =|z|and θ = arg (z )] = 3 + 3i = 3(1 + i )
1 If ‘θ’ is the argument of z, then
and w = ei θ …(iv) Im(z )
r tan θ = [Qz is in the first quadrant]
1 Re(z )
Now, z ⋅ w = re− i( π / 2 + θ ) ⋅ eiθ 3 π
r = =1⇒ θ=
3 4
= ei( − π / 2 − θ + θ ) = e− i( π / 2) = − i [Qe− i θ = cos θ − i sin θ]
1 4. Given that, z = cos θ + i sin θ = e iθ
and z w = re i( π / 2 + θ ) ⋅ e− iθ 15 15 15
r
= ei( π / 2 + θ − θ ) = e i ( π / 2) = i
∴ ∑ Ιµ(ζ 2 µ −1 ) = ∑ Ιµ( ε ιθ )2 µ −1 = ∑ Ιµ ε ι ( 2 µ −1 ) θ
µ =1 µ =1 µ =1

Key Idea Use, e i θ = cos θ + i sin θ


= sin θ + sin 3 θ + sin 5 θ + ... + sin 29 θ
2.  θ + 29 θ   15 × 2 θ 
π sin   sin  
3  1 π π i  2   2 
Given, z = +   i = cos + i sin = e 6 =
2  2  6 6  2 θ
sin  
 2 
so, (1 + iz + z5 + iz 8 )9
sin (15 θ )sin (15 θ ) 1
 π 5π 8π
i 
9
= =
i i
= 1 + ie 6 + e 6 + ie 6  sin θ 4 sin 2°
 
  5. Let z = |a + bω + cω 2|
π π 5π π 4π 9 π
 i i i i i   i  ⇒ z 2 = |a + bω + cω 2|2 = (a 2 + b2 + c2 − ab − bc − ca )
= 1 + e 2 ⋅ e 6 + e 6 + e 2 ⋅ e 3  Q i = e 2 
  1
    or z 2 = {(a − b)2 + (b − c)2 + (c − a )2} …(i)
9 2
 2π 5π 11 π 
i i i
= 1 + e 3 + e 6 + e 6  Since, a , b, c are all integers but not all simultaneously
 
  equal.
⇒ If a = b then a ≠ c and b ≠ c
  2π 2π   5π 5π 
= 1 +  cos + i sin  +  cos + i sin  Because difference of integers = integer
  3 3   6 6
⇒ (b − c)2 ≥ 1 {as minimum difference of two consecutive
 11π 11π  
9
integers is (± 1)} also (c − a )2 ≥ 1
+  cos + i sin 
 6 6   and we have taken a = b ⇒ (a − b)2 = 0
9 1
 1 i 3 3 1 3 i From Eq. (i), z 2 ≥ (0 + 1 + 1)
= 1 − + − + i+ −  2
 2 2 2 2 2 2
9
⇒ z2 ≥ 1
1 3i   π π
9
= +  =  cos + i sin  Hence, minimum value of|z | is 1 .
2 2   3 3
6. Given, (1 + ω 2)n = (1 + ω 4 )n
= cos 3π + i sin 3π [Q for any natural number ‘n’
⇒ (− ω )n = (− ω 2)n [Qω3 = 1 and 1 + ω + ω 2 = 0]
(cos θ + i sin θ )n = cos(nθ ) + i sin(nθ )]
⇒ ω =1
n

= −1 ⇒ n = 3 is the least positive value of n.


rankershalt.com

Complex Numbers 25

1 1 1 11. (1 + ω )7 = (1 + ω ) (1 + ω )6
7. Let ∆ = 1 −1 − ω ω 2
2
= (1 + ω ) (−ω 2)6 = 1 + ω
1 ω2 ω ⇒ A + Bω = 1 + ω
Applying R2 → R2 − R1 ; R3 → R3 − R1 ⇒ A = 1, B = 1
6
1 1 1   2 kπ 2 kπ 

=  0 −2 − ω 2
ω −1 
2 12. ∑ sin 7
− i cos
7 

 k =1
 0 ω −1
2 ω −1  6
 2 kπ 2 kπ 
= (−2 − ω )(ω − 1) − (ω − 1)
2 2 2 = ∑ − i  cos 7
+ i sin
7 

k =1
= − 2ω + 2 − ω3 + ω 2 − (ω 4 − 2ω 2 + 1)
 6 i2kπ 
= 3 ω 2 − 3 ω = 3ω (ω − 1) [Q ω 4 = ω ] = − i  ∑ e 7  = − i { ei2π / 7 + e i4 π / 7 + e i6π /7
z π k = 1 
8. Since, arg 1 = + e i8π / 7 + e i10π / 7 + e i12π / 7 }
z2 2
z1 π π  (1− ei12 π / 7 )
⇒ = cos + i sin = i = − i  ei 2 π / 7 
z2 2 2  1− ei 2 π / 7 
z1n
∴ = (i )n ⇒ i n = 1 [Q|z2| = |z1| = 1]  ei2π / 7 − ei14π / 7 
z2n = −i i2π / 7  [Q e i14π /7= 1]
 1−e 
⇒ n = 4k
 e i2π / 7 − 1 
Alternate Solution = −i  i2π / 7 
=i
z π
B (z2)
1 − e 
Since, arg 2 =
z1 2 13. (P) PLAN e iθ⋅ e iα
=e i( θ + α )

π
z2 z2 i 2 π 2 kπ  2π 
i  ( k + j)
∴ = e 2 Given zk = e
i
10 ⇒ zk ⋅ z j = e  10 
z1 z1 O A (z1)
z2 zk is 10th root of unity.
⇒ =i [Q|z1| = |z2| = 1]
z1 ⇒ zk will also be 10th root of unity.
 z2 
n
Taking, z j as zk, we have zk ⋅ z j = 1 (True)
⇒   =i
n
 z1  e iθ
(Q) PLAN = e i( θ − α )
e iα i 
2kπ

2π  π
∴ z1 and z2 are nth roots of unity. z = zk / z1 = e  10

10 
=e
i
5
( k − 1)

z1n = z2n = 1 π
n
 z2  For k = 2; z = e
i
5 which is in the given set (False)
⇒   =1
 z1 
(R) PLAN
⇒ in = 1 (i)1 − cos 2 θ = 2 sin2 θ
⇒ n = 4k, where k is an integer. (ii)sin 2 θ = 2 sin θ cos θ and
9. We know that, 5 −1
(i)cos 36° =
1 3 4
ω=− + i
2 2 5 + 1| 1 − z1 || 1 − z2 | K | 1 − z9 |
(ii)cos 108° =
334 365 4 10
 1 i 3  1 i 3
∴ 4 + 5 − +  + 3 − +   2 πk 2 πk 
NOTE | 1 − zk | = 1 − cos − i sin
 2 2   2 2   10 10 
= 4 + 5 ω334 + 3 ω365  πk  πk πk  πk
= 2 sin sin − i cos = 2 sin
 10  10 10  10
= 4 + 5 ⋅ (ω3 )111 ⋅ ω + 3 ⋅ (ω3 )121 ⋅ ω 2
Now, required product is
= 4 + 5 ω + 3 ω2 [Q ω3 = 1] π 2π 3π 8π 9π
29 sin ⋅ sin ⋅ sin K sin ⋅ sin
= 1 + 3 + 2 ω + 3 ω + 3 ω2 10 10 10 10 10
= 1 + 2 ω + 3 (1 + ω + ω 2) = 1 + 2 ω + 3 × 0 10
2
[Q 1 + ω + ω 2 = 0]  π 2π 3π 4π  5π
29 sin sin sin sin  sin
= 1 + (−1 + 3i ) = 3i  10 10 10 10  10
=
10. (1 + ω − ω 2)7 = (− ω 2 − ω 2)7 [Q 1 + ω + ω 2 = 0] 10
2
 π π 2π 2π 
= (−2ω 2)7 = (−2)7ω14= − 128 ω 2 29 sin cos ⋅ sin cos  ⋅1
 10 10 10 10 
=
10
rankershalt.com

26 Complex Numbers

z p + q − z p − zq + 1 = 0
2
1 π 1 2π  17. Given, …(i)
29  sin ⋅ sin 
2 5 2 5 ⇒ (z − 1)(z − 1) = 0
p q
=
10 Since, α is root of Eq. (i), either α p − 1 = 0 or α q − 1 = 0
25 (sin 36°⋅ sin 72° )2
= αp −1 αq − 1
10 ⇒ Either = 0 or =0 [as α ≠ 1]
α −1 α −1
25
= 2 (2 sin 36° sin 72° )2 ⇒ Either 1 + α + α 2 + ... + α p − 1 = 0
2 × 10
or 1 + α + K + αq −1 = 0
22
= (cos 36° − cos 108° )2 But α p − 1 = 0 and α q − 1 = 0 cannot occur
5
2 simultaneously as p and q are distinct primes, so
2 2   5 − 1  5 + 1  22 5 neither p divides q nor q divides p, which is the
=   +  = ⋅ =1
5  4   4   5 4 requirement for 1 = α p = α q.

(S) Sum of nth roots of unity = 0 18. Since, 1, a1 , a 2, ... , a n − 1 are nth roots of unity.
1 + α + α + α + K+ α =0
2 3 9 ⇒ (xn − 1) = (x − 1) (x − a1 ) (x − a 2) .... (x − a n − 1 )
9 xn − 1
⇒ = (x − a1 ) (x − a 2) ..... (x − a n − 1 )
1+ ∑ αk = 0 x−1
k =1
9
⇒ xn − 1 + xn − 2 + ..... + x2 + x + 1
 2 kπ 2 kπ  = (x − a1 ) (x − a 2) ..... (x − a n − 1 )
1+ ∑  cos
 10
+ i sin
10 
 =0
 xn − 1 
k =1 n −1
9 Q x − 1 = x + xn− 2 + ... + x + 1
2 kπ  
1+ ∑ cos
10
=0
On putting x = 1 , we get 1 + 1 + ... n times
k =1
9
2 kπ = (1 − a1 ) (1 − a 2) ..... (1 − a n − 1 )
So, 1− ∑ cos
10
=2 ⇒ (1 − a1 ) (1 − a 2)... (1 − a n − 1 ) = n
k =1

(P) → (i), (Q) → (ii), (R) → (iii), (S) → (iv) 19. Since, n is not a multiple of 3, but odd integers and
 1 ω ω 2 x3 + x2 + x = 0 ⇒ x = 0, ω , ω 2
 
14. Let A =  ω ω2 1  Now, when x = 0
ω 2
1 ω  ⇒ (x + 1)n − xn − 1 = 1 − 0 − 1 = 0

∴ x = 0 is root of (x + 1)n − xn − 1
0 0 0
Again, when x = ω
Now, A = 0 0 0 and Tr ( A ) = 0, A = 0
2
  ⇒ (x + 1)n − xn − 1 = (1 + ω )n − ω n − 1= −ω 2n − ω n − 1 = 0
0 0 0 [as n is not a multiple of 3 and odd]
A3 = 0 Similarly, x = ω 2 is root of {( x + 1)n − xn − 1 }
z+1 ω ω2 Hence, x = 0, ω , ω 2 are the roots of (x + 1)n − xn − 1
⇒ ω z + ω2 1 = [ A + zl] = 0 Thus, x3 + x2 + x divides (x + 1)n − xn − 1 .
ω 2
1 z+ω 20. Since, α, β are the complex cube roots of unity.
⇒ z =0 3 ∴ We take α = ω and β = ω 2.
⇒ z = 0, the number of z satisfying the given equation Now, xyz = (a + b)(aα + bβ )(aβ + bα )
is 1. = (a + b)[a 2αβ + ab(α 2 + β 2) + b2αβ ]
15. Here, Tr = (r − 1) (r − ω ) (r − ω ) ] = (r − 1)2 3
= (a + b)[a 2(ω ⋅ ω 2) + ab(ω 2 + ω 4 ) + b2(ω ⋅ ω 2)]
n
 n (n + 1) 
2
= (a + b)(a 2 − ab + b2) [Q1 + ω + ω 2 = 0 and ω3 = 1]
∴ Sn = ∑ (r3 − 1)=  2 
−n
= a3 + b3
r =1 2π
i
16. Since, cube root of unity are 1, ω , ω 2 given by 21. Priniting error = e 3

|x|2|+ | y|2 + |z|2


 1 3  1 3 Then, =3
A (1, 0), B − , −  ,C  − , −  (a )2 + (b)2 + |c|2
 2 2  2 2 2π
i
⇒ AB = BC = CA = 3 NOTE Here, w = e 3 , then only integer solution exists.
Hence, cube roots of unity form an equilateral triangle.

Download Chapter Test


http://tinyurl.com/y3avd3wd or
rankershalt.com

2
Theory of Equations
Topic 1 Quadratic Equations
Objective Questions I (Only one correct option) 7. If λ be the ratio of the roots of the quadratic equation in
x, 3m2x2 + m(m − 4)x + 2 = 0, then the least value of m for
1. If α and β are the roots of the quadratic equation, 1
 π which λ + = 1, is (2019 Main, 12 Jan I)
x2 + x sin θ − 2 sin θ = 0, θ ∈ 0,  , then λ
 2
(a) − 2 + 2 (b) 4 − 2 3
α +β
12 12
(c) 4 − 3 2 (d) 2 − 3
is equal to
(α −12 + β −12 )(α − β )24 (2019 Main, 10 April I) 8. If one real root of the quadratic equation
212 26 81x2 + kx + 256 = 0 is cube of the other root, then a value
(a) (b)
(sin θ + 8)12 (sin θ + 8)12 of k is (2019 Main, 11 Jan I)
212
212 (a) 100 (b) 144
(c) (d) (c) −81 (d) −300
(sin θ − 4)12
(sin θ − 8)6
9. If 5, 5r , 5r 2 are the lengths of the sides of a triangle, then
2. Let p, q ∈R. If 2 − 3 is a root of the quadratic equation,
r cannot be equal to (2019 Main, 10 Jan I)
x2 + px + q = 0, then (2019 Main, 9 April I) 5 7 3 3
(a) (b) (c) (d)
(a) q 2 − 4 p − 16 = 0 4 4 2 4
(b) p2 − 4q − 12 = 0 10. The value of λ such that sum of the squares of the roots
(c) p2 − 4q + 12 = 0 of the quadratic equation, x2 + (3 − λ )x + 2 = λ has the
(d) q 2 + 4 p + 14 = 0 least value is (2019 Main, 10 Jan II)
4 15
(a) (b) 1 (c) (d) 2
3. If m is chosen in the quadratic equation 9 8
(m2 + 1)x2 − 3x + (m2 + 1)2 = 0 such that the sum of its
roots is greatest, then the absolute difference of the
11. The number of all possible positive integral values of α
for which the roots of the quadratic equation,
cubes of its roots is (2019 Main, 9 April II)
6x2 − 11x + α = 0 are rational numbers is
(a) 10 5 (b) 8 5 (2019 Main, 9 Jan II)
(c) 8 3 (d) 4 3 (a) 5 (b) 2
4. If α and β are the roots of the equation x2 − 2x + 2 = 0, (c) 4 (d) 3
n
α 12. Let α and β be two roots of the equation x2 + 2x + 2 = 0,
then the least value of n for which   = 1 is
 β then α 15 + β15 is equal to (2019 Main, 9 Jan I)
(a) 2 (b) 5 (2019 Main, 8 April I) (a) 256 t(b) 512
(c) 4 (d) 3 (c) −256 (d) −512
5. The number of integral values of m for which the 13. Let S = { x ∈ R : x ≥ 0 and 2| x − 3| + x ( x − 6) + 6 = 0 .
equation (1 + m2)x2 − 2(1 + 3m)x + (1 + 8m) = 0, has no Then, S (2018 Main)
real root is (2019 Main, 8 April II) (a) is an empty set
(a) 3 (b) infinitely many (b) contains exactly one element
(c) 1 (d) 2 (c) contains exactly two elements
6. The number of integral values of m for which the (d) contains exactly four elements
quadratic expression, (1 + 2m) x2 − 2(1 + 3m) 14. If α , β ∈C are the distinct roots of the equation
x + 4(1 + m), x ∈ R, is always positive, is x2 − x + 1 = 0, then α 101 + β107 is equal to (2018 Main)
(2019 Main, 12 Jan II)
(a) −1 (b) 0
(a) 6 (b) 8 (c) 7 (d) 3
(c) 1 (d) 2
rankershalt.com

28 Theory of Equations

15. For a positive integer n, if the quadratic equation, 24. If a , b, c are the sides of a triangle ABC such that
x(x + 1) + (x + 1) (x + 2) + ... + (x + n − 1) (x + n ) = 10n x2 − 2 (a + b + c) x + 3λ (ab + bc + ca ) = 0 has real roots,
has two consecutive integral solutions, then n is then (2006, 3M)
equal to
(c) λ ∈  ,  (d) λ ∈  , 
(2017 Main) 4 5 4 5 1 5
(a) λ < (b) λ >
(a) 12 (b) 9 (c) 10 (d) 11 3 3  3 3  3 3
16. The sum of all real values of x satisfying the equation 25. If one root is square of the other root of the equation
2
+ 4 x − 60
(x2 − 5x + 5)x = 1 is (2016 Main) x2 + px + q = 0, then the relation between p and q is
(a) 3 (b) − 4 (c) 6 (d) 5 (a) p3 − q(3 p − 1) + q2 = 0 (2004, 1M)
π π (b) p3 − q(3 p + 1) + q2 = 0
17. Let − < θ < − . Suppose α 1 and β1 are the roots of the (c) p3 + q(3 p − 1) + q2 = 0
6 12
equation x2 − 2x secθ + 1 = 0 , and α 2 and β 2 are the roots (d) p3 + q(3 p + 1) + q2 = 0
of the equation x2 + 2x tan θ − 1 = 0. If α 1 > β1 and 26. The set of all real numbers x for which x2 − |x + 2| + x > 0
α 2 > β 2, then α 1 + β 2 equals (2016 Adv.) is (2002, 1M)
(a) 2(secθ − tan θ) (b) 2secθ (a) (− ∞ , − 2) ∪ (2, ∞ ) (b) (− ∞ , − 2 ) ∪ ( 2 , ∞ )
(c) −2tanθ (d) 0 (c) (− ∞ , − 1) ∪ (1, ∞ ) (d) ( 2 , ∞ )
18. Let α and β be the roots of equation x2 − 6x − 2 = 0. If 27. The number of solutions of log 4 (x − 1) = log 2(x − 3) is
a − 2a 8
a n = α − β , for n ≥ 1, then the value of 10
n n
is (a) 3 (b) 1 (2001, 2M)
2a 9 (c) 2 (d) 0
(a) 6 (b) – 6 (2015 Main) 28. For the equation 3x2 + px + 3 = 0, p > 0, if one of the root
(c) 3 (d) – 3 is square of the other, then p is equal to (2000, 1M)
19. In the quadratic equation p(x) = 0 with real coefficients (a) 1 /3 (b) 1 (c) 3 (d) 2 /3
has purely imaginary roots. Then, the equation 29. If α and β (α < β) are the roots of the equation
p[ p(x)] = 0 has (2014 Adv.)
x2 + bx + c = 0, where c < 0 < b, then (2000, 1M)
(a) only purely imaginary roots
(a) 0 < α < β (b) α < 0 < β < |α|
(b) all real roots
(c) two real and two purely imaginary roots (c) α < β < 0 (d) α < 0 < |α|< β
(d) neither real nor purely imaginary roots 30. The equation x + 1 − x − 1 = 4x − 1 has (1997C, 2M)
20. Let α and β be the roots of equation px2 + qx + r = 0, (a) no solution
1 1 (b) one solution
p ≠ 0. If p, q and r are in AP and + = 4, then the
α β (c) two solutions
value of|α − β|is (2014 Main) (d) more than two solutions
3 5
61 2 17 34 2 13 (log 2 x )2 + log 2 x −
(a) (b) (c) (d) 31. The equation x 4 4 = 2 has (1989; 2M)
9 9 9 9
(a) atleast one real solution
21. Let α and β be the roots of x2 − 6x − 2 = 0, with α > β. If (b) exactly three real solutions
a10 − 2a 8
a n = α n − β n for n ≥ 1 , then the value of is (c) exactly one irrational solution
2a 9 (d) complex roots
(a) 1 (b) 2 (c) 3 (d) 4 (2011) 32. If α and β are the roots of x2 + px + q = 0 and α 4 , β 4are
22. Let p and q be real numbers such that p ≠ 0, p ≠ q and 3
the roots of x2 − rx + s = 0, then the equation
p ≠ − q. If α and β are non-zero complex numbers
3
x2 − 4qx + 2q2 − r = 0 has always (1989, 2M)
satisfying α + β = − p and α 3 + β3 = q, then a quadratic (a) two real roots
α β (b) two positive roots
equation having and as its roots is (2010)
β α (c) two negative roots
(a) ( p3 + q) x2 − ( p3 + 2q) x + ( p3 + q) = 0 (d) one positive and one negative root
(b) ( p3 + q) x2 − ( p3 − 2q) x + ( p3 + q) = 0 2 2
( p3 − q) x2 − (5 p3 − 2q) x + ( p3 − q) = 0
33. The equation x − =1 − has
(c) x−1 x−1 (1984, 2M)
(d) ( p3 − q) x2 − (5 p3 + 2q) x + ( p3 − q) = 0
(a) no root (b) one root
23. Let α, β be the roots of the equation x2 − px + r = 0 and (c) two equal roots (d) infinitely many roots
α (x − a )(x − b)
, 2 β be the roots of the equation x2 − qx + r = 0 . Then, 34. For real x, the function will assume all real
2 (x − c)
the value of r is (2007, 3M)
2 2 values provided (1984, 3M)
(a) ( p − q) (2q − p ) (b) (q − p ) (2 p − q) (a) a > b > c (b) a < b < c
9 9
2 2 (c) a > c < b (d) a ≤ c ≤ b
(c) (q − 2 p ) (2q − p ) (d) (2 p − q) (2q − p )
9 9
rankershalt.com

Theory of Equations 29

35. The number of real solutions of the equation Analytical & Descriptive Questions
|x|2−3|x| + 2 = 0 is (1982, 1M)
(a) 4 (b) 1 (c) 3 (d) 2 45. If x2 − 10ax − 11b = 0 have roots c and d. x2 − 10cx − 11d = 0
have roots a and b, then find a + b + c + d. (2006, 6M)
36. Both the roots of the equation
(x − b) (x − c) + (x − a ) (x − c) + (x − a ) (x − b) = 0 46. If α , β are the roots of ax + bx + c = 0, (a ≠ 0) and
2

are always (1980, 1M) α + δ , β + δ are the roots of Ax2 + Bx + C = 0, ( A ≠ 0) for


(a) positive some constant δ, then prove that
b2 − 4ac B2 − 4 AC
(b) negative = (2000, 4M)
(c) real a2 A2
(d) None of the above 47. Let f (x) = Ax2 + Bx + C where, A , B, C are real
37. Let a > 0, b > 0 and c > 0. Then, both the roots of the numbers. prove that if f (x) is an integer whenever x is
equation ax2 + bx + c = 0 (1979, 1M) an integer, then the numbers 2 A , A + B and C are all
(a) are real and negative integers. Conversely, prove that if the numbers
(b) have negative real parts 2 A , A + B and C are all integers, then f (x) is an integer
(c) have positive real parts whenever x is an integer. (1998, 3M)
(d) None of the above 48. Find the set of all solutions of the equation
2|y| − |2y − 1 − 1| = 2y − 1 + 1 (1997 C, 3M)
Assertion and Reason
49. Find the set of all x for which
For the following question, choose the correct answer 2x 1
from the codes (a), (b), (c) and (d) defined as follows : > (1987, 3M)
2x + 5x + 2
2
x +1
(a) Statement I is true, Statement II is also true;
Statement II is the correct explanation of Statement I 50. Solve| x2 + 4x + 3 | + 2x + 5 = 0 (1987, 5M)
(b) Statement I is true, Statement II is also true; 51. For a ≤ 0, determine all real roots of the equation
Statement II is not the correct explanation of
Statement I x2 − 2a|x − a|− 3a 2 = 0 (1986, 5M)
(c) Statement I is true; Statement II is false 2
−3 2
−3
52. Solve for x : (5 + 2 6 )x + (5 − 2 6 )x = 10 (1985, 5M)
(d) Statement I is false; Statement II is true
38. Let a , b, c, p, q be the real numbers. Suppose α , β are the 53. If one root of the quadratic equation ax + bx + c = 0 is 2

roots of the equation x + 2 px + q = 0.


2 equal to the nth power of the other, then show that
1 1
1 n n +1 n +1
and α , are the roots of the equation ax2 + 2bx + c = 0, (ac ) + (a c)n
+ b =0 (1983, 2M)
β
where β 2 ∉ { −1, 0, 1}. 54. If α and β are the roots of x + px + q = 0 and γ , δ are the
2

Statement I ( p2 − q) (b2 − ac) ≥ 0 roots of x2 + rx + s = 0, then evaluate (α − γ ) (β − γ )


Statement II b ∉ pa or c ∉ qa . (2008, 3M) (α − δ ) (β − δ ) in terms of p, q, r and s. (1979, 2M)

55. Solve 2 log x a + log ax a + 3 logb a = 0 ,


Fill in the Blanks where a > 0, b = a 2 x (1978 , 3M )
39. The sum of all the real roots of the equation 56. If α and β are the roots of the equation
|x − 2|2 + |x − 2| − 2 = 0 is…… . (1997, 2M)
x2 + px + 1 = 0 ; γ , δ are the roots of x2 + qx + 1 = 0, then
40. If the products of the roots of the equation q2 − p2 = (α − γ )(β − γ )(α + δ )(β + δ ) (1978, 2M)
x2 − 3kx + 2e2log k − 1 = 0 is 7, then the roots are real for
k =K . (1984, 2M) Passage Type Questions
41. If 2 + i 3 is a root of the equation x2 + px + q = 0, where Let p, q be integers and let α , β be the roots of the equation,
p and q are real, then ( p, q) = (…,…). (1982, 2M) x2 − x − 1 = 0 where α ≠ β. For n = 0, 1, 2, …… , let
42. The coefficient of x99 in the polynomial a n = pα n + qβ n.
(x − 1)(x − 2)... (x − 100) is.... (1982, 2M) FACT : If a and b are rational numbers and a + b 5 = 0, then
a = 0 = b. (2017 Adv.)
True/False 57. a12 =
43. If P (x) = ax2 + bx + c and Q (x) = − ax2 + bx + c, where (a) a11 + 2a10 (b) 2a11 + a10
ac ≠ 0, then P (x) Q (x) has atleast two real roots. (c) a11 − a10 (d) a11 + a10
(1985, 1M)
58. If a 4 = 28, then p + 2q =
44. The equation 2x + 3x + 1 = 0 has an irrational root.
2
(a) 14 (b) 7 (c) 21 (d) 12
(1983, 1M)
rankershalt.com

30 Theory of Equations

Topic 2 Common Roots


Objective Questions I (Only one correct option) 3. A value of b for which the equations x2 + bx − 1 = 0,
1 If α , β and γ are three consecutive terms of a x2 + x + b = 0 have one root in common is (2011)

non-constant GP such that the equations (a) − 2 (b) −i 3 (c) i 5 (d) 2


αx2 + 2βx + γ = 0 andx2 + x − 1 = 0 have a common root,
then, α(β + γ) is equal to (2019 Main, 12 April II) Fill in the Blanks
(a) 0 4. If the quadratic equations x2 + ax + b = 0 and
(b) αβ x2 + bx + a = 0 (a ≠ b) have a common root, then the
(c) αγ numerical value of a + b is… . (1986, 2M)
(d) βγ
2. If the equations x2 + 2x + 3 = 0 and ax2 + bx + c = 0, True/False
a , b, c ∈ R have a common root, then a : b : c is 5. If x − r is a factor of the polynomial f (x) = a n xn,
(a) 1 : 2 : 3 (b) 3 : 2 : 1 (2013 Main) + a n − 1 x n−1 + K + a 0 repeated m times (1 < m ≤ n ), then
(c) 1 : 3 : 2 (d) 3 : 1 : 2 r is a root of f ′ (x) = 0 repeated m times. (1983, 1M)

Topic 3 Transformation of Roots


Objective Question I (Only one correct option) Analytical & Descriptive Question
1. Let α , β be the roots of the equation,(x − a )(x − b) = c, c ≠ 0. 2. Let a, b and c be real number s with a ≠ 0 and let α , β be
Then the roots of the equation (x − α ) (x − β ) + c = 0 are the roots of the equation ax2 + bx + c = 0. Express the
(a) a , c (b) b , c (c) a , b (d) a + c, b + c (1992, 2M) roots of a3 x2 + abcx + c3 = 0 in terms of α , β. (2001, 4M)

Topic 4 Graph of Quadratic Expression


Objective Questions I (Only one correct option)
1. Let P(4, − 4) and Q(9, 6) be two points on the parabola, 4. If a ∈ R and the equation −3 (x − [x])2 + 2 (x − [x])
y = 4x and let X be any point on the arc POQ of this
2
+ a 2 = 0 (where, [x] denotes the greatest integer ≤ x) has
parabola, where O is the vertex of this parabola, such no integral solution, then all possible values of a lie in
that the area of ∆PXQ is maximum. Then, this the interval (2014 Main)
maximum area (in sq units) is (2019 Main, 12 Jan I) (a) (−1, 0) ∪ (0, 1) (b) (1, 2)
125 75 (c) (−2 , − 1) (d) (−∞ , − 2) ∪ (2 , ∞ )
(a) (b)
2 2
625 125 5 For all ‘x’, x2 + 2ax + (10 − 3a ) > 0, then the interval in
(c) (d)
4 4 which ‘a’ lies is (2004, 1M)
(a) a < − 5 (b) −5 < a < 2 (c) a > 5 (d) 2 < a < 5
2. Consider the quadratic equation, (c − 5) x − 2cx + (c − 4)
2

= 0, c ≠ 5. Let S be the set of all integral values of c for 6. If b > a, then the equation (x − a ) (x − b) − 1 = 0 has
which one root of the equation lies in the interval (0, 2) (a) both roots in (a , b) (2000, 1M)
and its other root lies in the interval (2, 3). Then, the (b) both roots in ( − ∞ , a )
number of elements in S is (2019 Main, 10 Jan I) (c) both roots in (b, + ∞ )
(a) 11 (b) 10 (d) one root in (−∞ , a ) and the other in (b, ∞ )
(c) 12 (d) 18 7. If the roots of the equation x2 − 2ax + a 2 + a − 3 = 0 are
3. If both the roots of the quadratic equation real and less than 3, then (1999, 2M)
x2 − mx + 4 = 0 are real and distinct and they lie in the (a) a < 2 (b) 2 ≤ a ≤ 3 (c) 3 < a ≤ 4 (d) a> 4
interval [1, 5] then m lies in the interval
(2019 Main, 9 Jan II) 8. Let f (x) be a quadratic expression which is positive for
(a) (4, 5) (b) (−5, − 4) all real values of x.If g (x) = f (x) + f ′ (x) + f ′ ′ (x), then for
(c) (5, 6) (d) (3, 4) any real x (1990, 2M)
(a) g (x) < 0 (b) g (x) > 0 (c) g (x) = 0 (d) g (x) ≥ 0
rankershalt.com

Theory of Equations 31

Analytical & Descriptive Questions 11. Find all real values of x which satisfy x2 − 3x + 2 > 0 and
x2 − 2x − 4 ≤ 0. (1983, 2M)
9. If x2 + (a − b)x + (1 − a − b) = 0 where a , b ∈ R, then find
the values of a for which equation has unequal real roots Integer Answer Type Question
for all values of b. (2003, 4M)
12. The smallest value of k, for which both the roots of the
10. Let a , b, c be real. If ax2 + bx + c = 0 has two real roots α equation x2 − 8kx + 16 (k2 − k + 1) = 0 are real, distinct
and β, where α < − 1 and β > 1, then show that and have values atleast 4, is …… . (2009)
1 + +  < 0
c b
a a  (1995, 5M)

Topic 5 Some Special Forms


Objective Questions I (Only one correct option) Then, the quadratic equation ax2 + bx + c = 0 has
1. The number of real roots of the equation (a) no root in (0, 2) (b) atleast one root in (1, 2)
5 + |2 − 1| = 2 (2 − 2) is
x x x
(2019 Main, 10 April II) (c) a double root in (0, 2) (d) two imaginary roots
(a) 1 (b) 3
(c) 4 (d) 2 Objective Questions II
2. All the pairs (x, y) that satisfy the inequality (One or more than one correct option)
2
sin 2 x − 2 sin x + 5

1
≤ 1 also satisfy the equation
9. Let S be the set of all non-zero real numbers α such that
sin 2 y
4 (2019 Main, 10 April I)
the quadratic equation αx2 − x + α = 0 has two distinct
real roots x1 and x2 satisfying the inequality x1 – x2 < 1.
(a) 2|sin x| = 3 sin y (b) sin x = |sin y|
Which of the following interval(s) is/are a subset of S?
(c) sin x = 2 sin y (d) 2 sin x = sin y (2015 Adv.)
(a)  – , –
1 
(b)  – , 0 (c)  0,
3. The sum of the solutions of the equation 1 1 1   1 , 1
  (d)  
 2 5  5   5  5 2
| x − 2| + x ( x − 4) + 2 = 0 (x > 0) is equal to
(2019 Main, 8 April I) 10. Let a ∈ R and f : R → R be given by f (x) = x5 − 5x + a.
(a) 9 (b) 12 Then,
(c) 4 (d) 10 (a) f (x) has three real roots, if a > 4
4. The real number k for which the equation, (b) f (x) has only one real root, if a > 4
2x3 + 3x + k = 0 has two distinct real roots in [0, 1] (c) f (x) has three real roots, if a < − 4
(2013 Main) (d) f (x) has three real roots, if −4 < a < 4
(a) lies between 1 and 2 (b) lies between 2 and 3
(c) lies between − 1and 0 (d) does not exist Passage Based Problems
5. Let a , b, c be real numbers, a ≠ 0. If α is a root of Read the following passage and answer the questions.
a 2x2 + bx + c = 0, β is the root of a 2x2 − bx − c = 0 and Passage I
0 < α < β, then the equation a 2x2 + 2bx + 2c = 0 has a root
γ that always satisfies (1989, 2M) Consider the polynomial f ( x ) = 1 + 2x + 3x 2+ 4x3 . Let s be
α+β β the sum of all distinct real roots of f ( x ) and let t =| s|.
(a) γ = (b) γ = α +
2 2 (2010)
(c) γ = α (d) α < γ < β 11. The real numbers s lies in the interval
6. If a + b + c = 0, then the quadratic equation (a)  − , 0
1
(b)  − 11, −  (c)  − , − 
3 3 1
(d)  0,
1

3ax + 2bx + c = 0 has
2
(1983, 1M)  4   4   4 2  4
(a) at least one root in (0, 1) 12. The area bounded by the curve y = f (x) and the lines
(b) one root in (2, 3) and the other in (−2, − 1) x = 0, y = 0 and x = t , lies in the interval
(c) imaginary roots
(a)  , 3 (b)  ,  (d)  0, 
3 21 11 21
(c) (9, 10)
(d) None of the above 4   64 16   64 
7. The largest interval for which 13. The function f ′ (x) is
x12 − x9 + x4 − x + 1 > 0 is
(a) increasing in  − t, −  and decreasing in  − , t 
(1982, 2M) 1 1
(a) −4 < x ≤ 0 (b) 0 < x < 1  4  4 
(c) −100 < x < 100 (d) −∞ < x < ∞
(b) decreasing in  − t, −  and increasing in  − , t 
1 1
8. Let a , b, c be non-zero real numbers such that  4  4 
1 (c) increasing in (−t , t )
∫0 (1 + cos x)(ax2 + bx + c)dx
8
(1981, 2M)
(d) decreasing in (−t , t )
2
= ∫ (1 + cos 8 x)(ax2 + bx + c)dx
0
rankershalt.com

32 Theory of Equations

Passage II 16. For k > 0, the set of all values of k for which kex − x = 0
If a continuous function f defined on the real line R, has two distinct roots, is
(a)  0,  (b)  , 1
assumes positive and negative values in R, then the 1 1
equation f ( x ) = 0 has a root in R. For example, if it is  e e 
known that a continuous function f on R is positive at some (c)  , ∞ 
1
(d) (0, 1)
point and its minimum values is negative, then the e 
equation f ( x ) = 0 has a root in R. Consider f ( x ) = kex − x for
all real x where k is real constant. (2007, 4M) True/False
14. The line y = x meets y = ke for k ≤ 0 at
x 17. If a < b < c < d, then the roots of the equation (x − a )
(a) no point (b) one point (x − c) + 2 (x − b) (x − d ) = 0 are real and distinct.
(c) two points (d) more than two points (1984, 1M)

15. The positive value of k for which ke − x = 0 has only one


x Analytical & Descriptive Question
root is 18. Let −1 ≤ p < 1. Show that the equation 4x3 − 3x − p = 0
1
(a) (b) 1 (c) e (d) log e 2 has a unique root in the interval [1/2, 1] and identify it.
e (2001, 4M)

Answers
Topic 1
1. (a) 2. (b) 3. (b) 4. (c)
Topic 2
5. (a) 6. (c) 7. (c) 8. (d)
9. (b) 10. (d) 11. (d) 12. (c) 1. (d) 2. (a) 3. (b) 4. (–1)
13. (c) 14. (c) 15. (d) 16. (a) 5. False
17. (c) 18. (c) 19. (d) 20. (d) Topic 3
21. (c) 22. (b) 23. (d) 24. (a) 1. (c) 2. x = α 2β, αβ 2
25. (a) 26. (b) 27. (b) 28. (c)
Topic 4
29. (b) 30. (a) 31. (b) 32. (a)
1. (d) 2. (a) 3. (a) 4. (a)
33. (a) 34. (d) 35. (a) 36. (c)
37. (b) 38. (b) 39. 4 40. k = 2
5. (b) 6. (d) 7. (a) 8. (b)
9. a > 1 11. x ∈ [1 − 5, 1 ) ∪ [1 + 5, 2 ) 12. k = 2
41. ( −4, 7 ) 42. –5050 43. True 44. False
45. 1210 48. y ∈ { −1 } ∪ [1, ∞ ) Topic 5
 2 1 1. (a) 2. (b) 3. (d) 4. (d)
49. x ∈ ( −2, − 1 ) ∪  − , −  50. −4 and ( − 1 − 3 )
 3 2 5. (d) 6. (a) 7. (d) 8. (b)
51. x = {a (1 − 2 ), a ( 6 − 1 )} 52. ±2, ± 2 9. (a,d) 10. (b, d) 11. (c) 12. (a)
54. (q − s ) 2 − rqp − rsp + sp 2 + qr 2 55. x = a −1/2 or a − 4 /3 13. (b) 14. (b) 15. (a) 16. (a)
1 
56. q − p
2 2
57. (d) 58. (d) 17. True 18. x = cos cos−1 p
3 
rankershalt.com

Hints & Solutions


Topic 1 Quadratic Equations ⇒ (x − 1)2 = − 1
1. Given quadratic equation is ⇒ x−1 = ± i [where i = −1]
 π ⇒ x = (1 + i ) or (1 − i )
x2 + x sin θ − 2 sin θ = 0, θ ∈ 0, 
 2 Clearly, if α = 1 + i, then β = 1 − i
n
and its roots are α and β. α
According to the question   = 1
So, sum of roots = α + β = − sin θ  β
n
and product of roots = αβ = − 2 sin θ  1 + i
⇒   =1
⇒ αβ = 2(α + β ) …(i) 1 − i
α 12 + β12 n
Now, the given expression is −12  (1 + i )(1 + i )
(α + β −12)(α − β)24 ⇒   =1 [by rationalization]
 (1 − i )(1 + i ) 
α +β12 12
α +β 12 12
n
= = 12  1 + i 2 + 2i   2i 
n
 1 1 24  β + α 12 ⇒   =1⇒   =1⇒ i =1
n
 12 + 12 (α − β)  12 12  (α − β)24  1 − i2   2
α β   α β 
So, minimum value of n is 4. [Q i 4 = 1]
12 12
 αβ   αβ 
= 2
=  Key Idea
 (α − β )   (α + β ) 2
− 4αβ  5.
(i) First convert the given equation in quadratic equation.
 
12 (ii) Use, Discriminant, D = b 2 − 4 ac < 0
2(α + β)
=  [from Eq. (i)]
 (α + β ) 2
− 8 (α + β)  Given quadratic equation is
 2 
12
 2 
12
(1 + m2)x2 − 2(1 + 3m)x + (1 + 8m) = 0 …(i)
=  =  [Q α + β = − sin θ] Now, discriminant
 (α + β ) − 8  − sin θ − 8
D = [−2(1 + 3m)]2 − 4(1 + m2)(1 + 8m)
212
= = 4 [(1 + 3m)2 − (1 + m2)(1 + 8m)]
(sin θ + 8)12
= 4 [1 + 9m2 + 6m − (1 + 8m + m2 + 8m3 )]
2. Given quadratic equation is x2 + px + q = 0, where = 4 [−8m3 + 8m2 − 2m]
p, q ∈R having one root 2 − 3 , then other root is 2 + 3 = − 8m(4m2 − 4m + 1) = − 8m(2m − 1)2
(conjugate of 2 − 3 ) [Q irrational roots of a quadratic
equation always occurs in pairs] According to the question there is no solution of the
quadratic Eq. (i), then
So, sum of roots = − p = 4 ⇒ p = −4
and product of roots = q = 4 − 3 ⇒ q = 1 D <0
Now, from options p2 − 4q − 12 = 16 − 4 − 12 = 0 ∴ −8m(2m − 1)2 < 0 ⇒ m > 0
So, there are infinitely many values of ‘m’ for which,
3. Given quadratic equation is
there is no solution of the given quadratic equation.
(m2 + 1)x2 − 3x + (m2 + 1)2 = 0 …(i)
6. The quadratic expression
Let the roots of quadratic Eq. (i) are α and β, so ax2 + bx + c, x ∈ R is always positive,
3
α+β= 2 and αβ = m2 + 1 if a > 0 and D < 0.
m +1
So, the quadratic expression
According to the question, the sum of roots is greatest (1 + 2m) x2 − 2 (1 + 3m)x + 4(1 + m), x ∈ R will be
and it is possible only when ‘‘(m2 + 1) is minimum’’ and
‘‘minimum value of m2 + 1 = 1, when m = 0’’. always positive, if 1 + 2m > 0 …(i)
∴α + β = 3 and αβ = 1, as m = 0 and D = 4(1 + 3m)2 − 4(2m + 1) 4(1 + m) < 0 …(ii)
Now, the absolute difference of the cubes of roots From inequality Eq. (i), we get
= |α 3 − β3| 1
m>− …(iii)
= |α − β||α 2 + β 2 + αβ| 2
= (α + β )2 − 4αβ |(α + β )2 − αβ| From inequality Eq. (ii), we get
1 + 9m2 + 6m − 4 (2m2 + 3m + 1) < 0
= 9 − 4 |9 − 1|= 8 5
⇒ m2 − 6m − 3 < 0
4. Given, α and β are the roots of the quadratic equation, ⇒ [m − (3 + 12 )][m − (3 − 12 )] < 0
x2 − 2x + 2 = 0 6 ± 36 + 12
[Q m2 − 6m − 3 = 0 ⇒ m = = 3 ± 12]
⇒ (x − 1)2 + 1 = 0 2
rankershalt.com

34 Theory of Equations

⇒ 3 − 12 < m < 3 + 12 …(iv) 1 − 5 1 + 5


⇒ r ∈ ,  ...(i)
From inequalities Eqs. (iii) and (iv), the integral values  2 2 
of m are 0, 1, 2, 3, 4, 5, 6 + – +
Hence, the number of integral values of m is 7. 1– √ 5 1+ √ 5
7. Let the given quadratic equation in x, 2 2
3m2x2 + m(m − 4)x + 2 = 0, m ≠ 0 have roots α and β, then Similarly, b+ c>a
m(m − 4) 2
α +β = − 2
and αβ = ⇒ 5r + 5r 2 > 5
3m 3m2
⇒ r2 + r − 1 > 0
α
Also, let =λ   − 1 − 5    −1 + 5  
β r −    r −   >0
1 α β   2    2 
Then, λ + =1 ⇒ + =1 (given)
λ β α  2 −1 ± 1 + 4 − 1 ± 5 
Q r + r − 1 = 0 ⇒ r = = 
⇒ α 2 + β 2 = αβ  2 2 
⇒ (α + β )2 = 3αβ
 −1 − 5   −1 + 5 
m (m − 4)2
2
2 ⇒ r ∈  − ∞,  ∪ , ∞ …(ii)
⇒ =3  2   2 
9m4 3m2
+ – +
⇒ (m − 4)2 = 18 [Qm ≠ 0]
⇒ m − 4 = ±3 2 –1– √ 5 –1+ √ 5
2 2
⇒ m =4±3 2
The least value of m = 4 − 3 2 and c+ a >b
⇒ 5r 2 + 5 > 5r
8. Given quadratic equation is
⇒ r − r + 1 >0
2
81x2 + kx + 256 = 0 2 2
1  1  1
Let one root be α, then other is α 3 . ⇒ r2 − 2 ⋅ r +   + 1 −   > 0
2  2  2
k 256
Now, α + α 3 = − and α ⋅ α 3 = 2
81 81  1 3
⇒ r −  + > 0
b  2 4
[Q for ax2 + bx + c = 0, sum of roots = −
a ⇒ r ∈R … (iii)
c
and product of roots = ] From Eqs. (i), (ii) and (iii), we get
a
 −1 + 5 1 + 5
 4
4
4 r ∈ , 
⇒ α4 =   ⇒α = ±  2 2 
 3 3
∴ k = − 81 (α + α 3 )
= − 81 α (1 + α 2) –∞ –1– √ 5 ∞
1– √ 5 –1+ √ 5 1+ √ 5
 4  16
= − 81  ±  1 +  = ± 300 2 2 2 2
 3  9
7
and is the only value that does not satisfy.
9. Let a = 5, b = 5r and c = 5r 2 4
We know that, in a triangle sum of 2 sides is always 10. Given quadratic equation is
greater than the third side. x2 + (3 − λ )x + 2 = λ
∴ a + b > c; b + c > a and c + a > b x2 + (3 − λ )x + (2 − λ ) = 0 … (i)
Now, a+ b>c Let Eq. (i) has roots α and β, then α + β = λ − 3 and
⇒ 5 + 5r > 5r 2 ⇒ 5r 2 − 5r − 5 < 0 αβ = 2 − λ
⇒ r − r −1 <0
2 b
[Q For ax2 + bx + c = 0, sum of roots = −
 1 − 5   1 + 5  a
⇒ r −    r −   <0 c
and product of roots = ]
  2    2  a
[Q roots of ax2 + bx + c = 0 are given by Now, α 2 + β 2 = (α + β )2 − 2αβ
− b ± b2 − 4ac = (λ − 3)2 − 2(2 − λ )
x= and r 2 − r − 1 = 0 = λ2 − 6λ + 9 − 4 + 2λ
2a
= λ2 − 4λ + 5 = (λ2 − 4λ + 4) +1
1± 1+4 1± 5
⇒r = = ] = (λ − 2)2 + 1
2 2
Clearly, a + β 2 will be least when λ = 2.
2
rankershalt.com

Theory of Equations 35

11. For the roots of quadratic equation ax2 + bx + c = 0 to be  [1 + i 2 − 2i ]8 [1 + i 2 + 2 i ]8 


=− + 
rational D = (b − 4ac) should be perfect square.
2
 1−i 1+ i 
In the equation 6x2 − 11x + α = 0  (−2 i )8 (2 i )8 
=− +
a = 6, b = − 11 and c = α
 1−i 1 + i 
∴For roots to be rational
 1 1 
D = (− 11)2 − 4(6) (α) should be a perfect square. = − 28  +  [Q i 4n = 1, n ∈ Z ]
⇒ D(α) = 121 − 24α should be a perfect square 1 − i 1 + i 
Now,  2  2 
= − 256  2
= − 256 = − 256
− 2 
D(1) = 121 − 24 = 97 is not a perfect square.  1 (i ) 
D(2) = 121 − 24 × 2 = 73 is not a perfect square. 13. We have, 2| x − 3| + x ( x − 6) + 6 = 0
D(3) = 121 − 24 × 3 = 49 is a perfect square. Let x − 3 = y
D(4) = 121 − 24 × 4 = 25 is a perfect square. ⇒ x = y+3
D(5) = 121 − 24 × 5 = 1 is a perfect square. ∴ 2| y| + ( y + 3)( y − 3) + 6 = 0
and for α ≥ 6, D(α) < 0, hence imaginary roots. ⇒ 2| y| + y2 − 3 = 0
∴For 3 values of α (α = 3, 4, 5), the roots are rational. ⇒ | y|2 + 2| y| − 3 = 0
12. We have, x2 + 2x + 2 = 0 ⇒ (| y| + 3)(| y| − 1) = 0
⇒ | y| ≠ − 3 ⇒ | y| = 1
−2 ± 4 −8
⇒ x= [Q roots of ax2 + bx + c = 0 are ⇒ y= ±1 ⇒ x −3 = ±1
2
⇒ x = 4, 2 ⇒ x = 16, 4
−b± b2 − 4ac
given by x = ] 14. We have, α , β are the roots of x2 − x + 1 = 0
2a
Q Roots of x2 − x + 1 = 0 are −ω ,−ω 2
⇒ x = −1 ± i
∴ Let α = − ω and β = − ω 2
Let α = − 1 + i and β = − 1 − i.
⇒ α 101 + β107 = (− ω )101 + (− ω 2)107 = − (ω101 + ω 214 )
Then, α 15 + β15 = (−1 + i )15 + (− 1 − i )15 = − (ω 2 + ω ) [Q ω3 = 1]
= − [(1 − i )15 + (1 + i )15 ] = − (−1) [Q1 + ω + ω 2 = 0]
 =1
i  
15 15
 1 i    1
= −  2 −   +  2 +  
  2 2    2 2    15. Given quadratic equation is
x(x + 1) + (x + 1)(x + 2) + ... + (x + n − 1) (x + n ) =10n
  π π 
15 
 2  cos − i sin    ⇒ (x2 + x2 + ... + x2) + [(1 + 3 + 5 + ... + (2n − 1)]x
 4  + [(1 ⋅ 2 + 2 ⋅ 3 + ... + (n − 1)n ] = 10n
= −  4
15

   π π   n (n 2 − 1)
+  2  cos + i sin    ⇒ nx + n x +
2 2
− 10n = 0
  4 4   3
n −1
2
 15π 15π   15π 15π   ⇒ x2 + nx + − 10 = 0
= − ( 2 )15  cos − i sin  +  cos + i sin  3
  4 4   4 4  
⇒ 3x2 + 3nx + n 2 − 31 = 0
[using De’ Moivre’s theorem
Let α and β be the roots.
(cos θ ± i sin θ )n = cos nθ ± i sin nθ, n ∈ Z ]
Since, α and β are consecutive.
 15π   1 
= − ( 2 )15 2 cos = − ( 2 )15 2 × ∴ |α − β| = 1 ⇒ (α − β )2 = 1
 4   2 
Again, (α − β ) = (α + β ) − 4αβ
2 2
 15π  π π 1 
 − 3n 
2
 n 2 − 31
Q cos 4 = cos 4π − 4  = cos 4 = 2  ⇒ 1=  − 4 
   3   3 
= − ( 2 )16 = − 28 = − 256. 4
⇒ 1 = n 2 − (n 2 − 31) ⇒ 3 = 3n 2 − 4n 2 + 124
Alternate Method 3
α 15 + β15 = (−1 + i )15 + (−1 − i )15 ⇒ n 2 = 121 ⇒ n = ± 11
= − [(1 − i )15 + (1 + i )15 ] ∴ n = 11 [Qn > 0]
 (1 − i )16 (1 + i )16  16. Given, (x2 − 5x + 5)x
2
+ 4 x − 60
=1
=− +
 1−i 1 + i 
Clearly, this is possible when
 [(1 − i )2]8 [(1 + i )2]8  I. x2 + 4x − 60 = 0 and x2 − 5x + 5 ≠ 0
=− +
 1−i 1 + i  or
II. x2 − 5x + 5 = 1
rankershalt.com

36 Theory of Equations

or Now, consider
III. x2 − 5x + 5 = − 1 and x2 + 4x − 60 = Even integer. a10 − 2a 8 α 10 − β10 − 2(α 8 − β 8 )
=
Case I When x2 + 4x − 60 = 0 2a 9 2(α 9 − α 9 )
⇒ x2 + 10x − 6x − 60 = 0
α 8 (α 2 − 2) − β 8 (β 2 − 2)
⇒ x(x + 10) − 6(x + 10) = 0 =
2(α 9 − β 9 )
⇒ (x + 10) (x − 6) = 0
⇒ x = − 10or x = 6 α 8 ⋅ 6 α − β 86 β 6 α 9 − 6 β 9 6
= = = =3
Note that, for these two values of x, x2 − 5x + 5 ≠ 0 2(α 9 − β 9 ) 2(α 9 − 6 β 9 ) 2
Case II When x2 − 5x + 5 = 1 Q α and β are the roots of 
⇒ x2 − 5x + 4 = 0  x2 − 6x − 2 = 0 or x2 = 6x + 2 
⇒ x2 − 4x − x + 4 = 0 ⇒ α2 = 6 α + 2 ⇒ α2 − 2 = 6 α
and β = 6 β + 2 ⇒ β − 2 = 6 β 
2 2
⇒ x(x − 4) − 1 (x − 4) = 0
⇒ (x − 4) (x − 1) = 0 ⇒ x = 4or x = 1 Alternate Solution
Case III When x2 − 5x + 5 = − 1 Since, α and β are the roots of the equation
⇒ x2 − 5x + 6 = 0 x2 − 6x − 2 = 0.
⇒ x − 2x − 3x + 6 = 0
2
or x2 = 6x + 2
⇒ x(x − 2) − 3(x − 2) = 0
∴ α 2 = 6α + 2
⇒ (x − 2) (x − 3) = 0
⇒ α 10 = 6 α 9 + 2 α 8 ...(i)
⇒ x = 2 or x = 3
Similarly, β 10
=6β + 2β
9 8
…(ii)
Now, when x = 2, x2 + 4x − 60 = 4 + 8 − 60 = − 48, which is
an even integer. On subtracting Eq. (ii) from Eq. (i), we get
When x = 3, x2 + 4x − 60 = 9 + 12 − 60 = − 39, which is not α 10 − β10 = 6(α 9 − β 9 ) + 2(α 8 − β 8 ) (Q a n = α n − β n)
an even integer. ⇒ a10 = 6a 9 + 2a 8
Thus, in this case, we get x = 2. a − 2a 8
⇒ a10 − 2a 8 = 6a 9 ⇒ 10 =3
2a 9
Hence, the sum of all real values of
x = − 10 + 6 + 4 + 1 + 2 = 3 19. If quadratic equation has purely imaginary roots, then
coefficient of x must be equal to zero.
17. Here, x − 2x secθ + 1 = 0 has roots α 1 and β1.
2
Let p(x) = ax2 + b with a, b of same sign and a , b ∈ R.
2 sec θ ± 4 sec2 θ − 4
∴ α 1 , β1 = Then, p[ p(x)] = a (ax2 + b)2 + b
2 ×1
p(x) has imaginary roots say ix.
2 sec θ ± 2|tan θ|
= Then, also ax2 + b ∈ R and (ax2 + b)2 > 0
2
 π π ∴ a (ax2 + b)2 + b ≠ 0, ∀ x
Since, θ ∈  − ,−  ,
 6 12 Thus, p [ p(x)] ≠ 0, ∀ x
2 sec θ m 2 tan θ
i.e. θ ∈ IV quadrant = 20. PLAN If ax 2 + bx + c = 0 has roots α and β, then α + β = − b / a
2 c
∴ α 1 = sec θ − tan θ and β1 = sec θ + tan θ [as α 1 > β1] and α β = . Find the values of α + β and αβ and then put
a
and x2 + 2x tan θ − 1 = 0 has roots α 2 and β 2 . in (α − β )2 = (α + β )2 − 4αβ to get required value.
−2 tan θ ± 4 tan 2 θ + 4 Given, α and β are roots of px2 + qx + r = 0, p ≠ 0.
i.e. α 2, β 2 =
2 −q r
∴ α+β= , αβ = …(i)
∴ α 2 = − tan θ + sec θ p p
and β 2 = − tan θ − sec θ [as α 2 > β 2] Since, p, q and r are in AP.
Thus, α 1 + β 2 = −2 tan θ ∴ 2q = p + r ...(ii)
18. Given, α and β are the roots of the equation 1 1 α+β
Also, + =4 ⇒ =4
x2 − 6x − 2 = 0. α β αβ
∴ a n = α n − β n for n ≥ 1 −q 4r
⇒ α + β = 4 αβ ⇒ = [from Eq. (i)]
p p
a10 = α 10 − β10
⇒ q = − 4r
a 8 = α 8 − β8
On putting the value of q in Eq. (ii), we get
a 9 = α 9 − β9
⇒ 2 (−4r ) = p + r ⇒ p = − 9r
rankershalt.com

Theory of Equations 37

−q 4r 4r 4 ⇒ b2 + c2 − a 2< 2bc
Now, α+β= = = =−
p p −9 r 9 Similarly, c2+ a 2 − b2 < 2ca
r r 1 a 2 + b2 − c2 < 2ab
and αβ = = = and
p −9 r −9
⇒ a 2 + b2 + c2 < 2 (ab + bc + ca )
16 4 16 + 36
∴ (α − β )2 = (α + β )2 − 4 αβ = + = a 2 + b2 + c2
81 9 81 ⇒ <2 …(ii)
52 ab + bc + ca
⇒ (α − β ) =
2
81 From Eqs. (i) and (ii), we get
2 4
⇒ |α − β| = 13 3λ − 2 < 2 ⇒ λ<
9 3
a10 − 2a 8 (α 10 − β10 ) − 2 (α 8 − β 8 ) 25. Let the roots of x2 + px + q = 0 be α and α 2.
21. =
2a 9 2 (α 9 − β 9 ) ⇒ α + α2 = − p and α 3 = q
α (α − 2) − β (β − 2)
8 2 8 2
⇒ α (α + 1) = − p
=
2(α 9 − β 9 ) ⇒ α 3 {α 3 + 1 + 3α (α + 1)} = − p3 [cubing both sides]
Qα is root of x − 6 x − 2 = 0 ⇒ α − 2 = 6α
2 2
⇒ q (q + 1 − 3 p) = − p 3

[and β is root of x2 − 6 x − 2 = 0 ⇒ β 2 − 2 = 6 β] ⇒ p3 − (3 p − 1)q + q2 = 0


α 8 (6 α ) − β 8 (6 β ) 6 (α 9 − β 9 ) 26. Given, x2 − |x + 2| + x > 0 …(i)
= = =3
2 (α 9 − β 9 ) 2 (α 9 − β 9 )
Case I When x+ 2 ≥0
α 2 + β2 ∴ x − x − 2 + x > 0 ⇒ x2 − 2 > 0
2
22. Sum of roots = and product = 1
αβ
⇒ x < − 2 or x > 2
Given, α + β = − p and α 3 + β3 = q
⇒ x ∈ [−2, − 2 ) ∪ ( 2 , ∞ ) …(ii)
⇒ (α + β ) (α 2 − αβ + β 2) = q
Case II When x + 2 < 0
−q
∴ α 2 + β 2 − αβ = ...(i) ∴ x2 + x + 2 + x > 0
p
and (α + β )2 = p2 ⇒ x2 + 2x + 2 > 0

⇒ α 2 + β 2 + 2αβ = p2 ...(ii) ⇒ (x + 1)2 + 1 > 0


From Eqs. (i) and (ii), we get
p3 − 2q p3 + q which is true for all x.
α 2 + β2 = and αβ =
3p 3p ∴ x ≤ − 2 or x ∈ (−∞ , − 2) …(iii)
( p − 2q) x
3 From Eqs. (ii) and (iii), we get
∴ Required equation is, x2 − +1 =0
( p3 + q) x ∈ (−∞ , − 2 ) ∪ ( 2 , ∞ )

⇒ ( p + q) x − ( p − 2q) x + ( p + q) = 0
3 2 3 3 27. Given, log 4 (x − 1) = log 2(x − 3) = log 41/ 2 (x − 3)
23. The equation x − px + r = 0 has roots α, β and the
2
⇒ log 4 (x − 1) = 2 log 4 (x − 3)
α ⇒ log 4 (x − 1) = log 4 (x − 3)2
equation x − qx + r = 0 has roots , 2 β .
2
2
⇒ (x − 3)2 = x − 1
⇒ r = αβ and α + β = p,
⇒ x2 + 9 − 6x = x − 1
α 2q − p 2 (2 p − q)
and + 2β = q ⇒ β = and α = ⇒ x − 7x + 10 = 0
2
2 3 3
2 ⇒ (x − 2)(x − 5) = 0
⇒ αβ = r = (2q − p) (2 p − q) ⇒ x = 2, or x = 5
9
⇒ x = 5 [Q x = 2 makes log (x − 3) undefined].
24. Since, roots are real, therefore D ≥ 0
Hence, one solution exists.
⇒ 4 (a + b + c)2 − 12λ (ab + bc + ca ) ≥ 0
28. Let α , α 2 be the roots of 3x2 + px + 3 = 0
⇒ (a + b + c)2 ≥ 3λ (ab + bc + ca )
Now, S = α + α 2 = − p /3,
⇒ a 2 + b2 + c2 ≥ (ab + bc + ca ) (3λ − 2)
a 2 + b2+ c2 P = α3 = 1
⇒ 3λ − 2 ≤ …(i) ⇒ α = 1, ω , ω 2
ab + bc + ca
Now, α + α 2 = − p/3
b2 + c2 − a 2
Also, cos A = <1 ⇒ ω + ω 2 = − p/3
2bc
rankershalt.com

38 Theory of Equations

⇒ − 1 = − p/3 x2 − (a + b)x + ab
34. Let y =
⇒ p=3 x−c
⇒ yx − cy = x2 − (a + b)x + ab
29. Given, c<0 < b
⇒ x2 − (a + b + y)x + (ab + cy) = 0
Since, α + β = −b …(i)
For real roots, D ≥ 0
and αβ = c …(ii)
⇒ (a + b + y)2 − 4(ab + cy) ≥ 0
From Eq. (ii), c < 0 ⇒ α β < 0
⇒ (a + b)2 + y2 + 2(a + b) y − 4ab − 4cy ≥ 0
⇒ Either α is –ve, β is −ve or α is + ve, β is – ve.
⇒ y2 + 2(a + b − 2c) y + (a − b)2 ≥ 0
From Eq. (i), b > 0 ⇒ − b < 0 ⇒ α + β < 0 ⇒ the sum is
negative. which is true for all real values of y.
⇒ Modulus of negative quantity is > modulus of positive ∴ D ≤0
quantity but α < β is given. 4 (a + b − 2c) − 4 (a − b)2 ≤ 0
2

Therefore, it is clear that α is negative and β is positive ⇒ 4 (a + b − 2c + a − b)(a + b − 2c − a + b) ≤ 0


and modulus of α is greater than modulus of β ⇒ (2a − 2c)(2b − 2c) ≤ 0
⇒ α < 0 < β < |α| ⇒ (a − c)(b − c) ≤ 0
NOTE This question is not on the theory of interval in which root
lie, which appears looking at first sight. It is new type and first ⇒ (c − a )(c − b) ≤ 0
time asked in the paper. It is important for future. The actual ⇒ c must lie between a and b
type is interval in which parameter lie. i.e. a ≤ c ≤ b or b ≤ c ≤ a
30. Since, x + 1 − x − 1 = 4x − 1 35. Since, | x|2 − 3|x| + 2 = 0
⇒ (x + 1) + (x − 1) − 2 x2 − 1 = 4x − 1 ⇒ (|x| − 1) (|x| − 2) = 0
⇒ |x| = 1, 2
⇒ 1 − 2x = 2 x2 − 1 ⇒ 1 + 4x2 − 4x = 4x2 − 4
5 ∴ x = 1, − 1, 2, − 2
⇒ 4x = 5 ⇒ x =
4 Hence, four real solutions exist.
But it does not satisfy the given equation. 36. (x − a ) (x − b) + (x − b) (x − c) + (x − c) (x − a ) = 0
Hence, no solution exists. ⇒ 3x2 − 2 (a + b + c) x + (ab + bc + ca ) = 0
3
(log 2 x )2 + log 2 x −
5 Now, discriminant = 4 (a + b + c)2 − 12 (ab + bc + ca )
31. Given, x 4 4 = 2 = 4 { a 2 + b2 + c2 − ab − bc − ca }
3 5 = 2 {(a − b)2 + (b − c)2 + (c − a )2}
⇒ (log 2 x)2 + log 2 x − = log x 2
4 4 which is always positive.
3 5 1 Hence, both roots are real.
⇒ (log 2 x)2 + log 2 x − =
4 4 2 log 2 x
37. Since, a , b, c > 0 and ax2 + bx + c = 0
⇒ 3(log 2 x) + 4(log 2 x) − 5(log 2 x) − 2 = 0
3 2
−b b2 − 4ac
Put log 2 x = y ⇒ x= ±
∴ 3 y3 + 4 y2 − 5 y − 2 = 0 2a 2a
⇒ ( y − 1) ( y + 2) (3 y + 1) = 0 Case I When b2 − 4ac > 0
⇒ y = 1, −2 , −1 / 3 b2 − 4ac
−b
⇒ log 2 x = 1, − 2, − 1 / 3 ⇒ x= −
1 1 2a 2a
⇒ x = 2, 1/3 ,
2 4 −b b2 − 4ac
and + both roots, are negative.
32. Since, α , β are the roots of x + px + q = 0 and α , β are
2 4 4 2a 2a
the roots of x2 − rx + s = 0. Case II When b2 − 4ac = 0
⇒ α + β = − p ; α β = q; α 4 + β 4 = r and α 4β 4 = s −b
⇒ x= , i.e. both roots are equal and negative
Let roots of x2 − 4qx + (2q2 − r ) = 0 be α ′ and β′ 2a
Now, α ′ β ′ = (2q2 − r ) = 2 (αβ )2 − (α 4 + β 4 ) Case III When b2 − 4ac < 0
= − (α 4 + β 4 − 2α 2β 2) = − (α 2 − β 2)2 < 0 −b 4ac − b2
⇒ x= ±i
⇒ Roots are real and of opposite sign. 2a 2a
2 2 have negative real part.
33. Given, x − =1 − ⇒ x=1
x−1 x−1 ∴ From above discussion, both roots have negative real
But at x = 1, the given equation is not defined. parts.
Hence, no solution exist.
rankershalt.com

Theory of Equations 39

38. Given, x2 + 2 px + q = 0 ⇒ x = 1, 4 [4 is rejected]


∴ α + β = −2 p ... (i) ⇒ x=1 …(ii)
αβ = q ... (ii) Hence, the sum of the roots is 3 + 1 = 4.
And ax + 2bx + c = 0
2 Alternate Solution
1 2b Given,| x − 2|2 + | x − 2| − 2 = 0
∴ α+ =− ... (iii)
β a ⇒ (| x − 2| + 2) (| x − 2| − 1) = 0
α c ∴ | x − 2| = − 2, 1 [neglecting –2]
and = ... (iv)
β a ⇒ | x − 2| = 1 ⇒ x = 3, 1
Now, ( p2 − q) (b2 − ac) ⇒ Sum of the roots = 4
 1
2  40. Since, x2 − 3kx + 2e2 log k − 1 = 0 has product of roots 7.
 α + β 2    α +  
β α ⇒ 2e2 log k − 1 = 7
=   − αβ    −  a2
 
 −2   2  β ⇒ e2 log e k = 4
   ⇒ k2 = 4
 
2 ⇒ k =2 [neglecting −2]
(α − β )2  1
= α −  . a ≥ 0
2
41. If 2 + i 3 is one of the root of x + px + q = 0. Then,
2
16  β
other root is 2 − i 3.
∴ Statement I is true. ⇒ − p =2 + i 3 + 2 − i 3 = 4
 α + β a and q = (2 + i 3 ) (2 − i 3 ) = 7
Again, now pa = −   a = − (α + β )
 2  2 ⇒ ( p, q) = (−4, 7)
a  1 42. The coefficient of x99 in (x − 1)(x − 2)... (x − 100)
and b=− α + 
2  β
= − (1 + 2 + 3 + ... + 100)
1 100
Since, pa ≠ b ⇒ α + ≠α +β =− (1 + 100) = −50(101) = −5050
β 2
⇒ β 2 ≠ 1, β ≠ { −1, 0, 1}, which is correct. 43. P (x) ⋅ Q (x) = (ax2 + bx + c) (−ax2 + bx + c)
Similarly, if c ≠ qa
Now, D1 = b2 − 4ac and D2 = b2 + 4ac
α  1
⇒ a ≠ a α β ⇒ α β −  ≠ 0 Clearly, D1 + D2 = 2b2 ≥ 0
β  β
∴ Atleast one of D1 and D2 is (+ ve). Hence, atleast two
1
⇒ α ≠ 0 and β − ≠0 real roots.
β
Hence, statement is true.
⇒ β ≠ { −1 , 0 , 1 }
44. Given, 2x2 + 3x + 1 = 0
Statement II is true.
Here, D = (3)2 − 4 ⋅ 2 ⋅ 1 = 1 which is a perfect square.
Both Statement I and Statement II are true. But
Statement II does not explain Statement I. ∴ Roots are rational.
Hence, statement is false.
39. Given,|x − 2|2 + |x − 2| − 2 = 0
45. Here, a + b = 10c and c + d = 10a
Case I When x ≥ 2
⇒ (x − 2)2 + (x − 2) − 2 = 0 ⇒ (a − c) + (b − d ) = 10 (c − a )
⇒ x + 4 − 4x + x − 2 − 2 = 0
2 ⇒ (b − d ) = 11 (c − a ) …(i)
⇒ x − 3x = 0
2 Since, ‘c’ is the root of x2 − 10ax − 11b = 0
⇒ x (x − 3) = 0 ⇒ c2 − 10ac − 11b = 0 …(ii)

⇒ x = 0, 3 [0 is rejected] Similarly, ‘a’ is the root of

⇒ x=3 …(i) x2 − 10cx − 11d = 0


Case II When x < 2 ⇒ a 2 − 10ca − 11d = 0 …(iii)

⇒ { − (x − 2)}2 − (x − 2) − 2 = 0 On subtracting Eq. (iv) from Eq. (ii), we get

⇒ (x − 2) − x + 2 − 2 = 0
2 (c2 − a 2) = 11 (b − d ) …(iv)

⇒ x2 + 4 − 4x − x = 0 ∴ (c + a ) (c − a ) = 11 × 11 (c − a ) [from Eq. (i)]

⇒ x − 4x − (x − 4) = 0
2 ⇒ c + a = 121
⇒ x(x − 4) − 1 (x − 4) = 0 ∴ a + b + c + d = 10c + 10a
⇒ (x − 1) (x − 4) = 0 = 10 (c + a ) = 1210
rankershalt.com

40 Theory of Equations

b c − (3x + 2)
46. Since, α + β = − , αβ = ⇒ > 0 ; using number line rule
a a (2x + 1) (x + 1) (x + 2)
B C
and α+δ+β+δ=− , (α + δ ) (β + δ ) = – + – + –
A A
–2 –1 – 2 – 1
Now, α − β = (α + δ ) − (β + δ ) 3 2
⇒ (α − β )2 = [(α + δ ) − (β + δ )]2  2 1
∴ x ∈ (−2, − 1) ∪  − , − 
⇒ (α + β ) − 4αβ = (α + δ + β + δ ) − 4(α + δ ) (β + δ )
2 2  3 2
2 2
 b 4c  B  4C 50. Given,| x2 + 4x + 3|+ 2x + 5 = 0
⇒ −  − = −  −
 a a  A A Case I x2 + 4x + 3 > 0 ⇒ (x < − 3 or x > − 1)
2
b 4c B 4C 2
∴ x + 4x + 3 + 2x + 5 = 0
2
⇒ − = 2−
a2 a A A ⇒ x2 + 6x + 8 = 0 ⇒ (x + 4) (x + 2) = 0
b2 − 4ac B2 − 4 AC ⇒ x = − 4, − 2 [but x < − 3 or x > − 1]
⇒ =
a2 A2 ∴ x = − 4 is the only solution. …(i)
47. Suppose f (x) = Ax2 + Bx + C is an integer, whenever x Case II x2 + 4x + 3 < 0 ⇒ (−3 < x < − 1)
is an integer. ∴ − x2 − 4x − 3 + 2x + 5 = 0
∴ f (0), f (1), f (−1) are integers. ⇒ x2 + 2x − 2 = 0 ⇒ (x + 1)2 = 3
⇒ | x + 1| = 3
⇒ C , A + B + C , A − B + C are integers.
⇒ C , A + B, A − B are integers. ⇒ x = − 1 − 3, −1 + 3 [but x ∈ (−3, − 1)]
⇒ C , A + B, ( A + B) − ( A − B) = 2 A are integers. ∴ x = − 1 − 3 is the only solution. …(ii)
Conversely, suppose 2 A , A + B and C are integers. From Eqs. (i) and (ii), we get
Let n be any integer. We have, x = − 4 and (−1 − 3) are the only solutions.
 n (n − 1)  51. Here, a ≤0
f (n ) = An 2 + Bn + C = 2 A + ( A + B) n + C
 2  Given, x2 − 2 a | x − a | − 3 a 2 = 0
Since, n is an integer, n (n − 1) / 2 is an integer. Also, Case I When x ≥ a
2 A , A + B and C are integers. ⇒ x2 − 2a (x − a ) − 3a 2 = 0
We get f (n ) is an integer for all integer n. ⇒ x2 − 2ax − a 2 = 0
y −1 y −1
48. Given, 2 |y|
− |2 − 1| = 2 +1 ⇒ x = a ± 2a
Case I When y ∈ (−∞ , 0] [as a (1 + 2 ) < a and a (1 − 2 ) > a]
∴ 2 −y
+ (2 y −1
− 1) = 2 y −1
+1 ∴ Neglecting x = a (1 + 2 ) as x ≥ a
⇒ 2−y = 2 ⇒ x = a (1 − 2 ) …(i)
⇒ y = − 1 ∈ (−∞ , 0] …(i) Case II When x < a ⇒ x2 + 2a (x − a ) − 3a 2 = 0
Case II When y ∈(0, 1] ⇒ x2 + 2 ax − 5a 2 = 0 ⇒ x = − a ± 6a
∴ 2y + (2y − 1 − 1) = 2y − 1 + 1 [as a ( 6 − 1) < a and a (−1 − 6 ) > a]
⇒ 2 =2
y ∴ Neglecting x = a (−1 − 6 ) ⇒ x = a ( 6 − 1) ...(ii)
⇒ y = 1 ∈ (0, 1] …(ii) From Eqs. (i) and (ii), we get
Case III When y ∈ (1, ∞ ) x = { a (1 − 2 ), a ( 6 − 1)}
y −1 y −1 x 2 −3 2
−3
∴ 2 −2
y
+ 1 =2 +1 52. Given, (5 + 2 6 ) + (5 − 2 6 )x = 10 …(i)
y −1
⇒ 2 − 2 ⋅2
y
=0 2
−3 2
−3 1
Put y = (5 + 2 6 )x ⇒ (5 − 2 6 )x =
⇒ 2y − 2y = 0 true for all y > 1 …(iii) y
From Eqs. (i), (ii) and (iii), we get 1
From Eq. (i), y + = 10
y ∈{ −1} ∪ [1, ∞ ). y
2x 1 ⇒ y2 − 10 y + 1 = 0 ⇒ y=5±2 6
49. Given, >
2x2 + 5x + 2 x + 1 ⇒ (5 + 2 6 ) x 2 −3
=5 + 2 6
2x 1
⇒ − >0
2
−3
or (5 + 2 6 )x =5 −2 6
(2x + 1) (x + 2) (x + 1)
2x (x + 1) − (2x + 1) (x + 2) ⇒ x −3 =1
2
or x − 3 = −1
2

⇒ >0
(2x + 1) (x + 2) (x + 1) ⇒ x = ± 2 or x = ± 2
⇒ x = ± 2, ± 2
rankershalt.com

Theory of Equations 41

53. Let α , β are roots of ax2 + bx + c = 0 = (1 + γ 2 + γp)(1 − δp + δ 2) = (− qγ + γp)(−δp − δq)


Given, α = βn [Q γ 2 + qγ + 1 = 0 and δ 2 + qδ + 1 = 0]
c c = (q2 − p2)(γδ ) = q2 − p2 [Q γ δ = 1]
⇒ αβ = ⇒ βn + 1 =
a a
1/( n + 1 ) 57. α =α + 1
2
 c
⇒ β=  β2 = β + 1
 a
a n = pα n + qβ n
It must satisfy ax2 + bx + c = 0
2 /( n + 1 ) 1 /( n + 1 ) = p(α n − 1 + α n − 2) + q(β n − 1 + β n − 2)
 c  c
i.e. a  + b  + c=0 = an − 1 + an − 2
 a  a
∴ a12 = a11 + a10
a . c2/( n + 1) b. c1/( n + 1)
⇒ + 1/( n + 1) + c = 0 1+ 5 1− 5
a 2/( n + 1) a 58. α= ,β =
2 2
c1/( n + 1)  a . c1/( n + 1) c. a1/( n + 1) 
⇒  1/( n + 1) + b + 1/( n + 1)  = 0 a 4 = a3 + a 2
a1/( n + 1)  a c  = 2a 2 + a1
⇒ a n/( n + 1)c1/( n + 1) + b + cn/( n + 1)a1 / ( n + 1) = 0 = 3a1 + 2a 0
⇒ (a nc)1/( n + 1) + (cna )1/( n + 1) + b = 0 28 = p(3α + 2) + q(3β + 2)
54. Since, α , β are the roots of x2 + px + q = 0 3  3 5
28 = ( p + q) + 2 + ( p − q) 
and γ , δ are the roots of x + rx + s = 0 2 2   2 
∴ α + β = − p, αβ = q ∴ p− q =0
7
and γ + δ = − r, γδ = s and ( p + q) × = 28
2
Now, (α − γ )(α − δ )(β − γ )(β − δ )
⇒ p+ q =8
= [α 2 − (γ + δ ) α + γδ ][β 2 − (γ + δ ) β + γδ ]
⇒ p= q =4
= (α 2 + rα + s)(β 2 + rβ + s)
∴ p + 2q = 12
= (αβ )2 + r (α + β )αβ + s(α 2 + β 2) + αβr 2 + rs(α + β ) + s2
= q2 − rqp + s( p2 − 2q) + qr 2 − rsp + s2 Topic 2 Common Roots
= (q − s) − rqp − rsp + sp + qr
2 2 2
1 Given α, β and γ are three consecutive terms of a
55. The given equation can be rewritten as non-constant GP.
2 1 3 Let α = α, β = αr , γ = αr 2, { r ≠ 0, 1}
+ + = 0 [Q b = a 2x, given] and given quadratic equation is
log a x log a ax log a a 2x
αx2 + 2 βx + γ = 0 …(i)
2 1 3
⇒ + + =0 On putting the values of α,β, γ in Eq. (i), we get
log a x 1 + log a x 2 + log a x αx2 + 2αrx + αr 2 = 0
2 1 3 ⇒ x2 + 2rx + r 2 = 0
⇒ + + = 0, where t = log a x
t 1+ t 2+ t ⇒ (x + r )2 = 0
⇒ 2 (1 + t ) (2 + t ) + 3 t (1 + t ) + t (2 + t ) = 0 ⇒ x=−r
⇒ 6 t 2 + 11 t + 4 = 0 Q The quadratic equations αx2 + 2 βx + γ = 0 and
x2 + x − 1 = 0 have a common root, so x = − r must be root
⇒ (2 t + 1) (3 t + 4) = 0 of equation x2 + x −1 = 0, so
1 4
⇒ t=− or − r2 − r − 1 = 0 …(ii)
2 3 Now, α (β + γ ) = α (αr + αr 2)
1 4
∴ log a x = − or log a x = − = α 2 (r + r 2)
2 3
From the options,
⇒ x = a −1/ 2 βγ = αr ⋅ αr 2 = α 2r3 = α 2 (r + r 2)
−4/3
or x =a [Q r 2 − r − 1 = 0 ⇒ r3 = r + r 2]
∴ α (β + γ ) = βγ
56. Since, α + β = − p, αβ = 1 and γ + δ = − q, γδ = 1
2. Given equations are x2 + 2x + 3 = 0 …(i)
Now, (α − γ )(β − γ )(α + δ )(β + δ )
and ax + bx + c = 0
2
…(ii)
= {αβ − γ (α + β ) + γ 2}{αβ + δ (α + β ) + δ 2}
Since, Eq. (i) has imaginary roots, so Eq. (ii) will also
= {1 − γ (− p) + γ 2}{1 + δ ( − p) + δ 2} have both roots same as Eq. (i).
rankershalt.com

42 Theory of Equations

Thus,
a b c
= = ⇒ x = α β α , α β β are the roots
1 2 3 ⇒ x = α 2β , αβ 2 are the roots
Hence, a : b : c is 1 : 2 : 3. Divide the Eq. (i) by a3 , we get
3. If a1x + b1x + c1 = 0
2
b c  c
3
x2 + ⋅ ⋅ x+   =0
and a 2x2 + b2x + c2 = 0 a a  a
have a common real root, then ⇒ x2 − (α + β ) ⋅ (αβ ) x + (αβ )3 = 0
⇒ (a1c2 − a 2c1 ) = (b1c2 − b2c1 ) (a1b2 − a 2b1 )
2
⇒ x2 − α 2βx − αβ 2 x + (αβ )3 = 0
x + bx − 1 = 0
2
⇒ x (x − α 2β ) − αβ 2 (x − α 2β ) = 0
∴  have a common root.
x2 + x + b = 0  ⇒ (x − α 2β )(x − αβ 2) = 0
⇒ (1 + b)2 = (b2 + 1) (1 − b) ⇒ x = α 2β , αβ 2 which is the required answer.
⇒ b2 + 2b + 1 = b2 − b3 + 1 − b Alternate Solution
⇒ b + 3b = 0
3 Since, a3 x2 + abcx + c3 = 0
∴ b (b2 + 3) = 0 − abc ± (abc)2 − 4 . a3 ⋅ c3
⇒ x=
⇒ b = 0, ± 3 i 2 a3
4. Given equations are x2 + ax + b = 0 and − (b/a )(c/a ) ± (b/a )2(c/a )2 − 4(c/a )3
⇒ x=
x2 + bx + a = 0 have common root 2

On subtracting above equations, we get (α + β ) (α β ) ± (α + β )2 (α β )2 − 4(α β )3


⇒ x=
(a − b) x + (b − a ) = 0 2
⇒ x=1 (α + β )(αβ ) ± α β (α + β )2 − 4 αβ
⇒ x=
∴ x = 1 is the common root. 2
⇒ 1 + a + b =0  (α + β ) ± (α − β )2 
⇒ a + b = −1 ⇒ x = αβ  
 2 
5. Since, (x − r ) is a factor of the polynomial
 (α + β ) ± (α − β ) 
⇒ x = αβ
f (x) = a nxn + a n − 1xn − 1 + ... + a 0  2 
Then, x = r is root of f ′ (x) = 0 repeated (m − 1) times. α + β + α − β α + β − α + β 
⇒ x = αβ ,
Hence, statement is false.  2 2 
2α 2β 
Topic 3 Transformation of Roots ⇒ x = αβ ,
 2 2 
1. Given, α , β are the roots of (x − a )(x − b) − c = 0 ⇒ x = α 2 β , α β 2 which is the required answer.
⇒ (x − a )(x − b) − c = (x − α ) (x − β )
⇒ (x − a )(x − b) = (x − α )(x − β ) + c Topic 4 Graph of Quadratic Expression
⇒ a , b are the roots of equation (x − α )(x − β ) + c = 0 1. Given parabola is y2 = 4x,
2. Since, ax2 + bx + c = 0 has roots α and β. Since, X lies on the parabola, so let the coordinates of X
be (t 2, 2t ). Thus, the coordinates of the vertices of the
⇒ α + β = − b /a triangle PXQ are P (4,–4), X (t 2,2t ) and Q (9, 6).
and αβ = c / a Y
Now, a3 x2 + abcx + c3 = 0 …(i) Q(9,6)
y2=4x
X (t 2,2t)
2
On dividing the equation by c , we get
a3 2 abcx c3
x + 2 + 2 =0 X′ X
c2 c c O
2
 ax  ax
⇒ a   + b   + c=0
 c  c
P(4,–4)
ax
⇒ = α , β are the roots Y′
c
4 −4 1
c c 1
⇒ x = α , β are the roots ∴Area of ∆PXQ = t 2 2t 1
a a 2
9 6 1
rankershalt.com

Theory of Equations 43

1 3. According to given information, we have the following


= [4(2t − 6) + 4(t 2 − 9) + 1(6t 2 − 18t ]
2 graph
1 Y
= |[8t − 24 + 4t 2 − 36 + 6t 2 − 18t ]|
2
= |5t 2 − 5t − 30| = |5(t + 2) (t − 3)|
Now, as X is any point on the arc POQ of the parabola,
therefore ordinate of point X, 2t ∈ (− 4, 6) ⇒ t ∈ (− 2, 3). O 1 5
X
∴ Area of ∆PXQ = − 5(t + 2) (t − 3) = − 5t 2 + 5t + 30
[Q| x − a | = − (x − a ), if x < a] Now, the following conditions should satisfy
The maximum area (in square units) (i) D > 0 ⇒ b2 − 4ac > 0
25 − 4(− 5) (30)  125 ⇒ m2 − 4 × 1 × 4 > 0
=− = 4
 4(− 5)  ⇒ m2 − 16 > 0
[Q Maximum value of quadratic expression ⇒ (m − 4) (m + 4) > 0
D ⇒ m ∈ (− ∞ , − 4) ∪ (4, ∞ )
ax2 + bx + c, when a < 0 is − ]
4a (ii) The vertex of the parabola should lie
between x = 1and x = 5
2. Let f (x) = (c − 5)x2 − 2 cx + (c − 4) = 0. b m
Then, according to problem, the graph of y = f (x) will be ∴ − ∈ (1, 5) ⇒1 < < 5 ⇒ m∈ (2, 10)
2a 2
either of the two ways, shown below.
(iii) f (1) > 0 ⇒1 − m + 4 > 0
⇒ m < 5 ⇒ m ∈ (−∞ , 5)
 29
(iv) f (5) > 0 ⇒ 25 − 5m + 4 > 0 ⇒ 5m < 29 ⇒m ∈  − ∞, 
O 2 3
 5
Or From the values of m obtained in (i), (ii), (iii) and (iv), we
get m ∈ (4, 5).

O 2 3 –∞ ∞
–4 2 4 5 29/5

In both cases f (0). f (2) < 0 and f (2) f (3) < 0


Now, consider f (0) f (2) < 0 4. Put t = x − [x] = { X }, which is a fractional part function
and lie between 0 ≤ { X } < 1 and then solve it.
⇒ (c − 4) [4 (c − 5) − 4c + (c − 4)] < 0
Given, a ∈ R and equation is
⇒ (c − 4) (c − 24) < 0
−3 { x − [x]}2 + 2 { x − [x]} + a 2 = 0
⇒ c ∈ (4, 24) … (i)
+ – + Let t = x − [x], then equation is
4 24 −3 t 2 + 2 t + a 2 = 0

Similarly, f (2) ⋅ f (3) < 0 1 ± 1 + 3a 2


⇒ t=
⇒ [4 (c − 5) − 4c + (c − 4)] 3
[9(c − 5) − 6c + (c − 4)] < 0 Q t = x − [x] = { X } [fractional part]
⇒ (c − 24) (4c − 49) < 0 ∴ 0 ≤ t ≤1
+ – + 1 ± 1 + 3a 2
0≤ ≤1
49/4 24 3
 49  Taking positive sign, we get
⇒ c ∈ , 24 …(ii)
4  1 + 1 + 3a 2
0≤ <1 [Q{ x } > 0]
From Eqs. (i) and (ii), we get 3
 49 
c ∈ , 24 ⇒ 1 + 3a 2 < 2 ⇒ 1 + 3a 2 < 4
4 
∴Integral values of c are 13, 14, ……, 23. ⇒ a 2 − 1 < 0 ⇒ (a + 1) (a − 1) < 0
Thus, 11 integral values of c are possible. ∴ a ∈ (−1, 1), for no integer solution of a, we consider
(−1, 0) ∪ (0, 1)
rankershalt.com

44 Theory of Equations

5. As we know, ax2 + bx + c > 0 for all x ∈ R, iff ∴ g (x) = f (x) + f ′ (x) + f ′ ′ (x)
a > 0 and D < 0. ⇒ g (x) = ax2 + bx + c + 2ax + b + 2a
Given equation is x2 + 2ax + (10 − 3a ) > 0, ∀ x ∈ R ⇒ g (x) = ax2 + x (b + 2a ) + (c + b + 2a )
Now, D <0 whose discriminant
⇒ 4a − 4(10 − 3a ) < 0
2 = (b + 2a )2 − 4a (c + b + 2a )
⇒ 4(a 2 + 3a − 10) < 0 = b2 + 4a 2 + 4ab − 4ac − 4ab − 8a 2
⇒ (a + 5)(a − 2) < 0 ⇒ a ∈ (−5, 2) = b2 − 4a 2 − 4ac = (b2 − 4ac) − 4a 2 < 0 [from Eq. (i)]
∴ g (x) > 0 ∀ x, as a > 0 and discriminant < 0.
Thus, g (x) > 0, ∀ x ∈ R.
y = (x – a)(x – b) –1
9. Given,
6. x2 + (a − b) x + (1 − a − b) = 0 has real and unequal roots.
α a b β
⇒ D >0
1
⇒ (a − b) − 4(1) (1 − a − b) > 0
2

From graph, it is clear that one of the roots of


⇒ a 2 + b2 − 2ab − 4 + 4a + 4b > 0
(x − a )(x − b) − 1 = 0 lies in (−∞ , a ) and other lies in
(b, ∞ ). Now, to find the values of ‘a’ for which equation has
unequal real roots for all values of b.
7. Let f (x) = x − 2ax + a + a − 3
2 2
i.e. Above equation is true for all b.
Since, both root are less than 3. or b2 + b(4 − 2a ) + (a 2 + 4a − 4) > 0, is true for all b.
⇒ α < 3, β < 3 ∴ Discriminant, D < 0
⇒ Sum, S = α + β < 6 ⇒ (4 − 2a )2 − 4 (a 2 + 4a − 4) < 0
α+β ⇒ 16 − 16a + 4a 2 − 4a 2 − 16a + 16 < 0
⇒ <3
2 ⇒ −32a + 32 < 0 ⇒ a >1
2a
⇒ <3 10. Y
2 a<0
⇒ a <3 …(i) y = ax2 + bx + c
Again, product, P = αβ
⇒ P <9 ⇒ αβ < 9 –1 1
X
⇒ a2 + a − 3 < 9 α 0 β Y
⇒ a + a − 12 < 0
2
a>0
⇒ (a − 3) (a + 4) < 0
⇒ −4 < a < 3 …(ii) y = ax2 + bx + c
α β 3
Again, D = B − 4 AC ≥ 0
2
α β
X
–1 0 1
⇒ (−2a ) − 4 ⋅ 1 (a + a − 3) ≥ 0
2 2

⇒ 4a 2 − 4a 2 − 4a + 12 ≥ 0
⇒ −4a + 12 ≥ 0 ⇒ a ≤ 3 …(iii)
Again, a f (3) > 0
⇒ 1 [(3)2 − 2a (3) + a 2 + a − 3] > 0 From figure, it is clear that, if a > 0, then f (−1) < 0 and
⇒ 9 − 6a + a + a − 3 > 0
2 f (1) < 0 and if a < 0, f (−1) > 0 and f (1) > 0. In both
cases, af (−1) < 0 and af (1) < 0.
⇒ a 2 − 5a + 6 > 0
⇒ a (a − b + c) < 0 and a (a + b + c) < 0
⇒ (a − 2) (a − 3) > 0
On dividing by a 2, we get
∴ a ∈ (−∞ , 2) ∪ (3, ∞ ) …(iv)
b c b c
From Eqs. (i), (ii), (iii) and (iv), we get 1 − + < 0 and 1+ + <0
a a a a
a ∈ (−4, 2). On combining both, we get
NOTE There is correction in answer a < 2 should be −4 < a < 2 . b c
1 ± + <0
8. Let f (x) = ax2 + bx + c > 0, ∀ x ∈ R a a
b c
⇒ a >0 ⇒ 1 +  + < 0
and b2 − 4ac < 0 …(i) a a
rankershalt.com

Theory of Equations 45

11. Since, x2 − 3x + 2 > 0 and x2 − 2x − 4 ≤ 0 ∴The above inequality holds, iff


⇒ (x − 1) (x − 2) > 0 and x − 2x + 1 ≤ 5
2
(sin x − 1)2 + 4 = 2 = 2 sin 2 y
⇒ (x < 1 or x > 2) and (1 − 5 ≤ x ≤ 1 + 5 ) ⇒ sin x = 1 and sin 2 y = 1
∴ x ∈ [1 − 5 , 1) ∪ [1 + 5 , 2) ⇒ sin x = |sin y| [from the options]
12. (i) Given, x2 − 8kx + 16 (k2 − k + 1) = 0
3. Key Idea Reduce the given equation into quadratic equation.
Now, D = 64 { k2 − (k2 − k + 1)} = 64 (k − 1) > 0
k >1 Given equation is
b 8k | x − 2| + x ( x − 4) + 2 = 0
(ii) − >4 ⇒ >4 ⇒ k >1
2a 2 ⇒ | x − 2| + x − 4 x + 4 = 2
(iii) f (4) ≥ 0 ⇒ | x − 2| + ( x − 2)2 = 2
⇒ 16 − 32k + 16 (k2 − k + 1) ≥ 0 ⇒ (| x − 2|)2 + | x − 2| − 2 = 0
⇒ k − 3k + 2 ≥ 0
2
Let| x − 2| = y, then above equation reduced to
⇒ (k − 2) (k − 1) ≥ 0 y2 + y − 2 = 0 ⇒ y2 + 2 y − y − 2 = 0
⇒ k ≤1 or k ≥ 2 ⇒ y( y + 2) − 1( y + 2) = 0 ⇒ ( y + 2)( y − 1) = 0
⇒ y = 1, − 2
Hence, k =2
∴ y=1 [Q y = | x − 2| ≥ 0]
⇒ | x − 2| = 1
Topic 5 Some Special Forms ⇒ x −2 = ±1
1. Given equation 5 + |2x − 1| = 2x (2x − 2) ⇒ x = 3 or 1
Case I ⇒ x = 9 or 1
If 2x − 1 ≥ 0 ⇒ x ≥ 0 , ∴ Sum of roots = 9 + 1 = 10
then 5 + 2x − 1 = 2x (2x − 2) 4. Let f (x) = 2x3 + 3x + k
Put 2x = t, then On differentiating w.r.t. x, we get
5 + t − 1 = t 2 − 2t ⇒ t 2 − 3t − 4 = 0 f ′ (x) = 6x2 + 3 > 0, ∀ x ∈ R
⇒ t − 4t + t − 4 = 0 ⇒ t (t − 4) + 1(t − 4) = 0
2
⇒ f (x) is strictly increasing function.
⇒ t = 4 or − 1 ⇒ t = 4 (Q t = 2x > 0)
⇒ f (x) = 0 has only one real root, so two roots are not
⇒ 2 =4⇒x=2 >0
x
possible.
⇒ x = 2 is the solution.
5. Since, α is a root of a 2x2 + bx + c = 0
Case II
⇒ a 2α 2 + bα + c = 0 ... (i)
If 2x − 1 < 0 ⇒ x < 0 ,
and β is a root of a 2x2 − bx − c = 0
then 5 + 1 − 2x = 2x (2x − 2)
Put 2x = y, then 6 − y = y2 − 2 y ⇒ a 2β 2 − bβ − c = 0 ... (ii)
⇒ y2 − y − 6 = 0 ⇒ y2 − 3 y + 2 y − 6 = 0 Let f (x) = a 2x2 + 2bx + 2c
⇒ ( y + 2) ( y − 3) = 0 ⇒ y = 3 or − 2 ∴ f (α ) = a 2α 2 + 2bα + 2c
⇒ y = 3(as y = 2x > 0) ⇒ 2x = 3 = a 2α 2 − 2a 2α 2 = − a 2α 2
⇒ x = log 2 3 > 0 [from Eq. (i)]
So, x = log 2 3 is not a solution. and f (β ) = α 2β 2 + 2bβ + 2c
Therefore, number of real roots is one. = a 2β 2 + 2a 2β 2 = 3a 2β 2 [from Eq. (ii)]
2. Given, inequality is ⇒ f (α ) f (β ) < 0
sin 2 x − 2sin x + 5 1 f (x) must have a root lying in the open interval (α , β ).
2 ⋅ ≤1
4 sin 2 y ∴ α < γ <β

⇒2
(sin x − 1 )2 + 4
⋅2 −2sin 2 y
≤1 6. Let f (x) = ax3 + bx2 + cx + d …(i)
(sin x − 1 )2 + 4 2 ∴ f (0) = d and f (1) = a + b + c + d = d
⇒2 ≤ 22sin y
[Q a + b + c = 0]
⇒ (sin x − 1)2 + 4 ≤ 2 sin 2 y ∴ f (0) = f (1)
[if a > 1 and a ≤ a ⇒ m ≤ n]
m n f is continuous in the closed interval [0, 1] and f is
derivable in the open interval (0, 1).
Q Range of (sin x − 1) + 4 is [2, 2 2 ] 2
Also, f (0) = f (1).
and range of 2 sin 2 y is [0, 2]. ∴ By Rolle’s theorem, f ′ (α ) = 0 for 0 < α < 1
rankershalt.com

46 Theory of Equations

Now, f ′ (x) = 3ax2 + 2bx + c From Eqs. (i) and (ii), we get
⇒ f ′ (α ) = 3aα 2 + 2bα + c = 0  1 −1   1 1 
α ∈ − ,  ∪  , 
 2 5   5 2
∴ Eq. (i) has exist atleast one root in the interval (0, 1).
10. PLAN
Thus, f ′ (x) must have root in the interval (0, 1) or
(i) Concepts of curve tracing are used in this question.
3ax2 + 2bx + c = 0 has root ∈ (0, 1).
(ii) Number of roots are taken out from the curve traced.
7. Given, x12 − x9 + x4 − x + 1 > 0 Let y = x5 − 5x
Here, three cases arises: (i) As x → ∞, y → ∞ and as x → − ∞, y → − ∞
Case I When x ≤ 0 ⇒ x12 > 0, − x9 > 0, x4 > 0, − x > 0 (ii) Also, at x = 0, y = 0, thus the curve passes through
the origin.
∴ x12 − x9 + x4 − x + 1 > 0, ∀ x ≤ 0 …(i)
dy
(iii) = 5x4 − 5 = 5 (x4 − 1) = 5 (x2 − 1) (x2 + 1)
Case II When 0 < x ≤ 1 dx
x9 < x4 and x < 1 ⇒ − x9 + x4 > 0 and 1 − x > 0 = 5 (x − 1) (x + 1) (x2 + 1)
∴ x − x + x − x + 1 > 0, ∀ 0 < x ≤ 1
12 9 4
…(ii)
+ – +
Case III When x > 1 ⇒ x > x and x > x 12 9 4
–1 1
∴ x12 − x9 + x4 − x + 1 > 0, ∀ x > 1 …(iii) dy
From Eqs. (i), (ii) and (iii), the above equation holds for Now, > 0 in (− ∞ , − 1) ∪ (1, ∞ ), thus f (x) is
dx
all x ∈ R. increasing in these intervals.
8. Consider, dy
Also, < 0 in (− 1, 1), thus decreasing in (− 1, 1).
1 dx
f (x) = ∫ (1 + cos 8 x)(ax2 + bx + c) dx
0 (iv) Also, at x = − 1, dy /dx changes its sign from + ve to
Obviously, f (x) is continuous and differentiable in the –ve.
interval [1, 2]. ∴ x = − 1 is point of local maxima.
Similarly, x = 1 is point of local minima.
Also, f (1) = f (2) [given]
Local maximum value, y = (− 1)5 − 5 (−1) = 4
∴ By Rolle’s theorem, there exist atleast one point Local minimum value, y = (1)5 − 5(1) = − 4
k ∈ (1, 2), such that f ′ (k) = 0.
(–1, 4)
Now, f ′ (x) = (1 + cos 8 x)(ax2 + bx + c)
f ′ (k) = 0
–1
⇒ (1 + cos k)(ak + bk + c) = 0
8 2
(1, – 4)
⇒ ak2 + bk + c = 0 [as (1 + cos 8 k) ≠ 0]
Now, let y = − a
∴ x = k is root of ax2 + bx + c = 0,
As evident from the graph, if − a ∈ (− 4, 4)
where k ∈ (1, 2) i.e. a ∈ (− 4, + 4)
9. Given, x1 and x2 are roots of αx2 − x + α = 0. Then, f (x) has three real roots and if − a > 4
1 or − a < − 4, then f (x) has one real root.
∴ x1 + x2 = and x1x2 = 1
α i.e. for a < − 4 or a > 4, f (x) has one real root.
Also, x1 − x2 < 1 11. Given, f (x) = 4x3 + 3x2 + 2x + 1
⇒ |x1 − x2|2 < 1 ⇒ (x1 − x2)2 < 1 f ′ (x) = 2 (6x2 + 3x + 1)
or (x1 + x2)2 − 4x1x2 < 1
1 1 ⇒ D = 9 − 24 < 0
⇒ − 4 < 1 or <5 Hence, f (x) = 0 has only one real root.
α2 α2
 1 3 4
⇒ 5α 2 − 1 > 0 or ( 5 α − 1) ( 5 α + 1) > 0 f −  = 1 −1 + − > 0
 2 4 8
+ – +
 3 6 27 108
f −  = 1 − + −
–1/√5 1/√5
 4 4 16 64
64 − 96 + 108 − 108
 1  1  = <0
∴ α ∈  −∞ , −  ∪ , ∞ …(i) 64
 5  5 
 3 1
D >0 f (x) changes its sign in  − , − 
Also,  4 2
 1 1
⇒ 1 − 4α > 0 or α ∈  − , 
2
…(ii)  3 1
 2 2 Hence, f (x) = 0 has a root in  − , −  .
 4 2
rankershalt.com

Theory of Equations 47

1/ 2 t 3/ 4
12. ∫0 f (x) dx < ∫ f (x) dx < ∫ f (x) dx 16. For two distinct roots, 1 + ln k < 0 (k > 0)
0 0
1
Now, ∫ f (x) dx = ∫ (1 + 2x + 3x2 + 4x3 ) dx ln k < −1 ⇒ k<
e
= x + x2 + x3 + x4  1
Hence, k ∈ 0, 
 e

17. Let f (x) = (x − a ) (x − c) + 2 (x − b) (x − d )


1 Here, f (a ) = + ve
f (b) = − ve
1 t 3 f (c) = − ve
2 4 f (d ) = + ve
1/ 2 15 3 3/ 4 530 ∴ There exists two real and distinct roots one in the
⇒ ∫0 f (x) dx = > ,
16 4 ∫0 f (x) dx =
256
<3
interval (a , b) and other in (c, d ).
13. As, f ′′ (x) = 2 (12x + 3) Hence, statement is true.

1 18. Let f (x) = 4x3 − 3x − p …(i)


f ′ (x) > 0, when x > − and 3
4  1  1  1 4 3
Now, f   =4   −3   − p= − − p
1  2  2  2 8 2
f ′ (x) < 0, when x < − .
4
= − (1 + p)
∴ It could be shown as f (1) = 4(1)3 − 3(1) − p = 1 − p
1  1
⇒ f   . f (1) = − (1 + p)(1 − p)
 2
= ( p + 1)( p − 1) = p2 − 1
S
–3 –1 1 3 Which is ≤ 0 , ∀ p ∈ [−1, 1].
4 2 2 4 1 
∴ f (x) has atleast one root in ,1 .
2 
14. Let y = x intersect the curve y = kex at exactly one point
when k ≤ 0 . Now, f ′ (x) = 12x2 − 3 = 3 (2x − 1) (2x + 1)
Y 3 1  1 1 
=  x −   x +  > 0 in ,1
4 2  2 2 

⇒ f (x) is an increasing function in [1 /2, 1]


X′ X Therefore, f (x) has exactly one root in [1 /2, 1] for any
p ∈ [−1, 1].
Now, let x = cos θ
1   π
∴ x ∈ , 1 ⇒ θ ∈ 0,
Y′ 2   3 
15. Let f (x) = ke − x x
From Eq. (i),
f ′ (x) = kex − 1 = 0 4 cos3 θ − 3 cos θ = p ⇒ cos 3θ = p
⇒ x = − ln k ⇒ 3θ = cos −1 p
f ′ ′ (x) = ke x
1
⇒ θ = cos −1 p
∴ [ f ′ ′ (x)] x = − ln k = 1 > 0 3
f (− ln k) = 1 + ln k 1 
Hence, ⇒ cos θ = cos  cos −1 p
3 
For one root of given equation
1 + ln k = 0 1 
⇒ x = cos  cos −1 p
1 3 
⇒ k=
e

Download Chapter Test


http://tinyurl.com/y67a3jgc or
rankershalt.com

3
Sequences and Series
Topic 1 Arithmetic Progression (AP)
Objective Questions I (Only one correct option) (a)
1
(b)
1
+
1
(c) 1 (d) 0
mn m n
1. If a1 , a 2, a3 , ... , a n are in AP and a1 + a 4 + a7 + ... + a16
= 114 , then a1 + a 6 + a11 + a16 is equal to Analytical and Descriptive Question
(2019 Main, 10 April I)
(a) 64 (b) 76 5. If a1 , a 2 ..... , a n are in arithmetic progression, where
(c) 98 (d) 38 ai > 0, ∀ i, then show that
2. If 19th term of a non-zero AP is zero, then its (49th 1 1
+ + ...
term) : (29th term) is (2019 Main, 11 Jan II) a1 + a 2 a 2 + a3
(a) 1 : 3 (b) 4 : 1 1 n −1
(c) 2 : 1 (d) 3 : 1
+ =
an − 1 + an a1 + a n
3. For any three positive real numbers a , b and c, if (1982, 2M)
9 (25a 2 + b2) + 25 (c2 − 3ac) = 15b (3a + c), then (2017 Main)
(a) b, c and a are in GP True/False
(b) b, c and a are in AP
6. n1 , n2, K , n p are p positive integers, whose sum is an
(c) a , b and c are in AP
even number, then the number of odd integers among
(d) a , b and c are in GP
them is odd. (1985, 1M)
4. If Tr is the r th term of an AP, for r = 1, 2, 3, .... . If for
1
some positive integers m and n, we have Tm = and Integer Answer Type Question
n
1 7. The sides of a right angled triangle are in arithmetic
Tn = , then Tmn equals
m (1998, 2M) progression. If the triangle has area 24, then what is the
length of its smallest side? (2017 Adv.)

Topic 2 Sum of n Terms of an AP


Objective Questions I (Only one correct option) (a) − 153 (b) − 133
(c) − 131 (d) − 135
1. If a1 , a 2, a3 , ... are in AP such that a1 + a7 + a16 = 40,
then the sum of the first 15 terms of this AP is 4. If the sum and product of the first three terms in an AP
(2019 Main, 12 April II) are 33 and 1155, respectively, then a value of its 11th
(a) 200 (b) 280 (c) 120 (d) 150 term is (2019 Main, 9 April II)
(a) 25 (b) –36
2. Let S n denote the sum of the first n terms of an AP. If (c) –25 (d) –35
S 4 = 16 and S 6 = − 48, then S10 is equal to
(2019 Main, 12 April I) 5. Let the sum of the first n terms of a non-constant AP
n (n − 7)
(a) − 260 (b) − 410 (c) − 320 (d) − 380 a1 , a 2, a3 .....be 50n + A, where A is a constant.
2
3. For x ∈ R, let [x] denote the greatest integer ≤ x, then the If d is the common difference of this AP, then the
sum of the series ordered pair (d , a50 ) is equal to (2019 Main, 9 April I)
 1  1 1   1 2   1 99  (a) (A, 50 + 46A) (b) (50, 50 + 45A)
− 3  + − 3 − 100  + − 3 − 100  + … + − 3 − 100  is (c) (50, 50 + 46A) (d) (A, 50 + 45A)
(2019 Main, 12 April I)
rankershalt.com

Sequences and Series 49

6. The sum of all two digit positive numbers which when 14. Which one of the following is a correct statement ?
divided by 7 yield 2 or 5 as remainder is (a) Q1 , Q2 , Q3 ,... are in an AP with common difference 5
(2019 Main, 10 Jan I) (b) Q1 , Q2 , Q3 ,... are in an AP with common difference 6
(a) 1256 (b) 1465 (c) 1356 (d) 1365 (c) Q1 , Q2 , Q3 ,... are in an AP with common difference 11
30 (d) Q1 = Q2 = Q3 = ...
7. Let a1 , a 2, ..... a30 be an AP, S = ∑ ai and
i =1
15 Fill in the Blanks
T= ∑ a( 2 i − 1). If a5 = 27 and S − 2T = 75, 15. Let p and q be the roots of the equation x2 − 2x + A = 0
i =1
and let r and s be the roots of the equation
then a10 is equal to (2019 Main, 9 Jan I)
x2 − 18x + B = 0. If p < q < r < s are in arithmetic
(a) 42 (b) 57 progression, then A = … and B = … . (1997, 2M)
(c) 52 (d) 47
16. The sum of the first n terms of the series
8. Let bi > 1 for i = 1, 2, ... , 101 . Suppose log e b1 , log e b2, n (n + 1)2
... , log e b101 are in AP with the common difference log e 2 12 + 2 ⋅ 22 + 32 + 2 ⋅ 42 + 52 + 2 ⋅ 62 + K is , when
2
. Suppose a1 , a 2, ... , a101 are in AP, such that a1 = b1 and n is even. When n is odd, the sum is .... . (1988, 2M)
a51 = b51. If t = b1 + b2 + ... + b51 and
s = a1 + a 2 + ... + a51, then (2016 Adv.) 17. The sum of integers from 1 to 100 that are divisible by 2
(a) s > t and a101 > b101 (b) s > t and a101 < b101 or 5 is …… (1984, 2M)

(c) s < t and a101 > b101 (d) s < t and a101 < b101
Analytical & Descriptive Questions
9. If the sum of first n terms of an AP is cn 2, then the sum 18. The fourth power of the common difference of an
of squares of these n terms is (2009) arithmetic progression with integer entries is added to
n (4n 2 − 1) c2 n (4n 2 + 1) c2 the product of any four consecutive terms of it. Prove
(a) (b)
6 3 that resulting sum is the square of an integer.(2000, 4M)
n (4n 2 − 1) c2 n (4n 2 + 1) c2 19. The real numbers x1 , x2, x3 satisfying the equation
(c) (d)
3 6
x3 − x2 + βx + γ = 0 are in AP. Find the intervals in
10. If the sum of the first 2n terms of the AP series 2,5,8,..., which β and γ lie. (1996, 3M)
is equal to the sum of the first n terms of the AP series
57, 59, 61,..., then n equals (2001, 1M)
20. The interior angles of a polygon are in arithmetic
progression. The smallest angle is 120° and the common
(a) 10 (b) 12 difference is 5°. Find the number of sides of the polygon.
(c) 11 (d) 13 (1980, 3M)

Objective Question II Integer Answer Type Questions


(One or more than one correct option) 21. Suppose that all the terms of an arithmetic progression
4n k( k + 1 ) are natural numbers. If the ratio of the sum of the first
11. If S n = ∑ (−1) 2 k2. Then, S n can take value(s) seven terms to the sum of the first eleven terms is 6 : 11
2 (2013 Adv.)
and the seventh term lies in between 130 and 140, then
the common difference of this AP is (2015 Adv.)
(a) 1056 (b) 1088
(c) 1120 (d) 1332 22. A pack contains n cards numbered from 1 to n. Two
consecutive numbered cards are removed from the pack
Passage Based Problems and the sum of the numbers on the remaining cards is
1224. If the smaller of the numbers on the removed
Read the following passage and answer the questions. cards is k, then k − 20 is equal to (2013 Adv.)
Passage 23. Let a1 , a 2, a3 , K , a100 be an arithmetic progression with
p
Let V r denotes the sum of the first r terms of an
arithmetic progression (AP) whose first term is r and a1 = 3 and S p = ∑ ai , 1 ≤ p ≤ 100. For any integer n with
i =1
the common difference is (2r − 1). Let Tr = V r + 1 − V r and
Qr = Tr + 1 − Tr for r = 1, 2, . . . Sm
(2007, 8M) 1 ≤ n ≤ 20, let m = 5n. If does not depend on n, then a 2
Sn
12. The sum V1 + V 2 + ... + V n is
is equal to …… (2011)
1 1
(a) n (n + 1) (3n 2 − n + 1) (b) n (n + 1) (3n 2 + n + 2)
12 12 24. Let a1 , a 2, a3 , ... , a11 be real numbers satisfying a1 = 15,
1
(c) n (2n 2 − n + 1)
1
(d) (2n3 − 2n + 3) 27 − 2a 2 > 0 and a k = 2a k − 1 − a k − 2 for k = 3, 4, ... , 11.
2 3 a 2 + a 22 + K + a11
2
If 1 = 90, then the value of
13. Tr is always 11
(a) an odd number (b) an even number a1 + a 2 + K + a11
is…… (2010)
(c) a prime number (d) a composite number 11
rankershalt.com

50 Sequences and Series

Topic 3 Geometric Progression (GP)


Objective Questions I (Only one correct option) 8. Let f (x) = ax2 + bx + c, a ≠ 0 and ∆ = b2 − 4ac. If α + β,
1. Let a , b and c be in GP with common ratio r, where a ≠ 0 α 2 + β 2 and α 3 + β3 are in GP, then (2005, 1M)
1 (a) ∆ ≠ 0 (b) b∆ = 0 (c) c∆ = 0 (d) bc ≠ 0
and 0 < r ≤ . If 3a, 7b and 15care the first three terms of
2 9. Let a , b, c be in an AP and a 2, b2, c2 be in GP. If a < b < c
an AP, then the 4th term of this AP is 3
and a + b + c = , then the value of a is (2002, 1M)
(2019 Main, 10 April II) 2
2 7 1 1 1 1 1 1
(a) 5a (b) a (c) a (d) a (a) (b) (c) − (d) −
3 3 2 2 2 3 2 3 2 2
2. If three distinct numbers a , b and c are in GP and the 10. Let α , β be the roots of x2 − x + p = 0 and γ, δ be the roots
equations ax2 + 2bx + c = 0 and dx2 + 2ex + f = 0 have a of x2 − 4x + q = 0. If α , β , γ, δ are in GP, then the integer
common root, then which one of the following values of p and q respectively are (2001, 1M)
statements is correct? (2019 Main, 8 April II)
(a) − 2, − 32 (b) − 2,3 (c) − 6, 3 (d) − 6, − 32
d e f
(a) d , e and f are in GP (b) , and are in AP
a b c 11. If a , b, c, d and p are distinct real numbers such that
d e f (a 2 + b2 + c2) p2 − 2 (ab + bc + cd ) p
(c) d , e and f are in AP (d) , and are in GP
a b c
+ (b2 + c2 + d 2) ≤ 0, then a , b, c, d
3. The product of three consecutive terms of a GP is 512. If (a) are in AP (b) are in GP (1987, 2M)
4 is added to each of the first and the second of these
(c) are in HP (d) satisfy ab = cd
terms, the three terms now form an AP. Then, the sum
of the original three terms of the given GP is 12. If a , b, c are in GP, then the equations ax2 + 2bx + c = 0
(2019 Main, 12 Jan I) d e f
and dx2 + 2ex + f = 0 have a common root, if , , are
(a) 36 (b) 28 (c) 32 (d) 24 a b c
a3 in (1985, 2M)
4. Let a1 , a 2, .... , a10 be a GP. If = 25, then
a1 (a) AP (b) GP
a9 (c) HP (d) None of these
equals (2019 Main, 11 Jan I)
a5
13. The third term of a geometric progression is 4. The
(a) 53 (b) 2(52 ) (c) 4(52 ) (d) 54 product of the first five terms is (1982, 2M)
5. Let a , b and c be the 7th, 11th and 13th terms (a) 43 (b) 45
respectively of a non-constant AP. If these are also the (c) 44 (d) None of these
a
three consecutive terms of a GP, then is equal to
c Analytical & Descriptive Questions
(2019 Main, 9 Jan II)
14. Find three numbers a , b, c between 2 and 18 such that
7 1
(a) 2 (b) (c) 4 (d) (i) their sum is 25. (ii) the numbers 2, a , b are
13 2
consecutive terms of an AP. (iii) the numbers b, c, 18 are
6. If a , b and c be three distinct real numbers in GP and consecutive terms of a GP. (1983, 2M)
a + b + c = xb, then x cannot be (2019 Main, 9 Jan I) 15. Does there exist a geometric progression containing
(a) 4 (b) 2 (c) −2 (d) −3 27,8 and 12 as three of its term? If it exists, then how
7. If the 2nd, 5th and 9th terms of a non-constant AP are many such progressions are possible? (1982, 2M)
in GP, then the common ratio of this GP is (2016 Main) 16. If the mth, nth and pth terms of an AP and GP are equal
8 4 7
(a) (b) (c) 1 (d) and are x, y, z, then prove that xy − z ⋅ yz − x ⋅ z x − y = 1.
5 3 4 (1979, 3M)

Topic 4 Sum of n Terms & Infinite Terms of a GP


Objective Questions I (Only one correct option) 2. Let S n = 1 + q + q2 + K + qn and
2 n
20
1  q + 1  q + 1  q + 1
1. The sum ∑k 2k
is equal to (2019 Main, 8 April II)
Tn = 1 +   +
 2   2 
 +K+
 2 
 , where q is a
k =1
11 11 real number and q ≠ 1. If
(a) 2 − 19
(b) 1 − 101
C1 + 101C 2 ⋅ S1 + K + 101C101 ⋅ S100 = αT100, then α is
2 220 equal to (2019 Main, 11 Jan II)
3 21
(c) 2 − (d) 2 − (a) 2100 (b) 202
217 220
(c) 200 (d) 299
rankershalt.com

Sequences and Series 51

3. The sum of an infinite geometric series with positive Objective Question II


27
terms is 3 and the sum of the cubes of its terms is . (One or more than one correct option)
19
Then, the common ratio of this series is 10. Let S1 , S 2, ... be squares such that for each n ≥ 1 the
(2019 Main, 11 Jan I)
length of a side of S n equals the length of a diagonal of
4 2 2 1
(a) (b) (c) (d) S n + 1. If the length of a side of S1 is 10 cm, then for which
9 3 9 3
of the following values of n is the area of S n less than
4. Three positive numbers form an increasing GP. If the 1 sq cm? (1999, 3M)
middle term in this GP is doubled, then new numbers (a) 7 (b) 8 (c) 9 (d) 10
are in AP. Then, the common ratio of the GP is
(a) 2 + 3 (b) 3 + 2 (2014 Main) Analytical & Descriptive Questions
(c) 2 − 3 (d) 2 + 3 2 3 n
11. Let An =   −   +   + ... + (− 1)n − 1   ,
3 3 3 3
5. If (10) + 2(11) (10) + 3(11) (10) + ... + 10(11) = k(10) ,
9 1 8 2 7 9 9
 4  4  4  4
then k is equal to (2014 Main)
Bn = 1 − An. Find a least odd natural number n0 , so that
121 441
(a) (b) (c) 100 (d) 110 Bn > An , ∀ n ≥ n0. (2006, 6M)
10 100
12. If S1 , S 2, S3 , ... , S n are the sums of infinite geometric
6. The sum of first 20 terms of the sequence
0.7, 0.77, 0.777,… , is (2013 Main)
series, whose first terms are 1, 2, 3,..., n and whose
1 1 1 1
7 7 common ratios are , , ,... , respectively, then
(a) (179 − 10− 20 ) (b) (99 − 10− 20 ) 2 3 4 n+1
81 9
7 7 find the values of S12 + S 22 + S32 + ... + S 22n − 1. (1991, 4M)
(c) (179 + 10− 20 ) (d) (99 + 10− 20 )
81 9 13. The sum of the squares of three distinct real numbers,
7. An infinite GP has first term x and sum 5, then x which are in GP, is S 2. If their sum is a S, then show that
belongs to (2004, 1M) 1 
a 2 ∈  , 1 ∪ (1, 3)
(a) x < − 10 (b) −10 < x < 0 (c) 0 < x < 10 (d) x > 10 3  (1986, 5M)
8. Consider an infinite geometric series with first term a
and common ratio r. If its sum is 4 and the second term Integer Answer Type Questions
is 3 /4, then (2000, 2M) 14. Let S k , where k = 1, 2, , K , 100, denotes the sum of the
(a) a = 4/7, r = 3/7 (b) a = 2, r = 3 /8 k −1
(c) a = 3/2, r = 1/2 (d) a = 3, r = 1/4 infinite geometric series whose first term is and
k!
1 3 7 15 1
9. Sum of the first n terms of the series + + + + ... the common ratio is . Then, the value of
2 4 8 16 k
is equal to (1988, 2M) 1002 100

(a) 2n − n − 1 (b) 1 − 2−n (c) n + 2 − n − 1 (d) 2n + 1 100 !


+ ∑|(k2 − 3k + 1) Sk |is ……
k =1 (2010)

Topic 5 Harmonic Progression (HP)


Objective Questions I (Only one correct option) Assertion and Reason
1. If a1 , a 2, a3 ,… are in a harmonic progression with a1 = 5 For the following question, choose the correct
and a 20 = 25. Then, the least positive integer n for which answer from the codes (a), (b), (c) and (d) defined as
a n < 0, is (2012) follows:
(a) 22 (b) 23 (c) 24 (d) 25 (a) Statement I is true, Statement II is also true;
Statement II is the correct explanation of Statement I
2. If the positive numbers a , b, c,d are in AP. Then, (b) Statement I is true, Statement II is also true;
abc, abd , acd , bcd are (2001, 1M) Statement II is not the correct explanation of
(a) not in AP / GP / HP (b) in AP Statement I
(c) in GP (d) in HP (c) Statement I is true; Statement II is false
(d) Statement I is false; Statement II is true
3. Let a1 , a 2, ..., a10 be in AP and h1 , h2, equal to ..... , h10 be
5. Suppose four distinct positive numbers a1 , a 2, a3 , a 4
in HP. If a1 = h1 = 2 and a10 = h10 = 3, then a 4h7 is
(1999, 2M) are in GP. Let b1 = a1 , b2 = b1 + a 2, b3 = b2 + a3 and
b4 = b3 + a 4.
(a) 2 (b) 3 (c) 5 (d) 6
1 1 Statement I The numbers b1 , b2, b3 , b4 are neither in
4. If x > 1, y > 1, z > 1 are in GP, then , , AP nor in GP.
1 + ln x 1 + ln y
1 Statement II The numbers b1 , b2, b3 , b4 are in HP.
are in (1998, 2M)
1 + ln z (2008, 3M)
(a) AP (b) HP (c) GP (d) None of these
rankershalt.com

52 Sequences and Series

Fill in the Blank 8. Let a and b be positive real numbers. If a , A1 , A2, b are
in arithmetic progression, a , G1 , G2, b are in geometric
6. If cos (x − y), cos x and cos (x + y)’ are in HP. Then progression and a , H 1 , H 2, B are in harmonic
 y
cos x ⋅ sec   = K . (1997C, 2M) progression, then show that
 2 G1G2 A + A2 (2a + b)(a + 2b)
= 1 =
H 1H 2 H 1 + H 2 9ab (2002, 5M)
Analytical & Descriptive Questions
9. (i) The value of x + y + z is 15. If a , x, y, z , b are in AP
7. If a , b, c are in AP, a 2, b2, c2 are in HP, then prove that 1 1 1 5
while the value of + + is . If a , x, y, z , b are in
c x y z 3
either a = b = c or a , b, − form a GP.
2 (2003, 4M) HP, then find a and b.
(ii) If x, y, z are in HP, then show that
log (x + z ) + log (x + z − 2 y) = 2 log (x − z ). (1978, 3M)

Topic 6 Relation between AM, GM, HM and Some Special Series


Objective Questions I (Only one correct option) 7. The sum of the following series
1 +2 3
1 +2 +3
3 3 3 3 9 (12 + 22 + 32) 12 (12 + 22 + 32 + 42)
1. The sum of series 1 + + + ... 1+6+ +
1+2 1+2+3 7 9
13 + 23 + 33 + K + 153 1 15 (1 + 2 + ... + 5 )
2 2 2

+ − (1 + 2 + 3 + K + 15)is + + ... up to 15 terms is


1 + 2 + 3 + K + 15 2 11
(2019 Main, 9 Jan II)
equal to (2019 Main, 10 April II)
(a) 7510 (b) 7820
(a) 620 (b) 660 (c) 1240 (d) 1860 (c) 7830 (d) 7520
3 × 13 5 × (13 + 23 ) 12
2. The sum of series +
12 12 + 22 8. Let a1 , a 2, a3 , …, a 49 be in AP such that ∑ a 4k + 1 = 416
k=0
7 × (13 + 23 + 33 )
+ + .......... + upto 10th term, is and a 9 + a 43 = 66. If a12 + a 22 + … + a17
2
= 140 m, then m
12 + 22 + 32 (2019 Main, 10 April I) is equal to (2018 Main)
(a) 680 (b) 600 (a) 66 (b) 68 (c) 34 (d) 33
(c) 660 (d) 620
9. Let A be the sum of the first 20 terms and B be the sum
3. The sum of the series 1 + 2 × 3 + 3 × 5 + 4 × 7 +... upto of the first 40 terms of the series
11th term is (2019 Main, 9 April II) 12 + 2 ⋅ 22 + 32 + 2 ⋅ 42 + 52 + 2 ⋅ 62 + …
(a) 915 (b) 946 (c) 916 (d) 945 If B − 2 A = 100λ, then λ is equal to (2018 Main)
4. If the sum of the first 15 terms of the series (a) 232 (b) 248 (c) 464 (d) 496
3 3 3 3
 3  1  1  3 10. If the sum of the first ten terms of the series
  + 1  +  2  + 3 +  3  + ...
3
 4  2  4  4  3
2
 2
2
 1
2
 4
2
16
1  + 2  + 3  + 4 + 4  + K, is
2
m, then
is equal to 225 k, then k is equal to  5  5  5  5 5
(2019 Main, 12 Jan II) m is equal to (2016 Main)
(a) 108 (b) 27 (c) 54 (d) 9 (a) 102 (b) 101 (c) 100 (d) 99
1 + 2 + 3 + ... + k 5
5. Let S k = . If S12 + S 22 + ... + S10
2
= A, 11. If m is the AM of two distinct real numbers l and
k 12 n (l, n > 1) and G1 , G2 and G3 are three geometric means
then A is equal to (2019 Main, 12 Jan I)
between l and n, then G14 + 2G24 + G34 equals (2015)
(a) 156 (b) 301
(a) 4l2mn (b) 4lm2n (c) lmn 2 (d) l2m2n 2
(c) 283 (d) 303
12. The sum of first 9 terms of the series
6. Let x, y be positive real numbers and m, n positive 13 13 + 23 13 + 23 + 33
integers. The maximum value of the expression + + + ... is
1 1+3 1+3+5 (2015)
xm yn
is (a) 71 (b) 96 (c) 142 (d) 192
(1 + x ) (1 + y2n )
2m
(2019 Main, 11 Jan II)
 π tan 2 α
(a)
1
(b) 1 13. If α ∈ 0,  , then x2 + x + is always greater
 2 x2 + x
2
1 m+ n than or equal to (2003, 2M)
(c) (d)
4 6mn (a) 2 tan α (b) 1 (c) 2 (d) sec2 α
rankershalt.com

Sequences and Series 53

14. If a1 , a 2,... , a n are positive real numbers whose product Objective Question II
is a fixed number c, then the minimum value of (One or more than one correct option)
a1 + a 2 + ... + a n − 1 + 2a n is (2002, 1M)
(a) n (2c)1/ n (b) (n + 1)c1/ n 23. For a positive integer n let
1 1 1 1
(c) 2nc1/ n (d) (n + 1) (2c)1/ n a (n ) = 1 + + + + ... + n , then
2 3 4 (2 ) − 1 (1999, 3M)
15. If a , b, c are positive real numbers such that
a + b + c + d = 2 , then M = (a + b) (c + d ) satisfies the (a) a (100) ≤ 100 (b) a (100) > 100
relation (2000, 2M) (c) a (200)≤ 100 (d) a (200) > 100
(a) 0 < M ≤ 1 (b) 1 ≤ M ≤ 2 24. If the first and the (2n − 1)th term of an AP, GP and HP
(c) 2 ≤ M ≤ 3 (d) 3 ≤ M ≤ 4 are equal and their nth terms are a , b and c
16. The harmonic mean of the roots of the equation respectively, then (1988, 2M)
(a) a = b = c
(5 + 2 ) x2 − (4 + 5 ) x + 8 + 2 5 = 0 is (1999, 2M)
(b) a ≥ b ≥ c
(a) 2 (b) 4 (c) 6 (d) 8
(c) a + c = b
17. The product of n positive numbers is unity, then their (d) ac − b2 = 0
sum is (1991, 2M)
(a) a positive integer (b) divisible by n Fill in the Blanks
1
(c) equal to n + (d) never less than n
n 25. If x be is the arithmetic mean and y, z be two geometric
18. If a , b and c are distinct positive numbers, then the means between any two positive numbers, then
y3 + z3
expression (b + c − a ) (c + a − b) (a + b − c) − abc is = ...
(a) positive (b) negative (1991, 2M) xyz (1997C, 2M)
(c) non-positive (d) non-negative 26. If the harmonic mean and geometric mean of two
19. If x1 , x2,... , xn are any real numbers and n is any positive positive numbers are in the ratio 4 : 5. Then, the two
integer, then (1982, 1M) numbers are in the ratio… . (1992, 2M)
2 2
n  n  n  n 
(a) n ∑ xi2 <  ∑ xi  (b) n ∑ xi2 ≥  ∑ xi  True/False
   
i =1 i =1  i =1 i =1 
2 27. If x and y are positive real numbers and m, n are any
n  n 
(c) n ∑ xi2 ≥ n  ∑ xi  (d) None of these xn ym 1
  positive integers, then > .
i =1 i =1  (1 + x )(1 + y2m ) 4 (1989, 1M)
2n

Passage Based Problems 28. For 0 < a < x, the minimum value of function
log a x + log x a is 2.
Passage
Let A1 , G1 , H 1 denote the arithmetic, geometric and Analytical & Descriptive Questions
harmonic means, respectively, of two distinct positive
numbers. For n ≥ 2, let An − 1 and H n − 1 has arithmetic, 29. If a , b, c are positive real numbers, then prove that
geometric and harmonic means as An , Gn , H n , {(1 + a ) (1 + b) (1 + c)}7 > 77 a 4b4c4 (2004, 4M)
respectively. (2007, 8M)
30. Let a1 , a 2,.. be positive real numbers in geometric
20. Which one of the following statements is correct? progression. For each n, if An , Gn , H n are respectively,
(a) G1 > G2 > G3 > ... the arithmetic mean, geometric mean and harmonic
(b) G1 < G2 < G3 < ... mean of a1 , a 2, .... , a n. Then, find an expression for the
(c) G1 = G2 = G3 = ... geometric mean of G1 , G2, ... , Gn in terms of
(d) G1 < G3 < G5 < ... and G2 > G4 > G6 >... A1 , A2, ... , An , H 1 , H 2, ... , H n. (2001, 5M)
21. Which of the following statements is correct? 31. If p is the first of the n arithmetic means between two
(a) A1 > A2 > A3 >... numbers and q be the first on n harmonic means
(b) A1 < A2 < A3 <... between the same numbers. Then, show that q does not
(c) A1 > A3 > A5 >... and A2 < A4 < A6 <...  n + 1
2

(d) A1 < A3 < A5 <... and A2 > A4 > A6 >... lie between p and   p. (1991, 4M)
 n − 1
22. Which of the following statements is correct ?
(a) H1 > H 2 > H3 >... 32. If a > 0, b > 0 and c > 0, then prove that
(b) H1 < H 2 < H3 <... (a + b + c)
1 1 1
+ + ≥9 (1984, 2m)
(c) H1 > H3 > H5 >... and H 2 < H 4 < H 6 <... a b c
(d) H1 < H3 < H5 <... and H 2 > H 4 > H 6 >...
rankershalt.com

54 Sequences and Series

Integer Answer Type Question


33. Let a,b,c be positive integers such that b /a is an integer. If a,b,c are in geometric progression and the arithmetic mean of
a 2 + a − 14
a,b,c is b + 2, then the value of is
a+1 (2014 Adv.)
−5 −4 −3
34. The minimum value of the sum of real numbers a , a , 3a , 1, a and a with a > 0 is …… 8 10
(2011)

Answers
Topic 1 Topic 4
1. (b) 2. (d) 3. (b) 4. (c) 1. (a) 2. (a) 3. (b)
6. False 7. (6) 4. (d) 5. (c) 6. (c) 7. (c)
Topic 2 8. (d) 9. (c) 10. (b, c, d)
1
1. (a) 2. (c) 3. (b) 4. (c) 11. (7) 12. (2n )(2n + 1 )( 4n + 1 ) – 1 14. (4)
5. (a) 6. (c) 7. (c) 8. (b) 6
9. (c) 10. (c) 11. (a, d) 12. (b) Topic 5
13. (d) 14. (b) 15. (A = – 3 , B = 77) 1. (d) 2. (d) 3. (d) 4. (b)
n 2(n + 1 )  5. (c) 6. ± 2 9.(i) a = 1, b = 9
16.   17. (3050)
 2  12. 29
 1  1  Topic 6
19. β ∈ – ∞,  and γ ∈ – , ∞  20. (9) 21. (9)
 3  27  1. (a) 2. (c) 3. (b) 4. (b)
22. (5) 23. (9) 24. (0) 5. (d) 6. (c) 7. (b) 8. (c)
9. (b) 10. (b) 11. (b) 12. (b)
Topic 3
13. (a) 14. (a) 15. (a) 16. (b)
1. (c) 2. (b) 3. (b) 4. (d) 17. (d) 18. (b) 19. (b) 20. (c)
5. (c) 6. (b) 7. (b) 8. (c) 21. (a) 22. (b) 23. (a, d) 24. (a, b, d)
9. (d) 10. (a) 11. (b) 12. (a) 25. 2 26. 4:1 27. False 28. False
13. (b) 14. (a = 5 ) (b = 8 ) (c = 12 ) 15. Yes, infinite 34. (8)

Hints & Solutions


Topic 1 Arithmetic Progression (AP) 2. Let tn be the nth term of given AP. Then, we have t19 = 0
⇒ a + (19 − 1)d = 0 [Qtn = a + (n − 1)d ]
1. Key Idea Use nth term of an AP i.e. an = a + ( n − 1) d , simplify the ⇒ a + 18d = 0 …(i)
given equation and use result. t49 a + 48d
Now, =
t29 a + 28d
Given AP is a1 , a 2, a3 , … , a n
Let the above AP has common difference ‘d’, then − 18d + 48d
= [using Eq. (i)]
a1 + a 4 + a7 + … + a16 − 18d + 28d
= a1 + (a1 + 3d ) + (a1 + 6d ) + … + (a1 + 15d ) 30d
= = 3 :1
= 6a1 + (3 + 6 + 9 + 12 + 15)d 10d
∴6a1 + 45d = 114 (given) 3. We have,
⇒ 2a1 + 15d = 38 …(i) 225a 2 + 9b2 + 25c2 − 75ac − 45ab − 15bc = 0
Now, a1 + a 6 + a11 + a16 ⇒ (15a )2 + (3b)2 + (5c)2 − (15a )(5c) − (15a )(3b)
= a1 + (a1 + 5d ) + (a1 + 10d ) + (a1 + 15d ) − (3b)(5c) = 0
1
= 4a1 + 30d = 2(2a1 + 15d ) ⇒ [(15a − 3b) + (3b − 5c) + (5c − 15a ) ] = 0
2 2 2
2
= 2 × 38 = 76 [from Eq. (i)]
rankershalt.com

Sequences and Series 55

⇒ 15a = 3b, 3b = 5c and 5c = 15a Now, sum of first 15 terms is given by


∴ 15a = 3b = 5c 15
S15 = [2a1 + (15 − 1) d ]
a b c 2
⇒ = = =λ (say) 15
1 5 3 = [2a1 + 14d ] = 15 [a1 + 7d ]
⇒ a = λ , b = 5λ , c = 3λ 2
∴b, c, a are in AP. From Eq. (i), we have
40
1 a1 + 7d =
4. Let Tm = a + (m − 1) d = …(i) 3
n
40
1 So, S15 = 15 ×
and Tn = a + (n − 1) d = …(ii) 3
m
= 5 × 40 = 200
On subtracting Eq. (ii) from Eq. (i), we get
1 1 m−n 2. Given S n denote the sum of the first n terms of an AP.
(m − n ) d = − =
n m mn Let first term and common difference of the AP be ‘a’
1 and ‘d’, respectively.
⇒ d=
mn
∴ S 4 = 2[2a + 3d ] = 16 (given)
Again, Tmn = a + (mn − 1) d = a + (mn − n + n − 1) d  n 
= a + (n − 1) d + (mn − n ) d Q S n = 2 [2a + (n − 1)d ]
1 1 (m − 1)
= Tn + n (m − 1) = + =1 ⇒ 2a + 3d = 8 … (i)
mn m m
and S 6 = 3[2a + 5d ] = − 48 [given]
5. Since, a1 , a 2, ... , a n are in an AP. ⇒ 2a + 5d = − 16 … (ii)
∴ (a 2 − a1 ) = (a3 − a 2) = ... = (a n − a n − 1 ) = d On subtracting Eq. (i) from Eq. (ii), we get
1 1 1 2d = − 24
Thus, + + ... +
a1 + a 2 a 2 + a3 a n −1 + a n ⇒ d = − 12
So, 2a = 44 [put d = −12 in Eq. (i)]
 a 2 − a1   a3 − a 2   a n − a n −1 
=  +  + ... +   Now, S10 = 5[2a + 9d ]
    
 d   d   d  = 5[44 + 9(− 12)] = 5[44 − 108]
1 1 (a n − a1 ) (n − 1) = 5 × (− 64) = − 320
= ( a n − a1 ) = =
d d a n + a1 a n + a1 3. Given series is
 1  1 1   1 2   1 99 
6. Since, n1 , n2,... , n p are p positive integers, whose sum is − 3  + − 3 − 100  + − 3 − 100  + K ... + − 3 − 100 
even and we know that, sum of any two odd integers is
even. [where, [x] denotes the greatest integer ≤ x]
∴Number of odd integers must be even. Now,
Hence, it is a false statement.  1  1 1   1 2   1 66 
− 3 , − 3 − 100 , − 3 − 100 , …+ − 3 − 100 
7. Let the sides are a − d , a and a + d. Then,
a (a − d ) = 48 all the term have value − 1
and a − 2ad + d 2 + a 2 = a 2 + 2ad + d 2
2  1 67   1 68   1 99 
and − − , − − , …, − − all the term
⇒ a 2 = 4ad  3 100   3 100   3 100 
⇒ a = 4d have value − 2.
Thus, a = 8, d = 2  1  1 1   1 2   1 66 
So, −  + − − + − − + ... + − −
Hence, a − d =6  3   3 100   3 100   3 100 
= − 1 − 1 − 1 − 1 K 67 times.
Topic 2 Sum of n Terms of an AP = (− 1) × 67 = − 67
1. Let the common difference of given AP is ‘d’.  1 67   1 68   1 99 
and − − + − 3 − 100  + K + − 3 − 100 
Since, a1 + a7 + a16 = 40  3 100 
∴ a1 + a1 + 6d + a1 + 15d = 40 [Q a n = a1 + (n − 1) d ] = − 2 − 2 − 2 − 2 K 33 times
⇒ 3a1 + 21d = 40 …(i) = (−2) × 33 = −66
rankershalt.com

56 Sequences and Series


 1  1 1   1 2   1 99  On comparing corresponding term, we get
∴ −  + − 3 − 100  + − 3 − 100  + K + − 3 − 100 
 3 d 7
d = A and a1 − = 50 − A
= (− 67) + (− 66) = − 133. 2 2
Alternate Solution A 7
Q [− x] = − [x] − 1, if x ∉Integer, ⇒ a1 − = 50 − A [Qd = A]
2 2
 1  2  n − 1
and [x] + x +  + x +  + K + x + n  = [nx], ⇒ a1 = 50 − 3 A
 n  n
n ∈N. So a50 = a1 + 49d
So given series = (50 − 3 A ) + 49 A [Q d = A]
 1  1 1   1 2   − 1 99 
− 3  + − 3 − 100  + − 3 − 100  + … K +  3 − 100  = 50 + 46 A
Therefore, (d , a50 ) = ( A , 50 + 46 A )
 1    1 1  
=  −   − 1 +  −  + − 1
  3 100 
 3  
6. Clearly, the two digit number which leaves remainder 2
when divided by 7 is of the form N = 7k + 2 [by Division
 1 2    1 99   Algorithm]
+ −  +  − 1 + K +  −  +  − 1
 3 100    3 100  
1  For, k = 2, N = 16
= (− 1) × 100 −  × 100 = − 100 − 33 = − 133. k = 3, N = 23
3 
4. Let first three terms of an AP as a − d, a, a + d. M M
So, 3a = 33 ⇒ a = 11
k = 13, N = 93
∴ 12 such numbers are possible and these numbers
[given sum of three terms = 33 forms an AP.
and product of terms = 1155] 12
Now, S= [16 + 93] = 654
⇒ (11 − d )11(11 + d ) = 1155 [given] 2
⇒ 112 − d 2 = 105  n 
QS n = ( a + l )
⇒ d 2 = 121 − 105 = 16  2 
⇒ d = ±4 Similarly, the two digit number which leaves remainder
5 when divided by 7 is of the form N = 7k + 5
So the first three terms of the AP are either 7, 11, 15 or
For k = 1, N = 12
15, 11, 7.
k = 2, N = 19
So, the 11th term is either 7 + (10 × 4) = 47 M
or 15 + (10 × (−4)) = − 25. k = 13, N = 96
∴13 such numbers are possible and these numbers also
5 Key Idea Use the formula of sum of first n terms of AP, i.e forms an AP.
n 13
Sn = [2 a + ( n − 1) d ] Now, S′ = [12 + 96] = 702
2 2
 n 
QS n = ( a + l )
Given AP, is  2 
a1 , a 2, a3 ,… having sum of first n-terms Total sum = S + S′ = 654 + 702 = 1356
n 7. We have, S = a1 + a 2 + … + a30
= [2a1 + (n − 1)d ]
2 = 15[2a1 + 29d ] …(i)
[where, d is the common difference of AP] (where d is the common difference)
n (n − 7)  n 
= 50n + A (given) Q S n = 2 [2a + (n − 1)d ]
2
1 n−7 and T = a1 + a3 + … + a 29
⇒ [2a1 + (n − 1)d ] = 50 + A
2 2 15
= [2a1 + 14 × 2d )]
2
1  7  n
⇒ [2a1 + nd − d ] = 50 − A + A (Q common difference is 2d)
2  2  2
⇒ 2T = 15[2a1 + 28d ] …(ii)
 d  nd  7  n
⇒  a1 −  + = 50 − A + A From Eqs. (i) and (ii), we get
 2 2  2  2
S − 2T = 15d = 75 [QS − 2T = 75]
rankershalt.com

Sequences and Series 57

⇒ d =5 ⇒ 6n + 1 = 57 + n − 1 ⇒ 5n = 55
Now, a10 = a5 + 5d ∴ n = 11
= 27 + 25 = 52 11. PLAN Convert it into differences and use sum ofn terms of an AP,
n
8. If log b1 , log b2, ... , log b101 are in AP, with common i.e. S n = [2a + ( n − 1 )d ]
2
difference log e 2 , then b1 , b2, ... , b101 are in GP, with k( k + 1)
common ratio 2. 4n 2
∴ b1 = 20 b1 , b2 = 21 b1 , b3 = 22b1,…, b101 = 2100 b1 …(i) Now, Sn = ∑ (−1) ⋅ k2
k =1
Also, a1 , a 2, ... , a101 are in AP.
Given, a1 = b1 and a51 = b51 = − (1)2 − 22 + 32 + 42 − 52 − 62 + 72 + 82 + K
⇒ a1 + 50 D = 250 b1 = (32 − 12) + (42 − 22) + (72 − 52) + (82 − 62) + K
⇒ a1 + 50 D = 250 a1 [Q a1 = b1 ] …(ii) = 2{(4 + 6 + 12 + K ) + (6 + 14 + 22 + K )}
144424443 144424443
Now, t = b1 + b2 + K + b51 n terms n terms

(251 − 1) n n 
⇒ t = b1 …(iii) = 2  {2 × 4 + (n − 1) 8} + {2 × 6 + (n − 1) 8}
2 −1 2 2 
and s = a1 + a 2 + K + a51 = 2 [n (4 + 4n − 4) + n (6 + 4n − 4)]
51 = 2 [4n 2 + 4n 2 + 2n ] = 4n (4n + 1)
= (2a1 + 50 D ) …(iv)
2 Here, 1056 = 32 × 33, 1088 = 32 × 34,
∴ t = a1 (251 − 1) [Q a1 = b1 ] 1120 = 32 × 35, 1332 = 36 × 37
or t = 251 a1 − a1 < 251 a1 …(v) 1056 and 1332 are possible answers.
51
and s=
2
[a1 + (a1 + 50 D )] [from Eq. (ii)] r 1
12. Here, V r = [ 2r + (r − 1) ( 2r − 1)] = ( 2r3 − r 2 + r )
2 2
51 1
= [a1 + 250 a1 ] ∴ ΣV r = [ 2 Σr − Σr + Σr ]
3 2
2 2
51 51 50
= a1 + 2 a1 1   n (n + 1)  n (n + 1) ( 2n + 1) n (n + 1) 
2
2 2 = 2   − + 
∴ s > 251 a1 …(vi) 2   2  6 2 
From Eqs. (v) and (vi), we get s > t n (n + 1)
⇒ = [ 3n (n + 1) − ( 2n + 1) + 3]
Also, a101 = a1 + 100 D and b101 = 2100 b1 12
 250 a1 − a1  1
∴ a101 = a1 + 100   and b101 = 2100 a1 = n (n + 1) ( 3n 2 + n + 2)
 50  12
⇒ a101 = a1 + 251 a1 − 2a1 = 251 a1 − a1 1 1
13. V r + 1 − V r = (r + 1)3 − r3 − [(r + 1)2 − r 2] +
⇒ a101 < 251 a1 and b101 > 251 a1 2 2
⇒ b101 > a101 = 3r 2 + 2r − 1
9. Let S n = cn 2 ∴ Tr = 3r 2 + 2r − 1 = (r + 1) ( 3r − 1)
which is a composite number.
S n − 1 = c (n − 1)2 = cn 2 + c − 2cn
∴ Tn = 2cn − c [Q Tn = S n − S n − 1 ] 14. Since, Tr = 3r 2 + 2r − 1
Tn2 = (2 cn − c)2 = 4c2n 2 + c2 − 4c2n and Tr + 1 = 3 (r + 1)2 + 2 (r + 1) − 1
4c2 ⋅ n (n + 1) (2n + 1) ∴ Qr = Tr+1 − Tr = 3 [ 2r + 1] + 2 [1]
∴ Sum = Σ Tn2 = + nc2 − 2c2n (n + 1)
6 ⇒ Qr = 6 r + 5
2 c n (n + 1) (2n + 1) + 3nc − 6c n (n + 1)
2 2 2
⇒ Qr+ 1 = 6(r + 1) + 5
=
3 Common difference = Qr+ 1 − Qr = 6
nc2(4n 2 + 6n + 2 + 3 − 6n − 6) nc2(4n 2 − 1)
= = 15. Given, p + q = 2, pq = A
3 3
and r + s = 18, rs = B
10. According to given condition,
and it is given that p, q, r , s are in an AP.
S 2n = S′n
2n n Therefore, let p = a − 3d , q = a − d , r = a + d
⇒ [2 × 2 + (2n − 1) × 3] = [2 × 57 + (n − 1) × 2]
2 2 and s = a + 3d

1
(4 + 6n − 3) = (114 + 2n − 2) Since, p<q<r<s
2
rankershalt.com

58 Sequences and Series

We have, d >0 Now, a 2 − 5d 2 = a 2 − 9d 2 + 4d 2


Now, 2 = p + q = a − 3d + a − d = 2a − 4d = (a − 3d )(a + 3d ) + (2d )2
⇒ a − 2d = 1 …(i) = I ⋅ I + I2 [given]
Again, 18 = r + s = a + d + a + 3d = I2 + I2 = I2
18 = 2a + 4d =I [where, I is any integer]
⇒ 9 = a + 2d …(ii) Therefore, P = (I )2 = Integer
On subtracting Eq. (i) from Eq. (ii), we get 19. Since, x1 , x2, x3 are in an AP. Let x1 = a − d , x2 = a and
8 = 4d ⇒ d = 2 x3 = a + d and x1 , x2, x3 be the roots of x3 − x2 + βx + γ = 0
On putting in Eq. (ii), we get a = 5 ∴ Σα = a − d + a + a + d = 1
∴ p = a − 3d = 5 − 6 = − 1 ⇒ a = 1 /3 …(i)
q = a − d =5 −2 =3 Σαβ = (a − d ) a + a (a + d ) + (a − d ) (a + d ) = β …(ii)
r = a + d =5 + 2 = 7 and αβγ = (a − d ) a (a + d ) = − γ …(iii)
and s = a + 3d = 5 + 6 = 11 From Eq. (i),
Therefore, A = pq = − 3 and B = rs = 77 3a = 1 ⇒ a = 1 / 3
16. Here, 1 + 2 ⋅ 2 + 3 + 2 ⋅ 4 + 5 + ... upto n terms
2 2 2 2 2 From Eq. (ii), 3a − d 2 = β
2

n (n + 1)2 ⇒ 3 (1 / 3)2 − d 2 = β [from Eq. (i)]


= [when n is even] … (i)
2 ⇒ 1 /3 − β = d 2

When n is odd, 1 + 2 ⋅ 2 + 3 + 2 ⋅ 4 + 5 ... + n


2 2 2 2 2 2 NOTE In this equation, we have two variables β and γ but we have
only one equation. So, at first sight it looks that this equation
= {12 + 2⋅22 + 32 + 2 ⋅ 42 + ... + 2 (n − 1)2} + n 2 cannot solve but we know that d 2 ≥ 0, ∀ d ∈ R, then β can
 (n − 1) (n )2  be solved. This trick is frequently asked in IIT examples.
= + n
2
[from Eq. (i)] 1
 2  ⇒ −β ≥0 [Q d 2 ≥ 0]
3
n −1  (n + 1)
= n2  + 1 = n 2 1
 2  2 ⇒ β≤ ⇒ β ∈ [− ∞ , 1 / 3]
3
∴ 12 + 2⋅ 22 + 32 + 2 ⋅ 42 + ... upto n terms, when n is odd From Eq. (iii), a (a 2 − d 2) = − γ
n 2 (n + 1) 1 1 2 1 1 2
= ⇒  − d  = −γ ⇒ − d = −γ
2 3 9  27 3
17. Integers divisible by 2 are {2,4,6,8,10, ...,100}. 1 1 2 1
⇒ γ+ = d ⇒γ + ≥0
Integers divisible by 5 are {5,10,15, ...,100}. 27 3 27
Thus, sum of integers divisible by 2 ⇒ γ ≥ − 1 / 27
=
50
(2 + 100) = 50 × 51 = 2550  1 
⇒ γ ∈ − , ∞
2  27 
Sum of integers divisible by 5 Hence, β ∈ (− ∞ , 1 / 3] and γ ∈ [−1 / 27, ∞ )
20
= (5 + 100) = 10 × 105 = 1050 20. Since, angles of polygon are in an AP.
2
∴ Sum of all angles
Sum of integers divisible by 10 n
10 = (n − 2) × 180° = {2 (120° ) + (n − 1) 5° }
= (10 + 100) = 5 × 110 = 550 2
2 ⇒ 5n 2 − 125n + 720 = 0
∴ Sum of integers from 1 to 100 divisible by 2 or 5 ⇒ n 2 − 25n + 144 = 0
= 2550 + 1050 − 550 ⇒ (n − 9) (n − 16) = 0
= 2550 + 500 = 3050 ⇒ n = 9, 16
18. Let four consecutive terms of the AP are a − 3d , a − d , If n = 9, then largest angle = a + 8d = 160°
a + d , a + 3d, which are integers. Again, if n = 16, the n largest angle
Again, required product = a + 15d = 120° + 75 = 195°
P = (a − 3d )(a − d )(a + d )(a + 3d ) + (2d )4 which is not possible.
[by given condition] [since, any angle of polygon cannot be > 180°]
= (a 2 − 9d 2)(a 2 − d 2) + 16d 4 Hence, n = 9 [neglecting n = 16]
= a 4 − 10a 2d 2 + 9d 4 + 16d 4 = (a 2 − 5d 2)2
rankershalt.com

Sequences and Series 59

S7 6
21. Given, = and 130 < t7 < 140 Topic 3 Geometric Progression (GP)
S11 11
7 1.
[2a + 6d ] Key Idea Use nth term of AP i.e., an = a + ( n − 1) d , If a, A , b are in
2 6 7 (2a + 6d )
⇒ = ⇒ =6 AP, then 2A = a + b and nth term of G.P. i.e., an = ar n − 1.
11
[2a + 10d ] 11 (2a + 10d )
2 It is given that, the terms a , b, c are in GP with common
⇒ a = 9d …(i) 1
ratio r, where a ≠ 0 and 0 < r ≤ .
Also, 130 < t7 < 140 2
So, let, b = ar and c = ar 2
⇒ 130 < a + 6d < 140
⇒ 130 < 9d + 6d < 140 [from Eq. (i)] Now, the terms 3a, 7b and 15c are the first three terms
of an AP, then
⇒ 130 < 15d < 140
26 28 2( 7b) = 3a + 15 c
⇒ <d< [since, d is a natural number] ⇒ 14ar = 3a + 15ar 2 [as b = ar, c = ar 2]
3 3
⇒ 14r = 3 + 15r 2 [as a ≠ 0]
∴ d =9
⇒ 15r 2 − 14r + 3 = 0
22. Let number of removed cards be k and (k + 1). ⇒ 15r 2 − 5r − 9r + 3 = 0
n (n + 1)
∴ − k − (k + 1) = 1224 ⇒ 5r (3r − 1) − 3(3r − 1) = 0
2
⇒ (3r − 1) (5r − 3) = 0
⇒ n 2 + n − 4k = 2450 ⇒ n 2 + n − 2450 = 4k 1 3
⇒ r = or
⇒ (n + 50) (n − 49) = 4k 3 5
∴ n > 49  1 1
as, r ∈ 0, , so r =
Let n = 50  2 3
∴ 100 = 4k Now, the common difference of AP = 7b − 3a
⇒ k = 25 7  2a
= 7ar − 3a = a  − 3 = −
Now k − 20 = 5 3  3
23. Given, a1 = 3,m = 5n and a1 , a 2, …, is an AP.  −2a 
So, 4th term of AP = 3a + 3  =a
Sm S5 n  3 
∴ = is independent of n.
Sn Sn
2. (b) Given, three distinct numbers a , b and c are in GP.
5n
[2 × 3 + (5n − 1) d ] ∴ b2 = ac …(i)
5 {(6 − d ) + 5n }
= 2 = , and the given quadratic equations
n
[2 × 3 + (n − 1) d ] (6 − d ) + n
ax2 + 2bx + c = 0 …(ii)
2
independent of n dx2 + 2ex + f = 0 …(iii)
If 6 − d =0 ⇒ d =6 For quadratic Eq. (ii),
∴ a 2 = a1 + d = 3 + 6 = 9 the discriminant D = ( 2b)2 − 4ac
S
or If d = 0, then m is independent of n. = 4(b2 − ac) = 0 [from Eq. (i)]
Sn ⇒ Quadratic Eq. (ii) have equal roots, and it is equal to
b
∴ a2 = 9 x = − , and it is given that quadratic Eqs. (ii) and (iii)
a
24. a k = 2a k − 1 − a k − 2 have a common root, so
2
⇒ a1 , a 2, . . . , a11 are in an AP.  b  b
d  −  + 2e  −  + f = 0
a12 + a 22 + . . . + a11
2
11a 2 + 35 × 11d 2 + 10ad  a  a
∴ = = 90
11 11 ⇒ db2 − 2eba + a 2f = 0
⇒ 225 + 35 d 2 + 150 d = 90 ⇒ d (ac) − 2eab + a 2f = 0 [Q b2 = ac]
9 ⇒ dc − 2eb + af = 0 [Q a ≠ 0]
⇒ 35 d 2 + 150 d + 135 = 0 ⇒ d = − 3, −
7 ⇒ 2eb = dc + af
27 e dc af
Given, a2 < ⇒ 2 = 2+ 2
2 b b b
9 [dividing each term by b2]
∴ d = − 3 and d ≠ −
7  e d f
⇒ 2  = + [Q b2 = ac]
a1 + a 2 + . . . + a11 11  b a c
⇒ = [ 30 − 10 × 3] = 0
11 2
rankershalt.com

60 Sequences and Series

So,
d e f 11th term = b = A + 10d
, , are in AP.
a b c 13th term = c = A + 12d
Alternate Solution Q a, b, c are also in GP
Given, three distinct numbers a , b and c are in GP. Let ∴ b2 = ac
a = a, b = ar, c = ar 2 are in GP, which satisfies ⇒ ( A + 10d )2 = ( A + 6d ) ( A + 12d )
ax2 + 2bx + c = 0
⇒ A 2 + 20 Ad + 100d 2 = A 2 + 18 Ad + 72d 2
∴ ax2 + 2(ar )x + ar 2 = 0
⇒ 2 Ad + 28d 2 = 0
⇒ x2 + 2rx + r 2 = 0 [Q a ≠ 0]
⇒ 2d ( A + 14d ) = 0
⇒ (x + r )2 = 0 ⇒ x = − r.
⇒ d = 0 or A + 14d = 0
According to the question, ax2 + 2bx + c = 0 and
But d ≠0 [Q the series is non constant AP]
dx2 + 2ex + f = 0 have a common root.
⇒ A = − 14d
So, x = − r satisfies dx2 + 2ex + f = 0 ∴ a = A + 6d = − 14d + 6d = − 8d
∴ d (− r )2 + 2e(− r ) + f = 0 and c = A + 12d = − 14d + 12d = − 2d
⇒ dr 2 − 2er + f = 0 a − 8d
⇒ = =4
 
c  c c − 2d
⇒ d   − 2e  + f = 0
 a  b
6. Let b = ar and c = ar 2, where r is the common ratio.
d 2e f
⇒ − + =0 Then, a + b + c = xb
a b c
d f 2e ⇒ a + ar + ar 2 = xar
⇒ + = [Q c ≠ 0]
a c b ⇒ 1 + r + r 2 = xr … (i) [Q a ≠ 0]
a 1 + r + r2 1
3. Let the three consecutive terms of a GP are , a and ar. ⇒ x= =1+r +
r r r
Now, according to the question, we have 1
a We know that, r + ≥ 2 (for r > 0)
⋅ a ⋅ ar = 512 r
r 1
and r + ≤ − 2 (for r < 0) [using AM ≥ GM]
⇒ a3 = 512 r
⇒ a=8 … (i) 1
∴ 1+r + ≥ 3
Also, after adding 4 to first two terms, we get r
8 1
+ 4, 8 + 4, 8r are in AP or 1 + r + ≤ −1
r r
8 ⇒ x ≥ 3 or x ≤ −1
⇒ 2 (12) = + 4 + 8r
8r
⇒ x ∈ ( − ∞ ,−1] ∪ [3, ∞ )
2 
⇒ 24 = + 8r + 4 ⇒ 20 = 4  + 2r Hence, x cannot be 2.
r r 
2 Alternate Method
⇒ 5= + 2r ⇒ 2r 2 − 5r + 2 = 0
r From Eq. (i), we have
⇒ 2r 2 − 4r − r + 2 = 0 1 + r + r 2 = xr
⇒ 2r (r − 2) − 1(r − 2) = 0
⇒ r 2 + (1 − x )r + 1 = 0
⇒ (r − 2) (2r − 1) = 0
1 For real solution of r , D ≥ 0.
⇒ r = 2,
2 ⇒ (1 − x )2 − 4 ≥ 0
Thus, the terms are either 16, 8, 4 or 4, 8, 16. Hence, ⇒ x 2 − 2x − 3 ≥ 0
required sum = 28. ⇒ ( x − 3)( x + 1) ≥ 0
4. Let r be the common ratio of given GP, then we have the + – +
following sequence a1 , a 2 = a1r , a3 = a1r , ... , a10 = a1r
2 9 –1 3
Now, a3 = 25 a1 ⇒ x ∈ ( −∞ , − 1] ∪ [3, ∞ )
⇒ a1r 2 = 25 a1 ∴ x cannot be 2.
⇒ r 2 = 25 7. Let a be the first term and d be the common difference.
a a r8 Then, we have a + d, a + 4d, a + 8 d in GP,
Consider, 9 = 1 4 = r 4 = (25)2 = 54
a5 a1r i.e. (a + 4d ) 2 = (a + d ) (a + 8 d )
⇒ a + 16 d 2 + 8ad = a 2 + 8ad + ad + 8 d 2
2
5. Let A be the Ist term of AP and d be the common
difference. ⇒ 8 d 2 = ad
∴ 7th term = a = A + 6d ⇒ 8d = a [Q d ≠ 0]
[Q nth term = A + (n − 1)d]
rankershalt.com

Sequences and Series 61

Now, common ratio, 11. Here, (a 2 + b2 + c2) p2 − 2 (ab + bc + cd ) p


a + 4d 8 d + 4d 12 d 4
r= = = = + (b2 + c2 + d 2) ≤ 0
a+d 8d + d 9d 3
⇒ (a p − 2abp + b ) + (b p − 2bcp + c2)
2 2 2 2 2

8. Since, (α + β), (α + β ), (α + β ) are in GP.


2 2 3 3
+ (c2p2 − 2cdp + d 2) ≤ 0
⇒ (α + β ) = (α + β ) (α + β )
2 2 2 3 3
⇒ (ap − b) + (bp − c) + (cp − d ) ≤ 0
2 2 2
⇒ α 4 + β 4 + 2α 2β 2 = α 4 + β 4 + αβ3 + βα 3
[since, sum of squares is never less than zero]
⇒ αβ (α 2 + β 2 − 2αβ ) = 0
⇒ αβ (α − β )2 = 0 Since, each of the squares is zero.
⇒ α β = 0 or α = β ∴ (ap − b)2 = (bp − c)2 = (cp − d )2 = 0
c b c d
⇒ =0 or ∆ = 0 ⇒ p= = =
a a b c
⇒ c∆ = 0 ∴ a , b, c,d are in GP.
9. Since, a, b and c are in an AP. 12. Since, a , b, c are in GP.
Let a = A − D, b = A, c = A + D
⇒ b2 = ac
3
Given, a + b+ c= Given, ax + 2bx + c = 0
2
2
3 ⇒ ax + 2 ac x + c = 0
2
⇒ ( A − D) + A + ( A + D) =
2 c
⇒ ( a x+ c )2 = 0 ⇒ x = −
3 1 a
⇒ 3A = ⇒ A =
2 2 Since, ax2 + 2bx + c = 0 and dx2 + 2ex + f = 0 have
1 1 1 common root.
∴ The number are − D , , + D.
2 2 2 ∴ x = − c/a must satisfy.
2 2
1  1 1  dx2 + 2ex + f = 0
Also,  − D , ,  + D are in GP.
2  4 2  c c d 2e f
2 2 2 2 ⇒ d⋅ − 2e + f =0 ⇒ − + =0
 1 1  1  1 1  a a a ac c
∴   =  − D  + D ⇒ =  − D 2
 4 2  2  16  4  2e d f
⇒ = + [Q b2 = ac]
1 1 1 1 b a c
⇒ − D2 = ± ⇒ D2 = ⇒ D=±
4 4 2 2 d e f
Hence, , , are in an AP.
1 1 a b c
∴ a= ±
2 2 13. Here, t3 = 4 ⇒ ar 2 = 4
1 1
So, out of the given values, a = − is the right ∴ Product of first five terms = a ⋅ ar ⋅ ar 2 ⋅ ar3 ⋅ ar 4
2 2
= a5 r10 = (a r 2)5 = 45
choice.
α + β =1 λ + δ = 4 14. If a , b, c ∈ (2, 18), then
10.  and 
αβ = p λ δ = q a + b + c = 25 ....(i)
Let r be the common ratio. Since, 2, a , b are in AP.
Since, α , β , γ and δ are in GP. ⇒ 2a = b + 2 .... (ii)
Therefore, β = αr, γ = αr 2 and b, c, 18 are in GP.
and δ = αr3 ⇒ c2 = 18b ... (iii)
Then, α + αr = 1 ⇒ α (1 + r ) = 1 …(i) From Eqs. (i), (ii) and (iii),
b+2
and αr 2 + αr3 = 4 ⇒ αr 2(1 + r ) = 4 …(ii) + b + 18b = 25
2
From Eqs. (i) and (ii), r 2 = 4 ⇒ r = ± 2
⇒ 3b + 2 + 6 2 b = 50
Now, α ⋅ αr = p and αr 2 ⋅ αr3 = q
⇒ 3b + 6 2 b − 48 = 0
On putting r = − 2, we get
⇒ b + 2 2 b − 16 = 0
α = − 1, p = − 2 and q = − 32
⇒ b + 4 2 b − 2 2 b − 16 = 0
2
Again putting r = 2, we get α = 1 / 3 and p = − ⇒ b ( b + 4 2) − 2 2 ( b + 4 2) = 0
9
⇒ ( b − 2 2) ( b + 4 2) = 0
Since, q and p are integers.
⇒ b = 8, a = 5
Therefore, we take p = − 2 and q = − 32.
and c = 12
rankershalt.com

62 Sequences and Series

15. Let 27, 8, 12 be three terms of a GP. S 1 20 1 10 11


= 1 − 20 − 21 = 1 − 20 − 20 = 1 − 20
2 2 2 2 2 2
⇒ tm = 27, tn = 8 and t p = 12
11
AR m − 1 = 27, ARn − 1 = 8 ⇒ S =2 −
219
and AR p − 1 = 12
1/(m − n) 1/( n − p) 2. (a) We have, S n = 1 + q + q2 + … + qn and
 27 8
∴ R=  and R=  2 n
8  12  q + 1  q + 1  q + 1
Tn = 1 +   +  +…+  
1/(m − n) 1/( n − p)  2   2   2 
 27  2
⇒   =  Also, we have
8  3
101
C1 + 101
C2S1 + 101
C3S 2 + … + 101
C101S100 = αT100
⇒ 33/(m − n) ⋅ 31/( n − p) = 21/( n − p) ⋅ 23/(m − n)
3
+
1 ⇒ 101
C1 + 101
C2(1 + q ) + 101
C3 (1 + q + q 2 )
m− n n− p
3
⇒ 1 3
=1 +…+ 101
C101 (1 + q + q2 + … + q100)
+
n− p m− n = α ⋅ T100
2
(1 − q 2 )  1 − q3 
3 1 1 3 ⇒ 101
C1 + 101
C2 + C3 
101

∴ + = 0 and + =0 1− q  1− q 
m−n n− p n− p m−n
 1 − q4   1 − q101 
⇒ 3 (n − p) = n − m and 2n = 3 p – m + 101
C4   +…+ 101
C101  
Hence, there exists infinite GP for which 27, 8 and 12 as  1−q  1−q 
three of its terms.  1 − rn
= α ⋅ T100 [Qfor a GP, S n = a  ,r ≠1]
16. Let a , d be the first term and common difference of an 1−r
AP and b, r be the first term and common ratio of a GP. 1
⇒ [{ 101C1 + 101C2 + … + 101C101 }
Then, x = a + (m − 1) d and x = brm−1 1− q
y = a + (n − 1)d and y = br n−1 − { 101C1q + 101C2q 2 + … +101 C101q101 } = α ⋅ T100
z = a + ( p − 1)d and z = br p−1 1
⇒ [( 2101 − 1) − ((1 + q )101 − 1)] = αT100
Now, x − y = (m − n )d, y − z = (n − p)d (1 − q )
and z − x = ( p − m)d
[Q nC 0 + nC1 + … + nC n = 2n]
Again now, xy − z ⋅ yz − x ⋅ z x − y 2 101
− ( q + 1) 101

= [br m − 1 ]( n − p)d ⋅ [br n − 1 ]( p − m) d ⋅ [br p − 1 ](m − n)d ⇒ =α


1− q
[ n − p + p − m + m − n] d [(m − 1 )( n − p) + ( n − 1 )( p − m)+ ( p − 1 )(m − n)]d
=b ⋅r
 q + 1  q + 1
2
 q + 1 
100
= b0 ⋅ r 0 = 1 1 + +  +…+   
 2  2   2  

Topic 4 Sum of n Terms and Infinite   q + 1 
101
 1−   
Terms of a GP 2101 − ( q + 1)101  2  
⇒ = α 1 ⋅
20
 1 1− q  q+1 
1−
1. Let S = ∑ k 2k   2 
k =1  
1 2 3 4 20 q+1
S= + + + + … + 20 …(i) [Q q ≠ 1 ⇒ q + 1 ≠ 2 ⇒ ≠ 1]
2 22 23 24 2 2
 1 α [2101 − (q + 1)101 ]
On multiplying by   both sides, we get = ⇒ α = 2100
 2 (1 − q) ⋅ 2100
S 1 2 3 19 20
= + + + … + 20 + 21 …(ii) 3. Let the GP be a , ar , ar 2, ar3 , .... ∞; where a > 0 and
2 22 23 24 2 2
On subtracting Eq. (ii) from Eq. (i), we get 0 < r < 1.
Then, according the problem, we have
S 1 1 1 1 20
S− = + + + … + 20 − 21 3=
a
2 2 22 23 2 2 1−r
1 1  27
1 − 20  and = a3 + (ar )3 + (ar 2)3 + (ar3 )3 + ...
S 2 2  20 19
⇒ = − 21 
2 1 2 27 a3 a 
1− ⇒ = Q S ∞ = 
2 19 1 − r3  1 − r
 a (1 − r n ) 
Q sum of GP = , r < 1
 1 − r 
rankershalt.com

Sequences and Series 63

27 (3 (1 − r ))3  a  =
7
[(1 + 1 +…+ upto 20 terms)
⇒ = Q 3 = ⇒ a = 3 (1 − r )
19 1 − r3  1−r  9
27 27 (1 − r ) (1 + r 2 − 2r ) 1 1 1 
⇒ = −  + 2 + 3 + … + upto 20 terms 
19 (1 − r ) (1 + r + r 2)  10 10 10 

[Q (1 − r )3 = (1 − r ) (1 − r )2]  1   1   
20

⇒ r + r + 1 = 19 (r − 2r + 1)
2 2  1 −    
7 10   10  
= 20 − 
⇒ 18r 2 − 39r + 18 = 0 
9 1
⇒ 6r 2 − 13r + 6 = 0 1− 
⇒ (3r − 2) (2r − 3) = 0  10 
 
2 3  20 
∴ r = or r = (reject) [Q0 < r < 1]
3 2 Q ∑ = 20 and sum of n terms of 
 i =1 
4. Let a , ar , ar 2 are in GP, where (r > 1).  a (1 − r n ) 
 GP, S n = when (r < 1 ) 
On multiplying middle term by 2, we have  1 − r 
a , 2ar , ar 2 are in an AP.
7    1  
20
⇒ 4ar = a + ar 2 1
= 20 − 1 −   
 10 
⇒ r − 4r + 1 = 0 9  9 
2
 
4 ± 16 − 4 179 1 20 
⇒ r= =2 ± 3 7 1 7
2 =  +   = [179 + (10)− 20 ]
9  9 9  10  81
⇒ r =2 + 3 [since, AP is increasing]
5. Given, 7. We know that, the sum of infinite terms of GP is
k ⋅ 10 = 10 + 2 (11) (10) + 3(11) (10) + ... + 10(11)
9 9 1 8 2 7 9  a
 , |r| < 1
2 9 S ∞ = 1 − r
 11  11  11  ∞ ,|r| ≥ 1
⇒ k = 1 + 2   + 3  + ... + 10   ...(i)
 10  10  10
x
2 9 10 ∴ S∞ = =5 [|r| < 1]
 11  11  11  11  11 1−r
  k = 1   + 2   + ... + 9   + 10   …(ii)
 10  10  10  10  10 x
or 1−r=
On subtracting Eq. (ii) from Eq. (i), we get 5
 11 11  11
2
 11  11
9 10 5−x
k 1 −  = 1 + +   + ... +   − 10   ⇒ r= exists only when|r| < 1.
 10 10  10  10  10 5
5−x
 11 10  i.e. −1 < <1
1   − 1 5
 10 − 11   10    11
10
⇒ k  = − 10   or −10 < − x < 0
 10   11   10
 − 1 ⇒ 0 < x < 10
 10 
3
 a (r n − 1)  8. Since, sum = 4 and second term = .
Q In GP,sum of n terms = , when r > 1 4
 r −1  It is given first term a and common ratio r.
  11 10  11 
10
a 3
⇒ − k = 10 10   − 10 − 10    ⇒ = 4, ar =
  10  10 
 1−r 4
∴ k = 100 ⇒ r=
3
4a
6. Let S = 0.7 + 0.77 + 0.777 +…
a
7 77 777 ⇒ =4
= + + + … upto 20 terms 1−
3
10 102 103 4a
1 11 111  4a 2
=7 + + 3 + … upto 20 terms 
 10 10 2
10  ⇒ =4
4a − 3
79 99 999 
= + + +… upto 20 terms  ⇒ (a − 1) (a − 3) = 0
9 10 100 1000 
⇒ a = 1 or 3
7  1  1   1 
=  1 −  +  1 − 2 +  1 − 3  When a = 1, r = 3 / 4
9  10  10   10 
and when a = 3, r = 1 / 4
+…+ upto 20 terms]
rankershalt.com

64 Sequences and Series

1 3 7 15 12. Consider an infinite GP with first term 1, 2, 3, ..., n and


9. Sum of the n terms of the series + + + + ...
2 4 8 16 1 1 1 1
upto n terms can be written as common ratios , , , ... , .
2 3 4 n+1
 1  1  1  1 1
1 −  + 1 −  + 1 −  + 1 −  + ... upto n terms ∴ S1 = =2
 2  4  8  16 1 − 1 /2
1 1 1  2
= n −  + + + ... + n terms S2 = =3
2 4 8  1 − 1 /3
1 1
1 − n  M M M
2 2  = n + 2−n − 1 2n − 1
=n− S 2n − 1 = = 2n
1
1− 1 − 1 / 2n
2
∴ S12 + S 22 + S32 + ... + S 22n − 1
10. Let a n denotes the length of side of the square S n.
= 22 + 32 + 42 + ... + (2n )2
We are given, a n = length of diagonal of S n + 1. 1
⇒ an = 2 an + 1 = (2n ) (2n + 1) (4n + 1) − 1
6
a
⇒ an + 1 = n 13. Let three numbers in GP be a , ar , ar 2.
2
This shows that a1 , a 2, a3 , K form a GP with common ∴ a 2 + a 2r 2 + a 2r 4 = S 2 ...(i)
ratio 1 / 2. and a + ar + ar 2 = a S ...(ii)
n −1
 1 On dividing Eq. (i) by Eq. (ii) after squaring it, we get
Therefore, a n = a1  
 2 a 2 (1 + r 2 + r 4 ) S2
n −1 = 2 2
 1 a (1 + r + r )
2 2 2
aS
⇒ a n = 10   [ Q a1 = 10, given]
 2
(1 + r 2)2 − r 2 1
2( n − 1) ⇒ =
 1 (1 + r + r 2)2 a 2
⇒ a n2 = 100  
 2 (1 + r 2 − r ) 1
⇒ =

100
≤1 [Q a n2 ≤ 1, given] (1 + r 2 + r ) a 2
2n − 1 1
r+ +1
⇒ 100 ≤ 2n − 1 ⇒ a =
2 r
1
This is possible for n ≥ 8. r + −1
r
Hence, (b), (c), (d) are the correct answers. 1
Put r+ =y
11. Bn = 1 − An > An r
y+ 1
 3 
n
∴ = a2
1 −  −   y−1
  4  1
1 3
⇒ An < ⇒ < ⇒ y + 1 = a 2y − a 2
2 4 3 2
1+ a2 + 1  1 
4 ⇒ y= Q | y | = r + r > 2
n a2 − 1  
 3 1
⇒ −  > −
 4 6 a2 + 1
⇒ >2 [where , (a 2 − 1) ≠ 0]
Obviously, it is true for all even values of n. a2 −1
But for ⇒ |a 2 + 1| > 2|a 2 − 1|
3 1 ⇒ (a 2 + 1)2 − {2 (a 2 − 1)}2 > 0
n = 1, − <−
4 6
3
⇒ {(a 2 + 1) − 2 (a 2 − 1)}{(a 2 + 1) + 2 (a 2 − 1)} > 0
 3 27 1
⇒ (− a 2 + 3) (3a 2 − 1) > 0
n = 3,  −  = − <−
 4 64 6 1
5 ∴ < a2 < 3
 3 243 1 3
n = 5,  −  = − <− 1 
 4 1024 6 ∴ a 2 ∈  , 1 ∪ (1, 3) [Q a 2 ≠ 1]
k −1 3 
and for n = 7,
1
 3
7
2187 1 14. We have, S k = k ! =
−  = − >− 1−
1 (k − 1)!
 4 12288 6 k
Hence, minimum odd natural number n0 = 7.
rankershalt.com

Sequences and Series 65

1 1 1 6 ×1
Now, (k2 − 3k + 1) S k = {(k − 2) (k − 1) − 1} × ⇒ = +
(k − 1)! h7 2 − 54
1 1
= − 1 1 1 18
(k − 3)! (k − 1)! ⇒ = − ⇒ h7 =
100
h7 2 9 7
2
 1 1  100
⇒ ∑|(k2 − 3k + 1) S k|= 1 + 1 + 2 −  +  =4− ∴
7 18
a 4h7 = × =6
k =1
 99 ! 98 ! 100 ! 3 7
100
1002 4. Let the common ratio of the GP be r. Then,
100 ! k∑
⇒ + |(k2 − 3k + 1) S k|= 4
=1
y = xr and z = xr 2
⇒ ln y = ln x + ln r and ln z = ln x + 2 ln r
Topic 5 Harmonic Progression (HP) Let A = 1 + ln x, D = ln r
1 1 1 1 1 1
1. PLAN nth term of HP, t n = Then, = , = =
a + ( n − 1) n 1 + ln x A 1 + ln y 1 + ln x + ln r A + D
Here, a1 = 5, a 20 = 25 for HP 1 1 1
and = =
1 1 1 + ln z 1 + ln x + 2 ln r A + 2D
∴ = 5 and = 25
a a + 19d 1 1 1
Therefore, , , are in HP.
1 1 1 1 4 1 + ln x 1 + ln y 1 + ln z
⇒ + 19d = ⇒ 19d = − =−
5 25 25 5 25
5. Let a1 = 1, a 2 = 2 , ⇒ a3 = 4 , a 4 = 8
−4
∴ d= ∴ b1 = 1, b2 = 3, b3 = 7, b4 = 15
19 × 25
Clearly, b1 , b2, b3 , b4 are not in HP.
Since, an < 0
1 Hence, Statement II is false.
⇒ + (n − 1) d < 0
5 Statement I is already true.


1

4
(n − 1) < 0 ⇒ (n − 1) >
95 6. Since, cos (x − y), cos x and cos (x + y) are in HP.
5 19 × 25 4 2 cos (x − y) cos (x + y)
∴ cos x =
⇒ n >1 +
95
or n > 24.75 cos (x − y) + cos (x + y)
4
⇒ cos x (2 cos x ⋅ cos y) = 2 {cos 2 x − sin 2 y}
∴ Least positive value of n = 25
⇒ cos 2 x ⋅ cos y = cos 2 x − sin 2 y
2. Since, a , b, c, d are in AP. ⇒ cos x (1 − cos y) = sin 2 y
2

a b c d y y y
⇒ , , , are in AP. ⇒ cos 2 x ⋅ 2 sin 2 = 4 sin 2 ⋅ cos 2
abcd abcd abcd abcd 2 2 2
y
1 1 1 1 ⇒ cos 2 x ⋅ sec2 = 2
⇒ , , , are in AP. 2
bcd cda abd abc
y
⇒ bcd , cda , abd , abc are in HP. ∴ cos x ⋅ sec = ± 2
2
⇒ abc, abd , cda , bcd are in HP.
7. Since, a , b, c are in an AP.
3. Since, a1 , a 2, a3 , K , a10 are in AP. ∴ 2b = a + c
Now, a10 = a1 + 9d and a 2, b2, c2 are in HP.
⇒ 3 = 2 + 9d 2a 2c2  a + c
2
2a 2c2
⇒ b2 = ⇒   = 2 2
⇒ d = 1 / 9 and a 4 = a1 + 3d a 2 + c2  2  a +c
⇒ a 4 = 2 + 3(1 / 9) = 2 + 1 / 3 = 7 / 3 ⇒ (a 2 + c2)(a 2 + c2 + 2ac) = 8a 2c2
Also, h1 , h2, h3 , K , h10 are in HP. ⇒ (a 2 + c2) + 2ac(a 2 + c2) = 8a 2c2
1 1 1 1
⇒ , , ,K, are in AP. ⇒ (a + c ) + 2ac(a 2 + c2) + a 2c2 = 9a 2c2
2 2
h1 h2 h3 h10
⇒ (a 2 + c2 + ac)2 = 9a 2c2
Given, h1 = 2, h10 = 3
⇒ a 2 + c2 + ac = 3ac
1 1 1 1
∴ = + 9d1 ⇒ = + 9d1 ⇒ a 2 + b2 – 2ac = 0
h10 h1 3 2
1 ⇒ (a – c)2 = 0 ⇒ a = c
⇒ − = 9d1 and if a = c ⇒ b = c or a 2 + c2 + ac = – 3ac
6
⇒ d1 = −
1
and
1
=
1
+ 6d1 ⇒ a 2 + c2 + 2ac = –2ac
54 h7 h1 ⇒ (a + c)2 = –2ac
rankershalt.com

66 Sequences and Series

ac
⇒ 4b2 = –2ac ⇒ b2 = – Topic 6 Relation between AM, GM, HM
2
c and Some Special Series
Hence, are in GP.
a , b, –
2 1. Given series,
c 13 + 23 13 + 23 + 33
∴ Either a = b = c or a , b, − are in GP. S =1+ + + ... +
2 1+ 2 1+ 2+ 3
8. Since, a , A1 , A2, b are in AP. 13 + 23 + 33 + K + 153 1
− (1 + 2 + 3 + K + 15)
⇒ A1 + A2 = a + b 1 + 2 + 3 + K + 15 2
a , G1 , G2, b are in GP ⇒ G1G2 = ab = S1 − S 2 (let)
where,
and a , H 1 , H 2, b are in HP.
13 + 23 13 + 23 + 33
3ab 3ab S1 = 1 + + +K+
⇒ H1 = , H2 = 1+ 2 1+ 2+ 3
2b + a b + 2a
13 + 23 + 33 + K + 153
1 1 1 1
∴ + = + 1 + 2 + 3 + K + 15
H1 H 2 a b 2
 n( n + 1)
H 1 + H 2 A1 + A2 1 1  
⇒ = = + …(i) 15
13 + 23 + K + n3 15
 2 
H 1H 2 G1G2 a b = ∑ = ∑
n =1 1 + 2 + K + n n =1
n ( n + 1)
G1G2 ab
Now, = 2
H 1H 2  3ab   3ab   n + 1)
2
     n ( n + 1) n
n ( n
 2b + a   b + 2a  Q ∑ r3 =   and ∑ r = 
 r = 1  2  r =1 2 
(2a + b) (a + 2b)
= …(ii) 15
n( n + 1) 1 15
9ab = ∑ = ∑ (n 2 + n )
From Eqs. (i) and (ii), we get n =1 2 2 n =1
G1G2 A + A2 ( 2a + b) (a + 2b) 1 15 × 16 × 31 15 × 16 
= 1 = = +
H 1H 2 H 1 + H 2 9ab 2  6 2 
9. (i) Now, a + b = (a + x + y + z + b) − (x + y + z )
 n n (n + 1) (2n + 1) 
=
5
(a + b) − 15 Q ∑ r 2 = 
2  r = 1 6 
[since, a , x, y, z are in AP] 1
= [(5 × 8 × 31) + (15 × 8)]
5 2
∴ Sum = (a + b) ⇒ a + b = 10 …(i)
2 = (5 × 4 × 31) + (15 × 4)
1 1 1 1 1 = 620 + 60 = 680
Since, a , x, y, z , b are in HP, then , , , ,
are in AP. a x y z b 1
and S 2 = (1 + 2 + 3 + K + 15)
1 1  1 1 1 1 1   1 1 1 2
Now, + = + + + +  − + +  1 15 × 16
a b  a x y z b  x y z  = × = 60
2 2
5  1 1 5
=  +  − Therefore, S = S1 − S 2 = 680 − 60 = 620.
2  a b 3
a + b 10 9 × 10 2. Given series is
⇒ = ⇒ ab = [from Eq. (i)] 3 × 13 5 × (13 + 23 ) 7 × (13 + 23 + 33 )
ab 9 10 + + + ...
1 2
1 +2
2 2
12 + 22 + 33
⇒ ab = 9 …(ii)
So, nth term
On solving Eqs. (i) and (ii), we get
(3 + (n − 1)2)(13 + 23 + 33 ... + n3 )
a = 1, b = 9 Tn =
12 + 22 + 32 + K + n 2
(ii) LHS = log (x + z ) + log (x + z − 2 y)
2
  2 xz    2xz   n (n + 1)
= log (x + z ) + log x + z − 2   Q y = x + z  (2n + 1) ×  
  x + z   2 
=
n (n + 1)(2n + 1)
(x − z )2
= log (x + z ) + log 6
(x + z )
 n 3  n (n + 1)  2 n n (n + 1)(2n + 1) 
= 2 log (x − z ) = RHS Q Σ r =   and Σ r 2 = 
 r =1  2  r = 1 6 
rankershalt.com

Sequences and Series 67

3n (n + 1) 3 2 1 + 2 + 3 + ... + k
So, Tn =
= (n + n ) 5. Since, S k =
2 2 k
k ( k + 1) k + 1
Now, sum of the given series upto n terms = =
3 2k 2
S n = ΣTn = [Σn 2 + Σn ] 2
2  k + 1 1
So, S 2k =   = (k + 1)
2
… (i)
3  n (n + 1)(2n + 1) n (n + 1)   2  4
=  + 
2 6 2 5 10
Now, A = S 12 + S 22 + S 23 + ... S 10
2
= ∑ S 2k
3 10 × 11 × 21 10 × 11  12 k =1
∴ S10 = +
2  6 2  5 1 10
1
3

12
A=
4 ∑ (k + 1)2 = 4 [22 + 32 + 42 + ... 112]
k =1
= [(5 × 11 × 7) + (5 × 11)]
2 1 11 × (11 + 1) (2 × 11 + 1) 2
=  −1 
3 3 4 6 
= × 55(7 + 1) = × 55 × 8 = 3 × 55 × 4
n ( n + 1) ( 2n + 1)
2 2 [Q ∑ n 2 = ]
6
= 12 × 55 = 660
1 11 × 12 × 23  1
= − 1 = [(22 × 23) − 1]
3. (b) Given series is 4  6  4
1 + (2 × 3) + (3 × 5) + (4 × 7) + …upto 1 1
= [506 − 1] = [505]
11 terms. 4 4
Now, the rth term of the series is a r = r (2r − 1) 5 505
⇒ A= ⇒ A = 303
∴Sum of first 11-terms is 12 4
11 11 11 11
xm yn
S11 = ∑ r (2r − 1) = ∑ (2r 2 − r ) = 2 ∑ r 2 − ∑ r 6. Consider,
(1 + x )(1 + y2n )
2m
r =1 r =1 r =1 r =1

11 × (11 + 1)(2 × 11 + 1) 11 × (11 + 1) 1


=2 − =
6 2 ( x + x )( y n + y − n )
m −m

 n n (n + 1)(2n + 1) n
n (n + 1)  By using AM ≥ GM (because x , y ∈ R + ), we get
Q ∑ r 2 = and ∑ r = 
 r = 1 6 2  ( x m + x − m ) ≥ 2 and ( y n + y − n ) ≥ 2
r =1
1
 11 × 12 × 23  11 × 12 [Q If x > 0, then x + ≥ 2]
=  −  x
   2 
3 ⇒ ( x m + x − m )( y n + y − n ) ≥ 4
= (11 × 4 × 23) − (11 × 6) = 11(92 − 6) = 11 × 86 = 946 1 1
⇒ m −m −n

4. Given series is ( x + x )( y + y ) 4
n

 3
3
 1  1
3
 3
3 3 1
∴ Maximum value = .
  + 1  +  2  + 3 +  3  + ...
3
 4  2  4  4 4
3 3 3 3 7. General term of the given series is
 3  6  9  12
Let S =   +   +   +   3r (12 + 22 + K + r 2) 3r [r (r + 1) (2r + 1)]
 4  4  4  4 Tr = =
 15
3 2r + 1 6(2r + 1)
+   + … + upto 15 terms 1 3
 4 = (r + r ) 2

3 2
 3 15
1 15
=   [13 + 23 + 33 + 43 + 53 + ... + 153 ] Now, required sum = ∑ Tr = ∑ (r3 + r 2)
 4 2 r =1
3 2 r =1
 3  15 × 16
=    1  n (n + 1)  2 n (n + 1) (2n + 1) 
 4  2  =   + 
2  2 6 n = 15
 3  n ( n + 1)
2 
Q1 + 23 + 33 + ... + n3 =   , n ∈N 1  n (n + 1)  n 2 + n 2n + 1 
  2   =   + 
2  2  2 3 
n = 15
27 225 × 256
= × 1  n (n + 1) (3n 2 + 7n + 2) 
64 4 =  
2  2 6 n = 15
= 27 × 225
1 15 × 16 (3 × 225 + 105 + 2)
⇒ S = 27 × 225 = 225 k [given] = × × = 7820
2 2 6
⇒ k = 27.
rankershalt.com

68 Sequences and Series


2 2 2 2
8. We have, a1 , a 2, a3 , … a 49 are in AP.  8  12  16  24
12 =   +   +   + 42 +   + ... to 10 terms
 5 5 5 5
∑ a4k + 1 = 416 and a9 + a43 = 66 1 2
k =0
= 2 (8 + 12 + 16 + 20 + 242 + ... to 10 terms)
2 2 2

Let a1 = a and d = common difference 5


Q a1 + a 5 + a 9 + L + a 49 = 416 42 2
= 2 (2 + 32 + 42 + 52 + ... to 10 terms)
∴ a + (a + 4d) + (a + 8d) + …(a + 48d) = 416 5

13
(2a + 48d) = 416 42
= 2 (22 + 32 + 42 + 52 + ... + 112)
2 5
⇒ a + 24d = 32 …(i) 16
Also , a 9 + a 43 = 66 = ((12 + 22 + ... + 112) − 12)
25
∴ a + 8d + a + 42d = 66 16  11 ⋅ (11 + 1) (2 ⋅ 11 + 1) 
⇒ 2a + 50d = 66 =  − 1
25  6 
⇒ a + 25d = 33 …(ii)
16 16 16 16
Solving Eqs. (i) and (ii), we get = (506 − 1) = × 505 ⇒ m= × 505 = 101
25 25 5 25
a = 8 and d = 1
Now, a12 + a 22 + a 23 + L + a17 2
= 140m 11. Given, m is the AM of l and n.
8 + 9 + 10 + … + 24 = 140m
2 2 2 2 ∴ l + n = 2m ... (i)
and G1 , G2, G3 are geometric means between l and n.
⇒ (12 + 22 + 32 + … + 242) − (12 + 22 + 32 + … + 72) = 140m
24 × 25 × 49 7 × 8 × 15 l, G1 , G2, G3 , n are in GP.
⇒ − = 140m
6 6 Let r be the common ratio of this GP.
3× 7 × 8 × 5 1
⇒ (7 × 5 − 1) = 140m  n 4
6 ∴ G1 = lr, G2 = lr , G3 = lr , n − lr
2 3 4
⇒ r= 
 l
⇒ 7 × 4 × 5 × 34 = 140m
⇒ 140 × 34 = 140m ⇒ m = 34 Now, G14 + 2G24 + G34 = (lr )4 + 2(lr 2)4 + (lr3 )4
= l4 × r 4 (1 + 2r 4 + r 6 ) = l4 × r 4 (r 4 + 1)2
9. We have, 2
12 + 2⋅ 22 + 32 + 2⋅ 42 + 52 + 2⋅ 62 + … n  n + l
= l4 ×   = ln × 4 m = 4lm n
2 2

A = sum of first 20 terms l  l 


B = sum of first 40 terms 12. PLAN Write the nth term of the given series and simplify it to get its
∴A = 12 + 2⋅ 22 + 32 + 2⋅ 42 + 52 + 2⋅ 62 + … + 2⋅ 202 lowest form. Then, apply, S n = ∑ Tn
A = (12 + 22 + 32 + … + 202) + (22 + 42 + 62 + … + 202)
A = (12 + 22 + 32 + … + 202) + 41 ( 2 + 22 + 32 + …+102) 13 13 + 23 13 + 23 + 33
Given series is + + + ...
20 × 21 × 41 4 × 10 × 11 × 21 1 1+3 1+3+5
A= + Let Tn be the nth term of the given series.
6 6
20 × 21 20 × 41 × 63 13 + 23 + 33 + ... + n3
A= (41 + 22) = ∴ Tn =
6 6 1 + 3 + 5 + ... + upto n terms
Similarly 2
 n (n + 1) 
B = (12 + 22 + 32 + … + 402) + 41 ( 2 + 22 +…+ 202)  
 = (n + 1)
2
=
2
40 × 41 × 81 4 × 20 × 21 × 41
B= + n 2
4
6 6 9
40 × 41 40 × 41 × 123 (n + 1)2 1 2
B= (81 + 42) = S9 = ∑ 4 = 4 (2 + 32 + ... + 102) + 12 – 12]
6 6 n =1
Now, B − 2A = 100λ 1 10(10 + 1)(20 + 1)  384
40 × 41 × 123 2 × 20 × 21 × 63 = – 1 = = 96
∴ − = 100λ 4  6  4
6 6
40 40 π
⇒ (5043 − 1323) = 100λ ⇒ × 3720 = 100λ 13. Here, α ∈ (0, ) ⇒ tan α > 0
6 6 2
40 × 620 tan 2 α
⇒ 40 × 620 = 100λ ⇒λ = = 248 x2 + x +
100
x2 + x tan 2 α
10. Let S10 be the sum of first ten terms of the series. Then, ∴ ≥ x2 + x ⋅
2 x2 + x
we have
2 2 2 2 [using AM ≥ GM]
 3  2  1  4
S10 = 1  + 2  + 3  + 4 +
2
4  tan 2 α
 5  5  5  5 ⇒ x2 + x + ≥ 2 tan α
+ ... to 10 terms x2 + x
rankershalt.com

Sequences and Series 69

14. Given, a1 a 2 a3 ... a n = c Gn = An − 1 H n − 1 ,


2 An − 1 H n − 1
⇒ a1 a 2 a3 ... (a n − 1 )(2a n ) = 2c …(i) Hn =
a1 + a 2 + a3 + ... + 2a n An − 1 + H n − 1
∴ ≥ (a1 ⋅ a 2 ⋅ a3 ... 2a n )1/ n
n Clearly, G1 = G2 = G3 = ... = ab .
[using AM ≥ GM]
21. A2 is AM of A1 and H 1 and A1 > H 1
⇒ a1 + a 2 + a3 + ... + 2a n ≥ n (2c)1/ n [from Eq. (i)]
⇒ A1 > A2 > H 1
⇒ Minimum value of
A3 is AM of A2 and H 2 and A2 > H 2
a1 + a 2 + a3 + ... + 2a n = n (2c)1/ n ⇒ A2 > A3 > H 2
15. Since, AM ≥ GM, then M M M
(a + b) + (c + d ) ∴ A1 > A2 > A3 > ...
≥ (a + b)(c + d ) ⇒ M ≤ 1
2
Also, (a + b) + (c + d ) > 0 [Q a , b, c, d > 0]
22. As above, A1 > H 2 > H 1 , A2 > H 3 > H 2
∴ H 1 < H 2 < H 3 < ...
∴ 0 < M ≤1
1 1 1 1
16. Let α , β be the roots of given quadratic equation. Then, 23. Given, a (n ) = 1 +
+ + +K+ n
2 3 4 2 −1
4+ 5 8+2 5
α+β= and α β =  1 1  1 1  1 1
5+ 2 5+ 2 =1 +  +  +  + K +  +  + K+ 
 2 3  4 7  8 15
Let H be the harmonic mean between α and β, then  1 1 
+ K +  n −1 + K + n 
2 αβ 16 + 4 5 2 2 − 1
H = = =4
α+β 4+ 5  1 1  1 1 1  1 1 1
<1 +  +  +  + + K+  +  + + K + 
 2 2  4 4 4  8 8 8
17. Since, product of n positive numbers is unity.
 1 1 
⇒ x1 ⋅ x2 ⋅ x3 ... xn = 1 ... (i) + K +  n − 1 + ... + n − 1 
2 2 
x1 + x2 + ... + xn
Using AM ≥ GM, ≥ (x1 ⋅ x2 ... xn )1/ n 2 4 8 2n −1
n =1 + + + + K + n −1
2 4 8 2
⇒ x1 + x2 + ... + xn ≥ n (1)1/ n [from Eq. (i)]
= 1 + 1 + 1 + 1 + ... + 1 = n
Hence, sum of n positive numbers is never less than n. 1444 424444 3
(n ) times
18. Since, AM > GM
Thus, a(100) ≤ 100
(b + c − a ) + (c + a − b)
∴ > (b + c − a )(c + a − b)1/ 2 Therefore, (a) is the answer.
2
1  1 1  1 1
⇒ c > [(b + c − a )(c + a − b)] 1/ 2
…(i) Again, a (n ) = 1 + +  +  +  + K + 
2  3 4  5 8
Similarly b > [(a + b − c)(b + c − a )] 1/ 2
…(ii)
 1 1 1
and a > [(a + b − c)(c + a − b)]1/ 2 …(iii) + K +  n −1 + K + n − n
2 +1 2  2
On multiplying Eqs. (i), (ii) and (iii), we get
1  1 1  1 1 1
abc > (a + b − c)(b + c − a )(c + a − b) >1 + +  +  +  + +K+ 
2  4 4   8 8 8
Hence, (a + b − c)(b + c − a )(c + a − b) − abc < 0
 1 1 1
+ K +  n + K + n − n
19. Since, x1 , x2 ,…, xn are positive real numbers. 2 2  2
∴ Using nth power mean inequality 1 2 4 2n − 1 1
2 =1 + + + +K+ n − n
x12 + x22 + ... + xn2  x + x2 + ... + xn  2 4 8 2 2
≥ 1 
n  n  1 1 1 1 1  1 n
= 1 + + + + K + − n = 1 − n  +
2 2 2 2 2 2 2  2  2
n 2  n 2  n   n   n  14442444 3
⇒ ∑ xi ≥ ∑ xi ⇒ n  ∑ xi2 ≥  ∑ xi  n times
n  i = 1   i = 1     
 i =1   i =1   1  200
Therefore, a (200) > 1 − 200  + > 100
20. Let a and b are two numbers. Then,  2  2
a+b 2ab Therefore, (d) is also the answer.
A1 = ; G1 = ab ; H 1 =
2 a+b
24. Since, first and (2n − 1)th terms are equal.
An − 1 + H n − 1
An = , Let first term be x and (2n − 1) th term be y,
2
whose middle term is tn.
rankershalt.com

70 Sequences and Series

x+ y 1
Thus, in arithmetic progression, tn = =a log a x +
2 log a x
28. Since, > 1, using AM > GM
In geometric progression, tn = xy = b 2
2xy Here, equality holds only when x = a which is not
In harmonic progression, tn = =c
x+ y possible. So, log a x + log x a is greater than 2.
⇒ b2 = ac and a > b > c [using AM > GM > HM] Hence, it is a false statement.
Here, equality holds (i. e. a = b = c) only if all terms are 29. Here, (1 + a )(1 + b)(1 + c)
same. Hence, options (a), (b) and (d) are correct.
= 1 + a + b + c + ab + bc + ca + abc …(i)
25. Let the two positive numbers be a and b.
a + b + c + ab + bc + ca + abc
a+b Since, ≥ (a b c )
4 4 4 1/7
∴ x= [since, x is AM between a and b] … (i) 7
2
a y z [using AM ≥ GM]
and = = [since, y, z are GM’s between a and b] ⇒ a + b + c + ab + bc + ca + abc ≥ 7(a 4b4c4 )1/7
y z b
y2 z2 ⇒ 1 + a + b + c + ca + abc > 7(a 4b4c4 )1/7 …(ii)
∴ a= and b = From Eqs. (i) and (ii), we get
z y
On substituting the values of a and b in Eq. (i), we get (1 + a )(1 + b)(1 + c) > 7(a 4b4c4 )1/7

y2 z 2 or {(1 + a )(1 + b)(1 + c)}7 > 77 (a 4b4c4 )


2x = +
z y 30. Let Gm be the geometric mean of G1 , G2, ... , Gn.
y +z
3 3 ⇒ Gm = (G1 ⋅ G2 K Gn )1/ n
⇒ = 2x
yz = [(a1 ) ⋅ (a1 ⋅ a1r )1/ 2 ⋅ (a1 ⋅ a1r ⋅ a1r 2)1/3
y +z
3 3 K (a1 ⋅ a1r ⋅ a1r 2 K a1r n − 1 )1/ n ]1/ n
⇒ =2
xyz where, r is the common ratio of GP a1 , a 2, K , a n.
( n − 1 )n
26. Let the two positive numbers be ka and a , a > 0.
= [(a1 ⋅ a1. K n times ) (r 1/ 2
⋅r
3 /3
⋅r 6/ 4
Kr 2n
)]1/ n
Then, G = ka ⋅ a = k ⋅ a 1 3 n −1
+1+ + L+
and H =
2(ka )a
=
2ka = [a1n ⋅ r 2 2 2 ]1/ n

ka + a k + 1 1/ n
 1  ( n − 1 )n    n −1 
 
Again,
H 4
= [given] = a1 r 2  2   = a1 r 4  …(i)
G 5    
 
2ka
a1 + a 2 + ... + a n a1 (1 − r n )
k+1 4 2 k 4 Now, An = =
⇒ = ⇒ = n n (1 − r )
ka 5 k+1 5
n
⇒ 5 k = 2k + 2 and Hn =
1 1 1
⇒ 2k − 5 k + 2 = 0  + +K+ 
 a1 a 2 an
5 ± 25 − 16 5 ± 3 1
⇒ k= = = 2, n
4 4 2 =
1  1 1 
⇒ k = 4, 1 / 4. 1 + + K + n − 1 
a1  r r 
Hence, the required ratio is 4 : 1.
a1n (1 − r ) r n − 1
27. Using AM ≥ GM, =
1 − rn
1 + x2n
≥ 1 ⋅ x2n a1 (1 − r n ) a1n (1 − r )r n − 1
2 ∴ An ⋅ H n = × = a12r n − 1
n (1 − r ) (1 − r n )
1 + x2n
⇒ ≥ xn n n
2 ⇒ ∏ Ak H k = ∏ (a12r n − 1 )
x n
1 k =1 k =1
⇒ ≤
1+x 2n
2 = (a12 ⋅ a12 ⋅ a12K n times ) × r 0 ⋅ r1 ⋅ r 2K r n − 1
xn ⋅ ym 1 = a12n ⋅ r1 + 2 + K + ( n − 1)
∴ ≤ n ( n − 1) n −1
(1 + x ) (1 + y2m ) 4
2n
= a12nr 2 = [a1r 4 ]2 n

Hence, it is false statement.


rankershalt.com

Sequences and Series 71

= [Gm ]2n [from Eq. (i)]  n + 1


2

1/ 2 n ⇒ q < p or q >   p
 n   n − 1
Gm =  ∏ AkH k 
k = 1   n + 1 2 
Q   p > p
⇒ Gm = ( A1 A2 K AnH 1H 2 K H n )1/ 2n  n − 1  
2
31. Let two numbers be a and b and A1 , A2, ... , An be n  n + 1
Hence, q cannot lie between p and   p.
arithmetic means between a and b. Then,  n − 1
a , A1 , A2, ... , An , b are in AP with common difference
b−a 32. Since a , b, c > 0
d=
n+1 (a + b + c)
⇒ > (abc)1/3 …(i)
b−a 3
∴ p = A1 = a + d = a +
n+1 [using AM ≥ GM]
na + b 1 1 1
⇒ p= …(i) + + 1/3
n+1 a b c ≥  1 ⋅ 1 ⋅ 1
Also,   …(ii)
3  a b c
Let H 1 , H 2, ... , H n be n harmonic means between a and
b. [using AM ≥ GM]
1 1 1 1 1 On multiplying Eqs. (i) and (ii), we get
∴ , , , ... , , is an AP with common
a H1 H 2 Hn b  1 1 1
(a + b + c)  + + 
(a − b)  a b c 1
difference, D = . ≥ (abc)1/3
(n + 1) ab 9 (abc)1/3
1 1 1 1 (a − b)  1 1 1
∴ = +D ⇒ = + ∴ (a + b + c)  + +  ≥ 9
q a q a (n + 1) ab  a b c
1 nb + a
⇒ = 33. Plan
q (n + 1) ab (i) If a, b, c are in GP, then they can be taken as a, ar, ar 2
( n + 1) ab where r, (r ≠ 0) is the common ratio.
⇒ q= ... (ii)
nb + a x + x2 + K + xn
(ii) Arithmetic mean of x1, x2, K , xn = 1
n
From Eq. (i),
Let a , b, c be a , ar , ar 2, where r ∈ N
b = (n + 1) p − na.
a+ b+ c
Putting it in Eq. (ii), we get Also, =b+2
3
q { n (n + 1) p − n 2a + a } = (n + 1) a {(n + 1) p − na }
⇒ a + ar + ar 2 = 3 (ar ) + 6
⇒ n (n + 1) a 2 − {(n + 1)2 p + (n 2 − 1)q}a
⇒ ar 2 − 2ar + a = 6
+ n (n + 1) pq = 0 6
⇒ (r − 1)2 =
⇒ na 2 − {(n + 1) p + (n − 1)q} a + npq = 0 a
Since, a is real, therefore Since, 6 /a must be perfect square and a ∈ N .
{(n + 1) p + (n − 1)q}2 − 4n 2pq > 0 So, a can be 6 only.
⇒ (n + 1) p + (n − 1) q + 2 (n − 1) pq − 4n pq > 0
2 2 2 2 2 2
⇒ r −1 = ± 1 ⇒ r =2
⇒ (n + 1)2 p2 + (n − 1)2q2 − 2 (n 2 + 1) pq > 0 a 2 + a − 14 36 + 6 − 14
and = =4
2 (n 2 + 1)  n + 1
2 a+1 7
⇒ q2 − pq +   p2 > 0
(n − 1) 2
 n − 1 34. Using AM ≥ GM,
 2 2 a −5 + a −4 + a −3 + a −3 + a −3 + 1 + a 8 + a10
 n + 1   n + 1) 2
⇒ q − 1 + 
2
  pq +   p >0 8
  n − 1   n −1 1

 ≥ (a − 5 ⋅ a − 4 ⋅ a − 3 ⋅ a − 3 ⋅ a − 3 ⋅ 1 ⋅ a 8 ⋅ a10 ) 8
 n + 1 
2
⇒ (q − p) q −   p > 0 ⇒ a −5 + a −4 + 3a −3 + 1 + a 8 + a10 ≥ 8 ⋅ 1
  n − 1 

Hence, minimum value is 8.

Download Chapter Test


http://tinyurl.com/y6r7ns95 or
rankershalt.com

4
Permutations
and Combinations
7. In a collage of 300 students, every student reads
Topic 1 General Arrangement 5 newspapers and every newspaper is read by 60
students. The number of newspapers is (1998, 2M)
Objective Questions I (Only one correct option) (a) atleast 30
1. The number of four-digit numbers strictly greater than (b) atmost 20
4321 that can be formed using the digits 0, 1, 2, 3, 4, 5 (c) exactly 25
(repetition of digits is allowed) is (2019 Main, 8 April II) (d) None of the above
(a) 306 (b) 310 8. A five digits number divisible by 3 is to be formed using
(c) 360 (d) 288 the numbers 0, 1 , 2, 3 , 4 and 5, without repetition. The
2. How many 3 × 3 matrices M with entries from {0, 1, 2} total number of ways this can be done, is (1989, 2M)
are there, for which the sum of the diagonal entries of (a) 216 (b) 240
M T M is 5 ? (2017 Adv.) (c) 600 (d) 3125
(a) 198 (b) 162 (c) 126 (d) 135 9. Eight chairs are numbered 1 to 8. Two women and
3. The number of integers greater than 6000 that can be three men wish to occupy one chair each.
formed using the digits 3, 5, 6, 7 and 8 without First the women choose the chairs from amongst the
repetition is (2015 Main) chairs marked 1 to 4 and then the men select the chairs
(a) 216 (b) 192 from amongst the remaining. The number of possible
(c) 120 (d) 72 arrangements is
(a) 6C3 × 4 C2 (b) 4 P2 × 4 P3 (1982, 2M)
4. The number of seven-digit integers, with sum of the
digits equal to 10 and formed by using the digits 1, 2 and (c) 4 C2 + 4 P3 (d) None of these
3 only, is (2009) 10. The different letters of an alphabet are given. Words
(a) 55 (b) 66 (c) 77 (d) 88 with five letters are formed from these given letters.
Then, the number of words which have at least one
5. How many different nine-digit numbers can be formed
letter repeated, is (1980, 2M)
from the number 22 33 55 888 by rearranging its digits
so that the odd digits occupy even positions? (2000, 2M) (a) 69760 (b) 30240
(c) 99748 (d) None
(a) 16 (b) 36
(c) 60 (d) 180
Analytical & Descriptive Question
6. An n-digit number is a positive number with exactly n
digits. Nine hundred distinct n-digit numbers are to be 11. Eighteen guests have to be seated half on each side of a
formed using only the three digits 2,5 and 7. The smallest long table. Four particular guests desire to sit on one
value of n for which this is possible, is (1998, 2M) particular side and three other on the other side.
(a) 6 (b) 7 (c) 8 (d) 9 Determine the number of ways in which the sitting
arrangements can be made. (1991, 4M)
rankershalt.com

Permutations and Combinations 73

Match the Column


Match the conditions/expressions in Column I with statement in Column II.
12. Consider all possible permutations of the letters of the word ENDEANOEL. (2008, 6M)

Column I Column II
A. The number of permutations containing the word p. 5!
ENDEA, is
B. The number of permutations in which the letter E q. 2 × 5!
occurs in the first and the last positions, is
C. The number of permutations in which none of the r. 7 × 5!
letters D, L, N occurs in the last five positions, is
D. The number of permutations in which the letters A, E, s. 21 × 5!
O occur only in odd positions, is

Topic 2 Properties of Combinational and General Selections


Objective Questions I (Only one correct option) 20  20
Ci − 1 
3
k
1. The number of ways of choosing 10 objects out of 31
7. If ∑ 
i=1
20 20
 = , then k equals
Ci + Ci − 1  21 (2019 Main, 10 Jan I)
objects of which 10 are identical and the remaining 21
(a) 100 (b) 400 (c) 200 (d) 50
are distinct, is (2019 Main, 12 April I)
(a) 2 20
−1 (b) 2 21
(c) 2 20
(d) 2 20
+1 8. A man X has 7 friends, 4 of them are ladies and 3 are
men. His wife Y also has 7 friends, 3 of them are ladies
2. Suppose that 20 pillars of the same height have been and 4 are men. Assume X and Y have no common
erected along the boundary of a circular stadium. If the friends. Then, the total number of ways in which X and
top of each pillar has been connected by beams with the Y together can throw a party inviting
top of all its non-adjacent pillars, then the total number 3 ladies and 3 men, so that 3 friends of each of X and Y
of beams is (2019 Main, 10 April II)
are in this party, is (2017 Main)
(a) 180 (b) 210 (c) 170 (d) 190 (a) 485 (b) 468 (c) 469 (d) 484
3. Some identical balls are arranged in rows to form an 9. Let S = {1, 2, 3, …… , 9}. For k = 1, 2 , …… 5, let N k be the
equilateral triangle. The first row consists of one ball,
the second row consists of two balls and so on. If 99 more number of subsets of S, each containing five elements
identical balls are added to the total number of balls out of which exactly k are odd. Then
used in forming the equilateral triangle, then all these N1 + N 2 + N 3 + N 4 + N 5 = (2017 Adv.)
balls can be arranged in a square whose each side (a) 210 (b) 252 (c) 126 (d) 125
contains exactly 2 balls less than the number of balls 10. A debate club consists of 6 girls and 4 boys. A team of
each side of the triangle contains. Then, the number of 4 members is to be selected from this club including the
balls used to form the equilateral triangle is selection of a captain (from among these 4 members) for
(2019 Main, 9 April II) the team. If the team has to include atmost one boy, the
(a) 262 (b) 190 (c) 225 (d) 157 number of ways of selecting the team is (2016 Adv.)
4. There are m men and two women participating in a (a) 380 (b) 320 (c) 260 (d) 95
chess tournament. Each participant plays two games 11. Let Tn be the number of all possible triangles formed by
with every other participant. If the number of games
played by the men between themselves exceeds the joining vertices of an n-sided regular polygon. If
number of games played between the men and the Tn + 1 − Tn = 10, then the value of n is (2013 Main)
women by 84, then the value of m is (2019 Main, 12 Jan II) (a) 7 (b) 5 (c) 10 (d) 8
(a) 12 (b) 11 (c) 9 (d) 7 12. If r , s, t are prime numbers and p, q are the positive
5. If n C 4 , n C 5 and n C 6 are in AP, then n can be integers such that LCM of p, q is r 2s4 t 2 ,then the number
(2019 Main, 12 Jan II) of ordered pairs ( p, q) is (2006, 3M)
(a) 9 (b) 11 (c) 14 (d) 12 (a) 252 (b) 254 (c) 225 (d) 224
25 5
6. If ∑{
r = 0
50
Cr ⋅ 50 − r
C 25 − r } = K ( C 25 ),
50
13. The value of the expression C 4 +47
∑ 52− j
C 3 is
j =1
(1980, 2M)
then, K is equal to (2019 Main, 10 Jan II) (a) 47 C5 (b) 52C5
(a) 2 24
(b) 2 25
−1 (c) 225
(d) (25) 2
(c) 52C4 (d) None of these
rankershalt.com

74 Permutations and Combinations

Match Type Question 17. In a certain test, a i students gave wrong answers to at
least i questions, where i = 1, 2, K , k. No student gave
14. In a high school, a committee has to be formed from a more that k wrong answers. The total number of wrong
group of 6 boys M1 , M 2, M 3, M 4 , M 5, M 6 and 5 girls G1 , answers given is … . (1982, 2M)
G 2, G 3, G 4 , G 5.
(i) Let α1 be the total number of ways in which the True/False
committee can be formed such that the committee 18. The product of any r consecutive natural numbers is
has 5 members, having exactly 3 boys and 2 girls. always divisible by r !. (1985, 1M)
(ii) Let α 2 be the total number of ways in which the
committee can be formed such that the committee Analytical & Descriptive Questions
has at least 2 members, and having an equal
19. A committee of 12 is to be formed from 9 women and 8
number of boys and girls.
men. In how many ways this can be done if at least five
(iii) Let α 3 be the total number of ways in which the women have to be included in a committee ? In how
committee can be formed such that the committee many of these committees
has 5 members, at least 2 of them being girls.
(i) the women are in majority?
(iv) Let α 4 be the total number of ways in which the
(ii) the men are in majority? (1994, 4M)
committee can be formed such that the committee
has 4 members, having at least 2 girls such that both 20. A student is allowed to select atmost n books from n
M1 and G1 are NOT in the committee together. collection of (2n + 1) books. If the total number of ways
(2018 Adv.) in which he can select at least one books is 63, find the
List-I List-II value of n. (1987, 3M)

21. A box contains two white balls, three black balls and
P. The value of α1 is 1. 136
four red balls. In how many ways can three balls be
Q. The value of α 2 is 2. 189 drawn from the box, if at least one black ball is to be
included in the draw ? (1986, 2 12 M)
R. The value of α 3 is 3. 192
22. 7 relatives of a man comprises 4 ladies and 3
S. The value of α 4 is 4. 200 gentlemen, his wife has also 7 relatives ; 3 of them are
5. 381 ladies and 4 gentlemen. In how many ways can they
invite a dinner party of 3 ladies and 3 gentlemen so
6. 461 that there are 3 of man’s relative and 3 of the wife's
relatives? (1985, 5M)
The correct option is
(a) P → 4; Q → 6; R → 2; S → 1 23. m men and n women are to be seated in a row so that no
two women sit together. If m > n, then show that the
(b) P → 1; Q → 4; R → 2; S → 3 number of ways in which they can be seated, is
(c) P → 4; Q → 6; R → 5; S → 2 m ! (m + 1) !
.
(d) P → 4; Q → 2; R → 3; S → 1 (m − n + 1) ! (1983, 2M)

24. mn squares of equal size are arranged to form a


Integer Answer Type Question rectangle of dimension m by n where m and n are
15. Let n ≥ 2 be an integer. Take n distinct points on a circle natural numbers. Two squares will be called
and join each pair of points by a line segment. Colour the ‘neighbours’ if they have exactly one common side. A
line segment joining every pair of adjacent points by blue natural number is written in each square such that the
and the rest by red. If the number of red and blue line number in written any square is the arithmetic mean of
segments are equal, then the value of n is (2014 Adv) the numbers written in its neighbouring squares. Show
that this is possible only if all the numbers used are
Fill in the Blanks equal. (1982, 5M)
16. Let A be a set of n distinct elements. Then, the total 25. If n C r −1 = 36, n C r = 84 and n C r +1 = 126, then find the
number of distinct functions from A to A is…and out of values of n and r. (1979, 3M)
these… are onto functions. (1985, 2M)

Topic 3 Multinomial, Repeated Arrangement and Selection


Objective Question I (Only one correct option) 2. A committee of 11 members is to be formed from 8 males
and 5 females. If m is the number of ways the committee
1. The number of 6 digits numbers that can be formed
is formed with at least 6 males and n is the number of
using the digits 0, 1, 2,5, 7 and 9 which are divisible by
ways the committee is formed with atleast 3 females,
11 and no digit is repeated, is (2019 Main, 10 April I)
then (2019 Main, 9 April I)
(a) 60 (b) 72
(a) m = n = 68 (b) m + n = 68
(c) 48 (d) 36
(c) m = n = 78 (d) n = m − 8
rankershalt.com

Permutations and Combinations 75

3. Consider three boxes, each containing 10 balls labelled Integer Answer Type Questions
1, 2, …, 10. Suppose one ball is randomly drawn from
each of the boxes. Denote by ni , the label of the ball 9. Words of length 10 are formed using the letters A, B, C,
drawn from the ith box, (i = 1, 2, 3). Then, the number of D, E, F, G, H, I, J. Let x be the number of such words
ways in which the balls can be chosen such that where no letter is repeated; and let y be the number of
n1 < n2 < n3 is (2019 Main, 12 Jan I) such words where exactly one letter is repeated twice
y
(a) 82 (b) 120 (c) 240 (d) 164 and no other letter is repeated. Then, =
9x (2017 Adv.)
4. The number of natural numbers less than 7,000 which
can be formed by using the digits 0, 1, 3, 7, 9 (repitition 10. Let n be the number of ways in which 5 boys and 5 girls
of digits allowed) is equal to (2019 Main, 9 Jan II) can stand in a queue in such a way that all the girls
(a) 374 (b) 375 (c) 372 (d) 250 stand consecutively in the queue. Let m be the number
of ways in which 5 boys and 5 girls can stand in a queue
5. Consider a class of 5 girls and 7 boys. The number of in such a way that exactly four girls stand consecutively
different teams consisting of 2 girls and 3 boys that can m
be formed from this class, if there are two specific boys in the queue. Then, the value of is
n (2015 Adv.)
A and B, who refuse to be the members of the same
team, is (2019 Main, 9 Jan I) 11. Let n1 < n2 < n3 < n4 < n5 be positive integers such that
(a) 350 (b) 500 (c) 200 (d) 300 n1 + n2 + n3 + n4 + n5 = 20. The number of such distinct
arrangements (n1 , n2 , n3 , n4 , n5 ) is (2014 Adv.)
6. If all the words (with or without meaning) having five
letters, formed using the letters of the word SMALL and
arranged as in a dictionary, then the position of the Fill in the Blanks
word SMALL is (2016 Main) k(k + 1)
12. Let n and k be positive integers such that n ≥ .
(a) 46th (b) 59th 2
(c) 52nd (d) 58th The number of solutions (x1 , x2 ,... , xk ),
7. The letters of the word COCHIN are permuted and all x1 ≥ 1, x2 ≥ 2, ... , xk ≥ k for all integers satisfying
the permutations are arranged in an alphabetical order x1 + x2 + ... + xk = n is … (1996, 2M)
as in an English dictionary. The number of words that 13. Total number of ways in which six ‘+’ and four ‘–’ signs
appear before the word COCHIN, is (2007, 3M) can be arranged in a line such that no two ‘–’signs occur
(a) 360 (b) 192 (c) 96 (d) 48 together is… . (1988, 2M)

Numerical Value Analytical & Descriptive Question


8. The number of 5 digit numbers which are divisible by 4, 14. Five balls of different colours are to be placed in three
with digits from the set {1, 2, 3, 4, 5} and the repetition of boxes of different sizes. Each box can hold all five. In
digits is allowed, is ..................... . (2018 Adv.) how many different ways can we place the balls so that
no box remains empty? (1981, 4M)

Topic 4 Distribution of Object into Group


Objective Questions I (Only one correct option) 3.. From 6 different novels and 3 different dictionaries, 4
novels and 1 dictionary are to be selected and arranged
1. A group of students comprises of 5 boys and n girls. If
in a row on a shelf, so that the dictionary is always in
the number of ways, in which a team of 3 students can
the middle. The number of such arrangements is
randomly be selected from this group such that there is
(2018 Main)
at least one boy and at least one girl in each team, is
(a) atleast 1000
1750, then n is equal to (2019 Main, 12 April II)
(b) less than 500
(a) 28 (b) 27
(c) atleast 500 but less than 750
(c) 25 (d) 24
(d) atleast 750 but less than 1000
2. Let S be the set of all triangles in the xy-plane, each
4. The total number of ways in which 5 balls of different
having one vertex at the origin and the other two colours can be distributed among 3 persons so that each
vertices lie on coordinate axes with integral person gets at least one ball, is (2012)
coordinates. If each triangle in S has area 50 sq. units,
(a) 75 (b) 150 (c) 210 (d) 243
then the number of elements in the set S is
(2019 Main, 9 Jan II) 5. The number of arrangements of the letters of the word
(a) 36 (b) 32 BANANA in which the two N’s do not appear
(c) 18 (d) 9 adjacently, is (2002, 1M)
(a) 40 (b) 60 (c) 80 (d) 100
rankershalt.com

76 Permutations and Combinations

Analytical & Descriptive Questions 7. In how many ways can a pack of 52 cards be
2 (i) divided equally among four players in order
n !
6. Using permutation or otherwise, prove that is an (ii) divided into four groups of 13 cards each
(n !)n
(iii) divided in 4 sets, three of them having 17 cards each
integer, where n is a positive integer. (2004, 2M)
and the fourth just one card? (1979, 3M)

Topic 5 Dearrangement and Number of Divisors


Objective Question I (Only one correct option) Fill in the Blank
1. Number of divisors of the form (4n + 2), n ≥ 0 of the 2. There are four balls of different colours and four boxes of
integer 240 is (1998, 2M) colours, same as those of the balls. The number of ways
(a) 4 (b) 8 in which the balls, one each in a box, could be placed such
(c) 10 (d) 3
that a ball does not go to a box of its own colour is.... .
(1992, 2M)

Answers
Topic 1 22. (485) 25. (n = 9 and r = 3 )
1. (b) 2. (a) 3. (b) 4. (c)
5. (c) 6. (b) 7. (c) 8. (a)
Topic 3
1. (a) 2. (c) 3. (b) 4. (a)
9. (d) 10. (a) 11. 9
P4 × 9 P3 (11 )!
5. (d) 6. (d) 7. (c) 8. (625)
12. ( A → p; B → s; C → q ; D → q )
9. (5) 10. (5) 11. (7)
Topic 2 1
12. (2n − k + k − 2 )
2
13. (35 ways) 14. (300)
1. (c) 2. (c) 3. (b) 4. (a) 2
5. (c) 6. (c) 7. (a) 8. (a) Topic 4
9. (c) 10. (a) 11. (b) 12. (c) 1. (c) 2. (a) 3. (a) 4. (b)
13. (c) 14. (c) 15. (5) (52 )! (52 )! (52 )!
n 5. (a) 7. (i) 4
(ii) 4
(iii)
(13 !) 4 ! (13 !) 3 ! (17 ) 3
16. nn , ∑ ( −1 )n − r n Cr (r )n 17. 2n − 1 18. (True)
r =1
Topic 5
19. 6062, (i) 2702 (ii) 1008 20. n = 3 21. (64)
1. (a) 2. (9)

Hints & Solutions


Topic 1 General Arrangement
1. Following are the cases in which the 4-digit numbers Case-III
strictly greater than 4321 can be formed using digits 0, 4
1, 2, 3, 4, 5 (repetition of digits is allowed)
Case-I 4/5 0/1/2/3/4/5 2×6×6=72 numbers
2 ways 6 ways
4 3 2
Case-IV
2/3/4/5 4 ways 4 numbers 5
6×6×6=216 numbers
Case-II
0/1/2/3/4/5
4 3 6 ways

So, required total numbers = 4 + 18 + 72 + 216 = 310


3/4/5 0/1/2/3/4/5 3×6=18 numbers
3 ways 6 ways
rankershalt.com

Permutations and Combinations 77

2. Sum of diagonal entries of M T M is ∑ a i2. Case I Using digits 0, 1, 2, 4, 5


9 Number of ways = 4 × 4 × 3 × 2 × 1 = 96
∑a
i =1
2
i =5 Case II Using digits 1, 2, 3, 4, 5
Number of ways = 5 × 4 × 3 × 2 × 1 = 120
Possibilities
∴ Total numbers formed = 120 + 96 = 216
9!
I. 2, 1, 0, 0, 0, 0, 0, 0, 0, which gives matrices 9. Since, the first 2 women select the chairs amongst 1 to 4
7!
9! in 4 P2 ways. Now, from the remaining 6 chairs, three
II. 1, 1, 1, 1, 1, 0, 0, 0, 0, which gives matrices men could be arranged in 6 P3.
4! × 5!
∴ Total number of arrangements = 4 P2 × 6P3.
Total matrices = 9 × 8 + 9 × 7 × 2 = 198
10. Total number of five letters words formed from ten
3. The integer greater than 6000 may be of 4 digits or different letters = 10 × 10 × 10 × 10 × 10 = 105
5 digits. So, here two cases arise. Number of five letters words having no repetition
Case I When number is of 4 digits. = 10 × 9 × 8 × 7 × 6 = 30240
Four-digit number can start from 6, 7 or 8. ∴ Number of words which have at least one letter
repeated = 105 − 30240 = 69760
Thus, total number of 4-digit numbers, which are
greater than 6000 = 3 × 4 × 3 × 2 = 72 11. Let the two sides be A and B. Assume that four
Case II When number is of 5 digits. particular guests wish to sit on side A. Four guests who
Total number of five-digit numbers which are greater wish to sit on side A can be accommodated on nine
than 6000 = 5 ! = 120 chairs in 9 P4 ways and three guests who wish to sit on
side B can be accommodated in 9 P3 ways. Now, the
∴ Total number of integers = 72 + 120 = 192
remaining guests are left who can sit on 11 chairs on
4. There are two possible cases both the sides of the table in (11!) ways. Hence, the total
Case I Five 1’s, one 2’s, one 3’s number of ways in which 18 persons can be seated
7! = 9P4 × 9P3 × (11)!.
Number of numbers = = 42
5! 12. A. If ENDEA is fixed word, then assume this as a
Case II Four 1’s, three 2’s single letter. Total number of letters = 5
7! Total number of arrangements = 5 !.
Number of numbers = = 35
4!3! B. If E is at first and last places, then total number of
∴ Total number of numbers = 42 + 35 = 77 permutations = 7 !/ 2 ! = 21 × 5 !
C. If D, L, N are not in last five positions
5. X  X  X  X  X . The four digits 3, 3, 5, 5 can be ← D, L, N, N → ← E, E, E, A, O →
4!
arranged at () places in = 6 ways. 4! 5!
2 !2 ! Total number of permutations = × = 2 × 5 !
2! 3!
The five digits 2, 2, 8, 8, 8 can be arranged at (X ) places
5! D. Total number of odd positions = 5
in ways = 10 ways. 5!
2 !3 ! Permutations of AEEEO are .
Total number of arrangements = 6 × 10 = 60 3!
[since, events A and B are independent, therefore Total number of even positions = 4
A ∩ B = A × B] 4!
∴ Number of permutations of N, N, D, L =
6. Distinct n-digit numbers which can be formed using 2!
digits 2, 5 and 7 are 3n . We have to find n, so that 5! 4!
⇒ Total number of permutations = × = 2 × 5 !
3n ≥ 900 3! 2!
n− 2
⇒ 3 ≥ 100
⇒ n −2 ≥5 Topic 2 Properties of Combinational
⇒ n ≥ 7, so the least value of n is 7. and General Selections
7. Let n be the number of newspapers which are read by 1. Given that, out of 31 objects 10 are identical and
the students. remaining 21 are distinct, so in following ways, we can
choose 10 objects.
Then, 60n = (300) × 5
0 identical + 10 distincts, number of ways = 1 × 21
C10
⇒ n = 25
1 identical + 9 distincts, number of ways = 1 × 21
C9
8. Since, a five-digit number is formed using the digits
{0, 1, 2, 3, 4 and 5} divisible by 3 i.e. only possible when 2 identicals + 8 distincts, number of ways = 1 × 21
C8
sum of the digits is multiple of three. . . . . . .
. . . . . .
rankershalt.com

78 Permutations and Combinations

So, total number of ways in which we can choose 10 ⇒ m2 − 5m − 84 = 0


objects is ⇒ m2 − 12m + 7m − 84 = 0
21
C10 + 21
C9 + 21
C8 + K + 21
C 0 = x (let) … (i) ⇒ m(m − 12) + 7 (m − 12) = 0
⇒ m = 12 [Q m > 0]
⇒ C11 +
21 21
C12 + 21
C13 + K + 21
C 21 = x … (ii)
n n n
[Q C r = C n − r ]
n n
5. If C 4 , C 5 and C 6 are in AP, then
On adding both Eqs. (i) and (ii), we get 2 ⋅n C 5 = nC 4 + n C 6
2x = 21C 0 + 21C1 + 21C 2 + K + 21C10 [If a , b, c are in AP , then 2b = a + c]
n! n! n!
+ 21C11 + 21C12 + K + 21
C 21 ⇒ 2 = +
5 !(n − 5)! 4 !(n − 4)! 6 !(n − 6)!
⇒ 2x = 2 21
⇒ x=2 20
 n n! 
2. It is given that, there are 20 pillars of the same height Q C r = r !(n − r )! 
2  
have been erected along the boundary of a circular ⇒
5 ⋅ 4 !(n − 5) (n − 6)!
stadium.
1 1
Now, the top of each pillar has been connected by beams = +
with the top of all its non-adjacent pillars, then total 4 !(n − 4) (n − 5) (n − 6)! 6 ⋅ 5 ⋅ 4 ! (n − 6)!
number of beams = number of diagonals of 20-sided ⇒
2
=
1
+
1
polygon. 5(n − 5) (n − 4) (n − 5) 30
Q 20C 2 is selection of any two vertices of 20-sided polygon 2 30 + (n − 4) (n − 5)
⇒ =
which included the sides as well. 5(n − 5) 30 (n − 4) (n − 5)
So, required number of total beams = 20C 2 − 20 ⇒ 12 (n − 4) = 30 + n 2 − 9n + 20
[Q the number of diagonals in a n-sided closed ⇒ n 2 − 21n + 98 = 0
polygon = nC 2 − n] ⇒ n − 14n − 7n + 98 = 0
2
20 × 19
= − 20 ⇒ n (n − 14) − 7(n − 14) = 0
2 ⇒ (n − 7) (n − 14) = 0
= 190 − 20 = 170 ⇒ n = 7 or 14
25
3. Let there are n balls used to form the sides of equilateral 6. Given, Σ { 50C r .50− r C 25− r } = K 50C 25
triangle. r=0

According to the question, we have 


25 50 ! (50 − r )! 
n ( n + 1) ⇒ Σ  ×  = K C 25
50

+ 99 = ( n − 2 )2 r = 0  r !(50 − r )! (25 − r )! 25 !
2 25  50 ! 25 ! 
⇒ n 2 + n + 198 = 2 [ n 2 − 4 n + 4 ] ⇒ Σ  ×  = K C 25
50
r = 0  25 ! 25 ! r !(25 − r )!
⇒ n 2 − 9n − 190 = 0 [on multiplying 25 ! in
⇒ n 2 − 19n + 10n − 190 = 0 numerator and denominator.]
⇒ ( n − 19 )( n + 10 ) = 0 25  50 50 ! 
⇒ 50
C 25 Σ Cr = K
25 50
C 25 Q C 25 = 25 ! 25 ! 
⇒ n = 19, − 10 r = 0
 
25
⇒ n = 19 [Qnumber of balls n > 0] ⇒ K= Σ C r = 225
25
r = 0
Now, number of balls used to form an equilateral [Q nC 0 + nC1 + n C 2 + ....+ nC n = 2n ]
n (n + 1)
triangle is ⇒ K = 225
2
19 × 20 7. Given,
= = 190.
2 20  20
Ci − 1 
3
k
4. Since, there are m-men and 2-women and each ∑ i=1
 20
 Ci + Ci − 1 
20
 =
21
participant plays two games with every other 3
participant.
20
 20C i − 1  k
⇒ ∑  21
 Ci 
 =
21
(Q nC r + nC r − 1 = n +1
Cr )
∴ Number of games played by the men between i=1

themselves = 2 × mC 2  20 
3

20  Ci − 1  k  n n 
and the number of games played between the men and
the women =2 × mC1 × 2C1
⇒ ∑   = Q C r =

n−1
Cr − 1 

i=1  21 20C i − 1  21 r
Now, according to the question,  i 
3
 i
20
2 mC 2 = 2 mC1 2C1 + 84 k
m!
⇒ ∑  
 21
=
21
⇒ = m × 2 + 42 i=1
2 !(m − 2)! 20
1 k
⇒ m(m − 1) = 4m + 84 ⇒
(21)3 ∑i i=1
3
=
21
⇒ m2 − m = 4m + 84
rankershalt.com

Permutations and Combinations 79

Similarly, the number of ways to select t = 5


2
1  n (n + 1)  k
⇒ =
(21)3  2 
n = 20 21 ∴ Total number of ways = 5 × 9 × 5 = 225
 3  n (n + 1)  
2 5

 + 3
+ + 3
= 13. Here, 47 C 4 + ∑ 52 − j
C3
Q 1 2 K n   
 2  j =1
2
21  20 × 21 = 47C 4 + 51
C 3 + 50C 3 + 49C 3 + 48C 3 + 47
C3
⇒ k=   = 100
(21)3  2  = ( C4 +
47 47
C3 ) + C3 + C3 + C3 +
48 49 50 51
C3
∴ k = 100 [using C r + C r − 1 = n +1C r ]
n n

8. Given, X has 7 friends, 4 of them are ladies and 3 are = ( C 4 + C 3 ) + C 3 + 50C 3 +


48 48 49 51
C3
men while Y has 7 friends, 3 of them are ladies and 4 are
= ( C4 + C3 ) + C3 +
49 49 50 51
C3
men.
= ( 50C 4 + 50C 3 ) + 51
C3
∴ Total number of required ways
= 51C 4 + 51
C 3 = 52C 4
= 3C 3 × 4C 0 × 4C 0 × 3C 3 + 3C 2 × 4C1 × 4C1 × 3C 2
+ 3C1 × 4C 2 × 4C 2 × 3C1 + 3C 0 × 4C 3 × 4C 3 × 3C 0 14. Given 6 boys M1 , M 2 , M 3 , M 4 , M 5 , M 6 and
5 girls G1 , G2 , G3 , G4 , G5
= 1 + 144 + 324 + 16 = 485
(i) α 1 → Total number of ways of selecting 3 boys and 2
9. N i = 5C k × 4C 5 − k girls from 6 boys and 5 girls.
N1 = 5 × 1 i..e, 6C 3 × 5C 2 = 20 × 10 = 200
N 2 = 10 × 4 ∴ α 1 = 200
N 3 = 10 × 6 (ii) α 2 → Total number of ways selecting at least 2
member and having equal number of boys and girls
N4 = 5 × 4
i.e., 6C1 C1 + 6C 2 C 2 + 6C 3 C 3 + 6C 4 C 4 + 6C 5 C 5
5 5 5 5 5

N5 = 1 = 30 + 150 + 200 + 75 + 6 = 461


N 1 + N 2 + N 3 + N 4 + N 5 = 126 ⇒ α 2 = 461
10. We have, 6 girls and 4 boys. To select 4 members (iii) α 3 → Total number of ways of selecting 5 members
(atmost one boy) in which at least 2 of them girls
i.e. (1 boy and 3 girls) or (4 girls) = 6 C 3 ⋅4 C1 + 6C 4 …(i) i.e., 5C 2 C 3 + 5C 3 C 2+ 5C 4 C1 + 5C 5 C 0
6 6 6 6

Now, selection of captain from 4 members = 4 C1 …(ii) = 200 + 150 + 30 + 1 = 381


∴ Number of ways to select 4 members (including the α 3 = 381
selection of a captain, from these 4 members) (iv) α 4 → Total number of ways of selecting 4 members
= ( 6C 3 ⋅4 C1 + 6C 4 ) 4C1 in which at least two girls such that M1 and G1 are
= (20 × 4 + 15) × 4 = 380 not included together.
n +1 G1 is included → 4 C1 ⋅ 5C 2 + 4C 2 ⋅ 5C1 + 4C 3
11. Given, Tn = nC 3 ⇒ Tn + 1 = C3
= 40 + 30 + 4 = 74
∴ Tn +1 − Tn = n +1
C 3 − nC 3 = 10 [given] M 1 is included → 4C 2 ⋅ 5C1 + 4C 3 = 30 + 4 =34
n +1
⇒ n C 2 + nC 3 − nC 3 = 10 [Q nC r + nC r +1 = Cr +1 ] G1 and M 1 both are not included
⇒ n
C 2 = 10
4
C 4 + 4 C 3 ⋅ 5C1 + 4C 2 ⋅ 5C 2

⇒ n =5 1 + 20 + 60 = 81
∴ Total number = 74 + 34 + 81 = 189
12. Since, r , s, t are prime numbers. α 4 = 189
∴ Selection of p and q are as under Now, P → 4; Q → 6; R → 5; S → 2
p q Number of ways Hence, option (c) is correct.
r0 r2 1 way
15. PLAN Number of line segment joining pair of adjacent point = n
r1 r2 1 way Number of line segment obtained joining n points
r2 r 0 , r1 , r 2 3 ways on a circle = nC 2
∴ Total number of ways to select, r = 5 Number of red line segments = nC 2 − n
Selection of s as under Number of blue line segments = n
s0
s4
1 way ∴ C2 − n = n
n

s1 s4 1 way n (n − 1)
⇒ = 2n
s2 s4 1 way 2
s3 s4 1 way ⇒ n =5
s4 5 ways
∴ Total number of ways to select s = 9
rankershalt.com

80 Permutations and Combinations

16. Let A = { x1 , x2 , ... , xn }


∴ Number of functions from A to A is n n and out of these Case III When all three black balls are drawn
n
⇒ Number of ways = 3C 3 = 1
∑ (−1)
r =1
n−r n
C r (r )n are onto functions.
∴ Total number of ways = 45 + 18 + 1 = 64

17. The number of students answering exactly


22. The possible cases are
k (1 ≤ k ≤ n − 1) questions wrongly is 2n − k − 2n − k − 1 . The Case I A man invites 3 ladies and women invites 3
number of students answering all questions wrongly gentlemen.
is 20. Number of ways = 4C 3 ⋅4 C 3 = 16
Thus, total number of wrong answers
Case II A man invites (2 ladies, 1 gentleman) and
= 1 (2n − 1 − 2n − 2 ) + 2 (2n − 2 − 2n − 3 ) + K women invites (2 gentlemen, 1 lady).
+ (n − 1) (21 − 20 ) + 20 ⋅ n Number of ways = (4 C 2 ⋅3 C1 ) ⋅ (3C1 ⋅4 C 2 ) = 324
n−1 n− 2 n− 3
=2 +2 +2 + K + 2 + 20 = 2n − 1
1
Case III A man invites (1 lady, 2 gentlemen) and
18. Let r consecutive integers be x + 1, x + 2, K , x + r. women invites (2 ladies, 1 gentleman).
(x + r )(x + r − 1) K (x + 1) x ! Number of ways = (4 C1 ⋅3 C 2 ) ⋅ (3C 2 ⋅ 4C1 ) = 144
∴ (x + 1) (x + 2) K (x + r ) =
x!
(x + r )! r ! Case IV A man invites (3 gentlemen) and women
x + r
= ⋅ = C r ⋅ (r )! invites (3 ladies).
(x)! r !
Number of ways = 3C 3 ⋅3 C 3 = 1
Thus, (x + 1) (x + 2) K (x + r ) = x + rC r ⋅ (r )! , which is
clearly divisible by (r )! . Hence, it is a true statement. ∴ Total number of ways,
= 16 + 324 + 144 + 1 = 485
19. Given that, there are 9 women and 8 men, a committee
of 12 is to be formed including at least 5 women. 23. Since, m men and n women are to be seated in a row so
This can be done in that no two women sit together. This could be shown as
= (5 women and 7 men) + (6 women and 6 men) × M1 × M 2 × M 3 × ... × M m ×
+ (7 women and 5 men) + (8 women and 4 men) which shows there are (m + 1) places for n women.
+ (9 women and 3 men) ways
∴ Number of ways in which they can be arranged
Total number of ways of forming committee
= (m) ! m + 1 Pn
= (9C 5 . 8C7 ) + (9C 6. 8C 6 ) + (9C7 . 8C 5 )
(m)! ⋅ (m + 1)!
+ (9C 8 . 8C 4 ) + (9C 9. 8C 3 ) =
(m + 1 − n )!
= 1008 + 2352 + 2016 + 630 + 56 = 6062
24. Let mn squares of equal size are arrange to form a
(i) The women are in majority = 2016 + 630 + 56
rectangle of dimension m by n. Shown as, from figure.
= 2702
(ii) The man are in majority = 1008 ways
20. Since, student is allowed to select at most n books out of
(2n + 1) books.
2 n +1 2n +1 2 n +1
∴ C1 + C 2 + .... + C n = 63 ... (i) n
2 n +1 2n +1 2n +1
We know C0 + C1 + ..... + C 2 n + 1 = 22 n + 1
2n +1 2n + 1 2n + 1 2n +1
⇒ 2( C0 + C1 + C 2 + ... + C n ) = 22 n + 1 x6 x2
x5 x1 x3
2n +1 2n +1 2n +1
⇒ C1 + C 2 + ... + C n = (22 n − 1) .... (ii) x7 x4

From Eqs. (i) and (ii), we get m


neighbours of x1 are {x2 , x3 , x4 , x5} x5 are {x1 , x6 , x7 } and
22 n − 1 = 63
x7 are {x5 , x4 }.
⇒ 22 n = 64 x + x3 + x4 + x5 x + x6 + x7
⇒ x1 = 2 , x5 = 1
⇒ 2n = 6 4 3
x4 + x5
⇒ n =3 and x7 =
2
21. Case I When one black and two others balls are x + x6 + x7
∴ 4x1 = x2 + x3 + x4 + 1
drawn. 3
⇒ Number of ways = 3C1 ⋅6 C 2 = 45 x + x5
⇒ 12x1 = 3x2 + 3x3 + 3x4 + x1 + x6 + 4
2
Case II When two black and one other balls are drawn
⇒ 24x1 = 6x2 + 6x3 + 6x4 + 2x1 + 2x6 + x4 + x5
⇒ Number of ways = 3C 2 ⋅6 C1 = 18
⇒ 22x1 = 6x2 + 6x3 + 7x4 + x5 + 2x6
rankershalt.com

Permutations and Combinations 81

where, x1 , x2 , x3 , x4 , x5 , x6 are all the natural numbers 3. Given there are three boxes, each containing 10 balls
and x1 is linearly expressed as the sum of x2 , x3 , x4 , x5 , x6 labelled 1, 2, 3, … , 10.
where sum of coefficients are equal only if, all Now, one ball is randomly drawn from each boxes, and
observations are same. ni denote the label of the ball drawn from the ith box,
⇒ x2 = x3 = x4 = x5 = x6 (i = 1, 2, 3).
⇒ All the numbers used are equal. Then, the number of ways in which the balls can be
C n−r+1
n
chosen such that n1 < n2 < n3 is same as selection of 3
25. We know that, n r = different numbers from numbers {1, 2, 3, … , 10} = 10C 3
C r −1 r
84 7 n − r + 1 = 120.
⇒ = = [given]
36 3 r 4. Using the digits 0, 1, 3, 7, 9
⇒ 3n − 10r + 3 = 0 …(i) number of one digit natural numbers that can be formed
n
Cr 84 = 4,
Also given, n
=
C r +1 126 number of two digit natural numbers that can be
r+1 2 formed = 20,
⇒ =
n−r 3
⇒ 2n − 5r − 3 = 0 …(ii)
4×5
On solving Eqs. (i) and (ii), we get
r = 3 and n = 9 (Q 0 can not come in Ist box)
number of three digit natural numbers that can be
Topic 3 Multinomial, Repeated formed = 100
Arrangement and Selection
4×5× 5
1. Key Idea Use divisibility test of 11 and consider different situation
according to given condition. and number of four digit natural numbers less than
7000, that can be formed = 250
Since, the sum of given digits
0 + 1 + 2 + 5 + 7 + 9 = 24
Let the six-digit number be abcdef and to be divisible by
11, so the difference of sum of odd placed digits and sum 2×5× 5×5
of even placed digits should be either 0 or a multiple of
11 means|(a + c + e) − (b + d + f )|should be either 0 or (Q only 1 or 3 can come in Ist box)
a multiple of 11. ∴Total number of natural numbers formed
Hence, possible case is a + c + e = 12 = b + d + f (only) = 4 + 20 + 100 + 250 = 374
Now, Case I 5. Number of girls in the class = 5 and number of boys in
set { a , c, e} = {0, 5, 7} and set { b, d , f } = {1, 2, 9} the class = 7
So, number of 6-digits numbers = (2 × 2 !) × (3 !) = 24
Now, total ways of forming a team of 3 boys and 2 girls
[Q a can be selected in ways only either 5 or 7].
= 7C 3 ⋅5 C 2 = 350
Case II
Set { a , c, e} = {1, 2, 9} and set { b, d , f } = {0, 5, 7} But, if two specific boys are in team, then number of
ways = 5C1 ⋅5 C 2 = 50
So, number of 6-digits numbers = 3 ! × 3 ! = 36
So, total number of 6-digits numbers = 24 + 36 = 60 Required ways, i.e. the ways in which two specific boys
are not in the same team = 350 − 50 = 300.
2. Since there are 8 males and 5 females. Out of these 13
members committee of 11 members is to be formed. Alternate Method

According to the question, m = number of ways when Number of ways when A is selected and B is not
there is at least 6 males = 5C 2 ⋅5 C 2 = 100
= ( C 6 × C 5 ) + ( C7 × C 4 ) + ( C 8 × C 3 )
8 5 8 5 8 5
Number of ways when B is selected and A is not
= (28 × 1) + (8 × 5)+ (1 × 10) = 5C 2 ⋅5 C 2 = 100
= 28 + 40 + 10 = 78 Number of ways when both A and B are not selected
and n = number of ways when there is at least 3 females
= 5C 3 ⋅5 C 2 = 100
= ( 5C 3 × 8 C 8 ) + ( 5C 4 × 8 C7 ) + ( 5C 5 × 8 C 6 )
∴ Required ways = 100 + 100 + 100 = 300.
= 10 × 1 + 5 × 8 + 1 × 28 = 78
So, m = n = 78
rankershalt.com

82 Permutations and Combinations

6. Clearly, number of words start with A =


4!
= 12 Now, x1 ≤ x2 ≤ x3 ≤ x4 ≤ x5
2!
x1 x2 x3 x4 x5
Number of words start with L = 4 ! = 24
4! 0 0 0 0 5
Number of words start with M = = 12
2! 0 0 0 1 4
3! 0 0 0 2 3
Number of words start with SA = =3
2! 0 0 1 1 3
Number of words start with SL = 3 ! = 6 0 0 1 2 2
Note that, next word will be “SMALL”. 0 1 1 1 2
Hence, the position of word “SMALL” is 58th. 1 1 1 1 1
7. Arrange the letters of the word COCHIN as in the order So, 7 possible cases will be there.
of dictionary CCHINO.
12. The number of solutions of x1 + x2 + ... + xk = n
Consider the words starting from C.
There are 5! such words. Number of words with the two = Coefficient of t n in (t + t 2 + t 3 + ... )(t 2 + t 3 + ... )...
C’s occupying first and second place = 4 ! . (t k + t k +1 + ... )
1 + 2 + ... + k
Number of words starting with CH, CI, CN is 4! each. = Coefficient of t in t n
(1 + t + t + ... )k
2

k(k + 1)
Similarly, number of words before the first word Now, 1 + 2 + ... + k = =p [say]
starting with CO = 4! + 4! + 4! + 4! = 96. 2
1
The word starting with CO found first in the dictionary and 1 + t + t 2 + ... =
is COCHIN. There are 96 words before COCHIN. 1−t
8. A number is divisible by 4 if last 2 digit number is Thus, the number of required solutions
divisible by 4. = Coefficient of t n − p in (1 − t )− k
∴ Last two digit number divisible by 4 from (1, 2, 3, 4, 5) = Coefficient of t n − p in [1 + k C1 t + k +1 C 2t 2 + k + 2 C 3t 3 + ... ]
are 12, 24, 32, 44, 52 = k + n − p −1 C n − p = r C n − p
∴ The number of 5 digit number which are divisible by 1
where, r = k + n − p − 1 = k + n − 1 − k(k + 1)
4, from the digit (1, 2, 3, 4, 5) and digit is repeated is 2
5 × 5 × 5 × (5 ×1) = 625 1 1
= (2k + 2n − 2 + k − k) = (2n − k2 + k − 2)
2

2 2
9. x = 10 !
10 ! 10 ! y 10 13. Since, six ‘+’ signs are + + + + + +
y = 10C1 × 9C 8 × = 10 × 9 × ⇒ = =5
2! 2 9x 2 ∴ 4 negative sign has seven places to be arranged in
10. Here, B1 B2 B3 B4 B5 ⇒ 7
C 4 ways = 35 ways

Out of 5 girls, 4 girls are together and 1 girl is 14. Since, each box can hold five balls.
separate. Now, to select 2 positions out of 6 ∴ Number of ways in which balls could be distributed so
positions between boys = 6C 2 …(i) that none is empty, are (2, 2, 1) or (3, 1, 1).
4 girls are to be selected out of 5 = 5C 4 …(ii) i.e. ( 5C 2 3C 2 1C1 + 5C 3 C1 1C1 ) × 3 !
2

Now, 2 groups of girls can be arranged in 2 !ways. …(iii) = (30 + 20) × 6 = 300
Also, the group of 4 girls and 5 boys is arranged in 4 ! × 5 !
ways . …(iv) Topic 4 Distribution of Object into Group
Now, total number of ways = C × C × 2! × 4! × 5!
6 5
2 4 1. It is given that a group of students comprises of 5 boys
[from Eqs. (i), (ii), (iii) and (iv)] and n girls. The number of ways, in which a team of 3
∴ m = 6C 2 × 5C 4 × 2 ! × 4 ! × 5 ! students can be selected from this group such that each
and n = 5! × 6! team consists of at least one boy and at least one girls, is
m 6C 2 × 5C 4 × 2 ! × 4 ! × 5 ! 15 × 5 × 2 × 4 ! = (number of ways selecting one boy and 2 girls) +
⇒ = = =5 (number of ways selecting two boys and 1 girl)
n 6! × 5! 6 × 5 × 4!
= ( C1 × nC 2 ) ( C 2 × nC1 ) = 1750 [given]
5 5

11. PLAN Reducing the equation to a newer equation, where sum of


 n (n − 1)  5 × 4 
variables is less. Thus, finding the number of arrangements ⇒ 5 ×  + × n = 1750
becomes easier.  2   2 
As, n1 ≥ 1, n2 ≥ 2 , n3 ≥ 3, n4 ≥ 4, n5 ≥ 5 2
⇒ n (n − 1) + 4n = × 1750 ⇒ n 2 + 3n = 2 × 350
Let n1 − 1 = x1 ≥ 0, n2 − 2 = x2 ≥ 0, ..., n5 − 5 = x5 ≥ 0 5
⇒ New equation will be ⇒ n 2 + 3n − 700 = 0 ⇒ n 2 + 28n − 25n − 700 = 0
x1 + 1 + x2 + 2 + ... + x5 + 5 = 20 ⇒ n (n + 28) − 25(n + 28) = 0 ⇒ (n + 28) (n − 25) = 0
⇒ x1 + x2 + x3 + x4 + x5 = 20 − 15 = 5 ⇒ n = 25 [Q n ∈ N ]
rankershalt.com

Permutations and Combinations 83

2. According to given Y So, we should make cases


information, we have the (0, b) A B C  A B C 
following figure. B Case I  Case II 
1 1 2  1 2 2 
(Note that as a and b are
Number of ways to distribute 5 balls
integers so they can be
negative also). Here  3 !  3 !
O
X =  5C1 ⋅4 C1 ⋅3 C 3 ×  +  5C1 ⋅4 C 2 ⋅2 C 2 × 
O(0, 0), A (a , 0) and B(0, b) A (a, 0)  2!   2 !
are the three vertices of = 60 + 90 = 150
the triangle.
5. Total number of arrangements of word BANANA
Clearly, OA =| a |and OB =| b|. 6!
1 = = 60
∴Area of ∆OAB = | a ||b|. 3!2!
2
The number of arrangements of words BANANA in
But area of such triangles is given as 50 sq units. 5!
1 which two N’s appear adjacently = = 20
∴ |a || b| = 50 3!
2
Required number of arrangements = 60 − 20 = 40
⇒ |a || b| = 100 = 22 ⋅ 52
Number of ways of distributing two 2’s in|a |and| b| = 3 6. Here, n 2 objects are distributed in n groups, each group
containing n identical objects.
| a| | b|
∴ Number of arrangements
0 2
= n C n . n − nC n . n − 2n C n . n − 3nC n . n 2 − 2n
2 2 2 2
C n K nC n
1 1
(n 2 )! (n 2 − n )! n! (n 2 )!
2 0 = . K =
n ! (n 2 − n )! n ! (n 2 − 2n )! n ! ⋅ 1 (n !)n
⇒ 3 ways
Similarly, number of ways of distributing two 5’s in| a | ⇒ Integer (as number of arrangements has to be integer).
and|b|= 3 ways. 7. (i) The number of ways in which 52 cards be divided
∴ Total number of ways of distributing 2’s and 5’s equally among four players in order
= 3 × 3 = 9 ways (52)!
= 52C13 × 39C13 × 26C13 × 13C13 =
Note that for one value of | a | , there are 2 possible (13 !)4
values of a and for one value of|b|, there are 2 possible
(ii) The number of ways in which a pack of 52 cards
values of b.
can be divided equally into four groups of 13 cards
∴Number of such triangles possible = 2 × 2 × 9 = 36. 52
C13 × 39C13 × 26C13 × 13C13 (52)!
So, number of elements in S is 36. each = =
4! 4 !(13 !)4
3. Given 6 different novels and 3 different dictionaries. (iii) The number of ways in which a pack of 52 cards be
Number of ways of selecting 4 novels from 6 novels is divided into 4 sets, three of them having 17 cards
6! each and the fourth just one card
6
C4 = = 15
2 !4 ! 52
C17 × 35C18 × 18C17 × 1C1 (52)!
= =
Number of ways of selecting 1 dictionary is from 3 3! 3 !(17)3
3!
dictionaries is 3C1 = =3
1 !2 !
Topic 5 Dearrangement and Number of
∴ Total number of arrangement of 4 novels and 1 Divisors
dictionary where dictionary is always in the middle, is
15 × 3 × 4 ! = 45 × 24 = 1080 1. Since, 240 = 24 .3.5
4. Objects Groups Objects Groups ∴ Total number of divisors = (4 + 1)(2)(2) = 20
Distinct Distinct Identical Identical Out of these 2, 6, 10, and 30 are of the form 4n + 2.
Distinct Identical Identical Distinct 2. The number of ways in which the ball does not go its
 1 1 1 1
Description of Situation Here, 5 distinct balls are own colour box = 4 ! 1 − + − + 
distributed amongst 3 persons so that each gets at least  1 ! 2 ! 3 ! 4 !
one ball. i.e. Distinct → Distinct 1 1 1  12 − 4 + 1
= 4!  − +  = 24   =9
 2 6 24  24 

Download Chapter Test


http://tinyurl.com/y32hjn72 or
rankershalt.com

5
Binomial Theorem

Topic 1 Binomial Expansion and General Term


Objective Questions I (Only one correct option) 8. The sum of the coefficients of all even degree terms is x
18 in the expansion of (2019 Main, 8 April I)
1. The coefficient of x in the product
(1 + x)(1 − x) (1 + x + x ) is
10 2 9
(2019 Main, 12 April I) (x + x − 1 ) + (x − x − 1 ) , (x > 1) is equal to
3 6 3 6

(a) 84 (b) − 126 (a) 29 (b) 32 (c) 26 (d) 24


(c) − 84 (d) 126
9. The total number of irrational terms in the binomial
2. If the coefficients of x2 and x3 are both zero, in the expansion of (71/5 − 31/10 )60 is (2019 Main, 12 Jan II)
expansion of the expression (1 + ax + bx2) (1 − 3x)15 in
(a) 49 (b) 48 (c) 54 (d) 55
powers of x, then the ordered pair (a , b) is equal to
(2019 Main, 10 April I) 10. The ratio of the 5th term from the beginning to the 5th
(a) (28, 315) (b) (− 21, 714) term from the end in the binomial expansion of
10
(c) (28, 861) (d) (− 54, 315)  1 
 3 1 
3. The term independent of x in the expansion of 2 + 1
is (2019 Main, 12 Jan I)
 
1 x8   3
6
 2(3)3 
 −  . 2x2 − 2 is equal to
 60 81  x  (2019 Main, 12 April II)
1 1 1 1
(a) 1 : 2(6)3 (b) 1 : 4(16)3 (c) 4(36)3 : 1 (d) 2(36)3 : 1
(a) − 72 (b) 36 (c) − 36 (d) − 108
4. The smallest natural number n, such that the 11. The sum of the real values of x for which the middle
8
 1
n  x3 3
coefficient of x in the expansion of  x2 + 3  is nC 23 , is term in the binomial expansion of  +  equals
 x  3 x
(2019 Main, 10 April II) 5670 is (2019 Main, 11 Jan I)
(a) 35 (b) 23 (c) 58 (d) 38 (a) 4 (b) 0 (c) 6 (d) 8
5. If some three consecutive coefficients in the binomial 12. The positive value of λ for which the coefficient of x2 in
expansion of (x + 1)n in powers of x are in the ratio 2 : 15 :  λ
10

70, then the average of these three coefficients is the expression x2  x + 2 is 720, is
 x  (2019 Main, 10 Jan II)
(2019 Main, 9 April II)
(a) 964 (b) 227 (c) 232 (d) 625 (a) 3 (b) 5 (c) 2 2 (d) 4

6. If the fourth term in the binomial expansion of 13. If the third term in the binomial expansion of
6 (1 + xlog 2 x )5 equals 2560, then a possible value of x is
2 log x 
 + x 8  (x > 0) is 20 × 8 , then the value of x is
7
(2019 Main, 10 Jan I)
x  (2019 Main, 9 April I) 1 1
(a) 4 2 (b) (c) (d) 2 2
(a) 8−2 (b) 83 4 8
(c) 8 (d) 82 3
 1 − t6 
7. If the fourth term in the binomial expansion of 14. The coefficient of t in the expansion of 
4
 is
6 1−t
  1  1
  1+ log10 x   (2019 Main, 9 Jan II)
 x + x  is equal to 200, and x > 1, then the
12
(a) 12 (b) 10 (c) 15 (d) 14
 
  15. The sum of the coefficients of all odd degree terms in the
5 5
expansion of  x + x3 − 1 +  x − x3 − 1 , (x > 1) is
value of x is (2019 Main, 8 April II)
4
(a) 100 (b) 10 (2018 Main)
(c) 10 (d) 103 (a) −1 (b) 0 (c) 1 (d) 2
rankershalt.com

Binomial Theorem 85

16. The value of (21C1 − 10C1 ) + (21C 2 − 10C 2) 27. If the coefficients of x3 and x4 in the expansion of
+ ( C3 − C3 ) + ( C 4 − 10C 4 ) + ... + (21C10 − 10C10 ) is
21 10 21 (1 + ax + bx2)(1 − 2x)18 in powers of x are both zero, then
(2017 Main) (a , b) is equal to
(a) 221 − 211 (b) 221 − 210
(a)  16,
251
(b)  14,
251
(c) 220 − 29 (d) 220 − 210  
 3   3 
(c)  14,
272 
(d)  16,
17. If the number of terms in the expansion of 272 
 
 2 4
n  3   3 
1 − + 2 , x ≠ 0, is 28, then the sum of the
 x x 
coefficients of all the terms in this expansion, is Fill in the Blanks
(2016 Main) 28. Let n be a positive integer. If the coefficients of 2nd, 3rd,
(a) 64 (b) 2187 and 4th terms in the expansion of (1 + x)n are in AP,
(c) 243 (d) 729 then the value of n is… . (1994, 2M)
18. The sum of coefficients of integral powers of x in the 29. If (1 + a x)n = 1 + 8x + 24x2 + … , then a = … and n = K .
binomial expansion (1 − 2 x ) 50
is (2015 Main) (1983, 2M)
1 1 1 1
(a) (350 + 1) (b) (350 ) (c) (350 − 1) (d) (250 + 1) 30. For any odd integer n ≥ 1, n − (n − 1) + K 3 3
2 2 2 2
19. Coefficient of x11 in the expansion of + (− 1)n−113 = K
(1 + x2)4 (1 + x3 )7 (1 + x4 )12 is (2014 Adv.)
31. The larger of 9950 + 10050 and 10150 is ... .
(a) 1051 (b) 1106 (c) 1113 (d) 1120
Analytical & Descriptive Questions
20. The term independent of x in expansion of k
 x+1 x−1 
10
32. Prove that ∑ (−3)r − 1 3 nC 2r −1 = 0, where k = (3n )/ 2 and n
 2/ 3 −  is (2013 Main) r =1
x − x 1 /3 + 1 x − x 1 / 2 
is an even positive integer. (1993, 5M)
(a) 4 (b) 120 (c) 210 (d) 310 2n 2n
21. Coefficient of t 24 in (1 + t 2)12 (1 + t12) (1 + t 24 ) is 33. If ∑ a r (x − 2)r = ∑ br (x − 3)r and a k = 1 , ∀ k ≥ n, then
(2003, 1M) r=0 r=0
2n + 1
(a) 12C6 + 3 (b) 12C6 + 1 (c) 12C6 (d) 12C6 + 2 show that bn = Cn+ 1 (1992, 6M)

22. In the binomial expansion of (a − b) , n ≥ 5 the sum of n 34. Find the sum of the series
the 5th and 6th terms is zero. Then, a / b equals n  1 3r 7r 15
r

(2001, 1M) ∑ (− 1 ) r n
C r 
2
r
+
2 2r
+
23r
+
2 4r
... upto m terms .

n−5 n−4 5 6 r=0
(a) (b) (c) (d)
6 5 n−4 n−5 (1985, 5M)

23. If in the expansion of (1 + x) (1 − x) , the coefficients of


m n 35. Given, sn = 1 + q + q + K + q 2 n

x and x are 3 and −6 respectively, then m is euqal to


2
q + 1  q + 1
2
 q + 1
n

(1999, 2M) Sn = 1 + +  + ... +   ,q ≠1


2  2   2 
(a) 6 (b) 9 (c) 12 (d) 24
n+ 1 n+ 1 n+ 1
Prove that C1 + C 2 s1 + C3 s2
24. The expression [x + (x − 1) 3 1/ 2 5
] + [x − (x − 1) ]3 1/ 2 5
is a
n+ 1
polynomial of degree (1992, 2M)
+ ... + C n+ 1 sn = 2nS n
(1984, 4M)
(a) 5 (b) 6 (c) 7 (d) 8
10
25. The coefficient of x4 in  −
x 3
 is (1983, 1M) Integer Answer Type Question
2 x2
405 504 36. Let m be the smallest positive integer such that
(a) (b) the coefficient of x2 in the expansion of
256 259
450 (1 + x)2 + (1 + x)3 + K + (1 + x)49 + (1 + mx)50is (3n + 1)
(c) (d) None of these 51
263 C3 for some positive integer n. Then, the value of n is
(2016 Adv.)
26. Given positive integers r > 1, n > 2 and the coefficient of
37. The coefficient of x9 in the expansion of
(3r )th and (r + 2)th terms in the binomial expansion of (1 + x) (1 + x2) (1 + x3 ) ... (1 + x100 ) is (2015 Adv.)
(1 + x)2n are equal. Then, (1980, 2M)
(a) n = 2r (b) n = 2r + 1 38. The coefficients of three consecutive terms of (1 + x)n + 5
(c) n = 3r (d) None of these are in the ratio 5 : 10 : 14. Then, n is equal to (2013 Adv.)
rankershalt.com

86 Binomial Theorem

Topic 2 Properties of Binomial Coefficient


Objective Questions I (Only one correct option) (c) (−1)n/ 2 (n + 1) (d) None of these

1. Let (x + 10) + (x − 10)


50
= a 0 + a1x + a 2x + K + a50x ,
50 2 50

a2
Numerical Value
for all x ∈ R; then is equal to (2019 Main, 11 Jan II)
a0 11. Let X = (10C1 )2 + 2(10C 2)2 + 3(10C3 )2 + ... + 10(10C10 )2,
(a) 12.25 (b) 12.50 (c) 12.00 (d) 12.75 where 10
C r, r ∈{1, 2,... , 10} denote binomial
1
2. The value of r for which coefficients. Then, the value of X is .......... .
1430 (2018 Adv.)
20
Cr 20
C0 + 20
Cr−1 20
C1 + 20
Cr− 2 20C2 + .... + 20C 020C r
Fill in the Blank
is maximum, is (2019 Main, 11 Jan I)
(a) 15 (b) 10 (c) 11 (d) 20
12. The sum of the coefficients of the polynomial
403
(1 + x − 3x2)2163 is …. . (1982, 2M)
2 k
3. If the fractional part of the number is , then k is
15 15 Analytical & Descriptive Questions
equal to (2019 Main, 9 Jan I)
(a) 14 (b) 6 (c) 4 (d) 8
13. Prove that
 n  n  n  n − 1 k − 2  n  n − 2
4. For r = 0, 1, ... , 10, if Ar, Br and C r denote respectively 2k     − 2k −1     +2     − ...
 0  k  1  k − 1  2  k − 2
the coefficient of xr in the expansions of (1 + x)10, (1 + x)20
10  n  n − k  n
+ (−1)k     =   (2003, 4 M)
and (1 + x)30. Then, ∑ Ar (B10Br − C10 Ar ) is equal to
 k  0   k
r =1

(a) B10 − C10 2


(b) A10 (B10 − C10 A10 ) (2010) 14. For any positive integers m, n (with n ≥ m),
(c) 0 (d) C10 − B10  n
If   = nCm. Prove that
m
5. 30 30 30 30 30 30 30 30
0  10 −  1  11 +  2  12 + K + 20 30 is equal  n   n − 1  n − 2 m  n + 1
 + +   + ... +   =  
to (2005, 1M) m  m   m  m m + 1
(a) 30 C11 (b) 60
C10 or
(c) 30 C10 (d) 65
C55 Prove that
n −1  n  n − 1  n − 2
6. If C r = (k2 − 3) nC r + 1, then k belongs to (2004, 1M)   +2  +3  + ... + (n − m + 1)
m  m   m 
(a) (− ∞ , − 2] (b) [2, ∞ )
m  n + 2
(c) [ − 3 , 3 ] (d) ( 3 , 2]   =  (IIT JEE 2000, 6M)
m m + 2
10 20  , where  p = 0 if p > q,
m
7. The sum ∑  i  m − i  q
is 3!
n
 nC 
i=0 15. Prove that = ∑ (−1)r  r + 3 r  .
2(n + 3) r = 0  Cr 
maximum when m is equal to (2002, 1M) (1997C, 5M)
(a) 5 (b) 10 (c) 15 (d) 20 16. If n is a positive integer and
 n  n   n  (1 + x + x2)n = a 0 + a1x + ... + a 2n x2n.
8. For 2 ≤ r ≤ n,   + 2  +  is equal to
 r  r − 1  r − 2 (2000, 2M) Then, show that, a 02 − a12 + ... + a 22n = a n. (1994, 5M)
 n + 1  n + 1  n + 2  n + 2
(a)   (b) 2   (c) 2   (d)   17. Prove that C 0 − 2 ⋅ C1 + 3 ⋅ C 2 − ... + (−1) (n + 1) 2⋅ C n
2 2 n
 r − 1  r + 1  r   r 
= 0 , n > 2, where C r = nC r. (1989, 5M)
n n
1 r 18. If (1 + x) = C 0 + C1x + C 2 x + ... + C nx , then show
9. If a n = ∑ n , then ∑ nC r equals
n 2 n

r=0 C r r=0 that the sum of the products of the Ci’s taken two at a
(a) (n − 1) an (b) n an (1998, 2M) time represented by Σ Σ CiC j is equal to
(2n !)
(c)
1
n an (d) None of these 0 ≤ i < j ≤ n 22 n −1 − . (1983, 3M)
2 2 (n !)2
10. If C r stands for nC r, then the sum of the series 19. Prove that C12 − 2 ⋅ C 22 + 3 ⋅ C32 − ...−2n ⋅ C 22n = (−1)n n ⋅ C n
 n  n (1979, 4M)
2  !  !
 2  2 20. Prove that ( C 0 ) − ( C1 ) + ( C 2) − ... + (2 nC 2 n )2
2n 2 2n 2 2n 2
[C 02 − 2 C12 + 3 C 22 − ... + (−1)n (n + 1) C n2 ],
n! = (−1)n ⋅2n C n. (1978, 4M)
where n is an even positive integer, is (1986, 2M)
(a) (−1)n/ 2 (n + 2) (b) (−1)n (n + 1)
rankershalt.com

Binomial Theorem 87

Answers
Topic 1  2mn − 1 
31. (101 ) 50 34.  mn n 
1. (a) 2. (a) 3. (c) 4. (d) 2 (2 − 1 ) 
5. (c) 6.
(d) 7. (c) 8. (d) 36. (5) 37. (8) 38. (n = 6 )
9. (c) 10.
(c) 11. (b) 12. (d)
13. (b) 14.
(c) 15. (d) 16. (d) Topic 2
17. (d) 18.
(a) 19. (c) 20. (c) 1. (a) 2. (d) 3. (d) 4. (d)
21. (d) 22.
(b) 23. (c) 24. (c) 5. (c) 6. (d) 7. (c) 8. (d)
25. (a) 26.
(a) 27. (d) 28. (n = 7) 9. (c) 10. (a) 11. (646) 12. ( − 1 )
1
29. (a = 2, n = 4) 30. (n + 1 ) (2n − 1 )
2
4

Hints & Solutions


Topic 1 Binomial Expansion and General Term
1. Given expression is ⇒ 21a − b = 273 …(ii)
(1 + x) (1 − x) (1 + x + x )
10 2 9 From Eqs. (i) and (ii), we get
= (1 + x) (1 − x) [(1 − x) (1 + x + x2)]9 24a = 672 ⇒ a = 28
= (1 − x ) (1 − x )
2 3 9
So, b = 315
Now, coefficient of x18 in the product ⇒ (a , b) = (28, 315)

(1 + x) (1 − x)10 (1 + x + x2)9
3. Key Idea Use the general term (or (r + 1)th term) in the
= coefficient of x18 in the product (1 − x2) (1 − x3 )9 expansion of binomial (a + b) n
= coefficient of x 18
in (1 − x ) 3 9
i.e. T r + 1 = nC r a n − r b r
− coefficient of x16 in (1 − x3 )9
6
Since, (r + 1)th term in the expansion of  3
Let a binomial 2x2 − 2 , it’s (r + 1)th term
(1 − x3 )9 is 9C r (− x3 )r = 9C r (− 1)r x3 r  x 
r
Now, for x18, 3r = 18 ⇒ r = 6  3
= Tr + 1 = 6C r (2x2)6 − r  − 2
 x 
and for x16, 3r = 16
16 = 6C r (− 3)r (2)6 − r x12 − 2r − 2r
⇒ r= ∉N.
3 = 6C r (−3)r (2)6 − r x12 − 4r …(i)
9! 9 ×8 × 7 Now, the term independent of x in the expansion of
∴Required coefficient is C 6 = =
9
= 84
6 !3 ! 3 ×2 1 x8   2 3
6
 −   2x − 2
 60 81  x 
2. Given expression is (1 + ax + bx2)(1 − 3x)15 . In the
expansion of binomial (1 − 3x)15 , the (r + 1) th term is = the term independent of x in the expansion of
6
Tr + 1 = 15C r (−3x)r = 15C r (−3)r xr 1  2 3
2x − 2 + the term independent of x in the
Now, coefficient of x2, in the expansion of 60  x 
6
(1 + ax + bx2)(1 − 3x)15 is x8  2 3 
expansion of −  2x − 2
15
C 2(−3)2 + a15C1 (−3)1 + b 15C 0 (−3)0 = 0 (given) 81  x 
6
⇒ (105 × 9) − 45 a + b = 0 C3
= (− 3)3 (2)6 − 3 x12 − 4 (3 ) [put r = 3]
⇒ 45a − b = 945 …(i) 60
 1
Similarly, the coefficient of x3 , in the expansion of +  −  6C5 (−3)5 (2)6 − 5 x12 − 4 (5 ) x8 [put r = 5]
 81
(1 + ax + bx2)(1 − 3x)15 is
1 35 × 2(6)
C3 (−3)3 + a 15C 2(−3)2 + b 15C1 (−3)1 = 0
15
(given) = (− 3)3 23 +
⇒ − 12285 + 945a − 45b = 0 3 81
= 36 − 72 = − 36
⇒ 63a − 3b = 819
rankershalt.com

88 Binomial Theorem
6 −3
 2
n
∴ T4 = T3 + 1 = 6 C3   (xlog 8 x )3 = 20 × 87 (given)
4. Given binomial is  x2 + 3  , its (r + 1)th term, is
1
 x
 x  3
 2 3 log 8 x
r ⇒ 20   x = 20 × 87 [Q 6C3 = 20]
 1 1  x
Tr + 1 = nC r (x2)n − r  3  = nC rx2n − 2r 3 r 3 
x  x  log 2 x −3 
3 
⇒ 23 x [3(log 8 x )−3 ]= (23 )7 ⇒ x = (23 )6
= nC rx2n − 2r − 3 r = nC rx2n − 5 r
For the coefficient of x ,  1 
2n − 5r = 1 ⇒ 2n = 5r + 1 …(i) Q log an (x) = n log a x for x > 0; a > 0, ≠ 1
As coefficient of x is given as nC 23 , then either r = 23 or log x − 3 )
⇒ x( 2 = 218
n − r = 23 .
If r = 23, then from Eq. (i), we get On taking log 2 both sides, we get
2n = 5(23) + 1 (log 2 x − 3) log 2 x = 18
⇒ 2n = 115 + 1 ⇒ 2n = 116 ⇒ n = 58. ⇒ (log 2 x)2 − 3 log 2 x − 18 = 0
If n − r = 23, then from Eq. (i) on replacing the value of ‘ ⇒ (log 2 x)2 − 6 log 2 x + 3 log 2 x − 18 = 0
r’, we get 2n = 5(n − 23) + 1
⇒ log 2 x(log 2 x − 6) + 3 (log 2 x − 6) = 0
⇒ 2n = 5n − 115 + 1 ⇒ 3n = 114 ⇒ n = 38
So, the required smallest natural number n = 38. ⇒ (log 2 x − 6) (log 2 x + 3) = 0
⇒ log 2 x = −3, 6
Key Idea Use general term of Binomial expansion ( x + a) i.e. 1 2
⇒ x = 2 −3 , 2 6 ⇒ x =
n
5. ,8
n−r 8
Tr + 1 = C r 1 x
n
a r

6
 
Given binomial is (x + 1)n, whose general term, is   1  1 
Tr + 1 = nC r xr 7. Given binomial is   1 + log10 x  
 x +x 
12
According to the question, we have  
C r − 1 : nC r : nC r + 1 = 2 : 15 : 70
n Since, the fourth term in the given expansion is 200.
3
n
Cr − 1 2  1  2 1 3
Now, = 
∴ C3 x
6 1 + log10 x   12
x = 200
  
n
Cr 15 
 
n!  3 1
 2 (1 + log x ) + 4 
(r − 1)!(n − r + 1)! 2 ⇒ 20 × x  = 200
⇒ =
10
n! 15 3
+
1
2(1 + log10 x ) 4
r !(n − r )! ⇒ x = 10
r 2  3 1
⇒ = ⇒ 15r = 2n − 2r + 2 ⇒ 2(1 + log x) + 4  log10 x = 1
n − r + 1 15  10 
⇒ 2n − 17r + 2 = 0 …(i) [applying log10 both sides]
n!
⇒ [6 + (1 + log10 x)] log10 x = 4(1 + log10 x)
C n
15 r !(n − r )! 3
Similarly, n r = ⇒ = ⇒ (7 + log10 x) log10 x = 4 + 4 log10 x
Cr + 1 70 n! 14
(r + 1)!(n − r − 1)! ⇒ t 2 + 7t = 4 + 4t [let log10 x = t]
⇒ t + 3t − 4 = 0
2
r+1 3 ⇒ t = 1 , −4 = log10 x
⇒ = ⇒ 14r + 14 = 3n − 3r
n − r 14
⇒ x = 10, 10−4
⇒ 3n − 17r − 14 = 0 …(ii) Since, x>1 x = 10
On solving Eqs. (i) and (ii), we get 8.
n − 16 = 0 ⇒ n = 16 and r = 2 Key Idea Use formula :
C 2 + 16C316
C1 + 16 ( a + b) n + ( a − b) n =
Now, the average = 2 [ n C 0 a n + nC 2a n − 2b 2 + nC 4 a n − 4 b 4 + ...... ]
3
16 + 120 + 560 696
= = = 232 Given expression is (x + x3 − 1 )6 + (x − x3 − 1 )6
3 3
6 = 2 [6C 0x6 + 6C 2x4 ( x3 − 1 )2
6. Given binomial is  + x 8 
2 log x
x  + 6C 4x2( x3 − 1 )4 + 6C 6 ( x3 − 1 )6 ]
Since, general term in the expansion of (x + a )n is {Q (a + b)n + (a − b)n
Tr+ 1 = n C r xn− ra r = 2 [ C 0a + C 2a
n n n n−2 2
b + nC 4a n − 4b4 + …]}
rankershalt.com

Binomial Theorem 89

= 2 [6C 0x6 + 6C 2x4 (x3 − 1) + 6C 4x2(x3 − 1)2 + 6C 6 (x3 − 1)3 ] 12. The general term in the expansion of binomial
The sum of the terms with even power of x expression (a + b)n is Tr+ 1 = nC r a n− rbr, so the general
term in the expansion of binomial expression
= 2 [6C 0x6 + 6C 2(− x4 ) + 6C 4x8 + 6C 4x2 + 6C 6 (−1 − 3x6 )]  λ
10

= 2 [6C 0x6 − 6C 2x4 + 6C 4x8 + 6C 4x2 − 1 − 3x6 ] x2 x + 2 is


 x 
10 − r
Now, the required sum of the coefficients of even powers   λ 
r

of x in Tr+ 1 = x2  10C r ( x )10− r  2  =10C r x2 ⋅ x 2 λr x−2r


 x  
(x + x3 − 1 )6 + (x − x3 − 1 )6 10 − r
2+ − 2r
=10C r λr x 2
= 2 [6C 0 − 6C 2 + 6 C 4 + 6C 4 − 1 − 3] 10 − r
Now, for the coefficient of x2, put 2 + − 2r = 2
= 2 [1 − 15 + 15 + 15 − 1 − 3] = 2(15 − 3) = 24 2
10 − r
9. The general term in the binomial expansion of (a + b)n ⇒ − 2r = 0
2
is Tr + 1 = nC r a n − rbr. ⇒ 10 − r = 4r ⇒ r = 2
So, the general term in the binomial expansion of So, the coefficient of x2 is 10C 2 λ2 = 720 [given]
(71/5 − 31/10 )60 is 10 ! 2 10 ⋅ 9 ⋅ 8 ! 2
⇒ λ = 720 ⇒ λ = 720
Tr + 1 = C r (71/5 )60 − r (−31/10 )r
60
2!8! 2⋅ 8!
60 − r r r r
12 −
= 60
Cr 7 5 (−1 )r 310 = (−1)r 60
Cr 7 5 310 ⇒ 45 λ2 = 720
⇒ λ2 = 16 ⇒ λ = ± 4
The possible non-negative integral values of ‘r’ for ∴ λ =4 [λ > 0]
r r
which and are integer, where r ≤ 60, are
5 10 13. The (r + 1)th term in the expansion of (a + x)n is given
r = 0, 10, 20, 30, 40, 50, 60. by Tr + 1 = nC ra n − rxr
∴There are 7 rational terms in the binomial expansion ∴ 3rd term in the expansion of (1 + xlog 2 x )5 is
and remaining 61 − 7 = 54 terms are irrational terms. 5
C 2(1)5 − 2(xlog 2 x )2
10. Since, rth term from the end in the expansion of a ⇒ 5 C 2(1)5 − 2(xlog 2 x )2 = 2560 (given)
binomial (x + a )n is same as the (n − r + 2)th term from
⇒ 10 (xlog 2 x )2 = 2560
the beginning in the expansion of same binomial.
T5 T T4 + 1 ⇒ x( 2log 2 x ) = 256
∴Required ratio = = 5 =
T10 − 5 + 2 T7 T6 + 1 ⇒ log 2 x2log 2 x = log 2 256
4 (taking log 2 on both sides)
 1 
10
C 4 (21/3 )10− 4   ⇒ 2(log 2 x)(log 2 x) = 8 (Q log 2 256 = log 2 28 = 8)
T5  2(3)1/3  (log 2 x) = 4
2
⇒ = 6
T10 − 5 + 2  1  ⇒ log 2 x = ± 2
10
C 6 (21/3 )10− 6   ⇒ log 2 x = 2 or log 2 x = − 2
 2(3)1/3 
1
[QTr + 1 = n Cr x n − r a r ] ⇒ x = 4 or x = 2−2 =
6/3 1/3 6
4
2 (2(3) ) 3
= [Q 10C 4 = 10C 6 ] 1 − t 6
24/3 (2(3)1/3 )4 14. Clearly,   = (1 − t 6 )3 (1 − t )− 3
1−t
= 26/3 − 4/3 (2(3)1/3 )6 − 4
∴ Coefficient of t 4 in (1 − t 6 )3 (1 − t )−3
= 223
/
⋅ 22 ⋅ 323
/
= 4(6)23 /
= 4(36)1/3 = Coefficient of t 4 in (1 − t18 − 3t 6 + 3t12) (1 − t )− 3
So, the required ratio is 4(36)1/3 : 1 . = Coefficient of t 4 in (1 − t )− 3
 x3
8 = 3 + 4 − 1C 4 = 6C 4 = 15
3
11. In the expansion of  +  , the middle term is T4 + 1. (Q coefficient of xr in (1 − x)− n = n + r − 1C r)
3 x
15.
[Q Here, n = 8, which is even, therefore middle term Key Idea Use formula :
 n + 2 = ( a + b) n + ( a − b) n
=  th term] = 2 ( n C 0 a n + nC 2a n − 2b 2 + nC 4 a n − 4 b 4 + ...)
 2 
4
 x3   3 4 8 ⋅ 7 ⋅ 6 ⋅ 5 8 We have, (x + x3 − 1 )5 + (x − x3 − 1 )5 , x > 1
∴ 5670 = C4     =
8
x
 3   x 1⋅ 2⋅ 3⋅ 4
= 2(5 C 0x5 + 5C 2x3 ( x3 − 1 )2 + 5C 4x( x3 − 1 )4 )
  3
8− r r
QTr + 1 = 8Cr  x   3  = 2(x5 + 10x3 (x3 − 1) + 5x(x3 − 1)2)
 
  3  x 
  = 2(x5 + 10x6 − 10x3 + 5x7 − 10x4 + 5x)
⇒ x 8 = 34 ⇒ x = ± 3 Sum of coefficients of all odd degree terms is
So, sum of all values of x i.e + 3 and − 3 = 0 2 (1 − 10 + 5 + 5) = 2
rankershalt.com

90 Binomial Theorem

16. ( 21C1 − 10C1 ) + (21C 2 − 10C 2) + (21C3 − 10C3 ) 19. Coefficient of xr in (1 + x)n is nC r.
+ ... + (21C10 − 10C10 ) In this type of questions, we find different composition
= ( C1 + C 2 + ... + C10 ) − ( C1 + 10C 2 + ... + 10C10 )
21 21 21 10 of terms where product will give us x11.
1 Now, consider the following cases for x11 in
= (21C1 + 21C 2 + ... + 21C 20 ) − (210 − 1)
2 (1 + x2)4 (1 + x3 )7 (1 + x4 )12.
1 Coefficient of x0 x3 x8; Coefficient of x2 x9 x0
= (21C1 + 21C 2 + ... + 21C 21 − 1) − (210 − 1)
2 Coefficient of x4 x3 x4; Coefficient of x8 x3 x0
1 = 4C 0 × 7C1 × 12C 2 + 4C1 × 7C3 × 12C 0 + 4C 2 × 7C1
= (221 − 2) − (210 − 1) = 220 − 1 − 210 + 1 = 220 − 210
2 × 12C1 + 4C 4 × 7C1 × 12C 0
= 462 + 140 + 504 + 7 = 1113
17. Clearly, number of terms in the expansion of
10
 2 4
n
(n + 2) (n + 1)  x+1 ( x − 1) 
1 − + 2 is or n + 2C 2. 20.  − 
 x x  2 x
2/3
− x1/ 3 + 1 x − x1/ 2 
1 1
[assuming and 2 distinct] 10
x x  (x1/ 3 )3 + 13 {( x )2 − 1} 
=  23 − 
(n + 2) (n + 1) x ( x − 1) 
x − x + 1
/ 1/3
∴ = 28
2 10
 (x1/ 3 + 1)(x2/ 3 + 1 − x1/3 ) {( x )2 − 1} 
⇒ (n + 2) (n + 1) = 56 = (6 + 1) (6 + 2) ⇒ n = 6 = − 
 x23
/
− x1/3 + 1 x ( x − 1) 
Hence, sum of coefficients = (1 − 2 + 4)6 = 36 = 729
 ( x + 1) 
10
− 1/ 2 10
Note As
1 1
and 2 are functions of same variables, therefore = (x1/3 + 1) −  = (x − x )
1/3

x x  x 
number of dissimilar terms will be 2 n + 1, i.e. odd, which is not ∴ The general term is
possible. Hence, it contains error. 10 − r r

18. Let Tt +1 be the general term in the expension of Tr + 1 = 10C r (x1/3 )10 − r (− x− 1/ 2)r = 10C r (− 1)r x 3 2

(1 − 2 x )50 For independent of x , put


50 − r 10 − r r
∴ Tr+ 1 = 50
C r (1) (−2x ) =
1/ 2 r 50
C r2 x (−1)
r r/ 2 r
− = 0 ⇒ 20 − 2r − 3r = 0
3 2
For the integral power of x, r should be even integer. ⇒ 20 = 5r ⇒ r = 4
25
10 × 9 × 8 × 7
∴ Sum of coefficients = ∑ 50C 2r (2)2r ∴ T5 = 10C 4 = = 210
r= 0
4 ×3 ×2 ×1
1 1
= [(1 + 2)50 + (1 − 2)50 ] = (350 + 1)
21. Here, Coefficient of t 24 in {(1 + t 2)12(1 + t12)(1 + t 24 )}
2 2 = Coefficient of t 24 in {(1 + t 2)12 ⋅ (1 + t12 + t 24 + t36 )}
Alternate Solution = Coefficient of t 24 in
We have, {(1 + t 2)12 + t12(1 + t 2)12 + t 24 (1 + t 2)12};
(1 − 2 x )50 = C o − C12 x + C 2( 2x)2 + ... + C50 (2 x )50 ...(i) [neglecting t36 (1 + t 2)12]

(1 + 2 x )50 = C o + C12 x + C 2(2 x )2 + ... + C50 (2 x )50 = Coefficient of t 24 = (12C12 + C6 +


12
C0 ) = 2 +
12 12
C6
...(ii) 22. Given, T5 + T6 = 0
On adding Eqs. (i) and (ii), we get ⇒ n
C 4a n − 4b4 − nC5 a n − 5 b5 = 0
a nC5 n − 4
(1 − 2 x )50 + (1 + 2 x )50 ⇒ n
C 4a n − 4b4 = nC5 a n − 5 b5 ⇒ = =
b nC 4 5
= 2 [C 0 + C 2(2 x )2 + ... + C50 (2 x )50 ] ...(iii)
m (m − 1) 2
(1 − 2 x ) + (1 + 2 x )50
50
23. (1 + x)m (1 − x)n = 1 + mx + x + K

⇒  2 
2
 n (n − 1) 2 
= C 0 + C 2(2 x )2 + ... + C50 (2 x )50 1 − nx + 2
x − ...

On putting x = 1, we get m(m − 1) n (n − 1)  2
= 1 + (m − n ) x +  + − mn  x + K
(1 − 2 1 )50 + (1 + 2 1 )50  2 2 
= C 0 + C 2 + ... + C50 (2)50
2 term containing power of x ≥ 3.
(−1)50 + (3)50 Now, m − n =3 …(i)
⇒ = C 0 + C 2(2)2 + ... + C50 (2)50
2 [Q coefficient of x = 3, given]
1 + 350 1 1
⇒ = C 0 + C 2(2)2 + ... + C50 (2)50 and m(m − 1) + n (n − 1) − mn = − 6
2 2 2
rankershalt.com

Binomial Theorem 91

⇒ m(m − 1) + n (n − 1) − 2mn = − 12 Similarly, coefficient of x4 = 0


⇒ m2 − m + n 2 − n − 2mn = − 12 ⇒ 18
C 4 ⋅ 24 − a ⋅18 C3 23 + b ⋅18 C 2 ⋅ 22 = 0
⇒ (m − n )2 − (m + n ) = − 12 ∴ 32a − 3b = 240 …(ii)
⇒ m + n = 9 + 12 = 21 …(ii)
On solving Eqs. (i) and (ii), we get
On solving Eqs. (i) and (ii), we get m = 12 272
a = 16, b =
24. We know that, 3
(a + b)5 + (a − b)5 = 5C 0a5 + 5C1a 4b + 5C 2a3 b2 28. Let the coefficients of 2nd, 3rd and 4th terms in the
+ 5C3 a 2b3 + 5C 4ab4 + 5C5 b5 + 5C 0a5 − 5C1a 4b expansion of (1 + x)n is nC1, nC 2, nC3 .
+ 5C 2a3 b2 − 5C3 a 2b3 + 5C 4ab4 − 5C5 b5 According to given condition,
= 2 [a5 + 10a3 b2 + 5ab4 ] 2 (nC 2) = nC1 + nC3
∴ [x + (x − 1) ] + [x − (x − 1) ]
3 1/ 2 5 3 1/ 2 5 n (n − 1) n (n − 1) (n − 2)
⇒ 2 =n+
= 2 [x5 + 10x3 (x3 − 1) + 5x (x3 − 1)2] 1 ⋅2 1 ⋅2 ⋅3
(n − 1) (n − 2)
Therefore, the given expression is a polynomial of ⇒ n −1 =1 +
6
degree 7.
n 2 − 3n + 2
10 ⇒ n −1 =1 +
25. The general term in  −
x 3 6
 is
2 x2
10 − r r
⇒ 6n − 6 = 6 + n 2 − 3n + 2
 x  3 3r 10 − 3 r ⇒ n 2 − 9n + 14 = 0
tr + 1 = (−1) r 10
Cr    2 = (−1)
r 10
C r. ⋅x
 2 x  210 − r ⇒ (n − 2) (n − 7) = 0
For coefficient of x4, we put 10 − 3r = 4 ⇒ n =2
⇒ 3r = 6 or n=7
⇒ r =2 But C3 is true for n ≥ 3, therefore n = 7 is the answer.
n

10
x 3 32 29. Given,
∴ Coefficient of x4 in  − 2 = (−1)2. 10C 2. 8
2 x  2 (1 + ax)n = 1 + 8x + 24x2 + ...
45 × 9 405 n (n − 1) 2 2
= = ⇒ 1 + anx + a x + ... = 1 + 8x + 24x2 + ...
256 256 2!
26. In the expansion (1 + x)2n, t3 r = 2nC3 r − 1 (x)3 r − 1 n (n − 1)
∴ an = 8 and a 2 = 24
r+1
2
and tr + 2 = 2n
C r + 1 (x)
⇒ 8 (8 − a ) = 48
Since, binomial coefficients of t3 r and tr + 2 are equal. ⇒ 8 − a =6 ⇒ a =2
∴ 2n
C3 r −1 = 2nC r + 1 Hence, a =2 and n =4
⇒ 3r − 1 = r + 1 or 2n = (3r − 1) + (r + 1)
30. Since, n is an odd integer, (− 1)n−1 = 1
⇒ 2r = 2 or 2n = 4r
⇒ r =1 or n = 2r and n − 1, n − 3, n − 5, etc., are even integers, then
But r >1 n3 − (n − 1)3 + (n − 2)3 − (n − 3)3 + K + (− 1)n−1 ⋅ 13
∴ We take, n = 2r = n3 + (n − 1)3 + (n − 2)3 + K + 13
27. To find the coefficient of x3 and x4, use the formula of − 2 [(n − 1)3 + (n − 3)3 + K + 23 )]
coefficient of x in (1 − x) is (−1) Cr and then simplify.
r n rn
  n − 1 3  n − 3 3 
= Σ n3 − 2 × 23   +  +K+1 
3
In expansion of (1 + ax + bx2)(1 − 2x)18. 
 2   2  
Coefficient of x3 = Coefficient of x3 in (1 − 2x)18
[Q n − 1, n − 3, ... , are even integers]
+ Coefficient of x2 in a (1 − 2x)18
  n − 1 3 
+ Coefficient of x in b(1 − 2x)18 = Σ n3 − 16 Σ   
  2  
= 18
C3 ⋅ 23 + a 18C 2 ⋅ 22 − b 18C1 ⋅ 2
2 2
Given, coefficient of x3 = 0  n (n + 1)  1  n − 1  n − 1 
=  − 16     + 1 
 2  2  2   2 
⇒ C3 ⋅ 23 + a 18C 2 ⋅ 22 − b 18C1 ⋅ 2 = 0
18

18 × 17 × 16 18 × 17 2 1 2 16 (n − 1)2(n + 1)2
⇒ − ⋅8 + a ⋅ ⋅ 2 − b ⋅ 18 ⋅ 2 = 0 = n (n + 1)2 −
3 ×2 2 4 4 ×4 ×4
34 × 16 =
1 1
(n + 1)2 [n 2 − (n − 1)2] = (n + 1)2(2n − 1)
⇒ 17a − b = ..(i)
3 4 4
rankershalt.com

92 Binomial Theorem

31. Consider, (101)50 − (99)50 − (100)50 33. Let y = (x − a )m, where m is a positive integer, r ≤ m
= (100 + 1)50 − (100 − 1)50 − (100)50 dy d 2y
Now, = m(x − a )m − 1 ⇒ = m(m − 1) (x − a )m − 2
= {(100) 50
(1 + 0.01) 50
− (1 − 0.01) 50
− 1)} dx dx2
= (100) {2 ⋅ [ C1 (0.01) +
50 50 50
C3 (0.01)3 + K ] − 1} d3 y
⇒ = m(m − 1)(m − 2)(m − 3)(x − a )m − 4
= (100) {2 [ C3 (0.01) +
50 50 3 50
C5 (0.01)5 + ... ]} dx3
…………………………………
∴ (101)50 − {(99)50 + (100)50 } > 0 …………………………………
⇒ (101)50 > (99)50 + (100)50 On differentiating r times, we get
dr y
32. Since, n is an even positive integer, we can write = m(m − 1) ... (m − r + 1)(x − a )m − r
dxr
n = 2 m ,m = 1, 2, 3, K m!
3n 3(2m)
3m = (x − a )m − r = r !(mC r )(x − a )m − r
Also, k = = = 3m ∴ S = ∑ (−3)r − 1 ⋅6mC 2r − 1 (m − r )!
2 2 r =1 dr y
and for r > m, =0
i.e. S = (−3) 0 6m
C1 + (−3) 6m
C3 + K dxr
2n 2n
3 m − 1 6m
+ (−3) ⋅ C3m − 1 …(i) Now, ∑ a r (x − 2)r = ∑ br (x − 3)r [given]
From the binomial expansion, we write r=0 r=0

(1 + x)6 m = 6m
C0 + 6m
C1x + 6m
C2 x2 + K On differentiating both sides n times w.r.t. x, we get
6m − 1 2n 2n
6m
+ 6m 6m
C 6 m − 1x C6mx …(ii)
∑ a r (n !)rC n (x − 2)r − n = ∑ br (n !)rC n (x − 3)r − n
(1 − x)6 m = 6m
C0 + 6m
C1 (− x) + 6m
C 2(− x)2 + K r=n r=n
6m − 1 2n
+ 6m
C 6 m − 1 (− x) + 6m
C 6 m (− x)6 m …(iii)
On subtracting Eq. (iii) from Eq. (ii), we get
On putting x = 3, we get ∑ a r (n !)rC n = (bn )n !
r=n

(1 + x)6 m − (1 − x)6 m = 2 [6 mC1x + 6m


C3 x3 [since, all the terms except first on RHS become zero]
+ 6m
C5 x5 + K + 6 m C 6 m − 1x6 m − 1 ] ⇒ bn = nC n + n+1
Cn + n+ 2
Cn + K + 2n
Cn
(1 + x) 6m
− (1 − x) 6m
[Q a r = 1, ∀ r ≥ n ]
⇒ = 6mC1 + 6m
C3 x2 + 6m
C5 x4 + ...
2x = (n + 2C n + 1 + n+ 2
C n ) + K + 2n C n
+ 6 mC 6 m − 1 x6 m − 2 n+3
= Cn + 1 + K + 2n
Cn = ....
Let x2 = y 2n + 1
= Cn + 1 +
2n 2n
Cn = Cn + 1
(1 + y )6 m − (1 − y )6 m
⇒ = 6m
C1 + 6m
C3 y n
1 3r 7r 15r 
2 y 34. ∑ (−1)r nC r  r + 2r + 3 r + 4r + ... upto m terms 
+ 6m
C5 y2 + K + 6 m C 6 m − 1 y 3 m − 1 r=0 2 2 2 2 
n r n r
For the required sum we have to put y = − 3 in RHS.  1  3
= ∑ (−1)r nC r  2 + ∑ (−1)r nC r  4 +
(1 + −3 )6m − (1 − −3 )6m r=0 r=0
∴ S=
2 −3 n
 7
r

(1 + i 3 )6m − (1 − i 3 )6m
∑ (−1)r nC r  8  + ... upto m terms
r=0
= …(iv)
2i 3 n n n
 1  3  7
z = 1 + i 3 = r (cos θ + i sin θ ) = 1 −  + 1 −  + 1 −  + ... upto m terms
Let  2  4  8
⇒ r = |z| = 1 + 3 = 2  n 
and θ = π /3 using

∑ (−1)r nC rxr = (1 − x)n 
Now, z 6 m = [r (cos θ + i sin θ )]6 m r= 0 
= r 6 m (cos 6m θ + i sin 6m θ )  1  1
n
 1
n n
=   +   +   + ... upto m terms
Again, z = r (cos θ − i sin θ )  2  4  8
and (z )6 m = r 6 m (cos 6m θ − i sin 6m θ )   1 
m
1 −  n  
n
⇒ z 6 m − z 6 m = r 6 m (2i sin 6m θ )  1 2   2mn − 1
…(v) =   = mn n
 2  1  2 (2 − 1)
 1 − 2n 
From Eq. (i),
z 6 m − z 6 m r 6 m (2i sin 6 m θ )  
S= =
2i 3 2i 3 n+ 1 n+ 1 n+ 1 n +1
35. C1 + C 2 s1 + C3 s2 + ... + C n+ 1sn
26 m sin 6 m θ
= n+ 1
3 = ∑ n + 1C rsr −1,
= 0 as m ∈ z , and θ = π /3 r =1
rankershalt.com

Binomial Theorem 93

1 − qn + 1 Topic 2 Properties of Binomial Coefficient


where sn = 1 + q + q2 + ... + qn =
1−q
n+ 1
 1 − qr  1  n +1 n+ 1  1. We have,
∴ ∑ n + 1C r  1 − q  = 1 − q  ∑ n + 1C r − ∑ n + 1C r qr (x + 10)50 + (x − 10)50 = a 0 + a1x + a 2x2 + … + a50x50
r =1  r =1 r =1 
∴a 0 + a1x + a 2x2 + … + a50x50
1
= [(1 + 1)n + 1 − (1 + q)n + 1 ] = [(50C 0x50 + 50
C1x49 10 + 50
C 2 x48 ⋅ 102 + … + 50
C50 1050 )
1 −q
1 + (50C 0x50 − 50C1x49 10 + 50C 2 x48102− … + 50
C50 1050 )]
= [2n + 1 − (1 + q)n + 1 ] …(i)
1 −q = 2 [50C 0 x50 + 50C 2x48 ⋅ 102 + 50C 4x46 ⋅ 104
2 n +…+ 50
C50 ⋅ 1050 ]
 q + 1  q + 1  q + 1
Also, S n = 1 +   +  + ... +   By comparing coefficients, we get
 2   2   2 
n +1 a 2 = 2 50C 48 (10)48 ; a 0 = 2 50C50 (10)50 = 2(10)50
 q + 1
1−  a 2 2(50C 2)(10)48 50 ⋅ 49 (10)48
 2  2n + 1 − (q + 1)n + 1 ∴ = = 2
= = ... (ii) 2 (10)50 1 ⋅ 2 2 ⋅ (10)50
 q + 1 2n (1 − q) a0
1− 
 2  [Q 50
C 48 = 50C 2]
From Eqs. (i) and (ii), 50 × 49 5 × 49 245
= = = = 12 .25
n+ 1
C1 + n+ 1
C 2 s1 + n+ 1
C3 s2 + ... + n+ 1
C n + 1sn = 2 S n
n 2 ⋅ (10 × 10) 20 20
2
36. Coefficient of x in the expansion of 2. We know that,
{(1 + x)2 + (1 + x)3 + K + (1 + x)49 + (1 + mx)50} (1 + x)20 = 20
C0 + 20
C1x + 20
C 2x2 + ... +
⇒ 2C 2 + 3C 2 + 4C 2 + K + 49C 2 + 50C 2 ⋅ m2 20
C r − 1xr − 1 + 20
C rxr + ... + 20
C 20x20
= (3n + 1) ⋅51 C3
∴ (1 + x)20 ⋅ (1 + x)20 = (20C 0 + 20
C1x +
⇒ 50
C3 + C 2m = (3n + 1) ⋅ C3
50 2 51
r −1
[QrC r + r + 1C r + K+ nC r = n + 1C r + 1]
20
C 2x + ... +
2 20
C r − 1x + 20
C rx + ... +
r 20
c20x20 )
50 × 49 × 48 50 × 49 51 × 50 × 49 × (20C 0 + 20
C1x + ...+ 20
C r − 1xr − 1 + 20
C rxr
⇒ + × m2 = (3n + 1)
3 ×2 ×1 2 3 ×2 ×1 + ....+ 20C 20x20 )
⇒ m2 = 51n + 1
⇒ (1 + x) 40
= ( C0 .
20 20
Cr + 20
C1 20
C r − 1 ...
∴ Minimum value of m2 for which (51n + 1) is integer
(perfect square) for n = 5. 20
C r20C 0 ) xr + ...
∴ m2 = 51 × 5 + 1 ⇒ m2 = 256
On comparing the coefficient of xr of both sides, we get
∴ m = 16 and n = 5
Hence, the value of n is 5.
20
C 020C r + 20
C120C r − 1 + ... + 20
Cr 20
C0 = 40
Cr

37. Coefficient of x9 in the expansion of The maximum value of 40C r is possible only when r = 20
(1 + x)(1 + x )(1 + x ) K (1 + x ) =Terms having x
2 3 100 9
[Q nC n/2 is maximum when n is even]
= [199 ⋅ x9 , 198 ⋅ x ⋅ x8 , 198 ⋅ x2 ⋅ x7 , 198 ⋅ x3 ⋅ x6 , Thus, required value of r is 20.
198 ⋅ x4 ⋅ x5 , 197 ⋅ x ⋅ x2 ⋅ x6 , 197 ⋅ x ⋅ x3 ⋅ x5 ,197 ⋅ x2 ⋅ x3 ⋅ x4]
3. Consider,
∴ Coefficient of x9 = 8
2403 = 2400 + 3 = 8 ⋅ 2400 = 8 ⋅ (24 )100 = 8 (16)100= 8(1 + 15)100
38. Let the three consecutive terms in (1 + x)n + 5 be
= 8 (1 + 100
C1 (15) + 100
C 2(15)2 + … + 100
C100 (15)100 )
tr , tr + 1 , tr + 2 having coefficients
n+ 5 [By binomial theorem,
C r − 1 , n + 5C r , n + 5C r + 1.
Given, n + 5 C r − 1 : n + 5C r : n + 5C r + 1 = 5 : 10 : 14 (1 + x)n = nC 0 + nC1x + nC 2x2 + … nC nxn , n ∈ N ]
n+ 5
Cr 10
n+ 5
C r + 1 14 = 8 + 8 (100C1 (15) + 100
C 2(15)2 + … + 100
C100 (15)100 )
∴ = and =
n+ 5
C r −1 5 n+ 5
Cr 10 = 8 + 8 × 15λ
where λ =100 C1 +......+ 100C100 (15)99 ∈ N
n + 5 − (r − 1) n−r+5 7
⇒ =2 and = 2403 8 + 8 × 15λ 8
r r+1 5 ∴ = = 8λ +
15 15 15
⇒ n − r + 6 = 2 r and 5n − 5 r + 25 = 7r + 7
2403  8
⇒ n + 6 = 3 r and 5n + 18 = 12r ⇒  =
n + 6 5n + 18  15  15
∴ =
3 12 (where {⋅} is the fractional part function)
⇒ 4n + 24 = 5n + 18 ⇒ n = 6 ∴ k =8
rankershalt.com

94 Binomial Theorem

m
Alternate Method 10  20 
2 403
= 8 ⋅2 400
= 8(16) 100 ⇒ ∑  
m
 is the coefficient of x in the
 i  m − i
i=0
Note that, when 16 is divided by 15, gives remainder 1. expansion of (1 + x)30
m
∴ When (16)100 is divided by 15, gives remainder 1100 = 1 10  20  30 30
and when 8(16)100 is divided by 15, gives remainder 8.
i.e. ∑    = Cm =  
 i  m − i  m
…(i)
i=0

2403  8  n
∴ and we know that,   is maximum, when
 = .  r
 15  15
 n
(where {⋅} is the fractional part function)  n  r=2, if n ∈ even.
  =
⇒ k =8  r  max  n±1 .
 r = 2 , if n ∈ odd
4. A r = Coefficient of xr in (1 + x)10 = 10C r 30
Hence,   is maximum when m = 15.
B r = Coefficient of xr in (1 + x)20 = 20
Cr  m
C r = Coefficient of x in r
(1 + x) 30
= 30
Cr  n  n   n   n  n  
8.   + 2  +  =   +  
10 10 10  r  r − 1  r − 2  r   r − 1 
∴ ∑ Ar (B10 Br − C10 Ar ) = ∑ Ar B10 Br − ∑ Ar C10 Ar  n   n    n + 1  n + 1  n + 2
r =1 r =1 r =1
+  + =  +  = 
10 10  r − 1  r − 2   r   r − 1   r 
= ∑ 10C r 20C10 20C r − ∑ 10C r 30C10 10C r [Q nC r + nC r − 1 = n+1
Cr ]
r =1 r =1 n n
r n − (n − r )
10 10 9. Let b = ∑ n
= ∑ n
= ∑ 10C10 − r 20C10 20C r − ∑ 10C10 − r 30C10 10C r r=0
n
Cr r=0
n
Cr
r =1 r =1 1 n−r
10 10
=n ∑
C
− ∑ n
n
r=0 r = 0 Cr
∑ 10C10 − r ⋅ 20C r − 30C10 ∑ 10C10 − r 10C r
r
= 20
C10 n
n−r
r =1 r =1 = na n − ∑ n [Q nC r = nC n − r ]
r = 0 Cn − r
= 20
C10 ( 30C10 − 1) − 30C10 (20C10 − 1)
n
= na n − b ⇒ 2b = na n ⇒ b = an
= 30C10 − 20
C10 = C10 − B10 2
10. We have,
30 30 30 30 30 30 30 30 C 02 − 2C12 + 3C 22 − 4C32 + ... + (−1)n (n + 1) C n2
5. Let A =     −     +     − ... +    
 0  10  1  11  2  12 20 30
= [C 02 − C12 + C 22 − C32 + ... + (−1)nC n2 ]
∴ A = 30C 0 ⋅30 C10 − 30C1 ⋅30 C11 + 30
C 2 ⋅30 C12
− [ C12 − 2 C 22 + 3 C32 − ... + (−1)n nC n2 ]
−K + C 20 ⋅ C30
30 30 n
−1 n
n! n!
= Coefficient of x20 in (1 + x)30 (1 − x)30 = (−1)n/ 2 − (−1) 2
 n  n 2  n  n
= Coefficient of x20 in (1 − x2)30  !  !   !  !
 2  2  2  2
30
 n
∑ (−1)r
30 n!
= Coefficient of x20 in C r (x2)r = (−1)n/ 2 1 + 
r=0    
n n  2
 !  !
 2  2
= (−1) 10 30
C10 [for coefficient of x20, put r = 10]
 n  n
= C10
30 2 !  !
 2  2
n −1 ∴ [C 02 − 2 C12 + 3 C 22 − ... + (− 1)r (n + 1) C n2 ]
6. Given , C r = (k2 − 3) nC r + 1 n!
n −1 n n −1  n  n
⇒ C r = (k2 − 3) Cr 2  !  !
r+1  2  2 n! (n + 2)
= (−1)n/ 2 = (−1)n/ 2(n + 2)
r+1 n!    
n n 2
⇒ k2 − 3 =  !  !
n  2  2
r+1
[since, n ≥ r ⇒
≤ 1 and n , r > 0] 11. We have,
n
X = (10C1 )2 + 2(10C 2)2+ 3(10C3 )2 + ... + 10 (10C10 )2
⇒ 0 < k −3 ≤1 ⇒ 3 < k ≤4
2 2
10 10
⇒ k ∈ [−2, − 3 ) ∪ ( 3 , 2]
m
⇒ X= ∑ r (10C r )2 ⇒ X= ∑ r 10C r 10C r
10  20  r =1 r =1
7. ∑    m
 is the coefficient of x in the expansion of 10
 i   m − i 10 9  n n 
i=0 ⇒ X = ∑r× Cr − 1 10
Cr Q C r = r
n −1
Cr − 1 
(1 + x) (x + 1) ,
10 20
r =1
r 
rankershalt.com

Binomial Theorem 95

10
Again, we have to prove that
⇒ X = 10 ∑ 9C r − 1 10C r  n  n − 1  n − 2 m  n + 2
r =1
  +2  +3  + ... + (n − m + 1)   =  
10 m  m   m  m m + 2
⇒ X = 10 ∑ 9C r − 1 10C10 − r [Q nC r = nC n − r ]
 n  n − 1  n − 2 m
r =1 Let S1 =   + 2   +3  + ... + (n − m + 1)  
n−1 2n − 1 m  m   m  m
⇒ X = 10 × 19C 9 [Q C r − 1nC n − r = Cn − 1 ]
 n   n − 1  n − 2 m 
1 10 × C 9 19
C9 C9 19 19
=  +  +  + ... +   
Now, X= = = m  m   m  m
1430 1430 143 11 × 13 
 n − 1  n − 2 m 
19 × 17 × 16 + +  + ... +   
= = 19 × 34 = 646  m   m  m
8 
 n − 2 m  n − m + 1 rows
12. Sum of coefficients is obtained by putting x = 1 +  + ... +   
 m  m

i.e. (1 + 1 − 3)2163 = − 1 ......
Thus, sum of the coefficients of the polynomial m 
+  
(1 + x − 3x2)2163 is −1. m 
13. To show that  n + 1
Now, sum of the first row is  .
2k.n C 0.n C k − 2k − 1.n C1.n − 1 C k − 1 m + 1
+ 2k − 2.n C 2.n − 2 C k − 2 − K + (−1)k nC kn − kC 0 = nC k  n 
Sum of the second row is  .
Taking LHS m + 1
2k.n C 0.n C k − 2k −1.n C1 ⋅n − 1 C k − 1 + K + (−1)k.n C k.n − k C 0  n − 1
k Sum of the third row is  ,
m + 1
= ∑ (−1) r. k − r . n
2 Cr .n − r Ck − r
…………………………
r=0
k
n! (n − r )! m m + 1
Sum of the last row is   = 
= ∑ (−1)r 2k − r. r !(n − r )! . (k − r )!(n − k)! m m + 1
.
r=0
k  n + 1  n   n − 1 
n! k! S= + + 
= ∑ (−1)r. 2k − r. ⋅
(n − k)!. k ! r !(k − r )!
Thus,
m + 1 m + 1 m + 1
r=0

 k  m + 1  n + 1 + 1  n + 2
k
r. 1 .k
+K+ =   = 
= ∑ (− 1 ) r. k − r n
2 C k
. k
C r = 2 k.n
C k  ∑ (−1 )
2r
Cr  m + 1  m + 2  m + 2
r=0 r = 0 
k
[from Eq. (i) replacing n by n + 1 and m by m + 1]
 1
=2 k.n
C k 1 −  = n C k = RHS n n
Cr
 2 15. ∑ (−1)r r +3
Cr
r=0
 n  n − 1  n − 2 m  n + 1
14. Let S =   +  +  + ... +   =   …(i) n
n !⋅ 3 ! n
n!
m  m   m  m m + 1 = ∑ (−1)r = 3 ! ∑ (−1)r
r=0
(n − r ) ! (r + 3) ! r=0
(n − r ) ! (r + 3) !
It is obvious that, n ≥ m. [given]
n
3! (−1)r. (n + 3)!
∑ (n − r )!(r + 3)!
NOTE This question is based upon additive loop.
=
m m + 1 m + 2  n (n + 1)(n + 2)(n + 3) r=0
Now, S =   +  +  + ... +  
m  m   m  m 3! n
= ⋅ ∑ (−1)r ⋅n + 3C r + 3
m + 1 m + 1  m + 2  n (n + 1)(n + 2)(n + 3) r = 0
=  + +   + K 
  m + 1  m    m  m n+3
3 ! (− 1)3
 m m + 1  = ∑ (−1)s ⋅n + 3 C3
Q m = 1 = m + 1  (n + 1)(n + 2)(n + 3) s =3
    
m + 2 m + 2  n −3! n + 3 
= +  + ... +   =  ∑ (−1)s ⋅n + 3 C s
m + 1  m  m 
(n + 1)(n + 2)(n + 3)  s =` 0 

n+1
[Q nC r + nC r + 1 = Cr + 1 ] − n+3
C0 + n+3
C1 − n + 3C 2
m + 3  n
=  + ... +   −3!  (n + 3)(n + 2) 
m + 1 m = 0 − 1 + (n + 3) − 
= ............................... (n + 1)(n + 2)(n + 3)  2! 
 n   n   n + 1 −3! (n + 2)(2 − n − 3) 3!
=  +   =  , which is true. …(ii) = ⋅ =
m + 1 m m + 1 (n + 1)(n + 2)(n + 3) 2 2(n + 3)
rankershalt.com

96 Binomial Theorem
n n n
16. (1 + x + x2)n = a 0 + a1x + K + a 2nx2n …(i) = ∑ Ci ∑ C j − ∑ Ci2
i=0 j=0 i=0
Replacing x by −1 / x, we get
n
= 2n 2n − (2 nC n ) = 22 n − 2 nC n
 1 1 a1 a 2 a3 a 2n
22 n − 2 nC n
 1 − + 2 = a 0 − + 2 − 3 + K + 2n …(ii) (2n )!
 x x  x x x x ∴ ∑∑ CiC j =
2
= 22 n−1 −
2 (n !)2
0 ≤i< j≤n
Now, a 02 − a12 + a 22 − a32 + K + a 22n = coefficient of the
term independent of x in 19. We know that, (1 + x)2 n = C 0 + C1x + C 2 x2 + ... + C 2 n x 2 n
[a 0 + a1x + a 2x2 + K + a 2nx2n ] On differentiating both sides w.r.t. x, we get
 a a a 2n  2n (1 + x)2 n−1 = C1 + 2 ⋅ C 2 x + 3 ⋅ C3 x2
× a 0 − 1 + 22 − K + 2n  + ... + 2nC 2n x2 n−1 …(i)
 x x x  2n
= Coefficient of the term independent of x in  1 1 1 1
and 1 −  = C 0 − C1 ⋅ + C 2 ⋅ 2 − C3 ⋅ 3
n  x x x x
 1 1
(1 + x + x2)n 1 − + 2 1
 x x  + ... + C 2n ⋅ 2 n …(ii)
n x
 1 1
Now, RHS = (1 + x + x2)n 1 − + 2 On multiplying Eqs. (i) and (ii), we get
 x x 
2n
(1 + x + x2)n (x2 − x + 1)n [(x2 + 1)2 − x2]n  1
= = 2n (1 + x)2 n−1 1 − 
 x
x2n x2n
(1 + 2x + x − x )
2 4 2 n
(1 + x + x )
2 4 n
= [C1 + 2 ⋅ C 2x + 3 ⋅ C3 x2 + ... + 2n ⋅ C 2n x2n−1 ]
= =
x2n x2n   1  1  1 
× C 0 − C1   + C 2 2 − ..... + C 2 n  2 n  
Thus, a 0 − a1 + a 2 − a3 + K+ a 2n
2 2 2 2 2
  x  x  x 
= Coefficient of the term independent of x in  1
Coefficient of   on the LHS
1  x
(1 + x2 + x4 )n
x2n 1  1 
= Coefficient of in 2n  2 n  (1 + x)2 n−1 (x − 1)2 n
= Coefficient of x in (1 + x + x )
2n 2 4 n x x 
= Coefficient of t n in (1 + t + t 2)n = a n = Coefficient of x2 n−1 in 2n (1 − x2)2 n−1 (1 − x)
17. C 0 − 2 ⋅ C1 + 3 ⋅ C 2 − ... + (−1) (n + 1) ⋅ C n
2 2 n 2 = 2n (−1)n−1 ⋅ (2n − 1) C n−1 (−1)
(2n − 1)! (2n )!
n n = (−1)n (2n ) = (−1)n n ⋅n
= ∑ (−1) r(r + 1)2 nC r = ∑ (−1)r (r 2 + 2r + 1) nC r (n − 1)! n ! (n !)2
r=0 r=0 = − (−1)n n ⋅ C n …(iii)
n n n
 1
= ∑ (−1) r r 2 n
⋅ C r + 2 ∑ (−1) r ⋅ C r + r n
∑ (−1) r. n
Cr Again, the coefficient of   on the RHS
 x
r=0 r=0 r=0
n n
= − (C12 − 2 ⋅ C 22 + 3 ⋅ C32 − ... − 2n C 22n ) …(iv)
= ∑ (−1)r. r (r − 1) ⋅n C r + 3 ⋅ ∑ (−1)r. r ⋅ nC r From Eqs. (iii) and (iv),
r=0 r=0 n
+ ∑ (−1)r nC r C12 − 2 ⋅ C 22 + 3 ⋅ C32 − ... − 2n ⋅ C 22n = (−1)n n ⋅ C n
r=0 2n
20. (1 + x)2 n 1 − 
n n
1
 x
= ∑ (−1) n (n − 1)
r n− 2
C r − 2 + 3 ∑ (−1) n . r n −1
C r −1
r=2 r =1
n = [ C 0 + (2nC1 )x + (2nC 2)x2 + ...+ (2 nC 2 n )x2 n ]
2n

+ ∑ (−1) r n
Cr  1 1 1 
×  2 nC 0 − (2 nC1 ) + (2 nC 2) 2 + ... + (2 nC 2n ) 2n 
r=0  x x x 
= n (n − 1){ n − 2C 0 − n − 2C1 + n − 2C 2−... + (−1)n n − 2C n − 2} Independent terms of x on RHS
+ 3n { − n−1C 0 + n − 1C1 − n −1C 2 + ...+ (−1)n n − 1C n − 1} = (2nC 0 )2 − (2nC1 )2 + (2nC 2)2 − ...+ (2nC 2n )2
+ { C 0 − C1 + C 2 + ... + (−1)
n n n n n
Cn}  x − 1
2n
1
LHS = (1 + x)2n   = (1 − x2)2n
= n (n − 1) . 0 + 3n . 0 + 0, ∀n > 2 = 0, ∀n > 2  x  x2n
18. We know that, n n n n Independent term of x on the LHS = (−1)n ⋅2n C n.
2 ∑ ∑ Ci C j = ∑ ∑ Ci C j − ∑ ∑ CiC j
0 ≤i < j ≤ n i=0 j=0 i = 0 j=0

Download Chapter Test


http://tinyurl.com/yxkarvhw or
rankershalt.com

6
Probability

Topic 1 Classical Probability


Objective Questions I (Only one correct option) 7. If 12 identical balls are to be placed in 3 different boxes,
then the probability that one of the boxes contains excatly
1. A person throws two fair dice. He wins
3 balls, is (2015 Main)
` 15 for throwing a doublet (same numbers on the two 11 10 12 11
55  2 
(b) 55   (c) 220   (d) 22  
dice), wins ` 12 when the throw results in the sum of 2 1 1
(a)  
9, and loses ` 6 for any other outcome on the throw. 3  3  3  3  3
Then, the expected gain/loss (in `) of the person is 8. Three boys and two girls stand in a queue. The probability
(2019 Main, 12 April II)
that the number of boys ahead of every girl is atleast one
1 1 1
(a) gain (b) loss (c) loss (d) 2 gain more that the number of girls ahead of her, is (2014 Adv)
2 4 2
(a) 1 /2 (b) 1 /3 (c) 2 /3 (d) 3 /4
2. In a random experiment, a fair die is rolled until two
fours are obtained in succession. The probability that
9. Four fair dice D1 , D2, D3 and D4 each having six faces
the experiment will end in the fifth throw of the die is numbered 1, 2, 3, 4, 5 and 6 are rolled simultaneously. The
equal to (2019 Main, 12 Jan I) probability that D4 shows a number appearing on one of
175 225 D1 , D2 and D3 , is (2012)
(a) (b)
6 5
65 91 108 125 127
(a) (b) (c) (d)
200 150 216 216 216 216
(c) (d)
65 65 10. Let ω be a complex cube root of unity with ω ≠ 1. A fair die is
thrown three times. If r1, r2 and r3 are the numbers obtained
3. If there of the six vertices of a regular hexagon are
on the die, then the probability that ω r 1 + ω r2 + ω r3 = 0, is
chosen at random, then the probability that the
(2010)
triangle formed with these chosen vertices is
(a) 1/18 (b) 1/9 (c) 2/9 (d) 1/36
equilateral is
(2019 Main, 12 April I) 11. If three distinct numbers are chosen randomly from the
1 1 3 3 first 100 natural numbers, then the probability that all
(a) (b) (c) (d)
10 5 10 20 three of them are divisible by both 2 and 3, is (2004, 1M)
4 4 4 4
4. Let S = {1, 2, K , 20}. A subset B of S is said to be (a) (b) (c) (d)
55 35 33 1155
“nice”, if the sum of the elements of B is 203. Then,
the probability that a randomly chosen subset of S is 12. Two numbers are selected randomly from the set
‘‘nice’’, is (2019 Main, 11 Jan II) S = {1, 2, 3, 4, 5, 6} without replacement one by one. The
6 4 7 5 probability that minimum of the two numbers is less than
(a) (b) (c) (d)
220 220 220 220 4, is (2003, 1M)
(a) 1/15 (b) 14/15 (c) 1/5 (d) 4/5
5. If two different numbers are taken from the set {0, 1,
2, 3, …, 10}, then the probability that their sum as 13. If the integers m and n are chosen at random between 1
well as absolute difference are both multiple of 4, is and 100, then the probability that a number of the form
(2017 Main) 7m + 7n is divisible by 5, equals (1999, 2M)
6 12 14 7 1 1 1 1
(a) (b) (c) (d) (a) (b) (c) (d)
55 55 45 55 4 7 8 49
6. Three randomly chosen non-negative integers x, y 14. Seven white balls and three black balls are randomly
and z are found to satisfy the equation x + y + z = 10. placed in a row. The probability that no two black balls are
Then the probability that z is even, is (2017 Adv.) placed adjacently, equals (1998, 2M)
1 36 6 5 1 7 2 1
(a) (b) (c) (d) (a) (b) (c) (d)
2 55 11 11 2 15 15 3
rankershalt.com

98 Probability

15. Three of the six vertices of a regular hexagon are chosen Fill in the Blanks
at rondom. The probability that the triangle with three
vertices is equilateral, equals (1995, 2M) 21. Three faces of a fair die are yellow, two faces red and
(a) 1/2 (b) 1/5 (c) 1/10 (d) 1/20 one face blue. The die is tossed three times. The
probability that the colours, yellow, red and blue,
16. Three identical dice are rolled. The probability that the appear in the first, second and the third tosses
same number will appear on each of them, is (1984, 2M) respectively, is…… . (1992, 2M)
1 1 1 3
(a) (b) (c) (d) 1 + 3p 1 − p 1 − 2p
6 36 18 28 22. If , and are the probabilities of three
3 4 2
17. Fifteen coupons are numbered 1, 2, ..., 15, respectively. mutually exclusive events, then the set of all values of p
Seven coupons are selected at random one at a time is… . (1986, 2M)
with replacement. The probability that the largest
number appearing on a selected coupon is 9, is 23. A determinant is chosen at random from the set of all
6 7 7 determinants of order 2 with elements 0 or 1 only. The
(a)   (b)   (c)  
9 8 3
(d) None of these probability that the value of the determinant chosen is
 16   15   5
positive, is… . (1982, 2M)

Assertion and Reason True/False


For the following questions, choose the correct answer 24. If the letters of the word ‘ASSASSIN’ are written down
from the codes (a), (b), (c) and (d) defined as follows. at random in a row, the probability that no two S’s occur
(a) Statement I is true, Statement II is also true; together is 1/35.
Statement II is the correct explanation of Statement I
(b) Statement I is true, Statement II is also true; Analytical and Descriptive Questions
Statement II is not the correct explanation of
Statement I 25. An unbiased die, with faces numbered 1, 2, 3, 4, 5 and 6
(c) Statement I is true; Statement II is false is thrown n times and the list of n numbers showing up
(d) Statement I is false; Statement II is true is noted. What is the probability that among the
numbers 1, 2, 3, 4, 5 and 6 only three numbers appear in
18. Consider the system of equations this list? (2001, 5M)
ax + by = 0, cx + dy = 0, 26. If p and q are chosen randomly from the set {1, 2, 3, 4, 5,
where a , b, c, d ∈ {0, 1}. 6, 7, 8, 9 and 10} with replacement, determine the
Statement I The probability that the system of probability that the roots of the equation x2 + px + q = 0
equations has a unique solution, is 3/8. are real. (1997, 5M)
Statement II The probability that the system of 27. In how many ways three girls and nine boys can be
equations has a solution, is 1. (2008, 3M) seated in two vans, each having numbered seats, 3 in
the front and 4 at the back? How many seating
arrangements are possible if 3 girls should sit together
Passage Based Problems in a back row on adjacent seats? Now, if all the seating
Passage arrangements are equally likely, what is the probability
of 3 girls sitting together in a back row on adjacent
Box I contains three cards bearing numbers 1, 2, 3 ; box II
seats? (1996, 5M)
contains five cards bearing numbers 1, 2, 3, 4, 5; and box III
contains seven cards bearing numbers 1, 2, 3, 4, 5, 6, 7. A card 28. A box contains 2 fifty paise coins, 5 twenty five paise
is drawn from each of the boxes. Let xi be the number on the coins and a certain fixed number n (≥ 2) of ten and five
card drawn from the i th box i = 1, 2, 3. (2014 Adv.) paise coins. Five coins are taken out of the box at
random. Find the probability that the total value of
19. The probability that x1 + x2 + x3 is odd, is these 5 coins is less than one rupee and fifty paise.
29 53 57 1 (1988, 3M)
(a) (b) (c) (d)
105 105 105 2
29. Six boys and six girls sit in a row at random. Find the
20. The probability that x1 , x2 and x3 are in an arithmetic probability that
progression, is (i) the six girls sit together.
9 10 11 7
(a) (b) (c) (d) (ii) the boys and girls sit alternatively. (1978, 3M)
105 105 105 105
rankershalt.com

Probability 99

Topic 2 Addition and Subtraction Law of Probability


Objective Questions I (Only one correct option) Objective Questions II
1. For three events A , B and C, if P (exactly one of A or B (One or more than one correct option)
occurs) = P(exactly one of B or C occurs) = P (exactly one 8. For two given events A and B, P ( A ∩ B) is (1988, 2M)
1
of C or A occurs) = and P (all the three events occur (a) not less than P (A ) + P (B ) − 1
4 (b) not greater than P (A ) + P (B )
1
simultaneously) = , then the probability that atleast (c) equal to P (A ) + P (B ) − P (A ∪ B )
16 (d) equal to P (A ) + P (B ) + P (A ∪ B )
one of the events occurs, is (2017 Main)
7 7 7 3 9. If M and N are any two events, then the probability that
(a) (b) (c) (d) exactly one of them occurs is
32 16 64 16
3 1 1 (a) P (M ) + P (N ) − 2 P (M ∩ N ) (1984, 3M)
2. If P (B) = , P ( A ∩ B ∩ C ) = and P ( A ∩ B ∩ C ) = ,
(b) P (M ) + P (N ) − P (M ∪ N )
4 3 3
then P (B ∩ C ) is equal to (2002, 3M) (c) P (M ) + P (N ) − 2 P (M ∩ N )
1 1 1 1
(a) (b) (c) (d) (d) P (M ∩ N ) − P (M ∩ N )
12 6 15 9
3. If E and F are events with P (E ) ≤ P (F ) and Fill in the Blanks
P (E ∩ F ) > 0, then which one is not correct? (1998, 2M) 10. Three numbers are chosen at random without
(a) occurrence of E ⇒ occurrence of F replacement from {1, 2,…, 10}. The probability that the
(b) occurrence of F ⇒ occurrence of E minimum of the chosen number is 3, or their maximum
(c) non-occurrence of E ⇒ non-occurrence of F is 7, is … . (1997C, 2M)
(d) None of the above 11. P ( A ∪ B) = P ( A ∩ B) if and only if the relation between
4. For the three events A, B and C, P(exactly one of the P ( A ) and P (B) is… . (1985, 2M)
events A or B occurs) = P(exactly one of the events B or
C occurs) = P(exactly one of the events C or A occurs) True/False
= p and P(all the three events occurs simultaneously)
1 12. If the probability for A to fail in an examination is 0.2
= p2, where 0 < p < . Then, the probability of atleast and that of B is 0.3, then the probability that either A or
2
B fails is 0.5. (1989, 1M)
one of the three events A, B and C occurring is
(1996, 2M)
3p + 2p2 p + 3p2
Analytical and Descriptive Questions
(a) (b)
2 4 13. In a certain city only two newspapers A and B are
p + 3p2 3p + 2p2 published, it is known that 25% of the city population
(c) (d) reads A and 20% reads B, while 8% reads both A and B.
2 4
It is also known that 30% of those who read A but not B
5. If 0 < P ( A ) < 1, 0 < P (B) < 1 and P ( A ∪ B) = P ( A ) look into advertisements and 40% of those who read B
+ P (B) − P ( A ) P (B), then (1995, 2M) but not A look into advertisements while 50% of those
(a) P (B / A ) = P (B ) − P (A ) who read both A and B look into advertisements. What
(b) P (A ′ − B ′ ) = P (A ′ ) − P (B ′ ) is the percentage of the population reads an
(c) P (A ∪ B ) ′ = P (A ) ′ P (B ) ′ advertisement? (1984, 4M)
(d) P (A / B ) = P (A ) − P (B ) 14. A, B, C are events such that
6. The probability that at least one of the events A and B Pr ( A ) = 0.3, Pr (B) = 0.4, Pr (C ) = 0.8,
occurs is 0.6. If A and B occur simultaneously with Pr ( AB) = 0.08, Pr ( AC ) = 0.28 and Pr ( ABC ) = 0.09
probability 0.2, then P ( A ) + P (B ) is equal to (1987, 2M)
If Pr ( A ∪ B ∪ C ) ≥ 0.75, then show that Pr (BC ) lies in
(a) 0.4 (b) 0.8 (c) 1.2 (d) 1.4 the interval [ 0.23, 0.48 ]. (1983, 2M)
7. Two events A and B have probabilities 0.25 and 0.50,
15. A and B are two candidates seeking admission in IIT.
respectively. The probability that both A and B occur
The probability that A is selected is 0.5 and the
simultaneously is 0.14. Then, the probability that
probability that both A and B are selected is atmost 0.3.
neither A nor B occurs, is (1980, 1M)
Is it possible that the probability of B getting selected is
(a) 0.39 (b) 0.25 0.9? (1982, 2M)
(c) 0.11 (d) None of these
rankershalt.com

100 Probability

Pragraph Based Questions NONE of the remaining students gets the seat
previously allotted to him/her is
There are five students S1 , S 2, S3 , S 4 and S5 in a music class 3 1 7 1
(a) (b) (c) (d)
and for them there are five seats R1 , R2, R3 , R4and R5 40 8 40 5
arranged in a row, where initially the seat Ri is allotted to the
17. For i = 1, 2, 3, 4, let Ti denote the event that the students
student Si , i = 1, 2, 3, 4, 5. But, on the examination day, the
five students are randomly allotted the five seats. Si and Si+1 do NOT sit adjacent to each other on the day
(There are two questions based on Paragraph, the question of the examination. Then, the probability of the event
given below is one of them) (2018 Adv.) T1 ∩ T2 ∩ T3 ∩ T4 is
1 1 7 1
16. The probability that, on the examination day, the (a) (b) (c) (d)
student S1 gets the previously allotted seat R1, and 15 10 60 5

Topic 3 Independent and Conditional Probability


Objective Questions I (Only one correct option)
(a) E1 and E2 are independent
1. Assume that each born child is equally likely to be a boy
or a girl. If two families have two children each, then the (b) E2 and E3 are independent
conditional probability that all children are girls given (c) E1 and E3 are independent
that at least two are girls; is (2019 Main, 10 April I) (d) E1 , E2 and E3 are independent
1 1 1 1 1
(a) (b) (c) (d) 7. Let A and B be two events such that P ( A ∪ B) = ,
17 12 10 11 6
2. Four persons can hit a target correctly with 1 1
P ( A ∩ B) =and P ( A ) = , where A stands for the
1 1 1 1 4 4
probabilities , , and respectively. If all hit at the
2 3 4 8 complement of the event A. Then , the events A and B
target independently, then the probability that the are (2014 Main)
target would be hit, is (2019 Main, 9 April I) (a) independent but not equally likely
1 25 7 25 (b) independent and equally likely
(a) (b) (c) (d)
192 32 32 192 (c) mutually exclusive and independent
3. Let A and B be two non-null events such that A ⊂ B. (d) equally likely but not independent
Then, which of the following statements is always
correct. (2019 Main, 8 April I)
8. Four persons independently solve a certain problem
1 3 1 1
(a) P (A /B ) = P (B ) − P (A ) (b) P (A/B ) ≥ P (A ) correctly with probabilities , , , . Then, the
2 4 4 8
(c) P (A/B ) ≤ P (A ) (d) P (A/B ) = 1 probability that the problem is solved correctly by
4. Two integers are selected at random from the set { 1, 2, atleast one of them, is (2013 Adv)
…… , 11}. Given that the sum of selected numbers is 235 21 3 253
(a) (b) (c) (d)
even, the conditional probability that both the numbers 256 256 256 256
are even is (2019 Main, 11 Jan I)
2 1 7 3
9. An experiment has 10 equally likely outcomes. Let A
(a) (b) (c) (d) and B be two non-empty events of the experiment. If A
5 2 10 5 consists of 4 outcomes, then the number of outcomes
5. An unbiased coin is tossed. If the outcome is a head, that B must have, so that A and B are independent, is
then a pair of unbiased dice is rolled and the sum of the (a) 2, 4 or 8 (b) 3, 6 or 9 (2008, 3M)
numbers obtained on them is noted. If the toss of the (c) 4 or 8 (d) 5 or 10
coin results in tail, then a card from a well-shuffled
pack of nine cards numbered 1, 2, 3, …, 9 is randomly 10. Let E c denotes the complement of an event E. If E, F, G
picked and the number on the card is noted. The are pairwise independent events with P (G ) > 0 and
probability that the noted number is either 7 or 8 is P (E ∩ F ∩ G ) = 0 . Then, P (E c ∩ F c|G ) equals(2007, 3M)
(2019 Main, 10 Jan I) (a) P (E c ) + P (F c ) (b) P (E c ) − P (F c )
15 13 19 19 (c) P (E c ) − P (F ) (d) P (E ) − P (F c )
(a) (b) (c) (d)
72 36 72 36 11. One Indian and four American men and their wives are
6. Let two fair six-faced dice A and B be thrown to be seated randomly around a circular table. Then, the
simultaneously. If E1 is the event that die A shows up conditional probability that Indian man is seated
four, E 2 is the event that die B shows up two and E3 is adjacent to his wife given that each American man is
the event that the sum of numbers on both dice is odd, seated adjacent to his wife, is (2007, 3M)
then which of the following statements is not true? 1 1 2 1
(a) (b) (c) (d)
(2016 Main) 2 3 5 5
rankershalt.com

Probability 101

12. A fair die is rolled. The probability that the first time Objective Questions II
1 occurs at the even throw, is (2005, 1M)
(a) 1/6 (b) 5/11 (c) 6/11 (d) 5/36
(One or more than one correct option)
1 1
13. There are four machines and it is known that exactly 21. Let X andY be two events such that P (X ) = , P (X /Y ) =
two of them are faulty. They are tested, one by one, in 3 2
2
a random order till both the faulty machines are and P (Y /X ) = . Then
identified. Then, the probability that only two tests 5 (2017 Adv.)
4 1
are needed, is (1998, 2M) (a) P (Y ) = (b) P (X ′/Y ) =
1 1 1 1 15 2
(a) (b) (c) (d) 2 1
3 6 2 4 (c) P (X ∪Y ) = (d) P (X ∩ Y ) =
5 5
14. A fair coin is tossed repeatedly. If tail appears on first
four tosses, then the probability of head appearing on 22. If X and Y are two events such that
1 1 1
fifth toss equals (1998, 2M) P (X / Y ) = , P (Y /X ) = and P (X ∩ Y ) . Then, which of
1 1 31 1 2 3 6
(a) (b) (c) (d) the following is/are correct? (2012)
2 32 32 5
(a) P (X ∪ Y ) = 2/3
15. If from each of the three boxes containing 3 white and (b) X and Y are independent
1 black, 2 white and 2 black, 1 white and 3 black balls,
(c) X and Y are not independent
one ball is drawn at random, then the probability that
(d) P (X c ∩ Y ) = 1/3
2 white and 1 black balls will be drawn, is
(1998, 2M) 23. Let E and F be two independent events. The probability
13 1 1 3 11
(a) (b) (c) (d) that exactly one of them occurs is and the probability of
32 4 32 16 25
2
16. The probability of India winning a test match against none of them occurring is . If P (T ) denotes the
25
West Indies is 1/2. Assuming independence from
probability of occurrence of the event T, then (2011)
match to match the probability that in a 5 match 4 3 1 2
series India’s second win occurs at third test, is (a) P (E ) = , P (F ) = (b) P (E ) = , P (F ) =
(1995, 2M)
5 5 5 5
2 1 3 4
(a) 1/8 (b) 1/4 (c) 1/2 (d) 2/3 (c) P (E ) = , P (F ) = (d) P (E ) = , P (F ) =
5 5 5 5
17. An unbiased die with faces marked 1, 2, 3, 4, 5 and 6 is
24. The probabilities that a student passes in Mathematics,
rolled four times. Out of four face values obtained, the
Physics and Chemistry are m, p and c, respectively. Of
probability that the minimum face value is not less
these subjects, the students has a 75% chance of passing
than 2 and the maximum face value is not greater
in atleast one, a 50% chance of passing in atleast two and
than 5, is
a 40% chance of passing in exactly two. Which of the
(a) 16/81 (b) 1/81 (1993, 1M)
following relations are true? (1999, 3M) (2011)
(c) 80/81 (d) 65/81 19 27
(a) p + m + c = (b) p + m + c =
18. A student appears for tests I, II and III. The student is 20 20
successful if he passes either in tests I and II or tests I 1 1
(c) pmc = (d) pmc =
and III. The probabilities of the student passing in 10 4
1
tests I, II and III are p, q and , respectively. If the 25. If E and F are the complementary events of E and F
2
1 respectively and if 0 < P (F ) < 1, then (1998, 2M)
probability that the student is successful, is , then
2 (a) P (E / F ) + P (E / F ) = 1 (b) P (E / F ) + P (E / F ) = 1
1 (c) P (E / F ) + P (E / F ) = 1 (d) P (E / F ) + P (E / F ) = 1
(a) p = q = 1 (b) p = q = (1986, 2M)
2 26. Let E and F be two independent events. If the probability
1 that both E and F happen is 1/12 and the probability that
(c) p = 1, q = 0 (d) p = 1, q =
2 neither E nor F happen is 1/2. Then,
19. If A and B are two independent events such that (a) P (E ) = 1 / 3, P (F ) = 1 / 4 (1993, 2M)
P ( A ) > 0, and P (B) ≠ 1, then P ( A / B ) is equal to (b) P (E ) = 1 / 2, P (F ) = 1 / 6
(a) 1 − P (A / B ) (b) 1 − P (A / B ) (1982, 2M) (c) P (E ) = 1 / 6, P (F ) = 1 / 2
(d) P (E ) = 1 / 4, P (F ) = 1 / 3
1 − P (A ∪ B ) P (A )
(c) (d)
P (B ) P (B ) 27. For any two events A and B in a sample space
(1991, 2M)
20. The probability that an event A happens in one trial of P (A ) + P (B ) − 1
(a) P   ≥
A
an experiment, is 0.4. Three independent trials of the , P (B ) ≠ 0 is always true
 B P (B )
experiments are performed. The probability that the
(b) P (A ∩ B ) = P (A ) − P (A ∩ B ) does not hold
event A happens atleast once, is (1980, 1M)
(c) P (A ∪ B ) = 1 − P (A )P (B ), if A and B are independent
(a) 0.936 (b) 0.784
(d) P (A ∪ B ) = 1 − P (A )P (B ), if A and B are disjoint
(c) 0.904 (d) None of these
rankershalt.com

102 Probability

28. If E and F are independent events such that 0 < P (E ) < 1 result is a tail, a card from a well-shuffled pack of
and 0 < P (F ) < 1, then (1989, 2M) eleven cards numbered 2, 3, 4, …, 12 is picked and the
number on the card is noted. What is the probability
(a) E and F are mutually exclusive
that the noted number is either 7 or 8? (1994, 5M)
(b) E and F c (the complement of the event F) are
independent 39. A lot contains 50 defective and 50 non-defective bulbs.
(c) E c and F c are independent Two bulbs are drawn at random, one at a time, with
(d) P (E / F ) + P (E c / F ) = 1 replacement. The events A, B, C are defined as :
A = ( the first bulb is defective)
Fill in the Blanks B = (the second bulb is non-defective)
29. If two events A and B are such that P ( A c ) = 0.3, P (B) = 0.4 C = (the two bulbs are both defective or both
and P ( A ∩ Bc ) = 0.5, then P [B / ( A ∪ Bc )] = K . (1994, 2M) non-defective).
Determine whether
30. Let A and B be two events such that P ( A ) = 0.3 and
(i) A, B, C are pairwise independent.
P ( A ∪ B) = 0.8. If A and B are independent events, then
P (B) = … . (1990, 2M)
(ii) A, B, C are independent. (1992, 6M)

31. A pair of fair dice is rolled together till a sum of either 5 or 40. In a multiple-choice question there are four
7 is obtained. Then, the probability that 5 comes before 7, alternative answers, of which one or more are correct.
is… . (1989, 2M)
A candidate will get marks in the question only if he
ticks the correct answers. The candidates decide to
32. Urn A contains 6 red and 4 black balls and urn B contains tick the answers at random, if he is allowed upto three
4 red and 6 black balls. One ball is drawn at random from chances to answer the questions, find the probability
urn A and placed in urn B. Then, one ball is drawn at that he will get marks in the question. (1985, 5M)
random from urn B and placed in urn A. If one ball is
drawn at random from urn A, the probability that it is 41. A and B are two independent events. The probability
1
found to be red, is…. (1988, 2M) that both A and B occur is and the probability that
6
33. A box contains 100 tickets numbered 1, 2, …,100. Two 1
neither of them occurs is . Find the probability of the
tickets are chosen at random. It is given that the 3
maximum number on the two chosen tickets is not more occurrence of A. (1984, 2M)
than 10. The maximum number on them is 5 with
probability… . (1985, 2M) 42. Cards are drawn one by one at random from a well
shuffled full pack of 52 playing cards until 2 aces are
obtained for the first time. If N is the number of cards
Analytical and Descriptive Questions required to be drawn, then show that
34. If A and B are two independent events, prove that (n − 1)(52 − n )(51 − n )
Pr { N = n } =
P ( A ∪ B) ⋅ P ( A′ ∩ B ′ ) ≤ P (C ), where C is an event 50 × 49 × 17 × 13
defined that exactly one of A and B occurs. (2004, 2M)
where, 2 < n ≤ 50. (1983, 3M)
35. A is targeting to B, B and C are targeting to A. 43. An anti-aircraft gun can take a maximum of four shots
2 1
Probability of hitting the target by A, B and C are , and at an enemy plane moving away from it. The
3 2 probabilities of hitting the plane at the first, second,
1
, respectively. If A is hit, then find the probability that B third and fourth shot are 0.4, 0.3, 0.2, and 0.1,
3 respectively. What is the probability that the gun hits
hits the target and C does not. (2003, 2M) the plane? (1981, 2M)
36. For a student to qualify, he must pass atleast two out of 44. A box contanis 2 black, 4 white and 3 red balls. One
three exams. The probability that he will pass the 1st ball is drawn at random from the box and kept aside.
exam is p. If he fails in one of the exams, then the From the remaining balls in the box, another ball is
p
probability of his passing in the next exam, is drawn at random and kept beside the first.
2 This process is repeated till all the balls are
otherwise it remains the same. Find the probability that drawn from the box. Find the probability that the balls
he will qualify. (2003, 2M) drawn are in the sequence of 2 black, 4 white and
37. A coin has probability p of showing head when tossed. It is 3 red. (1979, 2M)
tossed n times. Let pn denotes the probability that no two
(or more) consecutive heads occur. Prove that p1 = 1, Integer Answer Type Question
p2 = 1 − p2 and pn = (1 − p). pn − 1 + p(1 − p) pn − 2 , ∀ n ≥ 3.
45. Of the three independent events E1 , E 2 and E3 , the
(2000, 5M)
probability that only E1 occurs is α , only E 2 occurs is β
38. An unbiased coin is tossed. If the result in a head, a pair
of unbiased dice is rolled and the number obtained by and only E3 occurs is γ. Let the probability p that none
adding the numbers on the two faces is noted. If the of events E1 , E 2 or E3 occurs satisfy the equations
rankershalt.com

Probability 103

(α − 2 β ), p = αβ and (β − 3γ ) p = 2 βγ. All the given 1 1 1


losing a game against T2 are , and , respectively. Each
probabilities are assumed to lie in the interval (0, 1). 2 6 3
probability of occurrence of E1 team gets 3 points for a win, 1 point for a draw and 0 point for
Then, is equal to a loss in a game. Let X andY denote the total points scored by
probability of occurrence of E3
teams T1 and T2, respectively, after two games. (2016 Adv.)

Passage Type Questions 46. P (X > Y ) is


1 5 1 7
(a) (b) (c) (d)
Passage 4 12 2 12
Football teams T1 and T2 have to play two games against each 47. P (X = Y ) is
other. It is assumed that the outcomes of the two games are 11 1 13 1
independent. The probabilities of T1 winning, drawing and (a) (b) (c) (d)
36 3 36 2

Topic 4 Law of Total Probability and Baye’s Theorem


Objective Question I (Only one correct option) (a)
3
(b)
6
(c)
20
(d)
9
5 7 23 20
1. A pot contain 5 red and 2 green balls. At random a ball
is drawn from this pot. If a drawn ball is green then put
a red ball in the pot and if a drawn ball is red, then put a
Objective Question II
green ball in the pot, while drawn ball is not replace in (One or more than one correct option)
the pot. Now we draw another ball randomnly, the 5. A ship is fitted with three engines E1 , E 2 and E3 . The
probability of second ball to be red is (2019 Main, 9 Jan II)
27 26 21 32 engines function independently of each other with
(a) (b) (c) (d) respective probabilities 1/2, 1/4 and 1/4. For the ship to
49 49 49 49
be operational atleast two of its engines must function.
2. A bag contains 4 red and 6 black balls. A ball is drawn at Let X denotes the event that the ship is operational and
random from the bag, its colour is observed and this ball let X1, X 2 and X3 denote, respectively the events that the
along with two additional balls of the same colour are engines E1, E 2 and E3 are functioning.
returned to the bag. If now a ball is drawn at random
Which of the following is/are true? (2012)
from the bag, then the probability that this drawn ball
is red, is (2018 Main) (a) P [X1c| X ] = 3 / 16
3 2 1 3 7
(a) (b) (c) (d) (b) P [exactly two engines of the ship are functioning] =
10 5 5 4 8
5
3. A computer producing factory has only two plants T1 (c) P [X | X 2 ] =
16
and T2. Plant T1 produces 20% and plant T2 produces 7
80% of the total computers produced. 7% of computers (d) P [X | X1 ] =
16
produced in the factory turn out to be defective. It is
known that P(computer turns out to be defective, given
that it is produced in plant T1) = 10P (computer turns Assertion and Reason
out to be defective, given that it is produced in plant T2), For the following questions, choose the correct answer
where P (E ) denotes the probability of an event E. A from the codes (a), (b), (c) and (d) defined as follows.
computer produced in the factory is randomly selected (a) Statement I is true, Statement II is also true;
and it does not turn out to be defective. Then, the Statement II is the correct explanation of Statement I
probability that it is produced in plant T2, is (2016 Adv.) (b) Statement I is true, Statement II is also true;
36 47 Statement II is not the correct explanation of
(a) (b)
73 79 Statement I
78 75 (c) Statement I is true; Statement II is false
(c) (d)
93 83 (d) Statement I is false; Statement II is true
4 6. Let H 1 , H 2,... , H n be mutually exclusive events with
4. A signal which can be green or red with probability
5 P (H i ) > 0, i = 1, 2,... , n . Let E be any other event with
1 0 < P (E ) < 1.
and respectively, is received by station A and then
5
Statement I P (H i/E ) > P (E/H i ) ⋅ P (H i ) for
transmitted to station B. The probability of each station
3 i = 1, 2, . . . , n
receiving the signal correctly is . If the signal received n
4
at station B is green, then the probability that the
Statement II ∑ P (Hi ) = 1 (2007, 3M)
i =1
original signal green is (2010)
rankershalt.com

104 Probability

Passage Based Problems 14. If P (ui ) ∝ i, where i = 1, 2, 3,... , n , then lim P (W ) is


n→ ∞
equal to
Passage I (a) 1 (b)
2
Let n1 and n2 be the number of red and black balls, respectively 3
in box I. Let n3 and n4 be the number of red and black balls, 1 3
(c) (d)
respectively in box II. (2015 Adv.) 4 4
7. One of the two boxes, box I and box II was selected at 15. If P (ui ) = c , where c is a constant, then P (un / W ) is
random and a ball was drawn randomly out of this box. equal to
The ball was found to be red. If the probability that this 2 1 n 1
(a) (b) (c) (d)
1
red ball was drawn from box II, is , then the correct n+1 n+1 n+1 2
3
16. If n is even and E denotes the event of choosing even
option(s) with the possible values of n1 , n2, n3 and n4
 1
is/are numbered urn P (ui )= , then the value of P ( W /E ) is
 n 
(a) n1 = 3, n2 = 3, n3 = 5, n4 = 15
(a) n + 2 n+ 2 n 1
(b) n1 = 3, n2 = 6, n3 = 10, n4 = 50 (b) (c) (d)
2n + 1 2 (n + 1) n+1 n+1
(c) n1 = 8, n2 = 6, n3 = 5, n4 = 20
(d) n1 = 6, n2 = 12, n3 = 5, n4 = 20
Analytical and Descriptive Questions
8. A ball is drawn at random from box I and transferred to
box II. If the probability of drawing a red ball from box I,
17. A person goes to office either by car, scooter, bus or
1 3 2 1
1 train probability of which being , , and ,
after this transfer, is , then the correct option(s) with 7 7 7 7
3
respectively. Probability that he reaches offices late, if
the possible values of n1 and n2 is/are
2 1 4 1
(a) n1 = 4 and n2 = 6 (b) n1 = 2 and n2 = 3 he takes car, scooter, bus or train is , , and ,
(c) n1 = 10 and n2 = 20 (d) n1 = 3 and n2 = 6
9 9 9 9
respectively. Given that he reached office in time,
Passage II then what is the probability that he travelled by a car ?
LetU 1 andU 2 be two urns such thatU 1 contains 3 white and 2 (2005, 2M)
red balls andU 2 contains only 1 white ball. A fair coin is tossed. 18. A bag contains 12 red balls and 6 white balls. Six balls
If head appears then 1 ball is drawn at random from U 1 and are drawn one by one without replacement of which at
put intoU 2. However, if tail appears then 2 balls are drawn at least 4 balls are white. Find the probability that in the
random from U 1 and put into U 2. Now, 1 ball is drawn at next two drawn exactly one white ball is drawn. (Leave
random from U 2. (2011) the answer in nC r). (2004, 4M)
9. The probability of the drawn ball fromU 2 being white, is 19. A box contains N coins, m of which are fair and the rest
13 23 19 11 are biased. The probability of getting a head when a fair
(a) (b) (c) (d)
30 30 30 30 coin is tossed, is 1/2, while it is 2/3 when a biased coin is
10. Given that the drawn ball from U 2 is white, the tossed. A coin is drawn from the box at random and is
tossed twice. The first time it shows head and the second
probability that head appeared on the coin is
17 11 15 12 time it shows tail. What is the probability that the coin
(a) (b) (c) (d) drawn is fair? (2002, 5M)
23 23 23 23
20. An urn contains m white and n black balls. A ball is
Passage III drawn at random and is put back into the urn along
A fair die is tossed repeatedly until a six is obtained. Let X with k additional balls of the same colour as that of the
denote the number of tosses required. (2009) ball drawn. A ball is again drawn at random.
11. The probability that X = 3 equals What is the probability that the ball drawn now is
25 25 5 125 white? (2001, 5M)
(a) (b) (c) (d)
216 36 36 216 21. Eight players P1 , P2, K , P8 play a knock-out tournament.
12. The probability that X ≥ 3 equals It is known that whenever the players Pi and Pj play,
125 25 5 25
(a) (b) (c) (d) the player Pi will win if i < j. Assuming that the players
216 36 36 216 are paired at random in each round, what is the
13. The conditional probability that X ≥ 6 given X > 3 equals probability that the player P4 reaches the final?
125 25 5 25 (1999, 10M)
(a) (b) (c) (d)
216 216 36 36 22. Three players, A, B and C, toss a coin cyclically in that
Passage IV order (i.e. A, B, C, A, B, C, A, B, …) till a head shows.
There are n urns each containing (n + 1) balls such that the ith Let p be the probability that the coin shows a head. Let
urn contains ‘i’white balls and (n + 1 − i) red balls. Let ui be the α, β and γ be, respectively, the probabilities that A, B
event of selecting ith urn, i = 1, 2, 3, ... , n and W denotes the and C gets the first head. Prove that β = (1 − p) α .
event of getting a white balls. (2006, 5M)
Determine α, β and γ (in terms of p). (1998, 8M)
rankershalt.com

Probability 105

23. Sixteen players S1 , S 2, K , S16 play in a tournament. 1


copied it, is . Find the probability that he knew the
They are divided into eight pairs at random from each 8
pair a winner is decided on the basis of a game played answer to the question given that he correctly answered
between the two players of the pair. Assume that all the it. (1991, 4M)
players are of equal strength. 25. An urn contains 2 white and 2 blacks balls. A ball is
(i) Find the probability that the player S1 is among the drawn at random. If it is white it is not replaced into the
eight winners. urn. Otherwise it is replaced along with another ball of
(ii) Find the probability that exactly one of the two players the same colour. The process is repeated. Find the
S1 and S2 is among the eight winners. (1997C, 5M) probability that the third ball drawn is black. (1987, 4M)
24. In a test an examinee either guesses or copies of knows 26. A lot contains 20 articles. The probability that the lot
the answer to a multiple choice question with four contains exactly 2 defective articles is 0.4 and the
1
choices. The probability that he make a guess is and probability that the lot contains exactly 3 defective
3 articles is 0.6. Articles are drawn from the lot at random
1
the probability that he copies the answer is . The one by one without replacement and are tested till all
6 defective articles are found. What is the probability that
probability that his answer is correct given that he the testing procedure ends at the twelfth testing?
(1986, 5M)

Topic 5 Probability Distribution and Binomial Distribution


Objective Questions I (Only one correct option) probability of hitting the target at least once is greater
5
1. For an initial screening of an admission test, a than , is
6 (2019 Main, 10 Jan II)
candidate is given fifty problems to solve. If the
probability that the candidate can solve any problem is (a) 6 (b) 3 (c) 5 (d) 4
4 7. Two cards are drawn successively with replacement
, then the probability that he is unable to solve less
5 from a well shuffled deck of 52 cards. Let X denote the
than two problem is (2019 Main, 12 April II) random variable of number of aces obtained in the two
201  1 
49
316  4 
48
54  4 
49
164  1 
48 drawn cards. Then, P (X = 1) + P (X = 2) equals
(a)   (b)   (c)   (d)   (2019 Main, 9 Jan I)
5  5 25  5  5  5 25  5  25 52 49 24
(a) (b) (c) (d)
2. Let a random variable X have a binomial distribution 169 169 169 169
k
with mean 8 and variance 4. If P (X ≤ 2) = , then k is 8. A box contains 15 green and 10 yellow balls. If 10 balls
216 are randomly drawn one-by-one with replacement, then
equal to (2019 Main, 12 April I) the variance of the number of green balls drawn is
(a) 17 (b) 121 (c) 1 (d) 137 (2017 Main)
12 6
3. Minimum number of times a fair coin must be tossed so (a) (b) 6 (c) 4 (d)
5 25
that the probability of getting atleast one head is more
than 99% is (2019 Main 10 April II) 9. A multiple choice examination has 5 questions. Each
(a) 8 (b) 6 (c) 7 (d) 5
question has three alternative answers of which exactly
one is correct. The probability that a student will get 4
4. The minimum number of times one has to toss a fair or more correct answers just by guessing is (2013 Main)
coin so that the probability of observing atleast one head 17 13 11 10
(a) (b) (c) (d)
is atleast 90% is (2019 Main, 8 April II) 35 35 35 35
(a) 2 (b) 3 (c) 5 (d) 4
10. India plays two matches each with West Indies and
5. In a game, a man wins ` 100 if he gets 5 or 6 on a throw Australia. In any match the probabilities of India
of a fair die and loses ` 50 for getting any other number getting points 0, 1 and 2 are 0.45, 0.05 and 0.50,
on the die. If he decides to throw the die either till he respectively. Assuming that the outcomes are
gets a five or a six or to a maximum of three throws, independent. The probability of India getting at least 7
then his expected gain/loss (in rupees) is points, is (1992, 2M)
(2019 Main, 12 Jan II) (a) 0.8750 (b) 0.0875 (c) 0.0625 (d) 0.0250
400 400 400
(a) loss (b) loss (c) 0 (d) gain 11. One hundred identical coins, each with probability p, of
3 9 3 showing up heads are tossed once. If 0 < p < 1 and the
6. If the probability of hitting a target by a shooter in any probability of heads showing on 50 coins is equal to that
1 of heads showing on 51 coins, then the value of p is
shot, is , then the minimum number of independent
3 (1988, 2M)
shots at the target required by him so that the (a) 1/2 (b) 49/101 (c) 50/101 (d) 51/101
rankershalt.com

106 Probability

Fill in the Blanks 15. A is a set containing n elements. A subset P of A is


chosen at random. The set A is reconstructed by
12. If the mean and the variance of a binomial variate X are replacing the elements of P. A subset Q of A is again
2 and 1 respectively, then the probability that X takes a chosen at random. Find the probability that P and Q
value greater than one is equal to… . (1991, 2M) have no common elements. (1991, 4M)
13. For a biased die the probabilities for the different faces 16. Suppose the probability for A to win a game against B is
to turn up are given below 0.4. If A has an option of playing either a ‘best of 3
Face 1 2 3 4 5 6 games’ or a ‘best of 5 games’’ match against B, which
0.1 0.32 0.21 0.15 0.05 0.17
option should choose so that the probability of
Probability
his winning the match is higher? (no game ends in a
This die is tossed and you are told that either face 1 or draw). (1989, 5M)
face 2 has turned up. Then, the probability that it is face 17. A man takes a step forward with probability 0.4 and
1, is… . (1981, 2M)
backwards with probability 0.6. Find the probability
that at the end of eleven steps he is one step away from
Analytical & Descriptive Questions the starting point. (1987, 3M)
14. Numbers are selected at random, one at a time, from the
two-digit numbers 00, 01, 02, …, 99 with replacement. Integer Type Question
An event E occurs if and only if the product of the two
digits of a selected number is 18. If four numbers are 18. The minimum number of times a fair coin needs to be
selected, find probability that the event E occurs at least tossed, so that the probability of getting atleast two
3 times. (1993, 5M) heads is atleast 0.96, is (2015 Adv.)

Answers
Topic 1 39. (i) A, B and C are pairwise independent 40.
1
1. (c) 2. (a) 3. (a) 4. (d) 5
5. (a) 6. (c) 7. (a) 8. (a) 1 1 1
41. or 43. 0.6976 44. 45. 6
9. (a) 10. (c) 11. (d) 12. (d) 3 2 1260
13. (a) 14. (b) 15. (c) 16. (b) 46. (b) 47. (c)
17. (d) 18. (b) 19. (b) 20. (c)
1 1 1 3 Topic 4
21. 22. ≤ p ≤ 23. 24. False
36 3 2 16 1. (d) 2. (b) 3. (c) 4. (c)
(3n − 3. 2n + 3 ) × 6C 3 1 5. (b, d) 6. (d) 7. (b) 8. (d)
25. 26. 0.62 27.
6n 91 9. (b) 10. (d) 11. (a) 12. (b)`
10 (n + 2 ) 1 1
28. 1 − n + 7 29. (i) (ii) 13. (d) 14. (b) 15. (a) 16. (b)
C5 132 462 1
17.
Topic 2 7
1. (b) 2. (a) 3. (c) 4. (a)
12
C 2 ⋅6 C 4 10C1 ⋅2 C1 12C1 ⋅6 C 5 11C1 ⋅1 C1 9m
18. ⋅ 12 + 18 ⋅ 12 19.
5. (c) 6. (c) 7. (a) 8. (a, b, c) 18
C6 C2 C6 C2 8N + m
11 m 4
9. (a, c) 10. 11. P ( A ∩ B ) 12. False 20. 21.
40 m+n 35
13. 13.9% 15. No 16. (a) 17. (c) p p (1 − p ) p − 2p 2 + p 3
22. α = ,β = ,γ =
Topic 3 1 − (1 − p ) 3
1 − (1 − p ) 3
1(1 − p ) 3
1. (d) 2. (b) 3. (b) 4. (a) 1 8 24 23 99
5. (c) 6. (d) 7. (a) 8. (a) 23. (i) (ii) 24. 25. 26.
2 15 29 30 1900
9. (d) 10. (c) 11. (c) 12. (b)
13. (b) 14. (a) 15. (a) 16. (b)
Topic 5
1. (c) 2. (d) 3. (c) 4. (d)
17. (a) 18. (c) 19. (b) 20. (b)
5. (c) 6. (c) 7. (a) 8. (a)
21. (a, b) 22. (a,b) 23. (a, d) 24. (b, c) 11
9. (c) 10. (b) 11. (d) 12.
25. (a, d) 26. (a, d) 27. (a, c) 28. (b, c, d) 16
1 5 2 32 n
29. 30. 31. 32. 5 97  3
4 7 5 55 13. 14. 15.  
21 25 4  4
1 1 193
33. 35. 36. 2 p 2 – p 3 38. 16. Best of 3 games 17. 11
C 6( 0 .24 ) 5 18. (8)
9 2 792
rankershalt.com

Hints & Solutions


Topic 1 Classical Probability S − { 1, 2, 4 }
1. It is given that a person wins ∴Number of favourables cases = 5
5
`15 for throwing a doublet (1, 1) (2, 2), (3, 3), Hence, required probability = 20
(4, 4), (5, 5), (6, 6) and win `12 when the throw results in 2
sum of 9, i.e., when (3, 6), (4, 5), 5. Total number of ways of selecting 2 different numbers
(5, 4), (6, 3) occurs. from {0, 1, 2, ..., 10} = 11C 2 = 55
Also, losses `6 for throwing any other outcome, i.e., Let two numbers selected be x and y.
when any of the rest 36 − 6 − 4 = 26 outcomes occurs. Then, x + y = 4m …(i)
Now, the expected gain/loss and x − y = 4n …(ii)
= 15 × P (getting a doublet) + 12 × P (getting sum 9) ⇒ 2x = 4(m + n ) and 2 y = 4(m − n )
− 6 × P (getting any of rest 26 outcome) ⇒ x = 2(m + n )and y = 2(m − n )
 6  4  26 ∴x and y both are even numbers.
= 15 ×  + 12 ×  − 6 × 
 36  36  36
x y
5 4 26 15 + 8 − 26
= + − = 0 4, 8
2 3 6 6
23 − 26 3 1 1 2 6, 10
= = − = − , means loss of `
6 6 2 2 4 0, 8
6 2, 10
2. Since, the experiment should be end in the fifth throw of
the die, so total number of outcomes are 65 . 8 0, 4
Now, as the last two throws should be result in two fours 10 2, 6
4 4 6
∴Required probability =
(i) (ii) (iii) (iv) (v) 55
So, the third throw can be 1, 2, 3, 5 or 6 (not 4). Also, 6. Sample space → 12C 2
throw number (i) and (ii) can not take two fours in Number of possibilities for z is even.
succession, therefore number of possibililites for throw z = 0 ⇒ 11C1
(i) and (ii) = 62 − 1 = 35 z = 2 ⇒ 9C1
[Q when a pair of dice is thrown
z = 4 ⇒ 7C1
then (4, 4) occur only once].
5 × 35 175 z = 6 ⇒ 5C1
Hence, the required probability = = 5 z = 8 ⇒ 3C1
65 6
z = 10 ⇒ 1C1
3. Since, there is a regular hexagon, then the number of Total = 36
ways of choosing three vertices is 6C3 . And, there is only 36 6
two ways i.e. choosing vertices of a regular hexagon ∴ Probability = =
66 11
alternate, here A1, A3 , A5 or A2, A4, A6 will result in an
equilateral triangle. 7. We have mentioned that boxes are different and one
particular box has 3 balls.
A1
11
A6 C3 × 29 55  2
12
A2 Then, number of ways = =  
312 3  3
A5 A3
8. Total number of ways to arrange 3 boys and 2 girls are
A4 5!.
∴Required probability According to given condition, following cases may arise.
2 2 2 ×3 ×2 ×3 ×2 1 B G G B B
= 6 = = =
C3 6 ! 6 × 5 × 4 × 3 × 2 × 1 10 G G B B B
3 !3 ! G B G B B
4. Number of subset of S = 220 G B B G B
20(21) B G B G B
Sum of elements in S is 1 + 2 + .....+20 = = 210 So, number of favourable ways = 5 × 3 ! × 2 ! = 60
2
 n (n + 1)  60 1
Q 1 + 2+ ...... + n = ∴ Required probability = =
 2  120 2
Clearly, the sum of elements of a subset would be 203, if 9. PLAN As one of the dice shows a number appearing on one of P1, P2
we consider it as follows and P3.
S − { 7}, S − {1, 6} S − {2, 5}, S − {3, 4}
rankershalt.com

108 Probability

Thus, three cases arise. 13. 71 = 7, 72 = 49, 73 = 343, 74 = 2401, …


(i) All show same number.
Therefore, for 7r, r ∈ N the number ends at unit place
(ii) Number appearing on D 4 appears on any one of
D1, D 2 and D 3. 7, 9, 3, 1, 7, …
(iii) Number appearing on D 4 appears on any ∴ 7m + 7n will be divisible by 5 if it end at 5 or 0.
two of D1, D 2 and D 3. But it cannot end at 5.
Sample space = 6 × 6 × 6 × 6 = 64 favourable events Also for end at 0.
= Case I or Case II or Case III For this m and n should be as follows
Case I First we should select one number for D4 m n
which appears on all i.e. 6C1 × 1.
1 4r 4r − 2
Case II For D4 there are 6C1 ways. Now, it appears 2 4r − 1 4r − 3
on any one of D1 , D2 and D3 i.e. 3 C1 × 1.
3 4r − 2 4r
For other two there are 5 × 5 ways. 4 4r − 3 4r − 1
⇒ 6
C1 × 3C1 × 1 × 5 × 5
For any given value of m, there will be 25 values of n.
Case III For D4 there are 6C1 ways now it appears on Hence, the probability of the required event is
any two of D1 , D2 and D3
100 × 25 1
⇒ 3
C 2 × 12 =
100 × 100 4
For other one there are 5 ways.
NOTE Power of prime numbers have cyclic numbers in their unit
⇒ 6
C1 × 3C 2 × 12 × 5 place.
6
C1 + 6C1 × 3C1 × 52 + 6C1 × 3C 2 × 5
Thus, probability = 14. The number of ways of placing 3 black balls without any
64
restriction is 10C3 . Since, we have total 10 places of
6 (1 + 75 + 15)
= putting 10 balls in a row. Now, the number of ways in
64 which no two black balls put together is equal to the
91 number of ways of choosing 3 places marked ‘—’ out of
= eight places.
216
10. Sample space A dice is thrown thrice, n (s) = 6 × 6 × 6. —W—W—W—W —W—W—W—
r r This can be done in 8C3 ways.
Favorable events ω r 1 + ω 2 + ω 3 = 0
8
C3 8 × 7 ×6 7
i.e. (r1 , r2, r3 ) are ordered 3 triples which can take ∴ Required probability = = =
values,
10
C3 10 × 9 × 8 15
(1, 2 , 3), (1, 5, 3), (4, 2 , 3), (4, 5, 3)
 i.e. 8 ordered pairs 15. Three vertices out of 6 can be chosen in 6C3 ways.
(1, 2 , 6), (1, 5, 6), (4, 2 , 6), (4, 5, 6)
So, total ways = 6C3 = 20
and each can be arranged in 3 ! ways = 6
8 ×6 2 Only two equilateral D
∴ n (E ) = 8 × 6 ⇒ P (E ) = = triangles can be formed
6 ×6 ×6 9
∆AEC and ∆BFD. E C
11. Since, three distinct numbers are to be selected from
first 100 natural numbers. ∴ Favourable ways = 2
⇒ n (S ) = 100C3 So, required probability
2 1
E(favourable events) = All three of them are divisible by both = = F B
2 and 3 . 20 10
⇒ Divisible by 6 i.e. {6, 12, 18, …, 96} 16. Since, three dice are rolled. A
Thus, out of 16 we have to select 3. ∴ Total number of cases S = 6 × 6 × 6 = 216
∴ n (E ) = 16C3 and the same number appear on each of them = 6C1 = 6
16
C 4 6 1
∴ Required probability = 100 3 = ∴ Required probability = =
C3 1155 216 36

12. Here, two numbers are selected from {1, 2, 3, 4, 5, 6} 17. Since, there are 15 possible cases for selecting a coupon
and seven coupons are selected, the total number of
⇒ n (S ) = 6 × 5 {as one by one without replacement}
cases of selecting seven coupons = 157
Favourable events = the minimum of the two numbers
is less than 4. n (E ) = 6 × 4 {as for the minimum of the It is given that the maximum number on the selected
two is less than 4 we can select one from (1, 2, 3, 4) and coupon is 9, therefore the selection is to be made from
other from (1, 2, 3, 4, 5, 6) the coupons numbered 1 to 9. This can be made in 97
n (E ) 24 4 ways. Out of these 97 cases, 87 does not contain the
∴ Required probability = = =
n (S ) 30 5 number 9.
rankershalt.com

Probability 109

Thus, the favourable number of cases = 97 − 87. 1 + 3p 1 − p 1 − 2p


22. Since, , and are the probability of
97 − 87 3 4 2
∴ Required probability = mutually exclusive events.
157
1 + 3p 1 − p 1 −2p
18. The number of all possible determinants of the form ∴ + + ≤1
3 4 2
a b ⇒ 4 + 12 p + 3 − 3 p + 6 − 12 p ≤ 12
= 24 = 16
c d
⇒ 13 − 3 p ≤ 12
Out of which only 10 determinants given by 1
⇒ p≥ ...(i)
1 1 0 0 1 1 0 0 0 1 1 0 1 0 3
, , , , , , ,
1 1 0 0 0 0 1 1 0 1 1 0 0 0 1 + 3p 1− p 1 −2p
and 0 ≤ ≤ 1, 0 ≤ ≤ 1, 0 ≤ ≤1
0 1 0 0 3 4 2
, ⇒ 0 ≤ 1 + 3 p ≤ 3, 0 ≤ 1 − p ≤ 4, 0 ≤ 1 − 2 p ≤ 2
0 0 1 0
1 2 1 1
Vanish and remaining six determinants have non-zero ⇒ − ≤ p ≤ , 1 ≥ p ≥ −3 , ≥ p ≥ − ...(ii)
6 3 3 3 2 2
values. Hence, the required probability = = From Eqs. (i) and (ii), 1 / 3 ≤ p ≤ 1 / 2
16 8
Statement I is true. 23. Since, determinant is of order 2 × 2 and each element is
Statement II is also true as the homogeneous equations 0 or 1 only.
have always a solution and Statement II is not the
∴ n (S ) = 24 = 16
correct explanation of Statement I.
and the determinant is positive are
Number of favourable outcomes
19. PLAN Probability = 1 0 1 1 1 0
Number of total outcomes , ,
0 1 0 1 1 1
As, x1 + x2 + x3 is odd.
∴ n (E ) = 3
So, all may be odd or one of them is odd and other two
are even. 3
Thus, the required probability =
16
∴ Required probability
8!
2
C1 × 3C1 × 4C1 + 1C1 × 2C1 × 4C1 + 2C1 × 2C1 × 3C1 24. Total number of ways to arrange ‘ASSASSIN’ is .
+ 1C1 × 3C1 × 3C1 4 !⋅ 2 !
= First we fix the position ⊗ A ⊗ A ⊗ I ⊗ N ⊗.
3
C1 × C1 × C1
5 7

24 + 8 + 12 + 9 Number of ways in which no two S’s occur together


= 4! 5
105 = × C4
53 2!
= 4! × 5 × 4! × 2! 1
105 ∴ Required probability = =
2! × 8! 14
20. Since, x1 , x2, x3 are in AP.
Hence, it is a false statement.
∴ x1 + x3 = 2x2
So, x1 + x3 should be even number.
25. Let us define a onto function F from A : [ r1 , r2, K , rn] to
B: [1, 2, 3], where r1 , r2, K , rn are the readings of n
Either both x1 and x3 are odd or both are even.
throws and 1, 2, 3 are the numbers that appear in the n
2
C1 × 4C1 + 1C1 × 3C1 throws.
∴ Required probability = 3
C1 × 5C1 × 7C1 Number of such functions, M = N − [n (1) − n (2) + n (3)]
11 where, N = total number of functions
=
105 and n (t ) = number of function having exactly t
elements in the range.
21. According to given condition,
3 1 Now, N = 3n, n (1) = 3 . 2n, n(2) = 3, n(3) = 0
P ( yellow at the first toss) ==
6 2 ⇒ M = 3n − 3 . 2n + 3
2 1
P (red at the second toss) = = Hence, the total number of favourable cases
6 3
= (3n − 3 . 2n + 3) . 6C3
1
and P (blue at the third toss) = (3n − 3 . 2n + 3) × 6C3
6 ∴ Required probability =
Therefore, the probability of the required event 6n
1 1 1 1
= × × =
26. The required probability = 1 − (probability of the event
2 3 6 36 that the roots of x2 + px + q = 0 are non-real).
rankershalt.com

110 Probability

The roots of x2 + px + q = 0 will be non-real if and only if So, the number of ways of selecting five coins, so
p2 − 4q < 0, i.e. if p2 < 4 q that the total value of the coins is less than one
The possible values of p and q can be possible according rupee and fifty paise is n + 7C5 − 10(n + 2)
n+7
to the following table. C5 − 10(n + 2)
∴ Required probability = n+7
Value of q Value of p Number of pairs of p, q C5
10 (n + 2)
1 1 1 =1 − n+7
C5
2 1, 2 2
29. (i) The total number of arrangements of six boys and
3 1, 2, 3 3
six girls = 12 !
4 1, 2, 3 3 6! × 7! 1
∴ Required probability = =
5 1, 2, 3, 4 4 (12)! 132
6 1, 2, 3, 4 4 [since, we consider six girls at one person]
7 1, 2, 3, 4, 5 5 2 ×6! ×6! 1
(ii) Required probability = =
8 1, 2, 3, 4, 5 5 (12)! 462
9 1, 2, 3, 4, 5 5
Topic 2 Addition and Subtraction Law of
10 1, 2, 3, 4, 5, 6 6
Probability
Therefore, the number of possible pairs = 38 1. We have, P (exactly one of A or B occurs)
Also, the total number of possible pairs is 10 × 10 = 100
= P ( A ∪ B) − P ( A ∩ B)
38
∴ The required probability = 1 − = 1 − 0.38 = 0.62 = P ( A ) + P (B) − 2P ( A ∩ B)
100
According to the question,
27. We have 14 seats in two vans and there are 9 boys and 3 1
P ( A ) + P (B) − 2P ( A ∩ B) = …(i)
girls. The number of ways of arranging 12 people on 14 4
seats without restriction is 1
14 ! P (B) + P (C ) − 2P (B ∩ C ) = …(ii)
14
P12 = = 7(13 !) 4
2! 1
and P (C ) + P ( A ) − 2P (C ∩ A ) = …(iii)
Now, the number of ways of choosing back seats is 2. 4
and the number of ways of arranging 3 girls on adjacent On adding Eqs. (i), (ii) and (iii), we get
seats is 2(3!) and the number of ways of arranging 9 2 [P ( A ) + P (B) + P (C ) − P ( A ∩ B) − P (B ∩ C )
boys on the remaining 11 seats is 11 P9 ways. 3
− P (C ∩ A )] =
Therefore, the required number of ways 4
4 ⋅ 3 ! 11 ! ⇒ P ( A ) + P (B) + P (C ) − P ( A ∩ B) − P (B ∩ C )
= 2. (2 .3 !).11 P9 = = 12 !
2! 3
− P (C ∩ A ) =
Hence, the probability of the required event 8
∴P (atleast one event occurs)
12 ! 1
= = = P(A ∪ B ∪ C )
7 ⋅ 13 ! 91 = P ( A ) + P (B) + P (C ) − P ( A ∩ B) − P (B ∩ C )
28. There are (n + 7) coins in the box out of which five coins − P (C ∩ A ) + P ( A ∩ B ∩ C )
n+7 3 1 7  1
can be taken out in C5 ways. = + = Q P(A ∩ B ∩ C ) =
8 16 16  16 
The total value of 5 coins can be equal to or more than
one rupee and fifty paise in the following ways. 3 1
2. Given, P (B) = , P ( A ∩ B ∩ C ) =
(i) When one 50 paise coin and four 25 paise coins are 4 3
chosen. A B
(A ∩ B ∩ C)
(ii) When two 50 paise coins and three 25 paise coins
(A ∩ B ∩ C)
are chosen.
(iii) When two 50 paise coins, 2 twenty five paise coins (B ∩ C)
and one from n coins of ten and five paise.
∴ The total number of ways of selecting five coins so C
that the total value of the coins is not less than one
rupee and fifty paise is 1
and P(A ∩ B ∩ C ) =
(2C1 ⋅5 C5 ⋅n C 0 ) + (2C 2 ⋅5 C3 ⋅n C 0 ) + (2C 2 ⋅5 C 2 ⋅n C1 ) 3
= 10 + 10 + 10n = 10 (n + 2) which can be shown in Venn diagram.
∴ P (B ∩ C ) = P (B) − { P ( A ∩ B ∩ C + P ( A ∩ B ∩ C ))}
rankershalt.com

Probability 111

3  1 1 3 2 1 ∴ P ( A ∩ B)min. , when P ( A ∪ B)max = 1


= − +  = − =
4  3 3 4 3 12 ⇒ P ( A ∩ B) ≥ P ( A ) + P (B) − 1
3. It is given that, P (E ) ≤ P (F ) ⇒ E ⊆ F …(i) ∴ Option (a) is true.
and P (E ∩ F ) > 0 ⇒ E ⊂ F …(ii) Again, P ( A ∪ B) ≥ 0
(a) occurrence of E ⇒ occurrence of F [from Eq. (i)] ∴ P ( A ∩ B)max. , when P ( A ∪ B)min. = 0
(b) occurrence of F ⇒ occurrence of E [from Eq. (ii)] ⇒ P ( A ∩ B) ≤ P ( A ) + P (B)
(c) non-occurrence of E ⇒ occurrence of F ∴ Option (b) is true.
Hence, option (c) is not correct. [from Eq. (i)] Also, P ( A ∩ B) = P ( A ) + P (B) − P ( A ∪ B), Thus, (c) is
4. We know that, also correct.
P (exactly one of A or B occurs) Hence, (a), (b), (c) are correct options.
= P ( A ) + P (B) − 2P ( A ∩ B) 9. P(exactly one of M, N occurs)
∴ P ( A ) + P (B) − 2P ( A ∩ B) = p …(i) = P{(M ∩ N ) ∪ (M ∩ N )} = P (M ∩ N ) + P (M ∩ N )
Similarly, P (B) + P (C ) − 2P (B ∩ C ) = p …(ii) = P (M ) − P (M ∩ N ) + P (N ) − P (M ∩ N )
and P (C ) + P ( A ) − 2P (C ∩ A ) = p …(iii) = P (M ) + P (N ) − 2P (M ∩ N )
On adding Eqs. (i), (ii) and (iii), we get Also, P(exactly one of them occurs)
2 [P ( A ) + P (B) + P (C ) − P ( A ∩ B) = {1 − P (M ∩ N )}{1 − P (M ∪ N )}
− P (B ∩ C ) − P (C ∩ A )] = 3 p = P (M ∪ N ) − P (M ∩ N ) = P (M ) + P (N ) − 2P (M ∩ N )
⇒ P ( A ) + P (B) + P (C ) − P ( A ∩ B) Hence, (a) and (c) are correct answers.
3p
− P (B ∩ C ) − P (C ∩ A ) = …(v) 10. Let E1 be the event getting minimum number 3 and E 2
2
be the event getting maximum number 7.
It also given that, P ( A ∩ B ∩ C ) = p2 …(v)
∴ P(at least one of the events A, B, and C occurs) Then, P (E1 ) = P (getting one number 3 and other two
= P ( A ) + P (B) + P (C ) − P ( A ∩ B) from numbers 4 to 10)
1
C1 × 7C 2 7
− P (B ∩ C ) − P (C ∩ A ) + P ( A ∩ B ∩ C ) = 10 =
C3 40
3p
= + p2 [from Eqs. (iv) and (v)] P (E 2) = P(getting one number 7 and other two from
2 numbers 1 to 6)
3 p + 2 p2 1
C1 × 6C 2 1
= = 10 =
2 C3 8
5. Since, P ( A ∩ B) = P ( A ) ⋅ P (B) and P (E1 ∩ E 2) = P(getting one number 3, second
number 7 and third from 4 to 6)
It means A and B are independent events, so A ′ and B ′
are also independent.
1
C1 × 1C1 × 3C1 1
= 10
=
∴ P ( A ∪ B) ′ = P ( A ′∩ B ′ ) = P ( A )′ ⋅ P (B)′ C3 40
Alternate Solution ∴ P (E1 ∪ E 2) = P (E1 ) + P (E 2) − P (E1 ∩ E 2)
7 1 1 11
P ( A ∪ B)′ = 1 − P ( A ∪ B) = 1 − { P ( A ) + P (B) − P ( A ) ⋅ P (B)} = + − =
40 8 40 40
= {1 − P ( A )}{1 − P (B)} = P ( A )′ P (B)′
11. P ( A ∪ B) = P ( A ) + P (B) − P ( A ∩ B)
6. Given, P ( A ∪ B) = 0.6 , P ( A ∩ B) = 0.2
If P ( A ∪ B) = P ( A ∩ B),
∴ P ( A ) + P (B ) = [1 − P ( A )] + [1 − P (B)]
then P ( A ) and P (B) are equals.
= 2 − [P ( A ) + P (B)]
Since, P ( A ∪ B) = P ( A ∩ B) ⇒ A and B are equals sets
= 2 − [P ( A ∪ B) + P ( A ∩ B)]
Thus, P ( A ) and P (B) is equal to P ( A ∩ B).
= 2 − [0.6 + 0.2] = 1.2
12. Given, P (A fails in examination) = 0.2
7. Given, P ( A ) = 0.25, P (B) = 0.50, P ( A ∩ B) = 0.14 and P (B fails in examination) = 0.3
∴ P ( A ∪ B) = P ( A ) + P (B) − P ( A ∩ B) P ( A ∩ B) = P ( A )P (B) = (0.2) (0.3)
= 0.25 + 0.50 – 0.14 = 0.61
∴ P ( A ∪ B) = P ( A ) + P (B) − P ( A ∩ B)
Now, P ( A ∪ B) = 1 − P ( A ∪ B) = 1 − 0.61 = 0.39
= 0.2 + 0.3 − 0.06 = 0.44
8. We know that, Hence, it is a false statement.
P ( A ∩ B) = P ( A ) + P (B) − P ( A ∪ B)
13. Let P ( A ) and P (B) denote respectively the percentage of
Also, P ( A ∪ B) ≤ 1 city population that reads newspapers A and B.
rankershalt.com

112 Probability

Then, = 5 ! − [4C1 4 ! 2 ! − (3 C1 3 ! 2 ! + 3C1 3 ! 2 ! 2 !)


25 1 20 1 + (2C1 2 ! 2 ! + 4C1 2 ⋅ 2 !) − 2]
P ( A) = = , P (B) = = ,
100 4 100 5 ⇒ n (T1 ∩ T2 ∩ T3 ∩ T4 )
8 2
P ( A ∩ B) = = , = 120 − [192 − (36 + 72) + (8 + 16) − 2]
100 25
= 120 − [192 − 108 + 24 − 2] = 14
1 2 17
P ( A ∩ B) = P ( A ) − P ( A ∩ B) = − = , 14 7
4 25 100 ∴ Required probability = =
120 60
1 2 3
P ( A ∩ B) = P (B) − P ( A ∩ B) = − =
5 25 25
Let P (C ) be the probability that the population who Topic 3 Independent and Conditional
reads advertisements. Probability
∴ P (C ) = 30% of P ( A ∩ B) + 40% of P ( A ∩ B) 1. Let event B is being boy while event G being girl.
1
+ 50% of P ( A ∩ B) According to the question, P (B) = P (G ) =
2
[since, A ∩ B, A ∩ B and A ∩ B are all mutually
exclusive] Now, required conditional probability that all children
3 17 2 3 1 2 139 are girls given that at least two are girls, is
⇒ P (C ) = × + × + × = = 13 . 9% All 4 girls
10 100 5 25 2 25 1000 =
(All 4 girls ) + (exactly 3 girls + 1 boy)
14. We know that, + (exactly 2 girls + 2 boys)
P ( A ) + P (B) + P (C ) − P ( A ∩ B) − P (B ∩ C )  1
4
 
− P (C ∩ A ) + P ( A ∩ B ∩ C ) = P ( A ∪ B ∪ C )  2 1 1
= = =
⇒ 0.3 + 0.4 + 0.8 – {0.08 + 0.28 + P (BC )} + 0.09  1
4 3
 1  1 4  1  1
2 2
1 + 4 + 6 11
  + C3     + C 2   
4
= P(A ∪ B ∪ C )  2  2  2  2  2
⇒ 1.23 − P (BC ) = P ( A ∪ B ∪ C )
where, 0.75 ≤ P ( A ∪ B ∪ C ) ≤ 1 2. Key Idea Use P ( A) = 1 − P ( A) and condition of independent
events i.e P ( A ∩ B) = P ( A) ⋅ P ( B)
⇒ 0.75 ≤ 1.23 − P (BC ) ≤ 1
⇒ − 0.48 ≤ − P (BC ) ≤ − 0.23 Given that probability of hitting a target independently
⇒ 0.23 ≤ P (BC ) ≤ 0.48 by four persons are respectively
1 1 1 1
15. Given, P ( A ) = 0.5 and P ( A ∩ B) ≤ 0.3 P1 = , P2 = , P3 = and P4 =
2 3 4 8
⇒ P ( A ) + P (B) − P ( A ∪ B) ≤ 0.3 Then, the probability of not hitting the target is
⇒ P (B) ≤ 0.3 + P ( A ∪ B) − P ( A ) ≤ P ( A ∪ B) − 0.2  1  1  1  1
= 1 −  1 −  1 −  1 − 
 2  3  4  8
[since, P ( A ∪ B) ≤ 1 ⇒ P ( A ∪ B) − 0.2 ≤ 0.8 ]
[Q events are independent]
∴ P (B) ≤ 0.8 1 2 3 7 7
= × × × =
⇒ P (B) cannot be 0.9. 2 3 4 8 32
16. Here, five students S1 , S 2, S3 , S 4 and S5 and five seats Therefore, the required probability of hitting the target
= 1 − (Probability of not hitting the target)
R1 , R2, R3 , R4 and R5
7 25
∴Total number of arrangement of sitting five students is =1− =
32 32
5 ! = 120
P ( A ∩ B)
Here, S1 gets previously alloted seat R1 3. We know that, P( A / B) =
∴S 2, S3 , S 4 and S5 not get previously seats. P (B)
Total number of way S 2, S3 , S 4 and S5 not get previously [by the definition of conditional probability]
seats is Q A⊂B
 1 1 1 1  1 1 1 ⇒ A∩B= A
4 ! 1 − + − +  = 24 1 − 1 + − +  P ( A)
 1 ! 2 ! 3 ! 4 !  2 6 24 ∴ P( A / B) = …(i)
 12 − 4 + 1 P (B)
= 24   =9 As we know that, 0 ≤ P (B) ≤ 1
 24 
1 P ( A)
9 3 ∴ 1≤ < ∞ ⇒ P ( A) ≤ <∞
∴ Required probability = = P (B) P (B)
120 40
P ( A)
⇒ ≥ P ( A) …(ii)
17. Here, n (T1 ∩ T2 ∩ T3 ∩ T4 ) P (B)
Total = − n (T1 ∪ T2 ∪ T3 ∪ T4 ) Now, from Eqs (i) and (ii), we get
⇒ n (T1 ∩ T2 ∩ T3 ∩ T4 ) P( A/B) ≥ P(A)
rankershalt.com

Probability 113

4. In {1, 2, 3, ...., 11} there are 5 even numbers and 6 odd 3


= = P (E1 ) ⋅ P (E3 )
numbers. The sum even is possible only when both are 36
odd or both are even. and P (E1 ∩ E 2 ∩ E3 ) = P [getting 4 on die A, 2 on die B
Let A be the event that denotes both numbers are even and sum of numbers is odd]
and B be the event that denotes sum of numbers is even. = P (impossible event) = 0
Then, n ( A ) = 5C 2 and n (B) = 5C 2 + 6C 2 Hence, E1, E 2 and E3 are not independent.
Required probability 1 1 1
P ( A ∩ B) 5
C / 11C 7. Given, P ( A ∪ B) = , P ( A ∩ B) = , P ( A ) =
P ( A / B) = = 6 2 5 2 6 4 4
P (B) ( C 2 + C 2) 1 5
11 ∴ P ( A ∪ B) = 1 − P ( A ∪ B) = 1 − =
C2 6 6
5
C2 10 2 1 3
= 6 = = and P ( A) = 1 − P ( A ) = 1 − =
C 2 + C 2 15 + 10 5
5
4 4
1 ∴ P ( A ∪ B) = P ( A ) + P (B) − P ( A ∩ B)
5. Clearly, P (H ) = Probability of getting head =
2 5 3 1
1 = + P (B) −
and P (T ) = Probability of getting tail = 6 4 4
2 1
⇒ P (B) = ⇒ A and B are not equally likely
Now, let E1 be the event of getting a sum 7 or 8, when a 3
pair of dice is rolled. 1
P ( A ∩ B) = P ( A ) ⋅ P (B) =
Then, E1 = {(6, 1), (5, 2), (4, 3), (3, 4), (2, 5), 4
(1, 6), (6, 2), (5, 3), (4, 4), (3, 5), (2, 6)} So, events are independent.
⇒ P (E1 ) = Probability of getting 7 or 8 when a pair of 8. PLAN It is simple application of independent event, to solve a
11 certain problem or any type of compitition each event in
dice is thrown =
36 independent of other.

Also, let P (E 2) = Probability of getting 7 or 8 when a Formula used


2 P ( A ∩ B) = P ( A ) ⋅ P (B), when A and B are independent
card is picked from cards numbered 1, 2, ...., 9 =
9 events.
∴Probability that the noted number is 7 or 8 Probability that the problem is solved correctly by
= P ((H ∩ E1 ) or (T ∩ E 2)) atleast one of them = 1 − (problem is not solved by all)
= P (H ∩ E1 ) + P (T ∩ E 2) ∴ P (problem is solved) = 1 − P (problem is not solved)
[Q (H ∩ E1 ) and (T ∩ E 2) are mutually exclusive] = 1 − P ( A ) ⋅ P (B ) ⋅ P (C ) ⋅ P (D )
= P (H ) ⋅ P (E1 ) + P (T ) ⋅ P (E 2)
 1 1 3 7 21 235
[Q{ H , E1 } and {T , E 2} both are sets of =1 −  ⋅ ⋅ ⋅  =1 − =
 2 4 4 8 256 256
independent events]
1 11 1 2 19 2
= × + × = 9. Since, P ( A ) =
2 36 2 9 72 5
For independent events,
6. Clearly, E1 = {(4, 1), (4, 2), (4, 3), (4, 4), (4, 5), (4, 6)}
P ( A ∩ B) = P ( A )P (B)
E 2 = {(1, 2), (2, 2), (3, 2), (4, 2), (5, 2), (6, 2)} 2
⇒ P ( A ∩ B) ≤
and E3 = {(1, 2), (1, 4), (1, 6), (2, 1), (2, 3), (2, 5), 5
(3, 2), (3, 4), (3, 6), (4, 1), (4, 3), (4, 5), 1 2 3 4
(5, 2), (5, 4), (5, 6), (6, 1), (6, 3), (6, 5)} ⇒ P ( A ∩ B) = , , ,
10 10 10 10
6 1 6 1
⇒ P (E1 ) = = , P (E 2) = = [maximum 4 outcomes may be in A ∩ B]
36 6 36 6 1
18 1 (i) Now, P ( A ∩ B) =
and P (E3 ) = = 10
36 2 1
Now, P (E1 ∩ E 2) = P (getting 4 on die A and 2 on die B) ⇒ P ( A ) . P (B) =
10
1 1 5 1
= = P (E1 ) ⋅ P (E 2) ⇒ P (B) = × = , not possible
36 10 2 4
P (E 2 ∩ E3 ) = P (getting 2 on die B and sum of numbers (ii) Now, P ( A ∩ B) =
2

2
× P (B) =
2
on both dice is odd) 10 5 10
3 5
= = P (E 2) ⋅ P (E3 ) ⇒ P (B) = , outcomes of B = 5
36 10
P (E1 ∩ E3 ) = P (getting 4 on die A and sum of numbers 3
(iii) Now, P ( A ∩ B) =
on both dice is odd) 10
rankershalt.com

114 Probability

3 2 3 15. P (2 white and 1 black) = P (W1W 2B3 or W1B2W3 or


⇒ P ( A )P (B) = ⇒ × P (B) =
10 5 10
B1W 2W3 )
3
P (B) = , not possible = P (W1W 2B3 ) + P (W1B2W3 ) + P (B1W 2W3 )
4
4 4 = P (W1 )P (W 2)P (B3 ) + P (W1 )P (B2)P (W3 )
(iv) Now, P ( A ∩ B) = ⇒ P ( A ) . p(B) = + P (B1 )P (W 2)P (W3 )
10 10
3.2.3 3.2.1 1.2.1 1 13
⇒ P (B) = 1 , outcomes of B = 10. = + + = (9 + 3 + 1) =
4 4 4 4 4 4 4 4 4 32 32
E ∩F c c
P (E c ∩ F c ∩ G )
10. P   = 16. Given, P (India wins) = 1/2
 G  P (G )
P (G ) − P (E ∩ G ) − P (G ∩ F ) ∴ P (India losses ) = 1 / 2
= Out of 5 matches India’s second win occurs at third test.
P (G )
P (G ) [1 − P (E ) − P (F )] ⇒ India wins third test and simultaneously it has won
= [Q P (G ) ≠ 0] one match from first two and lost the other.
P (G )
∴ Required probability = P (LWW ) + P (WLW )
= 1 − P (E ) − P (F ) = P (E c ) − P (F ) 3 3
 1  1 1
=  +  =
11. Let E = event when each American man is seated  2  2 4
adjacent to his wife
17. Let A = getting not less than 2 and not greater than 5
and A = event when Indian man is seated adjacent
to his wife 4
⇒ A ={2, 3, 4, 5} ⇒ P ( A) =
Now, n ( A ∩ E ) = (4 !) × (2 !)5 6

Even when each American man is seated adjacent to his But die is rolled four times, therefore the probability in
wife. getting four throws
n (E ) = (5 !) × (2 !)4  4  4  4  4 16
Again, =     =
 6  6  6  6 81
 A  n ( A ∩ E ) (4 !) × (2 !)
5
2
∴ P  = = =
 E n (E ) (5 !) × (2 !)4 5 18. Let A, B and C denote the events of passing the tests I,
II and III, respectively.
Alternate Solution
Evidently A, B and C are independent events.
Fixing four American couples and one Indian man in
between any two couples; we have 5 different ways in According to given condition,
which his wife can be seated, of which 2 cases are 1
= P [( A ∩ B) ∪ ( A ∩ C )]
favourable. 2
2 = P ( A ∩ B) + P ( A ∩ C ) − P ( A ∩ B ∩ C )
∴ Required probability =
5 = P ( A ) P (B) + P ( A ) ⋅ P (C ) − P ( A ) ⋅ P (B) ⋅ P (C )
12. Let E be the event of getting 1 on a die. 1 1
= pq + p ⋅ − pq ⋅
1 5 2 2
⇒ P (E ) = and P (E ) =
6 6 ⇒ 1 = 2 pq + p − pq ⇒ 1 = p(q + 1) …(i)
∴ P (first time 1 occurs at the even throw) The values of option (c) satisfy Eq. (i).
= t2 or t4 or t6 or t8 ... and so on [Infact, Eq. (i) is satisfied for infinite number of values
= { P (E )P (E )} + { P (E ) P (E ) P (E ) P (E )} + K ∞ of p and q. If we take any values of q such that 0 ≤ q ≤ 1,
5 1
then, p takes the value . It is evident that,
3
 5  1  5  1  5  1
5
36 5 q+1
=    +     +     +K∞ = = 1
 6  6  6  6  6  6 25 11 0< ≤ 1 i.e. 0 < p ≤ 1. But we have to choose correct
1−
36 q+1
answer from given ones.]
13. Probability that only two tests are needed = Probability
that the first machine tested is faulty × Probability that 19. Since, P ( A / B ) + P ( A / B ) = 1
2 1 1 ∴ P(A / B) = 1 − P(A / B)
the second machine tested is faulty = × =
4 3 6 20. Given that, P ( A ) = 0.4, P ( A ) = 0.6
14. The event that the fifth toss results in a head is P(the event A happens at least once)
independent of the event that the first four tosses result = 1 − P (none of the event happens)
in tails. = 1 − (0.6) (0.6) (0.6) = 1 − 0.216 = 0.784
∴ Probability of the required event = 1 / 2
rankershalt.com

Probability 115

1
21. P (X ) =
3
 X  P (X ∩ Y ) 1
P  = =
Y  P (Y ) 2 E F

 Y  P (X ∩ Y ) 2
P  = =
 X P (X ) 5
2
P (X ∩ Y ) = 2
15 Neither of them occurs =
25
4
P (Y ) = 2
15 ⇒ P (E ∩ F ) = …(ii)
4 2 25
− 11
 X ′  P (Y ) − P (X ∩ Y ) 15 15 1 From Eq. (i), P (E ) + P (F ) − 2 P (E ∩ F ) = …(iii)
P  = = =
Y  P (Y ) 4 2 25
15 2
From Eq. (ii), ( 1 − P (E )) ( 1 − P (F )) =
1 4 2 7 7 25
P (X ∪ Y ) = + − = =
3 15 15 15 15 2
⇒ 1 − P (E ) − P (F ) + P (E ) ⋅ P (F ) = …(iv)
22. PLAN 25
P ( A ∩ B) From Eqs. (iii) and (iv),
(i) Conditional probability, i.e. P( A / B) =
P( B) 7 12
P (E ) + P (F ) = and P (E ) ⋅ P (F ) =
(ii) P ( A ∪ B) = P( A ) + P( B) − P ( A ∩ B) 5 25
(iii) Independent event, then P ( A ∩ B) = P( A ) ⋅ P( B) 7  12
∴ P (E ) ⋅ − P (E ) =
1 Y  1  5  25
Here, P (X /Y ) = ,P  =
2  X 3
7 12
and P (X ∩ Y ) = 6 ⇒ (P (E ))2 − P (E ) + =0
5 25
 X  P (X ∩ Y )  3  4
∴ P  = ⇒ P (E ) − P (E ) − =0
Y  P (Y )  5   5 
1 1 /6 1
⇒ = ⇒ P (Y ) = …(i) 3 4 4 3
2 P (Y ) 3 ∴ P (E ) = or ⇒ P (F ) = or
5 5 5 5
Y  1 P (X ∩ Y ) 1
P  = ⇒ =
 X 3 P (X ) 3 24. Let A, B and C respectively denote the events that the
1 1 student passes in Maths, Physics and Chemistry.
⇒ = P (X ) It is given,
6 3
1 P ( A ) = m, P (B) = p and P (C ) = c and
∴ P (X ) = …(ii)
2 P (passing atleast in one subject)
P (X ∪ Y ) = P (X ) + P (Y ) − P (X ∩ Y ) = P ( A ∪ B ∪ C ) = 0.75
1 1 1 2
= + − = …(iii) ⇒ 1 − P ( A′ ∩ B ′ ∩ C ′ ) = 0.75
2 3 6 3
P (X ∩ Y ) =
1 1 1 1
and P (X ) ⋅ P (Y ) = ⋅ = Q [P ( A ) = 1 − P ( A )
6 2 3 6 and [P ( A ∪ B ∪ C ] = P ( A′ ∩ B′ ∩ C′ )]
⇒ P (X ∩ Y ) = P (X ) ⋅ P (Y )
⇒ 1 − P ( A′ ) . P (B ′ ) . P (C′ ) = 0.75
i.e. independent events
Q A, B and C are independent events, therefore A′, B′
∴ P (X c ∩ Y ) = P (Y ) − P (X ∩ Y ) and C ′ are independent events.
1 1 1
= − = ⇒ 0.75 = 1 − (1 − m) (1 − p) (1 − c)
3 6 6
⇒ 0 .25 = (1 − m) (1 − p) (1 − c) …(i)
23. E F
Also, P (passing exactly in two subjects)= 0.4
⇒ P ( A ∩ B ∩ C ∪ A ∩ B ∩ C ∪ A ∩ B ∩ C ) = 0.4
⇒ P ( A ∩ B ∩ C ) + P ( A ∩ B ∩ C ) + P ( A ∩ B ∩ C ) = 0.4
⇒ P ( A ) P (B) P (C ) + P ( A )P (B ) P (C )

11 + P ( A ) P (B) P (C ) = 0.4
P (E ∪ F ) − P (E ∩ F ) = …(i)
25 ⇒ pm (1 − c) + p(1 − m) c + (1 − p) mc = 0.4
[i.e. only E or only F] ⇒ pm − pmc + pc − pmc + mc − pmc = 0.4 …(ii)
rankershalt.com

116 Probability

Again, P (passing atleast in two subjects) = 0.5 ⇒


1
= 1 − { P (E ) + P (F )} +
1
⇒ P(A ∩ B ∩ C ) + P(A ∩ B ∩ C ) 2 12
1 1 7
+ P ( A ∩ B ∩ C ) + P ( A ∩ B ∩ C ) = 0.5 ⇒ P (E ) + P (F ) = 1 − + = …(ii)
2 12 12
⇒ pm(1 − c) + pc(1 − m) + cm(1 − p) + pcm = 0.5
On solving Eqs. (i) and (ii), we get
⇒ pm − pcm + pc − pcm + cm − pcm + pcm = 0.5 1 1
⇒ ( pm + pc + mc) − 2 pcm = 0.5 …(iii) either P (E ) = and P (F ) =
3 4
From Eq. (ii), 1 1
or P (E ) = and P (F ) =
pm + pc + mc − 3 pcm = 0.4 …(iv) 4 3
From Eq. (i),
27. We know that,
0.25 = 1 − (m + p + c) + ( pm + pc + cm) − pcm …(v)
 A  P ( A ∩ B) P ( A ) + P (B) − P ( A ∪ B)
P  = =
On solving Eqs. (iii), (iv) and (v), we get  B P (B) P (B)
p + m + c = 1.35 = 27 / 20
Since, P ( A ∪ B) < 1
Therefore, option (b) is correct.
⇒ − P ( A ∪ B ) > −1
Also, from Eqs. (ii) and (iii), we get pmc = 1 / 10
⇒ P ( A ) + P (B) − P ( A ∪ B) > P ( A ) + P (B) − 1
Hence, option (c) is correct.
P ( A ) + P (B) − P ( A ∪ B) P ( A ) + P (B) − 1
P (E ∩ F ) P (E ∩ F ) ⇒ >
25. (a) P (E / F ) + P (E / F ) = + P (B) P (B)
P (F ) P (F )
 A  P ( A ) + P (B) − 1
P (E ∩ F ) + P (E ∩ F ) ⇒ P  >
=  B P (B)
P (F ) Hence, option (a) is correct.
P (F )
= =1 The choice (b) holds only for disjoint i.e. P ( A ∩ B) = 0
P (F )
Finally, P ( A ∪ B) = P ( A ) + P (B) − P ( A ∩ B)
Therefore, option (a) is correct. = P ( A ) + P (B) − P ( A ) ⋅ P (B),
P (E ∩ F ) P (E ∩ F )
(b) P (E / F ) + P (E / F ) = + if A , B are independent
P (F ) P (F )
= 1 − {1 − P ( A )} {1 − P (B)} = 1 − P ( A ) ⋅ P (B )
P (E ∩ F ) P (E ∩ F )
= + ≠1 Hence, option (c) is correct, but option (d) is not correct.
P (F ) 1 − P (F )
28. Since, E and F are independent events. Therefore,
Therefore, option (b) is not correct. P (E ∩ F ) = P (E ) ⋅ P (F ) ≠ 0, so E and F are not mutually
P (E ∩ F ) P (E ∩ F ) exclusive events.
(c) P (E / F ) + P (E / F ) = +
P (F ) P (F ) Now, P (E ∩ F ) = P (E ) − P (E ∩ F ) = P (E ) − P (E ) ⋅ P (F )
P (E ∩ F ) P (E ∩ F ) = P (E ) [1 − P (F )] = P (E ) ⋅ P (F )
= + ≠1
P (F ) 1 − P (F ) and P (E ∩ F ) = P (E ∪ F ) = 1 − P (E ∪ F )
Therefore, option (c) is not correct. = 1 − [1 − P (E ) ⋅ P (F )]
P (E ∩ F ) P (E ∩ F ) [Q E and F are independent]
(d) P (E / F ) + P (E / F ) = +
P (F ) P (F ) = P (E ) ⋅ P (F )
P (E ∩ F ) + P (E ∩ F ) So, E and F as well as E and F are independent events.
=
P (F ) P (E ∩ F ) + P (E ∩ F )
Now, P (E / F ) + P (E / F ) =
P (F ) P (F )
= =1
P (F ) P (F )
= =1
Therefore, option (d) is correct. P (F )
1 29. P ( A c ) = 0.3 [given]
26. Both E and F happen ⇒ P (E ∩ F ) =
12 ⇒ P ( A ) = 0.7
1 P (B) = 0.4 [given]
and neither E nor F happens ⇒ P (E ∩ F ) =
2 ⇒ P (Bc ) = 0.6 and P ( A ∩ Bc ) = 0.5 [given]
But for independent events, we have Now, P ( A ∪ Bc ) = P ( A ) + P (Bc ) − P ( A ∩ Bc )
1 = 0.7 + 0.6 − 0.5 = 0.8
P (E ∩ F ) = P (E ) P (F ) = …(i)
12 P{ B ∩ ( A ∪ Bc )}
∴ P [B / ( A ∪ B ] =
c
and P (E ∩ F ) = P (E ) P (F ) P ( A ∪ Bc )
= {1 − P (E )}{(1 − P (F )} P{(B ∩ A ) ∪ (B ∩ Bc )} P{(B ∩ A ) ∪ φ } P (B ∩ A )
= 1 − P (E ) − P (F ) + P (E )P (F ) = = =
0.8 0.8 0.8
rankershalt.com

Probability 117

1 2
= [P ( A ) − P ( A ∩ Bc )] 35. Given, P ( A ) = probability that A will hit B =
0.8 3
0.7 − 0.5 0.2 1 1
= = = P (B) = probability that B will hit A =
0.8 0.8 4 2
1
30. P ( A ∪ B) = P ( A ) + P (B) − P ( A ) P (B), as A and B are P (C ) = probability that C will hit A =
3
independent events. P (E ) = probability that A will be hit
⇒ 0.8 = (0.3) + P (B) − (0.3) P (B) 1 2 2
5 ⇒ P (E ) = 1 − P (B ) ⋅ P (C ) = 1 − ⋅ =
⇒ 0.5 = (0.7) P (B) ⇒ P (B) = 2 3 3
7 Probability if A is hit by B and not by C
1.2
31. 5 can be thrown in 4 ways and 7 can be thrown in 6 P (B) . P (C ) 2 3 1
ways, hence number of ways of throwing neither 5 nor 7 = P (B ∩ C / E ) = = =
P (E ) 2 2
is 36 − (4 + 6) = 26
∴ Probability of throwing a five in a single throw with a 3
4 1 36. Let Ei denotes the event that the students will pass the
pair of dice = = and probability of throwing neither
36 9 ith exam, where i = 1, 2, 3
26 13
5 nor 7 = = and E denotes the student will qualify.
36 18
Hence, required probability ∴ P (E ) = [P (E1 ) × P (E 2 / E1 )]
1 + [P (E1 ) × P (E 2′ /E1 ) × P (E3 / E 2′ )]
2
 1  13  1  13  1 2 + [P (E1′ ) × P (E 2 / E ′1 ) × P (E3 / E 2)]
=   +     +     + ... = 9 =
 9  18  9  18  9 13 5
1− p
= p + p(1 − p) . + (1 − p) . . p
2 p
18 2 2
32. Let R be drawing a red ball and B for drawing a black 2 p2 + p2 − p3 + p2 − p3
ball, then required probability ⇒ P (E ) = = 2 p2 − p3
2
= RRR + RBR + BRR + BBR
37. Since, pn denotes the probability that no two (or more)
6 5 6 6 6 5
= × × + × ×  consecutive heads occur.
 10 11 10  10 11 10
⇒ pn denotes the probability that 1 or no head occur.
4 4 7  4 7 6
For n = 1 , p1 = 1 because in both cases we get less than
+ × × + × × 
 10 11 10  10 11 10
two heads (H, T).
640 32
= = For n = 2, p2 = 1 − p (two heads simultaneously occur).
1100 55
= 1 − p(HH ) = 1 − pp = 1 − p2
33. Let A be the event that the maximum number on the
two chosen tickets is not more than 10, and B be the For n ≥ 3, pn = pn − 1 (1 − p) + pn − 2(1 − p) p
event that the minimum number on them is 5 ⇒ pn = (1 − p) pn − 1 + p(1 − p) pn − 2
5
C Hence proved.
∴ P ( A ∩ B) = 100 1
C2 38. Let, E1 = the event noted number is 7
10
and
C
P ( A ) = 100 2 E 2 = the event noted number is 8
C2 H = getting head on coin
 B  P ( A ∩ B) T = be getting tail on coin
Then P  =
 A P ( A) ∴ By law of total probability,
5
= 10
C1 1
= P (E1 ) = P (H ) ⋅ P (E1 / H ) + P (T ) ⋅ P (E1 / T )
C2 9 and P (E 2) = P (H ) ⋅ P (E 2 / H ) + P (T ) ⋅ P (E 2 / T )
34. Here, P ( A ∪ B) . P ( A′ ∩ B ′ ) where, P (H ) = 1 / 2 = P (T )

⇒ { P ( A ) + P (B) − P ( A ∩ B)}{ P ( A′ ) . P (B ′ )} P (E1/H ) = probability of getting a sum of 7 on two dice


[since A, B are independent, so A ′ , B ′ are independent] Here, favourable cases are
∴ P ( A ∪ B) . P ( A ′ ∩ B ′ ) ≤ { P ( A ) + P (B)}. { P ( A′ ) . P (B′ )} {(1, 6), (6, 1), (2, 5), (5, 2), (3, 4), (4, 3)}.
= P ( A ) . P ( A′ ) . P (B′ ) + P (B) . P ( A′ ) . P (B′ ) 6 1
∴ P (E1 / H ) = =
≤ P ( A ) . P (B′ ) + P (B) . P ( A′ ) …(i) 36 6
[Q P ( A′ ) ≤ 1 and P (B ′ ) ≤ 1] Also, P (E1 / T ) = probability of getting 7 numbered
⇒ P ( A ∪ B) . P ( A ′ ∩ B ′ ) ≤ P ( A ) . P (B ′ ) + P (B) . P ( A′ ) card out of 11 cards
⇒ P ( A ∪ B) . P ( A′ ∩ B ′ ) ≤ P (C ) 1
=
[Q P (C ) = P ( A ) . P (B ′ ) + P (B). P ( A′ )] 11
rankershalt.com

118 Probability

P (E 2 / H ) = probability of getting a sum of 8 on two dice 1


Similarly, P (B ∩ C ) = , P (C ∩ A ) =
1
Here, favourable cases are 4 4
{(2, 6), (6, 2), (4, 4), (5, 3), (3, 5)}. and P ( A ∩ B ∩ C ) = 0.
Since, P ( A ∩ B) = P ( A )P (B), P (B ∩ C ) = P (B)P (C )
5
∴ P (E 2 / H ) = and P (C ∩ A ) = P (C )P ( A ).
36
Therefore, A, B and C are pairwise independent.
P (E 2 / T ) = probability of getting ‘8’ numbered
card out of 11 cards Also, P ( A ∩ B ∩ C ) ≠ P ( A )P (B)P (C ) therefore A, B and
C cannot be independent.
= 1 / 11
 1 1  1 1  1 1 17 40. The total number of ways to answer the question
∴ P (E1 ) =  ×  +  ×  = + =
 2 6  2 11 12 22 132 = 4C1 + 4C 2 + 4C3 + 4C 4 = 24 − 1 = 15
1 5  1 1  P(getting marks) = P( correct answer in I chance)
and P (E 2) =  ×  +  ×  + P(correct answer in II chance)
 2 36  2 11
+ P( correct answer in III chance)
1  91  91
=   = 1  14 1   14 13 1  3 1
2  396 729 = + ⋅  + ⋅ ⋅ = =
15  15 14  15 14 13 15 5
Now, E1 and E 2 are mutually exclusive events.
1 1
Therefore, 41. Given, P ( A ) ⋅ P (B) = , P ( A ) ⋅ P (B ) =
6 3
17 91 193 1
P (E1 or E 2) = P (E1 ) + P (E 2) = + = ∴ [1 − P ( A )] [1 − P (B)] =
132 792 792 3
39. Let D1 denotes the occurrence of a defective bulb in Ist Let P ( A ) = x and P (B) = y
draw. 1 1
50 1 ⇒ (1 − x)(1 − y) = and xy =
Therefore, P (D1 ) = = 3 6
100 2 1 1
⇒ 1 − x − y + xy = and xy =
and let D2 denotes the occurrence of a defective bulb in 3 6
IInd draw. 5 1
50 1 ⇒ x+ y= and xy =
Therefore, P (D2) = = 6 6
100 2
5  1
and let N 1 denotes the occurrence of non-defective bulb ⇒ x  − x =
6  6
in Ist draw.
50 1 ⇒ 6x2 − 5x + 1 = 0
Therefore, P (N 1 ) = =
100 2 ⇒ (3x − 1)(2x − 1) = 0
Again, let N 2 denotes the occurrence of non-defective 1 1
bulb in IInd draw. ⇒ x = and
50 1 3 2
Therefore, P (N 2) = = 1 1
100 2 ∴ P ( A ) = or
Now, D1 is independent with N 1and D2 is independent 3 2
with N 2 . 42. P (N th draw gives 2nd ace)
According to the given condition, = P{ 1 ace and (n − 2) other cards are drawn in (N − 1)
A = {the first bulb is defective} = { D1D2, D1N 2} draws} × P { N th draw is 2nd ace}
B = {the second bulb is non-defective} = { D1N 2, N 1N 2} 4 ⋅ (48)! ⋅ (n − 1)! (52 − n )! 3
= ⋅
and C = {the two bulbs are both defective} (52)! ⋅ (n − 2)! (50 − n )! (53 − n )
= { D1D2, N 1N 2}
4(n − 1)(52 − n )(51 − n ) ⋅ 3
Again, we know that, =
52 ⋅ 51 ⋅ 50 ⋅ 49
A ∩ B = { D1N 2}, B ∩ C = { N 1N 2}. (n − 1) (52 − n ) (51 − n )
=
C ∩ A = { D1D2} and A ∩ B ∩ C = φ 50 ⋅ 49 ⋅ 17 ⋅ 13
Also, P ( A ) = P{ D1D2} + P{ D1N 2} 43. Let P (H 1 ) = 0.4, P (H 2) = 0.3, P (H 3 ) = 0.2, P (H 4 ) = 0.1
= P (D1 )P (D2) + P (D1 )P (N 2)
P (gun hits the plane)
 1  1  1  1 1 = 1 − P(gun does not hit the plane)
=   +   =
 2  2  2  2 2
= 1 − P (H 1 ) ⋅ P (H 2) ⋅ P (H 3 ) ⋅ P (H 4 )
1 1
Similarly, P (B) = and P (C ) = = 1 − (0.6) (0.7) (0.8) (0.9) = 1 − 0.3024 = 0.6976
2 2
44. Since, the drawn balls are in the sequence black, black,
 1  1 1
Also, P ( A ∩ B) = P (D1N 2) = P (D1 )P (N 2) =     = white, white, white, white, red, red and red.
 2  2 4
rankershalt.com

Probability 119

Let the corresponding probabilities be 4  A 6


P (E1 ) = , P  =
p1 , p2,... , p9 10  E1  12
2 1 4 3 2 6  A 4
Then, p1 = , p2 = , p3 = , p4 = , p5 = ⇒ P (E 2) = , P  =
9 8 7 6 5 10  E 2 12
1 3 2
p6 = , p7 = , p8 = , p9 = 1 By law of total probability
4 3 2
 A  A
∴ Required probabilitie P ( A ) = P (E1 ) × P   + P (E 2) × P  
 E1   E 2
p1 . p2 . p3 ⋅ K ⋅ p9
4 6 6 4 24 + 24 48 2
 2  1  4   3   2  1   3  2  1 = × + × = = =
=                (1) = 10 12 10 12 120 120 5
 9  8  7  6   5  4   3  2  1260
3. Let x = P (computer turns out to be defective, given that
45. PLAN
it is produced in plant T2)
Forthe events to be independent,
 D
P( E1 ∩ E 2 ∩ E 3 ) = P( E1 ) ⋅ P( E 2 ) ⋅ P( E 3 ) ⇒ x= P   …(i)
 T2
P( E1 ∩ E 2 ∩ E 3 ) = P(only E1 occurs)
= P( E1 ) ⋅ (1 − P( E 2 )) (1 − P( E 3 )) where, D = Defective computer
Let x, y and z be probabilities of E1 , E 2 and E3 , ∴ P (computer turns out to be defective given that is
respectively. produced in plant T1) = 10x
∴ α = x (1 − y) (1 − z ) …(i)  D
β = (1 − x) ⋅ y(1 − z ) …(ii) i.e. P   = 10x …(ii)
 T1 
γ = (1 − x) (1 − y)z …(iii)
20 80
⇒ p = (1 − x) (1 − y) (1 − z ) …(iv) Also, P (T1 ) =
and P (T2) =
100 100
Given, (α − 2 β ) p = αβ and ( β − 3γ ) p = 2 βγ …(v)
7
From above equations, x = 2 y and y = 3z Given, P (defective computer) =
100
∴ x = 6z 7
x i.e. P (D ) =
⇒ =6 100
z
Using law of total probability,
46. Here, P (X > Y ) = P (T1win ) P (T1 win )  D  D
P (D ) = 9(T1 ) ⋅ P   + P (T2) ⋅ P  
+ P (T1 win ) P (draw ) + P (draw ) P (T1 win )  T1   T2
 1 1  1 1  1 1 5
= ×  + ×  + ×  = 7  20   80 
 2 2  2 6  6 2 12 ∴ =  ⋅ 10x +  ⋅x
100  100  100
47. P [X = Y ] = P (draw ) ⋅ P (draw ) 1
+ P (T1 win ) P (T2 win ) + P (T2 win ) ⋅ P (T1 win ) ⇒ 7 = (280)x ⇒ x = …(iii)
40
= (1 / 6 × 1 / 6) + (1 / 2 × 1 / 3) + (1 / 3 × 1 / 2) = 13 / 36  D  D  10
1
∴ P  = and P   =
Topic 4 Law of Total Probability and  T2 40  T1  40
Baye’s Theorem  D 1 39  D 10 30
⇒ P   =1− = and P   = 1 − = …(iv)
1. Let A be the event that ball drawn is given and B be the  T2 40 40  T1  40 40
event that ball drawn is red.
2 5
∴ P ( A ) = and P (B) = Using Baye’s theorem,
7 7
T  P (T2 ∩ D )
Again, let C be the event that second ball drawn is red. P  2 =
 D  P (T1 ∩ D ) + P (T2 ∩ D )
∴ P (C ) = P ( A ) P (C / A ) + P (B) P (C / B)
2 6 5 4  D
= × + × P (T2) ⋅ P  
7 7 7 7  T2
=
12 + 40 32  D  D
= = P (T1 ) ⋅ P   + P (T2) ⋅ P  
49 49  T1   T2
2. Key idea Use the theorem of total probability 80 39

Let E1 = Event that first ball drawn is red 100 40 78
= =
E 2 = Event that first ball drawn is black 20 30
⋅ +
80 39 93

A = Event that second ball drawn is red 100 40 100 40
rankershalt.com

120 Probability

4. From the tree diagram, it follows that P (E1 ∩ E 2 ∩ E3 ) + P (E1 ∩ E 2 ∩ E3 ) + P (E1 ∩ E 2 ∩ E3 )


=
S P (E 2)
4 1 1 1 3 1 1 1 1 1 1
⋅ ⋅ + ⋅ ⋅ + ⋅ ⋅
5 5 5
= 2 4 4 2 4 4 2 4 4 =
1 8
G R
3 1 3 1 4
1 1 1 1 3 1 1 1 3
4 4 4 4 ⋅ ⋅ + ⋅ ⋅ + ⋅ ⋅
P (X ∩ X1 ) 2 4 4 2 4 4 2 4 4
(d) P (X / X1 ) = =
AG AR AR 1 AG P (X1 ) 1 /2
3 1 3 11 3 4 3
7
4 4 4 44 4 4 =
BG BR BG BR BG 16
46 6. Statement I If P (H i ∩ E ) = 0 for some i, then
P (BG ) =
80 H   E
P  i = P   = 0
10 5  E  Hi 
P (BG|G ) = =
16 8 If P (H i ∩ E ) ≠ 0, ∀ i = 1, 2, K , n , then
5 4 1  H  P (H i ∩ E ) P (H i )
P (BG ∩ G ) = × = P  i = ×
8 5 2  E P (H i ) P (E )
1
 E
1 80 20 P   × P (H i )
∴ P (G|BG ) = 2 = × =  Hi   E
P (BG ) 2 46 23 = > P   ⋅ P (H i ) [Q0 < P (E ) < 1]
P (E )  Hi 
5. PLAN It is based on law of total probability and Bay’s Law.
Hence, Statement I may not always be true.
Description of Situation It is given that ship would
Statement II Clearly, H 1 ∪ H 2 ∪ . . . ∪ H n = S
work if atleast two of engines must work. If X be event
[sample space]
that the ship works. Then, X ⇒ either any two of
E1 , E 2, E3 works or all three engines E1 , E 2, E3 works. ⇒ P (H 1 ) + P (H 2) + . . . + P (H n ) = 1
Hence, Statement II is ture.
1 1 1
Given, P (E1 ) = , P (E 2) = , P (E3 ) = Passage I
2 4 4
P (E1 ∩ E 2 ∩ E3 ) + P (E1 ∩ E 2 ∩ E3 )  7.
∴ P(X) =  
n1 Red n3 Red
+ P (E1 ∩ E 2 ∩ E3 ) + P (E1 ∩ E 2 ∩ E3 ) 
 1 1 3 1 3 1 1 1 1  1 1 1 n2 Black n4 Black
= ⋅ ⋅ + ⋅ ⋅ + ⋅ ⋅ + ⋅ ⋅ 
 2 4 4 2 4 4 2 4 4  2 4 4
Box I Box II
= 1 /4
Let A = Drawing red ball
Now, (a) P (X1c / X )
∴ P ( A ) = P (B1 ) ⋅ P ( A / B1 ) + P (B2) ⋅ P ( A / B2)
1 1 1
 X1c ∩ X  P (E1 ∩ E 2 ∩ E3 ) 2 ⋅ 4 ⋅ 4 1 1  n1  1  n3 
=P = = = =  +  
2  n1 + n2 2  n3 + n4 
 P (X )  P (X ) 1 8
4 1
Given, P (B2 / A ) =
3
P (B2) ⋅ P (B2 ∩ A ) 1
(b) P (exactly two engines of the ship are functioning) ⇒ =
P ( A) 3
P (E1 ∩ E 2 ∩ E3 ) + P (E1 ∩ E 2 ∩ E3 ) + P (E1 ∩ E 2 ∩ E3 )
= 1  n3 
P (X )  
2  n3 + n4  1
1 1 3 1 3 1 1 1 1 ⇒ =
⋅ ⋅ + ⋅ ⋅ + ⋅ ⋅ 1  n1  1  n3  3
7   +  
=2 4 4 2 4 4 2 4 4= 2  n1 + n2 2  n3 + n4 
1 8
4 n3 (n1 + n2) 1
⇒ =
 X  P (X ∩ X 2) n1 (n3 + n4 ) + n3 (n1 + n2) 3
(c) P   =
 X 2 P (X 2) Now, check options, then clearly options (a) and (b)
P (ship is operating with E 2 function ) satisfy.
=
P (X 2)
rankershalt.com

Probability 121

8. 10. P (Head appeared/white from U 2)


1 Red
(n1 – 1) Red (n3 + 1) Red  3C1 2C1 2
C 1
C 
5 × 2 + 5 1 × 2 1
= P (H ) .  1
C C1 C 1 C1 
n2 Black n4 Black 23 / 30
3 2 1
Box I Box II  ×1 + × 
= 
1 5 5 2
1 Black
2 20 / 30
n1 Red n3 Red 12
or =
23
(n2 – 1) Black (n4 + 1) Black Passage III
Box I Box II 5 5 1 25
11. P (X = 3) = ⋅ ⋅ =
1 6 6 6 216
∴ P (drawing red ball from B1) = 5 5 25
3 12. P (X ≥ 3) = ⋅ ⋅ 1 =
 n1 − 1   n1   n2   n1  1 6 6 36
⇒   +   = P {(X > 3) / (X ≥ 6)} ⋅ P (X ≥ 6)
 n1 + n2 − 1  n1 + n2  n1 + n2  n1 + n2 − 1 3 13. P {(X ≥ 6) / (X > 3)} =
P (X > 3)
n12 + n1n2 − n1 1   5  1  5 6  1 
⇒ = 5
(n1 + n2) (n1 + n2 − 1) 3 1 ⋅   ⋅   +   ⋅   + ... ∞ 
 6  6  6  6
Clearly, options (c) and (d) satisfy. =   = 25
  5 3 1  5 4 1  36
Passage II   ⋅ +   ⋅ + ... ∞ 
3W 1W  6 6 6 6 
Initial
2R Passage IV
U1 U2 14. Here, P (ui ) = ki, Σ P (ui ) = 1
Head appears 2
2W ⇒ k=
2W n (n + 1)
1W
2R n
2i 2
U1 U2 2 Cases lim P (W ) = lim
n→ ∞ n→ ∞
∑ n (n + 1)2
or i=1
3W 1W 2n (n + 1)(2n + 1)
= lim = 2 /3
1R n→∞ 6n (n + 1)2
1R 1R
U1 U2 n
 un  n + 1 2
Tail appears 15. P   = =
W  Σi n+1
1W 3W 3W 2W 2W
1W n+1
2W 0R 2R 1R 1R
2R W  2 + 4 + 6 + ... n+2
16. P   = =
U1 U2 U1 U2 U1 U2  E n (n + 1) 2 (n + 1)
2
3 Cases
17. As, the statement shows problem is to be related to
9. Now, probability of the drawn ball from U 2 being white
Baye’s law.
is
Let C , S , B, T be the events when person is going by car,
 3C 2
C 2
C 1
C  scooter, bus or train, respectively.
P (white / U 2) = P (H ) ⋅  5 1 × 2 1 + 5 1 × 2 1 
 1C C1 C 1 C1  1 3 2 1
∴ P (C ) = , P (S ) = , P (B) = , P (T ) =
 3C 3
C 2
C 1
C 3
C1 ⋅ 2C1 C1 
2 7 7 7 7
+ P (T )  5 2 × 3 2 + 5 2 × 3 1 + × 
5 3 Again, L be the event of the person reaching office late.
 C2 C2 C2 C2 C2 C2 
1 3 2 1 ∴ L be the event of the person reaching office in time.
=  ×1 + × 
2 5 5 2  L 7  L 8  L 5
Then, P   = , P   = , P   =
13 1 1 6 2  23 C 9 S 9  B 9
+  ×1 + × + × =
2 10 10 3 10 3  30
 L 8
and P  =
T 9
rankershalt.com

122 Probability

 L =
1 8
[( C 2)(6C 2)(4C 2)(2C 2)]
P   ⋅ P (C )
C C 4!
∴ P  = 1 8! 6! 4!
 L  L  L  L = × × × ×1
P   ⋅ P (C ) + P   ⋅ P (S ) + P   ⋅ P (B) 4 ! 2 !6 ! 2 !4 ! 2 !2 !
C S  B 1 8×7 6 ×5 4 ×3 8 × 7 ×6 ×5
 L = × × × = = 105
+ P   ⋅ P (T ) 4 ! 2 ! ×1 2 ! × 1 2 ! × 1 2 .2 .2 .2
T
Now, atleast two players certainly reach the second
7 1
× round between P1, P2 and P3 and P4 can reach in final if
9 7 1
= = exactly two players play against each other between P1,
7 1 8 3 5 2 8 1 7
× + × + × + × P2, P3 and remaining player will play against one of the
9 7 9 7 9 7 9 7 players from P5 , P6, P7, P8 and P4 plays against one of the
18. Let A1 be the event exactly 4 white balls have been remaining three from P5 …P8.
drawn. A2 be the event exactly 5 white balls have been This can be possible in
drawn. 3
C 2 × 4C1 × 3C1 = 3 . 4 . 3 = 36 ways
A3 be the event exactly 6 white balls have been drawn. ∴ Probability that P4 and exactly one of P5 ... P8 reach
B be the event exactly 1 white ball is drawn from two 36 12
second round = =
draws. Then, 105 35
 B  B  B If P1 , Pi , P4 and Pj , where i = 2 or 3 and j = 5 or 6 or 7
P (B) = P   P ( A1 ) + P   P ( A2) + P   P ( A3 )
 A1   A2   A3  reach the second round, then they can be paired in 2
1 4
 B pairs in ( C 2) (2C 2) = 3 ways. But P4 will reach the
But P   = 0 2!
 A3  final, if P1 plays against Pi and P4 plays against Pj .
[since, there are only 6 white balls in the bag] Hence, the probability that P4 will reach the final round
1
 B  B from the second =
∴ P (B) = P   P ( A1 ) + P   P ( A2) 3
 A1   A2  12 1 4
∴ Probability that P4 will reach the final is × = .
12
C 2.6 C 4 10
C1.2 C1 C1.6 C5 . 11C1.1 C1
12 35 3 35
= . +
18
C6 12
C2 18
C6 12
C2 22. Let q = 1 − p = probability of getting the tail. We have,
α = probability of A getting the head on tossing firstly
19. Let E be the event that coin tossed twice, shows head at
first time and tail at second time and F be the event that = P (H 1 or T1T2T3 H 4 or T1T2T3T4T5T6H 7 or … )
coin drawn is fair. = P (H ) + P (H )P (T )3 + P (H )P (T )6 + …
P (E / F ) ⋅ P (F ) P (H ) p
P (F / E ) = = =
P (E / F ) ⋅ P (F ) + P (E / F ′ ) ⋅ P (F ′ ) 1 − P (T )3 1 − q3
1 1 m
⋅ ⋅ Also,
= 2 2 N
1 1 m 2 1 N −m β = probability of B getting the head on tossing secondly
⋅ ⋅ + ⋅ ⋅ = P (T1H 2 or T1T2T3T4H 5 or T1T2T3T4T5T6T7H 8 or …)
2 2 N 3 3 N
m = P (H ) [P (T ) + P (H )P (T )4 + P (H )P (T )7 + K ]
4 9m = P (T )[P (H ) + P (H )P (T )3 + P (H )P (T )6 + ... ]
= =
m 2 (N − m) 8N + m p(1 − p)
+ = q α = (1 − p) α =
4 9 1 − q3
20. Let W1 = ball drawn in the first draw is white. Again, we have
B1 = ball drawn in the first draw in black. α + β + γ =1
W 2 = ball drawn in the second draw is white. p + p(1 − p)
⇒ γ = 1 − (α + β ) = 1 −
Then , P (W 2) = P (W1 ) P (W 2 / W1 ) + P (B1 )P (W 2 / B1 ) 1 − q3
 m   m+ k   n   m  p + p(1 − p)
=    +    =1 −
 m + n   m + n + k  m + n   m + n + k 1 − (1 − p)3
m(m + k) + mn m (m + k + n ) m 1 − (1 − p)3 − p − p(1 − p)
= = = =
(m + n ) (m + n + k) (m + n ) (m + n + k) m + n 1 − (1 − p)3
21. The number of ways in which P1 , P2, K , P8 can be paired 1 − (1 − p)3 − 2 p + p2 p − 2 p2 + p3
γ= =
in four pairs 1 − (1 − p)3 1 − (1 − p)3
rankershalt.com

Probability 123

p p(1 − p) 1 1 1
Also, α= , β= ⇒ P (E3 ) = 1 − − =
1 − (1 − p)3
1 − (1 − p)3 3 6 2
23. (i) Probability of S1 to be among the eight winners If E1 has already occured, then the examinee guesses.
Since, there are four choices out of which only one is
= (Probability of S1 being a pair ) correct, therefore the probability that he answer
× (Probability of S1 winning in the group) correctly given that he has made a guess is 1/4.
1
1 1 i.e. P ( A / E1 ) =
=1 × = [since, S1 is definitely in a group] 4
2 2 1
It is given that, P ( A / E 2) =
(ii) If S1 and S 2 are in the same pair, then exactly one 8
wins.
and P ( A / E3 ) = probability that he answer correctly
If S1 and S 2 are in two pairs separately, then exactly given that he know the answer = 1
one of S1 and S 2 will be among the eight winners. If
By Baye’s theorem, we have
S1 wins and S 2 loses or S1 loses and S 2 wins.
P (E3 ) ⋅ P ( A / E3 )
Now, the probability of S1 , S 2 being in the same pair P (E3 / A ) =
 
and one wins
P (E1 ) ⋅ P ( A / E1 ) + P (E 2) ⋅ P ( A / E 2) 
= (Probability of S1 , S 2 being the same pair)  + P (E3 ) ⋅ P ( A / E3 ) 

× (Probability of anyone winning in the pair).
1
and the probability of S1 , S 2 being the same pair ×1
2 24
n (E ) ∴ P (E3 / A ) = =
=  1 1  1 1  1  29
 ×  +  ×  +  × 1
n (S )  3 4  6 8  2 
where, n (E ) = the number of ways in which 16 25. Let Bi = ith ball drawn is black.
persons can be divided in 8 pairs.
Wi = ith ball drawn is white, where i = 1, 2
(14)! (16)!
∴ n (E ) = and n (S ) = and A = third ball drawn is black.
(2 !) ⋅ 7 !
7
(2 !)8 ⋅ 8 !
We observe that the black ball can be drawn in the third
∴ Probability of S1 and S 2 being in the same pair
draw in one of the following mutually exclusive ways.
(14)! ⋅ (2 !)8 ⋅ 8 ! 1 (i) Both first and second balls drawn are white and
= =
(2 !)7 ⋅ 7 !⋅ (16)! 15 third ball drawn is black.
i.e. (W1 ∩ W 2) ∩ A
The probability of any one wining in the pairs of
(ii) Both first and second balls are black and third ball
S1 , S 2 = P (certain event) = 1
drawn is black.
∴ The pairs of S1 , S 2 being in two pairs separately i.e. (B1 ∩ B2) ∩ A
and S1 wins, S 2 loses + The probability of S1 , S 2 being (iii) The first ball drawn is white, the second ball drawn
in two pairs separately and S1 loses, S 2 wins. is black and the third ball drawn is black.
 (14)!   (14)!  i.e. (W1 ∩ B2) ∩ A
 (2 !)7 ⋅ 7 !  1 1  (2 !)7 ⋅ 7 !  1 1 (iv) The first ball drawn is black, the second ball drawn
= 1 − × × + 1 − × ×
(16)!  2 2  (16)!  2 2 is white and the third ball drawn is black.
 
 (2 !)8 ⋅ 8 !   (2 !)8 ⋅ 8 !  i.e. (B1 ∩ W 2) ∩ A
1 14 × (14)! 7 ∴ P ( A ) = P [{(W1 ∩ W 2) ∩ A } ∪{(B1 ∩ B2) ∩ A }
= × =
2 15 × (14)! 15 ∪ {(W1 ∩ B2) ∩ A } ∪ {(B1 ∩ W 2) ∩ A }]
1 7 8 = P{(W1 ∩ W 2) ∩ A } + P{(B1 ∩ B2) ∩ A }
∴ Required probability = + =
15 15 15 + P{(W1 ∩ B2) ∩ A } + P{(B1 ∩ W 2) ∩ A }
24. Let E1 , E 2, E3 and A be the events defined as = P (W1 ∩ W 2) ⋅ P ( A / (W1 ∩ W 2)) + P (B1 ∩ B2)
E1 = the examinee guesses the answer ∴ P ( A / (B1 ∩ B2)) + P (W1 ∩ B2) ⋅ P ( A / (W1 ∩ B2))
E 2 = the examinee copies the answer + P (B1 ∩ W 2) ⋅ P ( A / (B1 ∩ W 2))
E3 = the examinee knows the answer  2 1  2 3 4
=  ×  ×1 +  ×  ×
 4 3  4 5 6
and A = the examinee answer correctly
1 1  2 2 3  2 2 3
We have, P (E1 ) = , P (E 2) = + ×  × + ×  ×
3 6  4 3 4  4 5 4
1 1 1 3 23
Since, E1 , E 2, E3 are mutually exclusive and exhaustive = + + + =
events. 6 5 4 20 30
∴ P (E1 ) + P (E 2) + P (E3 ) = 1
rankershalt.com

124 Probability

26. The testing procedure may terminate at the twelfth 1 1


∴ q= ⇒ p =1−q =
testing in two mutually exclusive ways. 2 2
1
I : When lot contains 2 defective articles. Now, n× = 8 ⇒ n = 16
2
II : When lot contains 3 defective articles. 16
 1
Let A = testing procedure ends at twelth testing P (X = r ) =16 C r  
 2
A1 = lot contains 2 defective articles ∴ P (X ≤ 2) = P (X = 0) + P (X = 1) + P (X = 2)
A2 = lot contains 3 defective articles 16 16 16
 1  1  1
∴ Required probability =16 C 0   + 16 C1   + 16 C 2 
 2  2  2
= P ( A1 ) ⋅ P ( A / A1 ) + P ( A2) ⋅ P ( A / A2) 1 + 16 + 120 137 k
= = 16 = 16 (given)
Here, P ( A / A1 ) = probability that first 11 draws contain 216 2 2
10 non-defective and one-defective and twelfth draw
⇒ k = 137
contains a defective article.
18
C10 × 2C1 1 3. As we know probability of getting a head on a toss of a
= × …(i) 1
20
C11 9 fair coin is P (H ) = = p (let)
2
P ( A / A2) = probability that first 11 draws contains 9 Now, let n be the minimum numbers of toss required to
non-defective and 2-defective articles and twelfth draw get at least one head, then required probability
17
C 9 × 3C 2 1 = 1 − (probability that on all ‘n’ toss we are getting tail)
contains defective = 20
× … (ii) n
C11 9  1  1
=1 −   Q P (tail) = P (Head ) =
 2  2 
∴ Required probability
= (0.4)P ( A / A1 ) + 0.6 P ( A / A2) According to the question,
n n
 1 99  1 99
0.4 × 18C10 × 2C1 1 0.6 × 17C 9 × 3C 2 1 99 1−  > ⇒  <1 −
= 20
× + 20
× =  2 100  2 100
C11 9 C11 9 1900 n
 1 1
⇒   < ⇒ 2n > 100
 2 100
Topic 5 Probability Distribution and ⇒ n=7 [for minimum]
Binomial Distribution
4. The required probability of observing atleast one head
1. Given that, there are 50 problems to solve in an
admission test and probability that the candidate can = 1 − P (no head)
4 1
solve any problem is = q (say). So, probability that the =1 − n [let number of toss are n]
5 2
4 1  1
candidate cannot solve a problem is p = 1 − q = 1 − = . Q P (Head) = P (Tail) =
5 5  2 
1 90
According to the question, 1 − n ≥
Now, let X be a random variable which denotes the 2 100
number of problems that the candidate is unable to 1 1
⇒ n ≤ ⇒ 2 ≥ 10 ⇒ n ≥ 4
n
solve. Then, X follows binomial distribution with 2 10
1
parameters n = 50 and p = . So, minimum number of times one has to toss a fair coin
5 so that the probability of observing atleast one head is
Now, according to binomial probability distribution atleast 90% is 4.
concept
r 50 − r 5. Let p and q represents the probability of success and
 1  4
P (X = r ) = 50C r     , r = 0, 1, ... , 50 failure in a trial respectively. Then,
 5  5 2 1 4 2
p = P (5 or 6) = = and q = 1 − p = = .
6 3 6 3
∴Required probability Now, as the man decides to throw the die either till he
= P (X < 2) = P (X = 0) + P (X = 1) gets a five or a six or to a maximum of three throws, so
 4
50
449  4
49
 4 50 54  4
49 he can get the success in first, second and third throw or
= 50C 0   + 50C1 =    +  =   not get the success in any of the three throws.
 5 (5)50  5 5 5  5  5
So, the expected gain/loss (in `)
2. Let for the given random variable ‘X’ the binomial = ( p × 100) + qp(− 50 + 100)
probability distribution have n-number of independent + q2p(− 50 − 50 + 100) + q3 (− 50 − 50 − 50)
trials and probability of success and failure are p and q 1   2 1  2  1
2
 2
3
respectively. According to the question, =  × 100 +  ×  (50) +     (0) +   (− 150)
3   3 3  3  3  3
Mean = np = 8 and variance = npq = 4
rankershalt.com

Probability 125

100 100 1200 10. India play 4 matches and getting at least 7 points. It can
= + +0−
3 9 27 only be possible in WWWD or WWWW position, where W
900 + 300 − 1200 1200 − 1200 represents two points and D represents one point.
= = =0
27 27 Therefore, the probability of the required event
6. The probability of hitting a target at least once = 4C3 (0.05) (0.5)3 + 4C 4 (0.5)4
= 1 − (probability of not hitting the target in any trial) = [4(0.05) + 0.5 ] (0.5)3 = 0.0875
= 1 − nC 0 p0qn
11. Let X be the number of coins showing heads. Let X be a
where n is the number of independent trials and p and q binomial variate with parameters n = 100 and p.
are the probability of success and failure respectively.
Since, P (X = 50) = P (X = 51)
[by using binomial distribution]
1 1 2 ⇒ 100
C50 p50 (1 − p)50 = 100C51 ( p)51 (1 − p)49
Here, p= and q =1− p =1− =
3 3 3 (100) ! (51 !) × (49 !) p p 51
0 n ⇒ ⋅ = ⇒ =
 1  2 5 (50 !) (50 !) 100 ! 1− p 1 − p 50
According to the question, 1 − nC 0     >
 3  3 6 51
n n ⇒ p=
 2 5  2 1 101
⇒   <1− ⇒   <
 3 6  3 6 12. For Binomial distribution, mean = np
Clearly, minimum value of n is 5.
and variance = npq
7. Let p = probability of getting an ace in a draw = ∴ np = 2 and npq = 1 [given]
probability of success ⇒ q = 1 / 2 and p + q = 1
and q = probability of not getting an ace in a draw = ⇒ p = 1 /2
probability of failure ∴ n = 4, p = q = 1 / 2
4 1
Then, p= = Now, P (X > 1) = 1 − { P (X = 0) + P (X = 1)}
52 13
0 4 1 3
1 12  1  1  1  1
and q =1 − p=1 − = = 1 − 4C 0     − 4C1    
13 13  2  2  2  2
Here, number of trials, n = 2 1 4 11
=1 − − =
Clearly, X follows binomial distribution with parameter 16 16 16
1 0 .1 0 .1 5
n = 2 and p = . 13. Probability (face 1) = = =
13
x 2− x
0 .1 + 0 .32 0 .42 21
 1   12
Now, P (X = x) = 2C x     , x = 0, 1, 2 14. Let E be the event that product of the two digits is 18,
 13  13
therefore required numbers are 29 , 36, 63 and 92.
∴ P (X = 1) + P (X = 2)
4
1
 1   12
2
 1   12
0 Hence, p = P (E ) =
= 2C1     + 2C 2    100
 13  13  13  13
and probability of non-occurrence of E is
 12  1
=2  + q = 1 − P (E ) = 1 −
4
=
96
 169 169
100 100
24 1 25
= + = Out of the four numbers selected, the probability that
169 169 169
the event E occurs atleast 3 times, is given as
8. Given box contains 15 green and 10 yellow balls. P = 4C3 p3 q + 4C 4 p4
∴Total number of balls = 15 + 10 = 25 3 4
 4   96   4  97
15 3 =4     +  = 4
P(green balls) = = = p = Probability of success  100  100  100 25
25 5
10 2 15. Since, set A contains n elements. So, it has 2n subsets.
P(yellow balls) = = = q = Probability of unsuccess
25 5 ∴ Set P can be chosen in 2n ways, similarly set Q can be
and n = 10 = Number of trials. chosen in 2n ways.
3 2 12 ∴ P and Q can be chosen in (2n )(2n ) = 4n ways.
∴Variance = npq = 10 × × =
5 5 5
1 Suppose, P contains r elements, where r varies from 0 to
9. Probability of guessing a correct answer, p = and n. Then, P can be chosen in nC r ways, for 0 to be disjoint
3
from A, it should be chosen from the set of all subsets of
probability of guessing a wrong answer, q = 2 /3
set consisting of remaining (n − r ) elements. This can be
∴ The probability of guessing a 4 or more correct done in 2n − r ways.
4 5
 1 2  1 2 1 11 ∴ P and Q can be chosen in nC r ⋅ 2n − r ways.
answers = 5C 4   ⋅ + 5C5   = 5 ⋅ 5 + 5 = 5
 3 3  3 3 3 3
rankershalt.com

126 Probability

But, r can vary from 0 to n. Clearly, P1 > P2. Therefore, first option i.e. ‘best of 3
∴ Total number of disjoint sets P and Q games’ has higher probability of winning the match.
n
17. The man will be one step away from the starting point,
= ∑ nC r2n − r = (1 + 2)n = 3n if
r=0
n (i) either he is one step ahead or (ii) one step behind the
3 n  3 starting point.
Hence, required probability = = 
4 n  4 The man will be one step ahead at the end of eleven
16. Case I When A plays 3 games against B. steps, if he moves six steps forward and five steps
backward. The probability of this event is
In this case, we have n = 3, p = 0.4 and q = 0.6 11
C 6 (0.4)6 (0.6)5 .
Let X denote the number of wins. Then,
The man will be one step behind at the end of eleven
P (X = r ) = 3C r (0.4)r (0.6)3 − r; r = 0, 1, 2, 3
steps, if he moves six steps backward and five steps
∴ P1 = probability of winning the best of 3 games forward. The probability of this event is 11C 6 (0.6)6 (0.4)5 .
= P (X ≥ 2) ∴ Required probability
= P (X = 2) + P (X = 3) = 11C 6 (0.4)6 (0.6)5 + C 6 (0.6)6 (0.4)5 = 11C 6 (0.24)5
11

= 3C 2(0.4)2(0.6)1 + 3C3 (0.4)3 (0.6)0 18. Using Binomial distribution,


= 0.288 + 0.064 = 0.352 P (X ≥ 2) = 1 − P (X = 0) − P (X = 1)
Case II When A plays 5 games against B. n  n −1
 1  1  1
In this case, we have = 1 −   −  nC1 ⋅   ⋅   
 2  2  2
 
n = 5, p = 0.4 and q = 0.6
1 1  1 + n 
Let X denotes the number of wins in 5 games. = 1 − n − nC1 ⋅ n = 1 −  n 
2 2  2 
Then,
P (X = r ) = 5C r (0.4)r (0.6)5 − r , where r = 0, 1, 2K ,5 Given, P (X ≥ 2) ≥ 0. 96
(n + 1) 24
∴ P2 = probability of winning the best of 5 games ∴ 1− ≥
2n 25
= P (X ≥ 3)
n+1 1
= P (X = 3) + P (X = 4) + P (X = 5) ⇒ ≤
2n 25
= 5C3 (0.4)3 (0.6)2 + 5C 4 (0.4)4 (0.6) + 5C5 (0.4 )5 (0.6)0
∴ n =8
= 0.2304 + 0.0768 + 0.1024 = 0.31744

Download Chapter Test


http://tinyurl.com/y4kwqyyl or
rankershalt.com

7
Matrices and Determinants

Topic 1 Types of Matrices, Addition, Subtraction,


Multiplication and Transpose of a Matrix
Objective Question I (Only one correct option) 1 0 0
1. If A is a symmetric matrix and B is a skew-symmetric 6. Let P =  4 1 0 and I be the identity matrix of order 3.
2 3  16 4 1
matrix such that A + B =  , then AB is equal to
5 −1 q + q32
If Q = [qij ] is a matrix, such that P50 − Q = I, then 31
(2019 Main, 12 April I) q21
 −4 −2 4 −2 equals (2016 Adv.)
(a)   (b) 
 −1 4   −1 −4 (a) 52 (b) 103
(c) 201 (d) 205
 4 −2  −4 2
(c)   (d)  1 2 2
 1 −4 1 4
7. If A = 2 1 −2 is a matrix satisfying the equation
 0 2y 1 
 c 2 b 
2. The total number of matrices A = 2x y −1 ,
AA = 9 I, where, I is 3 × 3 identity matrix, then the
T
2x − y 1  ordered pair (a, b) is equal to (2015 Main)
(x, y ∈ R, x ≠ y) for which AT A = 3I3 is (a) (2, − 1) (b) (−2, 1)
(2019 Main, 9 April II) (c) (2, 1) (d) (−2, − 1)
(a) 2 (b) 4 (c) 3 (d) 6  3 /2 1 /2  1 1
cos α − sin α  8. If P =  , A =   and Q = PAP , then
T
3. Let A= , (α ∈ R) such that  −1 /2 3 / 2 0 1
sin α cos α  T 2005
P Q P is
(2005, 1M)
0 −1
A32 =  . Then, a value of α is  1 2005  1 2005
(a)  (b) 
1 0  (2019 Main, 8 April I) 0 1   2005 1 
π π π
(a) (b) 0 (c) (d)  1 0  1 0
32 64 16 (c)   (d)  
 2005 1  0 1
1 0 0
4. Let P = 3 1 0 and Q = [qij ] be two 3 × 3 matrices 9. If A = α 0
1
and B = 
1 0
1
, then value of α for which
1 5
9 3 1 A 2 = B, is (2003, 1M)
q + q31
such that Q − P = I3 . Then, 21
5
is equal to (a) 1 (b) –1
q32 (2019 Main, 12 Jan I) (c) 4 (d) no real values
(a) 10 (b) 135 (c) 9 (d) 15 10. If A and B are square matrices of equal degree, then which
0 2q r  one is correct among the following? (1995, 2M)
5. Let A =  p q −r . If AAT = I3 , then| p|is (a) A + B = B + A
(b) A + B = A − B
 p − q r  (2019 Main, 11 Jan I) (c) A − B = B − A
1 1 1 1
(a) (b) (c) (d) (d) AB = BA
5 2 3 6
rankershalt.com

128 Matrices and Determinants

Objective Question II 16. Let ω be a solution of x3 − 1 = 0 with Im (ω ) > 0. If a = 2


One or more than one correct option) with b and c satisfying Eq. (i) then the value of
3 1 3
+ + is
11. Let X and Y be two arbitrary, 3 × 3, non-zero, ω a ωb ω c
skew-symmetric matrices and Z be an arbitrary, 3 × 3, (a) − 2 (b) 2
non-zero, symmetric matrix. Then, which of the (c) 3 (d) − 3
following matrices is/are skew-symmetric?
Passage II
(2015 Adv.)
Let p be an odd prime number and T p be the following set
(a)Y Z − Z Y
3 4 4 3
(b) X + Y 44 44
of 2 × 2 matrices
(c) X 4 Z 3 − Z 3 X 4 (d) X 23 + Y 23
 
Tp =  A = 
a b
; a , b, c ∈ { 0, 1, 2, K , p − 1}
12. For 3 × 3 matrices M and N , which of the following   c a  (2010)
statement(s) is/are not correct ? (2013 Adv.)
T
17. The number of A in T p such that det (A) is not divisible
(a) N M N is symmetric or skew-symmetric, according as
M is symmetric or skew-symmetric by p, is
(b) MN − NM is symmetric for all symmetric matrices M (a) 2 p 2 (b) p3 − 5 p
and N (c) p3 − 3 p (d) p3 − p 2
(c) M N is symmetric for all symmetric matrices M and N 18. The number of A in T p such that the trace of A is not
(d) (adj M ) (adj N ) = adj (MN ) for all invertible matrices M divisible by p but det ( A ) is divisible by p is
and N (a) ( p − 1) ( p 2 − p + 1) (b) p3 − ( p − 1)2
13. Let ω be a complex cube root of unity with ω ≠ 0 and (c) ( p − 1)2 (d) ( p − 1) ( p 2 − 2)
P = [ pij ] be an n × n matrix with pij = ωi+ j . Then, P 2 ≠ 0 19. The number of A in T p such that A is either symmetric or
when n is equal to (2013 Adv.)
skew-symmetric or both and det (A) is divisible by p is
(a) 57 (b) 55 (c) 58 (d) 56
(a) ( p − 1)2 (b) 2 ( p − 1)
(c) ( p − 1)2 + 1 (d) 2 p − 1
Passage Based Problems NOTE The trace of a matrix is the sum of its diagonal entries.
Passage I
Let a , b and c be three real numbers satisfying Analytical and Descriptive Questions
1 9 7 a  b c
20. If matrix A =  b  , where a , b, c are real
[a b c] 8 2 7 = [0 0 0]
c a
...(i)  c 
  a b
7 3 7 (2011) positive numbers, abc = 1 and AT A = I , then find the
14. If the point P (a , b, c), with reference to Eq. (i), lies on value of a3 + b3 + c3 . (2003, 2M)

the plane 2x + y + z = 1, then the value of 7a + b + c is


(a) 0 (b) 12 (c) 7 (d) 6
Integer Type Question
15. Let b = 6, with a and c satisfying Eq. (i). If α and β are −1 + 3 i
21. Let z = , where i = −1, and r , s ∈ {1, 2, 3}. Let
the roots of the quadratic equation ax + bx + c = 0, then
2 2
∞ n (− z ) z 
r 2s
 1 1 P =  2s  and I be the identity matrix of order 2.
∑  α + β is equal to  z zr 
n= 0
6 Then, the total number of ordered pairs (r , s) for which
(a) 6 (b) 7 (c) (d) ∞ P 2 = − I is (2016 Adv.)
7

Topic 2 Properties of Determinants


Objective Questions I (Only one correct option) 2. The sum of the real roots of the equation
1. A value of θ ∈ (0, π / 3), for which x −6 −1
2 − 3x x − 3 = 0, is equal to
1 + cos 2 θ sin 2 θ 4 cos 6θ
− 3 2x x + 2 (2019 Main, 10 April II)
cos θ
2
1 + sin 2 θ 4 cos 6θ = 0, is
(a) 0 (b) − 4
cos 2 θ sin 2 θ 1 + 4 cos 6θ
(2019 Main, 12 April II) (c) 6 (d) 1
π π 7π 7π
(a) (b) (c) (d)
9 18 24 36
rankershalt.com

Matrices and Determinants 129

x sin θ cos θ Then, the number of elements in S, is (2019 Main, 10 Jan II)
3. If ∆1 = − sin θ −x 1 (a) 4 (b) 2
cos θ 1 x (c) 10 (d) infinitely many
2 b 1
x sin 2θ cos 2θ 
10. Let A = b b + 1 b, where b > 0. Then, the minimum
2
and ∆ 2 = − sin 2θ −x 1 , x ≠ 0,
1 b 2 
cos 2θ 1 x
det ( A )
 π value of is
then for all θ ∈ 0,  b (2019 Main, 10 Jan II)
 2 (2019 Main, 10 April I) (a) − 3 (b) −2 3 (c) 2 3 (d) 3
(a) ∆1 + ∆ 2 = − 2(x3 + x − 1) 11. Let d ∈ R, and
(b) ∆1 − ∆ 2 = − 2x3  −2 4+ d (sin θ ) − 2 
(c) ∆1 + ∆ 2 = − 2x3
(d) ∆1 − ∆ 2 = x(cos 2θ − cos 4θ) A =  1 (sin θ ) + 2 d  , θ ∈ [θ , 2π ]. If

1 1 1 2 1 3 1 n − 1 1 78  5 (2 sin θ ) − d (− sin θ ) + 2 + 2d 
4. If  . .  ...  = ,then the
0 1 0 1 0 1 0 1  0 1  the minimum value of det(A) is 8, then a value of d is
(2019 Main, 10 Jan I)
1 n 
inverse of   is (a) −5 (b) −7 (c) 2( 2 + 1) (d) 2( 2 + 2)
0 1  (2019 Main, 9 April I)
x − 4 2x 2x 
 1 0  1 −13  1 0  1 −12 
(a)   (b)   (c) 13 1 (d)   12. If 2x x − 4 2x  = ( A + Bx)(x − A )2, then the
12 1  0 1    0 1   
 2x 2x x − 4
5. Let α and β be the roots of the equation x2 + x + 1 = 0. ordered pair ( A , B) is equal to (2018 Main)
Then, for y ≠ 0 in R, (a) (−4, − 5) (b) (−4, 3) (c) (−4, 5) (d) (4, 5)
y+1 α β 13. Let ω be a complex number such that 2ω + 1 = z, where
α y+β 1 is equal to 1 1 1
β 1 y+α (2019 Main, 9 April I) z = − 3. If 1 −ω 2 − 1 ω 2 = 3 k, then k is equal to
(a) y( y2 − 1) (b) y ( y2 − 3) (c) y3 − 1 (d) y3 1 ω2 ω7 (2017 Main)
1 1 1 (a) − z (b) z (c) − 1 (d) 1
6. Let the numbers 2, b, c be in an AP and A = 2 b c . 14. If α, β ≠ 0 and f (n ) = α + β andn n

4 b2 c2
3 1 + f (1) 1 + f (2)
If det( A ) ∈ [2, 16], then c lies in the interval 1 + f (1) 1 + f (2) 1 + f (3)
(2019 Main, 8 April II)
1 + f (2) 1 + f (3) 1 + f (4)
(a) [3, 2 + 23 / 4 ] (b) (2 + 23 / 4 , 4) (c) [4, 6] (d) [2, 3)
 1 sin θ 1  = K (1 − α )2(1 − β )2 (α − β )2, then K is equal to (2014 Main)
 3π 5π 
7. If A = − sin θ sin θ ; then for all θ ∈ 
1
1 , , (a) αβ (b) (c) 1 (d) −1
 4 4 αβ
 − 1 − sin θ 1 
15. Let P = [aij ] be a 3 × 3 matrix and let Q = [bij ], where
det( A ) lies in the interval (2019 Main, 12 Jan II)
bij = 2i + j aij for 1 ≤ i , j ≤ 3. If the determinant of P is 2,
(a)  , 3 (b)  , 4 (c)  0,  (d)  1, 
3 5 3 5 then the determinant of the matrix Q is (2012)
 2   2   2   2  (a) 210 (b) 211 (c) 212 (d) 213
a−b−c
16. If A = α 2
2a 2a
and| A3| = 125, then the value of α is
8. If 2b b−c−a 2b 2 α
  (2004, 1M)
2c 2c c−a −b (a) ± 1 (b) ± 2 (c) ± 3 (d) ± 5
= (a + b + c) (x + a + b + c)2, x ≠ 0 and a + b + c ≠ 0, then 17. The number of distinct real roots of (2001, 1M)
x is equal to (2019 Main, 11 Jan II)
 sin x cos x cos x  π π
(a) − (a + b + c) (b) − 2(a + b + c) sin x cos x  = 0 in the interval − ≤ x ≤ is
 cos x
(c) 2(a + b + c) (d) abc  cos x cos x sin x  4 4

9. Let a1 , a 2, a3 ..... , a10 be in GP with ai > 0 for (a) 0 (b) 2


i = 1, 2, ..... ,10 and S be the set of pairs (r , k), r , k ∈ N (c) 1 (d) 3
(the set of natural numbers) for which  1 x x+1 
18. If f (x) =  2x x (x − 1) (x + 1) x ,
log e a1r a 2k log e a 2ra3k log e a3r a 4k
 3x (x − 1) x (x − 1) (x − 2) (x + 1) x (x − 1) 
log e a 4r a5k log e a5r a 6k log e a 6r a7k =0
then f (100) is equal to (1999, 2M)
log e a7r a 8k log e a 8r a 9k log e a 9r k
a10
(a) 0 (b) 1 (c) 100 (d) –100
rankershalt.com

130 Matrices and Determinants

19. The parameter on which the value of the determinant Numerical Value
 1 a a2 
 
26. Let P be a matrix of order 3 × 3 such that all the entries in
 cos ( p − d ) x cos px cos ( p + d ) x  P are from the set { − 1, 0, 1}. Then, the maximum possible
sin ( p − d ) x sin px sin ( p + d ) x 
value of the determinant of P is ......... .
does not depend upon, is (1997, 2M)
(a) a (b) p (c) d (d) x Fill in the Blanks
xp + y x y 27. For positive numbers x, y and z, the numerical value of the
20. The determinant yp + z y z = 0, if 1 log x y log x z
0 xp + y yp + z (1997C, 2M) determinant log y x 1 log y z is…… .
(a) x, y, z are in AP (b) x, y, z are in GP log z x log z y 1 (1993, 2M)
(c) x, y, z are in HP (d) xy, yz , zx are in AP 1 a − bc
2
a
21. Consider the set A of all determinants of order 3 with 28. The value of the determinant 1 b b2 − ca is … .
entries 0 or 1 only. Let B be the subset of A consisting  

1 c c2 − ab 
of all determinants with value 1. Let C be the subset
(1988, 2M)
of A consisting of all determinants with value –1.
Then, x 3 7
(a) C is empty (1981, 2M)
29. Given that x = − 9 is a root of 2 x 2  = 0, the other two
(b) B has as many elements as C
7 6 x
(c) A = B ∪ C roots are... and... . (1983, 2M)

(d) B has twice as many elements as C 1 4 20 


30. The solution set of the equation
1 − 2 5 
 = 0 is… .
Objective Question II 1 2x 5x2
(1981, 2M)
(One or more than one correct option)  λ2 + 3λ λ − 1 λ + 3
22. Which of the following is(are) NOT the square of a 31. Let pλ + qλ + rλ + sλ + t = 
4 3 2
 λ+1 − 2λ λ − 4 

3 × 3 matrix with real entries? (2017 Adv.)  λ −3 λ + 4 3λ 
1 0 0  1 0 0  be an identity in λ , where p,q,r,s and t are constants.
(a)  0 1 0  (b)  0 − 1 0  Then, the value of t is…. .
    (1981, 2M)
 0 0 −1  0 0 −1
− 1 0 0   1 0 0 True/False
(c)  0 − 1 0  (d)  0 1 0 1 a a2
    1 a bc
 0 0 −1  0 0 1 32. The determinants 1 b ca and 1 b b2 are not
23. Which of the following values of α satisfy the equation 1 c ab 1 c c2
(1 + α )2 (1 + 2α )2 (1 + 3 α )2
identically equal. (1983, 1M)
(2 + α )2 (2 + 2α )2 (2 + 3 α )2 = − 648 α ?
(3 + α )2 (3 + 2α )2 (3 + 3 α )2 Analytical and Descriptive Questions
(2015 Adv.)
(a) −4 (b) 9 (c) −9 (d) 4 33. If M is a 3 × 3 matrix, where M T M = I and det (M ) = 1,
24. Let M and N be two 3 × 3 matrices such that then prove that det (M − I ) = 0 (2004, 2M)
MN = NM . Further, if M ≠ N 2 and M 2 = N 4, then 34. Let a , b, c be real numbers with a + b + c = 1 . Show
2 2 2
(2014 Adv.)
that the equation
(a) determinant of (M + MN ) is 0
2 2

(b) there is a 3 × 3 non-zero matrix U such that  ax − by − c bx + ay cx + a 


(M 2 + MN 2 ) U is zero matrix  bx + ay − ax + by − c cy + b  = 0 represents a
(c) determinant of (M 2 + MN 2 ) ≥ 1  cx + a cy + b − ax − by + c 
(d) for a 3 × 3 matrix U, if (M 2 + MN 2 ) U equals the zero straight line. (2001, 6M)
matrix, then U is the zero matrix
35. Prove that for all values of θ
 a b aα + b
25. The determinant  b c bα + c  sin θ cos θ sin 2θ
aα + b bα + c 0 
 2π   2π   4π 
is equal to zero, then (1986, 2M) sin θ +  cos θ +  sin 2θ +  =0
 3  3  3
(a) a, b, c are in AP
 2π   2π   4π 
(b) a, b, c are in GP sin θ −  cos θ −  sin 2 θ − 
 3  3  3 (2000, 3M)
(c) a, b, c are in HP
(d) (x − α ) is a factor of ax2 + 2bx + c
rankershalt.com

Matrices and Determinants 131

36. Suppose, f (x) is a function satisfying the following  A 3 6


conditions  8 9 C is divisible by k. (1990, 4M)
2 B 2 
(a) f (0) = 2, f (1) = 1
(b) f has a minimum value at x = 5 / 2, and  a −1 n 6 
2ax − 1 2ax + b + 43. Let ∆ a =  4n − 2 
 2ax  (a − 1)3
1 2
2n 2 
(c) for all x, f ′ (x) =  b b+ 1 −1   (a − 1) 3n3 3n 2 − 3n 
 2(ax + b) 2ax + 2b + 1 2ax + b  n
where a, b are some constants. Determine the constants
a, b and the function f (x). (1998, 3M)
Show that ∑ ∆ a = c ∈ constant. (1989, 5M)
a =1
bc ca ab
44. Show that
37. Find the value of the determinant p q r , where  xC x+1 x+ 2
r
x
Cr + 1 x
Cr + 2   xCr Cr + 1 C r + 2
1 1 1  yC y
Cr + 1 y
C r + 2 =  y C r y+1
Cr + 1 C r + 2
y+ 2
a , b and c are respectively the pth , qth and rth terms of a z r z z
 z z +1 z+2

C Cr + 1 Cr + 2   Cr Cr + 1 C r + 2
harmonic progression. (1997C, 2M)  r
(1985, 3M)
38. Let a > 0, d > 0. Find the value of the determinant
1 1 1 45. If α be a repeated root of a quadratic equation f (x) = 0
and A (x), B (x) and C (x) be polynomials of degree 3, 4
a a (a + d ) (a + d ) (a + 2d )
1 1 1 and 5 respectively, then show that
(a + d ) (a + d ) (a + 2d ) (a + 2d ) (a + 3d )  A (x) B (x) C (x) 
1 1 1  A (α ) B (α ) C (α ) 
 A′ (α ) B′ (α ) C′ (α ) 
(a + 2d ) (a + 2d ) (a + 3d ) (a + 3d ) (a + 4d )
is divisible by f (x), where prime denotes the
39. For all values of A , B, C and P , Q , R, show that derivatives. (1984, 3M)
(1994, 4M)
 cos ( A − P ) cos ( A − Q ) cos ( A − R)  46. Without expanding a determinant at any stage, show

 cos (B − P ) cos (B − Q ) cos (B − R) = 0 that
 cos (C − P ) cos (C − Q ) cos (C − R) 
 x2 + x x+1 x−2 
40. For a fixed positive integer n, if  2x2 + 3x − 1 3x 3x − 3  = xA + B
 2
(n + 1)! (n + 2)!  2x − 1 2x − 1 
 n!  x + 2x + 3 
D =  (n + 1)! (n + 2)! (n + 3)! ,
 (n + 2)! (n + 3)! (n + 4)!  where A and B are determinants of order 3 not
involving x. (1982, 5M)
 D 
then show that  3
− 4 is divisible by n. (1992, 4M) 47. Let a, b, c be positive and not all equal. Show that the
 (n !) 
 a b c
p b c value of the determinant  b c a is negative.
41. If a ≠ p, b ≠ q, c ≠ r and a q c = 0  c a b (1981, 4M)
a b r
Integer Type Question
p q r
Then, find the value of + + . (1991, 4M)
p−a q−b r−c 48. The total number of distincts x ∈ R for which
x x2 1 + x3
42. Let the three digit numbers A28, 3B9 and 62C, where A,
B and C are integers between 0 and 9, be divisible by a 2x 4x2
1 + 8x3 = 10 is (2016 Adv.)
fixed integer k. Show that the determinant 3x 9x2 1 + 27x3

Topic 3 Adjoint and Inverse of a Matrix


Objective Questions I (Only one correct option) et e− t cos t
 t
2. If A = e −e cos t − e−t sin t
−t
5 2α 1  et 2e− t sin t
1. If B = 0 2 1  is the inverse of a 3 × 3 matrix A, then 
e− t sin t 
α 3 −1 −t −t 
the sum of all values of α for which det ( A ) + 1 = 0, is − e sin t + e cos t  then A is
(2019 Main, 12 April I)
−t
−2e cos t 

(a) 0 (b) −1 (c) 1 (d) 2 (2019 Main, 9 Jan II)
rankershalt.com

132 Matrices and Determinants

(a) invertible only when t = π each of a , b and c is either ω or ω 2. Then, the number of
(b) invertible for every t ∈ R distinct matrices in the set S is (2011)
(c) not invertible for any t ∈ R (a) 2 (b) 6 (c) 4 (d) 8
π
(d) invertible only when t = 11. Let M and N be two 3 × 3 non-singular skew-symmetric
2
matrices such that MN = NM . If PT denotes the
3. Let A and B be two invertible matrices of order 3 × 3. If
transpose of P, then M 2N 2(M T N )−1 (MN −1 )T is equal to
det( ABAT ) = 8 and det( AB− 1 ) = 8, then det(BA − 1BT ) is (a) M 2 (b) −N 2 (c) −M 2 (d) MN (2011)
equal to (2019 Main, 11 Jan II)
1 1 1 0 0
(a) 1 (b)
4
(c)
16
(d) 16
12. If A = 0 1 1, 6 A −1 = A 2 + cA + dI , then (c, d ) is
cos θ − sin θ  0 −2 4 (2005, 1M)
4. If A =   , then the matrix
sin θ cos θ  (a) (− 6, 11) (b) (− 11, 6)
π (c) (11, 6) (d) (6, 11)
A −50 when θ = , is equal to
12 (2019 Main, 9 Jan I)
 1 3  3 1 Objective Questions II
 − 
(a)  2 2  
(b)  2 2 (One or more than one correct option)
 
3 1 1 3
−    3 −1 −2
 2 2   2 
13. Let P = 2 0 α , where α ∈ R. Suppose Q = [qij ] is a
2
 3 1   1 3
 − 3 −5 0 
(c)  2 2  
(d)  2 2 
  matrix such that PQ = kI , where k ∈ R, k ≠ 0 and I is the
1 3 3 1
−   
 2 2   2 2  k k2
identity matrix of order 3. If q23 = − and det (Q ) = ,
 2 −3  8 2
5. If A =   , then adj (3 A + 12 A ) is equal to
2
then (2016 Adv.)
 −4 1  (2017 Main) (a) α = 0, k = 8 (b) 4α − k + 8 = 0
 72 − 84  51 63 (c) det (P adj (Q )) = 29 (d) det (Q adj (P )) = 213
(a)   (b)  
 − 63 51   84 72 14. Let M be a 2 × 2 symmetric matrix with integer entries.
 51 84  72 − 63
(c)   (d)   Then, M is invertible, if (2014 Adv.)
 63 72  − 84 51  (a) the first column of M is the transpose of the second row
5a − b of M
6. If A =  and A adj A = AAT , then 5a + b is equal
3 2  (b) the second row of M is the transpose of the first column
of M
to (2016 Main) (c) M is a diagonal matrix with non-zero entries in the
(a) − 1 (b) 5 (c) 4 (d) 13 main digonal
7. If A is a 3 × 3 non-singular matrix such that AAT = AT A (d) the product of entries in the main diagonal of M is not
the square of an integer
and B = A −1 AT , then BBT is equal to (2014 Main)
(a) I + B (b) I (c) B −1 (d) (B −1 )T 1 4 4 
1 α 3 15. If the adjoint of a 3 × 3 matrix P is 2 1 7, then the
8. If P = 1 3 3 is the adjoint of a 3 × 3 matrix A and 1 1 3 
possible value(s) of the determinant of P is/are
2 4 4
(a) − 2 (b) − 1
| A | = 4 , then α is equal to (2013 Main)
(c) 1 (d) 2
(a) 4 (b) 11 (c) 5 (d) 0
9. If P is a 3 × 3 matrix such that P = 2P + I, where PT is
T
Integer Answer Type Question
the transpose of P and I is the 3 × 3 identity matrix, then 16. Let k be a positive real number and let
 x  0
 2k − 1 2 k 2 k 
there exists a column matrix, X =  y ≠ 0 such that  
 z  0 A= 2 k 1 − 2k  and
(2012)
 − 2 k 2k − 1 
 0 
(a) PX =  0 (b) PX = X (c) PX = 2X (d) PX = − X  0 2k − 1 k 
   
 0 B =  1 − 2k 0 2 k 
10. Let ω ≠ 1 be a cube root of unity and S be the set of all  − k −2 k 0 
 
 1 a b
If det (adj A ) + det (adj B) = 106, then [k] is equal to……
non-singular matrices of the form  ω 1 c , where (2010)
ω 2 ω 1 
rankershalt.com

Matrices and Determinants 133

Topic 4 Solving System of Equations


Objective Questions I (Only one correct option)
1. If [x] denotes the greatest integer ≤ x , then the has a non-trivial solution, is (2019 Main, 8 April I)
1
system of liner equations [sin θ ]x + [− cos θ ] y = 0, (a) −1 (b) (c) 2 (d) 0
2
[cot θ ]x + y = 0
(2019 Main, 12 April II) 7. The set of all values of λ for which the system of linear
π 2π  equations x − 2 y − 2z = λx,x + 2 y + z = λyand − x − y = λz
(a) have infinitely many solutions if θ ∈  , 
2 3 has a non-trivial solution (2019 Main, 12 Jan II)
7π 
and has a unique solution if θ ∈  π , . (a) contains exactly two elements.
 6 (b) contains more than two elements.
(b) has a unique solution if (c) is a singleton.
 π 2π   7π  (d) is an empty set.
θ∈ ,  ∪ π, 
2 3   6 8. An ordered pair (α , β) for which the system of linear
 π 2π  equations (2019 Main, 12 Jan I)
(c) has a unique solution if θ ∈  , 
2 3  (1 + α )x + βy + z = 2
 7π  αx + (1 + β ) y + z = 3
and have infinitely many solutions if θ ∈  π , 
 6 ax + βy + 2z = 2
(d) have infinitely many solutions if has a unique solution, is
(a) (2, 4) (b) (− 4, 2) (c) (1, − 3) (d) (−3, 1)
 π 2π   7π 
θ ∈  ,  ∪ π,  9. If the system of linear equations
2 3   6
2x + 2 y + 3z = a
2. Let λ be a real number for which the system of linear 3x − y + 5z = b
equations
x − 3 y + 2z = c
x + y + z = 6, 4x + λy − λz = λ − 2 and
where a , b, c are non-zero real numbers, has more than
3x + 2 y − 4z = − 5 one solution, then (2019 Main, 11 Jan I)
has infinitely many solutions. Then λ is a root of the (a) b − c − a = 0 (b) a + b + c = 0
quadratic equation (2019 Main, 10 April II) (c) b − c + a = 0 (d) b + c − a = 0
(a) λ2 − 3λ − 4 = 0 (b) λ2 + 3λ − 4 = 0
10. The number of values of θ ∈ (0, π ) for which the system of
(c) λ2 − λ − 6 = 0 (d) λ2 + λ − 6 = 0
linear equations
3. If the system of linear equations x + 3 y + 7z = 0,
x+ y+ z =5 − x + 4 y + 7z = 0,
x + 2 y + 2z = 6 (sin 3θ )x + (cos 2θ ) y + 2z = 0
x + 3 y + λz = µ,(λ , µ ∈ R), has infinitely many has a non-trivial solution, is (2019 Main, 10 Jan II)
solutions, then the value of λ + µ is (a) two (b) three (c) four (d) one
(2019 Main, 10 April I) 11. If the system of equations
(a) 7 (b) 12 x+ y+z=5 x + 2 y + 3z = 9
(c) 10 (d) 9
x + 3 y + αz = β
4. If the system of equations 2x + 3 y − z = 0, x + ky − 2z = 0 has infinitely many solutions, then β − α equals
and 2x − y + z = 0 has a non-trivial solution (x, y, z ), (2019 Main, 10 Jan I)
x y z (a) 8 (b) 18 (c) 21 (d) 5
then + + + k is equal to
y z x (2019 Main, 9 April II)
1 1 3
(a) −4 (b) (c) − (d) 12. If the system of linear equations
2 4 4
x − 4 y + 7z = g, 3 y − 5z = h, − 2x + 5 y − 9z = k
5. If the system of linear equations is consistent, then (2019 Main, 9 Jan II)
x − 2 y + kz = 1 , 2x + y + z = 2 ,3x − y − kz = 3 (a) 2 g + h + k = 0 (b) g + 2h + k = 0
has a solution (x, y, z ), z ≠ 0, then (x, y) lies on the (c) g + h + k = 0 (d) g + h + 2k = 0
straight line whose equation is (2019 Main, 8 April II) 13. The system of linear equations
(a) 3x − 4 y − 4 = 0 (b) 3x − 4 y − 1 = 0 x + y + z = 2, 2x + 3 y + 2z = 5
(c) 4x − 3 y − 4 = 0 (d) 4x − 3 y − 1 = 0
2x + 3 y + ( a 2 − 1)z = a + 1 (2019 Main, 9 Jan I)
6. The greatest value of c ∈ R for which the system of (a) has infinitely many solutions for a = 4
linear equations x −cy − cz = 0, cx − y + cz = 0, (b) is inconsistent when a = 4
cx + cy − z = 0 (c) has a unique solution for|a| = 3
(d) is inconsistent when|a| = 3
rankershalt.com

134 Matrices and Determinants

14. If the system of linear equations (b) Statement I is true, Statement II is also true;
x + ky + 3z = 0, 3x + ky − 2z = 0 Statement II is not the correct explanation of
Statement I
2x + 4 y − 3z = 0 (c) Statement I is true; Statement II is false.
xz
has a non-zero solution ( x , y , z ), then is equal to (d) Statement I is false; Statement II is true.
y2 (2018 Main) 23. Consider the system of equations x − 2 y + 3 z = −1 ,
(a) −10 (b) 10 (c) −30 (d) 30
x − 3 y + 4z = 1 and − x + y − 2z = k
15. The system of linear equations Statement I The system of equations has no solution
x + λy − z = 0; λx − y − z = 0; x + y − λz = 0 for k ≠ 3 and
has a non-trivial solution for (2016 Main) 1 3 −1
(a) infinitely many values of λ (b) exactly one value of λ Statement II The determinant −1 −2 k ≠ 0 , for
(c) exactly two values of λ (d) exactly three values of λ 1 4 1
16. The set of all values of λ for which the system of linear k ≠ 0. (2008, 3M)
equations 2x1 − 2x2 + x3 = λx1, 2x1 − 3x2 + 2x3 = λx2 and
− x1 + 2x2 = λx3 has a non-trivial solution (2015 Main) Objective Questions II (Only or More Than One)
(a) is an empty set  b1 
(b) is a singleton set 24. Let S be the set of all column matrices  b2  such that b1,
(c) contains two elements
b3 
(d) contains more than two elements
b2, b3 ∈R and the system of equations (in real variables)
17. The number of value of k, for which the system of
equation − x + 2 y + 5z = b1
(k + 1) x + 8 y = 4 y ⇒ kx + (k + 3) y = 3k − 1 2x − 4 y + 3z = b2
(2013 Main) x − 2 y + 2z = b3
has no solution, is has at least one solution. Then, which of the following
(a) infinite (b) 1 (c) 2 (d) 3 system(s) (in real variables) has (have) at least one
 b1 
18. The number of 3 × 3 matrices A whose entries are either solution for each  b2  ∈ S ?
 x  1  
0 or 1 and for which the system A  y = 0 has exactly  b3 
 z  0 (a) x + 2 y + 3z = b1 , 4 y + 5z = b2 and x + 2 y + 6z = b3
(b) x + y + 3z = b1 , 5x + 2 y + 6z = b2 and − 2x − y − 3z = b3
two distinct solutions, is (2010)
(c) − x + 2 y − 5z = b1 , 2x − 4 y + 10z = b2 and x − 2 y + 5z = b3
(a) 0 (b) 29 − 1 (c) 168 (d) 2 (d) x + 2 y + 5z = b1 , 2x + 3z = b2 and x + 4 y − 5z = b3
19. Given, 2x − y + 2z = 2, x − 2 y + z = − 4, x + y + λz = 4,
then the value of λ such that the given system of Fill in the Blank
equations has no solution, is (2004, 1M) 25. The system of equations λx + y + z = 0 , − x + λy + z = 0
(a) 3 (b) 1 (c) 0 (d) –3 and − x − y + λz = 0 will have a non-zero solution, if real
20. The number of values of k for which the system of values of λ are given by ... (1982, 2M)
equations (k + 1) x + 8 y = 4k and kx + (k + 3) y = 3k − 1
has infinitely many solutions, is/are (2002, 1M)
Analytical and Descriptive Questions
(a) 0 (b) 1 (c) 2 (d) ∞ a 0 1 a 1 1 f  a 2
26. A = 1 c b, B = 0 c , U =  g , V = 0 
d
21. If the system of equations x + ay = 0, az + y = 0 and 1 d b  f h 
g h  0 
ax + z = 0 has infinite solutions, then the value of a is  
(a) –1 (b) 1 If there is a vector matrix X, such that AX = U has
(c) 0 (d) no real values infinitely many solutions, then prove that BX = V
cannot have a unique solution. If a f d ≠ 0. Then, prove
22. If the system of equations x − ky − z = 0, kx − y − z = 0, that BX = V has no solution. (2004, 4M)
x + y − z = 0 has a non-zero solution, then possible
values of k are (2000, 2M)
27. Let λ and α be real. Find the set of all values of λ for
which the system of linear equations
(a) –1, 2 (b) 1, 2 (c) 0, 1 (d) –1, 1
λx + (sin α ) y + (cos α )z = 0,
Assertion and Reason x + (cos α ) y + (sin α )z = 0
and − x + (sin α ) y − (cos α )z = 0
For the following questions, choose the correct answer from
has a non-trivial solution.
the codes (a), (b), (c) and (d) defined as follows.
For λ = 1, find all values of α. (1993, 5M)
(a) Statement I is true, Statement II is also true;
Statement II is the correct explanation of Statement I
rankershalt.com

Matrices and Determinants 135

28. Let α 1 , α 2, β1 , β 2 be the roots of ax2 + bx + c = 0 and 33. Given, x = cy + bz , y = az + cx, z = bx + ay, where x, y,
px2 + qx + r = 0, respectively. If the system of equations z are not all zero, prove that a 2 + b2 + c2 + 2ab = 1.
α 1 y + α 2z = 0 and β1 y + β 2z = 0 has a non-trivial solution, (1978, 2M)
b2 ac Integer Answer Type Question
then prove that 2 = . (1987, 3M)
q pr
34. For a real number α , if the system
29. Consider the system of linear equations in x, y, z  1 α α 2  x   1 
(sin 3θ ) x − y + z = 0, (cos 2θ ) x + 4 y + 3z = 0 and     
 α 1 α   y = −1
2x + 7 y + 7z = 0 α 2 α 1   z   1 
Find the values of θ for which this system has non-trivial     
solution. (1986, 5M) of linear equations, has infinitely many solutions,
then 1 + α + α 2 = (2017 Adv.)
30. Show that the system of equations, 3x − y + 4z = 3,
x + 2 y − 3z = − 2 and 6x + 5 y + λz = − 3 has atleast one 0  −1 
solution for any real number λ ≠ − 5. Find the set of 35. Let M be a 3 × 3 matrix satisfying M 1 =  2 ,
solutions, if λ = − 5. (1983, 5M) 0  3 
31. For what values of m, does the system of equations 1 1 1  0 
3x + my = m and 2x − 5 y = 20 has a solution satisfying the M −1 =  1 , and M 1 =  0  ,
   
conditions x > 0, y > 0? (1979, 3M )
 0  −1 1 12
32. For what value of k, does the following system of equations
Then, the sum of the diagonal entries of M is …
possess a non-trivial solution over the set of rationals
(2011)
x + y − 2z = 0, 2x − 3 y + z = 0, and x − 5 y + 4z = k
Find all the solutions. (1979, 3M )

Answers
Topic 1 Topic 3
1. (b) 2. (b) 3. (c) 4. (a) 1. (c) 2. (b) 3. (c) 4. (c)
5. (b) 6. (b) 7. (d) 8. (a) 5. (b) 6. (b) 7. (b) 8. (b)
9. (d) 10. (a) 11. (c, d) 12. (c, d) 9. (d) 10. (a) 11. (c) 12. (a)
13. (b, c, d) 14. (d) 15. (b) 16. (a) 13. (b,c) 14. (c, d) 15. (a,d) 16. (4)
17. (d) 18. (c) 19. (d) 20. (4)
21. (1) Topic 4
1. (a) 2. (c) 3. (c) 4. (b)
Topic 2 5. (c) 6. (b) 7. (c) 8. (a)
1. (a) 2. (a) 3. (c) 4. (b) 9. (a) 10. (a) 11. (a) 12. (a)
5. (d) 6. (c) 7. (a) 8. (b) 13. (d) 14. (b) 15. (d) 16. (c)
9. (d) 10. (c) 11. (a) 12. (c) 17. (d) 18. (a) 19. (b) 20. (b)
21. (a) 22. (d) 23. (a) 24. (a,d)
13. (a) 14. (c) 15. (d) 16. (c) π
17. (c) 18. (a) 19. (b) 20. (b) 25. λ=0 27. − 2 ≤ λ ≤ 2 , α = nπ , nπ +
4
21. (b) 22. (a, c) 23. (b, c) 24. (a, b) n π
29. θ = nπ , nπ + ( −1 ) , n ∈ Z
25. (b,d) 26. (4) 27. (0) 28. (0) 6
29. (2 and 7) 30. {–1,2} 31. (0) 32. False 4 − 5k 13k − 9
30. x= ,y = ,z = k
 1 5 1 2 5  7 7
36. a = , b = − and f ( x ) = x − x + 2 37. (0)
 4 4 4 4  15
31. m < − or m > 30
 4d 4  2
38.   41. (2)
 a (a + d ) 2(a + 2d ) 3(a + 3d ) 2(a + 4d ) 32. (k = 0, the given system has infinitely many solutions)
48. (2) 34. (1) 35. (9)
rankershalt.com

136 Matrices and Determinants

Hints & Solutions


Topic 1 Types of Matrices, Addition, cos α − sin α 
3. Given, matrix A =  
Subtraction and Transpose of a sin α cos α 
Matrix cos α − sin α cos α − sin α 
∴ A2 = 
1. Given matrix A is a symmetric and matrix B is a sin α cos α sin α cos α 
skew-symmetric.  cos2 α − sin 2 α − cosα sin α − sin α cosα 
= 
∴ A = A and B = − B
T T  sin α cosα + cosα sin α − sin 2 α + cos2 α 
2 3  cos 2 α − sin 2 α 
Since, A + B =   (given)… (i) = 
5 − 1 sin 2 α cos 2 α 
On taking transpose both sides, we get Similarly,
T cos(nα ) − sin(nα )
2 3  An =  , n ∈ N
( A + B)T =   sin(nα ) cos(nα ) 
5 − 1
cos(32 α ) − sin(32 α ) 0 −1
2 5  ⇒ A32 =  =  (given)
⇒ A +B = sin(32 α ) cos(32 α )  1 0 
T T
 … (ii)
3 − 1 So, cos(32 α ) = 0 and sin(32 α ) = 1
Given, AT = A and BT = − B π π
⇒ 32 α = ⇒ α =
2 5  2 64
⇒ A−B= 
3 − 1 4. Given matrix
On solving Eqs. (i) and (ii), we get 1 0 0 0 0 0 1 0 0
2 4  0 − 1 P = 3 1 0 = 3 0 0 + 0 1 0
 
A=  and B = 1 0 
4 − 1    9 3 1 9 3 0 0 0 1
2 4  0 − 1  4 − 2 ⇒ P = X + I (let)
So, AB =   =  Now, P5 = (I + X )5
4 − 1 1 0  − 1 − 4
= I + 5C1 (X ) + 5C 2(X 2) + 5C3 (X 3 ) + …
2. Given matrix [Q I = I , I ⋅ A = A and (a + x)n = nC 0a n +
n

 0 2y 1  n
C1a n − 1x + ...+ nC nxn]
A = 2x y −1 , (x, y ∈ R, x ≠ y) 0 0 0 0 0 0 0 0 0
2x − y 1  Here, X = 3 0 0 3 0 0 = 0 0 0
2

for which 9 3 0 9 3 0 9 0 0


AT A = 3I3 0 0 0 0 0 0 0 0 0
and X 3 = X 2 ⋅ X = 0 0 0 3 0 0 = 0 0 0
 0 2x 2x   0 2 y 1  3 0 0
⇒ 2 y y − y 2x y −1 = 0 3 0 9 0 0 9 3 0 0 0 0
    
0 0 0
 1 −1 1  2x − y 1  0 0 3
⇒ X = X = 0 0 0
4 5
8x2 0 0 3 0 0
  0 0 0
⇒  0 6 y2 0 = 0 3 0 0 0 0 0 0 0
 0 0 3 0 0 3
 So, P = I + 5 3 0 0 + 10 0 0 0
5  

Here, two matrices are equal, therefore equating the 9 3 0 9 0 0
corresponding elements, we get  1 0 0
8x2 = 3 and 6 y2 = 3 =  15 1 0
3 135 15 1
⇒ x=±
8  2 0 0
1 and Q = I + P5 =  15 2 0 = [q ij ]
and y=±
2 135 15 2
Q There are 2 different values of x and y each. ⇒ q21 = 15, q31 = 135 and q 32 = 15
So, 4 matrices are possible such that AT A = 3I3 . q + q31 15 + 135 150
Hence, 21 = = = 10
q32 15 15
rankershalt.com

Matrices and Determinants 137

5. Given, AAT = I q31 + q32 20400 + 200 20600


Thus, = = = 103
0 2q r   0 p p  1 0 0 q21 200 200
⇒  p q − r  2q q − q = 0 1 0 1 2 2 1 2 a 
 p − q r   r − r r  0 0 1 7. Given, A = 2 1 −2, AT = 2 1 2  and
0 + 4q2 + r 2 0 + 2q2 − r 2 0 − 2q2 + r 2  1 0 0 a 2 b  2 −2 b 
 
⇒ 0 + 2q2 − r 2 p2 + q2 + r 2 p2 − q2 − r 2  = 0 1 0 1 2 2  1 2 a
0 − 2q2 + r 2 p2 − q2 − r 2 p2 + q2 + r 2 0 0 1
    AAT = 2 1 −2 2 1 2 
We know that, if two matrices are equal, then a 2 b  2 −2 b 
corresponding elements are also equal, so  9 0 a + 4 + 2b 
4q2 + r 2 = 1 = p2 + q2 + r 2, …(i) 
= 0 9 2a + 2 − 2b
2q2 − r 2 = 0 ⇒ r 2 = 2q2 …(ii)
a + 4 + 2b 2a + 2 − 2b a 2 + 4 + b2 
and p2 − q 2 − r 2 = 0 …(iii)
Using Eqs. (ii) and (iii), we get It is given that, AAT = 9I
p2 = 3 q 2 …(iv)  9 0 a + 4 + 2b  1 0 0
Using Eqs. (ii) and (iv) in Eq. (i), we get ⇒  0 9 2a + 2 − 2b = 9 0 1 0
4q2 + 2q2 = 1 a + 4 + 2b 2a + 2 − 2b a 2 + 4 + b2  0 0 1
⇒ 6q2 = 1
 9 0 a + 4 + 2b  9 0 0
⇒ 2 p2 = 1  2a + 2 − 2b = 0 9 0
[using Eq. (iv)]
⇒ 0 9
1 1
p = ⇒ | p| =
2
a + 4 + 2b 2a + 2 − 2b a + 4 + b  0 0 9
2 2
2 2
1 0 0 On comparing, we get
6. Here, P =  4 1 0 a + 4 + 2b = 0 ⇒ a + 2b = − 4 ...(i)
16 4 1 2a + 2 − 2b = 0 ⇒ a − b = − 1 …(ii)
1 0 0  1 0 0  1 0 0 and a 2 + 4 + b2 = 9 …(iii)
∴ P 2 =  4 1 0  4 1 0 =  4 + 4 1 0 On solving Eqs. (i) and (ii), we get
16 4 1 16 4 1 16 + 32 4 + 4 1 a = − 2, b = − 1
 1 0 0 This satisfies Eq. (iii)

=  4 ×2 0
1 …(i) Hence, (a , b) ≡ (−2,−1)
16 (1 + 2) 4 × 2 1  3 /2 −1 /2   3 /2 1 /2 
8. Now, PT P =   
 1 0 0  1 0 0  1 /2 3 /2  −1 /2 3 /2

and P =  4 × 2
3
1 0  4 1 0 1 0
16 (1 + 2) 4 × 2 1 16 4 1 ⇒ PT P =  
0 1
 1 0 0
⇒ PT P = I
=  4 ×3 1 0 ...(ii)
⇒ PT = P −1
16 (1 + 2 + 3) 4 × 3 1
Since, Q = PAPT
From symmetry,
∴ PTQ 2005 P = PT [(PAPT )(PAPT ) K 2005 times ] P
 1 0 0
= (PT P ) A (PT P ) A (PT P ) K (PT P ) A (PT P )
P50 =  4 × 50 1 0 144444444 42444444444 3
2005 times
16 (1 + 2 + 3 + ... + 50) 4 × 50 1 = IA 2005 = A 2005
Q P50 − Q = I [given] 1 1
  ∴ A= 
 1 − q11 − q12 − q13  1 0 0 0 1
∴  200 − q21 1 − q22 − q23  = 0 1 0 1 1 1 1 1 2
  A2 =  =
16 × 50
(51) − q31 200 − q32 1 − q33  0 0 1 0 1 0 1 0 1
 2 
16 × 50 × 51 1 2 1 1 1 3
A3 =  =
⇒ 200 − q21 = 0,
2
− q31 = 0,
0 1 0 1 0 1
200 − q32 = 0 ……………………
∴ q21 = 200, q32 = 200, q31 = 20400 ……………………
rankershalt.com

138 Matrices and Determinants

1 2005 p p12  ω 2 ω3  ω 2 1 
A 2005 =  P = [ pij ]2× 2 =  11 = 3 =
0 1  4
 p21 p22 ω ω   1 ω 

1 2005 ω 2 1  ω 2 1 
∴ PTQ 2005 P =  P2 =   
0 1   1 ω  1 ω
ω 4 + 1 ω 2 + ω 
9. Given, A = α 0
1
, B = 
1 0
1 ⇒ P2 =  2 2
≠0
1 5 ω + ω 1 + ω 
α 0  α 0  α 2 0 
⇒ A 2 =  = When n = 3
1 1 1 1 α + 1 1
ω 2 ω3 ω 4  ω 2 1 ω 
Also, given, A = B 2    
P = [ pij ]3 × 3 = ω3 ω 4 ω5  =  1 ω ω 2
 2 0 1 0
⇒ α =
ω 4 ω5 ω 6   ω ω 2 1 
   
α + 1 1 5 1
ω 2 1 ω  ω 2 1 ω  0 0 0
⇒ α 2 = 1 and α + 1 = 5   
Which is not possible at the same time. P =  1 ω ω 2  1 ω ω 2 = 0 0 0 = 0
2

 ω ω 2 1   ω ω 2 1  0 0 0
∴ No real values of α exists.     

10. If A and B are square matrices of equal degree, then ∴ P 2 = 0, when n is a multiple of 3.
A+ B=B+ A P 2 ≠ 0, when n is not a multiple of 3.
11. Given, X T = − X , Y T = − Y , Z T = Z ⇒ n = 57 is not possible.

(a) Let P =Y Z − Z Y 3 4 4 3 ∴ n = 55, 58, 56 is possible.


Then, P = (Y 3 Z 4 )T − (Z 4 Y 3 )T
T
14. As (a , b, c) lies on 2x + y + z = 1 ⇒ 2a + b + c = 1
= (Z T )4 (Y T )3 − (Y T )3 (Z T )4 ⇒ 2a + 6a − 7a = 1
= − Z 4Y 3 + Y 3 Z 4 = P
⇒ a = 1, b = 6, c = − 7
∴ P is symmetric matrix.
∴ 7a + b + c = 7 + 6 − 7 = 6
(b) Let P = X 44 + Y 44
Then, PT = (X T )44 + (Y T )44 15. If b = 6 ⇒ a = 1 and c = − 7
= X 44 + Y 44 = P ∴ ax2 + bx + c = 0 ⇒ x2 + 6x − 7 = 0
∴ P is symmetric matrix.
⇒ (x + 7) (x − 1) = 0
(c) Let P = X 4Z 3 − Z 3 X 4
∴ x = 1, − 7
Then, PT = (X 4Z 3 )T − (Z 3 X 4 )T ∞ n 2
 1 1 6  6 1
= (Z T )3 (X T )4 − (X T )4 (Z T )3 ⇒∑  −  =1 + +   + L+ ∞ =
 1 7  7  7  6
= Z 3 X 4 − X 4Z 3 = − P n= 0 1−
∴ P is skew-symmetric matrix. 7
1
(d) Let P = X 23 + Y 23 = =7
1 /7
Then, PT = (X T )23 + (Y T )23 = − X 23 − Y 23 = − P
16. If a = 2, b = 12, c = − 14
∴ P is skew-symmetric matrix.
3 1 3
12. (a) (N T MN )T = N T M T (N T )T = N T M T N , is symmetric ∴ + +
ω a ωb ω c
if M is symmetric and skew-symmetric, if M is
3 1 3 3
skew-symmetric. ⇒ + 12 + −14 = 2 + 1 + 3ω 2 = 3ω + 1 + 3ω 2
(b) (MN − NM )T = (MN )T − (NM )T ω 2
ω ω ω
= NM − MN = − (MN − NM ) = 1 + 3 (ω + ω 2) = 1 − 3 = − 2
∴ Skew-symmetric, when M and N are symmetric. 17. The number of matrices for which p does not divide
(c) (MN )T = N T M T = NM ≠ MN Tr ( A ) = ( p − 1) p2 of these ( p − 1)2 are such that p
∴ Not correct. divides| A|. The number of matrices for which p divides
(d) (adj MN ) = (adj N ) ⋅ (adj M ) Tr ( A ) and p does not divides| A|are ( p − 1)2.
∴ Not correct. ∴ Required number = ( p − 1) p2 − ( p − 1)2 + ( p − 1)2
13. Here, P = [ pij ]n × n with pij = wi + j = p3 − p2
∴ When n = 1 18. Trace of A = 2a, will be divisible by p, iff a = 0.
P = [ pij ]1 × 1 = [ω ] 2 | A|= a 2 − bc, for (a 2 − bc) to be divisible by p. There are
⇒ P 2 = [ω 4 ] ≠ 0 exactly ( p − 1) ordered pairs (b, c) for any value of a.
∴ When n =2 ∴ Required number is ( p − 1)2.
rankershalt.com

Matrices and Determinants 139

a b Also, ω 2r + ω 4s = − 1
19. Given, A =  , a, b, c ∈ {0, 1, 2 ,... , p − 1}
c a  If r = 1, then ω 2 + ω 4s = − 1
If A is skew-symmetric matrix, then a = 0, b = − c which is only possible, when s = 1.
∴ | A|= − b2 As, ω2 + ω4 = − 1
Thus, P divides| A|, only when b = 0. ...(i) ∴ r = 1, s = 1
Again, if A is symmetric matrix, then b = c and Again, if r = 3, then
| A|= a 2 − b2 ω 6 + ω 4s = − 1
Thus, p divides| A|, if either p divides (a − b) or p ⇒ ω 4 s = −2 [never possible]
divides (a + b). ∴ r ≠3
p divides (a − b), only when a = b, ⇒ (r , s) = (1, 1) is the only solution.
i.e. a = b ∈ {0, 1, 2 ,... , ( p − 1)} Hence, the total number of ordered pairs is 1.
i.e. p choices ...(ii) Topic 2 Properties of Determinants
p divides (a + b). 1 + cos 2 θ sin 2 θ 4 cos 6 θ
⇒ p choices, including a = b = 0 included in Eq. (i). 1. Let ∆ = cos θ2
1 + sin 2 θ 4 cos 6 θ =0
∴ Total number of choices are ( p + p − 1) = 2 p − 1 cos 2 θ sin 2 θ 1 + 4 cos 6 θ
a b c
20. Given, A = b c a , abc = 1 and AT A = I …(i) Applying C1 → C1 + C 2, we get
c a b  2 sin 2 θ 4 cos 6 θ
Now, AT A = I ∆ = 2 1 + sin 2 θ 4 cos 6 θ =0
a b c  a b c  1 0 0 1 sin θ
2
1 + 4 cos 6 θ
⇒ b c a  b c a  = 0 1 0
c a b  c a b  0 0 1 Applying R1 → R1 − 2R3 and R2 → R2 − 2R3 , we get

 a 2 + b2 + c2 ab + bc + ca 0 − sin 2 θ − 2 − 4 cos 6 θ
ab + bc + ca 
⇒ ab + bc + ca a 2 + b2 + c2 ab + bc + ca  ∆ = 0 1 − sin 2 θ − 2 − 4 cos 6 θ = 0
ab + bc + ca ab + bc + ca 
 a 2 + b2 + c2  1 sin 2 θ 1 + 4 cos 6 θ
1 0 0 On expanding w.r.t. C1, we get
= 0 1 0 ⇒ sin 2 θ (2 + 4 cos 6 θ ) + (2 + 4 cos 6 θ ) (1 − sin 2 θ ) = 0
0 0 1 1 2π
⇒ 2 + 4 cos 6 θ = 0 ⇒ cos 6 θ = − = cos
⇒ a 2 + b2 + c2 = 1 and ab + bc + ca = 0 …(ii) 2 3
We know, a3 + b3 + c3 − 3abc 2π π   π 
⇒ 6θ = ⇒θ = Q θ ∈ 0, 3  
= (a + b + c)(a 2 + b2 + c2 − ab − bc − ca ) 3 9  
⇒ a + b + c = (a + b + c) (1 − 0) + 3
3 3 3
2. Given equation
[from Eqs. (i) and (ii)] x −6 −1
∴ a3 + b3 + c3 = (a + b + c) + 3 …(iii) 2 − 3x x − 3 = 0
Now, (a + b + c)2 = a 2 + b2 + c2 + 2(ab + bc + ca ) − 3 2x x + 2
=1 …(iv) On expansion of determinant along R1, we get
From Eq. (iii), a + b + c = 1 + 3 ⇒ a + b + c = 4
3 3 3 3 3 3 x [(− 3x) (x + 2) − 2x(x − 3)] + 6 [2(x + 2) + 3(x − 3)]
− 1 [2(2x) − (− 3x) (− 3)] = 0
−1 + i 3
21. Here, z= =ω ⇒ x [− 3x2 − 6x − 2x2 + 6x] + 6[2x + 4 + 3x − 9]
2
(−ω )r ω 2s  − 1 [4x − 9x] = 0
Q P =  2s  ⇒ x(− 5x2) + 6(5x − 5) − 1(− 5x) = 0
 ω ωr 
⇒ −5x3 + 30x − 30 + 5x = 0
(− ω )r ω 2s  (− ω )r ω 2s 
P 2 =  2s   ⇒ 5x3 − 35x + 30 = 0 ⇒ x3 − 7x + 6 = 0.
 ω ω r   ω 2s ωr  Since all roots are real
 ω 2r + ω 4s ω r + 2s [(− 1)r + 1] ∴ Sum of roots = −
coefficient of x2
=0
=  r + 2s  coefficient of x3
ω [(− 1) + 1] ω 4s + ω 2r
r

Given, P 2 = − I 3. Given determinants are
∴ ω 2r + ω 4s = − 1 and ω r + 2s [(− 1)r + 1] = 0 x sin θ cos θ
Since, r ∈{1, 2, 3} and (− 1)r + 1 = 0 ∆1 = − sin θ − x 1
⇒ r = {1, 3} cos θ 1 x
rankershalt.com

140 Matrices and Determinants

= − x3 + sin θ cos θ − sin θ cos θ + x cos 2 θ − x + x sin 2 θ On applying C 2 → C 2 − C1 and C3 → C3 − C1, we get
= − x3 y 0 0
x sin 2θ cos 2θ ∆ = α y + β −α 1 −α
and ∆ 2 = − sin 2θ −x 1 ,x≠0 β 1 −β y + α −β
cos 2θ 1 x = y[( y + (β − α )) ( y − (β − α )) − (1 − α ) (1 − β )]
= − x (similarly as ∆1)
3
[expanding along R1]
So, according to options, we get ∆1 + ∆ 2 = − 2x3 = y [ y − (β − α ) − (1 − α − β + αβ )]
2 2

4. Given = y [ y2 − β 2 − α 2 + 2αβ − 1 + (α + β ) − αβ ]
1 1 1 2 1 3 1 n − 1 1 78 = y [ y2 − (α + β )2 + 2αβ + 2αβ − 1 + (α + β ) − αβ ]
0 1 0 1 0 1 ... 0 =
     1  0 1 
= y[ y2 − 1 + 3 − 1 − 1] = y3 [Qα + β = −1 and αβ = 1]
1 1 1 2 1 2 + 1 1 1 1 
Q 0 1 0 1 = 0 1 
,
    6. Given, matrix A = 2 b c , so
1 2 + 1 1 3 1 3 + 2 + 1 4 b c 
2 2

0 1  0 1 = 0 ,
    1  1 1 1
: : : det( A ) = 2 b c
 2 2

: : : 4 b c 
1 1 1 2 1 3 1 n − 1
∴     ...  On applying, C 2 → C 2 − C1 and C3 → C3 − C1,
0 1 0 1 0 1 0 1 
1 0 0 
1 (n − 1) + (n − 2)+ ...+3 + 2 + 1 we get det( A ) = 2 b − 2 c − 2

=   
0 1  4 b − 4 c − 4
2 2

 n (n − 1)  1 78 b − 2 c − 2
=
1 = = 2 
2  b − 4 c − 4
2
0  0 1 
 1  b −2 c−2
Since, both matrices are equal, so equating =
(b − 2)(b + 2) (c − 2)(c + 2)
corresponding element, we get
 1 1 
n (n − 1) = (b − 2)(c − 2) 
= 78 ⇒ n (n − 1) = 156 b + 2 c + 2
2
[taking common (b − 2) from C1 and
= 13 × 12 = 13(13 − 1) (c − 2) from C 2]
⇒ n = 13 = (b − 2)(c − 2)(c − b)
1 13 −1 1 −13 Since, 2, b and c are in AP, if assume common difference
So, A=  = A = 0 1 
0 1    of AP is d, then
a b −1  d − b  b = 2 + d and c = 2 + 2d
[Q if|A|= 1and A =   , then A =  − c a  
 c d  
So, | A| = d (2d )d = 2d3 ∈ [2, 16] [given]
⇒ d ∈ [1, 8] ⇒ d ∈ [1, 2]
3

5. Given, quadratic equation is x2 + x + 1 = 0 having roots ∴ 2 + 2d ∈ [2 + 2, 2 + 4]


α , β.
= [4, 6] ⇒ c ∈ [4, 6]
Then, α + β = −1 and αβ = 1
 1 sin θ 1 
7. Given matrix A = − sin θ sin θ 
Now, given determinant
1
y+1 α β
 −1 − sin θ 1 
∆= α y+β 1 1 sin θ 1
β 1 y+α ⇒ det( A ) =| A|= − sin θ 1 sin θ
On applying R1 → R1 + R2 + R3 , we get −1 − sin θ 1
y+1+α +β y+1+α +β y+1+α +β = 1(1 + sin 2 θ ) − sin θ (− sin θ + sin θ ) + 1(sin 2 θ + 1)
∆= α y+β 1 ⇒| A| = 2 (1 + sin 2 θ ) …(i)
 3π 5π 
β 1 y+α As we know that, for θ ∈  , 
 4 4
y y y
 1 1
= α y+β 1 [Qα + β = −1] sin θ ∈  − , 
 2 2
β 1 y+α
rankershalt.com

Matrices and Determinants 141

 1  1   a r + kRr + 2k 
⇒ sin 2 θ ∈ 0,  ⇒ 1 + sin 2 θ ∈ 0 + 1, + 1 log e a r + kRk log e  
 2   
2  a r + k Rk 
 3   a r + k R4 r + 5 k 
⇒ 1 + sin 2 θ ∈ 1, 
 2 ⇒ log e a r + kR3 r + 4k log e  r + k 3 r + 4k 
a R 
3 
⇒ 2(1 + sin 2θ ) ∈ [2, 3) ⇒| A| ∈ [2, 3) ⊂  , 3  a r + kR7r + 8k 
2  log e a r + kR6r + 7k log e  r + k 6r + 7k 
a R 
a−b−c 2a 2a
 a r + kR2r + 3 k 
8. Let ∆ = 2b b−c−a 2b log e  
 a r + k Rk 
2c 2c c−a −b
 a r + kR5 r + 6k 
Applying R1 → R1 + R2 + R3 , we get log e  r + k 3 r + 4k  = 0
a+ b+ c a+ b+ c a+ b+ c a R 
∆= 2b b−c−a 2b  a r + k R8 r + 9 k 
log e  r + k 6r + 7k 
2c 2c c−a −b a R 
1 1 1 log e (a r + kRk ) log e Rr + k log e R2r + 2k
= (a + b + c) 2b b − c − a 2b ⇒ log e a r + kR3 r + 4k log e Rr + k log e R2r + 2k = 0
2c 2c c−a −b log e a r + kR6r + 7k log e Rr + k log e R2r + 2k
(taking common (a + b + c) fromR1)
log e (a r + kRk ) log e Rr + k 2 log e Rr + k
Applying C 2 → C 2 − C1 and C3 → C3 − C1 , we get
⇒ log e (a r + kR3 r + 4k ) log e Rr + k 2 log e Rr + k = 0
1 0 0
log e (a r + kR6r + 7k ) log e Rr + k 2 log e Rr + k
∆ = (a + b + c) 2b − (a + b + c) 0
2c 0 − (a + b + c) [Q log mn = n log m and here
log e R2r + 2k = log e R2( r + k) = 2 log e Rr + k ]
Now, expanding along R1, we get
Q Column C 2 and C3 are proportional,
∆ = (a + b + c) 1. {(a + b + c)2 − 0 }
= (a + b + c)3 = (a + b + c)(x + a + b + c)2 (given) So, value of determinant will be zero for any value of
⇒ (x + a + b + c)2 = (a + b + c)2 (r , k), r , k ∈ N .
⇒ x + a + b + c = ± (a + b + c) ∴Set ‘S’ has infinitely many elements.
⇒ x = − 2(a + b + c) [Q x ≠ 0] 2 b 1

log e a1ra 2k log e a 2ra3k log e a3r a 4k


10. Given matrix, A = b b2 + 1 b, b > 0
1 b 2 
9. Given, log e a 4r a5k log e a5r a 6k log e a 6r a7k =0
2 b 1
log e a7ra 8k log e a 8r a 9k log e a 9r a10
k
So, det ( A ) =| A| = b b2 + 1 b
On applying elementary operations 1 b 2
C 2 → C 2 − C1 and C3 → C3 − C1, we get
= 2 [2(b2 + 1) − b2] − b(2b − b)
log e a1ra 2k log e a 2ra3k − log e a1ra 2k
+1(b2 − b2 − 1)
log e a 4r a5k log e a5r a 6k − log e a 4r a5k = 2[2b + 2 − b ] − b − 1
2 2 2

log e a7ra 8k log e a 8r a 9k − log e a7ra 8k = 2b2 + 4 − b2 − 1 = b2 + 3


log e a3r a 4k − log e a1ra 2k det( A ) b2 + 3 3
⇒ = = b+
log e a 6r a7k − log e a 4r a5k =0 b b b
k
log e a 9r a10 − log e a7ra 8k Now, by AM ≥ GM, we get
3
b+ 1/ 2
 a ra k   arak  b ≥  b × 3 {Q b > 0 }
log e a1ra 2k log e  2r 3k  log e  3r 4k   
2  b
 a1 a 2   a1 a 2 
3
 a5r a 6k   arak  ⇒ b+ ≥2 3
⇒ log e a 4r a5k log e  r k  log e  6r 7k  = 0 b
 a 4a5   a 4a5  det (A )
So, minimum value of =2 3
 a 8r a 9k   a 9r a10
k  b
log e a7ra 8k log e  r k  log e  r k 
 a7 a 8   a7 a 8  11. Given,
  m   −2 4+ d (sin θ ) − 2 
Q log e m − log e n = log e  n    
  A =  1 (sin θ ) + 2 d 
[Q a1 , a 2, a3 ....... , a10 are in GP, therefore put  5 (2 sin θ ) − d (− sin θ ) + 2 + 2d 
a1 = a , a 2 = aR, a3 = aR2, ... , a10 = aR9 ]
rankershalt.com

142 Matrices and Determinants

−2 4+ d (sin θ ) − 2 1 1 1
∴ | A| = 1 (sin θ ) + 2 d ⇒ 1 ω ω 2 = 3k
5 (2 sin θ ) − d (− sin θ ) + 2 + 2d 1 ω2 ω
−2 4+ d (sin θ ) − 2 [Q 1 + ω + ω 2 = 0 and ω7 = (ω3 )2 ⋅ ω = ω]
= 1 (sin θ ) + 2 d On applying R1 → R1 + R2 + R3 , we get
1 0 0 3 1 + ω + ω2 1 + ω + ω2
(R3 → R3 − 2R2 + R1 ) 1 ω ω2 = 3k
= 1 [(4 + d )d − (sin θ + 2) (sin θ − 2)] 1 ω 2
ω
(expanding along R3 )
= (d 2 + 4d − sin 2 θ + 4) 3 0 0
= (d 2 + 4d + 4) − sin 2 θ ⇒ 1 ω ω 2 = 3k
= (d + 2)2 − sin 2 θ 1 ω2 ω
Note that| A|will be minimum if sin 2 θ is maximum i.e. if ⇒ 3(ω 2 − ω 4 ) = 3k
sin 2 θ takes value 1.
⇒ (ω 2 − ω ) = k
Q | A|min = 8,  − 1 − 3i   − 1 + 3i 
therefore (d + 2)2 − 1 = 8 ∴ k=  −  = − 3i = − z
 2   2 
⇒ (d + 2) = 9
2

⇒ d+2=±3 14. PLAN Use the property that, two determinants can be multiplied
column-to-row or row-to-column, to write the given
⇒ d = 1, − 5 determinant as the product of two determinants and then
expand.
12. Given,
x − 4 2x 2x  Given, f (n ) = α n + β n, f (1) = α + β, f (2) = α 2 + β 2,
 2x x − 4 2x  = ( A + Bx)(x − A )2 f (3) = α 3 + β3 , f (4) = α 4 + β 4
  3 1+ f (1) 1 + f (2)
 2x 2x x − 4
Let ∆ = 1 + f (1) 1 + f (2) 1 + f (3)
⇒ Apply C1 → C1 + C 2 + C3
1 + f (2) 1 + f (3) 1 + f (4)
 5x − 4 2x 2x 
 5x − 4 x − 4 2x  = ( A + Bx)(x − A )2 3 1 + α + β 1 + α 2 + β2
 
 5x − 4 2x x − 4 ⇒ ∆= 1+α +β 1 + α 2 + β 2 1 + α 3 + β3
Taking common (5x − 4) from C1, we get 1 + α 2 + β2 1 + α 3 + β3 1 + α 4 + β 4

1 2x 2x  1 ⋅1 + 1 ⋅1 + 1 ⋅1 1 ⋅1 + 1 ⋅α + 1 ⋅β
(5x − 4)1 x − 4 2x  = ( A + Bx)(x − A )2 = 1 ⋅1 + 1 ⋅α + 1 ⋅β 1 ⋅1 + α ⋅α + β ⋅β
 
1 2x x − 4 1 ⋅ 1 + 1 ⋅ α 2 + 1 ⋅ β2 1 ⋅ 1 + α 2 ⋅ α + β2 ⋅ β
Apply R2 → R2 − R1 and R3 → R3 − R1 1 ⋅ 1 + 1 ⋅ α 2 + 1 ⋅ β2
1 2x 0  1 ⋅ 1 + α ⋅ α 2 + β ⋅ β2
∴ (5x − 4)0 − x − 4 0  = ( A + Bx)(x − A )2 1 ⋅ 1 + α 2 ⋅ α 2 + β2 ⋅ β2
 
0 0 − x − 4 1 1 1 1 1 1 1 1 1
2

Expanding along C1, we get = 1 α β 1 α β = 1 α β


(5x − 4)(x + 4)2 = ( A + Bx)(x − A )2 1 α 2 β2 1 α 2 β2 1 α 2 β2
Equating, we get, A = − 4 and B = 5
On expanding, we get ∆ = (1 − α )2(1 − β )2(α − β )2
13. Given, 2ω + 1 = z
But given, ∆ = K (1 − α )2(1 − β )2(α − β )2
⇒ 2ω + 1 = − 3 [Q z = − 3]
− 1 + 3i Hence, K (1 − α )2(1 − β )2(α − β )2 = (1 − α )2(1 − β )2(α − β )2
⇒ ω= ∴ K =1
2
Since, ω is cube root of unity. 15. PLAN It is a simple question on scalar multiplication, i.e.
− 1 − 3i k a1 k a2 k a3 a1 a2 a3
∴ ω2 = and ω3 n = 1 b1 b 2 b 3 = k b1 b 2 b 3
2 c1 c 2 c 3 c1 c 2 c 3
1 1 1
Now, 1 − ω 2 − 1 ω 2 = 3k Description of Situation Construction of matrix,
1 ω2 ω7  a11 a12 a13 
i.e. if a = [aij ]3 × 3 =  a21 a22 a23 
 a31 a32 a33 
rankershalt.com

Matrices and Determinants 143

 a11 a12 a13   1 a a2 


Here, P = [aij ]3 × 3 =  a 21 a 22 a 23  19. Let ∆ = cos ( p − d ) x cos px cos ( p + d) x 
 a31 a32 a33   
sin ( p − d ) x sin px sin ( p + d) x 
 b11 b12 b13  Applying C1 → C1 + C3
Q = [bij ]3 × 3 =  b21 b22 b23 
 b31 b32 b33  1 + a2 a a2
⇒ ∆ = cos ( p − d ) x + cos ( p + d ) x cos px cos ( p + d )x
where, bij = 2i + j aij
sin( p − d ) x + sin( p + d ) x sin px sin ( p + d )x
4 a11 8 a12 16 a13
∴ Q = 8 a 21 16 a 22 32 a 23 1 + a2 a a2
16 a31 32 a32 64 a33 ⇒ ∆ = 2 cos px cos dx cos px cos ( p + d ) x
2 sin px cos dx sin px sin ( p + d ) x
a11 a12 a13
= 4 × 8 × 16 2 a 21 2 a 22 2 a 23 Applying C1 → C1 − 2 cos dx C 2
4 a31 4 a32 4 a33 1 + a 2 − 2a cos dx a a2 
⇒ ∆ =
 0 cos px cos ( p + d ) x

a11 a12 a13 0 sin px sin ( p + d ) x 
= 29 × 2 × 4 a 21 a 22 a 23 
a31 a32 a33 ⇒ ∆ = (1 + a 2 − 2a cos dx) [sin ( p + d ) x cos px
= 212 ⋅ P = 212 ⋅ 2 = 213 − sin px cos ( p + d ) x]
⇒ ∆ = (1 + a 2 − 2a cos dx) sin dx
16. We know, | A n| = | A|n
which is independent of p.
Since, | A3| = 125 ⇒ | A|3 = 125
xp + y x y
⇒ α 2 = 5 ⇒ α2 − 4 = 5 ⇒ α = ± 3 20. Given, yp + z y z =0
2 α 
0 xp + y yp + z
 sin x cos x cos x 
17. Given,  cos x sin x cos x  = 0 Applying C1 → C1 − ( p C 2 + C3 )
 cos x cos x sin x  0 x y
Applying C1 → C1 + C 2 + C3 ⇒ 0 y z =0
 sin x + 2 cos x cos x cos x  − (xp2 + yp + yp + z ) xp + y yp + z
=  sin x + 2 cos x sin x cos x 
 sin x + 2 cos x cos x sin x  ⇒ − (xp2 + 2 yp + z ) (xz − y2) = 0
1 cos x cos x  ∴ Either xp2 + 2 yp + z = 0 or y2 = xz
= (2 cos x + sin x) 1 sin x cos x  = 0 ⇒ x, y, z are in GP.
1 cos x sin x 
21. Since, A is the determinant of order 3 with entries 0 or
Applying R2 → R2 − R1 , R3 → R3 − R1 1 only.
1 cos x cos x  Also, B is the subset of A consisting of all
⇒ (2 cos x + sin x) 0 sin x − cos x 0 = 0 determinants with value 1.
0 0 sin x − cos x 
[since, if we interchange any two rows or columns,
⇒ (2 cos x + sin x) (sin x − cos x)2 = 0
then among themself sign changes]
⇒ 2 cos x + sin x = 0 or sin x − cos x = 0
⇒ 2 cos x = − sin x or sin x = cos x Given, C is the subset having determinant with
π π value −1.
⇒ cot x = − 1 / 2 gives no solution in − ≤ x ≤
4 4 ∴ B has as many elements as C.
and sin x = cos x ⇒ tan x = 1 ⇒ x = π /4 22. For a matrix to be square of other matrix its
18. Given, determinant should be positive.
 1 x x+1  (a) and (c) → Correct
f (x) = 2x x (x − 1) (x + 1) x  (b) and (d) → Incorrect
 3x (x − 1) x (x − 1) (x − 2) (x + 1) x (x − 1) 
23. Given determinant could be expressed as product of two
Applying C3 → C3 − (C1 + C 2) determinants.
 1 x 0 (1 + α )2 (1 + 2α )2 (1 + 3α )2
= 2x x (x − 1) 0 = 0 i.e. (2 + α )2 (2 + 2α )2 (2 + 3α )2 = − 648 α
 3x (x − 1) x (x − 1)(x − 2) 0
(3 + α )2 (3 + 2α )2 (3 + 3α )2
∴ f (x) = 0 ⇒ f (100) = 0
rankershalt.com

144 Matrices and Determinants

1 + 2α + α2 1 + 4α + 4α2 1 + 6α + 9 α2 Similar contradiction occurs when


⇒ 4 + 4α + α 2
4 + 8α + 4α 2
4 + 12 α + 9 α 2 a1 = 1, a 2 = 1, a3 = 1, b2c1 = b3 c1 = b1c2 = 1
and b3 c2 = b1c3 = b1c2 = − 1
9 + 6 α + α 2 9 + 12 α + 4 α 2 9 + 18 α + 9 α 2
Now, for value to be 5 one of the terms must be zero but
= − 648 α that will make 2 terms zero which means answer
1 α α2 1 1 1 cannot be 5
⇒ 4 2 α α 2 ⋅ 2 4 6 = − 648 α 1 1 1
9 3α α2 1 4 9 Now, −1 1 1 =4
1 −1 1
1 1 1 1 1 1
Hence, maximum value is 4.
⇒ α 3 4 2 1 ⋅ 2 4 6 = − 648 α
1 log x y log x z
9 3 1 1 4 9
27. Let ∆ = log y x 1 log y z
⇒ −8 α 3 = − 648 α
log z x log z y 1
⇒ α − 81α = 0 ⇒ α (α 2 − 81) = 0
3

∴ α = 0, ± 9 1
log y log z
log x log x
24. PLAN (i) If A and B are two non-zero matrices and AB = BA, then
( A − B)( A + B) = A 2 − B 2. log x log z
= 1
(ii) The determinant of the product of the matrices is equal to log y log y
product of their individual determinants, i.e. | A B | = | A || B |. log x log y
1
Given, M 2 = N 4 ⇒ M2 − N 4 = 0 log z log z
⇒ (M − N 2) (M + N 2) = 0 [as MN = NM ] On dividing and multiplying R1 , R2, R3 by log x,
Also, M ≠ N2 log y, log z, respectively.
⇒ M + N2 =0 log x log y log z
1
⇒ det (M + N ) = 0 2 = log x log y log z = 0
log x log y log z
Also, det (M 2 + MN 2) = (det M) (det M + N 2) log x log y log z
= (det M) (0) = 0 1 a a 2 − bc 1 a a2 1 a bc
As, det (M + MN ) = 0
2 2
28. 1 b b − ca = 1 b b2 − 1 b ca
2

Thus, there exists a non-zero matrix U such that 1 c c2 − ab 1 c c2 1 c ab


(M 2 + MN 2) U = 0
aα + b 1 a bc a a 2 abc
 a b 1
25. Given,  b c bα + c  = 0 Now, 1 b ca = b b2 abc
abc
 aα + b bα + c 0  1 c ab c c2 abc
Applying C3 → C3 − (α C1 + C 2)
Applying R1 → aR1 , R2 → bR2, R3 → cR3
 a b 0 
  a a2 1 1 a a2
 b c 0 = 0 1
 aα + b b α + c − (aα 2
+ 2 b α + c) = ⋅ abc b b2 1 = 1 b b2
abc
c c2 1 1 c c2
⇒ − (aα 2 + 2bα + c) (ac − b2) = 0
⇒ aα 2 + 2bα + c = 0 or b2 = ac 1 a a 2 − bc
⇒ x − α is a factor of ax2 + 2bx + c or a , b, c are in GP. ∴ 1 b b2 − ca = 0
a1 b1 c1 1 c c2 − ab
26. Let Det (P ) = a 2 b2 c2 x 3 7
a3 b3 c3 29. Given, 2 x 2 =0
= a1 (b2c3 − b3 c2) − a 2 (b1c3 − b3 c1 ) + a3 (b1c2 − b2c1 ) 7 6 x
Now, maximum value of Det (P ) = 6
Applying R1 → R1 + R2 + R3
If a1 = 1, a 2 = − 1, a3 = 1, b2c3 = b1c3 = b1c2 = 1
x+9 x+9 x+9 1 1 1
and b3 c2 = b3 c1 = b2c1 = − 1
⇒ 2 x 2 = 0 ⇒ (x + 9) 2 x 2 = 0
But it is not possible as
7 6 x 7 6 x
(b2c3 ) (b3 c1 ) (b1c2) = − 1 and (b1c3 ) (b3 c2) (b2c1 ) = 1
i.e., b1b2b3 c1c2c3 = 1 and − 1 Applying C 2 → C 2 − C1 and C3 → C3 − C1
rankershalt.com

Matrices and Determinants 145

1 0 0 a 2x − aby − ac bx + ay cx + a
1
⇒ (x + 9) 2 x − 2 0 = 0 ⇒ (x + 9) (x − 2) (x − 7) = 0 ⇒ abx + a 2y − ax + by − c cy + b =0
a cy + b − ax − by + c
7 −1 x − 7 acx + a 2

⇒ x = − 9, 2, 7 are the roots. Applying C1 → C1 + bC 2 + cC3


∴ Other two roots are 2 and 7.  (a 2 + b2 + c2) x ay + bx cx + a 
1 2
1 4 20 ⇒ (a + b2 + c2) y by − c − ax b + cy  = 0
a b + cy c − ax − by 
 a +b +c
2 2 2
30. Given, 1 −2 5 = 0 
1 2x 5x2 ay + bx cx + a 
1x
⇒  y by − c − ax b + cy  = 0
⇒ 1 (− 10 x − 10x) − 4 (5x − 5) + 20 (2x + 2) = 0
2 2
a 1 b + cy c − ax − by 
⇒ −30x2 + 30x + 60 = 0
[Q a 2 + b2 + c2 = 1]
⇒ (x − 2) (x + 1) = 0
Applying C 2 → C 2 − bC1 and C3 → C3 − cC1
⇒ x = 2, − 1
1x ay a 
Hence, the solution set is { −1, 2}. ⇒  y − c − ax b = 0
a 1 cy − ax − by 
λ 2 + 3λ λ −1 λ + 3
31. Given, λ + 1 −2 λ λ − 4 x2 axy ax
1
λ −3 λ + 4 3λ ⇒ y − c − ax b =0
ax
1 cy − ax − by
= pλ 4 + qλ 3 + rλ 2 + sλ + t
Thus, the value of t is obtained by putting λ = 0. Applying R1 → R1 + yR2 + R3
 2 
0 −1 3 1 x + y +1
2
0 0
⇒ 1 0 −4 = t ⇒ 
 y − c − ax b 
= 0
ax 1 cy − ax − by
−3 4 0  
1
⇒ t =0 ⇒ [(x2 + y2 + 1) {(− c − ax)(− ax − by) − b(cy)}] = 0
ax
[Q determinants of odd order skew-symmetric matrix 1
is zero] ⇒ [(x2 + y2 + 1) (acx + bcy + a 2x2 + abxy − bcy)] = 0
ax
 1 a bc  a a 2 abc 1
1 ⇒ [(x2 + y2 + 1) (acx + a 2x2 + abxy)] = 0
32. Let ∆ =  1 b ca  = b b2 abc ax
  abc 1
 1 c ab  c c2 abc ⇒ [ax(x2 + y2 + 1) (c + ax + by)] = 0
ax
Applying R1 → aR1 , R2 → bR2, R3 → cR3 ⇒ (x2 + y2 + 1)(ax + by + c) = 0
a a2 1 1 a a2 ⇒ ax + by + c = 0
1 which represents a straight line.
= ⋅ abc b b2 1 = 1 b b2
abc sin θ cos θ sin 2θ
c c2 1 1 c c2
 2π   2π   4π 
sin θ +  cos θ +  sin 2θ + 
1 a bc 1 a a2  3   3   3
35. Let ∆ =
∴ 1 b ca = 1 b b2
 2π   2π   4π 
1 c ab 1 c c2 sin θ −  cos θ −  sin 2θ − 
 3   3  3
Hence, statement is false. Applying R2 → R2 + R3
33. Since, M T M = I and|M | = 1
∴ |M − I | = |IM − M T M | [Q IM = M ] sin θ cos θ sin 2θ
 2π   2π   4π 
⇒ |M − I | = |(I − M )M |= |(I − M ) ||M |= | I − M |
T T
sin θ +  cos θ +  sin 2θ + 
 3  3  3
= (− 1)3 |M − I |[Q I − M is a 3 × 3 matrix] =
 2π   2π   4π 
+ sin θ −  + cos θ −  + sin 2θ − 
= − |M − I |  3  3  3
⇒ 2|M − I| = 0  2π   2π   4π 
sin θ −  cos θ −  sin 2θ − 
⇒ |M − I | = 0  3  3  3
 ax − by − c bx + ay cx + a   2π   2π 
Now, sin θ +  + sin θ − 
34. Given,  bx + ay −ax + by − c cy + b = 0  3  3
 cx + a cy + b − ax − by + c
rankershalt.com

146 Matrices and Determinants

 2π 2π   2π 2π  1 2 5
θ + +θ−  θ + −θ +  Thus, f (x) = x − x+2
= 2 sin  3 3  cos  3 3  4 4
 2   2  37. Since, a , b, c are pth , qth and rth terms of HP.
   
1 1 1
2π  π ⇒ , , are in an AP.
= 2 sin θ cos = 2 sin θ cos  π −  a b c
3  3
1 
π = A + ( p − 1) D 
= − 2 sin θ cos = − sin θ a
3 1 
⇒ = A + ( q − 1) D  …(i)
 2π   2π  b 
and cos θ +  + cos θ − 
 3   3 1
= A + (r − 1) D 
c 
 2π 2π   2π 2π 
θ + +θ−  θ + −θ +  1 1 1
= 2 cos  3 3  cos  3 3  bc ca ab
 2   2  a b c
    Let ∆ = p q r = abc p q r [from Eq. (i)]
 2π   1 1 1 1 1 1 1
= 2 cos θ cos   = 2 cos θ  −  = − cos θ
 3  2
A + ( p − 1) D A + (q − 1) D A + (r − 1) D
 4π   4π 
and sin 2θ +  + sin 2θ −  = abc p q r
 3  3
1 1 1
 4π 4π   4π 4π 
 2θ + + 2θ −   2θ + − 2θ +  Applying R1 → R1 − ( A − D ) R3 − DR2
= 2 sin  3 3  cos  3 3 
 2   2  0 0 0 bc ca ab
    = abc p q r = 0 ⇒ p q r =0
4π  π
= 2 sin 2θ cos = 2 sin 2θ cos  π +  1 1 1 1 1 1
3  3
π 38. Given, a > 0, d > 0 and let
= − 2 sin 2θ cos = − sin 2θ
3 1 1 1
  a a (a + d ) (a + d ) (a + 2d )
 sin θ cos θ sin 2 θ  1 1 1
∴ ∆ =  − sin θ − cos θ − sin 2θ = 0 ∆=
(a + d ) (a + d ) (a + 2d ) (a + 2d ) (a + 3d )
 
sin θ − 2π  cos θ − 2π  sin 2θ − 4π   1 1 1
  3   3   
3  (a + 2d ) (a + 2d ) (a + 3d ) (a + 3d ) (a + 4d )
[since, R1 and R2 are proportional] 1
Taking common from R1 ,
2ax 2ax − 1 2ax + b + 1 a (a + d ) (a + 2d )
36. Given, f ′ (x) = b b+1 −1 1
from R2,
2 (ax + b) 2ax + 2b + 1 2ax + b (a + d )(a + 2d )(a + 3d )
Applying R3 → R3 − R1 − 2R2, we get 1
from R3
 2ax 2ax − 1 2ax + b + 1  (a + 2d ) (a + 3d )(a + 4d )
f ′ (x) =  b b+1 −1  1
 0 0 1 ⇒ ∆=
a (a + d )2(a + 2d )3 (a + 3d )2(a + 4d )
2ax − 1   2ax −1 
=
2ax
=
b+1   b
[C 2 → C 2 − C1]  (a + d )(a + 2d ) (a + 2d ) a 
 b 1
 (a + 2d )(a + 3d ) (a + 3d ) (a + d ) 
⇒ f ′ (x) = 2ax + b  (a + 3d )(a + 4d ) (a + 4d ) (a + 2d ) 
On integrating, we get f (x) = ax2 + bx + c, ⇒ ∆=
1
∆′
where c is an arbitrary constant. a (a + d )2(a + 2d )3 (a + 3d )2(a + 4d )
Since, f has maximum at x = 5 / 2.  (a + d )(a + 2d ) (a + 2d ) a 
where, ∆′ =  (a + 2d )(a + 3d ) (a + 3d ) (a + d ) 
⇒ f ′ (5 / 2) = 0 ⇒ 5a + b = 0 …(i)
 (a + 3d )(a + 4d ) (a + 4d ) (a + 2d ) 
Also, f (0) = 2 ⇒ c = 2 and f (1) = 1
⇒ a + b + c=1 …(ii) Applying R2 → R2 − R1 , R3 → R3 − R2
(a + d ) (a + 2d ) (a + 2d ) a
On solving Eqs. (i) and (ii) for a , b, we get
1 5 ⇒ ∆′ = (a + 2d ) (2d ) d d
a = ,b = − (a + 3d ) (2d ) d d
4 4
rankershalt.com

Matrices and Determinants 147

Applying R3 → R3 − R2 1 (n + 1) (n + 1) (n + 2) 
(a + d )(a + 2d ) (a + 2d ) a ∴ D = n ! (n + 1)! (n + 2) ! 1 (n + 2) (n + 2) (n + 3) 
1 (n + 3) (n + 3) (n + 4) 
∆′ = (a + 2d )2d d d
2d 2
0 0 Applying R2 → R2 − R1 and R3 → R3 − R2, we get
 1 (n + 1) (n + 1) (n + 2) 
Expanding along R3 , we get D = n !(n + 1)!(n + 2)! 0 2n + 4
1 
a + 2d a  2n + 6
∆′ = 2d 2 0 1 
 d d
Expanding along C1 , we get
∆′ = (2d 2)(d )(a + 2d − a ) = 4d 4
D = (n !)(n + 1)!(n + 2)![(2n + 6) − (2n + 4)]
4d 4
∴ ∆= D = (n !)(n + 1)! (n + 2)! [2]
a (a + d ) (a + 2d )3 (a + 3d )2(a + 4d )
2
On dividing both side by (n !)3
cos ( A − P ) cos ( A − Q ) cos ( A − R) D (n !)(n !)(n + 1)(n !)(n + 1)(n + 2)2
39. Let ∆ = cos (B − P ) cos (B − Q ) cos (B − R) ⇒ 3
=
(n !) (n !)3
cos (C − P ) cos (C − Q ) cos (C − R)
D
cos A cos P + sin A sin P cos ( A − Q ) ⇒ = 2(n + 1)(n + 1)(n + 2)
⇒ ∆ = cos B cos P + sin B sin P cos (B − Q ) (n !)3
cos C cos P + sin C sin P cos (C − Q ) D
⇒ = 2(n3 + 4n 2 + 5n + 2) = 2n (n 2 + 4n + 5) + 4
cos ( A − R) (n !)3
cos (B − R)
cos (C − R) D
⇒ − 4 = 2n (n 2 + 4n + 5)
(n !)3
 cos A cos P cos ( A − Q ) cos ( A − R) 
⇒ ∆ =  cos B cos P cos (B − Q ) cos (B − R)   D 
which shows that  − 4 is divisible by n.
 cos C cos P cos (C − Q ) cos (C − R)   (n !)
3

 sin A sin P cos ( A − Q ) cos ( A − R) 
+  sin B sin P cos (B − Q ) cos (B − R)  p b c
 sin C sin P cos (C − Q ) cos (C − R)  41. Let ∆ = a q c
 cos A cos ( A − Q ) cos ( A − R)  a b r
⇒ ∆ = cos P  cos B cos (B − Q ) cos (B − R) 
Applying R1 → R2 − R1 and R3 → R3 − R1 , we get
 cos C cos (C − Q ) cos (C − R) 
p b c
 sin A cos ( A − Q ) cos ( A − R) 
+ sin P  sin B cos (B − Q ) cos (B − R)  ∆= a− p q−b 0
 sin C cos (C − Q ) cos (C − R)  a−p 0 r−c
Applying C 2 → C 2 − C1 cos Q , C3 → C3 − C1 cos R in a− p q−b p b
first determinant and C 2 → C 2 − C1 sin Q and in =c + (r − c)
a−p 0 a− p q−b
second determinant
cos A sin A sin Q sin A sin R = − c (a − p) (q − b) + (r − c) [ p(q − b) − b(a − p)]
⇒ ∆ = cos P cos B sin B sin Q sin B sin R = − c (a − p) (q − b) + p(r − c) (q − b) − b(r − c)(a − p)
cos C sin C sin Q sin C sin R Since, ∆ = 0
⇒ − c (a − p) (q − b) + p(r − c) (q − b) − b(r − c) (a − p) = 0
 sin A cos A cos Q cos A cos R 
+ sin P sin B cos B cos Q cos B cos R  c p b
⇒ + + =0
 sin C cos C cos Q cos C cos R  r−c p−a q−b
 cos A sin A sin A  [on dividing both sides by (a − p)(q − b)(r − c)]
∆ = cos P sin Q sin R cos B sin B sin B  p b c
 cos C sin C sin C  ⇒ + +1+ + 1 =2
p−a q−b r−c
 sin A cos A cos A  p q r
+ sin P cos Q cos R sin B cos B cos B  ⇒ + + =2
 sin C cos C cos C  p−a q−b r−c

∆ =0 + 0 =0 42. We know, A 28 = A × 100 + 2 × 10 + 8


 n! (n + 1)! (n + 2)!  3B9 = 3 × 100 + B × 10 + 9
40. Given, D =  (n + 1)! (n + 2)! (n + 3)!  and 62 C = 6 × 100 + 2 × 10 + C
 (n + 2)! (n + 3)! (n + 4)! 
Since, A 28, 3B 9 and 62 C are divisible by k, therefore
Taking n !, (n + 1)! and (n + 2)! common from R1 , R2 there exist positive integers m1 , m2 and m3 such that,
and R3 , respectively. 100 × A + 10 × 2 + 8 = m1k , 100 × 3 + 10 × B + 9 = m2k
and 100 × 6 + 10 × 2 + C = m3 k … (i)
rankershalt.com

148 Matrices and Determinants

A 3 6  xC x
Cr + 1 x
Cr + 2 
r
∴ ∆= 8 9 C 44. Let ∆ =  y C r y
Cr + 1 y
Cr + 2 
z z z 
2 B 2 C Cr + 1 Cr + 2 
 r
Applying R2 → 100R1 + 10R3 + R2 Applying C3 → C3 + C 2
A 3 x+1
 xC x
Cr + 1 Cr + 2 
r
⇒ ∆ = 100 A + 2 × 10 + 8 100 × 3 + 10 × B + 9
∆ = y C r y
Cr + 1 y+1
Cr + 2 
2 B z z z+1

C Cr + 1 Cr + 2 
6  r
n+1
[Q nC r + nC r − 1 = Cr ]
100 × 6 + 10 × 2 + C
Applying C 2 → C 2 + C1
2
x+1 x+1
A 3 6  xC Cr + 1 Cr + 2 
r

= A28 3B9 62C ∆ = y C r y+1


Cr + 1 y+1
Cr + 2 
[from Eq. (i)] z z+1 z+1

C Cr + 1 Cr + 2 
2 B 2  r
A 3 6 A 3 6 Applying C3 → C3 + C 2
x+1 x+ 2
= m1k m2k m3 k = k m1 m2 m3  xC Cr + 1 Cr + 2 
r
2 B 2 2 B 2 ⇒ ∆ = y C r y+1
Cr + 1 y+ 2
Cr + 2  Hence proved.
z z +1 z+ 2

C Cr + 1 Cr + 2 
∴ ∆ = mk  r
Hence, determinant is divisible by k. 45. Since, α is repeated root of f (x) = 0.
 a −1 n 6 
∴ f (x) = a (x − α )2, a ∈ constant (≠ 0)
43. Given, ∆ a = 
 (a − 1 ) 2
2 n 2
4 n − 2 

 A (x) B(x) C (x) 
 (a − 1) 3n 3n − 3n 
3 3 2
Let φ (x) =  A (α ) B(α ) C (α ) 
 n   A ′ (α ) B ′ (α ) C ′ (α ) 
 ∑ (a − 1) n 6 
 an=1  To show φ (x) is divisible by (x − α )2, it is sufficient to
n
  show that φ (α ) and φ ′ (α ) = 0.
∴ ∑ a ∑
∆ = (a − 1 ) 2
2 n 2
4 n − 2
  A (α ) B(α ) C (α ) 
a=1 a=1
 n  ∴ φ (α ) =  A (α ) B(α ) C (α ) 
 ∑ (a − 1)3 3n 3n − 3n 
3 2
 A ′ (α ) B ′ (α ) C ′ (α ) 
 a=1 
=0 [Q R1 and R2 are identical]
n (n − 1) ′ ′ ) C ′ (x) 
n 6  A (x) B (x
2 Again, φ ′ (x) =  A (α ) B(α ) C (α ) 
n (n − 1)(2n − 1)
= 2n 2 4n − 2  A ′ (α ) B ′ (α ) C ′ (α ) 
6
n 2(n − 1)2  A ′ (α ) B ′ (α ) C ′ (α ) 
3n3 3n 2 − 3n φ ′ (α ) =  A (α ) B(α ) C (α ) 
4  A ′ (α ) B ′ (α ) C ′ (α ) 
=0 [Q R1 and R3 are identical]
1 1 6
n (n − 1) (2n − 1)
2
Thus, α is a repeated root of φ (x) = 0.
= 2n 4n − 2
2 3 Hence, φ (x) is divisible by f (x).
n (n − 1)
3n 2 3n 2 − 3n  x2 + x
2 x+1 x−2
1 1 6 46. Let ∆ =  2x2 + 3x − 1 3x 3x − 3
n3 (n − 1)  2 2x − 1 2x − 1
= 2n − 1 6n 12n − 6  x + 2x + 3
12
n − 1 6n 6n − 6 Applying R2 → R2 − (R1 + R3 ), we get
Applying C3 → C3 − 6 C1  x2 + x x+ 1 x−2 
1 1 0 ∆ =
 2 − 4 0 0  
n3 (n − 1)
= 2n − 1 6n 0 = 0  x + 2x + 3 2x − 1 2x − 1 
12
n − 1 6n 0 x2
Applying R1 → R1 + R2
n 4
⇒ ∑ ∆a = c [c = 0, i.e. constant]
x2
a=1 and R3 → R3 + R2, we get
4
rankershalt.com

Matrices and Determinants 149

 x x+ 1 x−2 ⇒ x3 (48x3 − 4 − 36x3 + 6) = 10


∆ =  −4 0 0 ⇒ 12x6 + 2x3 = 10

 2x + 3 2 x − 1 2 x −1 ⇒ 6x6 + x3 − 5 = 0
5
 x + 0 x + 1 x − 2 ⇒ x3 = , − 1
Applying R3 → R3 − 2R1 =  − 4 0 0  6
1/3
 3 −3 3   5
x=   ,−1
 6
 x x x  0 1 −2 
=  −4 0 0  +  −4 0 0 Hence, the number of real solutions is 2.
 3 −3 3   3 −3 3
 1 1 1  0 1 −2  Topic 3 Adjoint and Inverse of a Matrix
= x −4 0 0  +  −4 0 0
 3 −3 3   3 −3 3 1. Given matrix B is the inverse matrix of 3 × 3 matrix A,

⇒ ∆ = Ax + B 5 2α 1 
where B = 0 2 1 
 1 1 1
where, A =  −4 0 0 α 3 − 1
 3 −3 3 
We know that,
 0 1 −2 
B =  −4 1  −1 1 
and 0 0 det( A ) = Q det( A ) = det( A ) 
 3 −3 3 det(B)  
a b c Since, det( A ) + 1 = 0 (given)
47. Let ∆ =  b c a 1
+ 1 =0
c a b det(B)
Applying C1 → C1 + C 2 + C3 ⇒ det(B) = − 1
 a + b + c b c 1 b c ⇒ 5(− 2 − 3) − 2α (0 − α ) + 1 (0 − 2α ) = − 1
∆ =  a + b + c c a = (a + b + c) 1 c a
 a + b + c a b 1 a b ⇒ − 25 + 2α 2 − 2α = − 1
⇒ 2α 2 − 2α − 24 = 0
Applying R2 → R2 − R1 and R3 → R3 − R1 , we get
1 b c  ⇒ α 2 − α − 12 = 0
= (a + b + c) 0 c − b a − c  ⇒ (α − 4) (α + 3) = 0
0 a − b b − c 
⇒ α = − 3, 4
= (a + b + c) [− (c − b)2 − (a − b) (a − c)] So, required sum of all values of α is 4 − 3 = 1
= − (a + b + c) (a 2 + b2 + c2 − ab − bc − ca ) et e− t cos t e− t sin t
1 −t
= − (a + b + c) (2a 2 + 2b2 + 2c2 − 2ab − 2bc − 2ca ) 2. | A | = et −e cos t − e− t sin t − e sin t + e− t cos t
−t
2
et 2e− t sin t − 2e− t cos t
1
= − (a + b + c)[(a − b)2 + (b − c)2 + (c − a )2] 1 cos t sin t
2
= (et ) (e− t ) (e− t ) 1 − cos t − sin t − sin t + cos t
which is always negative.
1 2 sin t − 2 cos t
x x2 1 + x3 (taking common from each column)
48. Given, 2x 4x2 1 + 8x3 = 10 Aplying R2 → R2 − R1 and R3 → R3 − R1, we get
3x 9x2 1 + 27x3 [Qet − t = e0 = 1]
1 1 1 + x3 1 cos t sin t
⇒ x⋅ x 2
2 4 1 + 8x3 = 10 = e− t 0 − 2 cos t − sin t − 2 sin t + cos t
3 9 1 + 27x3 0 2 sin t − cos t − 2 cos t − sin t
= e− t ((2 cos t + sin t )2 + (2 sin t − cos t )2)
Apply R2 → R2 − 2R1 and R3 → R3 − 3R1, we get
(expanding along column 1)
1 1 1 + x3 −t
= e (5 cos t + 5 sin t )
2 2
x 0 2 − 1 + 6x3 = 10
3
= 5e− t (Qcos 2 t + sin 2 t = 1)
0 6 − 2 + 24x3 −t
⇒| A | = 5e ≠ 0 for all t ∈ R
2 6x3 − 1 ∴ A is invertible for all t ∈ R
⇒ x3 ⋅ = 10
6 24x3 − 2 [QIf| A | ≠ 0, then A is invertible]
rankershalt.com

150 Matrices and Determinants

3. Given,| ABAT| = 8  2 − 3
5. We have, A= 
⇒ | A||B|| AT| = 8 [Q|XY | = |X ||Y |] − 4 1 
∴ | A|2|B| = 8 …(i) [Q| AT| = | A|]  2 − 3  2 − 3
∴ A2 = A ⋅ A =   
Also, we have | AB−1| = 8 ⇒| A||B−1| = 8 − 4 1  − 4 1 
| A|  1   4 + 12 − 6 − 3
⇒ =8 …(ii) Q| A −1|=| A|−1 = =
|B|  | A| 
− 8 − 4 12 + 1 
On multiplying Eqs. (i) and (ii), we get  16 − 9
=
| A|3 = 8 ⋅ 8 = 43 − 12 13 
⇒ | A| = 4  16 − 9  2 − 3
Now, 3 A 2 + 12 A = 3  + 12 
⇒ |B| =
| A| 4 1
= = − 1213  
− 4 1 
8 8 2
 48 − 27  24 − 36
1  1 1  1 1 = +  
Now, |BA −1BT| = |B| |B| =     = − 36 39  − 48 12 
| A|  2 4  2 16
 72 − 63
cos θ − sin θ  = 
4. We have, A =   − 84 51 
sin θ cos θ 
51 63 
∴ | A| = cos 2 θ + sin 2 θ = 1 ∴ adj (3 A 2 + 12 A ) =  
 cos θ sin θ  84 72
and adj A =   5a − b
− sin θ cos θ  6. Given, A =  and A adj A = AAT
3 2 
a b  d − b
[Q If A =  , then adj A =  ] Clearly, A (adj A ) = A I 2
c d −c a 
[Q if A is square matrix of order n,
 cos θ sin θ   adj A 
⇒ A −1 =   Q A −1 =  then A (adj A ) = (adj A ) ⋅ A = A I n ]
− sin θ cos θ   | A|  5a − b
= I 2 = (10a + 3b) I 2
Note that, A −50 = ( A −1 )50 3 2
Now, A −2 = ( A −1 )( A −1 ) 1 0
 cos θ sin θ   cos θ sin θ  = (10a + 3b)  
⇒ A −2 =    0 1
− sin θ cos θ  − sin θ cos θ  10a + 3b 0 
= ...(i)

=
cos 2 θ − sin 2 θ cos θ sin θ + sin θ cos θ   0 10a + 3b

− cos θ sin θ − cos θ sin θ − sin 2 θ + cos 2 θ  5a − b  5a 3
and AAT = 
 cos 2 θ sin 2 θ  3 2  − b 2
= 
− sin 2 θ cos 2 θ  25a 2 + b2 15a − 2b
=  ...(ii)
Also, A −3 = ( A −2)( A −1 )  15a − 2b 13 
 cos 2 θ sin 2 θ   cos θ sin θ 
A −3 =    Q A (adj A ) = AAT
− sin 2 θ cos 2 θ  − sin θ cos θ  10a + 3b 0  25a 2 + b2 15a − 2b
∴ = 
=
 cos 3 θ sin 3 θ   0 10a + 3b  15a − 2b 13 

− sin 3 θ cos 3 θ  [using Eqs. (i) and (ii)]
 cos 50 θ sin 50 θ  ⇒ 15a − 2b = 0
Similarly, A −50 =  
− sin 50 θ cos 50 θ  ⇒ a=
2b
...(iii)
 25 25  15
 cos 6 π sin 6 π   π and 10a + 3b = 13 ...(iv)
=  when θ = 
25 25   12 On substituting the value of ‘a ’ from Eq. (iii) in
− sin π cos π
 6 6  Eq. (iv), we get
 π π   Q cos  25π  = cos 4π + π  = cos π   2b
    10 ⋅   + 3b = 13
 cos 6 sin 6    6   6 6   15 
=  
π π   25π   π π 20b + 45b
− sin cos  and sin   = sin 4π +  = sin  ⇒ = 13
 6 6    6   6  6 15
 3 1  65b
⇒ = 13
  15
= 2 2 
⇒ b =3
 −1 3
 2 2 
rankershalt.com

Matrices and Determinants 151

Now, substituting the value of b in Eq. (iii), we get 12. Every square matrix satisfied its characteristic
5a = 2 equation,
Hence, 5a + b = 2 + 3 = 5 1−λ 0 0
i.e. | A − λI | = 0 ⇒ 0 1−λ 1 =0
7. PLAN Use the following properties of transpose
( AB)T = BT AT , ( AT )T = A and A −1 A = I and simplify. If A is 0 −2 4−λ
non-singular matrix, then| A | ≠ 0.
⇒ (1 − λ ) {(1 − λ ) (4 − λ ) + 2} = 0
Given, AAT = AT A and B = A −1 AT
⇒ λ3 − 6λ2 + 11λ − 6 = 0
BBT = ( A −1 AT )( A −1 AT )T
⇒ A3 − 6 A 2 + 11 A − 6I = O …(i)
= A −1 AT A ( A −1 )T [Q ( AB)T = BT AT ] −1
Given, 6 A = A + cA + dI , multiplying both sides by
2
= A −1 AAT ( A −1 )T [Q AAT = AT A] A, we get
= IAT ( A −1 )T [Q A −1 A = I ]
6I = A3 + cA 2 + dA ⇒ A3 + cA 2 + dA − 6I = O …(ii)
= AT ( A −1 )T = ( A −1 A )T
On comparing Eqs. (i) and (ii), we get
[Q ( AB)T = BT AT ]
c = − 6 and d = 11
= IT = I
3 − 1 − 2
1 α 3
13. Here, P = 2 0 α 
8. Given, P = 1 3 3
3 − 5 0 
2 4 4
Now, |P| = 3(5α ) + 1(− 3α ) − 2(− 10)
∴ | P | = 1(12 − 12) − α (4 − 6) + 3 (4 − 6) = 2 α − 6 = 12α + 20 …(i)
Q P = adj ( A ) [given]  5α 2α − 10
T

∴ |P| = |adj A|= | A|2 = 16 [Q|adj A| = | A|n −1 ] ∴ 


adj (P ) = − 10 6 
12 
∴ 2 α − 6 = 16  − α − (3α + 4) 2 
⇒ 2 α = 22  5α − 10 −α 
⇒ α = 11 
=  2α 6 − 3α − 4 …(ii)
9. Given, PT = 2 P + I …(i) − 10 12 2 

∴ (P ) = (2 P + I ) = 2 P + I
T T T T As, PQ = kI
⇒ P =2P + IT ⇒ |P||Q| = |kI|
⇒ |P||Q| = k3
⇒ P = 2 (2 P + I ) + I
 k2  k2 
⇒ P = 4P + 3I or 3P = − 3I ⇒ |P|  = k3 given,|Q| = 
 2  2
⇒ PX = − IX = –X
⇒ |P| = 2k …(iii)
1 a b
Q PQ = kI
10. | A| ≠ 0, as non-singular ω 1 c ≠ 0
∴ Q = kp−1I
ω2 ω 1 adjP k(adj P )
= k⋅ = [from Eq. (iii)]
⇒ 1 (1 − cω ) − a (ω − cω 2) + b (ω 2 − ω 2) ≠ 0 |P| 2k
⇒ 1 − cω − aω + acω 2 ≠ 0  5 α − 10 −α 
adj P 1 
1 1 = = 2α 6 − 3α − 4
⇒ (1 − cω ) (1 − aω ) ≠ 0 ⇒ a ≠ ,c≠ 2 2
ω ω − 10 12 2 
⇒ a = ω , c = ω and b ∈{ω , ω 2} ⇒ 2 solutions − 3α − 4  k
∴ q23 = given, q23 = − 8 
11. Given, M T = − M , N T = − N and MN = NM ...(i) 2  
∴ M 2N 2 (M T N )− 1 (MN −1 )T (3α + 4) k
⇒ − =−
⇒ M 2 N 2N −1 (M T )−1 (N −1 )T ⋅ M T 2 8
⇒ M 2 N (NN −1 )(− M )−1 (N T )−1 (− M ) ⇒ (3α + 4) × 4 = k
⇒ M 2 N I (− M −1 ) ( − N )−1 (− M ) ⇒ 12α + 16 = k …(iv)
⇒ − M 2 NM −1N −1 M From Eq. (iii), |P|= 2k
⇒ − M ⋅ (MN )M −1N −1 M = − M (NM )M −1N −1 M ⇒ 12α + 20 = 2k [from Eq. (i)] …(v)
⇒ − MN (NM −1 )N −1 M = − M (N N −1 )M ⇒ − M 2 On solving Eqs. (iv) and (v), we get
α = − 1 and k = 4 …(vi)
NOTE Here, non-singular word should not be used, since there
is no non-singular 3 × 3 skew-symmetric matrix. ∴ 4α − k + 8 = − 4 − 4 + 8 = 0
∴ Option (b) is correct.
rankershalt.com

152 Matrices and Determinants

Now, |P adj (Q )| = |P||adj Q| ⇒ ∆ = [sin θ ] − [− cos θ ] [cot θ ]


2
 k2 k5 210  π 2π 
= 2k  = = = 29 When θ ∈ , 
 2 2 2 2 3 

∴ Option (c) is correct.  3 


sin θ ∈  , 1
14. PLAN A square matrix M is invertible, iff dem (M) or| M| ≠ 0.  2 
a b ⇒ [sin θ ] = 0 …(iii)
Let M = 
 b c  1
− cos θ ∈ 0, 
 2
a  b
(a) Given,   =   ⇒ a = b = c = α [let] ⇒ [− cos θ ] = 0 …(iv)
 b  c
 1 
α α  and cot θ ∈  − , 0
⇒ M =  3 
 ⇒ |M |= 0 ⇒ M is non-invertible.
α α  ⇒ [cot θ ] = − 1 …(v)
(b) Given, [b c] = [a b] So, ∆ = [sin θ ] − [− cos θ ] [cot θ ]
⇒ a = b = c=α [let] − (0 × (− 1)) = 0 [from Eqs. (iii), (iv) and (v)]
Again,|M | = 0  π 2π 
Thus, for θ ∈  ,  , the given system have infinitely
⇒ M is non-invertible. 2 3 
a 0 many solutions.
(c) As given M =   ⇒|M |= ac ≠ 0  7π   1 
 0 c When θ ∈  π,  , sin θ ∈  − , 0
[Q a and c are non-zero]  6  2 
⇒ M is invertible. ⇒ [sin θ ] = − 1
a b  3 
(d) M =   ⇒|M |= ac − b2 ≠ 0
− cos θ ∈  , 1 ⇒ [cos θ ] = 0
 b c  2 
Q ac is not equal to square of an integer.
and cot θ ∈ ( 3 , ∞ ) ⇒ [cot θ ] = n , n ∈ N .
M is invertible.
So, ∆ = − 1 − (0 × n ) = − 1
15. PLAN If| A n × n| = ∆, then|adj A| = ∆A − 1  7π 
Thus, for θ ∈  π,  , the given system has a unique
 6
1 4 4 
solution.
Here, adj P3 × 3 = 2 1 7
1 1 3  2. Given, system of linear equations
x+ y+ z =6 … (i)
⇒ |adj P | = | P |2 4 x + λy − λz = λ − 2 …(ii)
1 4 4 and 3x + 2 y − 4z = − 5 …(iii)
∴ | adj P | = 2 1 7 = 1 (3 − 7) − 4 (6 − 7) + 4 (2 − 1) has infinitely many solutions, then ∆ = 0
1 1 3 1 1 1
= − 4 + 4 + 4 = 4 ⇒ |P |= ± 2 ⇒ 4 λ − λ =0
16. | A | = (2k + 1)3 ,| B| = 0 3 2 −4
But det (adj A) + det (adj B) = 106 ⇒ 1(− 4λ + 2λ ) − 1(− 16 + 3λ) + 1(8 − 3λ) = 0
⇒ (2k + 1)6 = 106 ⇒ − 8λ + 24 = 0 ⇒ λ = 3
9 From, the option λ = 3, satisfy the quadratic equation
⇒ k= ⇒ [k] = 4 λ2 − λ − 6 = 0.
2
3. Given system of linear equations
Topic 4 Solving System of Equations x+ y+ z =5 …(i)
1. Given system of linear equations is x + 2 y + 2z = 6 …(ii)
[sin θ ] x + [− cos θ ] y = 0 …(i) x + 3 y + λz = µ …(iii)
and [cot θ ] x + y = 0 …(ii) (λ, µ ∈ R)
The above given system has infinitely many solutions,
where, [x] denotes the greatest integer ≤ x.
then the plane represented by these equations
[sin θ ] [− cos θ ] intersect each other at a line, means (x + 3 y + λz − µ )
Here, ∆=
[cot θ ] 1 = p(x + y + z − 5) + q (x + 2 y + 2z − 6)
= ( p + q)x + ( p + 2q) y + ( p + 2q)z − (5 p + 6q)
rankershalt.com

Matrices and Determinants 153

On comparing, we get and cx + cy − z = 0


p + q = 1, p + 2q = 3, p + 2q = λ We know that a homogeneous system of linear
and 5 p + 6q = µ equations have non-trivial solutions iff
So, ( p, q) = (−1, 2) ∆ =0
⇒ λ = 3 and µ = 7 1 − c − c
⇒ c −1 c = 0
⇒ λ + µ = 3 + 7 = 10  
c c − 1
4. Given system of linear equations
2x + 3 y − z = 0, ⇒ 1(1 − c2) + c(− c − c2) − c(c2 + c) = 0
⇒ 1 − c2 − c2 − c3 − c3 − c2 = 0
x + ky − 2z = 0
⇒ −2c3 − 3c2 + 1 = 0
and 2x − y + z = 0 has a non-trivial solution (x, y, z ).
⇒ 2c3 + 3c2 − 1 = 0
2 3 −1 ⇒ (c + 1)[2c2 + c − 1] = 0
∴ ∆ = 0 ⇒1 k −2 = 0
  ⇒ (c + 1)[2c2 + 2c − c − 1] = 0
2 −1 1  ⇒ (c + 1)(2c − 1)(c + 1) = 0
2(k − 2) − 3(1 + 4) − 1(−1 − 2k) = 0 1
⇒ c = − 1 or
2
⇒ 2k − 4 − 15 + 1 + 2k = 0
1
9 Clearly, the greatest value of c is .
⇒ 4 k = 18 ⇒ k = 2
2
7. The given system of linear equations is
So, system of linear equations is
x − 2 y − 2 z = λx
2x + 3 y − z = 0 …(i) x + 2 y + z = λy
2x + 9 y − 4z = 0 …(ii) − x − y − λz = 0,
and 2x − y + z = 0 …(iii) which can be rewritten as
(1 − λ )x − 2 y − 2z = 0
From Eqs. (i) and (ii), we get
⇒ x + (2 − λ ) y + z = 0
y 1 x + y + λz = 0
6 y − 3z = 0, =
z 2 Now, for non-trivial solution, we should have
From Eqs. (i) and (iii), we get 1 − λ −2 −2
x
4x + 2 y = 0 ⇒ = −
1 1 2 − λ 1 =0
y 2 1 1 λ
x x y 1 z  y 1 x 1 [Q If a1x + b1 y + c1z = 0; a 2x + b2y + c2z = 0
So, = × = − ⇒ = −4 Q z = 2 and y = − 2 
z y z 4 x   a3 x + b3 y + c3 z = 0]
x y z 1 1 9 1 a1 b1 c1 
∴ + + + k= − + −4 + = . 
y z x 2 2 2 2 has a non-trivial solution, then a 2 b2 c2 = 0
a3 b3 c3 
5. Given system of linear equations
x − 2 y + kz = 1 …(i) ⇒ (1 − λ ) [(2 − λ )λ − 1] + 2 [λ − 1] − 2 [1 − 2 + λ ] = 0
2x + y + z = 2 …(ii) ⇒ (λ − 1)[λ2 − 2λ + 1 + 2 − 2] = 0
⇒ (λ −1)3 = 0
and 3x − y − kz = 3 …(iii)
⇒ λ =1
has a solution (x, y, z ), z ≠ 0.
8. Given system of linear equations,
On adding Eqs. (i) and (iii), we get
(1 + α )x + βy + z = 2
x − 2 y + kz + 3x − y − kz = 1 + 3 αx + (1 + β ) y + z = 3
4x − 3 y = 4 αx + βy + 2z = 2
⇒ 4x − 3 y − 4 = 0 has a unique solution, if
This is the required equation of the straight line in 1+α β 1
which point (x, y) lies. α (1 + β ) 1 ≠ 0
α β 2
6. Key Idea A homogeneous system of linear equations have
non-trivial solutions iff ∆ = 0 Apply R1 → R1 − R3 and R2 → R2 − R3
Given system of linear equations is 1 0 −1
x − cy − cz = 0, 0 1 −1 ≠0
cx − y + cz = 0 α β 2
rankershalt.com

154 Matrices and Determinants

⇒ 1(2 + β ) − 0(0 + α ) − 1(0 − α ) ≠ 0 In given interval (0, π ),


⇒ α + β + 2 ≠0 … (i) 1
sin θ =
Note that, only (2, 4) satisfy the Eq. (i). 2
π 5π
9. We know that, if the system of equations ⇒ θ= , [Q sin θ ≠ 0, θ ∈ (0, π )]
6 6
a1x + b1 y + c1z = d1
a 2x + b2y + c2z = d2 Hence, 2 solutions in (0, π )
a3 x + b3 y + c3 z = d3 11. Since, the system of equations has infinitely many
has more than one solution, then D =0 and solution, therefore D = D1 = D2 = D3 = 0
D1 = D2 = D3 = 0. In the given problem, Here,
a 2 3 1 1 1
D1 = 0 ⇒ b − 1 5 = 0 D = 1 2 3 = 1 (2α − 9) − 1 (α − 3) + 1(3 − 2)
c −3 2 1 3 α
⇒ a (− 2 + 15) − 2(2b − 5c) + 3(− 3b + c) = 0
=α −5
⇒ 13a − 4b + 10c − 9b + 3c = 0
1 1 5
⇒ 13a − 13b + 13c = 0
⇒ a − b + c=0⇒b − a − c=0 and D3 = 1 2 9 = 1 (2 β − 27) − 1(β − 9) + 5 (3 − 2)
1 3 β
10. We know that,
the system of linear equations = β − 13
a1x + b1 y + c1z = 0 Now, D =0
a 2x + b2y + c2z = 0 ⇒ α −5 =0 ⇒ α =5
a3 x + b3 y + c3 z = 0 and D3 = 0 ⇒ β − 13 = 0
has a non-trivial solution, if ⇒ β = 13
a1 b1 c1 ∴ β − α = 13 − 5 = 8
a 2 b2 c2 = 0 1 −4 7
a3 b3 c3 12. (a) Here, D = 0 3 −5
−2 5 −9
Now, if the given system of linear equations
x + 3 y + 7z = 0 = 1(− 27 + 25) + 4(0 − 10) + 7(0 + 6)
− x + 4 y + 7z = 0, [expanding along R1]
and (sin 3 θ )x + (cos 2 θ ) y + 2z = 0 = − 2 − 40 + 42 = 0
has non-trivial solution, then ∴The system of linear equations have infinite many
1 3 7 solutions.
−1 4 7 =0 [Q system is consistent and does not have unique
solution as D = 0]
sin 3 θ cos 2 θ 2
⇒ D1 = D2 = D3 = 0
⇒ 1(8 − 7 cos 2 θ ) − 3 (− 2 − 7 sin 3 θ ) g −4 7
+ 7 (− cos 2 θ − 4 sin 3 θ ) = 0 Now, D1 = 0 ⇒ h 3 − 5 = 0
⇒ 8 − 7 cos 2 θ + 6 + 21 sin 3 θ k 5 −9
− 7 cos 2 θ − 28 sin 3 θ = 0
⇒ g (− 27 + 25) + 4(− 9h + 5k) + 7(5h − 3k) = 0
⇒ − 7 sin 3 θ − 14 cos 2 θ + 14 = 0
⇒ − 2 g − 36h + 20k + 35h − 21k = 0
⇒ − 7 (3 sin θ − 4 sin3 θ ) − 14 (1 − 2 sin 2 θ ) +14 = 0
⇒ − 2g − h − k = 0 ⇒2g + h + k = 0
[ Q sin 3 A = 3 sin A − 4 sin3 A and
cos 2 A = 1 − 2 sin 2 A] 13 According to Cramer’s rule, here
⇒ 28 sin3 θ + 28 sin 2 θ − 21 sin θ − 14 + 14 = 0 1 1 1 1 0 0
⇒ 7 sin θ [4 sin 2 θ + 4 sin θ − 3] = 0 D= 2 3 2 = 2 1 0
⇒ sin θ [4 sin 2 θ + 6 sin θ − 2 sin θ − 3] = 0 2 3 a −1
2
2 1 a −3
2

⇒ sin θ [2 sin θ (2 sin θ + 3) − 1 (2 sin θ + 3)] = 0 (Applying C 2 → C 2 − C1 and C3 → C3 − C1)


⇒ (sin θ ) (2 sin θ − 1) (2 sin θ + 3) = 0 = a2 −3 (Expanding along R1)
1 2 1 1 2 1 0
Now, either sin θ = 0 or
2 and D1 = 5 3 2 = 5 3 −1
 3  a + 1 3 a2 − 1 a + 1 3 a2 − 1 −3
Q sin θ ≠ − 2 as − 1 ≤ sin θ ≤ 1
 
(Applying C3 → C3 − C 2)
rankershalt.com

Matrices and Determinants 155

Hence, given system of linear equation has a non-trivial


2 0 0 solution for exactly three values of λ.
5
= 5 3− −1 16. Given system of linear equations
2
2x1 − 2x2 + x3 = λx1
a + 1 3 − (a + 1) a 2 − 1 − 3
…(i)
2 ⇒ (2 − λ )x1 − 2x2 + x3 = 0 …(ii)
1 2x1 − 3x2 + 2x3 = λx2
(Applying C 2 → C 2 − C1) …(iii)
2
⇒ 2x1 − ( 3 + λ )x2 + 2x3 = 0
2 0 0
1 − x1 + 2x2 = λx3
= 5 −1
2 ⇒ − x1 + 2x2 − λx3 = 0
5 a a2 − 4
a+1 − Since, the system has non-trivial solution.
2 2
2 − λ −2 1
1 5 a   2  =0
= 2  (a 2 − 4) +  −   [Expanding along R1] ∴  − (3 + λ ) 2 
 2 2 2 
 −1 2 λ 
a2 5 a
=2 − 2 + −  = a2 − 4 + 5 − a = a2 − a + 1 ⇒ (2 − λ )(3λ + λ2 − 4) + 2(−2λ + 2) + 1(4 − 3) − λ ) = 0
 2 2 2
⇒ (2 − λ )(λ2 + 3λ − 4) + 4(1 − λ ) + (1 − λ ) = 0
Clearly, when a = 4, then D = 13 ≠ 0 ⇒ unique solution
⇒ (2 − λ )(λ + 4)(λ − 1) + 5(1 − λ ) = 0
and
when|a| = 3, then D = 0 and D1 ≠ 0. ⇒ (λ − 1)[(2 − λ )(λ + 4) − 5] = 0
∴ When |a| = 3, then the system has no solution i.e. ⇒ (λ − 1)(λ2 + 2λ − 3) = 0
system is inconsistent. ⇒ (λ − 1)[(λ − 1)(λ + 8)] = 0
14. We have, ⇒ (λ − 1)2(λ + 3) = 0
x + ky + 3z = 0; 3x + ky − 2z = 0; 2x + 4 y − 3z = 0 ⇒ λ = 1, 1, − 3
System of equation has non-zero solution, if Hence, λ contains two elements.
1 k 3 
 3 k −2 = 0 17. Given equations can be written in matrix form
  AX = B
 2 4 −3
k+1 x
where, A = 
8  4k
⇒ (−3k + 8) − k(−9 + 4) + 3(12 − 2k) = 0 , X =   and B =
 k k + 3   y 3k − 1
⇒ −3k + 8 + 9k − 4k + 36 − 6k = 0
For no solution, A = 0 and (adj A) B ≠ 0
⇒ −4k + 44 = 0 ⇒ k = 11
k+1
Now, A = 
8 
Let z = λ , then we get =0
 k k + 3 
x + 11 y + 3λ = 0 …(i)
3x + 11 y − 2λ = 0 …(ii) ⇒ (k2 + 1)(k + 3) − 8k = 0
and 2x + 4 y − 3λ = 0 …(iii) k2 + 4k + 3 − 8k = 0
Solving Eqs. (i) and (ii), we get ⇒ k2 − 4k × 3 = 0
5λ −λ xz 5λ2 ⇒ (k − 1)(k − 3) = 0
x= , y= ,z=λ ⇒ 2= = 10
2 2 y  λ
2
⇒ k = 1, k = 3,
2 × − 
 2 k+3 −8 
Now adj A = 
 −k k + 1 
15. Given, system of linear equation is
k+3 − 8   4k
x + λy − z = 0; λx − y − z = 0; x + y − λz = 0 Now, (adj A)B =  
 −k k + 1   3 k + 1 
Note that, given system will have a non-trivial solution
only if determinant of coefficient matrix is zero,  (k + 3)(4k) − 8 (3k − 1)
= 
 − 4k + (k + 1)(3k − 1) 
2
1 λ −1
i.e. λ −1 −1 = 0  4k2 − 12k + 8 
= 
 − k + 2k − 1 
2
1 1 −λ
⇒ 1 (λ + 1) − λ(− λ2 + 1) − 1(λ + 1) = 0 Put k = 1
4 − 12 + 8 0
⇒ λ + 1 + λ3 − λ − λ − 1 = 0 (adj A) B =  = not true
⇒ λ3 − λ = 0 ⇒ λ(λ2 − 1) = 0 −1 + 2 − 1  0
⇒ λ = 0or λ = ± 1
rankershalt.com

156 Matrices and Determinants

Put k = 3 Applying C1 → C1 − C 2, C 2 → C 2 + C3
36 − 36 + 8  8   1 + k −k − 1 −1 
(adj A) B =  = ≠ 0 true
−9 + 6 − 1 −4 ⇒ 1 + k −2 −1  = 0
 0 0 −1 
Hence, required value of k is 3.
Alternate Solution ⇒ 2(k + 1) − (k + 1)2 = 0

Condition for the system of equations has no solution ⇒ (k + 1)(2 − k − 1) = 0 ⇒ k = ± 1


is NOTE There is a golden rule in determinant that n one’s ⇒
( n − 1) zero’s or n (constant) ⇒ ( n − 1) zero’s for all
a1 b1 c1
= ≠ constant should be in a single row or a single column.
a 2 b2 c2
k+1 8 4k
23. The given system of equations can be expressed as
∴ = ≠  1 −2 3  x   −1 
k + 3 3k − 1  1 −3 4  y =  1
k
k+1 8   
Take = −1 1 −2  z   k
k k+3
⇒ k2 + 4k + 3 = 8k Applying R2 → R2 − R1 , R3 → R3 + R1

⇒ k2 − 4k + 3 1 −2 3  x   −1 
⇒ (k –1) (k –3) = 0 ~ 0 −1 1  y =  2 
   
0 −1 1  z  k − 1
k = 1, 3
8 4.1 Applying R3 → R3 − R2
If k –1, then ≠ , false
1+3 2
1 −2 3  x   −1 
And, if k = 3, then
8

4.3
, true ~ 0 −1 1  y =  2 
6 9 −1
0 0 0  z  k − 3
Therefore, k = 3
When k ≠ 3 , the given system of equations has no
Hence, only one value of k exist. solution.
 x  1 ⇒ Statement I is true. Clearly, Statement II is also
18. Since, A  y = 0 is linear equation in three variables true as it is rearrangement of rows and columns of
 z  0  1 −2 3 
 1 −3 4 .
and that could have only unique, no solution or  
infinitely many solution. −1 1 −2
∴It is not possible to have two solutions.
24. We have,
Hence, number of matrices A is zero.
− x + 2 y + 5z = b1
19. Since, given system has no solution. 2x − 4 y + 3z = b2
∴ ∆ = 0 and any one amongst ∆ x , ∆ y , ∆ z is non-zero. x − 2 y + 2z = b3
 2 −1 2   2 −1 2 has at least one solution.
Let 1 −2 1  = 0 and ∆ z =  1 −2 −4  = 6 ≠ 0 −1 2 5
1 1 λ 1 1 4
∴ D= 2 −4 3
⇒ λ =1 1 −2 2
20. For infinitely many solutions, we must have and D1 = D2 = D3 = 0
k+1 8 4k
= = ⇒ k =1 b1 2 5
k k + 3 3k − 1 ⇒ D1 = b2 − 4 3
21. Given equations x + ay = 0, az + y = 0, ax + z = 0 has b3 − 2 2
infinite solutions.
= − 2b1 − 14b2 + 26b3 = 0
1 a 0  ⇒ b1 + 7b2 = 13b3 ...(i)
∴ 0 1 a = 0
a 0 1  1 2 3
(a) D = 0 4 5 = 1(24 − 10) + 1(10 − 12)
⇒ 1 + a3 = 0 or a = − 1
1 2 6
22. Since, the given system has non-zero solution.
= 14 − 2 = 12 ≠ 0
1 −k −1 
∴ −1 −1  = 0 Here, D ≠ 0 ⇒ unique solution for any b1, b2, b3 .
k
1 1 −1 
rankershalt.com

Matrices and Determinants 157

1 1 3 a 0 f
(b) D = 5 2 6 Again, | A3|= 1 c g =0 ⇒ g=h
1 d h
−2 −1 −3
a f 1
= 1(− 6 + 6) − 1 (− 15 + 12) + 3 (− 5 + 4) = 0 ⇒ | A2| = 1 g b =0 ⇒ g=h
For atleast one solution 1 h b
D1 = D2 = D3 = 0 f 0 1
b1 1 3 and | A1| = g c b =0 ⇒ g=h
Now, D1 = b2 2 6 h d b
b3 − 1 − 3 ∴ g = h , c = d and ab = 1 …(i)
= b1 (− 6 + 6) − b2(− 3 + 3) + b3 (6 − 6) Now, BX = V
=0 a 1 1
1 b1 3 |B| = 0 d c =0 [from Eq. (i)]
f g h
D2 = 5 b2 6
− 2 b3 − 3 [since, C 2 and C3 are equal]
∴ BX = V has no solution.
= − b1 (− 15 + 12) + b2(− 3 + 6) − b3 (6 − 15)
= 3b1 + 3b2 + 9b3 = 0 ⇒ b1 + b2 + 3b3 = 0 a2 1 1
|B1|= 0 d c =0 [from Eq. (i)]
not satisfies the Eq. (i) 0 g h
It has no solution.
[since, c = d and g = h]
−1 2 −5
a a2 1
(c) D = 2 − 4 10 |B2| = 0 0 c = a 2cf = a 2df [Q c = d ]
1 −2 5 f 0 h
= − 1(− 20 + 20) − 2(10 − 10) − 5(− 4 + 4) Since, adf ≠ 0 ⇒ |B2| ≠ 0
=0 |B| = 0 and |B2| ≠ 0
Here, b2 = − 2b1 and b3 = − b1 satisfies the Eq. (i)
∴ BX = V has no solution.
Planes are parallel.
1 2 5 27. Given, λx + (sin α ) y + (cos α ) z = 0
(d) D = 2 0 3 = 1(0 − 12) − 2 (− 10 − 3) + 5 (8 − 0) x + (cos α ) y + (sin α ) z = 0
1 4 −5 and − x + (sin α ) y − (cos α ) z = 0 has non-trivial
= 54 solution.
D ≠0 ∴ ∆ =0
It has unique solution for any b1, b2, b3 . λ sin α cos α
⇒ 1 cos α sin α = 0
25. Given system λx + y + z = 0, − x + λy + z = 0
−1 sin α − cos α
and − x − y + λz = 0
will have non-zero solution, if ⇒ λ (− cos 2 α − sin 2 α ) − sin α (− cos α + sin α )
λ 1 1 + cos α (sin α + cos α ) = 0
−1 λ 1 = 0 ⇒ − λ + sin α cos α + sin α cos α − sin 2 α + cos 2 α = 0
−1 −1 λ ⇒ λ = cos 2α + sin 2α
Q − a 2 + b2 ≤ a sin θ + b cos θ ≤ a 2 + b2 
⇒ λ (λ 2 + 1) − 1 (− λ + 1) + 1(1 + λ ) = 0  
⇒ λ3 + λ + λ − 1 + 1 + λ = 0 ∴ − 2≤λ≤ 2 …(i)
⇒ λ 3 + 3λ = 0 Again, when λ = 1, cos 2α + sin 2α = 1
⇒ λ (λ 2 + 3) = 0 ⇒ λ =0 1 1 1
⇒ cos 2α + sin 2α =
26. Since, AX = U has infinitely many solutions. 2 2 2
a 0 1 ⇒ cos (2α − π / 4) = cos π / 4
⇒| A| = 0 ⇒ 1 c b =0 ∴ 2α − π / 4 = 2n π ± π / 4
1 d b
⇒ 2α = 2nπ − π / 4 + π / 4 or 2α = 2nπ + π / 4 + π / 4
⇒ a (bc − bd ) + 1(d − c) = 0 ⇒ (d − c)(ab − 1) = 0
∴ α = nπ or nπ + π /4
∴ ab = 1 or d=c
rankershalt.com

158 Matrices and Determinants

28. Since, α 1 , α 2 are the roots of ax2 + bx + c = 0. ⇒ 3 (2λ + 15) + 1 (λ + 18) + 4 (5 − 12) ≠ 0
b c ⇒ 7 (λ + 5) ≠ 0
⇒ α1 + α 2 = − and α 1α 2 = ...(i) ⇒ λ ≠ −5
a a
Also, β1 , β 2 are the roots of px2 + qx + r = 0. Let z = − k, then equations become
q r 3x − y = 3 − 4k
⇒ β1 + β 2 = − and β1β 2 = ...(ii)
p p and x + 2 y = 3k − 2
On solving, we get
Given system of equations
4 − 5k 13k − 9
α 1 y + α 2z = 0 x= ,y= ,z=k
7 7
and β1 y + β 2 z = 0, has non-trivial solution.
31. Given system of equations are
α1 α 2 α 1 β1
∴ =0 ⇒ = 3x + my = m and 2x − 5 y = 20
β1 β 2 α 2 β2
3 m
α 1 + α 2 β1 + β 2 Here, ∆= = −15 − 2m
Applying componendo-dividendo, = 2 −5
α 1 − α 2 β1 − β 2
m m
⇒ (α 1 + α 2) (β1 − β 2) = (α 1 − α 2) (β1 + β 2) and ∆x = = −25m
20 −5
⇒ (α 1 + α 2)2 {(β1 + β 2)2 − 4 β 2 β 2}
3 m
= (β1 + β 2)2{(α 1 + α 2)2 − 4 α 1α 2} ∆y = = 60 − 2m
2 20
From Eqs. (i) and (ii), we get
If ∆ = 0, then system is inconsistent, i.e. it has no
b2  q2 4r  q2  b2 4c
 − =  −  solution.
a 2  p2 p  p2  a 2 a 
15
If ∆ ≠ 0, i.e. m ≠ , the system has a unique solution
b2q2 4b2r b2q2 4q2c 2
⇒ − = −
a 2p2 a 2p a 2p2 ap2 for any fixed value of m.
∆ − 25m 25m
b2r q2c b2 ac We have, x= x = =
⇒ = ⇒ 2= ∆ − 15 − 2 m 15 + 2m
a p q pr
∆y 60 − 2m 2m − 60
29. The system of equations has non-trivial solution, if ∆ = 0. and y= = =
∆ − 15 − 2m 15 + 2m
sin 3θ −1 1
25m
⇒ cos 2θ 4 3 = 0 For x > 0 , >0
15 + 2m
2 7 7
⇒ m >0
Expanding along C1 , we get 15
or m<− …(i)
sin 3θ ⋅ (28 − 21) − cos 2 θ (−7 − 7) + 2 (−3 − 4) = 0 2
⇒ 7 sin 3θ + 14 cos 2θ − 14 = 0 2m − 60 15
and y > 0, > 0 ⇒ m > 30 or m < − …(ii)
⇒ sin 3θ + 2 cos 2 θ − 2 = 0 2m + 15 2
⇒ 3 sin θ − 4 sin3 θ + 2 (1 − 2 sin 2 θ ) − 2 = 0 15
From Eqs. (i) and (ii), we get m < − or m > 30
⇒ sin θ (4 sin 2 θ + 4 sin θ − 3) = 0 2
⇒ sin θ (2 sin θ − 1) (2 sin θ + 3) = 0
1 32. Since, the given system of equations posses non-trivial
⇒ sin θ = 0, sin θ = 0 1 −2
2
solution, if 0 −3 1 = 0 ⇒ k = 0
[neglecting sin θ = − 3 / 2]
k −5 4
π
⇒ θ = nπ , nπ + (−1)n , n ∈ Z
6 On solving the equations x = y = z = λ [say]
30. The given system of equations ∴ For k = 0, the system has infinite solutions of λ ∈R.
3x − y + 4z = 3
x + 2 y − 3z = − 2 33. Given systems of equations can be rewritten as
6 x + 5 y + λz = − 3 − x + cy + by = 0, cx − y + az = 0 and bx + ay − z = 0
has atleast one solution, if ∆ ≠ 0. Above system of equations are homogeneous equation.
3 −1 4 Since, x, y and z are not all zero, so it has non-trivial
∴ ∆ = 1 2 −3 ≠ 0 solution.
6 5 λ
rankershalt.com

Matrices and Determinants 159

Therefore, the coefficient of determinant must be zero. 0 − 1  1  1


−1 c b ∴ M 1 =  2 ⇒ M − 1 =  1
   
∴ c −1 a = 0 0  3  0 − 1
b a −1 1  0
⇒ − 1 (1 − a ) − c (− c − ab) + b (ca + b) = 0
2 M ⋅ 1 =  0
⇒ a 2 + b2 + c2 + 2abc − 1 = 0 1 12
⇒ a 2 + b2 + c2 + 2abc = 1 a 2 − 1 a1 − a 2  1
⇒  b  =  2,  b − b  =  1,
1 α α 2
 2    1 2  
34. α 1 α =0  c2   3  c1 − c2  − 1
α2 α 1 a1 + a 2 + a3   0
 b + b + b  =  0
⇒ α 4 − 2α 2 + 1 = 0  1 2 3   
⇒ α2 = 1  c1 + c2 + c3  12
⇒ α = ±1 ⇒ a 2 = − 1, b2 = 2, c2 = 3, a1 − a 2 = 1,
But α = 1 not possible [Not satisfying equation] b1 − b2 = 1, c1 − c2 = − 1
∴ α = −1 ⇒ a1 + a 2 + a3 = 0, b1 + b2 + b3 = 0
Hence, 1 + α + α 2 = 1 c1 + c2 + c3 = 12
a1 a 2 a3  ∴ a1 = 0, b2 = 2, c3 = 7
35. Let M =  b1 b2 b3  ⇒ Sum of diagonal elements = 0 + 2 + 7 = 9
 c1 c2 c3 

Download Chapter Test


http://tinyurl.com/yxfq2tq7 or
rankershalt.com

8
Functions
Topic 1 Classification of Functions, Domain and
Range and Even, Odd Functions
Objective Questions I (Only one correct option) 7. The domain of definition of the function y (x) is given by
1. The domain of the definition of the function the equation 2x + 2y = 2 , is (2000, 1M)
1 (a) 0 < x ≤ 1 (b) 0 ≤ x ≤ 1
f (x) = + log10 (x3 − x) is (2019 Main, 9 April II)
4 − x2 (c) − ∞ < x ≤ 0 (d) −∞ < x < 1
(a) (−1, 0) ∪ (1, 2) ∪ (3, ∞ ) (b) (−2, − 1) ∪ (−1, 0) ∪ (2, ∞ ) 8. Let f (θ ) = sin θ (sin θ + sin 3 θ ). Then, f (θ ) (2000, 1M)
(c) (−1, 0) ∪ (1, 2) ∪ (2, ∞ ) (d) (1, 2) ∪ (2, ∞ ) (a) ≥ 0, only when θ ≥ 0 (b) ≤ 0, for all real θ
2. Let f (x) = a x (a > 0) be written as f (x) = f1 (x) + f2(x), (c) ≥ 0, for all real θ (d) ≤ 0, only when θ ≤ 0
where f1 (x) is an even function and f2(x) is an odd
9. The domain of definition of the function
function. Then f1 (x + y) + f1 (x − y) equals 1
(2019 Main, 8 April II) y= + x + 2 is
log10 (1 − x) (1983, 1M)
(a) 2f1 (x + y) ⋅ f2 (x − y) (b) 2f1 (x + y) ⋅ f1 (x − y)
(c) 2f1 (x) ⋅ f2 ( y) (d) 2f1 (x) ⋅ f1 ( y) (a) (− 3, − 2) excluding − 2. 5 (b) [0, 1] excluding 0.5
(c) (−2, 1) excluding 0 (d) None of these
3. Domain of definition of the function
π
f (x) = sin −1 (2x) + for real valued x, is Match the Columns
6 (2003, 2M)
Match the conditions / expressions in Column I with
(a)  − ,
1
(b)  − ,
1 1 1
statement in Column II.
 4 2   2 2 
x2 − 6x + 5
(c)  − ,
1
(d)  − ,
1 1 1 10. Let f (x) = .
  4
 2 9 4  x2 − 5x + 6 (2007, 6M)
x + x+2
2
Column I Column II
4. Range of the function f (x) = ; x ∈ R is
x2 + x + 1 (2003, 2M) A. If −1 < x < 1, then f ( x ) satisfies p. 0 < f (x ) < 1
(a) (1, ∞) (b) (1, 11/7)
(c) (1, 7/3] (d) (1, 7/5) B. If 1 < x < 2 , then f ( x ) satisfies q. f (x ) < 0

5. Let f (x) = (1 + b ) x + 2bx + 1 and let m (b) be the


2 2
C. If 3 < x < 5, then f ( x ) satisfies r. f (x ) > 0
minimum value of f (x). As b varies, the range of m (b) is D. If x > 5, then f ( x ) satisfies s. f (x ) < 1
(2001, 1M)

(b)  0, 
1
(a) [0, 1]
 2  Objective Question II
(c)  , 1
1 (One or more than one correct option)
(d) (0, 1]
 2 
2x − 1
log 2(x + 3) 11. If S is the set of all real x such that is
6. The domain of definition of f (x) = is 2x3 + 3x2 + x
x2 + 3x + 2
positive, then S contains (1986, 2M)
(2001, 1M)
(a)  − ∞ , −  (b)  − , − 
3 3 1
(a) R / {− 1, − 2}
 2  2 4
(b) (− 2, ∞ )
(c)  − ,  (d)  , 3
(c) R / {− 1, − 2, − 3} 1 1 1
(d) (− 3, ∞ ) / {− 1, − 2}  4 2 2 
rankershalt.com

Functions 161

Fill in the Blanks True/False


 4 − x2  15. If f1 (x) and f2(x) are defined on domains D1 and D2
12. If f (x) = sin log   , then the domain of
respectively, then f1 (x) + f2(x) is defined on D1 ∩ D2.
 1−x 
 
(1988, 1M)
f (x) is… . (1985, 2M)
 x2 Analytical & Descriptive Questions
13. The domain of the function f (x) = sin −1  log 2  is 16. Find the range of values of t for which
 2
given by ...
1 − 2x + 5x2  π π
 π2 
(1984, 2M) 2 sin t = , t∈ − , .
3x2 − 2x − 1  2 2 
14. The values of f (x) = 3 sin  − x2 lie in the

(2005, 2M)
 16 
(x + 1) (x − 3)
interval… (1983, 2M) 17. Let y= .
(x − 2)
Find all the real values of x for which y takes real
values. (1980, 2M)

Topic 2 Composite of Functions


Objective Questions I (Only one correct option) 6. Let a , b, c ∈ R. If f (x) = ax2+ bx + c be such that
 3 a + b + c = 3 and f (x + y) = f (x) + f ( y) + xy, ∀ x, y ∈ R,
1. For x ∈ 0,  , let f (x) = x , g (x) = tan x and 10
 2 then ∑ f (n ) is equal to
1 − x2  π n =1
h (x) = . If φ(x) = ((hof )og )(x), then φ   is equal to (2017 Main)
1+ x 2  
(20193Main, 12 April I) (a) 330 (b) 165 (c) 190 (d) 255
π 11π 7. Let f (x) = x2 and g (x) = sin x for all x ∈ R. Then, the set of
(a) tan (b) tan
12 12
7π 5π
all x satisfying ( fogogof )(x) = ( gogof )(x) , where
(c) tan (d) tan ( fog )(x) = f ( g (x)), is (2011)
12 12
(a) ± nπ, n ∈ {0, 1, 2, K }
2. Let f (x) = x2, x ∈ R. For any A ⊆ R, define
(b) ± nπ, n ∈ {1, 2, K }
g ( A ) = { x ∈ R : f (x) ∈ A }. If S = [0, 4], then which one of
the following statements is not true? (c) π /2 + 2nπ, n ∈ {...,− 2, − 1, 0, 1, 2, K }
(2019 Main, 10 April I) (d) 2nπ, n ∈ {..., − 2, − 1, 0, 1, 2, K }
αx
(a) f ( g (S)) = S (b) g (f (S)) ≠ S 8. Let f (x) = , x ≠ − 1. Then, for what value of α is
(c) g (f (S)) = g (S) (d) f(g(S)) ≠ f (S)
x+1
10 f [ f (x)] = x ? (2001, 1M)
3. Let ∑ f (a + k) = 16(210 − 1), where the function f (a) 2 (b) − 2 (c) 1 (d) −1
k =1
− 1 , x < 0
satisfies f (x + y) = f (x) f ( y) for all natural numbers x, y 
9. Let g (x) = 1 + x − [x] and f (x) =  0, x = 0 , then for all
and f (1) = 2. Then, the natural number ‘a’ is  1, x > 0
(2019 Main, 9 April I) 
(a) 2 (b) 4 x, f [ g (x)] is equal to (2001, 1M)
(c) 3 (d) 16 (a) x (b) 1
 1 − x  2x  (c) f (x) (d) g (x)
4. If f (x) = log e   ,|x| < 1, then f  is equal to
 1 + x  1 + x2 10. If g { f (x) } = |sin x| and f { g (x) } = (sin x )2, then
(2019 Main, 8 April I)
(a) 2f (x) (b) 2f (x2 ) (1998, 2M)
(c) (f (x))2 (d) −2f (x) (a) f (x) = sin 2 x, g (x) = x
1 (b) f (x) = sin x, g (x) = | x|
5. For x ∈ R − {0, 1}, let f1 (x) = , f2(x) = 1 − x and (c) f (x) = x2 , g (x) = sin x
x
1 (d) f and g cannot be determined
f3 (x) = be three given functions. If a function, J (x)
1 −x 1   x 
11. If f (x) = cos(log x), then f (x) ⋅ f ( y) −  f   + f (xy)
satisfies ( f2° J ° f1 )(x) = f3 (x), then J (x) is equal to 2   y 
(2019 Main, 9 Jan I)
has the value (1983, 1M)
(a) f2 (x) (b) f3 (x) 1
1 (a) −1 (b)
(c) f1 (x) (d) f3 (x) 2
x (c) − 2 (d) None of these
rankershalt.com

162 Functions

12. Let f (x) = | x − 1|. Then, (1983, 1M) (a) g (x) = ± 1 − x2 (b) g (x) = 1 − x2
(a) f (x2 ) = {f (x) }2 (c) g (x) = − 1 − x2 (d) g (x) = 1 + x2
(b) f (x + y) = f (x) + f ( y)
x+2
(c) f (| x|) = | f (x)| 16. If y = f (x) = , then
(d) None of the above x−1 (1984, 3M)
(a) x = f ( y) (b) f (1) = 3
Objective Questions II (c) y increases with x for x < 1
(One or more than one correct option) (d) f is a rational function of x

π π  Fill in the Blanks


13. Let f (x) = sin  sin  sin x  for all x ∈ R and
6 2 
π  π  π
g (x) = sin x for all x ∈ R. Let ( fog )(x) denotes f { g (x)} 17. If f (x) = sin 2 x + sin 2 x +  + cos x cos  x +  and
2  3  3
and (gof ) (x) denotes g{f (x)}. Then, which of the  5
g   = 1, then (g o f ) (x) = ... .
following is/are true? (2015 Adv.)  4 (1996, 2M)
(a) Range of f is  − ,  (b) Range of fog is  − , 
1 1 1 1
 2 2   2 2 
True/False
f (x ) π
(c) lim = 18. If f (x) = (a − xn )1/ n, where a > 0 and n is a positive
x → 0 g (x ) 6
(d) There is an x ∈ R such that ( gof ) (x) = 1 integer, then f [ f (x)] = x. (1983, 1M)

14. If f (x) = cos [π 2] x + cos [− π 2] x, where [x] stands for the


Analytical & Descriptive Questions
greatest integer function, then (1991, 2M)
(a) f ( π / 2) = − 1 (b) f ( π ) = 1 19. Find the natural number a for which
n
(c) f (− π ) = 0 (d) f ( π / 4) = 1
∑ f (a + k) = 16 (2n − 1),
15. Let g (x) be a function defined on [− 1, 1]. If the area of k =1

the equilateral triangle with two of its vertices at (0, 0) where the function f satisfies the relation
and [x, g (x)] is 3 / 4, then the function g (x) is f (x + y) = f (x) f ( y) for all natural numbers x , y and
(1989, 2M) further f (1) = 2. (1992, 6M)

Topic 3 Types of Functions


Objective Questions I (Only one correct option) 4. Let f : R → R be defined by f (x) =
x
,
1 + x2
1. If the function f : R − {1, − 1} → A defined by
x2 x ∈ R. Then, the range of f is (2019 Main, 11 Jan I)
f (x) = , is surjective, then A is equal to
(a)  − , 
1 1
1 − x2 (b) (−1, 1) − {0}
(2019 Main, 9 April I)  2 2 
(a) R − {−1}
(c) R −  − , 
1 1
(b) [0, ∞ ) (d) R − [−1, 1]
 2 2 
(c) R − [−1, 0)
(d) R − (−1, 0) 5. Let N be the set of natural numbers and two functions f
and g be defined as f , g : N → N such that
2. Let a function f : (0, ∞ ) → (0, ∞ ) be defined by
n + 1
1 ; if n is odd
f (x) = 1 − . Then, f is 
x f (n ) =  2
(2019 Main, 11 Jan II) n
 ; if n is even
(a) injective only  2
(b) both injective as well as surjective and g (n ) = n − (−1)n. Then, fog is (2019 Main, 10 Jan II)
(c) not injective but it is surjective (a) one-one but not onto (b) onto but not one-one
(d) neither injective nor surjective (c) both one-one and onto (d) neither one-one nor onto
3. The number of functions f from {1, 2, 3, … , 20} onto {1, 6. Let A = { x ∈ R : x is not a positive integer}. Define a
2, 3, … , 20} such that f (k) is a multiple of 3, whenever k 2x
is a multiple of 4, is (2019 Main, 11 Jan II) function f : A → R as f (x) = , then f is
x−1 (2019 Main, 9 Jan II)
(a) (15)! × 6!
(a) injective but not surjective
(b) 56 × 15
(b) not injective
(c) 5! × 6!
(c) surjective but not injective
(d) 65 × (15)!
(d) neither injective nor surjective
rankershalt.com

Functions 163

7. The function f : R → − ,  defined as f (x) =


1 1 x Codes
is
 2 2 1 + x2 A B C D
(2017 Main) (a) r p s q
(a) invertible (b) p r s q
(b) injective but not surjective (c) r p q s
(c) surjective but not injective (d) p r q s
(d) neither injective nor surjective 14. Let the functions defined in Column I have domain
8. The function f: [0, 3] → [1, 29], defined by (−π /2, π /2) and range (−∞ , ∞ ) (1992, 2M)

f (x) = 2x − 15 x + 36x + 1, is
3 2
(2012) Column I Column II
(a) one-one and onto (b) onto but not one-one A. 1+ 2x p. onto but not one-one
(c) one-one but not onto (d) neither one-one nor onto B. tan x q. one-one but not onto
x, if x is rational 0, if x is rational r. one-one and onto
9. f (x) =  , g (x) = 
0, if x is irrational x, if x is irrational s. neither one-one nor onto
Then, f − g is (2005, 1M)
(a) one-one and into (b) neither one-one nor onto Objective Question II
(c) many one and onto (d) one-one and onto (One or more than one correct option)
x π π
10. If f : [0, ∞ ) → [0, ∞ ) and f (x) =
1+ x
, then f is 15. Let f :  − ,  → R be given by
(2003, 2M)  
2 2
(a) one-one and onto (b) one-one but not onto f (x) = [log(sec x + tan x)]3 . Then,
(c) onto but not one-one (d) neither one-one nor onto (a) f (x) is an odd function
11. Let function f : R → R be defined by f (x) = 2 x + sin x (b) f (x) is a one-one function
for x ∈ R . Then, f is (2002, 1M) (c) f (x) is an onto function
(d) f (x) is an even function
(a) one-to-one and onto (b) one-to-one but not onto
(c) onto but not one-to-one (d) neither one-to-one nor onto
Fill in the Blanks
12. Let E = {1, 2, 3, 4} and F = {1, 2}. Then, the number of
onto functions from E to F is (2001, 1M)
16. There are exactly two distinct linear functions, …,
and… which map {– 1, 1} onto {0, 2}. (1989, 2M)
(a) 14 (b) 16
(c) 12 (d) 8
True/False
Match the Columns x2 + 4x + 30
17. The function f (x) = is not one-to-one.
Match the conditions/expressions in Column I with x2 − 8x + 18 (1983, 1M)
statement in Column II.
13. Let f1 : R → R, f2 : [0, ∞ ] → R, f3 : R → R and Analytical & Descriptive Question
f4 : R → [0, ∞ ) be defined by (2014 Adv.) 18. A function f : IR → IR, where IR, is the set of real
|x|, if x < 0 sin x, if x < 0 αx2 + 6x − 8
f1 (x) =  x ; f2(x) = x2; f3 (x) =  numbers, is defined by f (x) = .
 e , if x ≥ 0  x, if x ≥ 0 α + 6 x − 8 x2
 f [ f (x)], if x < 0 Find the interval of values of α for which is onto. Is the
and f4 (x) =  2 1
 f2[ f1 (x)] − 1, if x ≥ 0 functions one-to-one for α = 3 ? Justify your answer.
(1996, 5M)
Column I Column II 19. Let A and B be two sets each with a finite number of
A. f4 is p. onto but not one-one elements. Assume that there is an injective mapping
from A to B and that there is an injective mapping
B. f3 is q. neither continuous nor one-one
from B to A. Prove that there is a bijective mapping
C. f2of1 is r. differentiable but not one-one from A to B. (1981, 2M)
D. f2 is s. continuous and one-one
rankershalt.com

164 Functions

Topic 4 Inverse and Periodic Functions


Objective Questions I (Only one correct option) 7. Which of the following functions is periodic? (1983, 1M)

1. If X and Y are two non-empty sets where f : X → Y , is (a) f (x) = x − [ x ], where [x] denotes the greatest integer
less than or equal to the real number x
function is defined such that
(b) f (x) = sin (1 /x) for x ≠ 0, f (0) = 0
f (C ) = { f (x) : x ∈ C } for C ⊆ X and (c) f (x) = x cos x
f −1 (D ) = { x : f (x) ∈ D } for D ⊆ Y , (d) None of the above
for any A ⊆ Y and B ⊆ Y , then (2005, 1M)
−1
(a) f {f (A )} = A
Objective Question II
(b) f −1 {f (A )} = A, only if f (X ) = Y (One or more than one correct option)
(c) f {f −1 (B )} = B, only if B ⊆ f (x) b−x
8. Let f : (0, 1) → R be defined by f (x) = , where b is a
(d) f {f −1 (B )} = B 1 − bx
constant such that 0 < b < 1. Then, (2011)
2. If f (x) = sin x + cos x, g (x) = x2 − 1, then g { f (x) } is
(a) f is not invertible on (0, 1)
invertible in the domain (2004, 1M)
1
(b) f ≠ f −1 on (0, 1) and f ′ (b) =
π π π
(a)  0,  (b)  − ,  f ′ (0)
 2   4 4  (c) f = f −1 on (0, 1) and f ′ (b) =
1
π π f ′ (0)
(c)  − ,  (d) [0 , π ]
 2 2  (d) f −1 is differentiable on (0, 1)

3. Suppose f (x) = (x + 1)2 for x ≥ − 1. If g (x) is the function Assertion and Reason
whose graph is reflection of the graph of f (x) with
respect to the line y = x, then g (x) equals For the following questions, choose the correct answer
(2002, 1M) from the codes (a), (b), (c) and (d) defined as follows.
1 (a) Statement I is true, Statement II is also true;
(a) − x − 1, x ≥ 0 (b) ,x> −1
(x + 1)2 Statement II is the correct explanation of Statement I.
(c) x + 1 , x ≥ − 1 (d) x − 1, x ≥ 0 (b) Statement I is true, Statement II is also true;
1 Statement II is not the correct explanation of
4. If f : [1, ∞ ) → [2, ∞ ) is given by f (x) = x + , then f −1(x) Statement I.
x
equals (2001, 1M) (c) Statement I is true; Statement II is false.
x+ x −42 (d) Statement I is false; Statement II is true.
x
(a) (b)
2 1 + x2 9. Let F (x) be an indefinite integral of sin 2 x.
x− x2 − 4 Statement I The function F (x) satisfies
(c) (d) 1 + x2 − 4 F (x + π ) = F (x) for all real x.
2 Because
5. If the function f : [1, ∞ ) → [1, ∞ ) is defined by Statement II sin 2(x + π ) = sin 2 x, for all real x.
x ( x − 1)
f (x) = 2 , then f −1 (x) is (1999, 2M) (2007, 3M)
x ( x − 1)
(a)  
1
(b)
1
(1 + 1 + 4 log2 x ) Analytical & Descriptive Question
 2 2
1
10. Let f be a one-one function with domain { x, y, z } and
(c) (1 − 1 + 4 log2 x ) (d) not defined range {1, 2, 3}. It is given that exactly one of the
2
following statements is true and the remaining two
6. If f (x) = 3x − 5, then f −1 (x) (1998, 2M) are false f (x) = 1, f ( y) ≠ 1, f (z ) ≠ 2 determine f −1 (1) .
1 (1982, 2M)
(a) is given by
3x − 5
11. If f is an even function defined on the interval (− 5, 5),
x+ 5
(b) is given by then four real values of x satisfying the equation
3  x + 1
(c) does not exist because f is not one-one f (x) = f   are ………. .
 x + 2 (1996, 1M)
(d) does not exist because f is not onto
rankershalt.com

Functions 165

Answers
Topic 1 Topic 3
1. (c) 2. (d) 3. (a) 4. (c) 1. (c) 2. (d) 3. (a) 4. (a)
5. (d) 6. (d) 7. (d) 8. (c) 5. (b) 6. (a) 7. (c) 8. (b)
9. (c) 10. A → p; B → q; C → q; D → p 9. (d) 10. (b) 11. (a) 12. (a)
11. (a,d) 12. (–2,1) 13. (d) 14. A → q; B → r
 3  15. (a, b, c) 16. y = x + 1 and y = − x + 1
13. Domain ∈ [ −2,−1 ] ∪ [1, 2 ] 14. 0, 15. True
 2  17. True 18. 2 ≤ α ≤ 14, No
 π π 3π π 
16. t ∈ − , ∪ , 17. x ∈ [ −1, 2 ) ∪ [3, ∞ ) Topic 4
 2 10   10 2 
1. (c) 2. (b) 3. (d) 4. (a)
Topic 2 5. (b) 6. (b) 7. (a) 8. (b)
1. (b) 2. (c) 3. (c) 4. (a) 9. (d)
5. (b) 6. (a) 7. (b) 8. (d)  ± 3 ± 5
−1
9. (b) 10. (a) 11. (d) 12. (d) 10. f (1 ) = y 11.  
 2 
13. (a,b,c) 14. (a, c) 15. (b, c) 16. (a, d)
17. 1 18. True 19. (a = 3)

Hints & Solutions


Topic 1 Classification of Functions, 1 x 1  y 1
= a + x a + y
Domain and Range 2 a  a 
1  a x + a −x   a y + a −y 
1. Given function f (x) = + log10 (x3 − x) =2   = 2 f1 (x) ⋅ f1 ( y)
4 − x2  2  2 
For domain of f (x)
4 − x2 ≠ 0 ⇒ x ≠ ± 2 π
…(i) 3. Here, f (x) = sin −1 (2x) + , to find domain we must
and x3 − x > 0 6
⇒ x(x − 1)(x + 1) > 0 have,
π  π π
From Wavy curve method, sin −1 (2x) +
≥0 but − ≤ sin −1 θ ≤
6  2 2 
π π
+ + − ≤ sin −1 (2x) ≤
–∞ +∞ 6 2
– +1
–1 0 –  – π π –1 1
sin   ≤ 2x ≤ sin ⇒ ≤ 2x ≤
 6  2 2 2
x ∈ (−1, 0) ∪ (1, ∞ ) …(ii) –1 1
≤x≤
From Eqs. (i) and (ii), we get the domain of f (x) as 4 2
(−1, 0) ∪ (1, 2) ∪ (2, ∞ ). – 1 1
Q x∈ ,
 4 2 
2. Given, function f (x) = a x , a > 0 is written as sum of an
even and odd functions f1 (x) and f2(x) respectively. x2 + x + 2
4. Let y = f (x) = , x ∈R
a x + a −x a x − a −x x2 + x + 1
Clearly, f1 (x) = and f2(x) =
2 2 x2 + x + 2
∴ y=
So, f1 (x + y) + f1 (x − y) x2 + x + 1
1 1 1
= [a x + y + a − ( x + y ) ] + [a x − y + a − ( x − y ) ] y=1 + 2 [i.e. y > 1] …(i)
2 2 x + x+1
1 x y 1 ax ay  ⇒ yx2 + yx + y = x2 + x + 2
= a a + x y + y + x 
2 a a a a  ⇒ x ( y − 1) + x ( y − 1) + ( y − 2) = 0, ∀ x ∈ R
2

1  x y Since, x is real, D ≥ 0
1 1  1 y 
= a a + y + x  y + a  ⇒ ( y − 1 )2 − 4 ( y − 1 ) ( y − 2 ) ≥ 0
2   a  a a 
⇒ ( y − 1) {( y − 1) − 4 ( y − 2)} ≥ 0
rankershalt.com

166 Functions

⇒ ( y − 1) (− 3 y + 7) ≥ 0 A. If − 1 < x < 1 ⇒ 0 < f (x) < 1


⇒ 1≤ y≤
7
…(ii) B. If 1 < x < 2 ⇒ f (x) < 0
3 C. If 3 < x < 5 ⇒ f (x) < 0
 7
From Eqs. (i) and (ii), Range ∈ 1 , D. If x > 5 ⇒ 0 < f (x) < 1
 3 
2x − 1
11. Since, >0
5. Given, f (x) = (1 + b2) x2 + 2bx + 1 2x + 3x2 + x
3

 b 
2
b2 (2x − 1)
= (1 + b )  x +
2
 + 1 − ⇒ >0
 1 + b2 1 + b2 x(2x2 + 3x + 1)
(2x − 1)
m (b) = minimum value of f (x) =
1
is positive ⇒ >0
1 + b2 x (2x + 1) (x + 1)
and m (b) varies from 1 to 0, so range = (0, 1] + − + − +
−∞ ∞
log (x + 3) log 2 (x + 3) −1 −1/2 0 1/2
6. Given, f (x) = 2 2 =
(x + 3x + 2) (x + 1) (x + 2) Hence, the solution set is,
For numerator, x + 3 > 0 x ∈ (−∞ , − 1) ∪ (−1/2, 0) ∪ (1 / 2, ∞ )
⇒ x> −3 …(i) Hence, (a) and (d) are the correct options.
and for denominator, (x + 1) (x + 2) ≠ 0  4 − x2 
⇒ x≠ −1 ,−2 12. Given, f (x) = sin log  
…(ii)  1−x 
 
From Eqs. (i) and (ii),
Domain is (− 3 , ∞ ) /{ − 1, − 2} 4 − x2
For domain, > 0 , 4 − x2 > 0 and 1 − x ≠ 0
1−x
7. Given, 2x + 2y = 2, ∀ x , y ∈ R
But 2x , 2y > 0, ∀ x , y ∈ R ⇒ (1 − x) > 0 and 4 − x2 > 0
Therefore, 2x = 2 − 2y < 2 ⇒ 0 < 2x < 2 ⇒ x<1 and | x| < 2 ⇒ −2 < x < 1
Taking log on both sides with base 2, we get Thus, domain ∈ (−2, 1).
log 2 0 < log 2 2x < log 2 2 ⇒ − ∞ < x < 1  x2
13. Given, f (x) = sin −1  log 2 
8. It is given,  2

f (θ ) = sin θ (sin θ + sin 3 θ ) x2


For domain, −1 ≤ log 2 ≤ 1
= (sin θ + 3 sin θ − 4 sin3 θ ) sin θ 2
= (4 sin θ − 4 sin3 θ ) sin θ = sin 2 θ (4 − 4 sin 2 θ ) 1 x2
⇒ ≤ ≤2
= 4 sin 2 θ cos 2 θ = (2 sin θ cos θ )2 2 2
= (sin 2θ )2 ≥ 0 ⇒ 1 ≤ x2 ≤ 4
which is true for all θ. ⇒ 1 ≤ |x| ≤ 2
9. For domain of y, ⇒ Domain ∈ [−2, − 1] ∪ [1, 2]
1 − x > 0, 1 − x ≠ 1 and x+ 2 >0 π2
14. Given, f (x) = 3 sin − x2
⇒ x < 1, x ≠ 0 and x > −2 16
⇒ −2 < x < 1 excluding 0  π π
⇒ Domain ∈ − ,
⇒ x ∈ (−2, 1) − {0}  4 4 
(x − 1) (x − 5)  π2 
10. Given, f (x) = 1(−2x)
(x − 2) (x − 3) ∴ For range, f ′ (x) = 3 cos  − x2  ⋅ =0
 16 
  π2
The graph of f (x) is shown as : 2 − x2
16
Y  π2 
Where, cos  − x2 = 0 or x=0
 16 
 
y=1   π2  π2 π2
neglecting cos  − x2 = 0 ⇒

− x2 = 
X' X   16  16 4
0 1 2 3 5  3π 2 
⇒ x = −
2
, never possible 
 16 
Y' ⇒ x =0
rankershalt.com

Functions 167

π 3
Thus, f (0) = 3 sin = Topic 2 Composite of Functions and
4 2
Even, Odd Functions
 π  π
and f −  = f   =0 1. Given, for x ∈ (0, 3 / 2), functions
 4  4
f (x) = x … (i)
 3 
Hence, range ∈ 0, g (x) = tan x
 2 
… (ii)
1 − x2
15. Since, domains of f1 (x) and f2(x) are D1 and D2 . and h (x) = … (iii)
1 + x2
Thus, domain of [ f1 (x) + f2(x)] is D1 ∩ D2 .
Also given, φ(x) = ((hof )og )(x) = (hof ) ( g (x))
Hence, given statement is true.
= h ( f ( g (x)))
1 − 2x + 5x2  π π
16. Given, 2 sin t = ,t ∈ − , = h ( f (tan x))
3x − 2x − 1
2  2 2 
1 − ( tan x )2
= h ( tan x ) =
Put 2 sin t = y ⇒ − 2 ≤ y ≤ 2 1 + ( tan x )2
1 − 2 x + 5 x2 1 − tan x π 
∴ y= = = tan  − x
3 x2 − 2 x − 1 1 + tan x 4 
⇒ (3 y − 5)x2 − 2x( y − 1) − ( y + 1) = 0  π  π π
Now, φ   = tan  − 
 3  4 3
Since, x ∈ R − {1, − 1 / 3}
 3π − 4π   π
[as, 3x2 − 2x − 1 ≠ 0 ⇒ (x − 1)(x + 1 / 3) ≠ 0] = tan   = tan  − 
 12   12
∴ D ≥0
π  π
⇒ 4( y − 1)2 + 4(3 y − 5) ( y + 1) ≥ 0 = − tan   = tan  π − 
 12  12
⇒ y2 − y − 1 ≥ 0  11π 
2 = tan  
 1 5  12 
⇒ y−  − ≥0
 2 4
2. Given, functions f (x) = x2, x ∈ R
 1 5  1 5
⇒ y− −  y− +  ≥0 and g ( A ) = { x ∈ R : f (x) ∈ A }; A ⊆ R
 2 2  2 2 Now, for S = [0, 4]
1− 5 g (S ) = { x ∈ R : f (x) ∈ S = [0, 4]}
⇒ y≤ = { x ∈ R : x2 ∈ [0, 4]}
2
1+ 5 = { x ∈ R: x ∈ [−2, 2]}
or y≥ ⇒ g (S ) = [−2, 2]
2
1− 5 So, f ( g (S )) = [0, 4] = S
⇒ 2 sin t ≤ Now, f (S ) = { x2 : x ∈ S = [0, 4]} = [0, 16]
2
and g ( f (S )) = { x ∈ R : f (x) ∈ f (S ) = [0, 16]}
1+ 5
or 2 sin t ≥ = { x ∈ R : f (x) ∈ [0, 16]}
2
 π = { x ∈ R: x2 ∈ [0, 16]}
⇒ sin t ≤ sin  − 
 10 = { x ∈ R : x ∈ [−4, 4]} = [−4 ,4]
 3π  From above, it is clear that g ( f (S )) = g (S ).
or sin t ≥ sin  
 10  3. Given, f (x + y) = f (x) ⋅ f ( y)
π 3π Let f (x) = λx [where λ > 0]
⇒ t≤− or t≥
10 10 Q f (1) = 2 (given)
 π π  3π π  ∴ λ =2
Hence, range of t is − , − ∪ , .
 2 10   10 2  10 10  10 
So, Σ f (a + k) = Σ λa+ k = λa  Σ λk
(x + 1) (x − 3) k =1 k =1  k=1 
17. Since, y = takes all real values only
(x − 2) = 2a [21 + 22 + 23 + ......+210 ]
(x + 1) (x − 3) 2(2 − 1) 
10
when ≥0 = 2a  
(x − 2)  2 −1 
− + − + [by using formula of sum of n-terms of a GP having
−∞ ∞ first term ‘a’ and common ratio ‘r’, is
−1 2 3
⇒ −1 ≤ x < 2 or x≥3 a (r n − 1) 
Sn = , where r > 1
∴ x ∈ [−1, 2) ∪ [3, ∞ ). r −1 
rankershalt.com

168 Functions

⇒ 2a+ 1 (210 − 1) = 16 (210 − 1) (given) ⇒ 2ax2 − x2 = 0


⇒ 2a+ 1 = 16 = 24 ⇒ a + 1 = 4 ⇒ a = 3 1
⇒ a=
 1 − x 2
4. Given, f (x) = log e   ,|x| < 1, then Also, a + b + c = 3
 1 + x
1 5
 2x  ⇒ + b + 0 =3⇒ b =
 1−  2 2
 2x  1 + x2    2x  
 x + 5x
2
f  = log e Q  < 1 ∴ f (x) =
 1 + x2   1 + 2x   1 + x  
2
2
 
 1 + x2 
n 2 + 5n 1 2 5
 1 + x2 − 2 x  Now, f (n ) = = n + n
  2
2 2 2
1 + x2   (1 − x)2   1 − x
= log e 
10 10 10
= log   = log   1 5
 1 + x2 + 2x 
 
e
 (1 + x)2
e
 1 + x ∴ ∑ f (n ) = 2 ∑ n 2 + 2 ∑ n
n =1 n =1 n =1
 1+ x 2

1 10 × 11 × 21 5 10 × 11
 1 − x = ⋅ + ×
= 2 log e   [Q log e| A|m = m log e| A|] 2 6 2 2
 1 + x
385 275 660
  1 − x  = + = = 330
= 2 f (x) Q f (x) = log e  1 + x 
2 2 2
  7. f (x) = x2, g (x) = sin x
5. We have, ( gof )(x) = sin x2
1 1
f1 (x) = , f2(x) = 1 − x and f3 (x) = go( gof ) (x) = sin (sin x2)
x 1−x
Also, we have ( f2 o J o f1 )(x) = f3 (x) ( fogogof ) (x) = (sin (sin x2))2 …(i)
⇒ f2((J o f1 )(x)) = f3 (x) Again, ( gof ) (x) = sin x 2

⇒ f2(J ( f1 (x)) = f3 (x) ( gogof ) (x) = sin (sin x2) …(ii)


1 ( fogogof ) (x) = ( gogof ) (x)
⇒ 1 − J ( f1 (x)) = Given,
1−x
⇒ (sin (sin x2))2 = sin (sin x2)
1
[Q f2(x) = 1 − x and f3 (x) = ] ⇒ sin (sin x2) {sin (sin x2) − 1} = 0
1−x
⇒ sin (sin x2) = 0 or sin (sin x2) = 1
 1 1 1
⇒ 1 − J  = [Q f1 (x) = ] π
 x 1 − x x ⇒ sin x2 = 0 or sin x2 =
2
 1 1
⇒ J  = 1 − ∴ x2 = nπ
 x 1−x
π
1 − x−1 −x [sin x2 = is not possible as − 1 ≤ sin θ ≤ 1]
= = 2
1−x 1−x
x = ± nπ
1
Now, put = X , then αx
x 8. Given, f (x) =
−1 x+1
 1  αx 
J (X ) = X Qx= α 
1−
1  X   αx   x + 1
f [ f (x) ] = f   =
X  x + 1 αx
+1
−1 1 x+1
= =
X −1 1 − X α 2x
⇒ J (X ) = f3 (X ) or J (x) = f3 (x) x+1 α 2x
= = = x [given] …(i)
6. We have, f (x) = ax2 + bx + c α x + (x + 1) (α + 1) x + 1
Now, f (x + y) = f (x) + f ( y) + xy x+1
Put y = 0 ⇒ f (x) = f (x) + f (0) + 0 ⇒ α 2 x = (α + 1) x2 + x
⇒ f (0) = 0 ⇒ x [α 2 − (α + 1) x − 1] = 0
⇒ c=0 ⇒ x(α + 1)(α − 1 − x) = 0
⇒ α − 1 = 0 and α + 1 = 0
Again, put y = − x
⇒ α = −1
∴ f (0) = f (x) + f (− x) − x2
But α = 1 does not satisfy the Eq. (i).
⇒ 0 = ax2 + bx + ax2 − bx − x2
rankershalt.com

Functions 169

9. g (x) = 1 + x − [x] is greater than 1 π π 


sin  sin  sin x 
f (x)  6  2 
since x − [x] > 0 (c) lim = lim
x → 0 g (x) x→ 0 π
f [ g (x)] = 1, since f (x) = 1 for all x > 0 (sin x)
2
10. Let f (x) = sin 2 x and g (x) = x π π  π π 
sin  sin  sin x  sin  sin x
 6 2  6 2 
Now, fog (x) = f [ g (x)] = f ( x ) = sin 2 x = lim ⋅
x→ 0 π π  π 
and gof (x) = g [ f (x)] = g (sin 2 x) = sin 2 x = |sin x| sin  sin x  sin x
6 2  2 
Again, let f (x) = sin x , g (x) = | x| π π
=1 × ×1 =
fog (x) = f [ g (x)] = f (| x|) 6 6
∴Option (c) is correct.
= sin| x|≠ (sin x )2
(d) g{ f (x)} = 1
When f (x) = x2, g (x) = sin x π
⇒ sin { f (x)} = 1
fog (x) = f [ g (x)] = f (sin x ) = (sin x )2 2
( gof ) (x) = g [ f (x)] = g (x2) = sin x2 2
and ⇒ sin { f (x)} = ...(i)
π
= sin| x|≠|sin x|  1 1  π π 
But f (x) ∈ − , ⊂ − ,
11. Given, f (x) = cos (log x)  2 2   6 6 

1   x   1 1
∴ sin { f (x)} ∈ − , ...(ii)
∴ f (x) ⋅ f ( y) −  f   + f (xy)  2 2 
2  y 
2
1 ⇒ sin { f (x)} ≠ , [from Eqs. (i) and (ii)]
= cos (log x) ⋅ cos(log y) − [cos (log x − log y) π
2
i.e. No solution.
+ cos(log x + log y)]
1 ∴ Option (d) is not correct.
= cos (log x) ⋅ cos (log y) − [(2 cos (log x) ⋅ cos (log y)]
2 14. Since, f (x) = cos [π 2] x + cos [−π 2] x
= cos (log x) ⋅ cos (log y) − cos (log x) ⋅ cos (log y) = 0 ⇒ f (x) = cos (9) x + cos (−10) x
12. Given, f (x) = |x − 1| [using [π 2] = 9 and [− π 2] = − 10]
π
  9 π
∴ f (x2) = |x2 − 1| ∴ f   = cos + cos 5π = − 1
 2 2
and { f (x)}2 = (x − 1)2
f (π ) = cos 9π + cos 10π = − 1 + 1 = 0
⇒ f (x2) ≠ ( f (x))2, hence (a) is false.
f (− π ) = cos 9π + cos 10π = − 1 + 1 = 0
Also, f (x + y) = |x + y − 1|
 π 9π 10π 1 1
and f (x) = |x − 1|, f   = cos + cos = +0=
 4 4 4 2 2
f ( y) = | y − 1|
⇒ f (x + y) ≠ f (x) + f ( y), hence (b) is false. Hence, (a) and (c) are correct options.
f (|x|) = ||x| − 1| 15. Since, area of equilateral triangle =
3
(BC )2
and | f (x)| = ||x − 1|| = |x − 1| 4
3 3
∴ f (|x|) ≠| f (x)|, hence (c) is false. ⇒ = ⋅ [x2 + g 2(x)] ⇒ g 2(x) = 1 − x2
4 4
π π 
13. (a) f (x) = sin  sin  sin x  , x ∈ R A
 6  2  
π   π π π
= sin  sin θ , θ ∈ − , , where θ = sin x
6   2 2  2
 π π π
= sin α, α ∈ − , ,where α = sin θ B C
 6 6  6 (0,0) (x,g(x))
 1 1
∴ f (x) ∈ − ,
 2 2  ⇒ g (x) = 1 − x2 or − 1 − x2
 1 1 Hence, (b) and (c) are the correct options.
Hence, range of f (x) ∈ − ,
 2 2  x+2
16. Given , y = f (x) =
So, option (a) is correct. x−1
 π π  1 1
(b) f { g (x)} = f (t ), t ∈ − , ⇒ f (t ) ∈ − , ⇒ yx − y = x + 2 ⇒ x( y − 1) = y + 2
 2 2   2 2 
y+2
∴ Option (b) is correct. ⇒ x= ⇒ x = f ( y)
y−1
rankershalt.com

170 Functions

Here, f (1) does not exist, so domain ∈ R − {1} Again, let f (k) is true.
dy (x − 1) ⋅ 1 − (x + 2) ⋅ 1 ⇒ f (k) = 2k, for some k ∈ N .
=
dx (x − 1)2 Again, f (k + 1) = f (k) ⋅ f (1) [by definition]
3 = 2k ⋅ 2 [from induction assumption]
=−
(x − 1)2 = 2k + 1
⇒ f (x) is decreasing for all x ∈ R − {1}. Therefore, the result is true for n = k + 1. Hence, by
Also, f is rational function of x. principle of mathematical induction,
Hence, (a) and (d) are correct options. f (n ) = 2n , ∀ n ∈ N
n n n
17. f (x) = sin 2 x + sin 2(x + π / 3) + cos x cos (x + π / 3) Now, ∑ f (a + k) = ∑ f (a ) f (k) = f (a ) ∑ 2k
⇒ f (x) = sin 2 x + (sin x cos π / 3 + cos x sin π / 3)2 k =1 k =1 k =1

+ cos x cos (x + π / 3) 2 (2n − 1)


2 = f (a ) ⋅
 sin x ⋅ 1 cos x . 3  2 −1
⇒ f (x) = sin 2 x +  + 
 2 2  = 2a ⋅ 2 (2n − 1) = 2a + 1 (2n − 1)
n
+ cos x (cos x cos π / 3 − sin x sin π / 3) But ∑ f (a + k) = 16 (2n − 1) = 24 (2n − 1)
sin x 3 cos x 2 ⋅ 3
2 2 k =1
⇒ f (x) = sin 2 x + + + sin x cos x
4 4 4 Therefore, a + 1 =4 ⇒ a =3
2
cos x 3
+− cos x sin x ⋅ Topic 3 Types of Functions
2 2
sin 2 x 3 cos 2 x cos 2 x 1. Given, function f : R – {1, − 1} → A defined as
= sin 2 x + + +
4 4 2 x2
5 5 5 f (x) = =y (let)
= sin 2 x + cos 2 x = 1 − x2
4 4 4 ⇒ x2 = y(1 − x2) [Q x2 ≠ 1]
and gof (x) = g { f (x)} = g (5 / 4) = 1 ⇒ x (1 + y) = y
2

Alternate Solution y
⇒ x2 = [provided y ≠ −1]
f (x) = sin 2 x + sin 2(x + π / 3) + cos x cos (x + π / 3) 1+ y
⇒ f ′ (x) = 2 sin x cos x + 2 sin (x + π / 3) cos (x + π / 3) Q x2 ≥ 0
− sin x cos (x + π / 3) − cos x sin (x + π / 3) y
⇒ ≥ 0 ⇒ y ∈ (−∞ , − 1) ∪ [0, ∞ )
1+ y
= sin 2x + sin (2x + 2π / 3) − [sin (x + x + π / 3)]
 2x + 2x + 2π / 3  2x − 2x − 2π / 3 Since, for surjective function, range of f = codomain
= 2 sin   ⋅ cos  
 2   2  ∴Set A should be R − [−1, 0).
 (x − 1)
− sin (2x + π / 3)
|x − 1| − x , if 0 < x ≤ 1
= 2 [sin (2x + π / 3) ⋅ cos π / 3] − sin (2x + π / 3) 2. We have, f (x) = =
x x−1
 , if x > 1
 1  π  x
= 2 sin (2x + π / 3) ⋅ − sin 2x +  = 0
 
2  3 1
− 1, if 0 < x ≤ 1

⇒ f (x) = c, where c is a constant. = x
1
But f (0) = sin 2 0 + sin 2(π / 3) + cos 0 cos π / 3 1 − , if x > 1
 x
2
 3 1 3 1 5 Now, let us draw the graph of y = f (x)
=  + = + =
 
2 2 4 2 4 Note that when x → 0, then f (x) → ∞, when x = 1, then
f (x) = 0, and when x → ∞, then f (x) → 1
Therefore, ( gof ) (x) = g [ f (x)] = g(5 / 4) = 1
Y
18. Given, f (x) = (a − xn )1/ n
⇒ f [ f (x)] = [a − {(a − xn )1/ n }n ]1/ n = (xn )1/ n = x
∴ f [ f (x)] = x x=0

Hence, given statement is true. y=1


19. Let f (n ) = 2 for all positive integers n.
n

X
Now, for n = 1, f (1) = 2 = 2 ! O 1 y=0
⇒ It is true for n = 1.
rankershalt.com

Functions 171

Clearly, f (x) is not injective because if f (x) < 1, then f is  1 1


⇒ (1 − 2 y) (1 + 2 y) ≥ 0 ⇒ y ∈ − ,
many one, as shown in figure.  2 2 
Also, f (x) is not surjective because range of f (x) is [0, ∞ [ – + –
and but in problem co-domain is (0, ∞ ), which is wrong. –1/2 1/2
∴f (x) is neither injective nor surjective
 1 1
So, range is − , .
3. According to given information, we have if  2 2 
k ∈{4, 8, 12, 16, 20}
n + 1
Then, f (k) ∈ {3, 6, 9, 12, 15, 18}  , if n is odd
5. Given, f (n ) =  n 2
[Q Codomain ( f ) = {1, 2, 3, …, 20}]
 , if n is even,
Now, we need to assign the value of f (k) for 2
k ∈{4, 8, 12, 16, 20} this can be done in 6C5 ⋅ 5 ! ways
= 6 ⋅ 5 ! = 6 ! and remaining 15 element can be associated
and {
g (n ) = n − (−1)n = n + 1 , if n is odd
n − 1, if n is even
by 15 ! ways. {
Now, f ( g (n )) = ff ((n + 1), if n is odd
n − 1), if n is even
∴Total number of onto functions = = 15 ! 6 ! n + 1
x  , if n is odd
4. We have, f (x) = , x ∈R =  n 2− 1 + 1 n
1 + x2  = , if n is even
 2 2
Ist Method f (x) is an odd function and maximum
= f (x)
occur at x = 1
[Q if n is odd, then (n + 1) is even and
Y (1, 1/2)
y= 1 if n is even, then (n − 1) is odd]
2
Clearly, function is not one-one as f (2) = f (1) = 1
–1
O 1 X But it is onto function.
y =– 1 [Q If m ∈ N (codomain) is odd, then 2m ∈ N (domain)
(–1, 1/2) 2 such that f (2m) = m and
if m ∈ N codomain is even, then
From the graph it is clear that range of f (x) is 2m − 1 ∈ N (domain) such that f (2m − 1) = m]
 1 1 ∴Function is onto but not one-one
− ,
 2 2  6. We have a function f : A → R defined as, f (x) =
2x
1 x −1
IInd Method f (x) = One-one Let x1, x2 ∈ A such that
1
x+
x f (x1 ) = f (x2)
1 2x1 2x2
If x > 0, then by AM ≥ GM, we get x + ≥2 ⇒ =
x x1 − 1 x2 − 1
1 1 1 ⇒ 2x1x2 − 2x1 = 2x1x2 − 2x2
⇒ ≤ ⇒ 0 < f (x) ≤
1 2 2
x+ ⇒ x1 = x2
x
1
If x < 0, then by AM ≥ GM, we get x + ≤ −2 Thus, f (x1 ) = f (x2) has only one solution, x1 = x2
x ∴ f (x) is one-one (injective)

1
≥−
1 1
⇒ – ≤ f (x) < 0 2 ×2
1 Onto Let x = 2, then f (2) = =4
x+ 2 2 2 −1
x
But x = 2 is not in the domain, and f (x) is one-one
0
If x = 0, then f (x) = =0 function
1+0 ∴f (x) can never be 4.
1 1
Thus, − ≤ f (x) ≤ Similarly, f (x) can not take many values.
2 2
Hence, f (x) is into (not surjective).
 1 1
Hence, f (x) ∈ − , ∴f (x) is injective but not surjective.
 2 2 
x
IIIrd Method 7. We have, f (x) =
x 1 + x2
Let y = ⇒ yx2 − x + y = 0 1
1 + x2
 1 x
∴ f  = x = = f (x)
Q x ∈ R, so D ≥ 0  x 1 1 + x2
1+ 2
⇒ 1 − 4 y2 ≥ 0 x
rankershalt.com

172 Functions

 1  1 ∴ Range = Codomain ⇒ onto


∴ f   = f (2)or f   = f (3)and so on.
 2  3 Thus, f − g is one-one and onto.
So, f (x) is many-one function.
x 10. Given, f : [0, ∞ ) → [0, ∞ )
Again, let y = f (x) ⇒ y =
1 + x2 Here, domain is [0, ∞ ) and codomain is [0, ∞ ). Thus, to
check one-one
⇒ y + x y = x ⇒ yx − x + y = 0
2 2
x 1
As, x ∈R Since, f (x) = ⇒ f ′ (x) = > 0, ∀ x ∈ [0, ∞ )
1+ x (1 + x)2
∴ (− 1)2 − 4 ( y)( y) ≥ 0
⇒ 1 − 4 y2 ≥ 0 ∴ f (x) is increasing in its domain. Thus, f (x) is one-one
− 1 1 in its domain. To check onto (we find range)
⇒ y∈ ,
 2 2  Again, y = f (x) =
x
1+ x
− 1 1
∴ Range = Codomain = ,
 2 2  ⇒ y + yx = x
So, f (x) is surjective. y y
⇒ x= ⇒ ≥0
Hence, f (x) is surjective but not injective. 1− y 1− y

8. PLAN To check nature of function. Since, x ≥ 0, therefore 0 ≤ y < 1


(i) One-one To check one-one, we must check whether i.e. Range ≠ Codomain
f ′ ( x )> 0 or f ′ ( x )< 0 in given domain.
∴ f (x) is one-one but not onto.
(ii) Onto To check onto, we must check
Range = Codomain 11. Given, f (x) = 2x + sin x
Description of Situation To find range in given ⇒ f ' (x) = 2 + cos x ⇒ f ' (x) > 0 , ∀x ∈ R
domain [a , b], put f ′ (x) = 0 and find x = α 1, α 2, …,
which shows f (x) is one-one, as f (x) is strictly increasing.
α n ∈[a , b]
Since, f (x) is increasing for every x ∈ R,
Now, find { f (a ), f (α 1 ), f (α 2), K , f (α n ), f (b)}
∴ f (x) takes all intermediate values between (−∞ , ∞ ).
its greatest and least values gives you range.
Range of f (x) ∈ R.
Now, f : [0, 3] → [1, 29]
Hence, f (x) is one-to-one and onto.
f (x) = 2x3 − 15x2 + 36x + 1
12. The number of onto functions from
∴ f ′ (x) = 6x2 − 30x + 36 = 6 (x2 − 5x + 6)
E = {1, 2, 3, 4} to F = {1, 2}
= 6 (x − 2) (x − 3)
= Total number of functions which map E to F
+ − +
− Number of functions for which map f (x) = 1 and
2 3
f (x) = 2 for all x ∈ E = 24 − 2 = 14
For given domain [0, 3], f (x) is increasing as well as 13. PLAN
decreasing ⇒ many-one (i) For such questions, we need to properly define the
functions and then we draw their graphs.
Now, put f ′ (x) = 0
(ii) From the graphs, we can examine the function for continuity,
⇒ x = 2, 3 differentiability, one-one and onto.
Thus, for range f (0) = 1, f (2) = 29, f (3) = 28 − x, x < 0
f1 (x) =  x
⇒ Range ∈[1, 29] e , x ≥ 0
∴ Onto but not one-one. f2(x) = x2, x ≥ 0
 x, x ∈ Q sin x, x < 0
9. Let φ (x) = f (x) − g (x) =  f3 (x) = 
− x, x ∉ Q x, x≥0
Now, to check one-one.  f ( f (x)), x<0
Take any straight line parallel to X-axis which will f4 (x) =  2 1
 2 1
f ( f (x)) − 1 , x≥0
intersect φ(x) only at one point.
⇒ φ(x) is one-one. x2, x < 0
Now, f2( f1 (x)) =  2x
To check onto e , x ≥ 0
 x, x ∈ Q  x2 , x<0
As f (x) =  , which shows ⇒ f4 =  2x
− x, x ∉ Q e − 1 , x ≥ 0
y = x and y = − x for rational and irrational values 2x, x<0
As f4 (x) is continuous, f ′ 4 (x) =  2x
⇒ y ∈ real numbers. 2 e , x>0
rankershalt.com

Functions 173

f4(x) f (− x) = − [ln (sec x + tan x)]3


f (x) + f (− x) = 0
⇒ f (x) is an odd function.
x 16. Let y = ax + b and y = cx + d be two linear functions.
O

Graph for f4 (x) When x = − 1, y = 0 and x = 1, y = 2 , then


0 = −a + b and a + b = 2 ⇒ a = b = 1
f4′ (0) is not defined. Its range is [0, ∞ ).
∴ y=x+1 ...(i)
Thus, range = codomain = [0, ∞ ), thus f4 is onto.
Also, horizontal line (drawn parallel to X-axis) meets Again, when x = − 1, y = 2 and x = 1, y = 0, then
the curve more than once, thus function is not one-one. − c + d = 2 and c + d = 0
14. y = 1 + 2x is linear function, therefore it is one-one and ⇒ d = 1 and c = − 1
its range is (− π + 1, π + 1). Therefore, (1 + 2x) is one-one ∴ y= −x+ 1 …(ii)
but not onto so (A) → (q). Again, see the figure. Hence, two linear functions are y = x + 1 and y = − x + 1
x2 + 4x + 30
Y 17. Given, f (x) =
x2 − 8x + 18
y = 1 + 2x
 (x2 − 8x + 18) (2x + 4) 
 
− (x2 + 4x + 30) (2x − 8)
X′
−π π X ⇒ f ′ (x) = 
O (x2 − 8x + 18)2
2 2
2 (−6x2 − 12x + 156) −12 (x2 + 2x − 26)
= =
(x2 − 8x + 18)2 (x2 − 8x + 18)2
Y′
which shows f ′ (x) is positive and negative both.
It is clear from the graph that y = tan x is one-one and ∴ f (x) is many one.
onto, therefore (B) → (r).
Hence, given statement is true.
15. PLAN
αx2 + 6x − 8
(i) If f ′ ( x ) > 0, ∀x ∈ ( a, b ), then f( x ) is an increasing function in 18. Let y =
α + 6 x − 8 x2
( a, b ) and thus f( x ) is one-one function in ( a, b ) .
(ii) If range of f( x ) = codomain of f( x ) , then f( x ) is an onto ⇒ αy + 6xy − 8x2y = αx2 + 6x − 8
function.
⇒ − αx2 − 8x2y + 6xy − 6x + αy + 8 = 0
(iii) A function f( x ) is said to be an odd function, if
f( − x ) = − f( x ), ∀x ∈ R, i.e. ⇒ αx2 + 8x2y − 6xy + 6x − αy − 8 = 0
f( − x ) + f( x ) = 0, ∀ x ∈ R ⇒ x2 (α + 8 y) + 6x (1 − y) − (8 + αy) = 0
f (x) = [ln (sec x + tan x)] 3
Since, x is real.
3 [ln (sec x + tan x)] (sec x tan x + sec x)
2 2 ⇒ B2 − 4 AC ≥ 0
f ′ (x) =
(sec x + tan x) ⇒ 36 (1 − y)2 + 4 (α + 8 y) (8 + αy) ≥ 0
 −π π  ⇒ 9 (1 − 2 y + y2) + [8α + (64 + α 2) y + 8 αy2] ≥ 0
f ′ (x) = 3 sec x [ln (sec x + tan x)]2 > 0, ∀x ∈  , 
 2 2 ⇒ y2 (9 + 8α ) + y (46 + α 2) + 9 + 8α ≥ 0 …(i)
f (x) is an increasing function. ⇒ A > 0, D ≤ 0, ⇒ 9 + 8 α > 0
∴ f (x) is an one-one function. and (46 + α 2)2 − 4 (9 + 8α )2 ≤ 0
 π x  π π ⇒ α > − 9 /8
(sec x + tan x) = tan  +  , as x ∈  − ,  , then
 4 2  2 2 and [46 + α 2 − 2 (9 + 8α )][46 + α 2 + 2 (9 + 8α )] ≤ 0
 π x ⇒ α > − 9 /8
0 < tan  +  < ∞
 4 2 and (α 2 − 16α + 28) (α 2 + 16α + 64) ≤ 0
0 < sec x + tan x < ∞ ⇒ α > − 9 /8
⇒ − ∞ < ln (sec x + tan x) < ∞ and [(α − 2) (α − 14)] (α + 8)2 ≤ 0
− ∞ < [ln (sec x + tan x)]3 < ∞ ⇒ α > − 9 /8
⇒ −∞ < f (x) < ∞ and (α − 2) (α − 14) ≤ 0 [Q (α + 8)2 ≥ 0]
Range of f (x) is R and thus f (x) is an ont function. ⇒ α > − 9 /8
  
3 and 2 ≤ α ≤ 14
1
f (− x) = [ln (sec x − tan x)]3 = ln   ⇒ 2 ≤ α ≤ 14
  sec x + tan x 
rankershalt.com

174 Functions

αx2 + 6x − 8 x± x2 − 4
Thus, f (x) = will be onto, if 2 ≤ α ≤ 14 ⇒ f −1 (x) =
α + 6 x − 8 x2 2
Again, when α = 3 Since, the range of the inverse function is [1, ∞), then
3x + 6x − 8
2
x+ x2 − 4
f (x) = , in this case f (x) = 0 we take f −1 (x) =
3 + 6x − 8x2 2
⇒ 3x2 + 6x − 8 = 0 x − x2 − 4
If we consider f −1 (x) = , then f −1 (x) > 1
− 6 ± 36 + 96 − 6 ± 132 1 2
⇒ x= = = (− 3 ± 33 )
6 6 3 This is possible only if (x − 2)2 > x2 − 4

This shows that ⇒ x2 + 4 − 4x > x2 − 4


1  1  ⇒ 8 > 4x
f (− 3 + 33) = f (− 3 − 33) = 0
3  3  ⇒ x < 2, where x > 2
Therefore, f is not one-to-one. Therefore, (a) is the answer.
19. Since, there is an injective mapping from A to B, each 5. Let y = 2x ( x − 1), where y ≥ 1 as x ≥ 1
element of A has unique image in B.
Taking log 2 on both sides, we get
Similarly, there is also an injective mapping from B to log 2 y = log 2 2x ( x − 1)
A, each element of B has unique image in A or in other
⇒ log 2 y = x (x − 1)
words there is one to one onto mapping from A to B.
⇒ x2 − x − log 2 y = 0
Thus, there is bijective mapping from A to B.
1 ± 1 + 4 log 2 y
⇒ x=
Topic 4 Inverse and Periodic Functions 2
1. Since, only (c) satisfy given definition For y ≥ 1, log 2 y ≥ 0 ⇒ 4 log 2 y ≥ 0 ⇒ 1 + 4 log 2 y ≥ 1
i.e. f {f −1
(B)} = B ⇒ 1 + 4 log 2 y ≥ 1
Only, if B ⊆ f (x) ⇒ − 1 + 4 log 2 y ≤ − 1
⇒ 1 − 1 + 4 log 2 y ≤ 0
2. By definition of composition of function,
But x≥1
g ( f (x) ) = (sin x + cos x)2 − 1, is invertible
So, x = 1 − 1 + 4 log 2 y is not possible.
(i.e. bijective)
⇒ g { f (x) } = sin 2x is bijective. 1
Therefore, we take x = (1 + 1 + 4 log 2y )
 π π 2
We know, sin x is bijective, only when x ∈ − , .
 2 2  1
⇒ f −1 ( y) = (1 + 1 + 4 log 2 y )
π π 2
Thus, g { f (x) } is bijective, if − ≤ 2x ≤
2 2 1
π π ⇒ f −1 (x) = (1 + 1 + 4 log 2 x )
⇒ − ≤x≤ 2
4 4
6. Given, f (x) = 3x − 5 [given]
3. It is only to find the inverse.
Let y = f (x) = 3x − 5 ⇒ y + 5 = 3x
Let y = f (x) = (x + 1)2, for x ≥ − 1
y+5
± y = x + 1, x≥ −1 ⇒ x=
3
⇒ y = x+1 ⇒ y ≥ 0, x + 1 ≥ 0 y+5
−1
f ( y) =
⇒ x= y −1 3
⇒ f −1 ( y) = y −1 −1 x+5
⇒ f (x) =
⇒ f −1 (x) = x − 1 ⇒ x≥0 3
1 x2 + 1 7. Clearly, f (x) = x − [x] = { x}
4. Let y = x + ⇒ y=
x x which has period 1.
⇒ xy = x2 + 1 1
And sin , x cos x are non-periodic functions.
y± y2 − 4 x
⇒ x2 − xy + 1 = 0 ⇒ x =
2 b−x
8. Here, f (x) = , where 0 < b < 1, 0 < x < 1
y± y2 − 4 1 − bx
⇒ f −1 ( y) =
2 For function to be invertible, it should be one-one onto.
∴ Check Range :
rankershalt.com

Functions 175

b−x Case III When f (z ) ≠ 2 is true.


Let f (x) = y ⇒ y=
1 − bx If f (z ) ≠ 2 is true, then remaining statements are false.
⇒ y − bxy = b − x ⇒ x (1 − by) = b − y ∴ If f (x) ≠ 1 and f ( y) = 1
b− y But f is injective.
⇒ x= , where 0 < x < 1
1 − by Thus, we have f (x) = 2, f ( y) = 1 and f (z ) = 3
b− y b− y b− y Hence, f −1 (1) = y
∴ 0< <1 ⇒ > 0 and <1
1 − by 1 − by 1 − by 11. Since, f is an even function,
1 then f (− x) = f (x), ∀ x ∈ (− 5, 5)
⇒ y < b or y> …(i)
b  x + 1
(b − 1) ( y + 1) 1 Given , f (x) = f   …(i)
<0−1 < y< …(ii)  x + 2
1 − by b
 − x + 1
From Eqs. (i) and (ii), we get ⇒ f (− x) = f  
 − x + 2
 1
y ∈  − 1,  ⊂ Codomain  − x + 1
 b ⇒ f ( x) = f   [Q f (− x) = f (x)]
 − x + 2
1 − cos 2x
9. Given, F (x) = ∫ sin 2 x dx = ∫ dx Taking f −1 on both sides, we get
2
1 −x+1
F (x) = (2x − sin 2x) + C x=
4 −x+2
Since, F (x + π ) ≠ F (x) ⇒ − x2 + 2x = − x + 1
Hence, Statement I is false. ⇒ x − 3x + 1 = 0
2

But Statement II is true as sin 2 x is periodic with 3 ± 9 −4 3 ± 5


⇒ x= =
period π. 2 2
10. It gives three cases  x + 1
Again, f (x) = f  
 x + 2
Case I When f (x) = 1 is true.
In this case, remaining two are false.  x + 1
⇒ f (− x) = f   [Q f (− x) = f (x)]
∴ f ( y) = 1 and f (z ) = 2  x + 2
This means x and y have the same image, so f (x) is not Taking f −1 on both sides, we get
an injective, which is a contradiction. x+1
−x=
Case II When f ( y) ≠ 1 is true. x+2
If f ( y) ≠ 1 is true, then the remaining statements are ⇒ x2 + 3x + 1 = 0
false. −3 ± 9 −4 −3 ± 5
∴ f (x) ≠ 1 and f (z ) = 2 ⇒ x= =
2 2
i.e. both x and y are not mapped to 1. So, either both ±3± 5
associate to 2 or 3. Thus, it is not injective. Therefore, four values of x are .
2

Download Chapter Test


http://tinyurl.com/y47c8lwl or
rankershalt.com

9
Limit, Continuity
and Differentiability
0 ∞ cot x − cos x
Topic 1 and Form 8. lim equals (2017 Main)
0 ∞ x → π/ 2

1
(π − 2x)3
1
(a) (b)
Objective Questions I (Only one correct option) 24 16
1 1
x + 2 sin x (c) (d)
1. lim is 8 4
x→ 0
x + 2 sin x + 1 − sin 2 x − x + 1
2
sin(π cos 2 x)
(2019 Main, 12 April II)
9. lim is equal to (2014 Main)
x→ 0 x2
(a) 6 (b) 2 (c) 3 (d) 1 π
(a) (b) 1 (c) − π (d) π
x2 − ax + b 2
2. If lim = 5, then a + b is equal to
x→1 x−1 (2019 Main, 10 April II) (1 – cos 2x)(3 + cos x)
10. lim is equal to (2013 Main)
x→ 0 x tan 4x
(a) − 4 (b) 1 (c) − 7 (d) 5
1
x4 − 1 x3 − k3 (a) 4 (b) 3 (c) 2 (d)
3. If lim = lim 2 , then k is 2
x→1 x − 1 x → k x − k2 (2019 Main, 10 April I)
 x2 + x + 1 
(a)
4
(b)
3
(c)
3
(d)
8 11. If lim  − ax − b = 4, then
3 8 2 3
x→ ∞  x+1  (2012)
sin x 2 (a) a = 1, b = 4 (b) a = 1, b = − 4
4. lim equals (2019 Main, 8 April I) (c) a = 2, b = − 3 (d) a = 2, b = 3
x→ 0 2 − 1 + cos x
f (2h + 2 + h 2) − f (2)
(a) 4 2 (b) 2 12. lim , given that f ′ (2) = 6 and
h → 0 f (h − h 2 + 1 ) − f (1 )
(c) 2 2 (d) 4
cot3 x − tan x f ′ (1) = 4 , (2003, 2M)
5. lim is (a) does not exist (b) is equal to −3/2
π  π (2019 Main, 12 Jan I)
x→ cos  x +  (c) is equal to 3/2 (d) is equal to 3
4
 4
{(a − n ) nx − tan x} sin nx
(a) 4 2 (b) 4 (c) 8 (d) 8 2 13. If lim = 0, where n is non-zero
x→ 0 x2
x cot(4x) real number, then a is equal to
6. lim is equal to (2019 Main, 11 Jan II) (2003, 2M)
x → 0 sin 2 x cot 2 (2 x) n+1 1
(a) 0 (b) (c) n (d) n +
(a) 0 (b) 1 (c) 4 (d) 2 n n
(cos x − 1) (cos x − ex )
1 + 1 + y4 − 2 14. The integer n for which lim is a
7. lim (2019 Main, 9 Jan I)
x→ 0 xn
y→ 0 y4 finite non-zero number, is (2002, 2M)
1 (a) 1 (b) 2
(a) exists and equals
4 2 (c) 3 (d) 4
(b) does not exist x tan 2x − 2x tan x
(c) exists and equals
1 15. lim is (1999, 2M)
2 2
x→ 0 (1 − cos 2x)2
1 (a) 2 (b) −2
(d) exists and equals 1 1
2 2 ( 2 + 1) (c) (d) −
2 2
rankershalt.com

Limit, Continuity and Differentiability 177

1 − cos 2 (x − 1) x2
16. lim (1998, 2M) a − a 2 − x2 −
x→1 x −1 23. Let L = lim 4 , a > 0 . If L is finite, then
(a) exists and it equals 2 x→ 0 x4
(b) exists and it equals − 2 (a) a = 2
(c) does not exist because x − 1 → 0 (b) a = 1 (2009)
1
(d) does not exist because left hand limit is not equal to (c) L =
right hand limit 64
1
1 (d) L =
(1 − cos 2 x) 32
17. The value of lim 2 is (1991, 2M)
x→ 0 x
(a) 1 (b) −1
Fill in the Blanks
(c) 0 (d) None of these log (1 + 2h ) − 2 log (1 + h )
24. lim =K . (1997C, 2M)
sin[x] [x] ≠ 0 h→ 0 h2
18. If f (x) =  [x]
,  
 0, [x] ≠ 0 25. If f (x) = sin x, x ≠ nπ , n = 0, ± 1, ± 2, ... 
 2, other wise 
where, [x] denotes the greatest integer less than or
equal to x, then lim f (x) equals (1985, 2M) x2 + 1, x ≠ 0, 2
x→ 0  
(a) 1 (b) 0 and g (x) =  4, x = 0 , then lim g [f(x)] is ………
(c) −1 (d) None of these  5, x = 2 
x→0
 (1996, 2M)

1 2 n 
19. lim  + + ... +  is equal to (1984, 2M) 26. ABC is an isosceles triangle inscribed in a circle of radius
n → ∞  1 − n2 1−n 2
1 − n 2
r. If AB = AC and h is the altitude from A to BC, then the
1
(a) 0 (b) − ∆ABC has perimeter P = 2( 2hr − h 2 + 2hr ) and area
2 A
(c)
1
(d) None of these A = K . Also, lim 3 = K (1989, 2M)
h→ 0 P
2
20. If f (a ) = 2, f ′ (a ) = 1, g (a ) = − 1, g ′ (a ) = 2,  4  1 2 
  x sin   + x  
g (x) f (a ) − g (a ) f (x) x
then the value of lim is (1983, 1M) 27. lim  = …
x→ − ∞  (1 + |x|3 ) 
x→ a x−a
 
1   (1987, 2M)
(a) − 5 (b)
28. Let f (x) = (x + x − 16x + 20) / (x − 2) , if x ≠ 2
5 3 2 2
.
(c) 5 (d) None of these k , if x = 2
G (x) − G (1) If f (x) is continuous for all x, then k = … . (1981, 2M)
21. If G (x) = − 25 − x2, then lim has the value πx
x→1 x−1 (1983, 1M) 29. lim (1 − x) tan =….
1 1 x→1 2 (1978, 2M)
(a) (b)
24 5
(c) − 24 (d) None of these True/False
30. If lim [ f (x) g (x)] exists, then both lim f (x) and
Objective Question II x→ a x→ a
lim g (x) exist. (1981, 2M)
(One or more than one correct option) x→ a

22. For any positive integer n, define fn : (0, ∞ ) → R as Analytical & Descriptive Questions
−1 
n 1
fn (x) = Σ tan   for all x ∈ (0, ∞ ). (1983, 3M)
j =1  1 + (x + j) (x + j − 1) ax −1
31. Use the formula lim = log e a, to find
(Here, the inverse trigonometric function tan − 1 x x→ 0 x
2x − 1
 π π lim .
assumes values in  − ,  ). Then, which of the x → 0 (1 + x)1/ 2 − 1
 2 2 (1982, 2M)
following statement(s) is (are) TRUE? (2018 Adv.) (a + h )2 sin (a + h ) − a 2 sin a
32. Evaluate lim . (1980, 3M)
(a) ∑
5
tan 2 (f j (0)) = 55 h→ 0 h
j =1
x − sin x
(b) ∑
10
(1+ f ′ j (0)) sec2 (f j (0)) = 10 33. Evaluate lim . (1979, 3M)
j =1 x→ 0 x + cos 2 x
1
(c) For any fixed positive integer n, lim tan(fn (x)) = x −1
n x →∞
34. Evaluate lim  
. (1978, 3M)
x→1  2 x2 − 7 x + 5 
(d) For any fixed positive integer n, lim sec2 (fn (x)) = 1
x →∞
rankershalt.com

178 Limit, Continuity and Differentiability

Integer Type Questions 36. Let m and n be two positive integers greater than 1. If
 ecos ( α n ) − e
x sin (βx) 2
lim   = −  e , then the value of m is
35. Let α , β ∈ R be such that lim = 1 . Then, α→ 0  α m   2 n
x → 0 αx − sin x  
(2015 Adv.)
6 (α + β ) equals (2016 Adv)

Topic 2 1∞ Form, RHL and LHL


Objective Questions I (Only one correct option) 8. Let p = lim+ (1 + tan 2 x )1/ 2x , then log p is equal to
x→ 0 (2016 Main)
1. Let f : R → R be a differentiable function satisfying 1 1
1 (a) 2 (b) 1 (c) (d)
 1 + f (3 + x) − f (3) x 2 4
f ′ (3) + f ′ (2) = 0. Then lim  is equal
x→ 0  1 + f (2 − x) − f (2 )  9. Let α (a ) and β (a ) be the roots of the equation
to (2019 Main, 8 April II) ( 3 1 + a − 1) x2 − ( 1 + a − 1) x + (6 1 + a − 1) = 0, where
(a) e (b) e−1 (c) e2 (d) 1
a > − 1. Then, lim α (a ) and lim β (a ) are (2012)
a→ 0+ a→ 0 +
−1 5 1
π − 2 sin x (a) − and 1 (b) − and −1
2. lim is equal to 2 2
x→1 − 1−x (2019 Main, 12 Jan II) 7 9
(c) − and 2 (d) − and 3
π 2 1 2 2
(a) (b) (c) π (d)
2 π 2π 1

3. Let [x] denote the greatest integer less than or equal to 10. If lim [1 + x log (1 + b2)] x = 2b sin 2 θ, b > 0
x→ 0
x. Then,
and θ ∈ (− π , π ], then the value of θ is (2011)
tan(π sin 2 x) + (|x| − sin(x[x]))2
lim π π π π
x→ 0 x2 (2019 Main, 11 Jan I)
(a) ± (b) ± (c) ± (d) ±
4 3 6 2
(a) equals π (b) equals π + 1
 sin x 
11. For x > 0, lim (sin x)1/ x +  
(c) equals 0 (d) does not exist 1
 is (2006, 3M)
x→0
  x 
4. For each t ∈ R, let [t ] be the greatest integer less than or
equal to t. Then, (a) 0 (b) – 1 (c) 1 (d) 2
π  12. Let f : R → R be such that f (1) = 3 and f ′ (1) = 6. Then,
(1 − |x| + sin|1 − x|) sin  [1 − x]
2  1/ x
lim  f (1 + x) 
x → 1+ |1 − x|[1 − x] lim equals
x→ 0 
(2019 Main, 10 Jan I) (2002, 2M)
 f (1) 
(a) equals 0 (b) does not exist 1
(c) equals − 1 (d) equals 1 (a) 1 (b) e 2 (c) e2 (d) e3
5. For each x ∈ R, let [x] be the greatest integer less than or x−3 
x

equal to x. Then, 13. For x ∈ R , lim   is equal to


x→ ∞ x+ 2  (2000, 2M)
x([x] + |x|) sin [x]
lim is equal to (a) e (b) e−1 (c) e−5 (d) e5
x → 0− |x| (2019 Main, 9 Jan II)
(a) 0 (b) sin 1 Fill in the Blanks
(c) − sin 1 (d) 1 1/ x 2
 1 + 5x2
6. For each t ∈R, let [t ] be the greatest integer less than or 14. lim   =K .
x → 0  1 + 3 x2 
(1996, 1M)
equal to t. Then,
x+ 4
  1  2 15  x + 6
lim x   +   + … +  x  15. lim   = …… .
+  x x → ∞  x + 1
x→ 0    x (2018 Main) (1991, 2M)
(a) is equal to 0 (b) is equal to 15
(c) is equal to 120 (d) does not exist (in R) Analytical & Descriptive Question
1/ x
 π  
7. Let f (x) =
1 − x (1 + |1 − x|)  1 
cos   16. Find lim tan  + x  .
|1 − x|  1 − x x→ 0 
 4   (1993, 2M)
for x ≠ 1. Then
Integer Answer Type Question
(a) limx→ 1 + f (x) = 0
17. The largest value of the non-negative integer a for
(b) limx→ 1 − f (x) does not exist 1−x
(c) limx→ 1 − f (x) = 0  − ax + sin(x − 1) + a 1− 1
which lim  
x
= is (2014 Adv)
x→1  x + sin(x − 1 ) − 1  4
(d) limx→ 1 + f (x) does not exist
rankershalt.com

Limit, Continuity and Differentiability 179

Topic 3 Squeeze, Newton-Leibnitz’s Theorem and Limit Based on


Converting infinite Series into Definite Integrals
Objective Questions I (Only one correct option) Objective Questions II
1. If α and β are the roots of the equation (One more than one correct option)
n n x
375x2 − 25x − 2 = 0, then lim ∑ α r + nlim ∑ βr is equal  n
 n n (x + n )  x +
n→ ∞ →∞ n  n
r =1 r =1  ...  x +  
 2  n
to (2019 Main, 12 April I) 4. Letf (x) = lim   ,
n→ ∞  n 2  2 n 2 
(a)
21
(b)
29  n ! (x2 + n 2)  x2 +  ...  x + 2 
346 358   4  n  
1 7
(c) (d) for all x = 0. Then (2016 Adv.)
12 116
(a) f   ≥ f (1) (b) f   ≤ f  
1 1 2
sec 2 x  2  3  3
2. lim
∫2 f (t ) dt
equals f ′ (3) f ′ (2)
(c) f ′ (2) ≤ 0 (d) ≥
x→
π π2 f (3) f (2)
4 x2 −
16 (2007, 3M)
8
Numerical Value
(a) f (2)
π 5. For each positive integer n, let
1
2 1
(b) f (2) yn = ((n + 1) (n + 2) ... (n + n )) n .
π n
2  1 For x ∈ R, let [x] be the greatest integer less than or
(c) f  
π  2 equal to x. If lim yn = L, then the value of [L ] is
n→ ∞
(d) 4f (2) ................. . (2018 Adv.)

1 2n
r Fill in the Blank
3. lim
n→ ∞
∑ equals (1999, 2M)
n n +r
2 2 x2
∫0
r =1
cos 2t dt
(a) 1 + 5 (b) 5 − 1 6. lim =K
x→ 0 x sin x
(c) −1 + 2 (d) 1 + 2 (1997C, 2M)

Topic 4 Continuity at a Point


Objective Questions I (Only one correct option) 3. Let [.] denotes the greatest integer function and
π π f (x) = [tan 2 x], then (1993, 1M)
1. If the function f defined on  ,  by (a) lim f (x) does not exist
 6 3 x→ 0
 2 cos x − 1 π (b) f (x) is continuous at x = 0
 , x≠ (c) f (x) is not differentiable at x = 0
f (x) =  cot x − 1 4 is continuous,
(d) f ′ (0) = 1
 π
k, x= 2x − 1
 4 4. The function f (x) = [x] cos   π, [⋅] denotes the
then k is equal to (2019 Main, 9 April I)  2 
greatest integer function, is discontinuous at
1
(a) (b) 2 (a) all x (1993, 1M)
2
(b) all integer points
1
(c) 1 (d) (c) no x
2
(d) x which is not an integer
2. The function f (x) = [x]2 − [x2] (where, [x] is the greatest 5. If f (x) = x ( x + (x + 1), then (1985, 2M)
integer less than or equal to x), is discontinuous at
(a) f (x) is continuous but not differentiable at x = 0
(a) all integers (1999, 2M)
(b) f (x) is differentiable at x = 0
(b) all integers except 0 and 1 (c) f (x) is not differentiable at x = 0
(c) all integers except 0 (d) None of the above
(d) all integers except 1
rankershalt.com

180 Limit, Continuity and Differentiability

log (1 + ax) − log (1 − bx) 13. Find the values of a and b so that the function
6. The function f (x) = (1989)
x  x + a 2 sin x, 0 ≤ x ≤ π /4
is not defined at x = 0. The value which should be 
f (x) =  2x cot x + b, π /4 ≤ x ≤ π /2
assigned to f at x = 0, so that it is continuous at x = 0, is
a cos 2x − b sin x, π / 2 < x ≤ π
(a) a − b (b) a + b (1983, 1M) 
(c) log a + log b (d) None of these is continuous for 0 ≤ x ≤ π.
14. Let g (x) be a polynomial of degree one and f (x) be
Objective Questions II
 g (x), x≤0
(One or more than one correct option)  1/ x
defined by f (x) =  (1 + x) 
7. Let [x] be the greatest integer less than or equals to x.  (2 + x)  , x > 0
 
Then, at which of the following point(s) the function
f (x) = x cos (π (x + [x])) is discontinuous ? (2017 Adv.) Find the continuous function f (x) satisfying
(a) x = − 1 (b) x = 1 f ′ (1) = f (− 1). (1987, 6M)
(c) x = 0 (d) x = 2 15. Determine the values a, b, c, for which the function
8. For every pair of continuous function f , g : [0, 1] → R  sin (a + 1) x + sin x
 , for x < 0
such that max { f (x): x ∈ [0, 1]} = max { g (x): x ∈ [0,1]}. x

The correct statement(s) is (are) (2014 Adv.)
f (x) =  c, for x = 0
 (x + bx2) 1/ 2 − x1/ 2
(a) [f (c)]2 + 3f (c) = [ g (c)]2 + 3 g (c) for some c ∈[0,1]  , for x > 0
(b) [f (c)]2 + f (c) = [ g (c)]2 + 3 g (c) for some c ∈[0,1]  bx3/ 2
(c) [f (c)]2 + 3f (c) = [ g (c)]2 + g (c) for some c ∈[0,1] is continuous at x = 0. (1982, 3M)
(d) [f (c)]2 = [ g (c)]2 for some c ∈[0,1]
9. For every integer n, let a n and bn be real numbers. Let Match the Columns
π π
function f : R → R be given by 16. Let f1 : R → R, f2 :  −,  → R, f3 : (− 1, eπ / 2 − 2) → R and
 2 2
a + sin πx, for x ∈ [2n , 2n + 1]
f (x) =  n , f4 : R → R be functions defined by
bn + cos πx, for x ∈ (2n − 1, 2n ) 2
(i) f1 (x) = sin( 1− e− x ),
for all integers n.
 |sin x|
if x ≠ 0
If f is continuous, then which of the following hold(s) for (ii) f2 (x) =  tan − 1 x , where the inverse
all n ? (2012)  1 if x = 0
(a) an−1 − bn−1 = 0 (b) an − bn = 1 trigonometric function tan − 1 x assumes values in
(c) an − bn + 1 = 1 (d) an − 1 − bn = −1  − π , π ,
 
 2 2
Fill in the Blank (iii) f3 (x) = [sin(log e (x + 2))], where for t ∈R, [t ] denotes the
10. A discontinuous function y = f (x) satisfying x2 + y2 = 4 is greatest integer less than or equal to t,
 2
x sin   if x ≠ 0
1
given by f (x) = .... . (1982, 2M) (iv) f4 (x) =   x
 0 if x = 0
Analytical & Descriptive Questions
List-I List-II
 π
{1 + |sin x|} < x<0
a/|sin x |
, P. The function f1 is 1. NOT continuous at x = 0
 6
11. Let f (x) =  b, x=0 continuous at x = 0 and
 π Q. The function f2 is 2. NOT differentiable at
 e tan 2 x / tan 3 x
, 0<x<
6 x=0
Determine a and b such that f (x) is continuous at x = 0. differentiable at x = 0
The function f3 3.
(1994, 4M) R. and its derivative is
is
NOT continuous at x = 0

 1 − cos 4x , differentiable at x = 0
x<0 The function f4
 x2 S. 4. and its derivative is
12. Let f (x) = 
is
a, x=0 continuous at x = 0
 x
 , x>0 The correct option is
 16 + x − 4 (a) P → 2; Q → 3; R → 1; S → 4
(b) P → 4; Q → 1; R → 2; S → 3
Determine the value of a if possible, so that the function
is continuous at x = 0. (1990, 4M)
(c) P → 4; Q → 2; R → 1; S → 3
(d) P → 2; Q → 1; R → 4; S → 3
rankershalt.com

Limit, Continuity and Differentiability 181

Topic 5 Continuity in a Domain


Objective Question I (Only one correct option) 6. Let f: R → R be a function defined as
1. Let f : R → R be a continuously differentiable  5, if x≤1
1 a + bx, if 1 < x < 3
function such that f (2) = 6 and f′ (2) = . If 
f (x) = 
 b + 5x, if 3 ≤ x < 5
48
f ( x) 3
∫ 4t dt = (x − 2) g(x), then lim g (x) is equal to
6 x→ 2 (2019 Main, 12 April I)
 30, if x≥5

(a) 18 (b) 24 Then, f is (2019 Main, 9 Jan I)

(c) 12 (d) 36 (a) continuous if a = − 5 and b = 10


(b) continuous if a = 5 and b = 5
sin ( p + 1) x + sin x
(c) continuous if a = 0 and b = 5
 ,x<0
 x (d) not continuous for any values of a and b
2. If f (x) =  q, x=0 1
 x + x2 − x x>0 7. If f (x) = x − 1, then on the interval [0, π ] (1989, 2M)
 , 2
 x3/ 2
(a) tan [f (x)] and 1/ f (x) are both continuous
is continuous at x = 0 , then the ordered pair ( p, q) is (b) tan [f (x)] and 1/ f (x) are both discontinuous
equal to (2019 Main, 10 April I) (c) tan [f (x)] and f −1 (x) are both continuous
(a)  − , −  (b)  − , 
3 1 1 3 (d) tan [f (x)] is continuous but 1/ f (x) is not continuous
 2 2  2 2
(c)  , 
5 1
(d)  − , 
3 1 Objective Questions II
 2 2  2 2
(One or more than one correct option)
 a|π − x|+1, x ≤ 5
3. If the function f (x) =  is continuous at 8. The following functions are continuous on (0, π)
b|x − π|+3, x > 5 x 1
(a) tan x (b) ∫ t sin dt
x = 5, then the value of a − b is (2019 Main, 9 April II) 0 t (1991, 2M)
−2 2  1, 0 ≤ x≤ 3 π /4 x sin x, 0 < x ≤ π /2
(a) (b)
π+ 5 π+ 5 (c)  2 3π (d)  π π
2 sin x, < x< π sin ( π + x ), < x< π
2 2  9 4  2 2
(c) (d)
π −5 5− π
9. Let [x] denotes the greatest integer less than or equal to
4. If f (x) = [x] −  , x ∈ R where [x] denotes the greatest
x
x. If f (x) = [x sin πx], then f (x) is (1986, 2M)
4  (a) continuous at x = 0 (b) continuous in (−1, 0)
integer function, then (2019 Main, 9 April II) (c) differentiable at x = 1 (d) differentiable in (−1, 1)
(a) lim f (x) exists but lim f (x) does not exist
x→ 4 + x→ 4 −
(b) f is continuous at x = 4 Fill in the Blank
(c) Both lim f (x) and lim f (x) exist but are not equal  π 
x→ 4 − x→ 4 + 10. Let f (x) = [x] sin   , where [⋅] denotes the
(d) lim f (x) exists but lim f (x) does not exist  [x + 1]
x→ 4 − x→ 4 +
greatest integer function. The domain of f is…… and
5. Let f : [−1, 3] → R be defined as the points of discontinuity of f in the domain are…… .
(1996, 2M)
|x| + [x], −1 ≤ x < 1

f (x) =  x + |x|, 1 ≤ x < 2 Analytical & Descriptive Question
 x + [x], 2 ≤ x ≤ 3 ,
 (2019 Main, 8 April II)  x2
 , 0 ≤ x<1
where, [t ] denotes the greatest integer less than or 11. Let f (x) =  2
equal to t. Then, f is discontinuous at 2x2 − 3x + 3 , 1 ≤ x ≤ 2
(a) four or more points (b) only two points  2
(c) only three points (d) only one point Discuss the continuity of f , f ′ and f ′ ′ on [0, 2].
(1983, 2M)
rankershalt.com

182 Limit, Continuity and Differentiability

Topic 6 Continuity for Composition and Function


Objective Question II where, a and b are non-negative real numbers.
(One or more than one correct option) Determine the compositie function gof. If ( gof ) (x) is
continuous for all real x determine the values of a and b.
1
1. For the function f (x) = x cos , x ≥ 1, Further, for these values of a and b, is gof differentiable
x (2009) at x = 0 ? Justify your answer. (2002, 5M)
(a) for atleast one x in the interval
3. Let f (x) be a continuous and g (x) be a discontinuous
[1, ∞ ), f (x + 2) − f (x) < 2
function. Prove that f (x) + g (x) is a discontinuous
(b) lim f ′(x) = 1 function. (1987, 2M)
x→ ∞

(c) for all x in the interval [1, ∞ ), f (x + 2) − f (x) > 2 1 + x, 0 ≤ x ≤ 2


(d) f ′(x) is strictly decreasing in the interval [1, ∞ )
4. Let f (x) = 
3 − x, 2 < x ≤ 3

Analytical & Descriptive Questions Determine the form of g (x) = f [ f (x)] and hence find the
points of discontinuity of g, if any (1983, 2M)
 x + a , if x < 0
2. Let f (x) =  and 5. Let f (x + y) = f (x) + f ( y) for all x and y. If the function f (x)
|x − 1|, if x ≥ 0
is continuous at x = 0, then show that f (x) is continuous
 x + 1, if x < 0
g (x) =  at all x. (1981, 2M)
(x − 1) + b, if x ≥ 0
2

Topic 7 Differentiability at a Point


Objective Questions I (Only one correct option)  −1 , −2 ≤ x < 0
6. Let f (x) =  and
x − 1 , 0 ≤ x ≤ 2
2
1. Let f : R → R be differentiable at c ∈ R and f (c) = 0. If
g (x) = | f (x)|, then at x = c, g is (2019 Main, 10 April I) g (x) = | f (x)| + f (|x|). Then, in the interval (−2, 2), g is
(2019 Main, 11 Jan I)
(a) not differentiable
(a) not differentiable at one point
(b) differentiable if f ′ (c) ≠ 0
(b) not differentiable at two points
(c) not differentiable if f ′ (c) = 0
(c) differentiable at all points
(d) differentiable if f ′ (c) = 0
(d) not continuous
2. If f : R → R is a differentiable function and
7. Let f : (−1, 1) → R be a function defined by
f ( x)
lim 2t dt f (x) = max { − x , − 1 − x2 }. If K be the set of all points at
f (2) = 6, then
x→ 2 ∫ (x − 2)
is
6 (2019 Main, 9 April II) which f is not differentiable, then K has exactly
(2019 Main, 10 Jan II)
(a) 12f ′ (2) (b) 0
(c) 24f ′ (2) (d) 2f ′ (2) (a) three elements (b) five elements
(c) two elements (d) one element
3. Let f (x) = 15 − x − 10 ; x ∈ R. Then, the set of all values
max {|x|, x2}, |x| ≤ 2
of x, at which the function, g (x) = f ( f (x)) is not 8. Let f (x) = 
differentiable, is (2019 Main, 9 April I)  8 − 2|x|, 2 < |x| ≤ 4
(a) {5, 10, 15, 20} (b) {5, 10, 15} Let S be the set of points in the interval (−4, 4) at which f
(c) {10} (d) {10, 15} is not differentiable. Then, S (2019 Main, 10 Jan I)
(a) equals {−2, − 1, 0, 1, 2} (b) equals {−2, 2}
4. Let S be the set of all points in (− π , π ) at which the
(c) is an empty set (d) equals {−2,−1, 1, 2}
function, f (x) = min {sin x, cos x} is not differentiable.
Then, S is a subset of which of the following? 9. Let f be a differentiable function from R to R such that
3
(2019 Main, 12 Jan I)
π π π π π π | f (x) − f ( y)| ≤ 2|x − y|2 , for all x, y ∈ R. If f (0) = 1, then
(a) − , 0,  (b) − , − , ,  1
 4 4  2 4 4 2
∫f
2
3π π 3π π  3π π π 3π  (x) dx is equal to (2019 Main, 9 Jan II)

(c) − ,− , ,  
(d) − ,− , ,  0 1
 4 4 4 4  4 2 2 4 (a) 2 (b) (c) 1 (d) 0
2
5. Let K be the set of all real values of x, where the function
f (x) = sin| x| − | x| + 2(x − π ) cos| x| is not differentiable. 10. Let S = (t ∈ R : f (x) = |x − π |(
⋅ e|x| − 1)sin| x| is not
Then, the set K is equal to (2019 Main, 11 Jan II) differentiable at t}.Then, the set S is equal to (2018 Main)
(a) {0} (b) φ (an empty set) (a) φ (an empty set) (b) {0}
(c) { π } (d) {0, π } (c) { π } (d) {0, π }
rankershalt.com

Limit, Continuity and Differentiability 183

11. For x ∈ R, f (x) = |log 2 − sin x|and g (x) = f ( f (x)), then 21. Let f : R → R be any function. Define g : R → R by
(a) g is not differentiable at x = 0 (2016 Main) g (x) =| f (x)|, ∀ x. Then, g is (2000, 2M)
(b) g′ (0) = cos (log 2) (a) onto if f is onto
(c) g′ (0) = − cos (log 2) (b) one-one if f is one-one
(d) g is differentiable at x = 0 and g′ (0) = − sin (log 2) (c) continuous if f is continuous
12. If f and g are differentiable functions in (0, 1) satisfying (d) differentiable if f is differentiable
f (0) = 2 = g (1), g(0) = 0 and f (1) = 6, then for some 22. The function f (x) = (x2 − 1)| x2 − 3x + 2| + cos (| x|) is
c ∈] 0, 1 [ (2014 Main) not differentiable at (1999, 2M)
(a) 2f ′ (c) = g ′(c) (b) 2 f ′(c) = 3 g ′(c) (a) −1 (b) 0 (c) 1 (d) 2
(c) f ′(c) = g ′(c) (d) f ′(c) = 2 g ′(c) x
23. The set of all points, where the function f (x) = is
 2 π 1 + | x|
13. Let f (x) = x cos x , x ≠ 0, x ∈ R, then f is (2012)
differentiable, is (1987, 2M)
 0, x =0
(a) (− ∞ , ∞ ) (b) [0, ∞ )
(a) differentiable both at x = 0 and at x = 2 (c) (− ∞ , 0) ∪ (0, ∞ ) (d) (0, ∞ )
(b) differentiable at x = 0 but not differentiable at x = 2
24. There exists a function f (x) satisfying f (0) = 1,
(c) not differentiable at x = 0 but differentiable at x = 2
(d) differentiable neither at x = 0 nor at x = 2
f ′ (0) = − 1, f (x) > 0, ∀ x and (1982, 2M)
(a) f ′′ (x) < 0 , ∀ x (b) − 1 < f ′′ (x) < 0 , ∀ x
(x − 1) n
14. Let g (x) = ; 0 < x < 2, m and n are (c) − 2 ≤ f ′′ (x) ≤ − 1 , ∀ x (d) f ′′ (x) < − 2 , ∀ x
log cosm (x − 1)
integers, m ≠ 0, n > 0 and let p be the left hand 25. For a real number y, let [ y] denotes the greatest
derivative of|x − 1| at x = 1 . If lim g (x) = p , then integer less than or equal to y. Then, the function
x→1 + tan π [(x − π )]
f (x) = is (1981, 2M)
(a) n = 1, m = 1 (b) n = 1, m = −1 (2008, 3M) 1 + [x]2
(c) n = 2 , m = 2 (d) n > 2, m = n (a) discontinuous at some x
15. If f is a differentiable function satisfying (b) continuous at all x, but the derivative f ′ (x) does not
 1 exist for some x
f   = 0, ∀ n ≥ 1, n ∈ I ,then (2005, 2M)
 n (c) f ′(x) exists for all x, but the derivative f ′ ′ (x) does not
(a) f (x) = 0, x ∈ (0, 1] exist for some x
(b) f ′ (0) = 0 = f (0) (d) f ′(x) exists for all x
(c) f (0) = 0 but f ′ (0) not necessarily zero
(d)|f (x)|≤ 1, x ∈ (0, 1] Objective Questions II
16. Let f (x) = ||x|− 1|, then points where, f (x) is not (One or more than one correct option)
differentiable is/are (2005, 2M) 26. For every twice differentiable function f : R → [−2, 2]
(a) 0, ± 1 (b) ± 1 with ( f (0))2 + ( f ′ (0))2 = 85, which of the following
(c) 0 (d) 1 statement(s) is (are) TRUE ? (2018 Adv.)
17. The domain of the derivative of the functions (a) There exist r , s ∈ R , where r < s, such that f is one-one on
 tan −1 x , if | x| ≤ 1 the open interval (r , s)

f (x) = 1 is (2002, 2M) (b) There exists x0 ∈ (−4, 0) such that|f ′ (x0 )|≤ 1
2 (| x| − 1), if | x| > 1 (c) lim f (x) = 1
x→ ∞
(a) R − {0} (b) R − {1} (d) There exists α ∈ (−4, 4) such that f (α ) + f ′′(α ) = 0 and
(c) R − {−1} (d) R − {−1, 1} f ′ (α ) ≠ 0
18. Which of the following functions is differentiable 27. Let f : (0, π)→ R be a twice differentiable function such
at x = 0 ? (2001, 2M) f (x) sin t − f (t )sin x
that lim = sin 2 x for all x ∈ (0, π).
(a) cos (| x|) + | x| (b) cos (| x|) − | x| t→ x t−x
(c) sin (| x|) + | x| (d) sin (| x|) − | x|
 π π
19. The left hand derivative of f (x) = [x] sin (π x) at x = k, k If f   = − , then which of the following statement(s)
 6 12
is an integer, is (2001, 2M)
is (are) TRUE? (2018 Adv.)
(a) (−1)k (k − 1) π (b)(−1)k − 1 (k − 1) π π π
(c) (−1)k kπ (d) (−1)k − 1 kπ (a) f   =
 4 4 2
20. Let f : R → R be a function defined by f (x) = max { x, x3 }. x4
(b) f (x)< − x2 for all x∈ (0, π)
The set of all points, where f (x) is not differentiable, is 6
(a) {−1,1 } (b) {−1, 0 } (2001, 2M)
(c) There exists α ∈(0, π) such that f ′ (α) = 0
(c) {0,1 } (d) {−1, 0,1 } π π
(d) f ′′  + f   = 0
 2  2
rankershalt.com

184 Limit, Continuity and Differentiability

28. Let f : R → R, g : R → R and h : R → R be differentiable (a) f ′ (x) − 3 g ′ (x) = 0 has exactly three solutions in
functions such that f (x) = x3 + 3x + 2, g ( f (x)) = x and (−1, 0) ∪ (0, 2)
h ( g ( g (x))) = x for all x ∈ R. Then, (2016 Adv.) (b) f ′ (x) − 3 g ′ (x) = 0 has exactly one solution in (−1 , 0)
(a) g ′(2) =
1 (c) f ′ (x) − 3 g ′ (x) = 0 has exactly one solution in (0, 2)
15 (d) f ′ (x) − 3 g ′ (x) = 0 has exactly two solutions in (−1, 0)
(b) h ′(1) = 666 and exactly two solutions in (0, 2)
(c) h(0) = 16
33. Let f : [a , b] → [1, ∞ ) be a continuous function and
(d) h ( g (3)) = 36

29. Let a , b ∈ R and f : R → R be defined by 0 , if x< a
 x
f (x) = a cos (|x3 − x|) + b|x|sin (|x3 + x|). Then, f is g : R → R be defined as g (x) = ∫ f (t )dt , if a ≤ x ≤ b.
a
(2016 Adv.)  b
(a) differentiable at x = 0, if a = 0 and b = 1 ∫ f (t )dt , if x> b
 a
(b) differentiable at x = 1, if a = 1 and b = 0 Then, (2013)
(c) not differentiable at x = 0, if a = 1and b = 0 (a) g (x) is continuous but not differentiable at a
(d) not differentiable at x = 1, if a = 1and b = 1 (b) g (x) is differentiable on R
30. Let f : − , 2 → R and g : − , 2 → R be functions
1 1 (c) g (x) is continuous but not differentiable at b
 2   2  (d) g (x) is continuous and differentiable at either a or b
defined by f (x) = [x2 − 3] and g (x) =|x| f (x) + |4x − 7| f (x), but not both
where [ y] denotes the greatest integer less than or equal  π π
to y for y ∈ R. Then, (2016 Adv.)  −x− 2 , x≤− 2
 π
(a) f is discontinuous exactly at three points in  − , 2 34. If f (x) = − cos x, − < x ≤ 0, then
1
 2  2

(b) f is discontinuous exactly at four points in  − , 2
1  x − 1, 0 < x≤1
 2   ln x, x>1 (2011)
π
(c) g is not differentiable exactly at four points in  − , 2
1 (a) f (x) is continuous at x = −
 2  2
f (x) is not differentiable at x = 0
(d) g is not differentiable exactly at five points in  − , 2
1 (b)
 2  (c) f (x) is differentiable at x = 1
3
31. Let g : R → R be a differentiable function with (d) f (x) is differentiable at x = −
2
g (0) = 0, g′ (0) = 0 and g′ (1) =/ 0. (2015 Adv.)
35. Let f : R → R be a function such that
 x
 g (x), x =/ 0 f (x + y) = f (x) + f ( y), ∀x, y ∈ R. If f (x) is differentiable
Let f (x) = |x|
 0 , x=0 at x = 0, then (2011)
(a) f (x) is differentiable only in a finite interval containing
and h (x) = e|x| for all x ∈ R. Let ( foh ) (x) denotes f { h (x)} zero
and (hof )(x) denotes h { f (x)}. Then, which of the (b) f (x) is continuous for all x ∈ R
following is/are true? (c) f ′ (x) is constant for all x ∈ R
(a) f is differentiable at x = 0 (d) f (x) is differentiable except at finitely many points
(b) h is differentiable at x = 0 36. If f (x) = min { 1, x2, x3 }, then (2006, 3M)
(c) foh is differentiable at x = 0 (a) f (x) is continuous everywhere
(d) hof is differentiable at x = 0 (b) f (x) is continuous and differentiable everywhere
(c) f (x) is not differentiable at two points
32. Let f, g : [−1 , 2] → R be continuous functions which are
(d) f (x) is not differentiable at one point
twice differentiable on the interval (−1, 2). Let the
values of f and g at the points −1, 0 and 2 be as given in 37. Let h (x) = min { x, x2} for every real number of x, then
the following table: (a) h is continuous for all x (1998, 2M)

x = −1 x=0 x=2 (b) h is differentiable for all x


(c) h ′ (x) = 1, ∀ x > 1
f (x) 3 6 0 (d) h is not differentiable at two values of x
g (x) 0 1 −1 | x − 3|, x≥1
38. The function f (x) =  x2 3x 13 is (1988, 2M)
− + , x<1
In each of the intervals (−1, 0) and (0, 2), the function  4 2 4
( f − 3 g )″ never vanishes. Then, the correct statement(s)
(a) continuous at x = 1 (b) differentiable at x = 1
is/are (2015 Adv.)
(c) discontinuous at x = 1 (d) differentiable at x = 3
rankershalt.com

Limit, Continuity and Differentiability 185

39. The function f (x) = 1 + |sin x|is  x


45. For the function f (x) = 1 + e1/ x , x ≠ 0 ;
(1986, 2M)
(a) continuous no where
(b) continuous everywhere  0 , x=0
(c) differentiable at x = 0 the derivative from the right, f ′ (0+ ) = … and the
(d) not differentiable at infinite number of points derivative from the left, f ′ (0− ) = … . (1983, 2M)
40. If x + | y| = 2 y, then y as a function of x is (1984, 2M)  1
46. Let f (x) = (x − 1) sin (x − 1) − |x|, if x ≠ 1 be a real
2
(a) defined for all real x (b) continuous at x = 0
dy 1 − 1, if x = 1
(c) differentiable for all x (d) such that = for x < 0
dx 3 valued function. Then, the set of points, where f (x) is
Assertion and Reason not differentiable, is …. . (1981, 2M)

For the following questions, choose the correct answer


from the codes (a), (b), (c) and (d) defined as follows.
True/False
(a) Statement I is true, Statement II is also true; 47. The derivative of an even function is always an odd
Statement II is the correct explanation of Statement I function. (1983, 1M)
(b) Statement I is true, Statement II is also true;
Statement II is not the correct explanation of Analytical & Descriptive Questions
Statement I
 −1  x + c 1
b sin  2  , − 2 < x < 0
(c) Statement I is true; Statement II is false
(d) Statement I is false; Statement II is true 
48. f (x) = 
1
41. Let f and g be real valued functions defined on interval , x=0
 2
(−1, 1) such that g′′ (x) is continuous, g (0) ≠ 0, g′ (0) = 0, a x/ 2
−1
 e 0<x<
1
g′′ (0) ≠ 0, and f (x) = g (x)sin x .  ,
x 2
Statement I lim [ g (x) cos x − g (0) cosec x] = f ′′ (0). and
x→ 0 1
If f (x) is differentiable at x = 0 and|c| < , then find the
Statement II f ′ (0) = g (0). (2008, 3M)
2
value of a and prove that 64b2 = (4 − c2). (2004, 4M)
Match the Columns
49. If f : [−1, 1] → R and f ′ (0) = lim nf   and f (0) = 0.
1
42. In the following, [x] denotes the greatest integer less n→ ∞  n
than or equal to x. 2  1
Find the value of lim (n + 1) cos −1   − n, given that
Column I Column II n→ ∞π  n
A. x| x| p. continuous in (– 1, 1)   1  π
0 <  lim cos −1    < . (2004, 2M)
B. | x| q. differentiable in (– 1, 1) n → ∞  n  2

C. x + [x ] r. strictly increasing (– 1, 1) 50. Let α ∈ R. Prove that a function f : R → R is


D. | x − 1| +| x + 1|, s. not differentiable atleast at one differentiable at α if and only if there is a function
in ( −1, 1) point in (– 1, 1) g : R → R which is continuous at α and satisfies
f (x) − f (α ) = g (x) (x − α ) , ∀ x ∈ R. (2001, 5M)
(2007, 6M)
51. Determine the values of x for which the following
43. Match the conditions/expressions in Column I with
statement in Column II (1992, 2M)
function fails to be continuous or differentiable
 1 − x, x<1

Column I Column II f (x) = (1 − x) (2 − x), 1 ≤ x ≤ 2 . Justify your answer.
sin (π [x])  3 − x, x>2
A. p. differentiable everywhere  (1997, 5M)
B. sin{ π (x − [x])} q. no where differentiable  −  1 + 1 
r. not differentiable at 1 and −1 52. Let f (x) = x e  |x | x  , x ≠ 0
 , x=0
 0
Fill in the Blanks Test whether
44. Let F (x) = f (x) g (x) h (x) for all real x, where f (x), g (x) (i) f (x) is continuous at x = 0.
and h (x) are differentiable functions. At same point (ii) f (x) is differentiable at x = 0. (1997C, 5M)
x0 , F ′ (x0 ) = 21 F (x0 ), f ′ (x0 ) = 4 f (x0 ), g ′ (x0 ) = − 7 g (x0 ) 53. Let f [(x + y) / 2] = { f (x) + f ( y)} / 2 for all real x and y, if
and h ′ (x0 ) = kh (x0 ), then k = K . (1997C, 2M)
f ′ (0) exists and equals −1 and f (0) = 1 , find f (2).
(1995, 5M)
rankershalt.com

186 Limit, Continuity and Differentiability

54. A function f : R → R satisfies the equation  x −1


f (x + y) = f (x) f ( y) , ∀ x , y in R and f (x) ≠ 0 for any x in 2x2 − 7x + 5 , when x ≠ 1
60. Find f ′ (1) , if f (x) =  .
R. Let the function be differentiable at x = 0 and  1
− , when x = 1
f ′ (0) = 2 . Show that f ′ (x) = 2 f (x) , ∀ x in R. Hence,  3 (1979, 3M)
determine f (x). (1990, 4M)
61. If f (x) = x tan −1 x , find f ′ (1) from first principle.
55. Draw a graph of the function (1978, 3M)
y = [x] + |1 − x|, − 1 ≤ x ≤ 3.
Integer Answer Type Questions
Determine the points if any, where this function is not
differentiable. (1989, 4M)
62. Let f : R → R be a differentiable function such that
 π
56. Let R be the set of real numbers and f : R → R be such f (0) = 0, f   = 3 and f′ (0) = 1.
 2
that for all x and y in R, f (x) − f ( y)|2 ≤ (x − y)3 . Prove π
that f (x) is a constant. (1988, 2M) If g (x) = ∫ 2 [ f ′ (t ) cosec t − cot t cosec t f (t )] dt
x
57. Let f (x) be a function satisfying the condition  π
f (− x) = f (x), ∀ x. If f ′ (0) exists, find its value. (1987, 2M) for x ∈ 0, , then lim g (x) =
 2 x→ 0 (2017 Adv.)
58. Let f (x) be defined in the interval [− 2, 2] such that 63. Let f : R → R and g : R → R be respectively given by
 −1 , −2 ≤ x ≤ 0 f (x) =|x|+ 1 and g (x) = x2 + 1. Define h : R → R by
f (x) = 
x − 1 , 0 < x ≤ 2 max{ f (x), g (x)}, if x ≤ 0 .
h (x) = 
and g (x) = f (| x|) + | f (x)|.  min{ f (x), g (x)}, if x > 0 .
Test the differentiability of g (x) in (− 2, 2). (1986, 5M)
The number of points at which h (x) is not differentiable
59. Let f (x) = x − x − x + 1
3 2
is (2014 Adv.)

= max { f (t ) ; 0 ≤ t ≤ x}, 0 ≤ x ≤ 1 64. Let p (x) be a polynomial of degree 4 having extremum


and g (x) 
= 3 − x, 1 < x ≤ 2  p (x) 
at x = 1 , 2 and lim 1 + 2  = 2 . Then, the value of p(2)
Discuss the continuity and differentiability of the x→ 0  x 
function g (x) in the interval (0, 2). (1985, 5M) is ……… . (2010)

Topic 8 Differentiation
Objective Questions I (Only one correct option) π π π π
(a) −x (b) x − (c) −x (d) 2x −
6 6 3 3
1. If C1 + (22) 20C 2 + (32) 20C3 + ..... + (202)20C 20 = A (2β )
20
dy
, then the ordered pair ( A , β) is equal to 6. For x > 1, if (2x)2y = 4e2x − 2y , then (1 + log e 2x)2 is
dx
(2019 Main, 12 April II)
equal to (2019 Main, 12 Jan I)
(a) (420, 19) (b) (420, 18) (c) (380, 18) (d) (380, 19)
x log e 2x + log e 2 x log e 2x − log e 2
 sin x − cos x  x (a) (b)
2. The derivative of tan −1   , with respect to , x x
 sin x + cos x  2 (c) x log e 2x (d) log e 2x
  π dy
where  x ∈ 0,   is 7. If x log e (log e x) − x2 + y2 = 4( y > 0), then at x = e is
  2 (2019 Main, 12 April II) dx
2 1 equal to (2019 Main, 11 Jan I)
(a) 1 (b) (c) (d) 2
3 2 e (2e − 1) (1+ 2e ) (1+ 2e )
(a) (b) (c) (d)
 dy d y 2
4+e 2
2 4+e 2
4+e 2
2 4 + e2
3. If ey + xy = e, the ordered pair  , 2  at x = 0 is equal
 dx dx  8. Let f :R→ R be a function such that
to (2019 Main, 12 April I) f (x) = x3 + x2f ′ (1) + xf ′ ′ (2) + f ′′′ (3), x ∈ R.
(a)  , − 2  (b)  − , 2  (c)  , 2  (d)  − , − 2 
1 1 1 1 1 1 1 1 Then, f (2) equals (2019 Main, 10 Jan I)
e e   e e   e e   e e  (a) 30 (b) − 4 (c) − 2 (d) 8
4. If f (1) = 1, f′ (1) = 3, then the derivative of d 2y π
9. If x = 3 tan t and y = 3sec t, then the value of at t = ,
f ( f ( f (x))) + ( f (x))2 at x = 1 is (2019 Main, 8 April II) dx2 4
is (2019 Main, 9 Jan II)
(a) 12 (b) 9 (c) 15 (d) 33
1 1
2 (a) (b)
  3 cos x + sin x   π dy 6 6 2
5. If 2 y =  cot−1    , x ∈ 0,  then is
 cos x − 3 sin x    1 3
 2 dx (c) (d)
3 2 2 2
equal to (2019 Main, 8 April I)
rankershalt.com

Limit, Continuity and Differentiability 187

10. For x ∈ 0,  , if the derivative of 18. If x2 + y2 = 1 , then


1 (2000, 1M)
 4 (a) yy′ ′ − 2 ( y ′ )2 + 1 = 0 (b) yy′ ′ + ( y ′ )2 + 1 = 0
 6x x  (c) yy′ ′ + ( y ′ )2 − 1 = 0 (d) yy′ ′ + 2 ( y ′ )2 + 1 = 0
tan −1   is x ⋅ g (x), then g (x) equals
 1 − 9x3  x3 sin x cos x
(2017 Main)
9 3x x 3x 3 19. Let f (x) = 6 −1 0 , where p is constant.
(a) (b) (c) (d)
1 + 9x 3
1 − 9x
3
1 − 9x 3
1 + 9x3 p p2 p3
1 d3
11. If g is the inverse of a function f and f ′ (x) = , then Then, f (x) at x = 0 is (1997, 2M)
1 + x5 dx3
g′ (x) is equal to (2015)
(a) p (b) p + p 2
(c) p + p3 (d) independent of p
(a) 1 + x5 (b) 5 x4
(c)
1
(d) 1 + { g (x)}5 20. If y2 = P (x) is a polynomial of degree 3, then
1 + { g (x)}5
d  3 d 2y
dy 2 y  equals (1988, 2M)
12. If y = sec (tan − 1 x), then at x = 1 is equal to dx  dx2 
dx (2013)
(a) P ′ ′ ′ (x) + P ′ (x) (b) P ′ ′ (x) ⋅ P ′ ′ ′ (x)
1 1
(a) (b) (c) 1 (d) 2 (c) P (x) P ′ ′ ′ (x) (d) a constant
2 2
13. Let g (x) = log f (x), where f (x) is a twice differentiable Fill in the Blanks
positive function on (0, ∞ ) such that f (x + 1) = x f (x). dy
 1  1 21. If x exy = y + sin 2 x, then at x = 0, = ...… . (1996, 2M)
Then, for N = 1, 2, 3,... , g′′  N +  − g′′   is equal to dx
 2  2
 
22. Let f (x) = x|x|. The set of points, where f (x) is twice
1 1 1
(a) − 4 1 + + + ... + 2 (2008, 3M) differentiable, is … . (1992, 2M)
 9 25 ( 2 N − 1) 
 
23. If f (x) = |x − 2|and g (x) = f [ f (x)] , then g ′ (x) = K… for
1 1 1
(b) 4 1 + + + ... +  x > 2. (1990, 2M)
 9 25 (2 N − 1)2 
−1 
1 
 1 1 1  24. The derivative of sec  − 2  with respect to
(c) − 4 1 + + + ... + 2
 2x − 1
 9 25 (2 N + 1)  1
1 − x2at x = is …… . (1986, 2M)
 1 1 1  2
(d) 4 1 + + + ... + 2
9 25 (2 N + 1)  25. If f (x) = log x (log x), then f ′ (x) at x = e is ...... . (1985, 2M)

d 2x 26. If fr (x), gr (x), hr (x), r = 1, 2, 3 are polynomials in x such
14. equals
dy2 (2007, 3M)
that fr (a ) = gr (a ) = hr (a ), r = 1, 2, 3
−1 −1 −3 f1 (x) f2(x) f3 (x)
 d y
2  d y 2
 dy 
(a)  2  (b) −  2    and F (x) = g1 (x) g2(x) g3 (x) ,
 dx   dx   dx 
h1 (x) h2(x) h3 (x)
 d 2 y  dy − 2  d 2 y  dy − 3
(c)  2    (d) −  2    then F ′ (x) at x = a is …… . (1985, 2M)
 dx   dx   dx   dx 
 2x − 1 dy
15. If f ′′ (x) = − f (x), where f (x) is a continuous double 27. If y = f   and f ′ (x) = sin 2 x, then = ....... .
 x2 + 1 dx (1982, 2M)
differentiable function and g (x) = f ′ (x).
2 2
  x    x  Analytical & Descriptive Questions
If F (x) =  f    +  g    and F (5) = 5,
  2    2  ax2 bx c
28. If y = + + + 1,
then F (10) is (2006, 3M) (x − a ) (x − b) (x − c) (x − b) (x − c) (x − c)
(a) 0 (b) 5 (c) 10 (d) 25
y′ 1  a b c 
16. Let f be twice differentiable function satisfying Prove that =  + + . (1998, 8M)
y x  a − x b − x c − x
f (1) = 1, f (2) = 4, f (3) = 9, then (2005, 2M)
dy
(a) f ′ ′ (x) = 2, ∀ x ∈ (R ) 29. Find at x = − 1, when
dx
(b) f ′ (x) = 5 = f ′ ′ (x), for some x ∈ (1, 3)
π
(c) there exists atleast one x ∈ (1, 3) such that f ′ ′ (x) = 2 sin x 3
(sin y) 2 + sec−1 (2x) + 2x tan ln (x + 2) = 0.
(d) None of the above 2 (1991, 4M)
17. If y is a function of x and log (x + y) = 2xy, then the value 30. If x = sec θ − cos θ and y = secn θ − cos nθ, then show that
of y′ (0) is (2004, 1M)
 dy
2

(a) 1 (b) −1 (c) 2 (d) 0 (x2 + 4)   = n 2 ( y2 + 4). (1989, 2M)


 dx
rankershalt.com

188 Limit, Continuity and Differentiability

31. If α be a repeated roots of a quadratic equation f (x) = 0 34. Let f be a twice differentiable function such that
(1983, 3M)
and A (x), B(x) and C (x) be polynomials of degree 3, 4 and
f ′ ′ (x) = − f (x) , f ′ (x) = g (x) and
A (x) B(x) C (x) h (x) = [ f (x) ]2 + [ g (x)]2
5 respectively, then show that A (α ) B(α ) C (α ) is Find h (10), if h (5) = 11.
A ′ (α ) B ′ (α ) C ′ (α ) 3 dy
35. Let y = ex sin x + (tan x)x , find .
divisible by f (x), where prime denotes the derivatives. dx (1981, 2M)
(1984, 4M) 5x dy
36. Given, y = + cos (2x + 1), find .
2

32. Find the derivative with respect to x of the function 3 (1 − x)2 dx


(1980)
  2x  
y = (log cos x sin x) (log sin x cos x)−1 + sin −1    Integer Type Questions
  1 + x2  
π 37. Let f : R → R be a continuous odd function, which
at x = . 1
4 (1984, 4M) vanishes exactly at one point and f (1) = .
2
33. If (a + bx) ey/ x = x, then prove that x
2
Suppose that F (x) = ∫ f (t ) dt for all x ∈ [−1 , 2] and
2
d y  dy  –1
x3
= x − y . x
G (x) = ∫ t| f { f (t )}| dt x ∈ [−1 , 2].
dx 2  dx  (1983, 3M) –1
for all If
F (x) 1  1
lim = , then the value of f   is
x → 1 G (x) 14  2 (2015 Adv.)

Answers
Topic 1 2 π –π
11. a = ,b = e 2/3 12. a = 8 13. a = ,b =
1. (b) 2. (c) 3. (d) 4. (a) 3 6 12
5. (c) 6. (b) 7. (a) 8. (b) 2   
2 1 
9. (d) 10. (c) 11. (b) 12. (d) 3  log  3 – 6 x, x ≤ 0

13. (d) 14. (c) 15. (c) 16. (c) 14. f ( x ) =  1/ x
  1 + x  , x>0
17. (d) 18. (d) 19. (b) 20. (c)   2 + x 
21. (a) 22. (d) 23. (a, c) 24. –1
–3 1
25. 1 26. h 2hr − h ,
2 1
27. − 1 15. a = , c = and b ∈ R 16. (d)
128r 2 2
2
28. 7 29. 30. False 31. loge 4 Topic 5
π
1. (a) 2. (d) 3. (d) 4. (b)
32. a 2 cosα + 2a sin α 33. 0
5. (c) 6. (d) 7. (b) 8. (b, c)
−1
34. 35. (7) 36. (2) 9. (a,b, d) 10. x ∈ (– ∞,– 1 ) ∪ [ 0, ∞ ),[ − 1, 0 )
3
11. f and f ′′ are continuous and f ′ is discontinuous at x = {1, 2 }.
Topic 2
1. (d) 2. (b) 3. (d) 4. (a) Topic 6
5. (c) 6. (c) 7. (d) 8. (c) 1. (b, c, d)
9. (b) 10. (d) 11. (c) 12. (c) x + a + 1, if x < −a
13. (c) 14. e2 15. e5 16. e2 ( x + a − 1 ) 2, if a ≤ x < c

17. a =2 2. g{ f ( x )} =  2
x + b , if 0 ≤ x ≤ 1
( x − 2 ) + b, if x > 1
2
Topic 3
1. (c) 2. (a) 3. (b) 4. (b,c) a = 1, b = 0
5. (1) 6. 1 gof is differentiable at x = 0
4 − x , 2 < x ≤ 3
Topic 4 
4. g ( x ) = 2 + x, 0 ≤ x ≤ 1, discontinuous at x = {1, 2 }
1. (a) 2. (b) 3. (b) 4. (c) 2 − x, 1 < x ≤ 2
5. (c) 6. (b) 7. (a,b,d) 8. (a, d) 
Discontinuity of g at x = {1, 2 }
9. (b,d) 10. f ( x ) = 4 − x 2
rankershalt.com

Limit, Continuity and Differentiability 189

Topic 7 59. g ( x ) is continuous for all x ∈ ( 0, 2 ) − {1 ] and g ( x ) is


1. (b) 2. (a) 3. (b) 4. (c) differentiable for all x ∈ ( 0, 2 ) − {1 }
 2 1 π
5. (b) 6. (a) 7. (a) 8. (a) 60.  −  61.  +  62. (2) 63. (3)
9. (c) 10. (a) 11. (b) 12. (d)  9 2 4
13. (b) 14. (c) 15. (b) 16. (a) 64. p (2 ) = 0
17. (d) 18. (d) 19. (a) 20. (d)
21. (c) 22. (d) 23. (a) 24. (a) Topic 8
25. (d) 26. (a,b,d) 27. (b,c,d) 28. (b,c) 1. (b) 2. (d) 3.
(b) 4. (d)
29. (a,b) 30. (b,c) 31. (a,d) 32. (b,c) 5. (b) 6. (b) 7.
(b) 8. (c)
9. (b) 10. (a) 11.
(d) 12. (a)
33. (b, c) 34. (a, b, c, d) 35. (b, c) 36. (a, d)
13. (a) 14. (d) 15.
(b) 16. (c)
37. (a, c, d) 38. (a, b) 39. (b, d) 40. (a, b, d) 17. (a) 18. (b) 19.
(d) 20. (c)
41. (b) 21. 1 22. x ∈ R − { 0} 23.
1 24. –4
42. (A) → p, q, r, s; (B) → p, s; (C) → r, s; (D) → p, s 1 –2 ( x 2 – x – 1 )  2x – 1 
25. 26. 0 27. ⋅ sin 2  2 
43. (A) → p; (B) → r 44. (24) e (x 2 + 1)2  x + 1
45. f ′( 0 + ) = 0, f ′( 0 − ) = 1 3 −8 32
29. 32. + 33. 11
 2 π π2 –3 loge 2
16 + π 2
46. x = 0 47. True 48. (a = 1 ) 49. 1 − 
 π 3
35. e x sin x (3 x 3 cos x 3 + sin x 3 ) + (tan x ) x [2 x cosec 2 x + log (tan x )]
51. (1, 2 ) 52. (i) Yes (ii) No  5
53. ( − 1 ) 54. e 2x 55. { 0, 1, 2 ) 57. f ′( 0 ) = 0 3 (1 – x ) 2 – 2 sin ( 4 x + 2 ), x < 1
36.  –5 37. (7)
58. g ( x ) is differentiable for all x ∈ ( − 2, 2 ) − { 0, 1 )  – 2 sin ( 4 x + 2 ), x > 1
3 ( x – 1 )
2

Hints & Solutions


0 ∞ x2 − ax + b
Topic 1 and Form 2. It is given that lim =5 …(i)
0 ∞ x→1 x−1
1. Let Since, limit exist and equal to 5 and denominator is
x + 2 sin x 0  zero at x = 1 , so numerator x2 − ax + b should be zero at
P = lim 0 form x = 1,
x→ 0
x2 + 2 sin x + 1 − sin 2 x − x + 1
So 1 − a + b =0 ⇒ a =1 + b …(ii)
On rationalization, we get On putting the value of ‘a’ from Eq. (ii) in
(x + 2 sin x)
P = lim 2 Eq. (i), we get
x → 0 x + 2 sin x + 1 − sin 2 x + x − 1
x2 − (1 + b) x + b (x2 − x) − b(x − 1)
lim = 5 ⇒ lim =5
× ( x2 + 2 sin x + 1 + sin 2 x − x + 1 ) x→1 x−1 x→1 x−1
(x − 1) (x − b)
= lim ( x2 + 2 sin x + 1 + sin 2 x − x + 1 ) ⇒ lim = 5 ⇒ lim (x − b) = 5
x→ 0 x→1 x−1 x→1
x + 2 sin x
× lim ⇒ 1 − b =5
x→ 0 x − sin 2 x + 2 sin x + x
2
⇒ b = −4 …(iii)
x + 2 sin x 0 
= 2 × lim 0 form
On putting value of ‘b’ from Eq. (iii) to Eq. (ii), we get
x→ 0 x2 − sin 2 x + 2 sin x + x a = −3
Now applying the L′ Hopital’s rule, we get So, a+ b=−7
1 + 2 cos x
P = 2 × lim 3. Given,
x → 0 2 x − sin 2 x +2 cos x + 1
x4 − 1 x3 − k3
(1 + 2) lim = lim 2
=2 [on applying limit] x→1 x − 1 x → k x − k2
0 −0 + 2 + 1
(x − 1)(x + 1)(x2 + 1)
3
=2 × =2 ⇒ lim
3
x→1 x −1
x + 2 sin x (x − k)(x2 + k2 + xk)
⇒ lim =2 = lim
x→ 0
x + 2 sin x + 1 − sin 2 x − x + 1
2 x→ k (x − k)(x + k)
rankershalt.com

190 Limit, Continuity and Differentiability

3k2 7. Clearly,
⇒ 2 ×2 =
2k
1 + 1 + y4 − 2
8 lim
⇒ k= y→ 0 y4
3
sin 2 x 0  1 + 1 + y4 − 2 1 + 1 + y4 + 2
4. Given limit is lim 0 form = lim ×
x→ 0 2 − 1 + cos x y→ 0 y4 1 + 1 + y4 + 2
2
sin x  2 x
= lim Q 1 + cos x = 2 cos 2  [rationalising the numerator]
x→ 0 x
2 − 2 cos (1 + 1 + y ) − 2
4
2 = lim [Q (a + b) (a − b) = a 2 − b2]
y→ 0
sin 2 x y ( 1 + 1 + y + 2)
4 4
= lim
x→ 0  x
2 1 − cos  1 + y4 − 1 1 + y4 + 1
 2 = lim ×
y→ 0
y4 ( 1 + 1 + y4 + 2 ) 1 + y4 + 1
sin 2 x  x 2 x
= lim Q 1 − cos 2 = 2 sin 4 
x→ 0  x [again, rationalising the numerator]
2 × 2 sin 2 
 4 y4
= lim
x2 16 y→ 0
y4 ( 1 + 1 + y4 + 2 ) ( 1 + y4 + 1)
= lim 2
= =4 2 [lim sin x = lim x]
x→ 0 x→ 0 x→ 0
 x 2 2
1
2 2  =
 4
2 2 ×2
cot3 x − tan x (by cancelling y4 and then by direct substitution).
5. Given, limit = lim
x → π /4  π 1
cos  x +  = .
 4 4 2
1 − tan 4 x 1  1  cot x − cos x 1 cos x(1 − sin x)
= lim × Q cot x = 8. lim = lim ⋅
 tan x  (π − 2x)3 3
x→ π / 2 x → π/ 2 8
π
1 3
x → π /4
(cos x − sin x) tan x 
sin x − x
2 2 
(1 − tan 2 x) 2 (1 + tan 2 x)
= lim × π  π 
x → π / 4 cos x − sin x tan3 x cos  − h 1 − sin  − h 
1 2  2 
cos 2 x − sin 2 x 2 (sec2 x) = lim ⋅ 3
= lim × h→ 0 8
π  π π 
x → π / 4 cos x − sin x cos 2 x tan3 x sin  − h  − + h
2  2 2 
[Q 1 + tan 2 x = sec2 x]
1 sin h (1 − cos h )
(cos x − sin x) (cos x + sin x) 2 sec4 x = lim
= lim × 8 h→ 0 cos h ⋅ h3
x → π /4 (cos x − sin x) tan3 x
[Q (a 2 − b2) = (a − b) (a + b)]  h
sin h 2 sin 2 
4
2 sec x 1  2
= lim (cos x + sin x) = lim
x → π / 4 tan3 x 8 h→ 0 cos h ⋅ h3
2 ( 2 )4  1 1  h
=  +  [on applying limit] sin h ⋅ sin 2 
(1) 3  2 2 1  2
= lim
 2 4 h→ 0 h3 cos h
= 4 2   = 8.
 2  h
2

 sin h  
sin 
2 1 2 1 1 1 1 1
6. lim
x cot 4x
= lim
x
.
1 tan 2x = lim    h  ⋅ ⋅ = × =
2 2 4 h → 0  h    cos h 4 4 4 16
x→0 sin x. cot 2x x→0 tan 4x sin 2 x 1
2  2 
1 4x x tan 2 2x
= lim . sin(π cos 2 x) sin π (1 − sin 2 x)
x→0 4 (tan 4 x) sin 2 x x2 9. lim = lim
2 2 x→0 x2 x→ 0 x2
1 4x  x   tan 2x 4
= lim   .  . sin(π − π sin x)
2
x→ 0 4 (tan 4 x)  sin x  2x  1 = lim
x→ 0 x2
1 4  x tan x 
= ⋅ 1 ⋅1 ⋅1 ⋅ = 1 Q lim = 1 = lim sin(π sin 2 x)
4 1 x→ 0 sin x x→ 0 x  = lim [Q sin (π − θ ) = sin θ ]
x→ 0 x2
sin π sin 2 x  sin 2 x  sin θ 
= lim × (π )  2  = π Q lim = 1
x→ 0 π sin x
2
 x  θ→ 0 θ 
rankershalt.com

Limit, Continuity and Differentiability 191

(1 – cos 2x)(3 + cos x) 2 sin 2 x(3 + cos x)  x2 x4  


10. We have, lim = lim  1 − + − ... 
x→ 0 x tan 4x x→ 0 tan 4x  x   2 ! 4 !  
x× × 4x  −2 sin  
2
4x  2   x 2
x3 
2 sin 2 x (3 + cos x) 1 − 1 + x + + + ... 
= lim × lim ×   2! 3!  
x→ 0 x2 x→ 0 4 tan 4x = lim
lim x→ 0 xn
x→ 0 4x
4  sin θ tan   2 x  2x2 x3 
= 2 × ×1 = 1 and lim = 1  − 2 sin   − x − − − ...
4 Q θlim
→0 θ θ→ 0 θ 
 2  2! 3! 
= lim 2
x→ 0
=2  x
4   xn − 2
 2
11.  ∞  form
PLAN  
∞ x  x2 
 0, if n< m sin 2 1 + x + + K
 a0
2  3! 
a0x + a1x n−1
+ K + an if n = m = lim
 ,
n
x→ 0 2
lim =  b0  x
x→∞ b 0x + b1x
m m−1
+ K + bm  + ∞, if n> mand a0b 0 > 0
2   xn − 3
 2

 − ∞, if n>m and a0b 0 < 0
Above limit is finite, if n − 3 = 0, i.e. n = 3.
Description of Situation As to make degree of x tan 2x − 2x tan x
numerator equal to degree of denominator. 15. lim
x→ 0 (1 − cos 2x)2
 x2 + x + 1 
∴ lim  − ax − b = 4 NOTE In trigonometry try to make all trigonometric functions in
x→ ∞  x+1  same angle. It is called 3rd Golden rule of trigonometry.
x2 + x + 1 − ax2 − ax − bx − b 2 tan x
⇒ lim =4 x − 2x tan x
x→ ∞ x+1 1 − tan 2 x
= lim
x→ 0 (2 sin 2 x)2
x2(1 − a ) + x(1 − a − b) + (1 − b)
⇒ lim =4  1 
x→ ∞ x+1 2x tan x  −1
 1 − tan 2 x 
Here, we make degree of numerator = lim
x→ 0 4 sin 4 x
= degree of denominator
1 − 1 + tan 2 x 
∴ 1−a =0 ⇒ a =1 2x tan x  
 1 − tan x 
2
x (1 − a − b) + (1 − b) = lim
and lim =4 x→ 0 4 sin 4 x
x→ ∞ x+1
3
 tan x
 ⋅x
3
⇒ 1−a −b =4 x
2x tan3 x 1  x 
⇒ b = − 4 [Q (1 − a ) = 0] = lim = lim
x → 0 2 sin 4 x (1 − tan 2 x) x → 0 2 sin 4 x (1 − tan 2 x)
f (2h + 2 + h 2) − f (2) 3
12. Here, lim  tan x
h→ 0 f (h − h 2 + 1) − f (1)  
1  x  1 ⋅ (1)3 1
[Q f ′ (2) = 6 and f ′ (1) = 4, given] = lim = =
2 x → 0
 sin x
4
2(1) (1 − 0) 2
4
 (1 − tan x)
2
Applying L’Hospital’s rule, 
 x 
{ f ′ (2h + 2 + h 2)} ⋅ (2 + 2h ) − 0 f ' (2) ⋅ 2
= lim = 1 − cos 2 (x − 1)
h → 0 { f ′ (h − h 2 + 1 )} ⋅ (1 − 2 h ) − 0 f ' (1) ⋅ 1 16. LHL = lim−
6 .2 x→1 x −1
= =3 [using f ′ (2) = 6 and f ′ (1) = 4]
4 .1 2 sin 2 (x − 1) |sin (x − 1)|
= lim = 2 lim
{(a − n ) nx − tan x} sin nx x→1 − x −1 x → 1− x −1
13. Given, lim =0
x→ 0 x2 Put x = 1 − h , h > 0, for x → 1− , h → 0
 tan x  sin n x |sin (− h )|
⇒ lim (a − n ) n −  ×n =0 = 2 lim
x→ 0 x  nx h→ 0 −h
⇒ {(a − n ) n − 1} n = 0 sin h
= 2 lim =− 2
⇒ (a − n ) n = 1 h → 0 −h
1
⇒ a=n+ 1 − cos 2 (x − 1)
n Again, RHL = lim
x →1+ x−1
(cos x − 1) (cos x − ex ) |sin (x − 1)|
14. lim = lim 2
x→ 0 xn x →1+ x−1
rankershalt.com

192 Limit, Continuity and Differentiability

Put x = 1 + h, h > 0 = G′ (1) =


1
+ 24
For x → 1 , h → 0
|sin h | sin h  2x 
= lim 2 = lim 2 = 2 Q G (x) = − 25 − x2 ⇒ G ′ (x) = 
h→ 0 h h→ 0 h  2 25 − x2 
∴ LHL ≠ RHL. 22. We have,
lim
Hence, x→1 f (x) does not exist. n
 1 
fn (x) = ∑ tan − 1   for all x ∈ (0, ∞ )
1  1 + (x + j) (x + j − 1)
(1 − cos 2 x) j=1
2 1 .|sin x|
17. lim = lim n
 (x + j) − (x + j − 1) 
x→ 0 x x → 0 2 x ⇒ fn (x) = ∑ tan − 1  
j=1  1 + (x + j) (x + j − 1)
At x=0
1 . sin h 1 n
RHL = lim = ⇒ fn (x) = ∑ [tan − 1 (x + j) − tan − 1 (x + j − 1)]
h→ 0 2 h 2
j=1
1 . sin h 1
and LHL = lim =− ⇒ fn (x) = (tan − 1 (x + 1) − tan − 1 x)
h→ 0 2 −h 2
+ (tan − 1 (x + 2) − tan − 1 (x + 1))
Here, RHL ≠ LHL
+ (tan − 1 (x + 3) − tan − 1 (x + 2))
∴ Limit does not exist.
+ ... + (tan − 1 (x + n ) − tan − 1 (x + n − 1))
sin [x]
, [x] ≠ 0 ⇒ fn (x) = tan − 1 (x + n ) − tan − 1 x
18. Since, f (x) =  [x]
 0, [x] = 0 This statement is false as x ≠ 0. i.e., x ∈ (0, ∞ ).
sin [x] (b) This statement is also false as 0 ∉ (0, ∞ )
 , x ∈ R − [0, 1)
⇒ f (x) =  [x] (c) fn (x) = tan − 1 (x + n ) − tan − 1 x
 0, 0 ≤ x<1
lim tan( fn (x)) = lim tan(tan − 1 (x + n ) − tan − 1 x)
x→ ∞ x→ ∞
At x = 0,  n 
RHL = lim 0 = 0 ⇒ lim tan( fn (x)) = lim tan  tan − 1 
x→ 0 +
x→ ∞ x→ ∞  1 + nx + x 
2

sin [x] sin [0 − h ] = lim


n
=0
and LHL = lim = lim x → ∞ 1 + nx + x2
x→ 0 −
[x] h→ 0 [0 − h ]
sin (−1) ∴ (c) statement is false.
= lim = sin 1
h→ 0 −1 (d) lim sec2( fn (x)) = lim (1 + tan 2 fn (x))
Since, RHL ≠ LHL x→ ∞ x→ ∞
= 1 + lim tan 2( fn (x)) = 1 + 0 = 1
∴ Limit does not exist. x→ ∞
∴ (d) statement is true.
 1 2 n 
19. lim  + +K+  x2
n→∞  1 − n 2
1 − n 2
1 − n 2 a − a 2 − x2 −
23. L = lim 4 , a >0
1+2+3+K+ n n (n + 1) x→ 0 x4
= lim = lim
n→∞ (1 − n 2) n→∞ 2 (1 − n ) (1 + n )  1 1  
 1 x2 2  2 − 1 x4  x2
n 1 a − a ⋅ 1 − ⋅ 2 + ⋅ 4 − ... −
⇒ lim =−
n→∞ 2 (1 − n ) 2  2 a 2 a  4
 
20. Given, f (a ) = 2, f ′ (a ) = 1, g (a ) = − 1, g ′ (a ) = 2 = lim
x→ 0 x4
g (x) f (a ) − g (a ) f (x)
∴ lim x2 1 x4 x2
x→ a x−a + ⋅ 3 + ... −
= lim 2a 8 a 4 4
g ′ (x) f (a ) − g (a ) f ′ (x) x→ 0 x
= lim , Since, L is finite
x→ a 1 −0
[using L’ Hospital’s rule] ⇒ 2a = 4 ⇒ a = 2
1 1
= g ′ (a ) f (a ) − g (a ) f ′ (a ) ∴ L = lim =
x → 0 8 ⋅ a3 64
= 2 (2) − (−1) (1) = 5
log (1 + 2h ) − 2 log (1 + h ) 0 
21. Given, G (x) = − 25 − x2 24. lim 0 form
h→ 0 h2
G (x) − G (1) G ′ (x) − 0
∴ lim = lim Applying L’Hospital’s rule, we get
x→1 x−1 x→1 1 −0 2 2

[using L’ Hospital’s rule] 1 + 2h 1 + h
= lim
h→ 0 2h
rankershalt.com

Limit, Continuity and Differentiability 193

2 + 2h − 2 − 4h 3x2 + 2x − 16 6x + 2
= lim = lim = lim =7
h→ 0 2h (1 + 2h ) (1 + h ) x→ 2 2 (x − 2) x→ 2 2
−1 ∴ k=7
= lim = −1
h→ 0 (1 + 2h ) (1 + h ) πx
29. lim (1 − x) tan
sin x, x ≠ nπ , n = 0, ± 1, ± 2, K x→1 2
25. Given, f (x) = 
 2 , otherwise Put x−1 = y
{ f (x)}2 + 1 , f (x) ≠ 0, 2 π  π 
∴ − lim y tan ( y + 1) = − lim y − cot  y 
 y→ 0 2 y→ 0  2 
g [ f (x)] =  4 , f (x) = 0
 5 , f (x) = 2  π 
  y 
= lim  2 ⋅2 = 2
g [ f (x)] = (sin x) + 1, x ≠ , nπ = 0, ± 1, K
2
∴ y→ 0  π  π π
 5 , x = nπ 
tan y
2 
Now, lim g [ f (x)] = lim (sin 2 x) + 1 = 1
x→ 0 x→ 0 30. If lim [ f (x) g (x)] exists, then both lim f (x) and lim g (x)
x→ a x→ a x→ a
26. Given, P = 2 ( 2hr − h 2 + 2hr ) may or may not exist. Hence, it is a false statement.
A 2x − 1 1+ x+1 (2x − 1)( 1 + x + 1)
31. lim × = lim
x→ 0 1 + x −1 1+ x + 1 x→ 0 x
= log e (2) ⋅ (2)
h = 2 log e 2 = log e 4
r (a + h )2 sin (a + h ) − a 2 sin a
32. Here, lim
h–r

h→ 0 h
B C
D a 2[sin (a + h ) − sin a ]
= lim
h→ 0 h
h [2a sin (a + h ) + h sin (a + h )]
Here, BD = r 2 − (h − r )2 = 2hr − h 2 +
1 h
∴ A = . 2BD. h = ( 2hr − h 2) h  h  h
2 a 2 ⋅ 2 cos  a +  ⋅ sin
 2 2
A h 2hr − h 2 = lim + (2a + h ) sin (a + h )
∴ lim = h→ 0 h
lim 2⋅
h→ 0 P 3 h→ 0 8 ( 2hr − h 2 + 2hr )3 2
= a 2 cos a + 2a sin a
h3/ 2 ( 2r − h )
= lim lim(x − sin x)1/ 2
h→ 0 8 h3/ 2 ( 2r − h + 2r )3 x − sin x
33. lim = x→ 0
x→ 0 x + cos 2 x lim(x + cos 2 x)1/ 2
1 2r 1
= ⋅ = x→ 0
8 ( 2r + 2r )3
128 r 1/ 2
  sin x 
lim x 1 − 
 4 1 2 1 x→ 0   x  
 x sin + x  x4 sin + x2 =
27. lim  x  = lim x lim(0 + 1)1/ 2
x→ 0
x → −∞  1 + |x |3  x → −∞ 1 − x3
  0 .0
= =0
On dividing by x3 , we get 1
sin (1 / x) 1  x −1  1
+ 34. lim   = lim
1 x x→1  (x − 1)(2x − 5)  x→1 (2x − 5)
x 1+0 1
lim = = −1 =−
x → −∞ 1
−1 0 −1 3
3
x
x2 sin (βx)
 x3 + x2 − 16x + 20 35. Here, lim =1
x → 0 αx − sin x
28. f (x) =  (x − 2)2
, if x ≠ 2
  (βx)3 (βx)5 
k , if x = 2 x2βx − + − K
 3! 5! 
Since, continuous at x = 2. ⇒ lim =1
x→ 0  x3
x5 
x3 + x2 − 16x + 20 αx −  x − + − K
⇒ f (2) = lim ,[using L’Hospital’s rule]  3! 5! 
x→ 2 (x − 2)2
rankershalt.com

194 Limit, Continuity and Differentiability

 β3 x2 β 5 x4  1
x3 β − + − K  1 + f (3 + x) − f (3) x
 3! 5!  So, lim  =1
⇒ lim =1 x→ 0  1 + f (2 − x) − f (2 ) 
x→ 0 x3 x5
(α − 1)x +
+ −K
3! 5! lim π − 2 sin −1 x
2. Let L = , then
Limit exists only, when α − 1 = 0 x → 1− 1−x
⇒ α =1 …(i)
lim π − 2 sin −1 x π + 2 sin −1 x
 β3 2
x β5 x4  L= ×
x3 β − + − K x → 1− 1−x π + 2 sin −1 x
 3! 5! 
∴ lim =1 [on rationalization]
x→ 0  1 x 2 
x  −
3
− K lim π − 2 sin −1 x 1
3! 5!  = ×
x → 1− 1−x π + 2 sin −1 x
⇒ 6β = 1 …(ii)
π 
From Eqs. (i) and (ii), we get π − 2 − cos −1 x
6(α + β ) = 6α + 6 β
lim 2  1
= ×
=6 + 1 = 7 x → 1− 1−x π + 2 sin −1 x
 ecos ( α n ) − e   −1 −1 π
36. Given, lim  =−
e Q sin x + cos x = 2 
α→ 0 
 α m
 2
lim 2 cos −1 x lim 1
cos( α n ) −1 = − ×
− 1} cos(α ) − 1 − e x → 1−
n
e {e x→ 1 1−x π + 2 sin −1 x
⇒ lim ⋅ =
α→ 0 cos(α n ) − 1 αm 2 1 lim 2 cos x −1
 lim π
−1
= Q x → 1− sin x = 2 

n −
−2 sin 2 π x→ 1 1−x  
 ecos( α n ) −1 − 1 
⇒ lim e  ⋅ 2 = − e /2 Put x = cos θ, then as x → 1− , therefore θ → 0+
 lim
 cos(α ) − 1  α→ 0 αm
α→ 0 n
1 lim 2θ
Now, L = +
 α n 2 π θ→0 1 − cos θ
sin 2 
 2  α 2n −e 1 lim 2θ  2 θ
⇒ e × 1 × (−2) lim ⋅ m= = + Q 1 − cos θ = 2 sin 2 
α→ 0 α 2n 4α 2 2 π θ→0  θ
2 sin  
4  2
α 2n − m − e  θ
⇒ e × 1 × − 2 × 1 × lim = 2⋅  
α→ 0 4 2 1  2
= ⋅ 2 lim
For this to be exists, 2n − m = 0 2 π θ→ 0 +  θ
sin  
m  2
⇒ =2
n 1 2  lim θ 
= 2 2 = Q x → 0+ sin θ = 1
2 π π  
Topic 2 1∞ Form, RHL and LHL
1
3.
Key Idea lim f ( x) exist iff
 1 + f (3 + x) − f (3) x ∞
x→a
1. Let l = lim  [1 form] lim f ( x) = lim f ( x)
x→ 0  1 + f (2 − x) − f (2 )  x → a+ x → a−

1 1 + f (3 + x ) − f (3 ) 
lim 1 −  At x = 0,
x 1 + f ( 2 − x ) − f ( 2) 
⇒ l = ex→ 0
tan(π sin 2 x) + (|x| − sin(x [x]))2
 1 + f ( 2 − x ) − f ( 2) − 1 − f (3 + x ) + f (3 )  RHL = lim
lim 
x→ 0  x(1 + f ( 2 − x ) − f ( 2)) 

x → 0+ x2
= e
 f ( 2 − x ) − f (3 + x ) + f (3 ) − f ( 2)  tan(π sin 2 x) + (x − sin(x ⋅ 0))2
lim 
x(1 + f ( 2 − x ) − f ( 2))  = lim
=e x→ 0   x→ 0 +
x2
On applying L’Hopital rule, we get  Q|x| = x for x > 0 
and [x] = 0 for 0 < x < 1

lim 
− f ′( 2 − x )− f ′(3 + x )   
1 − xf ′( 2 − x ) + f ( 2 − x ) − f ( 2) 
l = ex→ 0  tan (π sin 2 x) + x2
= lim
On applying limit, we get x → 0+ x2
 − f ′( 2) − f ′(3 ) 
   tan(π sin 2 x) π sin 2 x 
l=e
 1 − 0 + f ( 2) − f ( 2) 
= e0 = 1 = lim  . + 1
x→ 0 
+
π sin x
2
x 2

rankershalt.com

Limit, Continuity and Differentiability 195

tan (π sin 2 x) sin 2 x π 


= π lim . lim +1 ( − h + sin h ) sin  (− 1)
x→ 0 +
π sin x
2
x→ 0 +
x2 2 
= lim
 tan x  h→ 0 + h (− 1)
Q xlim =1 
=π+1 
→0 x
 (Q [x] = − 1 for − 1 < x < 0 and h → 0+ ⇒ − h → 0− )
sin x
and xlim =1 
 →0 x  (− h + sinh)  − π
= lim sin  
h→ 0 + −h  2 
and LHL
tan (π sin 2 x) + (|x| − sin (x [x])2  sin h   h
= lim = lim   − lim+  
x→ 0 −
x2 h→ 0 +  h  h → 0  h

tan (π sin 2 x) + (− x − sin(x (− 1))2  sin h   h  sin h 


= lim = lim   − lim+   = 1 − 1 = 0 Q lim+ = 1
x→ 0 −
x2 h→ 0 +  h  h → 0  h  h→ 0 h 
 Q|x| = − x for x < 0  x([x] + | x|) sin [x] x([x] − x) sin [x]
and [x] = − 1 for − 1 < x < 0 5. lim = lim
  x → 0− | x| x → 0− −x
tan(π sin 2 x) + (x + sin(− x))2 (Q| x| = − x, if x < 0)
= lim
x → 0− x2 x(− 1 − x) sin (− 1)
= lim (Q lim [x] = − 1)
tan(π sin 2 x) + (x − sin x)2 x → 0− −x x → 0−
= lim
x → 0− x2 − x(x + 1) sin(− 1)
= lim = lim (x + 1)sin(− 1)
[Qsin (− θ ) = − sin θ] x→ 0 − −x x → 0−
 tan(π sin x) + x + sin x − 2x sin x
2 2 2
= lim   = (0 + 1) sin (− 1) (by direct substitution)
x → 0−  x2 
= − sin 1 (Qsin(− θ) = − sin θ)
 tan(π sin 2 x) sin 2 x 2x sin x
= lim  2
+1+ −  6. Key Idea Use property of greatest integer function [ x ] = x − { x }.
x → 0−  x x2 x2 
 tan (π sin 2 x) π sin 2 x sin 2 x sin x  1  2  15 
= lim  . + 1+ −2  lim x   +   + …+  
x→ 0 

π sin x
2
x2
x2 x    x  x
x→ 0 +  x 
tan(π sin 2 x) π sin 2 x We know, [x] = x − { x}
= lim . lim +
x→ 0 −
π sin x
2
x→ 0 −
x2 1  1 1 
∴  x  = x −  x 
sin 2 x sin x
1 + lim − 2 lim
x→ 0 −
x2 x → 0− x  n  n n 
= π + 1 + 1 −2 = π
Similarly,  x  = x −  x 
Q RHL ≠ LHL  1 1  2 2  15 15 
∴Given limit = lim x −   + −   + … −  
∴ Limit does not exist x→ 0 +  x  x  x x x  x 
4. Given,  1  2  15 
= lim (1 + 2 + 3+ ...+15) − x   +   + ... +  
π  x→ 0 +  x x  x 
(1 − |x| + sin|1 − x|) sin  [1 − x]
2   
lim n 
x →1+ |1 − x|[1 − x] Q 0 ≤  x  < 1, therefore 
= 120 − 0 = 120  
Put x = 1 + h , then n  n 
0 ≤ x  < x ⇒ lim x  = 0
 x x→ 0 +  x  
x → 1+ ⇒ h → 0+
π  1 − x( 1 + |1 − x|)  1 
(1 − |x| + sin|1 − x|) sin  [1 − x] 7. f (x) = cos  
2  |1 − x|  1 − x
∴ lim
x →1+ |1 − x|[1 − x] 1 − x(1 + 1 − x)  1 
Now, lim f (x) = lim cos  
π  x→1 − x→1 − 1−x  1 − x
(1 − |h + 1| + sin|− h|) sin  [− h ]
2 
= lim  1 
h→ 0 + |− h|[− h ] = lim (1 − x) cos   =0
x→1 −
 1 − x
π 
(1 − (h + 1) + sin h ) sin  [− h ] 1 − x(1 − 1 + x)  1 
2  and lim f (x) = lim cos  
= lim x−1
h→ 0 + h [− h ] x→1 + x→1 +  1 − x

(Q|− h| = h and|h + 1| = h + 1 as h > 0)  1 


= lim − (x + 1) ⋅ cos   , which does not exist.
x→1 +  x + 1
rankershalt.com

196 Limit, Continuity and Differentiability


1
Applying L’Hospital’s rule, we get
8. Given, p = lim (1 + tan 2 x ) 2x (1∞ form)  1 
x → 0+ x − 
 x2  sin x
tan 2 x
2 lim lim tan x
1  tan x  x → 0 − cosec x cot x
lim
+
lim 
2x → 0+ 
 1 e = ex → 0 x
= e0 = 1
x→ 0 2x x 
=e =e = e2 1/ x
1  f (1 + x)  1
12. Let y =  ⇒ log y = [log f (1 + x) − log f (1)]
f (1) 
1
∴ log p = log e2 =  x
2
 1 
9. PLAN To make the quadratic into simple form we should ⇒ lim log y = lim  ⋅ f ′ (1 + x)
x→ 0 x → 0  f (1 + x) 
eliminate radical sign.
Description of Situation As for given equation, [using L’ Hospital’s rule]
when a → 0 the equation reduces to identity in x. f (1) 6
= =
i.e. ax2 + bx + c = 0, ∀ x ∈ R or a = b = c → 0 f (1) 3
Thus, first we should make above equation independent  
⇒ log  lim y = 2 ⇒ lim y = e2
from coefficients as 0.  x→ 0  x→ 0

Let a + 1 = t . Thus, when a → 0, t → 1.


6
 x − 3
x
(1 − 3 / x)x e−3
13. For x ∈ R , lim   = lim = 2 = e−5
∴ (t 2 − 1) x2 + (t3 − 1)x + (t − 1) = 0 x→ ∞  x + 2 x→∞ (1 + 2 / x)x e
⇒ (t − 1) {(t + 1) x2 + (t 2 + t + 1) x + 1} = 0, as t → 1 2

 1 + 5x2
1/ x 2 lim [(1 + 5x2)1/5 x ]5
2x + 3x + 1 = 0
2
x→ 0 e5
14. lim   = = = e2
x → 0  1 + 3 x2  e3
2
⇒ 2x2 + 2x + x + 1 = 0 lim [(1 + 3x2)1/3 x ]3
x→ 0
⇒ (2x + 1) (x + 1) = 0
x+4 x+ 4
x = − 1, − 1 / 2  x + 6  5 
Thus, 15. lim   = lim 1 +  [1∞ form]
x → ∞  x + 1 x→ ∞  x + 1
or lim α (a ) = − 1 / 2
a→ 0 + 5 ( x + 4)
lim
and lim β (a ) = − 1 x→ ∞ x +1
a→ 0 + =e = e5

10. Here, lim {1 + x log (1 + b )} 2 1/ x
[1 form] 1/ x
x→ 0  π 
1 16. lim tan  + x 
lim {x log (1 + b 2 )} ⋅ x→ 0 4 
= ex → 0 x
1/ x
 π 
= elog (1 + b
2
)
= (1 + b2) …(i)  tan + tan x  1 + tan x 
1/ x
= lim  4 = lim 
π  
x→ 0 x → 0  1 − tan x 
Given, lim {1 + x log (1 + b )} 2 1/ x
= 2b sin θ2
1 − tan tan x 
x→ 0  4 
⇒ (1 + b ) = 2b sin θ
2 2
[(1 + tan x)1/tan x ]tan x/ x e1
1 + b2 = lim − −
= −1 = e2
∴ sin 2 θ = …(ii) x → 0 [(1 − tan x) 1/tan x
] tan x/ x
e
2b
1 sin x
b+ 1/ 2
17. PLAN lim =1
x→0
By AM ≥ GM, b ≥  b ⋅ 1 x
  (1 + x ) (1 − x )
2  b
sin (x − 1) + a (1 − x)  1− x 1
b2 + 1 Given, lim   =
x → 1  (x − 1 ) + sin (x − 1 )  4
⇒ ≥1 …(iii)
2b 1+ x
 sin (x − 1) 
From Eqs. (ii) and (iii),  (x − 1) − a  1
lim  =
sin 2 θ = 1 x→1 sin(x − 1)  4
π 1 + 
⇒ θ=± , as θ ∈ (− π , π ]  (x − 1) 
2 2
1 − a 1
 1
sin x ⇒   = ⇒ (a − 1)2 = 1
11. Here, lim (sin x) 1/ x
+ lim    2  4
x→ 0 x → 0  x
sin x
⇒ a = 2 or 0
 1 log (1/ x )
log  
 x
lim
x → 0 cosec x
 lim (sin x)1/ x → 0  Hence, the maximum value of a is 2.
= 0 + lim e =e x → 0 

x→ 0
as, (decimal) → 0
rankershalt.com

Limit, Continuity and Differentiability 197

Topic 3 Squeeze, Newton-Leibnitz’s Theorem and Limit Based


on Coverting infinite Serie s into Definite Integrals
1. Given α and β are roots of quadratic equation
375x2 − 25x − 2 = 0 Taking log on both sides, we get
x
25 1 
∴ α+β= = n 
… (i) n
 n n (x + n )  x +
n 
375 15  K x +  
 2  n 
2 log e { f (x)} = lim log 
and αβ = − … (ii) n →∞   n 2  2 n 2 
375  n !(x2 + n 2)  x2 +  K  x + 2 
n n   4  n  
Now, lim
n→ ∞
∑ α r + nlim
→∞
∑ βr
r =1 r =1  n
 1  
 Π x +  
= (α + α + α + K + upto infinite terms)+
2 3 x r =1  r / n
= lim ⋅ log  
(β + β 2 + β 3 + K + upto infinite terms)
n→ ∞ n  n  2 1  n 
 rΠ x +
=1 
 Π (r / n ) 
   (r / n )2  r = 1 
α β a
= + Q S ∞ = for GP
1 −α 1 −β  1−r 
α (1 − β) + β (1 − α ) α − αβ + β − αβ  
= =  n 
n x+
(1 − α ) (1 − β ) 1 − α − β + αβ
log  
1
(α + β ) − 2αβ
= x lim
n→ ∞ n ∑  n
r
2
r 
r =1
=   x2 + 2  
1 − (α + β ) + αβ   r  n 
1 −2
On substituting the value α + β = and αβ = from  r 
15 375 n  ⋅x + 1 
1
Eqs. (i) and (ii) respectively, = x lim
n→ ∞ n
∑ log  rn2 2 
we get r =1
 2 ⋅ x + 1
1 4 n 
+
15 375 29 29 1 Converting summation into definite integration, we get
= = = =
1 2 375 − 25 − 2 348 12 1  xt + 1 
1− − log e{ f (x)} = x ∫ log  2 2  dt
15 375 0  x t + 1
sec 2 x
Put, tx=z
∫ f (t ) dt 0  ⇒ xdt = dz
2
2. lim 0 form  1 + z  dz
x→
π π2 ∴
x
log e { f (x)} = x ∫ log  
x −2
4
16 0  1 + z 2 x
f (sec2 x) 2 sec x sec x tan x  1+ z
= lim ⇒
x
log e{ f (x)} = ∫ log   dz
x→ π /4 2x 0  1 + z 2
[using L’ Hospital’s rule]
2 f (2) 8 Using Newton-Leibnitz formula, we get
= = f (2)
π /4 π 1  1 + x
⋅ f ′ (x) = log   … (i)
1
2n
r 1
2n
r f (x)  1 + x2
3. Let I = lim
n→ ∞ n
∑ = lim
n→ ∞ n
∑ Here, at x = 1 ,
r =1 n +r
2 2
r =1 n 1 + (r / n )2
f ′ (1)
1
2n
r /n = log (1) = 0
= lim
n→ ∞
∑ f (1)
n r =1 1 + (r / n )2 ∴ f′ (1) = 0
2 x Now, sign scheme of f ′ (x) is shown below
=∫ dx = [ 1 + x2 ]20 = 5 −1
0
1 + x2 + –

x=1
4. Here,
x
 n ∴ At x = 1 , function attains maximum.
 n n (x + n )  x +
n  n
 … x +   Since, f (x) increases on (0, 1).
   n 
f (x) = lim 
2 ∴ f (1) > f (1 / 2)
,x>0
n→ ∞   n 2  n 2 
 n ! (x2 + n 2)  x2 +  K  x2 + 2  ∴ Option (a) is incorrect.
  4  n   f (1 / 3) < f (2 / 3)
∴Option (b) is correct.
rankershalt.com

198 Limit, Continuity and Differentiability

Also, f ′ (x) < 0, when x > 1 Topic 4 Continuity at a Point


⇒ f′ (2) < 0
1. Given function is
∴ Option (c) is correct.
 1 + x  2 cos x − 1 π
f ′ (x)  ,x≠
Also, = log  
f (x)  1 + x2 f (x) =  cot x − 1 4
 π
k , x=
f ′ (3) f ′ (2) 4  3  4
∴ − = log   − log  
f (3) f (2)  10  5
Q Function f (x) is continuous, so it is continuous at
= log (2 / 3) < 0 π
x= .
f ′ (3) f ′ (2) 4
⇒ <
f (3) f (2)  π
∴ f   = lim f (x)
∴ Option (d) is incorrect.  4  x→ π
4
5. We have, lim 2 cos x − 1
1 ⇒ k= π
yn =
[(n + 1) (n + 2) … (n + n)]1 / n x→ cot x − 1
n 4
and lim yn = L π π
n→ ∞ Put x = + h, when x → , then h → 0
1 4 4
⇒ L = lim [(n + 1) (n + 2) (n + 3) … (n + n)]1 / n
n→ ∞ n
π 
1 2 cos  + h − 1
 n  n lim 4 
L = lim  1 +  1 +
1 2  3  k=
⇒  1 +  ... 1 +  
n→ ∞   n  n  n  n  h→0 π 
cot + h − 1
1 n 
4 
⇒ log L = lim  log 1 +
1  2 
 + log 1 +  … log 1 +  
n→ ∞ n   n  n  n   1 1 
2 cos h − sin h − 1
n lim  2 2 
∑ log 1 + n 
1 r =
⇒ log L = lim
n→ ∞ h→0 cot h − 1
n r =1 −1
1
cot h + 1
⇒ log L = ∫0 1II × log
I
(1 + x) dx
[Q cos (x + y) = cos x cos y − sin x sin y and
1 cot x cot y − 1
(log(1 + x) ∫ dx  dx
d cot (x + y) =
⇒ log L = (x ⋅ log (1 + x))10 −∫ ]
0
 dx  cot y + cot x
[by using integration by parts] lim cos h − sin h − 1
=
⇒ log L = [x log(1 + x)]10 −∫
1 x
dx h→0 −2
01 + x
1 + cot h
1 x +1 1 
⇒ log L = log 2 − ∫0  x + 1 − x + 1  dx lim  (1 − cos h ) + sin h 
=  (sin h + cos h )
h→0  2 sin h 
⇒ log L = log 2 − [x]10 + [log(x + 1)]10
 2 sin 2 h + 2 sin h cos h 
⇒ log L = log 2 − 1 + log 2 − 0 lim  
= 2 2 2 (sin h + cos h )
4 4 
⇒ log L = log 4 − log e = log ⇒ L= ⇒ h→ 0 h
4 sin cos
h

e e  2 2 
4 
[L ] =   = 1  h h 
 e lim sin 2 + cos 2  1
= × (sin h + cos h ) ⇒ k =
x2 h → 0  h
 2
∫0 cos 2tdt
0  
2 cos
2 
6. lim 0 form 
x→ 0 x sin x
2. NOTE All integers are critical point for greatest integer function.
Applying L’Hospital’s rule, we get Case I When x ∈ I
cos 2(x2) ⋅ 2x − 0 2 ⋅ cos 2(x2) 2 f (x) = [x]2 − [x2] = x2 − x2 = 0
= lim = lim = =1
x → 0 x cos x + sin x x→ 0 sin x 1 + 1 Case II When x ∉ I
cos x +
x If 0 < x < 1, then [x] = 0
rankershalt.com

Limit, Continuity and Differentiability 199

and 0 < x2 < 1, then [x2] = 0 a log (1 + ax) b log (1 − bx)


= lim +
Next, if 1 ≤ x <2 ⇒ 1 ≤ x< 2
2 x→ 0 ax − bx
log (1 + x)
⇒ [x] = 1 and [x2] = 1 = a ⋅1 + b ⋅1 [using lim = 1]
x→ 0 x
Therefore, f (x) = [x]2 − [x2] = 0, if 1 ≤ x < 2
=a+b
Therefore, f (x) = 0, if 0 ≤ x < 2 ∴ f (0) = (a + b)
This shows that f (x) is continuous at x = 1. 7. f (x) = x cos(π (x + [x]))
Therefore, f (x) is discontinuous in (−∞ , 0) ∪ [ 2 , ∞ ) on
many other points. Therefore, (b) is the answer. At x = 0
lim f (x) = lim x cos(π (x + [x]) = 0
3. Given, f (x) = [tan 2 x] x→ 0 x →0
and f (x) = 0
Now, − 45° < x < 45°
∴It is continuous at x = 0 and clearly discontinuous at
⇒ tan (− 45° ) < tan x < tan (45° ) other integer points.
⇒ − tan 45° < tan x < tan (45° )
8. PLAN If a continuous function has values of opposite sign inside an
⇒ −1 < tan x < 1 interval , then it has a root in that interval.
⇒ 0 < tan 2 x < 1 f,g : [0, 1] → R
⇒ [tan 2 x] = 0 We take two cases.
i.e. f (x) is zero for all values of x from x = − 45° to 45°. Case I Let f and g attain their common maximum
Thus, f (x) exists when x → 0 and also it is continuous at value at p.
x = 0 . Also, f (x) is differentiable at x = 0 and has a value ⇒ f ( p) = g ( p),
of zero. where p ∈ [0, 1]
Therefore, (b) is the answer. Case II Let f and g attain their common maximum
2x − 1
4. Here, f (x) = [x] cos 
value at different points.
π
 2  ⇒ f (a ) = M and g (b) = M
  2x − 1 ⇒ f (a ) − g (a ) > 0 and f (b) − g (b) < 0
− cos  2  π , −1 ≤ x < 0
⇒ f (c) − g (c) = 0 for some c ∈ [0, 1] as f and g are
0 , 0 ≤ x<1
 continuous functions.
∴ f (x) =  cos  2x − 1 π , 1 ≤ x<2
  ⇒ f (c) − g (c) = 0 for some c ∈ [0, 1] for all cases. ...(i)
  2 
  2 x − 1  Option (a) ⇒ f (c) − g (c) + 3 [ f (c) − g (c)] = 0
2 2
2 cos  2  π , 2 ≤ x<3
 which is true from Eq. (i).
which shows RHL = LHL at x = n ∈ Integer as if x = 1 Option (d) ⇒ f 2(c) − g 2(c) = 0 which is true from Eq. (i)
 2x − 1 Now, if we take f (x) = 1 and g (x) = 1, ∀x ∈ [0, 1]
⇒ lim cos   π = 0 and lim− 0 = 0
x →1+  2  x→1
Options (b) and (c) does not hold. Hence, options (a) and
Also, f (1) = 0 (d) are correct.
∴ Continuous at x = 1. 9. f (2n ) = a n , f (2n + ) = a n
Similarly, when x = 2,
f (2n − ) = bn + 1
lim f (x) = lim f (x) = 0
x → 2+ x → 2− ⇒ a n − bn = 1
Thus, function is discontinuous at no x. f (2n + 1) = a n
Hence, option (c) is the correct answer. f {(2n + 1)− } = a n
5. Given, f (x) = x ( x + x + 1 ) f {(2n + 1)+ } = bn + 1 − 1
⇒ f (x) would exists when x ≥ 0 and x + 1 ≥ 0. ⇒ a n = bn + 1 − 1 or a n − bn + 1 = − 1
⇒ f (x) would exists when x ≥ 0. or a n − 1 − bn = − 1
∴ f (x) is not continuous at x = 0, 10. Given, x + y =4
2 2
⇒ y = 4 − x2
because LHL does not exist.
or f (x) = 4 − x2
Hence, option (c) is correct.
 {1 + |sin x|} a/|sin x | , π/6 < x < 0
6. For f (x) to be continuous, we must have 
11. f (x) =  b , x=0
f (0) = lim f (x) 
x→ 0
 e tan 2x /tan 3 x
, 0 < x < π /6
log (1 + ax) − log (1 − bx)
= lim
x→ 0 x Since, f (x) is continuous at x = 0.
rankershalt.com

200 Limit, Continuity and Differentiability

∴ RHL (at x = 0) = LHL (at x = 0) = f (0)  x  1 + x 


1/ x  − log  
⇒ lim e tan 2 h/tan 3 h
= lim {1 + |sin h |} a /|sin h|
=b 1 + x (1 + x)( 2 + x)  2 + x 
h→ 0 h→ 0 ∴ f ′ (x) =  
2 + x x2 
⇒ e 23
/
=e =b
a
 
 
∴ a = 2 /3
2 1  
2 
⇒ f ′ (1) =  − log   
and b = e 23
/
3 6  3 
12. Since, f (x) is continuous at x = 0. and f (−1) = − a + b = − a [from Eq. (i)]
2 1  2 
∴ f (0) = LHL ∴ − a =  − log   
3 6  3 
1 − cos 4h
⇒ a = lim 2   2 1
h→ 0 h2 3  log  3 − 6 x, x ≤ 0

2 sin 2 2h 4 Thus, f (x) =  1/ x
⇒ a = lim ×   1 + x
h→ 0 h2 4   , x>0
  2 + x
⇒ a =8
Now, to check continuity of f (x) (at x = 0).
13. Since, f (x) is continuous for 0 ≤ x ≤ π 1/ x
 π  π  1 + x
∴ RHL at x =  = LHL at x =  RHL = lim   =0
 4  4 x → 0  2 + x

 π π  π π 2   2 1 
⇒ 2 ⋅ cot + b =  + a 2 ⋅ sin  ∴ LHL = lim
 4 4  4 4 log  3 − 6  x = 0
x→ 0 3  
π π π
⇒ + b= + a ⇒ a−b= …(i) Hence, f (x) is continuous for all x.
2 4 4
 π  π sin (a + 1) x + sin x
Also, RHL at x =  = LHL at x =   , x<0
 2  2  x
15. Given, f (x) =  c , x=0
 2π π  π π   (x + bx2)1/ 2 − x1/ 2
⇒  a cos − b sin  = 2 ⋅ ⋅ cot + b
 2 2  2 2   , x>0
 bx3/ 2
⇒ −a−b=b is continuous at x = 0.
⇒ a + 2b = 0 …(ii) ⇒ (LHL at x = 0) = (RHL at x = 0) = f (0)
On solving Eqs. (i) and (ii), we get  sin (a + 1) x sin x 
π π ⇒ lim  +
a= and b = − x→ 0  x x 
6 12
(1 + bx)1/ 2 − 1
14. Let g (x) = ax + b be a polynomial of degree one. = lim =c
x→ 0 bx
 ax + b, x≤0 bx 1
 1/ x ⇒ (a + 1) + 1 = lim ⋅ =c
⇒ f (x) =  1 + x x→ 0 bx 1 + bx + 1
 2 + x , x > 0
 ⇒ a+2=
1
=c
Since, f (x) is continuous and f ′ (1) = f (−1) 2
3 1
∴ (LHL at x = 0) = (RHL at x = 0) ∴ a =− ,c=
2 2
1/ x
 x + 1 and b ∈R
⇒ lim (ax + b) = lim  
x→ 0 x→ 0  x + 2 2
16. (i) Given, f1 : R → R and f1 (x) = sin ( 1 − e−x )
⇒ b =0 …(i)
∴ f1 (x) is continuous at x = 0
Also, f ′ (1) = f (−1)
2 1 2
Now, f1 ′ (x) = cos 1 − e− x . (2xe− x )
1/ x
 1 + x
⇒ f (x) =   ,x>0 2 1− e −x 2
 2 + x
1 At x = 0
⇒ log f (x) = [log (1 + x) − log (2 + x)]
x f1 ′ (x) does not exists.
On differentiating both sides, we get ∴ f1 (x) is not differential at x = 0
 1 1    1 + x  Hence, option (2) for P.
x −  − 1 log    |sin x|
f ′ (x) 1 + x 2 + x    2 + x  , if x ≠ 0
= (ii) Given, f2 (x) =  tan −1 x
f (x) x2  1, if x = 0
rankershalt.com

Limit, Continuity and Differentiability 201

 − sin x
 tan −1 x
x< 0 Topic 5 Continuity in a Domain

f2 (x) = 
sin x f ( x)
⇒ −1
x> 0 1. Given ∫ 4t3 dt = (x − 2) g (x)
tan x 6
 f ( x)


1 x=0


g (x) = 6
4t3 dt
[provided x ≠ 2]
 (x − 2)
Clearly, f2 (x) is not continuous at x = 0. f ( x)

∴ Option (1)for Q.

So, lim g (x) = lim 6
4t3 dt
x→ 2 x→ 2 x−2
(iii) Given, f3 (x) = [sin (log e (x + 2))] , where [ ] is G.I.F.
 0 
and f3 : (−1, eπ / 2 − 2) → R Q 0 form as x → 2 ⇒ f (2) = 6
It is given − 1 < x < eπ / 2 − 2
4( f (x))3 f ′ (x)
⇒ − 1 + 2 < x + 2 < eπ/ 2 − 2 + 2 lim g (x) = lim
x→ 2 x→ 2 1
⇒ 1 < x + 2 < eπ/ 2
⇒ log e 1 < log e (x + 2) < log e eπ / 2  d φ 2( x ) 
π
Q dx ∫φ 1 ( x)
f (t ) dt = f (φ 2(x)), φ 2′ (x) − f (φ1 (x)) ⋅ φ1′ (x)

⇒ 0 < log e (x + 2) <
2 On applying limit, we get
π 1
⇒ sin 0 < sin log e (x + 2) < sin lim g (x) = 4( f (2))3 f ′ (2) = 4 × (6)3 ,
2 x→ 2 48
⇒ 0 < sin log e (x + 2) < 1
∴ [sin log e (x + 2)] = 0  1
Q f (2) = 6 and f ′ (2) = 48 
∴ f3 (x) = 0, f ′3 (x) = f3 ′ ′ (x) = 0
4 × 216
It is differentiable and continuous at x = 0. = = 18
∴Option (4) for R 48
 2 2. Given function
x sin   , if x ≠ 0
1
(iv) Given, f4 (x) =   x sin( p + 1)x + sin x
 0, if x = 0  , x<0
 x
 1 f (x) =  q , x =0
Now, lim f4 (x) = lim x2 sin   = 0
x→ 0 x→ 0  x 
 x+x − x
2
, x>0
 1  1  x3 / 2
f4′ (x) = 2x sin   − cos  
 x  x
is continuous at x =`0, then
f (0 + h ) − f (0)
For x = 0, f4′ (x) = lim f (0) = lim f (x) = lim f (x) …(i)
h→ 0 h x→ 0 − x→ 0 +

 1 sin( p + 1)x + sin x


h 2 sin   − 0 lim f (x) = lim
 h x→ 0 − x→ 0 − x
⇒ f4′ (x) = lim
h→ 0 h  sin(ax) 
= p + 1 + 1 = p + 2 Q lim = a
 1  x→ 0 x 
⇒ f4′ (x) = lim h sin   = 0
h→ 0  h x + x2 − x
and lim f (x) = lim
  1  − cos  1  , x ≠ 0 x→ 0 + x3/ 2
x→ 0 +
f4 ′ (x) =     
2x sin
Thus,  x  x x [(1 + x)1/ 2 − 1]
 0, x=0 = lim
+
x→ 0 x x
  1  − cos  1    1 1  
Again, lim f ′ (x) = lim  2x sin    
x→ 0 x→ 0   x  x    − 1 
1
1 + x + 2  2 
x2 + .... − 1
does not exists.  2 2! 
 
 
Since, lim cos   does not exists.
1
= lim
x→ 0  x x→ 0 + x
Hence, f ′ (x) is not continuous at x = 0. [Q (1 + x)n
∴Option (3) for S.
n (n − 1) 2 n (n − 1(n − 2)) 3
= 1 + nx + x + x + ... ,|x|< 1]
1 ⋅2 1 ⋅2 ⋅3
rankershalt.com

202 Limit, Continuity and Differentiability

 1 1   − x − 1 , − 1 ≤ x < 0
  − 1   x,
1 2  2  1 0 ≤ x<1
= lim  + x + ... = 
x→ 0 +  2 2!  2 =  2x, 1 ≤ x<2
 
   x + 2, 2 ≤ x < 3

From Eq. (i), we get
 6, x=3
1 1
f (0) = q = and lim f (x) = p + 2 =
2 x→ 0 − 2 [Q if n ≤ x < n + 1, ∀ n ∈ Integer, [x] = n]
⇒ p=−
3 Q lim f (x) = − 1 ≠ f (0) [Q f (0) = 0]
x→ 0 −
2
Q lim f (x) = 1 ≠ f (1) [Q f (1) = 2]
 3 1 x→1 −
So, ( p, q ) =  − , 
 2 2 Q lim f (x) = 4 = f (2) = lim f (x) = 4 [Q f (2) = 4]
x→ 2− x→ 2 +
3. Given function
and lim f (x) = 5 ≠ f (3) [Q f (3) = 6]
a|π − x| + 1, x ≤ 5 x→ 3 −
f (x) = 
 b|x − π| + 3, x > 5
∴ Function f (x) is discontinuous at points 0, 1 and 3.
and it is also given that f (x) is continuous at x = 5.
6.
Clearly, f (5) = a (5 − π ) + 1 …(i) Key Idea A function is said to be continuous if it is continuous at
each point of the domain.
lim f (x) = lim[a|π − (5 − h )| + 1]
x→5 − h→ 0
= a(5 − π ) + 1 …(ii) We have,
and lim f (x) = lim[b| (5 + h ) − π| + 3]  5 if x≤1
x→5 + h→ 0 a + bx if 1 < x < 3

= b(5 − π ) + 3 …(iii) f (x) = 
Q Function f (x) is continuous at x = 5.  b + 5x if 3 ≤ x < 5
 30 if x≥5
∴f (5) = lim f (x) = lim f (x)
x→5 + x→5 − Clearly, for f (x) to be continuous, it has to be continuous
⇒ a (5 − π ) + 1 = b(5 − π ) + 3 at x = 1, x = 3 and x = 5
⇒ (a − b)(5 − π ) = 2 [Q In rest portion it is continuous everywhere]
2 ∴ lim (a + bx) = a + b = 5 …(i)
⇒ a−b=
5−π x →1+

[Q lim f (x) = lim f (x) = f (1)]


4. Given function f (x) = [x] −  , x ∈ R
x
x→1 − x→1 +
4 
 4 + h  lim (b + 5x) = b + 25 = 30 …(ii)
Now, lim f (x) = lim  [4 + h ] −   x →5−
x→ 4 + h→ 0   4  [Q lim f (x) = lim f (x) = f (5)]
x →5− x→5 +
[Q put x = 4 + h, when x → 4+ , then h → 0]
On solving Eqs. (i) and (ii), we get b = 5 and a = 0
= lim(4 − 1) = 3
h→ 0 Now, let us check the continuity of f (x) at x = 3.
 4 − h  lim (a + bx) = a + 3b = 15
and lim f (x) = lim [4 − h ] −   Here,
x→ 4 − h→ 0   4  x →3−

[Q put x = 4 − h, when x → 4−
and lim (b + 5x) = b + 15 = 20
then h → 0] x→3 +
= lim(3 − 0) = 3
h→ 0
4  Hence, for a = 0 and b = 5, f (x) is not continuous at x = 3
and f (4) = [4] −   = 4 − 1 = 3
4  ∴f (x) cannot be continuous for any values of a and b.
Q lim f (x) = f (4) = lim f (x) = 3 1
7. Given, f (x) = x − 1 for 0 ≤ x ≤ π
x→ 4 − x→ 4 + 2
So, function f (x) is continuous at x = 4. − 1 , 0 ≤ x < 2
∴ [ f (x)] = 
5. Given function f : [−1, 3] → R is defined as  0, 2 ≤ x ≤ π
|x| + [x], −1 ≤ x < 1 tan (−1), 0 ≤ x < 2
 ⇒ tan [ f (x)] = 
f (x) =  x + |x|, 1 ≤ x < 2  tan 0, 2≤x≤π
 x + [x ], 2 ≤ x ≤ 3
 ∴ lim tan [ f (x)] = − tan 1
x→ 2−
rankershalt.com

Limit, Continuity and Differentiability 203

and lim tan [ f (x)] = 0  π 


x→ 2 + 10. f (x) = [x] sin  
 [x + 1]
So, tan f (x) is not continuous at x = 2.
1 x−2 1 2 We know that, [x] is continuous on R ~ I, where I
Now, f (x) = x − 1 ⇒ f (x) = ⇒ =  π 
2 2 f (x) x − 2 denotes the set of integers and sin   is
Clearly, 1 /f (x) is not continuous at x = 2 .  [x + 1]
discontinuous for [x + 1] = 0.
 1 
So, tan [ f (x)] and   are both discontinuous at x = 2 . ⇒ 0 ≤ x+ 1 <1 ⇒ −1 ≤ x<0
 f (x) 
Thus, the function is defined in the interval.
π
8. The function f (x) = tan x is not defined at x = , so  x2
2  , 0 ≤ x<1
f (x) is not continuous on (0, π ). 11. Given, f (x) =  2 …(i)
1 2x2 − 3x + 3 , 1 ≤ x≤2
(b) Since, g (x) = x sin is continuous on (0, π ) and the  2
x
integral function of a continuous function is Clearly, RHL (at x = 1) = 1 / 2 and LHL (at x = 1) = 1 / 2
continuous, Also, f (x) = 1 / 2
x  1
∴ f (x) = ∫ t sin  dt is continuous on (0, π ). ∴ f (x) is continuous for all x ∈[0, 2].
0  t
3π On differentiating Eq. (i), we get

 1, 0<x≤  x , 0 ≤ x<1
(c) Also, f (x) =  4 f ′ (x) =  …(ii)
 2x 3π
2 sin   , <x<π 4x − 3 , 1 ≤ x ≤ 2
  9 4 Clearly, RHL (at x = 1) for f ′ (x) = 1
We have, lim f (x) = 1 and LHL (at x = 1) for f ′ (x) = 1
3π−
x→ Also, f (1) = 1
4
 2x Thus, f ′ (x) is continuous for all x ∈[0, 2].
lim f (x) = lim 2 sin   = 1
3π + x→
3π  9 Again, differentiating Eq. (ii), we get
x→
1 , 0 ≤ x < 1
4 4

So, f (x) is continuous at x = 3π / 4. f ′ ′ (x) = 


4 , 1 ≤ x ≤ 2
⇒ f (x) is continuous at all other points.
π  π π Clearly, RHL (at x = 1) ≠ LHL (at x = 1)
(d) Finally, f (x) = sin (x + π ) ⇒ f   = − Thus, f ′ ′ (x) is not continuous at x = 1.
2  2 2
or f ′ ′ (x) is continuous for all x ∈ [0, 2] − {1}.
π  π  3π  π
lim f (x) = lim f  − h = lim sin  − h =
π
− h→ 0 2  h→ 0 2  2  2
x→  
 2 Topic 6 Continuity for Composition and
π  Function
and lim f (x) = lim f  + h 1 1 1 1
x→ ( π / 2)
+ h→ 0 2  1. Given, f (x) = x cos , x ≥ 1 ⇒ f ′ (x) = sin + cos
x x x x
π  3π  π 1  1
sin  = lim
+ h = ⇒ f ′ ′ (x) = − cos  
2  2  2
h→ 0
x3  x
So, f (x) is not continuous at x = π / 2.
Now, lim f ′ (x) = 0 + 1 = 1 ⇒ Option (b) is correct.
x→ ∞
9. We have, for −1 < x < 1 1
Now, x ∈ [1 , ∞ ) ⇒ ∈ (0 , 1] ⇒ f ′ ′ (x) < 0
⇒ 0 ≤ x sin π x ≤ 1 / 2 x
∴ [x sin πx] = 0 Option (d) is correct.
As f ′ (1) = sin 1 + cos 1 > 1
Also, x sin πx becomes negative and numerically less
than 1 when x is slightly greater than 1 and so by f ′ (x) is strictly decreasing and lim f ′ (x) = 1
x→ ∞
definition of [x].
So, graph of f ′ (x) is shown as below.
f (x) = [x sin π x] = − 1, when 1 < x < 1 + h
Thus, f (x) is constant and equal to 0 in the closed Y
interval [–1, 1] and so f (x) is continuous and
differentiable in the open interval (–1, 1). (1, sin1+cos1)

At x = 1, f (x) is discontinuous, since lim (1 − h ) = 0


h→ 0 1
and lim (1 + h ) = − 1 X
h→ 0 0 1
∴ f (x) is not differentiable at x = 1.
Hence, (a), (b) and (d) are correct answers.
rankershalt.com

204 Limit, Continuity and Differentiability

Now, in [x , x + 2], x ∈ [1 , ∞ ), f (x) is continuous and Now, RHL (at x = 2) = 2 and LHL (at x = 2) = 0
differentiable so by LMVT, Also, RHL (at x = 1) = 1 and LHL (at x = 1) = 3
f (x + 2) − f (x) Therefore, f (x) is discontinuous at x = 1, 2
f ′ (x) =
2 ∴ f [ f (x)] is discontinuous at x = {1, 2}.
As, f ′ (x) > 1
For all x ∈ [1 , ∞ )
5. Since, f (x) is continuous at x = 0.
f (x + 2) − f (x) ⇒ lim f (x) = f (0)
⇒ > 1 ⇒ f (x + 2) − f (x) > 2 x→ 0
2 ⇒ f (0 ) = f (0− ) = f (0) = 0
+
…(i)
For all x ∈ [1 , ∞ ) To show, continuous at x = k
 f (x) + 1 , if f (x) < 0 RHL = lim f (k + h ) = lim [ f (k) + f (h )] = f (k) + f (0+ )
2. gof (x) =  h→ 0 h→ 0
{ f (x ) − 1 } 2
+ b , if f (x) ≥ 0
+ + x<−a = f (k) + f (0)
 x a 1 , if
 LHL = lim f (k − h ) = lim [ f (k) + f (− h )]
=  (x + a − 1) + b, 2
if − a ≤ x < 0 h→ 0 h→ 0
(| x − 1| − 1)2 + b, if x ≥ 0 = f (k) + f (0− ) = f (k) + f (0)

As gof (x) is continuous at x = − a ∴ lim f (x) = f (k)
x→ k
gof (− a ) = gof (− a + ) = gof (− a − ) ⇒ f (x) is continuous for all x ∈ R.
⇒ 1 + b =1 + b =1 ⇒ b =0
Also, gof (x) is continuous at x = 0 Topic 7 Differentiability at a Point
⇒ gof (0) = gof (0+ ) = gof (0− ) 1. Given function, g (x) = | f (x)|
⇒ b = b = (a − 1) + b ⇒ a = 1
2
where f : R → R be differentiable at c ∈ R and f (c) = 0,
 x + 2, if x < −1 then for function ‘g’ at x = c
 g (c + h ) − g (c)
Hence, gof (x) =  x2, if − 1 ≤ x < 0 g′ (c) = lim [where h > 0]
(| x − 1| − 1)2, if x ≥ 0 h→ 0 h

| f (c + h )| − | f (c)| | f (c + h )|
In the neighbourhood of x = 0 , gof (x) = x2, which is = lim = lim
h→ 0 h h → 0 h
differentiable at x = 0.
[as f (c) = 0 (given)]
3. As, f (x) is continuous and g (x) is discontinuous. f (c + h ) − f (c)
= lim [Qh > 0]
Case I g (x) is discontinuous as limit does not exist at h→ 0 h
x = k. f (c + h ) − f (c)
= lim
∴ φ (x) = f (x) + g (x) h→ 0 h
⇒ lim φ (x) = lim { f (x) + g (x)} = does not exist. = f ′ (c) [Q f is differentiable at x = c]
x→ k x→ k
∴ φ (x) is discontinuous. Now, if f ′ (c) = 0, then g (x) is differentiable at x = c,
Case II g (x) is discontinuous as, lim g (x) ≠ g (k). otherwise LHD (at x = c) and RHD
x→ k (at x = c) is different.
∴ φ (x) = f (x) + g (x).
2.
⇒ lim φ (x) = lim { f (x) + g (x)} = exists and is a finite Key Idea (i) First use L’ Hopital rule
x→ k x→ k
quantity (ii) Now, use formula
φ2 ( x )
but φ (k) = f (k) + g (k) ≠ lim { f (x) + g (x)}
∫ f (t ) dt = f [ φ2( x)] ⋅ φ′2 ( x) − f [ φ1( x)] ⋅ φ′1( x)
d
x→ k
∴ φ (x) = f (x) + g (x) is discontinuous, dx φ1 ( x )

whenever g (x) is discontinuous.


f ( x)
1 + x, 0 ≤ x ≤ 2
4. Given, f (x) =  f ( x) ∫ 2tdt 0 
3 − x, 2 < x ≤ 3 2tdt
Let l = lim
x→ 2 ∫ = lim 6
(x − 2) x→ 2 (x − 2) 0 form, as f (2) = 6
1 + f (x), 0 ≤ f (x) ≤ 2 6
∴ fof (x) = f [ f (x)] = 
3 − f (x), 2 < f (x) ≤ 3 On applying the L’ Hopital rule, we get

2 f (x) f ′ (x)  d 2
φ ( x)
1 + f (x), 0 ≤ f (x) ≤ 1 1 + (3 − x), 2 < x ≤ 3

 
fof = 1 + f (x), 1 < f (x) ≤ 2 = 1 + (1 + x), 0 ≤ x ≤ 1
l = lim
x→ 2 1
Q ∫ f (t )dt = f (φ 2(x)) ⋅ φ2′ (x)
 dx φ ( x )
3 − f (x), 2 < f (x) ≤ 3 3 − (1 + x), 1 < x ≤ 2 1
  − f (φ1 (x)) ⋅ φ1′ (x)]
4 − x, 2 < x ≤ 3 So, l = 2 f (2) ⋅ f ′ (2) = 12 f ′ (2) [Q f (2) = 6]

⇒ ( fof ) (x) = 2 + x, 0 ≤ x ≤ 1 f ( x)
2tdt
2 − x, 1 < x ≤ 2

∴ lim
x→ 2 ∫ x−2
= 12 f ′ (2)
6
rankershalt.com

Limit, Continuity and Differentiability 205

3. Given function is f (x) = 15 − |x − 10|, x ∈ R and  − 1, − 2 ≤ x < 0


We have, f (x) =  2
g (x) = f ( f (x)) x − 1 , 0 ≤ x ≤ 2
= f (15 −|x − 10|) and g (x) = | f (x)| + f (|x|)
= 15 − |15 − |x − 10| − 10|  1, −2 ≤ x<0
Clearly, | f (x)| =  2
= 15 − |5 − |x − 10|| |x − 1|, 0 ≤ x ≤ 2
15 − |5 − (x − 10)| , x ≥ 10  1, −2 ≤ x<0
=
15 − |5 + (x − 10)| , x < 10 
= − (x2 − 1), 0 ≤ x < 1
15 − |15 − x| , x ≥ 10  x2 − 1, 1 ≤ x≤2
= 
 15 − | x − 5| , x < 10 and f (|x|) = |x|2 − 1, 0 ≤ |x| ≤ 2
 15 + (x − 5) = 10 + x , x<5
 15 − (x − 5) = 20 − x , 5 ≤ x < 10 [Q f (|x|) = − 1 is not possible as|x| </ 0]

= = x2 − 1 , |x| ≤ 2 [Q |x|2 = x2]
 15 + (x − 15) = x , 10 ≤ x < 15
15 − (x − 15) = 30 − x , x ≥ 15 = x2 − 1 , − 2 ≤ x ≤ 2
∴ g (x) = | f (x)| + f (|x|)
From the above definition it is clear that g (x) is not
differentiable at x = 5, 10, 15.  1 + x2 − 1, −2 ≤ x<0

4. Let us draw the graph of y = f (x), as shown below = − (x − 1) + x − 1, 0 ≤ x < 1
2 2

 x2 − 1 + x2 − 1, 1 ≤ x≤2
y= cos x
y= sin x 
1
–3p  x2 , −2 ≤ x<0
–p 
4 O p
X = 0, 0 ≤ x<1
p p/4 2 (x2 − 1), 1 ≤ x ≤ 2
2 
–1 y= min {sin x, cos x} Now, let us draw the graph of y = g (x), as shown in the
figure.
Clearly, the function f (x) = min {sin x, cos x} is not
− 3π π Y
(2,6)
differentiable at x = and [these are point of
4 4
(–2,4)
intersection of graphs of sin x and cos x in (− π , π ), on y=x2 y=0
y=2 (x2 –1)
 −3 π π 
which function has sharp edges]. So, S =  , , X′ X
 4 4 –2 –1 O 1 2
 −3 π − π 3 π π 
which is a subset of  , , , 
 4 4 4 4
5. We have, Y′
f (x) = sin|x| − |x| + 2 (x − π ) cos|x|
1
− sin x + x + 2(x − π ) cos x, if x < 0 [ Here, y = 2 (x2 − 1) or x2 = ( y + 2) represent a parabola
f (x) =  2
 sin x − x + 2(x − π ) cos x, if x ≥ 0 with vertex (0, − 2) and it open upward]
[Q sin(−θ ) = − sin θ and cos(−θ ) = cos θ ]
Note that there is a sharp edge at x = 1 only, so g (x) is
− cos x + 1 + 2 cos x − 2(x − π )sin x ; if x < 0 not differentiable at x = 1 only.
∴ f ′ (x) = 
 cos x − 1 + 2 cos x − 2(x − π ) sin x , if x > 0
Clearly, f (x) is differentiable everywhere except
possibly at x = 0 7. Key Idea This type of questions can be solved graphically.
[Q f ′ (x) exist for x < 0 and x > 0]
Here, Rf ′ (0) = lim (3 cos x − 1 − 2(x − π )sin x) Given, f : (−1, 1) → R, such that
x→ 0 +
=3 −1 −0 =2
f (x) = max {−|x|, − }
1 − x2

and Lf ′ (0) = lim (cos x + 1 − 2(x − π )sin x) On drawing the graph, we get the follwong figure.
x → 0−
=1 + 1 −0 =2 Y

Q Rf ′ (0) = Lf ′ (0) y=f(x)


O 1
So, f (x) is differentiable at all values of x. –1
–1 , –1 , 1 , –1 ,
⇒ K =φ √2 √2 √2 √2
6. Key Idea This type of problem can be solved graphically. y=–|x|
rankershalt.com

206 Limit, Continuity and Differentiability


[ Q graph of y = −| x|is For| x| ∈ (2, 4]
Y 8 − 2x, x ∈ (2, 4]
f (x) = 8 − 2|x|= 
8 + 2x, x ∈ [− 4, − 2)
X  Q 2 < |x| ≤ 4 
⇒|x| > 2 and |x| ≤ 4
 
 
and graph of y = − 1 − x2 Y
Y 5
4
X 3
y=8+2x 2 y=8–2x
1
\[Q x2 + y2 = 1 represent a complete circle]
X′ X
 1 –5 –4 –3 –2 –1 O 1 2 3 4 5
 − 1−x , −1 < x ≤ −
2
2
 1 1 Y′
⇒ f (x) =  −|x|, − <x≤
 2 2
1 Hence, the graph of y = f (x) is
 − 1 − x2 , < x<1
 2 4

2x
From the figure, it is clear that function have sharp

y=
8+
1 1
edges, at x = −

8–
, 0, 1

y=

2x
2 2
∴ Function is not differentiable at 3 points.
–4 –2 –1 1 2 4
8. Key Idea This type of problem can be solved graphically

max {|x|, x2}, From the graph it is clear that at x = − 2, − 1, 0, 1, 2 the


|x| ≤ 2
We have, f (x) =  curve has sharp edges and hence at these points f is not
 8 − 2|x|, 2 < |x| ≤ 4 differentiable.
3
Let us draw the graph of y = f (x)
9. Given,| f (x) − f ( y)|≤ 2| x − y|2 , ∀ x, y ∈ R
For| x| ≤ 2 f (x) = max{|x|, x2}
1
Let us first draw the graph of y =|x|and y = x2 as shown | f (x) − f ( y)|
⇒ ≤ 2| x − y|2
in the following figure. | x − y|
y=x2 (dividing both sides by|x − y|)
y=|x| Put x = x + h and y = x, where h is very close to zero.
2 1
f (x + h ) − f (x)
⇒ lim ≤ lim 2|(x + h ) − x|2
1 h→ 0 (x + h ) − x h→ 0
1
f (x + h ) − f (x)
–2 –1 0 1 2 ⇒ lim ≤ lim 2| h |2
–1 h→ 0 h h→ 0

f (x + h ) − f (x)
–2 ⇒ lim ≤0
h→ 0 h
[substituting limit directly on right hand
Clearly, y =|x|and y = x2 intersect at x = − 1, 0, 1
side and using lim | f (x)| = lim f (x) ]
Now, the graph of y = max {|x|, x2} for|x| ≤ 2 is x→ a x→ a

 f (x + h ) − f (x) 
⇒| f ′ (x)| ≤ 0 Q lim = f ′ (x)
 h→ 0 h 
⇒| f ′ (x)| = 0 (Q| f ′ (x)|can not be less than zero)
⇒ f ′ (x) = 0 [Q| x| = 0 ⇔ x = 0]
⇒ f (x) is a constant function.
Since, f (0) = 1, therefore f (x) is always equal to 1.
1 1
Now, ∫ ( f (x))2dx = ∫ dx = [x]10 = (1 − 0) = 1
0 0
rankershalt.com

Limit, Continuity and Differentiability 207

10. We have, Here, students generally gets confused in defining


f (x) = |x − π|( ⋅ e|x| − 1)sin|x| modulus. To check differentiable at x = 0,

 (x − π )(e − 1)sin x, f (0 + h ) − f (0)
x<0 R{ f ′ (0)} = lim
x
 h→ 0 h
f (x) = − (x − π )(e − 1)sin x, 0 ≤ x < π
x

 (x − π )(ex − 1)sin x, π
 x≥π h 2 cos −0
h π
= lim = lim h ⋅ cos =0
We check the differentiability at x = 0 and π. h→ 0 h h→ 0 h
We have, π
(x − π ) (e−x − 1) cos x + (e−x − 1) sin x h 2 cos  −  − 0
f (0 − h ) − f (0)  h
 + (x − π ) sin xe−x (−1), x < 0 L {f ′ (0)} = lim = lim =0
− [(x − π )(ex − 1) cos x + (ex − 1) sin x
h→ 0 −h h→ 0 −h
f ′ (x) = 
+ (x − π ) sin xex ],0 < x < π So, f (x) is differentiable at x = 0.

 (x − π )(ex
− 1 ) cos x + (ex − 1) sin x To check differentiability at x = 2,
 + (x − π ) sin xex , x > π
f (2 + h ) − f (2)
Clearly, R{ f ′ (2)} = lim
h→ 0 h
lim f ′ (x) = 0 = lim f ′ (x)
x→ 0 − x→ 0 +  π   π 
(2 + h )2 cos   −0 (2 + h )2 ⋅ cos  
 2 + h  2 + h
and lim f ′ (x) = 0 = lim f ′ (x) = lim = lim
x→ π − x→ π + h→ 0 h h→ 0 h
∴ f is differentiable at x = 0 and x = π π π 
(2 + h )2 ⋅ sin  − 
Hence, f is differentiable for all x.  2 2 + h
= lim
h→ 0 h
11. We have, f (x) = log 2 − sin x and g (x) = f ( f (x)), x ∈ R
 πh 
Note that, for x → 0, log 2 > sin x (2 + h )2 ⋅ sin  
 2 (2 + h ) π
∴ f (x) = log 2 − sin x = lim ⋅ =π
h→ 0 π 2 (2 + h)
⇒ g (x) = log 2 − sin ( f (x)) h⋅
2 (2 + h )
= log 2 − sin (log 2 − sin x)
f (2 − h ) − f (2)
Clearly, g (x) is differentiable at x = 0 as sin x is and L { f ′ (2)} = lim
differentiable. h→ 0 −h
Now, g′ (x) = − cos (log 2 − sin x) (− cos x) π π
(2 − h )2 ⋅ cos − 22 ⋅ cos
= cos x ⋅ cos (log 2 − sin x) 2−h 2
= lim
⇒ g′ (0) = 1 ⋅ cos (log 2) h→ 0 −h
12. Given, f (0) = 2 = g (1), g(0) = 0 and f (1) = 6  π 
(2 − h )2 −  − cos  −0
f and g are differentiable in (0, 1).  2 − h
= lim
Let h (x) = f (x) − 2 g (x) …(i)
h→ 0 −h
π π 
h (0) = f (0) − 2 g (0) = 2 − 0 = 2 − (2 − h )2 ⋅ sin  − 
 2 2 − h
and h (1) = f (1) − 2 g (1) = 6 − 2(2) = 2 = lim
h→ 0 h
h (0) = h (1) = 2
 πh 
Hence, using Rolle’s theorem, (2 − h ) ⋅ sin  −
2

 2 (2 − h ) −π
h′ (c) = 0, such that c ∈ (0, 1) = lim × =−π
h→ 0 −π 2 (2 − h)

Differentiating Eq. (i) at c , we get 2 (2 − h )
⇒ f ′ (c) − 2 g′ (c) = 0 Thus, f (x) is differentiable at x = 0 but not at x = 2.
⇒ f ′ (c) = 2 g′ (c)
(x − 1)n
13. PLAN To check differentiability at a point we use RHD and LHD at
14. Given, g (x) = ; 0 < x < 2, m ≠ 0, n are
log cosm (x − 1)
a point and if RHD = LHD, then f( x ) is differentiable at the
x − 1 , x ≥ 1
point. integers and| x − 1| = 
Description of Situation 1 − x, x < 1
f (x + h ) − f (x) The left hand derivative of|x − 1|at x = 1 is p = − 1.
As, R{ f ′ (x)} = lim
h→ 0 h Also, lim g (x) = p = − 1
f (x − h ) − f (x) x →1+
and L{ f ′ (x)} = lim (1 + h − 1)n
h→ 0 −h ⇒ lim = −1
h→ 0 log cosm (1 + h − 1)
rankershalt.com

208 Limit, Continuity and Differentiability

hn ∴ Domain of f ′ (x) ∈ R − { −1, 1}


⇒ lim = −1
h→ 0 log cosm h sin h − h
18. RHD of sin (| x|) − | x| = hlim
→0 = 1 −1 = 0
hn h
⇒ lim = −1 [Q f (0) = 0]
h → 0 m log cos h
LHD of sin (| x|) − | x|
n ⋅ hn − 1
⇒ lim = −1 sin| − h | − | − h | sin h − h
h→ 0 1 = lim = =0
m (− sin h ) h→ 0 −h −h
cos h
[using L’Hospital’s rule] Therefore, (d) is the answer.
n−2
 n h  n hn − 2 19. Given, f (x) = [x] sin π x
⇒ lim  −  ⋅ = − 1 ⇒   lim =1
h → 0  m  tan h   m h → 0  tan h  If x is just less than k, [x] = k − 1
   
 h   h  ∴ f (x) = (k − 1) sin π x.
n (k − 1) sin π x − k sin π k
⇒ n = 2 and =1 ⇒ m = n =2 LHD of f (x) = lim
m x→ k x−k
(k − 1) sin π x
15. Given, f (1) = f   = f   = K = lim f   = 0
1 1 1
= lim ,
 2  3 n→ ∞  n x→ k x−k
 1 (k − 1) sin π (k − h )
as f   = 0; n ∈ integers and n ≥ 1. = lim
 n h→ 0 −h
 1 [where x = k − h ]
⇒ lim f   = 0 ⇒ f (0) = 0
n→ ∞  n (k − 1) (−1)k − 1 ⋅ sin h π
= lim = (−1) (k − 1) π
k
Since, there are infinitely many points in h→ 0 −h
neighbourhood of x = 0.
∴ f (x) = 0 20. Given, f (x) = max { x, x3 } considering the graph
⇒ f ′ (x) = 0 separately, y = x3 and y = x
⇒ f ′ (0) = 0 NOTE y = x 3 is odd order parabola and y = x is always
intersect at (1, 1) and ( −1, − 1).
Hence, f (0) = f ′ (0) = 0
x in (− ∞ , − 1]
 3 in (−1, 0]
16. Using graphical transformation. Now, f (x) = xx in (0, 1]
As, we know that, the function is not differentiable at x3 in (1, ∞ )

sharp edges. in (− ∞ , − 1]
1
 2 in (−1, 0]
Y Y ⇒ f ′ (x) = 3x
1 in (0, 1]
y = |x| – 1 3x2 in (1, ∞ )

O Y
X X y =x
O 1 –1 1 A (1, 1)
–1 –1
y =x 3
(i) y = x – 1 X′ X
(ii) y = |x| – 1 O
(–1, –1) B
Y
Y′

The point of consideration are


X
–1 O 1 f ′ (−1− ) = 1 and f ' (−1+ ) = 3

f ′ (−0 ) = 0 and f ′ (0+ ) = 1
f ′ (1− ) = 1 and f ' (1+ ) = 3
(iii) y = ||x| – 1|
Hence, f is not differentiable at −1, 0, 1.
In function, 21. Let h (x) = | x|, then g (x) = | f (x)| = h { f (x)}
y = || x| − 1|we have 3 sharp edges at x = − 1, 0, 1. Since, composition of two continuous functions is
Hence, f (x) is not differentiable at {0, ± 1}. continuous, g is continuous if f is continuous. So, answer
1 is (c).
2 (– x − 1), if x < − 1 (a) Let f (x) = x ⇒ g (x) = | x|
 −1
17. Given, f (x) =  tan x, if − 1 ≤ x ≤ 1 Now, f (x) is an onto function. Since, co-domain of x
1 is R and range of x is R. But g (x) is into function.
 2 (x − 1), if x > 1 Since, range of g (x) is [0, ∞ ) but co-domain is given R.
 Hence, (a) is wrong.
f (x) is discontinuous at x = − 1 and x = 1.
rankershalt.com

Limit, Continuity and Differentiability 209

(b) Let f (x) = x ⇒ g (x) = | x|. Now, f (x) is one-one f (x) − f (2)
and R f ′ (2) = lim
x → 2+
function but g (x) is many-one function. Hence, (b) is x−2
wrong.
(d) Let f (x) = x ⇒ g (x) = | x|. Now, f (x) is differentiable  cos x − cos 2 
= lim (x2 − 1) (x − 1) + 
for all x ∈ R but g (x) =|x|is not differentiable at x = 0 x → 2+  x−2 
Hence, (d) is wrong.
= (22 − 1) (2 − 1) − sin 2 = 3 − sin 2
22. Function f (x) = (x2 − 1)| x2 − 3x + 2| + cos (|x|) …(i)
So, L f ′ (2) ≠ R f ′ (2), f is not differentiable at x = 2
NOTE In differentiable of| f ( x )| we have to consider critical
points for which f ( x ) = 0. Therefore, (d) is the answer.
 x
 , x≥0
=  1 x+ x
| x|is not differentiable at x = 0 x
23. Given, f (x) =
1 + | x|  , x<0
cos (− x), if x < 0
but cos| x|=  1 − x
 cos x, if x ≥ 0
 (1 + x) ⋅ 1 − x ⋅ 1
 cos x, if x < 0  , x≥0
f ′ (x) =  (1 −(1x)+⋅ 1x)− x (−1)
2
⇒ cos| x| =  ∴
cos x, if x ≥ 0
 , x<0
Therefore, it is differentiable at x = 0 .  (1 − x)2
Now, | x2 − 3x + 2| = |(x − 1) (x − 2)|  1
 ,x≥0
(x − 1) (x − 2), if x < 1
⇒ f ′ (x) =  (1 +1 x)
2

=  (x − 1) (2 − x), if 1 ≤ x < 2  ,x<0
 (1 − x)
2
(x − 1) (x − 2), if 2 ≤ x

1
Therefore, ∴ RHD at x = 0 ⇒ lim =1
 (x2 − 1) (x − 1) (x − 2) + cos x,
x→ 0 (1 + x)2
if − ∞ < x < 1
 1
f (x) = − (x2 − 1) (x − 1) (x − 2) + cos x, if 1 ≤ x < 2 and LHD at x = 0 ⇒ lim =1
 (x2 − 1) (x − 1) (x − 2) + cos x, if 2 ≤ x < ∞
x→ 0 (1 − x)2

Hence, f (x) is differentiable for all x.
Now, x = 1, 2 are critical point for differentiability.
Because f (x) is differentiable on other points in its 24. Since, f (x) is continuous and differentiable where
domain. f (0) = 1 and f′ (0) = − 1, f (x) > 0, ∀ x.
Differentiability at x = 1 Thus, f (x) is decreasing for x > 0 and concave down.
f (x) − f (1) ⇒ f ′ ′ (x) < 0
L f ′ (1) = lim
x → 1− x−1 Therefore, (a) is answer.
 cos x − cos 1  tan π [(x − π )]
= lim (x2 − 1) (x − 2) + 25. Here, f (x) =
 1 + [x]2

x→1  x −1 
Since, we know π [(x − π )] = nπ and tan nπ = 0
= 0 − sin 1 = − sin 1
cos x − cos 1 d Q 1 + [x]2 ≠ 0
[ Q lim = (cos x) at x = 1 − 0
x → 1− x−1 dx ∴ f (x) = 0, ∀ x
= − sin x at x = 1 − 0 = − sin x at x = 1 = − sin 1] Thus, f (x) is a constant function.
f (x) − f (1) ∴f ′ (x), f ′ ′ (x) ,... all exist for every x , their value
and Rf ′ (1) = lim
x→1 + x−1
being 0.
 cos x − cos 1  ⇒ f ′ (x) exists for all x.
= lim − (x2 − 1) (x − 2) + 
+
x→1  x−1  26. We have,
= 0 − sin 1 = − sin 1 [same approach] ( f (0))2 + ( f ′ (0))2 = 85
Q Lf ′ (1) = Rf ′ (1). Therefore, function is differentiable and f : R → [− 2, 2]
at x = 1. (a) Since, f is twice differentiable function, so f is
f (x) − f (2) continuous function.
Again, Lf ′ (2) = lim
x→ 2 − x−2 ∴This is true for every continuous function.
 cos x − cos 2  Hence, we can always find x ∈ (r , s), where f (x) is
= lim − (x2 − 1) (x − 1) + 
x → 2−  x−2  one-one.
= − (4 − 1) (2 − 1) − sin 2 = − 3 − sin 2 ∴This statement is true.
rankershalt.com

210 Limit, Continuity and Differentiability

(b) By L.M.V.T On integrating, we get


f (b) − f (a ) f (b) − f (a ) f (x)
f ′ (c) = ⇒| f ′ (c)| = = − x + C ⇒ f (x) = − x sin x + C sin x
b−a b−a sin x
π  π π
It is given that x = , f   = −
f (0) − f (− 4) f (0) − f (− 4) 6  6 12
⇒ | f ′ (x0 )| = =
0+4 4  π π π π
∴ f   = − sin + C sin
 6 6 6 6
Range of f is [− 2, 2]
π π 1
f (0) − f (− 4) =− =− + C
− 4 ≤ f (0) − f (−4) ≤ 4 ⇒ 0 ≤
∴ ≤1 12 12 2
4 ⇒C=0
Hence,| f ′ (x0 )| ≤ 1. ∴ f (x) = − x sin x
(a) f (x) = − x sin x
Hence, statement is true.
π π π π
f   = − sin = − false
(c) As no function is given, then we assume  4 4 4 4 2
 85 x
f (x) = 2 sin   (b) f (x) = − x sin x
 2  x3
sin x > x − , ∀ x ∈ (0, π )
 85 x 6
∴ f ′ (x) = 85 cos   x4
 2  ⇒ − x sin x < − x2 +
6
Now, ( f (0)) + ( f ′ (0)) = (2 sin 0) + ( 85 cos 0)
2 2 2 2
x4
⇒ f (x ) < − x , ∀ x ∈ (0, π )
2
( f (0))2 + ( f ′ (0))2 = 85 6
It is true
and lim f (x) does not exists.
x→ ∞ (c) f (x) = − x sin x
Hence, statement is false. f ′ (x) = − sin x − x cos x
(d) From option b,| f ′ (x0 )| ≤ 1 and x0 ∈ (− 4, 0) f ′ (x ) = 0
⇒ − sin x − x cos x = 0
∴ ( f ′ (x0 ))2 ≤ 1
Hence, g (x0 ) = ( f (x0 ))2 + ( f ′ (x0 )2 ≤ 4 + 1 tan x = − x
[Q f (x0 ) ∈ [−2, 2]] Y
⇒ g (x0 ) ≤ 5
Now, let p ∈ (−4, 0) for which g ( p) = 5
Similarly, let q be smallest positive number q ∈ (0, 4) X′ π2 π1 X
o
3π/2
such that g (q) = 5
Hence, by Rolle’s theorem is ( p, q)
Y y=–x
g′ (c) = 0 for α ∈ (−4, 4) and since g (x) is greater than
5 as we move form x = p to x = q
⇒ Their exists α ∈(0, π) for which f′(α) = 0
and f (x))2 ≤ 4 ⇒ ( f ′ (x))2 ≥ 1in ( p, q) It is true
Thus, g′ (c) = 0 (d) f(x) = − x sin x
⇒ f′ f + f′ f′ ′ = 0 f ′(x) = − sin x − x cos x
f ′ ′ (x) = − 2 cos x + x sin x
So, f (α ) + f ′ ′ (α ) = 0 and f′ (α ) ≠ 0
π π π π
Hence, statement is true. f ′ ′   = , f   = −
 2 2  2 2
f (x) sin t − f (t ) sin x
= sin 2 x π π
f ′ ′   + f   = 0
27. Given, lim ∴
t→ x t −x  2  2
Using L′ Hospital rules It is true.
f (x) cos t − f ′ (t ) sin x
lim = sin 2 x 28. As, g ( f (x)) = x
t→ x 1
Thus, g (x) is inverse of f (x).
⇒ f (x) cos x − f ′ (x) sin x = sin 2 x
⇒ g ( f (x)) = x
⇒ f ′ (x) sin x − f (x) cos x = − sin 2 x
f ′ (x) sin x − f (x) cos x ⇒ g′ ( f (x)) ⋅ f ′ (x) = 1
⇒ = −1 1
sin 2 x ∴ g′ ( f (x)) = …(i)
 f (x)  f ′ (x)
⇒ d  = −1
 sin x [where, f ′ (x) = 3x2 + 3]
rankershalt.com

Limit, Continuity and Differentiability 211

When f (x) = 2, then and g (x) = | x| f (x) + |4x − 7| f (x)


x3 + 3x + 2 = 2 = (| x| + |4x − 7|) f (x)
⇒ x=0 = (| x| + |4x − 7|) [x2 − 3]
i.e. when x = 0, then f (x) = 2 (− x − 4x − 7) (− 3), − 1 / 2 ≤ x < 0
∴ g′ ( f (x)) = 2
1
at (0, 2)  (x − 4x + 7) (− 3), 0 ≤ x<1
3x + 3 
 (x − 4x + 7) (− 2), 1≤x< 2
1 
⇒ g′ (2) = =  (x − 4x + 7) (− 1), 2 ≤x< 3
3  (x − 4x + 7) (0) , 3 ≤ x < 7 /4
∴ Option (a) is incorrect. 
Now, h ( g ( g (x))) = x  (x + 4x − 7) (0), 7 /4 ≤ x < 2
 (x + 4x − 7) (1), x=2
⇒ h ( g ( g ( f (x))) = f (x) 
⇒ h ( g (x)) = f (x) …(ii) 15x + 21, − 1 /2 ≤ x < 0
As g ( f (x)) = x 9x − 21, 0 ≤ x<1

∴ h ( g (3)) = f (3) = 33 + 3(3) + 2 = 38 6x − 14, 1≤x< 2
∴ g (x) = 
∴ Option (d) is incorrect.
3x − 7, 2 ≤x< 3
From Eq. (ii), h ( g (x)) = f (x) 0, 3 ≤ x<2
⇒ h ( g ( f (x))) = f ( f (x)) 
5x − 7, x=2
⇒ h (x) = f ( f (x)) …(iii)
Now, the graphs of f (x)and g (x)are shown below.
[using g ( f (x)) = x]
Graph for f ( x)
⇒ h′ (x) = f ′ ( f (x)) ⋅ f ′ (x) …(iv)
Y
Putting x = 1, we get
h′ (1) = f ′ ( f (1)) ⋅ f ′ (1) = (3 × 36 + 3) × (6)
1
= 111 × 6 = 666
∴ Option (b) is correct.
X′ 0 X
Putting x = 0 in Eq. (iii), we get –1/2 1 √2 √3 2
h (0) = f ( f (0)) = f (2) = 8 + 6 + 2 = 16 –1
∴ Option (c) is correct. –2
29. Here, f (x) = a cos (|x3 − x|) + b|x|sin (|x3 + x|)
–3
If x3 − x ≥ 0
⇒ cos|x3 − x| = cos (x3 − x) Y′
x3 − x ≤ 0 Clearly, f (x) is discontinuous at 4 points.
⇒ cos|x3 − x| = cos (x3 − x) ∴ Option (b) is correct.
∴ cos (|x3 − x|) = cos (x3 − x), ∀ x ∈ R …(i) Graph for g ( x)
Again, if x3 + x ≥ 0 Y
⇒ |x|sin (|x3 + x|) = x sin (x3 + x) 21
x3 + x ≤ 0 27/2
⇒ |x|sin (|x3 + x|) = − x sin { − (x3 + x )}
3
∴ |x|sin (|x3 + x|) = x sin (x3 + x), ∀ x ∈ R …(ii)
⇒ f (x) = a cos (|x − x|) + b|x|sin (|x + x|)
3 3
X′ X
∴ f (x) = a cos (x3 − x) + bx sin (x3 + x) 0
…(iii) –1/2 3 √ 3–71 √2 √3 2
which is clearly sum and composition of differential 3 √ 2–7
functions.
Hence, f (x) is always continuous and differentiable. –8

30. Here, –12


− 3 , − 1 / 2 ≤ x < 1
− 2 , 1 ≤ x < 2 –21

f (x) = [x − 3] = [x ] − 3 = − 1,
2 2
2 ≤x< 3 Y′
 0, 3 ≤ x<2 Clearly, g (x) is not differentiable at 4 points, when
 x ∈ (− 1 / 2, 2).
 1, x=2
∴ Option (c) is correct.
rankershalt.com

212 Limit, Continuity and Differentiability

 g (x), x > 0 Given that, f : [a , b] → [1, ∞ )



31. Here, f (x) =  0, x=0 
− g (x), x < 0  0 , x<a

 x
 g′ (x), x ≥ 0 an g (x) = ∫ f (t )dt , a ≤ x ≤ b
f ′ (x) =  a
 b
− g′ (x), x < 0 ∫ f (t )dt , x > b
 a
∴ Option (a) is correct.
 ex , x ≥ 0 Now, g (a − ) = 0 = g (a + ) = g (a )
x
(b) h (x) = e =  −x x
e , x < 0 [as g (a + ) = lim ∫a f (t )dt = 0
+
x→ a
 ex , x≥0
⇒ h′ (x) =  − x a
− e , x < 0 and g (a ) = ∫ f (t )dt = 0]
a
⇒ h′ (0+ ) = 1and h′ (0− ) = − 1 b
g (b− ) = g (b+ ) = g (b) = ∫ f (t )dt
So, h (x) is not differentiable at x = 0. a

∴Option (b) is not correct. ⇒ g is continuous for all x ∈ R.


(c) ( foh )(x) = f { h (x)}, as h (x) > 0  0 , x<a
 g (ex ), x ≥ 0 
Now, g′ (x) =  f (x) , a < x < b
=  −x
 g (e ), x < 0  0 x>b
 ,
 ex g′ (ex ), x≥0 g′ (a − ) = 0
⇒ ( foh )′ (x) =  −x −x
 − e g′ (e ), x <0 but g′ (a + ) = f (a ) ≥ 1
+ −
⇒ ( foh )′ (0 ) = g′ (1), ( foh )′ (0 ) = − g′ (1) [Q range of f (x) is [1, ∞ ), ∀ x ∈ [a , b]]
So, ( foh )(x) is not differentiable at x = 0. ⇒ g is non-differentiable at x = a
∴ Option (c) is not correct. and g′ (b+ ) = 0
 e g( x ) , x ≠ 0
(d) (hof )(x) = e
f ( x)
= 0 but g′ (b− ) = f (b) ≥ 1
e = 1 , x = 0 ⇒ g is not differentiable at x = b.
g( x )
e −1  π π
Now, (hof )′ (0) = lim
h→ 0 x − x − 2 , x≤−
2
g( x )  π
e − 1 g (x) 34. f (x) =  − cos x, − < x ≤ 0
= lim ⋅ 2
h→ 0 g (x) x  x − 1, 0 < x≤1

−1 e g (x) − 0
g( x )
|x|  log x, x>1
= lim ⋅ lim ⋅ lim
h→ 0
g (x) h → 0 |x| h → 0 x π
Continuity at x = − ,
|x| 2
= 1 ⋅ g′ (0) ⋅ lim = 0, as g′ (0) = 0
h→ 0 x
 π  π π
f −  = − −  − = 0
∴ Option (d) is correct.  2  2 2
32. Let F (x) = f (x) − 3 g (x)  π 
RHL = lim − cos  − + h = 0
∴ F (−1) = 3, F (0) = 3 and F (2) = 3 h→ 0  2 
So, F ′ (x) will vanish atleast twice in (−1, 0) ∪ (0, 2).  π
∴ Continuous at x =  −  .
Q F ′′ (x) > 0 or < 0, ∀ x ∈ (−1, 0) ∪ (0, 2)  2
Hence, f ′ (x) − 3 g′ (x) = 0 has exactly one solution in Continuity at x = 0
(−1, 0) and one solution in (0, 2). f (0) = − 1
33. A function f (x) is continuous at x = a, RHL = lim (0 + h ) − 1 = − 1
h→ 0
if lim f (x) = lim f (x) = f (a ). ∴ Continuous at x = 0.
− +
x→ a x→ a
Continuity at x = 1,
Also, a function f (x) is differentiable at x = a, if
f (1) = 0
f (x) − f (a ) f (x) − f (a )
lim = lim RHL = lim log (1 + h ) = 0
x→ a− x−a x→ a + x−a h→ 0
∴ Continuous at x = 1
− +
i.e. f ′ (a ) = f ′ (a )
rankershalt.com

Limit, Continuity and Differentiability 213

 π 37. From the figure,


 − 1, x≤−
2 Y
 π y = min {x, x 2}
sin x, − < x ≤ 0
f ′ (x) =  2
 1, 0 < x≤1
 1 X′ X
 , x>1 O 1
 x
Differentiable at x = 0, LHD = 0, RHD = 1
∴ Not differentiable at x = 0 Y′
Differentiable at x = 1, LHD = 1, RHD = 1 h (x) is continuous all x, but h (x) is not differentiable at
two points x = 0 and x = 1. (due to sharp edges). Also
∴ Differentiable at x = 1. h′ (x) = 1, ∀ x > 1.
3
Also, for x = − Hence, (a), (c) and (d) is correct answers.
2
 |x − 3| , x≥1
π
⇒ f (x) = − x − 38. Here, f (x) =  x2 3x 13
2  4 − 2 + 4 , x < 1
3
∴ Differentiable at x = − ∴ RHL at x = 1, lim |1 + h − 3| = 2
2 h→ 0
LHL at x = 1,
35. f (x + y) = f (x) + f ( y), as f (x) is differentiable at x = 0.
(1 − h )2 3 (1 − h ) 13 1 3 13 14 3
⇒ f ′ (0) = k …(i) lim − + = − + = − =2
h→ 0 4 2 4 4 2 4 4 2
f (x + h ) − f (x)
Now, f ′ (x) = lim ∴ f (x) is continuous at x = 1
h→ 0 h

f (x) + f (h ) − f (x)  − (x − 3), 1 ≤ x<3
= lim 
h→ 0 h Again, f (x) =  (x − 3), x≥3
f (h ) 0   x2 3x 13
= lim 0 form  − + , x<1
h→ 0 h 4 2 4
Given, f (x + y) = f (x) + f ( y), ∀ x, y 
 −1 , 1 ≤ x<3
∴ f (0) = f (0) + f (0), 
∴ f ′ (x) =  1, x≥3
when x= y=0 ⇒ f (0) = 0) x 3
2 − 2 , x<1
Using L’Hospital’s rule,
f ′ (h ) RHD at x = 1 ⇒ −1 
= lim = f ′ (0) = k …(ii) ∴ 1 3  differentiable at x = 1.
h→ 0 1 LHD at x = 1 ⇒ − = − 1
2 2 
⇒ f ′ (x) = k, integrating both sides,
RHD at x = 3 ⇒ 1 
f (x) = kx + C , as f (0) = 0 Again, not differentiable at x = 3.
LHD at x = 3 ⇒ − 1
⇒ C =0 ∴ f (x) = kx
∴ f (x) is continuous for all x ∈ R and f ′ (x) = k, i.e. 39. We know that, f (x) = 1 + |sin x|could be plotted as,
constant for all x ∈ R. (1) y = sin x …(i)
Y
Hence, (b) and (c) are correct.
1
36. Here, f (x) = min { 1, x2, x3 } which could be graphically
shown as X′ X
Y y = x3 −π O π 2π 3π y = sin x
y= x2
−1

y=1
Y′
X′ X (2) y =|sin x| …(ii)
O 1
Y
1 y=|sin x|
Y′

⇒f (x) is continuous for x ∈ R and not differentiable at X′ X


x = 1 due to sharp edge. –2 π –π O π 2π 3π

Hence, (a) and (d) are correct answers. Y′


rankershalt.com

214 Limit, Continuity and Differentiability

(3) y = 1+|sin x| …(iii) D. |x − 1| + |x + 1| = 2 in (−1, 1)


⇒ The function is continuous and differentiable in
Y (−1, 1).
–2
43. We know, [x] ∈ I , ∀ x ∈ R.
y=1 +|sin x| Therefore, sin (π [x]) = 0, ∀ x ∈ R. By theory, we know
that sin (π [x]) is differentiable everywhere, therefore
–1
(A) ↔ (p).
X′ X Again, f (x) = sin{ π (x − [x])}
–π O π 2π 3π
Now, x − [x] = { x}
Y′ then π (x − [x]) = π { x}
which is not differentiable at x ∈ I.
Clearly, y = 1 + |sin x| is continuous for all x, but not Therefore, (B) ↔ (r ) is the answer.
differentiable at infinite number of points.. 44. Given, F (x) = f (x) ⋅ g (x) ⋅ h (x)
x + y = 2 y, when y > 0 On differentiating at x = x0, we get
40. Since, x + | y| = 2 y ⇒ 
 x − y = 2 y, when y < 0 F ′ (x0 ) = f ′ (x0 ) ⋅ g (x0 ) h (x0 ) + f (x0 ) ⋅ g′ (x0 ) h (x0 )
 y = x, when y > 0 ⇒ x > 0 + f (x0 ) g (x0 )h′ (x0 ) …(i)
⇒ 
 y = x /3, when y < 0 ⇒ x < 0 where, F ′ (x0 ) = 21 F (x0 ), f ′ (x0 ) = 4 f (x0 )
which could be plotted as, g′ (x0 ) = − 7 g (x0 ) and h ′ (x0 ) = k h (x0 )
Y On substituting in Eq. (i), we get
21 F (x0 ) = 4 f (x0 ) g (x0 )h (x0 ) − 7 f (x0 ) g (x0 ) h (x0 )
y = x, x > 0 + k f (x) g (x0 )h (x0 )
45° ⇒ 21 = 4 − 7 + k , [using F (x0 ) = f (x0) g (x0 ) h (x0 )]
X′ X
O ∴ k = 24
y =x , x < 0  x
, x ≠0
45. Given, f (x) = 1 + e1/ x
3
Y′
 0, x=0
Clearly, y is continuous for all x but not differentiable at
x = 0. h
−0
dy  1, x > 0 1 + e1/ h 1
Also, = ∴ Rf ′ (0) = f ′ (0+ ) = lim = lim =0
dx 1 /3, x < 0 h→ 0 h h → 0 1 + e1/ h

Thus, f (x) is defined for all x, continuous at x = 0, −h


−0
dy 1 − 1 + e−1/ h
differentiable for all x ∈ R − {0}, = for x < 0. and Lf ′ (0) = f ′ (0 ) = lim
dx 3 h→ 0 −h
g (x) cos x − g (0) 0  1 1
41. We have, lim 0 form = lim = =1
x→ 0 sin x h→ 0 1
1 + 1/ h 1+0
g′ (x) cos x − g (x)sin x e
= lim =0
x→ 0 cos x ∴ f ′ (0+ ) = 0 and f ′ (0− ) = 1
Since, f (x) = g (x)sin x  1
(x − 1)2 sin − | x|, if x ≠ 1
f ′ (x) = g′ (x) sin x + g (x) cos x 46. Given, f (x) =  (x − 1)
and f ′ ′ (x) = g′ ′ (x) sin x + 2 g′ (x) cos x − g (x) sin x  −1 , if x = 1
⇒ f ′ ′ (0) = 0
Thus, lim [ g (x) cos x − g (0) cosec x] = 0 = f ′ ′ (0)  1
 (x − 1) sin (x − 1) − x, 0 ≤ x − {1}
2
x→ 0
⇒ Statement I is true. 
 1
Statement II f ′ (x) = g′ (x) sin x + g (x) cos x As, f (x) = (x − 1)2 sin + x, x<0
 (x − 1)
⇒ f ′ (0) = g (0) −1 x=1
 ,
Statement II is not a correct explanation of Statement I. 
42. A. x|x| is continuous, differentiable and strictly Here, f (x) is not differentiable at x = 0 due to|x|.
increasing in (−1, 1) .
Thus, f (x) is not differentiable at x = 0.
B. |x| is continuous in (−1, 1) and not differentiable at
x = 0. 47. It is always true that differential of even function is and
C. x + [x] is strictly increasing in (−1, 1) and odd function.
discontinuous at x = 0 48. Since, f (x) is differentiable at x = 0.
⇒ not differentiable at x = 0.
⇒ It is continuous at x = 0.
rankershalt.com

Limit, Continuity and Differentiability 215

i.e. lim f (x) = lim f (x) = f (0) f (x) − f (α )


x → 0+ x → 0−
⇒ = g (x)
(x − α )
eah/ 2 − 1 eah/ 2 − 1 a a
Here, lim f (x) = lim = lim ⋅ = f (x) − f (α )
x→ 0 + h→ 0 h h→ 0 h 2 2 ⇒ lim = lim g (x)
a x→ α x−α x→ α
2
 c − h −1 c ⇒ f ′ (α ) = lim g (x) ⇒ f ′ (α ) = g (α )
Also, lim f (x) = lim b sin −1   = b sin x→ α
x → 0− h→ 0  2  2
c a 1 ⇒ f (x) is differentiable at x = α.
∴ b sin −1 = =
2 2 2 Conversely, suppose f is differentiable at α, then
⇒ a =1 f (x) − f (α )
lim exists finitely.
x→ α x−α
Also, it is differentiable at x = 0
 f (x) − f (α )
R f ′ (0+ ) = L f ′ (0– )  , x≠α
Let g (x) =  x−α
eh/ 2 − 1 1  f ′ (α ) , x=α

Clearly, lim g (x) = f ′ (α )
Rf ′ (0+ ) = lim h 2 [Q a = 1] x→ α
h→ 0 h
⇒ g (x) is continuous at x = α.
2eh/ 2 − 2 − h 1 Hence, f (x) is differentiable at x = α, iff g (x) is
= lim =
h→ 0 2h 2 8 continuous at x = α.
 c − h 1 51. It is clear that the given function
b sin −1   −
 2  2 b /2 (1 − x), x<1
and Lf ′ (0– ) = lim = 
h→ 0 −h 2
c f (x) = (1 − x)(2 − x), 1 ≤ x ≤ 2
1− (3 − x), x>2
4 
b 1 continuous and differentiable at all points except
∴ =
4−c 2 8 possibly at x = 1 and 2.
Continuity at x = 1,
⇒ 64b2 = (4 − c2)
LHL = lim f (x) = lim (1 − x)
⇒ a =1 and 64b2 = (4 − c2) x → 1− x → 1−

= lim [1 − (1 − h )] = lim h = 0
h→ 0 h→ 0
49. Here, lim (n + 1) cos −1   − n
2 1
and RHL = lim f (x) = lim (1 − x) (2 − x)
n→ ∞ π  n x →1+ x →1+

2  1  1   1 = lim [1 − (1 + h )] [2 − (1 + h )]
= lim n  1 +  cos −1   − 1 = lim nf   h→ 0
n→ ∞ π  n  n  n→ ∞  n
= lim − h ⋅ (1 − h ) = 0
h→ 0
 1 2  1  1
where, f   = 1 +  cos −1   − 1 = f ′ (0) ∴ LHL = RHL = f (1) = 0
 n π  n  n
Therefore, f is continuous at x = 1
  1 
given, f ′ (0) = nlim nf   
 n 
Differentiability at x = 1,
 →∞
f (1 − h ) − f (1)
2 1 L f ′ (1) = lim
∴ lim (n + 1) cos −1 − n = f ′ (0) …(i) h→ 0 −h
n→ ∞ π n 1 − (1 − h ) − 0  h 
2 = lim = lim   = −1
where, f (x) = (1 + x) cos −1 x − 1, f (0) = 0 h→ 0 −h h→ 0  − h 
π
f (1 + h ) − f (1)
and R f ′ (1) = lim
2  −1  h→ 0 h
⇒ f ′ (x) = (1 + x) + cos −1 x
π  1 − x2  [1 − (1 + h )] [(2 − (1 + h )] − 0
 = lim
h→ 0 h
2  π 2 − h (1 − h )
⇒ f ′ (0) = − 1 +  = 1 − …(ii)
= lim = lim (h − 1) = − 1
π  2 π h→ 0 h h→ 0
∴ From Eqs. (i) and (ii), we get Since, L [ f ′ (1)] = Rf ′ (1), therefore f is differentiable at
2  1 2 x = 1.
lim (n + 1) cos −1   − n = 1 −
n→ ∞ π  n π Continuity at x = 2,
50. Since, g (x) is continuous at x = α ⇒ lim g (x) = g (α ) LHL = lim f (x) = lim (1 − x) (2 − x)
x→ α x → 2− x → 2−

and f (x) − f (α ) = g (x) (x − α ) , ∀x ∈ R [given] = lim [1 − (2 − h )] [(2 − (2 − h )]


h→ 0
rankershalt.com

216 Limit, Continuity and Differentiability

= lim (− 1 + h ) h = 0 f (x + h ) − f (x)
h→ 0 Again, f ′ (x) = lim
h→ 0 h
and RHL = lim f (x) = lim (3 − x)
x → 2+ x → 2+  2x + 2h 
f  − f (x)
 2 
= lim [3 − (2 + h )] = lim (1 − h ) = 1 = lim
h→ 0 h→ 0 h→ 0 h
f (2x) + f (2h )
− f (x)
Since, LHL ≠ RHL, therefore f is not continuous at x = 2 2
= lim
as such f cannot be differentiable at x = 2. h→ 0 h
Hence, f is continuous and differentiable at all points
1   2x  2h  
except at x = 2 . 2 f   − 1 + 2 f   − 1 − f (x)
2   2   2 
 − 1 + 1  = lim
  h→ 0 h
 xe  x x  , x>0
  1 1
[from Eq. (i)]
52. Given, f (x) = xe  x x  , x < 0
− − + 
1
[2 f (x) − 1 + 2 f (h ) − 1] − f (x)
 0 , x=0 = lim 2
 h→ 0 h
 f (x) + f (h ) − 1 − f (x)
 = lim
 −2 h→ 0 h
xe x , x > 0 f (h ) − 1
 = lim = −1 [from Eq. (ii)]
= x , x<0 h→ 0 h
 0 , x=0
 ∴ f ′ (x) = −1, ∀x ∈ R

(i) To check continuity at x = 0,
⇒ ∫ f ′ (x)dx = ∫ − 1 dx

⇒ f (x) = − x + k, where, k is a constant.


LHL (at x = 0) = lim − h = 0
h→ 0 But f (0) = 1,
h
RHL = lim =0 therefore f (0) = −0 + k
h→ 0 e2/ h
⇒ 1=k
Also, f (0) = 0
⇒ f (x) = 1 − x, ∀x ∈ R ⇒ f (2) = −1
∴ f (x) is continuous at x = 0
54. We have, f (x + y) = f (x) ⋅ f ( y), ∀ x, y ∈ R.
(ii) To check differentiability at x = 0,
f (0 − h ) − f (0) ∴ f (0) = f (0) ⋅ f (0) ⇒ f (0) { f (0) − 1} = 0
L f ′ (0) = lim ⇒ f (0) = 1 [Q f (0) ≠ 0]
h→ 0 −h
f (0 + h ) − f (0)
(0 − h ) − 0 Since, f ′ (0) = 2 ⇒ lim =2
= lim =1 h→ 0 h
h→ 0 −h
f (h ) − 1
f (0 + h ) − f (0) ⇒ lim =2 [Q f (0) = 1] …(i)
R f ′ (0) = lim h→ 0 h
h→ 0 h f (x + h ) − f (x)
he−2 / h − 0 Also, f ′ (x) = lim
= lim =0 h→ 0 h
h→ 0 h f (x) ⋅ f (h ) − f (x)
= lim ,
∴ f (x) is not differentiable at x = 0. h→ 0 h
x + y f (x) + f ( y)
53. Given, f   = , ∀x, y ∈ R [using, f (x + y) = f (x) ⋅ f ( y)]
 2  2  f (h ) − 1 
= f (x) lim 
On putting y = 0, we get h→ 0 h 
 x f (x) + f (0) 1 ∴ f ′ (x) = 2 f (x) [from Eq. (i)]
f   = = [1 + f (x)] [Q f (0) = 1]
 2 2 2 f ′ (x)
⇒ =2
 x f (x)
⇒ 2 f   = f (x) + 1
 2
On integrating both sides between 0 to x, we get
 x x f ′ (x)
⇒ f (x) = 2 f   − 1, ∀ x, y ∈ R
 2
…(i)
∫ 0 f (x) dx = 2x
Since, f ′ (0) = −1, we get ⇒ log e | f (x)| − log e | f (0)| = 2x
f (0 + h ) − f (0) ⇒ log e| f (x)| = 2x [Q f (0) = 1]
lim = −1
h→ 0 h
⇒ log e | f (0)| = 0
f (h ) − 1
⇒ lim = −1 …(ii) ⇒ f (x) = e2 x
h→ 0 h
rankershalt.com

Limit, Continuity and Differentiability 217

55. y = [x] + |1 − x|, − 1 ≤ x ≤ 3  −1 , −2 ≤ x ≤ 0


58. Given that, f (x) = 
−1 + 1 − x, −1 ≤ x < 0 (x − 1), 0 < x ≤ 2
 0 + 1 − x, 0 ≤ x < 1 Since, x ∈ [−2, 2]. Therefore,|x|∈ [0, 2]

⇒ y= ⇒ f (|x|) = |x| − 1, ∀ x ∈ [−2, 2]
 1 + x − 1, 1 ≤ x < 2
 2 + x − 1, 2 ≤ x ≤ 3  x − 1, 0 ≤ x ≤ 2
⇒ f (|x|) = 
− x − 1 , − 2 ≤ x ≤ 0
 − x, −1 ≤ x < 0
 1 − x, 0 ≤ x < 1  1, −2 ≤ x < 0
 
⇒ y= Also, | f (x)| = 1 − x, 0 ≤ x < 1
 + x, 1 ≤ x<2
x − 1 , 1 ≤ x ≤ 2
 x + 1, 2 ≤ x < 3 

which could be shown as, Also, g (x) = f (|x|) + | f (x)|


 − x − 1 + 1 , −2 ≤ x ≤ 0
Y 
= x − 1 + 1 − x, 0 ≤ x < 1
4 x
1+ x − 1 + x − 1 , 1 ≤ x ≤ 2
3

 − x, −2 ≤ x ≤ 0
2 x 
1
g (x) =  0, 0 ≤ x<1
– 1 – 2(x − 1), 1 ≤ x ≤ 2
x x 
X′ –1 O X  − 1 , −2 ≤ x ≤ 0
1 2 3

Y′ ∴ g ′ (x) =  0, 0 ≤ x<1
 2, 1 ≤ x≤2

Clearly, from above figure, y is not continuous and not
differentiable at x = {0, 1, 2} . ∴ RHD (at x = 1) = 2, LHD (at x = 1) = 0
56. Since, | f ( y) − f (x)| ≤ (x − y)
2 3 ⇒ g (x) is not differentiable at x = 1.
Also, RHD (at x = 0) = 0, LHD at (x = 0) = − 1
| f ( y) − f (x)|2
⇒ ≤ (x − y) ⇒ g (x) is not differentiable at x = 0.
( y − x)2
Hence, g (x) is differentiable for all x ∈ (−2, 2) − {0, 1}
2
f ( y) − f (x)
⇒   ≤x− y …(i)
 y−x  59. Given, f (x) = x3 − x2 − x + 1
2
f ( y) − f (x) ⇒ f ′ (x) = 3x2 − 2x − 1 = (3x + 1) (x − 1)
⇒ lim   ≤ lim (x − y)
∴ f (x) is increasing for x ∈ (−∞ , − 1 /3) ∪ (1, ∞ )
y → x y−x  y→ x
and decreasing for x ∈ (−1 /3, 1)
⇒ | f ′ (x)|2 ≤ 0 max { f (t ); 0 ≤ t ≤ x}, 0 ≤ x ≤ 1
which is only possible, if| f ′ (x)| = 0 Also, given g (x) = 
 3 − x, 1 < x≤2
∴ f ′ (x) = 0
 f (x), 0 ≤ x ≤ 1
or f ′ (x) = Constant ⇒ g (x) = 
3 − x, 1 < x ≤ 2
57. Since, f (−x) = f (x) x3 − x2 − x + 1, 0 ≤ x ≤ 1
∴f (x) is an even function. ⇒ g (x) = 
 3 − x, 1 < x≤2
f (0 + h ) − f (0)
∴ f ′ (0) = lim At x = 1,
h→ 0 h
f (0 − h ) − f (0) RHL = lim (3 − x) = 2
= lim [Q f (− h ) = f (h )] x→1
h→ 0 −h and LHL = lim (x3 − x2 − x + 1) = 0
Since, f ′ (0) exists. x→1
∴ R f ′ (0) = L f ′ (0) ∴ It is discontinuous at x = 1.
f (h ) − f (0) f (h ) − f (0) 3x2 − 2x − 1, 0 ≤ x ≤ 1
⇒ lim = lim Also, g ′ (x) = 
h→ 0 h h→ 0 −h  −1 , 1 < x≤2
f (h ) − f (0) ⇒ +
g ′ (1 ) = − 1
⇒ 2 lim =0
h→ 0 h and g ′ (1− ) = 3 − 2 − 1 = 0
f (h ) − f (0)
⇒ lim =0 ∴ g (x) is continuous for all x ∈ (0, 2) − {1} and g (x) is
h→ 0 h
differentiable for all x ∈ (0, 2) − {1}.
∴ f ′ (0) = 0
rankershalt.com

218 Limit, Continuity and Differentiability

 x −1  3 sin x − f (x)
2x2 − 7x + 5 , when x ≠ 1 lim g (x) = lim
x→ 0 x→ 0  sin x


60. Given that, f (x) = 
1 3 cos x − f ′ (x)
− , when x = 1 = lim
 3 x→ 0 cos x
f (1 + h ) − f (1) 3 −1
RHD = lim = =2
h→ 0 h 1
 1 + h −1  1  63.
 − −   PLAN
 2(1 + h )2 − 7(1 + h ) + 5  3  (i) In these type of questions, we draw the graph of the function.
= lim
h→ 0 h (ii) The points at which the curve taken a sharp turn, are the
points of non-differentiability.
3h + 2 (1 + h )2 − 7(1 + h ) + 5 
= lim   Curve of f (x) and g (x) are
h→ 0 3 h {2 (1 + h )2 − 7 (1 + h ) + 5 }
 
 2h 2  2
= lim   =−
h→ 0  3 h (−3 h + 2 h 2 ) 9
f (1 − h ) − f (1)
LHD = lim
h→ 0 −h
 1 − h −1  1 
 − −  
 2(1 − h )2 − 7(1 − h ) + 5  3 
= lim
h→ 0 −h
−3h + 2(1 + h 2 − 2h ) − 7(1 − h ) + 5 h (x) is not differentiable at x = ± 1 and 0.
= lim As, h (x) take sharp turns at x = ± 1 and 0.
h→ 0 −3h [2(1 − h )2 − 7(1 − h ) + 5]
Hence, number of points of non-differentiability of h (x)
2h 2 2
= lim = − ∴ LHD = RHD is 3.
h→ 0 −3h (2h 2 + 3h ) 9
64. Let p (x) = ax4 + bx3 + cx2 + dx + e
2
Hence, required value of f ′ (1) = − . ⇒ p′ (x) = 4ax3 + 3bx2 + 2cx + d
9
61. Given, f (x) = x tan −1 x ∴ p′ (1) = 4a + 3b + 2c + d = 0 ... (i)
Using first principle, and p′ (2) = 32a + 12b + 4c + d = 0 ... (ii)
 f (1 + h ) − f (1)   p (x)
f ′ (1) = lim   Since, lim 1 + 2  = 2 [given]
h→ 0  h x→ 0  x 
 (1 + h ) tan −1 (1 + h ) − tan −1 (1)  ax4 + bx3 + (c + 1) x2 + dx + e
= lim   ∴ lim =2
h→ 0
 h  x→ 0 x2
 tan −1 (1 + h ) − tan −1 (1) h tan −1 (1 + h )  ⇒ c + 1 = 2, d = 0, e = 0
= lim  +  ⇒ c=1
h→ 0 h h
 
From Eqs. (i) and (ii), we get
1  h  
= lim  tan −1  −1
 + tan (1 + h ) 4a + 3b = − 2
h→ 0  h  2 + h 
and 32a + 12b = − 4
 −1  h  
 tan  2 + h   π ⇒
1
a = and b = − 1.
= lim  + 4
h→ 0  h  4
(2 + h ) ⋅ x4
 2 + h  ∴ p(x) = − x3 + x2
4
 −1  h   16
 tan   ⇒ p(2) = −8 + 4
1   2 + h  π 1 π
= lim + = + 4
h→ 0 2 + h  h  4 2 4
  ⇒ p(2) = 0
 (2 + h ) 
π
62. g (x) = ∫ 2 (f ′ (t ) cosec t − cot t cosec t f (t ))dt
Topic 8 Differentiation
x
1. We know,
 π π
∴ g (x) = f   cosec − f (x) cosec x (1 + x)n = nC 0 + nC1x + nC 2 x2 + ... + nC n xn
 2 2
On differentiating both sides w.r.t. x, we get
f (x)
⇒ g (x) = 3 − n (1 + x)n − 1 = nC1 + 2 nC 2 x + ... + n nC n xn − 1
sin x
rankershalt.com

Limit, Continuity and Differentiability 219

On multiplying both sides by x, we get On differentiating both sides w.r.t. x, we get


n x(1 + x)n − 1 = nC1x + 2nC 2x2 + ... + n nC nxn dy dy
ey +x + y=0 …(ii)
Again on differentiating both sides w.r.t. x, dx dx
we get dy  y 
⇒ =− y  …(iii)
n [(1 + x)n − 1 + (n − 1) x (1 + x)n − 2] dx  e + x
= nC1 + 22 nC 2x + ... + n 2 nC nxn − 1
Now putting x = 1 in both sides, we get Again differentiating Eq. (ii) w.r.t. ‘x’, we get
2
n
C1 + (22) nC 2 + (32) nC3 + ... + (n 2) nC n d 2y y  dy d 2y dy dy
ey + e   +x 2+ + =0 …(iv)
dx 2  dx dx dx dx
n −1 n−2
= n (2 + (n − 1) 2 )
For n = 20, we get Now, on putting x = 0 in Eq. (i), we get
ey = e1
C1 + (22) 20C 2 + (32) 20C3 + ... + (20)2
20 20
C 20
⇒ y=1
= 20(219 + (19) 218 )
On putting x = 0, y = 1 in Eq. (iii), we get
= 20 (2 + 19) 218 = 420 (218 )
dy 1 1
= A (2B ) (given) =− =−
dx e+0 e
On comparing, we get
( A , B) = (420, 18) dy 1
Now, on putting x = 0, y = 1 and = − in
dx e
 sin x − cos x  −1  tan x − 1 
2. Let f (x) = tan −1   = tan   Eq. (iv), we get
 sin x + cos x  tan x + 1
d 2y 1  1
2
 d 2y  1  1
[dividing numerator and denominator e1 + e  −  + 0 2  +  −  +  −  = 0
dx 2  e  dx   e  e
 π
by cos x > 0, x ∈ 0,  ]
 2 d 2y 1
⇒ = 2
 π  2
dx ( 0, 1) e
 tan x − tan 
= tan −1  4 
  π   dy d 2y  1 1
 1 +  tan  (tan x) So,  , 2  at (0, 1) is  − , 2
 4   dx dx   e e
  π  4. Let y = f ( f ( f (x))) + ( f (x))2
= tan −1 tan  x −  
  4 
On differentiating both sides w.r.t. x, we get
 tan A − tan B  dy
Q = tan ( A − B) = f ′ ( f ( f (x))) ⋅ f ′ ( f (x)) ⋅ f ′ (x) + 2 f (x) f ′ (x)
 1 + tan A tan B  dx
 π [by chain rule]
Since, it is given that x ∈ 0,  , so
 2 So, dy = f ′ ( f ( f (1))) ⋅ f ′ ( f (1)) ⋅ f ′ (1) + 2 f (1) f ′ (1)
π  π π dx at x=1
x − ∈− , 
4  4 4 ∴
dy
= f ′ ( f (1)) ⋅ f ′ (1) ⋅ (3) + 2(1)(3)
 π  π π dx x = 1
Also, for  x −  ∈  − ,  ,
 4  4 4 [Q f (1) = 1 and f′ (1) = 3]
Then, = f ′ (1) ⋅ (3) ⋅ (3) + 6
−1   π π = (3 × 9) + 6 = 27 + 6 = 33
f (x) = tan  tan  x −   = x −
  4 4 5. Given expression is
  π π 
2 2
−1   3 cos x + sin x    3 cot x + 1 
Q tan tan θ = θ, for θ ∈  − 2 , 2   2 y =  cot−1    =  cot−1  
    cos x − 3 sin x     cot x − 3  
x
Now, derivative of f (x) w.r.t. is [dividing each term of numerator and
2
d ( f (x)) df (x) denominator by sin x]
=2 2
d (x / 2) d (x)   π 

−1
 cot cot x + 1   π 
=  cot  6 
d  π Q cot 6 = 3 
=2 × x −  = 2   cot x − cot π  
dx  4   6 
3. 2
Key Idea Differentiating the given equation twice w.r.t. ‘x’.   π   cot A cot B + 1 
=  cot−1  cot − x   Q cot( A − B) = 
  6   cot B − cot A 
Given equation is
ey + xy = e …(i)
rankershalt.com

220 Limit, Continuity and Differentiability

 π 
2
π = 4 + e2 − e log e (1) = 4 + e2 − 0
  − x , 0<x<
= e2 + 4
 6  6
= 2
 π π  π π ⇒ y = e2 + 4 [Q y > 0]
+  − x  , <x<
 6  6 2
∴ At x = e and y = e + 4, 2

 π + θ , − π < θ < 0  dy 2e − 1 − 0 2e − 1
Q cot−1 (cot θ ) =  θ , = =
  0 <θ < π  [using Eq. (ii)]
 dx 2 e2 + 4 2 e2 + 4
 θ − π , π < θ < 2π 
 
8. We have,
 π 
2
π
  − x , 0 < x < f (x) = x3 + x2f ′ (1) + xf ′ ′ (2) + f ′ ′ ′ (3)
 6  6
⇒2y =  2
⇒ f ′ (x) = 3x2 + 2xf ′ (1) + f ′ ′ (2) … (i)
 7π − x , π < x < π ⇒ f ′ ′ (x) = 6x + 2 f ′ (1) … (ii)

 6 
 6 2 ⇒ f ′ ′ ′ (x) = 6 … (iii)
 π  π ⇒ f′ ′ ′ (3) = 6
2 − x (−1), 0 < x <
dy   6  6 Putting x = 1 in Eq. (i), we get
⇒2 =
dx   7π  π π f ′ (1) = 3 + 2 f ′ (1) + f ′ ′ (2) … (iv)
2 − x (−1), <x<
  6  6 2 and putting x = 2 in Eq. (ii), we get
 π π f ′ ′ (2) = 12 + 2 f ′ (1) …(v)
dy  x − 6 , 0 < x < 6 From Eqs. (iv) and (v), we get
⇒ =
dx x − 7π , π < x < π f ′ (1) = 3 + 2 f ′ (1) + (12 + 2 f ′ (1))
 6 6 2 ⇒ 3 f′ (1) = − 15
6. Given equation is ⇒ f′ (1) = − 5
(2x)2y = 4 ⋅ e2x − 2y ... (i) ⇒ f′ ′ (2) = 12 + 2 (− 5) = 2 [using Eq. (v)]
On applying ‘ log e ’ both sides, we get ∴ f (x) = x3 + x2f ′ (1) + xf ′ ′ (2) + f ′ ′ ′ (3)
log e (2x)2y = log e 4 + log e e2x − 2y ⇒ f (x) = x3 − 5x2 + 2x + 6
2 y log e (2x) = log e (2)2 + (2x − 2 y) ⇒ f (2) = 23 − 5(2)2 + 2(2) + 6 = 8 − 20 + 4 + 6 = − 2
[Q log e n = m log e n and log e ef ( x ) = f (x)]
m
9. We have, x = 3 tan t and y = 3 sec t
⇒ (2 log e (2x) + 2) y = 2x + 2 log e (2) dy d
x + log e 2 (3 sec t )
⇒ y= dy dt dt
Clearly, = =
1 + log e (2x) dx dx d (3 tan t )
On differentiating ‘y’ w.r.t. ‘x’, we get dt dt
2 3 sec t tan t tan t
(1 + log e (2x))1 − (x + log e 2) = = = sin t
dy
= 2x 3 sec2 t sec t
dx (1 + log e (2x))2 d 2y d  dy d  dy dt
1 and =   =  ⋅
1 + log e (2x) − 1 − log e 2 dx2 dx  dx dt  dx dx
= x
d  dy d
(1 + log e (2x))2   (sin t )
dt  dx dt cos t cos3 t
dy  x log e (2x) − log e 2 = = = 2
=
So, (1 + log e (2x))2 = dx d 3
 (3 tan t ) 3 sec t
dx  x  dt dt
π
7. We have, x log e (log e x) − x2 + y2 = 4, which can be cos3
d 2y  π 4 = 1 1
written as Now,  at t =  = =
dx2  4 3 3(2 2 ) 6 2
y2 = 4 + x2 − x log e (log e x) … (i)
 6x x  − 1  2 ⋅ (3 x ) 
3/ 2
Now, differentiating Eq. (i) w.r.t. x, we get 10. Let y = tan − 1   = tan  3/ 2 2 
dy 1 1  1 − 9x3  1 − (3x ) 
2y = 2x − x . − 1 ⋅ log e (log e x)
dx log e x x  2x 
= 2 tan − 1 (3x3/ 2) Q 2 tan − 1 x = tan − 1 
[by using product rule of derivative]  1 − x2 
1
2x − − log e (log e x) ∴
dy
= 2⋅
1 3
⋅ 3 × (x)1/ 2 =
9
⋅ x
 
dy log ex 1 + (3x )3/ 2 2
1 + 9x3
⇒   = … (ii) dx 2
 dx 2y
9
∴ g (x) =
Now, at x = e, y2 = 4 + e2 − e log e (log e e) 1 + 9x3
[using Eq. (i)]
rankershalt.com

Limit, Continuity and Differentiability 221

11. Here, g is the inverse of f (x).   x   x 1


+ 2  g    ⋅ g′   ⋅ = 0 [from Eq.(i)]
⇒ fog (x) = x   2   2 2

On differentiating w.r.t. x, we get ∴ F (x) is constant ⇒ F (10) = F (5) = 5


1 16. Let, g (x) = f (x) − x2
f ′{ g (x)} × g′ (x) = 1 ⇒ g′ (x) =
f ′ ( g (x)) ⇒ g (x) has atleast 3 real roots which are x = 1, 2, 3
1  1  [by mean value theorem]
= Q f ′ (x) =  ⇒ g ′ (x) has atleast 2 real roots in x ∈ (1, 3)
1  1 + x5 
⇒ g ′ ′ (x) has atleast 1 real roots in x ∈ (1, 3)
1 + { g (x)}5
⇒ f ′ ′ (x) – 2 . 1 = 0. for atleast 1 real root in x ∈ (1, 3)
⇒ g′ (x) = 1 + { g (x)}5 ⇒ f ′ ′ (x) = 2, for atleast one root in x ∈ (1, 3)
12. Given, y = sec (tan− 1 x ) 17. Given that, log (x + y) = 2xy …(i)
2
Let tan − 1 x = θ x ∴ At x = 0, ⇒ log ( y) = 0 ⇒ y = 1
1+ x dy
⇒ x = tan θ ∴ To find at (0, 1)
θ dx
∴ y = sec θ = 1 + x2
1
On differentiating Eq. (i) w.r.t. x, we get
On differentiating w.r.t. x, we get
dy 1 1  dy dy
= ⋅ 2x 1 +  = 2x + 2y ⋅ 1
dx 2 1 + x2 x+ y  dy dx
dy 1 dy 2 y (x + y) − 1
At x = 1, = ⇒ =
dx 2 dx 1 − 2 (x + y) x

13. Since, f (x) = e g ( x ) ⇒ e g ( x + 1)= f (x + 1) = xf (x) = xe g ( x )  dy


⇒   =1
 dx ( 0, 1)
and g (x + 1) = log x + g (x)
i.e. g (x + 1) − g (x) = log x ... (i) 18. Given, x2 + y2 = 1
1 On differentiating w.r.t. x, we get
Replacing x by x − , we get
2 2x + 2 yy′ = 0
 1  1  1 ⇒ x + yy′ = 0.
g  x +  − g  x −  = log  x −  = log (2x − 1) − log 2
 2  2  2
Again, differentiating w.r.t. x, we get
 1  1 −4
∴ g′′  x +  − g′′  x −  = ... (ii) 1 + y ′ y ′ + yy ′′ = 0
 2  2 (2x − 1)2
⇒ 1 + ( y′ )2 + yy′′ = 0
x3 sin x cos x
On substituting, x = 1, 2, 3,... , N in Eq. (ii) and adding,
we get 19. Given, f (x) = 6 −1 0
 1  1  1 1 1  p p2 p3
g′′  N +  − g′′   = − 4 1 + + + ... + 2
.
 2  2  9 25 (2N − 1) 
On differentiating w.r.t. x, we get
−1
dx 1  dy 3x2 cos x − sin x x3
14. Since, = =  sin x cos x
dy dy / dx  dx f ′ (x) = 6 −1 0 + 0 0 0
−1
d  dx d  dy dx 2 3 2
⇒   =   p p p p p p3
dy  dy dx  dx dy
d 2x  d 2y  dy − 2 dx  d 2y  dy −3 x3 sin x cos x
⇒ = −       = −  2  
dy2  dx   dx  dy
2
 dx   dx + 6 −1 0
0 0 0
15. Since, f ′′ (x) = − f (x)
d 3x2 cos x − sin x
⇒ { f ′ (x)} = − f (x) ⇒ f ′ (x) = 6 −1 0
dx
2
⇒ g′ (x) = − f (x) [Q g (x) = f ′ (x), given]…(i) p p p3
2 2
  x    x  6x − sin x − cos x
Also, F (x) =  f    +  g   

 2     2  ⇒ f ′ ′ (x) = 6 −1 0 +0+0
  x   x 1 p p2 p3
⇒ F ′ (x) = 2  f    ⋅ f ′   ⋅
  2   2 2
rankershalt.com

222 Limit, Continuity and Differentiability

6 − cos x sin x 23. Given, f (x) = | x − 2|


and f ′ ′ ′ (x) = 6 −1 0 +0+0 ∴ g (x) = f [ f (x)] =||x − 2|− 2|
p p2 p3
When, x>2
6 −1 0 g (x) = |(x − 2) − 2| = | x − 4| = x − 4
∴ f ′ ′ ′ (0) = 6 −1 0 = 0 = independent of p ∴ g′ (x) = 1 when x > 2
p p2 p3
24. Let u = sec−1  − 2  and v = 1 − x2
1
20. Since, y2 = P (x)  2x − 1
On differentiating both sides, we get Put x = cos θ
2 yy1 = P ′ (x), ∴ u = sec−1 (− sec 2 θ ) and v = sin θ
Again, differentiating, we get ⇒ u = π − 2θ [Q sec–1 (− x) = π − sec−1 x]
2 yy2 + 2 y12 = P ′ ′ (x) and v = sin θ
du
⇒ 2 y3 y2 + 2 y2y12 = y2P ′ ′ (x) ⇒ = −2

⇒ 2 y3 y2 = y2P ′ ′ (x) − 2 ( yy1 )2 dv
an d = cos θ
{ P ′ (x)}2 dθ
⇒ 2 y y2 = P (x) ⋅ P ′ ′ (x) −
3
2 du 2  du 
⇒ =− ,   = −4
Again, differentiating, we get dv cos θ  dv  θ = π /3
d 3
2 ( y y2) = P ′ (x) ⋅ P ′ ′ (x) + P (x) ⋅ P ′ ′ ′ (x) 25. Given, f (x) = log x (log x)
dx log (log x)
2P ′ (x) ⋅ P ′ ′ (x) ∴ f (x) =
− log x
2
d 3 On differentiating both sides, we get
⇒ 2 ( y y2) = P (x) ⋅ P ′ ′ ′ (x)
dx  1 1 1
(log x)  ⋅  − log (log x) ⋅
d  3 d 2y  log x x x
⇒ 2  y ⋅ 2  = P (x) ⋅ P ′ ′ ′ (x) f ′ (x) =
dx  dx  (log x)2
21. Given, xexy = y + sin 2 x  1 1 1
…(i) 1 ⋅  ⋅  − log (1) ⋅
 1 e e
On putting x = 0, we get ∴ f ′ (e) =
(1)2
0 . e0 = y + 0
1
⇒ y=0 ⇒ f ′ (e) =
e
f1 (x) f2 (x) f3 (x)
On differentiating Eq. (i) both sides w.r.t. x, we get 26. Given, F (x) = g1 (x) g2 (x) g3 (x)
 dy  dy
1 . exy + x ⋅ exy  x ⋅ + y = + 2 sin x cos x h1 (x) h2 (x) h3 (x)
 dx  dx
f1 ′ (x) f2 ′ (x) f3 ′ (x)
On putting x = 0, y = 0, we get ∴ F ′ (x) = g1 (x) g2 (x) g3 (x)
 dy  h1 (x) h2 (x) h3 (x)
e0 + 0(0 + 0) =   + 2 sin 0 cos 0
 dx ( 0, 0) f1 (x) f2 (x) f3 (x) f1 (x) f2 (x) f3 (x)
 dy  + g1 ′ (x) g2 ′ (x) g3 ′ (x) + g1 (x) g2 (x) g3 (x)
⇒  dx  =1
0, 0 h1 (x) h2 (x) h3 (x) h1 ′ (x) h2 ′ (x) h3 ′ (x)
⇒ F ′ (a ) = 0 + 0 + 0 = 0
22. Given, f (x) = x|x|
[Q fr (a ) = gr (a ) = hr (a ) ; r = 1, 2, 3]
 2 if x ≥ 0
⇒ f (x) = x , 2  2x − 1
− x , if x < 0
27. Given, y= f  2 
f (x) is not differentiable at x = 0 but all R − {0} .  x + 1
f ′ (x) = sin 2 x
f ′ (x) = 2x, x>0 and
Therefore,
 − 2x, x<0 dy  2x − 1 d  2x − 1
∴ = f′  2 ⋅  2 
⇒ f ′ ′ (x) = 2, x>0 dx  x + 1 dx  x + 1
− 2 , x<0
 2x − 1  (x2 + 1) ⋅ 2 − (2x − 1) (2x) 
Therefore, f (x) is twice differentiable for all x ∈ R − {0}. = sin 2 2  ⋅ 
 x + 1  (x2 + 1)2 
rankershalt.com

Limit, Continuity and Differentiability 223

 2x − 1 −2x2 + 2x + 2 and
dy
= n secn − 1 θ ⋅ sec θ tan θ − n cos n − 1 θ ⋅ (− sin θ )
= sin 2  2 ⋅
 x + 1 (x2 + 1)2 dθ
dx
−2 (x2 − x − 1)  2x − 1 ⇒ = tan θ (sec θ + cos θ )
= sin 2  2  dθ
(x2 + 1)2  x + 1 dy
and = n tan θ (secn θ + cos n θ )
ax2 bx c dθ
28. y= + + +1
(x − a ) (x − b) (x − c) (x − b) (x − c) (x − c) dy n (secnθ + cos n θ )
⇒ =
ax2 bx x dx sec θ + cos θ
= + +
(x − a ) (x − b) (x − c) (x − b) (x − c) (x − c)  dy
2
n 2 (secnθ + cos n θ )2
∴   =
ax 2
x  b   dx (sec θ + cos θ )2
= +  + 1
(x − a ) (x − b) (x − c) (x − c)  x − b 
n 2 {(secn θ − cos n θ )2 + 4} n 2 ( y2 + 4)
ax2 x x = =
= + ⋅ {(sec θ − cos θ )2 + 4} (x2 + 4)
(x − a ) (x − b) (x − c) (x − c) (x − b) 2
 a   dy
x2 x3 ⇒ (x2 + 4)   = n 2( y2 + 4)
=  + 1 ⇒ y =  dx
(x − c) (x − b)  x − 1  (x − a ) (x − b) (x − c)
A (x) B(x) C (x)
⇒ log y = log x3 − log (x − a ) (x − b) (x − c)
31. Let φ (x) = A (α ) B (α ) C (α ) …(i)
⇒ log y = 3 log x − log (x − a ) − log (x − b) − log(x − c)
A ′ (α ) B ′ (α ) C ′ (α )
On differentiating, we get
y′ 3 1 1 1 Given that, α is repeated root of quadratic equation
= − − − f (x) = 0.
y x x − a x−b x− c
y ′ 1 1  1 1  1 1  ∴ We must have f (x) = (x − α )2 ⋅ g (x)
⇒ = − + − + −  A ′ (x) B ′ (x) C ′ (x)
y  x x − a   x x − b  x x − c
y′ −a b c ∴ φ ′ (x) = A (α ) B (α ) C (α )
⇒ = − −
y x (x − a ) x (x − b) x (x − c) A ′ (α ) B ′ (α ) C ′ (α )
y′ a b c A ′ (α ) B ′ (α ) C ′ (α )
⇒ = + +
y x (a − x) x (b − x) x (c − x) ⇒ φ ′ (α ) = A (α ) B (α ) C (α ) = 0
y′ 1  a b c  A ′ (α ) B ′ (α ) C ′ (α )
⇒ =  + + 
y x  a − x b − x c − x
⇒ x = α is root of φ′ (x).
π
3
sin x ⇒ (x − α ) is a factor of φ′ (x) also.
29. Here, (sin +
y) 2 sec−1 (2x) + 2x tan {log (x + 2)} = 0 or we can say (x − α )2 is a factor of f (x).
2
On differentiating both sides, we get ⇒ φ (x) is divisible by f (x).
π
π π  2x  
32. Given,y = (log cos xsin x) ⋅ (logsin x cos x)−1 + sin −1 
sin x
(sin y) 2 ⋅ log (sin y) ⋅ cos x ⋅ 
2 2   1 + x2 
 π  2
 log e (sin x) −1  2 x 
π   sin x  −1
  dy
2 
+ sin x (sin y) ⋅ cos y ⋅ ∴ y=  + sin  
 2  dx  log e (cos x)  1 + x2
3 2 2 ⋅ sec {log (x + 2)}
x 2
(log e (cos x) ⋅ cot x 
+ ⋅ + 
2 (2|x|) 4x2 − 1 (x + 2) dy  log e (sin x) + loge ( in x) ⋅ tan x ) 2
⇒ =2  ⋅ +
+ 2x log 2 ⋅ tan {log (x + 2)} = 0
dx  log e (cos x) {log e (cos x)}2  1 + x2

 3
Putting  x = − 1, y = −  , we get 
 π   1 
 2 ⋅ log   
2  dy   2  2
 3 ⇒   = 2 1 ⋅ 2 
+
−   dx  x = π    log 1   1 + π
2
dy  π  3  4  
= =   2   16
dx  3
2
π π −3
2
8 32
1−  =− +
 π  log e 2 16 + π 2

30. Given, x = sec θ − cos θ and y = secn θ − cos n θ  x 


33. Given, (a + bx) ey/ x = x ⇒ y = x log  
On differentiating w.r.t. θ respectively, we get  a + bx
dx ⇒ y = x [log (x) − log (a + bx)]
= sec θ tan θ + sin θ …(i)

rankershalt.com

224 Limit, Continuity and Differentiability

On differentiating both sides, we get From Eqs. (i), (ii) and (iii), wet get
dy 1 b  dy 3
= ex sin x (3x3 ⋅ cos x3 + sin x3 ) + (tan x)x
=x −  + 1 [log (x) − log (a + bx)]
dx  x a + bx dx

  [2x cosec 2x + log (tan x)]


dy a
⇒x = x2   + y 5x
dx  x (a + bx) 36. Given, y = + cos 2(2x + 1)
3|1 − x|
ax
⇒ x y1 = + y …(ii)  5x
a + bx
3(1 − x) + cos (2x + 1), x < 1
2

Again, differentiating both sides, we get ⇒ y=


5x
 + cos 2(2x + 1), x > 1
 (a + bx) ⋅ 1 − x ⋅ b  3(x − 1)
x y2 + y1 = a   + y1
 (a + bx)2  The function is not defined at x = 1.
2 2
a x 5  (1 − x) − x(−1) 
⇒ x3 y2 =    − 2 sin (4x + 2), x < 1
(a + bx)2 dy 3  (1 − x)2 
⇒ =
2 dx  5  (x − 1) − x(1) 
 ax  − 2 sin (4x + 2), x > 1
⇒ x3 y2 =   [from Eq. (ii)]  3  (x − 1)2 
 (a + bx) 
⇒ x3 y2 = (xy1 − y)2  5
− 2 sin (4x + 2), x < 1
2 dy  3 (1 − x)2
d 2y  dy  ⇒ =
⇒ x3 = x − y dx − 5
dx2  dx  − 2 sin (4x + 2), x > 1
 3 (x − 1)2
34. Given, h (x) = [ f (x)]2 + [ g (x)]2 F (x) 1
37. Here, lim =
⇒ h ′ x = 2 f (x) ⋅ f ′ (x) + 2 g (x) ⋅ g ′ (x) x→1
G (x) 14
= 2 [ f (x) ⋅ g (x) − g (x) ⋅ f (x)] F ′ (x) 1
⇒ lim = [using L’Hospital’s rule]…(i)
=0 [Q f ′ (x) = g (x) and g′ (x) = − f (x)] x→1 G′ (x) 14
x
∴ h (x) is constant. As F (x) = ∫ f (t ) dt ⇒ F ′ (x) = f (x) …(ii)
–1
⇒ h (10) = h (5) = 11 x
x sin x 3
and G (x) = ∫ t f { f (t )} dt
35. Since, y = e + (tan x) , then
x –1

3 ⇒ G′ (x) = x f { f (x)} …(iii)


y = u + v, where u = ex sin x and v = (tan x)x F (x) F ′ (x) f (x)
∴ lim = lim = lim
dy  du dv x→1 G (x) x→1 G′ (x) x→1 x f { f (x)}
⇒ = +  …(i)
dx  dx dx f (1) 1 /2
3
= = ...(iv)
Here, u = ex sin x and log v = x log (tan x) 1 f { f (1)} f (1 / 2)
F (x) 1
On differentiating both sides w.r.t. x, we get Given, lim =
x→1 G (x) 14
du 3
= ex sin x ⋅ (3x3 cos x3 + sin x3 ) …(ii) 1
dx
∴ 2 = 1 ⇒ f  1 = 7
1 dv x ⋅ sec2x  
and ⋅ = + log (tan x)  1 14  2
f 
v dx tan x  2
dv
= (tan x)x [2x ⋅ cosec (2x) + log (tan x)] …(iii)
dx

Download Chapter Test


http://tinyurl.com/y6dl84lx or
rankershalt.com

10
Application of Derivatives

Topic 1 Equations of Tangent and Normal


Objective Questions I (Only one correct option) 6. If θ denotes the acute angle between the curves,
x y = 10 − x2 and y = 2 + x2 at a point of their intersection,
1. If the tangent to the curve y = 2 , x ∈ R, (x ≠ ± 3 ), at then|tan θ|is equal to (2019 Main, 9 Jan I)
x −3 7 8 4 8
a point (α , β ) ≠ (0, 0) on it is parallel to the line (a) (b) (c) (d)
17 15 9 17
2x + 6 y − 11 = 0, then (2019 Main, 10 April II)
(a) | 6α + 2β | = 19
7. If the curves y2 = 6x, 9x2 + by2 = 16 intersect each other
(b) | 6α + 2β | = 9 at right angles, then the value of b is (2018 Main)
(c) | 2α + 6β | = 19 7
(a) 6 (b)
(d) | 2α + 6β | = 11 2
9
2. Let S be the set of all values of x for which the tangent to (c) 4 (d)
2
the curve y = f (x) = x − x − 2x at (x, y) is parallel to the
3 2

line segment joining the points (1, f (1)) and (−1, f (−1)), 8. The normal to the curve y(x − 2) (x − 3) = x + 6 at the
then S is equal to (2019 Main, 9 April I) point, where the curve intersects the Y -axis passes
through the point
(a)  , − 1 (b)  , 1
1 1 (2017 Main)

(a)  − , −  (b)  ,
3  3  1 1 1 1

 1   2 2 2 2
(d) − , − 1
1
(c) − , 1
 3   3   1
(c)  , − 
1
(d)  ,
1 1

2 3 2 3
3. If the tangent to the curve, y = x3 + ax − b at the point
(1, − 5) is perpendicular to the line, − x + y + 4 = 0, then  1 + sin x   π
9. Consider f (x) = tan −1   , x ∈ 0,  .
which one of the following points lies on the curve ?  1 − sin x   2
(2019 Main, 9 April I)
π
(a) (−2, 2) (b) (2, − 2) A normal to y = f (x) at x = also passes through the
6
(c) (−2, 1) (d) (2, − 1)
point (2016 Main)
4. The tangent to the curve y = x2 − 5x + 5, parallel to the 2π  π π
(a) (0, 0) (b)  0,  (c)  , 0 (d)  , 0
line 2 y = 4x + 1, also passes through the point  3 6  4 
(2019 Main, 12 Jan II)
10. The normal to the curve x2 + 2xy − 3 y2 = 0 at (1,1)
(a)  ,  (b)  , 
1 7 7 1
 4 2  2 4 (a) does not meet the curve again (2015 Main)

(c)  − , 7 (d)  , − 7


1 1 (b) meets in the curve again the second quadrant
 8  8  (c) meets the curve again in the third quadrant
(d) meets the curve again in the fourth quadrant
5. A helicopter is flying along the curve given by
y − x3/ 2 = 7, (x ≥ 0). A soldier positioned at the point 11. The point(s) on the curve y3 + 3x2 = 12 y, where the
1  tangent is vertical, is (are) (2002, 2M)
 , 7 wants to shoot down the helicopter when it is
2 
 11 
(a)  ± , − 2
4
nearest to him. Then, this nearest distance is (b)  ± , 0
 3   3 
(2019 Main, 10 Jan II)
(d)  ± , 2
1 7 5 1 7 1 4
(a) (b) (c) (d) (c) (0, 0)
 3 
3 3 6 6 3 2
rankershalt.com

226 Application of Derivatives

12. If the normal to the curve y = f (x) at the point Fill in the Blank

(3, 4) makes an angle with the positive X-axis, then 16. Let C be the curve y3 − 3xy + 2 = 0. If H is the set of
4
points on the curve C, where the tangent is horizontal
f ′ (3) is equal to (2000, 1M)
and V is the set of points on the curve C, where the
(a) –1 (b) –3/4 tangent is vertical, then H = K and V = K . (1994, 2M)
(c) 4/3 (d) 1
13. The normal to the curve x = a (cos θ + θ sin θ ), Analytical & Descriptive Questions
y = a (sin θ − θ cos θ ) at any point ‘ θ ’ is such that
17. If | f (x1 ) − f (x2)| ≤ (x1 − x2)2, ∀x1 , x2 ∈ R.
Find the
(1983, 1M)
(a) it makes a constant angle with the X-axis equation of tangent to the curve y = f (x) at the point
(1, 2). (2005, 4M)
(b) it passes through the origin
(c) it is at a constant distance from the origin 18. The curve y = ax3 + bx2 + cx + 5, touches the X-axis at
(d) None of the above P(−2, 0) and cuts the Y-axis at a point Q, where its
gradient is 3. Find a, b, c. (1994, 5M)
Objective Questions II 19. Tangent at a point P1 {other than (0, 0)} on the curve
(One or more than one correct option) y = x3 meets the curve again at P2. The tangent at P2
14. On the ellipse 4x + 9 y = 1, the point at which the
2 2 meets the curve at P3 and so on.
tangents are parallel to the line 8x = 9 y, are (1999, 2M) Show that the abscissa of P1, P2, P3 , …, Pn, form a GP.
(a)  ,  (b)  − ,
2 1 2 1 Also, find the ratio of

 5 5  5 5 [area (∆P1P2P3 )]/[area (∆P2P3 P4 )]. (1993, 5M)

(c)  − , −  (d)  , −
2 1 2 1 20. Find the equation of the normal to the curve

 5 5 5 5 y = (1 + x)y + sin −1 (sin 2 x) at x = 0. (1993, 4M)
15. If the line ax + by + c = 0 is a normal to the curve xy = 1, 21. Find all the tangents to the curve y = cos (x + y),
then −2π ≤ x ≤ 2π , that are parallel to the line x + 2 y = 0.
(a) a > 0, b > 0
(1985, 5M)
(b) a > 0, b < 0 (1986, 2M)
(c) a < 0, b > 0 Integer Answer Type Question
(d) a < 0, b < 0 22. The slope of the tangent to the curve ( y − x5 )2 = x (1 + x2)2
at the point (1, 3) is (2014 Adv.)

Topic 2 Rate Measure, Increasing and Decreasing Functions


Objective Questions I (Only one correct option) the instant when the depth of water in the tank is 10 m
is (2019 Main, 9 April II)
1. A spherical iron ball of radius 10 cm is coated with a 2 1
layer of ice of uniform thickness that melts at a rate of (a) (b)
π 5π
50 cm3 /min. When the thickness of the ice is 5 cm, then 1 1
the rate at which the thickness (in cm/min) of the ice (c) (d)
15π 10π
decreases, is (2019 Main, 10 April II)
(a)
1
(b)
1
(c)
1
(d)
5 4. Let f : [0, 2] → R be a twice differentiable function such
9π 18 π 36 π 6π that f ′ ′ (x) > 0, for all x ∈ (0, 2). If φ (x) = f (x) + f (2 − x) ,
then φ is (2019 Main, 8 April I)
2. Let f (x) = e − x and g (x) = x − x, ∀ x ∈ R. Then, the set
x 2
(a) increasing on (0, 1) and decreasing on (1, 2)
of all x ∈ R, where the function h (x) = ( fog ) (x) is
(b) decreasing on (0, 2)
increasing, is (2019 Main, 10 April II)
(c) decreasing on (0, 1) and increasing on (1, 2)
−1
(a)  0,  ∪ [1, ∞ ) (b)  −1,  ∪  , ∞ 
1 1 (d) increasing on (0, 2)
 2   2   2  5. If the function f given by
−1
(c) [0, ∞ ) (d)  , 0 ∪ [1, ∞ ) f (x) = x3 − 3(a − 2) x2 + 3ax + 7,
 2 
for some a ∈ R is increasing in (0, 1] and decreasing in
3. A water tank has the shape of an inverted right circular f (x) − 14
 1 [1, 5), then a root of the equation, = 0 (x ≠ 1) is
cone, whose semi-vertical angle is tan −1   . Water is (x − 1)2
 2
(2019 Main, 12 Jan II)
poured into it at a constant rate of 5 cu m/min. Then, the
(a) − 7 (b) 6 (c) 7 (d) 5
rate (in m/min) at which the level of water is rising at
rankershalt.com

Application of Derivatives 227

x d−x (c) increasing on (0, π / e), decreasing on ( π / e, ∞ )


6. Let f (x) = − , x ∈ R, where a, b
a +x 2
b + (d − x)2
2 2 (d) decreasing on (0, π / e), increasing on ( π / e, ∞ )
and d are non-zero real constants. Then, 16. Let f and g be increasing and decreasing functions
(2019 Main, 11 Jan II) respectively from [0, ∞ ) to [0, ∞ ) and h (x) = f { g (x)}. If
(a) f is an increasing function of x h(0) = 0, then h (x) − h (1) is (1987, 2M)
(b) f ′ is not a continuous function of x (a) always negative
(c) f is a decreasing function of x (b) always positive
(d) f is neither increasing nor decreasing function of x (c) strictly increasing
π π
7. If the function g : (−∞ , ∞ ) →  − ,  is given by (d) None of these
 
2 2
π Objective Questions II
g (u ) = 2 tan −1 (eu ) − . Then, g is (2008, 3M)
2 (One or more than one correct option)
(a) even and is strictly increasing in (0, ∞ )
17. If f : R → R is a differentiable function such that
(b) odd and is strictly decreasing in (−∞ , ∞ )
f ′ (x) > 2 f (x) for all x ∈ R, and f (0) = 1 then (2017 Adv.)
(c) odd and is strictly increasing in (−∞ , ∞ )
(d) neither even nor odd but is strictly increasing in (a) f (x) > e2x in (0, ∞ )
(−∞ , ∞ ) (b) f ′ (x) < e2x in (0, ∞ )
(c) f (x) is increasing in (0, ∞ )
8. If f (x) = x3 + bx2 + cx + d and 0 < b2 < c, then in (−∞ , ∞ )
(d) f (x) is decreasing in (0 ∞ )
(a) f (x) is strictly increasing function (2004, 2M)
(b) f (x) has a local maxima 18. If f : (0, ∞ ) → R be given by (2014 Adv.)
(c) f (x) is strictly decreasing function  1
x −  t +  dt
(d) f (x) is bounded f (x) = ∫ e t
.
1/ x t
9. The length of a longest interval in which the function
Then,
3 sin x − 4 sin3 x is increasing, is (2002, 2M)
π π (a) f (x) is monotonically increasing on [1, ∞ )
(a) (b) (b) f (x) is monotonically decreasing on [0,1)
3 2
(c) f (x) + f   = 0, ∀x ∈ (0, ∞ )
3π 1
(c) (d) π  x
2
(d) f (2x ) is an odd function of x on R
10. If f (x) = xex (1 − x ) , then f (x) is (2001, 2M)
19. If h (x) = f (x) − f (x)2 + f (x)3 for every real number x.
(a) increasing in [−1 / 2, 1] (b) decreasing in R
(c) increasing in R (d) decreasing in [−1 / 2, 1] Then, (1998, 2M)
(a) h is increasing, whenever f is increasing
11. For all x ∈ (0, 1) (2000, 1M) (b) h is increasing, whenever f is decreasing
(a) ex < 1 + x (b) log e (1+ x) < x (c) h is decreasing, whenever f is decreasing
(c) sin x > x (d) log ε x > x (d) Nothing can be said in general
12. Let f (x) = ∫ ex (x − 1) (x − 2) dx. Then, f decreases in the
Fill in the Blanks
interval (2000, 2M)
(a) (−∞ , − 2) (b) (−2, − 1) (c) (1, 2) (d) (2, ∞ ) 20. The set of all x for which log (1 + x) ≤ x is equal to ..... .
(1987, 2M)
13. The function f (x) = sin x + cos x increases, if
4 4

π π 3π 21. The function y = 2x − log|x| is monotonically


2
(a) 0 < x < (b) < x< (1999, 2M)
8 4 8 increasing for values of x ( ≠ 0), satisfying the
3π 5π 5π 3π inequalities… and monotonically decreasing for values
(c) < x< (d) < x<
8 8 8 4 of x satisfying the inequalities… . (1983, 2M)
x x
14. If f (x) = and g (x) = , where 0 < x ≤ 1, then in
sin x tan x
Match the Columns
this interval (1997, 2M)
(a) both f (x) and g (x) are increasing functions Directions (Q.Nos. 22-24) by appropriately matching
(b) both f (x) and g (x) are decreasing functions the information given in the three columns of the
(c) f (x) is an increasing function following table.
(d) g (x) is an increasing function Let f (x) = x + log e x − x log e x, x ∈ (0, ∞ )
log (π + x) Column 1 contains information about zeros of f (x), f ′ (x)
15. The function f (x) = is (1995, 1M)
log (e + x) and f ′′ (x).
(a) increasing on (0, ∞ ) Column 2 contains information about the limiting
behaviour of f (x), f ′ (x) and f ′′ (x) at infinity.
(b) decreasing on (0, ∞ )
rankershalt.com

228 Application of Derivatives


Column 3 contains information about increasing/decreasing Column I Column II
nature of f (x) and f ′ (x).
A. x + sin x p. increasing
Column-1 Column-2 Column-3 B. sec x q. decreasing
(I) f( x ) = 0 for some (i) lim f( x ) = 0 (P) f is increasing r. neither increasing nor decreasing
x→ ∞
x ∈ (1, e 2 ) in (0, 1)

(II) f ′ ( x ) = 0 for some (ii) lim f( x ) = − ∞


x→ ∞
(Q) f is decreasing Analytical & Descriptive Questions
x ∈ (1, e ) in (e, e 2) 3x (x + 1)  π
26. Prove that sin x + 2x ≥ , ∀ x ∈ 0,
 2 
(III) f ′ ( x ) = 0 for some (iii) lim f ′ ( x ) = − ∞ (R) f′ is increasing π
x→ ∞
x ∈ ( 0, 1) in (0, 1)
(Justify the inequality, if any used). (2004, 4M)
(IV) f ′′( x ) = 0 for some (iv) lim f ′′( x ) = 0 (S) f′ is decreasing
x→ ∞ 27. Using the relation 2 (1 − cos x) < x , x ≠ 0 or prove that
2
x ∈ (1, e ) in (e, e 2)
sin (tan x) ≥ x , ∀ x ∈ [0, π / 4]. (2003, 4M)
22. Which of the following options is the only INCORRECT 28. If −1 ≤ p ≤ 1, then show that the equation
combination? (2017 Adv.)
4x3 − 3x − p = 0 has a unique root in the interval [1 / 2, 1]
(a) (I) (iii) (P) (b) (II) (iv) (Q)
and identify it.
(c) (II) (iii) (P) (d) (III) (i) (R) (2001, 5M)
23. Which of the following options is the only CORRECT  xeax , x≤0
combination? 29. Let f (x) = 
(a) (I) (ii) (R) (b) (III) (iv) (P)  x + ax 2
− x3
, x >0
(c) (II) (iii) (S) (d) (IV) (i) (S) where, a is a positive constant. Find the interval in
24. Which of the following options is the only CORRECT which f ′ (x) is increasing. (1996, 3M)
combination? 30. Show that 2 sin x + 2 tan x ≥ 3x, where 0 ≤ x < π /2.
(a) (III) (iii) (R) (b) (IV) (iv) (S) (1990, 4M)
(c) (II) (ii) (Q) (d) (I) (i) (P)
31. Show that 1 + x log (x + x2 + 1 ) ≥ 1 + x2 ∀x ≥ 0.
25. Match the conditions/expressions in Column I with (1983, 2M)
π π
32. Given A = x :
statements in Column II.
≤ x ≤  and f (x) = cos x – x (1 + x). Find
Let the functions defined in Column I have domain  6 3
(1980, 2M)
(−π / 2, π / 2). f ( A ).

Topic 3 Rolle’s and Lagrange’s Theorem


Objective Questions I (Only one correct option) Analytical & Descriptive Question
1. If f : R → R is a twice differentiable function such that 3. If f (x) and g (x) are differentiable functions for 0 ≤ x ≤ 1,
 1 1 such that f (0) = 2, g (0) = 0
f ′′ (x) > 0 for all x ∈ R, and f   = , f (1) = 1, then
 2 2 (2017 Adv.) f (1) = 6, g (1) = 2
(a) f ′ (1) ≤ 0 (b) f ′ (1) > 1 Then, show that there exists c satisfying 0 < c < 1 and
1 1 f ′ (c) = 2 g′ (c).
(c) 0 < f ′ (1) ≤ (d) < f ′ (1) ≤ 1 (1982, 2M)
2 2

Assertion and Reason Topic 4 Maxima and Minima


2. Let f (x) = 2 + cos x, for all real x. Objective Questions I (Only one correct option)
Statement I For each real t, there exists a point c in
1. Let f (x) = 5 −|x − 2| and g (x) = |x + 1|, x ∈ R. If f (x)
[t , t + π ], such that f ' (c) = 0.
attains maximum value at α and g (x) attains minimum
Because (x − 1) (x2 − 5x + 6)
Statement II f (t ) = f (t + 2π ) for each real t. (2007, 3M) value of β, then lim is equal to
x → − αβ x2 − 6x + 8
(a) Statement I is correct, Statement II is also correct; (2019 Main, 12 April II)
Statement II is the correct explanation of Statement I (a) 1/2 (b) − 3 / 2 (c) − 1 / 2 (d) 3/2
(b) Statement I is correct, Statement II is also correct;
Statement II is not the correct explanation of 2. If the volume of parallelopiped formed by the vectors
$i + λ$j + k
$ , $j + λk
$ and λi$ + k
$ is minimum, then λ is equal
Statement I
(c) Statement I is correct; Statement II is incorrect to (2019 Main, 12 April I)
1 1
(d) Statement I is incorrect; Statement II is correct (a) − (b) (c) 3 (d) − 3
3 3
rankershalt.com

Application of Derivatives 229

3. If m is the minimum value of k for which the function 1 1


12. Let f (x) = x2 + 2
and g (x) = x − , x ∈ R − { −1, 0, 1}. If
f (x) = x kx − x is increasing in the interval [0, 3] and M
2 x x
f (x)
is the maximum value of f in the interval [0, 3] when h (x) = , then the local minimum value of h (x) is
k = m, then the ordered pair (m, M ) is equal to g (x) (2018 Main)
(2019 Main, 12 April I) (a) 3 (b) −3
(a) (4, 3 2 ) (b) (4, 3 3 ) (c) −2 2 (d) 2 2
(c) (3, 3 3 ) (d) (5, 3 6 ) 13. If 20 m of wire is available for fencing off a flower-bed in
4. If f (x) is a non-zero polynomial of degree four, having the form of a circular sector, then the maximum area (in
local extreme points at x = − 1, 0, 1, then the set sq. m) of the flower-bed is (2017 Main)
S = { x ∈ R : f (x) = f (0)} contains exactly (a) 12.5 (b) 10
(a) four rational numbers (2019 Main, 9 April I) (c) 25 (d) 30
(b) two irrational and two rational numbers 14. A wire of length 2 units is cut into two parts which
(c) four irrational numbers are bent respectively to form a square of side = x units
(d) two irrational and one rational number and a circle of radius = r units. If the sum of the
areas of the square and the circle so formed is
5. The height of a right circular cylinder of maximum
minimum, then (2016 Main)
volume inscribed in a sphere of radius 3 is
(2019 Main, 8 April II) (a) 2x = ( π + 4)r (b) (4 − π )x = πr
(a) 6 (b) 2 3 (c) x = 2r (d) 2x = r
2 1
(c) 3 (d) 3 15. The least value of α ∈ R for which 4αx2 + ≥ 1 , for all
3 x
6. If S1 and S 2 are respectively the sets of local x > 0, is (2016 Adv.)
1 1 1 1
minimum and local maximum points of the function, (a) (b) (c) (d)
64 32 27 25
f (x) = 9x4 + 12x3 − 36x2 + 25, x ∈ R, then
(a) S1 = {−2} ; S2 = {0,1} (2019 Main, 8 April I) 16. Let f (x) be a polynomial of degree four having extreme
S1 = {−2, 0} ; S2 = {1} lim  f (x) 
(b) values at x = 1 and x = 2. If x→ 0 1+ = 3, then f (2) is
(c) S1 = {−2,1} ; S2 = {0}  x2 
(d) S1 = {−1} ; S2 = {0, 2} equal to
7. The shortest distance between the line y = x and the (a) −8 (b) −4
(c) 0 (d) 4
curve y = x − 2 is
2
(2019 Main, 8 April I)
7 7 11 17. If x = − 1 and x = 2 are extreme points of
(a) 2 (b) (c) (d)
8 4 2 4 2 f (x) = α log|x| + βx2 + x, then (2014 Main)
1 1
8. The maximum area (in sq. units) of a rectangle having (a) α = − 6, β = (b) α = − 6, β = −
its base on the X-axis and its other two vertices on the 2 2
1 1
parabola, y = 12 − x2 such that the rectangle lies inside (c) α = 2, β = − (d) α = 2, β =
the parabola, is (2019 Main, 12 Jan I) 2 2
(a) 36 (b) 20 2 18. The number of points in (− ∞ , ∞ ) for which
(c) 32 (d) 18 3 x2 − x sin x − cos x = 0, is (2013 Adv.)
9. The maximum value of the function (a) 6 (b) 4 (c) 2 (d) 0
f (x) = 3x3 − 18x2 + 27x − 40 19. Let f, g and h be real-valued functions defined on the
2 2 2 2
on the set S = { x ∈ R : x2 + 30 ≤ 11x} is (2019 Main, 11 Jan I) interval [0, 1] by f (x) = ex + e− x , g (x) = x ex + e− x and
2 2
(a) 122 (b) − 122 (c) − 222 (d) 222 h (x) = x2ex + e− x . If a, b and c denote respectively, the
10. Let A (4, − 4) and B(9, 6) be points on the parabola, absolute maximum of f, g and h on [0, 1], then (2010)
y2 = 4x. Let C be chosen on the arc AOB of the parabola, (a) a = b and c ≠ b (b) a = c and a ≠ b
where O is the origin, such that the area of ∆ACB is (c) a ≠ b and c ≠ b (d) a = b = c
maximum. Then, the area (in sq. units) of ∆ACB, is 20. The total number of local maxima and local minima of
(2019 Main, 9 Jan II) (2 + x)3 , − 3 < x ≤ −1
1 the function f (x) =  2 is
(a) 31 (b) 32 (2008, 3M)
4 x 3 , −1 < x < 2
3 1
(c) 31 (d) 30 (a) 0 (b) 1 (c) 2 (d) 3
4 2
11. The maximum volume (in cu.m) of the right circular 21. If f (x) = x + 2bx + 2c and g (x) = − x − 2cx + b2, such
2 2 2

cone having slant height 3m is (2019 Main, 9 Jan I) that min f (x) > max g (x), then the relation between b
4 and c, is (2003, 2M)
(a) π (b) 2 3π
3 (a) No real value of b and c (b) 0 < c < b 2
(c) 3 3π (d) 6π (c)| c|< | b| 2 (d)| c|> | b| 2
rankershalt.com

230 Application of Derivatives

22. If f (x) = | x|, for 0 < | x| ≤ 2 . Then, at x = 0, f has (a) f (x) attains its minimum at x = 0
1 , for x=0 (b) f (x) attains its maximum at x = 0
(2000, 1M)
(a) a local maximum (b) no local maximum (c) f ′ (x) = 0 at more than three points in (− π , π )
(c) a local minimum (d) no extremum (d) f ′ (x) = 0 at exactly three points in (− π , π )
x2 − 1 31. Let f : R → (0, ∞ ) and g : R → R be twice differentiable
23. If f (x) = , for every real number x, then the
x2 + 1 functions such that f′ ′ and g′ ′ are continuous functions
minimum value of f (1998, 2M)
on R. Suppose f ′ (2) = g (2) = 0, f ′ ′(2) ≠ 0 and g′ (2) ≠ 0.
f (x) g (x)
(a) does not exist because f is unbounded If lim = 1, then
(b) is not attained even though f is bounded x → 2 f ′ (x) g′ (x) (2016 Adv.)
(c) is 1
(a) f has a local minimum at x = 2
(d) is –1
(b) f has a local maximum at x = 2
24. The number of values of x, where the function (c) f ′ ′ (2) > f (2)
f (x) = cos x + cos ( 2x) attains its maximum, is(1998, 2M) (d) f (x) − f ′ ′ (x) = 0, for atleast one x ∈ R
(a) 0 (b) 1 (c) 2 (d) infinite 32. The function f (x) = 2|x| + |x + 2| − ||x + 2| − 2|x|| has a
25. On the interval [0,1], the function x 25
(1 − x)75 takes its local minimum or a local maximum at x is equal to
maximum value at the point (1995, 1M) (2013 Adv.)
−2
(a) 0 (b) 1/4 (c) 1/2 (d) 1/3 (a) −2 (b) (c) 2 (d) 2/3
3
26. Find the coordinates of all the points P on the ellipse
x2 y2 33. A rectangular sheet of fixed perimeter with sides
+ = 1, for which the area of the ∆PON is having their lengths in the ratio 8 : 15 is converted into
a 2 b2 an open rectangular box by folding after removing
maximum, where O denotes the origin and N is the foot squares of equal area from all four corners. If the total
of the perpendicular from O to the tangent at P. area of removed squares is 100, the resulting box has
(1990, 10M)
maximum volume. The lengths of the sides of the
 ± a2 ± b2   ± a2 ± b2 
(a)  ,  (b)  ,  rectangular sheet are (2013 Adv.)
 a 2 + b2 a 2 + b2   a 2 – b2 a 2 – b2  (a) 24 (b) 32 (c) 45 (d) 60
 ± a2 ± b2   ± a2 ± b2   ex , 0 ≤ x≤1
(c)  ,  (d)  , 
 a 2 + b2 a 2 – b2   a 2 – b2 a 2 + b2   x
34. If f (x) = 2 − ex − 1 , 1 < x≤2 and g (x) = ∫ f (t ) dt ,
0
27. If P (x) = a 0 + a1x + a 2x +... + a nx is a polynomial in
2 4 2n  x−e , 2 < x≤3

a real variable x with 0 < a 0 < a1 < a 2 < K < a n. Then, x ∈ [1, 3], then (2006, 3M)
the function P (x) has (1986, 2M) (a) g (x) has local maxima at x = 1 + log e 2 and local
(a) neither a maximum nor a minimum minima at x = e
(b) only one maximum (b) f (x) has local maxima at x = 1and local minima at x = 2
(c) only one minimum (c) g (x) has no local minima
(d) only one maximum and only one minimum (d) f (x) has no local maxima
28. If y = a log x + bx2 + x has its extremum values at x = − 1 35. If f (x) is a cubic polynomial which has local maximum at
and x = 2 , then (1983, 1M) x = − 1. If f (2) = 18, f (1) = − 1 and f ′ (x) has local
1 minimum at x = 0, then
(a) a = 2, b = − 1 (b) a = 2, b = − (2006, 3M)
2 (a) the distance between (– 1, 2) and (a , f (a )), where x = a
1
(c) a = − 2, b = (d) None of the above is the point of local minima, is 2 5
2
(b) f (x) is increasing for x ∈[1, 2 5 ]
29. If p, q and r are any real numbers, then (1982, 1M) (c) f (x) has local minima at x = 1
(a) max ( p , q) < max ( p , q, r ) (d) the value of f (0) = 5
1
(b) min ( p , q) = ( p + q − | p − q|) 36. The function
2
x
(c) max ( p , q) < min ( p , q, r ) f (x) = ∫ t (et − 1) (t − 1) (t − 2)3 (t − 3)5 dt has a local
−1
(d) None of the above
minimum at x equals (1999, 3M)

Objective Questions II (a) 0 (b) 1 (c) 2 (d) 3

37. If f (x) = 3x + 12x − 1, − 1 ≤ x ≤ 2, then


2
(One or more than one correct option) (1993, 3M)
37 − x, 2 < x≤3
cos(2x) cos(2x) sin(2x) (a) f (x) is increasing on [–1, 2]
30. If f (x) =  
− cos x cos x − sin x , then (b) f (x) is continuous on [–1, 3]
 sin x sin x cos x  (2017 Adv.) (c) f ′(2) does not exist
(d) f (x) has the maximum value at x = 2
rankershalt.com

Application of Derivatives 231

Match the Columns 43. For the circle x2 + y2 = r 2, find the value of r for which
the area enclosed by the tangents drawn from the point
38. A line L : y = mx + 3 meets Y -axis at E (0, 3) and the P(6, 8) to the circle and the chord of contact is
arc of the parabola y2 = 16x, 0 ≤ y ≤ 6 at the point maximum. (2003, 2M)
F (x0 , y0 ). The tangent to the parabola at F (x0 , y0 )
44. Find a point on the curve x2 + 2 y2 = 6 whose distance
intersects the Y -axis at G (0, y1 ). The slope m of the
from the line x + y = 7, is minimum. (2003, 2M)
line L is chosen such that the area of the ∆EFG has a
local maximum 45. Let f (x) is a function satisfying the following conditions
Match List I with List II and select the correct answer (i) f (0) = 2, f (1) = 1
using the codes given below the list. (ii) f (x) has a minimum value at x = 5 / 2 and
2 ax 2ax − 1 2 ax + b + 1
Column I Column II
(iii) For all x, f ′ (x) = b b+ 1 −1
P. m= 1. 1/2 2 (ax + b) 2 ax + 2b + 1 2 ax + b
Q. Maximum area 2. 4
where, a and b are some constants. Determine the
of ∆EFG is
constants a , b and the function f (x). (1998, 8M)
R. y0 = 3. 2 46. Let C1 and C 2 be respectively, the parabolas x2 = y – 1
S. y1 = 4. 1 and y2 = x – 1. Let P be any point on C1 and Q be any
point on C 2. If P1 and Q1 is the reflections of P and Q,
Codes
respectively, with respect to the line y = x. Prove that P1
P Q R S
lies on C 2 Q1 lies on C1 and PQ ≥ min (PP1 , QQ1 ). Hence,
(a) 4 1 2 3
determine points P0 and Q0 on the parabolas C1 and C 2
(b) 3 4 1 2
respectively such that P0Q0 ≤ PQ for all pairs of points
(c) 1 3 2 4
(P , Q ) with P on C1 and Q on C 2.
(d) 1 3 4 2
47. If S is a square of unit area. Consider any quadrilateral
which has one vertex on each side of S. If a , b, c and d
Passage Based Problems denote the length of the sides of the quadrilateral, then
Consider the function f : (−∞ , ∞ ) → (−∞ , ∞ ) defined by prove that 2 ≤ a 2 + b2 + d 2 ≤ 4. (1997, 5M)
x2 − ax + 1 48. Determine the points of maxima and minima of the
f (x) = 2 ; 0 < a < 2. (2008, 12M)
x + ax + 1 1
functionf (x) = In x − bx + x2, x > 0, where b ≥ 0 is a
8
39. Which of the following is true ? constant. (1996, 5M)
(a) (2 + a )2 f ′ ′ (1) + (2 − a )2 f ′ ′ (−1) = 0 49. Let (h , k) be a fixed point, where h > 0 , k > 0. A straight
(b) (2 − a )2 f ′ ′ (1) − (2 + a )2 f ′ ′ (−1) = 0 line passing through this point cuts the positive
(c) f ′ (1) f ′ (−1) = (2 − a )2 directions of the coordinate axes at the points P and Q.
(d) f ′ (1) f ′ (−1) = − (2 + a )2 Find the minimum area of the ∆OPQ, O being the
origin. (1995, 5M)
40. Which of the following is true ?
(a) f (x) is decreasing on (−1, 1) and has a local minimum at 50. The circle x2 + y2 = 1 cuts the X-axis at P and Q. Another
x=1 circle with centre at Q and variable radius intersects
(b) f (x) is increasing on (−1, 1) and has a local maximum at the first circle at R above the X-axis and the line
x=1 segment PQ at S. Find the maximum area of the ∆QSR.
(c) f (x) is increasing on (−1, 1) but has neither a local (1994, 5M)
maximum nor a local minimum at x = 1  3 (b3 − b2 + b − 1)
(d) f (x) is decreasing on (−1, 1) but has neither a local − x + , 0 ≤ x≤1
51. Let f (x) =  (b2 + 3b + 2)
maximum nor a local minimum at x = 1 
 2x − 3, 1 ≤ x≤3
ex f ′ (t )
41. Let g (x) = ∫ dt. Which of the following is true? Find all possible real values of b such that f (x) has the
0 1 + t2
smallest value at x = 1. (1993, 5M)
(a) g ′ (x) is positive on (− ∞ , 0) and negative on (0, ∞ )
(b) g ′ (x) is negative on (− ∞ , 0) and positive on (0, ∞ ) 52. What normal to the curve y = x2 forms the shortest
(c) g ′ (x) changes sign on both (− ∞ , 0) and (0, ∞ ) chord? (1992, 6M)
(d) g ′ (x) does not change sign (− ∞ , ∞ ) 53. A window of perimeter (including the base of the arch)
is in the form of a rectangle surmounted by a
Analytical & Descriptive Questions semi-circle. The semi-circular portion is fitted with
42. If f (x) is twice differentiable function such that f (a ) = 0, coloured glass while the rectangular part is fitted with
f (b) = 2, f (c) = 1, f (d ) = 2, f (e) = 0, where a < b < c < d < e, clear glass. The clear glass transmits three times as
then the minimum number of zeroes of much light per square meter as the coloured glass does.
g (x) = { f ′ (x)}2 + f ′′ (x) ⋅ f (x) in the interval [a, e] is What is the ratio for the sides of the rectangle so that
the window transmits the maximum light? (1991, 4M)
rankershalt.com

232 Application of Derivatives

54. A point P is given on the circumference of a circle of is open at the top. The bottom of the container is a solid
radius r. Chord QR is parallel to the tangent at P. circular disc of thickness 2 mm and is of radius equal to
Determine the maximum possible area of the ∆PQR. the outer radius of the container.
(1990, 4M) If the volume of the material used to make the container
55. Find the point on the curve 4x + a y = 4a , 4 < a 2 < 8
2 2 2 2 is minimum, when the inner radius of the container is
V
that is farthest from the point (0, –2). (1987, 4M) 10 mm, then the value of is (2015 Adv.)
250 π
56. Let A ( p , − p) B(q , q), C (r , − r ) be the vertices of the
2 2 2
63. A vertical line passing through the point (h , 0)
triangle ABC. A parallelogram AFDE is drawn with
x2 y2
vertices D, E and F on the line segments BC, CA and intersects the ellipse + = 1 at the points P and Q. If
AB, respectively. Using calculus, show that maximum 4 3
1 the tangents to the ellipse at P and Q meet at the point
area of such a parallelogram is ( p + q)(q + r )( p − r ).
4 R.
(1986, 5M)
π π If ∆(h ) = area of the ∆ PQR, ∆1 = max ∆ (h ) and
57. Let f (x) = sin x + λ sin x, − < x < ⋅ Find the
3 2
1/ 2 ≤ h ≤ 1
2 2 8
intervals in which λ should lie in the order that f (x) has ∆ 2 = min ∆ (h ), then ∆1 − 8∆ 2 is equal to (2013 Adv)
1/ 2 ≤ h ≤ 1 5
exactly one minimum and exactly one maximum.
(1985, 5M) 64. Let f : R → R be defined as f (x) = | x| + | x2 − 1|. The
x total number of points at which f attains either a local
58. Find the coordinates of the point on the curve y = , maximum or a local minimum is (2012)
1 + x2
where the tangent to the curve has the greatest slope. 65. Let p(x) be a real polynomial of least degree which has a
(1984, 4M) local maximum at x = 1 and a local minimum at x = 3. If
p(1) = 6 and p(3) = 2, then p′ (0) is equal to (2012)
59. A swimmer S is in the sea at a distance d km from the
closest point A on a straight shore. The house of the 66. The number of distinct real roots of
swimmer is on the shore at a distance L km from A. He x4 − 4x3 + 12x2 + x − 1 = 0 is……
can swim at a speed of u km/h and walk at a speed of 67. Let f be a function defined on R (the set of all real
v km/h (v > u ). At what point on the shore should be numbers) such that f ′ (x) = 2010 (x − 2009) (x − 2010)2
land so that he reaches his house in the shortest (x − 2011)3 (x − 2012)4, ∀x ∈ R. If g is a function defined
possible time? (1983, 2M)
on R with values in the interval (0, ∞ ) such that
60. If ax2 + b / x ≥ c for all positive x where a > 0 and b > 0, f (x) = ln ( g (x)), ∀ x ∈ R, then the number of points in R
then show that 27ab2 ≥ 4c3 . (1982, 2M) at which g has a local maximum is…… (2010)

61. If x and y be two real variables such that x > 0 and xy = 1. 68. The maximum value of the expression
Then, find the minimum value of x + y. (1981, 2M)
1
is …… (2010)
sin 2 θ + 3 sin θ cos θ + 5 cos 2 θ
Integer Answer Type Questions 69. The maximum value of the function
62. A cylindrical container is to be made from certain solid f (x) = 2x3 − 15x2 + 36x − 48 on the set
material with the following constraints : It has a fixed A = { x | x2 + 20 ≤ 9x} is ……… . (2009)
inner volume of V mm3 , has a 2 mm thick solid wall and

Answers
Topic 1 Topic 2
1. (a) 2. (c) 3. (b) 4. (d) 1. (b) 2. (a) 3. (b) 4. (c)
5. (c) 6. (b) 7. (d) 8. (b) 5. (c) 6. (c) 7. (a) 8. (a)
9. (b) 10. (d) 11. (d) 12. (d) 9. (a) 10. (b) 11. (c) 12. (b)
13. (c) 14. (b, d) 15. (b, c) 13. (c) 14. (b) 15. (d) 16. (a, c)
16. H = φ, V = {1, 1 } 17. y − 2 = 0 17. (c, d) 18. (a, c) 19. x > −1
 1  1   1  1
1 3
18. a = – , b = – , c = 3 20. x ∈  − , 0 ∪  , ∞ , x ∈  − ∞, −  ∪  0, 
 2  2   2  2
2 4
19. 1 : 16 21. (d) 22. (c) 23. (c)
 2 a
20. y + x − 1 = 0 24. A → p, B → r 28. – ,
 a 3 
π − 3π
21. x + 2y = and x + 2y = 1 π  π 3 π  π 
2 2 31.  − 1 +  , − 1 +  
2 3  
3 2 6  6
22. (8)
rankershalt.com

Application of Derivatives 233

Topic 3 1 −5 1 5
44. (2, 1) 45. a = , b = ; f ( x ) = x 2 − x + 2
4 4 4 4
1. (b) 2. (b)
1 5  5 1 
46. P0 =  ,  and Q0 =  , 
Topic 4 2 4  4 2
1. (a) 2. (b) 3. (b) 4. (d) (b − b 2 − 1 ) 1
5. (b) 6. (c) 7. (c) 8. (c) 48. Maxima at x = and minima at x = (b + b 2 − 1 )
4 4
9. (a) 10. (a) 11. (b) 12. (d)
4 3
13. (c) 14. (c) 15. (c) 16. (c) 49. 2hk 50. 51. b ∈ ( −2, − 1 ) ∪ [1, ∞ ]
9
17. (c) 18. (c) 19. (d) 20. (c)
21. (d) 22. (a) 23. (d) 24. (b) 52. 2 x − 2y + 2 = 0, 2 x + 2y − 2 = 0
25. (b) 26. (a) 27. (c) 28. (b) 3 3 2
29. (b) 30. (b, c) 31. (a, d) 32. (a, b) 53. 6 : 6 + π 54. r sq. units 55. (0, 2)
4
33. (a, c) 34. (a, b) 35. (b, c) 36. (b, d)  3 3 ud
37. (a, b, c, d) 57. λ ∈  − ,  58. x = 0, y = 0 59. 61. (2)
 2 2 v − u2
2
38. (a) P → 4 Q → 1 R → 2 S → 3 39. (a)
62. (4) 63. (9) 64. (5) 65. (9)
40. (a) 41. (a) 42. 6 43. 5 units
66. (2) 67. (1) 68. (2) 69. (7)

Hints & Solutions


Topic 1 Equations of Tangent and Normal f (1) − f (−1) −2 − 0
(−1, f (−1)) is m = = = −1
1. Equation of given curve is 1 − (−1) 1+1
x dy
y= , x ∈ R, (x ≠ ± 3 ) …(i) Now, = 3x2 − 2x − 2
x −3
2 dx
[differentiating Eq. (i), w.r.t. ‘x’]
On differentiating Eq. (i) w.r.t. x, we get According to the question,
dy (x2 − 3) − x(2x) (− x2 − 3) dy
= = 2 =m
dx (x2 − 3)2 (x − 3)2 dx
It is given that tangent at a point (α , β ) ≠ (0, 0) on it is ⇒ 3x − 2x − 2 = −1 ⇒ 3x2 − 2x − 1 = 0
2

1
parallel to the line ⇒ (x − 1) (3x + 1) = 0 ⇒ x = 1 ,−
2x + 6 y − 11 = 0. 3
 1 
2 dy Therefore, set S = − , 1.
∴ Slope of this line = − =  3 
6 dx ( α , β)
α2 + 3 1 3. Given curve is y = x3 + ax − b …(i)
⇒ − =− passes through point P(1, − 5).
(α − 3)
2 2
3
⇒ 3α 2 + 9 = α 4 − 6 α 2 + 9 ∴ −5 =1+ a −b⇒b−a =6 …(ii)
and slope of tangent at point P(1, − 5) to the curve (i), is
⇒ α4 − 9 α2 = 0
⇒ α = 0, − 3, 3 dy
m1 = = [3x2 + a ](1, −5 ) = a + 3
⇒ α = 3 or − 3, [Q α ≠ 0] dx (1, −5 )
Now, from Eq. (i), Q The tangent having slope m1 = a + 3 at point P(1, − 5)
α 3 −3 1 1 is perpendicular to line − x + y + 4 = 0 , whose slope is
β= 2 ⇒ β= or = or −
α −3 9 −3 9 −3 2 2 m2 = 1.
According to the options,|6 α + 2 β | = 19 ∴ a + 3 = −1 ⇒ a = −4 [Q m1m2 = −1]
 1 Now, on substituting a = −4 in Eq. (ii), we get b = 2
at (α , β ) =  ± 3, ± 
 2 On putting a = −2 and b = 2 in Eq. (i), we get
y = x3 − 4x − 2
2. Given curve is y = f (x) = x3 − x2 − 2x ...(i)
Now, from option (2, − 2) is the required point which lie
So, f (1) = 1 − 1 − 2 = −2 on it.
and f (−1) = −1 − 1 + 2 = 0
4. The given curve is y = x2 − 5x + 5 …(i)
Since, slope of a line passing through (x1 , y1 ) and
y −y Now, slope of tangent at any point (x, y) on the curve is
(x2, y2) is given by m = tan θ = 2 1
x2 − x1 dy
= 2x − 5 …(ii)
∴Slope of line joining points (1, f (1)) and dx
[on differentiating Eq. (i) w.r.t. x]
rankershalt.com

234 Application of Derivatives

Q It is given that tangent is parallel to line Q x≥0


2 y = 4x + 1 ∴ x=
1
dy 3
So, = 2 [Q slope of line 2 y = 4x + 1 is 2] 3/ 2
dx  1
7 and So, y=  +7 [Q y = x3/ 2 + 7]
⇒ 2x − 5 = 2 ⇒ 2x = 7 ⇒ x =  3
2  1  1 3 / 2 
7
On putting x = in Eq. (i), we get Thus, the nearest point is  ,   + 7
3 3  
2
49 35 69 35 1 Now, the nearest distance
y= − +5 = − =−
4 2 4 2 4 2
2  3/ 2  2 3
Now, equation of tangent to the curve (i) at point  1 1  1  1  1
=  −  +  7 −   − 7 =   +  
 7 1  2 3   3   6  3
 ,−  and having slope 2, is
 2 4
1  7 1 1 1 3+4 7 1 7
y + = 2  x −  ⇒ y + = 2x − 7 = + = = =
4  2 4 36 27 108 108 6 3
29
⇒ y = 2x − …(iii) 6. Key Idea Angle between two curves is the angle between the
4
tangents to the curves at the point of intersection.
1 
On checking all the options, we get the point  , − 7
8  Given equation of curves are
satisfy the line (iii). y = 10 − x2 …(i)
5. The helicopter is nearest to the soldier, if the tangent to and y = 2 + x2 …(ii)
the path, y = x3/ 2 + 7, (x ≥ 0) of helicopter at point (x, y) is For point of intersection, consider
perpendicular to the line joining (x, y) and the position 10 − x2 = 2 + x2
1 
of soldier  , 7 . ⇒ 2x2 = 8
2 
⇒ x2 = 4
⇒ x=±2
Clearly, when x = 2 , then y = 6 (using Eq. (i)) and when
x = − 2, then y = 6
Thus, the point of intersection are (2, 6) and
(x, y) (−2, 6).
y=x3/2+7 Let m1 be the slope of tangent to the curve (i) and m2 be
the slope of tangent to the curve (ii)
(1/2, 7) dy dy
For curve (i) = −2x and for curve (ii) = 2x
dx dx
Q Slope of tangent at point (x, y) is
dy 3 1/ 2 ∴ At (2, 6), slopes m1 = − 4 and m2 = 4, and in that case
= x = m1 (let ) …(i)
dx 2 m2 − m1 4+4 8
1  |tan θ| = = =
and slope of line joining (x, y) and  , 7 is 1 + m1m2 1 − 16 15
2 
y−7
m2 = …(ii) At (−2, 6), slopes m1 = 4 and m2 = − 4 and in that case
1
x−
2 m2 − m1 −4 − 4 8
|tan θ| = = =
Now, m1 ⋅ m2 = −1 1 + m1m2 1 − 16 15
3 1/ 2  y − 7  7. We have, y2 = 6x
⇒ x   = −1 [from Eqs. (i) and (ii)]
2  x − (1 /2) dy dy 3
⇒ 2y =6 ⇒ =
3 1/ 2 x3/ 2 dx dx y
⇒ x = −1 [Q y = x3/ 2 + 7] 3
2 1
x− Slope of tangent at ( x1 , y1 ) is m1 =
2 y1
3 2 1 Also, 9x 2 + by 2 = 16
⇒ x = −x +
2 2 dy dy −9x
⇒ 3x2 + 2x − 1 = 0 ⇒ 18x + 2by =0 ⇒ =
dx dx by
⇒ 3x2 + 3x − x − 1 = 0 −9x1
⇒ 3x(x + 1) − 1(x + 1) = 0 Slope of tangent at ( x1 , y1 ) is m2 =
1 by1
⇒ x = , −1 Since, these are intersection at right angle.
3
rankershalt.com

Application of Derivatives 235

27x1  2π 
∴ m1m2 = − 1 ⇒ =1 which passes through 0,  .
by12  3
27x1
⇒ =1 [Q y12 = 6x1] 10. Given equation of curve is
6bx1 x2 + 2xy − 3 y2 = 0 …(i)
9
⇒ b= On differentiating w.r.t x, we get
2 x+ y
8. Given curve is 2x + 2xy′ + 2 y − 6 yy′ = 0 ⇒ y′ =
3y − x
y(x − 2)(x − 3) = x + 6 …(i)
Put x = 0 in Eq. (i), we get At x = 1, y = 1, y′ = 1
y(− 2) (− 3) = 6 ⇒ y = 1  dy
i.e.   =1
 dx (1, 1)
So, point of intersection is (0, 1).
x+6 Equation of normal at (1, 1) is
Now, y =
(x − 2)(x − 3) 1
y − 1 = − (x − 1) ⇒ y − 1 = − (x − 1)
dy 1 (x − 2)(x − 3) − (x + 6)(x − 3 + x − 2) 1
⇒ =
dx (x − 2)2(x − 3)2 ⇒ x+ y=2 …(ii)
 dy 6 + 30 36 On solving Eqs. (i) and (ii) simultaneously, we get
⇒   = = =1
 dx ( 0, 1) 4 ×9 36 ⇒ x2 + 2x(2 − x) − 33(2 − x)2 = 0
⇒ x + 4x − 2x2 − 3(4 + x2 − 4x) = 0
2
∴ Equation of normal at (0, 1) is given by
⇒ − x2 + 4x − 12 − 3x2 + 12x = 0
−1
y−1 = (x − 0) ⇒ − 4x2 + 16x − 12 = 0
1
⇒ 4x2 − 16x + 12 = 0
⇒ x+ y−1 =0
⇒ x2 − 4x + 3 = 0
 1 1
which passes through the point  ,  . ⇒ (1 − 1)(x − 3) = 0
 2 2
∴ x = 1, 3
1 + sin x  π Now, when x = 1, then y = 1
9. We have, f (x) = tan − 1 , x ∈ 0, 
1 − sin x  2 and when x = 3, then y = − 1
2 ∴ P = (1, 1) and Q = (3, − 1)
 x x
 cos + sin  Hence, normal meets the curve again at (3, –1) in fourth
 2 2
⇒ f (x) = tan − 1 2 quadrant.
 x x
 cos − sin  Alternate Solution
 2 2 Given, x2 + 2xy − 3 y2 = 0
 x x ⇒ (x − y)(x + 3 y) = 0
 cos + sin 
−1
= tan  2 2  ⇒ x − y = 0 or x + 3 y = 0
 cos x − sin x  Equation of normal at (1, 1) is
 2 2
y − 1 = − 1(x − 1) ⇒ x + y − 2 = 0
 x x x π
Q cos > sin for 0 < <  It intersects x + 3 y = 0 at (3, –1) and hence normal meets
 2 2 2 4 the curve in fourth quadrant.
 x
−1
 1 + tan  x + y = 2 Y'
y=x
= tan  2  x + 3y = 0
 1 − tan x 
 2 (1,1)
X' X
  π x  π x
= tan − 1 tan  +   = + O
  4 2  4 2 (3,–1)
1  π 1
⇒ f ′ (x) = ⇒ f′   = Y'
2  6 2
π 11. Given, y + 3x = 12 y
3 2
...(i)
Now, equation of normal at x = is given by
6 On differentiating w.r.t. x, we get
  π  π
⇒ 3 y2
dy
+ 6x = 12
dy
 y − f    = − 2  x − 
 6  6 dx dx
π π  dy 6x
   π π π 4π π  ⇒ =
⇒  y −  = − 2  x −  Q f   = + = = dx 12 − 3 y2
 3   
6   6  4 12 12 3 
rankershalt.com

236 Application of Derivatives

dx 12 − 3 y2 On putting value of h in Eq. (ii), we get


⇒ =
dy 6x 4(− 2k)2 + 9 k2 = 1
dx ⇒ 16k2 + 9k2 = 1
For vertical tangent, =0
dy ⇒ 25k2 = 1
⇒ 12 − 3 y2 = 0 ⇒ y=±2 1
⇒ k2 =
4 25
On putting, y = 2 in Eq. (i), we get x = ± and again 1
3 ⇒ k=±
putting y = − 2 in Eq. (i), we get 3x = − 16, no real
2 5
solution. Thus, the point, where the tangents are parallel to
 4   2 1 2 1
So, the required point is  ± , 2 . 8x = 9 y are  − ,  and  , −  .
 3   5 5 5 5
dy Therefore, options (b) and (d) are the answers.
12. Slope of tangent y = f (x) is = f ′ (x)(3 , 4)
dx 1
15. Given, xy = 1 ⇒ y =
Therefore, slope of normal x
1 1 dy 1
=− =− ⇒ =− 2
f ′ (x)(3 , 4) f ′ (3) dx x
1  3π  Thus, slope of normal = x2 (which is always positive) and
But − = tan   [given] a
f ′ (3)  4 it is given ax + by + c = 0 is normal, whose slope = − .
b
−1  π π
⇒ = tan  +  = − 1 ⇒
a
− > 0 or
a
<0
f ′ (3)  2 4
b b
∴ f ′ (3) = 1 Hence, a and b are of opposite sign.
13. Given, x = a (cos θ + θ sin θ ) 16. Given, y3 − 3xy + 2 = 0
and y = a (sin θ − θ cos θ )
On differentiating w.r.t. x, we get
dx
∴ = a (− sin θ + sin θ + θ cos θ ) = a θ cos θ dy dy
dθ 3 y2 − 3x − 3y = 0
dy dx dx
and = a (cos θ − cos θ + θ sin θ ) dy dy 3y
dθ ⇒ (3 y2 − 3x) = 3 y ⇒ = 2
dy dy dx dx 3 y − 3x
= a θ sin θ ⇒ = tan θ
dθ dx For the points where tangent is horizontal, the slope of
Thus, equation of normal is tangent is zero.
y − a (sin θ − θ cos θ ) − cos θ dy 3y
= i.e. =0 ⇒ =0
x − a (cos θ + θ sin θ ) sin θ dx 3 y − 3x
2

⇒ − x cos θ + a θ sin θ cos θ + a cos 2 θ ⇒ y = 0 but y = 0 does not satisfy the given equation of
= y sin θ + θ a sin θ cos θ − a sin 2 θ the curve, therefore y cannot lie on the curve.
⇒ x cos θ + y sin θ = a So, H =φ [null set]
whose distance from origin is dy
For the points where tangent is vertical, =∞
|0 + 0 − a | dx
=a
cos 2 θ + sin 2 θ y
⇒ =∞
y2 − x
14. Given, 4x2 + 9 y2 = 1 …(i)
⇒ y2 − x = 0
On differentiating w.r.t. x, we get ⇒ y2 = x
dy
8x + 18 y =0 On putting this value in the given equation of the curve,
dx we get
dy 8x 4x
⇒ =− =− y3 − 3 ⋅ y2 ⋅ y + 2 = 0
dx 18 y 9y
⇒ − 2 y3 + 2 = 0
The tangent at point (h , k) will be parallel to 8x = 9 y, ⇒ y − 1 = 0 ⇒ y3 = 1
3
then
⇒ y=1, x=1
4h 8
− = Then, V = {1, 1}
9k 9
⇒ h = − 2k 17. As | f (x1) − f (x2)| ≤ (x1 − x2)2, ∀x1 , x2 ∈ R
Point (h , k) also lies on the ellipse. ⇒ | f (x1 ) − f (x2)| ≤ |x1 − x2|2 [as x2 =|x|2 ]
∴ 4h 2 + 9k2 = 1 ...(ii)
rankershalt.com

Application of Derivatives 237

f (x1 ) − f (x2) 19. Let any point P1 on y = x3 be (h , h3 ).


∴ ≤ |x1 − x2|
x1 − x2 Then, tangent at P1 is
 f (x1 ) − f (x2)  y − h3 = 3h 2(x − h ) ...(i)
⇒ lim   ≤ lim |x1 − x2|
x1 → x2  x1 − x2  x1 → x2 It meets y = x at P2.
3

⇒ | f ′ (x1 )| ≤ 0, ∀x1 ∈ R On putting the value of y in Eq. (i), we get


∴ | f ′ (x)| ≤ 0, which shows| f ′ (x)| = 0 x3 − h3 = 3h 2(x − h )
[as modulus is non negative or| f ′ (x)| ≥ 0] ⇒ (x − h ) (x + xh + h 2) = 3h 2(x − h )
2

∴ f ′ (x) = 0 or f (x) is constant function. ⇒ x2 + xh + h 2 = 3h 2 or x = h


⇒ Equation of tangent at (1, 2) is ⇒ x2 + xh − 2h 2 = 0
y−2 ⇒ (x − h ) (x + 2h ) = 0
= f ′ (x)
x−1 ⇒ x = h or x = − 2h
or y−2 =0 [Q as f ′ (x) = 0] Therefore, x = − 2h is the point P2,
⇒ y − 2 = 0 is required equation of tangent. which implies y = − 8h3
18. Given, y = ax3 + bx2 + cx + 5 touches X-axis at P(−2, 0) Hence, point P2 ≡ (−2h , − 8h3 )
which implies that X-axis is tangent at (–2, 0) and the Again, tangent at P2 is y + 8h3 = 3 (−2h )2(x + 2h ).
curve is also passes through (–2, 0). It meets y = x3 at P3
The curve cuts Y-axis at (0, 5) and gradient at this point ⇒ x + 8h = 12h 2(x + 2h )
3 3
is given 3, therefore at (0, 5) slope of the tangent is 3.
⇒ x2 − 2hx − 8h 2 = 0
dy
Now, = 3ax2 + 2bx + c ⇒ (x + 2h ) (x − 4h ) = 0 ⇒ x = 4h ⇒ y = 64h3
dx
Therefore, P3 ≡ (4h , 64h3 )
Since, X-axis is tangent at (–2, 0).
Similarly, we get P4 ≡ (− 8h , − 83 h3 )
∴  dy  =0
 dx x = − 2 Hence, the abscissae are h, –2h, 4h, –8h,…, which form
a GP.
⇒ 0 = 3a (−2)2 + 2b (−2) + c Let D ′ = ∆ P1 P2 P3 and D ′ ′ = ∆ P2 P3 P4
⇒ 0 = 12a − 4b + c …(i) h h3 1
Again, slope of tangent at (0, 5) is 3. 1
−2h −8h3 1
dy 2
∴ =3 D ′ ∆ P1P2 P3 4h 64h3 1
dx ( 0, 5 ) = =
D ′ ′ ∆ P2 P3 P4 −2 h −8h3 1
⇒ 3 = 3a (0) + 2b (0) + c
2 1
4h 64h3 1
⇒ 3=c …(ii) 2
−8h −512h3 1
Since, the curve passes through (–2, 0).
h h3 1
0 = a (−2)3 + b(−2)2 + c(−2) + 5 1
−2h −8h3 1
⇒ 0 = − 8a + 4b − 2c + 5 … (iii) 2
4h 64h3 1
From Eqs. (i) and (ii), =
12a − 4b = − 3 … (iv) h h3 1
1
× (−2) × (−8) −2h −8h3 1
2
4h 64h3 1
From Eqs. (ii) and (iii),
−8 a + 4 b = 1 … (v) D′ 1
⇒ = = 1 : 16 which is the required ratio.
On adding Eqs. (iv) and (v), we get D′ ′ 16
4a = − 2 ⇒ a = − 1 / 2 20. Given, y = (1 + x)y + sin −1 (sin 2 x)
On putting a = − 1 / 2 in Eq. (iv), we get Let y = u + v, where u = (1 + x)y , v = sin −1 (sin 2 x).
12(−1 / 2) − 4b = − 3 On differentiating w.r.t.x, we get
dy du dv
⇒ − 6 − 4b = − 3 = + …(i)
dx dx dx
⇒ − 3 = 4b
Now, u = (1 + x)y
⇒ b = − 3 /4
On taking logarithm both sides, we get
∴ a = − 1 / 2 , b = − 3 / 4 and c = 3
log e u = y log e (1 + x)
rankershalt.com

238 Application of Derivatives

1 du y dy y−0 1 π
⇒ = + {log e (1 + x)} and = − ⇒ 2y = − x +
u dx 1 + x dx x + 3π / 2 2 2
 y  3π

du
= (1 + x)y  +
dy
log e (1 + x) …(ii) and 2y = − x −
2
dx 1 + x dx  π
⇒ x + 2y =
Again, v = sin −1 (sin 2 x) 2
⇒ sin v = sin x
2 3π
and x + 2y = −
dv 2
⇒ cos v = 2 sin x cos x
dx are the required equations of tangents.

22. Slope of tangent at the point (x1 , y1 ) is 



dv
=
1 dy
(2 sin x cos x)  .
dx cos v  dx ( x
1 , y1 )
dv 2 sin x cos x 2 sin x cos x
⇒ = = …(iii) Given curve, ( y − x5 )2 = x (1 + x2)2
dx 1 − sin 2 v 1 − sin 4 x  dy 
⇒ 2 ( y − x5 )  − 5x4 = (1 + x2)2 + 2x (1 + x2) ⋅ 2x
From Eq. (i),  dx 
dy  y dy  2 sin x cos x Put x = 1 and y = 3, dy /dx = 8
= (1 + x)y  + log e (1 + x) +
dx  1 + x dx  1 − sin 4 x

dy y(1 + x)
y −1
+ 2 sin x cos x / 1 − sin 4 x
Topic 2 Increasing and Decreasing
⇒ = Functions
dx 1 − (1 + x)y log e (1 + x)
1. Let the thickness of layer of ice is x cm, the volume of
At x = 0,
spherical ball (only ice layer) is
y = (1 + 0)y + sin −1sin (0) = 1 4
1 −1 V = π [(10 + x)3 − 103 ] …(i)
dy 1 (1 + 0) + 2 sin 0 ⋅ cos 0 / (1 − sin 4 0) 3
∴ =
dx 1 − (1 + 0)1 log e (1 + 0) On differentiating Eq. (i) w.r.t. ‘t’, we get
dV 4 dx
dy = π (3(10 + x)2) = − 50 [given]
⇒ =1 dt 3 dt
dx
[− ve sign indicate that volume is decreasing as
Again, the slope of the normal is time passes].
1 dx
m=− = −1 ⇒ 4π (10 + x)2 = − 50
dy / dx dt
Hence, the required equation of the normal is At x = 5 cm
dx
y − 1 = (−1) (x − 0) [4π (10 + 5)2] = − 50
dt
i.e. y+ x−1 =0 dx 50 1 1
⇒ =− =− =− cm /min
21. Given, y = cos (x + y) dt 225(4π ) 9(2π ) 18π
 dy  dy So, the thickness of the ice decreases at the rate of
⇒   = − sin (x + y) ⋅ 1 +  …(i)
 dx  dx 1
cm /min.
Since, tangent is parallel to x + 2 y = 0 , 18π
dy 1 2. The given functions are
then slope =−
dx 2 f (x) = ex − x,
1  1
From Eq. (i), − = − sin (x + y) ⋅ 1 −  and g (x) = x2 − x, ∀ x ∈ R
2  2
Then, h (x) = ( fog )(x) = f ( g (x))
⇒ sin (x + y) = 1, which shows cos (x + y) = 0.
Now, h′ (x) = f ′ ( g (x)) ⋅ g′ (x)
∴ y=0 2
π 3π = (eg( x ) − 1) ⋅ (2x − 1) = (e( x − x ) − 1) (2x − 1)
⇒ x+ y= or −
2 2 = (ex( x − 1) − 1) (2x − 1)
π 3π Q It is given that h (x) is an increasing function, so
∴ x= or −
2 2 h′ (x) ≥ 0
π   3π  ⇒ (ex( x − 1) − 1)(2x − 1) ≥ 0
Thus, required points are  , 0 and  − , 0
2   2  Case I (2x − 1) ≥ 0 and (ex( x − 1) − 1) ≥ 0
∴ Equation of tangents are ⇒
1
x ≥ and x(x − 1) ≥ 0
y−0 1 2
=− ⇒ x ∈ [1 / 2, ∞ ) and x ∈ (− ∞ , 0] ∪ [1, ∞ ), so x ∈ [1, ∞ )
x − π /2 2
rankershalt.com

Application of Derivatives 239

Case II (2x − 1) ≤ 0 and [ex( x − 1) − 1] ≤ 0 Now, for φ(x) to be increasing,


1  1
⇒ x ≤ and x(x − 1) ≤ 0 ⇒ x ∈  −∞ , and x ∈ [0, 1] φ′ (x) ≥ 0
2  2 
⇒ f ′ (x) − f ′ (2 − x) ≥ 0 [using Eq. (i)]
 1 ⇒ f ′ (x) ≥ f ′ (2 − x) ⇒ x > 2 − x
So, x ∈ 0,
 2 
[Q f′ is a strictly increasing function]
 1 ⇒ 2x > 2 ⇒ x>1
From, the above cases, x ∈ 0, ∪ [1, ∞ ). Thus, φ(x) is increasing on (1, 2).
 2 
Similarly, for φ(x) to be decreasing,
φ′ (x) ≤ 0
3. Key Idea Use formula : ⇒ f ′ (x) − f ′ (2 − x) ≤ 0
1 [using Eq. (i)]
Volume of cone = πr 2h, where r = radius and h = height of the
3 ⇒ f ′ (x) ≤ f ′ (2 − x)
cone. ⇒ x<2 − x [Q f′ is a strictly increasing
Given, semi-vertical angle of right circular cone function]
 1 ⇒ 2x < 2
= tan −1   ⇒ x<1
 2
Thus, φ(x) is decreasing on (0, 1).
 1
Let α = tan −1  
 2 5. Given that function,
f (x) = x3 − 3 (a − 2) x2 + 3ax + 7, for some a ∈ R is
1
⇒ tan α = increasing in (0, 1] and decreasing in [1, 5).
2
r 1 r f′ (1) = 0 [Q tangent at x = 1 will be
⇒ = [from fig. tan α = ] parallel to X-axis]
h 2 h
⇒ (3x2 − 6(a − 2) x + 3a )x = 1 = 0
1
⇒ r= h …(i) ⇒ 3 − 6(a − 2) + 3a = 0
2
⇒ 3 − 6a + 12 + 3a = 0
r
⇒ 15 − 3a = 0
⇒ a =5
So, f (x) = x3 − 9x2 + 15x + 7
l h ⇒ f (x) − 14 = x3 − 9x2 + 15x − 7
α ⇒ f (x) − 14 = (x − 1) (x2 − 8x + 7) = (x − 1) (x − 1)(x − 7)
f (x) − 14
⇒ = (x − 7) …(i)
(x − 1)2
1 2
Q Volume of cone is (V ) = πr h f (x) − 14
Now, = 0, (x ≠ 1)
3 (x − 1)2
2
1 1  1 ⇒ x− 7 =0
∴ V = π  h (h ) = πh3 [from Eq. (i)] [from Eq. (i)]
3 2  12 ⇒ x=7

On differentiating both sides w.r.t. ‘t’, we get 6. We have,


x (d − x)
dV 1 dh f (x) = − 2
= π (3h 2) (a + x )
2 2 1/ 2
(b + (d − x)2)1/ 2
dt 12 dt
dh 4 dV Differentiating above w.r.t. x, we get
⇒ = 1 2x
dt πh 2 dt (a 2 + x2)1/ 2 − x
dh 4 dV 2 (a 2 + x2)1/ 2
⇒ = ×5 [Q given = 5 m3 /min] f ′ (x) =
dt πh 2 dt (a 2 + x2)
2(d − x )(−1)
Now, at h = 10 m, the rate at which height of water (b2 + (d − x ) 2 )1/ 2 (−1) − (d − x )
dh 2(b2 + (d − x ) 2 )1/ 2
level is rising = −
dt h = 10 ( b2 + ( d − x ) 2 )
4 1 [by using quotient rule of derivative]
= ×5= m/min a 2 + x2 − x2 b2 + (d − x)2 − (d − x)2
π (10)2 5π = +
(a 2 + x2)3/ 2 (b2 + (d − x)2)3/ 2
4. Given, φ (x) = f (x) + f (2 − x), ∀ x ∈ (0, 2)
a2 b2
⇒ φ′ (x) = f ′ (x) − f ′ (2 − x) …(i) = + > 0,
(a 2 + x2)3/ 2 (b2 + (d − x)2)3/ 2
Also, we have f ′ ′ (x) > 0 ∀ x ∈ (0, 2)
⇒ f ′ (x) is a strictly increasing function ∀ x ∈R
∀ x ∈ (0, 2). Hence, f (x) is an increasing function of x.
rankershalt.com

240 Application of Derivatives

π Option (b) Let g (x) = log e (1 + x) − x, 0 < x < 1


7. Given, g (u ) = 2 tan −1 (eu ) − for u ∈ (−∞ , ∞ )
2 1 x
g′ (x) = −1 = − < 0 for 0 < x < 1
π π 1+ x 1+ x
g (− u ) = 2 tan −1 (e− u ) − = 2 (cot−1 (eu )) −
2 2 ⇒ g (x) decreases for 0 < x < 1
π −1 u  π
⇒ g (x) < g (0) for 0 < x < 1
= 2  − tan (e ) −
2  2
⇒ log e (1 + x) − x < 0 for 0 < x < 1
= π / 2 − 2 tan −1 (eu ) = − g (u )
or log e (1 + x) < x for 0 < x < 1
∴ g (− u ) = − g (u )
Therefore, option (b) is the answer.
⇒ g (u ) is an odd function.
Option sin x > x
We have, g (u ) = 2 tan −1 (eu ) − π 2
Let h (x) = sin x − x
2 eu
g′ (u ) = h′ (x) = cos x − 1
1 + e 2u
For x ∈(0, 1), cos x − 1 < 0
g′ (u ) > 0, ∀ x ∈ R [Q eu > 0]
⇒ h (x) is decreasing function.
So, g′ (u ) is increasing for all x ∈ R.
⇒ h (x) < h (0)
8. Given, f (x) = x3 + bx2 + cx + d ⇒ sin x − x < 0
⇒ f ′ (x) = 3x2 + 2bx + c ⇒ sin x < x,which is not true.
As we know that, if ax2 + bx + c > 0, ∀x, then a > 0 and Option (d) p(x) = log x − x
D < 0. 1
p′ (x) = − 1 > 0, ∀x ∈ (0, 1)
Now, D = 4b2 − 12c = 4(b2 − c) − 8c x
[where, b2 − c < 0 and c > 0] Therefore, p′ (x) is an increasing function.
∴ D = (–ve) or D < 0 ⇒ p(0) < p(x) < p(1)
⇒ f ′ (x) = 3x2 + 2bx + c > 0 ∀x ∈ (− ∞ , ∞ ) ⇒ − ∞ < log x − x < −1
[as D < 0 and a > 0] ⇒ log x − x < 0
Hence, f (x) is strictly increasing function. ⇒ log x < x
9. Let f (x) = 3 sin x − 4 sin x = sin 3x
3
Therefore, option (d) is not the answer.
The longest interval in which sin x is increasing is of 12. Let f (x) = ∫ ex (x − 1) (x − 2) dx
length π .
⇒ f ′ (x) = ex (x − 1) (x − 2)
So, the length of largest interval in which f (x) = sin 3x is
π + − +
increasing is .
3 1 2
10. Given, f (x) = xex (1 − x ) ∴ f ′ (x) < 0 for 1 < x < 2
⇒ f ′ (x) = e x (1 − x )
+ xe x (1 − x )
(1 − 2x) ⇒ f (x) is decreasing for x ∈(1, 2).
=e x (1 − x )
[1 + x (1 − 2x)] 13. Given, f (x) = sin 4 x + cos 4 x
= ex (1 − x ) (1 + x − 2x2) On differentiating w.r.t. x, we get
x (1 − x )
= −e (2x − x − 1)
2
f ′ (x) = 4 sin3 x cos x − 4 cos3 x sin x
x (1 − x ) = 4 sin x cos x (sin 2 x − cos 2 x)
= −e (x − 1) (2x + 1)
 1  = 2 sin 2x (− cos 2x)
which is positive in  − , 1 .
 2  = − sin 4x
 1  Now, f ′ (x) > 0 , if sin 4x < 0
Therefore, f (x) is increasing in − , 1 .
 2 
⇒ π < 4x < 2π
11. π π
PLAN Inequation based upon uncompatible function. This type ⇒ <x< …(i)
of inequation can be solved by calculus only. 4 2
Option (a) Let f (x) = ex − 1 − x. ⇒ Option (a) is not proper subset of Eq. (i), so it is not
then f ′ (x) = e − 1 > 0, ∀x ∈ (0, 1)
x correct.
π 3π
⇒ f (x) increase in (0, 1) Now, <x<
4 8
⇒ f (x) > f (0) for 0 < x < 1
Since, option (b) is the proper subset of Eq. (i), so it is
⇒ ex − 1 − x > 0 or ex > 1 + x for 0 < x < 1 correct.
rankershalt.com

Application of Derivatives 241

14. Given, g (x) =


x
, where 0 < x ≤ 1 From Eqs. (i) and (iv), f ′ (x) < 0
tan x ∴ f (x) is decreasing for x ∈ (0, ∞ ).
Now, g (x) is continuous in [0, 1] and differentiable in
]0, 1[. 16. Let F (x) = h (x) − h (1) = f { g (x)} − f { g (1)}
For 0 < x < 1, ∴ F ′ (x) = f ′ { g (x)} ⋅ g ′ (x)
tan x − x sec x 2 = (+ ) (− ) = − ve
g′ (x) =
tan 2 x [since, f (x) is an increasing function f ′ ( g (x)) is + ve and
Again, H (x) = tan x − x sec2 x, 0 ≤ x ≤ 1 g (x) is decreasing function g ′ ( f (x)) is −ve ]
Since, f ′ (x) is –ve.
Now, H (x) is continuous in [0, 1] and differentiable in
]0, 1[. ∴ f (x) is decreasing function.
For 0 < x < 1, H (x) = tan x − x sec2x, 0 ≤ x ≤ 1 When 0 ≤ x<1
⇒ H ′ (x) = sec x − sec x − 2x sec x tan x
2 2 2 ⇒ h (x) − h (1) = + ve
= − 2x sec x tan x < 0
2 When x ≥ 1,
Hence, H (x) is decreasing function in [0, 1]. ⇒ h (x) − h (1) = − ve
Thus, H (x) < H (0) for 0 < x < 1 Hence, for x > 0,
⇒ H (x) < 0 for 0 < x<1 h (x) − h (1) is neither always positive nor always
⇒ g′ (x) < 0 for 0 < x<1 negative, so it is not strictly increasing throughout.
⇒ g (x) is decreasing function in (0, 1]. Therefore, option (d) is the answer.
dy
Therefore, g (x) =
x
is a decreasing function in 17. f ′ (x) > 2 f (x) ⇒ > 2dx
tan x y
f ( x) x
0 < x ≤ 1. dy
Also, g (x) < g (0) for 0 < x ≤ 1
⇒ ∫ y
> 2∫ dx
1 0


x
< 1 for 0 < x ≤ 1 ln( f (x)) > 2x
tan x ∴ f (x) > e2x
⇒ x < tan x for 0 < x≤1 Also, as f ′ (x) > 2 f (x)
0 < x≤1 ∴ f ′ (x) > 2c2x > 0
f (x) = 
x /sin x for
Now, let  1
 1 for x=0 − t + 
 t
x e
Now, f is continuous in [0, 1] and differentiable in ]0, 1[. 18. Given, f (x) = ∫ 1 dt
x
t
For 0 < x < 1,  1 1 
− x +  − + x 
sin x − x cos x (tan x − x) cos x e  x 
 −1  e x

f ′ (x) = = > 0 for 0 < x < 1 f ′ (x) = 1 ⋅ −  2
sin 2 x sin 2 x x  x  1 /x
⇒ f (x) increases in [0, 1].  1
− x + 
 1
− x + 
 1
− x + 
 x  x  x
x e e 2e
Thus, f (x) = increases in 0 < x ≤ 1. = + =
sin x x x x
Therefore, option (c) is the answer. As f ′ (x) > 0, ∀x ∈ (0, ∞ )
log (π + x) ∴ f (x) is monotonically increasing on (0, ∞ ).
15. Given, f (x) =
log(e + x) ⇒ Option (a) is correct and option (b) is wrong.
1 1  1  1
− t +  − t + 
log (e + x) ⋅ − log (π + x) ⋅ x e  t  t
π+x e+ x  1 1/ x e
f ′ (x) = …(i) Now, f (x) + f   = ∫ dt + ∫ dt
 x 1/x t x t
[log (e + x)]2
When x > 0, π + x > e + x = 0, ∀x ∈ (0, ∞ )
 1
− t + 
∴ log (π + x) > log (e + x) ... (ii) 2x e t

1 1 Now, let g (x) = f (2x ) = ∫ dt


2− x
and > ... (iii) t
e+ x π + x  1
− t + 
2− x  t
−x e
On multiplying Eqs. (ii) and (iii), we get g (− x) = f (2 ) = ∫ dt = − g (x)
2x t
log (π + x) log (e + x)
> …(iv) ∴ f (2x ) is an odd function.
e+ x π+x
rankershalt.com

242 Application of Derivatives

19. Given, h (x) = f (x) − f (x)2 + f (x)3 ⇒ f (x) = 0 for some x ∈ (1, e2)
On differentiating w.r.t. x, we get ∴ I is correct
h ′ (x) = f ′ (x) − 2 f (x) ⋅ f ′ (x) + 3 f 2(x) ⋅ f ′ (x) 1 1
f ′ (x) = 1 + − ln x − 1 = − ln x
= f ′ (x)[1 − 2 f (x) + 3 f (x)] 2 x x
 2 1 f ′ (x) > 0 for (0, 1)
= 3 f ′ (x) ( f (x))2 − f (x) +
 3 3  f ′ (x) < 0 for (e, ∞ )
 1 1 1
2 ∴ P and Q are correct, II is correct, III is incorrect.
= 3 f ′ (x)  f (x) −  + −  −1 1
 3  3 9 f ′′ (x) = 2 −

x x
 1
2
3 − 1
= 3 f ′ (x)  f (x) −  +  f ′′ (x) < 0 for (0, ∞ )
 3 9 
 ∴ S, is correct, R is incorrect.
 1
2
2 IV is incorrect.
= 3 f ′ (x)  f (x) −  + 
 3  9 lim f (x) = − ∞

x→∞
NOTE h ′ ( x ) < 0, if f ′ ( x ) < 0 and h ′ ( x ) > 0, if f ′ ( x ) > 0 lim f ′ (x) = − ∞
x→∞
Therefore, h (x) is an increasing function, if f (x) is
lim f ′′ (x) = 0
increasing function and h (x) is decreasing function, if x→∞
f (x) is decreasing function. ∴ ii, iii, iv are correct.
Therefore, options (a) and (c) are correct answers.
22. (d) 23. (c)
20. Let f (x) = log (1 + x) − x 24.(c)
1 x π π
⇒ f ′ (x) = −1 = − 25.
d x
(x + sin x) = 1 + cos x = 2 cos 2 > 0 for − < x < .
1+ x 1+ x dx 2 2 2
⇒ f ′ (x) > 0 Therefore, x + sin x is increasing in the given interval.
when −1 < x<0 Therefore, (A)→ (p) is the answer.
d
and f ′ (x) < 0 Again, (sec x) = sec x tan x which is > 0 for 0 < x < π / 2
dx
when x>0 −π
and < 0 for < x<0
∴ f (x) is increasing for −1 < x < 0. 2
⇒ f (x) < f (0) Therefore, sec x is neither increasing nor decreasing in
⇒ log (1 + x) < x the given interval. Therefore, (B)→(r) is the answer.
Again, f (x) is decreasing for x > 0. 3x (x + 1)
26. Let f (x) = sin x + 2x −
π
⇒ f (x) < f (0)
On differentiating w.r.t. x, we get
⇒ log (1 + x) < x
(6x + 3)
∴ log (1 + x) ≤ x, ∀x > − 1 ⇒ f ′ (x) = cos x + 2 −
π
 2x2 − log x, x >0 6  π
21. Here, y= 2 ⇒ f ′ ′ (x) = − sin x − < 0, ∀x ∈ 0,
2 x − log (− x), x <0 π  2 
 1  π
dy 4x − x , x > 0 ∴ f ′ (x) is decreasing for all x ∈ 0,
 2 
.
⇒ =
dx 4x − 1 , x < 0
⇒ f ′ (x) > 0 [ Q x < π /2]
 x
⇒ f ′ (x) > f ′ (π / 2)
4x2 − 1 (2x − 1) (2x + 1)
= , x ∈ R − {0} = ∴ f (x) is increasing.
x x
 1  1  Thus, when x ≥ 0, f (x) ≥ f (0)
∴ Increasing when x ∈  − , 0 ∪  , ∞ 3x(x + 1)
 2  2  ⇒ sin x + 2x − ≥0
π
 1  1
and decreasing when x ∈  −∞ , −  ∪ 0,  . 3x (x + 1)
 2  2 ⇒ sin x + 2x ≥
π
Solutions. (22-24)
27. Let f (x) = sin (tan x) − x
f (x) = x + ln x − x ln x
f ′ (x) = cos (tan x) ⋅ sec2x − 1
f (1) = 1 > 0
= cos (tan x) (1 + tan 2 x) − 1
f (e2) = e2 + 2 − 2e2 = 2 − e2 < 0
rankershalt.com

Application of Derivatives 243

= tan 2 x {cos (tan x)} + cos (tan x) − 1 (ax + 1) eax , if x < 0


2 Hence, f ′ (x) =  1, if x = 0
tan x 1 + 2ax − 3x2,
> tan 2 x cos (tan x) − if x > 0
2
 Now, we can say without solving that, f ′ (x) is
x2 
Q2 (1 − cos x) < x , x ≠ 0 ⇒ cos x > 1 − 2 
2
continuous at x = 0 and hence on R. We have,
 tan 2 x  f ′ ′ (x) = ae + a (ax + 1) e , if x < 0
ax ax
 ⇒ cos (tan x) > 1 −  2a − 6x, if x > 0
 2 
f ′ (x) − f ′ (0)
 1 and Lf ′ ′ (0) = lim
f ′ (x) > tan 2 x cos (tan x) − x → 0− x−0
 2 
> tan x [cos (tan x) − cos (π / 3)] > 0
2 (ax + 1)eax − 1
= lim
− x
⇒ f (x) is increasing function, for all x ∈ [0, π / 4] x→ 0

As f (0) = 0 ⇒ f (x) ≥ 0, for all x ∈ [0, π / 4]  eax − 1 


= lim aeax +
⇒ sin (tan x) ≥ x x → 0−  x 
28. Given, −1 ≤ p ≤ 1 eax − 1
= lim aeax + a ⋅ lim
− − ax
Let f (x) = 4x3 − 3x − p = 0 x→ 0 x→ 0

1 3 = ae0 + a (1) = 2a
Now, f (1 / 2) = − − p = − 1 − p ≤ 0 [Q p ≥ − 1] f ′ (x) − f ′ (0)
2 2 and Rf ′ ′ (0) = lim
Also, f (1) = 4 − 3 − p = 1 − p ≥ 0 [Q p ≤ 1] x → 0+ x+0
∴ f (x) has atleast one real root between [1 /2, 1]. (1 + 2ax − 3x2) − 1
= lim
Also, f ′ (x) = 12x2 − 3 > 0 on [1 / 2, 1] x → 0+ x
⇒ f ′ (x) increasing on [1 /2, 1] 2ax − 3x2
= lim = lim 2a − 3x = 2a
+
⇒ f has only one real root between [1 /2, 1]. x→ 0 x x → 0+

To find a root, we observe f (x) contains 4x − 3x, which is


3
Therefore, Lf ′ ′ (0) = Rf ′ ′ (0) = 2a
multiple angle formula for cos 3θ. a (ax + 2) eax , if x < 0
∴ Put x = cos θ Hence, f ′ ′ (x) = 2a , if x = 0
2a − 6x, if x > 0
⇒ 4 cos3 θ − 3 cos θ − p = 0
⇒ p = cos 3 θ ⇒ θ = (1 / 3) cos −1 ( p) Now, for x < 0, f ′ ′ (x) > 0, if ax + 2 > 0
1  ⇒ For x < 0, f ′ ′ (x) > 0, if x > − 2 / a
∴ Root is cos  cos −1 ( p) .
3  2
⇒ f ′ (x) > 0, if − < x < 0
a
29. NOTE This type is asked in 1983 and repeat after 13 years.
and for x > 0, f ′ ′ (x) > 0, if 2a − 6x > 0
At x = 0, LHL = lim f (x) = lim xeax = 0
x→ 0 −
x→ 0 − ⇒ for x > 0, f ′ ′ (x) > 0, if x < a /3
and RHL = lim f (x) = lim (x + ax2 − x3 ) = 0 Thus, f (x) increases on [–2/a, 0] and on [0, a/3].
x → 0+ x → 0+
Therefore, LHL = RHL = 0 = f (0)  2 a
Hence, f (x) increases on − , .
 a 3 
So, f (x) is continuous at x = 0.
 ax y = f (x) = 2 sin x + 2 tan x − 3x
f ′ (x) = 1 ⋅ e + axe 2 , if x < 0
ax 30. Let
Also,
 1 + 2 ax − 3 x , if x > 0 ⇒ f ′ (x) = 2 cos x + 2 sec2 x − 3
f (x) − f (0) 0 ≤ x < π / 2, f ′ (x) > 0
and Lf ′ (0) = lim For
x → 0− x−0
Thus, f (x) is increasing.
xeax − 0 When x ≥ 0, f (x) ≥ f (0)
= lim = lim eax = e0 = 1
x → 0− x x → 0−
⇒ 2 sin x + 2 tan x − 3x ≥ 0 + 0 − 0
f (x) − f (0) ⇒ 2 sin x + 2 tan x ≥ 3x
and Rf ′ (0) = lim
x → 0+ x+0
31. Let f (x) = 1 + x log (x + x2 + 1 ) − 1 + x2
x + ax2 − x3 − 0
= lim  
x →0 + x 1 + x 
 x + 1 
2
= lim 1 + ax − x2 = 1 
x → 0+
∴ f ′ (x) = x ⋅ + log (x + x2 + 1 )
x+ x +1
2
Therefore, Lf ′ (0) = Rf ′ (0) = 1 ⇒ f ′ (0) = 1
rankershalt.com

244 Application of Derivatives

x x x
− = + log (x + x2 + 1 ) − Topic 4 Maxima and Minima
x +1
2
x +1
2
x +1
2
1. Given functions are f (x) = 5 − |x − 2|
⇒ f ′ (x) = log (x + x + 1)
2
and g (x) = | x + 1|, where x ∈ R.
Clearly, maximum of f (x) occurred at x = 2, so α = 2.
⇒ f ′ (x) ≥ 0 [Q log (x + x + 1 ) ≥ 0]
2
and minimum of g (x) occurred at x = − 1, so β = − 1.
∴ f (x) is increasing for x ≥ 0. ⇒ αβ = − 2
(x − 1) (x2 − 5x + 6)
⇒ f (x) ≥ f (0) Now, lim
x → − αβ x2 − 6 x + 8
⇒ 1 + x log (x + 1 + x2 ) − 1 + x2 ≥ 1 + 0 − 1 (x − 1) (x − 3) (x − 2)
= lim [Qαβ = − 2]
⇒ 1 + x log (x + 1 + x2 ) ≥ 1 + x2 , ∀ x ≥ 0
x→ 2 (x − 4) (x −2)
(x − 1) (x − 3) (2 − 1) (2 − 3) 1 × (− 1) 1
π π = lim = = =
32. Given, A = x : ≤x≤  x→ 2 (x − 4) (2 − 4) (− 2) 2
 6 3
and f (x) = cos x − x − x2 Key Idea Volume of parallelopiped formed by the vectors a,
2.
⇒ f ' (x) = − sin x − 1 − 2x = − (sin x + 1 + 2x) b and c is V = [ a b c].

π π  Given vectors are i$ + λ$j + k


$ , $j + λk
$ and λi$ + k
$ , which
which is negative for x ∈ ,
 6 3  forms a parallelopiped.
∴ f ' (x) < 0 ∴Volume of the parallelopiped is
or f (x) is decreasing. 1 λ 1
  π  π  V = 0 1 λ = 1 + λ3 − λ
Hence, f ( A ) =  f   , f   
  3  6  λ 0 1
1 π  π 3 π  π 
=  − 1 +  , − 1 +   ⇒ V = λ3 − λ + 1
 2 3  3  2 6  6 
On differentiating w.r.t. λ, we get
dV
= 3 λ2 − 1
Topic 3 Rolle’s and Lagrange’s Theorem dλ
1. f ′ (x) is increasing dV
For maxima or minima, =0
1  dλ
For some x in  , 1
2  ⇒ λ=±
1
f ′ (x) = 1 [LMVT] 3
∴ f′ (1) > 1  1
2 3 > 0 , for λ =
d 2V  3
2. Given, f (x) = 2 + cos x, ∀x ∈ R and = 6λ = 
dλ2 2 3 < 0 , for λ = −
1
Statement I There exists a point ∈ [t , t + r ], where  3
f ′ (c) = 0 d 2V 1
Q is positive for λ = , so volume ‘V ’ is minimum
Hence, Statement I is true. dλ2 3
Statement II f (t ) = f (t + 2π ) is true. But statement II 1
for λ =
is not correct explanation for statement I. 3
3. Since, f (x) and g (x) are differentiable functions for 3. Given function f (x) = x kx − x2 … (i)
0 ≤ x ≤ 1.
f (1) − f (0) the function f (x) is defined if kx − x ≥ 0 2

∴ f ′ (c) =
1 −0 ⇒ x2 − kx ≤ 0

Using Lagrange’s Mean Value theorem, ⇒ x ∈ [0, k] … (ii)


6 −2 because it is given that f (x) is increasing in interval
=4 x ∈ [0, 3], so k should be positive.
1 −0
g (1) − g (0) Now, on differentiating the function f (x) w.r.t. x, we get
and g ′ (c) = x
1 −0 f ′ (x) = kx − x2 + × (k − 2x)
2 −0 2 kx − x2
= =2
1 −0 2(kx − x2) + kx − 2x2 3kx − 4x2
= =
⇒ f ′ (c) = 2 g′ (c)
2 kx − x2 2 kx − x2
rankershalt.com

Application of Derivatives 245

as f (x) is increasing in interval x ∈ [0, 3], so For maxima or minima,


dV
=0
f ′ (x) ≥ 0 ∀ x ∈ (0, 3) dθ
⇒ 54π [2 sin θ cos θ − sin θ ] = 0
2 3
⇒ 3kx − 4x2 ≥ 0
⇒ sin θ [2 cos 2 θ − sin 2 θ ] = 0
⇒ 4x2 − 3kx ≤ 0   π 
⇒ tan 2 θ = 2 Q θ ∈ 0, 2  
 3k  3k   
⇒ 4x x −  ≤ 0 ⇒ x ∈ 0, (as k is positive)
 4  4 
⇒ tan θ = 2
3k 2 1
So, 3≤ ⇒k ≥4 ⇒ sin θ = and cos θ = …(ii)
4 3 3
⇒ Minimum value of k = m = 4 From Eqs. (i) and (ii), we get
1
and the maximum value of f in [0, 3] is f (3). h =6 =2 3
3
Q f is increasing function in interval x ∈ [0, 3]
Q M = f (3) = 3 4 × 3 − 32 = 3 3 6. Given function is
f (x) = 9x4 + 12x3 − 36x2 + 25 = y (let)
Therefore, ordered pair (m, M ) = (4, 3 3 ) For maxima or minima put
dy
=0
dx
4. The non-zero four degree polynomial f (x) has dy
extremum points at x = −1, 0, 1, so we can assume ⇒ = 36x3 + 36x2 − 72x = 0
dx
f ′ (x) = a (x + 1)(x − 0) (x − 1) = ax(x2 − 1) ⇒ x3 + x2 − 2x = 0
where, a is non-zero constant.
⇒ x[x2 + x − 2] = 0
f ′ (x) = ax3 − ax
a a ⇒ x[x + 2x − x − 2] = 0
2

⇒ f (x) = x4 − x2 + C ⇒ x[x(x + 2) − 1(x + 2)] = 0


4 2
⇒ x(x − 1)(x + 2) = 0
[integrating both sides]
⇒ x = − 2, 0, 1
where, C is constant of integration.
Now, since f (x) = f (0) By sign method, we have following
– + – +
a 4 a 2 x4 x2
⇒ x − x +C =C ⇒ = –2 0 1
4 2 4 2
⇒ x2(x2 − 2) = 0 ⇒ x = − 2 , 0, 2 dy
Since, changes it’s sign from negative to positive at
Thus, f (x) = f (0) has one rational and two irrational dx
roots. x = ‘−2 ’ and ‘1’, so x = − 2, 1 are points of local minima.
dy
Also, changes it’s sign from positive to negative at
dx
Key Idea
5. x = 0, so x = 0 is point of local maxima.
(i) Use formula of volume of cylinder, V = πr 2h
∴S1 = { −2, 1} and S 2 = {0}.
where, r = radius and h = height
(ii) For maximum or minimum, put first 7. Given equation of curve is
derivative of V equal to zero y2 = x − 2 …(i)
Let a sphere of radius 3, which inscribed a right circular and the equation of line is
cylinder having radius r and height is h, so y=x …(ii)
h Y y=x
From the figure, = 3 cos θ y2=x–2
2
M
⇒ h = 6 cos θ P(t2+2, t)
and r = 3 sin θ …(i)
r
O X
(2, 0)

h
θ 3 Consider a point P (t 2 + 2, t ) on parabola (i).
h/2
For the shortest distance between curve (i) and line
(ii), the line PM should be perpendicular to line (ii) and
r parabola (i), i.e. tangent at P should be parallel to
y = x.
Q Volume of cylinder V = πr 2h
= π (3 sin θ )2(6 cos θ ) = 54π sin 2 θ cos θ .
rankershalt.com

246 Application of Derivatives


dy
= Slope of tangent at point P to curve (i) ⇒ x2 − 11x + 30 ≤ 0
dx at point P ⇒ (x − 5) (x − 6) ≤ 0 ⇒ x ∈ [5, 6]
=1 [Q tangent is parallel So, S = [5, 6]
to line y = x] Note that f (x) is increasing in [5, 6]
1 [Q f ′ (x) > 0 for x ∈[5, 6]
⇒ =1 ∴f (6) is maximum, where
2yP
dy f (6) = 3(6)3 − 18(6)2 + 27(6) − 40 = 122
[differentiating the curve (i), we get2 y =1]
dx 10. According to given information, we have the following
1 1 figure.
⇒ =1⇒t = [Q P (x, y) = P (t + 2, t )]
2
2t 2
 9 1
So, the point P is  ,  .
 4 2
9 − 1
4 2
Now, minimum distance = PM =
2
[Q distance of a point P (x1 , y1 ) from a line
|ax1 + by1 + c |
ax + by + c = 0 is  For y2 = 4ax, parametric coordinates of a point is (at 2,
a 2 + b2  2at).
7 ∴For y2 = 4x, let coordinates of C be (t 2, 2t).
= units
4 2 t 2 2t 1
1
8. Equation of parabola is given, y = 12 − x2 Then, area of ∆ABC = 9 6 1
2
or x2 = − ( y − 12). 4 −4 1
1
Note that vertex of parabola is (0, 12) and its open = |t 2(6 − (− 4)) − 2t (9 − 4) + 1(− 36 − 24)|
downward. 2
1 10 2
Let Q be one of the vertices of rectangle which lies on = |10t 2 − 10t − 60| = |t − t − 6|= 5| t 2 − t − 6|
2 2
parabola. Then, the coordinates of Q be (a, 12− a 2)
Let, A (t ) = 5| t 2 − t − 6| ...(i)
Clearly, A (4, − 4) ≡ A (t12, 2t1 ) ⇒ 2t1 = − 4
⇒ t1 = − 2
and B(9, 6) ≡ B(t22,2t2) ⇒ 2t2 = 6 ⇒ t2 = 3
Since, C is on the arc AOB, the parameter ‘t’ for point
C ∈ (− 2, 3).
Let f (t ) = t 2 − t − 6 ⇒ f ′ (t ) = 2t − 1
1
Now, f ′ (t ) = 0 ⇒ t =
2
Then, area of rectangle PQRS 1
= 2 × (Area of rectangle PQMO) Thus, for A (t ), critical point is at t =
2
[due to symmetry aboutY -axis] 2
 1  1 1 125 1
= 2 × [a (12 − a 2)] = 24a − 2a3 = ∆(let). Now,A   = 5   − − 6 = = 31 [Using Eq. (i)]
 2  2 2 4 4
The area function ∆ a will be maximum, when
d∆
=0 11. Let h = height of the cone,
da
r = radius of circular base
⇒ 24 − 6a 2 = 0
= (3)2 − h 2 [Q l2 = h 2 + r 2]
⇒ a2 = 4 ⇒ a = 2 [Q a > 0]
So, maximum area of rectangle = 9 − h2 …(i)
PQRS = (24 × 2) − 2 (2)3
= 48 − 16 = 32 sq units
9. We have,
3

h
l=

f (x) = 3x3 − 18x2 + 27x − 40


⇒ f ′ (x) = 9x2 − 36x + 27
= 9(x2 − 4x + 3) = 9 (x − 1) (x − 3) …(i) r
Also, we have S = { x ∈ R : x2 + 30 ≤ 11 x}
Clearly, x2 + 30 ≤ 11x
rankershalt.com

Application of Derivatives 247

1 2 14. According to given information, we have Perimeter of


Now, volume (V ) of cone = πr h
3 square + Perimeter of circle = 2 units
1 ⇒ 4 x + 2 πr = 2
⇒ V (h ) = π (9 − h 2)h [From Eq. (i)]
3 1 − 2x
1 ⇒ r= ...(i)
= π [9h − h3 ] …(ii) π
3 Now, let A be the sum of the areas of the square and the
For maximum volume V ′ (h ) = 0 and V ′′ (h ) < 0. circle. Then,
Here, V ′ (h ) = 0 ⇒ (9 − 3h 2) = 0 A = x2 + π r 2
⇒ h= 3 [Q h </ 0] (1 − 2x) 2
1 = x2 + π
and V ′′ (h ) = π (−6h ) < 0 for h = 3 π2
3 (1 − 2x)2
Thus, volume is maximum when h = 3 ⇒ A (x) = x2 +
π
Now, maximum volume dA
1 Now, for minimum value of A (x), =0
V ( 3 ) = π (9 3 − 3 3 ) [from Eq. (ii)] dx
3 2 (1 − 2x) 2 − 4x
= 2 3π ⇒ 2x + ⋅ (− 2) = 0 ⇒ x =
π π
12. We have, 2
⇒ πx + 4x = 2 ⇒ x = ...(ii)
1 1 f(x) π+4
f(x) = x2 + and g( x ) = x − ⇒ h( x ) =
x2 x g( x ) Now, from Eq. (i), we get
2
1  1
1 −2⋅
2
x2 + 2  x −  + 2
x =  x π + 4 π + 4 −4 1
∴ h( x ) = r= = = ...(iii)
1 1 π π(π + 4) π + 4
x− x−
x x From Eqs. (ii) and (iii), we get x = 2r
 1 2
⇒ h( x ) =  x −  + 15. Here, to find the least value of α ∈ R, for which
 x  1
x− 4 αx2 +
1
≥ 1, for all x > 0.
x x
1  1 2
x − > 0,  x −  + ∈ [2 2 , ∞ ) i.e. to find the minimum value of α when
x  x x − 1 1
y = 4αx + ; x > 0 attains minimum value of α.
2
x x
1  1 2 dy 1
x − < 0,  x −  + ∈ ( −∞ , 2 2 ] ∴ = 8αx − 2 …(i)
x  x x − 1 dx x
x d 2y 2
Now, = 8α + 3 …(ii)
∴ Local minimum value is 2 2. dx 2
x
dy
13. Total length = 2r + r θ = 20 When = 0, then 8x3α = 1
θ dx
20 − 2r
⇒ θ=  1
1/3
d 2y
r r r At x =   , = 8α + 16α = 24α, Thus, y attains
 8α  dx2
Now, area of flower-bed, 1/3
 1
1 minimum when x =   ; α > 0.
A = r 2θ  8α 
2 rθ 1/3
1 2 20 − 2r   1
⇒ A= r   ∴ y attains minimum when x =   .
2  r   8α 
23
/
⇒ A = 10r − r 2  1
i.e. 4α   + (8α )1/3 ≥ 1
dA  8α 
∴ = 10 − 2r
dr ⇒ α 1/3 + 2α 1/3 ≥ 1
dA 1
For maxima or minima, put = 0. ⇒ 3α 1/3 ≥ 1 ⇒ α ≥
dr 27
1
⇒ 10 − 2r = 0 ⇒ r = 5 Hence, the least value of α is .
27
1 20 − 2 (5) 
∴ Amax = (5)2
2  5  16. PLAN Any function have extreme values (maximum or minimum) at
its critical points, where f ′ ( x ) = 2.
1
= × 25 × 2 = 25 sq. m Since, the function have extreme values at x = 1 and
2 x = 2.
rankershalt.com

248 Application of Derivatives

∴ f ′ (x) = 0 at x = 1 and x = 2 Put g′ (x) = 0 ⇒ x cos x = 0


⇒ f′ (1) = 0 and f′ (2) = 0 π 3π 5π 7π
∴ x = 0, , , ,
Also, it is given that, 2 2 2 2
lim  f (x)  g (x)
x→ 0 1 + =3
 x2  /2
f (x)
⇒ 1 + lim
x→ 0 2 = 3
x
lim f (x)
⇒ x→ 0 =2
x2 –5 π/ 2 –π –π/2 O π/2 π 5π/2
⇒ f (x) will be of the form ax4 + bx3 + 2x2.
[Q f (x) is of four degree polynomial]
Y′
Let f (x) = ax4 + bx3 + 2x2 3π 7π
⇒ f ′ (x) = 4ax3 + 3bx2 + 4x At x = 0, , ,... , f ′′ (x) > 0, so minimum
2 2
⇒ f ′ (1) = 4a + 3b + 4 = 0 ...(i) π 5π 9π
and f ′ (2) = 32a + 12b + 8 = 0 At x = , , ,... , f ′ (x) < 0, so maximum
2 2 2
⇒ 8a + 3b + 2 = 0 ...(ii)
So, graph of f (x) and g (x) are shown as
On solving Eqs. (i) and (ii),
Y
1
we get a = , b = −2 f (x)
2 g (x)
x4
∴ f (x) = − 2x3 + 2x2
2
⇒ f (2) = 8 − 16 + 8 = 0 X′ X
–π/2 O π/2
17. Here, x = − 1 and x = 2 are extreme points of
f (x) = α log|x| + βx2 + x, then
α Y′
f ′ (x) = + 2 βx + 1
x So, number of solutions are 2.
f ′ (−1) = − α − 2 β + 1 = 0 …(i) 2 2
19. Given function, f (x) = ex + e− x , g (x) = xex + e− x and
2 2

[at extreme point, f ′ (x) = 0] 2 2


h (x) = x2ex + e− x are strictly increasing on [0, 1].
α
f ′ (2) = + 4 β + 1 = 0 …(ii) Hence, at x = 1, the given function attains absolute
2
maximum all equal to e + 1 / e .
On solving Eqs. (i) and (ii), we get ⇒ a=b=c
1
α = 2, β = − (2 + x)3 , if − 3 < x ≤ −1
2 20. Given, f (x) = 
 x 2 / 3 , if − 1 < x < 2
18. PLAN The given equation contains algebraic and trigonometric
functions called transcendental equation. To solve 3 (x + 2)2 , if − 3 < x ≤ −1

transcendental equations we should always plot the graph for ⇒ f ′ (x) = 2 − 1
LHS and RHS. 3 x
3 , if − 1 < x < 2
Here, x2 = x sin x + cos x
Y
Y x2

X′ X X′ X
O (–3,0) (–1,0)
Y′
Y′
Let f (x) = x2 and g (x) = x sin x + cos x
Clearly, f ′ (x) changes its sign at x = −1 from +ve to −ve
We know that, the graph for f (x) = x2
and so f (x) has local maxima at x = −1 .
To plot, g (x) = x sin x + cos x Also, f ′ (0) does not exist but f ′ (0 − ) < 0 and f ′ (0 + ) < 0.
g′ (x) = x cos x + sin x − sin x It can only be inferred that f (x) has a possibility of a
g′ (x) = x cos x …(i) minima at x = 0 . Hence, the given function has one local
maxima at x = −1 and one local minima at x = 0 .
g′′ (x) = − x sin x + cos x …(ii)
rankershalt.com

Application of Derivatives 249

21. Given f (x) = x2 + 2bx + 2c2 and g (x) = − x2 − 2cx + b2 26. Let the coordinates of P be (a cos θ , b sin θ )
Then, f (x) is minimum and g (x) is maximum at Equations of tangents at P is
 −b −D 
x = and f (x) =  , respectively. Y
 4a 4a 
N
− (4b2 − 8c2) P
∴ min f (x) = = (2c2 − b2)
4
(4c2 + 4b2) X' X
and max g (x) = − = (b2 + c2) O M
4(−1)
Now, min f (x) > max g (x) K
⇒ 2c − b > b + c
2 2 2 2
Y'
⇒ c2 > 2b2 x y
⇒ | c| > 2| b| cos θ + sin θ = 1
a b
22. It is clear from figure that at x = 0, f (x) is not continuous. Again, equation of normal at point P is
Y ax secθ − by cosec θ = a 2 − b2

y=x Let M be foot of perpendicular from O to PK, the normal


y = –x at P.
1
X′ X Area of ∆OPN = (Area of rectangle OMPN )
O
2
1
= ON ⋅ OM
2
Y′
1 ab
Here, f (0) > RHL at x = 0 and f (0) > LHL at x = 0 Now, ON = =
cos θ sin θ
2 2
b cos θ + a 2 sin 2 θ
2 2

So, local maximum at x = 0. 2


+ 2
a b
x2 − 1 2
23. Given, f (x) = =1 − 2 [perpendicular from O, to line NP]
x2 + 1 x +1
a 2 − b2 (a 2 – b2) ⋅ cos θ ⋅ sin θ
2 and OM = =
f (x) will be minimum, when 2 is maximum, a 2 sec2 θ + b2cosec2θ a 2 sin 2 θ + b2 cos 2 θ
x +1
ab(a 2 − b2) cos θ ⋅ sin θ
i.e. when x2 + 1 is minimum, Thus, area of ∆OPN =
2(a 2 sin 2 θ + b2 cos 2 θ )
i.e. at x = 0.
ab(a 2 − b2) tan θ
∴ Minimum value of f (x) is f (0) = − 1 =
2(a 2 tan 2 θ + b2)
24. The maximum value of f (x) = cos x + cos ( 2x) is 2
tan θ
which occurs at x = 0. Also, there is no other value of x Let f (θ ) = [0 < θ < π / 2]
α 2 tan 2 θ + b2
for which this value will be attained again.
sec2 θ( a 2 tan 2 θ + b2) − tanθ (2a 2 tanθ sec2θ)
25. Let f (x) = x25 (1 − x)75 , x ∈[0, 1] f ' (θ ) =
(a 2 tan 2 θ + b2)2
⇒ f ′ (x) = 25 x24 (1 − x)75 − 75x25 (1 − x)74
sec2 θ (a 2 tan 2 θ + b2 – 2a 2 tan 2 θ )
= 25x24 (1 − x)74 [(1 − x) − 3x] =
(a 2 tan 2 θ + b2)2
= 25x24 (1 − x)74 (1 − 4x)
sec2 θ( a tan θ + b)(b – a tan θ )
For maximum value of f (x), put f ′ (x) = 0 =
(a 2 tan 2 θ + b2)2
⇒ 25x24 (1 − x)74 (1 − 4x) = 0
1 For maximum or minimum, we put
⇒ x = 0, 1,
4 f′ (θ ) = 0 ⇒ b – a tan θ = 0
Also, at x = 0, y = 0 [sec2 θ ≠ 0, a tan θ + b ≠ 0, 0 < θ < π / 2 ]
At x = 1, y = 0 ⇒ tan θ = b / a
x = 1 / 4, y > 0  −1
Also, f ′ (θ )> 0, if 0 < θ−1< tan (b / a )
and at
∴ f (x) attains maximum at x = 1 / 4.  < 0 , if tan (b / a ) < θ < π /2
rankershalt.com

250 Application of Derivatives

Therefore, f (θ ) has maximum, when cos 2x cos 2x sin 2x


 b b 30. f (x) = 
− cos x cos x − sin x

θ = tan −1   ⇒ tan θ =
 a a  sin x sin x cos x 
b a cos 2x(cos x + sin x) − cos 2x
2 2
Again, sin θ = , cos θ =
a 2 + b2 a 2 + b2 (− cos 2 x + sin 2 x) + sin 2x(− sin 2x)
= cos 2x + cos 4x
By using symmetry, we get the required points
f ′ (x) = − 2 sin 2x − 4 sin 4x = −2 sin 2x(1 + 4 cos 2x)
 ±a2 ± b2 
 ,  At x=0
 a 2 + b2 a 2 + b2  f ′ (x) = 0
27. Given, P (x) = a 0 + a1x2 + a 2 x4 + K+ a nx2n and f (x) = 2
Also, f ′ (x) = 0
where, a n > a n − 1 > a n − 2 > K > a 2 > a1 > a 0 > 0
sin 2x = 0
⇒ P ′ (x) = 2a1x + 4a 2 x3 + ... + 2na nx2n − 1 −1
or cos 2x =
= 2x (a1 + 2a 2 x2 + K + na nx2n − 2) …(i) 4
where, (a1 + 2a 2 x + 3a3 x + K + na nx
2 4 2n − 2
) > 0, ∀ x ∈ R. nπ 1
⇒ x= or cos 2x = −
P ′ (x) > 0, when x > 0 2 4
Thus,  f (x) ⋅ g (x)
P ′ (x) < 0, when x < 0 31. Here, lim =1
x → 2 f ′ (x) ⋅ g′ (x)
i.e. P ′ (x) changes sign from (–ve) to (+ve) at x = 0. f (x) g′ (x) + f ′ (x) g (x)
∴ P (x) attains minimum at x = 0. ⇒ lim =1
x → 2 f ′ ′ (x) g′ (x) + f ′ (x) g′′ (x)
Hence, it has only one minimum at x = 0 . [using L’ Hospital’s rule]
28. y = a log x + bx2 + x has extremum at x = − 1 and x = 2. f (2) g′ (2) + f ′ (2) g (2)
⇒ =1
dy f ′′ (2) g′ (2) + f ′ (2) g′′ (2)
∴ = 0, at x = − 1 f (2) g′ (2)
dx ⇒ =1 [Q f ′ (2) = g (2) = 0]
a f ′′ (2) g′ (2)
and x=2 ⇒ + 2bx + 1 = 0, at x = − 1
x ⇒ f (2) = f ′′ (2) … (i)
and x=2 ∴ f (x) − f ′′ (x) = 0, for atleast one x ∈ R.
∴ −a − 2b + 1 = 0 ⇒ Option (d) is correct.
a Also, f : R → (0, ∞ )
and + 4b + 1 = 0
2 ⇒ f (2) > 0
1 ∴ f ′′ (2) = f (2) > 0 [from Eq. (i)]
⇒ a = 2 and b = −
2 Since, f′ (2) = 0 and f ′′ (2) > 0
 p, if p > q ∴ f (x) attains local minimum at x = 2.
29. Since, max ( p, q) =  ⇒ Option (a) is correct.
 q, if q > p
 p, if p is greatest.
32. PLAN
 x , if x ≥
We know that,| x| = 
0
and max ( p, q, r ) =  q, if q is greatest.
 r , if r is greatest.  − x , if x < 0
 x − a, if x ≥ a
⇒ | x − a| = 
∴ max ( p, q) < max ( p, q, r ) is false.  x − a ),
− ( if x < a
 p − q , if p ≥ q and for non-differentiable continuous function, the maximum or
We know that, | p − q| = 
q − p , if p < q minimum can be checked with graph as

1
(p + q − p + q), if p ≥ q Y Y
1 2
∴ ( p + q −| p − q|) = 1
2  (p + q + p − q), if p < q
2
 q, if p≥q
=
 p, if p<q
1 X X
⇒ { p + q − | p − q|} = min ( p, q) O x=a x=a
2 Minimum at x = a Maximum at x = a
rankershalt.com

Application of Derivatives 251

Y On differntiating Eq. (i) w.r.t a, we get


dv
= 6a 2 − 46ax + 60x2
da
Again, differentiating,
d 2v
= 12a − 46x
X da 2
O x=a
 dv 
  = 0 ⇒ 6x − 23x + 15 = 0
2
Neither maximum Here,
 da 
nor minimum at x = a
Here, f (x) = 2|x|+ |x + 2| − ||x + 2| − 2|x|| 5
At a = 5 ⇒ x = 3,
6
− 2x − (x + 2) + (x − 2), if when x ≤ − 2
− 2 x + x + 2 + 3 x + 2 , if when − 2 < x ≤ −2 /3  d v
2

 ⇒  2 = 2 (30 − 23x)
 2  da 
= − 4x, if when − < x ≤ 0
 3  d 2v 
 4x, if when 0 < x ≤ 2 At x = 3,  2 = 2(30 − 69) < 0
 2x + 4, if when x > 2  da 

− 2x − 4, if x ≤ −2 5  d 2v 
∴ Maximum when x = 3, also at x = ⇒  2 > 0
 2x + 4, if −2 < x ≤ − 2 /3 6  da 

 2
=  − 4x, if − < x≤0 ∴ At x = 5 /6, volume is minimum.
 3
0 < x≤2 Thus, sides are 8x = 24 and 15x = 45
 4x, if
 2x + 4, if x>2 34. Given,
Graph for y = f (x) is shown as  ex , if 0 ≤ x ≤ 1
 x −1
f (x) = 2 − e , if 1 < x ≤ 2
 x−e , if 2 < x ≤ 3

x
and g (x) = ∫ f (t ) dt
0

⇒ g′ (x) = f (x)
Put g′ (x) = 0 ⇒ x = 1 + log e 2 and x = e.
 ex , if 0 ≤ x ≤ 1

Also, g′′ (x) =  − ex − 1 , if 1 < x ≤ 2
33. PLAN  1 , if 2 < x ≤ 3
The problem is based on the concept to maximise volume of
cuboid, i.e. to form a function of volume, say f( x )find f ′( x )and f ′ ′( x ). At x = 1 + log e 2,
Put f ′ ( x ) = 0 and check f ′ ′( x ) to be + ve or − ve for minimum and g′′ (1 + log e 2) = − elog e 2 < 0, g (x) has a local maximum.
maximum, respectively. Also, at x = e,
Here, l = 15x − 2a , b = 8x − 2a and h = a
a
g′′ (e) = 1 > 0, g (x) has a local minima.
Q f (x) is discontinuous at x = 1, then we get local
a maxima at x = 1 and local minima at x = 2.
Hence, (a) and (b) are correct answers.
8x–2a 8x
35. Since, f (x) has local maxima at x = − 1 and f ′ (x) has
15x – 2a
local minima at x = 0.
∴ f ′ ′ (x) = λx
15x On integrating, we get
∴ Volume = (8x − 2a ) (15x − 2a ) a x2
f ′ (x) = λ +c [Q f ′ (− 1) = 0]
V = 2a ⋅ (4x − a ) (15x − 2a ) …(i) 2
λ
⇒ + c = 0 ⇒ λ = − 2c …(i)
2
Again, integrating on both sides, we get
a
x3
8 x – 2a f (x) = λ + cx + d
6
15x – 2a
rankershalt.com

252 Application of Derivatives

 8 and lim f (x) = lim (37 − x)


⇒ f (2) = λ   + 2c + d = 18 …(ii) x → 2+ x → 2+
 6
= lim [37 − (2 + h )] = 35
λ h→ 0
and f (1) = + c+ d = −1 …(iii) and f (2) = 3 ⋅ 22 + 12 ⋅ 2 − 1 = 12 + 24 − 1 = 35
6
From Eqs. (i), (ii) and (iii), Therefore, LHL = RHL = f (2) ⇒ function is continuous
1 at x = 2 ⇒ function is continuous in −1 ≤ x ≤ 3.
f (x) = (19x3 − 57x + 34)
4 f (x) − f (2)
Now, Rf ′ (2) = lim
1
∴ f ′ (x) = (57x − 57) =
2 57
(x − 1) (x + 1) x→ 2 + x−2
4 4 f (2 + h ) − f (2)
= lim
For maxima or minima, put f ′ (x) = 0 ⇒ x = 1, − 1 h→ 0 h
1 37 − (2 + h ) − (3 × 22 + 12 × 2 − 1)
Now f ′ ′ (x) = (114x) = lim
4 h→ 0 h
At x = 1 , f ′ ′ (x) > 0, minima −h
At x = − 1, f ′ ′ (x) < 0 , maxima = lim = −1
h→ 0 h
∴ f (x) is increasing for [1, 2 5 ] . f (x) − f (2) f (2 − h ) − f (2)
∴ f (x) has local maxima at x = − 1 and f (x) has local and Lf ′ (2) = lim = lim
x→ 2− x − 2 h → 0 −h
minima at x = 1.
Also, f (0) = 34 /4 [3(2 − h )2 + 12(2 − h ) − 1] 
 − (3 × 22 + 12 × 2 − 1)
Hence, (b) and (c) are the correct answers. = lim
x
h→ 0 −h
36. f (x) = ∫ t (et − 1) (t − 1)(t − 2)3 (t − 3)5 dt [12 + 3h 2 − 12h + 24 − 12h − 1] − 35
−1
= lim
– – + – + h→ 0 −h
–∞ 0 1 2 3 ∞ 3h 2 − 24h + 35 − 35
= lim
d x h→ 0 −h
f ′ (x) =
dx ∫ −1 t (et − 1)(t − 1)(t − 2)3 (t − 3)5 dt 3h − 24
= lim = 24
= x(ex − 1)(x − 1)(x − 2)3 (x − 3)5 × 1
h→ 0 −1
 d ψ( x )  Since, Rf ′ (2) ≠ Lf ′ (2), f ′ (2) does not exist.
Q
 dx ∫ φ ( x) f (t ) dt = f { ψ (x)}ψ′ (x) − f { φ (x)} φ′ (x)
 Again, f (x) is an increasing in [–1, 2] and is decreasing
in (2, 3), it shows that f (x) has a maximum value at x = 2.
For local minimum, f ′ (x) = 0
Therefore, options (a), (b), (c), (d) are all correct.
⇒ x = 0, 1, 2, 3
Let f ′ (x) = g (x) = x(ex − 1)(x − 1)(x − 2)3 (x − 3)5 38. Here, y2 = 16x,0 ≤ y ≤ 6
Y
Using sign rule,
− + − + (4t 2, 8t)

1 2 3 E (0, 3)

This shows that f (x) has a local minimum at x = 1 and X


G O
x = 3 and maximum at x = 2 .
fy
Therefore, (b) and (d) are the correct answers.
Tangent at F , yt = x + at 2
37. For −1 ≤ x ≤ 2 , we have
At x = 0, y = at = 4 t
f (x) = 3x2 + 12x − 1
Also, (4 t 2, 8 t ) satisfy y = mx + c .
⇒ f ′ (x) = 6x + 12 > 0, ∀ − 1 ≤ x ≤ 2
⇒ 8 t = 4mt 2 + 3
Hence, f (x) is increasing in [–1, 2].
⇒ 4mt − 8t + 3 = 0
2
Again, function is an algebraic polynomial, therefore it
is continuous at x ∈ (−1, 2) and (2, 3). 0 3 1
1 1
For continuity at x = 2, ∴ Area of ∆ = 0 4t 1 = ⋅ 4t 2(3 − 4t )
2 2 2
lim f (x) = lim (3x2 + 12x − 1) 4t 8t 1
x → 2− x → 2−
= lim [3 (2 − h )2 + 12(2 − h ) − 1] A = 2[3t 2 − 4t3 ]
h→ 0 dA
= lim [3 (4 + h 2 − 4h ) + 24 − 12h − 1] ∴ = 2[6t − 12t 2] = − 12t (12t − 1)
h→ 0 dt
= lim (12 + 3h 2 − 12h + 24 − 12h − 1) – + –
h→ 0 0 1
= lim (3h 2 − 24h + 35) = 35 2
h→ 0
rankershalt.com

Application of Derivatives 253

∴ Maximum at t =
1
and 4mt 2 − 8t + 3 = 0 ⇒ g (x) = 0 ⇒ h (x) = 0
2 ⇒ f (x) = 0 or f ′ (x) = 0
⇒ m −4 + 3 =0 If f (x) = 0 has 4 minimum solutions.
⇒ m =1 f ′ (x) = 0 has 3 minimum solutions.
G (0, 4t ) ⇒ G (0,2) h (x) = 0 has 7 minimum solutions.
⇒ y1 = 2 ⇒ h′ (x) = g (x) = 0 has 6 minimum solutions.
(x0 , y0 ) = (4t 2, 8t ) = (1, 4)
43. To maximise area of ∆ APB, we know that, OP = 10 and
y0 = 4 sin θ = r /10 , where θ ∈ (0, π / 2) … (i)
 3 1 1
∴ Area = 2 −  = Y
 4 2 2 P(6,8)
(x + ax + 1) − 2ax
2
2ax θ
39. f (x) = =1 − 2
x2 + ax + 1 x + ax + 1 A
θ
 (x2 + ax + 1) ⋅ 2a − 2ax(2x + a )  r
f ′ (x) = −   Q
 (x2 + ax + a )2  X′ X
O
 −2ax + 2a  2  (x − 1)  2
= 2 2
= 2a  2 2
...(i)
 (x + ax + a )   (x + ax + 1) 
B

 (x2 + ax + 1)2 (2x) − 2(x2 − 1) 


 (x2 + ax + 1) (2x + a )  Y′
f ′ ′ (x) = 2a   1
 (x2 + ax + 1)4  ∴ Area = (2 AQ ) (PQ )
  2
2x (x2 + ax + 1) − 2 (x2 − 1) (2x + a )  = AQ . PQ = (r cos θ ) (10 − OQ )
= 2a   …(ii) = (r cos θ ) (10 − r sin θ )
 (x2 + ax + 1)3 
4a (a + 2) 4a = 10 sin θ cos θ (10 − 10 sin 2 θ ) [from Eq. (i)]
Now, f ′′ (1) = = ⇒ A = 100 cos3 θ sin θ
(a + 2)3 (a + 2)2
dA
4a (a − 2) − 4a ⇒ = 100 cos 4 θ − 300 cos 2 θ ⋅ sin 2 θ
and f ′′ (−1) = = dθ
(2 − a )3 (a − 2)2
dA
∴ (2 + a )2 f ′ ′ (1) + (2 − a )2 f ′ ′ (−1) = 4a − 4a = 0 Put =0

40. When x ∈ (−1, 1), ⇒ cos 2 θ = 3 sin 2 θ
x <1 ⇒x −1 <0
2 2
⇒ tan θ = 1 / 3
∴ f ′ (x) < 0, f (x) is decreasing. ⇒ θ = π /6
4a dA
Also, at x = 1, f ′′ (1) = >0 [Q0 < a < 2] At which < 0, thus when θ = π /6, area is maximum
(a + 2)2 dθ
So, f (x) has local minimum at x = 1. π
From Eq. (i), r = 10 sin = 5 units
f ′ (e ) x
x 6
41. g′ (x) = ⋅e
1 + (ex )2 x2 y2
44. Let us take a point P( 6 cos θ , 3 sin θ ) on
+ =1.
6 3
 e2x − 1   ex 
= 2a  2x 2
  Now, to minimise the distance from P to given straight
 (e + ae + 1)   1 + e 
x 2x
line x + y = 7, shortest distance exists along the
g′ (x) = 0, if e2x − 1 = 0, i.e. x = 0 common normal.
If x < 0, e2x < 1 ⇒ g′ (x) < 0 Y
y
d N
42. Let g (x) = [ f (x) ⋅ f ′ (x)]
x+y=7
dx P
X′ X
To get the zero of g (x), we take function O
h (x) = f (x) ⋅ f ′ (x)
between any two roots of h (x), there lies atleast one root
of h′ (x) = 0. Y′
rankershalt.com

254 Application of Derivatives

a 2 / x1 6 sec θ we have, f ′ (t ) = 4(t 2 + 1 − t )(2t − 1)


Slope of normal at P = = = 2 tan θ = 1
b2 / y1 6 cosec θ = 4[(t − 1 / 2)2 + 3 / 4][2t − 1]

So, cos θ =
2
and sin θ =
1 Now, f ′ (t ) = 0
3 3 ⇒ t = 1 /2
Hence, required point is P(2, 1). Also, f ′ (t ) < 0 for t < 1 / 2
2ax 2ax − 1 2ax + b + 1 and f ′ (t ) > 0 for t > 1 / 2
45. Given, f ′ (x) = b b+1 −1
Thus, f (t ) is least when t = 1 / 2.
2(ax + b) 2ax + 2b + 1 2ax + b
Corresponding to t = 1 / 2, point P0 on C1 is (1/2, 5/4) and
Applying R3 → R3 − R1 − 2R2, we get P1 (which we take as Q0) on C 2 are (5 / 4, 1 / 2). Note that
2ax 2ax − 1 2ax + b + 1 P0Q0 ≤ PQ for all pairs of (P , Q ) with P on C 2.
f ′ (x) = b b+1 −1 47. Let the square S is to be bounded by the lines x = ±1 / 2
0 0 1 and y = ±1 / 2.
2 2
 1 1 
⇒ f ′ (x) = 2ax + b We have, a 2 =  x1 −  +  − y1
 2 2 
On integrating both sides, we get Y
f (x) = ax2 + bx + c A(x , 1/2) 1/2
Since, maximum at x = 5 /2 ⇒ f ′ (5 /2) = 0
d
⇒ 5a + b = 0 …(i) D( −1/2, 1/2)
1/2
Also, f (0) = 2 ⇒ c=2 …(ii) X' X
−1/2 O B ( −1/2, y)
and f (1) = 1 ⇒ a + b + c = 1 …(iii) c
b
On solving Eqs. (i), (ii) and (iii), we get
1 5 C(x2 −1/2) −1/2
a = ,b = − ,c=2
4 4
Y′
1 2 5
Thus, f (x) = x − x + 2 1
4 4 = x12 − y12 − x1 − y1 +
2
46. Let coordinates of P be (t , t 2 + 1) 1
Similarly, b = x2 − y1 − x2 + y1 +
2 2 2

Reflection of P in y = x is P1 (t 2 + 1, t ) 2
1
which clearly lies on y2 = x − 1 c = x2 − y2 + x2 + y2 +
2 2 2
2
Similarly, let coordinates of Q be (s2 + 1, s) 1
Its reflection in y = x is d = x1 − y2 + x1 − y2 +
2 2 2
2
Q1 (s, s2 + 1), which lies on x2 = y − 1. ∴ a 2 + b2 + c2 + d 2 = 2(x12 + y12 + x22 + y22) + 2
We have, PQ12 = (t − s) + (t − s ) = P1Q
2 2 2 2 2
1
Therefore, 0 ≤ x12, x22, y12, y22 ≤
⇒ PQ1 = P1Q 4
Also PP1 || QQ1 [Q both perpendicular to y = x] 0 ≤ x12 + x22 + y12 + y22 ≤ 1
C1 Y x 2 = y −1 ⇒ 0 ≤ 2(x12 + x22 + y12 + y22) ≤ 2
P1 y=x But 2 ≤ 2(x12 + x22 + y12 + y22) + 2 ≤ 4
Q1
(0, 1) P Alternate Solution
Q y2− = 1 c2 = x22 + y22 ... (i)
O
X′ X
(1, 0) Y
C2 A(x , 1)
Y′ a
d B (1, y1)
Thus, PP1QQ1 is an isosceles trapezium.
1/2
Also, P lies on PQ1 and Q lies on P1Q , then (0, y2)D X
PQ ≥ min { PP1QQ1 } b
c
Let us take min { PP1QQ1 } = PP1
X′ X
∴ PQ 2 ≥ PP12 = (t 2 + 1 − t )2 + (t 2 + 1 − t 2) O C(x2 , 0)
= 2(t 2 + 1 − t 2) = f (t ) [say] Y′
rankershalt.com

Application of Derivatives 255

b2 = (1 − x2)2 + y12 ...(ii) From sign chart, it is clear that f ′ (x) has no change of
a 2 = (1 − y1 )2 + (1 − x1 )2 ...(iii) sign in left and right of x = 1/4.
d 2 = x12 + (1 − y2)2 ...(iv) Case III When b > 1, then
1 2 1 1
On adding Eqs. (i), (ii), (iii) and (iv), we get f ′ (x) = − b + 2x =  x2 − bx + 
8x x 2 16
a 2 + b2 + c2 + d 2 = { x12 + (1 − x1 )2} + { y12 + (1 − y1 )2}
2  
2
+ { x22 + (1 − x2)2} + { y22 + (1 − y2)2} b 1 2
=  x −  − (b − 1)
where x1 , y1 , x2, y2 all vary in the interval [0, 1]. x  4  16 
Now, consider the function y = x2 + (1 − x)2, 0 ≤ x ≤ 1 2  b 1   b 1 
= x − − b 2 − 1  x − + b 2 − 1 
differentiating ⇒
dy
= 2x − 2(1 − x). For maximum or x  4 4   4 4 
dx
2
minimum
dy
= 0. = (x − α ) (x − β )
dx x
1
⇒ 2x − 2(1 − x) = 0 ⇒ 2x − 2 + 2x = 0 where, α < β and α = (b − b2 − 1 ) and
4
⇒ 4x = 2 ⇒ x = 1 /2 1
β = (b + b2 − 1 ). From sign scheme, it is clear that
d 2y 4
Again, =2+2 =4 > 0, for 0 < x < α
dx2 
f ′ (x) < 0, for α < x < β
1
Hence, y is minimum at x = and its minimum value is > 0, for x > β

2
1/4. Clearly, value is maximum when x = 1. By the first derivative test, f (x) has a maxima at x = α
1
1 1 1 1 = (b − b2 − 1 )
∴Minimum value of a 2 + b2 + c2 + d 2 = + + + = 2 4
2 2 2 2 1
and maximum value is 1 + 1 + 1 + 1 = 4 and f (x) has a minima at x = β = (b + b2 − 1 )
4
48. f (x) is a differentiable function for x > 0. 49. Let equation of any line through the point (h , k) is
Therefore, for maxima or minima, f ′ (x) = 0 must satisfy. y − k = m(x − h ) … (i)
1
Given, f (x) = ln x − bx + x2, x > 0 For this line to intersect the positive direction of two
8 axes, m = tan θ < 0 , since the angle in anti-clockwise
1 1 direction from X-axis becomes obtuse.
⇒ f ′ (x) = . − b + 2x
8 x
Y
For f ′ (x) = 0
1
⇒ − b + 2x = 0 Q
(h, k)
8x
⇒ 16x2 − 8bx + 1 = 0
⇒ (4x − b)2 = b2 − 1 …(i) X′ X
O P
⇒ (4x − b)2 = (b − 1) (b + 1) [b ≥ 0, given] Y′
Case I 0 ≤ b < 1 , has no solution. Since, RHS is  k 
negative in this domain and LHS is positive. The line (i) meets X-axis at P  h − , 0 and Y-axis at
1  m 
Case II When b = 1, then x = is the only solution.
4 Q (0, k − mh ).
When b = 1,
1
1 2 1 1 2 1
2 Let A = area of ∆ OPQ = OP . OQ
f ′ (x) = − 1 + 2x =  x2 − x +  = x −  2
8x x 2 16 x  4 1 k
=  h −  (k − mh )
We have to check the sign of f ′ (x) at x = 1/4. 2 m
1  mh − k 1
Interval Sign of f′(x) Nature of f(x) =   (k − mh ) = − (k − mh )2
2 m  2m
−∞, 0 −ve ↓
1
 0, 1  + ve ↑ =− (k − h tan θ )2 [Qm = tan θ]
  2 tan θ
 4
1
 1, ∞ + ve ↑ =− (k2 + h 2 tan 2 θ − 2hk tan θ )
  2 tan θ
4 
1
= (2kh − k2 cot θ − h 2 tan θ )
2
rankershalt.com

256 Application of Derivatives

dA 1 To find the coordinates of point R, we have to solve it


⇒ = [− k2(− cosec2θ ) − h 2 sec2θ ]
dθ 2 with
1 x2 + y2 = 1 … (ii)
= [k2 cosec2θ − h 2 sec2θ ]
2 On subtracting Eq. (ii) from Eq. (i), we get
dA (x − 1)2 − x2 = r 2 − 1
To obtain minimum value of A, =0
dθ ⇒ x + 1 − 2x − x2 = r 2 − 1
2

⇒ k cosec θ − h sec θ = 0
2 2 2 2
⇒ 1 − 2x = r 2 − 1
2 2 2

k
=
h

k
= tan 2 θ 2 − r2
∴ x=
sin 2 θ cos 2 θ h2 2
k On putting the value of x in Eq. (i), we get
⇒ tan θ = ±
h  2 − r 2
2

Q tan θ < 0 , k > 0, h > 0 [given]   + y2 = 1


 2 
k
Therefore, tan θ = − (only possible value). 2
h  2 − r 2 (2 − r 2)2
d 2A 1 ⇒ y2 = 1 −   =1 −
Now, = [− 2 k 2
cosec2
θ cot θ − 2h 2 sec2 θ tan θ ]  2  4
dθ 2 2
r 4 − 4r 2 + 4
= − [k2(1 + cot2 θ ) cot θ + h 2(1 + tan 2 θ ) tan θ ] =1 −
4
 d 2A    h 2  − h 
⇒  2 = − k21 + 2    4 − r 4 + 4r 2 − 4
k   k =
 dθ  tan θ = − k/ h   4
 k 2   − k  4r 2 − r 4
+ h 2 1 + 2    =
 h   h  4
  k2 + h 2  h   h 2 + k2  k   r 2(4 − r 2)
= k2    + h 2    =
  k
2
  k  h2   h  4
 (k2 + h 2) h (h 2 + k2)(k)  r 4 − r2
= +  ⇒ y=
 k h  2
Again, we know that, coordinates of S are (1 − r , 0),
2 h k
= (k + h )
2
+ >0 [Q h, k > 0] therefore
 k h 
SQ = 1 − (1 − r ) = r
Therefore, A is least when tan θ = − k / h. Also, the least
value of A is Let A denotes the area of ∆ QSR, then
1  4−r 
2
1  −h   − k  A = r r 
A= 2hk − k2  − h 2  
2   k  h  2 

2 

1
= [2hk + kh + kh ] = 2hk 1 2
2 = r 4 − r2
50. Since x + y2 = 1 a circle S1 has centre (0, 0) and cuts
2 4
X-axis at P(–1, 0) and Q(1, 0) . Now, suppose the circle 1 4
⇒ A2 = r (4 − r 2)
S 2, with centre at Q(1, 0) has radius r. Since, the circle 16
has to meet the first circle, 0 < r < 2 . Let f (r ) = r 4 (4 − r 2) = 4r 4 − r 6
Again, equation of the circle with centre at Q(1, 0) and
⇒ f ′ (r ) = 16r3 − 6r5 = 2r3 (8 − 3r 2)
radius r is
For maxima and minima, put f ′ (r ) = 0
Y
⇒ 2r3 (8 − 3r 2) = 0
R S2 ⇒ r = 0, 8 − 3r 2 = 0
S1
r ⇒ r = 0, 3r 2 = 8
X′ O S X ⇒ r = 0, r 2 = 8/3
(–1,0)P Q(1,0)
2 2
⇒ r = 0, r =
3
[ Q0 < r < 2 , so r = 2 2 / 3]
Y′
Again, f ′ ′ (r ) = 48r 2 − 30r 4
(x − 1) + y = r
2 2 2
…(i)
rankershalt.com

Application of Derivatives 257

2 2  4 × 2  4 × 2
2
Then, t12 − t 2 = −
1
(t1 − t )
⇒ f ′ ′  = 48   − 30  
 3  3   3  2t
10 × 64 1
= 16 × 8 − = 128 −
640
=−
256
<0 ⇒ (t1 − t ) (t1 + t ) = − (t1 − t )
3 3 2 2t
1
2 2 ⇒ (t1 + t ) = −
Therefore, f (r ) is maximum when, r = 2t
3
1
Hence, maximum value of A ⇒ t1 = − t −
2t
2 2
1 2 2 2 2 1  8 8 Therefore, length of chord,
=   4−  =   . 4−
4 3  3 
4 3  3 L = AB2 = (t − t1 )2 + (t 2 − t12)2
2 12 − 8 2 . 2 4 4 3 = (t − t1 )2 + (t − t1 )2(t + t1 )2
= . = = =
3 3 3 3 3 3 9 = (t − t1 )2[1 + (t + t1 )2]
1  1 
2 2
 3 (b3 − b2 + b − 1)  
− x + , if 0 ≤ x ≤ 1 =  t + t +  1 +  t − t −  
51. Given, f (x) =   2t    2t  
(b2 + 3b + 2)
 2x − 3 , if 1 ≤ x ≤ 3
  1 
2
1   1 
3
⇒ L =  2t +   1 + 2 = 4t 2 1 + 2
is smallest at x = 1 .  2t   4t   4t 
So, f (x) is decreasing on [0, 1] and increasing on [1, 3].
On differentiating w.r.t. t, we get
Here, f (1) = − 1 is the smallest value at x = 1. 3 2
∴ Its smallest value occur as
dL  1   1   2 
= 8t 1 + 2 + 12t 2 1 + 2  − 3 
dt  4t   4t   4t 
(b3 − b2 + b − 1)
lim f (x) = lim (− x3 ) + 2
x → 1− x → 1– b2 + 3b + 2  1    1  3
= 2 1 + 2 4t 1 + 2  − 
 4t    4t  t 
In order this value is not less than –1, we must have
2 2
b3 − b2 + b − 1  1   2  1   1
≥0 = 2  1 + 2  4t −  = 4  1 + 2  2t − 
b2 + 3b + 2  4t   t  4t   t
dL
(b2 + 1) (b − 1) For maxima or minima, we must have =0
⇒ ≥0 dt
(b + 1) (b + 2)
1 1
⇒ 2t − = 0 ⇒ t2 =
t 2
1
⇒ t=±
∴ b ∈ (−2, − 1) ∪ [1, ∞ ] 2
2
Y dL  1  1  1
52. Now, = 8  1 + 2  − 3   2t − 
y = x2 dt 2  4t   2t   t
2
 1   1
+ 4  1 + 2  2 + 2
 4t   t 
B
 d 2L   1
2
⇒  2 = 0 + 4 1 +  (2 + 2) > 0
A  2
X′ X  dt  t = ± 1 / 2

Therefore, L is minimum, when t = ± 1 / 2.For t = 1 / 2 ,


Y′ point A is (1 / 2 , 1 / 2) and point B is (− 2 , 2). When
Any point on the parabola y = x2 is of the form (t , t 2). t = − 1 / 2 , A is (−1 / 2 , 1 / 2), B is ( 2 , 2).
dy  dy  Again, when t = 1 / 2 , the equation of AB is
Now, = 2x ⇒ = 2t
dx  dx 
x=t
y−2 x+ 2
=
Which is the slope of the tangent. So, the slope of the 1 1
−2 + 2
normal to y = x2 at A (t , t 2) is –1/2t. 2 2
Therefore, the equation of the normal to
 1  1 
y = x2 at A (t , t 2) is ⇒ ( y − 2 )  + 2  = (x + 2)  − 2
  2    2 
 1
y − t 2 =  −  (x − t ) …(i)
 2t  ⇒ − 2 y + 4 = 2x + 2
Suppose Eq. (i) meets the curve again at B(t1 , t12). ⇒ 2x + 2 y − 2 = 0
rankershalt.com

258 Application of Derivatives

and when t = − 1 / 2 , the equation of AB is P

y −2 x− 2
= r
− 2 −   − 2

1 1
2  2 O
r r
 1  1 
⇒ ( y − 2)  − − 2 = (x − 2 )  − 2
 2  2  Q R
N
⇒ 2 y − 4 = 2 (x − 2 ) M
⇒ 2x − 2 y + 2 = 0
53. Let 2b be the diameter of the circular portion and a be Let A denotes the area of ∆PQR.
the lengths of the other sides of the rectangle. 1
Then, A = ⋅ 2r sin θ (r + r cos θ )
Total perimeter = 2a + 4b + πb = K [say] …(i) 2
Now, let the light transmission rate (per square metre) ⇒ A = r 2(sin θ + sin θ cos θ )
1
of the coloured glass be L and Q be the total amount of ⇒ A = r 2(sin θ + sin 2θ )
transmitted light. 2
dA
⇒ = r (cos θ + cos 2θ )
2

Coloured dθ
glass d 2A
2b and = r 2(− sin θ − 2 sin 2θ )
dθ 2
dA
For maximum and minimum values of θ,we put =0
a Clear glass a dθ
⇒ cos θ + cos 2θ = 0 ⇒ cos 2θ = − cos θ
π
⇒ cos θ = cos (π − 2θ ) ⇒ θ =
2b 3
1 d 2A π
Then, Q = 2ab (3L ) + πb2(L ) Clearly, < 0 for θ =
2 dθ 2 3
L π
⇒ Q = (πb + 12ab)
2 Hence, the area of ∆PQR is maximum when θ = .
2 3
L The maximum area of ∆ PQR is given by
⇒ Q = [πb2 + 6b (K − 4b − πb)] [from Eq. (i)]
2  π 1 2π   3 3
A = r 2 sin + sin  = r 2  + 

L
Q = (6Kb − 24b2 − 5πb2)  3 2 3  2 4
2
On differentiating w.r.t. b, we get 3 3 2
= r sq units
dQ L 4
= (6K − 48b − 10πb)
db 2 55. Let P (a cos θ , 2 sin θ ) be a point on the ellipse
dQ
For maximum, put =0 x2 y2
db 4x2 + a 2y2 = 4a 2, i.e. + =1
a2 4
6K
⇒ b= …(ii) Let A(0, − 2) be the given point.
48 + 10 π
Then,
d 2Q L ( AP )2 = a 2 cos 2 θ + 4 (1 + sin θ )2
Now, = (− 48 − 10π ) < 0
db2 2 d
⇒ ( AP )2 = − a 2 sin 2θ + 8 (1 + sin θ ) ⋅ cos θ
Thus, Q is maximum and from Eqs. (i) and (ii), we get dθ
(48 + 10π ) b = 6 {2a + 4b + πb} d
2b 6 ⇒ ( AP )2 = [(8 − 2a 2) sin θ + 8 ] cos θ
∴ Ratio = = = 6 :6 + π dθ
a 6+ π d
For maximum or minimum, we put ( AP )2 = 0
54. Since, the chord QR is parallel to the tangent at P. dθ
∴ ON ⊥ QR ⇒ [(8 − 2a 2) sin θ + 8] cos θ = 0
4
Consequently, N is the mid-point of chord QR. ⇒ cos θ = 0 or sin θ =
a −4
2
∴ QR = 2QN = 2r sin θ
4
Also, ON = r cos θ [Q 4 < a 2 < 8 ⇒ > 1 ⇒ sin θ > 1, which is
a2 − 4 impossible]
∴ PN = r + r cos θ
rankershalt.com

Application of Derivatives 259

d2 =
1
area of ∆ABC
Now, ( AP )2 = − {(8 − 2a 2) sin θ + 8} sin θ
dθ 2 2
+ (8 − 2a 2) ⋅ cos 2 θ p2 − p 1
1 1 2
π d 2
= × q q 1
For θ = , we have ( AP )2 = − (16 − 2a 2) < 0 2 2 2
2 dθ 2 r −r 1
π
Thus, AP 2 i.e. AP is maximum when θ = .The point on Applying R3 → R3 − R1 and R2 → R2 − R1
2
the curve 4x2 + a 2y2 = 4a 2 that is farthest from the point p2 −p 1
1
 π π = q −p
2 2
q+ p 0
A(0, − 2) is  a cos , 2 sin  = (0, 2) 4
 2 2 r 2 − p2 − r + p 0
56. Let AF = x and AE = y, ∆ABC and ∆EDC are similar. p2 −p 1
1

AB AC
= = ( p + q) (r − p) q − p 1 0
ED CE 4
r + p −1 0
1
C
2 =
( p + q) (r − p) (− q − r )
(r ,–r) 4
1
= ( p + q) (q + r ) ( p − r )
a 4
b E D
π π
y
57. Let y = f (x) = sin3 x + λ sin 2 x, − < x <
2 2
Let sin x = t
A 2 x B
(p , –p ) F (q 2, –q) ∴ y = t3 + λt 2, − 1 < t < 1
c dy
⇒ = 3t 2 + 2tλ = t (3 t + 2λ )
dt
c b
⇒ = For exactly one minima and exactly one maxima dy/dt
x b− y must have two distinct roots ∈ (−1, 1).
c 2λ
⇒ bx = c (b − y) ⇒ x= (b − y) ⇒ t = 0 and t = − ∈ (−1, 1)
b 3
Let z denotes the area of par 2λ
⇒ −1 < − <1
allelogram AFDE. 3
z = xy sin A 3 3
Then, ⇒ − <λ<
c 2 2
⇒ z = (b − y) y ⋅ sin A …(i)
b  3 3
⇒ λ ∈− , 
 2 2
On differentiating w.r.t. y we get
dz c d 2z −2c x
= (b − 2 y) sin A and = sin A 58. Given, y=
dy b dy2 b 1 + x2
For maximum or minimum values of z, we must have dy (1 + x2) ⋅ 1 − x (2x) 1 − x2
⇒ = =
dz dx (1 + x2)2 (1 + x2)2
=0
dy dy
c b Let = g (x) [i.e. slope of tangent]
⇒ (b − 2 y) = 0 ⇒ y = dx
b 2
1 − x2
d 2z 2c ∴ g (x) =
Clearly, =− < 0, ∀ y (1 + x2)2
2
dy b
(1 + x2)2 ⋅ (−2x) − (1 − x2) ⋅ 2 (1 + x2) ⋅ 2x
b ⇒ g ′ (x) =
Hence, z is maximum, when y = . (1 + x2)4
2
b −2x (1 + x2) [(1 + x2) + 2 (1 − x2)] −2x (3 − x2)
On putting y = in Eq. (i), we get = =
2 (1 + x2)4 (1 + x2)3
the maximum value of z is For greatest or least values of m, we should have
c b b 1
z =  b −  ⋅ ⋅ sin A = bc sin A g′ (x) = 0 ⇒ x = 0, x = ± 3
b 2 2 4
rankershalt.com

260 Application of Derivatives

Now, b 2ax3 − b
∴ f ′ (x) = 2ax − =
(1 + x ) (6x − 6) − (2x − 6x) ⋅ 3 (1 + x ) ⋅ 2x
23 2 3 2 2
x2 x2
g′ ′ (x) =
(1 + x2)6 2b
⇒ f ′ ′ (x) = 2a + 3 > 0 [since, a , b are all positive]
x
At x = 0, g′ ′ (x) = − 6 < 0
1/3
∴ g′ (x) has a maximum value at x = 0.  b
Now, put f ′ (x) = 0 ⇒ x =   >0 [Q a , b > 0]
 2a 
⇒ (x = 0, y = 0) is the required point at which tangent to
the curve has the greatest slope. 1/3
 b
At x=  , f ′ ′ (x) = + ve
59. Let the house of the swimmer be at B.  2a 
∴ AB = L km  b
1/3
⇒ f (x) has minimum at x =   .
Let the swimmer land at C on the shore and let  2a 
AC = x km  b  1/3  2 /3
 = a  
b b
and f    + − c≥ 0
S   2a    2 a (b / 2a )1/3
1/3
 2a  3b
=  ⋅ − c≥0
2
x +d
2  b 2
d
1/3
 2a  3b
⇒   ⋅ ≥c
 b 2
A x C (L – x) B
On cubing both sides, we get
L
2a 27b3
⋅ ≥ c3
∴ SC = x2 + d 2 and CB = (L − x) b 8
Distance ⇒ 27ab2 ≥ 4c3
∴ Time =
Speed 61. Let f (x) = x + y, where xy = 1
Time from S to B = Time from S to C + Time from C to B 1
⇒ f (x) = x +
x +d 2
L−x
2 x
∴ T= +
u v 1 x2 − 1
⇒ f ′ (x) = 1 − =
1 L x x2 x2
Let f (x) = T = x + d2 +
2

u v v Also, f ′ ′ (x) = 2 / x3
1 1 ⋅ 2x 1 On putting f ′ (x) = 0, we get
⇒ f ′ (x) = ⋅ +0−
u 2 x2 + d 2 v
x = ± 1, but x > 0 [neglecting x = − 1]
For maximum or minimum, put f ′ (x) = 0 f ′ ′ (x) > 0, for x = 1
⇒ v2x2 = u 2 (x2 + d 2) Hence, f (x) attains minimum at x = 1, y = 1
u 2d 2 ⇒ (x + y) has minimum value 2.
⇒ x2 = 2
v − u2 62. Here, volume of cylindrical container, V = πr 2h …(i)
ud
∴ f ′ (x) = 0 at x = ± , (v > u ) and let volume of the material used be T.
v − u2
2
r+2
− ud
But x≠ 2
v2 − u 2 O r
ud
∴ We consider, x =
v2 − u 2
h
1 d2
Now, f ′ ′ (x) = > 0, ∀ x
u x2 + d 2 (x2 + d 2)
ud
Hence, f has minimum at x = .
v − u2
2
∴ T = π [(r + 2)2 − r 2] h + π (r + 2)2 × 2
b V
60. Given, ax2 + ≥ c, ∀ x > 0 ; a , b > 0 ⇒ T = π [(r + 2)2 − r 2] ⋅ 2 + 2π (r + 2)2
x πr
b V
Let f (x) = ax2 + − c [Q V = πr 2h ⇒ h = ]
x πr 2
rankershalt.com

Application of Derivatives 261

1  2 3 ⋅ sin3 θ 
2
 r + 2
⇒ T =V   + 2π (r + 2) − V
2
∴ ∆1 = ∆ max occurs at cos θ = = 
 r  4  cos θ 
On differentiating w.r.t. r, we get 1 45 5
When cos θ = =
dT  r + 2   −2  4 8
= 2V ⋅   ⋅   + 4π (r + 2)
dr  r   r2  1
∆ 2 = ∆ min occurs at cos θ =
dT 2
At r = 10, =0
dr  2 3 sin3 θ 
= 
 V  cos θ 
Now, 0 = (r + 2) ⋅ 4  π − 3 
 r  1 9
V When cos θ = =
⇒ =π 2 2
r3 8
∴ ∆1 − 8∆ 2 = 45 − 36 = 9
where r = 10 5
V
⇒ =π 64. PLAN
1000
(i) Local maximum and local minimum are those points at which
V f ′ ( x ) = 0, when defined for all real numbers.
or =4
250π (ii) Local maximum and local minimum for piecewise functions
are also been checked at sharp edges.
63. PLAN As to maximise or minimise area of triangle, we should find
x, if x ≥ 0
area in terms of parametric coordinates and use second Description of Situation y =|x|= 
derivative test.  − x, if x < 0
Here, tangent at P(2 cos θ , 3 sin θ ) is (x2 − 1), if x ≤ − 1 or x ≥ 1
Also, y =|x2 − 1|= 
 (1 − x ), if − 1 ≤ x ≤ 1
2

P (2 cos θ, √3 sin θ)  − x + 1 − x2 , if x ≤ − 1

− x + 1 − x , if − 1 ≤ x ≤ 0
2
y =|x|+ |x2 − 1|= 
x + 1 − x , if 0 ≤ x ≤ 1
O (h,0) 2
R
(2 sec θ, 0)
Q  x + x2 − 1 , if x ≥ 1
(2 sec θ, –√3 sin θ)  − x2 − x + 1, if x ≤ − 1
 2
x y  − x − x + 1, if − 1 ≤ x ≤ 0
cos θ + sin θ = 1 = 2
2 3 − x + x + 1, if 0 ≤ x ≤ 1
∴ R(2 sec θ , 0)  x2 + x − 1, if x≥1
⇒ ∆ = Area of ∆PQR which could be graphically shown as
1
= (2 3 sin θ ) (2 sec θ − 2 cos θ ) Y
2
= 2 3 ⋅ sin3 θ/cos θ …(i)
– x 2– x + 1 – x 2– x + 1 – x 2+ x +1 x 2+x –1
1
Since, ≤ h ≤1
2 1
1 X
∴ ≤ 2 cos θ ≤ 1 –1 –1/2 O 1/2 1
2
1 −1
1 1 Thus, f (x) attains maximum at x = , and f (x)
⇒ ≤ cos θ ≤ …(ii) 2 2
4 2 attains minimum at x = − 1, 0, 1.
d∆ 2 3 {cos θ ⋅ 3 sin 2 θ cos θ − sin3 θ (− sin θ )} ⇒ Total number of points = 5
∴ =
dθ cos 2 θ 65. PLAN If f( x ) is least degree polynomial having local maximum and
2 3 ⋅ sin 2 θ local minimum at α and β.
= [3 cos 2 θ + sin 2 θ ]
cos 2 θ Then, f ′ (x) = λ (x − α ) (x − β )
2 3 sin θ2
Here, p′ (x) = λ (x − 1) (x − 3) = λ (x2 − 4x + 3)
= ⋅ [2 cos 2 θ + 1]
cos 2 θ On integrating both sides between 1 to 3, we get
= 2 3 tan θ (2 cos θ + 1) > 0
2 2 3 3
∫1 p′ (x) dx = ∫1 λ (x − 4x + 3) dx
2
1 1
When ≤ cos θ ≤ , x 3 3
4 2 ⇒ ( p(x))31 = λ  − 2x2 + 3x
3 1
rankershalt.com

262 Application of Derivatives

 1  f (θ ) =
1
⇒ p(3) − p(1) = λ  (9 − 18 + 9) −  − 2 + 3  68. Let
 3  sin θ + 3 sin θ cos θ + 5 cos 2 θ
2

− 4 Again let, g (θ ) = sin 2 θ + 3 sin θ cos θ + 5 cos 2 θ


⇒ 2 −6 = λ  
3  1 − cos 2 θ  1 + cos 2 θ  3
= +5  + sin 2 θ
2  2  2
⇒ λ =3
⇒ p′ (x) = 3 (x − 1) (x − 3) 3
= 3 + 2 cos 2 θ + sin 2 θ
2
∴ p′ (0) = 9
9
66. f (x) = x − 4x3 + 12x2 + x − 1
4 ∴ g (θ )min = 3 − 4 +
4
f ′ (x) = 4x3 − 12x2 + 24x + 1 5 1
=3− =
f ′′ (x) = 12x2 − 24x + 24 = 12 (x2 − 2x + 2) 2 2
1
= 12 {(x − 1)2 + 1} > 0 ∀x ∴ Maximum value of f (θ ) = =2
12
⇒ f ′ (x) is increasing.
Since, f ′ (x) is cubic and increasing.
69. Given, A = { x|x2 + 20 ≤ 9x} = { x|x ∈ [4, 5]}
⇒ f ′ (x) has only one real root and two imaginary roots. Y
∴ f (x) cannot have all distinct roots.
⇒ Atmost 2 real roots.
O 2 3 4
Now, f (− 1) = 15, f (0) = − 1, f (1) = 9 X
5
∴ f (x) must have one root in (− 1, 0) and other in (0, 1). –16
⇒ 2 real roots. –20
–21
67. Let g (x) = e f ( x ), ∀ x ∈ R
⇒ g′ (x) = ef ( x ) ⋅ f ′ (x)
⇒ f ′ (x) changes its sign from positive to negative in the Now, f ′ (x) = 6(x2 − 5x + 6)
neighbourhood of x = 2009
Put f ′ (x) = 0 ⇒ x = 2, 3
⇒ f (x) has local maxima at x = 2009.
f (2) = − 20, f (3) = − 21, f (4) = − 16, f (5) = 7
So, the number of local maximum is one.
From graph, maximum value of f (x) on set A is f (5) = 7.

Download Chapter Test


http://tinyurl.com/yxhc5me3 or
rankershalt.com

11
Indefinite Integration
Topic 1 Some Standard Results
Objective Questions (Only one correct option) 5x
sin
1. Let α ∈ (0, π / 2) be fixed. If the integral 5. ∫ 2 dx is equal to
x
tan x + tan α sin
∫ tan x − tan α
dx = A (x) cos 2α + B (x) 2
(where, C is a constant of integration )
sin 2 α + C, where C is a constant of integration, then the (2019 Main, 8 April I)
functions A (x) and B (x) are respectively (a) 2x + sin x + 2 sin 2x + C
(2019 Main, 12 April II) (b) x + 2 sin x + 2 sin 2x + C
(a) x + α and log e|sin(x + α )| (c) x + 2 sin x + sin 2x + C
(b) x − α and log e|sin(x − α )| (d) 2x + sin x + sin 2x + C
(c) x − α and log e|cos(x − α )| 3x13 + 2x11
6. The integral ∫ dx is equal to (where C
(d) x + α and log e|sin(x − α )| (2x4 + 3x2 + 1)4
2x3 − 1 is a constant of integration) (2019 Main, 12 Jan II)
2. The integral ∫ dx is equal to
x4 + x (a)
x4
+C (b)
x12
+C
(here C is a constant of integration) (2019 Main, 12 April I) 6(2x + 3x + 1)
4 2 3
6(2x + 3x 2 + 1)3
4

|x3 + 1| (x3 + 1)2 x4 x12


(a)
1
log e +C (b)
1
log e +C (c) +C (d) +C
2 x2 2 |x3| (2x + 3x + 1)
4 2 3
(2x + 3x 2 + 1)3
4

x3 + 1 |x3 + 1| x+1
(c) log e
x
+C (d) log e
x2
+C 7. If ∫
2x − 1
dx = f (x) 2x − 1 + C, where C is a

  constant of integration, then f (x) is equal to


dx  x − 1 f (x)
3. If = A  tan −1 
∫ (x2 − 2x + 10)2  + 2  + C, (2019 Main, 11 Jan II)
  3  x − 2 x + 10  2 1
(a) (x + 2) (b) (x + 4)
where, C is a constant of integration, then 3 3
2 1
(2019 Main, 10 April I) (c) (x − 4) (d) (x + 1)
1 3 3
(a) A = and f (x) = 9 (x − 1)
27 1 − x2
(b) A =
1
and f (x) = 3 (x − 1) 8. If ∫ dx = A (x)( 1 − x2 )m + C,
81 x4
1 for a suitable chosen integer m and a function A (x),
(c) A = and f (x) = 3 (x − 1)
54 where C is a constant of integration, then ( A (x))m
1 equals
(d) A = and f (x) = 9 (x − 1)2 (2019 Main, 11 Jan I)
54 1 −1
(a) (b)
dx
1 9x4 3x3
4. If ∫ = xf (x)(1 + x6 )3 +C (c)
−1
(d)
1
x3 (1 + x6 )23
/
27x9 27x6
where, C is a constant of integration, then the function π
f (x) is equal to 9. Let n ≥ 2 be a natural number and 0 < θ < . Then,
(2019 Main, 8 April II) 2
1 1 1
(a) − (b) −
3 3 (sin n θ − sin θ ) n cos θ
6x
1 3
2x
∫ sin n + 1 θ
dθ is equal to
(c) − (d)
2x2 x2 (where C is a constant of integration)
(2019 Main, 10 Jan I)
rankershalt.com

264 Indefinite Integration


n+1 −1 1 − (sec x + tan x)2  + K
1
n 1 − 1  n (a) 
(a)   +C (sec x + tan x)11/ 2 11 7 
n 2 − 1 sin n + 1 θ 
(b)
1 1 − 1 2
(sec x + tan x)  + K
n+1 
1 +
n 1  n (sec x + tan x)11/ 2 11 7 
(b) 2  n −1 
+C
n −1  sin θ −1 1 + 1 2
(c)  (sec x + tan x)  + K
n+1 (sec x + tan x)11/ 2 11 7 
n 1 − 1  n
+C 1 1 + 1 2
(c)   (d) (sec x + tan x)  + K
n 2 − 1 sin n − 1 θ  
(sec x + tan x)11/ 2 11 7 
n+1
n 1 − 1  n
+C ex e− x
(d)   15. If I = ∫ dx, J = ∫ e− 4x + e−2x + 1 dx.
n 2 + 1 sin n − 1 θ  e4x + e2x + 1
5x + 7x 8 6
Then, for an arbitrary constant c, the value of J − I
10. If f (x) = ∫ dx, (x ≥ 0), and f (0) = 0, then
(x2 + 1 + 2x7 )2 equals (2008, 3M)
the value of f (1) is (2019 Main, 9 Jan I) 1 e4 x − e2x + 1 1 e2x + ex + 1
(a) log 4 x +c (b) log 2x +c
1 1 e + e2x + 1 e − ex + 1
(a) − (b) − 2 2
2 4
1 1 1 e2x − ex + 1 1 e4 x + e2x + 1
(c) (d) (c) log 2x +c (d) log 4 x +c
4 2 2 e + ex + 1 2 e − e2x + 1
11. For x2 ≠ nπ + 1, n ∈ N (the set of natural numbers), the x
16. If f (x) = for n ≥ 2 and g (x) = ( fofo ... of ) (x).
integral (1 + xn )1/ n 14243
f occurs n times
2 sin(x2 − 1) − sin 2(x2 − 1) Then, ∫ xn − 2g (x) dx equals
∫ x 2 sin(x2 − 1) + sin 2(x2 − 1) dx is equal to 1
(2007, 3M)

1 1−
(where C is a constant of integration ) (2019 Main, 9Jan I) (a) (1 + nxn ) n +c
1 n (n − 1)
(a) log e|sec(x2 − 1)| + C 1
2 1 1−
(b) (1 + nxn ) n + c
 x2 − 1 n −1
(b) log e sec   +C 1
 2  1 1+
(c) (1 + nxn ) n + c
1 n (n + 1)
(c) log e sec2 (x2 − 1) + C
2 1
1 1+
 x2 − 1 (d) (1 + nxn ) n + c
1 n+1
(d) log e sec2   +C
2  2  (x2 − 1) dx
17. The value of ∫ is (2006, 3M)
12. The integral x3 2 x4 − 2 x2 + 1
sin 2 x cos 2 x
∫ (sin5 x + cos3 x sin 2 x + sin3 x cos2 x dx (a) 2 2 −
2
+
1
+c (b) 2 2 +
2
+
1
+ c
x2 x4 x2 x4
+ cos x)5 2
1 2 1
is equal to (2018 Main) (c) 2− 2 + 4 + c (d) None of these
2 x x
1 −1
(a) +C (b) +C
3 (1 + tan3 x) 3 (1 + tan3 x)
1 −1
One or More Than One
(c) +C (d) +C
1 + cot3 x 1 + cot3 x 18. Let f : R → R and g : R → R be two non-constant
differentiable functions. If f ′ (x) = (e( f ( x ) − g( x )) ) g′ (x) for
(where C is a constant of integration) all x ∈ R and f (1) = g (2) = 1, then which of the following
dx
13. The value of ∫ 2 4 is statement(s) is (are) TRUE? (2018 Adv.)
x (x + 1)3/ 4 (2015 Main) (a) f (2) < 1 − log e 2 (b) f (2) > 1 − log e 2
1
x +4
1 4
1 (c) g (1) > 1 − log e 2 (d) g (1) < 1 − log e 2
(a)  4  + c (b) (x4 + 1) 4 +c
 x 
1
1 Numerical Value
 x4 + 1 4
(c) − (x + 4
1) 4 +c (d) −  4  + c 19. Let f : R → R be a differentiable function with f (0) = 1
 x 
2 and satisfying the equation f (x + y) = f (x) f ′ ( y)
sec x
14. ∫ (sec x + tan x)9/ 2 dx equals to + f ′ (x) f ( y) for all x, y ∈ R.
Then, the value of log e ( f (4)) is ....... . (2018 Adv.)
(for some arbitrary constant K) (2012)
rankershalt.com

Indefinite Integration 265

dx
Fill in the Blank 24. Evaluate ∫ x2(x4 + 1)3/ 4 . (1984, 2M)
4ex + 6e− x
20. If ∫ x dx = Ax + B log (9e2x − 4) + C, then A = K,
9e − 4e− x 25. Evaluate the following: (1980, 4M)
2
B = ... and C = K . 1  x
∫ ∫
(1989, 2M)
(i) 1 + sin  x dx (ii) dx
2  1−x
Analytical & Descriptive Questions x2
21. For any natural number m, evaluate 26. Integrate . (1979, 2M)
(a + bx)2
∫ (x + x2m + xm ) (2 x2m + 3 xm + 6)1/m dx, x > 0. (2002, 5M)
3m
27. Integrate
1/ 2 sin x ⋅ sin 2 x ⋅ sin 3x + sec2 x ⋅ cos 2 2x + sin 4 x ⋅ cos 4 x
 1 − x dx .

(1979, 1M)
22. Evaluate   ⋅ (1997C, 3M)
 1 + x x 28. Integrate the curve
x
. (1978, 1M)
1 + x4
1− x
23. Evaluate ∫ dx. (1985, 2 1 M) 29. Integrate
1
or
sin x
. (1978, 2M)
1+ x 2
1 − cot x sin x − cos x

Topic 2 Some Special Integrals


Objective Question I (Only one correct option) Analytical & Descriptive Questions
1. The integral ∫ sec 23
/ 4/3
x cosec x dx is equal to (here C is a 3. Find the indefinite integral
constant of integration)  1 ln (1 + 6 x )

(2019 Main, 9 April I)
3 +  dx. (1992, 4M)
(a) 3 tan −1/3
x+C (b) −3 tan −1/3
x+C  x+ 4
x 3
x+ x 
3
(c) −3 cot −1/3 x + C −4 /3
(d) − tan
4
x+C 4. Evaluate ∫( tan x + cot x ) dx. (1988, 3M)

2. Let I n = ∫ tan n x dx (n > 1). If (cos 2x)1/ 2


5. Evaluate ∫ sin x dx. (1987, 6M)
I 4 + I 6 = a tan5 x + bx5 + C, where C is a constant of
integration, then the ordered pair (a , b) is equal to 2 sin x − sin 2 x
6. If f (x) is the integral of , where x ≠ 0, then
(2017 Main) x3
(a)  − , 1 (b)  , 0 (c)  , − 1 (d)  − , 0
1 1 1 1 find lim f ′ (x). (1979, 3M)
 5  5  5   5  x→ 0

Topic 3 Integration by Parts


Objective Questions (Only one correct option) 3. The integral ∫ cos (log e x) dx is equal to (where C is a
5 − x2 − x2
1. If ∫xe dx = g (x)e + C, where C is a constant of constant of integration) (2019 Main, 12 Jan I)

[cos(log e x) + sin(log e x)] + C


x
integration, then g (− 1) is equal to (2019 Main, 10 April II) (a)
2
(a) − 1 (b) 1 (b) x [cos(log e x) + sin(log e x)] + C
1 5
(c) − (d) − (c) x [cos(log e x) − sin(log e x)] + C
2 2
(d) [sin(log e x) − cos(log e x)] + C
x
2. If ∫ esec x 2
(sec x tan x f (x) + (sec x tan x + sec2 x)) 3 1 −4 x 3
4. If ∫ x5 e−4x dx = e f (x) + C ,
dx = esec x f (x) + C, then a possible choice of f (x) is 48
(2019 Main, 9 April II)
where C is a constant of integration, then f (x) is equal to
1 1
(a) x sec x + tan x + (b) sec x + tan x + (2019 Main, 10 Jan II)
2 2 (a) − 4x3 − 1 (b) 4x3 + 1
(c) sec x + x tan x −
1
(d) sec x − tan x −
1 (c) − 2x3 − 1 (d) − 2x3 + 1
2 2
rankershalt.com

266 Indefinite Integration


1
 1 x + Analytical & Descriptive Questions
5. ∫ 1 + x −  e x dx is equal to
 x
(2014 Main)
 2x + 2 
x+
1
x+
1
7. Evaluate ∫ sin −1   dx. (2000, 5M)
(a) (x − 1) e x + c (b) x e x + c  4x2 + 8 x + 13 
 
1 1
x+ x+
(c) (x + 1) e x + c (d) − x e x + c 8. Find the indefinite integral
6. If ∫ f (x) dx = ψ (x), then ∫ x f (x ) dx is equal to
5 3  cos θ + sin θ 
∫ cos 2θ log cos θ − sin θ  dθ. (1994, 5M)
1 3
(a) [x ψ(x3 ) − ∫ x2ψ(x3 )dx] + c (2013 Main)
3 sin −1 x − cos −1 x
1 3 9. Evaluate ∫ dx. (1986, 2½ M)
(b) x ψ(x3 ) − 3 ∫ x3 ψ(x3 ) dx + c sin −1 x + cos −1 x
3
1 3
x ψ(x3 ) − ∫ x2ψ(x3 ) dx + c (x − 1) ex
(c)
3 10. Evaluate ∫ dx. (1983, 2M)
(x + 1)3
1 3
(d) [x ψ(x3 ) − ∫ x3 ψ(x3 ) dx] + c
3 11. Evaluate ∫ (elog x + sin x) cos x dx. (1981, 2M)

Topic 4 Integration, Irrational Function and Partial Fraction


Objective Questions (Only one correct option) cos3 x + cos5 x
2. The value of ∫ dx is (1995, 2M)
2x12 + 5x9 sin 2 x + sin 4 x
1. The integral ∫ dx is equal to
(x5 + x3 + 1)3 (2016 Main) (a) sin x − 6 tan −1 (sin x) + c
−x5 (b) sin x − 2 (sin x)−1 + c
(a) +C
(x + x + 1)
5 3 2 (c) sin x − 2 (sin x)−1 − 6 tan −1 (sin x) + c
x10 (d) sin x − 2 (sin x)−1 + 5 tan −1 (sin x) + c
(b) +C
2(x5 + x3 + 1)2
x5
Analytical & Descriptive Questions
(c) +C
2(x + x + 1)
5 3 2 x3 + 3x + 2
− x10
3. ∫ (x + 1)2 (x + 1)
2
dx. (1999, 5M)
(d) +C
2(x5 + x3 + 1)2 (x + 1)
4. Evaluate ∫ dx. (1996, 2M)
where, C is an arbitrary constant. x (1 + xex )2

Answers
Topic 1 ( x 4 + 1 )1/4
24. − +c
1. (b) 2. (c) 3. (c) 4. (b) x
x x
5. (c) 6. (b) 7. (b) 8. (c) 25. (i) 4 sin − 4 cos + c
4 4
9. (c) 10. (c) 11. (b) 12. (b)
 2 1 
13. (d) 14. (c) 15. (c) 16. (a) (ii) − 2  1 − x − (1 − x ) 3/2 + (1 − x ) 5/2  + c
 3 5 
17. (c) 18. (b,c) 19. (2)
1  a2 
3 35 26.  a + bx − 2a log (a + bx ) − + c
20. A = − , B = and C ∈R b 
3
a + bx 
2 36
1 cos 4 x cos 2 x cos 6 x
21. ⋅(2 x + 3 x 2m + 6 xm ) (m + 1)/m + c
3m 27. − − + + sin 2 x + tan x − 2 x
6 (m + 1 ) 16 8 24
3 x sin 4 x sin 8 x
1
22. 2 [cos−1 x − log| 1 + 1 − x | − log | x| ] + c + − +
2 128 128 1024
1 −1 1 x
23. −2 1 − x + cos−1 x + x (1 − x ) + c 28. tan ( x ) + c
2
29. log (sin x − cos x ) + + c
2 2 2
rankershalt.com

Indefinite Integration 267

Topic 2 1  cosθ + sin θ  1


8. sin 2 θ ln   + ln(cos2 θ ) + c
1. (b) 2. (b) 2  cosθ − sin θ  2
3 2/3 12 7/12 4 1/2 12 5/12 1 1/3 2
3. x − x + x − x + x − 4 x 1/ 4 − 7 x 1/ 6 9. [ x − x 2 − (1 − 2 x ) sin −1 x ] − x + c
2 7 3 5 2 π
− 12 x1/12 + (2 x1/2 − 3 x1/3 + 6 x1/6 + 11 ) ln (1 + x1/6 ) ex
10. +c
+ 12 ln (1 + x1/2 ) − 3 [ ln(1 + x1/6 )]2 + c (x + 1)2
 tan x − cot x  11. x sin x + cos x −
cos2 x
+c
4. 2 tan −1   +c 4
 2 

1 2 + 1 − tan 2 x Topic 4
5. − log| cot x + cot 2 x − 1 | + log +c
2 2 − 1 − tan x
2 1. (b) 2. (c)
1 1 3 x
6. (1) 3. − log| x + 1 | + log| x 2 + 1 | + tan −1 x + 2 +c
2 4 2 x +1
Topic 3
xe x 1
1. (d) 2. (b) 3. (a) 4. (a) 4. log + +c
1 + xe x 1 + xe x
5. (b) 6. (c)
−1  2x + 2 3
7. ( x + 1 ) tan   − log( 4 x + 8 x + 13 ) + c
2
 3  4

Hints & Solutions


Topic 1 Some Standard Results
tan x + tan α  π
1. Let I = ∫ dx, α ∈ 0,  2. Key Idea
tan x − tan α  2
(i) Divide each term of numerator and denominator by x 2.
sin x sin α
+ (ii) Let x 2 +
1
= t
cos x cos α
=∫ dx x
sin x sin α
− 2x3 − 1 2x − 1 / x2
cos x cos α Let integral is I = ∫ dx = ∫ 2 1 dx
x4 + x x +
sin x cos α + sin α cos x
=∫ dx x
sin x cos α − sin α cos x [dividing each term of numerator and
sin (x + α )
=∫ dx denominator by x2]
sin (x − α )
1   1 
Now, put x − α = t ⇒ dx = dt, so Put x2 + = t ⇒ 2x +  − 2  dx = dt
x   x 
sin (t + 2α )
I=∫ dt dt
sin t ∴ I=∫ = log e|(t )| + C
t
sin t cos 2 α + sin 2α cos t
=∫ dt  1
sin t = log e  x2 +  + C
 x
 cos t 
= ∫  cos 2 α + sin 2 α  dt
 sin t  x3 + 1
= log e +C
= t (cos 2 α ) + (sin 2 α ) log e |sin t |+ C x
= (x − α ) cos 2 α + (sin 2 α ) log e |sin (x − α )| + C dx dx
3. Let I = ∫
(x2 − 2x + 10)2 ∫ ((x − 1)2 + 32)2
=
= A (x) cos 2 α + B(x) sin 2 α + C (given)
Now on comparing, we get Now, put x − 1 = 3 tan θ ⇒ dx = 3 sec2θ dθ
A (x) = x − α and B(x) = log e |sin (x − α )| 3sec2θ dθ 3sec2θ dθ
So,I = ∫ =∫ 4
(3 tan θ + 3 )
2 2 2 2
3 sec4θ
rankershalt.com

268 Indefinite Integration

1 1 1 + cos 2θ 5x 5x x
27 ∫ 27 ∫
= cos 2θ dθ = dθ sin 2 sin cos
2 5. Let I = ∫ 2 dx = ∫ 2 2 dx
 1 + cos 2θ  x x x
Q cos θ =
2
sin 2 sin cos
2  2 2 2
1 1  sin 2θ  x
[multiplying by 2 cos in numerator and
54 ∫
= (1 + cos 2θ ) dθ = θ +  +C 2
54  2 
denominator]
1  x − 1 1  2 tan θ  sin 3x + sin 2x
= tan −1   +   +C =∫ dx
54  3  108  1 + tan 2 θ  sin x
[Q2 sin A cos B = sin( A + B) + sin( A − B) and
 2 tan θ 
Q sin 2θ =  sin 2 A = 2 sin A cos A]
 1 + tan 2 θ 
(3 sin x − 4 sin3 x) + 2 sin x cos x
=∫ dx
 x − 1 sin x
  [Q sin 3x = 3 sin x − 4 sin3 x]
1  x − 1 1  3 
= tan −1   + +C = ∫ (3 − 4 sin x + 2 cos x)dx
2
54  3  54  x − 1
2
1+  
 3  = ∫ [3 − 2(1 − cos 2x) + 2 cos x]dx
1  x − 1 1  x−1 
[Q2 sin 2 x = 1 − cos 2x]
= tan −1   +  2
+C
54  3  18  (x − 1) + 3 
2
= ∫ [3 − 2 + 2 cos 2x + 2 cos x]dx
1  x − 1 1  x−1 
= tan −1   +  2  +C = ∫ [1 + 2 cos 2x + 2 cos x]dx
54  3  18  x − 2x + 10
= x + 2 sin x + sin 2x + C
1  −1  x − 1  3(x − 1) 
= tan   + 2 +C 6. Let
54  3  x − 2x + 10 
3 2
+ 5
It is given, that 3x13 + 2x11 3
I=∫ dx = ∫ x x dx
  x − 1 f (x)  (2x4 + 3x2 + 1)4  3 1
4
I = A tan −1   + 2 +C 2 + 2 + 4 
  3  x − 2x + 10   x x 
1 [on dividing numerator and denominator by x16]
On comparing, we get A = and f (x) = 3(x − 1). 3 1
54 Now, put 2 + 2 + 4 = t
dx x x
4. Let I = ∫
x3 (1 + x6 )2/ 3 −6 4  3 2 dt
⇒  3 − 5  dx = dt ⇒  3 + 5  dx = −
dx dx x x  x x  2
=∫ =∫
1 
2/ 3
1 
2/ 3 − dt 1 t− 4 + 1 1
x3 ⋅ x4  6 + 1 x7  6 + 1 So, I = ∫ =− × +C= 3 +C
x  x  2t 4
2 −4 + 1 6t
1 1  3 1
Now, put + 1 = t3 = 3
+C Q t = 2 + x2 + x4 
x6  3 1
6 2 + 2 + 4 
6  x x 
⇒ − 7 dx = 3t 2dt
x x12
2 = +C

dx t
= − dt 6 (2x4 + 3x2 + 1)3
7
x 2
1 2
7. We have,
− t dt x+1
So, I = ∫ 2 1
= − ∫ dt ∫ 2x − 1
dx = f (x) 2x − 1 + C ...(i)
t2 2
1/3 x+1
1 1 1   3 1  Let I = ∫ dx
=− t + C = −  6 + 1 + C Q t = x6 + 1 2x − 1
2 2 x 
1 1 Put 2x − 1 = t 2 ⇒ 2dx = 2tdt ⇒ dx = tdt
=− (1 + x6 )1/3 + C
2 x2 t2 + 1
+1
= x ⋅ f (x) ⋅ (1 + x6 )1/3 + C 1
[given] I=∫ 2 tdt = ∫ (t 2 + 3) dt
On comparing both sides, we get t 2
1  t2 + 1 
f (x) = − 3 Q 2x − 1 = t ⇒ x =
2

2x  2 
rankershalt.com

Indefinite Integration 269

1  t3  1
t +1
=  + 3t + C = (t 2 + 9) + C u1/ ndu un
23  6 ⇒ I=∫ = +C
n −1 1 
2x − 1 (n − 1)  + 1
= (2x − 1 + 9) + C [Q t = 2x − 1 ] n 
6 n+1
2x − 1  1  n
= (2x + 8) + C n 1 − n−1 
6  t 
= +C
x+4 (n − 1) (n + 1)
= 2x − 1 + C
3 n+1
 1  n
On comparing it with Eq. (i), we get n 1 − n −1 
 sin θ 
x+4 = +C
f (x) = n2 − 1
3
 1 
8. We have, Q u = 1 − t n − 1 and t = sin θ 
1 − x2
∫ x4
dx = A (x) ( 1 − x2 )m + C … (i)
10. We have, f (x) = ∫
5x8 + 7x6
dx
(x + 1 + 2x7 )2
2
1 
x2 2 − 1  x8   x6 
1 − x2 x  5 14  + 7 14 
LetI = ∫ dx = ∫ dx x  x 
x4 x4 =∫ dx
2
1  x2 1 2x7 
x 2 −1  7 + 7 + 7
=∫ x 1 1
dx = ∫ 3 − 1 dx x x x 
x4 x x2 (dividing both numerator and denominator by x14)
1 −2 1
Put 2 − 1 = t 2 ⇒ 3 dx = 2t dt ⇒ 3 dx = − t dt 5x− 6 + 7x− 8
x x x = ∫ −5 dx
(x + x− 7 + 2)2
t3
∴ I = − ∫ t 2dt = − +C Let x− 5 + x− 7 + 2 = t
3
3/ 2 ⇒ (− 5x− 6 − 7x− 8 )dx = dt
1  1 − x2   1  
1/ 2
=− .  + C Q t =  2 − 1  ⇒ (5x− 6 + 7x− 8 )dx = − dt
3  x2   x   dt
∴ f (x) = ∫ − 2 = − ∫ t −2dt
1 1 t
=− ( 1 − x2 )3 + C …(ii)
3 x3 t− 2 + 1 t− 1 1
=− + C =− +C= +C
On comparing Eqs. (i) and (ii), we get −2 + 1 −1 t
1 1 x7
A (x) = − 3 and m = 3 = +C= 7 +C
3x −5 −7
x +x +2 2x + x2 + 1
1
∴ ( A (x))m = ( A (x))3 = − Q f (0) = 0
27 x9 0
∴ 0= + C ⇒C = 0
(sin θ − sin θ ) cos θ
n 1/ n 0+0+1
9. Let I = ∫ dθ
sin n + 1 θ ∴ f (x) =
x7
Put sin θ = t ⇒ cos θ dθ = dt 2x + x2 + 1
7

(t n − t )1/ n 1 1
∴ I=∫ dt ⇒ f (1) = =
tn + 1 2(1) + 1 + 1 4
7 2
1/ n
 n t 
t
   1 −  2 sin(x2 − 1) − sin 2(x2 − 1)
t n   11. Let I = ∫ x
=∫ 
dx
n+1
dt 2 sin(x2 − 1) + sin 2(x2 − 1)
t
t (1 − 1 / t n−1 )1/ n (1 − 1 / t n − 1 )1/ n x2 − 1
=∫ dt = ∫ dt Put = θ ⇒ x2 − 1 = 2θ ⇒ 2x dx = 2 dθ
n+1 2
t tn
1 ⇒ x dx = dθ
Put 1 − n −1 = u
t 2 sin 2 θ − sin 4 θ
Now, I = ∫ dθ
(n − 1) 2 sin 2 θ + sin 4 θ
or 1 − t −( n − 1) = u ⇒ dt = du
tn 2 sin 2 θ − 2 sin 2 θ cos 2 θ
dt du =∫ dθ
⇒ = 2 sin 2 θ + 2 sin 2 θ cos 2 θ
tn n − 1
(Qsin 2 A = 2 sin A cos A)
rankershalt.com

270 Indefinite Integration

2 sin 2 θ (1 − cos 2 θ ) Put sec x + tan x = t


=∫ dθ
2 sin 2 θ (1 + cos 2 θ ) ⇒ (sec x tan x + sec2 x) dx = dt
1 − cos 2 θ 2 sin 2 θ dt
=∫ dθ = ∫ dθ ⇒ sec x ⋅ t dx = dt ⇒ sec x dx =
1 + cos 2 θ t
2 cos 2 θ
1 1 1
[Q1 − cos 2 A = 2 sin 2 A and 1 + cos 2 A = 2 cos 2 A] ∴ sec x − tan x = ⇒ sec x =  t + 
t 2 t
=∫ tan θ d θ = ∫ tan θd θ
2
sec x ⋅ sec x dx
∴ I =∫
(sec x + tan x)9/ 2
 x2 − 1  x2 − 1 
= log e|sec θ| + C = log e sec   +C Q θ =  1 1 dt
 2   2  t +  ⋅
2 t t 1  1 1 
⇒ I=∫ = ∫  9/ 2 + 13/ 2 dt
12. We have, t 9/ 2 2 t t 
sin x ⋅ cos x 2 2
1 2 2 
I=∫ dx =− +
(sin5 x + cos3 x ⋅ sin 2 x + sin3 x ⋅ cos 2 x + cos5 x)2  + K
2  7 t7/ 2 11 t11/ 2 
sin 2 x cos 2 x
=∫ dx  1 1 
{sin x(sin x + cos 2 x) + cos3 x(sin 2 x + cos 2 x)}2
3 2 = − + 11/ 2 
+K
 7 (sec x + tan x) 11 (sec x + tan x) 
7/ 2
sin 2 x cos 2 x sin 2 x cos 2 x
=∫ dx = ∫ dx −1 1 1 
(sin x + cos x)
3 3 2
cos 6 x(1 + tan3 x)2 = 11/ 2 
+ (sec x + tan x)2 + K
(sec x + tan x) 11 7 
tan 2 x sec2 x
=∫ dx ex e3 x
(1 + tan3 x)2 15. Since, I=∫ dx and J = ∫ dx
Put tan3 x = t ⇒ 3 tan 2 x sec2 xdx = dt e4x
+ e +1
2x
1 + e2x + e4x
(e3 x − ex )
∴ I= ∫
1 dt ∴ J −I=∫ dx
3 (1 + t )2 1 + e2x + e4x
−1 −1 Put ex = u ⇒ ex dx = du
⇒ I= +C ⇒ I= +C
3 (1 + t ) 3 (1 + tan3 x)  1
 1 − 2
(u 2 − 1)  u  du
dx dx ∴ J −I = ∫ du = ∫
13. ∫ x2(x4 + 1)3/ 4 = ∫  1
3/ 4 1+ u + u
2 4 1
1 + 2 + u2
x5 1 + 4  u
 x   1
1  1 − 2
Put 1+ = t4  u  du
x4 =∫ 2
−4  1
⇒ dx = 4t3 dt u +  −1
 u
x5
1
dx Put u+ =t
⇒ = − t3 dt u
x5
 1
Hence, the integral becomes ⇒ 1 − 2 du = dt
 u 
1/ 4
− t3 dt  1
∫ t3
= − ∫ dt = − t + c = − 1 + 4 
 x 
+c
=∫
dt 1
= log
t −1
+c
t2 − 1 2 t+1
14. PLAN Integration by Substitution
I = ∫ f { g ( x )} ⋅ g ′ ( x )dx
1 u2 − u + 1
i.e. = log 2 +c
Put g ( x ) = t ⇒ g ′ ( x )dx = dt 2 u +u+1
∴ I = ∫ f(t )dt 1 e2x − ex + 1
= log 2x +c
Description of Situation Generally, students gets 2 e + ex + 1
confused after substitution, i.e. sec x + tan x = t.
x
Now, for sec x, we should use 16. Given, f (x) = for n ≥ 2
(1 + xn )1/ n
sec2 x − tan 2 x = 1
f (x) x
⇒ (sec x − tan x) (sec x + tan x) = 1 ∴ ff (x) = =
[1 + f (x)n ]1/ n (1 + 2 xn )1/ n
1
⇒ sec x − tan x = x
t and fff (x) =
(1 + 3xn )1/ n
sec2 dx
Here, I=∫ x
(sec x + tan x)9/ 2 ∴ g (x) = ( fofo ... of ) (x) =
14243 (1 + n xn )1/ n
n times
rankershalt.com

Indefinite Integration 271

xn − 1 dx ⇒
1
1 = 2 f ′ (0) ⇒ f′ (0) =
Let I = ∫ xn − 2g (x) dx = ∫
(1 + nxn )1/ n 2
d Put x = x and y = 0, we get
(1 + nxn ) f (x) = f (x) f ′ (0) + f ′ (x) f (0)
1 n 2 xn − 1 dx 1 dx
n2 ∫ (1 + nxn )1/ n n 2 ∫ (1 + nxn )1/ n
= = dx ⇒
1
f (x) = f (x) + f ′ (x)
2
1−
1 1 f ′ (x) 1
1 ⇒ f ′ (x) = f (x) ⇒ =
I= (1 + nxn ) n + c 2 f (x) 2
n (n − 1)
On integrating, we get
(x2 − 1) dx 1
17. Let I = ∫ log f (x) = x + C
2x4 − 2x2 + 1
x3 2
1
x
[dividing numerator and enominator by x5 ] ⇒ f (x) = Ae2 , where eC = A
1 1 If f (0) = 1, then A = 1
 3 − 5  dx
x x  1
=∫ Hence, f (x) = e2
x
2 1
2− 2+ 4 1
x x ⇒ log e f (x) = x
2
2 1
Put 2− 2+ 4 =t 1
x x ⇒ log e f (4) = × 4 = 2
2
4 4
⇒  3 − 5  dx = dt 4ex + 6e−x
x x  20. Given, ∫ dx = Ax + B log (9e2x − 4) + c
9ex − 4e−x
1 dt 1 t1/ 2
∴ I=
4 ∫ = ⋅
t 4 1 /2
+c
LHS = ∫
4e2x + 6
dx
9e2x − 4
1 2 1
= 2− 2+ 4 + c Let 4e2x + 6 = A (9e2x − 4) + B (18 e2x )
2 x x
⇒ 9 A + 18B = 4 and − 4 A = 6
18. We have, f ′ (x) = e( f ( x ) − g( x )) g′ (x) ∀ x ∈ R
3 35
e f ( x) ⇒ A=− and B=
⇒ f ′ (x) = g′ (x) 2 36
eg( x )
f ′ (x) g′ (x) A (9e2x − 4) + B (18e2x ) 1

ef ( x )
= g( x )
e
∴ ∫ 9e − 4
2x
dx = A ∫ 1 dx + B ∫
t
dt

⇒ e− f ( x ) f ′ (x) = e− g( x ) g′ (x) where t = 9e2x − 4


On integrating both side, we get
= A x + B log (9e2x − 4) + c
−f ( x ) − g( x )
e =e +C 3 35
At x = 1 =− x+ log (9e2x − 4) + c
2 36
e− f (1) = e− g(1) + C
3 35
e = e− g(1) + C
−1
[Q f (1) = 1] …(i) ∴ A =− ,B=
At x = 2 2 36
e− f ( 2) = e− g( 2) + C and c = any real number
⇒ e− f ( 2) = e−1 + C [Q g(2) = 1] …(ii)
21. For any natural number m , the given integral can be
From Eqs. (i) and (ii) written as
e− f ( 2) = 2e−1 − e− g(1) …(iii) (2x3 m + 3x2m + 6xm )1/ m
⇒ e− f ( 2) > 2e−1 I = ∫ (x3 m + x2m + xm ) dx
x
We know that, e− x is decreasing 1/ m
∴ − f (2) < log e 2 − 1 ⇒ I = ∫ (2 x3 m + 3x2m + 6 xm )
f (2) > 1 − log e 2 (x3 m − 1 + x2m − 1 + xm − 1 ) dx
⇒ e− g(1) + e− f (2) = 2e−1 [from Eq. (iii)]
Put 2 x3 m + 3x2m + 6xm = t
⇒ e− g(1) < 2 e−1
− g (1) < log e 2 − 1 ⇒ (6 mx3 m−1 + 6 mx2m−1 + 6 mxm −1 ) dx = dt
1
⇒ g (1) > 1 − log e 2 +1
dt 1 tm
∴ I=∫t 1 /m
= ⋅
19. Given, 6m 6m  1 
f (x + y) = f (x) f ′ ( y) + f ′ (x) f ( y), ∀x, y ∈ R  + 1
m 
and f (0) = 1
1
Put x = y = 0, we get = ⋅ (2 x3m + 3x2m + 6 xm )(m + 1)/m + c
6 (m + 1)
f (0) = f (0) f ′ (0) + f ′ (0) f (0)
rankershalt.com

272 Indefinite Integration

 1 − x
1/ 2
 x x x x
I=∫  ⋅
dx = ∫  cos + sin  dx = 4 sin − 4 cos + c
22. Let   4 4  4 4
 1 + x x
x2
Put x = cos 2 θ ⇒ dx = − 2 cos θ sin θ dθ (ii) Let I=∫ dx
1/ 2 1−x
 1 − cos θ  − 2 cos θ ⋅ sin θ
∴ I=∫   ⋅ dθ Put 1 − x = t 2 ⇒ − dx = 2 t dt
 1 + cos θ  cos 2 θ
(1 − t 2)2 ⋅ (−2t )
θ ∴ I=∫ dt
sin t
=∫ 2 ⋅ − 2 sin θ d θ
θ cos θ = − 2 ∫ (1 − 2t 2 + t 4 ) dt
cos
2  2t3 t5 
= − 2 t − +  +c
θ θ θ θ  3 5
2 sin ⋅ 2 sin ⋅ cos 2 sin 2
=−∫ 2 2 2 d θ −2
∫ cos θ 2 d θ  2 1 
= − 2  1 − x − (1 − x)3/ 2 + (1 − x)5/ 2 + c
θ
cos ⋅ cos θ  3 5 
2
1 − cos θ x2
= −2∫ dθ 26. Let I=
cos θ (a + bx)2
= 2∫ (1 − sec θ ) dθ = 2 [θ − log|sec θ + tan θ|] + c Put a + bx = t ⇒ b dx = dt
2
 t − a
 1 1   
⇒ I = 2 cos −1 x − log + −1 + c  b  dt 1  t 2 − 2 at + a 2
∴ I=∫ ⋅ = 3 ∫   dt
 x x  t 2
b b  t2 
 1 
⇒ I = 2 cos −1 x − log|1 + 1 − x|− log|x| + c 1  2 a a 2
 2  =
b3 ∫ 1 −
 t
+ 2  dt
t 
1− x
23. Let I=∫ dx 1  a 2
1+ x = 3
 t − 2 a log t −  + c
b  t
Put x = cos 2 θ ⇒ dx = − 2sin θ cos θ dθ
1 − cos θ 1  a2 
∴ I=∫ ⋅ (− 2 sin θ cos θ ) dθ =  a + bx − 2 a log (a + bx) − + c
1 + cos θ
3
b  a + bx 
θ θ 27. Let I1 = ∫ sin x sin 2 x sin 3x dx
= − ∫ 2 tan ⋅ sin θ cos θ dθ = − 2 ∫ 2 sin 2 ⋅ cos θ dθ
2 2
1
4∫
= − 2∫ (1 − cos θ ) cos θ dθ = − 2 ∫ ( cos θ − cos 2 θ ) dθ = (sin 4x + sin 2 x − sin 6x) dx

= − 2∫ cos θ dθ + ∫ (1 + cos 2 θ ) dθ =−
cos 4x cos 2 x cos 6 x
− +
16 8 24
sin 2θ
= − 2 sin θ + θ + +c I 2 = ∫ sec2 x ⋅ cos 2 2 x dx
2
= − 2 1 − x + cos −1 x + x (1 − x) + c = ∫ sec2 x (2 cos 2 x − 1)2dx
dx dx
Let I = ∫ 2 4
x (x + 1)3/ 4 ∫ 2 3 
24. = 3/ 4 = ∫ (4 cos 2 x + sec2 x − 4) dx
1
x ⋅ x 1 + 4 
 x  = ∫ (2 cos 2 x + sec2 x − 2) dx
4 = sin 2 x + tan x − 2 x
Put 1 + x− 4 = t ⇒ − dx = dt
x5
1/ 4
and I3 = ∫ sin 4 x cos 4 x dx
1 dt 1 t1/ 4  1
∴ I=− ∫ =− ⋅ + c = − 1 + 4 
 x 
+c 1
128 ∫
4 t3/ 4
4 1 /4 = (3 − 4 cos 4x + cos 8x) dx
(x4 + 1)1/ 4
=− +c =
3x sin 4x sin 8x
− +
x 128 128 1024
x ∴ I = I1 + I 2 + I3
25. (i) Let I=∫ 1 + sin dx
2 cos 4x cos 2 x cos 6 x
=− − + + sin 2 x + tan x − 2 x
x x x x 16 8 24
=∫ cos 2 + sin 2 + 2 sin cos dx
4 4 4 4 3x sin 4x sin 8 x
+ − +
128 128 1024
rankershalt.com

Indefinite Integration 273

I=∫
x dx 1
= ∫
2x  1 ln (1 + 6 x )
28. Let dx 3. Let I = ∫ 3 +  dx
1+ x 4
2 1 + (x2)2  x+ 4
x 3
x+ x 
Put x2 = u ⇒ 2 x dx = du ∴ I = I1 + I 2
1 du 1 1  
∴ I= ∫ = tan −1 (u ) + c = tan −1 (x2) + c where, I1 = ∫  3
1
 dx,
2 1 + u2 2 2  x+ 4
x
sin x
29. Let I = ∫ dx ln (1 + 6 x )
sin x − cos x I2 = ∫ dx
3
x+ x
Again, let sin x = A (cos x + sin x) + B(sin x − cos x),
 1 
then A + B = 1 and A − B = 0 Now, I1 = ∫  3  dx
 x+ 4
x
1 1
⇒ A= , B= Put x = t12 ⇒ dx = 12 t11dt
2 2
1 1 t11
(cos x + sin x) + (sin x − cos x) ∴ I1 = 12 ∫ dt
∴ I=∫ 2 2 dx t + t3
4

(sin x − cos x)
t 8 dt
1 cos x + sin x 1 = 12 ∫
= ∫ dx + ∫ 1 dx + c t+1
2 sin x − cos x 2
1 1 = 12∫ (t7 − t 6 + t5 − t 4 + t3 − t 2 + t − 1) dt
= log (sin x − cos x) + x + c
2 2 dt
+ 12∫
t+1
Topic 2 Some Special Integrals  t8 t7 t 6 t5 t 4 t3 t 2 
= 12  − + − + − + − t
2 4 8 7 6 5 4 3 2 
dx
1. Let I = ∫ sec3 x cos ec3 x dx = ∫ 2 4 + 12 ln (t + 1)
cos3 x sin3 x  ln (1 + 6 x ) 
dx and I 2 = ∫  3  dx
∫ 4
4 2
 x+ x 
 sin x 3
  cos3 x cos3 x Put x = u6 ⇒ dx = 6 u5 du
 cos x
ln (1 + u ) 5 ln (1 + u )
∴ I2 = ∫ 6u du = ∫ 2 . 6 u5 du
[dividing and multiplying by cos 4/3
x in denominator] u +u
2 3
u (1 + u )
dx sec2 x dx u3
=∫ 4
=∫ 4 =6 ∫ ln (1 + u ) du
(u + 1)
tan3 x cos 2 x (tan x)3
 u3 − 1 + 1
Now, put tan x = t ⇒ sec2 x dx = dt =6 ∫   ln(1 + u ) du
−4  u+1 
+1
dt t3  1 
∴I=∫ = +C = 6 ∫  u2 − u + 1 −
t 4/3 −4  ln (1 + u ) du
+1  u + 1
3
ln (1 + u )
1 −3 −
1 = 6 ∫ (u 2 − u + 1) ln (1 + u ) du − 6 ∫ du
= −3 +C = + C = −3 tan 3 x+C II I (u + 1)
1 1
t3 (tan x)3  u3 u 2 
=6  − + u ln (1 + u )
2. We have, I n = ∫ tan n x dx  3 2 
2 u3 − 3u 2 + 6 u 1
∴ I n + I n + 2 = ∫ tan n x dx + ∫ tan
n+ 2
x dx −∫ du − 6 [ln (1 + u )]2
u+1 2
= ∫ tan n x(1 + tan 2 x) dx = (2 u3 − 3 u 2 + 6 u ) ln (1 + u )
tan n + 1 x  11 u 
= ∫ tan n x sec2 x dx = +C −∫ 2 u 2 − 5 u +  du − 3 [ln (1 + u )]
2
n+1  u + 1
tan5 x = (2 u3 − 3 u 2 + 6 u ) ln (1 + u )
Put n = 4, we get I 4 + I 6 = +C
5  2 u3 5 2 
−  − u + 11u − 11 ln (u + 1) − 3 [ln (1 + u )]2
1  3 2 
∴ a = and b = 0
5
rankershalt.com

274 Indefinite Integration

3 23 12 7/12 12 5/12 cos θ


∴ I= x/ − x + 2x1/ 2 − x + 3x1/3 − 4x1/ 4 = − ∫ secθ dθ + 2 ∫ dθ
2 7 5 1 + cos 2 θ
− 6 x1/ 6 − 12 x1/12 + 12 ln (x1/12 + 1) cos θ
= − log|sec θ + tan θ | + 2 ∫ dθ
+ (2 x1/ 2 − 3x1/3 + 6 x1/ 6 ) ln (1 + x1/ 6 ) 2 − sin 2 θ
2 5  dt
−  x1/ 2 − x1/3 11 x1/ 6 − 11 ln (1 + x1/ 6 )
3 2 
= − log|sec θ + tan θ | + ∫ 2 − t 2 , where sin θ = t
−3 [ln (1 + x1/ 6 )]2 + c 1  2 + sin θ 
3 12 7/12 4 1/ 2 12 5/12 = − log|sec θ + tan θ |+ 2 ⋅ log  + c
= x2/3 − x + x − x 2 2  2 − sin θ 
2 7 3 5
1 = − log|cot x + cot2 x − 1 |
+ x1/3 − 4x1/ 4 − 7x1/ 6 − 12 x1/12
2  2 + 1 − tan 2 x 
1
+ (2 x 1/ 2
− 3x 1/3
+ 6x1/ 6
+ 11) ln (1 + x )1/ 6 + log  + c
 2 − 1 − tan x 
2 2
+ 12 ln (1 + x1/12) − 3 [ln (1 + x1/ 6 )]2 + c
2 sin x − sin 2 x
4. Let I=∫ ( tan x + cot x ) dx = ∫
tan x + 1
dx 6. Given, f (x) = ∫   dx
tan x  3
x 

Put tan x = t ⇒ sec x dx = 2t dt


2 2 On differentiating w.r.t. x, we get
2t 2 sin x − sin 2 x 2 sin x  1 − cos x
⇒ dx = dt f ′ (x) = =  
1 + t4 x3 x  x2 

t2 + 1 t2 + 1  x
2t 2 sin 2 
∴ I=∫ ⋅ dt = 2 ∫ t 4 + 1 dt  sin x  2
t2 t4 + 1 lim f ′ (x) = lim 2  
x→ 0 x→ 0  x   x2 
1 1  
1+ 1+
t2 t2
=2 ∫ dt = 2 ∫ 2
dt  x 
1  1  sin 2 
t2 + −2 + 2 t −  + ( 2)
2
t2  t = 4 ⋅ 1 ⋅ lim  2 =1
x→ 0 
 x 
2

Put t−
1 1
= u ⇒ 1 + dt = du 4 ×   
 2 
t t2
du
∴ I =2 ∫
u 2 + ( 2 )2 Topic 3 Integration by Parts
2
2  u 1. Let given integral, I = ∫ x5 e− x dx
⇒ I= tan −1   + c
2  2 Put x2 = t ⇒ 2xdx = dt
 tan x − cot x  1
= 2 tan −1   +c So, I = ∫ t 2e− t dt
 2  2
1
= [(− t 2e− t ) + ∫ e− t (2t ) dt ] [Integration by parts]
cos 2 x cos 2 x − sin 2 x 2
5. Let I = ∫ dx = ∫ dx
sin x sin 2 x 1
= [− t 2e− t + 2t (− e− t ) + ∫ 2e− t dt ]
=∫ cot2 x − 1 dx 2
1
= [− t 2e− t − 2te− t − 2e− t ] + C
Put cot x = sec θ ⇒ − cosec2x dx = sec θ tan θ dθ 2
sec θ ⋅ tan θ e− t 2
∴ I=∫ sec2 θ − 1 ⋅ dθ =− (t + 2t + 2) + C
− (1 + sec2 θ ) 2
2
sec θ ⋅ tan 2 θ e− x
=−∫ dθ =− (x4 + 2x2 + 2) + C [Q t = x2] …(i)
1 + sec2 θ 2
sin 2 θ Q It is given that,
=−∫ dθ 2 2
cos θ + cos3 θ I = ∫ x5 e− x dx = g (x) ⋅ e− x + C

1 − cos 2 θ By Eq. (i), comparing both sides, we get


=−∫ dθ
cos θ (1 + cos 2 θ ) 1
g (x) = − (x4 + 2x2 + 2)
2
(1 + cos 2 θ ) − 2 cos 2 θ
=−∫ dθ 1 5
cos θ (1 + cos 2 θ ) So, g(− 1) = − (1 + 2 + 2) = −
2 2
rankershalt.com

Indefinite Integration 275

2. Given, ∫ esec x [(sec x tan x) f (x) + (sec x tan x + sec2 x)]dx


1 1 1
x+ x+ x+
=∫e x dx + xe x −∫e x dx
= e sec x ⋅ f (x) + C
  1 x+ x
1
x+ 
1
On differentiating both sides w.r.t. x, we get Q ∫  1 −  e dx = e x
esec x [(sec x tan x) f (x) + (sec x tan x + sec2 x)]   x2  
= esec x f ′ (x) + esec x (sec x tan x) f (x) x+
1
x+
1
x+
1
x+
1

⇒ esec x (sec x tan x + sec2 x) = esec x f ′ (x) =∫e x dx + xe x − ∫ ex x dx = xe x +c


⇒ f ′ (x) = sec x tan x + sec2 x
6. Given, ∫ f ( x ) dx = ψ( x )
So, f (x) = ∫ f ′ (x)dx = ∫ (sec x tan x + sec2 x)dx
Let I = ∫ x5 f (x3 ) dx
= sec x + tan x + C
So, possible value of f (x) from options, is Put x3 = t
1 dt
f (x) = sec x + tan x + . ⇒ x2dx = …(i)
2 3
3. Let I = ∫ cos(log e x)dx 1
3∫
∴ I= t f (t ) dt
1
= x cos(log e x) − ∫ x(− sin(log e x)) ⋅ dx 1 d  
x = t ⋅ ∫ f (t ) dt − ∫  (t ) ∫ f (t ) dt  dt 
[using integration by parts]
3  dt  
= x cos(log e x) + ∫ sin(log e x) dx [integration by parts]
1
1 = [t ψ (t ) − ∫ ψ (t ) dt ]
= x cos(log e x ) + x sin(log e x ) − ∫ x(cos(log e x)) x dx 3
1 3
[again, using integration by parts] = [x ψ (x3 ) – 3 ∫ x2ψ (x3 ) dx] + c [from Eq. (i)]
3
⇒ I = x cos(log e x) + x sin((log e x) − I
1

x
I = [cos(log e x) + sin(log e x)] + C. = x3 ψ (x3 ) − ∫ x2ψ (x3 ) dx + c
2 3
3 1 −4 x 3  2x + 2 
4. Given, ∫ x5 e−4x dx = e f (x) + C 7. Let I = ∫ sin −1   dx
48  4x2 + 8 x + 13 
 
In LHS, put x3 = t
 2x + 2 
⇒ 3x2dx = dt −1   dx
3 1
= ∫ sin  (2x + 2)2 + 9 
So, ∫ x5 e−4x dx = ∫ t e−4t dt  
3
Put 2x + 2 = 3 tan θ ⇒ 2 dx = 3 sec2θ dθ
1  e−4t e−4t 
= t −∫  3 tan θ  3
dt 
3  −4 −4  ∴ I = ∫ sin −1  ⋅ sec2 θ dθ
 9 tan 2 θ + 9  2
[using integration by parts]  
−4 t −4 t
1  te e   3 tan θ  3 2
=  + +C = ∫ sin −1   ⋅ sec θ dθ
3  −4 −16   3 secθ  2
1 −4 t  sin θ  3 2
=− e [4t + 1] + C = ∫ sin −1   ⋅ sec θ dθ
48  cos θ ⋅ sec θ  2
3
e−4x 3
sin −1 (sin θ ) ⋅ sec2θ dθ
2∫
=− [4x3 + 1] + C [Q t = x3 ] =
48
3 3
∴ f (x) = −1 − 4x3 (comparing with given equation) = ∫ θ ⋅ sec2θ dθ = [θ ⋅ tan θ − ∫ 1 ⋅ tan θdθ]
2 2
1
 1 x + 3
5. ∫ 1 + x −  e x dx
 x
= [θ tan θ − log sec θ ] + c
2
3  2 x + 2  2 x + 2
1 1
x+  1 x+
=∫e = tan −1   ⋅
x dx + ∫ x 1 − x2 e x dx
2  3   3 

1 1 1 2 
x+ x+ d x+  2 x + 2 
=∫e x dx + xe x −∫ (x)e x dx − log 1 +   + c1
dx  3  

rankershalt.com

276 Indefinite Integration

 θ
 2 x + 2 
2 1
 2x + 2 3 =− cos 2θ + sin 2θ
= (x + 1) tan −1   − log 1 +    + c1 2 4
 3  4   3  
1 1
 2 x + 2 3 = − θ (1 − 2 sin 2 θ ) + sin θ 1 − sin 2 θ
= (x + 1) tan −1   − log (4x + 8 x + 13) + c
2
2 2
 3  4
1 −1 1
 3  = − sin x (1 − 2 x) + x 1−x …(ii)
let 2 log 3 + c1 = c
2 2
From Eqs. (i) and (ii),
 cos θ + sin θ  4  1 1 
8. I = ∫ cos 2 θ ln   dθ [given] I = − (1 − 2 x) sin −1 x + x − x2  − x + c
 cos θ − sin θ  π  2 2 
We integrate it by taking parts 2 −1
= [ x − x − (1 − 2 x) sin
2
x] − x + c
 cos θ + sin θ  π
ln   as first function
 cos θ − sin θ  (x − 1) ex
10. Let I = ∫ dx
sin 2 θ  cos θ + sin θ  (x + 1)3
= ln  
2  cos θ − sin θ  x + 1 − 2 x  1 2  x
I=∫ 3 
e dx = ∫  − 3
e dx
 (x + 1)   (x + 1) (x + 1) 
2
1 d   cos θ + sin θ  
2 ∫ dθ   cos θ − sin θ  
− ln    sin 2 θdθ …(i) 1 1
= ∫ ex ⋅ dx − 2 ∫ ex ⋅ dx
(x + 1) 2
(x + 1)3
d   cos θ + sin θ  
But ln   Applying integration by parts,
dθ   cos θ − sin θ  
 1 −2 
d = ⋅ ex − ∫ ex ⋅ dx
= [ln (cos θ + sin θ ) − ln (cos θ − sin θ )]  (x + 1)
2
(x + 1)3


1 (− sin θ − cos θ ) 1 ex
= . (− sin θ + cos θ ) − − 2 ∫ ex ⋅ dx = +c
(cos θ + sin θ ) cos θ − sin θ (x + 1)3
(x + 1)2
(cos θ − sin θ ) (cos θ − sin θ ) − (cos θ + sin θ )
(− sin θ − cos θ ) 11. Let I = ∫ (elog x + sin x) cos x dx
=
(cos θ + sin θ ) (cos θ − sin θ ) = ∫ (x + sin x) cos x dx
cos 2 θ − cos θ sin θ − sin θ cos θ + sin 2 θ + cos θ sin θ 1
+ cos 2 θ + sin 2 θ + cos θ ⋅ sin θ = ∫ x cos x dx + ∫ (sin 2x) dx
= 2
cos 2 θ − sin 2 θ cos 2 x
= (x ⋅ sin x − ∫ 1 ⋅ sin x dx) − +c
2 (cos 2 θ + sin 2 θ ) 2 4
= =
cos 2 θ cos 2θ = x sin x + cos x −
cos 2 x
+c
4
Therefore, on putting this value in Eq.(i), we get
1  cos θ + sin θ  1 2
 cos θ − sin θ  2 ∫
I= sin 2θ ln   − sin 2 θ dθ Topic 4 Integration, Irrational Function
2 cos 2 θ
and Partial Fraction
1  cos θ + sin θ  1
= sin 2 θ ln   + ln (cos 2 θ ) + c 2x12 + 5x9 2x12 + 5x9
2  cos θ − sin θ  2 1. Let I = ∫ dx = ∫ 15 dx
(x + x + 1)
5 3 3
x (1 + x− 2 + x− 5 ) 3
sin −1 x − cos −1 x
9. Let I = ∫ dx 2x− 3 + 5x− 6
sin −1 x + cos −1 x =∫ dx
(1 + x− 2 + x− 5 ) 3
π 
sin −1 x −  − sin −1 x Now, put 1 + x− 2 + x− 5 = t
2 
=∫ dx
π ⇒ (− 2x− 3 − 5x− 6 ) dx = dt
2 ⇒ (2x− 3 + 5x− 6 ) dx = − dt
2  π 4 dt
= ∫ 2 sin −1 x −  dx = ∫ sin
−1
x dx − x + c …(i) ∴ I = − ∫ 3 = − ∫ t − 3 dt
π  2 π t
Now, ∫ sin −1 x dx t− 3 + 1 1
=− +C= 2+C
−3 + 1 2t
Put x = sin 2 θ ⇒ dx = sin 2θ
θ cos 2 θ 1 x10
= ∫ θ ⋅ sin 2 θ dθ = − = +C
2
+ ∫ 2
cos 2θ dθ
2 (x + x3 + 1) 2
5
rankershalt.com

Indefinite Integration 277

cos3 x + cos5 x On putting x = 0, we get


2. Let I=∫ dx
sin 2 x + sin 4 x 0=B+C
(cos x + cos x) ⋅ cos x dx
2 4
⇒ B = − C = 1 /2
=∫
(sin 2 x + sin 4 x) x3 + 3x + 2 x+1 1 2
= − + 2
Put sin x = t ⇒ cos x dx = dt (x + 1) (x + 1) 2 (x + 1)
2 2 2
2 (x + 1 ) (x + 1)2
[(1 − t 2) + (1 − t 2)2] x3 + 3x + 2
∴ I=∫ dt ∴ I=∫ dx
t2 + t4 (x + 1)2 (x + 1)
2

1 − t 2 + 1 − 2t 2 + t 4 1 dx 1 x+1 dx
⇒ I=∫ dt =−
2 ∫ x + 1 + 2 ∫ x2 + 1 dx + 2 ∫ (x + 1)2
2
t2 + t4
1 1
2 − 3t 2 + t 4 ⇒I = − log|x + 1| + log|x2 + 1|
⇒ I=∫ dt …(i) 2 4
t 2 (t 2 + 1)
1
Using partial fraction for + tan −1 x + 2 I1 …(i)
2
y − 3y + 2
2
A B dx
=1 + + [where, y = t 2] where, I1 = ∫
y ( y + 1) y y+1 (x2 + 1)2
⇒ A = 2, B = − 6 Put x = tan θ
y2 − 3 y + 2 2 6 ⇒ dx = sec2θ dθ
∴ =1 + −
y ( y + 1) y y+1 sec θdθ2
1
∴ I1 = ∫
(tan 2θ + 1)2 ∫
= cos 2 θ dθ = ∫ (1 + cos 2 θ )dθ
 2 6  2
Now, Eq. (i) reduces to, I = ∫ 1 + 2 −  dt
 t 1 + t 2 1 1 
= θ + sin 2θ 
2 2  2 
=t− − 6 tan −1 (t ) + c
t 1 1 tan θ
= θ+ ⋅
2 2 2 (1 + tan 2 θ)
= sin x − − 6 tan −1 (sin x) + c
sin x 1 1 x
= tan −1 x + ⋅
x + 3x + 2
3
x + 2x + x + 2
3
2 2 (1 + x2)
3. =
(x2 + 1)2 (x + 1) (x2 + 1)2(x + 1) From Eq. (i),
x(x2 + 1) + 2(x + 1) 1 1 3
I = − log|x + 1| + log|x2 + 1|+ tan −1 x + 2
x
+c
=
(x2 + 1)2(x + 1) 2 4 2 x +1

x 2 (x + 1) ex (x + 1)
= + 4. Let I=∫ dx = ∫ x ex (1 + xex )2 dx
(x2 + 1)(x + 1) (x2 + 1)2 x (1 + xex )2
x Ax + B C Put 1 + xex = t ⇒ (ex + x ex ) dx = dt
Again, = +
(x + 1) (x + 1) (x2 + 1) (x + 1)
2
dt  1 1 1
∴ I=∫ =∫  − − 2  dt
⇒ x = ( Ax + B) (x + 1) + C (x + 1) 2 (t − 1)t 2
 t − 1 t t 

On putting x = − 1, we get 1
= log|t − 1| − log|t| + + c
t
−1 = 2 C ⇒ C = − 1 / 2
t − 1
= log + 1 + c
On equating coefficients of x2, we get  t  t
0= A+C
 xex  1
⇒ A = − C = 1 /2 = log  + +c
1 + xex
 1 + xex

Download Chapter Test


http://tinyurl.com/yyjx6q3z or
rankershalt.com

12
Definite Integration
Topic 1 Properties of Definite Integral
Objective Questions I (Only one correct option) 2 − x cos x
8. If f (x) = and g (x) = log e x, (x > 0) then the
2 + x cos x
1. A value of α such that π/ 4
α+1 value of the integral ∫ g ( f (x))dx is
−π/ 4
dx  9
∫ (x + α ) (x + α + 1)
= log e   is
 8
(2019 Main, 8 April I)
α (2019 Main, 12 April II) (a) log e 3 (b) log e e
1 1 (c) log e 2 (d) log e 1
(a) − 2 (b) (c) − (d) 2
2 2 9. Let f and g be continuous functions on
π /2 cot x
2. If ∫ dx = m(π + n ), then m ⋅ n is equal to [0, a] such that f (x) = f (a − x) and g (x) + g (a − x) = 4,
0 cot x + cosec x a
(2019 Main, 12 April I) then ∫ f (x) g (x) dx is equal to (2019 Main, 12 Jan I)
1 1 0
(a) − (b) 1 (c) (d) −1 a a
2 2 (a) 4∫ f (x) dx (b) ∫ f (x) dx
0 0
π /3
3. The integral ∫ sec 2/ 3 x cosec4/3 x dx is equal to
a
(c) 2∫ f (x) dx
a
(d) − 3∫ f (x) dx
π /6 0 0
(2019 Main, 10 April II)
e  e 
2x x
10. The integral ∫  
(a) 35 / 6 − 32/ 3 (b) 37/ 6 − 35 / 6 x
−    log e x dx is
(c) 35 /3 − 31/3 (d) 34/3 − 31/3 1  e  x
 

equal to (2019 Main, 12 Jan II)
4. The value of ∫ [sin 2x (1 + cos 3x)] dx, where [t ] denotes (a)
3
−e− 2
1 1 1
(b) − + − 2
1
0
2 2e 2 e 2e
the greatest integer function, is (2019 Main, 10 April I) 1 1 3 1 1
(a) − π (b) 2π (c) − 2π (d) π (c) −e− 2 (d) − −
2 e 2 e 2e2
1
π /4 dx
5. The value of the integral ∫ x cot−1 (1 − x2 + x4 )dx is 11. The integral ∫ equals
0
π /6 sin 2x(tan5 x + cot5 x)
(2019 Main, 9 April II) (2019 Main, 11 Jan II)
π 1 π 1 1 π 1 
(a)  − tan − 1  tan − 1 
(a) − log e 2 (b) − log e 2 1 1 
 (b) 
4 2 2 2 5 4  3 3 20  9 3
π π
(c) − log e 2 (d) − log e 2 1π −1 1  π
4 2 (c)  − tan   (d)

10 4  9 3 40
π/ 2 sin3 x
6. The value of ∫ dx is sin 2 x
0 sin x + cos x (2019 Main, 9 April I) 12. The value of the integral ∫
2
dx
−2  x  1
π −1 π−2 π −1 π−2 +
(a) (b) (c) (d)  π  2
2 8 4 4
x (where, [x] denotes the greatest integer less than or equal
7. Let f (x) = ∫ g (t )dt, where g is a non-zero even function. to x) is (2019 Main, 11 Jan I)
0 x (a) 4 − sin 4 (b) 4
If f (x + 5) = g (x), then ∫ f (t )dt equals (c) sin 4 (d) 0
0 (2019 Main, 8 April II) b
5 x+5 13. Let I = ∫ (x4 − 2x2) dx. If I is minimum, then the ordered
(a) 5 ∫ g (t )dt (b) ∫ g (t )dt a
x+5 5 pair (a , b) is (2019 Main, 10 Jan I)
x+5 5
(a) (− 2 , 0) (b) (0, 2 )
(c) 2 ∫ g (t )dt (d) ∫ g (t )dt (c) ( 2 , − 2 ) (d) (− 2 , 2 )
5 x+5
rankershalt.com

Definite Integration 279

π /3 tan θ 1 1−x
14. If ∫
1
dθ = 1 − , (k > 0), then the value of k 25. The value of the integral ∫ dx is (2004, 1M)
0 2k sec θ 2 0 1+ x
is (2019 Main, 9 Jan II) π π
(a) +1 (b) −1 (c) −1 (d) 1
1 2 2
(a) 1 (b)
2 
1/ 2  1 + x 
(c) 2 (d) 4 26. The integral ∫  [x] + log    dx equals (2002, 1M)
π
−1/ 2   1 − x 
15. The value of ∫ |cos x|3 dx is
(d) log  
(2019 Main, 9 Jan I) 1 1
0 (a) − (b) 0 (c) 1
2 4 4 2  2
(a) (b) − (c) 0 (d)
3 3 3 π cos 2 x
π /2 sin x 2 27. The value of ∫ dx, a > 0,is (2001, 1M)
−π 1 + a x
16. The value of ∫ dx is
−π / 2 1 + 2x
(2018 Main) (a) π (b) aπ
π π π π
(a) (b) (c) 4π (d) (c) (d) 2π
8 2 4 2
3 π/ 4 dx ecos x sin x , for | x|≤ 2
17. ∫π / 4 1 + cos x
is equal to 28. If f (x) = 
(2017 Main)  2 , otherwise
(a) − 2 (b) 2 (c) 4 (d) − 1 3
then ∫ f (x) dx is equal to (2000, 2M)
x2 cos x π /2 −2
18. The value of ∫ dxis equal to
−π / 2 1 + ex (a) 0 (b) 1 (c) 2 (d) 3
(2016 Adv.)
π2 π2 log x e2
(a) −2 (b) + 2 29. The value of the integral ∫ −1  e  dx is (2000, 2M)
4 4 e  x 
(c) π 2 − e− π / 2 (d) π 2 + eπ / 2 (a) 3 / 2 (b) 5 / 2 (c) 3 (d) 5
4 log x2
19. The integral ∫ dx is equal to 30. If for a real number y, [ y] is the greatest integer less
2 log x2 + log(36 − 12x + x2) (2015, Main) than or equal to y, then the value of the integral
3π/2
(a) 2 (b) 4
π /2
(c) 1 (d) 6
∫ π / 2 [2 sin x] dx is (1999, 2M)
20. The integral ∫ (2 cosec x)17dx is equal to (a) − π (b) 0
π /4 (2014 Adv.)
π π
log( 1 + 2) (c) − (d)
(a) ∫0 2(eu + e−u )16 du 2 2
log( 1 + 2) 3 π /4 dx
(b) ∫0 (eu + e−u )17 du 31. ∫ π /4 1 + cos x
is equal to
(1999, 2M)
log( 1 + 2) −u 17
∫0 (e − e ) du 1 1
u
(c) (a) 2 (b) −2 (c) (d) −
log( 1 + 2)
2 2
(d) ∫0 2(eu − e−u )16 du 32. Let f (x) = x − [x] , for every real number x, where [x] is
π 1

21. The integral ∫ 1 + 4 sin 2 − 4 sin


x x
dx is equal to
the integral part of x. Then, ∫ −1 f (x) dx is (1998, 2M)
2 2 (2014 Main)
0 (a) 1 (b) 2
2π 1
(a) π − 4 (b) − 4− 4 3 (c) 0 (d) −
3 2
(c) 4 3 − 4 (d) 4 3 − 4 − π /3 x
33. If g (x) = ∫ cos 4 t dt , then g (x + π ) equals (1997, 2M)
π /2  2 π − x 0
22. The value of the integral ∫  x + log  cos x dx
− π /2  π + x (a) g (x) + g ( π ) (b) g (x) − g ( π )
g (x )
is (2012) (c) g (x) g ( π ) (d)
π2 π2 π2 g( π)
(a) 0 (b) −4 (c) + 4 (d)
2 2 2 34. Let f be a positive function.
log 3 x sin x2 If I1 = ∫
k
x f [x (1 − x)] dx and
23. The value of ∫ log 2 sin x + sin (log 6 − x2 )
2
dx is (2011)
1−k
k
(a)
1
log
3
(b)
1
log
3
(c) log
3
(d)
1
log
3 I2 = ∫ f [x (1 − x)] dx, where 2k − 1 > 0.
1−k
4 2 2 2 2 6 2
I1
24. The value of Then, is (1997C, 2M)
0
I2
∫ −2 [x + 3x2 + 3x + 3 + (x + 1) cos (x + 1)] dx is (2005, 1M)
3
(a) 2 (b) k
(c) 1/2 (d) 1
(a) 0 (b) 3 (c) 4 (d) 1
rankershalt.com

280 Definite Integration


2π 3 3 3
35. The value of ∫0 [ 2 sin x ] dx, where [.] represents the 43. If ∫ x2 F ′ (x) dx = − 12 and
1 ∫1 x F ′ ′ (x) dx = 40, then the
greatest integral functions, is (1995, 2M) correct expression(s) is/are
5π 5π (a) 9f ′ (3) + f ′ (1) − 32 = 0
3
(b) ∫ f (x) dx = 12
(a) − (b) − π (c) (d) −2π 1
3 3
3
π x (c) 9f ′ (3) – f ′ (1) + 32 = 0 (d) ∫ f (x) dx = − 12
36. If f (x) = A sin   + B, 1
 2 
f ′   = 2 and ∫ f (x) dx =
1 1 2A
, then constants
Passage II
 2 0 π For every function f (x) which is twice differentiable, these
A and B are (1995, 2M) will be good approximation of
π π 2 3 4 4 b  b − a
(a) and
2 2
(b) and
π π
(c) 0 and −
π
(d) and 0
π ∫ a f (x) dx =  2 
 { f (a ) + f (b)},
π /2 dx
37. The value of ∫ is (1993, 1M) for more acurate results for c ∈ (a , b),
0 1 + tan3 x c−a b−c
F (c) = [ f (a ) − f (c)] + [ f (b) − f (c)]
(a) 0 (b) 1 2 2
(c) π / 2 (d) π / 4
a+b
38. Let f : R → R and g : R → R be continuous functions. When c =
2
Then, the value of the integral b b−a
π/ 2 ∫ a f (x) dx = 4 { f (a ) + f (b) + 2 f (c)} dx
∫ −π / 2 [ f (x) + f (− x)] [ g (x) − g (− x)] dx is (1990, 2M) (2006, 6M)
t (t − a )
(a) π (b) 1 (c) −1 (d) 0 ∫ a f (x) dx − 2 { f (t ) + f (a )}
44. If lim = 0,
39. For any integer n, the integral t→ a (t − a )3
π cos 2 x then degree of polynomial function f (x) atmost is
∫0 e cos3 (2n + 1) x dx has the value (1985, 2M)
(a) 0 (b) 1
(a) π (b) 1 (c) 3 (d) 2
(c) 0 (d) None of these 45. If f ′ ′ (x) < 0, ∀x ∈ (a , b), and (c, f (c)) is point of maxima,
π /2 cot x where c ∈ (a , b), then f ′ (c) is
40. The value of the integral ∫ dx is
0 cot x + tan x f (b) − f (a ) f (b) − f (a ) 
(a) (b) 3 
(a) π / 4 (b) π / 2 (1983, 1M) b− a  b − a 
(c) π (d) None of these  f (b) − f (a ) 
(c) 2 (d) 0
 b − a 
Assertion and Reason π /2
46. Good approximation of ∫ sin x dx, is
41. Statement I The value of the integral (2013 Main) 0

π /3 dx (a) π / 4 (b) π ( 2 + 1) / 4
∫π / 6 1 + tan x
is equal to π /6 .
(c) π ( 2 + 1) / 8 (d)
π
8
b b
Statement II ∫a f (x) dx = ∫a f (a + b − x) dx Objective Questions II
(a) Statement I is correct; Statement II is correct; (One or more than one correct option)
Statement II is a correct explanation for Statement I 47. Let f : R → (0, 1) be a continuous function. Then, which
(b) Statement I is correct; Statement II is correct;
of the following function(s) has (have) the value zero at
Statement II is not a correct explanation for
some point in the interval (0, 1) ? (2017 Adv.)
Statement I π
(c) Statement I is correct; Statement II is false x 2

(d) Statement I is incorrect; Statement II is correct (a) e − ∫ f (t ) sin t dt


x
(b) f (x) + ∫ f (t ) sin t dt
0 0
π
−x
Passage Based Questions 2
(c) x − ∫ f (t ) cos t dt (d) x9 − f (x)
Passage I 0
Let F : R → R be a thrice differentiable function. Suppose that k+1 k+1
48. If I = ∑ k = 1
98
F (1) = 0, F (3) = − 4 and F ′ (x) < 0 for all x ∈ (1, 3). Let ∫k x(x + 1)
dx , then
(2017 Adv.)
f (x) = xF (x) for all x ∈ R. (2015 Adv.)
(a) I > log e 99 (b) I < log e 99
42. The correct statement(s) is/are
(a) f ′ (1) < 0 (b) f (2) < 0 49 49
(c) I < (d) I >
(c) f ′ (x) =/ 0 for any x ∈ (1, 3) (d) f ′ (x) = 0 for some x ∈ (1, 3) 50 50
rankershalt.com

Definite Integration 281

49. Let f (x) = 7 tan 8 x + 7 tan 6 x – 3 tan 4 x – 3 tan 2 x for all 37 π π sin (π log x)
57. The value of ∫ dx is …… .
 π π 1 x (1997, 2M)
x ∈  – ,  . Then, the correct expression(s) is/are
 2 2 2π x sin 2n x
π/4 1 π/ 4
58. For n > 0, ∫ dx = …… .
(a) ∫ x f (x) dx = (b) ∫ f (x) dx = 0 0 sin x + cos 2n x
2n
(1996, 2M)
0 12 0 (2015 Adv.)
59. If for non-zero x, af (x) + bf   = − 5, where a ≠ b,
π/4 1 π/ 4 1 1
(c) ∫ x f (x) dx = (d) ∫ f (x) dx = 1  x x
0 6 0
2
192x3  1 then ∫ f (x) dx = …… .
50. Let f ′ (x) = for all x ∈ R with f   = 0. If 1 (1996, 2M)
2 + sin πx
4  2
3 x
1 60. The value of ∫ dx is …… .
m≤∫ f (x) dx ≤ M , then the possible values of m and M 2 5−x+ x
1/ 2 (1994, 2M)
are (2015 Adv.) 3 π /4 x
61. The value of ∫ dx …… .
(a) m = 13, M = 24 π /4 1 + sin x (1993, 2M)
1 1
(b) m = , M = 2
4 2 62. The value of ∫ |1 − x | dx is … . 2
−2 (1989, 2M)
(c) m = − 11, M = 0 1.5
∫0
2
(d) m = 1, M = 12 63. The integral [x ] dx, where [.] denotes the greatest
51. The option(s) with the values of a and L that satisfy the function, equals …… . (1988, 2M)

equation
∫0 et (sin 6 at + cos 4 at )dt
= L, is/are
π t Match the Columns
∫0 e (sin at + cos at )dt
6 4
(2015 Adv.) 64. Match the conditions/expressions in Column I with
e4 π − 1 e4 π + 1 statement in Column II.
(a) a = 2, L = π (b) a = 2, L = π
e −1 e +1 Column I Column II
e4 π − 1 e4 π + 1
log  
(c) a = 4, L = π (d) a = 4, L = π 1 dx 1 2
e −1 e +1 A. ∫ −1 1 + x 2 P.
2  3

2 log  
1 1 dx 2
52. The value(s) of ∫
x4 (1 − x)4
dx is (are) (2010) B. ∫0 Q.  3
1− x 2
1 + x2
0 3 dx π
(a)
22
−π (b)
2
(c) 0 (d)
71 3 π

C. ∫2 1− x 2
R.
3
7 105 15 2 2 dx π
π sin nx D. ∫1 S.
53. If I n = ∫ dx , n = 0 , 1 , 2 ,... , then x x2 −1 2
−π (1 + π x ) sin x (2009)
10 65. Match List I with List II and select the correct answer
(a) I n = I n + 2 (b) ∑ I2 m + 1 = 10 π using codes given below the lists. (2014)
m =1
10 List I List II
(c) ∑ I2 m = 0 (d) I n = I n + 1
P. The number of polynomials f( x ) with (i) 8
m =1
non-negative integer coefficients of degree
1
≤ 2, satisfying f( 0) = 0 and ∫ f( x ) dx = 1, is
Numerical Value 0

1/ 2 1+ 3 Q. The number of points in the interval (ii) 2


54. The value of the integral ∫ dx is [− 13, 13 ] at which f( x ) = sin( x 2 ) + cos( x 2 )
0 ((x + 1)2 (1 − x)6 )1/ 4
attains its maximum value, is
....... (2018 Adv.)
R. 2 3x 2 (iii) 4
Fill in the Blanks ∫−2 1 + e x dx equals
55. Let f : [1, ∞ ] → [2, ∞ ] be differentiable function such S.  1/ 2 1 + x   (iv) 0
x  ∫ cos 2 x log   dx 
that f (1) = 2. If 6∫ f (t ) = dt = 3x f (x) − x , ∀ x ≥ 1 then 3  − 1/ 2 1− x  
1 equals
 1/ 2 1 + x  
the value of f (2) is .... (2011)  ∫ cos 2 x log   dx 
sin x  0 1− x  
d e
56. Let F (x) = , x > 0.
dx x Codes
2
4 2 e sin x P Q R S P Q R S
If ∫ dx = F (k) − F (1), then one of the possible (a) (iii) (ii) (iv) (i) (b) (ii) (iii) (iv) (i)
1 x
(c) (iii) (ii) (i) (iv) (d) (ii) (iii) (i) (iv)
values of k is ..… . (1997, 2M)
rankershalt.com

282 Definite Integration

Analytical & Descriptive Questions 80. If f and g are continuous functions on [0, a ] satisfying
1 f (x) = f (a − x) and g (x) + g (a − x) = 2, then show that
(5050) ∫ (1 − x50 )100 dx a a
66. The value of 1
0
is (2006, 6M) ∫ 0 f (x) g(x) dx = ∫ 0 f (x) dx. (1989, 4M)

∫0 (1 − x ) 50 101
dx
81. Prove that the value of the integral,
2a
67. Evaluate
π |cos x |  1  1 
∫0 [ f (x) /{ f (x) + f (2a − x)}] dx is equal to a. (1988, 4M)

∫ 0e 2 sin  2 cos x + 3 cos  2 cos x  sin x dx.


 
1
(2005, 2M) 82. Evaluate ∫ 0 log[ (1 − x) + (1 + x)] dx. (1988, 5M)
π/ 3 π + 4x 3
68. Evaluate ∫ −π / 3  π
dx. (2004, 4M)
83. Evaluate ∫
π x dx
, 0 < α < π. 1
2 − cos |x|+  01 + cos α sin x (1986, 2 M)
 3 2

69. If f is an even function, then prove that π /2 x sin x cos x


π /2 π /4 84. Evaluate ∫ dx.
∫0 f (cos 2x) cos x dx = 2 ∫
0
f (sin 2x) cos x dx.
(2003, 2M)
0 cos 4 x + sin 4 x 1
(1985, 2 M)
2
π ecos x
70. Evaluate ∫ dx. 1/ 2 x sin −1 x
∫0
(1999, 3M)
0 ecos x + e− cos x 85. Evaluate dx. (1984, 2M)
1 − x2
1 −1  1  1
71. Prove that ∫ 0 tan   dx = 2 ∫ tan −1 x dx. 86. Evaluate ∫
π /4 sin x + cos x
dx.
1 − x + x 
2 0
0 9 + 16 sin 2x (1983, 3M)

Hence or otherwise, evaluate the integral π π π


1 −1 87. (i) Show that ∫ x f (sin x) dx = ∫ 0 f (sin x) dx.(1982, 2M)
∫ tan (1 − x + x ) dx.
2
0
(1998, 8M) 0 2
3/ 2

72. Integrate ∫
π /4
log (1 + tan x) dx.
(ii) Find the value of ∫ −1 | x sin πx| dx. (1982, 3M)
0
(1997C, 2M)
π 2x (1 + sin x) 1
∫ −π ∫ 0 (tx + 1 − x) dx,
n
73. Determine the value of dx. (1995, 5M) 88. Evaluate
1 + cos 2 x
where n is a positive integer and t is a parameter
74. Evaluate the definite integral independent of x. Hence, show that
1/ 3  x4   2x  1 k 1
∫ −1/   cos −1   dx. (1995, 5M) n−k
∫ 0 x (1 − x) dx = nC k (n + 1), for k = 0, 1,... , n(1981,
.
3 1 − x 
4
 1 + x2 4M)
3 2x5 + x4 − 2x3 + 2x2 + 1
75. Evaluate ∫2 (x2 + 1) (x4 − 1)
dx. (1993, 5M)
Integer Answer Type Questions
[x], x ≤ 2
76. A cubic f (x) vanishes at x = − 2 and has relative 89. Let f : R → R be a function defined by f (x) =  ,
minimum / maximum at x = − 1 and x = 1/3. 0 , x > 2
1 where [x] denotes the greatest integer less than or equal
If ∫ f (x) dx = 14 / 3, find the cubic f (x). (1992, 4M)
−1 2 xf (x2)
to x. If I = ∫ dx, then the value of (4I − 1)
π  −1 2 + f (x + 1 )
x sin (2x) sin  cos x
π 2  is
77. Evaluate ∫ dx . (1991, 4M)
(2015 Adv.)
0 2x − π 1 −1  12 + 9x2
90. If α = ∫ (e9x + 3 tan x
)  dx,
78. Show that , 0  1 + x2 
π /2 π /4
∫0 f (sin 2x) sin x dx = 2 ∫0 f (cos 2x) cos x dx.
(1990, 4M)
where tan −1 x takes only principal values, then the
 3π 
79. Prove that for any positive integer k, value of  log e|1 + α | −  is
 4 (2015 Adv.)
sin 2kx
= 2 [cos x + cos 3x + K + cos(2k − 1) x]
sin x 1  d2 
π /2
91. The value of ∫ 4x3  2
(1 − x2)5  dx is
∫0  
0 dx
Hence, prove that sin 2kx ⋅ cot x dx = π /2. (2014 Adv.)
(1990, 4M)
rankershalt.com

Definite Integration 283

Topic 2 Periodicity of Integral Functions


Objective Questions I (Only one correct option) (a) −
3
≤ g(2) <
1
(b) 0 ≤ g(2) < 2
π/ 2
dx 2 2
1. The value of ∫ , where [t ] denotes (c)
3
< g(2) ≤ 5 / 2 (d) 2 < g(2) < 4
− π / 2 [x] + [sin x] + 4
2
the greatest integer less than or equal to t, is
(2019 Main, 10 Jan II)
1 1 Analytical & Descriptive Questions
(a) (7 π − 5) (b) (7 π + 5) nπ + v
12 12 4.. Show that ∫ |sin x|dx = 2n + 1 − cos v, where n is a
3 3 0
(c) (4 π − 3) (d) (4 π − 3)
10 20 positive integer and 0 ≤ v < π . (1994, 4M)

2. Let T > 0 be a fixed real number. Suppose, f is a 5. Given a function f (x) such that it is integrable over
continuous function such that for all every interval on the real line and f (t + x) = f (x), for
T
x ∈ R. f (x + T ) = f (x). If I = ∫ f (x) dx, every x and a real t, then show that the integral
a+ t
∫ f (x) dx is independent of a.
0
3 + 3T
(1984, 4M)
then the value of ∫
a
f (2x) dx is (2002,1M)
3

(a)
3
I (b) I Integer Answer Type Question
2
6. For any real number x, let [x] denotes the largest
(c) 3I (d) 6I
integer less than or equal to x. Let f be a real valued
x 1
3. Let g (x) = ∫ f (t ) dt, where f is such that ≤ f (t ) ≤ 1 for function defined on the interval [− 10, 10] by
0 2
1  x − [x[, if f (x) is odd
t ∈ [0,1] and 0 ≤ f (t ) ≤ for t ∈ [1, 2]. Then, g(2) satisfies f(x) = 
2 1 + [x[− x, if f (x) is even
the inequality (2000, 2M) π 2 10
10 ∫− 10
Then, the value of f (x) cos πx dx is…… (2010)

Topic 3 Estimation, Gamma Function and


Derivative of Definite Integration
Objective Questions I (Only one correct option) (a) f   < and f   >
1 1 1 1
 2 2  3 3
x 1 2  1
1. If ∫ f (t ) dt = x2 + ∫x t f (t )dt , then f′   is
 2 (b) f   > and f   >
0 1 1 1 1
(2019 Main, 10 Jan II)
 2 2  3 3

(c) f   < and f   <


24 18 6 4 1 1 1 1
(a) (b) (c) (d)
25 25 25 5  2 2  3 3
2. Let f : [0, 2] → R be a function which is continuous on (d) f   >
1 1
and f   <
1 1
[0, 2] and is differentiable on (0, 2) with f (0) = 1.  2 2  3 3
x2 1  1
Let F (x) = ∫ f ( t ) dt, for x ∈ [0, 2]. If F ′ (x) = f ′ (x), ∀ 5. If ∫ sin x t 2 f (t ) dt = 1 − sin x, ∀ x ∈ (0, π / 2), then f  
0  3
x ∈ (0, 2), then F (2) equals (2014 Adv) is
(a) e2 − 1 (b) e4 − 1 (c) e − 1 (d) e4 (a) 3 (b) 3 (2005, 1M)
3. The intercepts on X-axis made by tangents to the curve, (c) 1/3 (d) None of these
x
y = ∫ | t | dt , x ∈ R, which are parallel to the line y = 2x, t2 2
∫ 0 x f (x) dx = 5 t
5
0 6. If f (x) is differentiable and , then
are equal to (2013 Main)
4
(a) ± 1 (b) ± 2 (c) ± 3 (d) ± 4 f   equals (2004, 1M)
 25
4. Let f be a non-negative function defined on the interval 2 5
x x (a) (b) −
[0, 1]. If ∫0 1 − ( f ′ (t ))2 dt = ∫ f (t ) dt , 0 ≤ x ≤ 1
0
and 5
5
2
(c) 1 (d)
f (0) = 0 , then (2009) 2
rankershalt.com

284 Definite Integration

x 2 + 1 −t 2
7. If f (x) = ∫ e dt, then f (x) increases in (2003, 1M) Passage Based Questions
x2
Let f (x) = (1 − x)2 sin 2 x + x2, ∀ x ∈ R and
(a) (2, 2 ) (b) no value of x x  2 (t − 1 ) 
(c) (0, ∞ ) (d) (−∞ ,0) g (x) = ∫  − ln t f (t ) dt ∀ x ∈ (1, ∞).
1  t+1 
1
8. If I (m, n ) = ∫ tm (1 + t )n dt, then the expression for
0 15. Consider the statements
I (m, n ) in terms of I (m + 1, n − 1) is (2003, 1M) P : There exists some x ∈ R such that,
(a)
2n

n
I (m + 1, n − 1)
f (x) + 2x = 2 (1 + x2).
m+1 m+1 Q : There exists some x ∈ R such that,
(b)
n
I (m + 1, n − 1) 2 f (x) + 1 = 2x (1 + x).
m+1 Then,
2n n (a) both P and Q are true (b) P is true and Q is false
(c) + I (m + 1, n − 1)
m+1 m+1 (c) P is false and Q is true (d) both P and Q are false
m
(d) I (m + 1, n − 1) 16. Which of the following is true?
m+1 (a) g is increasing on (1, ∞ )
x (b) g is decreasing on (1, ∞ )
9. Let f (x) = ∫ 2 − t dt. Then, the real roots of the
2
1 (c) g is increasing on (1, 2) and decreasing on (2, ∞ )
equation x − f '(x) = 0 are
2
(2002, 1M) (d) g is decreasing on (1, 2) and increasing on (2, ∞ )
1 1
(a) ±1 (b) ± (c) ± (d) 0 and 1
2 2 Fill in the Blank
x
10. Let f : (0, ∞ ) → R and F (x) = ∫ f (t ) dt. sec x cos x sec2x + cot x cosec x
0
If F (x2) = x2 (1 + x), then f (4) equals (2001, 1M)
17. f (x) = cos 2 x cos 2 x cosec2x .
2 2
5 1 cos x cos x
(a) (b) 7 (c) 4 (d) 2
4 π /2
x 1
Then, ∫ f (x) dx = K .
0
∫0 f (t ) dt = x + ∫ t f (t ) dt, then the value of f (1) is (1987, 2M)
11.
x
(1998, 2M)
1 1 Analytical & Descriptive Questions
(a) (b) 0 (c) 1 (d) −
ln t x
2 2 18. For x > 0, let f (x) = ∫ dt. Find the function
12. Let f : R → R be a differentiable function and f (1) = 4. 1+ t 1

f ( x) 2t f (x) + f (1 / x) and show that f (e) + f (1 / e) = 1 / 2, where


Then, the value of lim ∫ dt is ln t = log e t.
x→1 4 x −1 (1990, 2M)
(2000, 5M)

(a) 8 f ′(1) (b) 4 f ′(1) 19. Let a + b = 4, where a < 2 and let g (x) be a
(c) 2 f ′(1) (d) f ′(1) differentiable function. If
dg
> 0, ∀ x prove that
1 −x 2 dx
13. The value of the definite integral ∫ (1 + e ) dx is a b

(a) −1
0
∫ 0 g(x) dx + ∫ 0 g (x) dx increases as (b − a ) increases.
(1997, 5M)
(b) 2 (1981, 2M)
20. Determine a positive integer n ≤ 5, such that
(c) 1+ e−1 1 x
(d) None of the above ∫ 0e (x − 1)n dx = 16 − 6e
(1992, 4M)
x
Objective Question II 21. If ‘ f ’ is a continuous function with ∫ 0 f (t ) dt → ∞ as
(One or more than one correct option) |x|→ ∞ , then show that every line y = mx intersects the
sin( 2x ) −1 x
14. If g (x) = ∫ sin (t ) dt, then curve y2 + ∫ f (t ) dt = 2
sin x (2017 Adv.) 0 (1991, 2M)
π π
(a) g′  −  = 2 π (b) g′  −  = − 2 π 22. Investigate for maxima and minima the function,
 2  2
x
π π f (x) = ∫ [2(t − 1)(t − 2)3 + 3(t − 1)2(t − 2)2] dt.
(c) g′   = 2 π (d) g′   = − 2 π 1 (1988, 5M)
 2  2
rankershalt.com

Definite Integration 285

Topic 4 Limits as the Sum


Objective Question I (Only one correct option) Objective Questions II
 (n + 1)1/3 (n + 2)1/3 (2n )1/3  (One or more than one correct option)
1. lim  + + ..... +  is equal to
n→ ∞  n 4/3
n 4/3
n 4/3  4. For a ∈ R (the set of all real numbers), a ≠ − 1,
(2019 Main, 10 April I) (1a + 2a + K + n a ) 1
lim = .
(a)
4 4/3
(2) (b)
3 4/3 4
(2) − n→∞ (n + 1 )a−1 [(na + 1 ) + (na + 2 ) + K + (na + n )] 60
3 4 3 Then, a is equal to
(c)
3
(2)4/3 −
3
(d)
4 3/ 4
(2) −15 −17
(a) 5 (b) 7 (c) (d)
4 4 3 2 2
 n n n 1 n
n n−1
n
2. lim  + 2 + 2 + ... +  is equal to 5. Let S n = ∑ and Tn = ∑ 2 , for
n → ∞  n2 +1 2
n +2 2
n +3 2
n
5
k=0 n +2
kn + k 2
k=0 n + kn + k2
(2019 Main, 12 Jan II) n = 1, 2, 3, ... , then (2008, 4M)
(a) tan − 1 (3) (b) tan − 1 (2) π π
(a) Sn < (b) Sn >
(c) π / 4 (d) π /2 3 3 3 3
(n + 1)(n + 2) K 3n 
1/ n π π
3. lim   is equal to (c) Tn < (d) Tn >
n→ ∞  2n
n  (2016 Main)
3 3 3 3
18 27
(a) (b) Analytical & Descriptive Question
e4 e2
(c)
9
(d) 3 log 3 − 2  1 1 1
e2
6. Show that, lim  + +K+  = log 6.
n→ ∞ n + 1 n + 2 6n  (1981, 2M)

Answers
Topic 1 75.
1
log 6 −
1
76. f ( x ) = x 3 + x 2 − x + 2 77.
8
1. (a) 2. (d) 3. (b) 4. (a) 2 10 π2
5. (a) 6. (c) 7. (d) 8. (d) 1 1 π απ π2
82. log 2 − + 83. 84.
9. (c) 10. (a) 11. (c) 12. (d) 2 2 4 sin α 16
13. (d) 14. (c) 15. (d) 16. (d)  3 1 1  3π + 1 
17. (b) 18. (a) 19. (c) 20. (a) 85.  − π +  86. (log 3 ) 87. ( ii )
 12 2 20   π 2 
21. (d) 22. (b) 23. (a) 24. (c)
25. (b) 26. (a) 27. (c) 28. (c)  tn+ 1 − 1 
88.   89. (0) 90. (9)
29. (b) 30. (c) 31. (a) 32. (a) (t − 1 )(n + 1 ) 
33. (a) 34. (c) 35. (a) 36. (d)
91. (2)
37. (d) 38. (d) 39. (c) 40. (a)
41. (d) 42. (a, b, c) 43. (c, d) 44. (b) Topic 2
45. (a) 46. (c) 47. (c, d) 48. (b, d) 1. (d) 2. (c) 3. (b) 6. (4)
49. (a, b) 50. (*) 51. (a, c) 52. (a)
53. (a, b, c) 54. (2) 55. (8/3) 56. (k = 16) Topic 3
1. (a) 2. (b) 3. (a) 4. (c)
1  7 
57. (2) 58. π 2 59. a log 2 − 5a + b 5. (a) 6. (a) 7. (d) 8. (a)
a − b2
2  2 
 1 9. (a) 10. (c) 11. (a) 12. (a)
60.   61. π ( 2 − 1 ) 62. (4) 63. (2 − 2 )
 2 13. (d) 14. (*) 15. (c) 16. (b)
 15 π + 32 1 2
64. A → S; B → S; C → P; D → R 65. (d) 17. −   18. (ln x ) 20. (n = 3 )
 60  2 
24   1 e  1 
66. 5051 67. e cos   + sin   − 1 
5   2 7
2  2  22. At x = 1 and , f ( x ) is maximum and minimum respectively.
5
 4π  1  π
68.  tan −1    70. 71. log 2
 3  2  2 Topic 4
π π 1. (c) 2. (b) 3. (b) 4. (b, d)
72. (log 2 ) 73. π 2 74. [ π + 3 log (2 + 3 ) − 4 3 ] 5. (a, d)
8 12
rankershalt.com

Hints & Solutions


Topic 1 Properties of Definite Integral Alternate Solution
α+1 π /2 cot x
dx Let I = ∫
1. Let I = ∫ (x + α ) (x + α + 1) 0 cot x + cosec x
dx
α
α+1 cos x
(x + α + 1) − (x + α )
= ∫ (x + α ) (x + α + 1)
dx =∫
0
π /2
sin
cos x
x
1
dx
α
+
α+1 sin x sin x
 1 1 
= ∫  −  dx
 x + α x + α + 1 =∫
π / 2 cos x
dx
α
α +1
0 cos x + 1
= [ log e (x + α ) − log e (x + α + 1)] α
x
α+1 π /2
2 cos 2 −1
  x+ α  =∫ 2 dx
= log e  
 x + α + 1  α
0 x
 2 cos 2
2
2α + 1 2α
= log e − log e  θ θ
2α + 2 2α + 1 Q cos θ = 2 cos 2 − 1 and cos θ + 1 = 2 cos 2
 2 2 
 2α + 1 2α + 1  9 π /2
= log e  ×  = log e   (given) 1 2 x
 2α + 2 2α   8 =∫ 1 − sec  dx
0  2 2
(2 α + 1)2 9 π
π /2
⇒ = ⇒ 8 [4α 2 + 4α + 1] = 36 (α 2 + α )  x 1
4α (α + 1) 8 = x − tan = − 1 = (π − 2)
 
2 0 2 2
⇒ 8α 2 + 8α + 2 = 9α 2 + 9α Since, I = m(π − n )
⇒ α2 + α − 2 = 0 1
∴m(π − n ) = (π − 2)
⇒ (α + 2) (α − 1) = 0 2
⇒ α = 1, − 2 On comparing both sides, we get
From the options we get α = − 2 1
m = and n = − 2
π /2 cot x 2
2. Let I = ∫ dx
0 cot x + cosec x 1
Now, mn = × − 2 = − 1
cos x 2
π /2 π / 2 cos x π /3
=∫ sin x dx = ∫
∫ sec
dx 3. Let I= 2/3
x cosec4/3 x dx
0 cos x
+
1 0 1 + cos x
π /6
sin x sin x
π / 2 cos x(1 − cos x) π /3
=∫ 1
0 1 − cos 2 x
dx = ∫ cos 2/ 3
x sin 4/3 x
dx
π /6
π / 2 cos x − cos 2 x
=∫ dx π /3
sec2 x
= ∫
2
0 sin x dx
π /2 π/6
(tan x)4/ 3
=∫ (cosec x cot x − cot2 x) dx
0
π /2 [multiplying and dividing the denominator by cos 4/3 x]
=∫ (cosec x cot x − cosec2 x + 1) dx
0 Put, tan x = t, upper limit, at x = π / 3 ⇒ t = 3 and
= [− cosec x + cot x + x]π0 / 2 lower limit, at x = π / 6 ⇒ t = 1 / 3
π /2
  2 x  and sec2 x dx = dt
  − 2 sin 
2 
π /2
 cos x − 1   3
= x+ = x +  3
dt  t − 1/3 
 
sin x  0  x
2 sin cos 
x So, I= ∫ = 
t 4/3  − 1 / 3 1/
 2 2  0 1/ 3 3
π /2
 x π 1  1 
= x − tan = − 1 = [π − 2] = − 3  1/ 6 − 31/ 6
 2  0 2 2 3 
= m[π + n ] [given]
1 = 3 ⋅ 31/ 6 − 3 ⋅ 3− 1/ 6
On comparing, we get m = and n = − 2
2 = 37/ 6 − 35/ 6
∴ m⋅ n = − 1
rankershalt.com

Definite Integration 287

4. Given integral 6. Key Idea Use property of definite integral.



I = ∫ [sin 2x ⋅ (1 + cos 3x)]dx ∫a f ( x) dx = ∫a f ( a + b − x) dx
b b
0
π
= ∫ [sin 2x ⋅ (1 + cos 3x)]dx
0 π
sin3 x
2π Let I=∫ 2 dx …(i)
+ ∫ [sin 2x ⋅ (1 + cos 3x)]dx 0 sin x + cos x
π
= I1 + I 2 (let) ... (i) b b


On applying the property,
∫a f (x)dx = ∫a f (a + b − x) dx,
Now, I 2 = ∫ [sin 2x ⋅ (1 + cos 3x)]dx we get
π

let 2π − x = t, upper limit t = 0 and lower limit t = π π /2 cos3 x


I=∫ dx …(ii)
and dx = − dt 0 cos x + sin x
0
So, I 2 = − ∫ [− sin 2x ⋅ (1 + cos 3x)]dx On adding integrals (i) and (ii), we get
π
π/ 2 sin3 x + cos3 x
π
= ∫ [− sin 2x ⋅ (1 + cos 3x)]dx …(ii) 2I = ∫ dx
0
0 sin x + cos x
π π
I = ∫ [sin 2x ⋅ (1 + cos 3x)]dx (sin x + cos x) (sin 2 x + cos 2 x − sin x cos x)
∴ 0 =∫ 2 dx
π 0 sin x + cos x
+ ∫0 [− sin 2x ⋅ (1 + cos 3x)]dx π
 1 
[from Eqs. (i) and (ii)] =∫ 1 − (2 sin x cos x) dx
2
 20 
π
= ∫ (−1)dx] [Q [x] + [− x] = − 1, x ∉Integer] π
 1 
0
= ∫ 2 1 − sin 2x dx
0  2 
= −π
π/ 2
1
 1  π  1 π 1
5. Let I = ∫ x cot−1 (1 − x2 + x4 )dx = x + cos 2x =  − 0 + (−1 − 1) = −
 4   2  4 2 2
0
0

Now, put x2 = t ⇒ 2xdx = dt π 1 π −1


⇒ I= − =
Lower limit at x = 0, t = 0 4 4 4
x
Upper limit at x = 1, t = 1 7. Given, f (x) = ∫ g (t )dt
1
1
cot−1 (1 − t + t 2)dt
2 ∫0
∴I= 0

On replacing x by (−x), we get


1
1
 1   1 −x
= ∫ tan −1   dt Q cot−1 x = tan −1
20 1 − t + t 
2  x 
f (− x) = ∫ g(t )dt
0
1
1  t − (t − 1)  Now, put t = − u, so
= ∫ tan −1   dt
20  1 + t (t − 1) x x
f (− x) = − ∫ g (− u )du = − ∫ g (u )du = − f (x)
1 
1
= ∫ ( tan −1 t − tan −1 (t − 1)dt  0 0
2 0  [Q g is an even function]
 −1 x − y −1 −1  ⇒ f (− x) = − f (x) ⇒ f is an odd function.
Q tan 1 + xy = tan x − tan y
Now, it is given that f (x + 5) = g (x)
 
1 1 1 ∴ f (5 − x) = g (− x) = g (x) = f (x + 5)
Q ∫ tan −1 (t − 1)dt = ∫ tan −1 (1 − t − 1)dt = − ∫ tan −1 (t )dt
[Q g is an even function]
0 0 0
a a ⇒ f (5 − x) = f (x + 5) …(i)
because ∫ f (x)dx = ∫ f (a − x)dx x
0 0 Let I = ∫ f (t )dt
1
1 0
So, I = ∫ ( tan −1 t + tan −1 t )dt Put t = u + 5 ⇒ t − 5 = u ⇒ dt = du
20
x −5 x −5

∫ f (u + 5)du = ∫ g(u )du


1 1
−1 −1 t ∴ I=
= ∫ tan tdt = [t tan t ]10 −∫ dt
0 0
1 + t2 −5 −5
Put u = − t ⇒ du = − dt, we get
[by integration by parts method] 5 −x 5
π 1 π 1
= − [log e (1 + t 2)]10 = − log e 2
I=− ∫ g (− t )dt = ∫ g(t )dt
4 2 4 2 5 5 −x
rankershalt.com

288 Definite Integration


b a
Also, upper limit x = e ⇒ t = 1 and lower limit x = 1 ⇒
[Q − ∫ f (x)dx = ∫ f (x)dx and g is an even function] 1
a b
t=
5 e
1  1 1
I= ∫ f ′ (t )dt [by Leibnitz rule f ′ (x) = g (x)] 1
∴ I = ∫  t 2 −  ⋅ dt ⇒ I = ∫ (t − t −2) dt
5 −x 1/ e  t t 1/ e
1
= f (5) − f (5 − x) = f (5) − f (5 + x) [from Eq. (i)]   t 2 1   1   1  3 1
I =  +   =  + 1 −  2 + e  = − e − 2
   
 2 t 1  2  2
5 5 2 e 2 e
= ∫ f ′ (t )dt = ∫ g(t )dt e
5+ x 5+ x π /4 dx
11. Let I = ∫
8. The given functions are π /6 sin 2x(tan5 x + cot5 x)
2 − x cos x 
g (x) = log e x, x > 0 and f (x) = π /4 (1 + tan 2 x) tan5 x 2 tan x 
2 + x cos x =∫ dx Q sin 2x = 
π /6 2 tan x (tan10 x + 1)  1 + tan 2 x 
π /4
Let I=∫ g ( f (x))dx 1 π / 4 tan 4 x sec 2x
2 ∫π / 6 (tan10 x + 1)
−π / 4
= dx
π /4  2 − x cos x 
Then, I=∫ log e   dx …(i) Put tan5 x = t [Q sec 2x = 1 + tan 2 x]
−π / 4  2 + x cos x
⇒ 5 tan 4 x sec 2x dx = dt
Now, by using the property π π
b b x
∫ f (x)dx = ∫ f (a + b − x)dx, we get 6
5
4
a a  1 
t   1
π /4  2 + x cos x  3
I=∫ log e   dx …(ii)
−π / 4  2 − x cos x 
1 1 1 dt 1
On adding Eqs. (i) and (ii), we get ∴I= ⋅ ∫(1/ = (tan −1 (t ))(11/ 3 )5
2 5 3 )5 t 2 + 1 10
π /4   2 − x cos x   2 + cos x  
2I = ∫ log e   + log e   dx 1  −1  1  
−π / 4   2 x cos x 
−1
 2 + x cos x =  tan (1) − tan  
 10  9 3
π /4  2 − x cos x 2 + x cos x
=∫ log e  ×  dx 1 π −1  1  
−π / 4  2 + x cos x 2 − x cos x  =  − tan  
10  4 9 3
[Q log e A + log e B = log e AB] 2 sin 2 x
π /4 12. Let I = ∫ dx
⇒ 2I = ∫ log e (1)dx = 0 ⇒ I = 0 = log e (1) −2 1 x
−π / 4 +
a 2  π 
9. Let I = ∫ f (x) g (x) dx … (i)
0 sin 2 x
a Also, let f (x) =
1 x
= ∫0 f (a − x) g (a − x) dx +
2  π 
Q a f (x) dx = a 
 ∫0 ∫0 f (a − x) dx Then, f (− x) =
sin 2 (− x)
(replacing x by − x)
a 1  x
+ −
⇒ I= ∫0 f (x) [4 − g (x)] dx 2  π 
[Q f (x) = f (a − x) and g (x) + g (a − x) = 4]
sin 2 x   − [x], if x ∈ I
a a = Q [− x ] = 
= ∫0 4f (x) dx − ∫0 f (x) g (x) dx 1   x     − 1 − [x], if x ∉ I 
+  − 1−
a 2   π  
⇒ I = 4 ∫ f (x) dx − I [from Eq. (i)]
0
sin 2 x
⇒ 2I = 4 ∫ f (x) dx ⇒
a a
I = 2 ∫ f (x) dx. ⇒ f (− x ) = − = − f (x )
1 x
0 0 +
e 
e  x
x 2x 2  π 
10. Let I = ∫   −    log e x dx
1  e  x i.e. f (x) is odd function
 
x  a  a 0, if f (x) is odd function 
 x  x ∴ I = 0 Q ∫ f (x) dx =
Now, put   = t ⇒ x log e   = log t 2 f (x)dx, if f (x) is even function 
 e  e  − a  ∫0 
⇒ x (log e x − log e e) = log t b
  1  1 13. We have, I = ∫ (x4 − 2x2)dx
⇒ x  x + (log e x − log e e) dx = t dt
a
  Let f (x) = x4 − 2x2 = x2(x2 − 2)
1 1
⇒ (1 + log e x − 1) dx = dt ⇒ (log e x) dx = dt = x2(x − 2 ) (x + 2 )
t t
rankershalt.com

Definite Integration 289

Graph of y = f (x) = x4 − 2x2 is Y


y=|cos x|
Y
y=f(x)

X′ X
X O π/2 π
– √2 √2 ∴
O f(x) < 0 for – √2 < x < √2 Y′
+ – – + π
π
– √2 0 √2 I = ∫ |cos x|3 = 2 ∫ 2|cos x|3 dx
0 0

b π
Note that the definite integral ∫ (x4 − 2x2)dx represent (Q y = |cos x|is symmetric about x = )
a 2
the area bounded byy = f (x) , x = a, b and the X -axis. π
  π 
But between x = − 2 and x = 2 , f (x) lies below the = 2∫ 2 cos3 x dx Q cos x ≥ 0 for x ∈ 0, 2 
0  
X-axis and so value definite integral will be negative.
Also, as long as f (x) lie below the X-axis, the value of Now, as cos 3x = 4 cos3 x − 3 cos x
definite integral will be minimum. 1
∴ cos3 x = (cos 3x + 3 cos x)
∴(a , b) = (− 2 , 2 ) for minimum of I. 4
π /3 tan θ 1 π
14. We have, ∫ dθ = 1 − , (k > 0) 2 2
4 ∫0
0 2k sec θ 2 ∴I= ( cos 3x + 3 cos x) dx
tan θ
π /3 1 π /3 tan θ π
Let I = ∫02k sec θ
dθ =
2k ∫0 secθ

1 sin 3x 2
= + 3 sin x
1 π /3 (sin θ) 1 π /3 sin θ 2  3  0
2k ∫0 2k ∫0
= dθ = dθ
1 cos θ 1  1 3π π  1
(cos θ) 
cosθ =  sin + 3 sin − sin 0 + 3 sin 0 
2  3 2  
2  3 
Let cos θ = t ⇒ − sin θ dθ = dt ⇒ sin θ dθ = − dt
for lower limit, θ = 0 ⇒ t = cos 0 = 1 1  1  
=  (−1) + 3 − [0 + 0]
π
for upper limit, θ = ⇒ t = cos =
π 1 2 3  
3 3 2
 3π  π π 
Q sin 2 = sin  π + 2  = − sin 2 = − 1
1
1 1/ 2 − dt −1 1/ 2 −
⇒ I=
2k ∫1 t
=
2k ∫1 t 2dt  
1 1 1  4
 = − +3 =
− +1 2   3
1
2 3
1  t 2 
1
1
=−   =− [2 t ]12 b b
2k  − 1 + 1 2k 16. Key idea Use property = ∫ f (x)dx = ∫ f (a + b − x)dx
 2 1
a a
2  1  2  1 π /2 2
=−  2 − 1 = 1 −  sin x
2k   2 k  2 Let I= ∫ 1 + 2x
dx
1 −π/ 2
Q I = 1− (given) π π
2 
2 π / 2 sin  − + − x
2  1   2 2 
∴ 1 −
2k  2
 = 1−
1
2

2
2k
=1 ⇒ I= ∫ −
π
+
π
−x
dx
−π/ 2
1+2 2 2
⇒ 2 = 2k ⇒ 2k = 4 ⇒ k = 2
 b b 
15. We know, graph of y = cos x is Q ∫ f (x)dx = ∫ f (a + b − x)dx
 a a 
Y π /2
sin 2 x
⇒ I= ∫ 1 + 2−x dx
−π / 2
π /2
X′ π X 2x sin 2 x
O π/2 ⇒ I= ∫ 2x + 1
dx
−π / 2
π /2
 2 x + 1
Y′ ⇒ 2I = ∫ sin 2 x x
 2 + 1
 dx
−π / 2
∴ The graph of y =|cos x|is π /2
⇒ 2I = ∫ sin
2
x dx
−π / 2
rankershalt.com

290 Definite Integration


π /2 4log x dx
⇒ 2I = 2 ∫ sin 2 x dx [Qsin 2 x is an even function] ⇒ I=∫ …(i)
[log x + log(6 − x)]
2
0
4 log(6 − x)
π /2
⇒ I=∫ dx …(ii)
⇒ I= ∫ sin xdx 2 log(6 − x) + log x
2

Q b f (x)dx = b f (a + b − x)dx
 ∫a ∫a
0
π /2  a a  
⇒ I= ∫ cos 2xdx Q ∫ f (x)dx = ∫ f (a − x)dx On adding Eqs. (i) and (ii), we get
0  0 0 
4 log x + log(6 − x)
π /2
2I = ∫ dx
⇒ 2I = ∫ dx 2 log x + log(6 − x)

0 4
π ⇒ 2I = ∫ dx = [x]42 ⇒ 2I = 2
⇒ 2I = [x]π0 / 2 ⇒ I= 2
4 ⇒ 2I = 2 ⇒ I = 1
3 π /4 dx 3 π / 4 1 − cos x 20. PLAN This type of question can be done using appropriate
17. Let I = ∫ =∫ dx substitution.
π /4 1 + cos x π / 4 1 − cos 2 x
π /2
3 π /4 1 − cos x Given, I = ∫ (2 cosec x)17 dx
=∫ dx π /4
π /4 sin 2 x
π /2 217 (cosec x)16 cosec x (cosec x + cot x)
3 π /4 =∫ dx
=∫ (cosec x − cosec x cot x)dx
2
π /4 (cosec x + cot x)
π /4

= [− cot x + cosec x]3π π/ 4/ 4 Let cosec x + cot x = t


= [(1 + 2 ) − (− 1 + 2 )] = 2 ⇒ (− cosec x ⋅ cot x − cosec2x) dx = dt
π /2 x2 cos x and cosec x − cot x = 1 /t
18. Let I = ∫ dx …(i)
1
− π / 2 1 + ex
⇒ 2 cosec x = t +
t 16
Q b f (x) dx = b f (a + b − x) dx  1
 ∫a ∫a  1 t +  dt
∴ I=−∫ 217  t
2+1
π /2 x cos (− x)
2  2  t
⇒ I=∫ dx …(ii)  
−π / 2 1 + e− x Let t = eu ⇒ dt = eudu.

On adding Eqs. (i) and (ii), we get When t = 1, eu = 1 ⇒ u = 0


π /2 2  1 1  and when t = 2 + 1, eu = 2 + 1
2I = ∫ x cos x  +  dx
−π / 2
1 + e
x
1 + e− x  ⇒ u = ln ( 2 + 1 )
π /2 2 0 eudu
=∫ x cos x ⋅ (1) dx ⇒ I=−∫ 2(eu + e− u )16
−π / 2 ln ( 2 + 1 ) eu
 a a  ln ( 2 + 1 )
Q ∫ f (x) dx = 2 ∫ f (x) dx, when f (− x) = f (x) =2 ∫ (eu + e− u )16 du
−a
 0  0
π /2 2 x − a, x ≥a
⇒ 2I = 2∫ x cos x dx 21. PLAN Use the formula,| x − a | = 
0  − ( x − a ), x < a
Using integration by parts, we get to break given integral in two parts and then integrate separately.
2I = 2 [x2(sin x) − (2x) (− cos x) + (2) (− sin x)] π0 / 2 2
π  x π x
⇒ 2I =2
π2
− 2
 ∫0 1 − 2 sin  dx = ∫ |1 − 2 sin |dx
 2 0 2
4  π
 x π  x
π2 = ∫ 3 1 − 2 sin  dx − ∫ π 1 − 2 sin  dx
∴ I= −2 0  2  2 
3
4 π
π
 x 3  x
19. PLAN Apply the property =  x + 4 cos  −  x + 4 cos 
b b  
2 0  2 π
∫a f(x )dx = ∫a f(a + b − x )dx and then add. 3
π
4 log x 2 =4 3 −4 −
Let I=∫ dx 3
2log x + log(36 − 12x + x2)
2
π /2  2  π − x 
4 2 log x 22. I = ∫ x + log  π + x  cos x dx
=∫ dx − π /2
2 2 log x + log(6 − x)2  
a
2 log x dx
∫− a f (x) dx = 0, when f (− x) = − f (x)
4
=∫ As,
2 2 [log x + log(6 − x)]
rankershalt.com

Definite Integration 291

π /2 π /2
∴ I=∫ x2 cos x dx + 0 = 2∫ (x2 cos x) dx 1/ 2   1 + x 
− π /2 0 26. ∫ −1/ 2[x] + log 1 − x  dx
π /2
= 2 {(x2 sin x)π0 / 2 − ∫ 2x ⋅ sin x dx} 1/ 2 1/ 2  1 + x
0
=∫ [x] dx + ∫ −1/ 2log 1 − x dx
−1/ 2
π 2 π /2
=2  − 2 {(− x ⋅ cos x)0π / 2 − ∫ 1 ⋅ (− cos x) dx}
4 0  1/ 2   1 + x 
=∫ [x]dx + 0 Q log  1 − x  is an odd function 
−1/ 2
π 2  π  π  2 2
 
=2  − 2 (sin x)π0 / 2 = 2  − 2 =  − 4
  
4 4 2 0 1/ 2 0 1/ 2
   =∫ [x] dx + ∫0 [x] dx = ∫ (−1) dx + ∫0 (0) dx
−1/ 2 −1/ 2
23. Put x = t ⇒ x dx = dt /2
2
 1 1
= − [x]0−1/ 2 = − 0 +  = −
sin t ⋅
dt  2 2
log 3
∴ I=∫ 2 ...(i) π cos 2 x
log 2 sin t + sin (log 6 − t )
27. Let I = ∫ dx …(i)
b b
−π 1 + ax
Using, ∫a f (x) dx = ∫a f (a + b − x) dx −π cos 2(− x)
=∫ d (− x)
1 log 3 sin (log 2 + log 3 − t ) π 1 + a −x
=
2 ∫log 2 sin (log 2 + log 3 − t ) + sin dt
π cos 2 x
(log 6 − (log 2 + log 3 − t )) ⇒ I=∫ ax dx …(ii)
1 log 3 sin (log 6 − t )
−π 1 + ax
= ∫ dt
2 log 2 sin (log 6 − t ) + sin (t ) On adding Eqs. (i) and (ii), we get
log 3 sin (log 6 − t ) π 1 + ax 
∴ I=∫ dt …(ii) 2I = ∫   cos 2x dx
log 2 sin (log 6 − t ) + sin t −π  1 + a x 

On adding Eqs. (i) and (ii), we get π π 1 + cos 2x


=∫ cos 2 x dx = 2 ∫ dx
1 log 3 sin t + sin (log 6 − t ) −π 0 2
2 ∫log 2 sin (log 6 − t ) + sin t
2I = dt π π π
= ∫ (1 + cos 2x) dx = ∫ 1 dx + ∫ 0 cos 2x dx
0 0
1 log 3 1
⇒ 2I = (t ) = (log 3 − log 2) π /2
2 log 2 2 = [x]π0 + 2∫ cos 2x dx = π + 0
0
1  3
∴ I = log   ⇒ 2I = π ⇒ I = π /2
4  2
0
ecos x sin x, for | x| ≤ 2
24. Let I = ∫ [x + 3x + 3x + 3 + (x + 1) cos (x + 1)] dx
3 2 
−2 28. Given, f (x) =  2 , otherwise
0 
=∫ [(x + 1)3 + 2 + (x + 1) cos (x + 1)] dx 
−2 3 2 3

Put x + 1 = t
∴ ∫ −2 f (x) dx = ∫ −2 f (x) dx + ∫ 2 f (x) dx
⇒ dx = dt =∫
2 3

1 −2
ecos x sin x dx + ∫ 2 2 dx = 0 + 2 [x]32
∴ I=∫ (t + 2 + t cos t ) dt
3
−1 [Q ecos x sin x is an odd function]
Q 3 f (x) dx = 2
 ∫ −2
1 1 1
=∫ t3 dt + 2 ∫ = 2 [3 − 2] = 2
−1 −1
dt + ∫ −1 t cos t dt 

= 0 + 2 ⋅ 2 [x]10 + 0 e2 2
log e x  dx = 1  log e x  dx − e  log e x  dx
=4
29. ∫e −1
 x  ∫ e  x  ∫1  x 
− 1

[since, t3 and t cos t are odd functions]  since, 1 is turning point for 
 log e x 
1 1−x 1 1−x   for + ve and − ve values
25. I = ∫ dx = ∫ dx  x  
0 1+ x 0
1 − x2
log e x e 2 log x
dx + ∫  e 
1
1 1 1 x = −∫ dx
=∫ dx − ∫ dx e −1
x 1  x 
0
1 − x2 0
1 − x2 1 1 2
= − [(log e x)2]1e−1 + [(log e x)2] 1e
0 t 2 2
= [sin −1 x] 10+ ∫1 dt
t 1 1 5
= − {0 − (−1)2} + (22 − 0) =
[where, t 2 = 1 − x2 ⇒ t dt = − x dx] 2 2 2
−1 −1
= (sin 1 − sin 0) + [t ]10 = π /2 − 1
rankershalt.com

292 Definite Integration


1 0 1
30. The graph of y = 2 sin x for π /2 ≤ x ≤ 3π / 2 is given in ∴ ∫ −1 [x] dx = ∫ −1 [x] dx + ∫ 0 [x] dx
figure. From the graph, it is clear that 0 1
 2, if x = π /2 =∫ (−1) dx + ∫0 0 dx
−1
 1, if π / 2 < x ≤ 5π /6 1

[2 sin x] =  0, if 5π / 6 < x ≤ π
= − [x]−01 + 0 = −1; ∴ ∫ −1 f (x) dx = 1
−1, if π < x ≤ 7π / 6 x
 33. Given, g (x) = ∫ cos 4 t dt
−2, if 7π / 6 < x ≤ 3π /2 0
π+x
Y ⇒ g (x + π ) = ∫ cos 4 t dt
0
2
π π+x
= ∫ cos 4 t dt + ∫ cos 4 t dt = I1 + I 2
1 0 π

π π
X
π/2 5π/6 7π/6 3π/2 where, I1 = ∫ cos 4 t dt = g (π )
–1 0
π+x
–2 and I2 = ∫ cos 4 t dt
π

Put t=π+ y
3 π /2
Therefore, ∫ π /2 [2 sin x]dx ⇒ dt = dy
x
=∫
5 π /6
dx + ∫
π 7 π /6 3 π /2
I 2 = ∫ cos 4 ( y + π ) dy
π /2 5π / 6
0 dx + ∫π (−1) dx + ∫
7π / 6
(−2) dx 0
x x
= [x] 5π π/ 2/ 6
+ [− x] + 7π / 6
π [−2x] 37ππ //62 = ∫ (− cos y)4 dy = ∫ cos 4 y dy = g (x)
0 0
 5 π π   7 π   −2 ⋅ 3 π 2 ⋅ 7 π 
= −  + − + π +  +  ∴ g (x + π ) = g (π ) + g (x)
 6 2   6   2 6 
k
 5 1  7 7 
= π  −  + π 1 −  + π  − 3
 6 2 
34. Given, I1 = ∫ x f [x (1 − x)] dx
6 3  1−k
k
 5 − 3  1  7 − 9
=π
 6 
 + π −  + π 
 6  3 
 = −π / 2 ⇒ I1 = ∫ (1 − x) f [(1 − x) x] dx
1−k
3 π /4 dx k k
31. Let I=∫ …(i) =∫ f [(1 − x)]dx] − ∫ xf (1 − x)]dx
π /4 1 + cos x 1−k 1 −k

3 π /4 dx I1 1
⇒ I=∫ ⇒ I1 = I 2 − I1 ⇒ =
π /4 1 + cos (π − x) I2 2
3 π /4 dx
I=∫ …(ii) 35. It is a question of greatest integer function. We have,
π /4 1 − cos x subdivide the interval π to 2π as under keeping in view
that we have to evaluate [2 sin x ]
On adding Eqs. (i) and (ii), we get
Y
3 π /4  1 1 
2I = ∫  +  dx 1,π/2
π / 4  1 + cos x 1 − cos x
3 π /4  2 
⇒ 2I = ∫   dx X'
O
X
π /4  1 − cos x 
2
(0,π) 30° 30° (0,2π)
3 π /4
⇒ I=∫ cosec2x dx = [− cot x] 3π π/ 4/ 4
π /4
–1,3π/2 –1/2,11π/6
3π π –1/2,7π/6
 Y'
= − cot + cot = − (−1) + 1 = 2
 4 4  π 1
1 1 1 1
We know that, sin =
32. Let ∫ −1 f (x) dx = ∫ −1 (x − [x]) dx = ∫ −1 x dx − ∫ −1 [x] dx 6 2
π 7π
 1
1 ∴ sin  π +  = sin =−
=0−∫ [x] dx [Q x is an odd function]  6 6 2
−1
11π  π π 1
−1, if −1 ≤ x < 0 ⇒ sin = sin 2π −  = − sin = −
 6  6 6 2
But [x] =  0, if 0 ≤ x<1
 1, if 9π 3π
 x=1 ⇒ sin = sin = −1
6 6
rankershalt.com

Definite Integration 293

π
Hence, we divide the interval π to 2π as 39. Let I = ∫ ecos
2
x
⋅ cos3 {(2n + 1) x} dx
 7π   7π 11 π   11π 
0
π , ,  , ,  , 2π  f (a − x) = − f (x)
 6  6 6   6  a
0,
Using∫ f (x) dx =  a /2
 1  1  1 
sin x = 0, −  ,  − 1, −  ,  − , 0
0
2 ∫ 0 f (x) dx, f (a − x) = f (x)
 2  2  2  2
Again, let f (x) = ecos x
⋅ cos3 {(2n + 1) x}
⇒ 2 sin x = (0, − 1), (−2, − 1), (−1, 0) 2
⇒ [2 sin x] = − 1 ∴ f (π − x) = (ecos x
){ − cos3 (2n + 1) x} = − f (x)
7π / 6 11 π / 6
=∫ ∴ I =0
π
[2 sin x] dx + ∫ 7π / 6 [2 sin x] dx
π /2 cot x
2π 40. Let I = ∫ dx …(i)
+ ∫ 11π / 6 [2 sin x] dx 0 cot x + tan x
7π / 6 11 π / 6 2π π /2 tan x
=∫ ⇒ I=∫
π
(−1) dx + ∫ 7π / 6 (−2) dx + ∫ 11π / 6 (−1) dx 0 cot x + tan x
dx …(ii)

π  4π  π 10π 5π On adding Eqs. (i) and (ii), we get


=− −2  − =− =−
6  6  6 6 3 π /2
2I = ∫ 1 dx
πx
36. Given, f (x) = A sin   1 0
 + B, f ′   = 2 π
2  2
∴ I=
1 2A 4
and ∫ 0 f (x) dx = π π /3 dx
41. Let I = ∫ …(i)
π / 6 1 + tan x
Aπ πx  1  Aπ π Aπ
f ′ (x) = cos ⇒ f′  = cos =
2 2  2 2 4 2 2 π /3 dx
∴ I=∫
 1 Aπ 4 π /6
π 
But f′   = 2 ∴ = 2 ⇒ A= 1 + tan  − x
 2 2 2 π 2 
1 2A 1   π x  2A
Now, ∫ f (x) dx = ⇒ ∫  A sin   + B dx = π /3 dx
0 π 0   2   π =∫
π /6 1 + cot x
1
 2A πx  2A 2A 2A
⇒ − cos + Bx = ⇒ B+ =
 π  π π π π /3 tan xdx
2 0 ⇒ I=∫ …(ii)
π /6 1 + tan x
⇒ B =0
π /2 1 π /2 1 On adding Eqs. (i) and (ii), we get
37. Let I=∫ dx = ∫ dx π /3
0 1 + tan3 x 0 sin3 x 2I = ∫ dx
1+ π /6
cos3 x
cos3 x
π /2 ⇒ 2I = [x]ππ //36 dx
⇒ I=∫ dx …(i)
0 cos x + sin3 x
3
1 π π  π
⇒ I= − =
π  2  3 6  12
cos3  − x
π /2 2 
⇒ I=∫ dx Statement I is false.
0  π  π 
cos3  − x + sin3  − x b b
2  2  But ∫ a f (x)dx = ∫ a f (a + b − x)dx is a true statement by
π /2 sin3 x property of definite integrals.
⇒ I=∫ dx …(ii)
0 sin x + cos3 x
3
42. According to the given data,
On adding Eqs. (i) and (ii), we get F ′ (x) < 0, ∀x ∈ (1, 3)
π /2
2I = ∫ 1 dx ⇒ 2I = [x]π0 / 2 = π / 2 ⇒ I = π /4 We have, f (x) = x F (x)
0
⇒ f ′ (x) = F (x) + x F ′ (x) …(i)
π /2
38. Let I = ∫ [f (x) + f (− x) ] [ g (x) − g (− x)] dx ⇒ f ′ (1) = F (1) + F ′ (1 ) < 0
−π / 2
[given F (1) = 0 and F ′ (x) < 0]
Let φ (x) = [ f (x) + f (− x)] [ g (x) − g (− x)]
Also, f (2) = 2F (2) < 0 [using F (x) < 0,∀x ∈ (1, 3)]
⇒ φ (− x) = [ f (− x) + f (x)] [ g (− x) − g (x)]
Now, f ′ (x) = F (x) + x F ′ (x) < 0
⇒ φ (− x) = − φ (x)
[using F (x) < 0, ∀ x ∈ (1, 3)]
⇒ φ(x) is an odd function.
π /2
⇒ f ′ (x) < 0
∴ ∫ −π / 2 φ (x) dx = 0
rankershalt.com

294 Definite Integration


3 π
∫1 x F ′ (x) dx = − 12
2
43. Given, 2

3
⇒ [x2F (x)]31 − ∫ 2x ⋅ F (x) dx = − 12
(b) f (x) + ∫ f (t )sin t dt always positive
0
1
3 ∴Option (b) is incorrect.
⇒ 9 F (3) − F ( 1 ) − 2∫ f (x) dx = − 12 π
−x
1
2

3
[QxF (x) = f (x), given] (c) Let h (x) = x − ∫ f (t ) cos t dt,
− 36 − 0 − 2 ∫ f (x) dx = −12
0
⇒ π
1
3 3 3 2
∴ ∫1 f (x) dx = − 12 and ∫1 x F ′ ′ (x)dx = 40 h (0) = − ∫ f (t ) cos t dt < 0
0
3 π
⇒ [x3 F ′ (x)] 31 − ∫ 3x2F ′ (x) dx = 40 −1
1 2

⇒ [x2(xF ′ (x)] 31 − 3 × (−12) = 40


h (1) = 1 − ∫ f (t ) cos t dt > 0
0
⇒ { x2 ⋅ [ f ′ (x) − F (x)]} 31 = 4 ∴ Option (c) is correct.
⇒ 9 [ f ′ (3) − F (3)] − [ f ′ (1) − F (1)] = 4 (d) Let g (x) = x9 − f (x)
⇒ 9 [ f ′ (3) + 4] − [ f ′ (1) − 0] = 4 g (0) = − f (0) < 0
g (1) = 1 − f (1) > 0
⇒ 9 f ′ (3) − f ′ (1) = − 32
∴ Option (d) is correct.
t (t − a )
∫a f (x) dx −
2
{ f (t ) + f (a )} 98 k + 1
(k + 1)
44. Given, lim =0
t→ a (t − a ) 3 48. I = ∑ ∫ x(x + 1)
dx
k =1 k
Using L’Hospital’s rule, put t − a = h 98 k + 1
(k + 1)
a+ h h
∫ a f (x) dx − 2 { f (a + h ) + f (a )} Clearly, I> ∑ ∫ (x + 1)2
dx
k =1 k
⇒ lim =0
h→ 0 h3 98 k+1
1
1 h
f (a + h ) − { f (a + h ) + f (a )} − { f ′ (a + h )}
⇒ I> ∑ (k + 1) ∫ (x + 1)2
dx
k =1 k
⇒ lim 2 2 =0 98 98
h→ 0 3h 2  1 1  1
Again, using L’ Hospital’s rule,
⇒ I> ∑ (−(k + 1)) k + 2 − k + 1  ⇒ I> ∑k+2
k =1 k =1
1 1 h
f ′ (a + h ) − f ′ (a + h ) − f ′ (a + h ) − f ′ ′ (a + h ) ⇒
1
I > +…+
1
>
98
⇒ I>
49
lim 2 2 2 =0 3 100 100 50
h→ 0 6h
98 k + 1 98
k+1
h
− f ′′ (a + h ) Also, I< ∑ ∫ x(k + 1)
dx = ∑ [log e (k + 1) − log e k]
⇒ lim 2 =0 k =1 k k =1
h→ 0 6h
⇒ f ′′ (a ) = 0, ∀ a ∈ R I < log e 99
⇒ f (x) must have maximum degree 1. 49. Here, f (x) = 7 tan 8 x + 7 tan 6 x − 3 tan 4 x − 3 tan 2 x
45. F ′ (c) = (b − a ) f ′ (c) + f (a ) − f (b)  −π π 
for all x ∈  , 
 2 2
F ′ ′ (c) = f ′ ′ (c)(b − a ) < 0
f (b) − f (a ) ∴ f (x) = 7 tan 6 x sec2 x − 3 tan 2 x sec2 x
⇒ F ′ (c) = 0 ⇒ f ′ (c) =
b−a = (7 tan 6 x − 3 tan 2 x) sec2 x
π /4 π /4
π   π Now, ∫ x f (x)dx = ∫ x (7 tan 6 x − 3 tan 2 x) sec2 x dx
−0  0 +  
π /2  π 0 0 I II
46. ∫ sin x dx = 2 sin 0 + sin   + 2 sin 
2  π /4
0 4  2  2  = [x (tan x − tan x)]
7 3
0
   π /4
π −∫ 1 (tan7 x − tan3 x) dx
= (1 + 2 ) 0
8 π /4
47. x =0 − ∫ tan3 x (tan 4 x − 1)dx
0
Q e ∈ (1, e) in (0, 1) and ∫ f (t )sin t dt ∈ (0, 1) in (0, 1)
x
π /4
0 = –∫ tan 3 x (tan 2 x − 1) sec2 x dx
0
x
∴ ex − ∫ f (t )sin t dt cannot be zero. Put tan x = t ⇒ sec2 x dx = dt
π /4 1
∫0 x f (x)dx = − ∫ t3 (t 2 − 1) dt
0

So, option (a) is incorrect. 0
rankershalt.com

Definite Integration 295

1
1  t 4 t5  1 1 1 From Eqs. (i), (ii), (iii) and (iv), we get
= ∫ (t3 − t5 )dt =  −  = − =
0
4 5  0 4 6 12 I1 = I 2 + eπ ⋅ I 2 + e2π ⋅ I 2 + e3 π ⋅ I 2 = (1 + eπ + e2π + e3 π ) I 2

∫ e (sin at + cos at )dt
π /4 π /4 t 6 4
Also, ∫0 f (x) dx = ∫
0
(7 tan 6 x − 3 tan 2 x) sec2 x dx ∴ L = 0π
∫0 e (sin at + cos at )dt
t 6 4
1
= ∫ (7t − 3t )dt = [t
6 2 7
− t3 ]10 =0
0 = (1 + eπ + e2π + e3 π )
192 x3 192x3 192x3 1⋅ (e4π − 1)
50. Here, f ′ (x) = ∴ ≤ f ′ (x) ≤ = for a ∈ R
2 + sin πx eπ − 1
4
3 2
1
On integrating between the limits to x, we get 1 x4 (1 − x)4 1 (x4 − 1 ) (1 − x)4 + (1 − x)4
2 52. Let I = ∫ dx = ∫0 dx
0 1 + x2 (1 + x2)
192x3
x x x 192 x3
∫1/ 2 3 dx ≤ ∫1/ 2 f ′ (x)dx ≤ ∫1/ 2 2 dx 1
= ∫ (x2 − 1) (1 − x)4 dx + ∫
1 (1 + x2 − 2x)2
dx
192  4 1 
0 0 (1 + x2)
4 3
⇒  x −  ≤ f (x) − f (0) ≤ 24x −
12  16 2 1 4x2 
= ∫ (x2 − 1) (1 − x)4 + (1 + x2) − 4x +  dx
3 0
 (1 + x2) 
⇒ 16x4 − 1 ≤ f (x) ≤ 24x4 −
2 1 4 
= ∫  (x2 − 1) (1 − x)4 + (1 + x2) − 4x + 4 −  dx
1 0 1 + x2
Again integrating between the limits to 1, we get
2 1  6 4 
1 1 1  4 3
=∫  x − 4x + 5x − 4x + 4 −
5 4 2
 dx
 1 + x2
∫1/ 2 (16x − 1) dx ≤ ∫1/ 2 f (x) dx ≤∫1/ 2 24x − 2 dx
4 0

1
 x7 4x6 5x5 4x3 
+ 4x − 4 tan −1 x
1 1
16x5  1 24x5 3  = − + −
⇒  − x ≤ ∫ f (x)dx ≤  − x 7 6 5 3 0
 5 1/ 2 1/ 2  5 2 1/ 2
1 4 5 4 π  22
 11 2 1  33 6 = − + − + 4 − 4  − 0 = −π
⇒  +  ≤ ∫ f (x)dx ≤  +  7 6 5 3 4  7
5 5 1/ 2  10 10
π sin nx
1 53. Given In = ∫ dx …(i)
⇒ 2 .6 ≤ ∫ f (x) dx ≤ 3 . 9 −π (1 + π x ) sin x
1/ 2
b b
(*) None of the option is correct. Using ∫ a f (x) dx = ∫ a f (b + a − x) dx, we get

51. Let I1 = ∫ et (sin 6 at + cos 4 at )dt π π x sin nx
0
In = ∫ dx …(ii)
π t −π (1 + π x ) sin x
= ∫ e (sin 6 at + cos 4 at ) dt
0
2π On adding Eqs. (i) and (ii), we have
+∫ et (sin 6 at + cos 4 at ) dt π sin nx π sin nx
π
2I n = ∫ dx = 2 ∫ dx
3π t − π sin x 0 sin x
+∫ e (sin 6 at + cos 4 at ) dt
2π sin nx
4π t [Q f (x) = is an even function]
+∫ e (sin 6 at + cos 4 at ) dt sin x

π sin nx
∴ I1 = I 2 + I3 + I 4 + I5 …(i) ⇒ In = ∫ dx
0 sin x

Now, I3 = ∫ e (sin at + cos at ) dt
t 6 4
π sin (n + 2 )x − sin nx
π Now, I n + 2 − I n = ∫ dx
Put t = π + x⇒ dt = dx 0 sin x
π π 2 cos (n + 1 ) x ⋅ sin x
∴ I3 = ∫ e π +x
⋅ (sin 6 at + cos 4 at ) dt = eπ ⋅ I 2 …(ii) =∫ dx
0 0 sin x
3π t π
Now, I4 = ∫ e (sin 6 at + cos 4 at ) dt π sin (n + 1) x 
2π = 2 ∫ cos (n + 1) x dx = 2   =0
0
 (n + 1)  0
Put t = 2π + x ⇒ dt = dx
π ∴ In + 2 = In …(iii)
∴ I 4 = ∫ ex + 2π (sin 6 at + cos 4 at ) dt = e2π ⋅ I 2 …(iii) π sin nx
In = ∫
0
4π t
Since, dx
and I5 = ∫ e (sin 6 at + cos4 at ) dt
0 sin x

⇒ I1 = π and I 2 = 0
Put t = 3π + x From Eq. (iii) I1 = I3 = I5 = .... = π
π
∴ I5 = ∫ e3 π + x (sin 6 at + cos4 at ) dt = e3 π ⋅ I 2 …(iv) and I 2 = I 4 = I 6 = ... = 0
0
rankershalt.com

296 Definite Integration


10 10
⇒ 1 = F (k ) − F (1 )
[F (t )]16
⇒ ∑ I 2m + 1 = 10 π and ∑ I 2m = 0  d e sin x 
m=1 m =1
Q dx { F (x)} = x , given 
∴ Correct options are (a), (b), (c).  
1/ 2 1+ 3 ⇒ F (16) − F (1) = F (k) − F (1)
54. (2) Let I=∫ dx
0 [(x + 1)2 (1 − x)6 ]1/ 4 ∴ k = 16
37 π π sin (π log x)
1/ 2 1+ 3 57. Let I = ∫ dx
⇒ I= ∫0 1/ 4
dx 1 x
 1 − x  6
Put π log x = t
(1 − x )2    π
 1 + x   ⇒ dx = dt
x
1−x − 2 dx
Put =t ⇒ = dt ∴ I=∫
37π
sin (t ) dt = − [cos t ]37 π
= − [cos 37π − cos 0]
1+ x (1 + x )2 0
0

1 1 = − [(−1) − 1] = 2
when x = 0, t = 1, x = ,t =
2 3 2π x sin 2n x
58. Let I = ∫ dx …(i)
∴ 0 sin x + cos 2n x
2n

1/ 3 (1 + 3 ) dt 2π (2π − x)[sin (2π − x)]2n


I= ∫1 −2( t ) 6/ 4 I=∫
0 [sin (2π − x)]2n + [cos (2π − x)]2n
dx
a a
[Q ∫ f (x) dx = ∫ f (a − x) dx]
1/ 3
−(1 + 3 )  −2
⇒ I=  t 
0 0
2 1 2π (2π − x) ⋅ sin 2n x
I=∫ dx
⇒ I = (1 + 3 ) ( 3 − 1) ⇒ I = 3 − 1 = 2 0 sin 2n x + cos 2n x
1 x
55. Given, f (1) = and 6 ∫ f (t )dt = 3x f (x) − x3 , ∀ x ≥ 1
2π 2π sin 2n x 2π x sin 2n x
⇒ I=∫ dx − ∫ dx
3 1 0 sin x + cos x
2n 2n 0 sin n x + cos 2n x
2

Using Newton-Leibnitz formula.


2π 2π sin 2n x
Differentiating both sides ⇒ I=∫ dx − I [from Eq. (i)]
0 sin 2n x + cos 2n x
⇒ 6 f (x) ⋅ 1 − 0 = 3 f (x) + 3xf ′ (x) − 3x2
1 2π π sin 2n x
⇒ 3xf ′ (x) − 3 f (x) = 3x2 ⇒ f ’ (x) − f (x) = x ⇒ I=∫ dx
x 0 sin x + cos 2n x
2n

xf ’ (x) − f ’ (x) d x


⇒ =1 ⇒  =1  π π sin 2n x
x2 dx  x  ⇒ I = π ∫ dx
 0 sin x + cos x
2n 2n
On integrating both sides, we get
π sin 2n (2π − x) 
f (x)  1 +∫ dx
⇒ = x+ c Q f (1) = 0 sin (2 π − x) + cos 2n (2 π − x)
2n
x`  3  
1 2 2 using property 
= 1 + c ⇒ c = and f (x) = x2 − x  2a f (x) dx = a [f (x) + f (2a − x)]dx
3 3 3 ∫ 0 ∫0 
4 8
∴ f (2) = 4 − =  π sin 2n x dx
3 3 I = π ∫ dx
0 sin 2n x + cos 2n x
NOTE Here, f(1) = 2, does not satisfy given function. 
∴ f(1 ) =
1 π sin 2n x 
3
+∫ dx
0 sin x + cos x 
2n 2n
2 4 8
For that f (x) = x2 − x and f (2) = 4 − = π sin 2n x dx
3 3 3 ⇒ I = 2π ∫ dx
0 sin 2n x + cos 2n x
2
4 2e sin x
56. Given, ∫1 x
dx = F (k) − F (1)

 π /2
I = 4π ∫
sin 2n x dx 
dx …(ii)
Put x2 = t  0 sin 2n
x + cos 2n
x 
⇒ 2x dx = dt π /2 sin 2n (π / 2 − x)
⇒ I = 4π ∫ dx
16 e sin t dt 0 sin 2n (π / 2 − x) + cos 2n (π / 2 − x)
⇒ ∫1 2
t

2
= F (k) − F (1)
π /2 cos 2n x
sin t ⇒ I = 4π ∫ dx …(iii)
16 e 0 cos x + sin 2n x
2n
⇒ ∫1 t
dt = F (k) − F (1)
rankershalt.com

Definite Integration 297

On adding Eqs. (ii) and (iii), we get 3 π /4 π−x


=∫ dx
π / 2 sin 2n
x + cos x2n π /4 1 + sin (π − x)
2I = 4π ∫ dx
2n
x + cos 2n x 3 π /4 π 3 π /4 x
=∫ ∫ π /4
0 sin dx − dx
π /2 π
π /4 1 + sin x 1 + sin x
⇒ 2I = 4π ∫ 1 dx = 4π [x]π0 / 2 = 4π ⋅ 3 π /4 dx
2 = π∫
0
−I [from Eq. (i)]
⇒ I=π 2 π /4 1 + sin x
1 π 3 π /4 dx
59. Given, af (x) + bf (1 / x) =
x
−5 …(i) =
2 ∫ π /4 (1 + sin x)
Replacing x by 1 / x in Eq. (i), we get π 3 π /4 (1 − sin x)
af (1 / x) + bf (x) = x − 5 …(ii)
=
2 ∫ π /4 (1 + sin x) (1 − sin x)
dx

On multiplying Eq. (i) by a and Eq. (ii) by b, we get π 3 π /4 (1 − sin x)


1 
=
2 ∫ π /4 1 − sin 2 x
dx
a 2f (x) + abf (1 / x) = a  − 5 …(iii)
x 
π 3 π /4  1 sin x 
abf (1 / x) + b2f (x) = b (x − 5) …(iv) =
2 ∫ π /4  −  dx
 cos x cos 2 x
2

On subtracting Eq. (iv) from Eq. (iii), we get π 3 π /4


2 ∫ π /4
a = (sec2 x − sec x ⋅ tan x) dx
(a 2 − b2) f (x) = − bx − 5a + 5b
x π
= [tan x − sec x]3π π/ 4/ 4
1 a 
⇒ f (x) = 2  − bx − 5a + 5b 2
(a − b2)  x  π
= [− 1 − 1 − (− 2 − 2 )]
2 1 2 a  2
⇒ ∫ 1 f (x) dx = (a 2 − b2) ∫ 1  x − bx − 5a + 5b dx π
= (− 2 + 2 2 ) = π ( 2 − 1)
2 2
1  b 
= a log|x| − x2 − 5(a − b)x 2
2   ∫ −2 |1 − x |dx
2
(a − b ) 
2
2 1
62.
1 −1 1 2
= [a log 2 − 2b − 10 (a − b) =∫ (x2 − 1) dx + ∫ (1 − x2) dx + ∫ (x2 − 1) dx
(a − b2)2 −2 −1 1
−1 1 2
b   x3   x3   x3 
− a log 1 + + 5 (a − b) = − x + x −  + − x
2  3 3 3
  −2   −1  1
1  7   1 8   1 1  8 1 
= a log 2 − 5a + b =  − + 1 + − 2 + 1 − + 1 −  +  − 2 − + 1
(a 2 − b2)  2   3 3   3 3   3 3 
3 x =4
60. Let I=∫ dx …(i)
2 5−x+ x 1.5 1 2 1.5
63. ∫0 [x2] dx = ∫ 0 dx + ∫
0 1
1 dx + ∫ 2
2 dx
3 2+3−x
⇒ I=∫ dx = 0 + [x]1 2 + 2 [x]1.52
2 (2 + 3) − (5 − x) + 2 + 3 − x
= ( 2 − 1) + 2 (1.5 − 2 )
3 5−x
⇒ I=∫ dx …(ii) = 2 −1 + 3 −2 2
2 x + 5 −x
=2− 2
On adding Eqs. (i) and (ii), we get 1 dx
64. (A) Let I = ∫
−1 1 + x2
3 x+ 5−x 3 1
2I = ∫ dx ⇒ 2I = ∫ 1 dx = 1 ⇒ I =
2 5−x+ x 2 2 Put x = tan θ ⇒ dx = sec2 θ dθ
π /4 π
3 π /4 x ∴ I = 2∫ dθ =
61. Let I=∫ dx …(i) 0 2
π /4 1 + sin x
1 dx
 π 3π
 +

− x
(B) Let I=
1 − x2
∫0
3 π /4 4 4 
⇒ I=∫ dx
π /4  π 3π  Put x = sinθ
1 + sin  + − x
4 4  ⇒ dx = cosθ d θ
π/ 2 π
∫0
b b
[Q ∫ f (x) dx = ∫ f (a + b − x) dx] ∴ I= 1 dθ =
a a 2
rankershalt.com

298 Definite Integration

3 a
3 dx1  1 + x  (S) PLAN ∫− a f( x ) dx = 0
(C) ∫ 2 1 − x 2 2 log  1 − x  
=
If f( − x ) = − f( x ), i.e. f( x ) is an odd function.
2
1  4  3  1   2   1 + x
= log   − log    = log    Let f ( x ) = cos 2x log  
2  −2   
−1  2   3   1 − x
2 dx π π  1 − x
(D) ∫ = [sec−1 x]12 = −0 = f (− x) = cos 2x log   = − f (x)
1
x x −1 2 3 3  1 + x
65. (P) PLAN (i) A polynomial satisfying the given conditions is taken. Hence, f (x) is an odd function.
(ii) The other conditions are also applied and the number 1/ 2
of polynomial is taken out. So, ∫−1/ 2 f (x) dx = 0
Let f ( x ) = ax + bx + c
2
(P) → (ii); (Q) → (iii); (R) → (i); (S) → (iv)
f (0) = 0 ⇒ c = 0 1
1
66. Let I 2 = ∫ (1 − x50)101 dx,
∫0 f (x) dx = 1
0
Now,
1
= [(1 − x50 )101 ⋅ x]10 + ∫ 0 (1 − x 50 ⋅ x49⋅x dx
50 100
)
 ax 2 1
3
bx α β
⇒  +  =1 ⇒ + =1
 3 2 0 3 2 [using integration by parts]
1
⇒ 2a + 3b = 6 =0 − ∫ (50) (101) (1 − x50 )100 (− x50 ) dx
0

As a , b are non-negative integers. 1


= − (50) (101) ∫ (1 − x50 )101 dx
So, a = 0, b = 2 or a = 3, b = 0 0
1
∴ f (x) = 2x or f (x) = 3x2 + (50) (101) ∫ (1 − x50 )100 dx = 5050I 2 + 5050I1
0
(Q) PLAN Such type of questions are converted into only sine or
cosine expression and then the number of points of ∴ I 2 + 5050I 2 = 5050I1
maxima in given interval are obtained. (5050)I1
⇒ = 5051
f ( x ) = sin ( x 2 ) + cos ( x 2 ) I2
 1 1 
= 2 cos (x2) + sin (x2) 67. Let
 2 2  π  1  1 
I = ∫ e |cos x | 2 sin  cos x + 3 cos  cos x  sin x dx
 π π  0  2  2 
= 2 cos x2 cos + sin sin (x2)
 4 4  π 1 
⇒ I = ∫ e |cos x | ⋅ sin x ⋅ 2 sin  cos x dx
 π 0 2 
= 2 cos  x2 − 
 4 π |cos x | 1 
π π
+ ∫ 0e ⋅ 3 cos  cos x ⋅ sin x dx
2 
For maximum value, x2 − = 2nπ ⇒ x2 = 2nπ +
4 4 ⇒ I = I1 + I 2 …(i)
π 9π using 2 a 
⇒ x=± ,for n = 0 ⇒ x = ± , for n = 1
4 4  ∫ 0 f (x) dx 
  0, f (2a − x) = − f (x) 
So, f (x) attains maximum at 4 points in [− 13 , 13].  = a 
(R) PLAN  2 ∫ 0 f (x) dx, f (2a − x) = + f (x)
a a
(i) ∫− a f( x ) dx = ∫− a f( − x ) dx where, I1 = 0 [Q f (π − x) = − f (x)] …(ii)
a a π / 2 cos x 1 
(ii) ∫− a f( x ) dx = 2 ∫0 f( x ) dx , if f( − x ) = f( x ) , i.e. f is an even and I 2 = 6∫
0
e ⋅ sin x ⋅ cos  cos x dx
2 
function.
3x 2 1  t
2 I 2 = 6 ∫ et ⋅ cos   dt
I= ∫−2 1 + ex dx Now,
0  2
[put cos x = t ⇒ − sin x dx = dt]
2 3x2
and I=∫ dx  t  t 1 t t 
1
−2 1 + e− x = 6 e cos   + ∫ e sin dt 
  2 2 2 0
2  3x2 3x2(ex )
2I = ∫
1
⇒  +  dx   t 1  t et t 
−2  1 + ex ex + 1  = 6 et cos   +  et sin − ∫ cos dt 
 2 2  2 2 2 
2 2  0
2I = ∫ 3x2 dx ⇒ 2I = 2 ∫ 3x2 dx 1
−2 0  t 1 t I
= 6 et cos + et sin − 2
I = [x3 ]20 = 8  2 2 
2 0 4
rankershalt.com

Definite Integration 299

24   1 e  1  − π /4  π 
=
5 e cos  2 + 2 sin

  − 1
 2 
…(iii) ∴ 2I = − 2 ∫ π /4 f cos  − 2t  cos t dt
 2 
π /4
From Eq. (i), we get ⇒ 2I = 2 ∫ −π / 4 f (sin 2t ) cos t dt
24   1 e  1 
I= e cos   + sin   − 1 π /4
5   2 2  2  ∴ I= 2 ∫0 f (sin 2t ) cos t dt
π /3 π dx π /3 x3 dx
68. Let I = ∫ + 4∫ π ecos x
− π /3  π − π /3  π 70. Let I = ∫ dx …(i)
2 − cos |x|+  2 − cos |x|+  0 ecos x + e− cos x
 3  3
f (− x) = − f (x)
π ecos ( π − x )
a  0, =∫ dx
Using ∫ f (x) dx =  a 0 ecos ( π − x ) + e− cos ( π − x )
−a
2 ∫ 0 f (x) dx, f (− x) = f (x) a a
[Q∫ f (x) dx = ∫ f (a − x) dx]
0 0
π /3 π dx
∴ I =2 ∫ +0
 π π e− cos x
⇒ I=∫
0
2 − cos |x| +  0 e− cos x
dx …(ii)
 3 + ecos x
  On adding Eqs. (i) and (ii), we get
 x3 dx  π ecos x + e− cos x π
Q is odd  =∫ dx = ∫ 1 dx = [x] π0 = π
 2 − cos |x|+ π   0 ecos x
+e − cos x 0
  3 
⇒ I = π /2
π /3 dx
I = 2π ∫ 1 −1  1  1  1−x+ x 
0 2 − cos (x + π / 3) 71. ∫ 0 tan   dx = ∫ tan −1   dx
π
 1 − x + x2 0
1 − x(1 − x) 
Put x+ = t ⇒ dx = dt 1
3 = ∫ [ tan −1 (1 − x) − tan −1 x] dx
t 0
2 π /3 2 π /3
sec2 dt
dt 1 1
∴ I = 2π ∫ π /3 = 2π ∫ π /3 2 = ∫ tan −1 [1 − (1 − x) ] dx + ∫ 0 tan
−1
x dx
2 − cos t 1 + 3 tan 2
t 0

= 2∫ tan −1x dx Q ∫ f (x) dx = ∫ f (a − x) dx …(i)


2 1 a a

t t 0 
 0 0 
Put tan =u ⇒ sec2 dt = 2 du
2 2 1  1 
3 2 du 4π Now, ∫ tan −1   dx
[ 3 tan −1 3u ] 13 1 − x + x 
∫ 1/
2
⇒ I = 2π = 0
3 1 + 3u 2 3 3 1 π  1 
= ∫  − cot−1    dx
4π 4π  1 0 2 1 − x + x  
2
= (tan −1 3 − tan −1 1) = tan −1   
3 3  2
π 1
π + 4x3 4π = − ∫ tan −1 (1 − x + x2) dx
π /3  1
∫ −π /3 tan −1   2 0
∴ dx =
 π 3  2 1 π 1 1
2 − cos |x|+  ∴ ∫ tan −1 (1 − x + x2) dx = − ∫ tan −1 dx
 3  0 2 0 (1 − x + x2)
π /2 π
69. Let I = ∫ f (cos 2x) cos x dx …(i) = − 2I1
0 2
 π  π
1 1 x dx
∫ 0 tan ∫ 0 1 + x2
−1
⇒ I=∫
π /2 
f  cos 2  − x  ⋅ cos  − x dx where, I1 = x dx = [x tan−1 x ]10 −
0  2  2 
using a f (x) dx = a f (a − x) dx π 1 π 1
∫0 ∫0 =
− [log(1 + x 2 )]10 = − log 2
  4 2 4 2
I=∫
π /2 1 π π 1 
⇒ f (cos 2x) sin x dx …(ii) ∴ ∫ tan−1(1 − x + x 2 ) dx = − 2  − log 2 = log 2
0 0 2 4 2 
On adding Eqs. (i) and (ii), we get π /4
72. Let I = ∫ log (1 + tan x) dx …(i)
π /2 0
2I = ∫ f (cos 2x) (sin x + cos x) dx
0 π /4 π
I=∫ log (1 + tan ( − x)) dx
π /2 0 4
= 2 ∫0 f (cos 2x) [cos (x − π / 4)] dx
π /4  1 − tan x 
π ∴ I=∫ log 1 +  dx
Put − x + = t ⇒ − dx = dt 0  1 + tan x
4
rankershalt.com

300 Definite Integration

π /4  1 + tan x + 1 − tan x Now, cos −1 (− x) = π − cos −1 x for −1 ≤ x ≤ 1.


=∫ log   dx
0  1 + tan x  1/ 3 y4  −1  2 y 

∴ I=∫ π − cos   dy
2 
π /4  2  π /4 −1/ 3 1 − y4
 1 + y  
I=∫ log   dx ⇒ I = ∫ log 2 dx − I
0  1 + tan x 0
1/ 3 y4 1/ 3 y4  2y 
= π∫ dy − ∫ cos −1   dy
π π −1/ 3 1− y 4 −1/ 3 1− y 4
 1 + y2
⇒ 2I = log 2 ⇒ I = (log 2)
4 8
1/ 3 x4 1/ 3 x4  2x 
π 2 x (1 + sin x) =π∫ dx − ∫ cos −1   dx
73. Let I=∫ dx −1/ 3 1−x 4 −1/ 3 1 − x4  1 + x2 
− π 1 + cos 2 x

π π 1/ 3 x4
I=∫
2x
dx + ∫
2x sin x
dx ⇒ I = π∫ dx − I [from Eq. (i)]
− π 1 + cos 2 x − π 1 + cos 2 x −1/ 3 1 − x4
x4 1/ 3  1 
⇒ I = I1 + I 2 ⇒ 2I = π ∫
1/ 3
dx = π ∫ −1 +
−1/ 3 1 − x4 −1/ 3   dx
π
2x  1 − x4 
Now, I1 = ∫ dx
1 + cos 2 x
−π 1/ 3 1/ 3 dx
=−π∫ 1 dx + π ∫
2x −1/ 3 −1/ 3 1 − x4
Let f (x) =
1 + cos 2 x 1/ 3 dx
−2 x − 2x = − π [x]1−/1/3 3 + π I1, where I1 = ∫
−1/ 3 1 − x4
⇒ f (− x) = = = − f (x)
1 + cos 2(− x) 1 + cos 2 x
 1 1 2π
⇒ f (− x) = − f (x) which shows that f (x) is an odd ⇒ 2I = − π  +  + πI1 = − + π I1
 3 3 3
function.
∴ I1 = 0 1/ 3 dx 1/ 3 dx
2x sin x
Now, I1 = ∫ −1/ 3 1−x 4
=2 ∫
0 1 − x4
Again, let g (x) =
1 + cos 2 x [since, the integral is an even function]
2 (− x)sin (− x) 2x sin x 1 + 1 + x2 − x2
⇒ g (− x) = = = g (x) 1/ 3
1 + cos 2(− x) 1 + cos 2 x
= ∫0 (1 − x2) (1 + x2)
dx

⇒ g (− x) = g (x) which shows that g (x) is an even 1/ 3 1 1/ 3 1


function. = ∫0 1−x 2
dx + ∫
0 1 + x2
dx
π 2 x sin x π x sin x
∴ I2 = ∫ dx = 2 ⋅ 2 ∫ dx 1/ 3 1 1/ 3 1
− π 1 + cos x 0 1 + cos 2 x = ∫0 dx + ∫
2
dx
(1 − x) (1 + x) 0 (1 + x2)
π (π − x)sin (π − x) π (π − x)sin x
=4 ∫ dx = 4 ∫ dx 1 1/ 3 1 1 1/ 3 1 1/ 3 1
0 1 + [cos (π − x)]2 0 1 + cos 2x =
2 ∫0 1−x
dx + ∫
2 0 1+ x
dx + ∫
0 1 + x2
dx
π π sin x π x sin x
=4 ∫ dx − 4 ∫ dx 1/ 3
0 1 + cos 2x 0 1 + cos 2x  1 1 
= − ln|1 − x| + ln|1 + x| + tan −1 x
π sin x  2 2 
⇒ I 2 = 4π ∫ dx − I 2 0
0 1 + cos 2x 1/ 3
1   1 + x 
π sin x =   + [tan −1 x]01/ 3
⇒ 2I 2 = 4π ∫ dx ln 1 − x 
0 1 + cos 2x
2    0

cos x = t ⇒ − sin x dx = dt 1  1 + 1 / 3
 + tan −1 1
Put = ln 
−1 dt 1 dt 1 dt 2 1 − 1 / 3 3
∴ I2 = − 2 π ∫ =2 π ∫ =4 π ∫ 0 1 + t2
1 1 + t2 −1 1 + t 2
1  3 + 1  
 + π = 1 ln  ( 3 + 1)  + π
2

−1 −1 −1
= ln 
= 4π [tan t ] 10 = 4π [tan 1 − tan 0] 2  3 − 1 6 2  3 − 1  6
= 4π (π / 4 − 0) = π 2 1 π
= ln (2 + 3 ) +
∴ I = I1 + I 2 = 0 + π 2 = π 2 2 6
1/ 3  x4   2x  − 2π π π2
74. Let I = ∫   cos −1   dx …(i) ∴ 2I = + ln (2 + 3 ) +
− 1/ 3  1 − x4   1 + x2 3 2 6
Put x = − y ⇒ dx = − dy π
= [π + 3 ln (2 + 3 ) − 4 3 ]
−1/ 3 y4  −2 y  6
∴ I=∫ cos −1   (−1) dy π
1/ 3 1 − y4  1 + y2 ⇒ I= [π + 3 ln (2 + 3 ) − 4 3 ]
12
rankershalt.com

Definite Integration 301

3
 1  
Alternate Solution 3
1
π = log (x − 1) −  log (x + 1)
Since, cos −1 y = − sin −1 y 2  2 2   2
2
−1  2 x  π −1 2x π 1 2 1
= log − log
4
∴ cos   = − sin = − 2 tan −1 x
 1 + x2 2 1 + x2 2 2 1 2 3
π  From Eq. (i), I = I1 + I 2
1/ 3 x4 x4
I=∫ 2 ⋅ − 2 tan −1 x dx 1 1 1 4
−1/ 3
 1−x
4
1−x 4
 = log 2 − + log 2 − log
10 2 2 3
 x4  −1/ 2
Q 2 tan −1 x is an odd function   4
= log [2 ⋅ 21/ 2  ]−
1 1
= log 6 −
1
 1−x
4
  3 10 2 10
π
1
 1  76. Since, f (x) is a cubic polynomial. Therefore, f ′ (x) is a
∴ I =2⋅
2 ∫03  −1 +

 dx + 0
1 − x4  quadratic polynomial and f (x) has relative maximum
1
π 1/ 3  1 1  and minimum at x = and x = − 1 respectively,
=
2 ∫0  −2 +

+  dx
1 − x2 1 + x2
3
therefore, –1 and 1/3 are the roots of f ′ (x) = 0.

1/ 3  1  1 1
π 1 1+ x −1  ∴ f ′ (x) = a (x + 1)  x −  = a  x2 − x + x − 
= −2x + 2 ⋅ 1 log 1 − x + tan x  3  3 3
2  0
 2 1
π  2 3 + 1 π = a  x2 + x − 
=
1  3
− + log +  3
2  3 2 3 −1 6
Now, integrating w.r. t. x, we get
π  x3
= [π + 3 log (2 + 3 ) − 4 3 ] x2 x
12 f (x) = a  + − +c
3 3 3
3 2x5 + x4 − 2x3 + 2x2 + 1
75. Let I = ∫ dx where, c is constant of integration.
2 (x2 + 1) (x4 − 1)
Again, f (−2) = 0
3 2x5 − 2x3 + x4 + 1 + 2x2
=∫ dx  8 4 2
2 (x2 + 1) (x2 − 1)(x2 + 1) ∴ f (−2) = a  − + +  + c
 3 3 3
3 2x3 (x2 − 1) + (x2 + 1)2  −8 + 4 + 2 
=∫ dx ⇒ 0=a +c
2 (x2 + 1)2 (x2 − 1)  3 
3 2x3 (x2 − 1) 3 (x2 + 1)2 −2 a 2a
=∫ dx + ∫ 2 (x2 + 1)2(x2 − 1) dx ⇒ 0= + c ⇒ c=
2 (x + 1 )2 (x2 − 1 )
2
3 3
3 2x3 3 1 x 3
x 2
x 2a a 3
=∫ ∴ f (x) = a  + − + = (x + x2 − x + 2)
2 (x2 + 1 )2
dx + ∫ 2 (x2 − 1) dx 3 3 3 3 3
⇒ I = I1 + I 2 1 14
Again, ∫ f (x) dx = [given]
2x3
3 −1 3
Now, I1 = ∫ dx
2 (x2 + 1 )2 1 a 14
⇒ ∫ −1 3 (x + x2 − x + 2) dx =
3

Put x + 1 = t ⇒ 2x dx = dt
2 3
1 a 14
(t − 1) ⇒ ∫ −1 3 (0 + x + 0 + 2) dx = 3
10 10 1 10 1 2
∴ I1 = ∫ 2
dt = ∫ dt − ∫ dt
5 t 5 t 5 t2
10 [Q y = x3 and y = − x are odd functions]
1 
= [log t ]10 + a 1 2
2 x dx + 4∫ 1 dx =
1 14
3  ∫ 0
5  t  ⇒
5 0  3
1 1 1
= log 10 − log 5 + − a  2x3   14
10 5 ⇒  + 4x  =
1
3  3  0 3
= log 2 −
10 a 2  14 a  14 14
⇒  + 4 = ⇒   =
1 3 3 1 3 3  3 3 3 3
Again, I 2 = ∫ dx = ∫ dx
2 (x2 − 1 ) 2 (x − 1 )(x + 1 )
⇒ a =3
1 3 1 1 3 1 f (x) = x3 + x2 − x + 2
=
2 ∫ 2 (x − 1) dx − 2 ∫ 2 (x + 1) dx Hence,
rankershalt.com

302 Definite Integration

π  79. We know that,


x sin (2x) ⋅ sin  cos x
π 2 
77. Let I=∫ dx …(i) 2 sin x [cos x + cos 3x + cos 5x + K + cos (2k − 1) x]
0 (2x − π )
= 2 sin x cos x + 2 sin x cos 3x + 2 sin x cos 5x
π  + K + 2 sin x cos (2k − 1) x
(π − x) ⋅ sin 2 (π − x) ⋅ sin cos(π − x)
π  2 
Then I=∫ dx = sin 2x + (sin 4x − sin 2x) + (sin 6x − sin 4x)
0 2 (π − x) − π
+ K + {sin 2kx − sin (2k − 2) x}
…(ii)
= sin 2kx
π 
(π − x) ⋅ sin 2x ⋅ sin  cos x ∴ 2 [cos x + cos 3x + cos 5x + K + cos (2k − 1) x]
π 2 
⇒ I=∫ dx sin 2kx
0 π − 2x = …(i)
sin x
π 
(x − π ) sin 2x ⋅ sin  cos x Now, sin 2kx ⋅ cot x =
sin 2kx
⋅ cos x
π 2 
⇒ I=∫ dx … (iii) sin x
0 (2x − π )
= 2 cos x [cos x + cos 3x + cos 5x + K + cos (2k − 1) x]
On adding Eqs. (i) and (iii), we get [from Eq. (i)]
π π  = [2 cos 2 x + 2 cos x cos 3x + 2 cos x cos 5x +
2I = ∫ sin 2x ⋅ sin  cos x dx
0 2  K + 2 cos x cos (2k − 1) x ]
π π  = (1 + cos 2x) + (cos 4x + cos 2x)
⇒ 2I = 2 ∫ sin x cos x ⋅ sin  cos x dx
0 2  + (cos 6x + cos 4x) + ... + {cos 2kx + cos (2k − 2) x}
π π  = 1 + 2 [cos 2x + cos 4x + cos 6x + K + cos (2k − 2) x]
⇒ I = ∫ sin x cos x ⋅ sin  cos x dx
0 2  + cos 2kx
π /2
 π π 2  ∴∫ (sin 2kx) ⋅ cot x dx
put cos x = t ⇒ − sin x dx = dt ⇒ sin x dx = − dt
 2 2 π 
0
π /2 π/ 2
2 − π / 2 2t =∫ 1 ⋅ dx + 2 ∫ ( cos 2x + cos 4x K cos (2k − 2) x) dx
∴ I=− ∫ ⋅ sin t dt 0 0
π π /2 π π /2
4 π /2
+ ∫0 cos (2k) x dx
= 2∫ t sin t dt
π −π / 2 sin 2x sin 4x
π /2
π sin (2k − 2) x 
= +2 + +K+
(2k − 2)  0
4 4 8
⇒ I = 2 [− t cos t + sin t ]−π π/ 2/ 2 = 2 × 2 = 2 2  2 4
π π π
π /2
π /2 sin (2k) x  π
78. Let I=∫ f (sin 2x) sin x dx …(i) +
 
=
0 2k 0 2
a
π /2  π  π  80. Let I = ∫ f (x) ⋅ g (x) dx
Then, I = ∫ f sin 2  − x  sin  − x dx
0   2    2  0
a a
π /2 I = ∫ f (a − x) ⋅ g (a − x) dx = ∫ f (x) ⋅ {2 − g (x)} dx
=∫ f [sin 2x] ⋅ cos x dx …(ii) 0 0
0
[Q f (a − x) = f (x) and g (x) + g (a − x) = 2]
On adding Eqs. (i) and (ii), we get a a
π /2 = 2 ∫ f (x) dx − ∫ f (x) g (x) dx
2I = ∫ f (sin 2x)(sin x + cos x) dx 0 0
0 a
π /4
⇒ I = 2 ∫ f (x) dx − I
=2 ∫
0
f (sin 2x)(sin x + cos x) dx
0 a
⇒ 2I = 2 ∫ f (x) dx
π /4  π 0
=2 2 ∫ f (sin 2x) sin  x +  dx
0  4 a a
∴ ∫ 0 f (x) g(x) dx = ∫ 0 f (x) dx
π /4  π  π π
=2 2 ∫0 f sin 2  − x  sin  − x +  dx
 4  4 4 81. Let I=∫
2a f (x)
dx …(i)
π /4
0 f (x) + f (2a − x)
=2 2 ∫0 f (cos 2x) cos x dx
I=∫
2a f (2a − x)
dx …(ii)
π /4 0 f (2a − x) + f (x)
∴ I= 2 ∫0 f (cos 2x) cos x dx
On adding Eqs. (i) and (ii), we get
π /2 π /4
Hence, ∫ f (sin 2x) ⋅ sin x dx = 2 ∫
2a
0 0
f (cos 2x) cos x dx 2I = ∫ 1 dx = 2a ⇒ I = a
0
rankershalt.com

Definite Integration 303

1 ∞
82. Let I = ∫ log ( 1 − x + 1 + x ) dx π  −1  t + cos α  
0 I= tan  sin α  
sin α  0
Put x = cos 2θ
π
⇒ dx = − 2 sin 2θ dθ = [tan −1 (∞ ) − tan −1 (cot α )]
sin α
0
∴ I = −2 ∫ log [ 1 − cos 2θ + 1 + cos 2θ ] (sin 2θ ) dθ π π π  απ
π /4 =  −  − α   =
0 sin α 2 2  sin α
= −2 ∫ log [ 2 (sin θ + cos θ )] sin 2θ dθ
π /4 απ
∴ I=
= −2 ∫
0
[(log 2 ) sin 2θ sin α
π /4 π / 2 x sin x ⋅ cos x
+ log (sin θ + cos θ ) ⋅ sin 2θ ] dθ 84. Let I = ∫ dx
0 cos 4 x + sin 4 x

 − cos 2θ 
0 π  π  π 
= − 2 log 2 π /2  2
 − x sin  − x ⋅ cos  − x
 2   2  2 
π /4 ⇒ I=∫ dx
0 4 π  4 π 
0 sin  − x + cos  − x
−2 ∫ log (sin θ + cos θ ) ⋅ sin 2θ dθ 2  2 
π /4
I II π 
π /2  2
 − x ⋅ sin x cos x

  cos 2θ 
0
⇒ I=∫ dx
= log 2 − 2 − log (sin θ + cos θ ) ⋅  0 cos 4 x + sin 4 x
  2 π / 4
π π /2 sin x cos x π / 2 x sin x ⋅ cos x
 cos θ − sin θ − cos 2θ 
0 
⇒ I=
2 ∫0 sin x + cos x
4 4
dx − ∫
0 sin 4 x + cos 4 x
dx
−∫  ×  dθ 
π / 4  cos θ + sin θ 2   π π /2 sin x ⋅ cos x
 1 0 
=
2 ∫0 sin 4 x + cos 4 x
dx − I
= log ( 2 ) − 2 0 + ∫ ( cos θ − sin θ )2 dθ
 2 π /4 
π π / 2 tanx ⋅ sec2x
1 0
⇒ 2I= ∫0 tan 4 x + 1
dx
= log 2 − ∫ (1 − sin 2θ ) dθ 2
2 π /4
π 1 π /2 1
0 ⇒ 2I= ⋅ ∫ d (tan 2 x)
1  cos 2θ  2 2 0 1 + (tan 2 x)2
= log 2 − θ +
 2  π / 4
2 π π
⇒ 2 I = ⋅ [tan −1 t ]0∞ = (tan −1 ∞ − tan –1 0)
1 1 π 1 1 π 4 4
= log 2 −  −  = log 2 − + [where, t = tan 2 x]
2  2 4  2 2 4
π2
π x ⇒ I=
83. Let I = ∫ dx …(i) 16
0 1 + cos α sin x
1/ 2 x sin −1 x
π (π − x) 85. Let I = ∫ dx Put sin −1 x = θ ⇒ x = sin θ
⇒ I=∫ dx
0 1 + cos α sin (π − x)
0
1 − x2
π (π − x) ⇒ dx = cos θ dθ
⇒ I=∫ dx … (ii)
0 1 + cos α sin x π /6 θ sin θ π/6
∴ I=∫ ⋅ cos θ d θ = ∫ θ sin θ d θ
On adding Eqs. (i) and (ii), we get
0
1 − sin θ
2 0

π dx π/6
2 I = π∫ = [− θ cos θ ]π0 / 6 + ∫0 cos θ d θ
0 1 + cos α sin x

x
sec2 dx  π π   π  3π 1
π =  − cos + 0 + sin − sin 0 = − +
⇒ 2I=π ∫ 2  6 6   6  12 2
0 x x
(1 + tan 2 ) + 2 cos α tan π /4 (sin x + cos x)
2 2 86. Let I=∫ dx
0 9 + 16 sin 2x
x 2x
Put tan = t ⇒ sec dx = 2 dt π /4 sin x + cos x
2 2 I=∫ dx
∞ 2 dt
0 25 − 16 (sin x − cos x)2
∴ 2I = π ∫
0 1 + t 2 + 2 t cos α Put 4 (sin x − cos x) = t ⇒ 4 (cos x + sin x) dx = dt
0
∞ dt 1 0 dt 1 1 5 + t 
⇒ 2I = 2π ∫0 (t + cos α )2 + sin 2 α
∴ I=
4 ∫ −4 25 − t 2 = 4 ⋅ 2 (5) log 5 − t 
−4
rankershalt.com

304 Definite Integration

 5 + 0   On equating coefficient of t k on both sides, we get


I=
1   5 − 4 
 
log 5 − 0 − log 5 + 4 
C k ∫ (1 − x)n − k ⋅ xk dx =
40     1 1
  n
 0  n + 1
1  1 1 1
=  log 1 − log  = log 9 = (log 3)
40  9 40 20 1 n−k k 1
π
⇒ ∫ 0 (1 − x) x dx =
(n + 1)nC k
87. (i) Let I = ∫ x f (sin x) dx …(i)
0
[x], x ≤ 2
π 89. Here, f (x) = 
⇒ I=∫ (π − x) f (sin x) dx …(ii)  0, x > 2
0
2 x f (x2)
On adding Eqs. (i) and (ii), we get ∴ I=∫ dx
π −1 2 + f (x + 1)
2I=∫ π f (sin x) dx
0
0 x f (x2) 1 x f (x2)
π π π =∫ dx + ∫0 dx
∴ ∫0 x f (sin x) dx =
2 ∫ 0 f (sin x) dx −1 2 + f (x + 1) 2 + f (x + 1)
2 x f (x2) 3 x f (x2)
∫1 ∫
3/ 2
(ii) Let I=∫ |x sin πx| dx + dx + dx
−1 2 + f (x + 1) 2 2 + f (x + 1)
 x sin πx, −1 < x ≤ 1 2 x f (x2)
Since, |x sin πx| =  3 + ∫3 dx
− x sin πx, 1 < x < 2 2 + f (x + 1)
1 3/ 2 0 1 2 x⋅1
∴ I=∫ x sin πx dx + ∫1 − x sin πx dx =∫ 0 dx + ∫0 0 dx + ∫1 dx
−1 −1 2+0
1
 x cos πx  1  − cos πx 3 2
=2 −
 π 
0
− 2 ∫ 1⋅ 
0  π 
 dx + ∫ 2
0 dx + ∫3 0 dx

Q − 1 < x < 0 ⇒ 0 < x2 < 1 ⇒ [x2] = 0,


 − x cos πx 3/ 2 3 / 2  − cos πx 
− −∫   dx 0 < x < 1 ⇒ 0 < x2 < 1 ⇒ [x2] = 0,
 π  1  π 
 1   1 < x2 < 2 ⇒ [x2] = 1
1 3/ 2 1 < x< 2 ⇒
 1  2 sin πx   1  1 sin πx  2 < x + 1 < 1 + 2 ⇒ f (x+ 1) = 0,
=2   + ⋅ − −  −
 π  π  π  0  π  π  π 1
2 < x < 3 ⇒ 2 < x2 < 3 ⇒ f (x2) = 0,
2 2 1 1
= + (0 − 0) + + 2 (+ 1 − 0) and 3 < x < 2 ⇒ 3 < x2 < 4 ⇒ f (x2) = 0
π π2 π π
2
3 1  3π + 1 2 x  x2  1 1
= + 2=  ⇒ I=∫ dx =   = (2 − 1) =
π π  π2  1 2 4
 1 4 4
1 1
88. Let I = ∫ (t x + 1 − x)n dx = ∫ {(t − 1) x + 1 }n dx ∴ 4I = 1 ⇒ 4I − 1 = 0
0 0
1 ( 9 x + 3 tan −1 x )  12 + 9x2
 ((t − 1) x + 1)n + 1 
1
1  t n + 1 − 1 90. Here, α = ∫ e   dx
=  =   0  1 + x2 
 (n + 1) (t − 1)  0 n + 1  t − 1  Put 9x + 3 tan −1 x = t
=
1
(1 + t + t 2 + K + t n )  3 
…(i) ⇒ 9 +  dx = dt
n+1  1 + x2
1 1 9+3π /4 t
Again, I = ∫ (t x + 1 − x)n dx = ∫ [(1 − x) + t x]n dx ∴ α=∫ e dt = [et ]90 + 3 π / 4 = e9 + 3 π / 4 − 1
0 0 0
1 3π
=∫ [nC 0 (1 − x)n + nC1 (1 − x)n − 1 (t x) ⇒ log e 1 + α = 9 +
0 4

n
C 2(1 − x)n − 2(tx)2+ ... + nC n (tx)n + ] dx ⇒ log e α + 1 − =9
4
 n n 
rΣ
1
=∫ C r (1 − x)n − r (t x)r  dx 91. PLAN Integration by parts
=0
0
  d 
n ∫ f(x ) g (x ) dx = f(x ) ∫ g (x ) dx − ∫  dx [f(x )] ∫ g (x ) dx  dx
Σ C r ∫ (1 − x)n − r ⋅ xr dx t r
1
= n
…(ii)
r=0  0  1 d2
Given, I = ∫ 4x3 (1 − x2)5 dx
From Eqs. (i) and (ii), we get 0 I dx2 II
n 1
 
C r ∫ (1 − x)n − r ⋅ xr dx t r=
1 d d
Σ
1 1
n
(1 + t + K + t n ) = 4x3 (1 − x2)5 − ∫ 12 x2 (1 − x2)5 dx
r=0  0  n+1  dx  0 0 dx
rankershalt.com

Definite Integration 305

1 11 1 1
= 4x3 × 5 (1 − x2)4 (− 2x) We get ∫ 0 2 dt ≤ ∫ 0 f (t ) dt ≤ ∫ 0 1 dt
 0
1
− 12 [x2 (1 − x2)5 ]10 − ∫ 2x (1 − x2)5 dx
1 1
⇒ ≤ ∫ f (t ) dt ≤ 1 …(i)
 0  2 0
1 1
= 0 − 0 − 12 (0 − 0) + 12 ∫ 2x (1 − x2)5 dx Again, 0 ≤ f (t ) ≤ for t ∈ [1, 2] …(ii)
0 2
1
 (1 − x2)6   1 2 2 2
= 12 × −  = 12 0 + 6  = 2
⇒ ∫ 10 dt ≤ ∫ 1 f (t ) dt ≤ ∫ 1 dt
 6 0  
2 1
⇒ 0 ≤ ∫ f (t ) dt ≤
1 2
Topic 2 Periodicity of Integral Functions
π From Eqs. (i) and (ii), we get
dx
1. Let I = ∫ 2 1 1 2
≤ ∫ f (t ) dt + ∫ f (t ) dt ≤
3
−π [x] + [sin x] + 4 2 0 1 2
2
−1 dx 0 dx 1 3
=∫ ⇒ ≤ g (2) ≤
−π
[x] + [sin x] + 4
+ ∫−1 [x] + [sin x] + 4 2 2
2
π ⇒ 0 ≤ g (2) < 2
1 dx dx
+∫ +∫2 nπ + v π 2π
0 [x] + [sin x] + 4 1 [x] + [sin x] + 4 4. ∫0 |sin x|dx = ∫
0
|sin x|dx + ∫
π
|sin x|dx + ...
nπ nπ + v
− 2 , − π / 2 < x < −1 +∫ |sin x|dx + ∫ |sin x|dx

Q [x] =  0−,1, −1 ≤ x < 0
0 ≤ x<1
( n − 1 )π nπ

 1, 1 ≤ x < π /2 n
rπ nπ + v
=∑∫ |sin x| dx + ∫ nπ |sin x| dx
( r − 1 )π
 − 1 , − π / 2 < x < −1 r =1

and [sin x] = −01, , − 01 << xx << 10 rπ
Now to solve, ∫ ( r −1) π| sin x |dx , we have
 0, 1 < x < π / 2
x = ( r − 1)π + t
[Q For x < 0,−1 ≤ sin x < 0 and for x > 0, 0 < sin x ≤ 1]
−1 dx 0 dx 1 dx ⇒ sin x = sin [(r − 1) π + t ] = (−1)r − 1 sin t
So, I = ∫ −π +∫ +∫
− 2 − 1 + 4 − 1 − 1 − 1 + 4 0 0 + 0+4 and when x = (r − 1) π, t = 0 and when
2
π x = r π, t = π
dx
+∫ 2 rπ π r −1
1 1+0+4 ∴ ∫ ( r − 1)π|sin x| dx = ∫ 0 |(−1) sin t| dt
π π π
−1 dx dx dx dx = ∫ |sin t| dt = ∫
0 1
= ∫ −π ∫ −1 ∫0 4 ∫
+ + + 2
0 0
sin t dt
2
1 2 1 5
= [− cos t ] π0 = − cos π + cos 0 = 2
 π 1 1 1 π  nπ + v
=  −1 +  + (0 + 1) + (1 − 0) +  − 1 Again, ∫ | sin x|dx, putting x = nπ + t
 2 2 4 5 2  nπ

 1 1 1  π π  nπ + v v v
=  −1 + + −  +  + 
 2 4 5  2 10 
Then, ∫ nπ | sin x|dx = ∫ |(−1)n sin t|dt = ∫
0 0
sin t dt

−20 + 10 + 5 − 4 5π + π = [− cos t ]v0 = − cos v + cos 0 = 1 − cos v


= + n
20 10 nπ + v rπ nπ + v
∴∫ |sin x|dx = ∑ ∫ | sin x|dx + ∫ nπ |sin x|dx
9 3π 3 0 ( r − 1 )π
=− + = (4π − 3) r =1
20 5 20 n
nπ + v
3 + 3T 1 = ∑ 2 + ∫ nπ |sin x|dx
2. ∫3 f (2x) dx Put 2x = y ⇒ dx = dy
2 r =1

1 6 + 6T 6I = 2n + 1 − cos v
2∫6
∴ f ( y) dy = = 3I a+ t
2 5. Let φ (a ) = ∫ f (x) dx
a
x
3. Given, g (x) = ∫ f (t ) dt On differentiating w.r.t. a, we get
0
2 1 2 φ ′ (a ) = f (a + t ) ⋅ 1 − f (a ) ⋅ 1 = 0 [given, f (x + t ) = f (x)]
⇒ g (2) = ∫ f (t ) dt = ∫ f (t ) dt + ∫ f (t )dt
0 0 1
∴ φ (a ) is constant.
1 a+ t
≤ f (t ) ≤ 1 for t ∈ [0,1]
Now,
2
⇒ ∫a f (x) dx is independent of a.
rankershalt.com

306 Definite Integration

 x − [x], if [x] is odd.  1


2
 1
6. Given, f (x) =  2 −2   2 −2   2 − 1 3
1 + [x] − x, if [x] is even.  1
∴f′   =
 2
=
 4
= 2 = 2 = 24
f (x) and cos π x both are periodic with period 2 and both  2  2 2  1
2
 5
2 25 25
 1
1 +     1 +   
are even.  2   4   4  16
10 10 
∴ ∫ f (x) cos π x dx = 2 ∫ f (x) cos π x dx 2. PLAN Newton-Leibnitz’s formula
− 10 d  ψ (x )
f(t ) dt = f { ψ ( x )}  ψ ( x ) − f { φ ( x )}  φ ( x )
0 d d
Y dx  ∫ φ ( x )   dx   dx 
x2
Given, F (x) = ∫ f ( t ) dt
0
X ∴ F ′ (x) = 2x f (x)
–10 – 9 –2 –1 0 1 2 9 10
Also, F ′ (x) = f ′ (x)
⇒ 2x f (x) = f ′ (x)
2 f ′ (x)
= 10 ∫ f (x) cos π x dx ⇒ = 2x
f (x)
0 f ′ (x)
1 ⇒ ∫ f (x)
dx = ∫ 2x dx ⇒ In f (x) = x2 + c
Now, ∫ f (x) cos π x dx 2
+ c 2
0 ⇒ f (x) = ex ⇒ f (x) = K ex [K = ec ]
1 1
Now, f (0) = 1
= ∫ (1 − x) cos π x dx = − ∫ u cos π u du
∴ 1=K
0 0 2
2 2 Hence, f (x) = ex
and ∫ f (x) cos π x dx = ∫ (x − 1) cos πx dx 4
F (2) = ∫ et dt = [et ]40 = e4 − 1
0
1 1
1 x
3. Given, y = ∫ |t| dt
= − ∫ u cos π u du 0
0 dy
10 1 ∴ = | x|⋅ 1 − 0 = |x| [by Leibnitz’s rule]
40 dx
∴ ∫ f (x) cos πx dx = − 20 ∫ u cos π u du = π 2 x
− 10 0 Q Tangent to the curve y = ∫ |t| dt , x ∈ R are parallel
0

π2
10 to the line y = 2x
10 −∫10
⇒ f (x) cos π x dx = 4
∴ Slope of both are equal ⇒ x = ± 2
± 2
Points, y=∫ | t | dt = ± 2
0
Topic 3 Estimation, Gamma Function and Equation of tangent is
Derivative of Definite Integration y − 2 = 2 (x − 2) and y + 2 = 2 (x + 2)
x 1
1. Given, ∫ f (t ) dt = x2 + ∫ t 2f (t )dt For x intercept put y = 0, we get
0 x

On differentiating both sides, w.r.t. ‘x’, we get 0 − 2 = 2 (x − 2) and 0 + 2 = 2 (x + 2)


⇒ x=±1
f (x) = 2x + 0 − x2f (x)
x x
 ψ( x )   4. Given ∫ 1 − { f ′ (t )}2 dt = ∫ f (t ) dt , 0 ≤ x ≤ 1
d d d 0 0
Q  ∫ f (t )dt  = f (ψ (x)) ψ (x) − f (φ (x)) φ(x)
 dx φ ( x )  dx dx  Differentiating both sides w.r.t. x by using Leibnitz’s
 
rule, we get
2x
⇒ (1 + x2) f (x) = 2x ⇒ f (x) = 1 − { f ′ (x)}2 = f (x) ⇒ f ′ (x) = ± 1 − { f (x)}2
1 + x2
f ′ (x)
On differentiating w.r.t. ‘x’ we get ⇒ ∫ dx = ± ∫ dx ⇒ sin −1 { f (x)} = ± x + c
1 − { f (x)}2
(1 + x2)(2) − (2x) (0 + 2x)
f ′ (x) = Put x = 0 ⇒ sin −1 { f (0)} = c
(1 + x2)2
⇒ c = sin −1 (0) = 0 [Q f (0) = 0]
2 + 2x2 − 4x2 2 − 2x2
= =
(1 + x2)2 (1 + x2)2
rankershalt.com

Definite Integration 307

∴ f (x) = ± sin x  tm + 1  1
1
( n − 1) t
m+ 1

m + 1  0 ∫ 0
∴ I (m, n ) = (1 + t )n ⋅ − n (1 + t ) ⋅ dt
but f (x) ≥ 0, ∀ x ∈ [0, 1]  m+1
∴ f (x) = sin x 2n n 1
= − ∫ (1 + t )( n − 1) ⋅ tm + 1 dt
As we know that, m+1 m+1 0

sin x < x, ∀ x > 0 2n n


∴ I (m, n ) = − ⋅ I (m + 1, n − 1)
 1 1  1 1 m+1 m+1
∴ sin   < and sin   <
 2 2  3 3 x
9. Given, f (x) = ∫ 2 − t 2 dt ⇒ f ′ (x) = 2 − x2
 1 1  1 1 1
⇒ f   < and f   <
 2 2  3 3 Also, x2 − f ′ (x) = 0

5. Since ∫
1
t 2 f (t ) dt = 1 − sin x, thus to find f (x). ∴ x2 = 2 − x2 ⇒ x4 = 2 − x2
sin x
⇒ x4 + x2 − 2 = 0 ⇒ x = ± 1
On differentiating both sides using Newton Leibnitz
x
formula 10. Given, F (x) = ∫ f (t ) dt
d 1 d 0

dx ∫ sin x
i.e. t 2 f (t ) dt = (1 − sin x) By Leibnitz’s rule,
dx
⇒ {12 f (1)} ⋅ (0) − (sin 2 x) ⋅ f (sin x) ⋅ cos x = − cos x F ′ (x) = f (x) …(i)
1 But F (x ) = x (1 + x) = x + x
2 2 2 3
[given]
⇒ f (sin x) =
sin 2 x 3
 1 ⇒ F (x) = x + x 3/ 2
⇒ F ′ (x) = 1 + x1/ 2
For f   is obtained when sin x = 1 / 3 2
 3 3 1/ 2
⇒ f (x) = F ′ (x) = 1 + x [from Eq. (i)]
 1 2
i.e. f   = ( 3 )2 = 3
 3 3 1/ 2 3
⇒ f (4) = 1 + (4) ⇒ f (4) = 1 + × 2 = 4
t 2
2 5 2 2
6. Here, ∫0 x f (x) dx =
t x 1
∫ 0 f (t )dt = x + ∫ x t f (t ) dt
5 11. Given,
Using Newton Leibnitz's formula, differentiating both
sides, we get On differentiating both sides w.r.t. x, we get
d  d  f (x) 1 = 1 − x f (x) ⋅ 1 ⇒ (1 + x) f (x) = 1
t 2{ f (t 2)}  (t 2) − 0 ⋅ f (0)  (0) = 2t 4 1 1 1
 dt   dt  ⇒ f (x) = ⇒ f (1) = =
1+ x 1+1 2
⇒ t 2f (t 2)2t = 2t 4 ⇒ f (t 2) = t f ( x)
4 2  2 f ( x) 2t ∫4 2t dt
∴ f   =−

putting t = 12. lim ∫ dt = lim
 25 5 5  x→1 4 x−1 x→1 x−1
4 2 [using L’ Hospital’s rule]
⇒ f   = 2 f (x) ⋅ f ′ (x)
 25 5 = lim = 2 f (1) ⋅ f ′ (1)
x→1 1
x 2 + 1 −t 2
7. Given, f (x) = ∫ e dt = 8 f ′ (1) [Q f (1) = 4]
x2

On differentiating both sides using Newton’s Leibnitz’s 13. If f (x) is a continuous function defined on [a , b], then
formula, we get b
m (b − a ) ≤ ∫ f (x) dx ≤ M (b − a )
+ 1 )2   d 2 a
2 d 2 − ( x 2 )2
f ′ (x) = e− ( x  (x + 1) − e  (x )
 dx   dx  where, M and m are maximum and minimum values
2 2 2 2 respectively of f (x) in [a , b].
= e− ( x + 1)
⋅ 2x − e− ( x )
⋅ 2x 2

− ( x 4 + 2x 2 + 1 ) 2x 2 + 1
Here, f (x) = 1 + e−x is continuous in [0, 1].
= 2xe (1 − e ) 2 2

2x 2 + 1 − ( x 4 + 2x 2 + 1 )
Now, 0 < x < 1 ⇒ x2 < x ⇒ ex < ex ⇒ e− x > e− x
[where, e > 1, ∀ x and e > 0, ∀ x] 2 2
Again, 0 < x < 1 ⇒ x2 > 0 ⇒ ex > e0 ⇒ e− x < 1
∴ f ′ (x) > 0 2

which shows 2x < 0 or x < 0 ⇒ x ∈ (−∞ , 0) ∴ e− x < e− x < 1, ∀ x ∈ [0, 1]


2
1 ⇒ 1 + e− x < 1 + e− x < 2, ∀ x ∈ [0, 1]
8. Here, I (m, n ) = ∫ t m(1 + t )n dt reduce into I (m + 1, n − 1) 1 1 2 1
−x
0
⇒ ∫ 0 (1 + e ) dx < ∫ (1 + e− x ) dx < ∫ 2 dx
[we apply integration by parts taking (1 + t ) as first n 0 0

and t m as second function] 1 1 2


⇒ 2 − < ∫ (1 + e− x ) dx < 2
e 0
rankershalt.com

308 Definite Integration

14. g (x) = ∫
sin 2x
sin −1 (t )dt = (cosecx ⋅ cot x + sec2 x − cos x) ⋅ (cos3 x − cos x) ⋅ cos x
sin x
sin 2 x + cos3 x − cos3 x ⋅ sin 2 x 
g′ (x) = 2 cos 2x sin −1 (sin 2x) − cos x sin −1 (sin x) =−  ⋅ cos x ⋅ sin x
2 2

 sin 2 x ⋅ cos 2 x 
 π
g′   = − 2 sin −1 (0) = 0
 2 = − sin 2 x − cos3 x (1 − sin 2 x) = − sin 2 x − cos5 x
π /2 π /2
 π
g′  −  = − 2 sin −1 (0) = 0
 2
∴ ∫0 f (x) dx = − ∫0 (sin 2 x + cos5 x) dx

 n +2
m+ 1 
No option is matching.  π /2 
2 2
15. Here, f (x) + 2x = (1 − x)2 ⋅ sin 2 x + x2 + 2x …(i) Q

∫0 sinm x ⋅ cos n x dx =
m +n+2


where, P : f (x) + 2x = 2 (1 + x)2 2
…(ii)  2 
∴ 2 (1 + x2) = (1 − x)2 sin 2 x + x2 + 2x  3 1 6 1 
 ⋅ ⋅
⇒ (1 − x)2 sin 2 x = x2 − 2x + 2 π /2 2 2 2 2 
⇒ (1 − x)2 sin 2 x = (1 − x)2 + 1
∫0 f (x) dx = − 
2 2
+
7 

 2
 2 
⇒ (1 − x)2 cos 2 x = − 1
which is never possible.  
1 /2 ⋅ π 2 π  π 8  15π + 32
∴ P is false. =− + = −  +  = −  
 2 5 3 1
2⋅ ⋅ ⋅ ⋅ π   4 15  60 
Again, let Q : h (x) = 2 f (x) + 1 − 2x (1 + x)  2 2 2 
where, h (0) = 2 f (0) + 1 − 0 = 1 x ln t
h (1) = 2 f (1) + 1 − 4 = − 3, as h (0) h (1) < 0 18. f (x) = ∫ dt for x > 0 [given]
1 1+ t
⇒ h (x) must have a solution. 1/ x ln t
Now, f (1 / x) = ∫ dt
∴ Q is true. 1 1+ t
16. Here, f (x) = (1 − x)2 ⋅ sin 2 x + x2 ≥ 0, ∀ x. Put t = 1 / u ⇒ dt = (−1 / u 2) du
x  2 (t − 1 )  x ln (1 / u ) (−1 )
and g (x) = ∫  − log t f (t ) dt ∴ f (1 / x) = ∫ ⋅ 2 du
1 t + 1  1 1 + 1 /u u
2 (x − 1)  x ln u x ln t
⇒ g′ (x) =  − log x ⋅ f {
(x) …(i) =∫ du = ∫ dt
 (x + 1 )  + ve 1 u (u + 1) 1 t (1 + t )

For g′ (x) to be increasing or decreasing,  1 x ln t x ln t


2 (x − 1) Now, f (x) + f   = ∫ dt + ∫ 1 t (1 + t ) dt
let φ(x) = − log x  x 1 (1 + t )
(x + 1) x (1 + t ) ln t x ln t 1 1
4 1 − (x − 1)2 =∫ dt = ∫ dt = [(ln t )2]1x = (ln x)2
φ′ (x) = − = 1 t (1 + t ) 1 t 2 2
(x + 1) 2
x x (x + 1)2
Put x = e,
φ′ (x) < 0, for x > 1 ⇒ φ (x) < φ (1) ⇒ φ(x) < 0 …(ii)
 1 1 1
f (e) + f   = (ln e)2 = Hence proved.
From Eqs. (i) and (ii), we get  e 2 2
g′ (x) < 0 for x ∈ (1, ∞ )
19. Let t = b − a and a + b = 4 [given]
∴g (x) is decreasing for x ∈ (1, ∞ ).
⇒ t =4 − a − a
sec x cos x cosec x ⋅ cotx + sec2x ⇒ t = 4 − 2a
17. Given, f (x) = cos 2 x cos 2 x cosec2x ⇒ a =2 −
t
2
1 cos x cos 2 x 2
1 and t = b − (4 − b)
Applying R3 → R3 , ⇒ t = 2b − 4
cos x
t
sec x cos x cosec x ⋅ cot x + sec2 x ⇒ = b −2
2
f (x) = cos x cos 2 x cos 2 x cosec2 x t
⇒ b =2 +
sec x cos x cos x 2
Again, a <2 [given]
Applying R1 → R1 − R3 ⇒ f (x) π
0 0 cosec x ⋅ cot x + sec2x − cos x ⇒ 2 − <2
2
= cos x cos 2 x cos 2 x cosec2x π
⇒ >0 ⇒ t >0
sec x cos x cos x 2
rankershalt.com

Definite Integration 309

a b x
Now, ∫ g (x) dx + ∫ g (x) dx As x → ∞, ∫ f (t ) dt → ∞ for a particular x (say xn ), then
0 0 0
x
∫0
2 − t/ 2 2 + t/ 2
=∫ f (t ) dt = 2 and for this value of x, y = 0
0
g (x) dx + ∫0 g (x) dx
2 − t/ 2 2 + t/ 2
F (x) = ∫ The curve is symmetrical about X-axis.
Let
0
g (x) dx + ∫0 g (x) dx
Thus, we have that there must be some x, such that
1  t 1  t f (xn ) = 2.
For t > 0, F ′ (t ) = − g 2 −  + g 2 + 
2  2 2  2 Thus, y = mx intersects this closed curve for all values of
m.
[using Leibnitz’s rule] x
1  t 1  t 22. Given, f (x) = ∫ [2(t − 1) (t − 2)3 + 3 (t − 1)2(t − 2)2] dt
= g 2 +  − g 2 −  1
2  2 2  2
∴ f ′ (x) = [2 (x − 1) (x − 2)3 + 3 (x − 1)2(x − 2)2] ⋅ 1 − 0
dg
Again, > 0, ∀ x ∈ R [given] = (x − 1) (x − 2)2 [2 (x − 2) + 3 (x − 1)]
dx
= (x − 1) (x − 2)2 (5x − 7)
Now, 2 − t / 2 < 2 + t / 2 ∴ t > 0
+ – +
We get g (2 + t / 2) − g (2 − t / 2) > 0, ∀ t > 0
So, F ′ (t ) > 0, ∀ t > 0 1 7/5

Hence, F (t ) increases with t, therefore F (t ) increases as ∴ f (x) attains maximum at x = 1 and f (x) attains
7
(b − a ) increases. minimum at x = .
1 5
20. Let I n = ∫ ex (x − 1)n dx
0

Put x − 1 = t ⇒ dx = dt
Topic 4 Limits as the Sum
0 t+1 0 n t  (n + 1)1/3 (n + 2)1/3 (2n )1/3 
∴ In = ∫ e ⋅ t ndt = e ∫ t e dt 1. Let p = lim  + +…+ 
−1 −1 n→ ∞  n 4/3
n 4/3
n 4/3 

= e  [t net ] −01 − n ∫ t n − 1et dt


0 n
(n + r )1/3
 −1  = lim
n→ ∞
∑ n 4/3
r =1
= e 0 − (−1)n e−1 − n t n − 1et dt
0
 ∫ −1 
 r
1/3
1 +  n
1/3
0
= (−1)n + 1 − ne ∫ t n − 1et dt  n
n
−1 = lim ∑
n→ ∞
r =1 n 4/3
⇒ I n = (−1)n + 1 − nI n − 1 …(i)
n 1/3
1 x 1 1  r
For n = 1, I1 = ∫ e (x − 1) dx = [e (x
0
x
− 1)]10 − ∫ ex dx
0
= lim
n→ ∞ n ∑ 1 + n 
r =1
= e1 (1 − 1) − e0 (0 − 1) − [ex ] 10 = 1 − (e − 1) = 2 − e Now, as per integration as limit of sum.
Therefore, from Eq. (i), we get r 1
Let = x and = dx [Q n → ∞]
I 2 = (−1)2 + 1 − 2I1 = − 1 − 2(2 − e) = 2e − 5 n n
and I3 = (−1)3 + 1 − 3I 2 = 1 − 3(2e − 5) = 16 − 6e Then, upper limit of integral is 1 and lower limit of
integral is 0.
Hence, n = 3 is the answer.
x 1  1 n
 r 1 
21. Since, f is continuous function and ∫ f (t ) dt → ∞ , So, p = ∫ (1 + x)1/3 dx Q lim ∑ f  n  = ∫0 f (x) dx
0 n→ ∞ n
0  r =1 
as| x|→ ∞. To show that every line y = mx intersects the
x 1
curve y2 + ∫ f (t ) dt = 2 =
3  3 3
(1 + x)4/3 = (24/3 − 1) = (2)4/3 −
3
0
4  4 4 4
0
Y
(0,√2)
A 2. Clearly,
lim  n n n 1
X  + + + ...+ 
O
(Xp,0) n → ∞  n 2 + 12 n 2 + 22 n 2 + 32 5n 
lim  n n n n 
B (0,–√2) =  + + + ....+ 2 
n → ∞  n 2 + 12 n 2 + 22 n 2 + 32 n + (2n )2
At x = 0, y = ± 2 lim 2n n
= ∑
Hence, (0, 2 ), (0, − 2 ) are the point of intersection of n → ∞ r=1 n 2 + r 2
the curve with the Y-axis.
rankershalt.com

310 Definite Integration

lim 2n 1 1 2 dx 1a + 2a + K + n a 1
= ∑ ⋅ =∫ lim =
n→ ∞ r =1  r
2
n 0 1 + x2 n→ ∞ (n + 1) a −1
{(na + 1) + (na + 2) + K + (na + n )} 60
1+  
 n n
 pn
1  r p  ∑ ra
Q lim ∑ f   = ∫ 0 f (x)dx ⇒ lim
r =1
=
1
n→∞ n n
 r =1  n→ ∞ a −1  n (n + 1)  60
= [tan −1 x]20 = tan −1 2 (n + 1) ⋅ n 2a +
 2 
1 a
n
(n + 1) ⋅ (n + 2) K (3n ) n  r
3. Let l = lim   2∑  
 n
n→ ∞  n 2n  r =1 1
1 ⇒ lim a −1
=
n→ ∞
 1 60
 (n + 1) ⋅ (n + 2) ... (n + 2n ) n
1 +  ⋅ (2na + n + 1)
= lim   
n→ ∞  n 2n  n
1 n
1 a
 r 1 1
  n + 1  n + 2  n + 2n   n ⇒ lim 2 ∑  ⋅ lim   =
= lim      K  n→ ∞ n   n→ ∞  n a −1
n→ ∞   n   n   n    r =1   1
1 + 
 1  60
⋅ 2a + 1 + 
 n  n
Taking log on both sides, we get 1 a 1 1
1   1  2 2 n   ⇒ 2∫ (x ) dx ⋅ =
 0 1 ⋅ (2a + 1) 60
log l = lim log 1 +  1 +  ... 1 + 
n→ ∞ n   n  n  n   2 ⋅ [xa+ 1 ] 10 1
⇒ =
⇒ log l = lim
1 (2a + 1) ⋅ (a + 1) 60
n→ ∞ n 2 1
∴ = ⇒ (2a + 1) (a + 1) = 120
  1  2  2n   (2a + 1) (a + 1) 60
log 1 + n  + log 1 + n  + ... + log 1 + n  
  ⇒ 2a 2 + 3a + 1 − 120 = 0 ⇒ 2a 2 + 3a − 119 = 0
2n −17
1  r ⇒ (2a + 17) (a − 7) = 0 ⇒ a = 7,
⇒ log l = lim
n→ ∞ n ∑ log 1 + n  2
r =1
n
n

2
log l = ∫ log (1 + x) dx 5. Given, S n = ∑
k = 0 n + kn + k
2 2
0
2
 1     
⇒ log l = log (1 + x) ⋅ x − ∫ ⋅ x dx  
 1+ x 0 n   n

∑ n  k  k  2
1 1 1 1
= ∑ ⋅  < lim
2 x+ 1 −1 n  k k2  n → ∞
⇒ log l = [log (1 + x) ⋅ x]20 − ∫ dx k=0
1 + + 2 k=0
1 + +   
0 1+ x  n n   n  n 
2 1 
⇒ log l = 2 ⋅ log 3 − ∫ 1 −  dx =∫
1 1
dx
0  1 + x 0 1 + x + x2
⇒ log l = 2 ⋅ log 3 − [x − log 1 + x ]20 1
⇒ log l = 2 ⋅ log 3 − [2 − log 3]  2  2  1  
= tan −1  x +  
⇒ log l = 3 ⋅ log 3 − 2  3  3 2   0
⇒ log l = log 27 − 2
2  π π π π
27 = ⋅ −  = i.e. S n <
∴ l = elog 27 − 2 = 27 ⋅ e− 2 = 2 3  3 6 3 3 3 3
e
π
4. PLAN Converting Infinite series into definite Integral Similarly, Tn >
3 3
h( n)
i.e. lim
n→ ∞ n  1 1 1  5n 1
6n  ∑
h ( n) 6. lim  + +K+ =
1  r n→ ∞  n + 1 n+2 n+r
lim
n→ ∞ n
∑ f  n  = ∫ f(x )dx r =1
r = g ( n) 5n
1 1
lim
g ( n) = lim
n→ ∞ n ∑ r
n→ ∞ n r =1 1 + 
r  n
where, is replaced with x.
n 5 dx
=∫ = [log (1 + x)] 50 = log 6 − log 1 = log 6
Σ is replaced with integral. 0 1+ x

Here, Download Chapter Test


http://tinyurl.com/y6h4j36z or
rankershalt.com

13
Area
Topic 1 Area Based on Geometrical Figures
Without Using Integration
Objective Questions I (Only one correct option)
1. If the area enclosed between the curves y = kx2 and 7. The triangle formed by the tangent to the curve
x = ky2, (k > 0), is 1 square unit. Then, k is f (x) = x2 + bx − b at the point (1,1) and the coordinate axes,
(2019 Main, 10 Jan I) lies in the first quadrant. If its area is 2 sq units, then the
1 2 3 value of b is (2001, 2M)
(a) 3 (b) (c) (d)
3 3 2 (a) – 1 (b) 3 (c) – 3 (d) 1
2. The area (in sq units) of the region {(x, y) : y2 ≥ 2x
and x2 + y2 ≤ 4x, x ≥ 0, y ≥ 0} is (2016 Main) Objective Questions II
(a) π −
4
(b) π −
8 (One or more than one correct option)
3 3
4 2 π 2 2
8. Let P and Q be distinct points on the parabola y2 = 2x such
(c) π − (d) − that a circle with PQ as diameter passes through the vertex
3 2 3
O of the parabola. If P lies in the first quadrant and the
3. The common tangents to the circle x2 + y2 = 2 and area of ∆OPQ is 3 2, then which of the following is/are the
the parabola y2 = 8x touch the circle at the points coordinates of P ? (2015 Adv.)
P,Q and the parabola at the points R, S. Then, the
(c)  ,
1 1 
area (in sq units) of the quadrilateral PQRS is (a) (4 , 2 2 ) (b) (9 , 3 2 )  (d) (1, 2 )
4 2
(2014 Adv.)
(a) 3 (b) 6 (c) 9 (d)15
Numerical Value
4. The area of the equilateral triangle, in which three
coins of radius 1 cm are placed, as shown in the 9. A farmer F1 has a land in the shape of a triangle with
figure, is vertices at P(0, 0), Q(1, 1) and R(2, 0). From this land, a
neighbouring farmer F2 takes away the region which lies
between the sides PQ and a curve of the form y = xn (n > 1).
If the area of the region taken away by the farmer F2 is
exactly 30% of the area of ∆PQR, then the value of n is
.................... . (2018 Adv.)

(2005, 1M)
(a) (6 + 4 3 ) sq cm (b) (4 3 − 6) sq cm Fill in the Blanks
(c) (7 + 4 3 ) sq cm (d) 4 3 sq cm 10. The area of the triangle formed by the positive X-axis and
5. The area of the quadrilateral formed by the the normal and the tangent to the circle x2 + y2 = 4 at (1, 3 )
tangents at the end points of latusrectum to the is … . (1989, 2M)
x2 y2 11. The area enclosed within the curve|x| + | y| = 1 is ....... .
ellipse + = 1, is
9 5 (2003, 1M) (1981, 2M)
(a) 27/4 sq units (b) 9 sq units Analytical & Descriptive Question
(c) 27/2 sq units (d) 27 sq units 1
12. Let O (0, 0), A (2, 0) and B (1, ) be the vertices of a
6. The area (in sq units) bounded by the curves 3
y = | x | − 1 and y = − | x | + 1 is (2002, 2M) triangle. Let R be the region consisting of all those points P
(a) 1 (b) 2 (c) 2 2 (d) 4 inside ∆ OAB which satisfy d (P , OA ) ≥ min
rankershalt.com

312 Area

{ d (P , OB), d (P , AB)}, where d denotes the distance 14. The area of the region bounded by the curve y = f (x),
from the point to the corresponding line. Sketch the the X-axis and the lines x = a and x = b, where
region R and find its area. (1997C, 5M) − ∞ < a < b < − 2, is
b x
(a) ∫ dx + bf (b) − af (a )
Passage Based Questions a 3[{f (x)}2 − 1]
b x
Consider the functions defined implicity by the
equation y3 − 3 y + x = 0 on various intervals in the real
(b) − ∫a 3[{f (x)}2 − 1] dx + bf (b) − af (a)
line. If x ∈ (−∞ , − 2) ∪ (2, ∞ ), the equation implicitly (c) ∫
b x
dx − bf (b) + af (a )
defines a unique real-valued differentiable function a 3[{f (x)}2 − 1]
y = f (x). If x ∈ (− 2, 2), the equation implicitly defines a b x
unique real-valued differentiable function y = g (x), (d) − ∫a 3[{f (x)}2 − 1] dx − bf (b) + af (a)
satisfying g (0) = 0. (2008, M)
1
13. If f (− 10 2 ) = 2 2 , then f′′ (− 10 2 ) is equal to 15. ∫− 1 g ′ (x) dx is equal to
4 2 4 2 4 2 4 2 (a) 2 g(− 1) (c) − 2 g(1)
(a) (b) − (c) (d) − (b) 0 (d) 2 g(1)
73 32 73 32 73 3 73 3

Topic 2 Area Using Integration


Objective Questions I (Only one correct option) 8. The area (in sq units) of the region
1. If the area (in sq units) bounded by the parabola A = {(x, y) ∈ R × R|0 ≤ x ≤ 3,
1 0 ≤ y ≤ 4, y ≤ x2 + 3x} is (2019 Main, 8 April I)
y2 = 4λx and the line y = λx, λ > 0, is , then λ is equal to 53 59 26
9 (a) (b) 8 (c) (d)
(2019 Main, 12 April II) 6 6 3
(a) 2 6 (b) 48 (c) 24 (d) 4 3 9. The area (in sq units) of the region bounded by the
2. If the area (in sq units) of the region parabola, y = x2 + 2 and the lines, y = x + 1, x = 0 and
{(x, y): y2 ≤ 4x, x + y ≤ 1, x ≥ 0, y ≥ 0} is a 2 + b, then x = 3, is (2019 Main, 12 Jan I)
a − b is equal to (2019 Main, 12 April I) (a)
15
(b)
17
(c)
21
(d)
15
10 8 2 2 4 2 4
(a) (b) 6 (c) (d) −
3 3 3 10. The area (in sq units) in the first quadrant bounded by
3. The area (in sq units) of the region bounded by the the parabola, y = x2 + 1, the tangent to it at the point
curves y = 2 and y = | x + 1|, in the first quadrant is
x
(2, 5) and the coordinate axes is (2019 Main, 11 Jan II)
(2019 Main, 10 April II) 14 187 8 37
3 3 1 3 1 (a) (b) (c) (d)
(a) (b) log e 2 + (c) (d) − 3 24 3 24
2 2 2 2 log e 2
11. The area (in sq units) of the region bounded by the curve
4. The area (in sq units) of the region x2 = 4 y and the straight line x = 4 y − 2 is
 y2  (2019 Main, 11 Jan I)
A = (x, y) : ≤ x ≤ y + 4 is 7 9 5 3
2 (a) (b) (c) (d)
  (2019 Main, 9 April II) 8 8 4 4
53
(a) 30 (b) (c) 16 (d) 18 12. The area of the region A = {(x, y); 0 ≤ y ≤ x| x| + 1 and
3
− 1 ≤ x ≤ 1} in sq. units, is (2019 Main, 9 Jan II)
5. The area (in sq units) of the region 4 1 2
A = {(x, y) : x2 ≤ y ≤ x + 2} is (2019 Main, 9 April I) (a) 2 (b) (c) (d)
3 3 3
13 9 31 10
(a) (b) (c) (d) 13. The area (in sq units) bounded by the parabola
6 2 6 3
y = x2 − 1, the tangent at the point (2, 3) to it and the
6. Let S (α ) = {(x, y) : y ≤ x, 0 ≤ x ≤ α} and A(α ) is area of the
2
Y -axis is (2019 Main, 9 Jan I)
region S(α ). If for λ, 0 < λ < 4, A (λ ) : A (4) = 2 : 5, then λ 8 56 32 14
(a) (b) (c) (d)
equals (2019 Main, 8 April II) 3 3 3 3
1 1 1 1

(a) 2 
4 3
(b) 4 
2 3
(c) 4 
4 3
(d) 2 
2 3 14. Let g (x) = cos x2, f (x) = x and α , β (α < β) be the roots of
 25   5  25   5 the quadratic equation 18x2 − 9πx + π 2 = 0. Then, the
area (in sq units) bounded by the curve y = ( gof )(x) and
7. The tangent to the parabola y = 4x at the point where it
2
the lines x = α, x = β and y = 0, is (2018 Main)
intersects the circle x2 + y2 = 5 in the first quadrant, 1 1
passes through the point (a) ( 3 − 1) (b) ( 3 + 1)
2 2
(a)  ,
3
(b)  ,
7
(c)  − ,
4
(d)  − ,
1 3 1 1 1 1 1
    (c) ( 3 − 2 ) (d) ( 2 − 1)
4 4 4 4  3 3  4 2 2 2
rankershalt.com

Area 313

15. The area (in sq units) of the region 25. The area enclosed between the curves y = ax2 and
{(x, y) : x ≥ 0, x + y ≤ 3, x2 ≤ 4 y and y ≤ 1+ x } is x = ay2 (a > 0) is 1 sq unit. Then, the value of a is
(2017 Main) (2004, 1M)
59 3 7 5
(a) (b) (c) (d) 1 1 1
12 2 3 2 (a) (b) (c) 1 (d)
3 2 3
16. Area of region {(x, y)} ∈ R2 : y ≥ |x + 3|,
the
26. The area bounded by the curves y = f (x),the X-axis and
5 y ≤ (x + 9) ≤ 15} is equal to (2016 Adv)
the ordinates x = 1 and x = b is (b − 1) sin (3b + 4). Then,
1 4 3 5
(a) (b) (c) (d) f (x) is equal to (1982, 2M)
6 3 2 3
(a) (x − 1) cos (3x + 4)
17. The area (in sq units) of region described by (x, y) y2 ≤ 2x (b) 8sin (3x + 4)
and y ≥ 4x − 1 is (2015 JEE Main) (c) sin (3x + 4) + 3(x − 1) cos (3x + 4)
7 5 15 9 (d) None of the above
(a) (b) (c) (d)
32 64 64 32 27. The slope of tanget to a curve y = f (x) at [x, f (x)] is 2x + 1.
18. The area (in sq units) of the region described by If the curve passes through the point (1, 2), then the
A = {(x, y) : x2 + y2 ≤ 1 and y2 ≤ 1 − x } is (2014 Main) area bounded by the curve, the X-axis and the line x = 1
π 4 π 4 π 2 π 2 is
(a) + (b) − (c) − (d) +
2 3 2 3 2 3 2 3 3 4 5 1
(a) (b) (c) (d)
2 3 6 12
19. The area enclosed by the curves y = sin x + cos x and
 π
y = | cos x − sin x|over the interval 0, is (2014 Adv.) Objective Questions II
 2 
(One or more than one correct option)
(a) 4( 2 − 1) (b) 2 2 ( 2 − 1)
(c) 2 ( 2 + 1) (d) 2 2 ( 2 + 1) 28. If the line x = α divides the area of region
20. The area (in sq units) bounded by the curves R = {(x, y) ∈ R : x ≤ y ≤ x, 0 ≤ x ≤ 1} into two equal
2 3

y = x, 2 y − x + 3 = 0, X-axis and lying in the first parts, then (2017 Adv.)


quadrant, is (2013 Main, 03) (a) 2α 4 − 4α 2 + 1 = 0 (b) α 4 + 4α 2 − 1 = 0
27 1 1
(a) 9 (b) 6 (c) 18 (d) (c) < α < 1 (d) 0 < α ≤
4 2 2
21. Letf : [−1, 2] → [0, ∞ ) be a continuous function such that 29. If S be the area of the region enclosed by
2
2 y = e− x , y = 0, x = 0 and x = 1. Then,
f (x) = f (1 − x), ∀x ∈ [−1, 2]. If R1 = ∫ xf (x) dx and R2 are (2012)
−1 1 1
(a) S ≥ (b) S ≥ 1 −
the area of the region bounded by y = f (x), x = − 1, x = 2 e e
and the X-axis. Then, (2011) 1 1 1 1  1 
(c) S ≤  1 +  (d) S ≤ + 1 − 
(a) R1 = 2R2 (b) R1 = 3R2 4 e 2 e  2
(c) 2R1 = R2 (d) 3R1 = R2
30. Area of the region bounded by the curve y = ex and lines
22. If the straight line x = b divide the area enclosed by x = 0 and y = e is (2009)
y = (1 − x)2, y = 0 and x = 0 into two parts R1 (0 ≤ x ≤ b) e
1
and R2(b ≤ x ≤ 1) such that R1 − R2 = . Then, b equals
(a) e − 1 (b) ∫1 ln (e + 1 − y) dy
1 e
4 (c) e − ∫ e dx x
(d) ∫1 ln y dy
3 1 1 1 0
(a) (b) (c) (d) (2011)
4 2 3 4 31. For which of the following values of m, is the area of the
23. The area of the region between the curves region bounded by the curve y = x − x2 and the line
9
1 + sin x 1 − sin x y = mx equals ? (1999, 3M)
y= and y = and bounded by the 2
cos x cos x
(a) – 4 (b) –2 (c) 2 (d) 4
π
lines x = 0 and x = is
4 (2008, 3M) Analytical & Descriptive Questions
2 −1 t 2 −1 4t
(a) ∫ dt (b) ∫ dt 4a 2 4a 1  f (−1)  2
3a + 3a 
32. If 4b2 4b 1  f (1)  = 3b2 + 3b ,
0 0
(1 + t 2 ) 1 − t 2 (1 + t 2 ) 1 − t 2
2+1 2+1    2 
 4c2 4c 1  f (2)  3c + 3c 
4t t
(c) ∫ dt (d) ∫ dt
0 0  
(1 + t 2 ) 1 − t 2 (1 + t 2 ) 1 − t 2
f (x) is a quadratic function and its maximum value
24. The area bounded by the curves y = (x − 1)2, y = (x + 1)2 occurs at a point V. A is a point of intersection of y = f (x)
1 with X-axis and point B is such that chord AB subtends
and y = is (2005, 1M)
4 a right angle at V. Find the area enclosed by f (x) and
1 2 1 1 chord AB. (2005, 5M)
(a) sq unit (b) sq unit (c) sq unit (d) sq unit
3 3 4 5
rankershalt.com

314 Area

33. Find the area bounded by the curves x2 = y, x2 = − y 1 1


and deduce < An < . (1996, 3M)
and y = 4x − 3.
2
(2005, 4M) 2n + 2 2n − 2
34. A curve passes through (2,0) and the slope of tangent at 42. Consider a square with vertices at (1,1), (–1, 1), (–1, –1)
(x + 1)2 + y − 3 and (1, –1). If S is the region consisting of all points
point P (x, y) equals .
(x + 1) inside the square which are nearer to the origin than to
any edge. Then, sketch the region S and find its area.
Find the equation of the curve and area enclosed by the (1995, 5M)
curve and the X-axis in the fourth quadrant. (2004, 5M)
43. In what ratio, does the X-axis divide the area of the
35. Find the area of the region bounded by the curves region bounded by the parabolas y = 4x − x2 and
y = x2, y = | 2 − x2| and y = 2 , y = x2 − x ? (1994, 5M)
which lies to the right of the line x = 1. (2002, 5M)
44. Sketch the region bounded by the curves y = x2 and
36. Let b ≠ 0 and for j = 0,1,2..., n. If S j is the area of the
y = 2 / (1 + x2). Find its area. (1992, 4M)
region bounded by the Y-axis and the curve
jπ ( j + 1)π 45. Sketch the curves and identify the region bounded by
xe = sin by,
ay
≤ y≤ . Then, show that x = 1 /2, x = 2, y = log x and y = 2x . Find the area of this
b (b)
region. (1991, 4M)
S 0 , S1 , S 2,... , S n are in geometric progression. Also,
find their sum for a = − 1 and b = π . (2001, 5M) 46. Compute the area of the region bounded by the curves
37. If f (x) is a continuous function given by log x
y = ex log x and y = , where log e = 1. (1990, 4M)
ex
f (x) =  2 |x| ≤ 1
2x,
x + ax + b, |x| > 1 47. Find all maxima and minima of the function
y = x (x − 1)2, 0 ≤ x ≤ 2 .
Then, find the area of the region in the third quadrant
bounded by the curves x = −2 y2 and y = f (x) lying on the Also, determine the area bounded by the curve
left on the line 8x + 1 = 0. (1999, 5M) y = x (x − 1)2, the Y-axis and the line x = 2 . (1989, 5M)

38. Let C1 and C 2 be the graphs of functions y = x2 and 48. Find the area of the region bounded by the curve
y = 2x, 0 ≤ x ≤ 1, respectively. Let C3 be the graph of a C: y = tan x, tangent drawn to C at x = π / 4 and the
function y = f (x), 0 ≤ x ≤ 1, f (0) = 0. For a point P on C1, X-axis. (1988, 5M)
let the lines through P, parallel to the axes, meet C 2 and 49. Find the area bounded by the curves
C3 at Q and R respectively (see figure). If for every x2 + y2 = 25, 4 y = |4 − x2|and x = 0 above the X-axis.
position of P(on C1) the areas of the shaded regions OPQ (1987, 6M)
and ORP are equal, then determine f (x). (1998, 8M) 50. Find the area bounded by the curves x2 + y2 = 4,
Y
x2 = − 2 y and x = y. (1986, 5M)
(0,1) (1/2,1) (1,1)
C2 C1 51. Sketch the region bounded by the curves y = 5 − x2 and
y = |x − 1|and find its area. ( 1985, 5M)
Q P 52. Find the area of the region bounded by the X-axis
π π
and the curves defined by y = tan x, − ≤x≤ and
X 3 3
(0,0) O C (1,0) π π
3 y = cot x, ≤x≤ . (1984, 4M)
R 6 3
39. Letf (x) = max { x2, (1 − x)2, 2x (1 − x)}, where 0 ≤ x ≤ 1. 53. Find the area bounded by the X-axis, part of the curve
 8
Determine the area of the region bounded by the curves y = 1 + 2 and the ordinates at x = 2 and x = 4. If the
y = f (x), X-axis, x = 0 and x = 1. (1997, 5M)
 x 
ordinate at x = a divides the area into two equal parts,
40. Find all the possible values of b > 0, so that the area of then find a. (1983, 3M)
the bounded region enclosed between the parabolas
x2 54. Find the area bounded by the curve x2 = 4 y and the
y = x − bx2 and y = is maximum. (1997C, 5M) straight line x = 4 y − 2.
b
et + e– t et – e– t
41. If An is the area bounded by the curve y = (tan x) and n
55. For any real t, x = ,y= is a point on the
π 2 2
the lines x = 0, y = 0 and x = . hyperbola x2 – y2 = 1. Find the area bounded by this
4
1 hrperbola and the lines joining its centre to the points
Then, prove that for n > 2 , An + An + 2 = corresponding to t1 and – t1. (1982, 3M)
n+1
rankershalt.com

Area 315

Answers
Topic 1  761
37.   sq units 38. f ( x ) = x 3 − x 2, 0 ≤ x ≤ 1
1. (b) 2. (b) 3. (d) 4. (a)  192
5. (d) 6. (b) 7. (c) 8. (a,d) 17 1 
39. sq unit 40. b = 1 42. (16 2 − 20 ) sq units
9. (4) 10. 2 3 sq units 27 3 

12. (2 − 3 ) sq unit  2
11. 2 sq units 13. (b) 43. 121 : 4 44.  π −  sq units
 3
14. (a) 15. (d)
4 − 2 5 3  e 2 − 5
Topic 2 45.  − log 2 +  sq units 46.   sq units
 log 2 2 2  4e 
1. (c) 2. (b) 3. (d) 4. (d)
 4 10 
5. (b) 6. (c) 7. (b) 8. (c) 47. y max = , y min = 0, sq units
 27 3 
9. (a) 10. (d) 11. (b) 12. (a)
13. (a) 14. (a)  1    4 
48.  log 2 −  sq units 49. 4 + 25 sin −1    sq units
15. (d) 16. (c) 17. (d) 18.(a)   4     5 
19. (b) 20. (a) 21. (c) 22.(b) 1   5π 1
50.  − π  sq units 51.  −  sq units
23. (b) 24. (a) 25. (a) 26.(c) 3   4 2
27. (c) 28. (a, c) 29. (b, d) 30.(b, c, d) 1  9
125 1 52.  loge 3 sq units 53. 2 2 54. sq units
31. (b, d) 32. sq units 33. sq unit 2  8
4 3 3
34. y = x – 2 x, sq units
2 e 2t1 − e −2t1 1 2t1
3 55. − (e − e −2t1 − 4t1 )
4 4
 20 − 12 2   π (1 + e ) (en + 1 − 1 ) 
35.   sq units 36.  ⋅
 3   (1 + π )
2
e − 1 

Hints & Solutions


Topic 1 Area Based on Geometrical Figures On substituting y2 = 2x in Eq. (ii), we get
Without Using Integration x2 + 2x = 4x ⇒ x2 = 2x ⇒ x = 0 or x = 2
1. We know that, area of region bounded by the parabolas ⇒ y = 0 or y = ± 2 [using Eq. (i)]
16 Now, the required area is the area of shaded region, i.e.
x2 = 4ay and y2 = 4bx is (ab) sq units.
3 Y
On comparing y = kx2 and x = ky2 with above equations,
1 1
we get 4a = and 4b = A (2, 2)
x 2 + y 2 = 4x
k k
1 1 X′ X
⇒ a= and b = (0, 0) B (2,0)
4k 4k y 2 = 2x
∴ Area enclosed between y = kx2 and x = ky2 is
16  1   1  1 Y′
   = 2
    Area of a circle 2
3 4k 4k 3k Required area = − ∫ 2x dx
1 4 0
⇒ = 1 [given, area = 1 sq.unit]  x3/ 2 
2
3k2 π (2)2 2
= − 2 ∫ x1/ 2dx = π − 2  
1 1 4 0
 3 /2  0
⇒ k2 = ⇒ k=±
3 3 2 2  8
=π− [2 2 − 0] =  π −  sq unit
1 3  3
⇒ k= [Q k > 0]
3 3. PLAN (i) y = mx + a / m is an equation of tangent to the parabola
y 2 = 4ax .
2. Given equations of curves are y2 = 2x (ii) A line is a tangent to circle, if distance of line from centre is
which is a parabola with vertex (0, 0) and axis parallel equal to the radius of circle.
to X-axis. . ..(i) (iii) Equation of chord drawn from exterior point ( x 1, y1 ) to a
And x2 + y2 = 4x circle/parabola is given by T = 0.
which is a circle with centre (2, 0) and radius = 2 ...(ii) 1
(iv) Area of trapezium = (Sum of parallel sides)
2
rankershalt.com

316 Area
2 By symmetry, the quadrilateral is a rhombus.
Let equation of tangent to parabola be y = mx +
m Y
It also touches the circle x2 + y2 = 2. A

 2  L(ae, √ b2(1 – e2))


∴  = 2
m 1 + m 
2
X′ X
D O B
⇒ m4 + m2 = 2 ⇒ m4 + m2 − 2 = 0 L′
⇒ (m2 − 1) (m2 + 2) = 0
C
⇒ m = ± 1,m2 = − 2 [rejected m2 = − 2]
Y′
So, tangents are y = x + 2, y = − x − 2.
They, intersect at (−2, 0). So, area is four times the area of the right angled triangle
formed by the tangent and axes in the Ist quadrant.
R
Y  5
∴ Equation of tangent at 2,  is
P  3
T(–2,0)
2 5 y x y
X′
O X x+ ⋅ =1 ⇒ + =1
9 3 5 9 /2 3
Q ∴ Area of quadrilateral ABCD
Y′ =4 [area of ∆ AOB]
S
1 9 
Equation of chord PQ is −2x = 2 ⇒ x = − 1 = 4  ⋅ ⋅ 3 = 27 sq units
2 2 
Equation of chord RS is O = 4(x − 2) ⇒ x = 2
6. The region is clearly square with vertices at the points
∴ Coordinates of P, Q, R, S are
(1, 0), (0, 1), (– 1, 0) and (0, – 1).
P (−1, 1), Q (−1, − 1), R(2, 4), S (2, − 4) Y
(2 + 8) × 3
∴ Area of quadrilateral = = 15 sq units
2
(0,1) y = –IxI + 1
4. Since, tangents drawn from external points to the circle y = IxI –1
subtends equal angle at the centre. X′ X
A
(1,0)
(–1,0)
(0,–1)

O3 Y′
∴ Area of square = 2 × 2 = 2 sq units

O1 O2 7. Let y = f (x) = x2 + bx − b
1cm 1cm Y

B 30° 30° C B
3 cm D 2 cm E 3 cm
(1,1)
∴ ∠ O1BD = 30° P
OD
In ∆O1BD, tan 30° = 1 ⇒ BD = 3 cm O A
X
BD
Also, DE = O1O2 = 2 cm and EC = 3 cm The equation of the tangent at P (1, 1)
Now, BC = BD + DE + EC = 2 + 2 3 to the curve 2 y = 2x2 + 2bx − 2b is
y + 1 = 2x ⋅ 1 + b (x + 1) − 2b
3 3
⇒ Area of ∆ABC = (BC )2 = ⋅ 4 (1 + 3 )2 ⇒ y = (2 + b) x − (1 + b)
4 4
Its meet the coordinate axes at
= (6 + 4 3 ) sq cm
1+ b
x2 y2 xA = and yB = − (1 + b)
5. Given, + =1 2+ b
9 5 1
To find tangents at the end points of latusrectum, we ∴Area of ∆ OAB = OA × OB
2
find ae.
1 (1 + b)2
i.e. ae = a 2 − b2 = 4 = 2 =− × =2 [given]
2 (2 + b)
 4 5 ⇒ (1 + b)2 + 4(2 + b) = 0 ⇒ b2 + 6b + 9 = 0
and b2(1 − e2) = 5 1 −  =
 9 3 ⇒ (b + 3)2 = 0 ⇒ b = − 3
rankershalt.com

Area 317
8. Since, ∠ POQ = 90° Thus, area of ∆ formed by (0, 0) (1, 3 ) and (4, 0)
Y 0 0 1
t 12 1 1
P 2 , t1 = 1 3 1 = |(0 − 4 3 )|= 2 3 sq units
2 2
4 0 1
11. The area formed by| x| + | y| = 1 is square shown as below
X′ X :
O
Y
(0, 0)

t 22
Q 2 , t2 −x + y = 1 x+y=1
Y′
X' X
t1 − 0 t2 − 0 −1 O 1
⇒ ⋅ 2 = − 1 ⇒ t1t2 = − 4 …(i)
t12 t x+y=1 x−y=1
−0 2 −0
2 2
Q ar (∆OPQ ) = 3 2 Y'
0 0 1 ∴ Area of square = ( 2 )2 = 2 sq units
1 2 1  t12t2 t1t22
∴ t1 / 2 t1 1 = ± 3 2 ⇒  −  =±3 2
2 2 2 2 2  12. Let the coordinates of P be (x, y) .
t2 / 2 t2 1
1 4
⇒ ( −4t1 + 4t2) = ± 3 2 ⇒ t1 + = 3 2 [Q t1 > 0 for P]
4 t1
⇒ t12 − 3 2t 1 + 4 = 0 ⇒ (t1 − 2 2 ) (t1 − 2 ) = 0
⇒ t1 = 2 or 2 2
∴ P (1, 2 ) or P (4, 2 2 )
9. We have, y = xn , n > 1
Q P ( 0, 0 ) Q (1, 1) and R( 2, 0 ) are vertices of ∆ PQR.
y Equation of line OA be y = 0.
Q(1,1) Equation of line OB be 3 y = x.
Equation of line AB be 3 y = 2 − x.
x

d (P , OA ) = Distance of P from line OA = y


y=

F2 F1
y = xn | 3 y − x|
x′ x d (P , OB) = Distance of P from line OB =
P(0,0) (1,0) R(2,0) 2
| 3 y + x − 2|
d (P , AB) = Distance of P from lineAB =
2
Given, d (P , OA ) ≤ min { d (P , OB), d (P , AB)}
y′  | 3 y − x| | 3 y + x − 2|
y ≤ min  , 
∴ Area of shaded region = 30% of area of ∆ PQR  2 2 
1
⇒ ∫ ( x − xn ) dx =
30 1
× × 2×1 | 3 y − x| | 3 y + x − 2|
0
⇒ y≤ and y ≤
100 2 2 2
1
 x2 xn + 1  3 1 1  3 | 3 y − x|
⇒ − Case I When y ≤ [since, 3 y − x < 0]
 = ⇒  −  = 2
2 n + 1 0 10  2 n + 1 10
x − 3y
1 1 3 2 1 y≤ ⇒ (2 + 3 ) y ≤ x ⇒ y ≤ x tan 15°
⇒ = − = = ⇒ n + 1= 5 ⇒ n = 4 2
n + 1 2 10 10 5 | 3 y + x − 2|
Case II When y ≤ ,
2
10. Equation of tangent at the point (1, 3 ) to the curve
2 y ≤ 2 − x − 3 y [since, 3 y + x − 2 < 0]
x2 + y2 = 4 is x + 3y = 4
⇒ (2 + 3 ) y ≤ 2 − x ⇒ y ≤ tan 15°⋅ (2 − x)
whose X-axis intercept (4, 0).
Y
Y
B (1, 1/ 3)
P (1,√3)

X′ P
X
(0,0) O A (4,0)
A
X′ X
O (0, 0) C (2, 0)
(1, 0)

Y′ Y′
rankershalt.com

318 Area

From above discussion, P moves inside the triangle as Topic 2 Area Using Integration
shown below :
⇒ Area of shaded region 1. Given, equation of curves are
= Area of ∆OQA y2 = 4λx …(i)
1 and y = λx …(ii)
= (Base) × (Height)
2 λ> 0
1
= (2) (tan 15° ) = tan 15° = (2 − 3 ) sq unit Area bounded by above two curve is, as per figure
2
Y
13. Given, y3 − 3 y + x = 0 A y2=4λx
dy dy
⇒ 3 y2 −3 + 1 =0 …(i)
dx dx O
X
2 dy 2 2
 dy d 2y
⇒ 3 y  2 + 6 y   − 3 =0 …(ii)
 dx   dx dx2
y=λx
At x = − 10 2 , y = 2 2
On substituting in Eq. (i) we get
dy dy the intersection point A we will get on the solving Eqs. (i)
3(2 2 )2 ⋅ − 3⋅ + 1 =0 and (ii), we get
dx dx
λ2x2 = 4λx
dy 1
⇒ =− ⇒
4
x = , so y = 4.
dx 21
λ
Again, substituting in Eq. (ii), we get
A  , 4
4
So,
d 2y  1 d 2y
2
λ 
3(2 2 )2 + 6 (2 2 ) ⋅  −  − 3 ⋅ 2 = 0
dx 2  21 dx Now, required area is
2 4/ λ
d y 12 2
⇒ 21 ⋅ =− = ∫ (2 λx − λx) dx
dx2 (21)2 0
4/ λ
d 2y − 12 2 − 4 2  
⇒ = = 3 2  x3 / 2   x2 
4/ λ
dx2 (21)3 7 ⋅3 =2 λ  −λ 
3 
b b    2 0
14. Required area = ∫ y dx = ∫ f (x) dx  2 0
a a 2
4 4 4 λ  4
= [ f (x) ⋅ x]ba
b
− ∫ f ′ (x)x dx = λ −  
a 3 λ λ 2  λ
b 32 8 32 − 24 8
= bf (b) − af (a ) − ∫ f ′ (x)x dx = − = =
a 3λ λ 3λ 3λ
b xdx 1
= bf (b) − af (a ) + ∫a 3[{ f (x)}2 − 1] It is given that area =
9
8 1
 dy −1 −1  ⇒ =
3λ 9
Q f ′ (x) = dx = = 
 3 ( y2
− 1 ) 3 [{ f (x )} 2
− 1 ]  ⇒ λ = 24

15. Let I = ∫
1
g′ (x) dx = [ g (x)]1− 1 = g (1) − g (− 1) 2. Given region is {(x, y) : y2 ≤ 4x, x + y ≤ 1, x ≥ 0, y ≥ 0}
−1
B(0,1)
Since, y3 − 3 y + x = 0 …(i)
and y = g (x) P
y2=4x
∴ { g (x)}3 − 3 g (x) + x = 0 [from Eq. (i)] x+y=1
At x = 1, { g (1)}3 − 3 g (1) + 1 = 0 …(ii)
X
At x = − 1, { g (− 1)}3 − 3 g (− 1) − 1 = 0 …(iii) O A(1,0)
On adding Eqs. (i) and (ii), we get Now, for point P, put value of y = 1 − x to y2 = 4x, we get
{ g (1)}3 + { g (− 1)}3 − 3 { g (1) + g (− 1)} = 0 (1 − x)2 = 4x ⇒ x2 + 1 − 2x = 4x
⇒ [ g (1) + g (− 1)][{ g (1)} + { g (− 1)} − g (1) g (− 1) − 3] = 0
2 2
⇒ x2 − 6x + 1 = 0
⇒ g (1) + g (− 1) = 0
6 ± 36 − 4
⇒ g (1) = − g ( − 1) ⇒ x=
2
∴ I = g (1) − g (− 1)
= 3 ± 2 2.
= g (1) − { − g (1)} = 2 g (1)
rankershalt.com

Area 319

Since, x-coordinate of P less than x-coordinate of point ⇒ y2 = 2x …(i)


A(1, 0). and x = y + 4 ⇒ y = x − 4 …(ii)
∴ x=3 −2 2 Graphical representation of A is
Now, required area Y
y2
3 −2 2 1 =x
=∫ 2 x dx + ∫3 − 2 2 (1 − x) dx Q 2
0

3 −2 2 1
x3/ 2  x2  X' X
=2 + x −  O
3 /2 0  2 3 − 2 2
P
(3 − 2 2 ) 2
(3 − 2 2 ) 3/ 2 +  1 −  − (3 − 2 2 ) +

4
4 1
=

y+
 2

x=
3 2 Y'
4 1 1
= [( 2 − 1)2]3/ 2 + − 3 + 2 2 + (9 + 8 − 12 2 ) On substituting y = x − 4 from Eq. (ii) to Eq. (i), we get
3 2 2
(x − 4)2 = 2x
4 5 17 ⇒ x − 8x + 16 = 2x
2
= ( 2 − 1) − + 2 2 +
3
−6 2
3 2 2 ⇒ x2 − 10x + 16 = 0
4 ⇒ (x − 2)(x − 8) = 0
= (2 2 − 3(2) + 3( 2 ) − 1) − 4 2 + 6
3 ⇒ x = 2, 8
4 8 2 10 ∴ y = − 2, 4 [from Eq. (ii)]
= (5 2 − 7) − 4 2 + 6 = −
3 3 3 So, the point of intersection of Eqs. (i) and
=a 2+b (given) (ii) are P (2, − 2) and Q(8, 4).
8 10 Now, the area enclosed by the region A
So, on comparing a = and b = − 4
3 3 4
 y2   y2 y3 
8 10
= ∫  ( y + 4) −
2
dy = 
2
+ 4 y − 
6  −2
∴ a−b= + =6 −2 
3 3
= 
16 64   4 8
+ 16 −  −  − 8+ 
3. Given, equations of curves  2 6  2 6
 x + 1 ,x ≥ − 1 32 4
= 8 + 16 − − 2+ 8−
y = 2x and y = | x + 1| =  3 3
− x − 1 , x < − 1 = 30 − 12 = 18 sq unit.
Q The figure of above given curves is 5. Given region is A = {(x, y) : x2 ≤ y ≤ x + 2}
Y y=x+1 Now, the region is shown in the following graph
(1,2) Y y=x+2
y=–x–1 y=2x x2=y

(0,1) B(2,4)

X′ X (0,2)
(–1,0) O A
(–2,0)
In first quadrant, the above given curves intersect each X' X'
–1 O 2
other at (1, 2).
1 Y'
So, the required area = ∫ ((x + 1) − 2x ) dx
0 For intersecting points A and B
 x2 2x 
1
 ax  Taking, x2 = x + 2 ⇒ x2 − x − 2 = 0
= + x−  ∫ = + C
x
a dx
log e 2  0
Q ⇒ x2 − 2x + x − 2 = 0
2  log e a 
⇒ x(x − 2) + 1(x − 2) = 0
1 2 1  ⇒ x = −1, 2 ⇒ y = 1, 4
= +1− + 
 2 log e 2 log e 2 So, A(−1, 1) and B (2, 4).
2
3 1
= − Now, shaded area = ∫ [(x + 2) − x
2
] dx
2 log e 2
−1
 y2   x2 x3 
2

=  + 2x −  =  + 4 −  −  − 2 + 
4. Given region A = (x, y) : ≤ x ≤ y + 4 4 8 1 1
 2   2 3  −1  2 3   2 3
y2
∴ =x 1 9 1 1 9
2 =8− − = 8 − − 3 = 5 − = sq units
2 3 2 2 2
rankershalt.com

320 Area

6. Given, S (α ) = {(x, y) : y2 ≤ x, 0 ≤ x ≤ α } and and points of intersection of curves y = x2 + 3x and y = 4


are (1, 4) and (−4, 4)
A(α ) is area of the region S(α )
Now required area
Y
1
y2=x
1
 x3 3x2 
3
= ∫ (x2 + 3x)dx +
∫ 4 dx =  3 + 2  + [4x]1
3

0 1 0
1 3 2+9 11 59
= + + 4(3 − 1) = +8 = +8= sq units
X 3 2 6 6 6
O
A(λ) 9. Given equation of parabola is y = x2 + 2, and the line is
y = x+1
y y=x2 +2

x=λ y=x+1
(0,2)
λ
x 3/ 2 λ
4 3/ 2
Clearly, A (λ ) = 2∫ x dx = 2   =3λ 1
0  3 / 2 0
A (λ ) 2 x
Since, = , (0 < λ < 4) 1 O (3,0)
A (4) 5
λ3/ 2 2
3 2 The required area = area of shaded region
 λ  2
⇒ = ⇒   =  3 3
43/ 2 5  4  5 =∫ ((x2 + 2) − (x + 1)) dx = ∫ (x2 − x + 1) dx
0 0
1/3 1/3 3
λ 4 4  x3 x2   27 9 
⇒ =  ⇒ λ =4   =  − + x =  − + 3 − 0
4  25  25
3 2 0  3 2 
7. Given equations of the parabola y2 = 4x …(i) 9 9 15
= 9 − + 3 = 12 − = sq units
and circle x + y =5
2 2
…(ii) 2 2 2
So, for point of intersection of curves (i) and (ii), put 10. Given, equation of parabola is y = x2 + 1, which can be
y2 = 4x in Eq. (ii), we get written as x2 = ( y − 1). Clearly, vertex of parabola is
x2 + 4x − 5 = 0 (0, 1) and it will open upward.
⇒ x2 + 5x − x − 5 = 0 y+5
⇒ (x − 1)(x + 5) = 0 Now, equation of tangent at (2, 5) is = 2x + 1
2
⇒ x = 1, − 5
[Q Equation of the tangent at (x1 , y1 ) is given by
For first quadrant x = 1 , so y = 2 . 1
T = 0. Here, ( y + y1 ) = xx1 + 1]
Now, equation of tangent of parabola (i) at point (1, 2) 2
is T = 0 y = 4x − 3
⇒ 2 y = 2(x + 1)
y= 4x–3
⇒ x− y+ 1 =0
Y
 3 7
The point  ,  satisfies, the equation of line
 4 4
P (2, 5)
x− y+ 1 =0
8. Given, y ≤ x2 + 3x
2 2
 3 9  3  9 (0, 1)
⇒ y ≤ x +  − ⇒ x +  ≥  y + 
 2 4  2  4 R
O X
Q (2, 0)
Since, 0 ≤ y ≤ 4 and 0 ≤ x ≤ 3 3,
0
∴The diagram for the given inequalities is 4
Y
y=x2+3x
Required area = Area of shaded region
2
y=4 = ∫ y(parabola) dx − (Area of ∆PQR)
0
–3/2 2
O X = ∫ (x2 + 1) dx − (Area of ∆PQR)
(–3, 0) 1 3 0
9 2

4 x=3  x3  1 3
= + x − 2 −  ⋅ 5
 3 0 2  4
rankershalt.com

Area 321

1 We need to calculate the shaded area, which is equal to


[Q Area of a triangle = × base × height]
2 0 1
∫−1 (− x + 1)dx + ∫ (x2 + 1) dx
2
8  1  5 0
=  + 2 − 0 −   5
3  2  4 0 1
 x3   x3 
14 25 112 − 75 37 = − + x +  + x
= − = =  3 − 1  3 0
3 8 24 24
  (− 1)3   1 
11. Given equation of curve is x2 = 4 y, which represent a =  0 − − + (− 1)  +   + 1 − 0
   
parabola with vertex (0, 0) and it open upward.   3    3 
1  4
x2 = −  − 1 +
Y y= 3  3
4
x+2 2 4
y=
4
= + =2
3 3
B
A
X′ X 13. Given, equation of parabola is y = x2 − 1, which can be
–1 O 2 rewritten as x2 = y + 1 or x2 = ( y − (−1)).
⇒ Vertex of parabola is (0, − 1) and it is open upward.
Equation of tangent at (2, 3) is given by T = 0
Y′ y + y1
⇒ = x x1 − 1, where, x1 = 2
Now, let us find the points of intersection of x2 = 4 y and 2
4y = x + 2 and y1 = 3.
For this consider, x2 = x + 2 y+3
⇒ x2 − x − 2 = 0 ⇒ = 2x − 1
2
⇒ (x − 2) (x + 1) = 0 ⇒ y = 4x − 5
⇒ x = − 1, x = 2
1
When x = − 1, then y = y=x2–1
4 (2, 3)
and when x = 2, then y = 1
Thus, the points of intersection are A  − 1,  and B (2, 1).
1
 4 2
Now, required area = area of shaded region
2
= ∫−1 {y (line) − y (parabola)} dx (0, –1)
y=4x–5
2
2x + 2 x 2
1 x x  2 3
= ∫−1 4
−  dx =  + 2x −
4 42

3  −1
1  8  1 1 
= 2 + 4 −  −  − 2 + 
4   3  2 3  Now, required area = area of shaded region
1 1  1 1 9 2
= ∫ (y(parabola) − y(tangent)) dx
= 8 − −3 = 5− = sq units.
4  2  4  2  8 0
2
= ∫ [(x2 − 1) − (4x − 5)] dx
12. We have, 0
A = {(x, y) : 0 ≤ y ≤ x| x|+ 1and − 1 ≤ x ≤ 1} 2
= ∫ (x2 − 4x + 4) dx = ∫ (x − 2)2 dx
2

When x ≥ 0, then 0 ≤ y ≤ x2 + 1 0 0
2
and when x < 0, then 0 ≤ y ≤ − x2 + 1 (x − 2)3 (2 − 2)3 (0 − 2)3 8
= = − = sq units.
Now, the required region is the shaded region. 3 0
3 3 3
y
14. We have,
2 ⇒ 18x2 − 9πx + π 2 = 0
⇒ 18x − 6πx − 3πx + π 2 = 0
2
y=–x2+1 1 y=x2+1
(6x − π )(3x − π ) = 0
π π
x ⇒ x= ,
–1 1 6 3
y=0 π
Now, α < β α= ,
6
[Q y = x2 + 1⇒ x2 = ( y − 1), parabola with vertex (0, 1) and
π
y = − x2 + 1⇒ x2 = − ( y − 1) , β=
parabola with vertex (0,1) but open downward] 3
rankershalt.com

322 Area

Given, g(x) = cos x 2 and f(x) = x Shown as


Y
y = gof (x)
∴ y = g ( f (x)) = cos x
(0, 9/5)
Area of region bounded by x = α,x = β, y = 0 and curve
y = g ( f (x)) is X
π /3 0
(–9, 0)
A= ∫ cos x dx
x=6
π /6
A = [sin x]ππ //36
π π 3 1 ∴ {(x, y) ∈ R2 : y ≥ |x + 3|, 5 y ≤ (x + 9) ≤ 15}
A = sin − sin = −
3 6 2 2 Y
 3 − 1 (1,2)
A=  C
 2  4,1 )
B (–
15. Required area (1,0)
X′ X
1 2 2 x2 –9 (–4,0) A E (–3,0) 0 D 6
=∫ (1 + x )dx + ∫1 (3 − x)dx − ∫ dx
0 0 4
Y
y=1+√x Y′
(0, 3) (1, 2)
4y=x 2 ∴ Required area = Area of trapezium ABCD
(0, 1)
− Area of ABE under parabola
(2, 1)
x+y=3
− Area of CDE under parabola
1 −3 1
X′
(0, 0) (1, 0)(2, 0) (3, 0)
X = (1 + 2) (5) − ∫ − (x + 3) dx − ∫ (x + 3) dx
2 − 4 − 3
−3 1
Y′    
15  (− 3 − x)3/ 2   (x + 3)3/ 2 
= −  −
2  
1 2 2
 x3/ 2   x2   x3  3 3
= x +  + 3x −  −    −   
3 /2  0  2  1 12  0  2 − 4  2  –3

15 2 2 15 2 16 15 18 3
 2  1  8  = + [0 − 1] − [8 − 0] = − − = − =
= 1 +  + 6 − 2 − 3 +  −   2 3 3 2 3 3 2 3 2
 3  2  12
5 3 2 3 5 17. Given region is {(x, y) : y2 ≤ 2x and y ≥ 4x − 1}
= + − = 1 + = sq units
3 2 3 2 2 y2 ≤ 2x repressents a region inside the parabola
16. Here, {(x, y) ∈ R2 : y ≥ |x + 3|, 5 y ≤ (x + 9) ≤ 15} y2 = 2x …(i)
and y ≥ 4x − 1 represents a region to the left of the line
∴ y ≥ x+3
y = 4x − 1 …(ii)
 x + 3 , when x ≥ − 3 The point of intersection of the curves (i) and (ii) is
⇒ y≥ 
 − x − 3 , when x ≤ − 3 (4x − 1)2 = 2x ⇒ 16x2 + 1 − 8x = 2x
1 1
 x + 3 , when x ≥ − 3 ⇒ 16x2 − 10x + 11 = 0 ⇒ x = ,
or y2 ≥  2 8
− 3 − x, when x ≤ − 3 1 
So, the points where these curves intersect are  , 1
Shown as 2 
 1 1
and  ,  .
y2=–x–3
Y  8 2
y2=x+3 Y
−1
4x

1,1
y=

2
1 y 2 = 2x
X′ X
–3 0
1
2
−1 1
Y′ −1 2 2 1
X′ X
O
Also, 5 y ≤ (x + 9) ≤ 15 1 −1
,
−1 8 2
⇒ (x + 9) ≥ 5 yand x ≤ 6 2

−1
Y′
rankershalt.com

Area 323

π /4
1  y + 1 y2 ∴ Area bounded = ∫ {(sin x + cos x) − (cos x − sin x)} dx
∴ Required area = ∫  −  dy 0
−1/ 2  4 2 π /2

1  y2  1
−1 + ∫π / 4 {(sin x + cos x) − (sin x − cos x)} dx
=  + y − ( y3 )1−1/ 2 π /4 π /2
42  −1/ 2 6 =∫ 2 sin x dx + ∫π / 4 2 cos x dx
0
1  1  1 1  1  1 = − 2 [cos x]π0 / 4 + 2 [sin x ⋅ n ]ππ // 24
=  + 1 −  −   − 1 + 
4  2 8 2  6  8 = 4 − 2 2 = 2 2( 2 − 1) sq units
1 3 3  1 9 
=  + −   20. Given curves are y = x …(i)
4 2 8  6 8 
1 15 3 9 and 2y − x + 3 = 0 …(ii)
= × − = sq units Y
4 8 16 32
x
18. Given, A = {(x, y) : x2 + y2 ≤ 1 and y2 ≤ 1 − x} y=
=0
Y x +3
X' 2y – X
3
–3
2
X′ X
(–1,0) (0,1) Y'

On solving Eqs. (i) and (ii), we get


Y′ 2 x − ( x )2 + 3 = 0
1 2 1 ⇒ ( x )2 − 2 x − 3 = 0
Required area = πr + 2∫ (1 − y2)dy
2 0 ⇒ ( x − 3) ( x + 1) = 0 ⇒ x =3

1 [since, x = − 1 is not possible]
1 y3 
= π (1)2 + 2  y −  ∴ y=3
2  3 0
Hence, required area
 π 4 3 3
=  +  sq units = ∫ (x2 − x1 ) dy = ∫ {(2 y + 3) − y2} dy
 2 3 0 0
3
 y3 
19. PLAN To find the bounded area between y = f( x ) and y = g ( x ) =  y2 + 3 y − = 9 + 9 − 9 = 9 sq units
between x = a to x = b.  3  0
2
Y
f(x) 21. R1 = ∫ x f (x) dx …(i)
−1
g(x) b b

g(x)
Using ∫ a f (x) dx = ∫ a f (a + b − x) dx
f(x) 2
R1 = ∫ (1 − x) f (1 − x) dx
X −1
O a c b 2
∴ R1 = ∫ (1 − x) f (x) dx …(ii)
c b −1
∴ Area bounded = ∫a[g (x ) − f(x )]dx + ∫c [f(x ) − g (x )]dx [f (x) = f (1 − x), given]
b
= ∫a | f( x ) − g ( x )|dx Given, R2 is area bounded by f (x), x = − 1 and x = 2.
2
π ∴ R2 = ∫ f (x) dx …(iii)
Here, f (x) = y = sin x + cos x, when 0 ≤ x ≤ −1
2
and g (x) = y = | cos x − sin x| On adding Eqs. (i) and (ii), we get
 π 2R1 = ∫
2
cos x − sin x, 0 ≤ x ≤ f (x) dx …(iv)
 4 −1
=
π π From Eqs. (iii) and (iv), we get
sin x − cos x, ≤x≤
 4 2 2R1 = R2
could be shown as
Y
22. Here, area between 0 to b is R1 and b to 1 is R2.
y = sin x + cos x b 1 1
√2
f(x)
= √2 sin x + π
∴ ∫0 (1 − x)2 dx − ∫b (1 − x)2 dx =
4
4 b 1
1  (1 − x)3   (1 − x)3  1
⇒  −3  −  −3  = 4
g(x) g(x)  0  b
X
O π/4 π/2
rankershalt.com

324 Area

1 1 1 25. As from the figure, area enclosed between the curves is


⇒− [(1 − b)3 − 1] + [0 − (1 − b)3 ] =
3 3 4 OABCO.
2 1 1 1 1 Thus, the point of intersection of
⇒ − (1 − b) = − + = −
3
⇒ (1 − b)3 =
3 3 4 12 8 y = ax2 and x = ay2
1 1
⇒ (1 − b) = ⇒ b= y = ax2
2 2 Y
π / 4  1 + sin x 1 − sin x 
23. Required area = ∫  −  dx x
0  cos x cos x  B y 2= a
 1 + sin x 1 − sin x  (1/a,1/a)
C
Q > >0
 cos x cos x 
A
 x x  X' X
 2 tan 2 tan  O D
 1+ 2 1− 2 
 1 + tan 2x
1 + tan 2 
x
π /4 
=∫ 2 − 2  dx Y'
0  2x 2x

 1 − tan 1 − tan 
2 2  ⇒ x = a (ax2)2

 x x  1 1
1 + tan 2
1 + tan 2  ⇒ x = 0, ⇒ y = 0,
 2 2  a a
 x  1 1
x So, the points of intersection are (0, 0) and  ,  ⋅
π /4
 1 + tan 1 − tan   a a
=∫  2 − 2  dx
 1 − tan x ∴ Required area OABCO = Area of curve OCBDO
1 + tan 
0 x

 2 2 – Area of curve OABDO
x x x 1/a  x 2
π /4
1 + tan − 1 + tan π /4
2 tan ⇒ ∫ 0  a − ax  dx = 1 [given]
=∫ 2 2 dx = ∫ 2 dx
0 x 0 x 1/ a
1 − tan 2 1 − tan 2  1 x3/ 2 ax3 
2 2 ⇒  ⋅ − =1
π  a 3 /2 3  0
x 1 x tan 4t dt
Put tan = t ⇒ sec2 dx = dt = ∫ 8
2 1
2 2 2 0
(1 + t 2) 1 − t 2 ⇒ 2
− =1
3a 3a 2
2 −1 4t dt π
As ∫0 (1 + t 2) 1 − t 2
[Q tan
8
= 2 − 1] ⇒ a2 =
1
3
⇒ a=
1
[Q a > 0]
3
b
24. The curves y = (x − 1)2, y = (x + 1)2 and y = 1 /4 are 26. Since, ∫ f (x) dx = (b − 1) sin (3b + 4)
1
shown as
2 On differentiating both sides w.r.t. b, we get
Y y = (x + 1)2 y = (x – 1)
f (b) = 3(b − 1) ⋅ cos (3b + 4) + sin (3b + 4)
∴ f (x) = sin (3x + 4) + 3(x − 1) cos (3x + 4)
dy
1/4 P
27. Given, = 2x + 1
dx
R Q On integrating both sides
y = 1/4

–1 –1/2 O 1/2 1
X ∫ dy = ∫ (2x + 1) dx
⇒ y = x2 + x + C which passes through (1, 2)
where, points of intersection are ∴ 2 =1+1+C
1 1 1 1 ⇒ C =0
(x − 1)2 = ⇒ x = and (x + 1)2 = ⇒ x = −
4 2 4 2 ∴ y = x2 + x
 1 1  1 1
i.e. Q  ,  and R  − ,  Y
 2 4  2 4
1/ 2  1 y = x (x + 1)
∴ Required area = 2 ∫ (x − 1)2 − dx
0  4 
1/ 2
 (x − 1)3 1 
=2  − x X' X
 3 4 0 −1 O x=1
 1 1  1  8 1
= 2 − − −  − − 0  = = sq unit
 8 ⋅ 3 8  3   24 3 Y'
rankershalt.com

Area 325

Thus, the required area bounded by X-axis, the curve 1 1  1 1


∴ + 1 −  ≥S ≥1 − [from Eqs. (i) and (ii)]
and x = 1 2 e  2  e
 x3 x 2 1
30. Shaded area = e −  ∫ ex dx = 1
1 1
= ∫ (x2 + x)dx =  + 
3 2 0
0 0
e
1 1 5
= + = sq unit
Also, ∫1 ln (e + 1 − y) dy [put e + 1 − y = t ⇒ − dy = dt]
3 2 6 1 e e
1 α =∫ ln t (− dt ) = ∫ ln t dt = ∫ ln y dy = 1
28. ∫0 (x − x3 )dx = 2∫ (x − x3 )dx
0
e 1 1

1 α2 α4 31. Case I When m = 0


= 2 − 
4  2 4 In this case, y = x − x2 …(i)
and y=0 …(ii)
2α 4 − 4α 2 + 1 = 0
are two given curves, y > 0 is total region above X-axis.
4 − 16 − 8
⇒ α2 = (Q α ∈ (0, 1)) Therefore, area between y = x − x2 and y = 0
4
1 is area between y = x − x2 and above the X-axis
α =1 −
2
Y
2
29. PLAN (i) Area of region f( x ) bounded between x = a to x = b is
A

X
O B

y = f (x)
1
1  x2 x3  1 1 1 9
a a 1 a 2 a3 D ∴ A = ∫ (x − x2) dx =  −  = − = ≠
0
 2 3 0 2 3 6 2
b
∫a f(x )dx = Sum of areas of rectangle shown in shaded part. Hence, no solution exists.
(ii) If f( x )≥ g ( x ) when defined in [a, b ], then Case II When m < 0
b b In this case, area between y = x − x2 and y = mx is
∫a f(x )dx ≥ ∫a g (x )dx
2
OABCO and points of intersection are (0,0) and
Description of Situation As the given curve y = e − x {1 − m, m(1 − m)}.
1 −m
cannot be integrated, thus we have to bound this function by
∴ Area of curve OABCO = ∫ [x − x2 − mx] dx
using above mentioned concept. 0
2
Graph for y = e− x Y

1
A
1

√e B
X
X (0, 0) O {1 – m, m (1 – m)}
O 1 1

√2 C
y= y = mx
x – x2
Since, x2 ≤ x when x ∈ [0, 1]
2
⇒ − x2 ≥ − x or e− x ≥ e− x 1 −m
 x2 x3 
1 − x2 1 −x = (1 − m) − 
∴ ∫0 e dx ≥ ∫ e
0
dx
 2 3 0
1 1 1 1
⇒ S ≥ − (e− x )10 = 1 − …(i) = (1 − m)3 − (1 − m)3 = (1 − m)3
e 2 3 6
1 − x2
∫0 e dx ≤ Area of two rectangles 1 9
Also, ∴ (1 − m) =
3
[given]
6 2
 1  1 1
⇒ (1 − m)3 = 27
≤ 1 ×  + 1 − ×
 2  2 e ⇒ 1 −m =3
1 1  1 ⇒ m = −2
≤ + 1 −  …(ii)
2 e 2
rankershalt.com

326 Area

Case III When m > 0 ⇒ t =4


In this case, y = mx and y = x − x2 intersect in (0,0) and ∴ B (8, − 15)
{(1 – m), m( 1 – m)} as shown in figure − (3x + 6)
So, equation of chord AB is y = .
2
Y
8  4 − x2 3x + 6
∴ Required area = ∫ −2 
 4
+
2 
 dx
y = mx
8
 x3 3x2 
1– m = x − + + 3x
A O (0,0)
X  12 4  −2

 128  2 
B = 8 − + 48 + 24 −  −2 + + 3 − 6 
y = x − x2  3  3 

125
= sq units
0 3
∴ Area of shaded region = ∫ (x − x2 − mx) dx
1 −m 33. The region bounded by the curves y = x2, y = − x2 and
 x2 x 3 0 y2 = 4 x − 3 is symmetrical about X-axis, where y = 4x − 3
= (1 − m) −  meets at (1, 1).
 2 3 1−m
∴ Area of curve (OABCO )
1 1
= − (1 − m) (1 − m)2 + (1 − m)3
= 2 ∫ x2 dx − ∫ ( 4x − 3 ) dx
1 1
2 3  0 3/ 4 
1
= − (1 − m) 3
y = x2 Y
6
9 1
⇒ = − (1 − m)3 [given]
2 6 (1,1) A
⇒ (1 − m)3 = − 27
X' B (3/4, 0)
⇒ (1 − m) = − 3 X
O
⇒ m =3 + 1 =4 C
Therefore, (b) and (d) are the answers. y 2 = 4x – 3

4a 2 4a 1 Y′
 f (−1) 3a + 3a 
2 y = –x 2
32. Given, 4b2 4b 1  f (1)  = 3b2 + 3b 
 2   x3  1  (4x − 3)3/ 2 1 
 4c2 4c 1  f (2)  3c + 3c  = 2   −   
 
 3  0  3 ⋅ 4 / 2  3/ 4 
⇒ 4a 2 f (−1) + 4a f (1) + f (2) = 3a 2 + 3a , …(i)
4b2 f (−1) + 4b f (1) + f (2) = 3b2 + 3b …(ii)  1 1
=2  − 
 3 6
and 4c2 f (−1) + 4cf (1) + f (2) = 3c2 + 3c …(iii)
1 1
where, f (x) is quadratic expression given by, = 1 ⋅ = sq unit
6 3
f (x) = ax2 + bx + c and Eqs. (i), (ii) and (iii).
⇒ 4x2 f (−1) + 4x f (1) + f (2) = 3x2 + 3x 34. Here, slope of tangent,
dy (x + 1)2 + y − 3
or {4 f (−1) − 3} x + {4 f (1) − 3} x + f (2) = 0
2
…(iv) =
dx (x + 1)
As above equation has 3 roots a, b and c.
dy ( y − 3)
So, above equation is identity in x. ⇒ = (x + 1) + ,
dx (x + 1)
i.e. coefficients must be zero.
⇒ f (−1) = 3 / 4, f (1) = 3 / 4, f (2) = 0 …(v) Put x + 1 = X and y − 3 = Y
dy dY
Q f (x) = ax2 + bx + c ⇒ =
dx dX
∴ a = − 1 / 4, b = 0 and c = 1, using Eq. (v) dY Y
4 − x2 ∴ =X +
Thus, f (x) = shown as, dX X
4 dY 1
Let A (−2, 0), B = (2t , − t 2 + 1) ⇒ − Y =X
dX X
1
Since, AB subtends right angle at vertex V (0, 1). ∫ − dX 1
IF = e X = e− log X =
1 −t 2 X
⇒ ⋅ = −1
2 2t
rankershalt.com

Area 327

1 1 36. Given, x = (sin by) e−ay


X ∫
∴ Solution is, Y ⋅ = X ⋅ dX + c
X
Now, −1 ≤ sin by ≤ 1
Y
⇒ =X +c ⇒ − e− ay ≤ e− ay sin by ≤ e− ay
X
⇒ − e− ay ≤ x ≤ e− ay
Y Y

x = e – ay x = e – ay
y = x 2 _ 2x
S3
S2
S1
S0
X′ X X′ X
O 2 O

Y′
Y′
y − 3 = (x + 1)2 + c(x + 1), which passes through (2, 0). In this case, if we take a and b positive, the values − e− ay
⇒ − 3 = (3)2 + 3c and e− ay become left bond and right bond of the curve
⇒ c= −4 and due to oscillating nature of sin by, it will oscillate
∴ Required curve between x = e− ay and x = − e−ay
( j + 1 ) π /b
y = (x + 1)2 − 4(x + 1) + 3 Now, Sj = ∫ sin by ⋅ e−ay dy
jπ /b
⇒ y = x2 − 2x
since, I = sin by ⋅ e−ay dy 
  x3  
2
 ∫ 
∴ Required area =  ∫ (x2 − 2x)dx=  
2 − ay
− x2  I = − e 
 0  3  0 (a sin by + b cos by)
  a 2 + b2 
8 4
= − 4 = sq units  − 1  −a ( j + 1)π
3 3 ∴ S j = 2 2
e b
a + b 

35. The points in the graph are {a sin ( j + 1)π + b cos ( j + 1) π}
A (1, 1), B ( 2, 0), C (2, 2), D ( 2, 2) − ajπ 
Y
−e b (a sin jπ + b cos jπ ) 
a

y = |2 – x 2| 1 − ( j + 1 )π
y = x2 Sj = − [e b {0 + b(−1) j + 1 }
D (√2,2) a + b2
2

C (2,2)
− e− ajπ / b{0 + b(− 1) j }]
y=2  −
a
jπ 
− j b  −aπ 
=
A b ( 1 ) e  e b + 1 
 a 2 + b2  
X′ X  
x = 1 B (√2,0)  
[Q (−1) j + 2 = (−1)2 (−1) j = (−1) j ]
Y′ a
− jπ
be b  − aπ 
∴ Required area =  e b + 1
a + b 
2 2 

2 2
=∫ { x2 − (2 − x2)} dx + ∫ {2 − (x2 − 2)} dx
1 2 a
− jπ  − aπ 
be b  e b + 1
2 2  
=∫ (2x2 − 2) dx + ∫ (4 − x2) dx   a
1 2 − jπ
Sj a 2 + b2 e b
2x3 
2
 x 3 2 ∴ = − π
= a
− ( j − 1 )π  
a a
= − 2x + 4x −  Sj −1 − ( j − 1 )π
3 3 be b  e b + 1 e b
 1  2  
 
4 2 2   8 2 2
= − 2 2 − + 2 + 8 − − 4 2 + a 2 + b2
 3 3   3 3  a
− π
 20 − 12 2  =e b = constant
=  sq units
 3  ⇒ S 0 , S1 , S 2, K , S j form a GP.
For a = − 1 and b = π
rankershalt.com

328 Area
1
. πj 38. Refer to the figure given in the question. Let the
π ⋅ eπ  1 ⋅π 
 eπ + 1 = π ⋅ e (1 + e)
j
Sj = coordinates of P be (x, x2), where 0 ≤ x ≤ 1.
(1 + π 2)   (1 + π 2)
  For the area (OPRO ),
n n Upper boundary: y = x2 and
π ⋅ (1 + e) π (1 + e)
⇒ ∑ Sj = (1 + π )2
∑ e j = (1 + π 2) (e0 + e1 + ...+ en ) lower boundary : y = f (x)
j=0 j=0 Lower limit of x : 0
π (1 + e) (en + 1 − 1) Upper limit of x : x
= ⋅ x x
(1 + π 2) e−1 ∴ Area (OPRO ) = ∫0 t 2 dt − ∫
0
f (t ) dt
|x| ≤ 1
37. Given, f (x) =  2x2 ,  t3 
x
 x + ax + b,|x| > 1
x
=   − ∫ f (t ) dt
Y
3
 0 0

1 x3 x
x=–
3 ∫0
x= – 2y2 8 = − f (t ) dt
y = f (x)
For the area (OPQO ),
X′ –2 –1 X The upper curve : x = y
O
and the lower curve : x = y /2
Lower limit of y : 0
and upper limit of y : x2
x2 x2 t
Y′ ∴ Area (OPQO ) = ∫ t dt − ∫ dt
0 0 2
x2 + ax + b, if x < − 1
 2 2 1 2
⇒ f (x) = 2x, if − 1 ≤ x < 1 = [t3/ 2]0x − [t 2]x0
3 4
x2 + ax + b, if x ≥ 1
 2 3 x4
= x −
f is continuous on R, so f is continuous at –1 and 1. 3 4
According to the given condition,
lim f (x) = lim f (x) = f (−1)
x → −1 − x → −1 + x3 x 2 x4
− ∫ f (t ) dt = x3 −
3 0 3 4
and lim f (x) = lim f (x) = f (1)
x → 1− x →1+ On differentiating both sides w.r.t. x, we get
x2 − f (x) ⋅ 1 = 2x2 − x3
⇒ 1 − a + b = − 2 and 2 = 1 + a + b
⇒ f (x) = x3 − x2, 0 ≤ x ≤ 1
⇒ a − b = 3 and a + b = 1
∴ a =2, b = −1
39. We can draw the graph of y = x2, y = (1 − x2) and
y = 2x(1 − x) in following figure
 x2 + 2x − 1, if x < −1
 Y
Hence, f (x) =  2x, if −1 ≤ x < 1
(1, 1)
 x2 + 2x − 1, if x≥1 (0, 1) 2
y = (1 – x)
Next, we have to find the points x = − 2 y2 and y = f (x). y = x2
Q (1/2, 1/2)
The point of intersection is (–2, –1).
A B
−1/ 8  − x 
∴ Required area = ∫ − f (x) dx
−2  2 y = 2x (1 – x)
 
−1/ 8 −x −1 −1/ 8 X′ X
=∫ dx − ∫ (x + 2x − 1)dx − ∫
2
2x dx O 1/3 2/3 1
−2 2 −2 −1
−1 Y′
2  x3 
=− [(− x)3/ 2]−−12/ 8 −  + x2 − x  − [x2] −−11/ 8 Now, to get the point of intersection of y = x2 and
3 2   3   −2 y = 2x (1 − x), we get
2   1
3/ 2   1  x2 = 2x (1 − x)
=−   − 2  −  − + 1 + 1
3/ 2
⇒ 3x2 = 2x
3 2 8  3
⇒ x (3x − 2) = 0
 8  1 
+  − + 4 + 2 − −1 ⇒ x = 0, 2 / 3
 3  64 
Similarly, we can find the coordinate of the points of
2 5 63
= [2 2 − 2−9/ 2] + + intersection of
3 3 64 y = (1 − x2) and y = 2x (1 − x) are x = 1 / 3 and x = 1
63 509 761
= + = sq units
16 × 3 64 × 3 192
rankershalt.com

Area 329

From the figure, it is clear that, ∴ We consider only b = 1.


 (1 − x)2, if 0 ≤ x ≤ 1 / 3 dA
 Sign scheme for around b = 1 is as shown below :
f (x) = 2x (1 − x), if 1 / 3 ≤ x ≤ 2 / 3 db
 x2 , if 2 / 3 ≤ x ≤ 1 – + –
∴ The required area 0 1
1
A=∫ f (x) dx From sign scheme, it is clear that A is maximum at
0
b = 1.
1/3 2 /3 1 π /4
=∫ (1 − x)2 dx + ∫ 1/ 3 2x (1 − x) dx + ∫ 2/ 3 x 2
dx 41. We have, An = ∫ (tan x)n dx
0 0
2 /3
 1 
1/ 3
 2x3  1 3 
1
Since, 0 < tan x < 1, when 0 < x < π /4
= − (1 − x)3 + x2 −  + x
 3  3 1/ 3 3  2 / 3 We have, 0 < (tan x)n+ 1 < (tan x)n for each n ∈N
0 
π /4 π /4
 1  2 3 1    2 2 2 3
 2  1 2
2
 1 
3 ⇒ ∫0 (tan x)n+ 1 dx < ∫ (tan x)n dx
= −   +  +   −   −  +   
0
   3  3  3
 3 3 3  3 3 3  ⇒ An+ 1 < An
1 1  2
3
Now, for n > 2
+  (1) −    π /4
3 3  3 An + An + 2 = ∫ [( tan x)n + (tan x)n + 2] dx
  0
19 13 19 17 π /4
= + + = sq unit =∫ (tan x)n (1 + tan 2 x) dx
81 81 81 27 0
x2 Y
40. Eliminating y from y = and y = x − bx2, we get n
b y = (tan x)
x2 = bx − b2x2
b
⇒ x = 0,
1 + b2
Y (π/4, 1)
x2
y=
b
B
X
y = x – bx 2 O π/4

X′ X π /4
–1 O b =∫ (tan x)n sec2 x dx
2 0
1+ b
π /4
 1 
= (tan x)n + 1 
 (n + 1 ) 0
Y′ 1 1
= (1 − 0) =
(n + 1) n+1
Thus, the area enclosed between the parabolas
b/(1 + b )2  x2 Since, An + 2 < An + 1 < An,
A=∫  x − bx2 −  dx then An + An + 2 < 2 An
0  b
1
b/(1 + b )2 ⇒ < 2 An
 x2 x3 1 + b2  1 b2 n+1
= − ⋅ = ⋅
2 3 b  0 6 (1 + b2)2 1
⇒ < An …(i)
2n + 2
On differentiating w.r.t. b, we get
1
dA 1 (1 + b2)2 ⋅ 2b − 2b2 ⋅ (1 + b2) ⋅ 2b Also, for n > 2 An + An < An + An − 2 =
= ⋅ n −1
db 6 (1 + b2)4
1
1 b (1 − b2) ⇒ 2 An <
⋅ = n −1
3 (1 + b2)3
1
dA ⇒ An < …(ii)
For maximum value of A, put =0 2n − 2
db
1 1
⇒ b = − 1, 0, 1, since b > 0 From Eqs. (i) and (ii), < An <
2n + 2 2n − 2
rankershalt.com

330 Area

42. The equations of the sides of the square are as follow : 1 1


= (3 − 2 2 ) + (3 − 2 2 )3/ 2
AB : y = 1, BC : x = − 1, CD : y = − 1, DA : x = 1 2 3
1 1
Y = (3 − 2 2) + [( 2 − 1)2]3/ 2
2 3
B(–1,1) A(1,1) 1 1
= (3 − 2 2 ) + ( 2 − 1)3
2 3
1 1
(0,1/2) = (3 − 2 ) + [2 2 − 1 − 3 2 ( 2 − 1)]
2 3
1 1
X′ X = (3 − 2 2 ) + [5 2 − 7]
(–1/2,0) O (1/2,0) 2 3
1 1
(0,–1/2) = [9 − 6 2 + 10 2 − 14] = [4 2 − 5] sq units
6 6
1
Similarly, area OEGO = (4 2 − 5) sq units
C(–1,–1) D(1,–1) 6
Therefore, area of S lying in first quadrant
Y′
2 1
= (4 2 − 5) = (4 2 − 5) sq units
Let the region be S and (x, y) is any point inside it. 6 3
Then, according to given conditions, 4 1
Hence, S = (4 2 − 5) = (16 2 − 20) sq units
x2 + y2 < |1 − x|,|1 + x|,|1 − y|,|1 + y| 3 3
⇒ x2 + y2 < (1 − x)2, (1 + x)2, (1 − y)2, (1 + y)2 43. Given parabolas are y = 4x − x2
⇒ x2 + y2 < x2 − 2x + 1, x2 + 2x + 1, and y = − (x − 2)2 + 4
y2 − 2 y + 1, y2 + 2 y + 1 or (x − 2)2 = − ( y − 4)
⇒ y < 1 − 2x, y < 1 + 2x, x < 1 − 2 y and x2 < 2 y + 1
2 2 2
Therefore, it is a vertically downward parabola with
Now, in y2 = 1 − 2x and y2 = 1 + 2x, the first equation vertex at (2,4) and its axis is x = 2
represents a parabola with vertex at ( 1/2,0) and second 2
 1 1
equation represents a parabola with vertex ( –1/2, 0) and y = x2 − x ⇒ y =  x −  −
 2 4
and in x2 = 1 − 2 y and x2 = 1 + 2 y, the first equation
Y y = x2 − x
represents a parabola with vertex at (0, 1/2) and second
equation represents a parabola with vertex at (0, –1 /2) . (2,4)
Therefore, the region S is lying inside the four parabolas
y2 = 1 − 2x, y2 = 1 + 2x, x2 = 1 + 2 y, x2 = 1 − 2 y
Y
A(1,1)
(0,1)
y = 4x − x2
y
2
=
2x

X' X
–1

O 1 5/2
G (1/2,–1/4)
I E x2
=
2y
–1
Y'
X 2
O H F (1,0)  1 1
⇒ x −  = y +
 2 4
where, S is the shaded region.
1 1
Now, S is symmetrical in all four quadrants, therefore This is a parabola having its vertex at  , −  ⋅
2 4
S = 4 × Area lying in the first quadrant.
1
Now, y2 = 1 − 2x and x2 = 1 − 2 y intersect on the line Its axis is at x = and opening upwards.
2
y = x. The point of intersection is E ( 2 − 1, 2 − 1).
Area of the region OEFO The points of intersection of given curves are
= Area of ∆ OEH + Area of HEFH 4x − x2 = x2 − x ⇒ 2x2 = 5x
1 5
= ( 2 − 1 )2 + ∫
1/ 2
1 − 2x dx ⇒ x (2 − 5x) = 0 ⇒ x = 0,
2 2 −1 2
1  2 1 
1/ 2 Also, y = x2 − x meets X-axis at (0,0) and (1, 0).
= ( 2 − 1)2 + (1 − 2x)3/ 2 ⋅ (− 1)
2  3 2  2−1 5/ 2
1 1 ∴ Area, A1 = ∫ [(4x − x2) − (x2 − x)] dx
= (2 + 1 − 2 2 ) + (1 + 2 − 2 2 )3/ 2 0
2 3
rankershalt.com

Area 331

5/ 2
=∫ (5x − 2x2) dx 45. The required area is the shaded portion in following
0
figure
5/ 2 2 3
5 2 2 3  5  5 2  5
= x − x =   − .  y = 2x
2 3  0
Y
2  2 3  2
5 25 2 125
= ⋅ − ⋅
2 4 3 8
125  2 125 y = loge x
= 1 −  = sq units
8  3 24
X′ X
This area is considering above and below X-axis both. O 1/2 1 2
Now, for area below X-axis separately, we consider
1
1  x2 x3  1 1 1
A2 = − ∫ (x2 − x) dx =  −  = − = sq units
0
2 3 0 2 3 6
Y′
Therefore, net area above the X-axis is
125 − 4 121 ∴ The required area
A1 − A2 = = sq units 2
24 24 2  2x 
=∫ (2x − log x) dx =  − (x log x − x)
Hence, ratio of area above the X-axis and area below 1/ 2  log 2  1/ 2
X-axis
121 1 4 − 2 5 3
= : = 121 : 4 = − log 2 +  sq units
24 6  log 2 2 2
44. The curve y = x2 is a parabola. It is symmetric about 46. Both the curves are defined for x > 0.
Y-axis and has its vertex at (0, 0) and the curve Both are positive when x > 1 and negative when 0 < x < 1.
2 We know that, lim (log x) → −∞
y= is a bell shaped curve. X-axis is its asymptote
1 + x2 x→ 0 +
log x
and it is symmetric about Y-axis and its vertex is (0, 2). Hence, lim → −∞. Thus, Y-axis is asymptote of
x→ 0 + ex
Y
second curve.
And lim ex log x [(0) × ∞ form]
y = x2 x→ 0 +
e log x  ∞ 
= lim − form
A 2 x→ 0 + 1 / x 
 ∞ 
 1
e 
(–1, 1) C B (1,1)  x
y= 2 2 = lim =0 [using L’Hospital’s rule]
1+ x x→ 0 +  1
 − 2
X′ X  x 
O M
Thus, the first curve starts from (0, 0) but does not
Since, y = x2 …(i) include (0, 0).
2 Now, the given curves intersect, therefore
and y= …(ii) log x
1 + x2 ex log x =
2 ex
⇒ y= i.e. (e2x2 − 1) log x = 0
1+ y
1
⇒ y2 + y − 2 = 0 ⇒ x = 1, [Qx > 0]
e
⇒ ( y − 1) ( y + 2) = 0 ⇒ y = − 2, 1 Y
But y ≥ 0, so y = 1 ⇒ x = ± 1
log x
Therefore, coordinates of C are (–1, 1) and coordinates y=
ex
of B are (1,1).
y = ex log x
∴ Required area OBACO = 2 × Area of curve OBAO
X′ X
O 1/e 1
 1 2 1 
= 2 ∫ dx − ∫ x2 dx
 01 + x
2 0

  3 1 
x 2π 1   2
= 2 [2 tan −1 x]10 −    = 2 − = π −  sq unit
 3   4 3   3 Y′
 0 
rankershalt.com

332 Area

∴ The required area π 


Hence, equation of tangent at A  , 1 is
1  (log x)  4 
=∫  − ex log x dx
1/ e  ex  y−1 π
= 2 ⇒ y − 1 = 2x −
1 1 x − π /4 2
1  (log x)2   x2   e2 − 5
=   − e  (2 log x − 1) =   sq units
e  2 1/ e 4 1/ e  4e  Y y = tan x

47. Given, y = x (x − 1) 2

dy A
⇒ = x ⋅ 2 (x − 1) + (x − 1)2 O 1
dx X′ X
B L
Y y = x(x – 1) 2 −1
2
Y′
4 max
27 π 
⇒ (2x − y) =  − 1
X′ X 2 
O 1/3 1 min
∴ Required area is OABO
π /4
=∫ ( tan x) dx − area of ∆ ALB
0
Y′
1
= (x − 1) ⋅ (2x + x − 1) = [log|sec x|]π0 / 4 −
⋅ BL ⋅ AL
2
= (x − 1) (3x − 1) 1  π π − 2
+• − • + = log 2 −  −  ⋅1
2 4 4 
1 /3 1  1
∴ Maximum at x = 1 / 3 =  log 2 −  sq unit
 4
2
1  2 4
ymax =  −  = 49. Given curves, x2 + y2 = 25, 4 y = |4 − x2| could be
3  3 27
sketched as below, whose points of intersection are
Minimum at x = 1
(4 − x2)2
ymin = 0 x2 + = 25
16
Now, to find the area bounded by the curve y = x (x − 1)2,
Y
the Y-axis and line x = 2 .
4y = 4 – x 2
Y
5
4y = x 2 – 4 4y =x 2 – 4
C B
2

4 O
27 X′ X
–5 –4 –2 2 4
X′ X
O 1 A x 2 + y 2 = 25
x=2

Y′ –5
2
∴ Required area = Area of square OABC − ∫ y dx Y′
0
2
= 2 × 2 − ∫ x (x − 1)2 dx ⇒ (x2 + 24) (x2 − 16) = 0
0
⇒ x=±4
 x (x − 1)3  2 1 2   4 2  4 − x2 
= 4 −   − ∫ (x − 1)3 ⋅ 1 dx ∴ Required area = 2 ∫ 25 − x2 dx − ∫   dx
 3 0 3 0  
0 0  4 

x (x − 1)4 
2
4  x2 − 4 
= 4 −  (x − 1)3 − −∫   dx
3 12  0 2  4  
2 1 1  10  x  x 
4
=4− − + = sq units = 2  25 − x2 +
25
sin −1   
3 12 12  3
2 2  5 
0
dy
48. Given, y = tan x ⇒ = sec2x 
2
 
4
dx 1 x3  1  x3
 dy
− 4 x − −
3  0 4 3 − 4 x 
∴   =2
4    2 
 dx x = π
4
rankershalt.com

Area 333

 25  4  1  8  ⇒ x = 2, − 1
= 2 6 + sin −1    − 8 −
  2  5  4  3  ∴ Required area
2 1 2
1  64   8   =∫ 5 − x2 dx − ∫ ( − x + 1) dx − ∫ (x − 1) dx
−   − 16 −  − 8  −1 −1 1
4 3   3  
2 1 2
x 5  x   − x2   x2 
 25  4 4 4 4  = 5 − x2 + sin −1    −  + x −  − x
= 2 6 + sin −1   − − −   2 2  
5  −1  2  −1  2 1
 2  5 3 3 3 
 5 2  5 −1  − 1  
  4  = 1 + sin −1  − −1 + sin   
= 4 + 25 sin −1    sq units  2 5   2  5 
  5 
 1 1   1 
−  − + 1 + + 1 − 2 − 2 − + 1
50. Given curves are x2 + y2 = 4, x2 = − 2 y and x = y.  2 2   2 
5 2 1 1
Y = sin −1 + sin −1  −
2 5 5 2
2 y=x
5  2 1 1 4 1
= sin −1  1− + 1−  −
2  5 5 5 5 2
x + y2 = 4
5 1  5π 1
−√ 2 √2 = sin −1 (1) − =  −  sq units
2  4 2
O
X′ X 2
−2 2
 π π
tan x, − ≤ x ≤
 3 3
52. Given, y = 
π π
 cot x, ≤x≤
−2  6 2
x 2 − √ 2y which could be plotted as Y-axis.
Y′
Y
Thus, the required area
2 0 2 − x2 y = cot x y = tan x
= ∫− 2
4 − x2 dx − ∫− 2
x dx − ∫0 2
dx

2   x2 0   x3  2 π/2 π
X′
=2 ∫ 4 − x2 dx −     −  O π/4
X
  − 2  
0 2 3 2 0 –π/2 –π/4
π/3
2
x 4 x 2
=2  4 − x2 − sin −1  − 1 −
2 2 2 0 3
5
= (2 − π ) − Y′
3
π /4 π /3
1  ∴ Required area = ∫ ( tan x) dx + ∫ π / 4 ( cot x) dx
=  − π  sq units 0
3 
= [− log|cos x|]0π / 4 + [log sin x] ππ //34
51. Given curves y = 5 − x and y = |x − 1| could be
2
 1   3 1
= −  log − 0 +  log − log 
sketched as shown, whose point of intersection are  2   2 2
5 − x2 = (x − 1)2 3 1
Y = log − 2 log
2 2
3 1 1 
= log − log =  log e 3 sq units
y = –x + 1 y=x–1 2 2 2 
a 8 4  8
53. Here, ∫ 2 1 + x2 dx = ∫ a 1 + 2 dx
 x 
X′ X a 4
√5 –1 1 2 √5  8  8
⇒ x− = x−
 x  2  x  a
1
Y′  8  8
⇒  a −  − (2 − 4) = (4 − 2) −  a − 
⇒ 5 − x = x2 − 2x + 1
2  a  a
⇒ 2x − 2x − 4 = 0
2
rankershalt.com

334 Area

8 8 16 Y
⇒ a− + 2 =2 − a + ⇒ 2a − =0
a a a
⇒ 2 (a − 8) = 0
2 P (t1)
⇒ a = ±2 2 [neglecting –ve sign]
∴ a =2 2 A
X′ X
54. The point of intersection of the curves x2 = 4 y and −1 C 1 N
x = 4 y − 2 could be sketched are x = − 1 and x = 2.
∴ Required area Q (−t1)
2  x + 2   x2 
= ∫   −    dx
−1  4   4 Y′

3 2 Required area
1  x2 x
=  2 + 2x − 3   e t1 + e – t1 
4   −1 = 2 area of ∆PCN – ∫ 2 ydx
 8  1 1   1 
1  
= 2 + 4 − 3 −  2 − 2 + 
4  3  1  et1 + e– t1   et1 – e– t1  t1 dy 
=2     –∫ y ⋅ dt 
1 10  −7  1 9 9
2  2  2  dt
1
= −   = ⋅ = sq units 
4  3  6   4 2 8
 e2t1 – e–2t1 t1  et – e– t  
 et1 + e– t1 et1 – e– t1  =2 –∫  dt 
55. Let P =  ,   8  2  
 2 2  0

e2t1 – e–2t1 1 t1 2t
 e– t + et1 e– t1 – et  = – ∫ (e + e–2t – 2)dt
and Q =  ,  4 2 0
 2 2 
e2t1 – e–2t1 1  e2t e–2t 
We have to find the area of the region bounded by the = –  – – 2t 
4 22 2 
curve x2 – y2 = 1 and the lines joining the centre x = 0,
y = 0 to the points (t1 ) and (– t1 ). e2t1 – e–2t1 1 2t1
= – (e – e–2t1 – 4t1 )
4 4

Download Chapter Test


http://tinyurl.com/y5eq3q4r or
rankershalt.com

14
Differential Equations

Topic 1 Solution of Differential Equations by Variable


Separation Method
Objective Questions I (Only one correct option) dy 1 − y2
7. The differential equation = determines a
1. Let f be a differentiable function such that f (1) = 2 and dx y
f ′ (x) = f (x) for all x ∈ R. If h (x) = f ( f (x)), then h′ (1) is family of circles with (2007, 3M)
equal to (2019 Main, 12 Jan II) (a) variable radii and a fixed centre at (0, 1)
(a) 4e2 (b) 4e (c) 2e (d) 2e2 (b) variable radii and a fixed centre at (0, – 1)
dy (c) fixed radius 1 and variable centres along the X-axis
2. The solution of the differential equation, = (x − y)2, (d) fixed radius 1 and variable centres along the Y-axis
dx
when y(1) = 1, is (2019 Main, 11 Jan II) 2 + sin x  dy
8. If y = y (x) and   = − cos x , y (0) = 1, then
y + 1  dx
2− y 1+ x − y  π
(a) log e = 2( y − 1) (b) − log e =x+ y−2 y   equals (2004, 1M)
2− x 1− x + y  2
2− x 1− x + y (a) 1/3 (b) 2/3
(c) log e =x− y (d) − log e = 2(x − 1) (c) − 1 / 3 (d) 1
2− y 1+ x − y
9. A solution of the differential equation
3. Let f : [0, 1] → R be such that f (xy) = f (x). f ( y), for all  dy
2
dy
x, y ∈ [0, 1] and f (0) ≠ 0. If y = y (x) satisfies the   −x + y = 0 is (1999, 2M)
 dx dx
dy
differential equation, = f (x) with y(0) = 1, then (a) y = 2 (b) y = 2x
dx
 1  3 (c)y = 2x − 4 (d) y = 2x2 − 4
y   + y   is equal to
 4  4 (2019 Main, 9 Jan II) 10. The order of the differential equation whose general
(a) 5 (b) 3 (c) 2 (d) 4 solution is given by y = (c1 + c2) cos (x + c3 ) − c4 ex + c5 ,
where c1 , c2, c3 , c4 , c5 are arbitrary constants, is
dy  π
4. If (2 + sin x) + ( y + 1) cos x = 0 and y(0) = 1, then y  (a) 5 (b) 4 (1998, 2M)
dx  2 (c) 3 (d) 2
is equal to (2017 Main)
(a)
1
(b) −
2
(c) −
1
(d)
4 Objective Questions II
3 3 3 3 (One or more than one correct option)
5. If y = y(x) satisfies the differential equation 11. Let f : [0, ∞ ) → R be a continuous function such that
( )
−1
  x x− t
8 x 9 + x dy =  4 + 9 +

x

dx, x>0 and f (x) = 1 − 2x + ∫0 e f (t ) dt for all x ∈ [0, ∞ ). Then,

y(0) = 7, then y(256) = (2017 Adv.) which of the following statement(s) is (are) TRUE?
(2018 Adv.)
(a) 16 (b) 3 (a) The curve y = f (x) passes through the point (1, 2)
(c) 9 (d) 80 (b) The curve y = f (x) passes through the point (2, − 1)
13
1 (c) The area of the region
6. The value of ∑  π (k − 1)π   π kπ 
is equal {(x, y) ∈ [0, 1] × R : f (x) ≤ y ≤ 1 − x2 } is
π−2
k =1 sin  +  sin  +  4
4 6  4 6 (d) The area of the region
π −1
to (2016 Adv.) {(x, y) ∈ [0, 1] × R : f (x) ≤ y ≤ 1 − x2 } is
(a) 3 − 3 (b) 2(3 − 3) (c) 2( 3 − 1) (d) 2(2 + 3) 4
rankershalt.com

336 Differential Equations

12. Let y(x) be a solution of the differential equation Analytical & Descriptive Questions
(1 + ex ) y′ + yex = 1. If y(0) = 2, then which of the dP (x)
following statement(s) is/are true? (2015 Adv.) 17. If P(1) = 0 and > P (x), ∀ x ≥ 1 , then prove that
dx
(a) y (−4) = 0 P (x) > 0, ∀ x > 1. (2003, 4M)
(b) y (−2) = 0 18. Let y = f (x) be a curve passing through (1, 1) such that
(c) y(x) has a critical point in the interval (−1 , 0) the triangle formed by the coordinate axes and the
(d) y(x) has no critical point in the interval (−1 , 0) tangent at any point of the curve lies in the first
13. Consider the family of all circles whose centres lie on the quadrant and has area 2 unit. Form the differential
straight line y = x.If this family of circles is represented by equation and determine all such possible curves.
the differential equation Py′ ′+ Qy′ + 1 = 0, where P , Q are (1995, 5M)
dy d 2y
the functions of x, y and y′ (here, y′ = , y′ ′ = ), then
dx dx2 Integer Answer Type Question
which of the following statement(s) is/are true? (2015 Adv.)
19. Let f : R → R be a continuous function, which satisfies
(a) P = y + x x
(b) P = y − x f (x) = ∫ f (t ) dt . Then, the value of f (ln 5) is … .
0 (2009)
(c) P + Q = 1 − x + y + y′ + ( y′ )2
(d) P – Q = x + y – y′ – ( y′ )2
Passage Based Problems
14. The differential equation representing the family of
curves y2 = 2c (x + c), where c is a positive parameter, Passage
is of (1999, 3M) Let f : [0, 1] → R (the set of all real numbers) be a function.
(a) order 1 (b) order 2 Suppose the function f is twice differentiable,
(c) degree 3 (d) degree 4 f ( 0) = f (1) = 0 and satisfies
f ′ ′ (x) − 2 f ′ (x) + f (x) ≥ ex , x ∈ [0, 1] (2013 Adv.)
Numerical Value −x
20. If the function e f (x) assumes its minimum in the
15. Let f : R → R be a differentiable function with f (0) = 0. interval [0, 1] at x = 1 / 4, then which of the following is
If y = f (x) satisfies the differential equation true?
dy 1 3 1
= (2 + 5 y) (5 y − 2), then the value of lim f (x) is ...... . (a) f ′ (x) < f (x), < x< (b) f ′ (x) > f (x), 0 < x <
dx x→− ∞ (2018 Adv.) 4 4 4
1 3
(c) f ′ (x) < f (x), 0 < x < (d) f ′ (x) < f (x), < x<1
Assertion and Reason 4 4
For the following question, choose the correct answer 21. Which of the following is true?
from the codes (a), (b), (c) and (d) defined as follows. 1 1
(a) 0 < f (x) < ∞ (b) − < f (x ) <
(a) Statement I is true, Statement II is also true; 2 2
Statement II is the correct explanation of Statement I. 1
(c) − < f (x ) < 1 (d) − ∞ < f (x) < 0
(b) Statement I is true, Statement II is also true; 4
Statement II is not the correct explanation of 22. Which of the following is true?
Statement I. (a) g is increasing on (1, ∞ )
(c) Statement I is true; Statement II is false. (b) g is decreasing on (1, ∞ )
(d) Statement I is false; Statement II is true.
(c) g is increasing on (1, 2) and decreasing on (2, ∞ )
16. Let a solution y = y(x) of the differential equation (d) g is decreasing on (1, 2) and increasing on (2, ∞ )
2
x x2 − 1 dy − y y2 − 1 dx = 0 satisfy y(2) = 23. Consider the statements.
3
I. There exists some x ∈ R such that, f (x) + 2x = 2(1 + x2)
 π
Statement I y(x) = sec sec−1 x −  and II. There exists some x ∈ R such that,
 6
1 2 3 1 2 f (x) + 1 = 2x (1 + x)
Statement II y(x) is given by = − 1− 2 (a) Both I and II are true (b) I is true and II is false
y x x
(2008, 3M)
(c) I is false and II is true (d) Both I and II are false
rankershalt.com

Topic 2 Linear Differential Equation and


Exact Differential Equation
Objective Questions I (Only one correct option) 8. If a curve passes through the point (1, − 2) and has slope
1. The general solution of the differential equation x2 − 2 y
of the tangent at any point (x, y) on it as , then
( y − x )dx − xydy = 0 (x ≠ 0) is (where, C is a constant of
2 3
x
integration) (2019 Main, 12 April II) the curve also passes through the point
(a) y2 − 2x2 + Cx3 = 0 (b) y2 + 2x3 + Cx2 = 0 (2019 Main, 12 Jan II)
(c) y2 + 2x2 + Cx3 = 0 (d) y2 − 2x3 + Cx2 = 0 (a) ( 3 , 0) (b) (− 1, 2)
(c) (− 2 , 1) (d) (3, 0)
 1
2. Consider the differential equation, y2dx +  x −  dy = 0. 9. Let y = y(x) be the solution of the differential equation,
 y
dy
If value of y is 1 when x = 1, then the value of x for which x + y = x log e x, (x > 1). If 2 y(2) = log e 4 − 1, then y(e)
y = 2, is (2019 Main, 12 April I)
dx
5 1 3 1 1 1 3 is equal to (2019 Main, 12 Jan I)
(a) + (b) − (c) + (d) − e e e2 e e2
2 e 2 e 2 e 2 (a) − (b) − (c) (d)
2 2 4 4
3. Let y = y(x) be the solution of the differential equation,
dy  π π 10. If y(x) is the solution of the differential equation
+ y tan x = 2x + x2 tan x, x ∈− , , such that dy  2x + 1 −2x
dx  2 2 +  y = e , x > 0,
dx  x 
y(0) = 1. Then (2019 Main, 10 April II)
1
π π where y (1) = e−2, then
(a) y ′   − y ′  −  = π −
(2019 Main, 11 Jan I)
2 (b) 2
 4  4
(a) y(x) is decreasing in  , 1
1
π π
y ′   + y ′  −  = − 2 2 
 4  4
(b) y(x) is decreasing in (0, 1)
π π π2 π π (c) y(log e 2) = log e 4
(c) y   + y −  = + 2 (d) y   − y −  = 2
 4  4 2  4  4 log e 2
(d) y(log e 2) =
4. If y = y(x) is the solution of the differential equation 4
dy  π π 11. Let f be a differentiable function such that
= (tan x − y) sec2 x, x ∈  − ,  , such that y (0) = 0,
dx  2 2 3 f (x)  1
f ′ (x) = 7 − , (x > 0) and f (1) ≠ 4. Then, lim x f 
 π 4 x x→ 0 +  x
then y  −  is equal to (2019 Main, 10 April I)
 4 (2019 Main, 10 Jan II)
4
1 1 1 (a) does not exist (b) exists and equals
(a) − 2 (b) −e (c) 2 + (d) e − 2 7
e 2 e (c) exists and equals 0 (d) exists and equals 4
π
5. If cos x − y sin x = 6x, 0 < x <  and y  = 0, then
dy x dy 3 1  −π π   π 4
dx  2  3 12. If + y= ,x ∈  ,  and y  = , then
dx cos 2 x cos 2 x  3 3  4 3
 π  π
y  is equal to (2019 Main, 9 April II)
y −  equals
 6  4 (2019 Main, 10 Jan I)
π2 π2 π2 π2 1 4 1 1
(a) (b) − (c) − (d) − (a) + e6 (b) − (c) + e3 (d)
2 3 2 3 4 3 2 3 3 3 3
6. The solution of the differential equation 13. If y = y(x) is the solution of the differential equation,
dy
x + 2 y = x2(x ≠ 0) with y(1) = 1, is (2019 Main, 9 April I) dy  1
dx x + 2 y = x2 satisfying y(1) = 1, then y  is equal to
dx  2
x2 3 x3 1
(a) y = + (b) y = + (2019 Main, 9 Jan I)
4 4x2 5 5x2 (a)
13
(b)
1
(c)
49
(d)
7
3 1 4 1
(c) y = x2 + (d) y = x3 + 16 4 16 64
4 4x2 5 5x2
14. Let y = y(x) be the solution of the differential equation
7. Let y = y(x) be the solution of the differential equation, dy
dy sin x + y cos x = 4x, x ∈ (0, π ).
(x + 1)
2
+ 2x(x2 + 1) y = 1 such
2
that y(0) = 0. If dx
dx  π  π
π If y  = 0, then y  is equal to
a y(1) = , then the value of ‘a’ is (2019 Main, 8 April I)  2  6 (2018 Main)
32 4 −8 2 8 4 2
1 1 1 (a) π 2
(b) π (c) − π 2 (d) − π
(a) (b) (c) 1 (d) 9 3 9 3 9 9
4 2 16
rankershalt.com

338 Differential Equations

15. If a curve y = f (x) passes through the point (1, − 1) and dy


21. If y (t ) is a solution of (1 + t ) − ty = 1 and y (0) = − 1,
satisfies the differential equation, y(1 + xy)dx = x dy, dt
 1 then y (1) is equal to (2003, 1M)
then f  −  is equal to
 2 (2016 Main) (a) −1 / 2 (b) e + 1 / 2
2 4 2 4 (c) e − 1 / 2 (d) 1 / 2
(a) − (b) − (c) (d)
5 5 5 5
Objective Questions II
16. Let y(x) be the solution of the differential equation
dy (One or more than one correct option)
(x log x) + y = 2x log x, (x ≥ 1). Then, y(e) is equal to
dx 22. Let f : (0, ∞ ) → R be a differentiable function such that
(2015 Main) f (x)
f ′ (x) = 2 − for all x ∈ (0, ∞ ) and f (1) ≠ 1. Then
(a) e (b) 0 (c) 2 (d) 2e x (2016 Adv.)
17. The function y = f (x) is the solution of the differential (a) lim f ′  = 1
1
x → 0+  x 
dy xy x + 2x 4
equation + = in (−1, 1) satisfying
(b) lim x f   = 2
1
dx x2 − 1 1 − x2 x → 0+  x
3 /2
f (0) = 0. Then, ∫ 3
f (x) dx is (2014 Adv.) (c) lim x2f ′(x) = 0
− x → 0+
2
π 3 π 3 π 3 π 3 (d)|f (x)|≤ 2 for all x ∈ (0, 2)
(a) − (b) − (c) − (d) −
3 2 3 4 6 4 6 2 23. If y(x) satisfies the differential equation
18. Let f : [1 /2, 1] → R (the set of all real numbers) be a y′ − y tan x = 2 x sec x and y(0), then (2012)
π π2 π π2
positive, non-constant and differentiable function such (a) y   = (b) y′   =
that f ′ (x) < 2 f (x) and f (1 / 2) = 1 . Then, the value of  4 8 2  4  18
1
π π2 π 4π 2π 2
∫ f (x) dx lies in the interval
1/ 2 (2013 Adv.) (c) y   =

 3
(d) y′   =

 3
+
9 3 3 3
(a) (2e − 1, 2e) (b) (e − 1, 2e − 1)
e−1 e − 1
(c)  , e − 1 (d)  0,  Analytical & Descriptive Question
 2   2 
24. Let u (x) and v (x) satisfy the differential equations
19. Let f (x) be differentiable on the interval (0, ∞) such that du dv
t 2f (x) − x2f (t ) + p (x) u = f (x) and + p (x) v = g (x), where
f (1) = 1, and lim = 1 for each x > 0 . Then, dx dx
t→ x t−x p (x), f (x) and g (x) are continuous functions. If
f (x) is (2007, 3M) u (x1 ) > v (x1 ) for some x1 and f (x) > g (x) for all x > x1,
1 2x2 1 4x2 prove that any point (x, y) where x > x1 does not satisfy
(a) + (b) − +
3x 3 3x 3 the equations y = u (x) and y = v (x). (1997, 5M)
1 2 1
(c) − + (d)
x x2 x Integer Answer Type Question
20. If x dy = y (dx + y dy), y (1) = 1 and y (x) > 0. Then, y (−3) 25. Let y′ (x) + y(x) g′ (x) = g (x) g′ (x), y(0) = 0, x ∈ R, where
is equal to d f (x)
(2005, 1M) f ′ (x) denotes and g (x) is a given non-constant
(a) 3 (b) 2 dx
(c) 1 (d) 0 differentiable function on R with g (0) = g (2) = 0. Then,
the value of y(2) is …… (2011)

Topic 3 Applications of Homogeneous Differential Equations


Objective Questions I (Only one correct option) 2. The curve amongst the family of curves represented by
1. Given that the slope of the tangent to a curve y = y(x) at the differential equation, (x2 − y2)dx + 2xydy = 0, which
2y passes through (1, 1), is (2019 Main, 10 Jan II)
any point (x, y) is. If the curve passes through the
x2 (a) a circle with centre on the Y-axis
centre of the circle x2 + y2 − 2x − 2 y = 0, then its (b) a circle with centre on the X-axis
equation is (2019 Main, 8 April II) (c) an ellipse with major axis along the Y-axis
(d) a hyperbola with transverse axis along the X-axis.
(a) x2 log e|y| = − 2(x − 1) (b) x log e|y|= x − 1
(c) x log e|y| = 2(x − 1) (d) x log e|y| = − 2(x − 1)
rankershalt.com

Differential Equations 339

3. Let the population of rabbits surviving at a time Analytical & Descriptive Questions
t be governed by the differential equation
dp(t ) 1
= p(t ) − 200. If p(0) = 100, then p(t ) is equal to 9. If length of tangent at any point on the curve y = f (x)
dt 2 intercepted between the point and the X-axis is of
(2014 Main)
t t
length 1. Find the equation of the curve. (2005, 4M)

(a) 400 − 300 e 2 (b) 300 − 200 e 2 10. A right circular cone with radius R and height H
t t
− contains a liquid which evaporates at a rate
(c) 600 − 500 e 2 (d) 400 − 300 e 2
proportional to its surface area in contact with air
π (proportionality constant = k > 0). Find the time after
4. A curve passes through the point 1,  . Let the slope of which the cone is empty.
 6 (2003, 4M)
y  y 11. A hemispherical tank of radius 2 m is initially full of
the curve at each point (x, y) be + sec   , x > 0.
x  x water and has an outlet of 12 cm2 cross-sectional area at
Then, the equation of the curve is (2013 Adv.) the bottom. The outlet is opened at some instant. The
flow through the outlet is according to the law
(a) sin   = log x + (b) cosec   = log x + 2
y 1 y
 x 2  x v (t ) = 0.6 2 gh (t ), where v (t ) and h (t) are respectively
the velocity of the flow through the outlet and the
 2y 
(c) sec   = log x + 2 (d) cos   = log x +
2y 1
height of water level above the outlet at time t and g is
 x   x  2 the acceleration due to gravity. Find the time it takes to
5. At present, a firm is manufacturing 2000 items. It is empty the tank. (2001, 10M)
estimated that the rate of change of production P with Hint Form a differential equation by relating the
respect to additional number of workers x is given by decreases of water level to the outflow.
dP
= 100 − 12 x. If the firm employees 25 more 12. A country has food deficit of 10%. Its population grows
dx
workers, then the new level of production of items is continuously at a rate of 3% per year. Its annual food
production every year is 4% more than that of the last
(2013 Main)
year. Assuming that the average food requirement per
(a) 2500 (b) 3000 (c) 3500 (d) 4500
person remains constant, prove that the country
will become self- sufficient in food after n years, where n
Objective Questions II is the smallest integer bigger than or equal to
(One or more than one correct option) ln 10 − ln 9
.
6. A solution curve of the differential equation ln (1.04) − (0.03) (2000, 10M)
dy
(x + xy + 4x + 2 y + 4)
2
− y2 = 0, x > 0, passes through 13. A curve passing through the point (1, 1) has the property
dx that the perpendicular distance of the origin from the
the point (1, 3). Then, the solution curve (2016 Adv.)
normal at any point P of the curve is equal to the
(a) intersects y = x + 2 exactly at one point distance of P from the X-axis. Determine the equation of
(b) intersects y = x + 2 exactly at two points the curve. (1999, 10M)
(c) intersects y = (x + 2)2
(d) does not intersect y = (x + 3)2 14. A and B are two separate reservoirs of water. Capacity
of reservoir A is double the capacity of reservoir B. Both
7. Tangent is drawn at any point P of a curve which passes the reservoirs are filled completely with water, their
through (1, 1) cutting X-axis and Y-axis at A and B, inlets are closed and then the water is released
respectively. If BP : AP = 3 : 1, then (2006, 3M) simultaneously from both the reservoirs. The rate of
dy
(a) differential equation of the curve is 3x + y =0 flow of water out of each reservoir at any instant of time
dx is proportional to the quantity of water in the reservoir
dy
(b) differential equation of the curve is 3x − y=0 at the time.
dx
One hour after the water is released, the quantity of
(c) curve is passing through  , 2
1
1
8  water in reservoir A is 1 times the quantity of water in
2
(d) normal at (1, 1) is x + 3 y = 4.
reservoir B. After how many hours do both the
reservoirs have the same quantity of water ?
Fill in the Blank (1997, 7M)
8. A spherical rain drop evaporates at a rate proportional 15. Determine the equation of the curve passing through
to its surface area at any instant t. The differential the origin in the form y = f (x), which satisfies the
equation giving the rate of change of the rains of the dy
differential equation = sin (10x + 6 y) (1996, 5M)
rain drop is …. . (1997C, 2M) dx
rankershalt.com

340 Differential Equations

Match the Columns


16. Match the conditions/expressions in Column I with statements in Column II. (2006, 6M)

Column I Column II
π /2
A.
∫0 {cos x cot x − log (sin x ) p. 1
cos x sin x
(sin x ) }dx

B. Area bounded by − 4 y 2 = x and x − 1 = − 5 y 2 q. 0


C. The angle of intersection of curves y = 3 x − 1 log x and y = x x − 1 is r. 3e y / 2
dy 2 s. 4
D. If = passing through (1, 0), then ( x + y + 2 ) is
dx x + y 3

Answers
Topic 1 17. (b) 18. (d) 19. (a) 20. (a)
1. (b) 2. (d) 3. (b) 4. (a) 21. (a) 22. (a) 23. (a, d) 25. (0)
5. (b) 6. (c) 7. (c) 8. (a) Topic 3
9. (c) 10. (c) 11. (b, c) 12. (a, c)
1. (c) 2. (b) 3. (a) 4. (a)
13. (b, c)
5. (c) 6. (a, d) 7. (a, c)
14. (a, c) 15. (0.40) 16. (b)  
 dr  1 + 1 −y2
d 2y dy 8.  = − λ  9.  1 − y 2 − log = ± x + c
18. Differential Equation: 2 = 0, x 2 +1 = 0  dt   
dx dx  1 − 1 −y2 
Curves : x + y = 2, xy = 1  14 π × 10 5 
 H
19. (0) 20. (c) 21. (d) 22. (b) 10. T =  11.  unit
 k  27 g 
23. (c)
 1 
Topic 2 13. ( x 2 + y 2 = 2 x ) 14.  log 3  
 2 
 4
1. (b) 2. (b) 3. (a) 4. (d)
1 4  3  3  5x
5. (b) 6. (a) 7. (d) 8. (a) 15. tan −1  tan  4 x + tan −1  − −
3 5  4 5  3
9. (c) 10. (a) 11. (d) 12. (a)
13. (c) 14. (c) 15. (d) 16. (c) 16. A → p; B → s; C → q; D → r

Hints & Solutions


Topic 1 Solution of Differential Equations Q f (1) = 2
by Variable Separation Method So, ln (2) = 1 + C [using Eq. (i)]
1. Given that, f ′ (x) = f (x) ⇒ C = ln 2 − ln e [Q ln e = 1]
f ′ (x)  2  A
⇒ =1 ⇒ C = ln   [Q ln A − ln B = ln   ]
f (x)  e  B
f ′ (x) From Eq. (i), we get
⇒ ∫ f (x)
dx = ∫ 1 ⋅ dx
 2
ln| f (x)|= x + ln  
 e
[by integrating both sides w.r.t. x]
⇒ Put f (x) = t ⇒ f ′ (x)dx = dt  2
⇒ ln| f (x)|− ln   = x
dt  e
∴ ∫ t = ∫ 1 dx ef (x) A
⇒ ln =x [Q ln A − ln B = ln ]
 dx  2 B
⇒ ln|t|= x + C Q ∫ x = ln|x|+ C  e
⇒ f (x) = ex [Q ln a = b ⇒ a = eb , a > 0]
⇒ ln| f (x)|= x + C …(i) 2
[Q t = f (x)]
rankershalt.com

Differential Equations 341

 e e  dy
= f (x) ⇒
dy
=1
⇒ | f (x)|= 2ex −1 Q 2 f (x) = 2 | f (x)| 
So,
  dx dx
f (x) = 2ex −1 or −2ex −1 ⇒ ∫ dy = ∫ dx
Now, h (x) = f ( f (x)) ⇒ y= x + C
⇒ h′ (x) = f ′ ( f (x)) ⋅ f ′ (x) Q y(0) = 1
[on differentiating both sides w.r.t. ‘x’] ∴ 1 =0 + C
⇒ h′ (1) = f ′ ( f (1)) ⋅ f ′ (1) ⇒ C =1
= f ′ (2) ⋅ f ′ (1) [Q f (1) = 2 (given)] ∴ y=x+1
= 2e2−1 ⋅ 2e1−1 [Q f ′ (x) = 2ex −1 or −2ex −1]  1 1 5  3 3 7
Now, y   = + 1 = and y   = + 1 =
= 4e  4 4 4  4 4 4
dy  1  3 5 7
2. We have, = (x − y)2 which is a differential equation of ⇒ y  + y  = + =3
dx  4  4 4 4
the form dy
dy 4. We have, (2 + sin x) + ( y + 1) cos x = 0
= f (ax + by + c) dx
dx dy cos x − cos x
Put x − y = t ⇒ + y=
dx 2 + sin x 2 + sin x
dy dt dy dt
⇒ 1− = ⇒ =1 − which is a linear differential equation.
dx dx dx dx
cos x
dt dy ∫ dx
⇒ 1− =t 2
[ Q = (x − y)2] ∴ IF = e 2 + sin x
= elog ( 2 + sin x ) = 2 + sin x
dx dx
dt dt ∴Required solution is given by
= 1 − t2 ⇒ ∫
1 − t2 ∫
⇒ = dx
dx − cos x
y ⋅ (2 + sin x) = ∫ ⋅ (2 + sin x)dx + C
[separating the variables] 2 + sin x
1  1+ t  ⇒ y(2 + sin x) = − sin x + C
⇒ log e   =x+C
2  1 − t Also, y(0) = 1
 dx 1 a+x  ∴ 1(2 + sin 0) = − sin 0 + C
∫ 2 = log e + C
 a − x2
2 a a − x  ⇒ C =2
1  1 + x − y π
⇒ log e   =x+C [Q t = x − y] 2 − sin
2 − sin x  π 2 =1
2  1 − x + y ∴ y= ⇒ y  =
2 + sin x  2 π 3
Since, y = 1 when x = 1, therefore 2 + sin
2
1  1 + 0 dy 1
log e   =1 + C 5. =
2  1 + 0 dx 8 x 9 + x 4+ 9+ x
⇒ C = −1 [Q log 1 = 0]
1  1 + x − y ⇒ y= 4 + 9+ x + c
∴ log e   = x −1
2  1 − x + y
Now, y(0) = 7 + c
1−x+ y ⇒ c=0
⇒ − log e = 2(x − 1)
1+ x− y
y(256) = 4 + 9 + 16 = 4 + 5 = 3
1
[Q log = log x−1 = − log x] 13
1
x 6. Here, ∑  π (k − 1 )π  π kπ 
3. Given, f (xy) = f (x) ⋅ f ( y), ∀ x, y ∈ [0, 1] ...(i) k = 1 sin
 +  sin  + 
4 6  4 6
Putting x = y = 0 in Eq. (i), we get
Converting into differences, by multiplying and dividing
f (0) = f (0) ⋅ f (0)
 π k π   π (k − 1)π   π
⇒ f (0) [ f (0) − 1] = 0 by sin  +  − + , i.e. sin   .
 4 6  4 6   6
⇒ f (0) = 1 as f (0) ≠ 0
 π π  π π 
Now, put y = 0 in Eq. (i), we get sin  + k  −  + (k − 1) 
 4 6 4
13
6 
f (0) = f (x) ⋅ f (0) ∴ ∑ π  π π π π 
⇒ f (x) = 1 k =1 sin sin  + (k − 1)  sin  + k  
6  4 6 4 6 
rankershalt.com

342 Differential Equations

  π kπ  π π  (a) y = 2 ⇒
dy
=0
sin  4 + 6  cos  4 + (k − 1) 6   dx
 π π  π kπ  
− sin  + (k − 1)  cos  +  On putting in Eq. (i),
13
 4 6 4 6  
= 2∑ 02 − x (0) + y = 0
π π π π
k =1 sin  + (k − 1)  sin  + k  ⇒ y = 0 which is not satisfied.
4 6 4 6
dy
13 (b) y = 2x ⇒ =2
 π π π π 
= 2 ∑ cot  + (k − 1)  − cot  + k   dx
k =1   4 6   4 6 
On putting in Eq. (i),
  π  π π  (2)2 − x ⋅ 2 + y = 0
= 2 cot   − cot  +  
  4  4 6  ⇒ 4 − 2x + y = 0
⇒ y = 2x which is not satisfied.
  π π  π 2π  
+ cot  +  − cot  +  dy
  4 6 4 6  (c) y = 2x − 4 ⇒ =2
dx
 π π π π  
+ K + cot  + 12  − cot  + 13   On putting in Eq. (i)
 4 6 4 6  
(2)2 − x − 2 + y
 π π π 
= 2 cot − cot  + 13   4 − 2x + 2x − 4 = 0 [Q y = 2x − 4]
 4  4 6 
y = 2x − 4 is satisfied.
  29 π     5 π  (d) y = 2x2 − 4
= 2 1 − cot   = 2 1 − cot 2π + 
  12     12   dy
= 4x
 5π  5π  dx
= 2 1 − cot Q cot 12 = (2 − 3 )
 12  On putting in Eq. (i),
= 2 (1 − 2 + 3 ) (4x)2 − x ⋅ 4x + y = 0
= 2 ( 3 − 1) ⇒ y = 0 which is not satisfied.
dy 1 − y2 10. Given, y = (c1 + c2) cos (x + c3 ) − c4 ex + c5 …(i)
7. Given, =
dx y ⇒ y = (c1 + c2) cos (x + c3 ) − c4 e ⋅ e x c5

y c1 + c2 = A , c3 = B, c4ec5 = c
⇒ ∫ 1 − y2
dy = ∫ dx Now, let
⇒ y = A cos (x + B) − cex …(ii)
⇒ − 1 − y = x + c ⇒ (x + c) + y = 1
2 2 2
On differentiating w.r.t. x, we get
Here, centre (– c, 0) and radius = 1 dy
= − A sin ( A + B) − cex …(iii)
dy − cos x ( y + 1) dx
8. Given, =
dx 2 + sin x Again, on differentiating w.r.t. x, we get
dy − cos x d 2y
⇒ = dx = − A cos (x + B) − cex …(iv)
y + 1 2 + sin x dx2
d 2y
On integrating both sides ⇒ = − y − 2 cex …(v)
dy cos x dx2
∫ y + 1 = − ∫ 2 + sin x dx ⇒
d 2y
+ y = − 2 cex
dx2
⇒ log ( y + 1) = − log (2 + sin x) + log c
Again, on differentiating w.r.t. x, we get
When x = 0, y = 1 ⇒ c = 4
d3 y dy
⇒ y+1=
4 + = − 2 cex …(vi)
2 + sin x dx3 dx
d3 y dy d 2y
 π 4 ⇒ + = + y [from Eq. (v)]
∴ y  = −1 dx2 dx dx2
 2 3
which is a differential equation of order 3.
 π 1
⇒ y  =
 2 3 11. We have,
x
∫0 e
x −t
f ( x ) = 1 − 2x + f ( t ) dt
9. Given differential equation is
 dy
2
dy On multiplying e− x both sides, we get
  −x + y=0 …(i) x
 dx dx e− x f ( x ) = e− x − 2xe− x + ∫ e− t f ( t ) dt
0
rankershalt.com

Differential Equations 343

On differentiating both side w.r.t. x, we get Given, y (0) = 2


e− x f ′ ( x ) − e− x f ( x ) = − e− x − 2e− x + 2xe− x + e− x f ( x ) ⇒ 2 + e0 ⋅ 2 = 0 + C
⇒ f ′ ( x ) − 2 f ( x ) = 2x − 3 ⇒ C =4
[dividing both sides by e− x ] ∴ y (1 + ex ) = x + 4
Let f(x) = y
x+4
dy ⇒ y=
⇒ f ′ (x) = 1 + ex
dx
dy −4 + 4
∴ − 2 y = 2x − 3 Now at x = − 4, y = =0
dx 1 + e−4
which is linear differential equation of the form ∴ y(−4) = 0 …(i)
dy
+ Py = Q. Here, P = −2 and Q = 2x − 3. For critical points,
dy
=0
dx dx
IF = e∫ = e∫
−2 dx
= e −2x
P dx
Now, dy (1 + ex ) ⋅ 1 − (x + 4)ex
∴ Solution of the given differential equation is i.e. = =0
dx (1 + ex )2
y ⋅ e−2x = ∫ ( 2x − 3) e−2x dx + C
I II ⇒ ex (x + 3) − 1 = 0
− ( 2x − 3) ⋅ e−2x e −2x e− x = (x + 3)
y ⋅ e −2x = + 2∫ dx + C or
2 2
y = e–x Y y=x+3
[by using integration by parts]
−2x − ( 2x − 3) e−2x e−2x (–1, e)
⇒ y⋅e = − +C
2 2 (–1, 2)
⇒ y = (1 − x ) + Ce2x X′ X
–1 O
On putting x = 0 and y = 1, we get
1=1+C ⇒ C = 0
∴ y=1− x
y = 1 − x passes through ( 2, − 1) Y′

Now, area of region bounded by curve y = 1 − x2 and


Clearly, the intersection point lies between (− 1, 0).
y = 1 − x is shows as
∴ y(x) has a critical point in the interval (− 1, 0).
y
B (0, 1) 13. Since, centre lies on y = x.
y= 1–x2 ∴ Equation of circle is
x2 + y2 − 2ax − 2ay + c = 0
A
x′ x On differentiating, we get
–1,0 O (1,0)
2x + 2 yy′ − 2a − 2ay′ = 0
y=1–x
⇒ x + yy′ − a − ay′ = 0
y′
x + yy′
∴ Area of shaded region ⇒ a=
1 + y′
= Area of 1st quadrant of a
circle − Area of ∆ OAB Again differentiating, we get
π 1 (1 + y′ )[1 + yy′ ′+ ( y′ )2] − (x + yy′ ) ⋅ ( y′ ′ )
= (1)2 − × 1 × 1 0=
4 2 (1 + y′ )2
π 1 π −2
= − = ⇒ (1 + y′ ) [1 + ( y′ )2 + yy′ ′ ] − (x + yy′ ) ( y′′ ) = 0
4 2 4
⇒ 1 + y′ [( y′ )2 + y′ + 1] + y′′ ( y − x) = 0
Hence, options b and c are correct.
On comparing with Py′′ + Qy′ + 1 = 0, we get
12. Here, (1 + ex ) y′ + y ex = 1
P = y−x
dy dy
⇒ + ex ⋅ + yex = 1 and Q = ( y′ )2 + y′ + 1
dx dx
⇒ dy + ex dy + yex dx = dx 14. Given, y2 = 2c (x + c) …(i)
⇒ dy + d (e y) = dx
x On differentiating w.r.t. x, we get
dy dy
On integrating both sides, we get 2y = 2c ⇒ c = y
dx dx
y + ex y = x + C
rankershalt.com

344 Differential Equations

On putting this value of c in Eq. (i), we get 2 π π


At x = 2 , y = ; = +c
dy  dy  3 6 3
y2 = 2 y x + y  π
dx  dx  ⇒ c=−
3/ 2
6
dy  dy  π
⇒ y=2 ⋅ x + 2 y1/ 2   Now, y = sec sec−1 x − 
dx  dx  6
3/ 2
dy  dy  3
⇒ y − 2x =2 y   1
dx  dx = cos cos −1 − cos −1 
 x 2 
2 3
 dy  dy
⇒  y − 2x  = 4 y     3 1 3 
 dx  dx = cos cos −1  + 1− 2 1 − 
  2 x x 4  
Therefore, order of this differential equation is 1 and
degree is 3. 3 1 1
y= + 1− 2
15. We have, 2x 2 x
dy
= ( 2 + 5 y ) ( 5 y − 2) dP (x)
dx 17. Given, P(1) = 0 and − P (x) > 0, ∀ x ≥ 1 ...(i)
dx
 
dy 1  dy  On multiplying Eq. (i) by e− x , we get
⇒ = dx ⇒   = dx
25 y 2 − 4 25  y 2 − 4  e− x ⋅
d
P (x) ⋅
d −x
e >0
 25  dx dx
On integrating both sides, we get d
⇒ (P (x) ⋅ e−x ) > 0
1 dy dx
25 ∫  2
2
= ∫ dx
⇒ P (x) ⋅ e−x is an increasing function.
y − 2
⇒ P (x) ⋅ e− x > P (1) ⋅ e−1 , ∀ x ≥ 1
 5
1 1 y − 2/ 5 ⇒ P (x) > 0, ∀ x > 1 [Q P (1) = 0 and e−x > 0]
⇒ × log = x+C
25 2 × 2 y + 2/ 5 18. Equation of tangent to the curve y = f (x) at point
5 dy
5y − 2 A (x, y) is Y − y = (X − x)
⇒ log = 20( x + C ) dx
5y + 2 y

5y − 2 (1, 1)
⇒ = Ae20x [Q e20C = A]
5y + 2
Q
(x, y) y = f(x)
when x = 0 ⇒ y = 0, then A = 1 A
5 y −2 x ′' x
∴ = e20x O P
5y + 2
5 f ( x ) −2
lim = lim e20x y′
x → −∞ 5 f(x) + 2 x → −∞
 dx 
whose, x-intercept  x − y ⋅ , 0
5 f(x) − 2  dy 
⇒ lim = 0
n → −∞ 5 f(x) + 2  dy
y-intercept 0, y − x 
⇒ lim 5 f ( x ) −2 = 0  dx
n→−∞
Given, ∆OPQ = 2
2
⇒ lim f (x) = = 0.4 1  dx  dy
n→ − ∞ 5 ⇒ ⋅ x − y   y − x  = 2
2  dy  dx
dy y y − 1
2
16. Given, =  1 dy
dx x x2 − 1 ⇒  x − y  ( y − xp) = 4, where p=
 p dx
dy dx ⇒ p2x2 − 2 pxy + 4 p + y2 = 0
∫ y y −12
=∫
x x −12
⇒ ( y − px)2 + 4 p = 0
⇒ sec −1
y = sec −1
x+ c ∴ y − px = 2 − p
⇒ y = px + 2 − p …(i)
rankershalt.com

Differential Equations 345

On differentiating w.r.t. x, we get  1


⇒ e− x f ′ (x) − e− x f (x) < 0, x ∈ 0, 
dp  1 dp  4
p= p+ ⋅ x + 2 ⋅   ( − p)−1/ 2 ⋅ ( −1) 1
dx  2 dx ⇒ f ′ (x) < f (x) ,0 < x <
dp 4
⇒ { x − ( − p)−1/ 2 } = 0
dx 21. Here, f ′′ (x) − 2 f ′ (x) + f (x) ≥ ex
dp
⇒ = 0 or x = ( − p)−1/ 2 ⇒ f ′′ (x)e− x − f ′ (x)e− x − f ′ (x)e− x + f (x)e− x ≥ 0
dx
dp d d
If = 0 ⇒ p= c ⇒ { f ′ (x)e− x } − { f (x)e− x } ≥ 1
dx dx dx
On putting this value in Eq. (i), we get y = cx + 2 − c ⇒
d
{ f ′ (x)e− x − f (x)e− x } ≥ 1
This curve passes through (1, 1). dx
⇒ 1 = c + 2 −c d 2 −x
⇒ { e f (x)} ≥ 1, ∀x ∈ [0, 1]
⇒ c= −1 dx2
∴ y=−x+2 ∴ φ (x) = e− x f (x) is concave function.
⇒ x+ y=2 f (0) = f (1) = 0
Again, if x = (− p)−1/ 2 ⇒ φ(0) = 0 = f (1)
1
⇒ − p = 2 putting in Eq. (i) ⇒ φ (x) < 0
x
⇒ e− x f (x) < 0
−x 1
y= 2
+ 2⋅ ⇒ xy = 1 ∴ f (x) < 0
x x
22. Here, f (x) = (1 − x)2 ⋅ sin 2 x + x2 ≥ 0, ∀ x
Thus, the two curves are xy = 1 and x + y = 2.
x  2 (t − 1 ) 
19. From given integral equation, f (0) = 0 . and g (x) = ∫  − log t f (t )dt
1  t + 1 
Also, differentiating the given integral equation w.r.t. x
2(x − 1) 
f ′ (x) = f (x) ⇒ g′ (x) =  − log x ⋅ f{
(x) …(i)
 (x + 1 )  + ve
If f (x) ≠ 0
f ′ (x) For g′ (x) to be increasing or decreasing.
⇒ =1 ⇒ log f (x) = x + c
f (x) 2(x − 1)
Let φ(x) = − log x
⇒ f (x) = ecex x+1
4 1 − (x − 1)
Q f (0) = 0 ⇒ ec = 0 , a contradiction φ′ (x) = − =
(x + 1) 2
x x (x + 1)2
∴ f (x) = 0, ∀ x ∈ R
φ′ (x) < 0, ∀ x > 1
⇒ f (ln 5) = 0
⇒ φ (x) < φ (1)
Alternate Solution
x ⇒ φ(x) < 0 …(ii)
Given, f (x) = ∫ f (t ) dt
0 From Eqs. (i) and (ii), g′ (x) < 0, x ∈ (1, ∞ )
⇒ f (0) = 0 and f ′ (x) = f (x) ∴g (x) is decreasing on x ∈ (1, ∞ ).
If f (x) ≠ 0 23. Here, f (x) + 2x = (1 − x)2 ⋅ sin 2 x + x2 + 2x …(i)
f ′ (x)
⇒ = 1 ⇒ ln f (x) = x + c where, I: f (x) + 2x = 2(1 + x) 2
…(ii)
f (x)
∴ 2(1 + x2) = (1 − x)2 sin 2 x + x2 + 2x
⇒ f (x) = ec ⋅ ex
⇒ (1 − x)2 sin 2 x = x2 − 2x + 2
Q f (0) = 0
⇒ (1 − x)2 sin 2 x = (1 − x)2 + 1
⇒ ec = 0, a contradiction
⇒ (1 − x)2 cos 2 x = − 1
∴ f (x) = 0, ∀ x ∈ R
which is never possible.
⇒ f (ln 5) = 0
∴I is false.
20. Let φ(x) = e−x f (x) Again, let h (x) = 2 f (x) + 1 − 2x(1 + x)
 1 h (0) = 2 f (0) + 1 − 0 = 1
Here, φ′ (x) < 0, x ∈ 0,  where,
 4
h(1) = 2(1) + 1 − 4 = − 3 as [h (0)h (1) < 0]
1  ⇒ h (x) must have a solution.
and φ′ (x) > 0, x ∈  , 1
4 
∴ II is true.
rankershalt.com

346 Differential Equations

1 − 1/ y
Topic 2 Linear Differential Equation and ⇒ x e− 1/ y = e + e− 1/ y + C … (i)
y
Exact Differential Equation Now, at y = 1, the value of x = 1, so
1. Given differential equation is 1
1 ⋅ e− 1 = e− 1 + e− 1 + C ⇒ C = −
( y2 − x3 ) dx − xy dy = 0, (x ≠ 0) e
dy
⇒ xy − y2 = − x3 On putting the value of C, in Eq. (i), we get
dx
1 e1/ y
dy dt dy 1 dt x= +1−
Now, put y2 = t ⇒ 2 y = ⇒y = y e
dx dx dx 2 dx
x dt 1 e1/ 2 3 1
∴ − t = − x3 So, at y = 2, the value of x = +1− = −
2 dx 2 e 2 e
dt 2
⇒ − t = − 2x2 3. Given differential equation is
dx x
dy
which is the linear differential equation of the form + y tan x = 2x + x2 tan x , which is linear differential
dt dx
+ Pt = Q. dy
dx equation in the form of + Py = Q .
2 dx
Here, P = − and Q = − 2x2.
x Here, P = tan x and Q = 2x + x2 tan x
∴IF = e∫ tan x dx = elog e (sec x ) = sec x
2
−∫ dx 1
Now, IF = e x =
x2 Now, solution of linear differential equation is given as
Q Solution of the linear differential equation is y × IF = ∫ (Q × IF)dx + C
(IF) t = ∫ Q (IF)dx + λ [where λ is integrating constant]
∴ y(sec x) = ∫ (2x + x2 tan x) sec x dx + C
t  2  = − 2 ∫  x2 × 2  dx + λ
1 1

x   x  = ∫ (2x sec x) dx + ∫x
2
sec x tan x dx + C
t
⇒ = − 2x + λ Q ∫ x2 sec x tan x dx = x2 sec x − ∫ (2x sec x) dx
x2
y2
⇒ + 2x = λ [Q t = y2] Therefore, solution is
x2
y sec x = 2∫ x sec x dx + x2 sec x − 2∫ x sec x dx + C
⇒ y2 + 2x3 − λx 2 = 0
or y2 + 2x3 + Cx 2 = 0 [let C = − λ] ⇒ y sec x = x2 sec x + C …(i)
Q y(0) = 1 ⇒ 1(1) = 0(1) + C ⇒ C = 1
2. Given differential equation is Now, y = x2 + cos x [from Eq. (i)]
 1 and y′ = 2x − sin x
y2dx +  x −  dy = 0
 y According to options,
 π  − π   π 1

dx 1 1
+ 2 x = 3 , which is the linear differential y′   − y′   = 2  − 
dy y  4  4    4 2
y
dx   π 1
equation of the form + Px = Q. − 2 −  +  =π− 2
dy   4 2
1 1  π  π   π 1    π 1
Here, P = and Q = 3 and y′   + y′  −  = 2  −  + 2 −  +  =0
y2 y  4  4   4 2   4 2
 π  π π π2 1 π2
1 1 2
∫ 2 dy − 1
and y  + y −  = + + + = + 2
Now, IF = e y
=e y
 4  4  16 2 16 2 4
∴The solution of linear differential equation is  π  π π π2
2
1 1
and y  − y −  = + − − =0
x ⋅ (IF) ∫ Q (IF)dy + C  4  4  16 2 16 2

1 − 1/ y 4. Given differential equation


⇒ x e− 1/ y = ∫ e dy + C dy
y3 = (tan x − y)sec2 x
dx
1 1
∴ x e− 1/ y = ∫ (− t ) et dt + C [Qlet − = t ⇒ + 2 dy = dt] ⇒
dy
+ (sec2 x) y = sec2x tan x,
y y dx
= − tet + ∫ e dt + C
t
[integration by parts] which is linear differential equation of the form
dy
= − tet + et + C + Py = Q,
dx
where P = sec2 x and Q = sec2 x tan x
rankershalt.com

Differential Equations 347

2
IF = e∫
sec x dx which is a linear differential equation of the form
= etan x
dy
So, solution of given differential equation is + Py = Q
dx
y × IF = ∫ (Q × IF)dx + C
2
Here, P = and Q = x
y(etan x ) = ∫ etan x ⋅ sec2 x tan x dx + C x
Let tan x = t ⇒ sec2x dx = dt 2
∫ dx
yetan x = ∫ et ⋅ t dt + C = tet − ∫ et dt + C ∴ IF = e x = e2log x = x2
[using integration by parts method] Since, solution of the given differential equation is
= et (t − 1) + C
y × IF = ∫ (Q × IF) dx + C
⇒ y ⋅ etan x = etan x (tan x − 1) + C [Q t = tan x]
Q y(0) = 0 x4
∴ y(x2) = ∫ (x × x2) dx + C ⇒ yx2 = +C
⇒ 0 = 1(0 − 1) + C ⇒ C =1 4
∴ y ⋅ etan x = etan x (tan x − 1) + 1 1 3
Q y(1) = 1, so 1 = + C ⇒C =
π 4 4
Now, at x = −
4 x 4
3 x 2
3
ye−1 = e−1 (−1 − 1) + 1 ∴ yx2 = + ⇒y= + 2
4 4 4 4x
⇒ ye−1 = − 2e−1 + 1⇒ y = e − 2
7. Given differential equation is
dy
(x2 + 1)2 + 2x(x2 + 1) y = 1
5. Key Idea (i) First convert the given differential equation into
dy dx
linear differential equation of the form + Py = Q
dx dy  2x  1
⇒ +  y =
(ii) Find IF dx  1 + x2  (1 + x2 )2
(iii) Apply formula, y( IF) = ∫ Q( IF) dx + C [dividing each term by (1 + x2 )2] …(i)
This is a linear differential equation of the form
Given differential equation
dy
dy + P⋅y =Q
cos x − (sin x) y = 6x dx
dx
2x 1
dy 6x Here, P = and Q =
⇒ − (tan x) y = , which is the linear (1 + x2 ) (1 + x2 )2
dx cos x
2x
differential equation of the form ∫ x2
dx
dy ∴Integrating Factor (IF) = e 1 +
+ Px = Q,
x2)
dx = eln(1 + = (1 + x2 )
6x
where P = − tan x and Q = and required solution of differential Eq. (i) is given by
cos x
So, IF = e ∫
− tanx dx
= e− log(sec x ) = cos x
y ⋅ (IF) = ∫ Q (IF)dx + C
∴Required solution of differential equation is 1
cos x x2
⇒ y(1 + x2 ) = ∫
(1 + x2 )2
(1 + x2 )dx + C
y(cos x) = ∫ (6x) dx + C = 6 + C = 3x2 + C
dx
cos x 2 ⇒ y(1 + x2 ) = ∫ +C
 π 1 + x2
Given, y  = 0
 3 ⇒ y(1 + x2 ) = tan −1 (x) + C
2
π π
2
y(0) = 0
0 = 3  + C ⇒C = −
Q
So,
 3 3 ∴ C=0
π 2
∴ y(1 + x2 ) = tan −1 x [Q C = 0]
∴ y(cos x) = 3x2 −
3 tan −1 x
π ⇒ y=
Now, at x = 1 + x2
6  tan −1 x 
 3 π2 π2 π2 π2 ⇒ ay = a  
y  =3 − =− ⇒ y=−  1 + x2 
 2  36 3 4 2 3
[multiplying both sides by a]
6. Given differential equation is Now, at x = 1
dy
+ 2 y = x2, (x ≠ 0)
x π
dx  tan −1 (1)  aπ π
a y (1) = a  = a 4 = = (given)
dy  2  1+ 1  2 8 32
⇒ +   y = x,
dx  x 1 1
∴ a= ⇒a =
4 16
rankershalt.com

348 Differential Equations

8. We know that, slope of the tangent at any point (x, y) on From Eqs. (iii) and (iv), we get
the curve is C =0
dy x2 − 2 y So, required solution is
= (given)
dx x x2 x2
yx = log e x −
dy 2 2 4
⇒ + y=x …(i)
dx x e2 e2
Now, at x = e, ey(e) = log e e −
which is a linear differential equation of the form 2 4
dy
+ P (x) ⋅ y = Q (x), [where, y(e) represents value of y at x = e]
dx e
2 ⇒ y(e) = [Q log e e = 1].
where P (x) = and Q (x) = x 4
x
Now, integrating factor dy  2x + 1 − 2x
10. We have, + y=e
2
∫ dx dx  x 
(IF) = e∫
P ( x )dx
= e x = e2log e x dy
2 which is of the form + Py = Q, where
= elog e x [Q m log a = log am ] dx
2x + 1
= x2 [Q elog e f ( x ) = f (x)] P= and Q = e−2x
x
and the solution of differential Eq. (i) is  1 + 2x  1 
∫  dx ∫  + 2 dx
Now, IF = e∫ =e  x 
= e x 
Pdx
y(IF) = ∫ Q (x)(IF)dx + C ⇒ y(x ) = ∫ x ⋅ x dx + C 2 2

= eln x + 2x = eln x . e2x = x. e2x


x4
⇒ yx = +C 2
…(ii) and the solution of the given equation is
4
y ⋅ (IF) = ∫ (IF ) Q dx + C
Q The curve (ii) passes through the point (1, − 2),
therefore ⇒ y(xe2x ) = ∫ (x e2x . e−2x ) dx + C
1 9
−2 = + C ⇒C = − x2
4 4 = ∫ x dx + C = +C … (i)
2
∴ Equation of required curve is 4 yx = x4 − 9.
2
1 −2
Now, checking all the option, we get Since, y = e when x = 1
2
only ( 3 , 0) satisfy the above equation. 1 −2 2 1
∴ e . e = + C ⇒ C = 0 (using Eq. (i))
9. Given differential equation is 2 2
dy x2 x
+ y = x log e x, (x > 1)
x ∴ y (xe ) =
2x
⇒ y = e−2x
dx 2 2
dy 1 −2x x −2x 1 

dy 1
+ y = log e x …(i) Now, = e + e (− 2) = e−2x  − x < 0,
dx x dx 2 2 2 
1
Which is a linear differential equation. if < x < 1 [by using product rule of derivative]
1 2
∫ dx
So, if = e x = elog e x = x and y(log e 2) =
log e 2 −2log e 2 1
e = log e 2 elog e 2
−2

Now, solution of differential Eq. (i), is 2 2


1 −2 1
= . log e 2 ⋅ 2 = log e 2
y × x = ∫ (log e x) x dx + C 2 8
3 f (x)
x2 x2 1 11. Given, f ′ (x) = 7 − , (x > 0)
⇒ yx = log e x − ∫ × dx + C 4 x
2 2 x
dy
[using integration by parts] On putting f (x) = y and f ′ (x) = , then we get
dx
x2 x2 dy 3 y
⇒ yx = log e x − +C … (ii) = 7−
2 4 dx 4 x
Given that, 2 y(2) = log e 4 − 1 … (iii) dy 3
⇒ + y=7 …(i)
On substituting, x = 2, in Eq. (ii), dx 4x
we get which is a linear differential equation of the form
4 4 dy 3
2 y(2) = log e 2 − + C, + Py = Q, where P = and Q = 7.
2 4 dx 4x
[where, y(2) represents value of y at x = 2] ∫ dx
3
Now, integrating factor (IF) = e 4x
⇒ 2 y(2) = log e 4 − 1 + C … (iv) 3
log x
[Q m log a = log am ]
3/ 4
= e4 = elog x = x3/ 4
rankershalt.com

Differential Equations 349

and solution of differential Eq. (i) is given by 13. Given differential equation can be rewritten as
dy  2
y(IF) = ∫ (Q ⋅ (IF))dx + C +   ⋅ y = x, which is a linear differential equation of
dx  x
yx3/ 4 = ∫ 7x3/ 4dx + C dy 2
the form + Py = Q, where P = and Q = x.
3 dx x
+1
x4 Now, integrating factor
⇒ yx 3/ 4
=7 +C 2
3 ∫
+1 dx 2
(IF) = e x = e2log x = elog x = x2
4
7 [Q elog f ( x ) = f (x) ]
⇒ y x3/ 4 = 4x 4 + C and the solution is given by
⇒ y = 4x + C x−3/ 4 y(IF ) = ∫ (Q × IF ) dx + C
So, y = f (x) = 4x + C ⋅ x−3/ 4
⇒ yx2 = ∫ x3 dx + C
 1 4
Now, f  = + C ⋅ x3/ 4
 x x x4
⇒ yx2 = +C …(i)
 1 4 
∴ lim x f   = lim x + Cx3/ 4 = lim (4 + Cx7/ 4 ) = 4 4
x→ 0 +  x x→ 0 +  x  x→ 0 + Since, it is given that y = 1 when x = 1
12. Given, differential equation is ∴ From Eq. (i), we get
dy  3  1 1 3
+ y= , which is a linear differential 1 = + C ⇒C = …(ii)
dx  cos 2 x cos 2 x 4 4
dy 3 ∴ 4x2y = x4 + 3 [using Eqs. (i) and (ii)]
equation of the form + Py = Q, where P = and
dx cos 2 x x4 + 3
1 ⇒ y=
Q= . 4x2
cos 2 x 1
Now, Integrating factor +3
 1 16 49
3
∫ 2 dx Now, y  = =
 2
IF = e cos x = e∫
3 sec 2 x dx 1 16
= e3 tan x and the solution of 4×
4
differential equation is given by
y(IF) = ∫ (Q. (IF)) dx 14. We have,
dy dy
sin x + y cos x = 4x ⇒ + y cot x = 4xcosec x
⇒ e3 tan x . y = ∫ e3 tan x sec2 x dx … (i) dx dx
This is a linear differential equation of form
dy
Let I = ∫ e3 tan x sec2 x dx + Py = Q
dx
Put 3 tan x = t where P = cot x, Q = 4x cosec x
⇒ 3 sec2 x dx = dt
Now, IF = e∫ Pdx = e∫
cotxdx
= elogsin x = sin x
et et e3 tan x
∴ I = ∫ dt = +C= +C Solution of the differential equation is
3 3 3
y ⋅ sin x = ∫ 4x cosec x sin xdx + C
From Eq. (i)
e3 tan x ⇒ y sin x = ∫ 4xdx + C = 2x2 + C
e3 tan x . y = +C
3 π
Put x = , y = 0, we get
It is given that when, 2
π 4 π2 π2
x = , y is C=− ⇒ y sin x = 2x2 −
4 3 2 2
3 4 e3 π
⇒ e = +C Put x=
3 3 6
⇒ C = e3  1  π 2 π 2
∴ y  = 2  −
e3 tan x  2  36  2
Thus, e3 tan x y = + e3
3 π2 8π 2
⇒ y= − π2 ⇒ y = −
π e−3 9 9
Now, when x = − , e−3 y = + e3
4 3 Alternate Method
dy
1   π  We have, sin x + y cos x = 4x, which can be written as
⇒ y = e6 + Q tan  − 4  = − 1 dx
3   d
(sin x ⋅ y) = 4x
dx
rankershalt.com

350 Differential Equations

On integrating both sides, we get At x=1 ⇒ c=2


d ⇒ y ⋅ log x = 2 [x log x − x] + 2
∫ dx (sin x ⋅ y) ⋅ dx = ∫ 4x ⋅ dx
At x = e, y = 2(e − e) + 2
4 x2
⇒ y ⋅ sin x = + C ⇒ y ⋅ sin x = 2x2 + C ⇒ y=2
2
π dy
17. PLAN (i) Solution of the differential equation + Py = Q is
Now, as y = 0 when x = dx
2
y ⋅ (IF) = ∫ Q ⋅ (IF) dx + c
π2
∴ C=− where, IF = e ∫
P dx
2
a a
2 π
2
⇒ y ⋅ sin x = 2x −
(ii) ∫− a f( x ) dx = 2 ∫0 f( x ) dx , if f( − x ) = f( x )
2
π Given differential equation
Now, putting x = , we get
6 dy x x4 + 2 x
 2 + 2 y=
 
1 π π2 π2 8π 2 dx x − 1
y  = 2  − ⇒ y= − π2 = − 1 − x2
 2  36  2 9 9
This is a linear differential equation.
15. Given differential equation is x
∫ x2 − 1
dx 1
ln | x 2 − 1 |
y(1 + xy) dx = x dy IF = e = e2 = 1 − x2
⇒ y dx + xy2 dx = x dy x(x3 + 2)
⇒ Solution is y 1 − x2 = ∫ ⋅ 1 − x2 dx
x dy − y dx 1−x 2
⇒ = x dx
y2 x5
or y 1 − x2 = ∫ (x4 + 2x) dx = + x2 + c
( y dx − x dy)  x 5
⇒ − = x dx ⇒ − d   = x dx
y 2
 y x5
f (0) = 0 ⇒ c = 0 ⇒ f (x) 1 − x2 = + x2
On integrating both sides, we get 5
3 /2 3 /2 x2
x x2 Now, ∫ f (x) dx = ∫ dx
− = +C …(i) − 3 /2 − 3 /2
1 − x2
y 2
[using property]
Q It passes through (1, − 1). 3 /2 x2
1 1 =2 ∫ dx
∴ 1= +C ⇒ C = 0
1 − x2
2 2
x x2 1 π /3 sin 2 θ
Now, from Eq. (i) − = + =2 ∫ cos θ dθ [taking x = sin θ ]
y 2 2 0 cos θ
2x π /3 π /3
⇒ x2 + 1 = − =2 ∫ sin 2 θ dθ = ∫ (1 − cos 2θ ) dθ
y 0 0
π /3
2x  sin 2θ  π sin 2π / 3 π 3
⇒ y=− 2 = θ −  = − = −
x +1  2 0 3 2 3 4
 1 4 18. PLAN Whenever we have linear differential equation containing
∴ f −  =
 2 5 inequality, we should always check for increasing or
decreasing,
16. Given differential equation is dy dy
i.e. for + Py < 0 ⇒ + Py > 0
dy dx dx
(x log x) + y = 2x log x
Multiply by integrating factor, i.e. e ∫
Pdx
dx and convert into
dy y total differential equation.
⇒ + =2 Here, f ′ (x) < 2 f (x), multiplying by e− ∫ 2dx
dx x log x
d
This is a linear differential equation. f ′ (x) ⋅ e−2x − 2e−2x f (x) < 0 ⇒ ( f (x) ⋅ e−2x ) < 0
dx
1
∫ x log x dx 1 
∴ IF = e = elog(log x ) = log x ∴ φ (x) = f (x)e−2x is decreasing for x ∈  , 1
2 
Now, the solution of given differential equation is given
1
by Thus, when x >
2
y ⋅ log x = ∫ log x ⋅ 2dx
 1  1
φ (x) < φ   ⇒ e−2x f (x) < e−1 ⋅ f  
⇒ y ⋅ log x = 2∫ log xdx  2  2
⇒ y ⋅ log x = 2 [x log x − x] + c
rankershalt.com

Differential Equations 351

 1 Since, y (0) = − 1
⇒ f (x) < e2x −1 ⋅ 1, given f   = 1
 2 ⇒ −1 ⋅ e0 (1 + 0) = − e0 + c
1 1
⇒ 0<∫ f (x) dx < ∫ e2x − 1 dx c=0
1/ 2 1/ 2
1 1
1 ∴ y=− ⇒ y (1) = −
1  e2x − 1  (1 + t ) 2
⇒ 0<∫ f (x) dx <  
1/ 2  2  1/ 2 f (x)
22. Here, f ′ (x) = 2 −
1 e−1 x
⇒ 0 < ∫ f (x) dx <
1/ 2 2 dy y
or + = 2 [i.e. linear differential equation in y]
t f (x) − x f (t )
2 2 dx x
19. Given, lim =1 1
t→ x t−x ∫ dx
Integrating Factor, IF = e x = elog x = x
⇒ x2f ′ (x) − 2x f (x) + 1 = 0
∴ Required solution is y ⋅ (IF) = ∫ Q (IF)dx + C
x2f ′ (x) − 2x f (x) 1
⇒ + 4 =0
(x2)2 x ⇒ y(x) = ∫ 2(x) dx + C
d  f (x) 1 ⇒ yx = x2 + C
⇒   =− 4
dx  x2  x C
∴ y=x+ [QC ≠ 0, as f (1) ≠ 1]
On integrating both sides, we get x
1  1
f (x) = cx2 + (a) lim f ′   = lim (1 − Cx2) = 1
3x x→ 0 +  x x → 0 +
2 ∴ Option (a) is correct.
Also, f (1) = 1, c=
3  1
2 1 (b) lim x f   = lim (1 + Cx2) = 1
Hence, f (x) = x2 + x → 0+  x x → 0 +
3 3x
∴ Option (b) is incorrect.
20. Given, x dy = y(dx + y dy), y > 0 (c) lim x2f ′ (x) = lim (x2 − C ) = − C ≠ 0
x → 0+ x → 0+
⇒ x dy − y dx = y2dy
∴ Option (c) is incorrect.
x dy − y dx  x
⇒ = dy ⇒ d   = − dy C
y2  y (d) f (x) = x + , C ≠ 0
x
On integrating both sides, we get For C > 0, lim f (x) = ∞
x x → 0+
=− y+ c …(i) ∴ Function is not bounded in (0, 2).
y
∴ Option (d) is incorrect.
Since, y (1) = 1 ⇒ x = 1, y = 1
23. PLAN Linear differential equation under one variable.
∴ c=2
+ Py = Q; IF = e ∫
dy Pdx
x
Now, Eq. (i) becomes, + y = 2 dx
y ∴ Solution is, y (IF) = ∫ Q ⋅ (IF) dx + C
Again, for x = − 3 y ′ − y tan x = 2x sec x and y (0) = 0
⇒ –3 + y2 = 2 y dy
⇒ − y tan x = 2x sec x
⇒ y2 − 2 y − 3 = 0 dx
⇒ ( y + 1) ( y − 3) = 0 ∴ IF = ∫ e− tan x dx = elog|cos x| = cos x
As y > 0, take y = 3, neglecting y = − 1. Solution is y ⋅ cos x = ∫ 2x sec x ⋅ cos x dx + C
dy  t  1
21. Given, −  y= and y (0) = − 1 ⇒ y ⋅ cos x = x2 + C
dt  1 + t  (1 + t )
As y(0) = 0 ⇒ C = 0
Which represents linear differential equation of first ∴ y = x2 sec x
order.
   π π2
t
∫ −  1 + t  dt Now, y  =
∴ IF = e  = e−t + log (1 + t ) = e− t ⋅ (1 + t )  4 8 2
Required solution is,  π π π2
⇒ y′   = +
1  4
ye− t (1 + t ) = ∫ ⋅ e− t (1 + t ) dt + c = ∫ e− t dt + c 2 8 2
1+ t
 π  2π  π  4π 2π
2 2
−t −t y  = ⇒ y′   = +
⇒ ye (1 + t ) = − e +c  3 9  3 3 3 3
rankershalt.com

352 Differential Equations

24. Let w (x) = u (x) − v (x) …(i) Topic 3 Applications of Homogeneous


and h (x) = f (x) − g (x) Differential Equations
On differentiating Eq. (i) w.r.t. x dy 2 y
dw du dv 1. Given, =
= − dx x2
dx dx dx dy 2
= { f (x) − p (x) ⋅ u (x)} − { g (x) − p (x) v (x)} [given]
⇒ ∫ y = ∫ x2dx [integrating both sides]

= { f (x) − g (x)} − p (x) [u (x) − v (x)] ⇒


2
log e| y| = − + C …(i)
dw x
⇒ = h (x) − p (x) ⋅ w (x) …(ii)
dx Since, curve (i) passes through centre (1, 1) of the circle
dw x2 + y2 − 2x − 2 y = 0
⇒ + p (x) w (x) = h (x) which is linear differential
dx 2
equation . ∴ log e (1) = − + C ⇒ C = 2
1
The integrating factor is given by ∴ Equation required curve is
IF = e∫
p ( x ) dx
= r (x) [let] 2
log e| y| = − + 2 [put C = 2 in Eq. (i)]
x
On multiplying both sides of Eq. (ii) of r (x), we get
dw ⇒ x log e| y| = 2(x − 1)
r (x) ⋅ + p (x) (r (x)) w (x) = r (x) ⋅ h (x)
dx 2. Given differential equation is
d  dr  (x2 − y2)dx + 2xy dy = 0, which can be written as
⇒ [r (x) w (x)] = r (x) ⋅ h (x) Q dx = p (x) ⋅ r (x)
dx dy y2 − x2
=
r (x) = e∫ dx 2xy
P ( x ) dx
Now, > 0, ∀ x
Put y = vx [Q it is in homogeneous form]
and h (x) = f (x) − g (x) > 0, for x > x1
dy dv
d ⇒ = v+ x
Thus, [r (x) w (x)] > 0, ∀ x > x1 dx dx
dx
Now, differential equation becomes
r (x) w (x) increases on the interval [x, ∞ [ dv v2x2 − x2 dv (v2 − 1)x2
Therefore, for all x > x1 v+ x = ⇒ v+ x =
dx 2x(vx) dx 2vx2
r (x) w (x) > r (x1 ) w (x1 ) > 0 dv v2 − 1 v2 − 1 − 2v2
⇒ x = −v=
[Q r (x1 ) > 0 and u (x1) > v (x1)] dx 2v 2v
⇒ w (x) > 0 ∀ x > x1 dv 1 + v2 2v dv dx
⇒ x =− ⇒ ∫ = −∫
⇒ u (x) > v (x) ∀ x > x1 [Q r (x) > 0] dx 2v 1+ v 2
x
Hence, there cannot exist a point (x, y) such that x > x1 ⇒ ln (1 + v2) = − ln x − ln C
and y = u (x) and y = v (x).  f ′ (x) 
Q ∫ f (x) dx ⇒ ln| f (x)|+ C 
dy  
25. + y ⋅ g′ (x) = g (x) g′ (x)
dx ⇒ ln|(1 + v2)Cx|= 0 [Q ln A + ln B = ln AB]
IF = e∫
g ′( x ) dx
= e g( x ) ⇒ (1 + v )Cx = 1
2
[log e x = 0 ⇒ x = e0 = 1]
∴ Solution is y (e g( x ) ) = ∫ g (x) ⋅ g′ (x) ⋅ e g( x ) dx + C y
Now, putting v = , we get
x
Put g (x) = t, g′ (x) dx = dt
 y2 
y(eg( x ) ) = ∫ t ⋅ et dt + C 1 + 2  Cx = 1 ⇒ C (x2 + y2) = x
 x 
= t ⋅ et − ∫ 1 ⋅ et dt + C = t ⋅ et − et + C Q The curve passes through (1, 1), so
g( x )
= ( g (x) − 1) e +C g( x ) 1
ye …(i) C (1 + 1) = 1 ⇒ C =
Given, y(0) = 0, g (0) = g (2) = 0 2
∴ Eq. (i) becomes, Thus, required curve is x2 + y2 − 2x = 0, which represent
a circle having centre (1, 0)
y(0) ⋅ eg( 0) = ( g (0) − 1) ⋅ eg( 0) + C
⇒ 0 = (− 1) ⋅ 1 + C ⇒ C = 1 ∴ The solution of given differential equation represents
a circle with centre on the X-axis.
∴ y(x) ⋅ eg( x ) = ( g (x) − 1) eg( x ) + 1
dp 1
⇒ y(2) ⋅ eg( 2) = ( g (2) − 1) eg( 2) + 1, where g(2) = 0 3. Given, differential equation is − p(t ) = −200 is a
dt 2
⇒ y(2) ⋅ 1 = (− 1) ⋅ 1 + 1
linear differential equation.
y(2) = 0
rankershalt.com

Differential Equations 353

−1 dy
Here, p(t ) = , Q (t ) = −200 ⇒ [(x + 2)2 + y(x + 2)] − y2 = 0
2 dx
 1 t Put x + 2 = X and y = Y , then
∫ −   dt −
IF = e  2 = e 2 dY
(X 2 + XY ) −Y 2 = 0
Hence, solution is dX
p (t ) ⋅ IF = ∫ Q (t ) ⋅ IF dt ⇒ X 2dY + XYdY − Y 2dX = 0
t t ⇒ X 2dY + Y (XdY − YdX ) = 0
− −
p(t )⋅ e 2 = ∫ −200 ⋅ e 2dt dY XdY − YdX
⇒ − =
t t
Y X2
− −
p(t )⋅ e 2 = 400 e 2 +K Y 
⇒ − d (log|Y |) = d  
 X
⇒ p(t ) = 400 + ke−1/ 2
If p(0) = 100, then k = − 300 On integrating both sides, we get
t Y
⇒ p(t ) = 400 − 300 e2 − log|Y| = + C, where x + 2 = X and y = Y
X
4. PLAN To solve homogeneous differential equation, i.e. substitute y
y ⇒ − log| y| = +C …(i)
=v x+ 2
x
dy dv Since, it passes through the point (1, 3).
∴ y = vx ⇒ =v +x
dx dx ∴ − log 3 = 1 + C
Here, slope of the curve at (x, y) is ⇒ C = − 1 − log 3 = − (log e + log 3)
dy y  y
= + sec   = − log 3e
dx x  x
∴ Eq. (i) becomes
y
Put =v y
x log| y| + − log (3e) = 0
x+2
dv dv
∴ v+ x = v + sec (v) ⇒ x = sec (v) | y| y
dx dx ⇒ log   + =0 …(ii)
 3e  x + 2
dv dx dx
⇒ ∫ sec v = ∫ x ⇒ ∫ cos v dv = ∫ x Now, to check option (a), y = x + 2 intersects the curve.
 y |x + 2| x + 2 |x + 2|
⇒ sin v = log x + log c ⇒ sin   = log(cx) ⇒ log  + = 0 ⇒ log   = −1
 x  3e  x + 2  3e 
 π  π |x + 2| 1
As it passes through 1,  ⇒ sin   = log c ⇒ = e−1 =
 6  6 3e e
1 ⇒ |x + 2| = 3 or x + 2 = ± 3
⇒ log c =
2
∴ x = 1, − 5 (rejected), as x > 0 [given]
 y 1
∴ sin   = log x + ∴ x = 1 only one solution.
 x 2
Thus, (a) is the correct answer.
dP
5. Given, = (100 − 12 x ) ⇒dP = (100 − 12 x ) dx To check option (c), we have
dx
| y| y
On integrating both sides, we get y = (x + 2)2 and log   + =0
 3e  x + 2
∫ dP = ∫ (100 − 12 x ) dx
|x + 2|2  (x + 2 )2 |x + 2|2 
⇒ log  + = 0 ⇒ log   = − (x + 2)
P = 100x − 8x 3/ 2
+C  3e  x+2  3e 
When x = 0, then P = 2000 ⇒ C = 2000 (x + 2)2 3e
Now, when x = 25, then is ⇒ = e−( x + 2) or (x + 2)2 ⋅ ex + 2 = 3e ⇒ex+ 2 =
3e (x + 2)2
P = 100 × 25 − 8 × (25)3/ 2 + 2000 Y
e x +2
= 2500 − 8 × 125 + 2000
= 4500 − 1000 = 3500 e2
dy
6. Given, (x2 + xy + 4x + 2 y + 4) − y2 = 0 3e /4
dx 3e /( x + 2)2
dy X
⇒ [(x2 + 4x + 4) + y(x + 2)] − y2 = 0 O
dx
rankershalt.com

354 Differential Equations

Clearly, they have no solution. dr


= − λ is required differential equation.
To check option (d), y = (x + 3) 2 dt
|x + 3|2  (x + 3)2  dx
2
i.e. log  + =0 9. Since, the length of tangent = y 1 +   =1
 3e  (x + 2)  dy
To check the number of solutions.   dx 
2
⇒ y2 1 +    = 1
(x + 3) 2
  dy 
Let g (x) = 2 log (x + 3) + − log (3e) 
(x + 2)
dy y
 (x + 2) ⋅ 2 (x + 3) − (x + 3)2 ⋅ 1 ∴ =±
2 dx 1 − y2
∴ g′ (x) = +  −0
x+ 3  (x + 2)2 
1 − y2
=
2
+
(x + 3)(x + 1) ⇒ ∫ y
dy = ± ∫ x dx
x+3 (x + 2)2
Clearly, when x > 0, then, g′ (x) > 0 1 − y2
⇒ ∫ y
dy = ± x + C
∴ g (x) is increasing, when x > 0.
Thus, when x > 0, then g (x) > g (0) Put y = sin θ ⇒ dy = cos θ dθ
cos θ
 3 9
g (x) > log   + > 0 ∴ ∫ sin θ ⋅ cos θ dθ = ± x + C
 e 4
cos 2 θ
Hence, there is no solution. Thus, option (d) is true. ⇒ ∫ sin 2 θ
⋅ sin θ dθ = ± x + C
7. Since, BP : AP = 3 : 1. Then, equation of tangent is
Again put cos θ = t ⇒ − sin θ dθ = dt
Y − y = f ′ (x) (X − x)
t2
The intercept on the coordinate axes are ∴ −∫ dt = ± x + C
1 − t2
 y 
A x − , 0
 f ′ (x)   1 
⇒ ∫ 1 −

 dt = ± x + C
1 − t 2
and B [0, y − x f ′ (x)]
1+ t
Since, P is internally intercepts a line AB, ⇒ t − log =± x+C
1−t
 y 
3 x −  + 1 ×0
 f ′ (x) 1 + 1 − y2
∴ x= ⇒ 1 − y2 − log =±x+C
3+1 1 − 1 − y2
Y
(1, 1) 10. Given, liquid evaporates at a rate proportional to its
B
3 (x, y) surface area.
P y = f (x) dV
⇒ ∝ −S …(i)
1 dt
X
A 1 2
We know that, volume of cone = πr h
dy y dy 1 3
⇒ = ⇒ =− dx
dx − 3x y 3x R
On integrating both sides, we get
xy3 = c
Since, curve passes through (1, 1), then c = 1. r
H
∴ xy = 1
3

1 h
At x= ⇒ y=2
8
Hence, (a) and (c) are correct answers.
8. Since, rate of change of volume ∝ surface area and surface area = πr 2
1

dV
∝ SA or V = πr 2h and S = πr 2 …(ii)
dt 3
R r
⇒ 4 πr 2 ⋅
dr
= − λ 4 πr 2 Where, tan θ = and = tan θ …(iii)
dt H h
rankershalt.com

Differential Equations 355

From Eqs. (ii) and (iii), we get −2πr3 0

1
⇒ T=
A (0.6) 2 gr ∫1 [1 − (1 + t14 − 2t12)] dt1
V = πr3 cot θ and S = πr 2
…(iv)
3
−2πr3 0
On substituting Eq. (iv) in Eq. (i), we get ⇒ T=
A (0.6) 2 gr ∫1 [1 − 1 − t14 + 2t12] dt1
1 dr
cot θ ⋅ 3r 2 = − kπ r 2 2πr3 0
3 dt
0 T
⇒ T=
A (0.6) 2 gr ∫1 (t14 − 2t12) dt1
⇒ cot θ ∫ dr = − k ∫ dt 0
R 0
2πr3  t15 2t13 
⇒ cot θ (0 − R) = − k (T − 0) ⇒ T=  − 
A (0.6) 2 gr 5 3 1
⇒ R cot θ = kT ⇒ H = kT [from Eq. (iii)]
2π ⋅ r5/ 2  1 2
H ⇒ T= ⋅ 0 − − 0 + 
⇒ T= 6  5 3
k A   2 gr
 10
H
∴ Required time after which the cone is empty, T =
k 2π ⋅ 25/ 2 (102) 5/ 2 2 1 
⇒ T=  − 
12 ⋅ ⋅ 2 ⋅ g 3 5 
3
11. Let O be the centre of hemispherical tank. Let at any
instant t, water level be BAB1 and at t + dt, water level 5
is B′ A′ B1. Let ∠ O1OB1 = θ. 2π × 105 ⋅ 4 ⋅ 5 10 − 3 
=
O O1 (12 × 3) g  15 
θ
2π × 105 × 7 14π × 105
B
A
B1 = = unit
B ′1 3⋅3⋅ g ⋅3 27 g
B' A′
12. Let X 0 be initial population of the country andY 0 be its
initial food production. Let the average consumption be
a unit. Therefore, food required initially aX 0. It is given
O2
 90 
Y p = aX 0   = 0.9 aX 0 …(i)
⇒ AB1 = r cos θ and OA = r sin θ decrease in the water  100
volume in time dt = π AB12 ⋅ d (OA )
Let X be the population of the country in year t.
[πr 2 is surface area of water level and d (OA ) is depth
dX
of water level] Then, = Rate of change of population
dt
= πr 2 ⋅ cos 2 θ ⋅ r cos θ dθ
3
= πr3 ⋅ cos3 θ dθ = X = 0.03 X
100
Also, h (t ) = O2A = r − r sin θ = r (1 − sin θ ) dX dX
⇒ = 0.03 dt ⇒ ∫ = ∫ 0.03 dt
Now, outflow rate Q = A ⋅ v (t ) = A ⋅ 0.6 2 gr (1 − sin θ ) X X
Where, A is the area of the outlet. ⇒ log X = 0.03 t + c
Thus, volume flowing out in time dt. ⇒ X = A ⋅ e0. 03 t , where A = ec
⇒ Q dt = A ⋅ (0.6) ⋅ 2 gr ⋅ 1 − sin θ dt At t = 0, X = X 0, thus X 0 = A
We have, πr3 cos3 θ dθ = A (0.6) ⋅ 2 gr ⋅ 1 − sin θ dt ∴ X = X 0 e0. 03 t

πr3 cos3 θ Let Y be the food production in year t.


⇒ ⋅ dθ = dt t
A (0.6) 2 gr (1 − sin θ )  4 
Then, Y = Y 0 1 +  = 0.9aX 0 (1.04)
t
 100
Let the time taken to empty the tank be T.
Q Y 0 = 0.9 aX 0 [from Eq. (i)]
π/ 2 πr3 cos3 θ
Then, T =∫ ⋅ dθ Food consumption in the year t is aX 0 e 0. 03 t
.
0 A (0.6) ⋅ 2 gr 1 − sin θ
Again, Y − X ≥0 [given]
− πr3 π/ 2 1 − sin 2 θ (− cos θ )
∫0 ⇒ 0.9 X 0 a (1.04) > a X 0 e
t 0. 03 t
= dθ
A (0.6) 2 gr 1 − sin θ
(1.04)t 1 10
⇒ > = .
Let t1 = 1 − sin θ e0. 03 t 0.9 9
− cos θ
⇒ dt1 = dθ Taking log on both sides, we get
1 − sin θ
t[log (1.04) − 0.03] ≥ log 10 − log 9
−2 πr 3 0 log 10 − log 9
∴ T=
A (0.6) 2 gr ∫1 [1 − (1 − t12)2] dt1 ⇒ t≥
log (1.04) − 0.03
rankershalt.com

356 Differential Equations

Thus, the least integral values of the year n, when the dV


14. ∝ V for each reservoir.
country becomes self-sufficient is the smallest integer dt
log 10 − log 9 dV dV A
greater than or equal to . ∝ − VA ⇒ = − K 1V A
log (1.04) − 0.03 dx dt
13. Equation of normal at point (x, y) is [K 1 is the proportional constant]
dx V ′A dV A t
Y − y=−
dy
(X − x) …(i) ⇒ ∫V A VA
= − K 1 ∫ dt
0

V ′A
Distance of perpendicular from the origin to Eq. (i) ⇒ log = − K 1t ⇒ V A′ = V A ⋅ e−K1 t …(i)
dx VA
y+ ⋅x
dy Similarly for B, VB′ = VB ⋅ e−K 2t …(ii)
=
 dx
2 On dividing Eq. (i) by Eq. (ii), we get
1+   V A′ V A − (K1
 dy = ⋅e − K2) t
VB′ VB
Also, distance between P and X-axis is |y|.
dx It is given that at t = 0, V A = 2 VB and at
y+ ⋅x 3 3
dy t = , V A′ = VB′
∴ = | y| 2 2
2
 dx 3 − (K1 − K 2 )t 3
1+   Thus, = 2⋅ e ⇒ e− (K1 − K 2 ) = …(iii)
 dy 2 4
 Now, let at t = t0 both the reservoirs have some quantity
 dx 
2
dx 2 dx
⇒ y2 + ⋅ x + 2xy = y2 1 +    of water. Then,
dy dy   dy 
V A′ = VB′
−(K − K 2 )
 dx
2
dx From Eq. (iii), 2e =1
⇒   (x − y ) + 2xy
2 2
=0 t0
 dy dy  3
⇒ 2⋅   =1
 4
dx  dx 2 
⇒  dy (x − y ) + 2xy = 0
2
t0 = log3/ 4 (1 / 2)
dy  
dy
dx dy y2 − x2 15. Given, = sin (10x + 6 y)
⇒ = 0 or = dx
dy dx 2xy
dx Let 10x + 6 y = t …(i)
But =0 dy  dt 
dy ⇒ 10 + 6 = 
dx  dx
⇒ x = c, where c is a constant.
dy 1  dt 
Since, curve passes through (1, 1), we get the equation ⇒ =  − 10
of the curve as x = 1. dx 6  dx 
dy y2 − x2 Now, the given differential equation becomes
The equation = is a homogeneous equation.
dx 2xy 1  dt 
sin t =  − 10
dy dv 6  dx 
Put y = vx ⇒ =v+ x
dx dx dt
⇒ 6 sin t = − 10
dv v2x2 − x2 dx
v+ x = dt
dx 2x2v ⇒ = 6 sin t + 10
dx
dv v2 − 1 v2 − 1 − 2v2 v2 + 1
⇒x = −v= =− dt
dx 2v 2v 2v ⇒ = dx
6 sin t + 10
−2 v dx
⇒ dv = On integrating both sides, we get
v2 + 1 x
1 dt
2 ∫ 3 sin t + 5
⇒ c1 − log (v + 1) = log|x|
2
=x+ c …(ii)
 y2 
⇒ log| x|(v2 + 1) = c1 ⇒ | x|  2 + 1 = ec1 dt dt
I1 = ∫
3 sin t + 5 ∫
x  Let =
 2 tan t / 2 
⇒ x2 + y2 = ± ec1 x or x2 + y2 = ± ecx is passing through 3  +5
 1 + tan t / 2
2
(1, 1).
∴ 1 + 1 = ± ec ⋅ 1 (1 + tan 2 t / 2) dt
=∫
⇒ ± ec = 2  t 2 t
6 tan + 5 + 5tan 
Hence, required curve is x2 + y2 = 2x .  2 2
rankershalt.com

Differential Equations 357

π /2
Put tan t /2 = u 17. A. I = ∫
x
(sin x)cos {cos x ⋅ cot x − log (sin x)sin x } dx
0
1 2 du
⇒ sec2 t / 2 dt = du ⇒ dt = π /2 d
2 sec2 t / 2 =∫ (sin x)cos x dx = 1
0 dx
2 du 2 du
⇒ dt = ⇒ dt = B. The point of intersection of − 4 y2 = x and
1 + tan 2 t / 2 1 + u2
x − 1 = − 5 y2 is (− 4, − 1) and (− 4, 1).
2 (1 + u 2)du 2 du Y
∴ I1 = ∫ = ∫
(1 + u 2) (5u 2 + 6u + 5) 5 6
u + u+1
2
(– 4, 1)
5
2 du
=
5 ∫
6 9 9
u + u+ 2
− +1
5 25 25 X′ X
(1, 0)
2 du 2 5  u + 3 / 5
= ∫ = ⋅ tan −1   (– 4, –1)
5  3
2
 4
2
5 4  4 /5 
u +  +  
 5  5

1 5u + 3  1 5 tan t / 2 + 3 
= tan −1   = tan −1  
Y′
2  4  2  4 ∴Required area
= 2 ∫ (1 − 5 y2) dy − ∫ − 4 y2 dy
On putting this in Eq. (ii), we get 1 1

 t   0 0 
5 tan + 3 
−1 
1 1
2  y3 
=x+ c 4
tan  1
4 4 = 2 ∫ (1 − y2)dy = 2  y −  = sq units
  0
 3 0 3
 
 t  C. The point of intersection y = 3x −1 log x and y = xx − 1
5 tan 2 + 3  is (1, 0).
⇒ tan −1   = 4x + 4c
 4  dy 3x −1
Hence, = + 3x −1 ⋅ log 3 ⋅ log x
  dx x
1  dy
⇒ [5 tan (5x + 3 y) + 3] = tan (4x + 4c) ∴   =1
4  dx (1, 0)
⇒ 5 tan (5x + 3 y) + 3 = 4 tan (4x + 4c)
For y = xx − 1
When x = 0, y = 0, we get dy
⇒ = xx (1 + log x)
5 tan 0 + 3 = 4 tan (4c) dx
3  dy
⇒ = tan 4c ∴   =1
4  dx (1, 0)
3 If θ is angle between the curves, then tan θ = 0.
⇒ 4c = tan −1
4
⇒ θ = 0°
 3
Then, 5 tan (5x + 3 y) + 3 = 4 tan 4x + tan −1  D.
dy
=
2

dx x y
− =
 4 dx x + y dy 2 2
4  3 3 1
⇒ tan (5x + 3 y) = tan 4x + tan  − ⇒ xe− y/ 2 = ⋅ ∫ y ⋅ e− y/ 2dy
5  4 5 2
4 1  ye− y 2 e− y/ 2 
  −1 3   3  ⇒ xe− y/ 2 =  − + k
⇒ 5x + 3 y = tan −1  tan 4x + tan −  2  − 1 / 2 (1 / 2)2 
5   4  5 
⇒ x + y + 2 = key/ 2
−1 4   −1 3   3
⇒ 3 y = tan  tan 4x + tan   −  − 5x It passing through (1, 0).
 
5 4   5
⇒ k =3
1 4   3  3  5x ∴ x + y + 2 = 3e y/ 2
⇒ y= tan −1  tan 4x + tan −1  −  −
3 5   4  5  3

Download Chapter Test


http://tinyurl.com/yym2wuku or
rankershalt.com

15
Straight Line and
Pair of Straight Lines

Topic 1 Various Forms of Straight Line


Objective Questions I (Only one correct option) 5. If the two lines x + (a − 1) y = 1 and
1. A straight line L at a distance of 4 units from the origin 2x + a 2y = 1, (a ∈ R − {0, 1}) are perpendicular, then the
makes positive intercepts on the coordinate axes and the distance of their point of intersection from the origin is
perpendicular from the origin to this line makes an angle (2019 Main, 9 April II)
of 60° with the line x + y = 0. Then, an equation of the line 2 2
(a) (b)
L is (2019 Main, 12 April II) 5 5
(a) x + 3 y = 8 2 2
(c) (d)
(b) ( 3 + 1) x + ( 3 − 1) y = 8 2 5 5
(c) 3x + y = 8 6. Slope of a line passing through P (2, 3) and
(d) ( 3 − 1)x + ( 3 + 1) y = 8 2 intersecting the line, x + y = 7 at a distance of 4 units
from P, is (2019 Main, 9 April I)
2. The equation y = sin x sin(x + 2) − sin 2(x + 1) represents a
1− 5 7−1
straight line lying in (2019 Main, 12 April I) (a) (b)
1+ 5 7+1
(a) second and third quadrants only
(b) first, second and fourth quadrants 1− 7 5−1
(c) (d)
(c) first, third and fourth quadrants 1+ 7 5+1
(d) third and fourth quadrants only
7. If a point R(4, y, z ) lies on the line segment joining the
3. Lines are drawn parallel to the line 4x − 3 y + 2 = 0, at a points P(2, − 3, 4) and Q(8, 0, 10), then the distance of R
3 from the origin is
distance from the origin. Then which one of the (2019 Main, 8 April II)
5 (a) 2 21 (b) 53
following points lies on any of these lines? (c) 2 14 (d) 6
(2019 Main, 10 April I)
8. Suppose that the points (h , k), (1, 2) and (−3, 4) lie on
(a)  − , −  (b)  − , 
1 2 1 2 the line L1. If a line L 2 passing through the points (h , k)
 4 3  4 3 and (4, 3) is perpendicular to L1, then k /h equals
 1
(c)  , − 
1
(d)  , 
1 1 (2019 Main, 8 April II)
4 3  4 3 1 1
(a) − (b)
7 3
4. The region represented by |x − y| ≤ 2 and |x + y| ≤ 2 is
(c) 3 (d) 0
bounded by a (2019 Main, 10 April I)
9. A point on the straight line, 3x + 5 y = 15 which is
(a) rhombus of side length 2 units
equidistant from the coordinate axes will lie only in
(b) rhombus of area 8 2 sq units (2019 Main, 8 April I)
(c) square of side length 2 2 units (a) IV quadrant (b) I quadrant
(d) square of area 16 sq units (c) I and II quadrants (d) I, II and IV quadrants
rankershalt.com

Straight Line and Pair of Straight Lines 359

10. If a straight line passing through the point P(− 3, 4) is 19. Let k be an integer such that the triangle with vertices
such that its intercepted portion between the (k, − 3k), (5, k) and (− k, 2) has area
coordinate axes is bisected at P, then its equation is 28 sq units. Then, the orthocentre of this triangle is at
(2019 Main, 12 Jan II) the point (2017 Main)
x− y+ 7= 0
(a)  2, −  (b)  1,
3
(c)  1, −  (d)  2,
(a) 1 3 1
 
(b) 4x − 3 y + 24 = 0  2  4  4  2
(c) 3x − 4 y + 25 = 0
(d) 4x + 3 y = 0
20. Let a , b, c and d be non-zero numbers. If the point of
intersection of the lines 4ax + 2ay + c = 0 and
11. If the straight line, 2x − 3 y + 17 = 0 is perpendicular to 5bx + 2by + d = 0 lies in the fourth quadrant and is
the line passing through the points (7, 17) and (15, β), equidistant from the two axes, then (2014 Main)
then β equals (2019 Main, 12 Jan I) (a) 2bc − 3ad = 0 (b) 2bc + 3ad = 0
35 35
(a) (b) − 5 (c) − (d) 5 (c) 2ad − 3bc = 0 (d) 3bc + 2ad = 0
3 3
21. If PS is the median of the triangle with vertices P(2, 2),
12. If in a parallelogram ABDC, the coordinates of A , B and
Q(6, – 1) and R(7, 3), then equation of the line passing
C are respectively (1, 2), (3, 4) and (2, 5), then the through (1, – 1) and parallel to PS is (2014 Main, 2000)
equation of the diagonal AD is (2019 Main, 11 Jan II)
(a) 4x − 7 y − 11 = 0 (b) 2x + 9 y + 7 = 0
(a) 3x + 5 y − 13 = 0 (c) 4x + 7 y + 3 = 0 (d) 2x − 9 y − 11 = 0
(b) 3x − 5 y + 7 = 0 22. The x-coordinate of the incentre of the triangle that has
(c) 5x − 3 y + 1 = 0 the coordinates of mid-points of its sides as (0, 1), (1, 1)
(d) 5x + 3 y − 11 = 0 and (1, 0) is (2013 Main)
2
13. The tangent to the curve, y = xex passing through (a) 2 + 2 (b) 2 − 2 (c) 1 + 2 (d) 1 − 2
the point (1, e) also passes through the point 23. A straight line L through the point (3, −2) is inclined at
(2019 Main, 10 Jan II) an angle 60° to the line 3x + y = 1. If L also intersects
(a)  , 2e
4 the X-axis, then the equation of L is (2011)
(b) (3, 6e)
3  (a) y + 3x + 2 − 3 3 = 0 (b) y − 3x + 2 + 3 3 = 0
(d)  , 2e (c) 3 y − x + 3 + 2 3 = 0 (d) 3 y + x − 3 + 2 3 = 0
5
(c) (2, 3e)
3 
24. The locus of the orthocentre of the triangle formed by
14. Two sides of a parallelogram are along the lines, the lines (1 + p) x − py + p (1 + p) = 0 ,
x + y = 3 and x − y + 3 = 0. If its diagonals intersect at (2, (1 + q) x − qy + q (1 + q) = 0 and y = 0, where p ≠ q , is (2009)
4), then one of its vertex is (2019 Main, 10 Jan II) (a) a hyperbola (b) a parabola
(a) (3, 6) (b) (2, 6) (c) an ellipse (d) a straight line
(c) (2, 1) (d) (3, 5) 25. Let O(0, 0), P(3, 4) and Q(6, 0) be the vertices of a ∆OPQ.
15. The shortest distance between the point  , 0 and the The point R inside the ∆OPQ is such that the triangles
3
2  OPR, PQR and OQR are of equal area. The coordinates
curve y = x , (x > 0), is (2019 Main, 10 Jan I) of R are (2007, 3M)

(a)  , 3 (b)  3,


2
(c)  3,
4
(d)  ,
3 5 3 5 4 4 2
(a) (b) (c) (d)   
2 4 2 2 3   3  3 3 3
16. If the line 3x + 4 y − 24 = 0 intersects the X-axis at the 26. Orthocentre of triangle with vertices (0, 0), (3, 4) and
(4, 0) is (2003, 2M)
point A and theY -axis at the point B, then the incentre
(a)  3,  (c)  3, 
of the triangle OAB, where O is the origin, is 5 3
(b) (3, 12) (d) (3, 9)
(2019 Main, 10 Jan I)  4  4
(a) (4, 3) (b) (3, 4)
27. The number of integer values of m, for which the
(c) (4, 4) (d) (2, 2)
x-coordinate of the point of intersection of the lines
17. A point P moves on the line 2x − 3 y + 4 = 0. If Q(1, 4) and 3x + 4 y = 9 and y = mx + 1 is also an integer, is (2001, 1M)
R(3, − 2) are fixed points, then the locus of the centroid of (a) 2 (b) 0 (c) 4 (d) 1
∆PQR is a line (2019 Main, 10 Jan I)
28. A straight line through the origin O meets the parallel
2 3 lines 4x + 2 y = 9 and 2x + y + 6 = 0 at points P and Q
(a) with slope (b) with slope
3 2 respectively. Then, the point O divides the segment PQ
(c) parallel to Y -axis (d) parallel to X-axis in the ratio (2000, 1M)
18. A straight line through a fixed point (2, 3) intersects the (a) 1 : 2 (b) 3 : 4 (c) 2 : 1 (d) 4 : 3
coordinate axes at distinct points P and Q. If O is the 29. The incentre of the triangle with vertices (1, 3 ), (0, 0)
origin and the rectangle OPRQ is completed, then the and (2, 0) is (2000, 2M)
locus of R is (2018 Main)
 3  2 3
(b)  ,
2 1 
(d)  1,
1 
(a) 3x + 2 y = 6 (b) 2x + 3 y = xy (a)  1,   (c)  ,  
 2   3 3 3 2   3
(c) 3x + 2 y = xy (d) 3x + 2 y = 6xy
rankershalt.com

360 Straight Line and Pair of Straight Lines

30. If A0 , A1 , A2, A3 , A4 and A5 be a regular hexagon 39. The point (4, 1) undergoes the following three
inscribed in a circle of unit radius. Then, the product of transformations successively
the lengths of the line segments A0 A1 , A0 A2 and A0 A4 is I. Reflection about the line y = x.
(a) 3 / 4 (b) 3 3 (1998, 2M) II. Transformation through a distance 2 units along
3 3 the positive direction of X-axis.
(c) 3 (d)
2 π
III. Rotation through an angle about the origin in the
31. If the vertices P , Q , R of a ∆PQR are rational points, 4
which of the following points of the ∆PQR is/are always counter clockwise direction.
rational point(s) (1998, 2M) Then, the final position of the point is given by the
(a) centroid (b) incentre coordinates (1980, 1M)
(c) circumcentre (d) orthocentre
(a) 
1 7 
,  (b) (− 2, 7 2)
(A rational point is a point both of whose coordinates are  2 2
(c)  −
rational numbers) 1 7 
,  (d) ( 2, 7 2)
32. If P (1, 2), Q (4, 6), R (5, 7) and S (a , b) are the vertices of  2 2
a parallelogram PQRS, then (1998, 2M) 40. The points (−a , − b), (0, 0), (a , b) and (a 2, a3 ) are
(a) a = 2, b = 4 (b) a = 3, b = 4 (a) collinear (1979, 2M)
(c) a = 2, b = 3 (d) a = 3, b = 5 (b) vertices of a rectangle
33. The diagonals of a parallelogram PQRS are along the (c) vertices of a parallelogram
lines x + 3 y = 4 and 6x − 2 y = 7. Then, PQRS must be a (d) None of the above
(a) rectangle (b) square (1998, 2M)
(c) cyclic quadrilateral (d) rhombus Objective Questions II
34. The graph of the function cos x cos (x + 2) − cos (x + 1) 2 (One or more than one correct option)
is (1997, 2M) 41. Let a, λ, µ ∈R. Consider the system of linear equations
(a) a straight line passing through (0, − sin 1) with slope 2
2
ax + 2 y = λ and 3x − 2 y = µ.
(b) a straight line passing through (0, 0)
Which of the following statement(s) is/are correct?
(c) a parabola with vertex (1, − sin 2 1) (2016 Adv.)
π
(d) a straight line passing through the point  , − sin 2 1 (a) If a = − 3, then the system has infinitely many
2 
solutions for all values of λ and µ
and parallel to the X-axis (b) If a ≠ − 3, then the system has a unique solution for all
35. The orthocentre of the triangle formed by the lines values of λ and µ
xy = 0 and x + y = 1,is (1995, 2M) (c) If λ + µ = 0, then the system has infinitely many
solutions for a = − 3
(a)  ,
1
(b)  , 
1 1 1
 (d) If λ + µ ≠ 0, then the system has no solution for a = − 3
2 2  3 3
(c) (0, 0) (d)  , 
1 1 42. For a > b > c > 0, the distance between (1, 1) and the
 4 4 point of intersection of the lines ax + by + c = 0 and
36. If the sum of the distance of a point from two bx + ay + c = 0 is less than 2 2. Then, (2014 Adv.)
perpendicular lines in a plane is 1, then its locus is (a) a + b − c > 0 (b) a − b + c < 0
(a) square (b) circle (1992, 2M) (c) a − b + c > 0 (d) a + b − c < 0
(c) straight line (d) two intersecting lines 43. All points lying inside the triangle formed by the points
37. Line L has intercepts a and b on the coordinate axes. (1, 3), (5, 0) and (– 1, 2) satisfy (1986, 2M)
When, the axes are rotated through a given angle, (a) 3x + 2 y ≥ 0 (b) 2x + y − 13 ≥ 0
keeping the origin fixed, the same line L has intercepts (c) 2x − 3 y − 12 ≤ 0 (d) −2x + y ≥ 0
p and q, then (1990, 2M)
1 1 1 1 Fill in the Blanks
(a) a 2 + b2 = p 2 + q2 (b) + = +
a2 b2 p2 q2
1 1 1 1 44. Let the algebraic sum of the perpendicular distance
(c) a 2 + p 2 = b2 + q2 (d) 2
+ 2
= 2
+ 2 from the points (2, 0) , (0, 2) and (1, 1) to a variable
a p b q
straight line be zero, then the line passes through a
38. If P = (1, 0), Q = (−1, 0) and R = (2, 0) are three given fixed point whose coordinates are… . (1991, 2M)
points, then locus of the points satisfying the relation 45. The orthocentre of the triangle formed by the lines
SQ 2 + SR2 = 2 SP 2, is (1988, 2M) x + y = 1, 2 x + 3 y = 6 and 4x − y + 4 = 0 lies in quadrant
(a) a straight line parallel to X-axis number… . (1985, 2M)
(b) a circle passing through the origin
46. If a, b and c are in AP, then the straight line
(c) a circle with the centre at the origin
a x + by + c = 0 will always pass through a fixed point
(d) a straight line parallel to Y-axis whose coordinates are (…) . (1984, 2M)
rankershalt.com

Straight Line and Pair of Straight Lines 361

47. y = 10x is the reflection of y = log10 x in the line whose 59. A line cuts the X-axis at A (7, 0) and the Y-axis at
equation is .... . (1984, 2M) B (0, − 5). A variable line PQ is drawn perpendicular to
AB cutting the X-axis in P and the Y-axis in Q. If AQ
True/False and BP inters at R, find the locus of R. (1990, 4M)

48. The lines 2x + 3 y + 19 = 0 and 9x + 6 y − 17 = 0 cut the 60. Let ABC be a triangle with AB = AC. If D is mid point of
coordinate axes in concyclic points. (1988, 1M) BC, the foot of the perpendicular drawn from D to AC
49. No tangent can be drawn from the point (5/2, 1) to the and F the mid-point of DE. Prove that AF is
circumcircle of the triangle with vertices (1, 3 ), perpendicular to BE. (1989, 5M)
(1, − 3 ) and (3, 3 ). (1985, 1M) 61. The equations of the perpendicular bisectors of the
50. The straight line 5x + 4 y = 0 passes through the point of sides AB and AC of a ∆ABC are x − y + 5 = 0 and
intersection of the straight lines x + 2 y − 10 = 0 and x + 2 y = 0, respectively. If the point A is (1, – 2), find the
2x + y + 5 = 0. (1983, 1M) equation of the line BC. (1986, 5M)

62. One of the diameters of the circle circumscribing the


Analytical & Descriptive Questions rectangle ABCD 4 y = x + 7. If A and B are the points
51. A straight line L through the origin meets the line (−3, 4) and (5, 4) respectively, then find the area of
x + y = 1 and x + y = 3 at P and Q respectively. Through rectangle. (1985, 3M)
P and Q two straight lines L1 and L 2 are drawn, parallel
to 2x − y = 5 and 3x + y = 5, respectively. Lines L1 and L 2
63. Two sides of a rhombus ABCD are parallel to the lines
y = x + 2 and y = 7x + 3. If the diagonals of the rhombus
intersect at R, show that the locus of R as L varies, is a
intersect at the point (1, 2) and the vertex A is on the
straight line. (2002, 5M)
Y-axis, find possible coordinates of A. (1985, 5M)
52. A straight line L with negative slope passes through the 64. Two equal sides of an isosceles triangle are given by the
point (8, 2) and cuts the positive coordinate axes at equations 7x − y + 3 = 0 and x + y − 3 = 0 and its third
points P and Q. Find the absolute minimum value of side passes through the point (1, −10). Determine the
OP + OQ, as L varies, where O is the origin. (2002, 5M) equation of the third side.
53. For points P = (x1 , y1 ) and Q = (x2, y2) of the coordinate (1984, 4M)

plane, a new distance d (P , Q ) is defined by 65. The vertices of a triangle are


d (P , Q ) = | x1 − x2| + | y1 − y2|. [at1t2, a (t1 + t2)], [at2 t3 , a (t2 + t3 )],
Let O = (0, 0) and A = (3, 2). Prove that the set of points [at3 t1 , a (t3 + t1 )].
in the first quadrant which are equidistant (with Find the orthocentre of the triangle. (1983, 3M)
respect to the new distance) from O and A consists of the 66. The ends A , B of a straight line segment of constant
union of a line segment of finite length and an infinite
ray. Sketch this set in a labelled diagram. (2000, 10M) length c slide upon the fixed rectangular axes OX , OY
respectively. If the rectangle OAPB be completed, then
54. A rectangle PQRS has its side PQ parallel to the line show that the locus of the foot of the perpendicular
y = mx and vertices PQ and S on the lines y = a , x = b drawn from P to AB is
and x = − b, respectively. Find the locus of the vertex R. x2 / 3 + y2 / 3 = c2 / 3 (1983, 2M)
(1996, 2M)
55. A line through A (−5, − 4) meets the line x + 3 y + 2 = 0, 67. The points (1, 3) and (5, 1) are two opposite vertices of a
rectangle. The other two vertices lie on the line
2x + y + 4 = 0 and x − y − 5 = 0 at the points B, C and D y = 2x + c. Find c and the remaining vertices. (1981, 4M)
respectively. If (15 / AB)2 + (10 / AC )2 = (6 / AD )2, find 68. Two vertices of a triangle are (5, − 1) and (−2, 3). If the
the equation of the line. (1993, 5M)
orthocentre of the triangle is the origin, find the
56. Determine all values of α for which the point (α , α 2) lies coordinates of the third vertex. (1978, 3M)
inside the triangles formed by the lines 2x + 3 y − 1 = 0, 69. One side of a rectangle lies along the line
4x + 7 y + 5 = 0. Two of its vertices are (−3, 1) and (1, 1).
x + 2 y − 3 = 0, 5x − 6 y − 1 = 0 (1992, 6M)
Find the equations of the other three sides. (1978, 3M)
57. Find the equations of the line passing through the point
(2, 3) and making intercept of length 3 unit between the Integer Answer Type Question
lines y + 2x = 2 and y + 2x = 5. (1991, 4M) 70. For a point P in the plane, let d1 (P ) and d2(P ) be the
58. Straight lines 3x + 4 y = 5 and 4x − 3 y = 15 intersect at distances of the point P from the lines x − y = 0 and
the point A. Points B and C are chosen on these two x + y = 0, respectively. The area of the region R
lines such that AB = AC. Determine the possible consisting of all points P lying in the first quadrant of
equations of the line BC passing through the point the plane and satisfying 2 ≤ d1 (P ) + d2(P ) ≤ 4, is
(2014 Adv.)
(1, 2). (1990, 4M)
rankershalt.com

Topic 2 Angle between Straight Lines and Equation of


Angle Bisector
Objective Questions II
(One or more than one correct option)
1. A ray of light along x + 3 y = 3 gets reflected upon (c) Statement I is true; Statement II is false
reaching X-axis, the equation of the reflected ray is (d) Statement I is false; Statement II is true
(2013 Main) 4. Lines L1 : y − x = 0 and L 2 : 2x + y = 0 intersect the line
(a) y = x + 3 (b) 3 y = x − 3
L3 : y + 2 = 0 at P and Q, respectively. The bisector of the
(c) y = 3x − 3 (d) 3 y = x − 1
acute angle between L1 and L 2 intersects L3 at R.
2. Consider three points Statement I The ratio PR : RQ equals 2 2 : 5 .
P = {– sin ( β – α ) – cos β}, Q = {cos( β – α ),sin β} Because
and R = {cos ( β – α + θ ) sin ( β – θ )}, Statement II In any triangle, bisector of an angle
π
where 0 < α , β , θ < . Then, divides the triangle into two similar triangles. (2007, 3M)
4 (2008, 4M)
(a) P lies on the line segment RQ Fill in the Blank
(b) Q lies on the line segment PR
(c) R lies on the line segment QP 5. The vertices of a triangle are A (− 1, − 7), B(5, 1) and
(d) P, Q, R are non-colinear C (1, 4). The equation of the bisector of the angle ABC
3. Let P = (–1, 0), and Q (0, 0) and R = (3, 3 3 ) be three is… . (1993, 2M)

point. Then, the equation of the bisector of the angle


PQR is (2001, 1M)
Analytical & Descriptive Questions
(a)
3
x+ y = 0 (b) x + 3y = 0
6. The area of the triangle formed by the intersection of
2 line parallel to X-axis and passing through (h, k) with
3 the lines y = x and x + y = 2 is 4h 2. Find the locus of point
(c) 3 x + y = 0 (d) x + y=0
2 P. (2005)

Assertion and Reason 7. Find the equation of the line which bisects the obtuse
angle between the lines x − 2 y + 4 = 0 and 4x − 3 y + 2 = 0.
For the following questions choose the correct answer (1993, 2M)
from the codes (a), (b), (c) and (d) defined as follows.
8. Lines L1 ≡ ax + by + c = 0 and L 2 ≡ lx + my + n = 0
(a) Statement I is true, Statement II is also true;
Statement II is the correct explanation of Statement I intersect at the point P and makes an angle θ with each
(b) Statement I is true, Statement II is also true; other. Find the equation of a line L different from L 2
Statement II is not the correct explanation of which passes through P and makes the same angle θ
Statement I with L1.y (1988, 5M)

Topic 3 Area and Family of Concurrent Lines


Objective Questions I (Only one correct option) 3. Two sides of a rhombus are along the lines, x − y + 1 = 0
1. A triangle has a vertex at (1, 2) and the mid-points of and 7x − y − 5 = 0. If its diagonals intersect at (− 1, − 2),
the two sides through it are (−1, 1) and (2, 3). Then, the then which one of the following is a vertex of this
centroid of this triangle is (2019 Main, 12 April II) rhombus? (2016 Main)
(a) (− 3, − 9) (b) (− 3, − 8)
(a)  1,
7
(b)  , 2 (c)  , 1 (d)  ,
1 1 1 5
 
 3 3  3  3 3 (c)  , −  (d)  − , − 
1 8 10 7
3 3  3 3
2. Consider the set of all lines px + qy + r = 0 such that
3 p + 2q + 4 r = 0. Which one of the following statements 4. Area of the parallelogram formed by the lines
is true? (2019 Main, 9 Jan I) y = mx, y = mx + 1, y = nx and y = nx + 1 equals
(2001, 1M)
(a) Each line passes through the origin. |m + n| 2
 3 1 (a) (b)
(b) The lines are concurrent at the point  ,  (m − n ) 2 |m + n|
 4 2
(c) The lines are all parallel 1 1
(c) (d)
(d) The lines are not concurrent |m + n| |m − n|
rankershalt.com

Straight Line and Pair of Straight Lines 363

5. The points 0,  , (1, 3) and (82, 30) are vertices of


8 Column I Column II
 3 (1986, 2M) (A) L1, L2, L3 are concurrent, if (p) k= −9
(a) an obtuse angled triangle 6
(B) One of L1, L2, L3 is parallel to k=−
(b) an acute angled triangle (q)
at least one of the other two, if 5
(c) a right angled triangle 5
(C) L1, L2, L3 form a triangle, if
(r) k=
(d) None of the above 6
6. The straight lines x + y = 0, 3x + y − 4 = 0, x + 3 y − 4 = 0 (D) L1, L2, L3 do not form a (s) k=5
form a triangle which is (1983, 1M) triangle, if
(a) isosceles
(b) equilateral Fill in the Blank
(c) right angled 10. The set of lines ax + by + c = 0, where 3a + 2b + 4c = 0 is
(d) None of the above
concurrent at the point… . (1982, 2M)
7. Given the four lines with the equations
x + 2 y − 3 = 0, 3x + 4 y − 7 = 0, True/False
2x + 3 y − 4 = 0,4x + 5 y − 6 = 0, then (1980, 1M)
x1 y1 1 a1 b1 1
(a) they are all concurrent
(b) they are the sides of a quadrilateral 11. If x2 y2 1 = a 2 b2 1 ,then the two triangles
(c) only three lines are concurrent x3 y3 1 a3 b3 1
(d) None of the above with vertices (x1 , y1 ), (x2, y2), (x3 , y3 ) and (a1 , b1 ), (a 2, b2),
(a3 , b3 ) must be congruent. (1985, 1M)
Objective Question II
(One or more than one correct option) Analytical & Descriptive Questions
8. Three lines px + qy + r = 0, qx + ry + p = 0 and 12. Using coordinate geometry, prove that the three
rx + py + q = 0 are concurrent, if (1985, 2M) altitudes of any triangle are concurrent. (1998, 8M)
(a) p + q + r = 0 (b) p 2 + q2 + r 2 = pr + rq 13. The coordinates of A , B, C are (6, 3), (−3, 5), (4, − 2)
(c) p3 + q3 + r3 = 3 pqr (d) None of these
respectively and P is any point (x, y). Show that the
ratio of the areas of the triangles ∆ PBC and ∆ ABC is
Match the Columns x+ y−2
. (1983, 2M)
Match the conditions/expressions in Column I with 7
statement in Column II. 14. A straight line L is perpendicular to the line in
9. Consider the lines given by 5x − y = 1. The area of the triangle formed by the line L
L1 : x + 3 y − 5 = 0, L 2 : 3x − ky − 1 = 0, and the coordinate axes is 5. Find the equation of the
L3 : 5x + 2 y − 12 = 0 line L. (1980, 3M)

Topic 4 Homogeneous Equation of Pair of Straight Lines


Objective Questions I (Only one correct option)
1. Let a and b be non-zero and real numbers. Then, the 2. Area of triangle formed by the lines x + y = 3 and angle
equation (ax2 + by2 + c) (x2 − 5xy + 6 y2) = 0 represents bisectors of the pair of straight lines x2 − y2 + 2 y = 1 is
(2008, 3M) (2004, 2M)
(a) four straight lines, when c = 0 and a , b are of the (a) 2 sq units (b) 4 sq units
same sign (c) 6 sq units (d) 8 sq units
(b) two straight lines and a circle, when a = b and c is of
sign opposite to that of a
Analytical & Descriptive Question
(c) two straight lines and a hyperbola, when a and b are 3. Show that all chords of curve 3x2 − y2 − 2x + 4 y = 0,
of the same sign and c is of sign opposite to that of a which subtend a right angle at the origin pass through
(d) a circle and an ellipse, when a and b are of the same a fixed point. Find the coordinates of the point.
sign and c is of sign opposite to that of a (1991, 4M)
rankershalt.com

364 Straight Line and Pair of Straight Lines

Topic 5 General Equation of Pair of Straight Lines


Objective Question I (Only one correct option)
1. Let PQR be a right angled isosceles triangle, right (a) 3x2 − 3 y2 + 8xy + 20x + 10 y + 25 = 0
angled at P (2, 1). If the equation of the line QR is (b) 3x2 − 3 y2 + 8xy − 20x − 10 y + 25 = 0
2x + y = 3, then the equation representing the pair of (c) 3x2 − 3 y2 + 8xy + 10 x + 15 y + 20 = 0
lines PQ and PR is (1999, 2M) (d) 3x2 − 3 y2 − 8xy − 10 x − 15 y − 20 = 0

Answers
Topic 1 65. [( −a, a (t1 + t 2 + t 3 + t1t 2t 3 )] 67. c = − 4,( 4, 4 ),(2, 0 )
1. (d) 2. (d) 3. (b) 4. (c) 68. ( − 4, − 7 )
5. (d) 6. (c) 7. (c) 8. (c) 69. 7 x − 4y + 25 = 0, 4 x + 7y = 11 = 0, 7 x − 4y − 3 = 0
9. (c) 10. (b) 11. (d) 12. (c)
70. 6 sq units
13. (a) 14. (a) 15. (d) 16. (d)
17. (a) 18. (c) Topic 2
19. (d) 20. (c) 21. (b) 22. (b) 1. (b) 2. (d) 3. (c) 4. (c)
23. (b) 24. (d) 25. (c) 26. (c) 5. 7y = x + 2
27. (a) 28. (b) 29. (d) 30. (c) 6. 2 x = ± (y − 1 )
31. (a) 32. (c) 33. (d) 34. (d) 7. ( 4 + 5 ) x − (2 5 + 3 ) y + ( 4 5 + 2 ) = 0
35. (c) 36. (a) 37. (b) 38. (d) 8. 2 (al + bm )(ax + by + c ) − (a 2 + b 2 ) (lx + my + n ) = 0
39. (c) 40. (a) 41. (b, c, d) 42. (a, c)
43. (a, c) 44. (1, 1) 45. Ist 46. (1, − 2 ) Topic 3
47. (y = x ) 48. True 49. True 50. True 1. (b) 2. (b) 3. (c) 4. (d)
5. (d) 6. (a) 7. (c) 8. (a, c)
52. (OP + OQ = 18)
9. A → s; B → p, q; C → r; D → p, q, s
54. (m 2 − 1 ) x − my + b (m 2 + 1 ) + am = 0
 3 1
10.  ,  11. False 14. x + 5y = ±5 2
55. 2 x + 3y + 22 = 0
3 1
56. − < α < −1 ∪ < α < 1  4 2
2 2
57. x = 2 and 3 x + 4y = 18 Topic 4
58. x − 7y + 13 = 0 and 7 x + y − 9 = 0 1. (b) 2. (a)
3. (1, − 2 )
59. x + y − 7 x + 5y = 0
2 2
61. 14 x + 23y − 40 = 0
 5 Topic 5
62. 32 sq units 63.  0,  or ( 0, 0 )
 2 1. (b)
64. x − 3y − 31 = 0 or 3 x + y + 7 = 0

Hints & Solutions


Topic 1 Various Forms of Straight Line tan θ = − 1 = tan (180° − 45° )
1. According to the question, we have the following figure. ⇒ tan θ = tan 135°
⇒ θ = 135°
(0,b) ⇒ α + 60° = 135°
M ⇒ α = 75°
Since, line L having perpendicular distance OM = 4.
60°
α (a,0) So, equation of the line ‘L’ is
O x+ x cos α + y sin α = 4
y= x y
0 — + — =1 ⇒ x cos 75° + y sin 75° = 4
a b
⇒ x cos (45° + 30° ) + y sin (45° + 30° ) = 4
Let θ be the inclination of the line x + y = 0. Then,
rankershalt.com

Straight Line and Pair of Straight Lines 365

 3 1   3 1  5.
⇒ x − + y + =4 Key Idea
2 2 2 2  2 2 2 2 (i) If lines are perpendicular to each other, then
⇒ ( 3 − 1) x + y ( 3 + 1) = 8 2 product of their slopes is −1, i.e. m1m2 = − 1
(ii) Distance between two points ( x 1, y 1) and (x 2, y 2)
Key Idea Use formulae : = (x 2 − x 1) 2 + (y 2 − y 1) 2
2. 2sin A sin B = cos( A − B) − cos( A + B) and cos 2 θ = 1 − 2 sin2 θ
Given, lines x + (a − 1) y = 1
Given equation is y = sin x sin(x + 2) − sin 2(x + 1) and 2x + a 2y = 1, where a ∈ R − {0, 1}
1 1 are perpendicular to each other
= [cos 2 − cos(2x + 2)] − [1 − cos(2x + 2)]
2 2  1   2
∴ −  × −  = −1
[Q2 sin A sin B = cos( A − B) − cos( A + B) and  a − 1  a 2
cos 2 θ = 1 − 2 sin 2 θ ⇒ 2 sin 2 θ = 1 − cos 2 θ] [Q If lines are perpendicular, then product of their
1 1 1 1 slopes is −1]
= cos 2 − cos(2x + 2) − + cos(2x + 2)
2 2 2 2 ⇒ a 2(a − 1) = − 2 ⇒ a3 − a 2 + 2 = 0
1 1 ⇒ (a + 1)(a 2 − 2a + 2) = 0 ⇒ a = − 1
= (cos(2) − 1) = − (2 sin 2(1))
2 2 ∴Equation of lines are
x − 2y = 1 …(i)
= − sin 2(1) < 0 ⇒ y < 0
and 2x + y = 1 …(ii)
and as we know that y < 0, is in third and fourth On solving Eq. (i) and Eq. (ii), we get
quadrants only. 3 1
x = and y = −
3. Since, equation of a line parallel to line ax + by + c = 0 is 5 5
ax + by + k = 0 ∴ Point of intersection of the lines (i) and (ii) is
∴Equation of line parallel to line 3 1
P , −  .
4x − 3 y + 2 = 0 is 4x − 3 y + k = 0 …(i)
5 5
3 1
Now, distance of line (i) from the origin is Now, required distance of the point P  , −  from
| k| 3 5 5
=
4 + 3
2 2 5 9 1 10 2
origin = + = =
25 25 25 5
[as per question’s requirement]
⇒ | k| = 3 6. Let the slope of line is m, which is passing through
⇒ k=± 3 P(2, 3).
So, possible lines having equation, either 4x − 3 y + 3 = 0 Y
or 4x − 3 y − 3 = 0
8
Now, from the given options the point  − ,  lies on the
1 2
 4 3 7
line 4x − 3 y + 3 = 0. 6

4. The given inequalities are 5


|x − y| ≤ 2 and|x + y| ≤ 2 . 4 R
d
On drawing, the above inequalities, we get a square 3 P(2,
3 )
Y 2 θ
4 Q
1
(0, 2)
X′ X
1 2 3 4 5 6 7 8 9
(–2, 0) (2, 0)
X′ X Y′ x+y=7
O
Since, the distance of a point (x1 , y1 ) from the line
ax + by1 + c
(0, –2) ax + by + c = 0 is d = 1 .
a 2 + b2
Y′ ∴The distance of a point P (2, 3) from the line
x + y − 7 = 0, is
Now, the area of shaded region is equal to the area of a
|2 + 3 − 7| 2
square having side length (2 − 0)2 + (0 − 2)2 = 2 2 d= = = 2
1+ 1 2
units.
Now, in ∆PRQ,
QR = 16 − d 2 = 16 − 2 = 14
rankershalt.com

366 Straight Line and Pair of Straight Lines

d 2 1 m+ 1  m2 − m1  9. Given equation of line is 3x + 5 y = 15 …(i)


∴ tanθ = = = = Q tanθ = 
QR 14 7 1− m  1 + m1 m2  Clearly, a point on the line (i), which is equidistance
m+ 1 1 from X and Y -axes will lie on the line either y = x or
⇒ =±
1− m 7 y = − x.
m+ 1 1 m+ 1 1 Y
⇒ = or =−
1− m 7 1− m 7
B
1− 7 −1 − 7 y=x
⇒ m= or m=
1+ 7 7 −1 (0, 3)
7. Given points are P(2, − 3, 4), Q(8, 0, 10) and R(4, y, z ). y=–x A
Now, equation of line passing through points P and Q is (5, 0)
x − 8 y − 0 z − 10 X
= = 0
6 3 6 3x+5y=15
[Since equation of a line passing through two points
In the above figure, points A and B are
A (x1 , y1 , z1 ) and B (x2 , y2 , z2 ) is given by
on the line (i) and are equidistance from the coordinate
x − x1 y − y1 z − z1 
= =  axes.
x2 − x1 y2 − y1 z2 − z1 
 15 15
x − 8 y z − 10 On solving line (i) and y = x, we get A  , .
⇒ = = …(i) 8 8
2 1 2
Similarly, on solving line (i) and y = − x, we get
Q Points P , Q and R are collinear, so
4 − 8 y z − 10  15 15
= = B − , .
2 1 2  2 2
z − 10 So, the required points lie only in I and II quadrants.
⇒ −2 = y =
2
10. Let the equation of required line having intercepts a
⇒ y= −2
and b with the axes is
and z=6
x y
So, point R is (4, − 2, 6), therefore the distance of point R + =1 …(i)
a b
from origin is Y
OR = 16 + 4 + 36 B (0,b)
= 56 = 2 14
8. Given, points (1, 2), (−3, 4) and (h , k) are lies on line L1, b P (–3,4)
so slope of line L1 is
4− 2 k − 2
m1 = = A (a , 0 )
−3 − 1 h − 1 X
O a
−1 k − 2
⇒ m1 = = …(i)
2 h −1 Now, according to given information,
⇒ 2(k − 2) = − 1(h − 1) P is the mid-point of AB
P =  ,  = (−3, 4)
a b
⇒ 2k − 4 = − h + 1 ∴ [given]
 2 2
⇒ h + 2k = 5 …(ii)
and slope of line L2 joining points (h , k ) and ⇒ (a , b) = (−6, 8)
3− k On putting the value of a and b in Eq. (i), we get
(4, 3), is m2 = …(iii) x y
4− h + = 1 ⇒ 8x − 6 y = −48
−6 8
Since, lines L1 and L2 are perpendicular to each other. ⇒ 4x − 3 y + 24 = 0
∴ m1 m2 = − 1
 
11. Slope of the line 2x − 3 y + 17 = 0 is
− 1 3 − k
⇒   = − 1[from Eqs. (i) and (iii)]
 2   4 − h 
2
= m1, (let) and the slope of line joining the points (7, 17)
3
⇒ 3 − k = 8 − 2h β − 17 β − 17
and (15, β ) is = = m2 (let)
⇒ 2h − k = 5 …(iv) 15 − 7 8
On solving Eqs. (ii) and (iv), we get According to the question, m1m2 = − 1
(h , k ) = (3, 1) 2 β − 17
⇒ × = − 1 ⇒ β − 17 = − 12 ⇒ β = 5.
k 3 3 8
So, = =3
h 1
rankershalt.com

Straight Line and Pair of Straight Lines 367

12. According to given information, we have the following 0 + x2 3 + y2


⇒ 2= ;4=
figure 2 2
C(2, 5) D ⇒ x2 = 4 and y2 = 5
∴Thus, C ≡ (4, 5)
P
Now, equation of line BC is given by
( y − y1 ) = m (x − x1 )
A(1, 2) B(3, 4)
y − 5 = 1(x − 4)
[line BC is parallel to x − y + 3 = 0 and slope
We know that, diagonals of a parallelogram intersect at
(−1)
mid-point. of x − y + 3 = 0 is = 1]
(−1)
∴ P = Mid-point of BC and so, P ≡  , 
5 9
 2 2 ⇒ y = x+1 …(iii)
Now, equation of AD is. and equation of line DC is
9 y − 5 = −1 (x − 4)
−2
( y − 2) = 2 (x − 1) [line DC is parallel to x + y = 3 and
5 −1
−1 slope of x + y = 3 is = −1]
2 1

5
y − 2 = (x − 1) ⇒ x+ y = 9 …(iv)
3 On solving Eqs. (i) and (iii), we get B (1, 2) and on
⇒ 3 y − 6 = 5x − 5 solving Eqs. (ii) and (iv), we get D (3, 6)
⇒ 5x − 3 y + 1 = 0 15. Let P (x1 , y1 ) be any point on the curve y = x .
x2
13. Given equation of curve is y = xe …(i) Clearly,y1 = x1 ⇒ x1 = y12 [Q (x1 , y1 ) lies on y = x]
Note that (1, e) lie on the curve, so the point of contact is ∴ The point is P ( y12 , y1 )
(1, e).
Now, let the given point be A  , 0 , then
3
Now, slope of tangent, at point (1, e), to the curve (i) is 2 
=  x(2x) ex + ex 
dy 2 2
2
  (1 , e ) PA =  y2 − 3  + y2
dx (1 , e )  1 
 2
1

= 2e + e = 3e
9
Now, equation of tangent is given by = y14 − 3 y12 + + y12
4
( y − y1 ) = m (x − x1 )
y − e = 3e(x − 1) ⇒ y = 3ex − 2e 9
= y14 − 2 y12 +
4
4 
On checking all the options, the option  , 2e satisfy
3  = ( y12 − 1)2 +
5
the equation of tangent. 4
Clearly, PA will be least when
14. According to given information, we have the following
figure y12 − 1 = 0
5 5
D C (x2 , y2 )
⇒ PAmin = 0+ =
4 2
M 16. Given equation of line is
x–y+3=0 (2,4)
3x + 4 y − 24 = 0
B For intersection with X-axis put y = 0
A x+y=3 ⇒ 3x − 24 = 0
[Note that given lines are perpendicular to each other as ⇒ x=8
m1 × m2 = −1] For intersection withY -axis, put x = 0
Clearly, point A is point of intersection of lines ⇒ 4 y − 24 = 0 ⇒ y = 6
x+ y = 3 …(i)
∴ A(8, 0) and B (0, 6)
and x − y = −3 …(ii)
So, A = (0, 3) [solving Eqs. (i) and (ii)] B(0,6)
Now, as point M (2, 4) is mid-point of line joining the
points A and C, so
0 + x2 3 + y2 
(2, 4) =  , 
 2 2 
  x1 + x2 y1 + y2   O
Qmid- point =  2 ,  A(8,0)
 2  
rankershalt.com

368 Straight Line and Pair of Straight Lines

Let AB = c = 82 + 62 = 10 19. Given, vertices of triangle are (k, − 3k), (5, k) and (− k, 2).
OB = a = 6 k − 3k 1
1
and OA = b = 8 ∴ 5 k 1 = ± 28
2
Also, let incentre is (h k ), then −k 2 1
ax + bx2 + cx3 k − 3k 1
h= 1 (here, x1 = 8, x2 = 0, x3 = 0)
a+ b+ c ⇒ 5 k 1 = ± 56
6 × 8 + 8 × 0 + 10 × 0 48 −k 2 1
= = =2
6 + 8 + 10 24 ⇒ k(k − 2) + 3k(5 + k) + 1(10 + k2) = ± 56
ay1 + by2 + cy3 ⇒ k2 − 2k + 15k + 3k2 + 10 + k2 = ± 56
and k= (here, y1 = 0, y2 = 6, y3 = 0)
a+ b+ c ⇒ 5k2 + 13k + 10 = ± 56
6 × 0 + 8 × 6 + 10 × 0 48 ⇒ 5k2 + 13k − 66 = 0
= = =2
6 + 8 + 10 24 or 5k2 + 13k − 46 = 0
⇒ k =2 [Qk ∈ I]
∴Incentre is (2, 2).
Thus, the coordinates of vertices of triangle are
17. Let the coordinates of point P be (x1 , y1 ) A(2, − 6), B(5, 2) and C (− 2, 2).
Q P lies on the line 2x − 3 y + 4 = 0 Now, equation of altitude from vertex A is
∴ 2x1 − 3 y1 + 4 = 0 −1
2x + 4 y − (− 6) = (x − 2) ⇒ x = 2 …(i)
⇒ y1 = 1 …(i)  2 −2 
3  
 − 2 − 5
Now, let the centroid of ∆PQR be G (h , k ), then
x + 1+ 3 Y
h= 1
3
⇒ x1 = 3h − 4 …(ii) C (–2, 2) D
B (5, 2)
y + 4− 2
and k= 1 (2, 1/2)
3
E
2x1 + 4
+ 2 X′ X
3 O
⇒ k= [from Eq. (i)]
3 A (2, –6)
2x + 4 + 6
⇒ 3k = 1
3 Y′
⇒ 9k − 10 = 2x1 …(iii)
Now, from Eqs. (ii) and (iii), we get Equation of altitude from vertex C is
−1
2(3h − 4) = 9k − 10 y−2 = [x − (− 2)]
⇒ 6h − 8 = 9k − 10 2 − (− 6) 
 5 −2 
⇒ 6h − 9k + 2 = 0  
Now, replace h by x and k by y. ⇒ 3x + 8 y − 10 = 0 …(ii)
⇒ 6x − 9 y + 2 = 0, which is the required locus and slope of 1
On solving Eqs. (i) and (ii), we get x = 2 and y = .
this line is
2 Q slope of ax + by + c = 0 is − a  2
3  b   1
∴ Orthocentre = 2, 
18.  2

R(α, β) 20. Let coordinate of the intersection point in fourth


Q(0, β)
quadrant be (α, − α).
(2, 3)
Since, (α, − α) lies on both lines 4 ax + 2ay + c = 0 and
5bx + 2by + d = 0.
−c
P(α, 0) ∴ 4 aα − 2aα + c = 0 ⇒ α = …(i)
2a
−d
x y and 5 bα − 2 bα + d = 0 ⇒ α = …(ii)
Equation of line PQ is + =1 3b
α β
From Eqs. (i) and (ii), we get
Since this line is passes through fixed point (2, 3). −c −d
2 3 = ⇒ 3bc = 2ad
∴ + =1 2a 3b
α β
⇒ 2ad − 3bc = 0
∴Locus of R is 2β + 3α = αβ
i.e. 2 y + 3x = xy ⇒ 3x + 2 y = xy
rankershalt.com

Straight Line and Pair of Straight Lines 369

7 + 6 3 − 1  13 
21. Coordinate of S =  ,  = , 1 and
y+2
=
− 3− 3
 2 2  2  x − 3 1 + (− 3)( 3 )
[QS is mid-point of line QR] ⇒ y+ 2 =0
P (2,2) y+ 2 −2 3
and = = 3
x−3 1 − 3
y+ 2 = 3 x−3 3
Neglecting, y + 2 = 0, as it does not intersect Y-axis.
Q R
(6,–1)
S
(7,3)
24. Given, lines are (1 + p) x − py + p (1 + p) = 0 ... (i)
−2 and (1 + q) x − qy + q (1 + q) = 0 ... (ii)
Slope of the line PS is .
9 On solving Eqs. (i) and (ii), we get
Required equation passes through (1 , − 1) and parallel C { pq , (1 + p) (1 + q)}
to PS is ∴ Equation of altitude CM passing through C and
−2 perpendicular to AB is
y+1= (x − 1)
9 x = pq ... (iii)
⇒ 2x + 9 y + 7 = 0  1 + q 
Q Slope of line (ii) is  .
 q 
22. Given mid-points of a triangle are (0, 1), (1, 1) and
(1, 0). Plotting these points on a graph paper and −q
∴ Slope of altitude BN (as shown in figure) is .
make a triangle. 1+ q
Y
So, the sides of a triangle will be 2, 2 and 22 + 22 i.e.
2 2.
Y N C
1)
e(
Lin H(h,k) (-p, 0)
C (0,2) X
A O M B
Line (2)
2 (0, 1) (1, 1)

X′ X
B (1, 0) A (2, 0) −q
(0, 0) 2 ∴ Equation of BN is y − 0 = (x + p)
1+ q
Y′
−q
⇒ y= (x + p) ... (iv)
2 × 0 + 2 2 ⋅0 + 2 ⋅2 (1 + q)
x-coordinate of incentre =
2+2+2 2 Let orthocentre of triangle be H(h, k), which is the point
2 2− 2 of intersection of Eqs. (iii) and (iv).
= × =2 − 2
2+ 2 2− 2 On solving Eqs. (iii) and (iv), we get
x = pq and y = − pq
23. A straight line passing through P and making an angle
⇒ h = pq and k = − pq
of α = 60º , is given by
y − y1 ∴ h+ k =0
= tan (θ ± α ) ∴ Locus of H (h, k) is x + y = 0 .
x − x1
25. Since, triangle is isosceles, hence centroid is the desired
point.
Y
P (3, –2)
60°
⇒ tan α = √3
60° P (3, 4)

√3x + y = 1
R
⇒ 3 x+ y=1 X' X
⇒ y = − 3 x + 1, then tan θ = − 3 (0, 0) O Q (6, 0)
y+2 tan θ ± tan α
⇒ =
x − 3 1 m tan θ tan α Y'
y+2 − 3+ 3  4
= ∴ Coordinates of R 3,  .
x − 3 1 − (− 3)( 3 )  3
rankershalt.com

370 Straight Line and Pair of Straight Lines


2
26. To find orthocentre of the triangle formed by (0, 0) (3, 4)  1
2
 3
and (4, 0). ( A 0 A 2 )2 =  1 +  + 0 − 
 2   2
B(3, 4) 2
Y  3
2
 3 9 3 12
=   + −  = + = =3
 2  2 4 4 4
P
y) ⇒ A0 A2 = 3
(3,
H 2
 1
2
 3
and ( A 0 A 4 )2 =  1 +  +  0 + 
A  2  2
X' O X
Q (4, 0) 2
(0, 0)  3  3 9 3 12
=  +  = + = =3
Y'  2  4 4 4 4

Let H be the orthocentre of ∆OAB ⇒ A0 A4 = 3


∴ (slope of OP i.e. OH) ⋅(slope of BA) = −1 Thus, ( A0 A1 ) ( A0 A2) ( A0 A4 ) = 3
 y − 0  4 − 0 31. Since, the coordinates of the centroid are
⇒   ⋅  = −1
 3 − 0   3 − 4  x1 + x2 + x3 y1 + y2 + y3 
 ,  , then the centroid is always
4  3 3 
⇒ − y = −1 a rational point.
3
3 32. PQRS is a parallelogram if and only if the mid point of
⇒ y=
4 the diagonals PR is same as that of the mid-point of QS.
 3 That is, if and only if
∴ Required orthocentre = (3, y) = 3, 
 4 1+5 4+ a 2+ 7 6+ b
= and =
2 2 2 2
27. On solving equations 3x + 4 y = 9 and y = mx + 1, we get
⇒ a = 2 and b = 3.
5
x= 33. Slope of line x + 3 y = 4 is − 1 / 3
3 + 4m
and slope of line 6x − 2 y = 7 is 3.
Now, for x to be an integer,
 −1 
3 + 4m = ± 5 or ± 1 Here, 3 ×   = −1
 3
The integral values of m satisfying these conditions are
Therefore, these two lines are perpendicular which
−2 and −1.
show that both diagonals are perpendicular.
28. Now, distance of origin from 4x + 2 y − 9 = 0 is Hence, PQRS must be a rhombus.
| − 9| 9
= 34. Let y = cos x cos (x + 2) − cos 2 (x + 1)
4 +2
2 2 20
= cos (x + 1 − 1) cos (x + 1 + 1) − cos 2 (x + 1)
and distance of origin from 2x + y + 6 = 0 is = cos 2 (x + 1) − sin 2 1 − cos 2 (x + 1) ⇒ y = − sin 2 1
|6| 6
= This is a straight line which is parallel to X-axis.
22 + 12 5 It passes through (π / 2, − sin 2 1).
9 / 20 3 35. Orthocentre of right angled triangle is at the vertex of
Hence, the required ratio = =
6/ 5 4 right angle. Therefore, orthocentre of the triangle is at
(0, 0).
29. Let the vertices of triangle be A(1, 3 ), B(0, 0) and
C (2, 0). Here, AB = BC = CA = 2 . 36. By the given conditions, we can take two perpendicular
lines as x and y axes. If (h , k) is any point on the locus,
Therefore, it is an equilateral triangle. So, the incentre
then |h| + |k| = 1.Therefore, the locus is |x| + | y| = 1.
coincides with centroid.
This consist of a square of side 1.
0 + 1 + 2 0 + 0 + 3
∴ I ≡ ,  Hence, the required locus is a square.
 3 3 
37. Since, the origin remains the same. So, length of the
⇒ I ≡ (1, 1 / 3 ) perpendicular from the origin on the line in its position
x y x y
 3
2 2 + = 1 and + = 1 are equal. Therefore,
30. Now, ( A0 A1 )2 = 1 −  + 0 − 
1 a b p q
 2  2 1 1 1 1 1 1
= ⇒ + = +
 1
2
 3 1 3
2
1 1 1 1 a 2 b 2 p2 q 2
=   +   = + = 1 ⇒ A0 A1 = 1 + +
 2  2 4 4 a 2 b2 p2 q 2
rankershalt.com

Straight Line and Pair of Straight Lines 371

38. Let the coordinate of S be (x, y).  −c −c 


The distance between (1, 1) and  , 
 a + b a + b
Q SQ 2 + SR2 = 2SP 2
⇒ (x + 1) + y2 + (x − 2)2 + y2 = 2 [(x − 1)2 + y2]
2 i.e. less than 2 2.
⇒ x2 + 2x + 1 + y2 + x2 − 4x + 4 + y2 = 2(x2 − 2x + 1 + y2)  c  
2
c 
2

3 ⇒ 1 +  + 1 +  <2 2
⇒ 2x + 3 = 0 ⇒ x = −  a + b  a + b
2
 a + b + c
Hence, it is a straight line parallel to Y-axis. ⇒   2 <2 2
 a+b 
39. Let B, C , D be the position of the point A(4, 1) after the
three operations I, II and III, respectively. Then, B is ⇒ a + b + c < 2a + 2b
(1, 4), C (1 + 2, 4) i.e. (3, 4). The point D is obtained from ⇒ a + b−c>0
C by rotating the coordinate axes through an angle π/4 From Eqs. (i) and (ii), option (a) is correct.
in anti-clockwise direction.
Therefore, the coordinates of D are given by 43. Since, 3x + 2 y ≥ 0 …(i)
π π 1 where (1, 3) (5, 0) and (− 1, 2) satisfy Eq. (i).
X = 3 cos − 4 sin = −
4 4 2 ∴ Option (a) is true.
π π 7
and Y = 3 sin + 4 cos = Again, 2x + y − 13 ≥ 0
4 4 2
is not satisfied by (1, 3),
 1 7
∴ Coordinates of D are  − , . ∴ Option (b) is false.
 2 2
2x − 3 y − 12 ≤ 0
40. The point O(0, 0) is the mid-point of A (− a , − b) and is satisfied for all points,
B(a , b). Therefore, A , O , B are collinear and equation of ∴ Option (c) is true.
line AOB is
b and − 2x + y ≥ 0
y= x is not satisfied by (5, 0),
a
Since, the fourth point D (a 2, ab) satisfies the above ∴ Option (d) is false.
equation. Thus, (a) and (c) are correct answers.
Hence, the four points are collinear.
44. Let the variable straight line be ax + by + c = 0 ...(i)
41. Here, ax + 2 y = λ where, algebraic sum of perpendiculars from (2, 0), (0, 2)
and 3x − 2 y = µ and (1, 1) is zero.
For a = − 3, above equations will be parallel or 2a + 0 + c 0 + 2b + c a + b + c
∴ + + =0
coincident, i.e. parallel for λ + µ ≠ 0and coincident, if a 2 + b2 a 2 + b2 a 2 + b2
λ + µ = 0 and if a ≠ − 3, equations are intersecting, i.e.
⇒ 3a + 3b + 3c = 0
unique solution.
⇒ a + b + c=0 ... (ii)
42. PLAN Application of inequality sum and differences, along with
From Eqs. (i) and (ii) ax + by + c = 0 always passes
lengths of perpendicular. For this type of questions involving
inequality we should always ckeck all options. through a fixed point (1, 1).
Situation analysis Check all the inequalities 45. Let H (h , k) be orthocentre.
according to options and use length of perpendicular
⇒ (slope of AH ) ⋅ (slope of BC ) = − 1
from the point (x1 , y1) to ax + by + c = 0
ax1 + by1 + c A (–3/7, 16/7)
i.e.
a 2 + b2
0
4=

2x

As a > b > c>0


+
+

a − c > 0 and b > 0


3y
–y

H (h,k)
4x

⇒ a + b−c>0
6

…(i)
a − b > 0 and c > 0 ...(ii) x+y=1 C
B
a + c− b > 0 (–3/5, 8/5) (–3, 4)
∴ Option (c) are correct.
 16 
Also, the point of intersection for ax + by + c = 0 and k− 
⇒ 7  ⋅ (−1) = −1
bx + ay + c = 0  3
 h+ 
 −c –c   7
i.e.  , 
 a + b a + b
rankershalt.com

372 Straight Line and Pair of Straight Lines

16 3 51. Let the equation of straight line L be


⇒ k− =h+
7 7 y = mx
19
⇒ h−k=− ...(i)  1 m 
7 P≡ , 
 m + 1 m + 1
Also, (slope of CH) ⋅(slope of AB ) = − 1
k −4  3 3m 
⇒ ⋅ (4) = − 1 Q≡ , 
h+3  m + 1 m + 1
⇒ 4k − 16 = − h − 3 Y
⇒ h + 4k = 13 …(ii)
3 22 L1
On solving Eqs. (i) and (ii), we get h = , k =
7 7 R
L2 Q
 3 22
∴ Orthocentre  ,  L
7 7  P
Hence, this coordinate lies in the first quadrant. X
O
46. Since, a , b, c are in AP.
x+y=1 x+y=3
∴ 2b = a + c
or a − 2b + c = 0which satisfy ax + by + c = 0 Now, equation of
∴ ax + by + c = 0 always pass through a fixed point m −2
L1 : y − 2x = …(i)
(1, − 2). m+1
47. y = 10x is reflection of y = log10 x about y = x. and equation of
3m + 9
L 2 : y + 3x = …(ii)
48. Since, a1x + b1 y + c1 = 0 and a 2 x + b2y + c2 = 0 cuts the m+1
coordinate axes at concyclic points. By eliminating m from Eqs. (i) and (ii), we get locus of R
⇒ a1a 2 = b1b2 as x − 3 y + 5 = 0, which represents a straight line.
or a1b2 + b1a 2 = 0 52. Let L : ( y − 2) = m(x − 8), m < 0
Given lines are, 2x + 3 y + 19 = 0
 2 
and 9x + 6 y − 17 = 0 The ponts P and Q are 8 – , 0 and (0, 2 – 8 m),
 m 
Here, a1 = 2, b1 = 3, c1 = 19 respectively.
and a 2 = 9, b2 = 6, c2 = − 17  2  2 
Then,OP + OQ = 8 –  + (2 − 8 m) = 10 + – + (–8 m)
∴ a1a 2 = 18  m   m 
and b1b2 = 18 [using AM ≥ GM]
⇒ a1a 2 = b1b2. Thus, points are concyclic.
 2  2
Hence, given statement is true. ⇒   + (– 8 m) ≥ 2 16 [Q and −8m are positive]
 – m m
49. Since, (1, 3 ), (1, − 3 ) and (3, 3 ) form a right angled 2 
triangle at (1, 3 ) ⇒ –  + 8 m ≥ 8
m 
∴ Equation of circumcircle taking (3, 3 ) and (1, − 3 ) as
2 
end points of diameter. ⇒ 10 –  + 8 m ≥ 10 + 8
m 
∴ (x − 3) (x − 1) + ( y − 3 ) ( y + 3 ) = 0
⇒ OP + OQ ≥ 18
⇒ x2 − 4x + 3 + y2 − 3 = 0
⇒ x + y − 4x = 0
2 2 53. NOTE d : (P, Q ) = | x1 − x 2 | + | y1 − y2 |.

5  25 It is new method of representing distance between two


At point  , 1 , S1 = + 1 − 10 < 0
2  4 points P and Q and in future very important in coordinate
geometry.
∴ Point (5 / 2, 1) lies inside the circle.
Now, let P (x, y) be any pont in the first quadrant. We
Hence, no tangent can be drawn.
have
Hence, given statement is true.
d (P , 0) = | x − 0| + | y − 0| = | x| + | y| = x + y
50. The point of intersection of x + 2 y = 10 and 2x + y + 5 = 0 [Q x, y > 0]
 20 25 d (P , A ) = | X − 3| + |Y − 2| [given]
is  − ,  which clearly satisfy 5x + 4 y = 0.
 3 3 d (P , 0) = d (P , A ) [given]
Hence, given statement is true. ⇒ x + y =| x − 3| + | y − 2| …(i)
Case I When 0 < x < 3, 0 < y < 2
rankershalt.com

Straight Line and Pair of Straight Lines 373

In this case, Eq. (i) becomes 1 k −α 1


Slope of RQ = − ⇒ =− …(ii)
x+ y=3 − x+ 2 − y m h−b m
⇒ 2x + 2 y = 5 From Eq. (i), we get
or x + y = 5 /2 α − a = m (b + h )
Case II When 0 < x < 3, y ≥ 2 ⇒ α = a + m (b + h ) …(iii)
Now, Eq. (i) becomes and from Eq. (ii), we get
x+ y=3 − x+ y−2 1
⇒ 2x = 1 k −α = − (h − b)
m
⇒ x = 1 /2 1
Case III When x ≥ 3, 0 < y < 2 ⇒ α =k+ (h − b) …(iv)
m
Infinite segment From Eqs. (iii) and (iv), we get
Y
x = 1/2 1
a + m (b + h ) = k + (h − b)
(1/2,2)

2 y=2 m
x + ite s

⇒ am + m2 (b + h ) = km + (h − b)
Fin
y = egm

1 ⇒ (m2 − 1) h − mk + b (m2 + 1) + am = 0
5/2 ent

Hence, the locus of vertex is


,

X (m2 − 1) x − my + b (m2 + 1) + am = 0
O 1/2 (5/2,0) 3
55. Let equation of line AC is
Now, Eq. (i) becomes y+4 x+5
= =r
x+ y= x−3 + 2 − y sin θ cos θ
⇒ 2 y = − 1 or y = − 1 / 2 Let line AE make angle θ with X-axis and intersects
Hence, no solution. x + 3 y + 2 = 0 at B at a distance r1 and line 2x + y + 4 = 0
Case IV When x ≥ 3, y ≥ 2 at C at a distance r2 and line x − y − 5 = 0 at D at a
In this case, case I changes to distance r3 .
x+ y= x−3 + y−2⇒ 0 = −5 ∴ AB = r1 , AC = r2, AD = r3 .
which is not possible. −5 − 3 × 4 + 2  I′ 
r1 = − Q r = − (a cos θ + b sin θ ) 
Hence, the solution set is 1 ⋅ cos θ + 3 ⋅ sin θ  
{(x, y)| x = 12, y ≥ 2 } ∪ {(x, y)}| 15
⇒ r1 = …(i)
x + y = 5 / 2, 0 < x < 3, 0 < y > 2} cos θ + 3 sin θ
The graph is given in adjoining figure. 2 × (−5) + 1 (−4) + 4
Similarly, r2 = −
54. Let the coordinates of Q be (b, α ) and that of S be (−b, β). 2 cos θ + 1 ⋅ sin θ
Dx–y
Suppose, PR and SQ meet in G. Since, G is mid point of –5=
0
SQ, its x-coordinate must be 0. Let the coordinates of R r3
be (h, k). =0
y+4
C 2x +
Since, G is mid point of PR, the x-coordinate of P must
be −h and as P lies on the line y = a, the coordinates of P r2
are (−h , a ) . Since, PQ is parallel to y = mx, slope of B x+ 3 y + 2 = 0
PQ = m r1
α−a
⇒ =m …(i) A (–5,–4)
b+ h
Again, RQ ⊥ PQ
E
Y 10
P(–h,a) ⇒ r2 = …(ii)
2 cos θ + sin θ
−5 × 1 − 4 (−1) − 5
and r3 = −
S(–b,β) cos θ − sin θ
O 6
X′ X ⇒ r3 = …(iii)
G cos θ − sin θ
But it is given that,
2 2 2
Q (– b,α)  15   10   6 
  +  = 
 AB  AC   AD 
Y′ 2 2 2
R(h,k)  15  10  6
⇒   +  = 
 r1   r2   r3 
rankershalt.com

374 Straight Line and Pair of Straight Lines

⇒ (cos θ + 3 sin θ )2 + (2 cos + sin θ )2 = (cos θ − sin θ )2 Therefore, 2α 2 + α − 3 < 0


[from Eqs. (i), (ii) and (iii)] ⇒ 2α (α − 1) + 3 (α − 1) < 0
⇒ cos 2 θ + 9 sin 2 θ + 6 cos θ sin θ + 4 cos 2 θ ⇒
2
(2α + 3) (α − 1) < 0 ⇒ − < α < 1
+ sin 2 θ + 4 cos θ sin θ = cos 2 θ + sin 2 θ − 2 cos θ sin θ 3
⇒ 4 cos 2 θ + 9 sin 2 θ + 12 sin θ cos θ = 0 On solving Eqs. (i), (ii) and (iii), we get the common
3 1
⇒ (2 cos θ + 3 sin θ )2 = 0 answer is − < α < − 1 ∪ < α < 1.
2 2
⇒ 2 cos θ + 3 sin θ = 0
57. Let l makes an angle α with the given parallel lines and
⇒ cos θ = − (3 / 2) sin θ intercept AB is of 3 units.
On substituting this in equation of AC, we get l
y+4 x+5
= B
sin θ − sin θ
3 y + 2x = 2
2 3 3/√5
⇒ −3 ( y + 4) = 2 (x + 5)
⇒ −3 y − 12 = 2x + 10 A α y + 2x = 5
C
⇒ 2x + 3 y + 22 = 0 (2,3)
which is the equation of required straight line.
Now, distance between parallel lines
56. Given lines are 2x + 3 y − 1 = 0 …(i) |5 − 2| 3
= =
x + 2y − 3 = 0 …(ii) 1 +2
2 2 5
5x − 6 y − 1 = 0 …(iii) 1 2
∴ sin α =
, cos α =
A(–7,5) 5 5
1
tan α =
0

, and
1=

2
x+
y–

⇒ Equation of straight line passing through (2, 3 ) and


2y
+3

making an angle α with y + 2x = 5 is


–3

P (a, a2)
2x

y−3
= tan (θ + α )
0

x−2
B(1/3,1/9) 5x – 6y – 1 = 0 C(5/4,7/8) y−3 tan θ + tan α
⇒ =
On solving Eqs. (i), (ii) and (iii), we get the vertices of a x − 2 1 − tan θ tan α
 1 1  5 7 y−3 tan θ − tan α
triangle are A(−7, 5), B  ,  and C  ,  . and =
 3 9  4 8 x − 2 1 + tan θ tan α
Let P (α , α 2) be a point inside the ∆ABC. Since, A and P y−3 3 y−3 1
⇒ =− and =
are on the same side of 5x − 6 y − 1 = 0, both x−2 4 x−2 0
5 (−7) − 6 (5) − 1 and 5α − 6α 2 − 1 must have the same
sign, therefore ⇒ 3x + 4 y = 18 and x=2
5α − 6α 2 − 1 < 0 58. Let m1 and m2 be the slopes of the lines 3x + 4 y = 5 and
⇒ 6 α 2 − 5α + 1 > 0 4x − 3 y = 15, respectively.
⇒ (3α − 1) (2α − 1) > 0 3 4
Then, m1 = − and m2 =
1 1 4 3
⇒ α < or α > …(iv) Clearly, m1m2 = − 1. So, lines AB and AC are at right
3 2
 5 7 angle. Thus, the ∆ABC is a right angled isosceles
Also, since P (α , α 2) and C  ,  lie on the same side of triangle.
 4 8 Y
 5  7
2x + 3 y − 1 = 0, therefore both 2   + 3   − 1 and C
 4  8
4x − 3y − 15 = 0

2 α + 3α − 1 must have the same sign.


2
45°
Therefore, 2α + 3α 2 − 1 > 0
 1
⇒ (α + 1) α −  > 0
 3
⇒ α < − 1 ∪ α > 1 /3 …(v) 45°
X
 1 1 A (3,−1) 3x+ 4y – 5 = 0 B
and lastly  ,  and P (α , α 2) lie on the same side of the
 3 9
1  1 Hence, the line BC through (1, 2) will make an angle of
line therefore, + 2   − 3 and α + 2α 2 − 3 must have
3  9 45° with the given lines. So, the possible equations of
the same sign. BC are
rankershalt.com

Straight Line and Pair of Straight Lines 375

m ± tan 45° Then, equation of AC is


( y − 2) = (x − 1)
1 m m tan 45° x y
+ =1 …(i)
3 a h
where, m = slope of AB = −
4 and equation of DE ⊥ AC and passing through origin is
3 x y
− ±1 − =0
4 h a
⇒ ( y − 2) = (x − 1)
 3 hy
1 m −  ⇒ x= …(ii)
 4 a
−3 ± 4 On solving, Eqs. (i) and (ii), we get the coordinates of
⇒ ( y − 2) = (x − 1) point E as follows
4±3
hy y a 2h

1
( y − 2) = (x − 1) + =1 ⇒ y= 2
7 a 2
h a + h2
and ( y − 2) = − 7 (x − 1)  ah 2 a 2h 
∴Coordinate of E =  2 , 2 
⇒ x − 7 y + 13 = 0  a + h a + h 2
2

and 7x + y − 9 = 0 Since, F is mid-point of DE.


59. The equation of the line AB is  ah 2 a 2h 
x y ∴Coordinate of F  , 2 
+ =1 … (i) 2 (a + h ) 2 (a + h ) 
2 2 2
7 −5
⇒ 5x − 7 y = 35 ∴ Slope of AF ,
a 2h
Equation of line perpendicular to AB is h−
2 (a 2 + h 2) 2h (a 2 + h 2) − a 2h
7x + 5 y = λ …(ii) m1 = =
ah 2 − ah 2
It meets X-axis at P(λ/7, 0) and Y-axis at Q(0, λ /5). 0−
x 5y 2 (a + h )
2 2

The equations of lines AQ and BP are + = 1 and


7 λ − (a 2 + 2h 2)
⇒ m1 = …(iii)
7x y ah
− = 1, respectively.
λ 5 2
a h
−0
Let R(h , k) be their point of intersection of lines a + h2
2
a 2h
AQ and BP. and slope of BE , m2 = =
ah 2
ah + a3 + ah 2
2
h 5k +a
Then, + =1 a +h
2 2
7 λ
ah
and
7h k
− =1 ⇒ m2 = 2 …(iv)
λ 5 a + 2h 2

1  h 1  k From Eqs. (iii) and (iv), m1m2 = − 1 ⇒ AF ⊥ BE


⇒ 1 −  = 1 +  [on eliminating λ]
5k  7  7h  5 61. Let the coordinates of B and C be (x1 , y1 ) and (x2, y2)
respectively. Let m1 and m2 be the slopes of AB and AC,
⇒ h (7 − h ) = k(5 + k)
respectively. Then,
⇒ h 2 + k2 − 7h + 5k = 0 A(1,–2)
Hence, the locus of a point is
x_y+5=0 x + 2y = 0
x2 + y2 − 7x + 5 y = 0.
F E
60. Let BC be taken as X-axis with origin at D, the
mid-point of BC and DA will be Y-axis.
Given, AB = AC
Let BC = 2a, then the coordinates of B and C are (– a, 0) B C
(x1, y1) (x2, y2)
and (a , 0) let A (0, h ).
y1 + 2
Y m1 = slope of AB =
x1 − 1
A y +2
and m2 = slope of AC = 2
x2 − 1

E Let F and E be the mid point of AB and AC, respectively.


F Then, the coordinates of E and F are
X' X
B D C  x + 1 y2 − 2  x + 1 y1 − 2
E 2 ,  and F  1 ,  , respectively.
 2 2   2 2 
Y'
rankershalt.com

376 Straight Line and Pair of Straight Lines

Now, F lies on x − y + 5 = 0. 63. Let the coordinates of A be (0, α ). Since, the sides AB
x1 + 1 y1 − 2 and AD are parallel to the lines y = x + 2 and y = 7x + 3,
⇒ − = −5
2 2 respectively.
⇒ x1 − y1 + 13 = 0 …(i) Y
Since, AB is perpendicular to x − y + 5 = 0.
D C
∴ (slope of AB) ⋅ (slope of x − y + 5 = 0) = − 1.
y1 + 2
⇒ ⋅ (1) = − 1
x1 − 1

E(
1,
⇒ y1 + 2 = − x1 + 1

2)
A (0, α) B
⇒ x1 + y1 + 1 = 0 …(ii)
On solving Eqs. (i) and (ii), we get X′ X
O
x1 = − 7, y1 = 6.
So, the coordinates of B are (−7, 6). Y′
Now, E lies on x + 2 y = 0.
∴ The diagonal AC is parallel to the bisector of the angle
x2 + 1  y − 2
∴ +2 2  =0 between these two lines. The equation of the bisectors
2  2  are given by
x− y+2 7x − y + 3
⇒ x2 + 2 y2 − 3 = 0. …(iii) =±
2 50
Since, AC is perpendicular to x + 2 y = 0
∴ (slope of AC) ⋅ (slope of x + 2 y = 0) = − 1 ⇒ 5 (x − y + 2) = ± (7x − y + 3)
y2 + 2  1 ⇒ 2x + 4 y − 7 = 0 and 12x − 6 y + 13 = 0.
⇒ ⋅ −  = − 1 Thus, the diagonals of the rhombus are parallel to the
x2 − 1  2
lines 2x + 4 y − 7 = 0 and 12x − 6 y + 13 = 0.
⇒ 2x2 − y2 = 4 … (iv)
2 12
On solving Eqs. (iii) and (iv), we get ∴ Slope of AE = − or
4 6
11 2
x2 = and y2 = 2 −α 1 2 −α
5 5 ⇒ =− or =2
1 −0 2 1 −0
 11 2
So, the coordinates of C are  , . 5
 5 5 ⇒ α= or α = 0.
2
Thus, the equation of BC is Hence, the coordinates are (0, 5 /2) or (0, 0).
2 /5 − 6
y −6 = (x + 7) 64. The equation of any line passing through (1, − 10) is
11 / 5 + 7
⇒ − 23 ( y − 6) = 14 (x + 7) y + 10 = m (x − 1).
⇒ 14x + 23 y − 40 = 0 Since, it makes equal angles, say θ, with the given lines,
therefore
62. Let O be the centre of circle and M be mid-point of AB. m−7 m − (−1) 1
tan θ = = ⇒ m = or –3
1 + 7m 1 + m(−1) 3
A M B Hence, the equations of third side are
(– 3, 4) (5, 4) 1
y + 10 = (x − 1) or y + 10 = − 3 (x − 1)
O
3
i.e. x − 3 y − 31 = 0 or 3x + y + 7 = 0
D C 65. Let ABC be a triangle whose vertices are
(a, b)
A [at1t2, a (t1 + t2)], B [at2t3 , a (t2 + t3 )] and
C [at1t3 , a (t1 + t3 )].
Then, OM ⊥ AB ⇒ M (1, 4) a (t 2 + t3 ) − a (t1 + t3 ) 1
Then, Slope of BC = =
Since, slope of AB = 0 at 2t3 − at1t3 t3
Equation of straight line MO is x = 1 and equation of a (t1 + t3 ) − a (t1 + t2) 1
Slope of AC = =
diameter is 4 y = x + 7. at1t3 − at1t2 t1
⇒ Centre is (1, 2). Also, O is mid-point of BD So, the equation of a line through A perpendicular to BC
 α + 5 β + 4 is y − a (t1 + t2) = − t3 (x − at1t2) … (i)
⇒ ,  = (1, 2) ⇒ α = − 3, β = 0
 2 2  and the equation of a line through B perpendicular to
∴ AD = (−3 + 3)2 + (4 − 0)2 = 4 AC is
y − a (t2 + t3 ) = − t1 (x − at2t3 ) …(ii)
and AB = 64 + 0 = 8 The point of intersection of Eqs. (i) and (ii), is the
Thus, area of rectangle = 8 × 4 = 32 sq units orthocentre.
rankershalt.com

Straight Line and Pair of Straight Lines 377

On subtracting Eq. (ii) from Eq. (i), we get x = − a. ⇒ c = −4


On putting x = − a in Eq. (i), we get ∴ Other two vertices lies on y = 2x − 4
y = a (t1 + t2 + t3 + t1t2t3 ) Let the coordinate of B be (x, 2x − 4).
Hence, the coordinates of the orthocentre are ∴ Slope of AB ⋅ Slope of BC = − 1
[− a , a (t1 + t2 + t3 + t1t2t3 )].  2 x − 4 − 3  2 x − 4 − 1
⇒   ⋅  = −1
66. Let OA = a and OB = b. Then, the coordinates of A and B  x−1   x−5 
are (a, 0) and (0, b) respectively and also, coordinates of ⇒ (x2 − 6x + 8) = 0
P are (a , b). Let θ be the foot of perpendicular from P on
⇒ x = 4, 2
AB and let the coordinates of Q (h , k). Here, a and b are
the variable and we have to find locus of Q. ⇒ y = 4, 0
Given, AB = c ⇒ AB2 = c2 Hence, required points are (4, 4), (2, 0).
⇒ OA + OB2 = c2 ⇒ a 2 + b2 = c2
2
…(i)
Since, PQ is perpendicular to AB. 68. Let the coordinates of third vertex be C (a , b).
Y C (a,b)

B
(0,b) P(a,b)
Q

)
(h

0,0
,k

H(
)

X′ X
O A(a,0)
Y′
⇒ Slope of AB ⋅ Slope of PQ = − 1 A (5,–1) B (–2,3)
0−b k−b
⇒ ⋅ = −1 Since, CH is ⊥ AB,
a −0 h − a
 b  4 
⇒ bk − b2 = ah − a 2 ∴     = −1
 a   −7 
⇒ ah − bk = a 2 − b2 …(ii)
x y ⇒ 4b = 7a ...(i)
Equation of line AB is + = 1. Also, AH ⊥ BC
a b
 1  3 − b 
Since, Q lies on AB, therefore ∴ −    = −1
h k  5   −2 − a 
+ =1
a b ⇒ 3 − b = −10 − 5a ...(ii)
⇒ bh + ak = ab …(iii) On solving Eqs. (i) and (ii), we get
On solving Eqs. (ii) and (iii), we get a = −4 , b = −7
h k 1
= = 69. Since, the side AB is perpendicular to AD.
ab2 + a (a 2 − b2) − b(a 2 − b2) + a 2b a 2 + b2
D C (1,1)
h k 1
4 x + 7y + 5 = 0

⇒ 3
= 3 = 2 [from Eq. (i)]
a b c
⇒ a = (hc2)1/3 and b = (kc2)1/3
On substituting the values of a and b in a 2 + b2 = c2,
we get h 2 / 3 + k2 / 3 = c2 / 3
Hence, locus of a point is x2 / 3 + y2 / 3 = c2 / 3 . A (–3,1) B
67. Since, diagonals of rectangle bisect each other, so ∴ Its equation is of the form 7x − 4 y + λ = 0
mid point of (1, 3) and (5, 1) must satisfy y = 2x + c, i.e. Since, it passes through (−3, 1).
(3, 2) lies on it. ∴ 7(−3) − 4(1) + λ = 0.
⇒ λ = 25
D C(5,1)
∴ Equation of AB is
7x − 4 y + 25 = 0
Now, BC is parallel to AD. Therefore, its equation is
y= 4x + 7 y + λ = 0
2x
A(1,3) B + Since, it passes through (1, 1).
c
∴ 4(1) + 7(1) + λ = 0
⇒ 2 =6 + c
⇒ λ = − 11
rankershalt.com

378 Straight Line and Pair of Straight Lines

∴ Equation of BC is 4x + 7 y − 11 = 0 3. The line segment QR makes an angle of 60° with the


Now, equation of DC is 7x − 4 y + λ = 0 positive direction of X-axis.
⇒ 7(1) − 4(1) + λ = 0 So, the bisector of the angle PQR will make an angle
⇒ λ = −3 of 60° with the negative direction of X-axis it will
∴ 7x − 4 y − 3 = 0 therefore have angle of inclination of 120° and so, its
equation is
70. PLAN Distance of a point ( x 1, y1 ) from ax + by + c = 0 is given by
y − 0 = tan 120° (x − 0)
ax 1 + by1 + c
. ⇒ y = − 3x
a2 + b 2
⇒ y + 3x = 0
y=x
x<y 4. It is not necessary that the bisector of an angle will
2√2 divide the triangle into two similar triangles, therefore,
statement II is false.
Now, we verify Statement I.
√2 x≥y
∆ OPQ, OR is the internal bisector of ∠POQ.
PR OP
√2 2√2 ∴ =
RQ OQ
Let P (x, y) is the point in first quadrant.
PR 22 + 22 2 2
x− y x+ y ⇒ = =
Now, 2 ≤ + ≤4 RQ 12 + 22 5
2 2
2 2 ≤ |x − y| + | x + y| ≤ 4 2 1 − (− 7)
5. Equation of the line AB is y − 1 = (x − 5)
Case I x ≥ y 5 − (− 1)
8 4
2 2 ≤ (x − y) + (x + y) ≤ 4 2 ⇒ x ∈[ 2, 2 2] ⇒ y − 1 = (x − 5) ⇒ y − 1 = (x − 5)
6 3
Case II x < y
⇒ 3 y − 3 = 4x − 20
2 2 ≤ y − x + (x + y) ≤ 4 2 ⇒ 3 y − 4x + 17 = 0
y ∈[ 2, 2 2]
Equation of the line BC is
⇒ A = (2 2 )2 − ( 2 )2 = 6 sq units 4 −1 3
y−4 = (x − 1) ⇒ y−4 = − (x − 1)
1 −5 4
Topic 2 Angle between Straight Lines and ⇒ 4 y − 16 = − 3x + 3 ⇒ 3x + 4 y − 19 = 0
Equation of Angle Bisector
Again, equation of the bisectors of the angles between
1. Take any point B (0, 1) on given line. two given lines AB and BC are
3 y − 4x + 17 4 y + 3x − 19
B (0, 1) √ 3y =x – 1 =±
3 +4
2 2
42 + 32
⇒ 3 y − 4x + 17 = ± (4 y + 3x − 19)
A (√ 3, 0) ⇒ 3 y − 4x + 17 = 4 y + 3x − 19
and 3 y − 4x + 17 = − (4 y + 3x − 19)
B′ (0, –1) ⇒ 36 = y + 7x and 7 y − x = 2
Out of these two, equation of the bisector of angle ABC is
Equation of AB′ is
7 y = x + 2.
−1 −0
y−0 = (x − 3 )
0− 3 6. Here, the triangle formed by a line parallel to X-axis
passing through P (h , k) and the straight line y = x and
⇒ − 3y = − x + 3 y = 2 − x could be as shown below:
⇒ x − 3y = 3
Y
⇒ 3y = x − 3
y=x
2. For collinear points (1, 1)
− sin( β − α ) − cos β 1 A

∆= cos( β − α ) sin β 1 (k, k)


C (2, −k, k)
B
cos( β − α + θ ) sin( β – θ ) 1 p (h, k)
Y=K
X′
O
Clearly, ∆ ≠ 0 for any value of α , β , θ. y=x y=2−x
Y
Hence, points are non-collinear.
rankershalt.com

Straight Line and Pair of Straight Lines 379

Since, area of ∆ABC = 4h 2 On substituting the value of λ in Eq. (i), we get


1
AB ⋅ AC = 4h 2 (a 2 + b2)
(ax + by + c) − (lx + my + n ) = 0
2 2 (al + bm)
where, AB = 2 |k – 1| and AC = 2 (|k – 1|) ⇒ 2 (al + bm) (ax + by + c) − (a 2 + b2) (lx + my + n ) = 0
1 which is the required equation of line L.
⇒ ⋅ 2(k − 1)2 = 4h 2
2
⇒ 4h 2 = (k − 1)2 Topic 3 Area and Family of
⇒ 2h = ± (k − 1) Concurrent Lines
The locus of a point is 2x = ± ( y − 1).
1. Let a ∆ABC is such that vertices
7. Given equations of lines are A (1, 2), B(x1 y1 ) and C (x2, y2).
x − 2 y + 4 = 0 and 4x − 3 y + 2 = 0
A(1,2)
Here, a1a 2 + b1b2 = 1(4) + (−2)(−3) = 10 > 0
For obtuse angle bisector, we take negative sign.
x −2y + 4 4x − 3 y + 2
∴ =−
5 5 B(x1,y1) C(x2,y2)
⇒ 5 (x − 2 y + 4) = − (4x − 3 y + 2)
⇒ (4 + 5 )x − (2 5 + 3) y + (4 5 + 2) = 0 It is given that mid-point of side AB is (− 1, 1).
x1 + 1
8. Since, the required line L passes through the So, = −1
2
intersection of L1 = 0 and L 2 = 0. y1 + 2
L2 and =1
2
⇒ x1 = − 3 and y1 = 0
A P θ So, point B is (− 3, 0)
L
(x1,y1) θ Also, it is given that mid-point of side AC is
(2, 3), so
x2 + 1 y +2
L1 = 2 and 2 =3
2 2
So, the equation of the required line L is ⇒ x2 = 3 and y2 = 4
L1 + λL 2 = 0. So, point C is (3, 4).
i.e. (ax + by + c) + λ (lx + my + n ) = 0 … (i) Now, centroid of ∆ABC is
where, λ is a parameter. 1 + (− 3) + 3 2 + 0 + 4 
G  , 1 
 = G  , 2
Since, L1 is the angle bisector of L = 0 and L 2 = 0.  3 3  3 

∴ Any point A (x1 , y1 ) on L1 is equidistant from L1 = 0 2 Given, px + qy + r = 0 is the equation of line such that
and L 2 = 0. 3 p + 2q + 4r = 0
| lx1 + my1 + n |
⇒ Consider, 3 p + 2q + 4r = 0
l2 + m 2 3 p 2q
⇒ + + r=0
|(ax1 + by1 + c) + λ (lx1 + my1 + n )| 4 4
= …(ii)
(a + λl)2 + (b + λm)2 (dividing the equation by 4)
p   + q  + r = 0
3 1

But, A (x1 , y1 ) lies on L1. So, it must satisfy the equation  4  2
of L1 , ie, ax1 + by1 + c1 = 0. ⇒  ,
3 1
 satisfy px + qy + r = 0
On substituting ax1 + by1 + c = 0 in Eq. (ii), we get 4 2
So, the lines always passes through the point  ,  .
3 1
| lx1 + my1 + n | |0 + λ (lx1 + my1 + n )|  4 2
=
l2 + m 2 (a + λl)2 + (b + λm)2 3. As the given lines x − y + 1 = 0 and 7x − y − 5 = 0 are not
parallel, therefore they represent the adjacent sides of
⇒ λ2(l2 + m2) = (a + λl)2 + (b + λm)2
the rhombus.
(a + b )
2 2
∴ λ=− On solving x − y + 1 = 0 and 7x − y − 5 = 0, we get x = 1
2 (al + bm)
and y = 2. Thus, one of the vertex is A(1, 2).
rankershalt.com

380 Straight Line and Pair of Straight Lines

D C (x, y)
6. The points of intersection of three lines are

=0
A (1, 1), B(2, − 2), C (−2, 2).

y–5
7x – (–1, –2) Now, | AB| = 1 + 9 = 10 ,
|BC| = 16 + 16 = 4 2,
A x − y + 1=0 B and |CA| = 9 + 1 = 10
(1, 2) ∴ Triangle is an isosceles.
Let the coordinate of point C be (x, y). 7. Given lines, x + 2 y − 3 = 0 and 3x + 4 y − 7 = 0 intersect
x+1 y+2 at (1,1), which does not satisfy 2x + 3 y − 4 = 0 and
Then, −1 = and − 2 =
2 2 4x + 5 y − 6 = 0.
⇒ x + 1 = − 2 and y = − 4 − 2 Also, 3x + 4 y − 7 = 0 and 2x + 3 y − 4 = 0 intersect at
⇒ x= −3 (5, –2) which does not satisfy x + 2 y − 3 = 0 and
and y= −6 4x + 5 y − 6 = 0 .
Hence, coordinates of C = (− 3, − 6) Lastly, intersection point of x + 2 y − 3 = 0 and
Note that, vertices B and D will satisfy x − y + 1 = 0 and 2x + 3 y − 4 = 0 is (–1, 2) which satisfy 4x + 5 y − 6 = 0.
Hence, only three lines are concurrent.
7x − y − 5 = 0, respectively.
8. Given lines px + qy + r = 0, qx + ry + p = 0
Since, option (c) satisfies 7x − y − 5 = 0, therefore
 1 − 8 and rx + py + q = 0 are concurrent.
coordinate of vertex D is  , . p q r
3 3 
∴ q r p =0
4. Let lines OB : y = mx r p q
CA : y = mx + 1
Applying R1 → R1 + R2 + R3 and taking common from
BA : y = nx + 1 R1
and OC : y = nx 1 1 1
The point of intersection B of OB and AB has x ( p + q + r) q r p = 0
1 r p q
coordinate ⋅
m−n
⇒ ( p + q + r ) ( p2 + q2 + r 2 − pq − qr − pr ) = 0
Y
⇒ p3 + q3 + r3 − 3 pqr = 0
A Therefore, (a) and (c) are the answers.

C D B 9. (A) Solving equations L1 and L3 ,


x y 1
X′
O
X = =
−36 + 10 −25 + 12 2 − 15
∴ x = 2, y = 1
Y′ L1 , L 2, L3 are concurrent, if point (2, 1) lies on L 2
Now, area of a parallelogram OBAC ∴ 6 − k −1 =0 ⇒ k =5
= 2 × area of ∆ OBA (B) Either L1 is parallel to L 2,or L3 is parallel to L 2,then
1 1 1 3 −k
= 2 × × OA × DB = 2 × × 1
=
3
or = ⇒ k = −9
2 2 m−n 3 −k 5 2
=
1
=
1 −6
or k=
m − n |m − n | 5
depending upon whether m > n or m < n. (C) L1 , L 2, L3 form a triangle, if they are not concurrent,
or not parallel.
5. Since, vertices of a triangle are (0, 8 / 3), (1, 3) and (82, 30) 6 5
∴ k ≠ 5, − 9, − ⇒ k =
0 8 /3 1 5 6
1 (D) L1 , L 2, L3 do not form a triangle, if
Now, 1 3 1
2 6
82 30 1 k = 5, − 9, − ⋅
5
1 8 
= − (1 − 82) + 1(30 − 246) 10. The set of lines ax + by + c = 0, where 3a + 2b + 4c = 0 or
2  3 
3 1  3 1
1 a + b + c = 0 are concurrent at  x = , y =  i.e.
= [216 − 216] = 0 4 2  4 2
2
comparing the coefficients of x and y.
∴ Points are collinear.  3 1
Thus, point of concurrency is  ,  .
 4 2
rankershalt.com

Straight Line and Pair of Straight Lines 381

Alternate Solution 1 λ 
⇒ λ⋅ =5
As, ax + by + c = 0, satisfy 3a + 2b + 4c = 0 which 2 5 
represents system of concurrent lines whose point of ⇒ λ2 = 50
concurrency could be obtained by comparison as, ⇒ |λ | = 5 2
3a 2 ∴ Equation of the line L is, x + 5 y = ± 5 2
ax + by + c ≡ + b+ c
4 4
3 1
⇒ x = , y = is point of concurrency. Topic 4 Homogeneous Equation of
4 2
 3 1
Pair of Straight Lines
∴  ,  is the required point.
 4 2 1. Let a and b be non-zero real numbers.
x1 y1 1 a1 b1 1 Therefore the given equation
11. Since, x2 y2 1 = a 2 b2 1 (ax2 + by2 + c) (x2 − 5xy + 6 y2) = 0 implies either
x3 y3 1 a3 b3 1
x2 − 5xy + 6 y2 = 0
represents area of triangles are equal, which does not ⇒ (x − 2 y)(x − 3 y) = 0
impies triangles are congrvent. Hence, given statement ⇒ x = 2y
is false. and x = 3y
12. Let the vertices of a triangle be, O (0, 0) A (a , 0) and represent two straight lines passing through origin or
B (b, c) equation of altitude BD is x = b. ax2 + by2 + c = 0 when c = 0 and a and b are of same
c signs, then
Slope of OB is .
b
b ax2 + by2 + c = 0,
Slope of AF is − . x=0
c
and y = 0.
Now, the equation of altitude AF is
b which is a point specified as the origin.
y − 0 = − (x − a )
c When, a = b and c is of sign opposite to that of a,
Suppose, BD and OE intersect at P. ax2 + by2 + c = 0 represents a circle.
  (a − b)  Hence, the given equation,
Coordinates of P are b, b  
  c  
(ax2 + by2 + c)(x2 − 5xy + 6 y2) = 0
a−b may represent two straight lines and a circle.
Let m1 be the slope of OP =
c
c 2. Given, x2 − y2 + 2 y = 1
and m2 be the slope of AB =
b−a ⇒ x2 = ( y − 1)2
⇒ x = y − 1 and x = − y + 1
 a − b  c 
Now, m1m2 =     = −1 Y
 c   b − a

We get, that the line through O and P is perpendicular (0,3)


to AB.
x+
y=

1
|x (5 + 2) + (−3) (−2 − y) + 4 ( y − 5)|
3

Area of ∆PBC 2 Angle bisector


13. = (0, 1) (2,1)
Area of ∆ABC 1 |6 (5 + 2) + (−3) (−2 − 3) + 4 (3 − 5)|
2 X′ X
|7x + 7 y − 14| 7|x + y − 2|  x + y − 2  −1 O 1 3
= = = y=x+1 y=
|42 + 15 − 8| 49  7  –x
+1
14. A straight line perpendicular to 5x − y = 1 is x + 5 y = λ. Y′
Angle bisector
Since, area of triangle = 5
Y From the graph, it is clear that equation of angle
bisectors are
y=1
λ/5 and x=0
X′ X
O λ ∴ Area of region bounded by x + y = 3, x = 0
1
and y = 1 is ∆ = × 2 × 2 = 2 sq units
Y′ 2
rankershalt.com

382 Straight Line and Pair of Straight Lines

3. The given curve is ∴ 90° + ∠ Q + ∠ R = 180°


3x − y − 2x + 4 y = 0
2 2
…(i) ⇒ ∠ Q = ∠ R = 45°
Y
Let y = mx + c be the chord of curve (i) which subtend
right angle at origin. Then, the combined equation of
Q
lines joining points of intersection of curve (i) and chord P(2,1)
90°
y = mx + c to the origin, can be obtained by the equation
of the curve homogeneous, i.e.
S
 y − mx  y − mx
3x2 − y2 − 2x   + 4y   =0
 c   c 
X' X
⇒ 3cx2 − cy2 − 2xy + 2mx2 + 4 y2 − 4mxy = 0 O
Y'
R
⇒ (3c + 2m) x2 − 2 (1 + 2m) y + (4 − c) y2 = 0
Now, slope of QR is −2 . [given]
Since, the lines represented are perpendicular to each
But QR ⊥ PS.
other.
∴ Slope of PS is 1/2.
∴ Coefficient of x2 + Coefficient of y2 = 0
Let m be the slope of PQ .
⇒ 3c + 2m + 4 − c = 0 m − 1 /2
⇒ c+ m + 2 =0 ∴ tan (± 45° ) =
1 − m (−1 / 2)
On comparing with y = mx + c 2m − 1
⇒ y = mx + c passes through (1, –2). ⇒ ±1=
2+m
⇒ m = 3, − 1 / 3
Topic 5 General Equation of Pair of ∴ Equations of PQ and PR are
Straight Lines y − 1 = 3 (x − 2)
1
1. Let S be the mid-point of QR and given ∆PQR is an and y − 1 = − (x − 2)
isosceles. 3
or 3 ( y − 1) + (x − 2) = 0
Therefore, PS ⊥ QR and S is mid-point of hypotenuse, Therefore, joint equation of PQ and PR is
therefore S is equidistant from P , Q , R.
[3 (x − 2) − ( y − 1)] [(x − 2) + 3 ( y − 1)] = 0
∴ PS = QS = RS
⇒ 3 (x − 2)2 − 3 ( y − 1)2 + 8 (x − 2) ( y − 1) = 0
Since, ∠ P = 90° and ∠ Q = ∠ R
⇒ 3x2 − 3 y2 + 8xy − 20x − 10 y + 25 = 0
But ∠ P + ∠ Q + ∠ R = 180°

Download Chapter Test


http://tinyurl.com/y5bdtgdz or
rankershalt.com

16
Circle
Topic 1 Equation of Circle
Objective Questions I (Only one correct option) 8. Let AB be a chord of the circle x2 + y2 = r 2 subtending a
1. A circle touching the X-axis at (3, 0) and making a right angle at the centre. Then, the locus of the centroid
intercept of length 8 on the Y -axis passes through the of the ∆ PAB as P moves on the circle, is (2001, 1M)
point (2019 Main, 12 April II) (a) a parabola (b) a circle
(a) (3, 10) (b) (3, 5) (c) an ellipse (d) a pair of straight lines
(c) (2, 3) (d) (1, 5) 9. The lines 2x − 3 y = 5 and 3x − 4 y = 7 are diameters of a
2. Let O(0, 0) and A(0, 1) be two fixed points, then the locus circle of area 154 sq units. Then, the equation of this
of a point P such that the perimeter of ∆AOP is 4, is circle is (1989, 2M)
(2019 Main, 8 April I) (a) x2 + y 2 + 2x − 2 y = 62 (b) x2 + y 2 + 2x − 2 y = 47
(a) 8x − 9 y + 9 y = 18
2 2
(b) 9x − 8 y + 8 y = 16
2 2 (c) x2 + y 2 − 2x + 2 y = 47 (d) x2 + y 2 − 2x + 2 y = 62
(c) 9x2 + 8 y 2 − 8 y = 16 (d) 8x2 + 9 y 2 − 9 y = 18 10. AB is a diameter of a circle and C is any point on the
3. If a circle of radius R passes through the origin O and circumference of the circle. Then, (1983, 1M)
intersects the coordinate axes at A and B, then the locus (a) the area of ∆ ABC is maximum when it is isosceles
of the foot of perpendicular from O on AB is (b) the area of ∆ ABC is minimum when it is isosceles
(2019 Main, 12 Jan II) (c) the perimeter of ∆ ABC is minimum when it is
(a) (x2 + y 2 )2 = 4R 2x 2 y 2 isosceles
(b) (x2 + y 2 )3 = 4R 2x 2 y 2 (d) None of the above
(c) (x2 + y 2 )(x + y) = R 2xy 11. The centre of the circle passing through the point (0, 1)
(d) (x2 + y 2 )2 = 4Rx 2 y 2 and touching the curve y = x2 at (2, 4) is (1983, 1M)
4. A square is inscribed in the circle
(a)  − ,
16 27 
(b)  − ,
16 53 
x2 + y2 − 6x + 8 y − 103 = 0 with its sides parallel to the  
 5 10   7 10 
coordinate axes. Then, the distance of the vertex of this
(c)  −
16 53 
square which is nearest to the origin is ,  (d) None of the above
(2019 Main, 11 Jan II)
 5 10 
(a) 6 (b) 13 (c) 41 (d) 137
Objective Questions II
5. If the area of an equilateral triangle inscribed in the
circle, x + y + 10x + 12 y + c = 0
2 2 (One or more than one correct option)
is 27 3 sq units, then c is equal to (2019 Main, 10 Jan II) 12. Circle(s) touching X-axis at a distance 3 from the origin
(a) 20 (b) −25 (c) 13 (d) 25 and having an intercept of length 2 7 on Y-axis is/are
6. Let the orthocentre and centroid of a triangle be A(−3, 5) (a) x2 + y 2 − 6x + 8 y + 9 = 0 (2013 Adv.)

and B(3, 3), respectively. If C is the circumcentre of this (b) x2 + y 2 − 6x + 7 y + 9 = 0


triangle, then the radius of the circle having line (c) x2 + y 2 − 6x − 8 y + 9 = 0
segment AC as diameter, is (2018 Main) (d) x2 + y 2 − 6x − 7 y + 9 = 0

(a) 10 (b) 2 10 (c) 3


5
(d)
3 5 13. Let L1 be a straight line passing through the origin and
2 2 L 2 be the straight line x + y = 1. If the intercepts made
7. The centre of circle inscribed in square formed by the by the circle x2 + y2 − x + 3 y = 0 on L1 and L 2 are equal,
lines x2 − 8x + 12 = 0 and y2 − 14 y + 45 = 0, is (2003, 1M) then which of the following equation can represent L1?
(a) x + y = 0 (b) x − y = 0 (1999, 1M)
(a) (4, 7) (b) (7, 4)
(c) x + 7 y = 0 (d) x − 7 y = 0
(c) (9, 4) (d) (4, 9)
rankershalt.com

384 Circle

Fill in the Blanks Integer Answer Type Questions


14. The lines 3x − 4 y + 4 = 0 and 6x − 8 y − 7 = 0 tangents to 22. For how many values of p, the circle
the same circle. The radius of this circle is… . (1984, 2M) x2 + y2 + 2x + 4 y − p = 0 and the coordinate axes have
15. If A and B are points in the plane such that PA / PB = k exactly three common points? (2017 Adv.)
(constant) for all P on a given circle, then the value of k 23. The straight line 2x − 3 y = 1 divide the circular region
cannot be equal to ……… . (1982, 2M)
x2 + y2 ≤ 6 into two parts. If
 3   5 3   1 1   1 1  
True/False S = 2,  ,  ,  ,  ,−  ,  ,  , then the number of
 4   2 4   4 4   8 4  
16. The line x + 3 y = 0 is a diameter of the circle point (s) in S lying inside the smaller part is ... .
x2 + y2 − 6x + 2 y = 0. (1989, 1M)
Paragraph Based Questions
Analytical & Descriptive Questions Let S be the circle in the XY -plane defined by the equation
17. Let C be any circle with centre (0, 2 ). Prove that x2 + y2 = 4. (2018 Adv.)
at most two rational points can be there on C.
(There are two questions based on above Paragraph, the
(A rational point is a point both of whose coordinates are
question given below is one of them)
rational numbers.) (1997, 5M)
18. Consider a curve ax2 + 2hxy + by2 = 1 and a point P not 24. Let E1E 2 and F1F2 be the chords of S passing through
on the curve. A line drawn from the point P intersect the the point P0 (1, 1) and parallel to the X-axis and the
curve at points Q and R. If the product PQ ⋅ QR is Y -axis, respectively. Let G1G2 be the chord of S passing
independent of the slope of the line, then show that the through P0 and having slope −1. Let the tangents to S at
curve is a circle. (1997, 5M) E1 and E 2 meet at E3 , then tangents to S at F1 and F2
meet at F3 , and the tangents to S at G1 and G2 meet at
19. A circle passes through three points A, B and C with the G3 . Then, the points E3 , F3 and G3 lie on the curve
line segment AC as its diameter. A line passing through
(a) x + y = 4
A intersects the chord BC at a point D inside the circle. If
angles DAB and CAB are α and β respectively and the (b) (x − 4)2 + ( y − 4)2 = 16
distance between the point A and the mid-point of the (c) (x − 4) ( y − 4) = 4
line segment DC is d, prove that the area of the circle is (d) xy = 4
πd 2 cos 2 α 25. Let P be a point on the circle S with both coordinates
(1996, 5M) being positive. Let the tangent to S at P intersect the
cos α + cos β + 2 cos α cos β cos (β − α )
2 2
coordinate axes at the points M and N . Then, the
20. If (mi , 1 / mi ), mi > 0, i = 1, 2, 3, 4 are four distinct points mid-point of the line segment MN must lie on the
on a circle , then show that m1 m2 m3 m4 = 1. (1989, 2M) curve
(a) (x + y)2 = 3xy
21. The abscissae of the two points A and B are the roots of
the equation x + 2ax − b = 0 and their ordinates are the
2 2 (b) x23
/
+ y23/
= 24/3
roots of the equation y2 + 2 py − q2 = 0. Find the equation (c) x + y = 2xy
2 2

and the radius of the circle with AB as diameter. (d) x2 + y2 = x2 y2


(1984, 4M)

Topic 2 Relation between Two Circles


Objective Questions I (Only one correct option) 3. If a variable line, 3x + 4 y − λ = 0 is such that the two
1. If the circles x + y + 5Kx + 2 y + K = 0
2 2
and circles x2 + y2 − 2x − 2 y + 1 = 0
2 (x2 + y2) + 2Kx + 3 y −1 = 0, (K ∈ R), intersect at the and x2 + y2 − 18x − 2 y + 78 = 0
points P and Q, then the line 4x + 5 y − K = 0 passes are on its opposite sides, then the set of all values of λ is
through P and Q, for (2019 Main, 10 April I) the interval (2019 Main, 12 Jan I)
(a) no values of K (a) [13, 23] (b) (2, 17)
(b) exactly one value of K (c) [12, 21] (d) (23, 31)
(c) exactly two values of K
4 Let C1 and C 2 be the centres of the circles
(d) infinitely many values of K
x2 + y2 − 2x − 2 y − 2 = 0 and x2 + y2 − 6x − 6 y + 14 = 0
2. If a tangent to the circle x2 + y2 = 1 intersects the respectively. If P and Q are the points of intersection of
coordinate axes at distinct points P and Q, then the locus these circles, then the area (in sq units) of the
of the mid-point of PQ is (2019 Main, 9 April I) quadrilateral PC1QC 2 is (2019 Main, 12 Jan I)

(a) x2 + y2 − 2x2 y2 = 0 (b) x2 + y2 − 2xy = 0 (a) 8 (b) 4


(c) 6 (d) 9
(c) x + y − 4x y = 0
2 2 2 2
(d) x + y − 16x y = 0
2 2 2 2
rankershalt.com

Circle 385

5. If the circles x2 + y2 −16x − 20 y + 164 = r 2 and Objective Question II


(x − 4)2 + ( y − 7)2 = 36 intersect at two distinct points, (One or more than one correct option)
then (2019 Main, 9 Jan II)
(a) 0 < r < 1 (b) r > 11 16. Let T be the line passing through the points P(− 2, 7)
(c) 1 < r < 11 (d) r = 11 and Q(2, − 5). Let F1 be the set of all pairs of circles
6. If one of the diameters of the circle, given by the (S1 , S 2) such that T is tangent to S1 at P and tangent to
equation, x2 + y2 − 4x + 6 y − 12 = 0, is a chord of a circle S 2 at Q, and also such that S1 and S 2 touch each other at
S, whose centre is at (−3, 2), then the radius of S is a point, say M. Let E1 be the set representing the locus
(2016 Main) of M as the pair (S1 , S 2) varies in F1. Let the set of all
(a) 5 2 (b) 5 3 (c) 5 (d) 10 straight line segments joining a pair of distinct points of
E1 and passing through the point R(1, 1) be F2. Let E 2 be
7. If one of the diameters of the circle the set of the mid-points of the line segments in the set
x2 + y2 − 2x − 6 y + 6 = 0 is a chord to the circle with F2. Then, which of the following statement(s) is (are)
centre (2, 1), then the radius of the circle is (2004, 1M) TRUE? (2018 Adv.)
(a) 3 (b) 2 (c) 3 (d) 2
(a) The point (− 2, 7) lies in E1
8. The number of common tangents to the circles
(b) The point  ,  does NOT lie in E2
4 7
x2 + y2 − 4x − 6 y − 12 = 0 and x2 + y2 + 6x + 18 y + 26 = 0  5 5
is (2015)
(c) The point  ,1 lies in E2
1
(a) 1 (b) 2 (c) 3 (d) 4
2 
9. Let C be the circle with centre at (1, 1) and radius 1. If T
(d) The point  0,
3
is the circle centred at (0, y) passing through origin and  does NOT lie in E1
 2
touching the circle C externally, then the radius of T is
equal to (2014 Main) 17. A circle S passes through the point (0, 1) and is
(a)
3
(b)
3
(c)
1
(d)
1 orthogonal to the circles (x − 1)2 + y2 = 16 and x2 + y2 = 1.
2 2 2 4 Then, (2014 Adv.)
10. If the circle x2 + y2 + 2x + 2ky + 6 = 0 and (a) radius of S is 8
x + y + 2ky + k = 0 intersect orthogonally, then k is
2 2 (b) radius of S is 7
(c) centre of S is (−7,1)
(a) 2 or − 3 / 2 (b) −2 or − 3 / 2 (2000, 2M)
(d) centre of S is (−8,1)
(c) 2 or 3 / 2 (d) −2 or 3 / 2
11. The ∆ PQR is inscribed in the circle x2 + y2 = 25. If Q and Passage Based Problems
R have coordinates (3, 4) and (− 4, 3) respectively, then
∠ QPR is equal to (2000, 2M) Passage
(a) π / 2 (b) π / 3 (c) π / 4 (d) π / 6 Let ABCD be a square of side length 2 unit. C 2 is the
circle through vertices A , B, C , D and C1 is the circle
12. The number of common tangents to the circles
touching all the sides of square ABCD. L is the line
x2 + y2 = 4 and x2 + y2 − 6x − 8 y = 24 is (1998, 2M)
through A. (2006, 5M)
(a) 0 (b) 1 (c) 3 (d) 4
18. If P is a point of C1 and Q is a point on C 2 , then
13. The angle between a pair of tangents drawn from a
point P to the circle PA 2 + PB2 + PC 2 + PD 2
is equal to
x2 + y2 + 4 x − 6 y + 9 sin 2 α + 13 cos2 α = 0 QA 2 + QB2 + QC 2 + QD 2
is 2α. The equation of the locus of the point P is (a) 0.75 (b) 1.25
(1996, 1M) (c) 1 (d) 0.5
(a) x2 + y2 + 4x − 6 y + 4 = 0 (b) x2 + y2 + 4x − 6 y − 9 = 0 19. A circle touches the line L and the circle C1 externally
(c) x2 + y2 + 4x − 6 y − 4 = 0 (d) x2 + y2 + 4x − 6 y + 9 = 0 such that both the circles are on the same side of the
14. If the two circles (x − 1)2 + ( y − 3)2 = r 2 and line, then the locus of centre of the circle is
x + y2 − 8x + 2 y + 8 = 0 intersect in two distinct points,
2 (a) ellipse (b) hyperbola
then (1989, 2M) (c) parabola (d) parts of straight line
(a) 2 < r < 8 (b) r < 2 (c) r = 2 (d) r > 2 20. A line M through A is drawn parallel to BD. Point S
15. If a circle passes through the point (a, b) and cuts the moves such that its distances from the line BD and the
circle x + y = k orthogonally, then the equation of the
2 2 2 vertex A are equal. If locus of S cuts M at T2 and T3 and
locus of its centre is (1988, 2M) AC at T1, then area of ∆T1T2T3 is
(a) 2ax + 2by − (a 2 + b2 + k 2 ) = 0 1 2
(a) sq unit (b) sq unit
(b) 2ax + 2by − (a 2 − b2 + k 2 ) = 0 2 3
(c) x2 + y2 − 3ax − 4by + a 2 + b2 − k 2 = 0 (c) 1sq unit (d) 2 sq units
(d) x2 + y2 − 2ax − 3by + (a 2 − b2 − k 2 ) = 0
rankershalt.com

386 Circle

Match the Columns Analytical & Descriptive Questions


21. Match the conditions/expressions in Column I with 22. Let C1 and C 2 be two circles with C 2 lying inside C1. A
statement in Column II. circle C lying inside C1 touches C1 internally and C 2
Column I Column II externally. Identify the locus of the centre of C.
(2001, 5M)
A. Two intersecting circles p. have a common tangent
23. Three circles touch one another externally. The
B. Two mutually external circles q. have a common normal tangents at their points of contact meet at a point whose
C. Two circles, one strictly inside r. do not have a common distance from a point of contact is 4. Find the ratio of the
the other tangent product of the radii to the sum of the radii of the circles.
D. Two branches of a hyperbola s. do not have a common (1992, 5M)
normal

Topic 3 Equation of Tangent, Normal and Length of Tangents


Objective Questions I (Only one correct option)
1. The common tangent to the circles x2 + y2 = 4 and 8. If the tangent at the point P on the circle
x2 + y2 + 6x + 8 y − 24 = 0 also passes through the point x2 + y2 + 6x + 6 y = 2 meets the straight line
(2019 Main, 9 April II ) 5x − 2 y + 6 = 0 at a point Q on the Y-axis, then the length
(a) (6, − 2) (b) (4, − 2) of PQ is (2002, 1M)

(c) (−6, 4) (d) (−4, 6) (a) 4 (b) 2 5 (c) 5 (d) 3 5

2. A rectangle is inscribed in a circle with a diameter lying 9. Let PQ and RS be tangents at the extremities of the
along the line 3 y = x + 7. If the two adjacent vertices of diameter PR of a circle of radius r. If PS and RQ
the rectangle are (–8, 5) and (6, 5), then the area of the intersect at a point X on the circumference of the circle,
rectangle (in sq units) is (2019 Main, 9 April II ) then 2r equals (2001, 1M)

(a) 72 (b) 84 (c) 98 (d) 56 PQ + RS


(a) PQ ⋅ RS (b)
2
3. The tangent and the normal lines at the point ( 3 , 1) to
the circle x2 + y2 = 4 and the X-axis form a triangle. The 2PQ ⋅ RS PQ 2 + RS 2
(c) (d)
area of this triangle (in square units) is PQ + RS 2
(2019 Main, 8 April II )
1 4 2 1
(a) (b) (c) (d) Objective Questions II
3 3 3 3
(One or more than one correct option)
4. The straight line x + 2 y = 1 meets the coordinate axes at
A and B. A circle is drawn through A , B and the origin. 10. Let RS be the diameter of the circle x2 + y2 = 1, where S
Then, the sum of perpendicular distances from A and B is the point (1, 0) . Let P be a variable point (other than R
on the tangent to the circle at the origin is and S) on the circle and tangents to the circle at S and P
(2019 Main, 11 Jan I ) meet at the point Q. The normal to the circle at P
5 5 intersects a line drawn through Q parallel to RS at point
(a) 2 5 (b) (c) 4 5 (d)
4 2 E. Then, the locus of E passes through the point(s)
(2016 Adv.)
5. If a circle C passing through the point
(a)  ,
1 1 
(b)  , 
1 1
(4, 0) touches the circle x + y + 4x − 6 y = 12 externally
2 2

 3 3  4 2
at the point (1, − 1), then the radius of C is
(c)  , −
1 
(d)  , − 
1 1 1
(2019 Main,10 Jan I ) 
(a) 5 (b) 2 5 (c) 57 (d) 4 3 3 4 2
6. If the tangent at (1, 7) to the curve x = y − 6 touches the
2
11. The circle C1 : x2 + y2 = 3 with centre at O intersects the
circle x + y + 16x + 12 y + c = 0, then the value of c is
2 2
parabola x2 = 2 y at the point P in the first quadrant. Let
(2018 Main) the tangent to the circle C1 at P touches other two circles
(a) 195 (b) 185 (c) 85 (d) 95 C 2 and C3 at R2 and R3 , respectively. Suppose C 2 and C3
7. Let ABCD be a quadrilateral with area 18, with side AB have equal radii 2 3 and centres Q2 and Q3 ,
parallel to the side CD and AB = 2 CD . Let AD be respectively. If Q2 and Q3 lie on the Y-axis, then
perpendicular to AB and CD. If a circle is drawn inside (a) Q2Q3 = 12 (2016 Adv.)
the quadrilateral ABCD touching all the sides, then its (b) R2R3 = 4 6
radius is (2007, 3M) (c) area of the ∆OR2R3 is 6 2
(a) 3 (b) 2 (c)
3
(d) 1 (d) area of the ∆PQ2Q3 is 4 2
2
rankershalt.com

Circle 387

Assertion and Reason


16. Points E and F are given by
12. Tangents are drawn from the point (17, 7) to the circle
 3 3  3 1
x + y = 169.
2 2
(a)  ,  , ( 3 , 0) (b)  ,  , ( 3 , 0)
 2 2  2 2
Statement I The tangents are mutually perpendicular.
 3 3  3 1  3 3   3 1
because (c)  , ,  ,  (d)  , ,  , 
 2 2  2 2  2 2   2 2
Statement II The locus of the points from which a
mutually perpendicular tangents can be drawn to the 17. Equations of the sides QR, RP are
given circle is x2 + y2 = 338. (2007, 3M)
(a) y =
2
x + 1, y = −
2
x − 1 (b) y =
1
x, y = 0
(a) Statement I is true, Statement II is true; Statement II 3 3 3
is correct explanation of Statement I 3 3
(c) y = x + 1, y = − x − 1 (d) y = 3x, y = 0
(b) Statement I is true, Statement II is true, Statement II 2 2
is not correct explanation of Statement I.
(c) Statement I is true, Statement II is false. Fill in the Blanks
(d) Statement I is false, Statement II is true.
18. A circle is inscribed in an equilateral triangle of side a.
The area of any square inscribed in this circle is… .
Passage Based Problems (1994, 2M)
Passage 1 19. If a circle passes through the points of intersection of
A tangent PT is drawn to the circle x2 + y2 = 4 at the the coordinate axes with the lines λx − y + 1 = 0 and
point P ( 3 , 1). A straight line L, perpendicular to PT is x − 2 y + 3 = 0, then the value of λ is ... . (1991,2M)
a tangent to the circle (x − 3)2 + y2 = 1. (2012)
Analytical & Descriptive Questions
13. A possible equation of L is
(a) x − 3 y = 1 (b) x + 3y = 1 20. Find the equation of circle touching the line
(c) x − 3 y = − 1 (d) x + 3y = 5 2x + 3 y + 1 = 0 at the point (1, –1) and is orthogonal to
the circle which has the line segment having end points
14. A common tangent of the two circles is (0, –1) and (–2, 3) as the diameter. (2004, 4M)
(a) x = 4 (b) y = 2
(c) x + 3 y = 4 (d) x + 2 2 y = 6
21. Find the coordinates of the point at which the circles
x2 − y2 − 4x − 2 y + 4 = 0 and x2 + y2 − 12x − 8 y + 36 = 0
Passage 2 touch each other. Also, find equations of common
A circle C of radius 1 is inscribed in an equilateral tangents touching the circles the distinct points.
∆PQR. The points of contact of C with the sides PQ, QR, (1993, 5M)
RP are D, E, F respectively. The line PQ is given by the 22. Two circles, each of radius 5 units, touch each other at
 3 3 3 (1, 2). If the equation of their common tangent is
equation 3 x + y − 6 = 0 and the point D is  , . 4x + 3 y = 10, find the equations of the circles. (1991, 4M)
 2 2
Further, it is given that the origin and the centre of C
are on the same side of the line PQ. (2008, 12M) Integer Answer Type Question
15. The equation of circle C is 23. The centres of two circles C1 and C 2 each of unit radius
(a) (x − 2 3 ) + ( y − 1) = 1
2 2 are at a distance of 6 units from each other. Let P be the
2 mid-point of the line segment joining the centres of C1
(b) (x − 2 3 )2 +  y +  = 1
1
 2  and C 2 and C be a circle touching circles C1 and C 2
externally. If a common tangents to C1 and C passing
(c) (x − 3 )2 + ( y + 1)2 = 1
through P is also a common tangent to C 2 and C , then the
(d) (x − 3 )2 + ( y − 1)2 = 1
radius of the circle C is … (2009)

Topic 4 Radical Axis and Family of Circle


Objective Questions I (Only one correct option) 2. The line x = y touches a circle at the point (1, 1). If the
1. The locus of the centres of the circles, which touch the circle also passes through the point (1, − 3), then its
circle, x2 + y2 = 1 externally, also touch the Y-axis and radius is (2019 Main, 10 April I)
lie in the first quadrant, is (2019 Main, 10 April II) (a) 3 2
(b) 2 2
(a) y = 1 + 2x, x ≥ 0 (b) y = 1 + 4x, x ≥ 0
(c) 2
(c) x = 1 + 2 y, y ≥ 0 (d) x = 1 + 4 y, y ≥ 0 (d) 3
rankershalt.com

388 Circle

3. Two circles with equal radii are intersecting at the Fill in the Blanks
points (0, 1) and (0, −1). The tangent at the point (0,1) to
one of the circles passes through the centre of the other 13. For each natural number k. Let C k denotes the circle
circle. Then, the distance between the centres of these with radius k centimetres and centre at origin. On the
circles is (2019 Main, 11 Jan I) circle C k a particle moves k centimetres in the
(a) 2 (b) 2 2 (c) 1 (d) 2 counter-clockwise direction. After completing its
motion on C k the particle moves to C k + 1 in the radial
4. Three circles of radii a , b, c(a < b < c) touch each other
direction. The motion of the particle continue in this
externally. If they have X-axis as a common tangent, manner. The particle starts at (1, 0). If the particle
then (2019 Main, 9 Jan I) crosses the positive direction of the X-axis for the first
1 1 1 time on the circle C n, then n = …
(a) a , b, c are in AP (b) = + (1997, 2M)
a b c
1 1 1 14. The intercept on the line y = x by the circle x2 + y2 − 2x = 0
(c) a , b , c are in AP (d) = +
b a c is AB. Equation of the circle with AB as a diameter is… .
(1996, 1M)
5. The circle passing through (1, − 2) and touching the axis
of x at (3, 0) also passes through the point (2013 Main) 15. If the circle C1 : x2 + y2 = 16 intersects another circle C 2
(a) (− 5, 2) (b) (2, − 5) of radius 5 in such a manner that the common chord is
(c) (5, − 2) (d) (− 2, 5) of maximum length and has a slope equal to 3/4, then
the coordinates of the centre of C 2 are… . (1988, 2M)
6. The circle passing through the point (− 1, 0) and
16. The points of intersection of the line 4x − 3 y − 10 = 0 and
touching the Y-axis at (0, 2) also passes through the
point (2011)
the circle x2 + y2 − 2x + 4 y − 20 = 0 are…and… .
(1983, 2M)
(a)  − , 0 (b)  − , 2 (c)  − , 
3 5 3 5
(d) (− 4, 0)
 2   2   2 2
Analytical & Descriptive Questions
7. The locus of the centre of circle which touches
( y − 1) + x = 1 externally and also touches X-axis, is
2 2 17. Consider the family of circles x2 + y2 = r 2 , 2 < r < 5. If in
(a) {x = 4 y, y ≥ 0} ∪ {(0, y), y< 0}
2
(2005, 2M) the first quadrant, the common tangent to a circle of
(b) x2 = y this family and the ellipse 4x2 + 25 y2 = 100 meets the
(c) y = 4x2 coordinate axes at A and B, then find the equation of the
(d) y2 = 4x ∪ (0, y), y∈R locus of the mid-points of AB. (1999, 5M)

8. If two distinct chords, drawn from the point ( p, q) on the 18. Consider a family of circles passing through two fixed
points A (3, 7) and B (6, 5). Show that the chords in
circle x + y = px + qy (where, pq ≠ 0) are bisected by
2 2
which the circle x2 + y2 − 4x − 6 y − 3 = 0 cuts the
the X-axis, then (1999, 2M) members of the family are concurrent at a point. Find
(a) p 2 = q2 (b) p 2 = 8q2 the coordinates of this point. (1993, 5M)
(c) p 2 < 8q2 (d) p 2 > 8q2
19. A circle touches the line y = x at a point P such that
9. The locus of the centre of a circle, which touches OP = 4 2, where O is the origin. The circle contains the
externally the circle x2 + y2 − 6x − 6 y + 14 = 0 and also point (− 10, 2) in its interior and the length of its chord
touches the Y-axis, is given by the equation (1993, 1M) on the line x + y = 0 is 6 2. Determine the equation of
(a) x2 − 6x − 10 y + 14 = 0 (b) x2 − 10x − 6 y + 14 = 0 the circle. (1990, 5M)
(c) y2 − 6x − 10 y + 14 = 0 (d) y2 − 10x − 6 y + 14 = 0
20. Let S ≡ x2 + y2 + 2 gx + 2 fy + c = 0 be a given circle. Find
10. The centre of a circle passing through the points (0, 0),
the locus of the foot of the perpendicular drawn from the
(1, 0) and touching the circle x2 + y2 = 9 is (1992, 2M)
origin upon any chord of S which subtends a right angle
(a) (3/2, 1/2) (b) (1/2, 3/2)
at the origin. (1988, 5M)
(c) (1/2, 1/2) (d) (1 / 2, − 21/ 2 )
21. Let a given line L1 intersect the X and Y-axes at P and Q
11. The equation of the circle passing through (1, 1) and
the points of intersection of x + y + 13x − 3 y = 0 and
2 2 respectively. Let another line L 2, perpendicular to L1,
2x2 + 2 y2 + 4x − 7 y − 25 = 0 is (1983, 1M)
cut the X and Y-axes at R and S, respectively. Show that
the locus of the point of intersection of the line PS and
(a) 4x2 + 4 y2 − 30x − 10 y = 25
QR is a circle passing through the origin. (1987, 3M)
(b) 4x2 + 4 y2 + 30x − 13 y − 25 = 0
(c) 4 x2 + 4 y2 − 17x − 10 y + 25 = 0 22. Find the equations of the circles passing through (−4, 3)
(d) None of the above and touching the lines x + y = 2 and x − y = 2. (1982, 3M)
12. Two circles x2 + y2 = 6 and x2 + y2 − 6x + 8 = 0 are given. 23. Find the equation of the circle which passes through the
Then the equation of the circle through their points of point (2, 0) and whose centre is the limit of the point of
intersection and the point (1, 1) is (1980, 1M) intersection of the lines 3x + 5 y = 1, (2 + c)x + 5c2y = 1 as c
(a) x2 + y2 − 6 x + 4 = 0 (b) x2 + y2 − 3x + 1 = 0 tends to 1. (1979, 3M)
(c) x2 + y2 − 4 y + 2 = 0 (d) None of these
rankershalt.com

Circle 389

Topic 5 Equation of Chord Bisected at a Point, Product of Pair of


Tangents, Chord of Contact of Tangents, Pole and
Equation of Polar
Objective Questions I (Only one correct option) Assertion and Reason
1. If the angle of intersection at a point where the two For the following questions, choose the correct answer
circles with radii 5 cm and 12 cm intersect is 90°, then from the codes (a), (b), (c) and (d) defined as follows.
the length (in cm) of their common chord is (a) Statement I is true, Statement II is also true;
(2019 Main, 12 April I) Statement II is the correct explanation of Statement I.
13 120 60 13
(a) (b) (c) (d) (b) Statement I is true, Statement II is also true;
5 13 13 2
Statement II is not the correct explanation of
2. The sum of the squares of the lengths of the chords Statement I
intercepted on the circle, x2 + y2 = 16, by the lines, (c) Statement I is true; Statement II is false
x + y = n, n ∈ N , where N is the set of all natural (d) Statement I is false; Statement II is true
numbers, is (2019, Main, 8 April I)
8. Consider L1 : 2x + 3 y + p − 3 = 0
(a) 320 (b) 105
(c) 160 (d) 210 L2 : 2x + 3 y + p + 3 = 0

3. The centres of those circles which touch the circle, where, p is a real number and
x2 + y2 − 8x − 8 y − 4 = 0, externally and also touch the C : x2 + y2 − 6x + 10 y + 30 = 0
X-axis, lie on (2016 Main) Statement I If line L1 is a chord of circle C, then line L 2
(a) a circle is not always a diameter of circle C.
(b) an ellipse which is not a circle Statement II If line L1 is a diameter of circle C, then
(c) a hyperbola line L 2 is not a chord of circle C. (2008, 3M)
(d) a parabola
4. The locus of the mid-point of the chord of contact of Fill in the Blanks
tangents drawn from points lying on the straight line
9. The chords of contact of the pair of tangents drawn from
4x − 5 y = 20 to the circle x2 + y2 = 9 is (2012) each point on the line 2x + y = 4 to the circle x2 + y2 = 1
(a) 20 (x2 + y2 ) − 36x + 45 y = 0 pass through the point… . (1997, 2M)
(b) 20 (x2 + y2 ) + 36x − 45 y = 0
10. The equation of the locus of the mid-points of the chords
(c) 36 (x2 + y2 ) − 20 y + 45 y = 0
of the circle 4x2 + 4 y2 − 12x + 4 y + 1 = 0 that subtend an
(d) 36 (x2 + y2 ) + 20x − 45 y = 0 angle of 2π / 3 at its centre is … . (1993, 2M)
5. Tangents drawn from the point P (1, 8) to the circle
11. The area of the triangle formed by the tangents from
x2 + y2 − 6x − 4 y − 11 = 0 touch the circle at the points A
the point (4, 3) to the circle x2 + y2 = 9 and the line
and B. The equation of the circumcircle of the ∆PAB is
(2009)
joining their points of contact is… . (1987, 2M)
(a) x2 + y2 + 4x − 6 y + 19 = 0 12. From the point A (0, 3) on the circle x2 + 4x + ( y − 3)2 = 0,
(b) x2 + y2 − 4x − 10 y + 19 = 0 a chord AB is drawn and extended to a point M such
(c) x2 + y2 − 2x + 6 y − 29 = 0 that AM = 2 AB. The equation of the locus of M is… .
(d) x2 + y2 − 6x − 4 y + 19 = 0 (1986, 2M)
6. The locus of the mid-point of a chord of the circle 13. The equation of the line passing through the points of
x2 + y2 = 4 which subtends a right angle at the origin, is intersection of the circles 3x2 + 3 y2 − 2x + 12 y − 9 = 0
(a) x + y = 2 (1984, 2M) and x2 + y2 + 6x + 2 y − 15 = 0 is… . (1986, 2M)
(b) x2 + y2 = 1
14. Let x2 + y2 − 4x − 2 y − 11 = 0 be a circle. A pair of
(c) x2 + y2 = 2
(d) x + y = 1 tangents from the point (4, 5) with a pair of radii form a
quadrilateral of area... . (1985, 2M)

Objective Question II 15. From the origin chords are drawn to the circle
(One or more than one correct option) (x − 1)2 + y2 = 1. The equation of the locus of the mid
points of these chords is… . (1985, 2M)
7. The equations of the tangents drawn from the origin to
the circle x2 + y2 + 2rx + 2hy + h 2 = 0, are (1988, 2M) Analytical & Descriptive Questions
(a) x = 0 16. Let 2x2 + y2 − 3xy = 0 be the equation of a pair of
(b) y = 0
tangents drawn from the origin O to a circle of radius 3
(c) (h 2 − r 2 ) x − 2rhy = 0
with centre in the first quadrant. If A is one of the points
(d) (h 2 − r 2 ) x + 2rhy = 0
of contact, find the length of OA. (2001, 5M)
rankershalt.com

390 Circle

17. Let T1 , T2 and be two tangents drawn from (−2, 0) onto 21. Lines 5x + 12 y − 10 = 0 and 5x − 12 y − 40 = 0 touch a
the circle C : x2 + y2 = 1. Determine the circles touching circle C1 of diameter 6. If the centre of C1 lies in the first
C and having T1 , T2 as their pair of tangents. Further, quadrant, find the equation of the circle C 2 which is
find the equations of all possible common tangents to concentric with C1 and cuts intercepts of length 8 on
these circles when taken two at a time. (1999, 10M) these lines. (1986, 5M)

18. C1 and C 2 are two concentric circles, the radius of C 2 22. Through a fixed point (h , k) secants are drawn to the
circle x2 + y2 = r 2. Show that the locus of the mid-points
being twice that of C1. From a point P on C 2, tangents PA
of the secants intercepted by the circle is
and PB are drawn to C1. Prove that the centroid of the
x2 + y2 = hx + ky. (1983, 5M)
∆PAB lies on C1. (1998, 8M)
23. Let A be the centre of the circle x2 + y2 − 2x − 4 y − 20 = 0.
19. Find the intervals of values of a for which the line
Suppose that, the tangents at the points B (1, 7) and D
y + x = 0 bisects two chords drawn from a point
(4, −2 ) on the circle meet at the point C. Find the area of
 1 + 2a 1 − 2a 
 ,  to the circle the quadrilateral ABCD. (1981, 4M)
 2 2 
Integer Answer Type Question
2x2 + 2 y2 − (1 + 2a ) x − (1 − 2a ) y = 0. (1996, 6M)
24. Two parallel chords of a circle of radius 2 are at a
20. Let a circle be given by distance 3 + 1 apart. If the chords subtend at the centre,
2x (x − a ) + y (2 y − b) = 0, (a ≠ 0, b ≠ 0) angles of π /k and

, where k > 0, then the value of [k]
Find the condition on a and b if two chords, each bisected k
by the X-axis can be drawn to the circle from (a , b /2). is…… (2010)
(1992, 6M) NOTE [k ] denotes the largest integer less than or equal to k]

Answers
Topic 1 Topic 4
1. (a) 2. (c) 3. (b) 4. (c) 1. (a) 2. (b) 3. (d) 4. (b)
5. (d) 6. (c) 7. (a) 8. (b) 5. (c) 6. (d) 7. (a) 8. (d)
9. (c) 10. (a) 11. (c) 12. (a, c) 9. (d) 10. (d) 11. (b) 12. (b)
3
13. (b, c) 14. 15. k ≠ 1 16. True 13. n = 7
4
21. x 2 + y 2 + 2ax + 2 py − (b 2 + q 2 ) = 0,  9 12  9 12
14. x 2 + y 2 − x − y = 0 15.  − ,  and  ,− 
 5 5 5 5 
radius = a 2 + p 2 + b 2 + q 2
16. ( −2, − 6 ) and ( 4, 2 ) 17. 4 x 2 + 25y 2 = 4 x 2y 2
22. 2 23. 2 24. (a) 25. (d)
18. x = 2 and y = 23 / 3 19. x 2 + y 2 + 18 x − 2y + 32 = 0
Topic 2 c
20. x 2 + y 2 + gx + f y + =0
1. (a) 2. (c) 3. (c) 4. (b) 2
5. (c) 6. (b) 7. (c) 8. (c) 22. x 2 + y 2 + 2 (10 ± 3 6 ) x + (55 ± 24 6 ) = 0
9. (d) 10. (a) 11. (c) 12. (b)
13. (d) 14. (a) 15. (a) 16. (a,d) 23. 25 ( x 2 + y 2 ) − 20 x + 2y − 60 = 0
17. (b, c) 18. (a) 19. (c) 20. (c)
Topic 5
21. A→p, q; B→p, q; C→q, r; D→q, r
1. (b) 2. (d) 3. (d) 4. (a)
22. Ellipse having foci are (a, b) and (0, 0)
5. (b) 6. (c) 7. (a,c) 8. (c)
23. 16 : 1
 1 1
9.  ,  10. 16 x 2 + 16y 2 − 48 x + 16y + 31 = 0
Topic 3  2 4
1. (a) 2. (b) 3. (c) 4. (d) 192
11. sq units
5. (a) 6. (d) 7. (b) 8. (c) 25
9. (a) 10. (a ,c) 11. (a, b, c) 12. (a) 12. x 2 + y 2 + 8 x − 6y + 9 = 0 13. 10 x − 3y − 18 = 0
13. (a) 14. (d) 15. (d) 16. (a)
14. 8 sq units 15. x + y − x = 0
2 2
16. 3 (3 + 10 )
a2 1
17. (a) 18. sq unit 19. λ = 2 or − 2 2
6 2  4  1 5  4
17.  x +  + y 2 =   ; y = ± x + 
20. 2 x 2 + 2y 2 − 10 x − 5y + 1 = 0 21. y = 0 and 7y − 24 x + 16 = 0  3  3 39  5
19. a ∈ ( −∞, − 2 ) ∪ (2, ∞ ) 20. a 2 > 2b 2
22. ( x − 5 ) 2 + (y − 5 ) 2 = 5 2 and ( x + 3 ) 2 + (y + 1 ) 2 = 5 2
23. 8 21. ( x − 5 ) 2 + (y − 2 ) 2 = 5 2 22. x 2 + y 2 = hx + ky
23. 75 sq units 24. 3
rankershalt.com

Hints & Solutions


Topic 1 Equation of Circle 3. Let the foot of perpendicular be P (h , k). Then, the slope
1. It is given that the circle touches the X-axis at (3, 0) and k
of line OP =
making an intercept of 8 on the Y -axis. h
Y Y

B B
P(h, k)
M
8 C
r r A
A X
X O
O (3,0)
Q Line AB is perpendicular to line OP, so slope of line
Let the radius of the circle is ‘r’, then the coordinates of h
AB = − [Qproduct of slopes of two
centre of circle are (3, r ). k
From the figure, we have perpendicular lines is (−1)]
CM = 3, Now, the equation of line AB is
CA = radius = r h
y − k = − (x − h ) ⇒ hx + ky = h 2 + k2
AB k
and AM = BM = =4 x y
2 or + =1
 h + k   h + k2
2 2 2
Then, r 2 = CM 2 + AM 2 = 9 + 16 = 25    
⇒ r=±5  h   k 
Now, the equation of circle having centre (3, ± 5) and
 h 2 + k2   h 2 + k2
radius = 5 is So, point A  , 0 and B0, 
(x − 3)2 + ( y ± 5)2 = 25  h   k 
Now, from the options (3, 10) satisfy the equation of Q ∆AOB is a right angled triangle, so AB is one of the
circle diameter of the circle having radius R (given).
(x − 3)2 + ( y − 5)2 = 25 ⇒ AB = 2R
2 2
2. Given vertices of ∆AOP are O(0, 0) and A(0, 1)  h 2 + k2  h 2 + k2
⇒   +  = 2R
Let the coordinates of point P are (x, y).  h   k 
Clearly, perimeter = OA + AP + OP = 4 (given)  1 1
⇒ (h 2 + k2)2  2 + 2 = 4R2
h k 
⇒ (0 − 0)2 + (0 − 1)2 + (0 − x)2 + (1 − y)2 + x2 + y2 = 4
⇒ (h + k2)3 = 4R2h 2k2
2

⇒1 + x2 + ( y − 1)2 + x2 + y2 = 4
On replacing h by x and k by y, we get
⇒ x + y − 2y + 1 +
2 2
x + y =3
2 2
(x2 + y2)3 = 4R2x2y2,
⇒ x2 + y2 − 2 y + 1 = 3 − x2 + y2 which is the required locus.
4. Given equation of circle is x2 + y2 − 6x + 8 y − 103 = 0,
⇒ x2 + y2 − 2 y + 1 = 9 + x2 + y2 − 6 x2 + y2
which can be written as (x − 3)2 + ( y + 4)2 = 128 = (8 2 )2
[squaring both sides]
∴ Centre = (3, − 4) and radius = 8 2
⇒ 1 − 2 y = 9 − 6 x2 + y2 Now, according to given information, we have the
following figure.
⇒ 6 x2 + y2 = 2 y + 8
Y
⇒ 3 x2 + y2 = y + 4
⇒ 9(x2 + y2) = ( y + 4)2 [squaring both sides] D C
O X
⇒ 9x2 + 9 y2 = y2 + 8 y + 16
45°
⇒ 9x2 + 8 y2 − 8 y = 16 G
Thus, the locus of point P (x, y) is (3,–4)

9x + 8 y − 8 y = 16
2 2
A B
rankershalt.com

392 Circle

For the coordinates of A and C. ⇒ 36 = 25 + 36 − c


x−3 y+ 4 ⇒ c = 25
Consider, = =±8 2
1 1
6. Key idea Orthocentre, centroid and circumcentre are
2 2 collinear and centroid divide orthocentre and
[using distance (parametric) form of line, circumcentre in 2 : 1 ratio.
x − x1 y − y1
= = r] We have orthocentre and centroid of a triangle be
cos θ sin θ A(−3, 5) and B(3, 3) respectively and C circumcentre.
⇒ x = 3 ± 8, y = − 4 ± 8
∴ A(− 5, − 12) and C (11, 4)
Similarly, for the coordinates of B and D, consider Clearly, AB = (3 + 3)2 + (3 − 5)2 = 36 + 4 = 2 10
x−3 y+ 4
= = ±8 2 [in this case, θ = 135°] We know that, AB : BC = 2 : 1
1 1
− ⇒ BC = 10
2 2
⇒ x = 3 m 8, y = − 4 ± 8 Now, AC = AB + BC = 2 10 + 10 = 3 10
∴ B (11, − 12) and D (− 5, 4) Since, AC is a diameter of circle.
AC
Now, OA = 25 + 144 = 169 = 13; ∴ r=
2
OB = 121 + 144 = 265 3 10 5
⇒ r= =3
OC = 121 + 16 = 137 2 2
and OD = 25 + 16 = 41 7. Given, circle is inscribed in square formed by the lines
5. Clearly, centre of the circumscribed circle is the x2 − 8x + 12 = 0 and y2 − 14 y + 45 = 0
centroid (G) of the equilateral triangle ABC. ⇒ x = 6 and x = 2, y = 5 and y = 9
which could be plotted as
[Q in an equilateral triangle circumcentre and centroid
coincide] Y D (2,9) C (6,9)
y=9
C

60°
y=5
(–5,–6) A (2,5) B (6,5)
G X X
r r O Y
120°
where, ABCD clearly forms a square.
A B
∴ Centre of inscribed circle
= Point of intersection of diagonals
Also, we know that = Mid-point of AC or BD
∆AGB ≅ ∆BGC ≅ ∆CGA [by SAS congruence rule]  2 + 6  5 + 9
=  ,  = (4, 7)
 2   2 
∴ ar (∆ABC ) = 3 ar (∆AGB)
1  ⇒ Centre of inscribed circle is (4, 7).
= 3  r 2 sin 120°
2  8. Choosing OA as X-axis, A = (r , 0), B = (0, r ) and any
1
[Q area of triangle = ab sin (∠C )] point P on the circle is (r cos θ , r sin θ ). If (x, y) is the
2 centroid of ∆ PAB, then
Q ar (∆ABC ) = 27 3 [given] Y B (0,r )
3 2 3
∴ r = 27 3 P
2 2
3
[sin 120° = sin (180° − 60° ) = sin 60° = ]
2 X′ O X
A (r,0)
⇒ r2 = 4 × 9
⇒ r =6
Now, radius of circle, Y′

r= g2 + f 2 − c 3x = r cos θ + r + 0
and 3 y = r sin θ + 0 + r
⇒ 6 = 25 + 36 − c
∴ (3x − r )2 + (3 y − r )2 = r 2
[Q in the given equation of circle 2 g = 10 Hence, locus of P is a circle.
and 2 f = 12 ⇒ g = 5 and f = 6]
rankershalt.com

Circle 393

9. Since, 2x − 3 y = 5 and 3x − 4 y = 7 are diameters of a Here, the length of intercept on Y-axis is ⇒ 2 f 2 − c


circle. and if circle touches X-axis
Their point of intersection is centre (1, − 1) . ⇒ g2 = c
Also given, πr 2 = 154 for x + y + 2 gx + 2 fy + c = 0
2 2

7 x2 + y2 + 2 gx + 2 fy + c = 0
⇒ r 2 = 154 × ⇒ r=7 Here,
22
∴ Required equation of circle is
A
(x − 1)2 + ( y + 1)2 = 72
2√7
⇒ x2 + y2 − 2x + 2 y = 47
10. Clearly, ∠C = 90° as angle in semi-circle is right angled. B
Now, area of triangle is maximum when AC = BC. A′ (3, 0)
i.e. Triangle is right angled isosceles. 2√7
C
B′

A B
passes through (3, 0).
⇒ 9 + 6g + c = 0 …(i)
g2 = c …(ii)
and 2 f2 − c =2 7
11. Let centre of circle be (h , k) .
f2 − c = 7 …(iii)
so that OA 2 = OB2
From Eqs. (i) and (ii), we get
B (0,1) g 2 + 6 g + 9 = 0 ⇒ ( g + 3 )2 = 0
⇒ g = − 3 and c = 9
∴ f 2 = 16 ⇒ f =±4
A (2,4)
O (h,k ) ∴ x + y − 6x ± 8 y + 9 = 0
2 2

13. Let equation of line L1 be y = mx. Intercepts made by L1


y = x2
T and L 2 on the circle will be equal i.e. L1 and L 2 are at the
same distance from the centre of the circle;
⇒ h 2 + (k − 1)2 = (h − 2)2 + (k − 4)2
Centre of the given circle is (1/2, – 3/2). Therefore,
⇒ 4h + 6k − 19 = 0 …(i)
m 3
k −4 + m+3
Also, slope of OA = and slope of tangent at (2, 4) to 1 /2 – 3 /2 – 1 2
h −2 = 2 2 ⇒ =
1+1 2
m –1 2 2 m2 + 1
y = x2 is 4.
and (slope of OA) ⋅ (slope of tangent at A) = − 1 ⇒ 8m2 + 8 = m2 + 6m + 9
k −4
∴ ⋅4 = − 1 ⇒ 7m – 6 m – 1 = 0 ⇒ (7m + 1)(m – 1) = 0
2
h −2
1
⇒ 4k − 16 = − h + 2 ⇒ m = – , m =1
7
h + 4k = 18 …(ii) Thus, two chords are x + 7 y = 0
On solving Eqs. (i) and (ii), we get and x – y = 0.
53 16
k= and h = − Therefore, (b) and (c) are correct answers.
10 5
7
 16 53 14. Since, 3x − 4 y + 4 = 0 and 3x − 4 y − = 0 are two parallel
∴ Centre coordinates are  − , . 2
 5 10
tangents. Thus, distance between them is diameter of
12. PLAN circle
7
4+
2 15 3
(–g, –f ) ⇒ Diameter = = =
3 +4
2 2 2 .5 2
3
and radius =
4
rankershalt.com

394 Circle

15. Since, P lies on circle and A and B are points in plane Here, ax12 + 2hx1 y1 + by12 − 1 ≠ 0, as (x1 , y1 ) does not lie on
PA Eq. (i),
such that, = k, then the locus of P is perpendiular
PB Also, a cos 2 θ + 2h sin θ cos θ + b sin 2 θ
bisector of AB. Thus, the value of k ≠ 1. = a + 2 h sin θ cos θ + (b − a )sin 2 θ
16. Since, centre of circle is (3, − 1) which lies on x + 3 y = 0 = a + sin θ {2h cos θ + (b − a ) sin θ }
⇒ x + 3 y = 0 is diameter of x + y − 6x + 2 y = 0
2 2
= a + sin θ ⋅ 4h 2 + (b − a )2 ⋅ (cos θ sin φ + sin θ cos φ )
Hence, given statement is true.
b−a
17. Equations of any circle C with centre at (0, 2 ) is given where, tan θ =
2h
by
= a + 4h 2 + (b − a )2 sin θ sin (θ + φ )
(x − 0)2 + ( y − 2 )2 = r 2
which will be independent of θ, if
or x2 + y2 − 2 2 y + 2 = r 2 …(i)
4h 2 + (b − a )2 = 0
where, r > 0.
⇒ h = 0 and b = a
Let (x1 , y1 ), (x2, y2), (x3 , y3 ) be three distinct rational
1
points on circle. Since, a straight line parallel to X-axis ∴ Eq. (i) reduces to x2 + y2 =
meets a circle in at most two points, either a
y1 , y2 or y1 , y3 . which is a equation of circle.
On putting these in Eq. (i), we get
19. Let the radius of the circle be r. Take X-axis along AC
x12 + y12 − 2 2 y1 = r 2 − 2 …(ii) and the O (0, 0) as centre of the circle. Therefore,
x22 + y22 − 2 2 y2 = r − 2
2
…(iii) coordinate of A and C are (− r , 0) and (r , 0), respectively.
Now, ∠ BAC = β , ∠ BOC = 2 β
x32 + y32 − 2 2 y3 = r 2 − 2 …(iv)
Therefore, coordinates of B are (r cos 2 β , r sin 2 β ).
On subtracting Eq. (ii) from Eq. (iii), we get And slope of AD is tan (β − α ).
p1 − 2 q1 = 0 Let (x, y) be the coordinates of the point D. Equation of
where, p1 = x22 + y22 − x12 − y12, AD is
Y
q1 = y2 − y1 B (r cos 2 β, r sin 2 β)
P
On subtracting Eq. (ii) from Eq. (iv), we get
D E
p2 − 2q2 = 0 X′ X
A(−r, 0) O C (r,0)
where p2 = x32 + y32 − x12 − y12, q2 = y3 − y1
Now, p1 , p2 , q1 , q2 are rational numbers. Also, either
q1 ≠ 0 or q2 ≠ 0. If q1 ≠ 0, then 2 = p1 / q1 and if q2 ≠ 0,
then 2 = p2 / q2. In any case 2 is a rational number. Y′
This is a contradiction. y = tan (β − α ) (x + r ) …(i)
18. The given circle is ax2 + 2hxy + by2 = 1 …(i) [Q slope = tan (β − α ) and point is (− r, 0)]
Let the point P not lying on Eq. (i) be (x1 , y1 ), let θ be Now, equation of BC is
the inclination of line through P which intersects the r sin 2 β − 0
y= (x − r )
given curve at Q and R. r cos 2 β − r
Then, equation of line through P is r ⋅ 2 sin β cos β
⇒ y= (x − r )
x − x1 y − y1 r (− 2 sin 2 β )
= =r
cos θ sin θ 2 sin β cos β
⇒ x = x1 + r cos θ , y = y1 + r sin θ ⇒ y= (x − r )
−2 sin 2 β
For points Q and R, above point must lie on Eq. (i).
⇒ y = − cot β (x − r ) …(ii)
⇒ a (x1 + r cos θ )2 + 2h (x1 + r cos θ ) ( y1 + r sin θ )
To obtain the coordinate of D, solve Eqs. (i) and (ii)
+ b ( y1 + r sin θ )2 = 1
simultaneously
⇒ (a cos 2 θ + 2h sin θ cos θ + b sin 2 θ )r 2 ⇒ tan (β − α ) (x + r ) = − cot β (x − r )
+ 2 (ax1 cos θ + hx1 sin θ + hy1 cos θ + by1 sin θ )r
⇒ x tan (β − α ) + r tan (β − α ) = − x cot β + r cot β
+ (ax12 + 2hx1 y1 + by12 − 1) = 0 ⇒ x [tan (β − α ) + cot β ] = r [cot β − tan (β − α )]
It is quadratic in r, giving two values of r as PQ and PR.
sin (β − α ) cos β   cos β sin(β − α ) 
a x12 + 2hx1 y1 + by12 − 1 ⇒ x + =r −
∴ PQ ⋅ PR =  cos (β − α ) sin β  
sin β cos (β − α ) 
a cos θ + 2h sin θ cos θ + b sin 2 θ
2
rankershalt.com

Circle 395

sin (β − α ) sin β + cos (β − α ) cos β   1


⇒ x  20. Let the points mi ,  ; i = 1, 2, 3, 4
 cos (β − α ) sin β   mi 
 cos β cos (β − α ) − sin β sin (β − α )  lie on a circle x2 + y2 + 2 gx + 2 fy + c = 0.
=r  1 2f
 sin β cos (β − α )  Then, mi2 + 2 + 2 gmi + + c=0;
mi mi
⇒ x [cos (β − α − β )] = r [cos (β − α + β )]
Since, mi4 + 2 gmi3 + cmi2 + 2 fmi + 1 = 0 ; i = 1, 2, 3, 4
r cos (2 β − α )
⇒ x= ⇒ m1 , m2, m3 and m4 are the roots of the equation
cos α
m4 + 2 gm3 + cm2 + 2 fm + 1 = 0
On putting this value in Eq. (ii), we get
1
 r cos (2 β − α )  ⇒ m1m2 m3m4 = = 1
y = − cot β  − r 1
 cos α 
21. Let (x1 , y1 ) and (x2, y2) be the coordinates of points A and
cos β ⋅ r  cos (2β − α ) − cos α 
⇒ y=− B, respectively.
sin β  cos α 
 It is given that x1 , x2 are the roots of x2 + 2ax − b2 = 0
 2β − α + α α − 2β + α  ⇒ x1 + x2 = − 2a and x1x2 = − b2 …(i)
2 sin   sin  
r cos β   2   2  Also, y1 and y2 are the roots of y2 + 2 py − q2 = 0
⇒ y=−   ⇒ y1 + y2 = − 2 p and y1 y2 = − q2 …(ii)
sin β  cos α 
  ∴ The equation of circle with AB as diameter is,
(x − x1 ) (x − x2) + ( y − y1 ) ( y − y2) = 0
r cos β 2 sin β ⋅ sin (α − β ) 
⇒ y=−
sin β  cos α 

⇒ x2 + y2 − (x1 + x2) x − ( y1 + y2) y + (x1x2 + y1 y2) = 0
⇒ x2 + y2 + 2ax + 2 py − (b2 + q2) = 0
= − 2r cos β sin (α − β ) / cos α
Therefore, coordinates of D are and radius = a 2 + p2 + b2 + q2
 r cos (2 β − α ) 2r cos β sin (α − β )
 ,−  22. The circle and coordinate axes can have 3 common
 cos α cos α  points, if it passes through origin. [ p = 0]
Thus, coordinates of E are If circle is cutting one axis and touching other axis.
 r cos (2 β − α ) + r cos α cos β sin (α − β ) Only possibility is of touching X-axis and cutting
 ,−r 
 2 cos α cos α  Y -axis. [ p = − 1]
 2β − α + α   2β − α − α 
2 cos   ⋅ cos   23. x2 + y2 ≤ 6 and 2x − 3 y = 1 is shown as
 2   2 
⇒ r ,
2 cos α L
cos β sin (β − α )
r
cos α
cos β ⋅ cos (β − α ) cos β sin (β − α ) 1/3 1/2
⇒ r ,r
cos α cos α
Since, AE = d, we get For the point to lie in the shade part, origin and the
2 2 point lie on opposite side of straight line L.
 cos β cos (β − α )   cos β sin (β − α ) 
d2 = r2  + 1 + r 2   ∴ For any point in shaded part L > 0 and for any point
 cos α   cos α 
inside the circle S < 0.
r2  3
= [cos 2 β cos 2 (β − α ) + cos 2 α Now, for 2,  L : 2x − 3 y − 1
cos 2 α  4
+ 2 cos β cos (β − α ) cos α + cos 2 β sin 2 (β − α )] 9 3
L :4 − − 1 = > 0
r2 4 4
= [cos 2 β {cos 2 (β − α ) + sin 2 (β − α ) } + cos 2 α
cos 2 α 9
+ 2cos β cosα cos ( β − α )] and S : x + y − 6, S : 4 +
2 2
−6 <0
16
r2
= [cos 2 β + cos 2 α + 2 cos α cos β cos (β − α )]  3
cos 2 α ⇒ 2,  lies in shaded part.
 4
d 2 cos 2 α
⇒ r2 =  5 3
cos β + cos α + 2 cos α cos β cos (β − α )
2 2 For  ,  , L : 5 − 9 − 1 < 0 [neglect]
 2 4
Therefore, area of the circle
 1 1 1 3 
π d 2 cos 2 α For  ,−  , L: + − 1 > 0
πr 2 =  4 4 2 4 
cos β + cos α + 2 cos α cos β cos (β − α )
2 2
rankershalt.com

396 Circle

 1 1 ∴ h=
1
and k =
1
∴  ,−  lies in the shaded part.
 4 4 cos θ sin θ
1 1
 1 1 1 3 ⇒ cos θ = and sin θ =
For  ,  , L : − − 1 < 0 [neglect] h k
 8 4 4 4 1 1 h 2 + k2
⇒ cos θ + sin θ = 2 + 2 ⇒ 1 = 2 2
2 2
⇒ Only 2 points lie in the shaded part. h k h ⋅k
24. ⇒ h 2 + k2 = h 2 k2
Y ∴ Mid-point of MN lie on the curve x2 + y2 = x2 y2
E3(0,4)
Topic 2 Relation between Two Circles
G1 (0,2) 1. Equation of given circles
F2(1, 3)
x2 + y2 + 5Kx + 2 y + K = 0 …(i)
E1(– 3,1) E2( 3,1) and 2(x2 + y2) + 2Kx + 3 y − 1 = 0
Po(1,1) 3 1
⇒ x2 + y2 + Kx + y − = 0 …(ii)
x′ x 2 2
(–2,0) O G2(2, 0)
On subtracting Eq. (ii) from Eq. (i), we get
1 1
4Kx + y + K + = 0
2 2
F1(1, – 3)
(0, –2)
⇒ 8Kx + y + (2K + 1) = 0 …(iii)

Y′
[Q if S1 = 0 and S 2 = 0 be two circles, then their common
chord is given by S1 − S 2 = 0.]
Eq. (iii) represents equation of common chord as it is
Equation of tangent at E1 (− 3 , 1) is
given that circles (i) and (ii) intersects each other at
− 3x + y = 4 and at E 2( 3 , 1) is points P and Q.
3x + y = 4 Since, line 4x + 5 y − K = 0 passes through point P and Q.
Intersection point of tangent at E1 and E 2 is (0, 4). 8K 1 2K + 1
∴ = =
∴ Coordinates of E3 is (0, 4) 4 5 −K
Similarly, equation of tangent at F1 (1, − 3 ) and ⇒ K =
1
[equating first and second terms]
F2(1, 3 ) are x − 3 y = 4 and x + 3 y = 4, respectively 10
and intersection point is (4, 0), i.e., F3 (4, 0) and equation and − K = 10K + 5
of tangent at G1 (0, 2) and G2(2, 0) are 2 y = 4 and 2x = 4, [equating second and third terms]
respectively and intersection point is (2, 2) i.e., G3 (2, 2). 5
Point E3 (0, 4), F3 (4, 0) and G3 (2, 2) satisfies the line ⇒ 11K + 5 = 0 ⇒ K = −
11
x + y = 4.
1 5
Q ≠ − , so there is no such value of K, for which line
25. We have, 10 11
x2 + y2 = 4 4x + 5 y − K = 0 passes through points P and Q.
Let P(2 cos θ , 2 sin θ ) be a point on a circle.
∴ Tangent at P is 2. Equation of given circle is x2 + y2 = 1, then equation of
2 cos θ x + 2 sin θ y = 4 tangent at the point (cos θ ,sin θ ) on the given circle is
⇒ x cos θ + y sin θ = 2 x cos θ + y sin θ = 1 …(i)
Y [QEquation of tangent at the point P(cos θ , sin θ ) to
N the circle x2 + y2 = r 2 is x cos θ + y sin θ = r]
P(2 cos θ, 2 sin θ)
Now, the point of intersection with coordinate axes are
P(sec θ , 0) and Q(0, cos ec θ ).
Q Mid-point of line joining points P and Q is
X′ X
O M  sec θ cos ecθ 
M ,  = (h , k) (let)
 2 2 
x2 + y2 = 4
1 1
So, cos θ = and sin θ =
2h 2k
Y′ Q sin 2 θ + cos 2 θ = 1
1 1 1 1
∴ + =1⇒ 2 + 2 =4
 2   2  4h 2 4k2
∴ The coordinates at M  , 0 and N 0,  h k
 cos θ   sin θ 
Let (h , k) is mid-point of MN
rankershalt.com

Circle 397

Now, locus of mid-point M is ⇒ C 2(4, 7) is the centre of 2nd circle and r2 = 6 is its
1 1 radius.
+ 2 =4
x 2
y Two circles intersect if|r1 − r2| < C1C 2 < r1 + r2
⇒ x2 + y2 − 4x2y2 = 0 ⇒| r − 6|< (8 − 4)2 + (10 − 7)2 < r + 6
3. The given circles, ⇒| r − 6| < 16 + 9 < r + 6
x2 + y2 − 2x − 2 y + 1 = 0 ... (i) ⇒| r − 6| < 5 < r + 6
and x + y − 18x − 2 y + 78 = 0,
2 2
... (ii) Now as, 5 < r + 6 always, we have to solve only
are on the opposite sides of the variable line | r − 6| < 5 ⇒ − 5 < r − 6 < 5
3x + 4 y − λ = 0. So, their centres also lie on the opposite
⇒ 6 − 5 < r < 5 + 6 ⇒ 1 < r < 11
sides of the variable line.
⇒ [3(1) + 4(1) − λ ] [3(9) + 4(1) − λ ] < 0 6. Given equation of a circle is x2 + y2 − 4x + 6 y − 12 = 0,
[Q The points P (x1 , y1 ) and Q (x2, y2) lie on the opposite whose centre is (2, − 3) and radius
sides of the line ax + by + c = 0, = 2 2 + (− 3) 2 + 12 = 4 + 9 + 12 = 5
if (ax1 + by1 + c)(ax2 + by2 + c) < 0]
⇒ (λ − 7)(λ − 31) < 0 S
⇒ λ ∈ (7, 31) ... (iii) Now, according to given information,
A (–3, 2) C
3(1) + 4(1) − λ we have the following figure.
Also, we have ≥ 1 + 1 −1
5 x2 + y2 − 4x + 6 y − 12 = 0 O
(2,–3)
QDistance of centre from the given line is  Clearly, AO ⊥ BC, as O is mid-point B
 ax1 + by1 + c  of the chord.
 greater than the radius,i.e. ≥ r
 a +b
2 2  Now, in ∆AOB, we have
⇒ |7 − λ|≥ 5 ⇒ λ ∈ (− ∞, 2] ∪ [12 , ∞ ) ... (iv) OA = (− 3 − 2) 2 + (2 + 3) 2
3(9) + 4(1) − λ = 25 + 25 = 50 = 5 2
and ≥ 81 + 1 − 78 and OB = 5
5
⇒ |λ − 31 | ≥ 10 ∴ AB = OA 2 + OB2 = 50 + 25 = 75 = 5 3
⇒ λ ∈ (− ∞ , 21] ∪ [41, ∞ ) ... (v) 7. Here, radius of smaller circle, AC = 12 + 32 − 6 = 2
From Eqs. (iii), (iv) and (v), we get
Clearly, from the figure the radius of bigger circle
λ ∈ [ 12, 21 ]
r 2 = 22 + [(2 − 1)2 + (1 − 3)2]
4. Given circles, r2 = 9 ⇒ r =3
x2 + y2 − 2x − 2 y − 2 = 0 … (i)
and x2 + y2 − 6x − 6 y + 14 = 0 … (ii)
A
are intersecting each other orthogonally, because
r
2(1)(3) + 2(1)(3) = 14 − 2 2
[Q two circles are intersected C C1 (2,1)
(1,3)
orthogonally if 2 g1 g2 + 2 f1 f2 = c1 + c2] 2

P B

2 2
C1 (3,3)
C2 8. PLAN Number of common tangents depend on the position of the
(1,1)
circle with respect to each other.
Q (i) If circles touch externally ⇒ C1C 2 = r1 + r2, 3 common
tangents.
(ii) If circles touch internally ⇒ C1C 2 = r2 − r1,1 common tangent.
So, area of quadrilateral
(iii) If circles do not touch each other, 4 common tangents.
PC1QC 2 = 2 × ar (∆PC1C 2).
Given equations of circles are
1  x2 + y2 − 4x − 6 y − 12 = 0
= 2 ×  × 2 × 2 = 4 sq units ...(i)
2 
x + y2 + 6x + 18 y + 26 = 0
2
...(ii)
5. Circle I is x2 + y2 − 16x − 20 y + 164 = r 2 Centre of circle (i) is C1 (2, 3) and radius
⇒ (x − 8)2 + ( y − 10)2 = r 2 = 4 + 9 + 12 = 5(r1 ) [say]
⇒C1 (8, 10) is the centre of Istcircle and r1 = r is its radius Centre of circle (ii) is C 2(–3, – 9) and radius
Circle II is (x − 4)2 + ( y − 7)2 = 36 = 9 + 81 – 26 = 8(r2) [say]
rankershalt.com

398 Circle

Now, C1C 2 = (2 + 3)2 + (3 + 9)2 Now, slope of OQ , m1 = 4 / 3 ,


slope of OR, m2 = − 3 / 4
⇒ C1C 2 = 52 + 122 Here, m1 m2 = − 1
⇒ C1C 2 = 25 + 144 = 13 Therefore, ∠ QOR = π /2
∴ r1 + r2 = 5 + 8 = 13 which implies that ∠ QPR = π /4

Also, C1C 2 = r1 + r2 12. Given, x2 + y2 = 4


Thus, both circles touch each other externally. Hence, Centre ≡ C1 ≡ (0, 0) and R1 = 2
there are three common tangents. Again, x2 + y2 − 6x − 8 y − 24 = 0, then C 2 ≡ (3, 4)
9. PLAN Use the property, when two circles touch each other and R2 = 7
externally, then distance between the centre is equal to sum Again, C1C 2 = 5 = R2 − R1
of their radii, to get required radius.
Therefore, the given circles touch internally such that,
Let the coordinate of the centre of T be (0, k). they can have just one common tangent at the point of
Distance between their centre contact.
k + 1 = 1 + (k − 1)2 [Q C1C 2 = k + 1] 13. Centre of the circle
x2 + y2 + 4x − 6 y + 9 sin 2 α + 13 cos2 α = 0
⇒ k + 1 = 1 + k2 + 1 − 2k
is C (−2, 3) and its radius is
Y (−2)2 + (3)2 − 9 sin 2 α − 13 cos2 α
C
= 13 − 13 cos2 α − 9 sin 2 α
C1 (1,1)
T (1– k)
= 13 sin 2 α − 9 sin 2 α = 4 sin 2 α = 2 sin α
(0,k) 1
X′ X A
O (1,0)
C2

Y′ , 3)
−2
P (h, k) C(
⇒ k + 1 = k2 + 2 − 2k
⇒ k2 + 1 + 2k = k2 + 2 − 2k B
1
⇒ k= Let (h , k) be any point P and
4
∠ APC = α , ∠ PAC = π /2
1
So, the radius of circle T is k, i. e. . That is, triangle APC is a right angled triangle.
4
AC 2 sin α
∴ sin α = =
10. Since, the given circles intersect orthogonally. PC (h + 2)2 + (k − 3)2
∴ 2 (1) (0) + 2 (k) (k) = 6 + k
⇒ (h + 2)2 + (k − 3)2 = 4
[Q 2 g1 g2 + 2 f1 f2 = c1 + c2]
⇒ h + 4 + 4h + k2 + 9 − 6k = 4
2
3
⇒ 2k − k − 6 = 0 ⇒ k = − , 2
2
⇒ h 2 + k2 + 4h − 6k + 9 = 0
2
Thus, required equation of the locus is
11. Let O is the point at centre and P is the point at
circumference. Therefore, angle QOR is double the x2 + y2 + 4x − 6 y + 9 = 0
angle QPR. So, it is sufficient to find the angle QOR. 14. As, the two circles intersect in two distinct points.
Y
⇒ Distance between centres lies between | r1 − r2| and
| r1 + r2|.
Q (3,4) i.e. | r − 3|< (4 − 1)2 + (−1 − 3)2 < | r + 3|
4,3)

⇒ | r − 3|< 5 < | r + 3| ⇒ r < 8 or r > 2


R (−

∴ 2 < r <8
X′ X
O (0,0)
15. Let x + y + 2 gx + 2 fy + c = 0, cuts x2 + y2 = k2
2 2

orthogonally.
⇒ 2 g1 g2 + 2 f1 f2 = c1 + c2
P
⇒ −2 g ⋅ 0 − 2 f ⋅ 0 = c − k 2
Y′ ⇒ c = k2 …(i)
rankershalt.com

Circle 399

Also, x2 + y2 + 2 gx + 2 fy + k2 = 0 passes through (a , b). 18. Let the, equation of circles are
∴ a 2 + b2 + 2 ga + 2 fb + k2 = 0 …(ii) C1 : (x − 1)2 + ( y − 1)2 = (1)2
⇒ Required equation of locus of centre is and C 2 : (x − 1)2 + ( y − 1)2 = ( 2 )2
−2ax − 2by + a + b + k = 0
2 2 2

or 2ax + 2by − (a 2 + b2 + k2) = 0 D(0, 2) C(2, 2)

16. It is given that T is tangents to S1 at P and S 2 at Q and S1 C1 P


x2 + x2 – 2y = 0
and S 2 touch externally at M. (1, 1)

A(0, 0) B(2, 0)
s1 M s2
∴ Coordinates of P(1 + cos θ , 1 + sin θ )
T and Q(1 + 2 cos θ , 1 + 2 sin θ )
P(–2,7) N Q(2,–5)
∴ PA 2 + PB2 + PC 2 + PD 2
∴ MN = NP = NQ = {(1 + cos θ )2 + (1 + sin θ )2} + {(cos θ − 1)2 + (1 + sin θ )2}

∴ Locus of M is a circle having PQ as its diameter of + {(cos θ − 1)2 + (sin θ − 1)2}


circle. + {(1 + cos θ )2 + (sin θ − 1)2} = 12
∴ Equation of circle Similarly, QA 2 + QB2 + QC 2 + QD 2 = 16
(x − 2) (x + 2) + ( y + 5) ( y − 7) = 0
Σ PA 2 12
⇒ x2 + y2 − 2 y − 39 = 0 ∴ = = 0.75
ΣQA 2 16
Hence, E1 : x2 + y2 − 2 y − 39 = 0, x ≠ ± 2
Locus of mid-point of chord (h , k) of the circle E1 is 19. Let C be the centre of the required circle.
xh + yk − ( y + k) − 39 = h 2 + k2 − 2k − 39
⇒ xh + yk − y − k = h 2 + k2− 2k C1
Since, chord is passing through (1, 1).
∴ Locus of mid-point of chord (h , k) is
C
h + k − 1 − k = h 2 + k2 − 2k
L
⇒ h 2 + k2 − 2k − h + 1 = 0
Locus is E 2 : x2 + y2 − x − 2 y + 1 = 0
A
Now, after checking options, (a) and (d) are correct.
17. PLAN Now, draw a line parallel to L at a distance of r1
(i) The general equation of a circle is (radius of C1) from it.
x 2 + y 2 + 2 gx + 2 f y + c = 0
Now, CC1 = AC ⇒ C lies on a parabola.
where, centre and radius are given by ( − g , − f ) and 20. Since, AG = 2
g 2 + f 2 − c , respectively. 1
∴ AT1 = T1G =
2
(ii) If the two circles x 2 + y 2 + 2 g 1x + 2 f1 y + c1 = 0 and
x 2 + y 2 + 2 g 2x + 2 f2 y + c 2 = 0 are orthogonal, then As, A is the focus, T1 is the vertex and BD is the directrix
2 g 1g 2 + 2 f1f2 = c1 + c 2. of parabola.

Let circle be x2 + y2 + 2 gx + 2 fy + c = 0
It passes through (0, 1). D C
∴ 1 + 2f + c = 0 …(i)
Orthogonal with x2 + y2 − 2x − 15 = 0 G
2 g (− 1) = c − 15 T1
T2
⇒ c = 15 − 2 g …(ii)
B
Orthogonal with x2 + y2 − 1 = 0 A
c=1 …(iii)
T3
⇒ g = 7 and f = − 1
Also, T2T3 is latusrectum.
Centre is (− g , − f ) ≡ (− 7, 1)
1
∴ Radius = g2 + f 2 − c ∴ T2T3 = 4 ⋅
2
= 49 + 1 − 1 = 7 1 1 4
∴ Area of ∆T1T2T3 = × × = 1 sq unit
2 2 2
rankershalt.com

400 Circle

21. (A) When two circles are intersecting they have a A


common normal and common tangent. R1 R2
C1 C2
(B) Two mutually external circles have a common
normal and common tangent. R1 R2
(C) When one circle lies inside of other, then they have a C B
common normal but no common tangent. R3 R3
(D) Two branches of a hyperbola have a common normal
C3
but no common tangent.
22. Let the given circles C1 and C 2 have centres O1 and O2
and radii r1 and r2, respectively.
Now, perimeter of a triangle
Let the variable circle C touching C1 internally, C 2
2s = R1 + R2 + R2 + R3 + R3 + R1
externally have a radius r and centre at O.
⇒ 2s = 2 (R1 + R2 + R3 )
⇒ s = R1 + R2 + R3
and ∆ = s (s − a ) (s − b) (s − c)
C2 C1
O2 = (R1 + R2 + R3 ) (R3 ) (R2) (R1 )
r2
R1 R2 R3 (R1 + R2 + R3 )
r From Eq. (i), 4=
R1 + R2 + R3
O1 O
R1 R2 R3 (R1 + R2 + R3 )
C ⇒ 16 =
(R1 + R2 + R3 )2
r1
R1 R2 R3
⇒ 16 =
R1 + R2 + R3

Now, OO2 = r + r2 and OO1 = r1 − r Topic 3 Equation of Tangent, Normal


⇒ OO1 + OO2 = r1 + r2 and Length of Tangents
which is greater than O1O2 as O1O2 < r1 + r2 1. Given circles are
[Q C 2 lies inside C1 ] x2 + y2 = 4, centre c1 (0, 0) and radius r1 = 2
⇒ Locus of O is an ellipse with foci O1 and O2. and x2 + y2 + 6x + 8 y − 24 = 0, centre c2(−3, − 4) and
Alternate Solution radius r2 = 7
Q c1c2 = 9 + 16 = 5 and|r1 − r2| = 5
Let equations of C1 be x2 + y2 = r12 and of C 2 be
Q c1c2 = |r1 − r2| = 5
(x − a )2 + ( y − b)2 = r22
∴circle x2 + y2 = 4 touches the circle
Let cetnre C be (h , k) and radius r, then by the given x2 + y2 + 6x + 8 y − 24 = 0 internally.
condition So, equation of common tangent is
(h − a )2 + (k − b)2 = r + r2 and h 2 + k2 = r1 − r S1 − S 2 = 0
⇒ 6x + 8 y − 20 = 0
⇒ (h − a )2 + (k − b)2 + h 2 + k2 = r1 + r2
⇒ 3x + 4 y = 10 …(i)
Required locus is The common tangent passes through the point (6, − 2),
(x − a )2 + ( y − b2) + x2 + y2 = r1 + r2 from the given options.
which represents an ellipse whose foci are (a , b) 2. Given points are (−8, 5) and (6, 5) in which y-coordinate
and (0, 0). is same, i.e. these points lie on horizontal line y = 5.
23. Suppose the circles have centres at C1 , C 2 and C3 with 3y=x+7
radius R1 , R2 and R3 , respectively. Let the circles touch
at A , B and C. Let the common tangents at A, B and C
(–8, β) (6, β)
meet at O. We have, OA = OB = OC = 4 [given]. Now, the
circle with centre at O and passing through A, B and C
is the incircle of the triangle C1 C 2 C3 (because
OA ⊥ C1 C 2).
Therefore, the inradius of ∆ C1 C 2 C3 is 4.
(–8, 5) (6, 5)

and r= …(i)
s
rankershalt.com

Circle 401

Let (−8, β) and (6, β ) are the coordinates of the other ⇒ x2 + y2 − x −


y
=0
vertices of rectangle as shown in the figure. 2
Since, the mid-point of line joining points (−8, 5) and y
Equation of tangent at (0, 0) is x + =0
(6, β ) lies on the line 3 y = x + 7. 2
 5 + β  −8 + 6 [Q equation of tangent at (x1 , y1 ) is given by T = 0.
∴ 3  = +7
 2  2
Here, T = 0
⇒ 15 + 3 β = − 2 + 14 1 1
⇒ 3β = −3 ⇒β = −1 ⇒ xx1 + yy1 − (x + x1 ) − ( y + y1 ) = 0]
2 4
Now, area of rectangle = |−8 − 6| × |β − 5|
⇒ 2x + y = 0
= 14 × 6 = 84
|2 ⋅ 1 + 1 ⋅ 0| 2
3. Let T = 0 and N = 0 represents the tangent and normal Now, AM = =
5 5
lines at the point P( 3 , 1) to the circle x2 + y2 = 4.
[Q distance of a point P (x1 , y1 ) from a line
Y
|ax1 + by1 + c |
ax + by + c = 0 is ]
N=0
a 2 + b2
P
 1
T=0 2 ⋅0 + 1  
1  2
and BN = =
X 5 2 5
O M A
2 1 4+1 5
∴ AM + BN = + = =
5 2 5 2 5 2
x2+y2=4 5. Equation of tangent to the circle
x2 + y2 + 4x − 6 y − 12 = 0 at (1, − 1) is given by
xx1 + yy1 + 2 (x + x1 ) − 3 ( y + y1 ) − 12 = 0, where x1 = 1
So, equation of tangent (T = 0) is and y1 = − 1
3x + y = 4 …(i) ⇒ x − y + 2 (x + 1) − 3( y − 1) − 12 = 0
For point A, put y = 0, we get ⇒ 3x − 4 y − 7 = 0
4 This will also a tangent to the required circle.
x=
3 Now, equation of family of circles touching the line
1 3x − 4 y − 7 = 0 at point (1, − 1) is given by
Q Area of required ∆OPA = (OA )(PM )
2 (x − 1)2 + ( y + 1)2 + λ (3x − 4 y − 7) = 0
1 4 So, the equation of required circle will be
= × ×1
2 3 (x − 1)2 + ( y + 1)2 + λ (3x − 4 y − 7) = 0, for some λ ∈ R
[QPM = y-coordinate of P] …(i)
2 Q The required circle passes through (4, 0)
= sq unit
3 ∴(4 − 1)2 + (0 + 1)2 + λ (3 × 4 − 4 × 0 − 7) = 0
⇒ 9 + 1 + λ (5) = 0 ⇒ λ = − 2
4. According to given information, we have the following
figure. Substituting λ = − 2 in Eq. (i), we get
Y
(x − 1)2 + ( y + 1)2 − 2 (3x − 4 y − 7) = 0
(0,1/2)
⇒ x2 + y2 − 8x + 10 y + 16 = 0
B On comparing it with
x2 + y2 + 2 gx + 2 fy + c = 0, we get
g = − 4, f = 5, c = 16
N
∴ Radius = g 2 + f 2 − c = 16 + 25 − 16 = 5
A
x
O 6. Key Idea Equation of tangent to the curve
y + y1
x 2 = 4 ay at ( x 1, y 1) is xx 1 = 4 a 
(1, 0) x+2y=1

 2 
M

From figure, equation of circle (diameter form) is Tangent to the curve x2 = y − 6 at (1, 7) is
y+ 7
 1 x= −6
(x − 1) (x − 0) + ( y − 0) y −  = 0 2
 2
⇒ 2x − y + 5 = 0 …(i)
rankershalt.com

402 Circle

Equation of circle is x2 + y2 + 16x + 12 y + c = 0 9. From figure, it is clear that ∆ PRQ and ∆ RSP are
Centre (−8, − 6) similar.
r = 82 + 62 − c = 100 − c
Since, line 2x − y + 5 = 0 also touches the circle.
2(−8) − (−6) + 5
∴ 100 − c =   r r
 22 + 12  P
θ
R
90 − θ

°
⇒ −16 + 6 + 5
100 − c =  
 5 
⇒ 100 − c = |− 5| X

θ

⇒ 100 − c = 5

90 °
θ
⇒ c = 95
Q S
1
7. 18 = (3α ) (2r )
2 PR PQ
∴ =
⇒αr = 6 RS RP
2r
Line, y = − (x − 2α ) is tangent to circle ⇒ PR2 = PQ ⋅ RS
α
(x − r )2 + ( y − r )2 = r 2 ⇒ PR = PQ ⋅ RS
2α = 3r , αr = 6 and r = 2 ⇒ 2r = PQ ⋅ RS
Y 10. Given, RS is the diameter of x2 + y2 = 1.
Here, equation of the tangent at P(cos θ , sin θ ) is
C (a, 2r)
D x cos θ + y sin θ = 1.
(0, 2r)

(r, r) P (cos θ, sin θ)

X′ X
(0, 0) A B (2a, 0) Q
E
Y′ (h, k)
R O S(1,0)
Alternate Solution (–1,0) x cos θ + y sin θ = 1
1
(x + 2x) × 2r = 18
2 x
r x−r
xr = 6 ... (i) B A
x−r
In ∆ AOB, tan θ = Intersecting with x = 1 ,
r 1 − cos θ
θ
and in ∆ DOC ,
r y=
2r sin θ
2x − r O
tan (90°− θ ) = 90
°−  1 − cos θ 
r r θ ∴ Q 1, 
x−r r  sin θ 
∴ =
r 2x − r C 2x–r D ∴ Equation of the line through Q parallel to RS is
2x
⇒ x (2x − 3r ) = 0 θ
2 sin 2
3r 1 − cos θ 2 θ
⇒ x= ... (ii) y= = = tan …(i)
2 sin θ θ θ 2
2 sin cos
From Eqs. (i) and (ii), we get 2 2
sin θ
r =2 Normal at P : y = ⋅x
cos θ
8. The line 5x − 2 y + 6 = 0 meets the Y-axis at the point
⇒ y = x tan θ …(ii)
(0, 3) and therefore the tangent has to pass through the
Let their point of intersection be (h , k).
point (0, 3) and required length θ
Then, k = tan and k = h tan θ
= x12 + y12 + 6x1 + 6 y1 − 2 2
 θ 
= 02 + 32 + 6 (0) + 6 (3) − 2  2 tan 
∴ k=h 2  ⇒ k = 2h ⋅ k
θ
 1 − tan 2  1 − k2
= 25 = 5
 2
rankershalt.com

Circle 403

⇒ k(1 − k2) = 2hk |0 + k − 3|


∴ =2 3
∴ Locus for point E : 2x = (1 − y2) …(iii) 2+1
1 ⇒ |k − 3| = 6
When x = , then
3 ⇒ k = 9, − 3
2 2 1
1 − y2 = ⇒ y2 = 1 − ⇒y=± ∴ Q2(0, 9)and Q3 (0, − 3)
3 3 3
Hence, Q2Q3 = 12
1 1
∴ Option (a) is correct.
∴ ,±  satisfy 2x = 1 − y .
2
3 3
Also, R2R3 is common internal tangent to C 2 and C3 ,
1
When x = , then and r2 = r3 = 2 3
4
2 1 1 ∴ R2R3 = d 2 − (r1 + r2)2 = 122 − (4 3 )2
1 − y2 = ⇒ y2 = 1 − ⇒ y=±
4 2 2 = 144 − 48 = 96 = 4 6
1 1
∴  , ±  does not satisfy 1 − y = 2x.
2
4 2
Q2 (0,9)
11. Given, C1 : x2 + y2 = 3 intersects the parabola x2 = 2 y.
R2
3 C2
P

− 3 O 3 d
R3
− 3

On solving x2 + y2 = 3 and x2 = 2 y, we get Q3 (0, −3)


y2 + 2 y = 3
⇒ y2 + 2 y − 3 = 0 C3
⇒ ( y + 3)( y − 1) = 0 ∴ Option (b) is correct.
∴ y = 1, − 3 [neglecting y = − 3, as − 3 ≤ y ≤ 3] Q Length of perpendicular from O(0, 0) to R2R3 is equal
∴ y=1 ⇒x= ± 2 to radius of C1 = 3.
⇒ P( 2 , 1) ∈ I quadrant 1 1
∴ Area of ∆OR2R3 = × R2R3 × 3 = × 4 6 × 3 = 6 2
Equation of tangent at P( 2 , 1)to C1 : x2 + y2 = 3 is 2 2
2x + 1 ⋅ y = 3 …(i) ∴ Option (c) is correct.
Now, let the centres of C 2 and C3 be Q2 and Q3 , and 1 2
Also, area of ∆PQ2Q3 = Q2Q3 × 2 = × 12 = 6 2
tangent at P touches C 2 and C3 at R2 and R3 shown as 2 2
below ∴ Option (d) is incorrect.
Y 12. As locus of point of intersection for perpendicular
tangents is directors circle.
i.e. x2 + y2 = 2r 2
Q2 Here, (17, 7) lie on directors circle x2 + y2 = 338
R2 C2 ⇒ Tangents are perpendicular.
13. Here, tangent to x2 + y2 = 4 at ( 3 , 1) is 3x + y = 4 …(i)
As, L is perpendicular to 3x + y = 4
⇒ x − 3 y = λ which is tangent to
O P( 2,1) (x − 3)2 + y2 = 1
X
C1 |3 − 0 − λ |
⇒ =1
Q3 1+3
C3 ⇒ |3 − λ | = 2
2x + y = 3 ⇒ 3 − λ =2,−2
∴ λ = 1, 5
Let Q2 be (0, k) and radius is 2 3. ⇒ L : x − 3 y = 1, x − 3 y = 5
rankershalt.com

404 Circle

E D
14. Here, equation of common tangent be
Y
30°
(√3,1)
X
O 2 3
F

∴ E and F are given by


x− 3 y−1
y = mx ± 2 1 + m2 = =1
cos 150° sin 150°
which is also the tangent to x− 3 y −1
and = =1
(x − 3)2 + y2 = 1 cos (−90° ) sin (−90° )
|3m − 0 + 2 1 + m2|  3 3
⇒ =1 ∴ E= ,  and F = ( 3 , 0)
m +1
2
 2 2
2 2
⇒ 3m + 2 1 + m2 = ± 1 + m2 17. Equation of QR, RP are y = x + 1 and y = – x − 1.
3 3
⇒ 3m = − 3 1 + m2
18. In an equilateral triangle, the radius of incircle
3m = − 1 + m 2 1
or = × median of the triangle
3
⇒ m2 = 1 + m2 or 9m2 = 1 + m2
1 a2 1 4a 2 − a 2 a
1 = a2 − = =
⇒ m ∈ φ or m = ± 3 4 3 4 2 3
2 2
Therefore, area of the square inscribed in this circle
1 1
∴ y=± x±2 1+ 2a 2 a 2
2 2 8 = 2 (radius of circle)2 = = sq unit
4 ⋅3 6
x 6
⇒ y=± ±
2 2 2 2 19. Since, the point of intersection of the coordinate axes
with the line λx − y + 1 = 0 and x − 2 y + 3 = 0 forms the
⇒ 2 2 y = ± (x + 6) circle.
∴ x + 2 2y = 6 ∴ (λx − y + 1) (x − 2 y + 3) = 0
15. Let centre of circle C be (h , k). represents a circle, if coefficient of x2 = coefficient of y2
and coefficient of xy = 0
3h + k − 6
Then, =1 ⇒ λ = 2 or −2 λ − 1 = 0
3+1
1
⇒ λ = 2 or λ = −
⇒ 3h + k − 6 = 2, − 2 2
⇒ 3h + k = 4 …(i) 20. The equation of circle having tangent 2x + 3 y + 1 = 0 at
[rejecting 2 because origin and centre of (1, − 1)
C are on the same side of PQ] ⇒ (x − 1)2 + ( y + 1)2 + λ (2x + 3 y + 1) = 0
The point ( 3 , 1) satisfies Eq. (i). x + y + 2x(λ − 1) + y(3λ + 2) + (λ + 2) = 0
2 2
…(i)
∴ Equation of circle C is (x − 3 )2 + ( y − 1)2 = 1. which is orthogonal to the circle having end point of
Clearly, points E and F satisfy the equations given in diameter (0, − 1) and (−2, 3).
option (d). ⇒ x(x + 2) + ( y + 1)( y − 3) = 0
16. Slope of line joining centre of circle to point D is or x2 + y2 + 2x − 2 y − 3 = 0 …(ii)
3 2(2λ − 2) 2(3λ + 2)
−1 ∴ ⋅1 + (−1) = λ + 2 − 3
1 2 2
tan θ = 2 =
3 3 3 ⇒ 2λ − 2 − 3λ − 2 = λ − 1
− 3
2 ⇒ 2λ = − 3
It makes an angle 30° with X-axis. ⇒ λ = − 3 /2
From Eq. (i) equation of circle,
∴ Points E and F will make angle 150° and −90° with
X-axis. 2x2 + 2 y2 − 10x − 5 y + 1 = 0
rankershalt.com

Circle 405

21. Two circles touch each other externally, if C1 C 2 = r1 + r2 22. We have,


and internally if C1 C 2 = r1 ~ r2. 4
Slope of the common tangent = −
Given circles are x2 + y2 − 4x − 2 y + 4 = 0, 3
3
whose centre C1 (2, 1) and radius r1 = 1 ∴ Slope of C1C 2 =
4
and x2 + y2 − 12x − 8 y + 36 = 0 4
whose centre C 2 (6, 4) and radius r2 = 4 If C1C 2 makes an angle θ with X-axis, then cos θ = and
5
The distance between the centres is 3
sin θ = .
(6 − 2)2 + (4 − 1)2 = 16 + 9 = 5 5
Y
⇒ C1 C 2 = r1 + r2
Therefore, the circles touch each other externally and at
the point of touching the point divides the line joining
the two centres internally in the ratio of their radii, 1 : 4. 5
1 × 6 + 4 × 2 14
Therefore, x1 = = 5 C2
1+4 5
C1
1 ×4 + 4 ×1 8 X′ θ
y1 = = X
1+4 5
4x + 3y = 10
Again, to determine the equations of common tangents Y′
touching the circles in distinct points, we know that, the
tangents pass through a point which divides the line So, the equation of C1 C 2 in parametric form is
joining the two centres externally in the ratio of their x−1 y−2
radii, i.e. 1 : 4. = …(i)
4 /5 3 /5
1 × 6 − 4 × 2 −2 2
Therefore, x2 = = = Since, C1 and C 2 are points on Eq. (i) at a distance of
1 −4 −3 3
5 units from P.
1 ×4 −4 ×1
and y2 = =0 So, the coordinates of C1 and C 2 are given by
1 −4 x−1 y−2
= = ± 5 ⇒ x=1 ± 4
Now, let m be the slope of the tangent and this line 4 /5 3 /5
passing through (2 / 3, 0) is and y = 2 ± 3.
y − 0 = m (x − 2 / 3) Thus, the coordinates of C1 and C 2 are (5, 5) and (−3, − 1),
2 respectively.
⇒ y − mx + m = 0
3 Hence, the equations of the two circles are
This is tangent to the Ist circle, if perpendicular (x − 5)2 + ( y − 5)2 = 52
distance from centre = radius. and (x + 3)2 + ( y + 1)2 = 52
1 − 2m + (2 / 3)m 23. (r + 1)2 = α 2 + 9
∴ =1 [Q C1 ≡ (2, 1) and r1 = 1]
1 + m2

⇒ 1 − 2m + (2 / 3) m = 1 + m2 3 C
P 2
4 1
⇒ 1−m = 1 + m2 C1
3
16 2 8 α r
⇒ 1+ m − m = 1 + m2
9 3 C
7 2 8
⇒ m − m =0
9 3
7 8
r2 + 8 = α 2
⇒ m  m −  =0
9 3
⇒ r 2 + 2r + 1 = r 2 + 8 + 9
24
⇒ m = 0, m = ⇒ 2r = 16 ⇒ r = 8
7
Hence, the equations of the two tangents are
Topic 4 Radical Axis and Family of Circle
24  2
y = 0 and y = x −  1. Let (h , k) be the centre of the circle and radius r = h, as
7  3
circle touch theY -axis and other circle x2 + y2 = 1 whose
⇒ y = 0 and 7 y − 24x + 16 = 0 centre (0, 0) and radius is 1.
rankershalt.com

406 Circle

Y Now, 2(α ) (− α ) + 2 ⋅ 0 ⋅ 0 = (− 1) + (− 1) ⇒ α = ± 1
[Q condition of orthogonality is 2 g1 g2 + 2 f1 f2 = c1 + c2]
(h,k) ∴ C1 (1, 0) and C 2(− 1, 0) ⇒ C1C 2 = 2
C r=h>O
r 4. According to given information, we have the following
k>O
for first figure.
quadrant
X
O
x2+y2=1
C
B
F
∴ OC = r + 1 c
b E
[Q if circles touch each other externally, D a
A
then C1C 2 = r1 + r2]
where A , B, C are the centres of the circles
⇒ h 2 + k2 = h + 1, h > 0
Clearly, AB = a + b (sum of radii) and BD = b − a
and k > 0, for first quadrant.
∴ AD = (a + b)2 − (b − a )2
⇒ h 2 + k2 = h 2 + 2h + 1
(using Pythagoras theorem in ∆ABD)
⇒ k2 = 2h + 1
= 2 ab
⇒ k = 1 + 2h , as k > 0
Similarly, AC = a + c and CE = c − a
Now, on taking locus of centre (h , k), we get
∴In ∆ACE, AE = (a + c)2 − (c − a )2 = 2 ac
y = 1 + 2x, x ≥ 0
Similarly, BC = b + c and CF = c − b
2. Since, the equation of a family of circles touching line
L =0 at their point of contact(x1 , y1 ) is ∴In ∆BCF, BF = (b + c)2 − (c − b)2 = 2 bc
(x − x1 )2 + ( y − y1 )2 + λ L = 0, where λ ∈ R. Q AD + AE = BF
∴Equation of circle, touches the x = y at point (1, 1) is ∴ 2 ab + 2 ac = 2 bc
(x − 1)2 + ( y − 1)2 + λ (x − y) = 0 1 1 1
⇒ + =
⇒ x2 + y2 + (λ − 2)x + (− λ − 2) y + 2 = 0 …(i) c b a
Q Circle (i) passes through point (1, − 3).
∴ 1 + 9 + (λ − 2) + 3(λ + 2) + 2 = 0 5. Let the equation of circle be
⇒ 4λ + 16 = 0 (x − 3)2 + ( y − 0)2 + λy = 0
⇒ λ = −4
So, equation of circle (i) at λ = − 4 , is Y
x2 + y2 − 6x + 2 y + 2 = 0
A (3, 0)
Now, radius of the circle = 9 + 1 − 2 = 2 2. X X
P
3. Clearly, circles are orthogonal because tangent at one (1, –2)
point of intersection is passing through centre of the Y
other. As it passes through (1, − 2)
Y ∴ (1 − 3)2 + (− 2)2 + λ (− 2) = 0
S2 S1 ⇒ 4 + 4 − 2λ = 0 ⇒ λ = 4
(0, 1)
r r ∴ Equation of circle is
X (x − 3)2 + y2 + 4 y = 0
(α, 0)
(–α, 0) By hit and trial method, we see that point (5, − 2)
(0,–1) satisfies equation of circle.
6. Equation of circle passing through a point (x1 , y1 ) and
Let C1 (α, 0) and C 2(− α , 0) are the centres. touching the straight line L, is given by
Then, S1 ≡ (x − α )2 + y2 = α 2 + 1 (x − x1 )2 + ( y − y1 )2 + λ L = 0
⇒ S1 ≡ x2 + y2 − 2 αx − 1 = 0 ∴ Equation of circle passing through (0, 2) and touching
x=0
[Q radius, r = (α − 0) + (0 − 1) ] 2 2

⇒ (x − 0)2 + ( y − 2)2 + λ x = 0 …(i)


and S 2 ≡ (x + α ) + y2 = α 2 + 1
2
Also, it passes through (− 1, 0) ⇒ 1 + 4 − λ = 0
⇒ S 2 ≡ x2 + y2 + 2αx − 1 = 0
∴ λ =5
rankershalt.com

Circle 407

Eq. (i) becomes, Then, coordinates of B are (− p + 2h , − q) and B lies on


x2 + y2 − 4 y + 4 + 5x = 0 the circle x2 + y2 = px + qy, we have
⇒ x2 + y2 + 5x − 4 y + 4 = 0, (− p + 2h )2 + (− q)2 = p (− p + 2h ) + q (− q)
For x-intercept put y = 0 ⇒ x2 + 5x + 4 = 0, ⇒ p2 + 4h 2 − 4 ph + q2 = − p2 + 2 ph − q2
(x + 1) (x + 4) = 0 ⇒ 2 p2 + 2q2 − 6 ph + 4h 2 = 0
∴ x = − 1, − 4 ⇒ 2h 2 − 3 ph + p2 + q2 = 0 …(i)
Hence, (d) option (–4, 0). There are given two distinct chords which are bisected
at X-axis then, there will be two distinct values of h
7. Let the locus of centre of circle be (h , k) touching satisfying Eq. (i).
( y − 1)2 + x2 = 1 and X-axis shown as
X So, discriminant of this quadratic equation must be > 0.
⇒ D >0
⇒ (− 3 p) − 4 ⋅ 2 ( p + q ) > 0
2 2 2

⇒ 9 p2 − 8 p2 − 8 q 2 > 0
⇒ p2 − 8 q 2 > 0 ⇒ p2 > 8 q 2
(0, 1) |k| (h, k )
O 1 A 9. Let (h , k) be the centre of the circle which touches the
|k| circle x2 + y2 − 6x − 6 y + 14 = 0 and Y-axis.
X′ X
O B The centre of given circle is (3, 3) and radius is
32 + 32 − 14 = 9 + 9 − 14 = 2
Y′
Clearly, from figure, Since, the circle touches Y-axis, the distance from its
Distance between C and A is always 1 + | k|, centre to Y-axis must be equal to its radius, therefore its
radius is h. Again, the circles meet externally, therefore
i.e. (h − 0)2 + (k − 1)2 = 1 + | k|, the distance between two centres = sum of the radii of
⇒ h 2 + k2 − 2k + 1 = 1 + k2 + 2| k| the two circles.
⇒ h 2 = 2| k| + 2k ⇒ x2 = 2| y| + 2 y Hence, (h − 3)2 + (k − 3)2 = (2 + h )2
h + 9 − 6h + k2 + 9 − 6k = 4 + h 2 + 4h
2
 y , y≥0
where | y| =  i.e. k2 − 10h − 6k + 14 = 0
− y , y < 0
Thus, the locus of (h, k) is
∴ x2 = 2 y + 2 y, y ≥ 0 y2 − 10x − 6 y + 14 = 0
and x2 = − 2 y + 2 y, y < 0 10. Let C1 (h , k) be the centre of the required circle. Then,
⇒ x2 = 4 y, when y ≥ 0
(h − 0)2 + (k − 0)2 = (h − 1)2 + (k − 0)2
and x2 = 0, when y < 0
⇒ h 2 + k2 = h 2 − 2h + 1 + k2
∴ {(x, y): x2 = 4 y, when y ≥ 0} ∪ {(0, y): y < 0}
⇒ −2 h + 1 = 0 ⇒ h = 1 / 2
8. NOTE In solving a line and a circle there oftengenerate a
Since, (0, 0) and (1, 0) lie inside the circle x2 + y2 = 9.
quadratic equation and further we have to apply
Therefore, the required circle can touch the given circle
condition of Discriminant so question convert from
internally.
coordinate to quadratic equation.
i.e. C1 ⋅ C 2 = r1 ~ r2
From equation of circle it is clear that circle passes
through origin. Let AB is chord of the circle.
Y ⇒ h 2 + k2 = 3 − h 2 + k2
1
⇒ 2 h 2 + k2 = 3 ⇒ 2 + k2 = 3
4
1 3 1 9
⇒ + k2 = ⇒ + k2 =
4 2 4 4
A (p,q)
⇒ k2 = 2 ⇒ k = ± 2
11. The required equation of circle is
C (h,0)  1 25
X′ X (x2 + y2 + 13x − 3 y) + λ  11x + y +  =0 …(i)
O  2 2
B
Its passing through (1, 1),
Y′ ⇒ 12 + λ (24) = 0
A ≡ ( p, q) ⋅ C is mid-point and coordinate of C is (h , 0) 1
⇒ λ=−
2
rankershalt.com

408 Circle

On putting in Eq. (i), we get The chord will be of maximum length, if φ = 0 or


11 1 25 16 2
x2 + y2 + 13x − 3 y − x− y− =0 h 2 + k2 − 9 = 0 ⇒ h 2 + h =9
2 4 4 9
⇒ 4x2 + 4 y2 + 52x − 12 y − 22x − y − 25 = 0 9
⇒ h=±
⇒ 4x2 + 4 y2 + 30x − 13 y − 25 = 0 5
12
∴ k= m
12. The required equation of circle is, S1 + λ (S 2 − S1 ) = 0. 5
∴ (x2 + y2 − 6) + λ (−6x + 14) = 0 9 12  9 12
Hence, centres are  , −  and  − , 
Also, passing through (1, 1). 5 5   5 5
⇒ − 4 + λ (8) = 0
16. For point of intersection, we put
1
⇒ λ= 3 y + 10
2 x= in x2 + y2 − 2x + 4 y − 20 = 0
4
∴ Required equation of circle is
2
x2 + y2 − 6 − 3x + 7 = 0  3 y + 10  3 y + 10
⇒   + y −2 
2
 + 4 y − 20 = 0
or x2 + y2 − 3x + 1 = 0  4   4 
13. It is given that, C1has centre (0, 0) and radius 1. ⇒ 25 y2 + 100 y − 300 = 0
⇒ y2 + 4 y − 12 = 0
Similarly, C 2 has centre (0, 0) and radius 2 and C k has ⇒ ( y − 2) ( y + 6) = 0
centre (0, 0) and radius k. ⇒ y = − 6, 2
Now, particle starts it motion from (1, 0) and moves 1 When y = −6 ⇒ x = − 2
radian on first circle then particle shifts from C1 to C 2. When y=2
After that, particle moves 1 radian on C 2 and then ⇒ x=4
particle shifts from C 2 to C3 . Similarly, particle move on ∴ Point of intersection are (−2, − 6) and (4, 2) .
n circles.
17. Equation of any tangent to circle x2 + y2 = r 2 is
Now, n ≥ 2π because particle crosses the X-axis for the
first time on C n, then n is least positive integer. x cos θ + y sin θ = r … (i)
Therefore, n = 7. Suppose Eq. (i) is tangent to 4x + 25 y = 100
2 2

14. Equation of any circle passing through the point of x2 y2


intersection of x2 + y2 − 2x = 0 and y = x is or + = 1 at (x1 , y1 )
25 4
(x2 + y2 − 2x) + λ ( y − x) = 0
xx1 yy
⇒ x2 + y2 − (2 + λ )x + λy = 0 Then, Eq. (i) and + 1 = 1 are identical
25 4
2 + λ − λ
Its centre is  , . y1
 2 2  x1 / 25 1
∴ = 4 =
⇒ For AB to be the diameter of the required circle the cos θ sin θ r
centre must be on AB, i.e. 25 cos θ 4 sin θ
2+ λ =−λ ⇒ x1 = , y1 =
r r
⇒ λ = −1 [Q y = x]
The line (i) meet the coordinates axes in A (r sec θ , 0)
Therefore, equation of the required circle is
and β (0, r cosec θ ). Let (h , k) be mid-point of AB.
x2 + y2 − (2 − 1) x − 1 ⋅ y = 0
r sec θ r cosecθ
⇒ x2 + y2 − x − y = 0 Then, h= and k =
2 2
15. Given, C1: x2 + y2 = 16 r r
Therefore, 2h = and 2k =
and let C 2 : (x − h )2 + ( y − k)2 = 25 cos θ sin θ
∴ Equation of common chords is S1 − S 2 = 0 25 4
∴ x1 = and y1 =
∴ 2hx + 2ky = (h 2 + k2 − 9) 2h 2k
h 3
∴ Its slope = − = [given] x2 y2
k 4 As (x1 , y1 ) lies on the ellipse + = 1, we get
25 4
If p be the length of perpendicular on it from the centre 1  625 1  4 
h 2 + k2 − 9   +   =1
(0, 0) of C1 of radius 4, then p = . 25  4h 2  4  k2
4h 2 + 4k2 25 1
⇒ + =1
Also, the length of the chord is 4h 2 k2
or 25k2 + 4h 2 = 4h 2 k2
2 r 2 − p2 = 2 4 2 − p2
Therefore, required locus is 4x2 + 25 y2 = 4x2y2
rankershalt.com

Circle 409

18. The equation of the circle on the line joining the points k+4
⇒ ⋅ (1) = − 1
A(3, 7) and B (6, 5) as diameter is h+4
(x − 3) (x − 6) + ( y − 7) ( y − 5) = 0 …(i) ⇒ k + 4 = − h −4
and the equation of the line joining the points A (3, 7) ⇒ h + k = −8 …(i)
7 −5
and B (6, 5) is y − 7 = (x − 3) Also, CP 2 = (h + 4)2 + (k + 4)2
3 −6
⇒ (h + 4)2 + (k + 4)2 = r 2 …(ii)
⇒ 2x + 3 y − 27 = 0 …(ii)
In ∆ACM , we have
Now, the equation of family of circles passing through 2
the point of intersection of Eqs. (i) and (ii) is  h + k
AC 2 = (3 2 )2 +  
S + λP = 0  2 
⇒ (x − 3)(x − 6) + ( y − 7)( y − 5) + λ (2x + 3 y − 27) = 0 ⇒ r 2 = 18 + 32
⇒ x2 − 6x − 3x + 18 + y2 − 5 y − 7 y + 35 ⇒ r =5 2 …(iii)
+ 2λx + 3λy − 27λ = 0 Also, CP = r
⇒ S1 ≡ x2 + y2 + x (2λ − 9) + y (3λ − 12) 
h − = r
k

+ (53 − 27λ ) = 0 …(iii)  2 
Again, the circle,which cuts the members of family of ⇒ h − k = ± 10 …(iv)
circles, is From Eqs. (i) and (iv), we get
S 2 ≡ x2 + y2 − 4x − 6 y − 3 = 0 …(iv) (h = − 9, k = 1)
and the equation of common chord to circles S1 and S 2 is or (h = 1, k = − 9)
S1 − S 2 = 0 Thus, the equation of the circles are
⇒ x (2λ − 9 + 4) + y (3λ − 12 + 6) + (53 − 27λ + 3) = 0 (x + 9)2 + ( y − 1)2 = (5 2 )2
⇒ 2λx − 5x + 3λy − 6 y + 56 − 27λ = 0 or (x − 1)2 + ( y + 9)2 = (5 2 )2
⇒ x + y2 + 18x − 2 y + 32 = 0
2
⇒ (− 5x − 6 y + 56) + λ (2x + 3 y − 27) = 0
or x2 + y2 − 2x + 18 y + 32 = 0
which represents equations of two straight lines
Clearly, (−10, 2) lies interior of
passing through the fixed point whose coordinates are
obtained by solving the two equations x2 + y2 + 18x − 2 y + 32 = 0
5x + 6 y − 56 = 0 and 2x + 3 y − 27 = 0, Hence, the required equation of circle, is
x2 + y2 + 18x − 2 y + 32 = 0
we get x = 2 and y = 23 /3
19. The parametric form of OP is 20. Let P (h , k) be the foot of perpendicular drawn from
x−0 y−0 origin O(0, 0) on the chord AB of the given circle such
= that the chord AB subtends a right angle at the origin.
cos 45° sin 45°

x2 + y2 + 2g x + 2fy + c = 0
Since, OP = 4 2
So, the coordinates of P are given by B
x−0 y−0
= = −4 2
cos 45° sin 45°
So, P(−4, − 4) P (h, k)
Let C (h , k) be the centre of circle and r be its radius.
Now, CP ⊥ OP A

Y O
y=x The equation of chord AB is
B (x2, y2) h
y − k = − (x − h ) ⇒ hx + ky = h 2 + k2
M k
3 3 √ 2 A (x , y )
(h,k)C 1 1 The combined equation of OA and OB is homogeneous
X equation of second degree obtained by the help of the
P O
given circle and the chord AB and is given by,
4 √2 2
 hx + ky  hx + ky
x2 + y2 + (2 gx + 2 fy)  2  + c 2  =0
4)

h + k  h + k 
2 2
4,−

y = −x
(−

Since, the lines OA and OB are at right angle.


rankershalt.com

410 Circle

∴Coefficient of x2 + Coefficient of y2 = 0 Now,  −g − f − 2


  −g + f − 2
 = 
 2 gh ch 2   2   2 
⇒ 1 + 2 + 2 2
 h +k 2
(h + k ) 
2
⇒ − g − f − 2 = ± (− g + f − 2)
 2f k ck 
2 ⇒ − g − f −2 = − g + f −2
+ 1 + 2 + 2 2 2
=0
+ 2
+ or − g − f −2 = g − f + 2
 h k (h k ) 
⇒ f = 0 or g = − 2
⇒ 2 (h 2 + k2) + 2 ( g h + f k) + c = 0
c Case I When f = 0
⇒ h 2 + k2 + gh + fk + = 0 From Eq. (iii), we get
2
∴ Required equation of locus is  − g − 2
 = g2 − c
c  2 
x + y + gx + fy + = 0
2 2
2 ⇒ ( g + 2)2 = 2 ( g 2 − c)
21. Let the equation of L1 be x cos α + y sin α = p1. ⇒ g 2 − 4 g − 4 − 2c = 0 …(iv)
Then, any line perpendicular to L1 is On putting f = 0 in Eq. (ii). we get
Y 25 − 8 g + c = 0 …(v)
L2 Eliminating c between Eqs. (iv) and (v), we get
Q g 2 − 20 g + 46 = 0
⇒ g = 10 ± 3 6 and c = 55 ± 24 6
S On substituting the values of g , f and c in Eq. (i), we get
X′ X
R O P x2 + y2 + 2 (10 ± 3 6 ) x + (55 ± 24 6 ) = 0
L1 Case II When g = − 2
Y′
From Eq. (iii), we get
x sin α − y cos α = p2
⇒ f 2 = 2 (4 + f 2 − c)
where, p2 is a variable.
⇒ f 2 − 2c + 8 = 0 …(vi)
Then, L1 meets X-axis at P ( p1 sec α ,0) and Y-axis at On putting g = − 2 in Eq. (ii), we get
Q (0, p1cosec α). c = − 6 f − 41
Similarly, L 2 meets X-axis at R ( p2 cosec α , 0) and On substituting c in Eq. (vi), we get
Y-axis at S (0, − p2 sec α ). f 2 + 12 f + 90 = 0
Now, equation of PS is,
x y x y This equation gives imaginary values of f.
+ =1 ⇒ − = secα …(i) Thus, there is no circle in this case.
p1 sec α − p2 sec α p1 p2
Hence, the required equations of the circles are
Similarly, equation of QR is
x y x y x2 + y2 + 2 (10 ± 3 6 ) x + (55 ± 24 6 ) = 0
+ =1 ⇒ + = cosec α …(ii)
p2cosec α p1cosec α p2 p1 23. Given lines are
3x + 5 y − 1 = 0 ...(i)
Locus of point of intersection of PS and QR can be
and (2 + c)x + 5c2y − 1 = 0 ...(ii)
obtained by eliminating the variable p2 from Eqs. (i)
x y 1
and (ii). ∴ = =
 x  x y −5 + 5 c2
− (2 + c) + 3 15c − 5c − 10
2

∴  − secα  + = cosec α
 p1  y p1 5(c2 − 1) 1−c
⇒ x= and y =
⇒ (x − p1 secα ) x + y2 = p1 y cosec α 5(3c2 − c − 2) 15c2 − 5c − 10
⇒ x + y − p1x sec α − p1 y cosec α = 0
2 2 2c
⇒ lim x = lim
c→1 c→1 6 c − 1
which is a circle through origin.
−1 2
22. Let the equation of the required circle be and lim y = lim ⇒ lim x =
c→1 c→1 30 c − 5 c→1 5
x2 + y2 + 2 gx + 2 fy + c = 0 …(i) 1
and lim y = −
It passes through (−4, 3). c→1 25
∴ 25 − 8 g + 6 f + c = 0 …(ii)  2 1
∴ Centre =  lim x, lim y =  , − 
Since, circle touches the line x + y − 2 = 0 and  c→1 c→1   5 25
x − y − 2 = 0. 2 2
 2  1 64 1 1601
∴  −g − f − 2  −g + f − 2
 =  = g2 + f 2 − c ∴ Radius = 2 −  + 0 +  = + =
…(iii)  5  25 25 625 25
 2   2 
rankershalt.com

Circle 411

2 2 1
 2  1 1601 Also, area of ∆C1 AC 2 = C1C 2 × AM
∴ Equation of the circle is  x −  +  y +  =
 5  25 625 2
1 AB  AB 
4x 2 y 4 1 1601 = × 13 × Q AM =
⇒ x2 + y2 − + + + − =0 2 2  2 
5 25 25 625 625 1
∴ × 13 × AM = 30 cm
⇒ 25(x2 + y2) − 20x + 2 y − 60 = 0 4
120
AM = cm
Topic 5 Equation of Chord Bisected at a 13
Point, Product of Pair of Tangents, 2. Given equation of line is x + y = n,n ∈ N …(i)
Chord of Contact of Tangent, Pole and equation of circle is x2 + y2 = 16 …(ii)
and Equations of Polar Now, for intercept, made by circle (ii) with line (i)

1. Let the length of common chord = AB = 2 AM = 2x


x2+y2=16
(0, 0)
A
12 5 4
d
C1 C2
M
12 5 A B x+y=n, n 0 N

B d <4
n
⇒ <4
Now, C1C 2 = AC12 + AC 22 … (i) 2
[Qcircles intersect each other at 90º] [Q d = perpendicular distance from (0, 0) to the line
|0 + 0 − n| n 
and C1C 2 = C1M + MC 2 x + y = n and it equal to = 
12 + 12 2
⇒ C1C 2 = 122 − AM 2 + 52 − AM 2 … (ii)
⇒ n <4 2 …(iii)
From Eqs. (i) and (ii), we get Q n ∈ N , so n = 1, 2, 3, 4, 5
AC12 + AC 22 = 144 − AM 2 + 25 − AM 2 Clearly, length of chord AB = 2 42 − d 2
⇒ 144 + 25 = 144 − x2 + 25 − x2 n2  n 
= 2 16 − Q d = 2 
⇒ 13 = 144 − x + 25 − x
2 2 2
∴ Sum of square of all possible lengths of chords (for
On squaring both sides, we get
n = 1, 2, 3, 4, 5)
169 = 144 − x2 + 25 − x2 + 2 144 − x2 25 − x2  1 
= 4 (16 × 5) − (12 + 22 + 32 + 42 + 52)
 2 
⇒ x2 = 144 − x2 25 − x2
5(6)(11)
Again, on squaring both sides, we get = 320 − 2 = 320 − 110 = 210
6
x4 = (144 − x2) (25 − x2) = (144 × 25) − (25 + 144)x2 + x4
3. Given equation of circle is
144 × 25 12 × 5 60
⇒ x2 = ⇒ x= = cm x2 + y2 − 8x − 8 y − 4 = 0, whose centre is C (4, 4) and
169 13 13
120 radius = 42 + 42 + 4 = 36 = 6
Now, length of common chord 2x = cm
13 Let the centre of required circle be C1 (x, y). Now, as it
Alternate Solution touch the X-axis, therefore its radius = y . Also, it
Given, AC1 = 12 cm and AC 2 = 5 cm touch the circle
In ∆C1 AC 2, x2 + y2 − 8x − 8 y − 4 = 0, therefore CC1 = 6 + y

C1C 2 = (C1 A )2 + ( AC 2)2 [Q∠C1 AC 2 = 90º, ⇒ (x − 4)2 + ( y − 4)2 = 6 + y


because circles intersects each other at 90º] ⇒ x2 + 16 − 8x + y2 + 16 − 8 y = 36 + y2 + 12 y
⇒ x − 8x − 8 y + 32 = 36 + 12 y
2
= (12)2 + (5)2 = 144 + 25
⇒ x2 − 8x − 8 y − 4 = 12 y
= 169 = 13 cm
Case I If y > 0, then we have
1
Now, area of ∆C1 AC 2 = AC1 × AC 2 x2 − 8x − 8 y − 4 = 12 y
2
1
= × 12 × 5 = 30 cm2 ⇒ x2 − 8x − 20 y − 4 = 0
2 ⇒ x2 − 8x − 4 = 20 y
rankershalt.com

412 Circle

⇒ (x − 4)2 − 20 = 20 y 6. We have to find locus of mid-point of chord and we know


⇒ (x − 4) = 20 ( y + 1,) which is a parabola.
2
perpendicular from centre bisects the chord.
Case II If y < 0, then we have ∴ ∠OAC = 45°
x − 8x − 8 y − 4 = − 12 y
2
A
⇒ x2 − 8x − 8 y − 4 + 12 y = 0
⇒ x2 − 8x + 4 y − 4 = 0 2 45°
⇒ x2 − 8x − 4 = − 4 y

(h , k )
O
⇒ (x − 4) 2 = 20 − 4 y C
⇒ (x − 4)2 = − 4( y − 5), which is again a parabola. 2
B
4. PLAN If S : ax 2 + 2 hxy + by 2 + 2 gx + 2 fy + C
then equation of chord bisected at P ( x 1, y1 ) is T = S1
or a xx 1 + h( xy1 + yx 1 ) + b yy1 + g ( x + x 1 ) + f ( y + y1 ) + C OC
2 2 In ∆ OAC , = sin 45°
= ax 1 + 2 hx 1 y1 + by1 + 2 gx 1 + 2 fy1 + C OA
2
Description of Situation As equation of chord of ⇒ OC = = 2
contact is T = 0 2
Equation of polar Also, h 2 + k2 = OC 2

P
Hence, x2 + y2 = 2 is required equation of locus of
mid-point of chord subtending right angle at the centre.
7. Since, tangents are drawn from origin. So, the equation
P' of tangent be y = mx.
Equation of chord of contact
⇒ Length of perpendicular from origin = radius
A
Here, equation of chord of contact w.r.t. P is ius
 4λ − 20 (0, 0) Rad
xλ + y.   =9 = r
 5  O (– r, h)
5λx + (4λ − 20) y = 45 …(i)

B
A
|mr + h |
l, 4l–20 P
Q ⇒ =r
9
y 2=

(h,k) (0,0)
5 m2 + 1
x 2+

B
⇒ m2r 2 + h 2 + 2mrh = r 2 (m2 + 1)
S1=T
r 2 − h 2
and equation of chord bisected at the point Q (h , k) is ⇒ m = , m = ∞
 2rh 
xh + yk − 9 = h 2 + k2 − 9
⇒ xh + ky = h 2 + k2 r 2 − h 2
…(ii) ∴ Equation of tangents are y =   x, x = 0
From Eqs. (i) and (ii), we get  2rh 
5λ 4λ − 20 45 Therefore (a) and (c) are the correct answers.
= = 2
h k h + k2 8. Equation of given circle C is
20h 9h (x − 3)2 + ( y + 5)2 = 9 + 25 − 30
∴ λ= and λ = 2 i.e. (x − 3)2 + ( y + 5)2 = 22
4h − 5k h + k2
20h 9h Centre = (3, − 5)
⇒ =
4h − 5k h 2 + k2 If L1 is diameter, then 2(3) + 3(−5) + p − 3 = 0 ⇒ p = 12
or 20 (h + k ) = 9 (4h − 5k)
2 2 ∴ L1 is 2x + 3 y + 9 = 0
or 20 (x2 + y2) = 36x − 45 y L 2 is 2x + 3 y + 15 = 0
5. For required circle, P (1, 8) Distance of centre of circle from L 2 equals
A
and O (3, 2) will be the end
2(3) + 3(−5) + 15 6
point of its diameter. (1, 8)P O(3 ,2) = <2 [radius of circle]
∴ (x − 1) (x − 3) + ( y − 8) ( y − 2) 2 +3
2 2 13
B
=0 ∴ L 2 is a chord of circle C.
⇒ x + y − 4x − 10 y + 19 = 0
2 2 Statement II is false..
rankershalt.com

Circle 413

9. A point on the line 2x + y = 4 is of the form (h , 4 − 2h ). But AB is the chord of circle


Equation of the chord of contact is T = 0 i.e. x2 + 4x + ( y − 3)2 = 0
hx + (4 − 2h ) y = 1 ⇒ (4 y − 1) + h (x − 2 y) = 0 Thus, B must satisfy above equation.
2
This line passes through the point of intersection of h 2 4h 1 
 1 1 ∴ + + (k + 3) − 3 = 0
4 y − 1 = 0 and x − 2 y = 0 i.e. through the point  ,  . 4 2 2 
 2 4
⇒ h 2 + k2 + 8h − 6k + 9 = 0
10. Given, 4x2 + 4 y2 − 12x + 4 y + 1 = 0 A ∴ Locus of M is the circle
⇒ x2 + y2 − 3x + y + 1 / 4 = 0
x2 + y2 + 8x − 6 y + 9 = 0
3 1 π/3
whose centre is  , −  and radius S C D
13. Equation of straight line passing through intersection
2 2
of two circles C1 and C 2 is (S1 − S 2) = 0.
2 2 2
 3  1 1 9 1 1 B ∴ x2 + y2 − x + 4 y − 3 − (x2 + y2 + 6x + 2 y − 15) = 0
=   + −  − = + − 3
 2  2 4 4 4 4
20
3 ⇒ x − 2 y − 12 = 0 ⇒ 10x − 3 y − 18 = 0
= 3
2
14. Here, length of tangent AB
Again, let S be a circle with centre at C and AB is given
chord and AD subtend angle 2π / 3 at the centre and D be = 42 + 52 − 4(4) − 2(5) − 11 = 2
the mid point of AB and let its coordinates are (h, k).
1 1 2π π B
Now, ∠DCA = (∠BCA ) = ⋅ =
2 2 3 3
2 4
Using sine rule in ∆ADC, (4, 5)
O
A
DA CA (2, 1)
=
sin π / 3 sin π / 2
3 3 C
⇒ DA = CA sin π / 3 = ⋅
2 2
Now, in ∆ACD ∴ Area of quadrilateral ABOC = 2 area of ∆ABO
9 27 9 1
CD = CA − AD = −
2 2
=2 = 2 ⋅ ( AB) (OB)
4 16 16 2
= 2 ⋅ 4 = 8 sq units
But CD 2 = (h − 3 / 2)2 + (k + 1 / 2)2
9 15. For the equation of circle x2 + y2 − 2x = 0. Let the
⇒ (h − 3 / 2)2 + (k + 1 / 2)2 = mid-point of chords be (h , k).
16
∴ Equation of chord bisected at the point is S1 = T .
Hence, locus of a point is
2 2 ∴ h 2 + k2 − 2h = xh + yk − (x + h ) which passes through
 3  1 9 (0, 0).
x −  +  y +  =
 2  2  16 ⇒ h 2 + k2 − 2h = − h
⇒ 16x2 + 16 y2 − 48x + 16 y + 31 = 0
11. Area of triangle formed by the tangents from the point M
(h , k) to the circle x2 + y2 = a 2 and their chord of contact (0, 0) O
(1, 0)
(h 2 + k2 − a 2)3/ 2
=a
h 2 + k2
Thus, area of triangle formed by tangents from (4, 3) to
the circle x2 + y2 = 9 and their chord of contact ∴ The required locus of a chord is x2 + y2 − x = 0
3 (42 + 32 − 9)3/ 2 3 (16 + 9 − 9)3/ 2 16. 2x2 + y2 − 3xy = 0 [given]
= =
42 + 32 25 ⇒ 2x − 2xy − xy + y = 0
2 2
3 (64) 192
= = sq units ⇒ 2x (x − y) − y (x − y) = 0
25 25
⇒ (2x − y) (x − y) = 0
12. Given, (x + 2)2 + ( y − 3)2 = 4
⇒ y = 2x, y = x
Let the coordinate be M (h , k), where B is mid-point of A are the equations of straight lines passing through
and M. origin.
 h k + 3 Now, let the angle between the lines be 2 θ and the line
⇒ B , 
2 2  y = x makes angle of 45° with X-axis.
rankershalt.com

414 Circle

Therefore, tan (45° + 2 θ ) = 2 (slope of the line y = 2x) From figure it is clear that, ∆OLS is a right triangle
Y y = 2x
with right angle at L.
Also, OL = 1 and OS = 2
y=x 1
3 C ∴ 1 sin (∠ LSO ) = ⇒ ∠ LSO = 30°
3 2
A Since, SA1 = SA2, ∆ SA1 A2 is an equilateral triangle.
The circle with centre at C1 is a circle inscribed in the
∆ SA1 A2. Therefore, centre C1 is centroid of ∆ SA1 A2.
This, C1 divides SM in the ratio 2 : 1. Therefore,
45°
X′
O
X coordinates of C1 are (−4 / 3, 0) and its radius
= C1M = 1 / 3
Y′ ∴ Its equation is (x + 4 / 3)2 + y2 = ( 1 / 3)2 …(i)
tan 45° + tan 2 θ 1 + tan 2 θ
⇒ =2 ⇒ =2 The other circle touches the equilateral triangle SB1B2
1 − tan 45° × tan 2 θ 1 − tan 2 θ ∆
externally. Its radius r is given by r = ,
(1 + tan 2 θ ) − (1 − tan 2 θ ) 2 − 1 1 s−a
⇒ = =
(1 + tan 2 θ ) + (1 − tan 2 θ ) (2 + 1) 3 1 3
where B1 B2 = a. But ∆ = (a ) (SN ) = a
2 tan 2 θ 1 1 2 2
⇒ = ⇒ tan 2 θ =
2 3 3 3 a
and s− a = a −a =
2 tan θ 1 2 2
⇒ =
1 − tan 2 θ 3 Thus, r =3
⇒ (2 tan θ ) ⋅ 3 = 1 − tan 2 θ ⇒ Coordinates of C 2 are (4, 0).
⇒ tan 2 θ + 6 tan θ − 1 = 0
∴ Equation of circle with centre at C 2 is
− 6 ± 36 + 4 × 1 × 1 − 6 ± 40
⇒ tan θ = = (x − 4)2 + y2 = 32 …(ii)
2 2
Equations of common tangents to circle (i) and circle C
⇒ tan θ = − 3 ± 10
π are

⇒ tan θ = − 3 + 10 Q0 < θ <
 4  1
x = − 1 and y = ± (x + 2) [T1 and T2]
Again, in ∆ OCA 3
3
tan θ = Equation of common tangents to circle (ii) and circle C
OA
3 3 3 (3 + 10 ) are
∴ OA = = = 1
tan θ (− 3 + 10 ) (− 3 + 10 ) (3 + 10 ) x = 1 and y = ± (x + 2) [T1 and T2]
3
3 (3 + 10 )
= = 3 (3 + 10 )
(10 − 9) Two tangents common to (i) and (ii) are T1 and T2 at O.
To find the remaining two transverse tangents to (i) and
17. (ii), we find a point I which divides the joint of C1 C 2 in
Y
the ratio r1 : r2 = 1 / 3 : 3 = 1 : 9
T1 Therefore, coordinates of I are (−4 / 5, 0)
B1 Equation of any line through I is y = m (x + 4 / 5). It will
touch (i) if
A1 L
m  − 4 + 4 − 0 
30° 
X′ X

S C1 M O N C2   3 5 = 1 ⇒ −
8m 1
= 1 + m2
A2 1+m 2 3  15  3
B2
⇒ 64 m2 = 25 (1 + m2)
T2 5
⇒ 39 m2 = 25 ⇒ m=±
Y′ 39
5  4
Therefore, these tangents are y = ± x + 
39  5
l
C1 C2 18. Let the coordinate of point P be (2r cos θ , 2r sin θ )
We have, OA = r , OP = 2r
Since, ∆ OAP is a right angled triangle.
rankershalt.com

Circle 415

Equation of the chord having (α , − α ) as mid points is


C1 T = S1
A C2
 1 + 2a   1 − 2a 
θ−φ ⇒ xα + y (− α ) −   (x + α ) −   (y − α )
θ  4   4 
P X
φ O  1 + 2a   1 − 2a 
= α 2 + (− α )2 −  α −  (− α )
B  2   2 

⇒ 4xα − 4 yα − (1 + 2a ) x − (1 + 2a ) α

cos φ = 1 / 2 ⇒ φ = π /3 − (1 − 2a ) y + (1 − 2a ) α
∴Coordinates of A are { r cos (θ − π / 3), r sin (θ − π / 3)} = 4α + 4α − (1 + 2a ) ⋅ 2α + (1 − 2a ) ⋅ 2α
2 2

and that of B are [r cos (θ + π / 3), r sin (θ + π / 3)] ⇒ 4αx − 4αy − (1 + 2a ) x − (1 − 2a ) y


If p, q is the centroid of ∆ PAB, then = 8 α 2 − (1 + 2a ) α + (1 − 2a ) α
1
p = [r cos (θ − π / 3) + r cos (θ + π / 3) + 2r cos θ ]
3 But this chord will pass through the point
1  1 + 2a 1 − 2a 
= [r {cos (θ − π / 3) + cos (θ + π / 3)} + 2r cos θ ]  , .
3  2 2 
  π π π π 
θ− +θ+ θ − −θ−   1 + 2a   1 − 2a  (1 + 2a ) (1 + 2a )
1  3 3 3 3  4α   −4α   −
= r 2 cos . cos  + 2r cos θ 
3  2 2  2   2  2
 
    (1 − 2a ) (1 − 2a )
1 −
= [r {2 cos θ cos π / 3} + 2r cos θ ] 2
3
1 = 8 α 2 − 2 2 aα
= [r ⋅ cos θ + 2r cos θ ] = r cos θ
3 ⇒ 2 α [(1 + 2a − 1 + 2a )] = 8 α 2 − 2 2 aα
1  π  π 1
and q = [r sin θ −  + r sin θ +  + 2r sin θ ] ⇒ 4 2 aα − [2 + 2 ( 2a )2] = 8 α 2 − 2 2aα
3  3  3 2
1  π  π [Q (a + b)2 + (a − b)2 = 2a 2 + 2b2]
= [r {sin θ −  + sin θ +  } + 2r sin θ ]
3  3  3 ⇒ 8 α 2 − 6 2 aα + 1 + 2a 2 = 0
  π π π π  But this quadratic equation will have two distinct
θ− +θ+ θ − − θ− 
1  3 3 3 3  roots, if
= r 2 sin . cos  + 2 r sin θ 
3  2 2   (6 2a )2 − 4 (8) (1 + 2a 2) > 0 ⇒ 72a 2 − 32 (1 + 2a 2) > 0
   
1 ⇒ 8a 2 − 32 > 0 ⇒ a2 − 4 > 0
= [r (2 sin θ cos π / 3) + 2r sin θ ]
3 ⇒a < −2 ∪a >2
1
= [r (sin θ ) + 2r sin θ ] = r sin θ Therefore, a ∈ (− ∞ , − 2) ∪ (2, ∞ ) .
3
Now, ( p, q) = (r cos θ , r sin θ) lies on x2 + y2 = r 2 which is 20. The given circle can be rewritten as
called C1. by
x2 + y2 − ax − =0 …(i)
19. Given, 2x2 + 2 y2 − (1 + 2a ) x − (1 − 2a ) y = 0 2
 1 + 2a   1 − 2a 
⇒ x 2 + y2 −   x−  y=0 Let one of the chord through (a , b / 2) be bisected at (h , 0).
 2   2 
Then, the equation of the chord having (h , 0) as
Since, y + x = 0 bisects two chords of this circle, mid mid-point is
points of the chords must be of the form (α , − α ). T = S1
Y a b
⇒ h ⋅ x + 0 ⋅ y − (x + h ) − ( y + 0) = h 2 + 0 − ah − 0
2 4
 a by a
y

⇒  −  − − h = h 2 − ah
+

h x …(ii)
 2
x

4 2
=
0

O It passes through (a , b / 2), then


X
 a b b a
 h −  a − ⋅ − h = h − ah
2
(α,−α)  2  4 2 2
3 a2 b2
⇒ h 2 − ah + + =0 …(iii)
2 2 8
(α,−α)
rankershalt.com

416 Circle

According to the given condition, Eq. (iii) must have two x2 + y2 = hx + ky


distinct real roots. This is possible, if the discriminant of Alternate Solution
Eq. (iii) is greater than 0. Let M be the mid-point of chord AB.
9 2  a 2 b2 a 2 b2
i.e. a −4  +  >0 ⇒ − >0 ⇒ CM ⊥ MP
4  2 8 4 2
⇒ (slope of CM) ⋅ (slope of MP) = − 1
⇒ a 2 > 2b2 y1 k − y1
21. Since, 5x + 12 y − 10 = 0 and 5x − 12 y − 40 = 0 are both ⇒ ⋅ = −1
x1 h − x1
perpendicular tangents to the circle C1. ⇒ ky1 − y12 = − hx1 + x12
∴ OABC forms a square. Hence, required locus is x2 + y2 = hx + ky
Let the centre coordinates be (h , k), where,
23. Equation of tangents at (1, 7) and (4, − 2) are
OC = OA = 3 and OB = 6 2
|5h + 12k − 10| |5h + 12k − 40| B (1,7) C
⇒ = 3 and =3
13 13
Q C1
D (4,–2)
A (1,2)
O (h,k)
3
40

C A
y=
5x

3
+

12

x + 7 y − (x + 1) − 2 ( y + 7) − 20 = 0
12

5x
y=

⇒ 5 y = 35 ⇒ y = 7
10

–15
B 5,
12 and 4x − 2 y − (x + 4) − 2 ( y − 2) − 20 = 0
⇒ 3x − 4 y = 20
⇒ 5h + 12k − 10 = ± 39 and 5h − 12k − 40 = ± 39 ∴ Point C is (16, 7).
on solving above equations. The coordinates which lie in ∴ Vertices of a quadrilateral are
I quadrant are (5, 2).
A (1, 2), B(1, 7), C (16, 7), D (4, − 2)
C2
∴ Area of quadrilateral ABCD
A
5 = Area of ∆ ABC + Area of ∆ ACD
4 (5,2) 1 1
3 O = × 15 × 5 + × 15 × 5 = 75 sq units
2 2
C1
4 π x
24. Let θ= ⇒ cos θ =
2k 2
B
5x − 12y = 10 3 +1−x
⇒ cos 2 θ =
2 x
∴ Centre for C1 (5, 2) θ
3 +1−x 2
⇒ 2 cos 2 θ − 1 = C
To obtain equation of circle concentric withC1 and making 2
an intercept of length 8 on5x + 12 y = 10 and5x − 12 y = 40  x2 3 +1−x
2
2θ √3 + 1 – x
∴ Required equation of circle C 2 has centre (5, 2) and ⇒ 2   −1 =
radius 5 is (x − 5)2 + ( y − 2)2 = 52  4 2
⇒ x2 + x − 3 − 3 = 0
22. Given,circle is x2 + y2 = r 2
− 1 ± 1 + 12 + 4 3
Equation of chord whose mid point is given, is ⇒ x=
T = S1 ⇒ xx1 + yy1 − r 2 = x12 + y12 − r 2 2
It also passes through (h, k) hx1 + ky1 = x12 + y12 − 1 ± 13 + 4 3 −1 + 2 3 + 1
= = = 3
∴ Locus of (x1 , y1 ) is 2 2
3 π
∴ cos θ =
⇒θ =
2 6
C (0, 0)
π π
P (h, k) ∴ Required angle = = 2 θ =
B k 3
A
M
(x1, y1) ⇒ k =3

Download Chapter Test


http://tinyurl.com/y4wkkqkf or
rankershalt.com

17
Parabola

Topic 1 Equation of Parabola and Focal Chord


Objective Questions I (Only one correct option) 6. Let( x , y ) be any point on the parabola y 2 = 4x . Let
1. If the area of the triangle whose one vertex is at the vertex P be the point that divides the line segment from
of the parabola, y + 4( x − a ) = 0 and the other two
2 2 (0, 0) to ( x , y ) in the ratio 1 : 3. Then, the locus of P
is (2011)
vertices are the points of intersection of the parabola and (a) x2 = y (b) y2 = 2x
Y -axis, is 250 sq units, then a value of ‘a’ is (c) y2 = x (d) x2 = 2 y
(2019 Main, 11 Jan, II)
(a) 5 5 (b) 5 7. Axis of a parabola is y = x and vertex and focus
(c) 5(21/3 ) (d) (10)2/3 are at a distance 2 and 2 2 respectively
2. A circle cuts a chord of length 4a on the X-axis and passes from the origin. Then, equation of the parabola
through a point on the Y-axis, distant 2b from the origin. is (2006, 3M)
Then, the locus of the centre of this circle, is (a) (x − y)2 = 8 (x + y − 2)
(2019 Main, 11 Jan, II) (b) (x + y)2 = 2 (x + y − 2)
(a) a parabola (b) an ellipse (c) (x − y)2 = 4 (x + y − 2)
(c) a straight line (d) a hyperbola (d) (x + y)2 = 2 (x − y + 2)
3. Axis of a parabola lies along X-axis. If its vertex and focus 8. The locus of the mid-point of the line segment
are at distances 2 and 4 respectively from the origin, on joining the focus to a moving point on the
the positive X-axis, then which of the following points parabola y 2 = 4ax is another parabola with
does not lie on it? (2019 Main, 9 Jan, I) directrix (2002, 1M)
(a) (4, −4) (b) (6, 4 2) (a) x = − a (b) x = −
a
(c) x = 0 (d) x =
a
(c) (8, 6) (d) (5, 2 6) 2 2
4. Let P be the point on the parabola, y 2 = 8x, which is at a 9. The equation of the directrix of the parabola
minimum distance from the centre C of the circle, y 2 + 4 y + 4x + 2 = 0 is (2001, 1M)
x 2 + ( y + 6)2 = 1. Then, the equation of the circle, passing (a) x = − 1 (b) x = 1
through C and having its centre at P is (2016 Main) (c) x = − 3 / 2 (d) x = 3 / 2
(a) x2 + y 2 − 4x + 8 y + 12 = 0 10. If the line x − 1 = 0 is the directrix of the parabola
(b) x2 + y 2 − x + 4 y − 12 = 0 y 2 − kx + 8 = 0, then one of the values of k is
x
(c) x2 + y 2 − + 2 y − 24 = 0 (2000, 2M)
4
1 1
(d) x2 + y 2 − 4x + 9 y + 18 = 0 (a) (b) 8 (c) 4 (d)
8 4
5. Let O be the vertex and Q be any point on the parabola
x 2 = 8 y. If the point P divides the line segment OQ
11. The curve described parametrically by
x = t 2 + t + 1, y = t 2 − t + 1 represents (1999, 2M)
internally in the ratio 1 : 3, then the locus of P is (2015)
(a) x2 = y (b) y2 = x (a) a pair of straight lines (b) an ellipse
(c) y2 = 2x (d) x2 = 2 y (c) a parabola (d) a hyperbola
rankershalt.com

418 Parabola

Assertion and Reason Integer Answer Type Questions


2
x 13. Let the curve C be the mirror image of the
12. Statement I The curve y = − + x + 1 is symmetric
2 parabola y 2 = 4x with respect to the line
with respect to the line x = 1. because (2007, 3M) x + y + 4 = 0. If A and B are the points of
Statement II A parabola is symmetric about its axis. intersection of C with the line y = − 5, then the
(a) Statement I is correct, Statement II is correct, Statement distance between A and B is (2015 Adv.)
II is a correct explanation for Statement I 14. Let S be the focus of the parabola y 2 = 8x and PQ
(b) Statement I is correct, Statement II is correct, Statement II be the common chord of the circle
is not a correct explanation for Statement I
x 2 + y 2 − 2x − 4 y = 0 and the given parabola. The
(c) Statement I is correct, Statement II is incorrect
(d) Statement I is incorrect, Statement II is correct
area of ∆PQS is (2012)

Topic 2 Equation of Tangents and Properties


Objective Questions I (Only one correct option) 7. The radius of a circle having minimum area,
1. If the line ax + y = c, touches both the curves x + y = 1 2 2 which touches the curve y = 4 − x 2 and the lines
and y 2 = 4 2x, then| c|is equal to y =|x|, is
(2017 Main)
(2019 Main, 10 April, II)
(a) 2 ( 2 + 1) (b) 2 ( 2 − 1)
1 1
(a) (b) 2 (c) 2 (d) (c) 4 ( 2 − 1) (d) 4 ( 2 + 1)
2 2
8. The slope of the line touching both the parabolas
2. The tangents to the curve y = ( x − 2)2 − 1 at its points of y 2 = 4x and x 2 = − 32 y is (2014 Main)
intersection with the line x − y = 3, intersect at the point 1 3 1 2
(2019 Main, 12 April II)
(a) (b) (c) (d)
2 2 8 3
(a)  , 1 (b)  − , − 1 (c)  , − 1 (d)  − , 1
5 5 5 5
2   2  2   2  9. The tangent at (1, 7) to the curves x 2 = y − 6x
touches the circle x 2 + y 2 + 16x + 12 y + c = 0 at
3. The area (in sq units) of the smaller of the two circles
(2005 2M)
that touch the parabola, y 2 = 4x at the point (1, 2) and (a) (6, 7) (b) (–6, 7)
the X-axis is (2019 Main, 9 April, II) (c) (6, –7) (d) (–6, –7)
(a) 8 π( 3 − 2 2 ) (b) 4 π(3 + 2) 10. The angle between the tangents drawn from the
(c) 8 π(2 − 2 ) (d) 4 π(2 − 2 ) point (1,4) to the parabola y 2 = 4x is (2004, 1M)
4. The equation of a tangent to the parabola, x = 8 y, 2 π π π π
(a) (b) (c) (d)
which makes an angle θ with the positive direction of 6 4 3 2
X-axis, is 11. The focal chord to y 2 = 16 x is tangent to
(2019 Main, 12 Jan, II) ( x − 6)2 + y 2 = 2, then the possible values of the
(a) y = x tan θ − 2 cot θ (b) x = y cot θ + 2 tan θ
slope of this chord are (2003, 1M)
(c) y = x tan θ + 2 cot θ (d) x = y cot θ − 2 tan θ
(a) {−1 , 1 } (b) {−2, 2}
5. Equation of a common tangent to the circle, (c) {−2, 1 / 2} (d) {2, − 1 / 2}
x 2 + y 2 − 6x = 0 and the parabola, y 2 = 4x, is
12. The equation of the common tangent to the curves
(2019 Main, 9 Jan, I)
y 2 = 8x and xy = − 1 is (2002, 1M)
(a) 3 y = 3x + 1 (b) 2 3 y = 12x + 1
(a) 3 y = 9x + 2 (b) y = 2x + 1
(c) 3y = x + 3 (d) 2 3 y = − x − 12 (c) 2 y = x + 8 (d) y = x + 2
6. Tangent and normal are drawn at P(16, 16) on the 13. The equation of the common tangent touching the
parabola y 2 = 16x, which intersect the axis of the circle ( x − 3)2 + y 2 = 9 and the parabola y 2 = 4x
parabola at Aand B, respectively. If C is the centre of the above the X-axis is (2001, 1M)
circle through the points P, A and B and ∠CPB = θ, then
(a) 3 y = 3x + 1 (b) 3 y = − (x + 3)
a value of tanθ is (2018 Main)
1 4 (c) 3 y = x + 3 (d) 3 y = − (3x + 1)
(a) (b) 2 (c) 3 (d)
2 3
rankershalt.com

Parabola 419

Assertion and Reason 16. The value of r is


1 t2 + 1 1 t2 − 1
14. Given A circle, 2x + 2 y = 5 and a parabola,
2 2 (a) − (b) (c) (d)
t t t t
y 2 = 4 5x.
17. If st = 1, then the tangent at P and the normal at S to
Statement I An equation of a common tangent to the parabola meet at a point whose ordinate is
these curves is y = x + 5. (t 2 + 1)2 a (t 2 + 1)2
(a) 3
(b)
5 2t 2t3
Statement II If the line, y = mx + ( m ≠ 0)is the
m a (t 2 + 1)2 a (t 2 + 2)2
(c) (d)
common tangent, then msatisfies m 4 − 3m 2 + 2 = 0. t3 t3
(2013 Main)
Fill in the Blank
(a) Statement I is correct, Statement II is correct,
Statement II is a correct explanation for Statement I 18. The point of intersection of the tangents at the ends of
(b) Statement I is correct, Statement II is correct, the latusrectum of the parabola y 2 = 4x is … .
Statement II is not a correct explanation for Statement I (1994, 2M)
(c) Statement I is correct, Statement II is incorrect Analytical & Descriptive Questions
(d) Statement I is incorrect, Statement II is correct
19. At any point P on the parabola y 2 − 2 y − 4x + 5 = 0 a
Objective Questions II tangent is drawn which meets the directrix at Q. Find
(One or more than correct option) the locus of point R, which divides QP externally in the
1
15. Equation of common tangent of ratio : 1 . (2004, 4M)
2
y = x 2 , y = − x 2 + 4x − 4 is (2006, 5M) 20. Find the shortest distance of the point ( 0, c) from the
(a) y = 4 (x − 1) parabola y = x 2, where 0 ≤ c ≤ 5. (1982, 2M)
(b) y = 0
(c) y = − 4 (x − 1)
(d) y = − 30x − 50 Integer Answer Type Question
21. Consider the parabola y 2 = 8x.Let ∆1 be the area of the
Passage Based Problems triangle formed by the end points of its latusrectum
1 
Passage and the point P  , 2 on the parabola and ∆ 2 be the
Let a, r, s, t be non-zero real numbers. Let 2 
P ( at 2 , 2at ),Q , R ( ar 2 , 2ar ) andS ( as2 , 2as) be distinct area of the triangle formed by drawing tangents at P
point on the parabola y 2 = 4ax. Suppose that PQ is ∆
and at the end points of the latusrectum. Then, 1 is
the focal chord and lines QR and PK are parallel, ∆2
where K is point ( 2a , 0). (2014, Adv.) (2011)

Topic 3 Equation of Normal and Properties


Objective Questions I (Only one correct option) Match the Columns
1. If the parabolas y = 4b( x − c) and y = 8ax have a
2 2
3. Match the conditions/expressions in Column I with
common normal, then which one of the following is a statement in Column II. Normals at P , Q , R are drawn
valid choice for the ordered triad ( a , b, c) ? to y 2 = 4x which intersect at ( 3, 0). Then,
(2019 Main, 10 Jan, I)
(a)  , 2, 0
1 Column I Column II
(b) (1, 1, 0)
2 
A. Area of ∆PQR p. 2
(d)  , 2, 3
1
(c) (1, 1, 3)
2  B. Radius of circumcircle of ∆PQR 5
q.
2
2. If x + y = k is normal to y 2
= 12x, then k is
C. Centroid of ∆PQR r.  , 0
(a) 3 (2000, 2M) 5
(b) 9 2 
(c) −9 D. Circumcentre of ∆PQR s.  , 0
2
(d) −3 3 
rankershalt.com

420 Parabola

(a) vertex is 
2a 
Objective Questions II , 0 (b) directrix is x = 0
 3 
(One or more than one correct option) 2a
(c) latusrectum is (d) focus is (a , 0)
4. Let P be the point on the parabola y 2 = 4x, which is at 3
the shortest distance from the centre S of the circle
x 2 + y 2 − 4x − 16 y + 64 = 0. Let Q be the point on the
Integer Answer Type Question
circle dividing the line segment SP internally. Then, 8. If the normals of the parabola y 2 = 4x drawn at the
(a) SP = 2 5 (2016 Adv.) end points of its latusrectum are tangents to the
(b) SQ : QP = ( 5 + 1) : 2 circle ( x − 3)2 + ( y + 2)2 = r 2, then the value of r 2 is
(c) the x-intercept of the normal to the parabola at P is 6 (2015 Adv.)
1
(d) the slope of the tangent to the circle at Q is Analytical & Descriptive Questions
2
5. A solution curve of the differential equation 9. Normals are drawn from the point P with slopes
dy m1 , m2 , m3 to the parabola y 2 = 4x. If locus of P with
( x + xy + 4x + 2 y + 4)
2
− y 2 = 0, x > 0, passes
dx m1m2 = α is a part of the parabola itself, then findα.
(2003, 4M)
through the point (1, 3). Then, the solution curve
(a) intersects y = x + 2 exactly at one point (2016 Adv.) 10. Three normals are drawn from the point ( c, 0) to the
(b) intersects y = x + 2 exactly at two points 1
curve y 2 = x. Show that c must be greater than .
(c) intersects y = (x + 2)2 2
(d) does not intersect y = (x + 3)2 One normal is always the X-axis. Find c for which
6. Let L be a normal to the parabola y 2 = 4x. If L passes the other two normals are perpendicular to each
through the point (9, 6), then L is given by (2011) other. (1991, 4M)
(a) y − x + 3 = 0 (b) y + 3x − 33 = 0 11. Find the equation of the normal to the curve x 2 = 4 y
(c) y + x − 15 = 0 (d) y − 2x + 12 = 0
which passes through the point (1, 2). (1984, 4M)
7. The tangent PT and the normal PN to the parabola
12. Suppose that the normals drawn at three different
y 2 = 4ax at a point P on it meet its axis at points T and
points on the parabola y 2 = 4x pass through the
N, respectively. The locus of the centroid of the triangle
PTN is a parabola, whose (2009)
point ( h , 0). Show that h > 2. (1981, 4M)

Topic 4 Diameter, Chord of Contact, Chord Bisected and


Product of Pair of Tangents
Objective Questions II radius r having AB as its diameter, then the slope of
(One or more than one correct option) the line joining A and B can be (2010)
1 1 2 2
(a) − (b) (c) (d) −
1. If one end of a focal chord of the parabola, y 2 = 16x is at r r r r
(1, 4), then the length of this focal chord is
(2019 Main, 9 April, I) Passage Based Problems
(a) 22 (b) 25 (c) 24 (d) 20
Passage
2. The length of the chord of the parabola x 2 = 4 y having
Let PQ be a focal chord of the parabola y 2 = 4ax. The
equation x − 2 y + 4 2 = 0 is (2019 Main, 10 Jan II) tangents to the parabola at P and Q meet at a point
(a) 8 2 (b) 2 11 (c) 3 2 (d) 6 3 lying on the line y = 2x + a , a > 0. (2013 Adv.)
3. If a chord, which is not a tangent, of the parabola 5. Length of chord PQ is
y 2 = 16x has the equation 2x + y = p, and mid-point (a) 7a (b) 5a
( h , k), then which of the following is(are) possible (c) 2a (d) 3a
value(s) of p, h and k? (2017 Adv.) 6. If chord PQ subtends an angle θ at the vertex of
y 2 = 4ax , then tanθ is equal to
(a) p = − 1, h = 1, k = − 3 (b) p = 2, h = 3, k = − 4 2 −2
(c) p = − 2, h = 2, k = − 4 (d) p = 5, h = 4, k = − 3 (a) 7 (b) 7
3 3
4. Let A and B be two distinct points on the parabola 2 −2
(c) 5 (d) 5
y 2 = 4x. If the axis of the parabola touches a circle of 3 3
rankershalt.com

Parabola 421

Analytical & Descriptive Questions intersect at points P , Q and R. Determine the ratio of
the areas of the triangles ABC and PQR. (1996, 3M)
7. The angle between a pair of tangents drawn from a
point P to the parabola y 2 = 4ax is 45°. Show that the 10. Show that the locus of a point that divides a chord of
slope 2 of the parabola y 2 = 4ax internally in the
locus of the point P is a hyperbola. (1998, 8M)
ratio 1 : 2 is a parabola. Find the vertex of this
8. From a point A common tangents are drawn to the parabola. (1995, 5M)
a2
circle x 2 + y 2 = and parabola y 2 = 4ax. Find the 11. Through the vertex O of parabola y 2 = 4x , chords OP
2
and OQ are drawn at right angles to one another.
area of the quadrilateral formed by the common
Show that for all positions of P, PQ cuts the axis of the
tangents, the chord of contact of the circle and the
parabola at a fixed point. Also, find the locus of the
chord of contact of the parabola. (1996, 2M)
middle point of PQ. (1994, 4M)
9. Points A, B and C lie on the parabola y 2 = 4ax. The
tangents to the parabola at A, B andC, taken in pairs,

Answers
Topic 1 Topic 3
1. (b) 2. (a) 3. (c) 4. (a) 1. (c) 2. (b) 3. A→ p; B→ q; C→ s; D→ r
5. (d) 6. (c) 7. (a) 8. (c) 4. (a, c, d) 5. (a, d) 6. (a, b, d) 7. (a, d)
9. (d) 10. (c) 11. (c) 12. (a)  3
8. (2) 9. (2) 10.   11. x + y = 3
13. (4) 14. (4)  4
Topic 2 12. (1)
1. (c) 2. (c) 3. (a) 4. (b) Topic 4
5. (c) 6. (b) 7. (c) 8. (a) 1. (b) 2. (d) 3. (b) 4. (c, d)
9. (d) 10. (c) 11. (a) 12. (d)  15a 2 
13. (c) 14. (a) 15. (a, b) 16. (d) 5. (b) 6. (d) 8.   9. (2)
 4 
17. (b) 18. (− 1, 0) 19. ( x + 1 ) (y − 1 ) 2 + 4 = 0
 2 8
1 1 10.  ,  11. y 2 = 2 ( x – 4 )
20. c − , ≤c ≤5 21. (2)  9 9
4 2

Hints & Solutions


Topic 1 Equation of Parabola and Focal Chord
From the given condition, we have
1. Vertex of parabola y = − 4(x − a ) is (a , 0).
2 2 2
Area of ∆ABC = 250
For point of intersection with Y-axis, put x = 0 in the 1 1
∴ (BC )(OA ) = 250 [Q Area = × base × height]
given equation of parabola. 2 2
This gives, y2 = 4a 2 ⇒ y = ± 2a 1
⇒ (4a )a 2 = 250 ⇒ a3 = 125 = 53
2
∴ a =5
Thus, the point of intersection are (0, 2a ) and (0, − 2a ).
2. According to given information, we have the following
Y figure.
B (0, 2a)
(0, 2b)
P
O A
X
(a2, 0)
4a
C(0, –2a) A O B
rankershalt.com

422 Parabola

Let the equation of circle be Thus, coordinate of point P are (2, − 4).
x2 + y2 + 2 gx + 2 fy + c = 0 …(i) Now , CP = 22 + (− 4 + 6)2 = 4 + 4 = 2 2
According the problem, Hence, the required equation of circle is
4a = 2 g − c2
…(ii) (x − 2)2 + ( y + 4)2 = (2 2 )2
[Q The length of intercepts made by the ⇒ x + 4 − 4x + y2 + 16 + 8 y = 8
2

⇒ x2 + y2 − 4x + 8 y + 12 = 0
circle x2 + y2 + 2 gx + 2 fy + c = 0
5. PLAN Any point on the parabola x 2 = 8 y is ( 4t ,2t 2 ). Point P divides
with X-axis is 2 g 2 − c] the line segment joining of O( 0, 0) and Q( 4t ,2t 2 ) in the ratio
1 : 3. Apply the section formula for internal division.
Also, as the circle is passing through P (0, 2b)
Equation of parabola is x2 = 8 y ...(i)
∴ 0 + 4b2 + 0 + 4bf + c = 0 [using Eq. (i)]
’ Let any point Q on the parabola (i) is (4t , 2t 2).
⇒ 4b2 + 4bf + c = 0 …(iii)
Let P (h , k) be the point which divides the line segment
Eliminating ‘c’ from Eqs. (ii) and (iii), we get joining (0, 0) and (4t , 2t 2) in the ratio 1 : 3.
4b2 + 4bf + g 2 − 4a 2 = 0 Y

[Q4a = 2 g − c ⇒ c = g − 4a ]
2 2 2
,k
)
p(h
So, locus of (− g , − f ) is 3 Q(4t, 2t 2)
1:
4b2 − 4by + x2 − 4a 2 = 0 X′ X
⇒ x2 = 4by + 4a 2 − 4b2 (0, 0) O

which is a parabola.
3. According to given information, we have the following Y′
figure. 1 × 4t + 3 × 0
Y ∴ h= ⇒ h=t
4
1 × 2t 2 + 3 × 0 t2
and k= ⇒ k=
X 4 2
(2, 0) (4, 0) 1 2
⇒ k = h ⇒ 2k = h 2
[Q t = h]
2
⇒ 2 y = x2, which is required locus.
6. By section formula,
Now, if the origin is shifted to (2, 0) and (X , Y ) are the x+0 y+0
coordinates with respect to new origin, then equation of h= ,k=
4 4
parabola is Y 2 = 4aX ,
∴ x = 4 h, y = 4 k
where, X = x − 2 and Y = y and a = 4 − 2 = 2
∴ y2 = 8(x − 2) Y (x, y)Q
Note that (8, 6) is the only point which does not satisfy 3
the equation.
P(h, k)
4. Centre of circle x2 + ( y + 6)2 = 1 is C (0, − 6). 1
X
(0, 0) O
Let the coordinates of point P be (2t 2, 4 t ).
Now, let D = CP y 2 = 4x

= (2t ) + (4 t + 6)
2 2 2
Substituting in y2 = 4 x,
⇒ D = 4 t 4 + 16 t 2 + 36 + 48 t (4 k)2 = 4 (4 h )
Squaring on both sides ⇒ k2 = h
⇒ D 2(t ) = 4 t 4 + 16 t 2 + 48 t + 36 or y = x is required locus.
2

Let F (t ) = 4 t 4 + 16 t 2 + 48 t + 36 7. Since, distance of vertex from origin is 2 and focus is


For minimum, F ′ (t ) = 0 2 2.
⇒ 16 t3 + 32t + 48 = 0 ∴ V (1, 1) and F (2, 2) (i.e. lying on y = x)
⇒ t3 + 2t + 3 = 0 where, length of latusrectum
⇒ (t + 1) (t 2 − t + 3) = 0 ⇒ t = − 1 = 4a = 4 2 [Q a = 2 ]
rankershalt.com

Parabola 423

∴ By definition of parabola 8 k
Directrix of original parabola is x =−
Y k 4
y=x 8 k
Now, x = 1 also coincides with x = −
P
k 4
F
(2, 2) On solving, we get k = 4
N
M 11. Given curves are x = t 2 + t + 1 …(i)
(1, 1) V and y=t −t+1 2
…(ii)
x+y–2=0 On subtracting Eq. (ii) from Eq. (i),
X′ X x − y = 2t
O
Thus, x = t2 + t + 1
2
 x − y  x − y
⇒ x=  +  +1
Y′  2   2 
PM 2 = (4a ) (PN ) ⇒ 4x = (x − y)2 + 2x − 2 y + 4
where, PN is length of perpendicular upon ⇒ (x − y)2 = 2 (x + y − 2)
x + y − 2 = 0, i.e. tangent at vertex ⇒ x + y − 2xy − 2x − 2 y + 4 = 0
2 2

(x − y)2  x + y − 2 Now, ∆ = 1⋅ 1⋅ 4 + 2 ⋅ (−1)(−1)(−1)


⇒ =4 2  
2  2  −1 × (−1)2 − 1 × (−1)2 − 4 (−1)2
⇒ (x − y)2 = 8 (x + y − 2) = 4 − 2 − 1 − 1 −4 = − 4
8. Let P (h , k) be the mid-point of the line segment joining ∴ ∆ ≠0
the focus (a , 0) and a general point Q (x, y) on the and ab − h 2 = 1 ⋅ 1 − (−1)2 = 1 − 1 = 0
parabola. Then, Hence, it represents a equation of parabola.
x+ a y x2 3 1
h= ,k= ⇒ x = 2h − a , y = 2k. 12. y = − + x + 1 ⇒ y − = − (x − 1 )2
2 2 2 2 2
Put these values of x and y in y2 = 4ax, we get ⇒ It is symmetric about x = 1.
4k2 = 4a (2h − a ) Hence, option (a) is correct.
⇒ 4k = 8ah − 4a 2 ⇒ k2 = 2ah − a 2
2
13. Let P (t 2 , 2t ) be a point on the curve y2 = 4x, whose
So, locus of P (h , k) is y2 = 2ax − a 2
image is Q(x , y) on x + y + 4 = 0, then
 a
⇒ y2 = 2a  x −  x − t 2 y − 2t −2(t 2 + 2t + 4)
 2 = =
1 1 12 + 12
a a
Its directrix is x − =− ⇒ x = 0. ⇒ x = − 2t − 4
2 2
9. Given, y + 4 y + 4x + 2 = 0
2 and y = − t2 − 4

⇒ ( y + 2 )2 + 4 x − 2 = 0 Y
 1
⇒ ( y + 2 )2 = − 4  x − 
 2 y 2 = 4x

1 (– 4, 0) O
Replace y + 2 =Y , x −=X X′ X
2
We have, Y 2 = − 4X
(0, – 4)
This is a parabola with directrix at X = 1
1 x+y+4=0
⇒ x− =1
2
3 Mirror image Y′
⇒ x=
2
Now, the straight line y = − 5 meets the mirror image.
10. Given, y2 = kx − 8
∴ − t2 − 4 = − 5
 8 ⇒ t2 = 1
⇒ y2 = k  x − 
 k ⇒ t=±1
Shifting the origin Y 2 = kX , where Y = y, X = x − 8 / k. Thus, points of intersection of A and B are (− 6, − 5) and
Directrix of standard parabola is X = −
k (−2, − 5).
4 ∴ Distance, AB = (−2 + 6)2 + (−5 + 5)2 = 4
rankershalt.com

424 Parabola

14. PLAN Parametric coordinates for y 2 = 4 ax are ( at 2,2 at ). Now, let (x1 , y1) be the point of intersection of tangents
of parabola (i) and line x − y = 3, then
P Equation of chord of contact of point (x1 , y1 ) w.r.t.
parabola (i) is
T =0
1
Q ⇒ ( y + y1 ) = xx1 − 2(x + x1 ) + 3
2
Description of Situation As the circle intersects the ⇒ y + y1 = 2x (x1 −2) − 4x1 + 6
parabola at P and Q. Thus, points P and Q should satisfy ⇒ 2x(x1 − 2) − y = 4 x1 + y1 − 6, this equation represent
circle. the line x − y = 3 only, so on comparing, we get
P ( 2 t 2, 4 t ) should lie on x2 + y2 − 2x − 4 y = 0 2(x1 − 2) − 1 4x1 + y1 − 6
= =
⇒ 4 t 4 + 16 t 2 − 4 t 2 − 16 t = 0 1 −1 3
5
⇒ 4 t 4 + 12 t 2 − 16 t = 0 ⇒ x1 = and y1 = − 1
2
⇒ 4 t (t3 + 3 t − 4) = 0 5 
⇒ 4t (t − 1) (t 2 + t + 4) = 0 So, the required point is  , − 1 .
2 
∴ t = 0, 1
3. Given parabola y2 = 4x …(i)
⇒ P (2, 4) and PQ is the diameter of circle.
1 1 So, equation of tangent to parabola (i) at point (1, 2) is
Thus, area of ∆PQS = ⋅ OS × PQ = ⋅ (2) ⋅ (4) = 4 2 y = 2(x + 1)
2 2
[Qequation of the tangent to the parabolay2 = 4ax at
a point (x1 , y1 ) is given by yy1 = 2a (x + x1 )]
Topic 2 Equation of Tangents and ⇒ y=x+1 …(ii)
Properties Now, equation of circle, touch the parabola at point (1, 2)
is
1. Key Idea Use the equation of tangent of slope (x − 1)2 + ( y − 2)2 + λ (x − y + 1) = 0
‘m’ to the parabola y 2 = 4 ax is y = mx +
a
and a line
⇒ x + y + (λ − 2)x + (−4 − λ ) y + (5 + λ ) = 0
2 2
…(iii)
m Also, Circle (iii) touches the x-axis, so g 2 = c
ax + by + c = 0 touches the circle  λ − 2
2
|c| ⇒   =5 + λ
x 2 + y 2 = r 2, if = r.  2 
a2 + b2
⇒ λ − 4λ + 4 = 4λ + 20
2

Since, equation of given parabola is y2 = 4 2x and ⇒ λ2 − 8λ − 16 = 0


equation of tangent line is ax + y = c or y = − ax + c, 8 ± 64 + 64
⇒ λ=
2 2 2
then c = = [Qm = slope of line = − a]
m −a ⇒ λ = 4 ± 32 = 4 ± 4 2
Now, radius of circle is r = g 2 + f 2 − c
[Q line y = mx + c touches the parabola
y2 = 4ax iff c = a / m]. ⇒ r =| f| [Q g 2 = c]
λ + 4 8 + 4 2 8 −4 2
Then, equation of tangent line becomes = = or
 2  2 2
2
y = − ax − …(i) 8 −4 2
a For least area r = = 4 − 2 2 units
2
So, area = πr 2 = π (16 + 8 − 16 2 ) = 8π (3 − 2 2 ) sq unit
Q Line (i) is also tangent to the circle x2 + y2 = 1 .
4. Given parabola is x2 = 8 y …(i)
2
− Now, slope of tangent at any point (x, y) on the parabola
a 2
∴ Radius = 1 = ⇒ 1 + a2 = − (i) is
1 + a2 a dy x
= = tan θ
2 dx 4
⇒ 1 + a2 = [squaring both sides] [Q tangent is making an angle θ with the positive
a2
direction of X-axis]
⇒ a 4 + a 2 − 2 = 0 ⇒ (a 2 + 2) (a 2 − 1) = 0
So,x = 4 tan θ
⇒ a2 = 1 [Q a 2 > 0, ∀ a ∈ R]
2 ⇒ 8 y = (4 tan θ )2
∴ | c| = = 2 [on putting x = 4 tan θ in Eq. (i)]
|a |
⇒ y = 2 tan 2 θ
2. Given equation of parabola is Now, equation of required tangent is
y = (x − 2)2 − 1 y − 2 tan 2 θ = tan θ (x − 4 tan θ )
⇒ y = x2 − 4x + 3 …(i) ⇒ y = x tan θ − 2 tan 2 θ ⇒ x = y cot θ + 2 tan θ
rankershalt.com

Parabola 425

5. We know that, equation of tangent to parabola y2 = 4ax A = (−16, 0) ; B = (24, 0)


is Q C is the centre of circle passing through PAB
a i.e. C = (4, 0)
y = mx + 16 − 0 16 4
m Slope of PC = = = = m1
16 − 4 12 3
∴Equation of tangent to the parabola y2 = 4x is
16 − 0 16
1 Slope of PB = = = − 2 = m2
y = mx + (Qa = 1) 16 − 24 −8
m
 m − m2 
⇒ m2x − my + 1 = 0 …(i) tan θ =  1 
Now, let line (i) is also a tangent to the circle. 1 + m1m2
Equation of circle x2 + y2 − 6x = 0  4 
Clearly, centre of given circle is (3, 0) and radius = 3  3+2 
⇒ tan θ =   ⇒ tan θ = 2
[Q for the circle x2 + y2 + 2 gx + 2 fy + c = 0, centre = 1 −  4 (2)
  3 
(− g , − f ) and radius = g 2 + f 2 − c]
∴ The perpendicular distance of (3, 0) from the line (i) 7. Let the radius of circle with least area be r.
is 3. Then, then coordinates of centre = (0, 4 − r ).
[Q Radius is perpendicular to the
tangent of circle]
|m ⋅ 3 − m ⋅ 0 + 1|
2
⇒ =3
(m2)2 + (−m)2
The length of perpendicular from a point (x1 , y1 ) to the
ax1 + by1 + c
line ax + by + c = 0 is .
a 2 + b2
3m2 + 1
⇒ =3 Since, circle touches the line y = x in first quadrant
m4 + m2 0 − (4 − r )
∴ = r ⇒ r −4 = ± r 2
⇒ 9m4 + 6m2 + 1 = 9(m4 + m2) 2
1 4 4
⇒ m ≈ ∞ or m = ± ⇒ r= or
3 2+1 1− 2
 1 
3+ 2 4  4 
 3m2 + 1  But r ≠ Q <0
Q lim = lim m = 3 1− 2  1 − 2 
 → ∞ → ∞ 
m
m +m
4 2 m
1+ 2
1 4
 m  ∴ r= = 4 ( 2 − 1)
2 +1
∴ Equation of common tangents are x = 0, 8. Let the tangent to parabola be y = mx + a / m, if it
x −x  1
y= + 3 and y = − 3  using y = mx +  touches the other curve, then D = 0, to get the value of m.
3 3  m
For parabola, y2 = 4x
i.e. x = 0, 3 y = x + 3 and 3 y = − x − 3
1
6. Equation of tangent and normal to the curve y2 = 16x Let y = mx + be tangent line and it touches the
m
at (16, 16) is x − 2 y + 16 = 0 and 2x + y − 48 = 0, parabola x2 = –32 y
respectively.
 1
∴ x2 = –32mx + 
Y  m
32
P
(16, 16)
⇒ x2 + 32 mx + =0
m
=0 θ 2x+
16 y– D =0
y+ 4
x–2 8  32
=
0 Q (32 m) – 4 ⋅   = 0 ⇒
2
m3 = 1 / 8
X′ X  m
A(–16, 0) C(4, 0) B(24, 0)
∴ m = 1 /2
9. The tangent at (1, 7) to the parabola x2 = y – 6x is
1
x (1) = ( y + 7) – 6
2
Y′
[replacing x2 → xx1 and 2 y → y + y1]
rankershalt.com

426 Parabola

⇒ 2x = y + 7 – 12  2 2
⇒ x mx +  = − 1 ⇒ mx2 + x+ 1 =0
⇒ y = 2x + 5 ... (i)  m m
which is also tangent to the circle Since, it has equal roots.
x + y + 16 x + 12 y + c = 0
2 2 ∴ D =0
4
i.e. x2 + (2x + 5)2 + 16x + 12(2x + 5) + C = 0 must have ⇒ − 4m = 0
equal rools i.e., α = β m2
⇒ 5x2 + 60x + 85 + c = 0 ⇒ m3 = 1
– 60 ⇒ m=1
⇒ α +β =
5 Hence, equation of common tangent is y = x + 2.
⇒ α = –6
∴ x = –6 and y = 2x + 5 = –7 1
13. Any tangent to y2 = 4x is of the form y = mx + ,
∴ Point of contact is (– 6, –7). m
a (Q a = 1) and this touches the circle (x − 3)2 + y2 = 9.
10. We know, tangent to y2 = 4a x is y = mx + . 1
m m (3) + − 0
1 If m =3
∴ Tangent to y2 = 4x is y = mx + m2 + 1
m
Since, tangent passes through (1, 4).
[Q centre of the circle is (3, 0) and radius is 3].
1
∴ 4 =m+ 3m2 + 1
m ⇒ = ± 3 m2 + 1
m
⇒ m2 – 4m + 1 = 0 (whose roots are m1 and m2)
⇒ 3m2 + 1 = ± 3m m2 + 1
∴ m1 + m2 = 4 and m1m2 = 1
⇒ 9m4 + 1 + 6m2 = 9m2 (m2 + 1)
and m1 – m2 = (m1 + m2)2 – 4m1m2
⇒ 9m4 + 1 + 6m2 = 9m4 + 9m2
= 12 = 2 3 ⇒ 3m2 = 1
Thus, angle between tangents 1
⇒ m =±
m – m1 2 3 π 3
tan θ = 2 = = 3 ⇒ θ=
1 + m1m2 1 + 1 3 If the tangent touches the parabola and circle above
the X-axis, then slope m should be positive.
11. Here, the focal chord of y2 = 16 x is tangent to circle 1 1
(x − 6)2 + y2 = 2 . ∴ m= and the equation is y = x+ 3
3 3
⇒ Focus of parabola as (a, 0) i.e. (4, 0) or 3 y = x + 3.
5
14. Equation of circle can be rewritten as x2 + y2 = .
Now, tangents are drawn from (4, 0) to (x − 6)2 + y2 = 2. 2
Since, PA is tangent to circle. 5
Centre → (0, 0) and radius →
AC 2 BC 2
∴ tan θ = slope of tangent = = = 1, or = −1
AP 2 BP Let common tangent be
2
y =16x 5
Y y = mx + ⇒ m2x – my + 5 = 0
m
Tangent
as focal chord The perpendicular from centre to the tangent is equal
A to radius of the circle.
√2 5 /m 5
P θ C(6,0) ∴ =
X′
θ
X 1+m 2 2
(4,0)
2 ⇒ m 1 + m2 = 2
B
⇒ m2(1 + m2) = 2
⇒ m4 + m2 – 2 = 0
Y′ ⇒ (m2 + 2)(m2 – 1) = 0
∴ Slope of focal chord as tangent to circle = ± 1 ⇒ m = ±1 [Q m2 + 2 ≠ 0, as m ∈ R ]
2
12. Tangent to the curve y2 = 8x is y = mx + . So, it must ∴ y = ± (x + 5 ), both statements are correct as m ± 1
m satisfies the given equation of Statement II.
satisfy xy = − 1
rankershalt.com

Parabola 427

m2 ∴ − 2 ( x + 1) = 2 ( x + 1)
15. The equation of tangent to y = x2, be y = mx − .
4 ⇒ 0 = 4 ( x + 1)
Putting in y = − x2 + 4x − 4,we should only get one value ⇒ −1= x ⇒ y= 0
of x i.e. Discriminant must be zero. Therefore, the required point is (− 1, 0).
m2
∴ mx − = − x2 + 4x − 4 19. Given equation can be rewritten as
4
( y − 1)2 = 4 (x − 1), whose parametric coordinates are
m2
⇒ x + x (m − 4) + 4 −
2
=0 x − 1 = t 2 and y − 1 = 2t
4
i.e. P (1 + t 2, 1 + 2t )
D =0 ∴ Equation of tangent at P is,
Now, (m − 4) − (16 − m2) = 0
2
t ( y − 1) = x − 1 + t 2, which meets the directrix x = 0 at Q.
⇒ 2m (m − 4) = 0 ⇒ m = 0, 4 1  1
⇒ y=1 + t − or Q 0, 1 + t − 
∴ y = 0 and y = 4 (x − 1) are the required tangents. t  t
Hence, (a) and (b) are correct answers. Let R (h , k) which divides QP externally in the ratio
1
16. PLAN 2
(i) If P( at , 2 at ) is one end point of focal chord of parabola : 1 or Q is mid-point of RP.
2
y 2 = 4ax , then other end point is  2 ,−  .
a 2a
t t  h + t2 + 1
⇒ 0= or t 2 = − (h + 1) …(i)
(ii) Slope of line joining two points ( x 1, y1 ) and ( x 2, y2 ) is given by 2
y2 − y1 1 k + 2t + 1 2
. and 1+ t− = or t = …(ii)
x 2 − x1
t 2 1−k
If PQ is focal chord, then coordinates of Q will be 4
From Eqs. (i) and (ii), + (h + 1) = 0
a 2a  (1 − k)2
 2 , − .
t t 
or (k − 1)2 (h + 1) + 4 = 0
Now, slope of QR = slope of PK ∴ Locus of a point is (x + 1) ( y − 1)2 + 4 = 0
2a
2ar + 20. Let the point Q ( x , x 2 ) on x 2 = y whose distance from
t = 2at ⇒ r + 1 /t t
= 2
a at 2
− 2 a r 2
− 1 / t 2
t −2 ( 0, c) is minimum.
ar − 2
2
t Now, PQ 2 = x 2 + ( x 2 − c)2
1 t 1 t2 − 2 2 Let f ( x ) = x 2 + ( x 2 − c)2 ... (i)
⇒ = 2 ⇒ r− = =t−
1 t −2 t t t f ′ ( x ) = 2x + 2 ( x 2 − c) ⋅ 2x
r−
t  1
= 2x (1 + 2x2 − 2c) = 4x  x2 − c + 
1 t2 − 1  2
⇒ r =t− = Y
t t
17. PLAN Equation of tangent and normal at ( at 2, 2 at ) are given by x2 = y
ty = x + at 2 and y + tx = 2 at + at 3, respectively. (0,c)P
x
Tangent at P : ty = x + at 2 or y = + at Q(x,x 2)
t
x 2a a X′ X
Normal at S : y + = + 3 O
t t t
2a a a (t 2 + 1)2 Y′
Solving, 2 y = at + + 3 ⇒ y=
t t 2t3  1   1 1
= 4x  x − c −   x + c − , when c >
18. The coordinates of extremities of the latusrectum of  2   2 2
y2 = 4x are (1, 2 ) and (1, − 2).
For maxima, put f ′ (x) = 0
Equations of tangents at these points are
 1 1
4(x + 1) 4x  x2 − c +  = 0 ⇒ x = 0, x = ± c −
y⋅2 = ⇒ 2 y = 2(x + 1) …(i)  2 2
2  2 1
4 (x + 1) Now, f ′ ′ (x) = 4 x − c + + 4x [2x]
and y (− 2) =  2 
2
1
⇒ − 2 y = 2(x + 1) …(ii) At x = ± c−
2
The point of intersection of these tangents can be
f ′ ′ (x) ≥ 0.
obtained by solving Eqs. (i) and (ii) simultaneously.
rankershalt.com

428 Parabola
∴ f (x) is minimum. 2. If y = mx + c is normal to the parabola y2 = 4ax, then
Hence, minimum value of f (x) =|PQ| c = − 2am − am3 .
2  2 
2 From given condition, y 2 = 12x
 1  1
=  c −  +   c −  − c ⇒ y2 = 4 ⋅ 3 ⋅ x
 2  2 
  ⇒ a =3
1  1 
2
1 1 And x+ y=k
= c− +  c − − c = c − , ≤ c≤5
2  2  4 2 ⇒ y = (−1) x + k
⇒ m = −1
21. As, we know area of ∆ formed by three points on
and c=k
parabola is twice the area of ∆ formed by
corresponding tangents i.e. area of ∆ PQR = 2 area of ∴ c = k = − 2 (3) (−1) − 3 (−1)3 = 9
∆ T1T2T3 . 3. Since, equation of normal to the parabola y2 = 4ax is
∆1 y + xt = 2at + at3 passes through (3, 0).
∴ ∆1 = 2∆ 2 or =2
∆2 ⇒ 3 t = 2 t + t3 [Q a = 1]
Y ⇒ t = 0, 1, − 1
∴ Coordinates of the normals areP (1, 2), Q (0, 0), R(1, − 2).
Thus,
T1
P 1
T2 A. Area of ∆PQR = ×1 ×4 = 2
2
Q
2 
X C. Centroid of ∆PQR =  , 0
3 

T3 Equation of circle passing through P , Q , R is


R
(x − 1)(x − 1) + ( y − 2)( y + 2) + λ (x − 1) = 0
⇒ 1 −4 − λ = 0
⇒ λ = −3
Topic 3 Equation of Normal and Properties
∴ Required equation of circle is
1. Normal to parabola y2 = 4ax is given by
x2 + y2 − 5x = 0
y = mx − 2am − am3 5  5
∴ Centre  , 0 and radius .
∴ Normal to parabola 2  2
y2 = 4b(x − c) is
4. Tangent to y 2 = 4x at (t 2, 2 t) is
y = m(x − c) − 2bm − bm3
Y
[replacing a by b and x by x − c] S(2, 8)
= mx − (2b + c)m − bm3 … (i) Q
P (t2, 2t)
and normal to parabola y2 = 8 ax is
y = mx − 4am − 2am3 …(ii) O
X
[replacing a by 2a]
y2=4x
For common normal, we should have
mx − 4am − 2am3 = mx − (2b + c)m − bm3 y(2 t ) = 2(x + t 2)
[using Eqs. (i) and (ii)] ⇒ yt = x + t 2 …(i)
4am + 2am3 = (2b + c)m + bm3 Equation of normal at P (t 2, 2 t) is
⇒ (2a − b)m3 + (4a − 2b − c)m = 0 y + tx = 2t + t3
⇒ m((2a − b)m2 + (4a − 2b − c)) = 0 Since, normal at P passes through centre of circle S(2, 8).
⇒ m =0 ∴ 8 + 2 t = 2 t + t3
2b + c − 4a c ⇒ t = 2, i.e. P(4, 4)
or m2 = = −2
2a − b 2a − b [since, shortest distance between two curves lie along
c their common normal and the common normal will pass
As, m2 > 0, therefore >2
2a − b through the centre of circle]
Note that if m = 0, then all options satisfy
(Q y = 0 is a common normal) and if common normal is ∴ SP = (4 − 2)2 + (4 − 8)2 = 2 5
other than the axis, then only option (c) satisfies
3 ∴Option (a) is correct.
[Q for option (c), 2a − b = = 3 > 2]
2 −1
rankershalt.com

Parabola 429

Also, SQ = 2 | y| y
y = (x + 2)2 and log   + =0
∴ PQ = SP − SQ = 2 5 − 2  3e  x + 2
SQ 1 5+1 |x + 2|2  (x + 2 )2
= = ⇒ log  + x+2 =0
Thus,
QP 5 −1 4
 3e 
∴Option (b) is wrong. |x + 2|2 
Now, x-intercept of normal is x = 2 + 22 = 6 ⇒ log   = − (x + 2)
 3e 
∴Option (c) is correct.
(x + 2)2
1 1
Slope of tangent = = ⇒ = e−( x + 2) or (x + 2)2 ⋅ ex + 2 = 3e
t 2 3e
3e
∴Option (d) is correct. ⇒ ex+ 2 =
(x + 2)2
dy
5. Given, ( x 2 + xy + 4x + 2 y + 4) − y2 = 0 Y
dx e x+ 2
dy
⇒ [(x + 4x + 4) + y(x + 2)]
2
− y2 = 0
dx e2
dy
⇒ [(x + 2)2 + y(x + 2)] − y2 = 0 3e /4
dx 3e /( x + 2)2
Put x + 2 = X and y = Y , then O
X
dY
(X 2 + XY ) −Y 2 = 0 Clearly, they have no solution.
dX
⇒ X 2dY + XYdY − Y 2dX = 0 To check option (d), y = (x + 3)2
⇒ X 2dY + Y (XdY − YdX ) = 0 |x + 3|2  (x + 3)2
i.e. log  + =0
⇒ −
dY XdY − YdX
=  3e  (x + 2)
Y X2 To check the number of solutions.
Y  (x + 3)2
⇒ − d (log|Y |) = d   Let g (x) = 2 log (x + 3) + − log (3e)
 X
(x + 2)
On integrating both sides, we get  (x + 2) ⋅ 2 (x + 3) − (x + 3)2 ⋅ 1
2
Y ∴ g′ (x) = +  −0
− log|Y| = + C, where x + 2 = X x+ 3  (x + 2)2 
X
and y = Y 2 (x + 3)(x + 1)
= +
y x+3 (x + 2)2
⇒ − log| y| = +C …(i)
x+ 2 Clearly, when x > 0, then, g′ (x) > 0
Since, it passes through the point (1, 3).
∴ − log 3 = 1 + C ∴ g (x) is increasing, when x > 0.
⇒ C = − 1 − log 3 = − (log e + log 3) = − log 3e Thus, when x > 0, then g (x) > g (0)
∴ Eq. (i) becomes
 3 9
log| y| +
y
− log (3e) = 0 g (x) > log   + > 0
 e 4
x+2
| y| y Hence, there is no solution.
⇒ log   + =0 …(ii)
 3e  x + 2 Thus, option (d) is true.
Now, to check option (a), y = x + 2 intersects the curve. 6. Normal to y 2 = 4x , is
|x + 2| x + 2 y = mx − 2m − m3 which passes through (9, 6).
⇒ log  + =0
 3e  x + 2 ⇒ 6 = 9m − 2m − m3
|x + 2| ⇒ m3 − 7m + 6 = 0
⇒ log   = −1
 3e  ⇒ m = 1, 2, − 3
|x + 2| 1 ∴ Equation of normals are,
⇒ = e−1 =
3e e y − x + 3 = 0, y + 3x − 33 = 0 and y − 2x + 12 = 0
⇒ |x + 2| = 3 or x + 2 = ± 3 7. Equation of tangent and normal at point P (at 2, 2at ) is
∴ x = 1, − 5 (rejected), as x > 0 [given]
ty = x = at 2 and y = − tx + 2at + at 2
∴ x = 1 only one solution.
Let centroid of ∆ PTN is R (h , k).
Thus, (a) is the correct answer.
at 2 + (− at 2) + 2a + at 2
To check option (c), we have ∴ h=
3
rankershalt.com

430 Parabola

Y 2 9. We know equation of normal to y2 = 4ax is


(at , 2at)
P y = mx − 2am − am3
2
(− at , 0) N (2a + at ,0)
2 Thus, equation of normal to y2 = 4x is,
X
T y = – tx + 2at + at
3
y = mx − 2m − m3 , let it passes through (h , k) .
2
ty = x + at ⇒ k = mh − 2m − m3
or m3 + m (2 − h ) + k = 0 …(i)
2at Here, m1 + m2 + m3 = 0,
and k= m1m2 + m2m3 + m3m1 = 2 − h ,
3
2 m1m2 m3 = − k where m1m2 = α
 3k 
⇒ 3h = 2a + a ⋅   k
 2a  ⇒ m3 = − , it must satisfy Eq. (i)
α
9k2
⇒ 3h = 2a + k3 k
4a ⇒ − 3 − (2 − h ) + k = 0
α α
⇒ 9k = 4a (3h − 2a )
2
⇒ k2 = α 2h − 2α 2 + α 3
∴ Locus of centroid is
⇒ y2 = α 2x − 2α 2 + α 3
4a  2a 
y2 = x −  On comparing with y2 = 4x
3  3
⇒ α2 = 4
 2a 
∴ Vertex  , 0 ; directrix and − 2α + α 3 = 0
2
 3 
⇒ α =2
2a a
x−=− 10. We know that, normal for y2 = 4ax is given by,
3 3
a y = mx − 2am − am3 .
⇒ x= ∴ Equation of normal for y2 = x is
3
4a m m3  1
and latusrectum = y = mx − − Qa =
2 4  4 
3
 a 2a  Since, normal passes through (c, 0).
∴ Focus  + , 0 , i.e. (a , 0).
3 3  m m3
∴ mc − − =0
8. End points of latusrectum are (a , ± 2a) i. e. (1, ± 2). 2 4
 1 m2
Equation of normal at (x1 , y1) is ⇒ m c − −  =0⇒ m =0
y − y1 y  2 4
=− 1
x − x1 2a  1
or m2 = 4  c − 
y−2 2 y+2 2  2
i. e. = − and =
x−1 2 x−1 2 ⇒ m = 0, the equation of normal is y = 0
⇒ x+ y=3 Also, m2 ≥ 0
and x− y=3 ⇒ c − 1 /2 ≥ 0 ⇒ c ≥ 1 /2
which is tangent to (x − 3)2 + ( y + 2)2 = r 2 At c = 1 /2 ⇒ m =0
Now, for other normals to be perpendicular to each
Y
other, we must have m1 ⋅ m2 = − 1
L(1, 2)
m2  1 
or +  − c = 0, has m1 m2 = − 1
4 2 
Normal
1 
X′
F
X  − c
O (1, 0) 2  1 1
Normal
⇒ = −1 ⇒ −c=−
1 /4 2 4
3
⇒ c=
L′(1, –2) 4
Y′ 11. Equation of normal to x 2 = 4 y is x = my − 2 m − m3
∴ Length of perpendicular from centre = Radius and passing through (1, 2).
|3 − 2 − 3| ∴ 1 = 2 m − 2 m − m3
⇒ =r ⇒ m = − 1 or
3
m = −1
12 + 12
Thus, the required equation of normal is ,
∴ r2 = 2 x = − y + 2 + 1 or x + y = 3 is required equation.
rankershalt.com

Parabola 431

12. If three different normals are drawn from (h , 0) to Again from Eqs. (i) and (ii), we have
y = 4x.
2  x 
x2 = 4 +4
Then, equation of normals are y = mx − 2m − m3  2 
which passes through (h , 0). ⇒ x2 − 2 2x − 16 = 0 … (v)
⇒ mh − 2m − m3 = 0 ⇒ h = 2 + m2 Let the roots of Eq. (v) be x1 and x2
where, 2 + m2 ≥ 2 Then, x1 + x2 = 2 2
∴ h > 2 [neglect equality as if 2 + m2 = 2 ⇒ m = 0] and x1x2 = −16 … (vi)
Therefore, three normals are coincident. Clearly, length of the chord AB
∴ h >2 = (x1 − x2)2 + ( y1 − y2)2

Topic 4 Diameter, Chord of Contact, Chord = (x1 + x2)2 − 4x1x2 + ( y1 + y2)2 − 4 y1 y2


Bisected and Product of Pair of [Q (a − b)2 = (a + b)2 − 4ab]
Tangents = 8 + 64 + 100 − 64
= 108 = 6 3 [from Eqs. (iv) and (vi)]
1. Key Idea (i) First find the focus of the given parabola
3. Equation of chord with mid-point (h , k).
y2 − y1
(ii) Then, find the slope of the focal chord by using m =
x2 − x1 T = S1
(iii) Now, find the length of the focal chord by using the yk − 8x − 8h = k2 − 16h
formula 4a cosec 2α. yk k2
2x − = 2h −
4 4
Equation of given parabola is y2 = 16x, its focus is (4, 0). Q 2x + y = p
Since, slope of line passing through (x1 , y1 ) and (x2, y2) ∴ k = − 4 and p = 2h − 4
y −y
is given by m = tan θ = 2 1 . where h =3
x2 − x1
p = 2 ×3 −4 = 2
∴ Slope of focal chord having one end point is (1, 4) is
 t12 + t22 
4. Here, coordinate M =  , t1 + t2 i.e. mid-point of
4 −0 4  2 
m = tan α = =− chord AB.
1 −4 3 2
(t2 , 2t2)
Y
[where, ‘α’ is the inclination of focal chord with X-axis.] B
Since, the length of focal chord = 4a cos ec2α 2
(t , 2t1)
M
1 r
∴ The required length of the focal chord A
X
= 16 [1 + cot α ] [Q a = 4 and cosec α = 1 + cot α ]
2 2 2 A′ P
r
 9  1 3
= 16 1 + = 25 units Q cot α = =−
 
16  
 tan α 4 
M′

B′
2. Given, equation of parabola is x2 = 4 y … (i)
MP = t1 + t2 = r ...(i)
and the chord is x − 2 y + 4 2 = 0 … (ii) 2 t2 − 2 t1 2
Also, mAB = 2 2 = [when AB is chord]
From Eqs. (i) and (ii), we have t2 − t1 t2 + t1
[ 2 ( y − 4)]2 = 4 y 2
⇒ 2 ( y − 4 )2 = 4 y ⇒ mAB = [from Eq. (i)]
r
⇒ ( y − 4 )2 = 2 y 2
Also, mA′ B ′ = − [when A′ B ′ is chord]
⇒ y2 − 8 y + 16 = 2 y r
⇒ y2 − 10 y +16 = 0 …(iii) Hence, (c, d) are the correct options.
  1 
Y 5. Since, R  – a , a  t −   lies on y = 2x + a.
  t
A x– √ 2 y+4 √ 2 =0
(x1 , y1 )
B (x2 , y2 ) P (at2, 2at)
X 1
O R – a, a t – —
t O

Let the roots of Eq. (iii) be y1 and y2


Then, y1 + y2 = 10 and y1 y2 = 16 … (iv) a , – 2a y2 = 4ax
t2 t
rankershalt.com

432 Parabola

 1 1 8. Equation of any tangent to the parabola, y2 = 4ax is


⇒ a ⋅  t −  = − 2a + a ⇒ t − = − 1
 t  t a
y = mx + .
Thus, length of focal chord m
 1
2  1
2  a2
= a  t +  = a  t −  + 4 = 5a This line will touch the circle x2 + y2 =
 t  t 2
  Y
2at − 02 E
6. mOP = 2 =
at − 0 t
Y x = − a/2 B
x=a
P (at2, 2at)
X' X
A(−a,0) K O L
q
X
O
C
Q a , –2a
t2 t D
Y'
−2a/t
mOQ = = −2 t  a
2
a2 2
a /t 2 If   = (m + 1)
 m 2
2  1
+ 2t 2 t +  1 1
t  t – 2 5 ⇒ = (m2 + 1)
∴ tan θ = = = m2 2
2
1 − ⋅2 t 1 − 4 3
⇒ 2 = m4 + m2
t
1 ⇒ m + m −2 =0
4 2

where t+ = 5 ⇒ (m2 − 1) (m2 + 2) = 0


t
⇒ m2 − 1 = 0, m2 = − 2
7. Let P (α,β) be any point on the locus. Equation of pair of
⇒ m=±1 [m2 = −2 is not possible]
tangents from P (α , β ) to the parabola y2 = 4ax is
Therefore, two common tangents are
[ βy − 2a (x + α )]2 = (β 2 − 4aα ) ( y2 − 4a x)
y = x + a and y = − x − a
[Q T 2 = S ⋅ S1 ] These two intersect at A (− a , 0).
⇒ β 2y2 + 4a 2 (x2 + α 2 + 2x ⋅ α ) −4a βy (x + α ) The chord of contact of A (− a , 0) for the circle
= β 2y2 − 4 β 2ax − 4aαy2 + 16a 2α x x2 + y2 = a 2 / 2 is (− a ) x + 0 ⋅ y = a 2/2
⇒ β 2y2 + 4a 2x2 + 4a 2 α 2 + 8x α a 2 ⇒ x = − a /2
= β 2y2 − 4β 2ax − 4aα y2 + 16a 2α x − 4a βxy − 4a βαy…(i) and chord of contact of A (− a , 0) for the parabola
Now, coefficient of x2 = 4a 2 y2 = 4ax is 0 ⋅ y = 2a (x − a ) ⇒ x = a
coefficient of xy = − 4a β Again, length of BC = 2 BK
coefficient of y2 = 4aα
= 2 OB2 − OK 2
Again, angle between the two of Eq. (i) is given as 45°
a2 a2 a2
2 h 2 − ab =2 − =2 =a
∴ tan 45° = 2 4 4
a+b
and we know that, DE is the latusrectum of the
2 h 2 − ab parabola, so its length is 4a.
⇒ 1=
a+ b Thus, area of the quadrilateral BCDE
⇒ a + b = 2 h 2 − ab 1
= (BC + DE ) (KL )
2
⇒ (a + b)2 = 4 (h 2 − ab) 2
1  3a  15a
⇒ (4a + 4aα )2 = 4 [4a 2 β 2 − (4a 2) (4aα )]
2 = (a + 4a )   =
2  2  4
⇒ 16a 2 (a + α )2 = 4 ⋅ 4a 2 [ β 2 − 4aα ]
⇒ α + 6aα + a 2 − β 2 = 0
2 9. Let the three points on the parabola be
⇒ (α + 3a )2 − β 2 = 8a 2 A ( at12 , 2at1 ), B( at22 , 2at2 ) and C( at32 , 2at3 ).
Thus, the required equation of the locus is Equation of the tangent to the parabola at (at 2, 2at ) is
(x + 3a )2 − y2 = 8a 2 which is a hyperbola. ty = x + at 2
rankershalt.com

Parabola 433

Therefore, equations of tangents at A and B are 10. Let A (t12, 2t1 ) and B(t22, 2t2) be coordinates of the end
t1 y = x + at12 …(i) points of a chord of the parabola y2 = 4x having slope 2.
and t 2y = x + at22 …(ii) Now, slope of AB is
From Eqs. (i) and (ii) 2t − 2t1 2(t 2 − t1 ) 2
m = 22 = =
t1 y = t2y − at22 + at12 t 2 − t12 (t 2 − t1 )(t2 + t1 ) t2 + t1
⇒ t1 y − t2y = at12 − at22 Y
⇒ y = a (t1 + t2) [Q t1 ≠ t2] 2 2t 1)
,
A (t 1
and t1 a (t1 + t2) = x + at12 [from Eq. (i)]
⇒ x = at1t2 1
Therefore, coordinates of P are (at1t2, a (t1 + t2)). P (h, k)
X′ X
Similarly, the coordinates of Q and R are respectively, O
2
[at2 t3 , a (t2 + t3 )] and [at1t3 , a (t1 + t3 )].
Let ∆1 = Area of the ∆ABC B( 2
t2 ,
2 2t
at1 2at1 1 2)

1 Y′
= at22 2at2 1
2
at32 2at3 1
But m =2 [given]
Applying R3 → R3 − R2 and R2 → R2 − R1 , we get ⇒ 2=
2
at12 2at1 1 t2 + t1
1 ⇒ t1 + t2 = 1
∆1 = a (t 22 − t12) 2a (t2 − t1 ) 0 …(i)
2
a (t32 − t22) 2a (t3 − t2) 0 Let P (h , k) be a point on AB such that, it divides AB
internally in the ratio 1 : 2.
1 a (t22 − t12) 2a (t2 − t1 )
= 2t12 + t22 2(2t1 ) + 2t2
2 a (t32 − t22) 2a (t3 − t2) Then, h= and k =
2+1 2+1
1 (t 2 − t1 ) (t2 + t1 ) (t2 − t1 )
= . a . 2a ⇒ 3h = 2t12 + t22 …(ii)
2 (t3 − t2) (t3 + t2) (t3 − t2)
and 3k = 4t1 + 2t2 …(iii)
t2 + t1 1
= a 2 (t2 − t1 ) (t3 − t2)
t3 + t2 1
On substituting value of t1 from Eq. (i) in Eq. (iii)
= a 2|(t2 − t1 ) (t3 − t2) (t1 − t3 )|
3k = 4 (1 − t2) + 2t2
Again, let ∆ 2 = area of the ∆PQR
⇒ 3k = 4 − 2t2
at1t2 a (t1 + t2) 1 3k
=
1
at2t3 a (t2 + t3 ) 1 ⇒ t2 = 2 − …(iv)
2 2
at3 t1 a (t3 + t1 ) 1 On substituting t1 = 1 − t2 in Eq. (ii), we get
t1t2 (t1 + t2) 1 3h = 2 (1 − t2 )2 + t22
1
= a⋅a t2t3 (t2 + t3 ) 1 = 2 (1 − 2t 2 + t22) + t22
2
t3 t1 (t3 + t1 ) 1  4 2
= 3t 22 − 4t2 + 2 = 3  t22 − t2 + 
 3 3
Applying R3 → R3 − R2, R2 → R2 − R1 , we get
t1t2 t1 + t2 1  2
2
2 4  2
2
2
a2 = 3  t2 −  + −  = 3  t2 −  +
= t2 (t3 − t1 ) t3 − t1 0  3  3 9  3  3
 
2
t3 (t1 − t2) t1 − t2 0 2  2
2
⇒ 3h − = 3  t2 − 
t1t2 t1 + t2 1 3  3
a2
= (t3 − t1 ) (t1 − t2) t2 1 0  2  3k 2
2
2 ⇒ 3  h −  = 3 2 − −  [from Eq. (iv)]
t3 1 0  9   2 3
a2 t 1
 2  4 3k
2
= (t3 − t1 ) (t1 − t2) 2 ⇒ 3 h −  =3  − 
2 t3 1  9 3 2 
a2  2 9  8
2
= |(t3 − t1 ) (t1 − t2) (t2 − t3 )| ⇒ h −  = k − 
2  9 4  9
∆1 a 2|(t2 − t1 ) (t3 − t2) (t1 − t3 )| 2
Therefore, = =2  8 4 2
∆ 2 1 a 2|(t − t ) (t − t ) (t − t )| ⇒ k −  = h − 
3 1 1 2 2 3  9 9 9
2
rankershalt.com

434 Parabola

On generalising, we get the required locus m


⇒ y + 4m = (x − 4 m2)
 8
2
4 2 1 − m2
 y −  = x − 
 9 9 9 ⇒ (1 − m2) y + 4 m − 4m3 = mx − 4m3
2 8 ⇒ mx − (1 − m2) y − 4 m = 0
This represents a parabola with vertex at  , .
9 9
11. Let the equation of chord OP be y = mx. This line meets X-axis, where y = 0
1 i.e. x = 4 ⇒ OL = 4 which is constant as independent of
Then, equation of chord will be y = − x and m.
m
Again, let (h , k) be the mid-point of PQ. Then,
P is point of intersection of y = mx and y2 = 4x is
4
 4 4 1 4m2 + 2
 2 ,  and Q is point intersection of y = − x and m
 m m m h=
2
y2 = 4x is (4m2, − 4m). 4
− 4m
Now, equation of PQ is and k= m
4 2
+ 4m
m  2 1
y + 4m = (x − 4 m2) ⇒ h = 2  m + 2
4  m 
− 4m 2
m2 1 
and k = 2  − m
Y m 
 1
2 
P ⇒ h = 2 m −  + 2
 m 
 
 1 
X′ X and k = 2  − m
O L m 
Q Eliminating m, we get
y 2 = 4x
2h = k2 + 8
Y′ or y2 = 2 (x − 4) is required equation of locus.

Download Chapter Test


http://tinyurl.com/y2us2kda or
rankershalt.com

18
Ellipse

Topic 1 Equation of Ellipse and Focal Chord


Objective Questions I (Only one correct option) 7. If P = (x, y), F1 = (3, 0), F2 = (− 3, 0)
1. An ellipse, with foci at (0, 2) and (0, − 2) and minor axis and 16x2 + 25 y2 = 400, then PF1 + PF2 equals (1998, 2M)
of length 4, passes through which of the following (a) 8 (b) 6 (c) 10 (d) 12
points? (2019 Main, 12 April II)
(a) ( 2 , 2) (b) (2, 2 ) Objective Questions II (Only one or More than one)
(c) (2, 2 2 ) (d) (1, 2 2 ) 8. Consider two straight lines, each of which is tangent to
2. In an ellipse, with centre at the origin, if the difference both the circle x2 + y2 = (1 /2) and the parabola y2 = 4x.
of the lengths of major axis and minor axis is 10 and one Let these lines intersect at the point Q. Consider the
of the foci is at (0, 5 3 ), then the length of its latus ellipse whose centre is at the origin O(0, 0) and whose
rectum is (2019 Main, 8 April I) semi-major axis is OQ. If the length of the minor axis of
(a) 5 (b) 10 (c) 8 (d) 6
this ellipse is 2, then which of the following
statement(s) is (are) TRUE? (2018 Adv.)
3. Let S and S′ be the foci of an ellipse and B be any one of (a) For the ellipse, the eccentricity is 1 / 2 and the length
the extremities of its minor axis. If ∆S′ BS is a right of the latus rectum is 1
angled triangle with right angle at B and area
(b) For the ellipse, the eccentricity is 1/2 and the length of
(∆S′ BS ) = 8 sq units, then the length of a latus rectum
of the ellipse is (2019 Main, 12 Jan II)
the latus rectum is 1/2
(a) 2 2 (b) 4 2 (c) The area of the region bounded by the ellipse between
1 1
(c) 2 (d) 4 the lines x = and x =1is ( π − 2)
2 4 2
4. Let the length of the latus rectum of an ellipse with its
(d) The area of the region bounded by the ellipse between
major axis along X-axis and centre at the origin, be 8. If 1 1
the distance between the foci of this ellipse is equal to the lines x = and x =1is ( π − 2)
2 16
the length of its minor axis, then which one of the
following points lies on it? (2019 Main, 11 Jan II)
(a) (4 2 , 2 3 ) (b) (4 3 , 2 2 ) Fill in the Blanks
(c) (4 2 , 2 2 ) (d) (4 3 , 2 3 ) 9. An ellipse has OB as a semi-minor axis. F and F ′ are its
5. The equation of the circle passing through the foci of foci and the angle FBF′ is a right angle. Then, the
x2 y2 eccentricity of the ellipse is …… . (1997, 2M)
the ellipse + = 1 and having centre at (0, 3) is
16 9 1
(2013 Main) 10. An ellipse has eccentricity and one focus at the point
2
(a) x2 + y 2 − 6 y − 7 = 0 (b) x2 + y 2 − 6 y + 7 = 0 1 
(c) x2 + y 2 − 6 y − 5 = 0 (d) x2 + y 2 − 6 y + 5 = 0 P  , 1 . Its one directrix is the common tangent,
2 
x2 y2 nearer to the point P, to the circle x2 + y2 = 1 and the
6. The ellipse E1 : + = 1 is inscribed in a rectangle R
9 4 hyperbola x2 − y2 = 1. The equation of the ellipse, in the
whose sides are parallel to the coordinate axes. standard form is…… . (1996, 2M)
Another ellipse E 2 passing through the point (0, 4)
circumscribes the rectangle R. The eccentricity of the x2 y2
11. Let P be a variable point on the ellipse + = 1 with
ellipse E 2 is (2012) a 2 b2
2 3 1 3 foci F1 and F2. If A is the area of the ∆ PF1F2, then the
(a) (b) (c) (d) maximum value of A is… . (1994, 2M)
2 2 2 4
rankershalt.com

436 Ellipse

Analytical & Descriptive Question at the origin and focus at F2 intersects the ellipse at
2 2
point M in the first quadrant and at point N in the
x y fourth quadrant.
12. Let P be a point on the ellipse 2
+ 2 = 1 , 0 < b < a. Let
a b (2016 Adv.)
the line parallel to Y-axis passing through P meet the 13. The orthocentre of ∆F1MN is
circle x2 + y2 = a 2 at the point Q such that P and Q are on
(a)  −
9 
(b)  , 0
2
the same side of X-axis. For two positive real numbers r , 0
 10  3 
and s, find the locus of the point R on PQ such that
(c)  , 0 (d)  , 6 
9 2
PR : RQ = r : s as P varies over the ellipse. (2001, 4M)
 10  3 
Passage Type Questions 14. If the tangents to the ellipse at M and N meet at R and
Passage the normal to the parabola at M meets the X-axis at Q,
Let F1 (x1 , 0) and F2 (x2, 0), for x1 < 0 and x2 > 0, be the foci of then the ratio of area of ∆MQR to area of the
quadrilateral MF1NF2 is
x2 y2
the ellipse + = 1. Suppose a parabola having vertex (a) 3 : 4 (b) 4 : 5
9 8
(c) 5 : 8 (d) 2 : 3

Topic 2 Equation of Tangent and Normal


Objective Questions I (Only one correct option) (a)
x2
+
y2
=1 (b)
1
+
1
=1
1. If the normal to the ellipse 3x + 4 y = 12 at a point P on it
2 2 4 2 4x2 2 y2
is parallel to the line, 2x + y = 4 and the tangent to the x2 y2 1 1
(c) + =1 (d) + =1
ellipse at P passes through Q(4, 4) then PQ is equal to 2 4 2x2 4 y2
(2019 Main, 12 April I) 7. Equation of a common tangent to the parabola y2 = 4x
(a)
5 5
(b)
61 and the hyperbola xy = 2 is (2019 Main, 11 Jan I)
2 2 (a) x + 2 y + 4 = 0 (b) x − 2 y + 4 = 0
221 157 (c) 4x + 2 y + 1 = 0 (d) x + y + 1 = 0
(c) (d)
2 2
8. The eccentricity of an ellipse whose centre is at the
2. The tangent and normal to the ellipse 3x2 + 5 y2 = 32 at the origin is 1/2. If one of its directrices is x = − 4, then the
point P(2, 2) meets the X-axis at Q and R, respectively.  3
equation of the normal to it at 1,  is
Then, the area (in sq units) of the ∆PQR is  2 (2017 Main)
(2019 Main, 10 April II)
16 14 34 68 (a) 2 y − x = 2 (b) 4x − 2 y = 1
(a) (b) (c) (d) (c) 4x + 2 y = 7 (d) x + 2 y = 4
3 3 15 15
x2 y2
9. The area (in sq units) of the quadrilateral formed by
3. If the line x − 2 y = 12 is tangent to the ellipse + =1 the tangents at the end points of the latusrectum to
a 2 b2 x2 y2
 −9  the ellipse + = 1 is
at the point 3,  , then the length of the latusrectum of 9 5 (2015 Main)
 2
27 27
the ellipse is (2019 Main, 10 April I)
(a) (b) 18 (c) (d) 27
4 2
(a) 8 3 (b) 9 (c) 5 (d) 12 2
10. The locus of the foot of perpendicular drawn from the
4. If the tangent to the parabola y = x at a point (α , β), (β > 0)
2
centre of the ellipse x2 + 3 y2 = 6 on any tangent to it is
is also a tangent to the ellipse, x + 2 y = 1, then α is equal
2 2 (2014 Main)
to (2019 Main, 9 April II) (a) (x2 − y2 )2 = 6x2 + 2 y2
(a) 2+1 (b) 2 −1 (c) 2 2 + 1 (d) 2 2 − 1 (b) (x2 − y2 )2 = 6x2 − 2 y2
(c) (x2 + y2 )2 = 6x2 + 2 y2
5. If the tangents on the ellipse 4x2 + y2 = 8 at the points (d) (x2 + y2 )2 = 6x2 − 2 y2
(1, 2) and (a , b) are perpendicular to each other, then a 2 is
equal to (2019 Main, 8 April I)
11. The normal at a point P on the ellipse x2 + 4 y2 = 16
128 64 4 2 meets the X-axis at Q. If M is the mid-point of the line
(a) (b) (c) (d) segment PQ, then the locus of M intersects the
17 17 17 17
latusrectum of the given ellipse at the points (2009)
6. If tangents are drawn to the ellipse x2 + 2 y2 = 2 at all  3 5 2  3 5 19 
(a)  ± ,±  (b)  ± ,± 
points on the ellipse other than its four vertices, then the  2 7  2 4 
mid-points of the tangents intercepted between the
 4 3
(c)  ± 2 3 , ±
1
coordinate axes lie on the curve (2019 Main, 11 Jan I)  (d)  ± 2 3 , ± 
 7  7 
rankershalt.com

Ellipse 437

12. The line passing through the extremity A of the major 43 7


(a) e12 + e22 = (b) e1 e2 =
axis and extremity B of the minor axis of the ellipse 40 2 10
x2 + 9 y2 = 9 meets its auxiliary circle at the point M. 5 3
(c)|e12 – e22| = (d) e1 e2 =
Then, the area (insqunits) of the triangle with vertices 8 4
at A, M and the origin O is (2009)
31 29 21 27
(a) (b) (c) (d) Analytical & Descriptive Questions
10 10 10 10
18. Find the equation of the common tangent in 1st
13. Tangents are drawn to the ellipse x2 + 2 y2 = 2, then the
quadrant to the circle x2 + y2 = 16 and the ellipse
locus of the mid-point of the intercept made by the
x2 y2
tangents between the coordinate axes is (2004, 1M) + = 1. Also, find the length of the intercept of the
1 1 1 1 25 4
(a) + =1 (b) + =1 tangent between the coordinate axes. (2005, 4M)
2x2 4 y2 4x2 2 y2
x2 y2 x2 y2 19. Prove that, in an ellipse, the perpendicular from a focus
(c) + =1 (d) + =1 upon any tangent and the line joining the centre of the
2 4 4 2
ellipse of the point of contact meet on the corresponding
x2
14. Tangent is drawn to ellipse + y2 = 1 at directrix. (2002, 5M)
27
(3 3 cos θ ,sin θ ) (where, θ ∈(0, π / 2). 20. Let ABC be an equilateral triangle inscribed in the
circle x2 + y2 = a 2. Suppose perpendiculars from A, B, C
Then, the value of θ such that the sum of intercepts on x2 y2
axes made by this tangent is minimum, is (2003, 1M) to the major axis of the ellipse 2 + 2 = 1, (a > b) meets
π π π π a b
(a) (b) (c) (d) the ellipse respectively at P , Q , R so that P , Q , R lie on
3 6 8 4
the same side of the major axis as A, B, C respectively.
15. If a > 2b > 0, then positive value of m for which Prove that, the normals to the ellipse drawn at the
y = mx − b 1 + m2 is a common tangent to x2 + y2 = b2 points P, Q and R are concurrent. (2000, 7M)
and (x − a )2 + y2 = b2 is (2002, 1M)
21. A tangent to the ellipse x2 + 4 y2 = 4 meets the ellipse
2b a − 4b
2 2
x2 + 2 y2 = 6 at P and Q. Prove that the tangents at P and
(a) (b)
a 2 − 4b2 2b Q of the ellipse x2 + 2 y2 = 6 are at right angles.
2b b (1997, 5M)
(c) (d)
a − 2b a − 2b 22. Let d be the perpendicular distance from the centre of
the ellipse x2 / a 2 + y2 / b2 = 1 to the tangent drawn at a
16. The number of values of c such that the straight line point P on the ellipse. If F1 and F2 are the two foci of the
x2
y = 4 x + c touches the curve + y2 = 1 is ellipse, then show that
4 (1998, 2M)
 b2 
(a) 0 (b) 2 (PF1 − PF2)2 = 4a 2 1 − 2
(c) 1 (d) ∞  d  (1995, 5M)

Objective Question II Integer Type Question


(One or more than one correct option) x2 y2
23. Suppose that the foci of the ellipse + = 1 are (f1 , 0)
17. Let E1 and E 2 be two ellipses whose centres are at the 9 5
and ( f2, 0), where f1 > 0 and f2 < 0. Let P1 and P2 be two
origin. The major axes of E1 and E 2 lie along the X-axis
parabolas with a common vertex at (0, 0) with foci at
and Y-axis, respectively. Let S be the circle
x2 + ( y − 1)2 = 2. The straight line x + y = 3 touches the ( f1 , 0) and (2 f2, 0), respectively. Let T1 be a tangent to P1
curves S, E1 and E 2 at P, Q and R, respectively. which passes through (2 f2, 0) and T2 be a tangent to P2
2 2 which passes through ( f1 , 0). If m1 is the slope of T1 and
Suppose that PQ = PR = . If e1 and e2 are the  1 
3 m2 is the slope of T2, then the value of  2 + m22 is
eccentricities of E1 and E 2 respectively, then the correct  m1 
expression(s) is/are (2015 Adv.) (2015 Adv.)
rankershalt.com

438 Ellipse

Topic 3 Equation of Chord of Contact, Chord Bisected


at a Given Point and Diameter
Passage Based Questions 2. The orthocentre of the ∆ PAB is
(a)  5,  (b)  , 
8 7 25
Passage  7 5 8 
Tangents are drawn from the point P( 3, 4) to the ellipse
(c)  ,  (d)  , 
11 8 8 7
x2 y2  5 5  25 5 
+ = 1 touching the ellipse at points A and B.
9 4 (2010)
3. The coordinates of A and B are
1. The equation of the locus of the point whose distance  8 2 161
(b)  − ,  9 8
from the point P and the line AB are equal, is (a) (3, 0) and (0, 2)  and  − , 
 5 15   5 5
(a) 9x2 + y2 − 6 xy − 54 x − 62 y +241 = 0
 8 2 161  9 8
(b) x2 + 9 y2 + 6 xy − 54 x + 62 y −241 = 0 (c)  − ,  and (0, 2) (d) (3, 0) and  − , 
 5 15   5 5
(c) 9x + 9 y − 6 xy − 54 x − 62 y −241 = 0
2 2

(d) x2 + y2 − 2 xy + 27 x + 31 y −120 = 0

Answers
Topic 1 Topic 2
1. (a) 2. (a) 3. (d) 4. (b) 1. (a) 2. (d) 3. (b) 4. (a)
5. (a) 6. (c) 7. (c) 8. (a, c) 5. (d) 6. (d) 7. (a) 8. (b)
2
 1 9. (d) 10. (c) 11. (c) 12. (d)
x − 
 3 (y − 1 ) 2 13. (a) 14. (b) 15. (a) 16. (b)
9. e = 1 / 2 10. + =1 11. b a 2 − b 2
1/9 1 / 12 2x 7 14 3
17. (a, b) 18. y = − +4 , 23. (4)
x 2 y 2 (r + s ) 2 3 3 3
12. + =1 13. (a) 14. (c)
a 2 (ar + bs ) 2 Topic 3
1. (a) 2. (c) 3. (d)

Hints & Solutions


Topic 1 Equation of Ellipse and Now, from the option the ellipse
x2 y2
+ = 1 passes
Focal Chord 4 8
through the point ( 2 , 2).
1. Let the equation of ellipse be
x 2
y 2 x2 y2
+ =1 …(i) 2. One of the focus of ellipse + = 1 is on Y-axis
a 2 b2 a 2 b2
(0, 5 3 )
Since, foci are at (0, 2) and (0, − 2), major axis is along
the Y -axis. ∴ be = 5 3 …(i)
[where e is eccentricity of ellipse]
So, be = 2 …(ii)
[where e is the eccentricity of ellipse] According to the question,
and 2a = length of minor axis = 4 [given] 2b − 2a = 10
⇒ a =2 …(iii) ⇒ b − a =5 …(ii)
2
a On squaring Eq. (i) both sides, we get
Q e2 = 1 − 2
b b2e2 = 75
2
 2 4  2  a 2  2 a2
∴   =1 − 2 Qe= ⇒ b21 − 2  = 75
 b b  b  Q e = 1 − 2 
 b   b 
8
⇒ =1 ⇒ b =82
b2 ⇒ b2 − a 2 = 75
x2 y2 ⇒ (b + a )(b − a ) = 75
Thus, equation of required ellipse is + =1
4 8 ⇒ b + a = 15 [from Eq. (ii)] …(iii)
rankershalt.com

Ellipse 439

On solving Eqs. (ii) and (iii), we get  b b


⇒ b   = 4 and = e
b = 10 and a = 5  a a
2a 2 2 × 25 ⇒ b(e) = 4
So, length of latusrectum is = = 5 units 1
b 10 ⇒ b = 4. …(i)
e
x2 y2
3. Let the ellipse be 2 + 2 = 1. Also, we know that b2 = a 2(1 − e2)
a b
b2  b 
⇒ = 1 − e2 ⇒ e2 = 1 − e2 Q =e
Then, according to given information, we have the a2  a 
following figure.
⇒ 2e2 = 1
Y 1
⇒ e= …(ii)
2
B (0,b)
From Eqs. (i) and (ii), we get
b =4 2
X
S′(–ae,o)O S(ae,0) b2 32
Now, a2 = = = 64
1−e 2
1−
1
2
2 2
x y
∴Equation of ellipse be + =1
b 64 32
Clearly, slope of line SB = and slope of line
− ae Now, check all the options.
b
S′B = Only (4 3 , 2 2 ), satisfy the above equation.
ae
x2 y2
Q Lines SB and S′ B are perpendicular, so 5. Given equation of ellipse is + =1
 b   b 16 9
  .   = −1 Y
 − ae  ae
[Q product of slopes of two
perpendicular lines is (−1)]
⇒ b2 = a 2e2 …(i) r
Also, it is given that area of ∆S′ BS = 8 X' X
1 2
∴ a =8
2
Y'
[QS′ B = SB = a because S′ B + SB = 2a and S′ B = SB]
⇒ a 2 = 16 ⇒ a = 4 9 7
…(ii) Here, a = 4, b = 3, e = 1 − ⇒
b 2 16 4
Q e2 = 1 − 2 = 1 − e2 [from Eq. (i)] 
a 7 
∴ Foci = (± ae, 0) =  ± 4 × , 0 = (± 7 ,0)
⇒ 2e2 = 1  4 
1
⇒ e2 = …(iii) Radius of the circle, r = (ae)2 + b2
2
From Eqs. (i) and (iii), we get = 7 + 9 = 16 = 4
 1  1 Now, equation of circle is
b2 = a 2  = 16  [using Eq. (ii)]
 2  2 (x − 0)2 + ( y − 3)2 = 16
⇒ b =8
2
∴ x2 + y2 − 6 y − 7 = 0
2
2b 2 ×8 x2 y2
Now, length of latus rectum = = = 4 units 6. PLAN Equation of an ellipse is 2
+ =1 [Qa > b]
a 4 a b2
x2 y2 b2
Eccentricity, e 2 = 1− [Q a > b]
4 Let the equation of ellipse be + =1 a2
a 2 b2
Y
Then, according the problem, we have
2b2
= 8 and 2ae = 2b
a A' A
2b2 X' X
[Length of latusrectum = and (– a, 0) (a, 0)
a
length of minor axis = 2b]
Y'
rankershalt.com

440 Ellipse

Description Situation As ellipse circumscribes the 1


0+ 0+
rectangle, then it must pass through all four vertices. 1 m
∴ =
Let the equation of an ellipse E 2 be 2 1 + m2
x2 y2
+ = 1, where a < b and b = 4.
a 2 b2 ⇒ m4 + m2 − 2 = 0 ⇒ m = ± 1
Y ∴ Equation of common tangents are
(0, 4) y = x + 1 and y = − x − 1
Intesection point of common tangent at Q ( −1, 0)
x2 y2
(–3, 2) (0, 2) (3, 2) ∴ Equation of ellipse + =1
1 1/ 2
where, a 2 = 1, b2 = 1 / 2
X' X
(–3, 0) O (3, 0) b2 1 1
E1 Now, eccentricity ( e) = 1 − = 1− =
(–3, –2) (0, –2) (3, –2) a2 2 2
 1
2 
E2 2b2  2
and length of latusrectum = = =1
(0, – 4) a 1
Y'
Y 0, –1 

Also, it passes through (3, 2). √2
9 4
⇒ + =1 [Q b = 4]
a 2 b2 X′ X
9 1 (–1,0) O 1  (1,0)
,0
⇒ + = 1 or a = 12
2 
√2
a2 4 x= 1
2 0, –1 
 √2
a 12 1 Y′ √2
Eccentricity of E 2 , e2 = 1 − =1− = [Q a < b]
b2 16 4
∴ Area of shaded region
1 1 1
∴ e= = 2∫ 1 − x 2 dx
2 1/ 2 2
1
7. Given, 16 x2 + 25 y2 = 400 [given] x 1 
= 2 1 − x 2 + sin−1 x
x 2
y 2
2 2 1/
⇒ + =1 2
25 16  π  1 π
= 2  0 +  −  +  
Now, PF1 + PF2 = Major axis = 2a [where, a = 5]   4  4 8 
= 2 × 5 = 10  π 1 π − 2
= 2 −  =
8. We have,  8 4 4 2
1
Equation of circle x 2 + y 2 = 9. Since, angle FBF′ is right angled.
2
∴ (Slope of FB) ⋅ (Slope of F ′ B) = − 1
and Equation of parabola y 2 = 4x
Y Y

y2=4x B (0, b)

X′ X X' X
Q O  x2 + y2 = 1 
 (– ae,0) F' O F (ae,0)
2

Y'
Y′
 0− b  0− b 
Let the equation of common tangent of parabola and ⇒   ⋅  = −1
 ae − 0  − ae − 0
circle is
1 b2
y = mx + ⇒ = − 1 ⇒ b2 = a 2e2
m − a 2e2
1
Since, radius of circle = ⇒ a 2(1 − e2) = a 2 e2
2
⇒ e2 = 1 / 2 ⇒ e = 1 / 2
rankershalt.com

Ellipse 441

10. There are two common tangents to the circle x2 + y2 = 1 So, A is maximum when x = 0.
and the hyperbola x2 − y2 = 1. These are x = 1 and x = − 1. b2 a 2 − b2
∴ Maximum of A = abe = ab 1 − = ab
But x = 1 is nearer to the point P(1 / 2, 1). a 2
a2
Therefore, directrix of the required ellipse is x = 1.
= b a 2 − b2
Now, if Q (x, y) is any point on the ellipse, then its
distance from the focus is PR r
12. Given, =
QP = (x − 1 / 2) + ( y − 1)
2 2 RQ s

and its distance from the directrix is| x − 1|. Y

By definition of ellipse,
2 Q(a cos θ, a sin θ)
 1 1 R (a cos θ,α)
QP = e|x − 1|⇒  x −  + ( y − 1)2 = |x − 1|
 2 2 (a cos θ,b sin θ)
P
2
 1 1 X' X
⇒  x −  + ( y − 1) = (x − 1)
2 2 O (0,0)
 2  4
1 1
⇒ x2 − x + + y2 − 2 y + 1 = (x2 − 2x + 1)
4 4
⇒ 4x2 − 4x + 1 + 4 y2 − 8 y + 4 = x2 − 2x + 1
Y'
⇒ 3x2 − 2x + 4 y2 − 8 y + 4 = 0
 1
2
1 α − b sin θ r
⇒ 3   x −  −  + 4 ( y − 1 )2 = 0 ⇒ =
 3 9 a sin θ − α s

2 ⇒ α s − b sin θ ⋅ s = ra sin θ − α r
 1 1
⇒ 3  x −  + 4 ( y − 1 )2 = ⇒ α s + α r = ra sin θ + b sin θ ⋅ s
 3 3
2 ⇒ α (s + r ) = sin θ (ra + bs)
 1 sin θ (ra + bs)
x −  ⇒ α=
 3 ( y − 1 )2
⇒ + =1 r+s
1 /9 1 / 12
Let the coordinates of R be (h, k).
x2 y2 h
11. Given, + =1 ∴ h = a cos θ ⇒ cos θ = …(i)
a 2 b2 a
Foci F1 and F2 are ( − ae, 0) and (ae, 0), respectively. Let (ar + bs) sin θ
and k =α =
P (x, y) be any variable point on the ellipse. r+s
The area A of the triangle PF1F2 is given by k (r + s)
⇒ sin θ = …(ii)
Y ar + bs
On squaring and adding Eqs. (i) and (ii), we get
P (X, Y )
h 2 k2 (r + s)2
sin 2 θ + cos 2 θ = +
a 2 (ar + bs)2
X' X
F1 O (ae, 0) F2 h 2 k2 (r + s)2
(–ae, 0) ⇒ 1= +
a 2 (ar + bs)2
x2 y2 (r + s)2
Hence, locus of R is + = 1.
Y' a 2
(ar + bs)2
x y 1 x2 y2
1 13. Here, + =1 …(i)
A= − ae 0 1 9 8
2
ae 0 1 has foci (± ae, 0)
1 where, a 2e2 = a 2 − b2
= (− y) (− ae × 1 − ae × 1) ⇒ a 2e2 = 9 − 8
2
⇒ ae = ± 1
1 x2
=− y (− 2ae) = a ey = ae ⋅ b 1 − 2 i.e. F1, F2 = (± 1, 0)
2 a
rankershalt.com

442 Ellipse

1 7 5 6
Y ∴ Area of ∆MQR = 6 −  6=
sq units
2 2 4
 3 , √6  1

2 and area of quadrilateral MF1NF2 = 2 × {1 − (− 1)} 6
y 2=4x
2
M = 2 6 sq units
3 Area of ∆MQR 5
F1 F2 2 P ∴ =
X′ O X Area of quadrilateral MF1NF2 8
(–3, 0) (–1,0) (1,0) (3, 0)

 x 2+ y 2=1   3 , – √6 
 Topic 2 Equation of Tangent and Normal
 2
9 8 N
1. Key Idea Equation of tangent and normal to the ellipse
x2 y2 xx yy 1
Y′
2
+ 2 = 1 at point p(x 1, y 1) is T = 0 ⇒ 21 + =1
a b a b2
Equation of parabola having vertex O(0, 0) and F2(1, 0) a2x b2 y
(as, x2 > 0) and − = a 2 − b 2 respectively.
x1 y1
y2 = 4x …(ii)
x 2
y2 Equation of given ellipse is 3x2 + 4 y2 = 12
On solving + = 1 and y = 4x, we get
2
9 8 x2 y2
⇒ + =1 … (i)
x = 3 / 2 and y = ± 6 4 3
Equation of altitude through M on NF1 is Now, let point P(2 cos θ , 3 sin θ ) , so equation of
y− 6 5 tangent to ellipse (i) at point P is
=
x − 3 /2 2 6 x cos θ y sin θ
+ =1 … (ii)
5 2 3
⇒ (y − 6) = (x − 3 / 2) …(iii)
2 6 Since, tangent (ii) passes through point Q(4, 4)
and equation of altitude through F1 is y = 0 …(iv) ∴2 cos θ +
4
sin θ = 1 … (iii)
 9  3
On solving Eqs. (iii) and (iv), we get  − , 0 as
 10 
and equation of normal to ellipse (i) at point P is
orthocentre. 4x 3y
− =4 −3
x2 y2 2 cos θ 3 sin θ
14. Equation of tangent at M (3 / 2, 6 ) to + = 1 is
9 8 ⇒ 2x sin θ − 3cosθy = sin θ cos θ … (iv)
3 x y
⋅ + 6 ⋅ =1 …(i) Since, normal (iv) is parallel to line, 2x + y = 4
2 9 8
∴ Slope of normal (iv) = slope of line, 2x + y = 4
which intersect X-axis at (6, 0).
2
Also, equation of tangent at N (3 / 2, − 6 ) is ⇒ tan θ = − 2 ⇒ tan θ = − 3 ⇒ θ = 120º
3 x y 3
⋅ − 6 =1 …(ii)
2 9 8  3 1
⇒ (sin θ , cos θ ) =  ,− 
Eqs. (i) and (ii) intersect on X-axis at R(6, 0) . …(iii)  2 2
− 6  3
Also, normal at M (3 / 2, 6 ) is y − 6 = x −   3
2  2 Hence, point P  − 1, 
 2
On solving with y = 0, we get Q(7 / 2, 0) …(iv) 2
 3
Y Now, PQ = (4 + 1)2 + 4 − 
 2
[given cordinates of Q ≡≡ (4, 4)]
M (3/2, √ 6 )
25 5 5
Q (7/2,0) = 25 + =
4 2
(–1, 0) (1, 0) R(6, 0)
X′
(–3, 0) F2 O F1 (3/2,0) X 2. Equation of given ellipse is
(3, 0) 3x2 + 5 y2 = 32 …(i)
Now, the slope of tangent and normal at point P(2, 2) to
N (3/2, – √ 6 ) the ellipse (i) are respectively
dy dx
mT = and mN = −
dx ( 2, 2) dy ( 2, 2)
Y′
rankershalt.com

Ellipse 443

On differentiating ellipse (i), w.r.t. x, we get ⇒ 2 yβ = x + β 2 [from Eq. (i)]


dy dy 3x x β
6x + 10 y =0 ⇒ =− ⇒ y= + …(ii)
dx dx 5y 2β 2
3x 3 5y 5 Since, line (ii) is also a tangent of the ellipse
So, mT = − = − and mN = =
5 y ( 2, 2) 5 3 y ( 2, 2) 3 x2 + 2 y2 = 1
2 2
 β 2 1  1
Now, equation of tangent and normal to the given ∴   = (1 )   +
 2  2 β 2
ellipse (i) at point P(2, 2) are
3 [Q condition of tangency of line y = mx + c to ellipse
( y − 2) = − (x − 2) x2 y2
5 + = 1 is c2 = a 2m2 + b2,
5 a 2 b2
and ( y − 2) = (x − 2) respectively. 1 1 β
3 here m= , a = 1, b = and c =
It is given that point of intersection of tangent and 2β 2 2 
normal are Q and R at X-axis respectively. β2 1 1
⇒ = +
4 4 β2 2
 16  4 
So, Q , 0 and R , 0 ⇒ β4 = 1 + 2 β2
3  5 
⇒ β4 − 2 β2 − 1 = 0
1
∴ Area of ∆PQR = (QR) × height 2± 4+4 2±2 2
2 ⇒ β2 = = =1 ± 2
1 68 68 2 2
= × ×2= sq units ⇒ β2 = 1 + 2 [Qβ 2 > 0]
2 15 15
Q α = β2 = 1 + 2
2 2
 16 4  68 68
[QQR =  −  =   = and height = 2] 5. Equation of given ellipse is
 3 5  15 15
4x2 + y2 = 8 …(i)
x2 y2 x2 y2
3. Key Idea Write equation of the tangent to the ellipse at any ⇒ + =1⇒ + =1
point and use formula for latusrectum of ellipse. 2 8 ( 2 )2 (2 2 )2
Now, equation of tangent at point (1, 2) is
Equation of given ellipse is
2x + y = 4 ...(ii)
x2 y2 x2 y2
2
+ 2 =1 …(i) [Q equation of tangent to the ellipse 2 + 2 = 1 at (x1 , y1 )
a b a b
 9 xx yy
Now, equation of tangent at the point 3, −  on the is 21 + 21 = 1]
 2 a b
ellipse (i) is and equation of another tangent at point (a , b) is
3x 9 y 4ax + by = 8 …(iii)
⇒ − =1 …(ii) Since, lines (ii) and (iii) are perpendicular to each other.
a 2 2b2
 2  4a 
x2
y2 ∴ −  × −  = −1
[Q the equation of the tangent to the ellipse + =1  1  b 
a 2 b2
xx1 yy [if lines a1x + b1 y + c1 = 0 and a 2x + b2y + c2 = 0
at the point (x1 , y1 ) is + 21 = 1 ]
a2 b  a  a 
are perpendicular, then  − 1   − 2 = − 1]
Q Tangent (ii) represent the line x − 2 y = 12, so  b1   b2 
1 2 12 ⇒ b = − 8a …(iv)
= =
3 9 1 Also, the point (a , b) lies on the ellipse (i), so
a 2 2b2 4a 2 + b2 = 8
⇒ a 2 = 36 and b2 = 27 ⇒ 4a 2 + 64a 2 = 8 [from Eq.(iv)]
8
2b2
2 × 27 ⇒ 68a = 8 ⇒ a =
2 2
Now, Length of latusrectum = = = 9 units 68
a 6 2
⇒ a2 =
4. Since the point (α , β) is on the parabola y2 = x, so 17
α = β2 …(i) 6. Given equation of ellipse is x2 + 2 y2 = 2 , which can be
Now, equation of tangent at point (α , β ) to the parabola x2 y2
y2 = x, is T = 0 written as + =1
1 2 1
⇒ yβ = (x + α )
2 Let P be a point on the ellipse, other than its four
[Q equation of the tangent to the parabola y2 = 4ax at a vertices. Then, the parametric coordinates of P be
point (x1 , y1 ) is given by yy1 = 2a (x + x1 )] ( 2 cos θ , sin θ )
rankershalt.com

444 Ellipse

1 a
Y 8. We have, e = and = 4
2 e
B ∴ a =2
  1 
2
 1
P (√2 cos θ, sin θ) Now, b2 = a 2(1 − e2) = (2)21 −    = 41 −  = 3
 2  4
 
X
A ⇒ b= 3
x2 y2
∴ Equation of the ellipse is + =1
Now, the equation of tangent at P is (2) 2
( 3 )2
x 2 cos θ y sin θ x2 y2
+ =1 ⇒ + =1
2 1 4 3
[Q equation of tangent at ( x1 , y1 ) is given by T = 0  3
xx yy  Now, the equation of normal at 1,  is
⇒ 21 + 21 = 1  2
a b 
a 2x b2y
x y − = a 2 − b2
⇒ + =1 x1 y1
2 sec θ cosec θ
4x 3y
∴ A ( 2 sec θ, 0) and B ( 0, cosec θ) ⇒ − =4 −3
1 (3 /2)
Let mid-point of AB be R( h , k), then
⇒ 4x − 2 y = 1
2 sec θ cosec θ
h= and k = 9. Given equation of ellipse is
2 2
x2 y2
2h = 2 sec θ and 2k = cosec θ + =1
9 5
1 1 ∴ a 2 = 9, b2 = 5 ⇒ a = 3, b = 5
⇒ cosθ = and sinθ =
2h 2k
b2 5 2
We know that, cos2 θ + sin2 θ = 1 Now, e= 1+ 2
= 1− =
a 9 3
1 1
∴ + =1 b2 5
2h 2 4k2 Foci = (± a e, 0) = (± 2, 0) and =
1 1 a 3
So, locus of (h , k) is + =1 Y
2x2 4 y2
P (0,3)
a
7. We know that, y = mx + is the equation of tangent to )
m , 5/3
the parabola y = 4ax.
2 −2
M ( L (2,
1 5/3)
∴ y = mx + is a tangent to the parabola
m X' X
(−2,0) O (2,0) Q
y = 4x.
2
[Qa = 1] (9/2, 0)
M'(−
Let, this tangent is also a tangent to the hyperbola 2,
− L' (2, − 5/3)
5/3
xy = 2 )
1
Now, on substituting y = mx + in xy = 2, we get
m Y'
 1
x mx +  = 2.
 m ∴Extremities of one of latusrectum are
⇒ m 2x 2 + x − 2m = 0
 5  − 5
2,  and 2, 
Note that tangent touch the curve exactly at one point,  3  3
therefore both roots of above equations are equal.
3  5
 1 ∴Equation of tangent at 2,  is
⇒ D = 0 ⇒ 1 – 4( m 2 ) ( − 2m ) ⇒ m3 =  −   3
 2
x(2) y(5 / 3)
1 + = 1 or 2x + 3 y = 9
⇒ m=− 9 5
2 9 
∴ Required equation of tangent is Since, Eq. (ii) intersects X and Y -axes at  , 0
x 2 
y=− −2 and (0, 3), respectively.
2 ∴ Area of quadrilateral = 4 × Area of ∆POQ
⇒ 2y = − x − 4
1 9 
⇒ x + 2y + 4 = 0 = 4 ×  × × 3 = 27 sq units
2 2 
rankershalt.com

Ellipse 445

x2 y2 Y
+
10. Equation of ellipse is x2 + 3 y2 = 6 or = 1. (0,2)
6 2 P (4 cosθ,2 sinθ)
x cos θ y sin θ
Equation of the tangent is + =1 (− 4,0) (4,0)
a b X′ X
Q(3cosθ,0)
Let (h , k) be any point on the locus.
h k
∴ cos θ + sin θ = 1 ...(i) (0,−2)
a b
Y′
−b
Slope of the tangent line is cot θ.
a 4 3
Slope of perpendicular drawn from centre (0,0) to (h , k) For given ellipse, e2 = 1 − =
16 4
is k / h.
3
Since, both the lines are perpendicular. ∴ e=
2
 k  b 
∴   ×  − cot θ = − 1 3
 h  a  ∴ x= ±4× = ± 2 3 [Q x = ± ae] ...(iv)
2
cos θ sin θ
⇒ = =α [say] On solving Eqs. (iii) and (iv), we get
ha kb
4 48 1
⇒ cos θ = αha × 12 + y2 = 1 ⇒ y2 = 1 − =
49 49 49
sin θ = αkb 1
h k y=±
From Eq. (i), (αha ) + (αkb) = 1 7
a b
 1
⇒ h 2α + k2α = 1 ∴ Required points  ± 2 3 , ±  .
 7
1
⇒ α= 12. Equation of auxiliary circle is
h 2 + k2
x2 + y2 = 9 ... (i)
Also, sin 2 θ + cos 2 θ = 1 x y
Equation of AM is + =1 ... (ii)
⇒ (αkb)2 + (αha )2 = 1 3 1
⇒ α k b +α h a =1
2 2 2 2 2 2
Y
2 2
kb h 2a 2
⇒ + 2 =1 M
(h + k )
2 2 2
(h + k2)2 ( 125 , 95 ( B(0,1)
2k2 6 h2
⇒ + 2 =1 [Q a 2 = 6, b2 = 2]
(h + k )
2 2 2
(h + k2)2 X' X
N O A(3,0)
⇒ 6x + 2 y = (x + y )
2 2 2 2 2

[replacing k by y and h by x ]
x2 y2
11. Given, + =1
16 4 Y'
Here, a = 4, b = 2
 12 9
Equation of normal On solving Eqs. (i) and (ii), we get M  − , .
 5 5
4x sec θ − 2 y cosec θ = 12
1 27
 7 cos θ  Now, area of ∆ AOM = ⋅ OA × MN = sq units
M , sin θ = (h , k) [say] 2 10
 2 
x2 y2
7 cos θ 13. Let the point P ( 2 cos θ, sin θ) on + = 1.
∴ h= 2 1
2
Y
2h
⇒ = = cos θ …(i) (mid-point of AB)
7 B(θ, cosec θ)
M
and k = sin θ …(ii) P(√2 cos θ, sin θ)
On squaring and adding Eqs. (i) and (ii), we get X′ X
A
4h 2
+ k2 = 1 [Q cos θ + sin θ = 1]
2 2
(√2 sec θ, 0)
49
4x2
Hence, locus is + y2 = 1 ... (iii)
49 Y′
rankershalt.com

446 Ellipse

x 2 x2 y2
Equation of tangent is, cos θ + y sin θ = 1 17. Here, E1: + = 1, (a > b)
2 a 2 b2
whose intercept on coordinate axes are x2 y2
E 2 : 2 + 2 = 1, (c < d ) and S : x2 + ( y − 1)2 = 2
A( 2 sec θ , 0) and B (0, cosec θ) c d
as tangent to E1 , E 2 and S is x + y = 3.
∴ Mid-point of its intercept between axes
 2 1  Y
 sec θ , cos ec θ = (h , k)

2
=
 2 2 

)2
R

–1
P

(y
1 1
⇒ cos θ = and sin θ =

+
(0, 1)

x2
2h 2k

S:
Q
Thus, focus of mid-point M is
X′ X
1 1 O E1
(cos 2θ + sin 2 θ ) = + x+y=3
2h 2 4k2
1 1
⇒ 2
+ 2 = 1, is required locus.
2x 4 y
E2
14. Given, tangent is drawn at (3 3 cos θ ,sin θ ) to Y′
x2 y2 Let the point of contact of tangent be (x1 , y1 ) to S.
+ = 1.
27 1 ∴ x ⋅ x1 + y ⋅ y1 − ( y + y1 ) + 1 = 2
x cos θ y sin θ or x x1 + y y1 − y = (1 + y1 ), same as x + y = 3.
∴ Equation of tangent is + = 1.
3 3 1 x1 y1 − 1 1 + y1
⇒ = =
3 3 1  1 1 3
Thus, sum of intercepts =  +  = f (θ ) [say] i.e. x1 = 1 and y1 = 2
 cos θ sin θ 
∴ P = (1, 2)
3 3 sin3 θ − cos3 θ 2 2
⇒ f ′ (θ ) = , put f ′ (θ ) = 0 Since, PR = PQ = . Thus, by parametric form,
sin 2 θ cos 2 θ 3
1 x −1 y −2 2 2
⇒ sin3 θ = cos3 θ = =±
33/ 2 −1 / 2 1 / 2 3
1 π π  5 4  1 8
⇒ tan θ = , i.e. θ = and at θ= , f ′ ′ (0) > 0 ⇒ x = , y =  and  x = , y = 
3 6 6  3 3  3 3
π  5 4  1 8
Hence, tangent is minimum at θ = . ∴ Q =  ,  and R= , 
6  3 3  3 3
15. Given, y = mx − b 1 + m2 touches both the circles, so Now, equation of tangent at Q on ellipse E1 is
distance from centre = radius of both the circles. x⋅5 y⋅4
+ =1
|ma − 0 − b 1 + m2 | |− b 1 + m2| a 2 ⋅ 3 b2 ⋅ 3
= b and =b On comparing with x + y = 3, we get
m2 + 1 m2 + 1
a 2 = 5 and b2 = 4
⇒ |ma − b 1 + m2 | =|− b 1 + m2 | b2 4 1
∴ e12 = 1 − 2 = 1 − = …(i)
a 5 5
⇒ m2a 2 − 2abm 1 + m2 + b2 (1 + m2) = b2 (1 + m2)
Also, equation of tangent at R on ellipse E 2 is
⇒ ma − 2b 1 + m2 = 0 x⋅1 y⋅8
+ 2 =1
⇒ m2a 2 = 4b2 (1 + m2) a ⋅3 b ⋅3
2

2b On comparing with x + y = 3, we get


∴ m= a 2 = 1, b2 = 8
a 2 − 4b2
a2 1 7
∴ e22 = 1 − 2 = 1 − = …(ii)
16. For ellipse, condition of tangency is c2 = a 2m2 + b2 b 8 8
x2 7 7
Given line is y = 4x + c and curve + y2 = 1 Now, e12 ⋅ e22 = ⇒ e1 e2 =
4 40 2 10
1 7 43
⇒ c2 = 4 × 42 + 1 = 65 and e12 + e22 = + =
5 8 40
⇒ c = ± 65
1 7 27
So, there are two different values of C. Also, e1 − e2 = − =
2 2
5 8 40
rankershalt.com

Ellipse 447

x2 y2 Point of intersection Q of Eqs. (i) and (ii) has x


18. Let the common tangent to x2 + y2 = 16 and + =1 a
25 4 coordinate, . Hence, Q lies on the corresponding
be e
a
y = mx + 4 1 + m2 …(i) directrix x = .
e
and y = mx + 25m2 + 4 …(ii) 20. Let the coordinates of A ≡ (a cos θ , b sin θ), so that the
Since, Eqs. (i) and (ii) are same tangent. coordinates of
B = { a cos (θ + 2π / 3), a sin (θ + 2π / 3)}
∴ 4 1 + m2 = 25m2 + 4
and C = { a cos (θ + 4π / 3), a sin (θ + 4π / 3)}
⇒ 16 (1 + m2) = 25 m2 + 4
According to the given condition, coordinates of P are
⇒ 9m2 = 12 (a cos θ b sin θ ) and that of Q are { a cos (θ + 2π / 3),
⇒ m = ± 2/ 3 b sin (θ + 2π / 3)} and that of R are
a cos (θ + 4π / 3), b sin (θ + 4π / 3)
Since, tangent is in Ist quadrant.
Y
∴ m <0 A (a cos θ, b sin θ)
B
⇒ m = − 2/ 3
So, the equation of the common tangent is P
X' Q X
2x 7 O
y=− +4 R
3 3
which meets coordinate axes at A(2 7 , 0) and C
 7 Y'
0, 4 .
 3 [Q it is given that P, Q, R are on the same side
2
of X-axis as A, B and C]
 7
∴ AB = (2 7 − 0)2 + 0 − 4  Equation of the normal to the ellipse at P is
 3 ax by
− = a 2 − b2
11 196 14 3 14 3 cos θ sin θ
= 28 + = = × = 1
3 3 3 3 3 or ax sin θ − by cos θ = (a 2 − b2) sin 2θ …(i)
2
19. Any point on the ellipse
Equation of normal to the ellipse at Q is
x2 y2
+ 2 = 1 be P (a cos θ , b sin θ )  2π   2π 
a 2
b ax sin θ +  − by cos θ + 
 3  3
The equation of tangent at point P is given by
1 2  4π 
x cos θ y sin θ = (a − b2)sin 2θ +  …(ii)
+ =1 2  3
a b
Equation of normal to the ellipse at R is
The equation of line perpendicular to tangent is
x sin θ y cos θ a x sin (θ + 4π / 3) − by cos (θ + 4π / 3)
− =λ 1
b a = (a 2 − b2) sin (2θ + 8π / 3) …(iii)
2
Since, it passes through the focus (ae, 0), then
But sin (θ + 4π / 3) = sin (2π + θ − 2π / 3)
ae sin θ
−0 = λ = sin (θ − 2π / 3)
b
ae sin θ and cos (θ + 4π / 3) = cos (2π + θ − 2π / 3)
⇒ λ= = cos (θ − 2π / 3)
b
x sin θ y cos θ ae sin θ and sin (2θ + 8π / 3) = sin (4π + 2θ − 4π / 3 )
∴ Equation is − = …(i) = sin (2θ − 4π / 3)
b a b
Now, Eq. (iii) can be written as
Equation of line joining centre and point of contact
P (a cos θ , b sin θ ) is ax sin (θ − 2π / 3) − by cos (θ − 2π / 3)
1
b = (a 2 − b2) sin (2θ − 4π / 3) …(iv)
y = (tan θ ) x …(ii) 2
a
rankershalt.com

448 Ellipse

For the lines (i), (ii) and (iv) to be concurrent, we must  hx ky 


2
 x2 y2   h 2 k2 
i.e.  + − 1 =  + − 1  + − 1 …(v)
have the determinant  6 3  6 3  6 3 

a sin θ − b cos θ h 2 1  h 2 k2 
In Eq. (v), coefficient of x2 = −  + − 1
 2π   2π  36 6  6 3 
∆1 = a sin θ +  − b cos θ + 
 3  3
h 2 h 2 k2 1 1 k2
 2π   2π  =
− − + = −
a sin θ −  − b cos θ −  36 36 18 6 6 18
 3   3
1 2 k2 1  h 2 k2 
(a − b2) sin 2 θ and coefficient of y2 = −  + − 1
2 9 3 6 3 
1 2
(a − b2) sin (2 θ + 4π / 3) = 0 k2 h 2 k2 1 h2 1
2 = − − + =− +
1 2 9 18 9 3 18 3
(a − b2) sin (2 θ − 4π / 3)
2 Again, coefficient of x2 + coefficient of y2
Thus, lines (i), (ii) and (iv) are concurrent. 1 1 1
=− (h 2 + k2) + +
x2 y2 18 6 3
21. Given, x2 + 4 y2 = 4 or + =1 …(i) 1 1
4 1 =− (9 cos θ + 9 sin 2 θ ) +
2
18 2
Equation of any tangent to the ellipse on (i) can be
9 1
written as =− + =0
x 18 2
cos θ + y sin θ = 1 …(ii)
2 which shows that two lines represent by Eq. (v) are at
Equation of second ellipse is right angles to each other.
Y 22. Let the coordinates of point P be (a cos θ , b sin θ ).
A Then, equation of tangent at P is
x y
Q cos θ + sin θ = 1 …(i)
√3 a b
P
1 We have, d = length of perpendicular from O to the
X′ X tangent at P
−2 O 2 √6
−√6 Y
−1
P (a cosθ, b sin θ)
−√3
d
Y′
X' X
x2 + 2 y2 = 6 F1(−ae,0) O F2(ae,0)
x2 y2
⇒ + =1 …(iii)
6 3
Suppose the tangents at P and Q meets at A (h , k). Y'
Equation of the chord of contact of the tangents through |0 + 0 − 1|
A (h , k) is d=
cos 2 θ sin 2 θ
hx ky
+ =1 …(iv) +
6 3 a2 b2
But Eqs. (iv) and (ii) represent the same straight line, so 1 cos 2 θ sin 2 θ
⇒ = +
comparing Eqs. (iv) and (ii), we get d a2 b2
h /6 k /3 1
= = 1 cos 2 θ sin 2 θ
cos θ / 2 sin θ 1 ⇒ = +
d2 a2 b2
⇒ h = 3 cos θ and k = 3 sin θ
 b2 
Therefore, coordinates of A are (3 cos θ ,3 sin θ ). We have to prove (PF1 − PF2)2 = 4a 2 1 − 2
 d 
Now, the joint equation of the tangents at A is given by
T 2 = SS1,  b2  4a 2b2
Now, RHS = 4a 2 1 − 2 = 4a 2 −
 d  d2
rankershalt.com

Ellipse 449

 cos 2 θ sin 2 θ 
= 4a 2 − 4a 2b2  +  Topic 3 Equation of Chord of Contact,
 a2 b2  Chord Bisected at a Given Point
= 4a 2 − 4b2 cos 2 θ − 4a 2 sin 2 θ and Diameter
= 4a 2(1 − sin 2 θ ) − 4b2 cos 2 θ 1
1. Equation of AB is y − 0 = − (x − 3)
= 4a cos θ − 4b cos θ
2 2 2 2 3
x + 3y −3 = 0
= 4 cos 2 θ (a 2 − b2) = 4 cos 2 θ ⋅ a 2e2 Q e = 1 − (b / a )2 
  ⇒ |x + 3 y − 3|2 = 10 [(x − 3)2 + ( y − 4)2]
Again, PF1 = e|a cos θ + a / e|= a | e cos θ + 1| On solving, we are getting
= a (e cos θ + 1) 9x2 + y2 − 6xy − 54x − 62 y + 241 = 0
[Q − 1 ≤ cos θ ≤ 1 and 0 < e < 1] 2. Equation of AB is
Similarly, PF2 = a (1 − e cos θ ) P (3, 4)
Therefore, LHS = (PF1 − PF2) 2

= [a (e cos θ + 1) − a (1 − e cos θ )]2


= (ae cos θ + a − a + ae cos θ )2
(− 95 , 85(
B A
= (2ae cos θ )2 = 4a 2e2 cos 2 θ (3, 0)
Hence, LHS = RHS
8
23. 8
T2 y − 0 = 5 (x − 3) = (x − 3)
9
− −3 − 24
Y
T1 5
1
⇒ y = − (x − 3)
3
(– 4, 0)
X′ O F1(2, 0) X ⇒ x + 3y = 3 ...(i)
F2 F2(–2, 0)
Equation of the straight line perpendicular to AB
through P is
y 2 = 8x
3x − y = 5
y2 = –16x Y′
Equation of PA is 2x − 3 = 0
Tangent to P1 passes through (2 f2, 0) i. e. (−4, 0). The equation of straight line perpendicular to PA
2  − 9 8 8
∴ T1 : y = m1x + through B  ,  is y = .
m1  5 5 5

⇒ 0 = −4m1 +
2  11 8
Hence, the orthocentre is  , .
m1  5 5
⇒ m12 = 1 / 2 …(i) 3. Figure is self-explanatory.
Also, tangent to P2 passes through (f1, 0) i.e. (2, 0). P
(−4) (3, 4)
⇒ T2 : y = m2x + D
m2
–9 , 8 B
4
⇒ 0 = 2m2 − 5 5
m2
F
⇒ m22 = 2 …(ii) A
1 (3, 0)
∴ + m22 = 2 + 2 = 4
m12

Download Chapter Test


http://tinyurl.com/y56azn65 or
rankershalt.com

19
Hyperbola

Topic 1 Equation of Hyperbola and Focal Chord


Objective Questions I (Only one correct option) (c) an ellipse whose eccentricity is
2
, when r > 1.
1. Let P be the point of intersection of the common tangents r+1
to the parabola y2 = 12x and the hyperbola 8x2 − y2 = 8. If (d) an ellipse whose eccentricity is
1
, when r > 1.
S and S′ denotes the foci of the hyperbola where S lies on r+1
the positive X-axis then P divides SS′ in a ratio
(2019 Main, 12 April I) 7. A hyperbola has its centre at the origin, passes through
(a) 13 : 11 (b) 14 : 13 the point (4, 2) and has transverse axis of length 4 along
(c) 5 : 4 (d) 2 : 1 the X-axis. Then the eccentricity of the hyperbola is
(2019 Main, 9 Jan II)
2. If 5x + 9 = 0 is the directrix of the hyperbola 2
(a) 2 (b)
16x2 − 9 y2 = 144, then its corresponding focus is 3
(2019 Main, 10 April II) 3
(c) (d) 3
(a)  − , 0 (b) (− 5, 0) (c)  , 0
5 5 2
(d) (5, 0)
 3  3 
π
8. Let 0 < θ < . If the eccentricity of the hyperbola
3. If a directrix of a hyperbola centred at the origin and 2
passing through the point (4, − 2 3 ) is 5x = 4 5 and its x2 y2
eccentricity is e, then − = 1 is greater than 2, then the length of its
(2019 Main, 10 April I) cos θ sin 2 θ
2

(a) 4e4 − 12e2 − 27 = 0 (b) 4e4 − 24e2 + 27 = 0 latus rectum lies in the interval (2019 Main, 9 Jan I)
(c) 4e4 + 8e2 − 35 = 0 (d) 4e4 − 24e2 + 35 = 0 3
(a)  1, (b) (3,∞)
4. If the vertices of a hyperbola be at (−2, 0) and (2, 0) and  2 
3 
one of its foci be at (−3, 0), then which one of the (c)  , 2 (d) (2, 3]
following points does not lie on this hyperbola?  2 
(2019 Main, 12 Jan I)
9. The eccentricity of the hyperbola whose length of the
(a) (2 6 , 5) (b) (6, 5 2 )
latusrectum is equal to 8 and the length of its conjugate
(c) (4, 15 ) (d) (− 6, 2 10 )
axis is equal to half of the distance between its foci, is
5. If a hyperbola has length of its conjugate axis equal to 5 (2017 Main)
and the distance between its foci is 13, then the 4 4
(a) (b)
eccentricity of the hyperbola is (2019 Main, 11 Jan II) 3 3
13 13 13 2
(a) (b) 2 (c) (d) (c) (d) 3
12 8 6 3
 y 2
x  2
10. Consider a branch of the hyperbola
6. Let S = (x, y) ∈ R2 : − = 1,
 1+ r 1−r  x2 − 2 y2 − 2 2x − 4 2 y − 6 = 0
where r ≠ ± 1. Then, S represents (2019 Main, 10 Jan II) with vertex at the point A. Let B be one of the end points
2 of its latusrectum. If C is the focus of the hyperbola
(a) a hyperbola whose eccentricity is , when
1−r nearest to the point A, then the area of the ∆ ABC is
0 < r < 1. (a) 1 − 2 / 3 sq unit (2008, 3M)
2 (b) 3 / 2 − 1 sq unit
(b) a hyperbola whose eccentricity is , when
r+1 (c) 1 + 2 /3 sq unit
0 < r < 1. (d) 3 /2 + 1 sq unit
rankershalt.com

Hyperbola 451

11. A hyperbola, having the transverse axis of length 2 sin θ , 16. An ellipse intersects the hyperbola 2x2 − 2 y2 = 1
is confocal with the ellipse 3x + 4 y = 12 . Then, its
2 2
orthogonally. The eccentricity of the ellipse is reciprocal
equation is (2007, 3M) to that of the hyperbola. If the axes of the ellipse are
(a) x2cosec2 θ − y2sec2 θ = 1 along the coordinate axes, then (2009)
(b) x2 sec2 θ − y2cosec2 θ = 1 (a) equation of ellipse is x2 + 2 y2 = 2
(c) x2 sin 2θ − y2cos2θ = 1 (b) the foci of ellipse are (±1, 0)
(d) x2cos2 θ − y2sin 2 θ = 1 (c) equation of ellipse is x2 + 2 y2 = 4
x2 y2 (d) the foci of ellipse are (± 2 , 0)
12. If e1 is the eccentricity of the ellipse + = 1 and e2 is
16 25
the eccentricity of the hyperbola passing through the Analytical & Descriptive Question
foci of the ellipse and e1 e2 = 1, then equation of the 17. A variable straight line of slope 4 intersects the
hyperbola is (2006, 3M) hyperbola xy = 1 at two points. Find the locus of the
2 2 2 2
x y x y point which divides the line segment between these two
(a) − =1 (b) − = −1
9 16 16 9 points in the ratio 1 : 2. (1997, 5M)
x2 y2
(c) − =1 (d) None of these
9 25 Match the List
2 2
x y x2 y2
13. For hyperbola − = 1, which of the 18. Let H : − = 1, where a > b > 0, be a hyperbola in the
cos 2 α sin 2 α a 2 b2
following remains constant with change in ‘ α ’ ? (2003, 1M) XY -plane whose conjugate axis LM subtends an angle
(a) Abscissae of vertices (b) Abscissae of foci of 60° at one of its vertices N . Let the area of the ∆ LMN
(c) Eccentricity (d) Directrix be 4 3.
x2 y2
14. The equation − = 1,|r| < 1 represents List-I List-II
1− r 1 + r (1981, 2M)
(a) an ellipse (b) a hyperbola The length of the
P. 1. 8
(c) a circle (d) None of these conjugate axis of H is
4
Objective Questions II Q. The eccentricity of H is 2.
3
(One or more than one correct option) 2
The distance between the
x2 y2 R. 3.
15. Let the eccentricity of the hyperbola 2 − 2 = 1 be foci of H is 3
a b
The length of the latus
reciprocal to that of the ellipse x2 + 4 y2 = 4. If the S. 4. 4
rectum of H is
hyperbola passes through a focus of the ellipse, then
x2 y2 The correct option is (2018 Adv.)
(a) the equation of the hyperbola is − =1 (2011)
3 2 (a) P → 4; Q → 2; R → 1; S → 3
(b) a focus of the hyperbola is (2, 0) (b) P → 4; Q → 3; R → 1; S → 2
5 (c) P → 4; Q → 1; R → 3; S → 2
(c) the eccentricity of the hyperbola is
3 (d) P → 3; Q → 4; R → 2; S → 1
(d) the equation of the hyperbola is x2 − 3 y2 = 3

Topic 2 Equation of Tangent and Normal


Objective Questions I (Only one correct option) 3. If the eccentricity of the standard hyperbola passing
through the point (4, 6) is 2, then the equation of the
1. The equation of a common tangent to the curves,
tangent to the hyperbola at (4, 6) is (2019 Main, 8 April II)
y = 16x and xy = − 4, is
2
(2019 Main, 12 April II)
(a) x − y + 4 = 0 (b) x + y + 4 = 0 (a) 3x − 2 y = 0 (b) x − 2 y + 8 = 0
(c) x − 2 y + 16 = 0 (d) 2x − y + 2 = 0 (c) 2x − y − 2 = 0 (d) 2x − 3 y + 10 = 0
2. If the line y = mx + 7 3 is normal to the hyperbola 4. The equation of a tangent to the hyperbola
x2 y2 4x2 − 5 y2 = 20 parallel to the line x − y = 2 is
− = 1, then a value of m is (2019 Main, 10 Jan I)
24 18 (2019 Main, 9 April I)
(a) x − y − 3 = 0 (b) x − y + 9 = 0
3 15 2 5
(a) (b) (c) (d) (c) x − y + 1 = 0 (d) x − y + 7 = 0
5 2 5 2
rankershalt.com

452 Hyperbola
5. Tangents are drawn to the hyperbola 4x2 − y2 = 36 at the 11. Consider the hyperbola H: x2 − y2 = 1 and a circle S with
points P and Q. If these tangents intersect at the centre N (x2 , 0). Suppose that H and S touch each other
point T(0, 3), then the area (in sq units) of ∆PTQ is at a point P(x1 , y1 ) with x1 > 1 and y1 > 0. The common
(a) 45 5 (b) 54 3 (2018 Main) tangent to H and S at P intersects the X-axis at point M.
(c) 60 3 (d) 36 5 If (l , m) is the centroid of ∆PMN , then the correct
expression(s) is/are (2015 Adv.)
6. If a hyperbola passes through the point P( 2 , 3 ) and dl 1 dm x1
has foci at (± 2, 0), then the tangent to this hyperbola at (a) = 1 − 2 for x1 > 1 (b) = for x1 > 1
dx1 3x1 dx1 3( x2 − 1)
P also passes through the point (2017 Main) 1
dl 1 dm 1
(a) (3 2 , 2 3 ) (b) (2 2 , 3 3 ) (c) = 1+ for x1 > 1 (d) = for y1 > 0
(c) ( 3 , 2 ) (d) (− 2 , − 3 ) dx1 3x12 dy1 3

x2 y2 x2 y2
7. Let P(6, 3) be a point on the hyperbola − = 1. If the 12. Tangents are drawn to the hyperbola − = 1,
a 2 b2 9 4
normal at the point P intersects the X-axis at (9, 0), then parallel to the straight line 2x − y = 1. The points of
the eccentricity of the hyperbola is (2011) contacts of the tangents on the hyperbola are (2012)

(a) 
1 
(b)  −
5 3 9 9 1 
(a) (b) (c) 2 (d) 3 ,  ,− 
2 2  2 2 2  2 2 2
8. If the line 2x + 6 y = 2 touches the hyperbola (c) (3 3 , − 2 2) (d) (− 3 3 , 2 2 )
x2 − 2 y2 = 4, then the point of contact is (2004, 1M)
Passage Based Problems
(a) ( − 2, 6) (b) ( − 5, 2 6 ) (c)  ,
1 1 
 (d) (4, − 6)
 2 6 x2 y2
The circle x 2 + y 2 − 8x = 0 and hyperbola − =1
9. Let P (a sec θ , b tan θ ) and Q (a sec φ , b tan φ ), where 9 4
π x2 y2 intersect at the points A and B. (2010)
θ+φ= , be two points on the hyperbola 2 − 2 = 1.
2 a b 13. Equation of the circle with AB as its diameter is
If (h , k) is the point of the intersection of the normals at (a) x2 + y2 − 12 x + 24 = 0 (b) x2 + y2 + 12 x + 24 = 0
P and Q, then k is equal to (1999, 2M) (c) x2 + y2 + 24 x − 12 = 0 (d) x2 + y2 − 24 x − 12 = 0
a 2 + b2  a 2 + b2  a 2 + b2  a 2 + b2  14. Equation of a common tangent with positive slope to the
(a) (b) −   (c) (d) −  
a  a  b  b  circle as well as to the hyperbola is
(a) 2 x − 5 y − 20 = 0 (b) 2 x − 5 y + 4 = 0
(c) 3 x − 4 y + 8 = 0 (d) 4 x − 3 y + 4 = 0
Objective Questions II
(One or more than one correct option) Integer Answer Type Question
x y 2 2 15. The line 2x + y = 1 is tangent to the hyperbola
10. If 2x − y + 1 = 0 is a tangent to the hyperbola − =1 x2 y2
a 2 16 − = 1. If this line passes through the point of
then which of the following CANNOT be sides of a right a 2 b2
angled triangle? (2017 Adv.) intersection of the nearest directrix and the X-axis,
(a) a, 4, 1 (b) 2a, 4, 1 (c) a, 4, 2 (d) 2a, 8, 1 then the eccentricity of the hyperbola is…… (2010)

Topic 3 Equation of Chord of Contact, Chord Bisected Diameter,


Asymptote and Rectangular Hyperbola
Objective Question I (Only one correct option) (a) x1 + x2 + x3 + x4 = 0
1. If x = 9 is the chord of contact of the hyperbola (b) y1 + y2 + y3 + y4 = 0
(c) x1 x2 x3 x4 = c4
x − y2 = 9, then the equation of the corresponding pair
2

of tangents is (1999, 2M) (d) y1 y2 y3 y4 = c4


(a) 9 x2 − 8 y2 + 18 x − 9 = 0 (b) 9 x2 − 8 y2 − 18 x + 9 = 0
(c) 9 x2 − 8 y2 − 18 x − 9 = 0 (d) 9 x2 − 8 y2 + 18 x + 9 = 0 Analytical & Descriptive Question
3. Tangents are drawn from any point on the hyperbola
Objective Question II x2 y2
(Only one or more than one correct option) − = 1 to the circle x2 + y2 = 9. Find the locus of
9 4
2. If the circle x2 + y2 = a 2 intersects the hyperbola xy = c2 mid-point of the chord of contact. (2005, 4M)

in four points P (x1 , y1 ), Q (x2 , y2 ), R (x3 , y3 ), S (x4 , y4 ),


then (1998, 2M)
rankershalt.com

Hyperbola 453

Answers
Topic 1 5. (a) 6. (b) 7. (b) 8. (d)
1. (c) 2. (b) 3. (d) 4. (b) 9. (d) 10. (a, c, d) 11. (a, b, d) 12. (a, b)
5. (a) 6. (c) 7. (b) 8. (b) 13. (a) 14. (b) 15. (2)
9. (c) 10. (b) 11. (a) 12. (b)
13. (b) 14. (b) 15. (b, d) 16. (a, b)
Topic 3
1. (b) 2. (a,b,c,d)
17. 16x 2 + y 2 + 10 xy = 2 18. (b)
x 2
y 2
(x + y 2 )2
2

Topic 2 3. − =
9 4 81
1. (a) 2. (c) 3. (c) 4. (c)

Hints & Solutions


Topic 1 Equation of Hyperbola and and given directrix is 5x + 9 = 0 ⇒ x = − 9 / 5
Focal Chord   5 
So, corresponding focus is  − 3   , 0 = (− 5, 0)
1 Equation of given parabola y = 12x 2
… (i)   3 

and hyperbola 8x − y = 82 2
… (ii) 3. Let the equation of hyperbola is
Now, equation of tangent to parabola y = 12x having
2 x2 y2
− =1 …(i)
3 a 2 b2
slope ‘m’ is y = mx + … (iii)
m Since, equation of given directrix is 5x = 4 5
and equation of tangent to hyperbola  a a
so 5  = 4 5 [Q equation of directrix is x = ]
 e e
x2 y2
− = 1 having slope ‘m’ is a 4
1 8 ⇒ = …(ii)
e 5
y = mx ± 12m2 − 8 …(iv)
and hyperbola (i) passes through point (4, − 2 3 )
Since, tangents (iii) and (iv) represent the same line
2 16 12
 3 so, − =1 …(iii)
∴ m2 − 8 =   a 2 b2
 m
⇒ m − 8m2 − 9 = 0
4 b2
The eccentricity e = 1 +
⇒ (m − 9) (m2 + 1) = 0
2
a2
⇒ m = ± 3.
b2
⇒ e2 = 1 +
Now, equation of common tangents to the parabola (i) a2
and hyperbola (ii) are y = 3x + 1 and y = − 3x − 1
⇒ a 2e2 − a 2 = b2 …(iv)
Q Point ‘P’ is point of intersection of above common
From Eqs. (ii) and (iv), we get
tangents,
16 4 16 2
∴ P(− 1 / 3, 0) e − e = b2 …(v)
and focus of hyperbola S(3, 0) and S′ (− 3, 0). 5 5
From Eqs. (ii) and (iii), we get
PS 3 + 1 / 3 10 5
Thus, the required ratio = = = = 16 12 5 12
PS′ 3 − 1 / 3 8 4 − =1 ⇒ 2 − 2 =1
16 2 b2 e b
e
2. Equation of given hyperbola is 5
16x2 − 9 y2 = 144 12 5 12 5 − e2
⇒ = 2 −1 ⇒ 2 =
x2 y2 b 2
e b e2
⇒ − =1 …(i)
9 16 12e2
So, the eccentricity of Eq. (i) ⇒ b2 = …(vi)
5 − e2
16 5
e= 1+ = From Eqs. (v) and (vi), we get
9 3  12e2 
16e4 − 16e2 = 5  ⇒ 16(e2 − 1)(5 − e2 ) = 60
x2 y2  5 − e2
[Q the eccentricity (e) of the hyperbola − = 1 is
a 2 b2
⇒ 4(5e2 − e4 − 5 + e2 ) = 15
1 + (b / a )2 ]
⇒ 4e4 − 24e2 + 35 = 0
rankershalt.com

454 Hyperbola

4. The vertices of hyperbola are given as (± 2, 0) and one of 7. Equation of hyperbola is given by
its foci is at (− 3, 0). x2 y2
− =1
∴ (a , 0) = (2, 0) and (− ae, 0), = (− 3, 0) a 2 b2
On comparing x-coordinates both sides, we get Q Length of transverse axis = 2a = 4
⇒ a = 2 and − ae = − 3 ∴ a=2
3 x2 y2
⇒ 2e = 3 ⇒ e = Thus, − = 1 is the equation of hyperbola
2 4 b2
9 b2  2 b2  Q It passes through (4, 2).
Also, =1 + ⇒ b2 = 5 Q e = 1 + 2 16 4 4 4 2
4 4  a  ∴ − 2 = 1 ⇒ 4 − 2 = 1 ⇒ b2 = ⇒ b =
4 b b 3 3
So, equation of the hyperbola is Now, eccentricity,
x2 y2 4
− =1 …(i)
4 5 b2 1 2
e= 1+ 2 = 1+ 3 = 1+ =
The point (6, 5 2 ) from the given options does not a 4 3 3
satisfy the above equation of hyperbola. x2 y2
2 2 8. For the hyperbola − = 1,
x y a 2 b2
5. We know that in − = 1, where b2 = a 2 (e2 − 1), the
a 2 b2 b2
length of conjugate axis is 2b and distance between the e= 1+ 2
a
foci is 2ae.
∴ For the given hyperbola,
∴According the problem, 2b = 5 and 2ae = 13
sin 2 θ
Now, b2 = a 2 (e2 − 1) e= 1+ >2
2 cos 2 θ
 5
⇒   =a e −a
2 2 2
(Q a 2 = cos 2 θ and b2 = sin 2 θ)
 2
⇒ 1 + tan 2 θ > 4
25 (2ae)2 ⇒ tan 2 θ > 3
⇒ = − a2
4 4 ⇒ tan θ ∈ (− ∞ , − 3 ) ∪ ( 3 , ∞ )
25 169
⇒ = − a2 [Q 2ae = 13] [x2 > 3 ⇒|x| > 3 ⇒ x ∈ (−∞ , − 3 ) ∪ ( 3 , ∞ )]
4 4
 π
169 − 25 144 But θ ∈ 0,  ⇒ tan θ ∈ ( 3 , ∞ )
⇒ a 2= = = 36  2
4 4
 π π
⇒ a =6 ⇒ θ ∈ , 
 3 2
Now, 2ae = 13
Now, length of latusrectum
⇒ 2 × 6 × e = 13 2b2 sin 2 θ
13 = =2 = 2 sin θ tan θ
⇒ e= a cos θ
12 Since, both sin θ and tan θ are increasing functions in
 y2 x2   π π
6. Given, S = (x, y) ∈ R2: − = 1  , .
1+ r 1−r  3 2
 
  ∴ Least value of latusrectum is
y2 x2
= (x, y) ∈ R2 : + = 1 π π 3  π
 1 + r r −1  = 2 sin ⋅ tan = 2 ⋅ ⋅ 3 =3 at θ = 
3 3 2  3
y2 x2 and greatest value of latusrectum is < ∞
For r > 1, + = 1, represents a vertical ellipse.
1 + r r −1
Hence, latusrectum length ∈ (3, ∞ )
[Q for r > 1, r − 1 < r + 1 and r − 1 > 0]
2b2
r −1 9. We have, = 8 and 2b = ae
Now, eccentricity (e) = 1 − a
r+1
⇒ b2 = 4a and 2b = ae
 2 2 2
x y a Consider, 2b = ae
Q For 2 + 2 = 1, a < b, e = 1 − 2 
 a b b  ⇒ 4b2 = a 2e2
(r + 1) − (r − 1) ⇒ 4a 2 (e2 − 1) = a 2e2
= ⇒ 4e2 − 4 = e2 [Q a ≠ 0]
r+1
⇒ 3 e2 = 4
2
= 2
r+1 ⇒ e= [Q e > 0]
3
rankershalt.com

Hyperbola 455

10. Given equation can be rewritten as focal chord ⇒ Foci = (± 1, 0)


(x − 2 )2 ( y + 2 )2 where, vertices are (± cos α , 0).
− =1
4 2 1
Eccentricity, ae = 1 or e =
2 3 cos α
For point A (x, y), e = 1 + =
4 2 Hence, foci remains constant with change in α.
⇒ x− 2 =2 x2 y2
14. Given equation is − = 1 , where| r | < 1
⇒ x=2 + 2 1−r 1+ r
⇒ 1 − r is (+ ve) and 1 + r is (+ ve)
Y
x2 y2
Conjugate ∴Given equation is of the form 2 − 2 = 1.
axis
B
a b
transverse
axis Hence, it represents a hyperbola when| r | < 1.
X′ X x2 y2
A C 15. Here, equation of ellipse is + =1
4 1
2
Y′ b 1 3
⇒ e2 = 1 − 2 = 1 − =
For point C (x, y), x − 2 = ae = 6 ⇒ x = 6 + 2 a 4 4
AC = 6 + 2 − 2 − 2 = 6 − 2 3
Now, ∴ e= and focus (± a e, 0) ⇒ (± 3 , 0)
2 2
b 2
and BC = = =1 x2 y2 b2
a 2 For hyperbola 2
− 2 = 1, e12 = 1 + 2
1 3 a b a
∴ Area of ∆ ABC = × ( 6 − 2) × 1 = − 1 sq unit
2 2 1 4 b2 4
where, e12 = = ⇒ 1+ =
11. The given ellipse is e2 3 a2 3
x2 y2 b2 1
+ = 1 ⇒ a = 2, b = 3 ∴ = …(i)
4 3 a2 3
1 and hyperbola passes through (± 3, 0)
∴ 3 = 4 (1 − e2 ) ⇒ e =
2 3
1 ⇒ = 1 ⇒ a2 = 3 …(ii)
So, ae=2 × =1 a2
2
From Eqs. (i) and (ii), b2 = 1 …(iii)
Hence, the eccentricity e1 of the hyperbola is given by 2 2
x y
e1 = cosec θ [Q ae = e sin θ ] ∴ Equation of hyperbola is − =1
3 1
⇒ b2 = sin 2 θ (cosec2θ − 1) = cos 2 θ  2 
Hence, equation of hyperbola is Focus is (± a e1 , 0) ⇒  ± 3 ⋅ , 0 ⇒ (± 2, 0)
 3 
x2 y2
− = 1 or x2 cosec2θ − y2sec2θ = 1. Hence, (b) and (d) are correct answers.
sin θ cos 2 θ
2
16. Given, 2x2 − 2 y2 = 1
x2 y2
12. The eccentricity of + = 1 is x2 y2
16 25 ⇒ − =1 ... (i)
16 3  1  1
e1 = 1 − =    
25 5  2  2
5 Eccentricity of hyperbola = 2
∴ e2 = [Q e1 e2 = 1]
3 So, eccentricity of ellipse = 1 / 2
⇒ Foci of ellipse (0, ± 3)
x2 y2 Let equation of ellipse be
⇒ Equation of hyperbola is − = − 1.
16 9 x2 y2
+ =1 [where a > b]
x2 y2 a 2 b2
13. Given equation of hyperbola is − =1
cos α sin 2 α
2 1 b2
∴ = 1− 2
Here, a 2 = cos 2 α and b2 = sin 2 α 2 a
x2 y2 b2 1
[i.e. comparing with standard equation − = 1] ⇒ = ⇒ a 2 = 2b2
a 2 b2 a2 2
We know that, foci = (± ae, 0) ⇒ x2 + 2 y2 = 2b2 ... (ii)
where, ae = a + b = cos α + sin α = 1
2 2 2 2
rankershalt.com

456 Hyperbola

Let ellipse and hyperbola intersect at ∴ Coordinates of P are


 1 1   1 1
A sec θ , tan θ  2⋅ + 1⋅ 
 2 2  2 t + t t t
 1 2
, 1 2
= (h , k) [say]
On differentiating Eq. (i), we get  2+1 2+1 
 
dy dy x  
4x − 4 y =0 ⇒ =
dx dx y 2t1 + t2 2t + t
∴h= and k = 2 1 ...(i)
dy sec θ 3 3t1 t2
= = cosec θ
dx at A tan θ  1  1
Also,  t1 ,  and  t2 ,  lie on y = 4 x + c.
and on differentiating Eq. (ii), we get  t1   t2
dy dy x 1 1 1
2x + 4 y =0 ⇒ =− = − cosec θ −
dx dx at A 2y 2 t2 t1 1
⇒ = =4 or t1 t2 = − 1 4 ... (ii)
Since, ellipse and hyperbola are orthogonal. t2 − t1 t1 t2
1 π k
∴ − cosec2 θ = − 1 ⇒ cosec2 θ = 2 ⇒ θ = ± From Eq. (i), t1 = 2h +
2 4 4
 1  1
∴ A 1 ,  or 1 , −  t1 = h −
k
 2  2 and ...(iii)
2
2
 1  k  k 1
Form Eq. (ii), 1 + 2   = 2b2 From Eqs. (ii) and (iii), − h −  2h +  = −
 2  2  4 4
⇒ b2 = 1  2h + k  8h + k 1
⇒ −    =−
Equation of ellipse is x + 2 y2 = 2.
2  2   4  4
 1  ⇒ ( 2h + k) ( 8h + k) = 2
Coordinates of foci (± ae , 0) =  ± 2 ⋅ , 0 = (± 1 , 0)
 2  ⇒ 16h 2 + k2 + 10hk = 2
If major axis is along Y-axis, then Hence, required locus is 16x2 + y2 + 10xy = 2.
1 a2 18. We have,
= 1 − 2 ⇒ b2 = 2a 2
2 b Equation of hyperbola
2x
∴ 2x2 + y2 = 2a 2 ⇒ Y ′ = − x2 y2
y − =1
−2 a 2 b2
⇒ y′ 
= Y

1
sec θ ,
1
tan θ  sin θ
 2 2  L (0,–b) x2 y2
– 2 =1
a2 b
As ellipse and hyperbola are orthogonal b
2 X′ 60° X
∴ − ⋅ cosec θ = − 1 O N(a,0)
sin θ b
1 π
⇒ cosec2 θ = ⇒ θ=± M (0,–b)
2 4
Y′
∴ 2x2 + y2 = 2a 2
1 5 It is given,
⇒ 2 + = 2a 2 ⇒ a 2 =
2 4 ∠ LNM = 60°
5 Area of ∆LMN = 4 3
∴ 2x + y = , corresponding foci are (0, ±1).
2 2 and
2 Now, ∆LNM is an equilateral triangle whose sides is 2b
Hence, option (a) and (b) are correct. ~ ∆MOL ; ∴ ∠NLO = ∠NMO = 60°]
[Q ∆LON =
17. Let y = 4x + c meets xy = 1 at two points A and B. 3
Y ∴ Area of ∆LMN = ( 2b)2
y = 4x + c 4
2:1 A ⇒ 4 3 = 3b2 ⇒ b = 2
P 1
xy = 1
X′ X
Also, area of ∆LMN = a (2b) = ab
2
B
⇒ 4 3 = a(2) ⇒ a = 2 3
(P) Length of conjugate axis = 2b = 2(2)= 4
Y′ b2 4 4 2
(Q) Eccentricity (e) = 1 + = 1+ = =
i.e. A (t1 , 1 t1 ), B (t2 , 1 t2 ) a2 12 2 3 3
rankershalt.com

Hyperbola 457

2 2
(R) Distance between the foci = 2ae = 2 × 2 3 × =8 ⇒ 5m2 = 4 ⇒ m = ±
3 5
2b2 2(4) 4 3. Let the equation of standard hyperbola is
(S) The length of latusrectum = = =
a 2 3 3 x2 y2
P → 4; Q → 3; R → 1; S → 2 − =1 …(i)
a 2 b2
Now, eccentricity of hyperbola is
Topic 2 Equation of Tangent and Normal b2
1+ =2 (given)
a2
1. Key Idea An equation of tangent having slope ⇒ a 2 + b2 = 4a 2
a
‘m’ to parabola y = 4 ax is y = mx +
2
. ⇒ b2 = 3a 2 …(ii)
m
Since, hyperbola (i) passes through the point (4, 6)
Given equation of curves are 16 36
y2 = 16x (parabola) …(i) ∴ − =1 …(iii)
a 2 b2
and xy = − 4 (rectangular hyperbola) …(ii)
On solving Eqs. (ii) and (iii), we get
Clearly, equation of tangent having slope ‘m’ to parabola
4 a 2 = 4 and b2 = 12 …(iv)
(i) is y = mx + …(iii)
m Now, equation of tangent to hyperbola (i) at point (4, 6),
Now, eliminating y from Eqs. (ii) and (iii), we get is
 4 4 4x 6 y
x mx +  = − 4 ⇒ mx2 + x + 4 = 0, − =1
 m m a 2 b2
which will give the points of intersection of tangent and 4x 6 y
⇒ − =1 [from Eq. (iv)]
rectangular hyperbola. 4 12
4
Since, line y = mx + is also a tangent to the y
m ⇒ x − = 1 ⇒ 2x − y − 2 = 0
rectangular hyperbola. 2
4 4. Given equation of hyperbola is
∴Discriminant of quadratic equation mx2 + x + 4 = 0,
m 4x2 − 5 y2 = 20
should be zero. which can be rewritten as
[Q there will be only one point of intersection]
x2 y2
 4
2 ⇒ − =1
⇒ D =   − 4 (m) (4) = 0 5 4
 m
The line x − y = 2 has slope, m = 1
⇒ m3 = 1 ⇒ m = 1
∴ Slope of tangent parallel to this line = 1
So, equation of required tangent is y = x + 4. x2 y2
2 2 We know equation of tangent to hyperbola − =1
2. Given equation of hyperbola, is
x

y
=1 …(i) a 2 b2
24 18 having slope m is given by
Since, the equation of the normals of slope m to the y = mx ± a 2m2 − b2
x2 y2 m(a 2 + b2 )
hyperbola 2 − 2 = 1, are given by y = mx m Here, a 2 = 5, b2 = 4 and m = 1
a b a 2 − b2m2 ∴Required equation of tangent is
∴ Equation of normals of slope m, to the hyperbola (i), ⇒ y= x± 5 −4
are
⇒ y = x ± 1 ⇒ x − y ± 1 =0
m(24 + 18)
y = mx ± …(ii) 5. Tangents are drawn to the hyperbola 4x2 − y2 = 36 at
24 − m2 (18)
the point P and Q.
Q Line y = mx + 7 3 is normal to hyperbola (i) Tangent intersects at point T (0, 3)
∴On comparing with Eq. (ii), we get
m(42) Y
± =7 3 T (0, 3)
24 − 18m2
6m
⇒ ± = 3
24 − 18m2
X
36m2 O
⇒ = 3 [squaring both sides]
24 − 18m2
⇒ 12m2 = 24 − 18m2 (–3√5, –12)Q P(3√5, –12)
S(0, –12)
⇒ 30m2 = 24
rankershalt.com

458 Hyperbola

Clearly, P Q is chord of contact. Slope for normal at the point (a sec θ , b tan θ ) will be
∴Equation of PQ is −3 y = 36 a 2b tan θ a
− = − sin θ
⇒ y = − 12 b2a sec θ b
Solving the curve 4x2 − y2 = 36 and y = − 12 ,
∴ Equation of normal at (a sec θ , b tan θ ) is
we get x= ±3 5 a
1 1 y − b tan θ = − sin θ (x − a sec θ )
Area of ∆PQT = × PQ × ST = (6 5 × 15) = 45 5 b
2 2
⇒ (a sin θ ) x + by = (a 2 + b2 ) tan θ
x2 y2
6. Let the equation of hyperbola be 2 − 2 = 1. ⇒ a x + b cosecθ = (a 2 + b2 ) sec θ …(i)
a b
x2 y2
∴ ae = 2 ⇒ a 2e2 = 4 Similarly, equation of normal to 2 − 2 = 1 at
⇒ a 2 + b2 = 4 ⇒ b2 = 4 − a 2 a b
(a sec φ, b tan φ ) is ax + b y cosec φ = (a 2 + b2 ) sec φ …(ii)
x2 y2
∴ − =1 On subtracting Eq. (ii) from Eq. (i), we get
a 2
4 − a2
b (cosec θ − cosec φ ) y = (a 2 + b2 ) (sec θ − sec φ )
Since, ( 2 , 3 ) lie on hyperbola.
2 3 a 2 + b2 sec θ − sec φ
∴ − =1 ⇒ y= ⋅
a2 4 − a2 b cosec θ − cosec φ
⇒ 8 − 2a 2 − 3a 2 = a 2 (4 − a 2 ) sec θ − sec φ sec θ − sec (π / 2 − θ )
But =
⇒ 8 − 5a 2 = 4a 2 − a 4 cosec θ − cosec φ cosec θ − cosec (π / 2 − θ )
⇒ a − 9a 2 + 8 = 0
4
[Q φ + θ = π / 2]
⇒ (a − 8)(a 4 − 1) = 0 ⇒ a 4 = 8, a 4 = 1
4
sec θ − cosec θ
= = −1
∴ a =1 sec θ − sec θ
x2 y2  a 2 + b2  a 2 + b2
Now, equation of hyperbola is − = 1.
1 3 Thus, y=−  , i.e. k = −  
 b   b 
∴ Equation of tangent at ( 2 , 3 ) is given by
3y y 10. Tangent ≡ 2x − y + 1 = 0
2x − = 1 ⇒ 2x − =1
3 3 x2 y2
Hyperbola ≡ − =1
which passes through the point (2 2 , 3 3 ). a 2 16
7. Equation of normal to hyperbola at (x1 , y1 ) is It point ≡ (a sec θ , 4 tan θ ),
x sec θ y tan θ
a 2x b2 y tangent ≡ − =1
+ = (a 2 + b2 ) a 4
x1 y1
On comparing, we get secθ = − 2a
a 2x b2 y tan θ = − 4 ⇒ 4a 2 − 16 = 1
∴ At (6, 3) = + = (a 2 + b2 )
6 3 17
∴ a=
a2⋅ 9 2
Q It passes through (9, 0). ⇒ = a 2 + b2
6 Substitute the value of a in option (a), (b), (c) and (d).
3a 2 a2 11.
⇒ − a 2 = b2 ⇒ =2 Y
b2
)

2
1
,y

2
1
x

b 1 3
P(

∴ e2 = 1 + 2 = 1 + ⇒ e=
a 2 2
M
8. The equation of tangent at (x1 , y1 ) is x x1 − 2 y y1 = 4, X′ X
(–1, 0) N(x2, 0)
(1, 0)
which is same as 2x + 6 y = 2.
x1 2y 4 x2 – y2 = 1
∴ =− 1 =
2 6 2
Y′
⇒ x1 = 4 and y1 = − 6
Equation of family of circles touching hyperbola at
Thus, the point of contact is (4, − 6 ). (x1 , y1 ) is (x − x1 )2 + ( y − y1 )2 + λ( x x1 − y y1 −1) = 0
x2 y2 Now, its centre is (x2 , 0).
9. Firstly, we obtain the slope of normal to − = 1 at
a 2 b2  − (λx1 − 2x1 ) – (−2 y1 − λy1 ) 
∴ , = (x2 , 0)
(a sec θ , b tan θ ). On differentiating w.r.t. x, we get  2 2 
2x 2 y dy dy b2 x ⇒ 2 y1 + λy1 = 0 ⇒ λ = − 2
− × = 0 ⇒ =
a 2 b2 dx dx a 2 y and 2x1 − λx1 = 2x2 ⇒ x2 = 2x1
rankershalt.com

Hyperbola 459

∴ P (x1 , x12 − 1 ) and N (x2 , 0) = (2x1 , 0) 9 1


or x1 = , y1 =
2 2 2
1 
As tangent intersect X-axis at M  ,0 .
 x1  x2 y2
13. The equation of the hyperbola is − = 1 and that of
Centroid of ∆PMN = (l, m) 9 4
circle is x2 + y2 − 8x = 0
 1 
 3x1 +  x2 x2 − 8x
 x y + 0 + 0  = (l, m) For their points of intersection, + =1
⇒ 1
, 1 9 4
 3 3 
  ⇒ 4x2 + 9x2 − 72x = 36
 
1 ⇒ 13x2 − 72x − 36 = 0
3x1 +
x1 ⇒ 13x2 − 78x + 6x − 36 = 0
⇒ l=
3 ⇒ 13x (x − 6) + 6 (x − 6) = 0
1 13
3− 2 ⇒ x=6, x= −
dl x1 6
On differentiating w.r.t. x1 , we get =
dx1 3 13
x=− not acceptable.
dl 1 x12 − 1 6
⇒ = 1 − 2 ,for x1 > 1 and m =
dx1 3x1 3 Now, for x = 6, y = ± 2 3
On differentiating w.r.t. x1 , we get Required equation is, (x − 6)2 + ( y + 2 3 ) ( y − 2 3 ) = 0
dm 2x1 x1 ⇒ x2 − 12x + y2 + 24 = 0
= = , for x1 > 1
dx1 2 × 3 x12 − 1 3 x12 − 1 ⇒ x2 + y2 − 12x + 24 = 0
y 14. Equation of tangent to hyperbola having slope m is
Also, m= 1
3
y = mx + 9m2 − 4 ...(i)
dm 1
On differentiating w.r.t. y1 , we get = , for y1 > 0 Equation of tangent to circle is
dy1 3
2 2 y = m (x − 4) + 16m2 + 16 ...(ii)
12. PLAN Equation of tangent to x 2 − y 2 = 1 is y = mx ± a 2m2 − b 2
a b 2
Eqs. (i) and (ii) will be identical for m = satisfy.
Description of Situation If two straight lines 5
a1 x + b1 y + c1 = 0 ∴ Equation of common tangent is 2x − 5 y + 4 = 0.

15. On substituting  , 0 in y = − 2x + 1, we get


a1 b1 c1 a
and a 2x + b2 y + c2 = 0 are identical. Then, = =
a 2 b2 c2 e 
Equation of tangent, parallel to y = 2x − 1 is 2a
0=− +1
y = 2x ± 9 (4) − 4 e
a 1
∴ y = 2x ± 32 …(i) ⇒ =
e 2
The equation of tangent at (x1 , y1 ) is
Also, y = − 2x + 1 is tangent to hyperbola.
xx1 yy1
− =1 …(ii) 1
9 4 ∴ 1 = 4a 2 − b2 ⇒ = 4 − (e2 − 1)
a2
From Eqs. (i) and (ii),
4
2x – y = 1 Tangent ⇒ = 5 − e2
Y e2
⇒ e4 − 5 e2 + 4 = 0 ⇒ (e2 − 4) (e2 − 1) = 0
P (x1, y1) ⇒ e=2, e=1
x x X
(–3, 0) O 12 (3, 0) Y
–1
x2 – y2
=1
9 4 F
X
F1
2
x1
=
−1
− y1
=
± 32 (– 1, 0
2 )
1
9 4
9 1 e = 1 gives the conic as parabola. But conic is given as
⇒ x1 = − and y1 = −
2 2 2 hyperbola, hence e = 2.
rankershalt.com

460 Hyperbola

Topic 3 Equation of Chord of Contact, and product of the roots x1 x2 x3 x4 = c4


Chord Bisected Diameter, Similarly, y1 + y2 + y3 + y4 = 0
Asymptote and Rectangular and y1 y2 y3 y4 = c4
Hyperbola Hence, all options are correct.
1. Let (h , k) be a point whose chord of contact with respect 3. Let any point on the hyperbola is (3 sec θ , 2 tan θ ).
to hyperbola x − y = 9 is x = 9.
2 2
∴ Chord of contact of the circle x2 + y2 = 9 with respect
We know that, chord of contact of (h , k) with respect to to the point (3 sec θ , 2 tanθ ) is,
hyperbola x2 − y2 = 9 is T = 0. (3 sec θ ) x + (2 tan θ ) y = 9 …(i)
⇒ h ⋅ x + k (− y) − 9 = 0 Let (x1 , y1 ) be the mid-point of the chord of contact.
∴ hx − ky − 9 = 0 ⇒ Equation of chord in mid-point form is
But it is the equation of the line x = 9. x x1 + yy1 = x12 + y12 …(ii)
This is possible when h = 1, k = 0 (by comparing both Since, Eqs. (i) and (ii) are identically equal.
equations). 3 sec θ 2 tanθ
Again equation of pair of tangents is ∴ =
x1 y1
T 2 = SS1 9
⇒ (x − 9)2 = (x2 − y2 − 9) (12 − 02 − 9) =
x12 + y12
⇒ x2 − 18x + 81 = (x2 − y2 − 9) (−8) 9x1
⇒ sec θ =
⇒ x2 − 18x + 81 = − 8x2 + 8 y2 + 72 3 (x12 + y12 )
⇒ 9x2 − 8 y2 − 18x + 9 = 0 9 y1
and tanθ =
2. It is given that, 2 (x12 + y12 )
x2 + y2 = a 2 …(i) Thus, eliminating ‘ θ ’ from above equation, we get
and xy = c
2
…(ii) 81 x12 81 y12
− =1
We obtain x + c /x = a
2 4 2 2
9 (x1 + y1 )
2 2 2
4 (x12 + y12 )2
⇒ x4 − a 2x2 + c4 = 0 …(iii) [Q sec2 θ − tan 2 θ = 1]
Now, x1 , x2 , x3 , x4 will be roots of Eq. (iii). x2 y2 (x2 + y2 )2
Therefore, Σ x1 = x1 + x2 + x3 + x4 = 0 ∴ Required locus is − = .
9 4 81

Download Chapter Test


http://tinyurl.com/y2khzcbp or
rankershalt.com

20
Trigonometrical Ratios
and Identities

Topic 1 Based on Trigonometric Formulae


Objective Questions I (Only one correct option) (a) 13 − 4 cos4 θ + 2 sin 2 θ cos2 θ
(b) 13 − 4 cos2 θ + 6 cos4 θ
1. The value of sin 10º sin 30º sin 50º sin 70º is
(c) 13 − 4 cos2 θ + 6 sin 2 θ cos2 θ
(2019 Main, 9 April II)
1 1 1 1 (d) 13 − 4 cos6 θ
(a) (b) (c) (d)
36 32 16 18 tan A cot A
8. The expression + can be written as
2. The value of cos 10° − cos 10° cos 50° + cos 50° is
2 2 1 − cot A 1 − tan A (2013 Main)
(2019 Main, 9 April I) (a) sin A cos A + 1 (b) sec A cosec A + 1
(a)
3
(1 + cos 20° ) (b)
3
+ cos 20° (c) tan A + cot A (d) sec A + cosec A
2 4 9. The number of ordered pairs (α , β), where α , β ∈ (−π , π )
(c) 3 / 2 (d) 3 / 4 1
satisfying cos (α − β ) = 1 and cos (α + β ) = is
3. If the lengths of the sides of a triangle are in AP and the e (2005, 1M)
greatest angle is double the smallest, then a ratio of (a) 0 (b) 1 (c) 2 (d) 4
lengths of the sides of this triangle is (2019 Main, 8 April II)
(a) 3 : 4 : 5 (b) 4 : 5 : 6 (c) 5 : 9 : 13 (d) 5 : 6 : 7
10. Given both θ and φ are acute angles and
1 1
3 5 π sin θ = , cos φ = , then the value of θ + φ belongs to
4. If cos(α + β) = , sin(α − β) = and 0 < α , β < , then 2 3 (2004, 1M)
5 13 4
π π π 2π
tan(2α ) is equal to (2019 Main, 8 April I) (a)  ,  (b)  , 
63 63 21 33  3 6  2 3
(a) (b) (c) (d)
52 16 16 52 2π 5π  5π 
(c)  , (d)  ,π
1
5. Let fk (x) = (sin k x + cos k x) for k = 1, 2, 3 ... . Then, for  3 6   6 
k
11. Which of the following numbers is rational? (1998, 2M)
all x ∈ R, the value of f4 (x) − f6 (x) is equal to
(2019 Main, 11 Jan I) (a) sin 15° (b) cos 15°
1 5 −1 1 (c) sin 15° cos 15° (d) sin 15° cos 75°
(a) (b) (c) (d)
12 12 12 4 12. 3 (sin x − cos x)4 + 6 (sin x + cos x)2 + 4 (sin 6 x + cos 6 x)
6. The value of equals (1995, 2M)
π π π π (a) 11 (b) 12 (c) 13 (d) 14
cos 2 ⋅ cos 3 ....... cos 10 ⋅ sin 10 is
2 2 2 2 (2019 Main, 10 Jan II)
13. The value of the expression 3 cosec 20° − sec 20° is
1 1 1 1
(a) (b) (c) (d) equal to (1988, 2M)
1024 2 512 256
(a) 2
π π
7. For any θ ∈  ,  , the expression (b) 2 sin 20° /sin 40°
 4 2
(c) 4
3 (sin θ − cos θ )4 + 6 (sin θ + cos θ )2 + 4 sin 6 θ equals (d) 4 sin 20° /sin 40°
(2019 Main, 9 Jan I)
rankershalt.com

462 Trigonometrical Ratios and Identities

14. The expression 21. For a positive integer n, let


  3π    θ
3 sin 4  − α  + sin 4 (3π + α ) fn (θ ) =  tan  (1 + sec θ )(1 + sec 2 θ )
  2    2
 6 π   (1 + sec 22 θ )... (1 + sec 2nθ ), then
−2 sin  + α  + sin 6 (5π − α ) (1999, 3M)
  2   π π
(a) f2   = 1 (b) f3   = 1
is equal to (1986, 2M)  16   32 
π π 
(a) 0 (b) 1 (c) f4   = 1 (d) f5   =1
(c) 3 (d) sin 4α + cos 6 α  64   128 
π 3π 5π 7π
15. 1 + cos  1 + cos  1 + cos  1 + cos  is Match the Column
 8  8  8 8
equal to (1984, 3M) Match the conditions/expressions in Column I with values
1 π in Column II.
(a) (b) cos
2 8
22. (sin 3 α ) / (cos 2 α ) is (1992, 2M)
1 1+ 2
(c) (d)
8 2 2
Column I Column II
16. Given A = sin 2 θ + cos 4 θ , then for all real values of θ A. positive p. (13 π / 48, 14 π / 48)
(1980, 1M)
3 B. negative q. (14 π / 48, 18 π / 48)
(a) 1 ≤ A ≤ 2 (b) ≤ A≤1
4 r. (18 π / 48, 23 π / 48)
13 3 13
(c) ≤ A≤1 (d) ≤ A ≤ s. (0, π / 2)
16 4 16
4
17. If tan θ = − , then sin θ is
3 (1978, 2M)
4 4 4 4 Fill in the Blanks
(a) − but not (b) − or
π  5π   7π 
5 5 5 5 23. If k = sin   sin   sin   , then the numerical
4
(c) but not −
4
(d) None of the above  18  18   18 
5 5 value of k is …… (1993, 2M)

Objective Questions II 24. The value of


π 3π 5π 7π 9π 11π 13π
sin ⋅ sin ⋅ sin ⋅ sin ⋅ sin ⋅ sin ⋅ sin
(One or more than one correct option) 14 14 14 14 14 14 14
2 is equal to …… . (1991, 2M)
18. Let f : (−1, 1) → R be such that f (cos 4 θ ) = for
2 − sec2 θ
 π  π π  1 Analytical & Descriptive Questions
θ ∈  0 ,  ∪  ,  . Then, the value(s) of f   is/are
 4  4 2  3 25. Prove that
(2012)
3 3 tan α + 2 tan 2α + 4 tan 4α + 8 cot 8α = cot α (1988, 2M)
(a) 1 − (b) 1 +  2π   4π   8π   16π 
2 2 26. Show that 16 cos   cos   cos   cos   =1
 15   15   15   15 
2 2
(c) 1 − (d) 1 + (1983, 2M)
3 3
π
27. Without using tables, prove that
19. For 0 < θ < , the solution(s) of 1
2 (sin 12° ) (sin 48° ) (sin 54° ) = .
6
 (m − 1) π   mπ  8 (1982, 2M)
∑ cosec θ + 4  cosec θ + 4  = 4 2 is/are 28. Prove that sin 2 α + sin 2 β – sin 2 γ = 2 sinα sin β sin γ ,
m =1 (2009)
π π where α + β + γ = π . (1978, 4M)
(a) (b)
4 6
(c)
π
(d)
5π Integer Answer Type Question
12 12 29. The number of all possible values of θ, where 0 < θ < π,
sin 4 x cos 4 x 1 for which the system of equations
20. If + = , then
2 3 5 (2009) ( y + z ) cos 3 θ = ( xyz ) sin 3 θ
(a) tan 2 x =
2 sin 8 x cos8 x
+ =
1 2 cos 3 θ 2 sin 3 θ
(b) x sin 3 θ = + `
3 8 27 125 y z
1 sin 8 x cos8 x 2
(c) tan x =
2
(d) + = and (xyz )sin 3 θ = ( y + 2z ) cos 3 θ + ysin 3 θ have a
3 8 27 125
solution (x0 , y0 , z0 ) with y0z0 ≠ 0, is …… (2010)
rankershalt.com

Trigonometrical Ratios and Identities 463

Topic 2 Graph and Conditional Identities


Objective Questions I (Only one correct option) True/False
π 4. If tan A = (1 − cos B) / sin B, then tan 2 A = tan B.
1. If α + β = and β + γ = α , then tan α equals (2001, 1M)
2 (1993, 1M)
(a) 2 (tan β + tan γ ) (b) tan β + tan γ
(c) tan β + 2 tan γ (d) 2 tan β + tan γ Analytical & Descriptive Questions
2. If α + β + γ = 2π ,then (1979, 2M) 5. In any triangle, prove that
α β γ α β γ A B C A B C
(a) tan + tan + tan = tan tan tan cot + cot + cot = cot cot cot
2 2 2 2 2 2 2 2 2 2 2 2 (2000, 3M)
α β β γ γ α
(b) tan tan + tan tan + tan tan = 1 6. ABC is a triangle such that
2 2 2 2 2 2
α β γ α β γ
sin( 2 A + B) = sin(C − A) = − sin( B + 2C ) = 1/ 2.
(c) tan + tan + tan = − tan tan tan
2 2 2 2 2 2 If A, B and C are in arithmetic progression, determine
the values of A , B and C. (1990, 5M)
(d) None of the above
7. Given α + β + γ = π, prove that
Fill in the Blank sin 2 α + sin 2 β − sin 2 γ = 2 sin α sin β cos γ. (1980, 3M)
n
8. If A + B + C = 180°, then prove that
3. Suppose sin3 x sin 3 x = ∑ Cm cos nx is an identity in x, tan A + tan B + tan C = tan A tan B tan C. (1979, 3M)
m= 0
where C 0 , C1 , K , C n are constants and C n ≠ 0. Then, the 4 5
9. If cos (α + β) = , sin (α − β) = and α , β lie between 0
value of n is… . (1981, 2M) 5 13
and π /4, find tan 2 α. (1979, 4M)

Topic 3 Maxima and Minima


Objective Question I (Only one correct option) (a) t1 > t2 > t3 > t4 (b) t4 > t3 > t1 > t2
(c) t3 > t1 > t2 > t4 (d) t2 > t3 > t1 > t4
1 The maximum value of
 π
3 cos θ + 5 sin θ −  Fill in the Blank
 6
for any real value of θ is (2019 Main, 12 Jan I) 3. If A > 0, B > 0 and A + B = π /3, then the maximum value
79 of tan A tan B is ........... (1993)
(a) (b) 34
2
(c) 31 (d) 19 Analytical & Descriptive Question
π sin x cos 3x
2. Let θ ∈ 0,  and t1 = (tan θ )tan θ , t2 = (tan θ )cot θ , 4. Prove that the values of the function do not
 4 sin 3x cos x
t3 = (cot θ )tan θ and t4 = (cot θ )cot θ , then (2006, 3M) lie between 1 / 3 and 3 for any real x. (1997, 5M)

Topic 4 Height & Distance


1. The angle of elevation of the top of a vertical tower ground when the top of the ladder is 1 m above the
standing on a horizontal plane is observed to be 45° ground is (2019 Main, 12 April I)
from a point A on the plane. Let B be the point 30 m 25 25
(a) 25 3 (b) (c) (d) 25
vertically above the point A. If the angle of elevation of 3 3
the top of the tower from B be 30°, then the distance (in
3. ABC is a triangular park with AB = AC = 100 m. A
m) of the foot of the tower from the point A is
(2019 Main, 12 April II) vertical tower is situated at the mid-point of BC. If the
(a) 15 (3 + 3 ) (b) 15 (5 − 3 ) angles of elevation of the top of the tower at A and B are
(c) 15 (3 − 3 ) (d) 15 (1 + 3 ) cot−1 (3 2 ) and cosec−1 (2 2 ) respectively, then the
height of the tower (in m) is (2019 Main, 10 April I)
2. A 2 m ladder leans against a vertical wall. If the top of
(a) 25 (b) 20
the ladder begins to slide down the wall at the rate 25 100
cm/s, then the rate (in cm/s) at which the bottom of the (c) 10 5 (d)
3 3
ladder slides away from the wall on the horizontal
rankershalt.com

464 Trigonometrical Ratios and Identities

4. Two poles standing on a horizontal ground are of 8. PQR is a triangular park with PQ = PR = 200 m. A TV
heights 5 m and 10 m, respectively. The line joining tower stands at the mid-point of QR. If the angles of
their tops makes an angle of 15º with the ground. elevation of the top of the tower at P , Q and R are
Then, the distance (in m) between the poles, is respectively 45°, 30° and 30°, then the height of the
(2019 Main, 9 April II) tower (in m) is (2018 Main)
5
(a) 5( 3 + 1) (b) (2 + 3 ) (a) 100 (b) 50
2
(c) 100 3 (d) 50 2
(c) 10( 3 − 1) (d) 5(2 + 3 )
5. Two vertical poles of heights, 20 m and 80 m stand apart 9. Let a vertical tower AB have its end A on the level
ground. Let C be the mid-point of AB and P be a point on
on a horizontal plane. The height (in m) of the point of
the ground such that AP = 2 AB. If ∠BPC = β, then tan β
intersection of the lines joining the top of each pole to
is equal to (2017 Main)
the foot of the other, from this horizontal plane is
6 1
(2019 Main, 8 April II) (a) (b)
(a) 15 (b) 16 (c) 12 (d) 18 7 4
2 4
6. If the angle of elevation of a cloud from a point P which (c) (d)
9 9
is 25 m above a lake be 30º and the angle of depression of
reflection of the cloud in the lake from P be 60º, then the 10. A man is walking towards a vertical pillar in a straight
height of the cloud (in meters) from the surface of the path, at a uniform speed. At a certain point A on the
lake is (2019 Main, 12 Jan II) path, he observes that the angle of elevation of the top of
(a) 50 (b) 60 (c) 45 (d) 42
the pillar is 30°. After walking for 10 min from A in the
same direction, at a point B, he observes that the
7. Consider a triangular plot ABC with sides AB = 7 m, angle of elevation of the top of the pillar is 60°. Then, the
BC = 5 m and CA = 6 m. A vertical lamp-post at the time taken (in minutes) by him, from B to reach the
mid-point D of AC subtends an angle 30° at B. The pillar, is (2016 Main)
height (in m) of the lamp-post is (2019 Main, 10 Jan I) (a) 6 (b) 10
2 3 (c) 20 (d) 5
(a) 21 (b) 2 21 (c) 7 3 (d) 21
3 2

Answers
Topic 1 6. A = 45 °, B = 60 °, C = 75 °
1. (c) 2. (d) 3. (b) 4. (b) 56
9.
5. (a) 6. (c) 7. (d) 8. (b) 33
9. (b) 10. (b) 11. (c) 12. (c)
Topic 3
13. (c) 14. (b) 15. (c) 16. (b)
1
17. (b) 18. (a, b) 19. (c, d) 20. (a, b) 1. (d) 2. (b) 3.
3
21. (a, b, c, d) 22. A → r; B → p
1 1 Topic 4
23. 24. 29. 3
8 64 1. (a) 2. (b) 3. (b) 4. (d)
Topic 2 5. (b) 6. (a) 7. (a) 8. (a)
1. (c) 2. (a) 3. 6 4. True 9. (c) 10. (d)

Hints & Solutions


Topic 1 Based on Trigonometric Formulae =
1 1 1 1
sin 30° = × =
1. We have, sin 10° sin 30° sin 50° sin 70° 8 8 2 16
= sin(30° )[sin(10° )sin(50° )sin(70° )] 2. We have, cos 2 10º − cos 10º cos 50º + cos 2 50º
1
= [sin(10° )sin(60° − 10° )sin(60° + 10° )] 1
2 = [2 cos 2 10º −2 cos 10º cos 50º +2 cos 2 50º ]
1 1  2
= sin(3(10° ))
2 4  1
= [1 + cos 20º − (cos 60º + cos 40º ) + 1 + cos 100º ]
1 2
[Qsin θ sin(60° − θ )sin(60° + θ ) = sin 3 θ]
4 [Q 2 cos 2 A = 1 + cos 2 A and
2 cos A cos B = cos( A + B) + cos( A − B)]
rankershalt.com

Trigonometrical Ratios and Identities 465

1  1   1 π π π
= 2 + cos 20º + cos 100º − − cos 40º Q cos 60º = ∴ 0 <α +β <+ =
2  2  
  2  4 4 2
3  π
=
1
+ (cos 20º − cos 40º ) + cos 100º ⇒ 0 <α +β <
2 2  2
π
1 3 20º + 40º 20°− 40°  Also, − < −β < 0
= − 2 sin sin + cos 100° 4
2 2 2 2 
π π
 C + D C − D  ∴ 0 − < α −β < + 0
Q cos C − cos D = −2 sin sin 4 4
 2 2 
π π
1 3  ⇒ − < α −β <
= − 2 sin 30º sin(−10º ) + cos(90º +10º ) 4 4
2 2 
 π  π π
∴ α + β ∈ 0,  and α − β ∈  − , 
1 3   2  4 4
= + sin 10º − sin 10º [Q cos (90º + θ ) = − sin θ ]

2 2 
 π
But sin(α − β ) > 0, therefore α − β ∈ 0,  .
1 3 3  4
= × =
2 2 4 5
Now, sin(α − β ) =
3. Let a , b and c be the lengths of sides of a ∆ABC such 13
5
that a < b < c. ⇒ tan(α − β ) = …(i)
Since, sides are in AP. 12
3
∴ 2b = a + c …(i) and cos(α + β ) =
Let ∠ A =θ 5
4
Then, ∠ C = 2θ [according to the question] ⇒ tan (α + β ) = …(ii)
3
So, ∠ B = π − 3θ …(ii)
Now, tan(2α ) = tan[(α + β ) + (α − β )]
On applying sine rule in Eq. (i), we get
4 5
2 sin B = sin A + sin C +
tan(α + β ) + tan(α − β ) 3 12
⇒ 2 sin(π − 3 θ ) = sin θ + sin 2 θ [from Eq. (ii)] = =
1 − tan(α + β ) tan(α − β ) 1 − 4 × 5
⇒ 2 sin 3 θ = sin θ + sin 2 θ 3 12
⇒2 [3 sin θ − 4 sin3 θ ] = sin θ + 2 sin θ cos θ [from Eqs. (i) and (ii)]
⇒6 − 8 sin 2 θ = 1 + 2 cos θ [Qsin θ can not be zero] 48 + 15 63
= =
⇒6 − 8(1 − cos 2 θ ) = 1 + 2 cos θ 36 − 20 16
⇒ 8 cos 2 θ − 2 cos θ − 3 = 0
5. We have,
⇒(2 cos θ + 1)(4 cos θ − 3) = 0
1
3 fk (x) = (sin k x + cos k x), k = 1, 2, 3, …
⇒cos θ = k
4
1 1
orcos θ = − (rejected). ∴f4 (x) = (sin 4 x + cos 4 x)
2 4
Clearly, the ratio of sides is a : b : c 1
= ((sin 2 x + cos 2 x)2 − 2 sin 2 x cos 2 x)
= sin θ : sin 3 θ : sin 2 θ 4
= sin θ : (3 sin θ − 4 sin3 θ ) : 2 sin θ cos θ 1 1  1 1
= 1 − (sin 2x)2 = − sin 2 2x
= 1 : (3 − 4 sin 2 θ ) : 2 cos θ 4 2  4 8

= 1 : (4 cos 2 θ − 1) : 2 cos θ 1
and f6 (x) = (sin 6 x + cos 6 x)
5 6 6
= 1 : : = 4 :5 :6 1
4 4 = {(sin 2 x + cos 2 x)3 − 3 sin 2 x cos 2 x
5 6
4. Given, sin(α − β) = (sin2 x + cos2 x )}
13
3  π 1 3 2 1 1
and cos(α + β ) = , where α , β ∈ 0,  = 1 − (2 sin x cos x)  = − sin 2x
2
5  4 6 4  6 8
π π 1 1 3 −2 1
Since, 0 < α < and 0 < β < Now, f4 (x) − f6 (x) = − = =
4 4 4 6 12 12
rankershalt.com

466 Trigonometrical Ratios and Identities

6. We know that,  1 1  1 1
and 0 <  cos φ =  < as 0 < <
sin (2 α )
n  3 2  3 2 
cos α ⋅ cos (2α ) cos(22α )…cos (2n−1α ) =
2n sin α π  1
⇒ θ= and cos −1 (0) > φ > cos −1  
π π π π 6  2
∴ cos 2 ⋅ cos 3 …cos 10 ⋅ sin 10
2 2 2 2   π 
  π 9  the sign changed as cos x is decreasing between 0, 2  
 
sin  210 2   π π π π π π 2π
=  sin 10 [Qhere, α = 10 and n = 9 ] ⇒ θ= and <φ < ⇒ <θ + φ <

2 sin   
9 π  2 2 6 3 2 2 3
  210    π 2π 
∴ θ∈ , 
2 3 
1  π 1 1
= sin   = 9 =
29  2 2 512 11. Since, sin 15° =
1
2 − 3 and cos 15° =
1
2+ 3
2 2
7. Given expression 1
= 3(sin θ − cos θ )4 + 6(sin θ + cos θ )2 + 4 sin 6 θ and sin 15° cos 75° = sin 15°⋅ sin 15° = (2 − 3 )
4
= 3((sin θ − cos θ )2)2 + 6(sin θ + cos θ )2 + 4(sin 2 θ )3
Therefore, all these values are irrational and
= 3(1 − sin 2 θ )2 + 6(1 + sin 2 θ ) + 4(1 − cos 2 θ )3 1
[Q1 + sin 2 θ = (cos θ + sin θ )2 sin 15° cos 15° = ⋅ 2 sin 15° cos 15°
2
and 1 − sin 2 θ = (cos θ − sin θ )2] 1 1
= ⋅ sin 30° = , which is rational.
= 3(1 + sin 2 θ − 2 sin 2 θ ) + 6(1 + sin 2 θ )
2 2
2 4
+ 4(1 − cos 6 θ − 3 cos 2 θ + 3 cos 4 θ ) 12. Given expression =
[Q (a − b)2 = a 2 + b2 − 2ab 3 (sin x − cos x)4 + 6 (sin x + cos x)2 + 4(sin 6 x + cos 6 x)
and (a − b) = a3 − b3 − 3a 2b + 3ab2]
3
= 3 (1 − sin 2x)2 + 6 (1 + sin 2x) + 4 {(sin 2 x + cos 2 x)3
= 3 + 3 sin 2 θ − 6 sin 2 θ + 6 + 6 sin 2 θ + 4
2
− 3 sin 2 x cos 2 x (sin 2 x + cos 2 x)}
− 4 cos 6 θ − 12 cos 2 θ + 12 cos 4 θ = 3 (1 − 2 sin 2x + sin 2x) + 6 + 6 sin 2x
2

= 13 + 3 sin 2 θ − 4 cos θ − 12 cos 2 θ + 12 cos 4 θ


2 6 + 4 (1 − 3 sin 2 x cos 2 x)
 3 
= 13 + 3(2 sin θ cos θ )2 − 4 cos 6 θ − 12 cos 2 θ (1 − cos 2 θ ) = 3 (1 − 2 sin 2x + sin 2 2x + 2 + 2 sin 2x) + 4 1 − ⋅ sin 2 2x
 4 
= 13 + 12 sin 2 θ cos 2 θ − 4 cos 6 θ − 12 cos 2 θ sin 2 θ
= 13 + 3 sin 2 2x − 3 sin 2 2x = 13
= 13 − 4 cos 6 θ
13. Given expression =
8. Given expression is
tan A cot A 3 cosec 20° − sec 20° = tan 60° cosec 20° − sec 20°
+ sin 60° cos 20° − cos 60°⋅ sin 20°
1 − cot A 1 − tan A =
sin A sin A cos A cos A cos 60°⋅ sin 20°⋅ cos 20°
= × + ×
cos A sin A − cos A sin A cos A − sin A sin (60° − 20° ) sin 40°
= =
1 sin3 A − cos3 A  cos 60°⋅ sin 20°⋅ cos 20° 1 ⋅ sin 20° cos 20°
=   2
sin A − cos A  cos A sin A 
2 sin 20° cos 20°
sin 2 A + sin A cos A + cos 2 A = =4
= 1
sin 20° cos 20°
sin A cos A 2
1 + sin A cos A
= = 1 + sec A cosec A 14. Given expression =
sin A cos A
  3π    π 
9. Since, cos (α − β ) = 1 3 sin 4  − α  + sin 4 (3π + α ) −2 sin 6  + α
  2    2 
⇒ α − β = 2nπ
+ sin 6 (5π − α )]
But −2 π < α − β < 2 π [as α , β ∈ (− π , π )]
= 3 (cos α + sin α ) − 2(cos α + sin 6 α )
4 4 6

∴ α −β =0 …(i) = 3 (1 − 2 sin 2 α cos 2 α ) − 2 (1 − 3 sin 2 α cos 2 α )


1
Given, cos (α + β ) =
e = 3 − 6 sin 2 α cos 2 α − 2 + 6 sin 2 α cos 2 α = 1
1
⇒ cos 2α = < 1, which is true for four values of α . 15. Given expression =
e
[as −2π < 2α < 2π]  π  3π   5π   7π 
1 + cos  1 + cos  1 + cos  1 + cos 
1  8  8 8 8
10. Since, sin θ =
2  π  3π   3π   π
π = 1 + cos  1 + cos  1 − cos  1 − cos 
and cos φ = ⇒
1
θ=  8  8  8  8
3 6
rankershalt.com

Trigonometrical Ratios and Identities 467

 π  3π   (m − 1) π   mπ 
= 1 − cos 2  1 − cos 2  cot θ +  − cot θ + 
 8   8
6
 4   4 
1 π  π
⇒ ∑ 1/ 2
=4 2
m =1
= 2 − 1 − cos  2 − 1 − cos 3 
4 4  4 6
  (m − 1) π   mπ  
1 π  π
⇒ ∑ cot θ +
 4
 − cot θ +
 
 =4
4  
= 1 − cos  1 − cos 3  m =1
4 4  4  π  π  2π 
⇒ cot (θ ) − cot θ +  + cot θ +  − cot θ + 
1 1 1 1 1 1  4  4  4
= 1 −  1 +  = 1 −  =
4 2  2 4  2 8  5π   6π 
+ ... + cot θ +  − cot θ +  =4
 4  4
16. Given, A = sin 2 θ + (1 − sin 2 θ )2
 3π 
⇒ cot θ − cot  + θ = 4
⇒ A = sin 4 θ − sin 2 θ + 1  2 
2
 1 3 ⇒ cot θ + tan θ = 4
⇒ A = sin 2 θ −  +
 2 4 ⇒ tan 2 θ − 4 tan θ + 1 = 0
2 ⇒ (tan θ − 2)2 − 3 = 0
 1 1
⇒ 0 ≤ sin 2 θ −  ≤ [Q 0 ≤ sin 2 θ ≤ 1] ⇒ (tan θ − 2 + 3 ) (tan θ − 2 − 3 ) = 0
 2 4
3 ⇒ tan θ = 2 − 3 or tan θ = 2 + 3
∴ ≤ A ≤1 π 5π   π 
4 ⇒ θ= ;θ= Q θ ∈ 0, 2  
12 12  
17. Since, tan θ < 0.
sin 4 x cos 4 x 1 sin 4 x (1 − sin 2 x)2 1
∴ Angle θ is either in the second or fourth quadrant. 20. + = ⇒ + =
2 3 5 2 3 5
Then, sin θ > 0 or < 0
sin x 1 + sin x − 2 sin x 1
4 4 2

4 4 ⇒ + =
∴ sin θ may be or − 2 3 5
5 5 6
⇒ 5 sin 4 x − 4 sin 2 x + 2 =
2 5
18. f (cos 4θ ) = …(i)
2 − sec2 θ ⇒ 25 sin 4 x − 20 sin 2 x + 4 = 0
1 ⇒ (5 sin 2 x − 2)2 = 0
At cos 4θ = 2
3 ⇒ sin 2 x =
1 5
⇒ 2 cos 2 2 θ − 1 = 3 2
3 cos x = , tan 2 x =
2
2 5 3
⇒ cos 2 2 θ =
3 sin 8 x cos 8 x 1
2 ∴ + =
⇒ cos 2 θ = ± …(ii) 8 27 125
3
2 ⋅ cos 2 θ
21. NOTE Multiplicative loop is very important approach in IIT
∴ f (cos 4 θ ) = Mathematics.
2 cos 2 θ − 1
 θ sin θ /2  1 
1 + cos 2 θ  tan  (1 + sec θ ) = ⋅ 1+

=  2  cos θ/2  cos θ 
cos 2 θ
(sin θ /2) 2 cos 2 θ /2
 1 3 =
⇒ f   =1 ± [from Eq. (ii)] (cos θ/2) cos θ
 3 2
(2 sin θ /2) cos θ /2 sin θ
π = = = tan θ
19. For 0 < θ < cos θ cos θ
2
6
(m − 1) π  mπ  ∴ fn (θ ) = (tan θ /2)(1 + sec θ )
 
∑ cosec θ + 4
 cosec θ +
 
 =4 2
4  (1 + sec 2 θ ) (1 + sec 22θ )... (1 + sec 2nθ )
m =1
= (tan θ )(1 + sec 2 θ )(1 + sec 22θ ).... (1 + sec 2nθ )
6
1 = tan 2 θ ⋅ (1 + sec 22θ )... (1 + sec 2nθ )
⇒ ∑  (m − 1)π   mπ 
=4 2
m =1 sin θ +  sin θ +  = tan (2nθ )
 4   4 
π  π  π
Now, f2  = tan 22 ⋅  = tan   = 1
 mπ  (m − 1) π    16  16   4
sin θ + − θ + 
6
 4  4 
⇒ ∑ π  (m − 1 ) π   mπ  
=4 2
Therefore, (a) is the answer.
m =1 sin sin θ +  sin θ +  π  π  π
4  4   4  f3   = tan 23 ⋅  = tan   = 1
 32  32  4
rankershalt.com

468 Trigonometrical Ratios and Identities

= 1 / 64
Therefore, (b) is the answer.
π  π  π
25. We know that,
f4   = tan 24 ⋅  = tan   = 1 1 − tan 2 θ  1 − tan 2 θ 
 64  64  4 cot θ − tan θ = =2  = 2 cot 2 θ …(i)
Therefore, (c) is the answer. tan θ  2 tan θ 
 π   5 π   π
f5   = tan 2 ⋅  = tan   = 1 LHS = tan α + 2 tan 2 α + 4 tan 4 α + 8 cot 8 α
 128  128  4
= − (cot α − tan α − 2 tan 2 α − 4 tan 4 α )
Therefore, (d) is the answer.
+ 8 cot 8 α + cot α
13 π 14 π 
22. In the interval  ,  , cos 2 α < 0 and sin 3 α > 0. = – (2 cot 2 α − 2 tan 2 α − 4 tan 4 α )
 48 48 
+ 8 cot 8 α + cot α
sin 3 α
⇒ is negative, therefore B → p. [from Eq. (i)]
cos 2 α
= − (2 (cot 2 α − tan 2 α ) − 4 tan 4 α )
 18 π 23 π  + 8 cot 8 α + cot α
Again, in the interval  ,  , both sin 3 α and
 48 48 
sin 3 α = − (2 (2 cot 4 α ) − 4 tan 4 α ) + 8 cot 8 α + cot α
cos 2 α are negative, so is positive, therefore A→ r. [from Eq. (i)]
cos 2 α
= − 4 (cot 4 α − tan 4α ) + 8 cot 8 α + cot α
23. Using the relation,
π  π  sin 3 θ = − 8 cot 8 α + 8 cot 8 α + cot α [from Eq. (i)]
sin θ sin  − θ sin  + θ =
3  3  4 = cot α = RHS
π 2π 4π 8π 16π 
Taking θ = , we get 26. 16  cos ⋅ cos ⋅ cos ⋅ cos 
18  15 15 15 15 
π
sin = 16 (cos A ⋅ cos 2 A cos 22 A ⋅ cos 23 A)
π 5π 7π 6 1
sin ⋅ sin ⋅ sin = =  2π 
where, A = 15 
18 18 18 4 8
Alternative Method
Given, k = sin 10° ⋅ sin 50° ⋅ sin 70°  2π 
sin 24  
= cos 80° ⋅ cos 40° ⋅ cos 20°  sin 24 A   15 
= 16  4 =
sin 23 A  2 sin A  2π 
= cos A ⋅ cos 2 A ⋅ cos 22 A = 3 sin  
2 sin A  15 
where, A = 20°  32π   2π 
sin 160° sin (180°–20° ) sin 20° 1 sin   sin 2π + 
= = = =  15   15 
= =
8 sin 20° 8 sin 20° 8 sin 20° 8  2π   2π 
sin   sin  
π 3π 5π 7π 9π 11π 13π  15   15 
24. sin ⋅ sin ⋅ sin ⋅ sin ⋅ sin ⋅ sin ⋅ sin
14 14 14 14 14 14 14  2π 
sin  
π 3π 5π  5π   15 
= sin ⋅ sin ⋅ sin ⋅ sin  π −  = =1
14 14 14  14   2π 
sin  
 3π   π  15 
⋅ sin  π −  ⋅ sin  π − 
 14   14 1
27. sin 12° sin 48° sin 54° = (2 sin 12° sin 48° ) sin 54°
π 3π 5π  π 3π 5π 
2 2
= sin 2 ⋅ sin 2 ⋅ sin 2 = sin ⋅ sin ⋅ sin  1
14 14 14  14 14 14  = [cos (36° ) − cos (60° )] sin 54°
2
2
 π π  π 3π   π 5π   1 1
=  cos  −  ⋅ cos  −  ⋅ cos  −  =  cos 36° −  sin 54°
  2 14   2 14   2 14   2 2
1
 3π 2π π
2 = (2 cos 36° sin 54° − sin 54° )
=  cos ⋅ cos ⋅ cos  4
 7 7 7 1
2
= (sin 90° + sin 18° − sin 54° )
 π 2π 4π  4
=  − cos ⋅ cos ⋅ cos  1 5 −1 5 + 1
 7 7 7 = 1 + − 
2 4 4 4 
 sin 23 π / 7 
= − 3 
 2 ⋅ sin π / 7 1 5 − 1 − 5 − 1
= 1 + 
4 4 
 1 sin 8π / 7
2
 8π  π π
= − ⋅  Q sin 7 = sin  π + 7  = − sin 7 
 8 sin π / 7   
rankershalt.com

Trigonometrical Ratios and Identities 469

1 1 1 α β γ α β γ
= 1 −  = ⇒ tan + tan + tan = tan tan tan
4  2 8 2 2 2 2 2 2
n
28. LHS = sin 2 α + sin 2 β − sin 2 γ 3. Given, sin3 x sin 3x = Σ Cm cos nx is an identity in x,
m= 0
= sin 2 α + (sin 2 β − sin 2 γ ) where, C 0 , C1 ,... , C n are constants.
= sin 2 α + sin ( β + γ ) sin ( β − γ ) 1
= sin 2 α + sin ( π − α )sin (β − γ ) [Qα + β + γ = π] sin3 x sin 3x = {3 sin x − sin 3x} ⋅ sin 3x
= sin 2 α + sin α sin ( β − γ ) 4
= sin α [sin α + sin ( β − γ )] 1 3 
=  ⋅ 2 sin x ⋅ sin 3x − sin 2 3x
= sin α [sin (π − ( β + γ )) + sin ( β − γ )] 4 2 
= sin α [sin ( β + γ ) + sin ( β − γ )] 1 3 1 
= sin α [2 sin β cos γ ] =  (cos 2x − cos x) − (1 − cos 6x)
4 2 2 
= 2 sin α sin β cos γ = RHS
1
29. Given equations can be written as = (cos 6x + 3 cos 2x − 3 cos x − 1)
8
cos 3 θ cos 3 θ
x sin 3 θ − − =0 ...(i) ∴ On comparing both sides, we get n = 6
y z
B
2 cos 3 θ 2 sin 3 θ 2 sin 2
x sin 3 θ − − =0 ...(ii) 1 − cos B 2
y z 4. Since, tan A = =
sin B B B
2 sin cos
2 1 2 2
and x sin 3 θ − cos 3 θ − (cos 3 θ + sin 3 θ ) = 0 ...(iii)
y z tan A = tan B / 2
Eqs. (ii) and (iii), implies ⇒ tan 2 A = tan B
2 sin 3 θ = cos 3 θ + sin 3 θ Hence, it is a true statement.
⇒ sin 3 θ = cos 3 θ 5. Since, A+ B+C=π
∴ tan 3 θ = 1 A B π C
⇒ + = −
π 5π 9π 2 2 2 2
⇒ 3θ =, ,
 A B  π C
4 4 4 ⇒ cot  +  = cot  − 
π 5π 9π 2 2  2 2
or θ= , ,
12 12 12 A B
cot ⋅ cot − 1
2 2 C
⇒ = tan
Topic 2 Graph and Conditional Identities B A 2
cot + cot
1. Given, α + β = π /2 2 2
A B C C A B
⇒ α = (π /2) − β ⇒ cot ⋅ cot ⋅ cot − cot = cot + cot
2 2 2 2 2 2
⇒ tan α = tan (π /2 − β )
A B C A B C
⇒ tan α = cot β ⇒ cot + cot + cot = cot cot cot
2 2 2 2 2 2
⇒ tan α tan β = 1
Again, β + γ =α [given]
6. Given, in ∆ABC , A , B and C are in an AP.
⇒ γ = (α − β ) ∴ A + C = 2B
⇒ tan γ = tan (α − β ) Also, A + B + C = 180° ⇒ B = 60°
tan α − tan β and sin (2 A + B) = sin (C − A )
⇒ tan γ = 1
1 + tan α tan β = − sin (B + 2C ) = …(i)
2
tan α − tan β
⇒ tan γ = ⇒ sin (2 A + 60° ) = sin (C − A ) = − sin (60° + 2C ) =
1
1+1 2
∴ 2 tan γ = tan α − tan β
⇒ 2 A + 60° = 30° , 150° [neglecting 30°, as not possible]
⇒ tan α = tan β + 2 tan γ
⇒ 2 A + 60° = 150° ⇒ A = 45°
α β  γ Again, from Eq. (i),
2. Since, + = π − 
2 2  2 sin (60° + 2C ) = − 1 / 2
 α β  γ ⇒ 60° + 2C = 210° , 330°
∴ tan  +  = tan  π − 
 2 2  2 ⇒ C = 75° or 135°
α β Also, from Eq. (i),
tan + tan sin (C − A ) = 1 / 2
⇒ 2 2 = − tan γ
α β ⇒ C − A = 30° , 150°
1 − tan tan 2
For A = 45° , C = 75°
2 2
rankershalt.com

470 Trigonometrical Ratios and Identities

and C = 135° [not possible] Therefore, (b) is the answer.


∴ C = 75° π
Hence, A = 45° , B = 60° , C = 75° 3. Since, A + B = and, we know product of term is
3
7. LHS = sin 2 α + sin 2 β − sin 2 γ = sin 2 α + (sin 2 β − sin 2 γ ) maximum, when values are equal.
= sin 2 α + sin (β + γ ) sin (β − γ ) ∴(tan A ⋅ tan B) is maximum.
= sin 2 α + sin (π − α )sin (β − γ ) [Qα + β + γ = π] When A = B = π /6
= sin 2 α + sin α sin (β − γ )
π π 1
= sin α [sin α + sin (β − γ )] i.e. y = tan
tan =
= sin α [sin (π − (β + γ )) + sin (β − γ )] 6 6 3
= sin α [sin (β + γ ) + sin (β − γ )] 4. Let y =
sin x cos 3x tan x
=
= sin α [2 sin β cos γ ] = 2 sin α sin β cos γ = RHS sin 3x cos x tan 3x
8. Since, A + B = 180° − C tan x tan x (1 − 3 tan 2 x)
⇒ y= =
∴ tan( A + B) = tan(180°− C ) tan 3x 3 tan x − tan3 x
tan A + tan B
⇒ = − tan C 1 − 3 tan 2 x
1 − tan A tan B = [Q x ≠ 0]
⇒ tan A + tan B = − tan C + tan A tan B tan C 3 − tan 2 x
⇒ tan A + tan B + tan C = tan A tan B tan C Put tan x = t
4 1 − 3t 2
9. Since, cos(α + β) = ⇒ y=
5
3 − t2
5
and sin(α − β ) = ⇒ 3 y − t 2 y = 1 − 3t 2
13
∴ tan(α + β ) =
3 ⇒ 3 y − 1 = t 2y − 3t 2
4 ⇒ 3 y − 1 = t 2 ( y − 3)
5
and tan(α − β ) =
12
Now, tan 2 α = tan[(α + β ) + (α − β )]
+ – +
3 5
+
tan(α + β ) + tan(α − β ) 56 1/3 3
= = 4 12 =
1 − tan(α + β ) ⋅ tan(α − β ) 1 − 3 ⋅ 5 33
3y − 1 3y − 1
4 12 ⇒ = t2 ⇒ >0
y−3 y−3
Topic 3 Maxima and Minima
π ∴ t2 > 0
1. Given expression 3 cos θ + 5 sin θ − 
 6 NOTE It is a brilliant technique to convert equation into inequation
and asked in IIT papers frequently. ⇒ y < 1 / 3 or y > 3.
 π π 
= 3 cos θ + 5 sin θ cos − sin cos θ This shows that y cannot lie between 1 / 3 and 3.
 6 6 
 3 1  Topic 4 Height & Distance
= 3 cos θ + 5  sin θ − cos θ
 2 2  1. According to the question, we have the following figure.
5 5 3 T
= 3 cos θ − cos θ + sin θ
2 2
1 5 3 xm
= cos θ + sin θ
2 2
30° B
Q The maximum value of a cos θ + b sin θ is a 2 + b2
P
1 5 3
So, maximum value of cos θ + sin θ is 30 m
2 2
2
45°
 1
2
5 3 1 75 76 S ym A
=   +  = + = = 19.
 2  2  4 4 4 Now, let distance of foot of the tower from the point A is
y m.
π
2. As when θ ∈ 0,  , tan θ < cot θ Draw BP ⊥ ST such that PT = x m.
 4 Then, in ∆TPB, we have
Since, tan θ < 1 and cot θ > 1 x
tan 30° =
∴ (tan θ )cot θ < 1 and (cot θ )tan θ > 1 y
∴ t4 > t1 which only holds in (b).
rankershalt.com

Trigonometrical Ratios and Identities 471

1 Q
⇒ x= y …(i)
3
x + 30 C
and in ∆TSA, we have tan 45° =
y
⇒ y = x + 30 …(ii) h
On the elimination of quantity x from Eqs. (i) and (ii), we
get 100 P
1
y= y + 30 α
3 l
β
 1 B
⇒ y 1 −  = 30
 3 100
A
30 3 30 3 ( 3 + 1)
⇒ y= = From the figure and given information, we have
3 −1 3 −1
β = cot−1 (3 2 )
30
= 3 ( 3 + 1) = 15 (3 + 3 ) and α = cosec−1 (2 2 )
2
Now, in ∆QPA,
2. Given a ladder of length l = 2m leans against a vertical l
wall. Now, the top of ladder begins to slide down the cot β =
h
wall at the rate 25 cm/s.
⇒ l = (3 2 )h …(i)
Let the rate at which bottom of the ladder slides away h
dx and in ∆BPQ, tan α =
from the wall on the horizontal ground is cm /s. BP
dt
wall BP (100)2 − l2
⇒ cot α = =
h h
l [Q p is mid-point of isosceles ∆ABC, AP ⊥ BC]
y Ladder
⇒ h 2 cot2 α = (100)2 − l2
⇒ h (cosec2α − 1) = (100)2 − (3 2h )2
2
[from Eq. (i)]
x
ground
⇒ h 2(8 − 1) = (100)2 − 18h 2
⇒ 25h 2 = (100)2
Q x + y =l
2 2 2
 100
2
⇒ h2 =   ⇒ h = 20 m
[by Pythagoras theorem]  5 
⇒ x + y =4
2 2
[Ql = 2m]… (i)
4. Given heights of two poles are 5 m and 10 m.
On differentiating both sides of Eq. (i) w.r.t. ‘t’, we get A
dx dy
2x + 2y =0
dt dt 5m
dx  y dy 15° E
⇒ = −  … (ii) B
dt  x  dt 10 m d

From Eq. (i), when y = 1m, then 5m


x + 1 = 4⇒x = 3⇒x = 3 m
2 2 2
[Q x > 0] 15° F
On substituting x = 3m and y = 1m in Eq. (ii), we get C d D
dx 1  25   dy  i.e. from figure AC = 10 m, DE = 5 m
=− −  m /s given = − 25 cm /sec
dt 3  100  dt  ∴ AB = AC − DE = 10 − 5 = 5 m
25 Let d be the distance between two poles.
= cm /s Clearly, ∆ABE ~ ∆ACF
3
[by AA- similarity criterion]
3. Given ABC is a triangular park with AB = AC = 100 m. ∴ ∠AEB = 15°
A vertical tower is situated at the mid-point of BC. In ∆ABE, we have
Let the height of the tower is h m. AB 3 −1 5  3 − 1
tan 15° = ⇒ = Q tan 15° = 
Now, according to given information, we have the BE 3+1 d  3 + 1
following figure. 5( 3 + 1)
⇒ d=
( 3 − 1)
rankershalt.com

472 Trigonometrical Ratios and Identities

3+1 3+1 25 + (25 + x)


⇒ d =5 × 3=
3 −1 3+1 3x
5(3 + 2 3 + 1) 5(2 3 + 4) ⇒ 3x = 50 + x
= =
3 −1 2 ⇒ 2x = 50 ⇒ x = 25 m.
2 × 5( 3 + 2) ∴ Height of cloud from surface
= = 5(2 + 3 ) m
2 = x + 25 = 50 m.
5. Let a first pole AB having height 20 m and second pole 7. According to given information, we have the following
PQ having height 80 m figure.
E
and ∠PBQ = α, ∠AQB = β A

c=7
b=6
D

80 m 30°
A
M
B a=5 C
20 m 1
h Clearly, length of BD =
2a 2 + 2c2 − b2,
α β 2
B N Q (using Appollonius theorem)
x y
where, c = AB = 7, a = BC = 5
and MN = hm is the height of intersection point from
and b = CA = 6
the horizontal plane
1
h 80 ∴ BD = 2 × 25 + 2 × 49 − 36
Q tan α = = [in ∆MNB and ∆PQB] …(i) 2
x x+ y
1 1
h 20 = 112 = 4 7 = 2 7
and tan β = = 2 2
y x+ y
Now, let ED = h be the height of the lamp post.
[in ∆MNQ and ∆ABQ] …(ii) E
From Eqs. (i) and (ii), we get
y
= 4 ⇒ y = 4x …(iii) h
x
From Eqs. (i) and (iii), we get 30°
h 80 80 B D
= ⇒ h= = 16 m
x x + 4x 5 Then, in ∆BDE , tan 30° =
h
BD
6. According to given information, we have the following 1 h
figure, ⇒ =
3 2 7
Q Cloud 2 7 2
⇒ h= = 21
xm 3 3
30° M P
P 60° 8.
25 m 25 m
Surface
y
200 m 45° 200 m
(25 +x)m
T

R Image of cloud
30° 90° 30°
Q M R
x
In ∆PQM , tan 30° = …(i)
y Let height of tower TM be h.
25 + (25 + x) TM
In ∆PRM, tan 60° = …(ii) In ∆PMT , tan 45° =
y PM
h
On eliminating ‘y’ from Eqs. (i) and (ii), we get ⇒ 1=
PM
rankershalt.com

Trigonometrical Ratios and Identities 473

⇒ PM = h 10. According to given information, we have the following


h figure
In ∆TQM, tan 30° = ; QM = 3h
QM Now, from ∆ACD and ∆BCD, we have
In ∆PMQ, PM 2 + QM 2 = PQ 2
D
h 2 + ( 3h )2 = ( 200)2
⇒ 4h 2 = ( 200)2
⇒ h = 100 m Pillar h

9. Let AB = h, then AP = 2h
30º 60º
h
and AC = BC = A x B y C
2
Again, let ∠CPA = α h
tan 30° =
B x+ y
h/2 h
and tan 60° =
h C y
h/2 β x+ y
⇒ h= ...(i)
α 3
A P
2h and h= 3 y ...(ii)
AB x+ y
Now, in ∆ABP, tan (α + β ) = From Eqs. (i) and (ii), = 3 y
AP 3

=
h 1
= ⇒ x +y = 3y
2h 2 ⇒ x − 2y = 0
h x
AC 1 ⇒ y=
Also, in ∆ACP, tan α = = 2 = 2
AP 2h 4
Q Speed is uniform
Now, tan β = tan[(α + β ) − α ]
and distance x covered in 10 min.
1 1 1
− x
tan(α + β ) − tan α 2 4 2 ∴Distance will be cover in 5 min.
= = =4 = 2
1 + tan(α + β ) tan α 1 + ×1 1 9 9
2 4 8 ∴ Distance y will be cover in 5 min.

Download Chapter Test


http://tinyurl.com/y4p5yoht or
rankershalt.com

21
Trigonometrical Equations
Topic 1 General Solution
Objective Questions I (Only one correct option) 8. If sum of all the solutions of the equation
1. Let S be the set of all α ∈ R such that the equation,  π  π  1
8 cos x ⋅  cos + x ⋅ cos − x −  = 1
cos 2x + α sin x = 2α − 7 has a solution. Then, S is equal  6  6  2
to (2019 Main, 12 April II) in [0, π ] is kπ, then k is equal to (2018 Main)
(a) R (b) [1, 4] 2 13 8 20
(a) (b) (c) (d)
(c) [3, 7] (d) [2, 6] 3 9 9 9
2. The number of solutions of the equation 9. If 5 (tan 2 x − cos 2 x) = 2 cos 2x + 9, then the value of
 5π 5π 
1 + sin 4 x = cos 2 3x, x ∈ − , is cos 4x is (2017 Main)
 2 2  (2019 Main, 12 April I) 3 1 2 7
(a) − (b) (c) (d) −
(a) 3 (b) 5 5 3 9 9
(c) 7 (d) 4
10. If 0 ≤ x < 2π, then the number of real values of x, which
3. Let S = {θ ∈ [−2π , 2π ] : 2 cos 2 θ + 3 sin θ = 0}, then the satisfy the equation
sum of the elements of S is (2019 Main, 9 April I) cos x + cos 2x + cos 3x + cos 4x = 0, is (2016 Main)
(a) 2π (b) π (a) 3 (b) 5
5π 13 π
(c) (d) (c) 7 (d) 9
3 6
π
4. If sin 4 α + 4 cos 4 β + 2 = 4 2 sin α cos β; 11. Let S = x ∈ (− π , π ): x =/ 0, ± . The sum of all
 2
α, β ∈ [0, π ], then cos(α + β ) − cos(α − β ) is equal to distinct solutions of the equation 3 sec x + cosec x
(2019 Main, 12 Jan II) + 2(tan x − cot x) = 0 in the set S is equal to (2016 Adv.)
(a) − 1 (b) 2 7π 2π
(a) − (b) −
(c) − 2 (d) 0 9 9
5. Let α and β be the roots of the quadratic equation 5π
(c) 0 (d)
x2 sin θ − x(sin θ cos θ + 1) + cos θ = 0 (0 < θ < 45º ) and 9

 (− 1)n  12. If P = {θ :sin θ − cos θ = 2 cos θ } and
α < β. Then, ∑ α n +  is equal to
n = 0 βn  (2019 Main, 11 Jan II) Q = {θ :sin θ + cos θ = 2 sin θ } be two sets. Then, (2011)
1 1 1 1 (a) P ⊂ Q and Q − P ≠ φ (b) Q ⊄ P
(a) − (b) +
1 − cosθ 1 + sin θ 1 − cosθ 1 + sin θ (c) P ⊄ Q (d) P = Q
1 1 1 1 n
− +
∑ br sin r θ, for every
(c) (d)
1 + cosθ 1 − sin θ 1 + cosθ 1 − sin θ 13. Let n be an odd integer. If sin nθ =
r=0
π
6. The sum of all values of θ ∈ 0,  satisfying value of θ, then (1998, 2M)
 2
3 (a) b0 = 1, b1 = 3 (b) b0 = 0, b1 = n
sin 2 2θ + cos 4 2θ = is (c) b0 = − 1, b1 = n (d) b0 = 0, b1 = n 2 − 3n + 3
4 (2019 Main, 10 Jan I)
3π 5π π 14. The general value of θ satisfying the equation
(a) (b) (c) (d) π
8 4 2 2 sin 2 θ − 3 sin θ − 2 = 0, is (1995,2M)
π π π
7. If 0 ≤ x < , then the number of values of x for which (a) nπ + (−1) n
(b) nπ + (−1) n

2 6 2
sin x − sin 2x + sin 3x = 0, is 5π 7π
(2019 Main, 9 Jan II) (c) nπ + (−1)n (d) nπ + (−1)n
(a) 2 (b) 3 (c) 1 (d) 4 6 6
rankershalt.com

Trigonometrical Equations 475

15. In a ∆ ABC , angle A is greater than angle B. If the Numerical Value


measures of angles A and B satisfy the equation
3 sinx − 4 sin3 x − k = 0, 0 < k < 1, then the measure of ∠ C
21. Let a , b, c be three non-zero real numbers such that the
 π π
is (1990, 2M) equation 3 a cos x + 2b sin x = c, x ∈ − , , has two
π π 2π 5π  2 2 
(a) (b) (c) (d)
3 2 3 6 π
distinct real roots α and β with α + β = . Then, the
16. The general solution of 3
b
sin x − 3 sin 2x + sin 3x = cos x − 3 cos 2x + cos 3x is value of is .................. .
π nπ π a (2018 Adv.)
(a) nπ + (b) + (1989, 2M)
8 2 8
nπ π 3 Integer Answer Type Question
(c) (−1)n + (d) 2nπ + cos−1
2 8 2 22. The number of distinct solutions of the equation
5
17. The general solution of the trigonometric equation cos2 2x + cos4 x + sin4 x + cos6 x + sin6 x = 2 in the
sin x + cos x = 1 is given by (1981, 2M) 4
(a) x = 2nπ ; n = 0, ± 1, ± 2, ... interval [0, 2π ] is (2015 Adv.)
(b) x = 2 nπ + π / 2 ; n = 0, ± 1, ± 2, ....
π π Fill in the Blank
(c) x = nπ + (− 1)n − ; n = 0, ± 1, ± 2 , ...
4 4 23. General value of θ satisfying the equation
(d ) None of the above
tan 2 θ + sec 2 θ = 1 is…… . (1996, 1M)
π
18. The equation 2 cos 2   sin 2 x = x2 + x−2, x ≤ has
x
 2 9 (1980, 1M)
True/False
(a) no real solution
(b) one real solution 24. There exists a value of θ between 0 and 2π that satisfies
(c) more than one real solution
the equation sin 4 θ − 2 sin 2 θ + 1 = 0. (1984, 1M)

(d) None of the above


Analytical & Descriptive Questions
Objective Questions II 25. Determine the smallest positive value of x ( in degrees)
(One or more than one correct option) for which
tan (x + 100° ) = tan (x + 50° ) tan (x) tan (x − 50° ).
19. Let α and β be non zero real numbers such that (1993, 5M)
2(cos β − cos α ) + cos α cos β = 1. Then which of the
following is/are true? (2017 Adv.)
26. If exp {(sin 2 x + sin 4 x + sin 6 x + ... ∞ ) log e 2}, satisfies
α β the equation x2 − 9x + 8 = 0,
(a) 3 tan   − tan   = 0
find the value of
 2  2 cos n x π
,0 < x < . (1991, 4M)
α β cos x + sin x
(b) tan   − 3 tan   = 0
2
 2  2
27. Consider the system of linear equations in x, y, z
α β
(c) tan   + 3 tan   = 0
 2  2 (sin 3 θ ) x − y + z = 0 ,
α β
(d) 3 tan   + tan   = 0 (cos 2 θ ) x + 4 y + 3z = 0,
 2  2
2x + 7 y + 7z = 0
20. The values of θ lying between θ = 0 and θ = π /2 and Find the values of θ for which this system has
satisfying the equation non-trivial solutions. (1986, 4M)

1 + sin 2 θ cos 2 θ 4 sin 4 θ 28. Find the values of x (− π , π ) which satisfy the equation
2
sin θ
2
1 + cos θ2
4 sin 4θ = 0, is 21 + | cos x | + | cos x | + ...
=4 (1984, 2M)
sin 2 θ cos 2 θ 1 + 4 sin 4θ 29. Find all the solutions of 4cos x sin x − 2 sin x = 3 sin x.
2 2
(1988, 3M)
(a) 7 π / 24 (b) 5 π / 24 (1983, 2M)
(c) 11π / 24 (d) π / 24 30. Solve 2 (cos x + cos 2x) + (1 + 2 cos x)sin 2x
= 2 sin x, − π ≤ x ≤ π (1978, 3M)
rankershalt.com

476 Trigonometrical Equations

Topic 2 Solving Equations with Graph


Objective Question I (Only one correct option) Objective Question II
1. All x satisfying the inequality (One or more than one correct option)
(cot− 1 x)2 − 7(cot− 1 x) + 10 > 0, lie in the interval 3. Let θ, φ ∈ [0, 2π ] be such that 2 cos θ (1 − sin φ ) = sin 2 θ
(2019 Main, 11 Jan II)
(− ∞ , cot 5) ∪ (cot 2, ∞ )  θ θ
(a)  tan + cot  cos φ − 1, tan (2π − θ ) > 0
(b) (cot 5, cot 4)  2 2
(c) (cot 2, ∞ ) 3
(d) (− ∞ , cot 5) ∪ (cot 4, cot 2)
and − 1 < sin θ < − . Then, φ cannot satisfy (2012)
2
2. The set of values of θ satisfying the inequation (a) 0 < φ <
π
(b)
π
< φ<

2 sin 2 θ − 5 sin θ + 2 > 0, where 0 < θ < 2π , is (2006, 3M) 2 2 3
π 5π 4π 3π 3π
(a)  0,  ∪  , 2 π  (c) < φ< (d) < φ < 2π
 6  6  3 2 2
 π   5π 
(b) 0, ∪ , 2π Analytical & Descriptive Question
 6   6 
π π
π
(c)  0,  ∪ 

, 2π 4. Find all values of θ in the interval  − ,  satisfying
 3   3   2 2
2
θ
(d) None of the above the equation (1 − tan θ )(1 + tan θ )sec2 θ + 2tan = 0.
(1996, 2M)

Topic 3 Problems Based on Maximum and Minimum


Objective Questions I (Only one correct option) 7. The equation (cos p − 1) x2 + (cos p) x + sin p = 0 in the
1. For x ∈ (0, π ), the equation sin x + 2 sin 2x − sin 3x = 3 has variable x, has real roots. Then, p can take any value in
the interval (1990, 2M)
(a) infinitely many solutions (2014 Adv.)
(a) (0, 2 π) (b) ( − π , 0)
(b) three solutions
π π
(c) one solution (c)  − ,  (d) (0, π )
 2 2
(d) no solution
2. The number of solutions of the pair of equations 8. The smallest positive root of the equation tan x − x = 0
2 sin 2 θ − cos 2θ = 0 and 2 cos 2 θ − 3 sin θ = 0 in the lies in (1987, 2M)
π π 3π 3π
interval [0, 2π] is (2007, 3M) (a)  0,  (b)  , π  (c)  π ,  (d)  , 2 π 
(a) 0 (b) 1  2 2   2   2 
(c) 2 (d) 4 9. The number of all possible triplets (a1 , a 2, a3 ) such that
3. The number of integral values of k for which the a1 + a 2 cos (2x) + a3 sin 2 (x) = 0, ∀ x is (1987, 2M)
equation 7 cos x + 5 sin x = 2k + 1 has a solution, is (a) 0 (b)1 (c) 3 (d) ∞
(a) 4 (b) 8 (2002, 1M)
(c) 10 (d) 12 Objective Questions II
4. The number of values of x in the interval [0, 5π ] (One or more than one correct option)
satisfying the equation 3 sin 2 x − 7 sin x + 2 = 0 is 4xy
(a) 0 (b) 5 (1998, 2M) 10. sec2 θ = is true if and only if
(x + y)2 (1996, 1M)
(c) 6 (d) 10
(a) x = y ≠ 0 (b) x = y, x ≠ 0
5. Number of solutions of the equation
(c) x = y (d) x ≠ 0, y ≠ 0
tan x + sec x = 2 cos x lying in the interval [0, 2π ] is
∞ ∞
(a) 0
(c) 2
(b) 1
(d) 3
(1993, 1M) 11. For 0 < φ < π / 2, if x = ∑ cos2n φ , y = ∑ sin 2n φ ,
n=0 n=0

6. The number of solutions of the equation
sin (ex ) = 5x + 5−x is (1991, 2M)
z= ∑ cos2n φ sin 2n φ , then (1993, 2M)
n=0
(a) 0 (b) 1
(c) 2 (d) infinitely many (a) xyz = xz + y (b) xyz = xy + z
(c) xyz = x + y + z (d) xyz = yz + x
rankershalt.com

Trigonometrical Equations 477

−π π
Integer Answer Type Questions 16. The larger of cos (log θ ) and log (cos θ ) if <θ < ,
2 2
12. The positive integer value of n > 3 satisfying the is …… . (1983, 1M)
1 1 1
equation = + is …… (2011)
 π  2π   3π  Analytical & Descriptive Questions
sin   sin   sin  
 n  n  n
17. Find the smallest positive number p for which the
π π
13. The number of values of θ in the interval  − ,  such equation cos ( p sin x) = sin ( p cos x) has a solution
 2 2
nπ x ∈[0, 2π ] . (1995, 5M)
that θ ≠ for n = 0, ±1, ± 2 and tan θ = cot 5θ as well as
5 18. Show that the equation e sin x − e – sin x − 4 = 0 has no real
sin 2 θ = cos 4θ is…… (2010) solution. (1982, 2M)
19. Find the coordinates of the points of intersection of the
Fill in the Blanks π π
curves y = cos x, y = sin 3x, if − ≤x≤ .
14. The set of all x in the interval [0, π ] for which 2 2 (1982, 3M)
20. For all θ in [0, π / 2], show that cos (sin θ ) ≥ sin(cos θ ).
2 sin 2 x − 3 sin x + 1 ≥ 0, is…… . (1987, 2M)
(1981, 4M)
π
21. Prove that 5 cosθ + 3 cos θ +  + 3 lies between − 4 and
15. The solution set of the system of equations
2π 3  3
x+ y= , cos x + cos y = , where x and y are real, 10. (1979, 3M)
3 2
is…… .

Answers
Topic 1 π π π
30. x = − π , − ,− , ,π
1. (d) 2. (b) 3. (a) 4. (c) 2 3 3
5. (b) 6. (c) 7. (a) 8. (b)
Topic 2
9. (d) 10. (c) 11. (c) 12. (d)
1. (c) 2. (a) 3. (a,c,d) 4. θ = ± π / 3
13. (b) 14. (d) 15. (c) 16. (b)
17. (c) 18. (a) 19. (b, c) 20. (a, c) Topic 3
π
21. (0.5) 22. (8) 23. θ = mπ , nπ ± 1. (d) 2. (c) 3. (b) 4. (c)
3
5. (c) 6. (a) 7. (d) 8. (c)
3 −1 9. (d) 10. (a, b) 11. (b, c) 12. 7
24. False 25. x = 30 ° 26.
2  π π  5π 
13. 3 14. x ∈ 0, ∪   ∪  , π
 π  π 2π   6  2  6 
27. θ = nπ or nπ + ( −1 )n   28. ± , ± 
 6  3 3  15. No solution
 π 16. cos (log θ )
29. { x : x = n π } ∪ x : x = n π + ( −1 )n 
 10  π
17. Smallest positive value of p =
2 2
  − 3π  
∪ x : x = n π + ( − 1 )n   π π π π   3π 3π 
  10   19.  , cos   , cos   − , cos 
8 8 4 4  8 8

Hints & Solutions


Topic 1 General Solution ⇒ (sin x − 2) (2 sin x + 4 − α ) = 0
1. The given trigonometric equation is ∴ 2 sin x + 4 − α = 0 [Q sin x − 2 ≠ 0]
cos 2x + α sin x = 2α − 7 α −4
⇒ sin x = …(i)
⇒ 1 − 2 sin 2x + α sin x = 2α − 7 2
[Q cos 2x = 1 − 2 sin 2 x] Now, as we know −1 ≤ sin x ≤ 1
⇒ 2 sin 2 x − α sin x + 2α − 8 = 0 α −4
∴ −1 ≤ ≤1 [from Eq. (i)]
⇒ 2(sin 2 x − 4) − α (sin x − 2) = 0 2
⇒ 2 (sin x − 2) (sin x + 2) − α (sin x − 2) = 0 ⇒ − 2 ≤ α − 4 ≤ 2 ⇒ 2 ≤ α ≤ 6 ⇒ α ∈ [2, 6]
rankershalt.com

478 Trigonometrical Equations

2. Given equation is 1 + sin 4 x = cos 2(3x) ⇒ x = cos θ , x = cosec θ


⇒ α = cos θ and β = cosec θ
Since, range of (1 + sin x) = [1, 2]
4
1
and range of cos 2(3x) = [0, 1] (Q For 0 < θ < 45° , < cos θ < 1 and 2 < cosecθ < ∞
2
So, the given equation holds if ⇒ cos θ < cos ecθ)
∞ ∞ ∞
1 + sin 4 x = 1 = cos 2(3x)  (−1)n  (−1)n
Now, consider, ∑ α n +  = ∑ α n
+ ∑ n
⇒ sin 4 x = 0 and cos 2 3x = 1 n = 0 βn  n = 0 n=0 β

 5π 5π  = (1 + α + α 2 + α 3 + .... ∞ )
Since, x ∈ − ,  
 2 2  1 1 1
+ 1 − + 2 − 3 + .... ∞
 β β β 
∴ x = − 2π, − π, 0, π, 2 π.
1 1 1 1
Thus, there are five different values of x is possible. = + = +
1 −α  1 1 − α 1 + 1
3. We have, θ ∈ [−2π , 2π ] 1 − − 
 β β
and 2 cos 2 θ + 3 sin θ = 0 1 1  1 
= + Q = sin θ 
⇒ 2 (1 − sin 2 θ ) + 3 sin θ = 0 1 − cos θ 1 + sin θ  β 
⇒ 2 − 2 sin 2 θ + 3 sin θ = 0 6. Given, sin 2 2 θ + cos 4 2 θ =
3
⇒ 2 sin θ − 3 sin θ − 2 = 0
2 4
3
⇒ (1 − cos 2 θ) + cos 2 θ =
2 4
(Q sin 2 x = 1 − cos 2 x)
⇒ 2 sin θ − 4 sin θ + sin θ − 2 = 0
2
4
⇒ 2 sin θ (sin θ − 2) + 1(sin θ − 2) = 0 ⇒ 4 cos 4 2 θ − 4 cos 2 2 θ + 1 = 0
⇒ (2 cos 2 2 θ − 1)2 = 0
⇒ (sin θ − 2) (2 sin θ + 1) = 0 1
−1 ⇒ 2 cos 2 2 θ − 1 = 0 ⇒ cos 2 2 θ =
∴ sin θ = [Q(sin θ − 2) ≠ 0] 2
2 1
⇒ cos 2 θ = ±
π π π π 2
∴ θ = 2π – , − π + , − , π +
6 6 6 6
[Q θ ∈ [−2π , 2π ]]  π
If θ ∈ 0,  , then 2 θ ∈ (0, π )
 2
Now, sum of all solutions 1
π π π π ∴ cos 2 θ = ±
= 2π − − π + − + π + = 2π 2
6 6 6 6 π 3π
⇒ 2θ = , ,
4. By applying AM ≥ GM inequality, on the numbers 4 4
sin 4 α , 4 cos 4β , 1 and 1, we get   3π   π π 1 
sin α + 4 cos β + 2
4 4 Q cos  4  = cos  π − 4  = − cos 4 = − 2 
≥ ((sin 4 α ) (4 cos 4 β ) ⋅ 1 ⋅ 1)1/ 4  
4 π 3π
⇒ θ= ,
⇒ sin 4 α + 4 cos 4 β + 2 ≥ 4 2 sin α cos β 8 8
But, it is given that π 3π π
Sum of values of θ = + =
sin 4 α + 4 cos 4 β + 2 = 4 2 sin α cos β 8 8 2
So, sin 4 α = 4 cos 4 β = 1 7. We have, sin x − sin 2x + sin 3x = 0
[Q In AM ≥ GM , equality holds when all given ⇒ (sin x + sin 3x) − sin 2x = 0
positive quantities are equal.]  x + 3x  x − 3x
⇒ 2 sin   cos   − sin 2x = 0
⇒ sin α = 1 and sin β =
1
…(i)  2   2 
2  C + D  C − D
[Q α , β ∈ [0, π ]] [Qsin C + sin D = 2 sin   cos  ]
 2   2 
Now, cos (α + β ) − cos (α − β ) = −2 sin α sin β ⇒ 2 sin 2x cos x − sin 2x = 0 [Qcos (− θ) = cos θ]
 C+D D −C  ⇒ sin 2x(2 cos x − 1) = 0
Q cos C − cos D = 2 sin sin
 2 2  ⇒ sin 2x = 0 or 2 cos x − 1 = 0
1 1
= −2 × 1 × [from Eq. (i)] ⇒ 2x = 0, π, ... or cos x =
2 2
π π
=− 2 ⇒ x = 0, ... or x =
2 3
5. Given,  π
x2 sin θ − x sin θ cos θ − x + cos θ = 0, In the interval 0,  only two values satisfy, namely
 2 
where 0 < θ < 45° π
⇒ x sin θ (x − cos θ ) − 1(x − cos θ ) = 0 x = 0 and x = .
3
⇒ (x − cos θ ) (x sin θ − 1) = 0
rankershalt.com

Trigonometrical Equations 479

8. Key idea Apply the identity ⇒ 2 cos 2x cos x + 2 cos 3x cos x = 0


cos(x + y) cos(x − y) = cos 2 x − sin 2 y ⇒ 2 cos x (cos 2x + cos 3x) = 0
and cos 3x = 4 cos3 x − 3 cos x  5x x
 π  π  1 ⇒ 2 cos x 2 cos cos  = 0
We have, 8 cos x cos  + x cos  − x −  = 1  2 2
 6  6  2
5x x
 π 1 ⇒ cos x ⋅ cos ⋅ cos = 0
⇒ 8 cos x cos 2 − sin 2 x −  = 1 2 2
 6 2
⇒ cos x = 0
3 1
⇒ 8 cos x − sin x −  = 1
2 5x
=0
4 2 or cos
2
3 1 2  x
⇒ 8 cos x − − 1 + cos x = 1 or cos = 0
4 2 
2
 −3 + 4 cos 2 x Now, cos x = 0
⇒ 8 cos x  =1
 4  π 3π
⇒ x= , [Q 0 ≤ x < 2π]
⇒ 2(4 cos3 x − 3 cos x) = 1 2 2
1 5x
⇒ 2 cos 3x = 1 ⇒ cos 3x = cos =0
2 2
π 5π 7π 5x π 3π 5π 7π 9π 11π
⇒ 3x = , , [0 ≤ 3x ≤ 3π] ⇒ = , , , , ...,
3 3 3 2 2 2 2 2 2 2
π 5π 7π π 3π 7π 9π
⇒ x= , , ⇒ x= , , π, , [Q 0 ≤ x < 2π]
9 9 9 5 5 5 5
π 5π 7π 13π 13π
Sum = + + = ⇒ kπ = x
9 9 9 9 9 and cos = 0
13 2
Hence, k= x π 3π 5π
9 ⇒ = , , , ...
2 2 2 2
9. Given, 5 (tan 2 x − cos 2 x) = 2 cos 2x + 9 ⇒ x=π [Q 0 ≤ x < 2π]
 2 sin 2 x  π 3π π 3π 7π 9π
⇒ 5  − cos 2 x = 2 cos 2x + 9 Hence, x= , , π, , , ,
 2 cos x
2
 2 2 5 5 5 5

 1 − cos 2x 1 + cos 2x 11. Given, 3 sec x + cosec x + 2(tan x − cot x) = 0,


⇒ 5 −  = 2 cos 2x + 9
 1 + cos 2x 2 
(− π < x < π ) − {0, ± π / 2}
Put cos 2x = y, we have ⇒ 3 sin x + cos x + 2 (sin 2 x − cos 2 x) = 0
 1 − y 1 + y ⇒ 3 sin x + cos x − 2 cos 2x = 0
5 −  = 2y + 9
1 + y 2  Multiplying and dividing by a 2 + b2, i.e. 3 + 1 = 2 ,
⇒ 5 (2 − 2 y − 1 − y2 − 2 y) = 2(1 + y)(2 y + 9) we get
 3 1 
⇒ 5(1 − 4 y − y2) = 2(2 y + 9 + 2 y2 + 9 y) 2 sin x + cos x − 2 cos 2x = 0
 2 2 
⇒ 5 − 20 y − 5 y2 = 22 y + 18 + 4 y2
 π π
⇒ 9 y2 + 42 y + 13 = 0 ⇒  cos x ⋅ cos + sin x ⋅ sin  − cos 2x = 0
 3 3
⇒ 9 y2 + 3 y + 39 y + 13 = 0
 π
⇒ 3 y (3 y + 1) + 13(3 y + 1) = 0 ⇒ cos  x −  = cos 2x
 3
⇒ (3 y + 1)(3 y + 13) = 0  π
∴ 2 x = 2 nπ ±  x −  since, cosθ = cos α 

1
y=− ,−
13  3  ⇒ θ = 2 nπ ± α 
3 3 π
1 13 ⇒ 2 x = 2 nπ + x −
∴ cos 2x = − , − 3
3 3 π
1  13  or 2 x = 2 nπ − x +
∴ cos 2x = − Q cos 2x ≠ − 3
3  3  π π
⇒ x = 2nπ − or 3x = 2nπ +
Now, cos 4x = 2 cos 2 2x − 1 3 3
 1
2
2 7 π
= 2 −  − 1 = − 1 = − ⇒ x = 2 nπ −
 3 9 9 3
2 nπ π
10. Given equation is cos x + cos 2x + cos 3x + cos 4x = 0 or x= +
3 9
⇒ (cos x + cos 3x) + (cos 2x + cos 4x) = 0
rankershalt.com

480 Trigonometrical Equations

−π π −5 π 7 π  A + B  A − B
∴ x= or x = , , ⇒ cos 3   = 0, sin 3   =0
3 9 9 9  2   2 
Now, sum of all distinct solutions But it is given that, A > B and 0 < 3 A < π, 0 < 3B < π.
−π π 5π 7π  A − B
= + − + =0 Therefore, sin 3   ≠0
3 9 9 9  2 
12. P = {θ : sin θ − cos θ = 2 cos θ }  A + B
Hence, cos 3   =0
 2 
⇒ cos θ ( 2 + 1) = sin θ
 A + B π
⇒ tan θ = 2 + 1 ⇒ 3  =
 2  2
⇒ Q = {θ : sin θ + cos θ } = 2 sin θ
π
⇒ sin θ ( 2 − 1) = cos θ ⇒ A+ B=
3
1 2 +1 2π
⇒ tan θ = × = ( 2 + 1) ⇒ C = π − ( A + B) =
2 −1 2 +1 3
∴ P =Q
n
16. Given, sin 3x + sin x − 3 sin 2x = cos 3x + cos x − 3 cos 2x
13. Given, sin nθ = ∑ br sin r
θ ⇒ 2 sin 2x cos x − 3 sin 2x = 2 cos 2x cos x − 3 cos 2x
r=0
⇒ sin 2x ( 2 cos x − 3) = cos 2x ( 2 cos x − 3)
Now, put θ = 0, we get 0 = b0 [Q 2 cos x − 3 ≠ 0]
n
⇒ sin 2x = cos 2x
∴ sin nθ = ∑ br sin r
θ
⇒ tan 2x = 1
r =1
π nπ π
sin nθ n
⇒ 2 x = nπ + ⇒ x= +
⇒ = ∑ br (sin θ )r − 1 4 2 8
sin θ r =1
17. Given, sin x + cos x = 1
Taking limit as θ → 0
On dividing and multiplying each terms by 2, we get
sin n θ n
⇒ lim = lim ∑ br (sin θ )r − 1 1 1 1
θ → 0 sin θ θ→ 0 sin x + cos x =
r =1 2 2 2
sin n θ π π 1
nθ⋅ ⇒ sin x cos = cos x sin =
nθ 4 4 2
⇒ lim = b1 + 0 + 0 + 0 + K
θ→ 0 sin θ π π
θ⋅    
θ ⇒ sin  x +  = sin  
 4  4
[Q other values becomes zero for higher powers of sin θ ] π π
n ⋅1 ⇒ x + = nπ + (−1)n
⇒ = b1 4 4
1 π π
⇒ x = nπ + (−1)n − , n ∈ I
⇒ b1 = n 4 4
14. Given, 2 sin θ − 3 sin θ − 2 = 0
2
π
18. Given equation is 2 cos 2   sin 2 x = x2 + x−2, x ≤
x
⇒ ( 2 sin θ + 1)(sin θ − 2) = 0  2 9
⇒ sin θ = − 1 / 2  x 1
LHS = 2 cos 2  sin 2 x < 2 and RHS = x2 + 2 ≥ 2
 2 x
[neglecting sin θ = 2, as|sin θ| ≤ 1]
∴ θ = nπ + (−1)n (7π / 6) ∴ The equation has no real solution.

15. Given, 3 sin x − 4 sin x = k , 0 < k < 1 which can also be


3 19. We have, 2(cos β − cos α ) + cos α cos β = 1
written as sin 3x = k. or 4(cos β − cos α ) + 2 cos α cos β = 2
It is given that A and B are solutions of this equation. ⇒ 1 − cos α + cos β − cos α cos β
Therefore,
= 3 + 3 cos α − 3 cos β − 3 cos α cos β
sin 3 A = k and sin 3B = k, where 0 < k < 1
⇒ (1 − cos α )(1 + cos β ) = 3(1 + cos α )(1 − cos β )
⇒ 0 < 3 A < π and 0 < 3B < π (1 − cos α ) 3(1 − cos β )
Now, sin 3 A = k and sin 3B = k ⇒ =
(1 + cos α ) 1 + cos β
⇒ sin 3 A − sin 3B = 0 α β
3 3 ⇒ = 3 tan 2
tan 2
⇒ 2 cos ( A + B) sin ( A − B) = 0 2 2
2 2 α β
∴ tan ± 3 tan = 0
2 2
rankershalt.com

Trigonometrical Equations 481

1 + sin 2 θ cos 2 θ 4 sin 4 θ 5 5


⇒ cos 2 2x − sin 2 2x = 0
20. Given, sin θ
2
1 + cos 2 θ 4 sin 4 θ =0 4 4
5 5 5
sin 2 θ cos θ
2
1 + 4 sin 4 θ ⇒ cos 2x − + cos 2 2x = 0
2
4 4 4
Applying R3 → R3 − R1 and R2 → R2 − R1, we get 5 5 1
⇒ cos 2x = ⇒ cos 2 2x =
2

1 + sin 2 θ cos 2 θ 4 sin 4 θ 2 4 2


−1 1 0 =0 ⇒ 2 cos 2 2x = 1
−1 0 1 ⇒ 1 + cos 4x = 1
⇒ cos 4x = 0, as 0 ≤ x ≤ 2π
Applying C1 → C1 + C 2, we get  π 3π 5π 7π 9π 11π 13π 15π 
∴ 4x =  , , , , , , , 
2 cos 2 θ 4 sin 4 θ 2 2 2 2 2 2 2 2 
0 1 0 =0 as 0 ≤ 4x ≤ 8π
−1 0 1  π 3π 5π 7π 9π 11π 13π 15π 
⇒ x= , , , , , , , 
8 8 8 8 8 8 8 8 
⇒ 2 + 4 sin 4 θ = 0
Hence, the total number of solutions is 8.
−1
⇒ sin 4 θ = 23. Given, tan 2 θ + sec 2θ = 1
2
 π ⇒ tan 2 θ +
1
=1
⇒ 4 θ = nπ + (−1)n  − 
 6 cos 2θ
nπ π 1 + tan 2 θ
⇒ + (−1)n+ 1  
θ= ⇒ tan 2 θ + =1
4  24 1 − tan 2 θ
7π 11π ⇒ tan 2 θ (1 − tan 2 θ ) + (1 + tan 2 θ ) = 1 − tan 2 θ
Clearly, θ = , are two values of θ lying between
24 24
⇒ 3 tan 2 θ − tan 4 θ = 0
0 and π /2.
⇒ tan 2 θ (3 − tan 2 θ ) = 0
21. We have, α , β are the roots of
⇒ tan θ = 0
3 a cos x + 2b sin x = c
∴ 3 a cos α + 2b sin α = c …(i) or tan θ = ± 3
and 3 a cos β + 2b sin β = c …(ii) Now, tan θ = 0 ⇒ θ = mπ , where m is an integer.
On subtracting Eq. (ii) from Eq. (i), we get  π
and tan θ = ± 3 = tan  ± 
3a (cos α − cos β ) + 2b(sin α − sin β ) = 0  3
  α + β   α − β π
⇒ 3 a  − 2 sin   sin   ⇒ θ = nπ ±
  2    2  3
  α + β   α − β π
+ 2b 2 cos   sin   =0 ∴ θ = mπ , nπ ± , where m and n are integers.
  2    2  3
 α + β  α + β sin 4 θ − 2 sin 2 θ + 1 = 2
⇒ 3 a sin   = 2b cos   24. Given,
 2   2 
 α + β 2b ⇒ (sin 2 θ − 1)2 = 2 ⇒ sin 2 θ = ± 2 + 1
⇒ tan   =
 2  3a which is not possible. Hence, given statement is false.
 π 2b  π  25. tan (x + 100° ) = tan (x + 50° ) tan x tan(x − 50° )
⇒ tan   = Q α + β = , given
 6 3 a  3  tan(x + 100° )
1 2b b 1 ⇒ = tan (x + 50° ) tan(x − 50° ).
⇒ = ⇒ = tan x
3 3a a 2 sin(x + 100° ) cos x sin(x + 50° ) sin(x − 50° )
b ⇒ ⋅ =
⇒ = 0.5 cos (x + 100° ) sin x cos(x + 50° ) cos (x − 50° )
a
5 sin(2x + 100° ) + sin 100° cos 100° − cos 2x
22. Here, cos 2 2x + (cos 4x + sin 4 x) + (cos 6x + sin 6 x) = 2 ⇒ =
4 sin(2x + 100° ) − sin 100° cos 100° + cos 2x
5 ⇒ [sin(2x + 100° ) + sin 100° ] [cos 100° + cos 2x]
⇒ cot 2x + [(cos 2 x + sin 2 x)2 − 2 sin 2 x cos 2 x]
4
= [cos 100° − cos 2x] × [sin(2x + 100° ) − sin 100° ]
+ (cos 2 x + sin 2 x)[(cos 2 x + sin 2 x)2 − 3 sin 2 x cos 2 x] = 2
5 ⇒ sin(2x + 100° ) ⋅ cos 100° + sin(2x + 100° ) ⋅ cos 2x
⇒ cos 2 2x + (1 − 2 sin 2 x cos 2 x) + (1 − 3 cos 2 x sin 2 x) = 2
4 + sin 100° cos 100° + sin 100° cos 2x
5 = cos 100° sin(2x + 100° ) − cos 100° sin 100°
⇒ cos 2 2x − 5 sin 2 x cos 2 x = 0
4 − cos 2x sin (2x + 100° ) + cos 2x sin 100°
rankershalt.com

482 Trigonometrical Equations

⇒ 2 sin(2x + 100° ) cos 2x + 2 sin 100° cos 100° = 0 ⇒ sin θ (4 sin 2 θ + 4 sin θ − 3) = 0
⇒ sin(4x + 100° ) + sin 100° + sin 200° = 0 ⇒ sin θ = 0
⇒ sin(4x + 100° ) + 2 sin 150° cos 50° = 0 ⇒ θ = nπ ...(i)

1
sin(4x + 100° ) + 2 ⋅ sin(90° − 50° ) = 0 or 4 sin 2 θ + 4 sin θ − 3 = 0
2 ⇒ ( 2 sin θ − 1) ( 2 sin θ + 3) = 0
⇒ sin(4x + 100° ) + sin 40° = 0 1 3
⇒ sin θ = [Qsin θ = − is not possible]
⇒ sin(4x + 100° ) = sin(− 40° ) 2 2
⇒ 4x + 100° = nπ + (− 1)n (− 40° ) n  π
∴ θ = nπ + (−1)   ...(ii)
⇒ 4x = n (180° ) + (− 1)n (− 40° ) − 100°  6
1 ∴ From Eqs. (i) and (ii), we get
⇒ x = [n (180° ) + (− 1)n (− 40° ) − 100° ]
4  π
θ = nπ or nπ + (−1)n  
The smallest positive value of x is obtained  6
when n = 1. 2
x | + |cos 3 x| + ....
28. Given, 2 1+ | cos x | + |cos = 22
1
Therefore, x = (180° + 40° − 100° ) 1
4
⇒ 21 −|cos x| = 22
1
⇒ x = (120° ) = 30° 1
4 ⇒ =2
1 − |cos x|
26. exp {(sin 2 x + sin 4 x + sin 6 x + ... ∞ ) log e 2}
1
sin 2 x sin 2 x
⇒ |cos x| =
2
⋅log e 2 log e 2 2
cos 2 x
= e 1 − sin x
=e 1
⇒ cos x = ±
tan 2 x 2
⇒ 2 satisfies x − 9x + 8 = 0
2
π 2π π 2π
⇒ x = 1, 8 ∴ ,x= ,− ,− [Q x ∈ (−π , π )]
2 2 3 3 3 3
∴ 2tan x
=1 and 2tan x
=8  π 2 π 
Thus, the solution set is ± , ± .
⇒ tan x = 0
2
and tan x = 3
2
 3 3 
2
 π
⇒ x=nπ tan 2 x =  tan 
and 29. Given, 4 cos 2 x sin x − 2 sin 2 x = 3 sin x
 3
⇒ 4 (1 − sin 2 x) sin x − 2 sin 2 x − 3 sin x = 0
π
⇒ x = nπ and x = nπ ± ⇒ 4 sin x − 4 sin3 x − 2 sin 2 x − 3 sin x = 0
3
π ⇒ − 4 sin3 x − 2 sin 2 x + sin x = 0
Neglecting x = nπ as 0 < x <
2 ⇒ − sin x (4 sin 2 x + 2 sin x − 1) = 0
π  π ⇒ sin x = 0 or 4 sin 2 x + 2 sin x − 1 = 0
⇒ x = ∈ 0, 
3  2 −2 ± 4 + 16
1 ⇒ sin x = sin 0 or sin x =
2 (4)
cos x 1 3 −1
2
∴ = = × −1 ± 5
cos x + sin x 1 3 1+ 3 3 −1 ⇒ x = nπ or sin x =
+ 4
2 2
π
cos x 3 −1 ⇒ x = nπ or sin x = sin
⇒ = 10
cos x + sin x 2
 3π 
or sin x = sin  − 
27. Since, the given system has non-trivial solution.  10 
sin 3 θ −1 1

∴ cos 2 θ 4 3 = 0 π  − 3π 
x = nπ , nπ + (−1)n , nπ + (−1)n  
2 7 7 10  10 
⇒ sin 3 θ (28 − 21) + 1 (7 cos 2 θ − 6) ∴ General solution set is
+ 1 (7 cos 2 θ − 8) = 0  π
{ x : x = nπ} ∪ x : x = nπ + (−1)n 
⇒ 7 sin 3 θ + 14 cos 2 θ − 14 = 0  10 
⇒ sin 3 θ + 2 cos 2 θ − 2 = 0   −3 π  
∪ x : x = nπ + (−1)n  
⇒ 3 sin θ − 4 sin θ + 2 (1 − 2 sin θ ) − 2 = 0
3 2
  10  
rankershalt.com

Trigonometrical Equations 483

30. Given that, 1


⇒ sin θ <
2 cos x + 2 cos 2x + sin 2x + sin 3x + sin x − 2 sin x = 0 2
∴ 2 cos x + 2 cos 2x + 2 sin x cos x + (sin 3x − sin x) = 0  π  5π 
∴ From the graph, θ ∈ 0,  ∪  , 2π
⇒ 2 cos x + 2 cos 2x + 2 sin x cos x + 2 cos 2x sin x = 0  6  6 
⇒ 2 cos x(1 + sin x) + 2 cos 2x(1 + sin x) = 0 3. PLAN It is based on range of sin x, i.e.
⇒ 2 (1 + sin x)(cos x + cos 2x) = 0 [− 1,1] and the internal for a < x < b.
 3x x Description of Situation As θ , φ ∈ [0, 2π ] and
⇒ 4 (1 + sin x) cos   cos = 0
 2 2 3
tan (2π − θ ) > 0, − 1 < sin θ < −
∴ 1 + sin x = 0 2
3x x tan (2π − θ ) > 0
or cos = 0 or cos = 0
2 2 ⇒ − tan θ > 0
If 1 + sin x = 0, then sin x = −1 ∴ θ ∈II or IV quadrant.

∴ x = 2 nπ + ...(i) 3
2 Also, − 1 < sin θ < −
2
3x 3x π
If cos = 0, then = (2n + 1) Y
2 2 2

2π – π/3
π + π/3
π 1
∴ x = (2 n + 1) ...(ii)
3 X
π 2π
y = –√3
O
x x π
And if cos = 0, then = ( 2 n + 1) –1 y = –1 2
2 2 2
∴ x = ( 2 n + 1)π ...(iii) 4π 5π
⇒ <θ < but θ ∈II or IV quadrant
But given interval is [−π , π ] . 3 3
π 3π 5π
Put n = −1 in Eq. (i), x = − ⇒ <θ < …(i)
2 2 3
π π  θ θ
Put n = 0, 1, − 1, − 2 in Eq. (ii), x =,π − ,− π Here, 2 cos θ (1 − sin φ ) = sin 2 θ  tan + cot  cos φ − 1
3 3  2 2
π π π  2θ 2 θ
Hence, the solution in [− π , π ] are − π , − , − , , π.  sin + cos 
2 3 3 2 2  cos φ − 1
⇒ 2 cos θ − 2 cos θ sin φ = sin θ  2

 sin θ cos θ 
Topic 2 Solving Equations with Graph  2 2 
1. Given, (cot−1 x)2 − 7(cot−1 x) + 10 > 0  1 
⇒ 2 cos θ − 2 cos θ sin φ = 2 sin 2 θ   cos φ − 1
⇒ (cot−1 x − 2)(cot−1 x − 5) > 0 (by factorisation)  sin θ 
⇒ cot−1 x < 2 or cot−1 x > 5
⇒ 2 cos θ + 1 = 2 sin φ cos θ + 2 sin θ cos φ
By wavy curve method,
⇒ 2 cos θ + 1 = 2 sin (θ + φ ) …(ii)
+ – +
3π 5π
cot−1 x =2 −1
cot x =5 From Eq. (i), <θ <
2 3
∴ cot−1 x ∈ (−∞ , 2) ∪ (5, ∞ ) ⇒ 2 cos θ + 1 ∈ (1, 2)
cot−1 x ∈ (0, 2) [Q Range of cot−1 x is (0, π )]
∴ x ∈ (cot 2, ∞ ) ∴ 1 < 2 sin (θ + φ ) < 2
1
2. Since, 2 sin 2 θ − 5 sin θ + 2 > 0 ⇒ < sin (θ + φ ) < 1 …(iii)
2
⇒ ( 2 sin θ − 1) (sin θ − 2) > 0 π 5π
⇒ <θ+ φ <
[where, (sin θ − 2) < 0, ∀ θ ∈ R] 6 6
∴ ( 2 sin θ − 1) < 0 13π 17π
or <θ+ φ <
Y 6 6
π 5π
∴ −θ < φ < −θ
6 6
y=1
2 13π  17π 
or −θ < φ <   −θ
X' π X 6  6 
O π 5π 2π
6 6  3π 2π   2π 7π   3π 5π 
⇒ φ ∈− , −  or  ,  , as θ ∈  , 
 2 3  3 6  2 3
Y'
rankershalt.com

484 Trigonometrical Equations

4. Given, (1 − tan θ )(1 + tan θ ) sec2 θ + 2tan


2
θ
=0 π
For x= ,
2 2
θ
⇒ (1 − tan 2 θ ) ⋅ (1 + tan 2 θ ) + 2tan =0 2
 π 1 π  1
tan 2 θ RHS = 4  cos −  sin = 4   (1) = 1 ≠ 0
⇒ 1 − tan θ + 2
4
=0  2 2 2  4
Put tan θ = x 2
∴ No solution of the equation exists.
∴ 1 − x + 2 =0
2 x
2. 2 sin 2 θ − cos 2 θ = 0
⇒ x −1 =2
2 x
1
NOTE 2 x and x 2 − 1 are uncompatible functions, therefore we
⇒ sin 2 θ =
4
have to consider range of both functions. Also, 2 cos 2 θ = 3 sin θ
Curves y = x2 − 1 and y = 2x 1
∴ sin θ = [Q sin θ + 2 ≠ 0]
Y 2
⇒ Two solutions exist in the interval [0, 2π].
3. We know that,
X′ X
–1 1
− a 2 + b2 ≤ a sin x + b cos x ≤ a 2 + b2
–1
Y′ ∴ − 74 ≤ 7 cos x + 5 sin x ≤ 74
It is clear from the graph that two curves intersect at i.e. − 74 ≤ 2k + 1 ≤ 74
one point at x = 3, y = 8.
Since, k is integer, − 9 < 2k + 1 < 9
Therefore, tan 2 θ = 3
⇒ − 10 < 2k < 8 ⇒ − 5 < k < 4
⇒ tanθ = ± 3
π ⇒ Number of possible integer values of k = 8.
⇒ θ=±
3 4. Given, 3 sin 2 x − 7 sin x + 2 = 0
⇒ 3 sin 2 x − 6 sin x − sin x + 2 = 0
Topic 3 Problems Based on ⇒ 3 sin x (sin x − 2) − 1 (sin x − 2) = 0
Maximum and Minimum ⇒ (3 sin x − 1) (sin x − 2) = 0
1. PLAN For solving this type of questions, obtain the LHS and RHS in 1
⇒ sin x = [Q sin x = 2 is rejected]
equation and examine, the two are equal or not for a given 3
interval.
1
Given, trigonometrical equation ⇒ x = nπ + (−1)n sin −1 , n ∈ I
3
(sin x − sin 3x) + 2 sin 2x = 3
For 0 ≤ n ≤ 5, x ∈ [0, 5π ]
⇒ −2 cos 2x sin x + 4 sin x cos x = 3
There are six values of x ∈ [0, 5π ] which satisfy the
 C + D  C − D equation 3 sin 2 x − 7 sin x + 2 = 0.
[Qsin C − sin D = 2 cos   sin   and
 2   2  π
5. tan x + sec x = 2 cos x, x ∉ ( 2n + 1)
sin 2θ = 2 sin θ cos θ ] 2
⇒ 2 sin x ( 2 cos x − cos 2x) = 3 ⇒ sin x + 1 = 2 cos 2 x
⇒ 2 sin x ( 2 cos x − 2 cos 2 x + 1) = 3 ⇒ sin x + 1 = 2 (1 − sin 2 x)
3 ⇒ 2 sin x + sin x − 1 = 0
2
1 
2

⇒ 2 sin x  − 2  cos x −   = 3 ⇒ ( 2 sin x − 1) (sin x + 1) = 0
 2 
2 1
2 ⇒ sin x = , sin x = − 1
 1 2
⇒ 3 sin x − 3 = 4  cos x −  sin x
 2 π 5π
⇒ x= ,
As x ∈ (0, π ) LHS ≤ 0 and RHS ≥ 0 6 6

For solution to exist, LHS = RHS = 0 or x=
2
Now, LHS = 0
π
⇒ 3 sin x − 3 = 0 but x ∉ (2n + 1)
2
⇒ sin x = 1 π 5π
π ∴ x= ,
⇒ x= 6 6
2 Hence, number of solutions are two.
rankershalt.com

Trigonometrical Equations 485

6. Given equation is sin (ex ) = 5x + 5−x is 10. We know that, sec2 θ ≥ 1


LHS = sin (ex ) < 1 , ∀ x ∈R ⇒
4xy
≥1
and RHS = 5 + 5 x −x
≥2 (x + y)2
−x
∴ sin (e ) = 5 + 5
x x
has no solution. ⇒ 4xy ≥ (x + y)2
7. Since, the given quadratic equation ⇒ (x + y) − 4xy ≤ 0
2

(cos p − 1) x + (cos p) x + sin p = 0


2 ⇒ (x − y)2 ≤ 0
has real roots. ⇒ x− y=0
∴ Discriminant, cos p − 4 sin p (cos p − 1) ≥ 0
2 ⇒ x= y
⇒ (cos p − 2 sin p) − 4 sin p + 4 sin p ≥ 0
2 2 Therefore, x + y = 2x [add x both sides]
⇒ (cos p − 2 sin p) + 4 sin p (1 − sin p) ≥ 0
2 But x + y ≠ 0 since it lies in the denominator,
Q 4 sin p (1 − sin p) > 0 for 0 < p < π ⇒ 2x ≠ 0
and (cos p − 2 sin p)2 ≥ 0 ⇒ x ≠0
Thus, (cos p − 2 sin p)2 + 4 sin p (1 − sin p) ≥ 0 Hence, x = y, x ≠ 0 is the answer.
for 0 < p< π . Therefore, (a) and (b) are the answers.
Hence, the equation has real roots for 0 < p < π . 11. For 0 < φ < / π / 2, we have
8. Let f (x) = tan x − x ∞

π x= ∑ cos2n φ = 1 + cos2 φ + cos4 φ + cos6 φ + K


We know, for 0 < x < n=0
2
It is clearly a GP with common ratio of cos 2 φ which is
⇒ tan x > x
≤ 1.
∴ f (x) = tan x − x has no root in (0, π / 2)  
1 1 a
For π / 2 < x < π , tan x is negative. Hence, x = = Q S∞ = , − 1 < r < 1
1 − cos 2 φ sin 2 φ  1−r 
∴ f (x) = tan x − x < 0
1
π  Similarly, y =
So, f (x) = 0 has no root in  , π  . cos 2 φ
2 
1
3π and z=
For < x < 2π , tan x is negative. 1 − sin 2 φ cos 2 φ
2
1 1
∴ f (x) = tan x − x < 0 Now, x + y = +
sin φ cos 2 φ
2
 3π 
So, f (x) = 0 has no root in  , 2π .
 2  cos 2 φ + sin 2 φ 1
= =
We have, f (π ) = 0 − π < 0 cos 2 φ sin 2 φ cos 2 φ sin 2 φ
 3π  3π 3π 1 1
and f   = tan − >0 Again, = 1 − sin 2 φ cos 2 φ = 1 −
 2 2 2 z xy
3π 1 xy − 1
∴ f (x) = 0 has at least one root between π and . ⇒ = ⇒ xy = xyz − z
2 z xy

9. Given, a1 + a 2 cos 2x + a3 sin 2 x = 0, ∀ x ⇒ xy + z = xyz …(i)


Therefore, (b) is the answer from Eq. (i).
 1 − cos 2x
⇒ a1 + a 2 cos 2x + a3   = 0, ∀ x [putting the value of xy]
 2 
⇒ xyz = x + y + z
 a   a 
⇒  a1 + 3  +  a 2 − 3  cos 2x = 0, ∀ x Therefore, (c) is also the answer.
 2  2
a a
⇒ a1 + 3 = 0 and a 2 − 3 = 0
2 2 12. Given, n > 3 ∈ Integer
k k 1 1 1
⇒ a1 = − , a 2 = , a3 = k, where k ∈ R and = +
2 2  π  2π   3π 
sin   sin   sin  
 k k   n  n  n
Hence, the solutions, are  − , , k , where k is any
 2 2  1 1 1
⇒ − =
real number. π 3π 2π
sin sin sin
Thus, the number of triplets is infinite. n n n
rankershalt.com

486 Trigonometrical Equations

3π π  2π  3
sin − sin ⇒ cos x + cos  − x =
⇒ n n = 1  3  2
π 3π 2π
sin ⋅ sin sin  1 3  3
n n n ⇒ cos x +  − cos x + sin x =
π 3π  2 2  2
sin ⋅ sin
 2π  π n n 1 3 3
⇒ 2 cos   ⋅ sin = ⇒ cos x + sin x =
 n n 2π 2 2 2
sin
n π
  3
2π 2π 3π ⇒ sin  + x = , which is never possible.
⇒ 2 sin ⋅ cos = sin 6  2
n n n
4π 3π Hence, no solution exists.
⇒ sin = sin
n n 16. Since, cos θ ≤ 1 ⇒ log (cos θ ) < 0
4π 3π and cos (log θ ) > 0
⇒ =π−
n n ∴ cos (log θ ) > log (cos θ )

⇒ =π ⇒ n=7 17. Given, cos ( p sin x) = sin ( p cos x) , ∀ x ∈ [0, 2π ]
n
π 
13. Given, tan θ = cot 5 θ ⇒ cos ( p sin x) = cos  − p cos x
2 
π  π 
⇒ tan θ = tan  − 5θ ⇒ p sin x = 2nπ ±  − p cos x , n ∈ I
2  2 
π [Q cos θ = cos α ⇒ θ = 2nπ ± α , n ∈ I ]
⇒ − 5 θ = nπ + θ
2 ⇒ p sin x + p cos x = 2nπ + π / 2
π p sin x − p cos x = 2nπ − π / 2, n ∈ I
⇒ 6 θ = − nπ or
2
⇒ p (sin x + cos x) = 2nπ + π / 2
π nπ
⇒ θ= − or p (sin x − cos x) = 2nπ − π / 2, n ∈ I
12 6
π π π
π  ⇒ p 2 (cos sin x + sin cos x) = 2nπ +
Also, cos 4 θ = sin 2 θ = cos  − 2 θ 4 4 2
2 
 π π  π
π  or p 2  cos sin x − sin cos x = 2nπ − , n ∈ I
⇒ 4 θ = 2 nπ ±  − 2 θ  4 4  2
2  (4n + 1)π
⇒ p 2 [sin (x + π / 4)] =
Taking positive sign, 2
π π
6 θ = 2 nπ + or p 2 [sin (x − π / 4)] = (4n − 1) , n ∈ I
2 2
nπ π − 1 ≤ sin (x ± π / 4) ≤ 1
⇒ θ= + Now,
3 12
⇒ − p 2 ≤ p 2 sin (x ± π / 4) ≤ p 2
Taking negative sign, (4n + 1) ⋅ π
π π ⇒ −p 2≤ ≤ p 2, n ∈I
2 θ = 2 nπ − ⇒ θ = nπ − 2
2 4 (4n − 1) π
or −p 2≤ ≤ p 2, n ∈I
Above values of θ suggest that there are only 3 common 2
solutions.
Second inequality is always a subset of first, therefore
14. Given, 2 sin 2 x − 3 sin x + 1 ≥ 0 we have to consider only first.
⇒ 2 sin 2 x − 2 sin x − sin x + 1 ≥ 0 It is sufficient to consider n ≥ 0, because for n > 0, the
solution will be same for n ≥ 0.
⇒ (2 sin x − 1) (sin x − 1) ≥ 0
If n ≥ 0, − 2 p ≤ (4n + 1) π / 2
⇒ 2 sin x − 1 ≤ 0 or sin x ≥ 1
⇒ (4n + 1) π / 2 ≤ 2 p
1
⇒ sin x ≤ or sin x = 1 For p to be least, n should be least.
2
 π  π  5π  ⇒ n =0
⇒ x ∈ 0, ∪  ∪ ,π π
 6   2   6  ⇒ 2 p ≥ π /2 ⇒ p≥
2 2

15. Given, x+ y= π
3 Therefore, least value of p =
3 2 2
and cos x + cos y =
2
rankershalt.com

Trigonometrical Equations 487

e sin x −
1
=4 π 
18. Given, = 2 sin  + θ
e sin x 4 
⇒ (e sin x )2 − 4 (e sin x ) − 1 = 0 π
⇒ cos θ + sin θ ≤ 2 <
4 ± 16 + 4 2
⇒ e sin x = =2 ± 5
2  π 
as, 2 = 1.4141, = 1.57 (approx)
 2 
But since, e ~ 2 .72 and we know, 0 < e sin x
<e
∴ e sin x = 2 ± 5 is not possible. π
⇒ cos θ + sin θ <
Hence, it does not exist any solution. 2
π
19. The point of intersection is given by Since, cos θ < − sin θ
π  2
sin 3x = cos x = sin  − x π
2  
⇒ sin (cos θ ) < sin  − sin θ
2 
π 
⇒ 3x = nπ + (−1)  − x n
2  ⇒ sin (cos θ ) < cos (sin θ )
(i) Let n be even i.e. n = 2 m ⇒ cos (sin θ ) > sin (cos θ )
π

π
3x = 2 mπ + −x 21. Let f (θ ) = 5 cos θ + 3 cos θ +  + 3
2  3
mπ π  π π
⇒ n= + ...(i) = 5 cos θ + 3  cos θ cos − sin θ sin  + 3
2 8  3 3
(ii) Let n be odd i.e. n = ( 2 m + 1)  1  3
= 5 cos θ + 3   cos θ − 3   sin θ + 3
π   2  2
∴ 3x = ( 2 m + 1) π −  − x
2 
13 3 3
π = cos θ − sin θ + 3
⇒ 3x = 2 mπ + +x 2 2
2
1
π ⇒ f (θ ) = (13 cos θ − 3 3 sin θ ) + 3
⇒ x = mπ + ...(ii) 2
4
π π Put r cos α = 13, r sin α = 3 3 , then
Now, − ≤x≤
2 2 r = 169 + 27
π π 3π = 196 = 14
⇒ x= , − [from Eqs. (i) and (ii)]
8 4 8 1
∴ f (θ ) = (r cos α cos θ − r sin α sin θ ) + 3
Thus, points of intersection are 2
1
π π  π π   3π 3π  = r cos (θ + α ) + 3
 , cos   , cos   − , cos  2
8 8  4 4  8 8
= 7 cos (θ + α ) + 3
 1 1 
20. We have, cos θ + sin θ = 2  cos θ + sin θ Now, −1 ≤ cos (θ + α ) ≤ 1
 2 2 
⇒ −7 ≤ 7 cos (θ + α ) ≤ 7
 π π 
= 2 sin ⋅ cos θ + cos ⋅ sin θ ⇒ − 4 ≤ f (θ ) ≤ 10
 4 4 

Download Chapter Test


http://tinyurl.com/y3wjwbog or
rankershalt.com

22
Inverse Circular Functions

Topic 1 Domain and Range


Objective Question I (Only one correct option) Fill in the Blank
1. The number of real solutions of 2. The greater of the two angles A = 2 tan −1 (2 2 − 1) and
−1 −1 π  1  3
tan x (x + 1) + sin x + x + 1 = is
2
B = 3 sin −1   + sin −1   is …… .
2 (1999, 2M)  3  5 (1989, 2M)
(a) zero (b) one (c) two (d) infinite

Topic 2 Properties of Inverse Functions


Objective Questions II 4. If α = 3sin −1   and β = 3 cos −1   , where the inverse
6 4
(One or more than one correct option)  
11 9  
trigonometric functions take only the principal values,
1. Let f (x) = log e (sin x), (0 < x < π ) and g (x) = sin − 1 (e− x ), then the correct option(s) is/are (2015 Adv.)
(x ≥ 0). If α is a positive real number such that (a) cos β > 0 (b) sin β < 0
a = ( fog )′ (α ) and b = ( fog )(α ), then (2019 Main, 10 April II) (c) cos (α + β ) > 0 (d) cos α < 0
(a) aα 2 − bα − a = 0 (b) aα 2 − bα − a = 1
(c) aα 2 + bα − a = − 2α 2 (d) aα 2 + bα + a = 0 5. If 0 < x < 1, then 1 + x2 [{ x cos (cot−1 x)
 19  n  + sin (cot−1 x)}2 − 1]1/ 2 is equal to (2008, 3M)
2. The value of cot  ∑ cot−1 1 + ∑ 2 p  is (a)
x
(b) x (c) x 1 + x2 (d) 1 + x2
  
n =1 p =1 1 + x2
23 21 (2019 Main, 10 Jan II)
(a) (b) 6. The value of x for which sin [cot−1 (1 + x)] = cos (tan −1 x)
22 19
19 22 is (2004, 1M)
(c) (d) 1 1
21 23 (a) (b) 1 (c) 0 (d) −
2 2
π
3. If cos −1   + cos −1   =  x >  , then x is equal to
2 3 3
 3x  4x 2   x2 x3   x4 x6 
4 7. If sin −1  x − + − K + cos −1  x2 − + − K
(2019 Main, 9 Jan I)  2 4   2 4 
145 146 π
(a) (b) = , for 0 <| x| < 2, then x equals (2001, 1M)
10 12 2
145 145 (a) 1/2 (b) 1 (c) –1/2 (d) –1
(c) (d)
12 11
rankershalt.com

Inverse Circular Functions 489

2π 
8. The principal value of sin −1 sin  is (1986, 2M)
 x 
function defined by f (x) = log e   and g : E 2 → R be
 3  x − 1

(a) −   x 
3 the function defined by g (x) = sin − 1  log e  .
2π   x −1 
(b)
3 (2018 Adv.)
π
(c)
3

List I List II
(d)  1   e 
3  − ∞, ∪  e − 1 , ∞
P. The range of f is 1.  1 − e  
Match the Columns Q. The range of g contains 2. (0, 1)
9. Match List I with List II and select the correct answer The domain of f 3.  1 1
− ,
 2 2 
using the code given below the lists. R. contains
List I List II S. The domain of g is 4. (− ∞ , 0) ∪ (0, ∞ )
P.   cos(tan−1 y) + y sin(tan−1 y) 2 
1/ 2
1. 1 5  e 
1  − ∞,
e − 1 
 2  + y 
4 5.
−1 −1
takes value 2 3 
 y  cot (sin y) + tan(sin y)  
 
1 e 
Q. If cos x + cos y + cos z = 0 = sin x + sin y + sin z, then 2. (− ∞ , 0) ∪  ,
 2 e − 1 
2 6.
x − y
possible value of cos is
2 The correct option is
R. If cos  π − x  cos 2 x + sin x sin2 x sec x 3. 1
(a) P → 4; Q → 2; R → 1; S → 1
 
4  2
(b) P → 3; Q → 3; R → 6; S → 5
π
= cos x sin2 x sec x + cos  + x  cos 2 x, then (c) P → 4; Q → 2; R → 1; S → 6
4 
possible value of sec x is (d) P → 4; Q → 3; R → 6; S → 5
S. If cot (sin−1 1 − x 2 ) = sin[tan−1( x 6 )], 4. 1
Numerical Value Based
x = 0. Then, possible value of x is
12. The number of real solutions of the equation
Codes  ∞ ∞
 x 
i
sin −1  ∑ xi+ 1 − x ∑   
P Q R S   2 
(a) 4 3 1 2 i = 1 i =1

(b) 4 3 2 1 π  ∞  x i ∞ 
= − cos −1  ∑  −  − ∑ (− x)i lying in the interval
(c) 3 4 2 1 2   2 
i = 1 i =1 
(d) 3 4 1 2
 1 1
10. Let (x, y) be such that  − ,  is ................... .
π  2 2
sin −1 (ax) + cos −1 ( y) + cos −1 (bxy) = . (2007)
2 (Here, the inverse trigonometric functions sin −1 x
Column I Column II  π π
and cos −1x assume values in − , and [0, π ],
 2 2 
A. If a = 1 and b = 0, then (x, y) p. lies on the circle x 2 + y 2 = 1
respectively.) (2018 Adv.)
B. If a = 1 and b = 1, then (x, y) q. lies on ( x 2 − 1)( y 2 − 1) = 0

C. If a = 1 and b = 2, then (x, y) r. lies on y = x Analytical & Descriptive Question


If a = 2 and b = 2, then (x, y) lies on ( 4x 2 − 1)( y 2 − 1) = 0 x2 + 1
D. s. 13. Prove that cos tan −1 [sin (cot−1 x)] = .
x2 + 2 (2002, 5M)
x
11. Let E1 = { x ∈ R : x ≠ 1 and > 0} and 1
x−1 14. Find the value of cos (2 cos −1 x + sin −1 x) at x = , where
5
   x  
E 2 = x ∈ E1 : sin − 1  log e    is a real number  0 ≤ cos −1 x ≤ π and − π / 2 ≤ sin −1 x ≤ π / 2. (1981, 2M)
   x − 1   
Integer Answer Type Question
(Here, the inverse trigonometric function sin − 1 x
15. If f : [ 0, 4π ] → [ 0, π ] be defined by f (x) = cos −1 (cos x).
 π π
assumes values in − , ).Let f : E1 → R be the Then, the number of points x ∈ [0, 4π ] satisfying the
 2 2  10 − x
equation f (x) = , is (2014 Adv.)
10
rankershalt.com

490 Inverse Circular Functions

Topic 3 Sum and Difference Formulae


Objective Questions I (Only one correct option)  2x  1
6. If tan −1 y = tan −1 x + tan −1   , where |x| < .
1 − x 
2
1. The value of sin −1 
12 −1  3  3
 − sin   is equal to
 13  5 Then, the value of y is
(2019 Main, 12 April I) 3x − x3 3x + x 2
(a) (b)
π − sin −1  
63 1 − 3x 2
1 − 3x 2
(a)
 65 
3x − x 3
3x + x3
π (c) (d)
− sin −1  
56
(b) 1 + 3x 2 1 + 3x 2
2  65 
π  23   
− cos−1  
9 n
(c)
2  65  7. The value of cot  ∑ cot−1 1 + ∑ 2k  is

n = 1 k =1  
π − cos−1  
33 (2013 Main)
(d)
 65  23 25 23 24
(a) (b) (c) (d)
−1 −1 y 25 23 24 23
2. If cos x − cos = α, where − 1 ≤ x ≤ 1, − 2 ≤ y ≤ 2,
2 8. If x, y and z are in AP and tan − 1 x, tan − 1 y and tan − 1 z are
y
x ≤ , then for all x, y, 4x2 − 4xy cos α + y2 is equal to also in AP, then (2013 Main)
2
(2019 Main, 10 April II)
(a) x = y = z (b) 2x = 3 y = 6z
(c) 6x = 3 y = 2z (d) 6x = 4 y = 3z
(a) 2 sin 2 α
(b) 4 cos2 α + 2x2 y2
 2 
9. The value of tan cos −1   + tan −1    is
(c) 4 sin 2 α 4
(1983, 1M)
(d) 4 sin 2 α − 2x2 y2   5  3 
π
3. If α = cos −1   , β = tan −1   , where 0 < α , β < , then
3 1 6 17
(a) (b)
 5  3 2 17 6
16
α − β is equal to (2019 Main, 8 April I) (c) (d) None of these
7
−1  9  −1  9 
(a) tan   (b) cos  
 5 10   5 10 
Fill in the Blanks
(d) sin −1 
9 
(c) tan −1  
9
  π
 14   5 10 
10. The numerical value of tan 2 tan −1   −  is … .
1
  5  4 (1984, 2M)
4. Considering only the principal values of inverse
 π
functions, the set A = x ≥ 0 : tan −1 (2x) + tan −1 (3x) =  11. If a, b, c are positive real numbers
 4 a (a + b + c) b (a + b + c)
(2019 Main, 12 Jan I) θ = tan −1 + tan −1
bc ca
(a) is an empty set
c (a + b + c)
(b) is a singleton + tan −1 .
(c) contains more than two elements ab
(d) contains two elements Then, tan θ equals …… . (1981, 2M)
5. If x = sin −1 (sin 10) and y = cos −1 (cos 10), then y − x is
12. Solve the following equation for x.
equal to (2019 Main, 9 Jan II)
π
(a) 0 (b) 10 (c) 7π (d) π tan −1 2x + tan −1 3x = (1978, 3M)
4

Answers
Topic 1 −2 6
12. (2) 13. 3 14. 15. 3
1. (c) 2. (A) 5

Topic 2 Topic 3
1. (b) 2. (b) 3. (c) 4. (b, c, d) 1. (b) 2. (c) 3. (d) 4. (b)
5. (c) 6. (d) 7. (b) 8. (c) 5. (d) 6. (a) 7. (b) 8. (a)
9. P → 4; Q → 3; R → 2; S → 1  7 1
9. (b) 10.  −  11. 0 12. x =
10. A → p; B → q; C → p; D → s 11. (a)  17 6
rankershalt.com

Hints & Solutions


Topic 1 Domain and Range Now, fog (x) = f ( g (x)) = f (sin − 1 (e− x ))
1. Given function is = log e (sin(sin − 1 (e− x )))
π = log e (e− x ) {Qsin(sin − 1 x) = x, if x ∈ [− 1, 1]}
tan −1 x(x + 1) + sin −1 x2 + x + 1 =
2 =−x …(i)
d
Function is defined, if and ( fog )′ (x) = (− x) = − 1 …(ii)
(i) x (x + 1) ≥ 0, since domain of square root function. dx
(ii) x2 + x + 1 ≥ 0, since domain of square root function. According to the question,
−1 Q a = ( fog )′ (α ) = − 1 [from Eq. (ii)]
(iii) x + x + 1 ≤ 1, since domain of sin function.
2

and b = ( fog ) (α ) = − (α ) [from Eq. (i)]


From (ii) and (iii), 0 ≤ x2 + x + 1 ≤ 1 ∩ x2 + x ≥ 0 for a positive real value ‘α’.
⇒ 0 ≤ x2 + x + 1 ≤ 1 ∩ x2 + x + 1 ≥ 1 Since, the value of a = − 1 and b = − α, satisfy the
⇒ x2 + x + 1 = 1 quadratic equation (from the given options)
aα 2 − bα − a = 1 .
⇒ x2 + x = 0
 19
 
⇒ x (x + 1) = 0 2. Consider, cot  ∑ cot−1 1 + Σ 2 p 
n

⇒ x = 0, x = − 1  
n =1
p =1 

2. Given, A = 2 tan −1 ( 2 2 − 1)  19   19 n (n + 1) 
= cot  ∑ cot−1(1 + n (n + 1)) Q ∑ p = 
  2
 1  3  n =1   n = 1 
and B = 3 sin −1   + sin −1  
 3  5  19 
−1
Here, A = 2 tan (2 2 − 1) = cot  ∑ cot−1 (1 + n + n 2)
 
 n =1 
= 2 tan −1 (2 × 1.414 − 1)  19 
1
= 2 tan −1 (1.828) = cot  ∑ tan −1 
 1 + n (n + 1)
 n =1
π 2π
∴ A > 2 tan −1 ( 3 ) = 2 ⋅ = 1
3 3 [ Q cot−1 x = tan −1 , if x > 0 ]
x
To find the value of B, we first say
 19  (n + 1) − n  
1 1 π = cot  ∑ tan −1 
sin −1 < sin −1 =   [put 1 = (n + 1) − n]
3 2 6  n =1  1 + n (n + 1) 
1 π 19
so that 0 < 3 sin −1 <
3 2 = cot ∑ (tan −1 (n + 1) − tan −1 n )
n =1
1  1 1
Now, 3 sin −1 = sin −1 3 ⋅ − 4 ⋅ 
3  3 27  −1 x − y −1 −1 
Q tan 1 + xy = tan x − tan y

 23
= sin −1  
 27 = cot [(tan −1 2 − tan −1 1) + (tan −1 3 − tan −1 2) +
 3 π ......+ (tan −1 20 − tan − 1 19)]
= sin −1 (0.851) < sin −1   = = cot (tan 20 − tan −1 1)
−1
 2 3
 3 π  π  π 
 3 = cot  − cot−1 20 −  − cot− 1 1 
sin −1   = sin −1 (0.6) < sin −1   =   2   2 
 5  2 3
π π 2π [Q tan −1 x + cot−1 x = π / 2 ]
∴ B< + = −1
= cot (cot 1 − cot 20) −1
3 3 3
2π 2π cot (cot−1 1) cot (cot−1 20) + 1
Thus, A > and B < =
3 3 cot (cot−1 20) − cot (cot−1 1)
cot A cot B + 1 
Hence, greater angle is A. [ Q cot ( A − B) =
cot B − cot A 
(1 × 20) + 1
Topic 2 Properties of Inverse Functions = [Qcot (cot−1 x) = x]
20 − 1
1. Given functions, f (x) = log e (sin x), (0 < x < π ) and 21
g (x) = sin − 1 (e− x ), x ≥ 0. =
19
rankershalt.com

492 Inverse Circular Functions


Key Idea Use the formula, ⇒ cot θ = x
3. cos −1 x + cos −1 y = cos −1( xy − 1 − x 2 1 − y 2 ) 1
⇒ sin θ = = sin (cot−1 x)
 2  3 π 1 + x2
We have, cos −1   + cos −1   =
 3x  4x 2 x
and cos θ = = cos (cot−1 x)
 2 3 4 9  π 1 + x2
⇒ cos −1  ⋅ − 1− 2 1− =
 3x 4x 9x 16x2  2 Now, 1 + x2 [{ x cos (cot−1 x) + sin (cot−1 x)}2 − 1]1/ 2
−1 −1 −1
[Q cos x + cos y = cos (xy − 1 − x 2
1 − y )]
2
 2 
1/ 2

2  x 1 
 1 9x2 − 4 16x2 − 9  π = 1+ x x + − 1
⇒ cos −1  2 − =  1 + x2 1 + x2  
 2x 12x2  2  
  1/ 2
 2 
1 + x2 

6 − 9x2 − 4 16x2 − 9
= cos
π
=0 = 1 + x2   − 1
 1 + x2 
 
12x2 2  
⇒ 9x2 − 4 16x2 − 9 = 6 = 1 + x 2 [1 + x 2 − 1]1/ 2 = x 1 + x 2
On squaring both sides,
6. Given, sin [cot−1 (1 + x)] = cos (tan −1 x) … (i)
⇒ (9x2 − 4)(16x2 − 9) = 36
and we know that,
⇒ 144x4 − 81x2 − 64x2 + 36 = 36
⇒ 144x4 − 145x2 = 0  1   1 
cot−1 θ = sin −1  and tan −1 θ = cos −1 
⇒ x2(144x2 − 145) = 0  1 + θ2   1 + θ2 
   
145 145
⇒ x = 0 or x = ± =± From Eq. (i),
144 12
 1   1 
3
But x > , x=
145 sin sin −1  = cos  cos −1
 1 + (1 + x)2   1 + x2 
4 12  

4. Here, α = 3 sin −1   and β = 3 cos −1   as


6 4 6 1 1 1
> ⇒ =
 11  9 11 2 1 + (1 + x) 2
1 + x2

6  1 π ⇒ 1 + x2 + 2x + 1 = x2 + 1
⇒ sin −1   > sin −1   =
 11  2 6 1
⇒ x=−
6 π 2
−1 
∴ α = 3 sin   > π
 11 2 7. We know that, sin −1 (α ) + cos −1 (α ) =
2
⇒ cos α < 0
Therefore, α should be equal in both functions.
 4
Now, β = 3 cos −1   x2 x3 x4 x6
 9 ∴ x−
+ − K = x2 − + −K
2 4 2 4
4 1  4  1 π
As < ⇒ cos −1   > cos −1   = x x2 x x2
9 2  9  2 3 ⇒ = ⇒ =
x x 2 2 + x 2 + x2
 4 1+ 1+
∴ β = 3 cos −1   > π 2 2 2 2
 9
2x 2x2
∴ cos β < 0 and sin β < 0 ⇒ =
2 + x 2 + x2

Now, α + β is slightly greater than . ⇒ 2x (2 + x2) = 2x2(2 + x)
2
⇒ 4x + 2x3 = 4x2 + 2x3
∴ cos (α + β ) > 0
⇒ x (4 + 2x2 − 4x − 2x2) = 0
5. We have, 0 < x<1 ⇒ Either x = 0 or 4 − 4x = 0
Let cot−1 x = θ
⇒ x=0 or x=1
C Q 0 < | x|< 2
∴ x=1 and x ≠ 0
√1 + y2
2π  −1  π 
1 8. sin −1 sin 
 = sin sin  π −  
 3   3 
θ π π

B x A = sin −1 sin  =
 3 3
rankershalt.com

Inverse Circular Functions 493

9. P. Here, innermost function is inverse. 10. A. If a = 1, b = 0, then sin −1 x + cos −1 y = 0


⇒ sin −1 x = − cos −1 y ⇒ x2 + y2 = 1
√1 + y2 B. If a = 1 and b = 1, then
y π
sin −1 x + cos −1 y + cos −1 xy =
1 θ
2
2 ⇒ cos −1 x − cos −1 y = cos −1 xy
−1
∴ Put tan y = θ ⇒ tan θ = y ⇒ xy + 1 − x2 1 − y2 = xy ⇒ (x2 − 1)( y2 − 1) = 0
1/ 2
 1  cos (tan −1 y) + y sin(tan −1 y) 2  C. If a = 1, b = 2 , then
 2⋅ −1 −1  + y 4
 π
 y  cot (sin y) + tan(sin y)   sin −1 x + cos −1 y + cos −1 (2 xy) =
2
  2 
1/ 2
⇒ cos −1 x – cos −1 y = cos −1 (2xy)
1 y2 
  +  
 1  1 + y2  ⇒ xy + 1 − x2 1 − y2 = 2xy ⇒ x2 + y2 = 1
1 + y2 
= 2  + y 
4
D. If a = 2 and b = 2, then
 y  1 − y2 y   π
  +  sin −1 (2x) + cos −1 ( y) + cos −1 (2xy) =
2 
  y 1− y   2
  ⇒ cos −1 (2x) − cos −1 ( y) = cos −1 (2xy)
1/ 2
1  ⇒ 2xy + 1 − 4x2 1 − y2 = 2xy
=  2 ⋅ y2(1 − y4 ) + y4  =1
 y  ⇒ (4x2 − 1) ( y2 − 1) = 0
Q. Given, cos x + cos y = − cos z 11. We have,
and sin x + sin y = − sin z  x 
E1 =  x ∈ R : x ≠ 1 and > 0
On squaring and adding, we get  x−1 
cos 2 x + sin 2 x + cos 2 y + sin 2 y + 2 cos x cos y x
∴ E1 = >0
+ 2 sin x sin y = 1 x−1
1 + – +
⇒ 2 + 2 [cos(x − y)] = 1 ⇒ cos (x − y) = −
2 0 1

2  x − y 1 E1 = x ∈ ( − ∞ , 0) ∪ (1, ∞ )
⇒ 2 cos   −1 = −
 2  2 and
   x  

 x − y 1 E2 =  x ∈ E1 : sin−1  loge    is a real number
2 cos 2   =   x − 1 
 2  2  
x− y 1 x
⇒ cos   = E2 = − 1 ≤ loge ≤1 ⇒
 2  2 x−1
x
 π  π  e −1 ≤ ≤e
R. cos 2x ⋅  cos  − x − cos  + x  + 2 sin 2 x x−1
 4  4 
x x 1
Now, ≥ e −1 ⇒ − ≥0
= 2 sin x ⋅ cos x x−1 x−1 e
⇒ cos 2x ⋅ ( 2 sin x) + 2 sin x = 2 sin x ⋅ cos x
2
ex − x + 1 x ( e − 1) + 1
⇒ ≥ 0⇒ ≥0
⇒ 2 sin x [cos 2x + 2 sin x − 2 cos x] = 0 e( x − 1) ( x − 1) e
⇒ sin x = 0, (cos x − sin x) (cos x + sin x − 2 ) = 0 + – +
–1 1
⇒ sec x = 1 or tan x = 1
e–1
⇒ sec x = 1 or 2
 1 
⇒ x ∈ − ∞, ∪ (1, ∞ )
S. cot (sin −1 1 − x2 ) = sin(tan −1 (x 6 ))  1− e
x x 6 x
⇒ = Also, ≤e
1−x 2
1 + 6x2 x−1
( e − 1)x − e
⇒ 1 + 6x2 = 6 − 6x2 ⇒ ≥0
x−1
5 5 + – +
⇒ 12x2 = 5 ⇒ x= =
12 2 3 1 e
e–1
(P) → 4, (Q) → 3, (R) → 2 or 4, (S) → 1
 e 
⇒ x ∈ ( − ∞ , 1) ∪  , ∞
e − 1 
rankershalt.com

494 Inverse Circular Functions

  1  e  Let f ( x ) = x3 + 2x 2 + 5x − 2
So, E2 =  −∞ ,
∪  e − 1 , ∞
e
 1−  f ′ ( x ) = 3x 2 + 4x + 5
∴ The domain of f and g are f ′ ( x ) > 0, ∀ x ∈ R
 1   e  ∴ x3 + 2x 2 + 5x − 2 has only one real roots
 −∞ , ∪ , ∞
 1 − e  e − 1  Therefore, total number of real solution is 2.
x 13. LHS = cos tan −1 [sin (cot−1 x)]
and Range of is R + − { 1}
x−1
  1  
⇒ Range of f is R − { 0} or ( −∞ , 0) ∪ ( 0, ∞ ) = cos tan −1 sin sin −1 
  1 + x2  
 π π  π   π 
Range of g is − ,  − { 0} or − , 0 ∪  0, 
 2 2  2   2  1  x2 + 1
= cos  tan −1 = = RHS
Now, P → 4, Q → 2, R → 1, S → 1  1 + x2  x2 + 2

12. We have,
14. Let f (x) = cos ( 2 cos −1 x + sin −1 x)
 ∞ ∞
 x 
i
−1
sin  Σ x i + 1 − x Σ     π  π
i = 1 i = 1  2  = cos  cos −1 x +  Q cos −1 x + sin −1 x =
   2  2 
π  ∞
 − x
i ∞  −1
= − sin (cos x)
= − cos−1  Σ   − Σ ( − x )i 

2 i = 1  2  i =1
 ⇒ f (x) = − sin (sin −1 1 − x2 )
 x   1  −1 1
x⋅  ⇒ f   = − sin sin 1 − 2
−1  x
2
⇒ sin  − 2  5  5 
1 − x 1 − x
   2 6 2 6
 2 = − sin sin −1  =−
 5  5
 −x 
π −1  2 (− x)  15. PLAN
= − cos  −
2 x 1 + x (i) Using definition of f( x ) = cos −1( x ) , we trace the curve
1 +  f( x ) = cos −1(cos x ) .
 2 
(ii) The number of solutions of equations involving trigonometric
 ∞ i +1 x2  and algebraic functions and involving both functions are
Σ
 i =1
Q x = x 2
+ x3
+ x 4
+ ... = 
 1 − x found using graphs of the curves.
using sum of infinite terms of GP  x ∈ [0, π]
x, if
 x2 x2  π 2π − x, x ∈ [π , 2π]
−1  x x  if
⇒ sin−1  −  = − cos  −  We know that, cos −1 (cos x) = 
1 − x 2 − x  2 1 + x 2 + x  − 2π + x, if x ∈ [2π, 3π]
 x2 x2  −1  x x  4π − x, if x ∈ [3π, 4π]
⇒ sin−1  −  = sin  − 
 1 − x 2 − x   1 + x 2 + x Y

 −1 π −1 
Q sin x = 2 − cos x π y = cos –1 (cos x)
10 – π π
y=
10 π/2 x π–

2 –x
x2 x2 x x 4π
π–

⇒ − = − (0, 1)
1− x 2− x 1+ x 2+ x
X
 2− x−1+ x (2 + x − 1 − x) π π 3π 2π 5π 3π 10 4π
⇒ x2   =x 2 2 2
 (1 − x ) ( 2 − x ) (1 + x ) ( 2 + x ) 10 – x x
y= =1–
x 1 10 10
⇒ = or x = 0
2 − 3x + x 2 2 + 3x + x 2 10 − x
From above graph, it is clear that y = and
⇒ x + 3x + 2x = x − 3x + 2
3 2 2
−1
10
y = cos (cos x) intersect at three distinct points, so
⇒ x + 2x 2 + 5x − 2 = 0 or x = 0
3
number of solutions is 3.
rankershalt.com

Inverse Circular Functions 495

Topic 3 Sum and Difference Formulae y2 x2y2 x2y2


⇒ 1 − x2 − + = cos 2 α + − xy cos α
4 4 4
y2
1. Key Idea Use formulae ⇒ x2 + − xy cos α = 1 − cos 2 α
4
(i) sin− 1 x − sin− 1 y
⇒ 4x2 − 4xy cos α + y2 = 4 sin 2 α
= sin− 1( x 1 − y 2 − y 1 − x 2 ) if x 2 + y 2 ≤ 1
3. Given, α = cos −1   and β = tan −1  
or if xy > 0 and x 2 + y 2 > 1 ∀x , y ∈ [ − 1, 1]
3 1
 5  3
(ii) sin− 1 x = cos − 1 1 − x 2 and
π
π where, 0 < α , β <
(iii) sin− 1θ + cos − 1θ = 2
2

We have, 5
 12  3 52 – 3 2 = 4
sin − 1   − sin − 1  
 13  5
α
 12 2 2 3
 3 3  12
= sin − 1  1−  − 1−  
 13  5 5  13  4
  Clearly, α = tan −1
3
[Qsin − 1 x − sin − 1 y = sin − 1 (x 1 − y2 − y 1 − x2),  4 1 
 − 
if x + y ≤ 1 or if xy > 0 and x + y > 1 ∀x, y ∈ [− 1, 1]]
2 2 2 2 −1 4 −1 1 −1  3 3 
So, α − β = tan − tan = tan
3 3   4 1 
 12 4 3 5   1 +  ×  
= sin − 1  × − ×   3 3 
 13 5 5 13
 −1 x − y 
 48 − 15 −1 −1
Q tan x − tan y = tan 1 + xy , if xy > − 1
= sin − 1  
 65   
−1 1 −1 9
 33 = tan = tan
= sin − 1   4 13
 65 1+
9
2
 33 250
= cos − 1 1 −  
 65 3
2 =
2 1
9+ 9
−1 3136 −1 −1
= cos [Qsin x = cos 1−x ]
2
α –β
4225 13
− 1  56 π − 1  56 9 9
= cos   = − sin   = sin−1 = sin−1
 65 2  65
9 + 13
2 2 250
 π
Q sin − 1 θ + cot− 1 θ =  9 
 2  = sin −1  
 5 10 
2. Given equation is 4. Given equation is
y π
cos − 1 x − cos − 1 = α, where − 1 ≤ x ≤ 1, tan − 1 (2x) + tan −1 (3x) = , x≥0
2 4
y π
− 2 ≤ y ≤ 2 and x ≤ ∴ ⇒ tan −1
5x
= , 6x2 < 1
2
1 − 6x 2
4
 y   x + y
cos − 1  x + 1 − x2 1 − ( y / 2)2 = α [Q tan − 1 x + tan − 1 y = tan −1   , xy < 1]
 2 
 1 − xy
[Q cos − 1 x − cos − 1 y = cos − 1 (xy + 1 − x2 1 − y2), 5x 1
⇒ = 1 , x2 <
| x|,| y| ≤ 1 and x + y ≥ 0] 1 − 6x 2
6
xy 1
⇒ + 1 − x2 1 − ( y / 2)2 = cos α ⇒ 6x2 + 5x − 1 = 0, 0 ≤ x < [Q x ≥ 0]
2 6
xy
⇒ 1 − x2 1 − ( y / 2)2 = cos α − ⇒ 6x2 + 6x − x − 1 = 0, 0≤x<
1
2 6
On squaring both sides, we get 1
 ⇒ 6x (x + 1) − 1 (x + 1) = 0, 0≤x<
y2 x2y2 xy 6
(1 − x2) 1 −  = cos 2 α + −2 cos α
 4 4 2
rankershalt.com

496 Inverse Circular Functions

⇒ (6x − 1)(x + 1) = 0,
1
0≤x< Let x = tan θ
6 π π
⇒ − <θ <
1 1 6 6
⇒ x = , − 1, 0≤x<
6 6 ∴ tan −1 y = θ + tan −1 (tan 2θ ) = θ + 2θ = 3θ
1 1
⇒ x= , [Q 0 ≤ x < ] ⇒ y = tan 3θ
6 6
3 tan θ − tan3 θ
So ‘A’ is a singleton set. ⇒ y=
1 − 3 tan 2 θ
∴ The solution of given differential equation represents
a circle with centre on the X-axis. 3x − x3
⇒ y=
5. The graph of y = sin − 1 (sin x) is 1 − 3x2
y  23  n 
7. We have, cot  ∑ cot−1 1 + ∑ 2k 
y=π–x y=–2π+x y=–x+3π  
n = 1 k =1 
x
y=

–π/2 3π/2
x  23 
π/2 π 2π 5π/2 3π
⇒ cot  ∑ cot−1 (1 + 2 + 4 + 6 + 8 + K + 2n )
n = 1 
∴ x = sin − 1 (sin 10) = − 10 + 3π ...(i)  23 
and the graph of y = cos − 1 (cos x) is ⇒ cot  ∑ cot−1 {1 + n (n + 1)}
n = 1 
Y
 23 
x

1
y=

⇒ cot  ∑ tan −1
y=

π+


x

–x
y=

–2

1 + n (n + 1) 
+

n = 1
–x

y=


X
O π 2π 3π 10 4π  23  (n + 1) − n 
⇒ cot  ∑ tan −1  
n = 1 1 + n (n + 1) 
∴ y = cos − 1 (cos 10) = − 10 + 4π ...(ii)
Now, from Eqs. (i) and (ii),  23 
⇒ cot  ∑ (tan −1 (n + 1) − tan −1 ln n )
y − x = (− 10 + 4π) − (− 10 + 3π) = π n = 1 
 2x 
6. Given, tan −1 y = tan −1 x + tan −1   ⇒ cot [(tan −1 2 − tan −1 1) + (tan −1 3 − tan −1 2)
 1 − x2
+ (tan −1 4 − tan −1 3)] + K + (tan −1 24 − tan −1 23)]
 
2x ⇒ cot (tan −1 24 − tan −1 1)
 x+ 2 
1  1−x   24 − 1   −1 23
where|x| < ⇒ tan −1 y = tan −1   ⇒ cot  tan −1  = cot  tan 
3 1 − x  2x  
  1 + 24 ⋅ (1)  25
  1 − x2 
  25 25
= cot  cot−1  =
  23 23
−1 −1 −1  x + y 
Q tan x + tan y = tan  1 − xy ,
 8. Since, x , y and z are in an AP.

where x > 0, y > 0 and xy < 1 ∴ 2y = x + z
 Also, tan −1
x, tan − 1 y and tan − 1 z are in an AP.
 x − x + 2x 3
∴ 2 tan − 1 y = tan − 1 x + tan − 1 (z )
= tan −1  
 1 − x2 − 2x2  2y   x+ z
⇒ tan − 1   = tan − 1  
 3x − x3   1 − y2   1 − xz 
−1 −1
tan y = tan  
 1 − 3 x2  x+ z x+ z
⇒ = ⇒ y2 = xz
1 − y2 1 − xz
3x − x 3
⇒ y=
1 − 3x2 Since x, y and z are in an AP as well as in a GP.
1 ∴ x= y=z
|x| <
3
 2   2 
9. tan cos −1   + tan −1    = tan tan −1   + tan −1   
1 1 4 3
⇒ − <x<
3 3   5  3    4  3 
 –1  4 −1  3  
Q cos  5 = tan  4 
 
rankershalt.com

Inverse Circular Functions 497

  3 2    a b c  
 +  a + b+ c + +  
   17  17   bc ca ab 
= tan tan  4 3   = tan tan −1    =
−1
 
 6  6
 − (a + b + c) a + b + c
3 2
1 − ⋅    

  4 3   abc 
= tan −1  
  1    1 − (a + b + c)  1 + 1 + 1
 
 −1  1  π  −1
 2⋅  π   a b c 
10. tan 2 tan   −  = tan tan  5  − 
  5 4  1 − 1  4 

  25  
 a + b+ c a + b+ c
  5  π  (a + b + c) − (a + b + c)
= tan tan −1   −  = tan −1 abc abc 
  12 4   
 (a + b + c) (ab + bc + ca ) 
1−
  5   π  abc 
tan tan −1    − tan  
  12    4
= ⇒ θ = tan −1 0 ⇒ tan θ = 0
  5  π π
1 + tan tan −1    tan 12. Given, tan −1 2x + tan −1 3x =
  12   4 4
5
−1  2x + 3x π
12 7 ⇒ tan −1   =
= =−  1 − 6x2  4
5 17
1+ ⋅1 5x
12 ⇒ =1
1 − 6x2
11. Given,
a (a + b + c) b (a + b + c) ⇒ 6x2 + 5x − 1 = 0
θ = tan −1 + tan −1
bc ac ⇒ (x + 1) (6x − 1) = 0
1
c (a + b + c) ⇒ x = −1 or
+ tan −1 6
ab
But x = −1 does not satisfy the given equation.
 −1 −1 −1 −1  x + y + z − xyz  
Q tan x + tan y + tan z = tan  1 − xy − yz − zx 
1
∴ We take x =
  6

Download Chapter Test


http://tinyurl.com/y5bu7cjl or
rankershalt.com

23
Properties of Triangles

Topic 1 Applications of Sine, Cosine,


Projection and Half Angle Formulae
Objective Questions I (Only one correct option) 6. In a ∆ ABC, among the following which one is true?
B + C
= a sin 
A
1. The angles A, B and C of a ∆ABC are in AP and (a) (b + c) cos  (2005, 1M)
2  2 
a : b = 1 : 3. If c = 4 cm, then the area (in sq cm) of
B + C
(b) (b + c) cos 
A
this triangle is (2019 Main, 10 April II)  = a sin
 2  2
2 4
(a) (b) 4 3 (c) 2 3 (d) 
(c) (b − c) cos 
B – C  A
3 3  = a cos  
 2   2
b+ c c+ a a+ b B − C
2. Given, = = for a ∆ABC with usual A
(d) (b − c) cos = a sin  
11 12 13 
2  2 
cos A cos B cos C
notation. If = = , then the ordered 7. If the angles of a triangle are in the ratio 4 : 1 : 1, then
α β γ
the ratio of the longest side to the perimeter is
triad (α , β , γ ) has a value (2019 Main, 11 Jan II) (2003, 1M)
(a) (19, 7, 25) (b) (3, 4, 5) (a) 3 : (2 + 3 ) (b) 1 : 3 : 2
(c) (5, 12, 13) (d) (7, 19, 25) (c) 1 : 2 + 3 (d) 2 : 3
3. In a triangle, the sum of lengths of two sides is x and 1 
the product of the lengths of the same two sides is y. 8. In a ∆ABC, 2ac sin ( A − B + C ) is equal to
2  (2000, 2M)
If x 2 − c 2 = y, where c is the length of the third side of
(a) a 2 + b2 − c2 (b) c2 + a 2 − b2
the triangle, then the circumradius of the triangle is (c) b2 − c2 − a 2 (d) c2 − a 2 − b2
(2019 Main, 11 Jan I)
c c π  P Q
(a) (b) 9. In a ∆PQR, ∠R = , if tan   and tan   are the
3 3 2  2  2
3 y
(c) y (d) roots of the equation ax + bx + c = 0 ( a ≠ 0), then
2
2 3
(a) a + b = c (b) b + c = a (1999, 2M)
4. ABCD is a trapezium such that AB and CD are (c) a + c = b (d) b = c
parallel and BC ⊥ CD, if ∠ ADB = θ, BC = p and
10. If in a ∆PQR, sin P, sin Q, sin R are in AP, then
CD = q, then AB is equal to (2013 Main)
(a) the altitudes are in AP (1998, 2M)
( p 2 + q2 ) sin θ p 2 + q2 cos θ (b) the altitudes are in HP
(a) (b)
p cos θ + q sin θ p cos θ + q sin θ (c) the medians are in GP
p 2 + q2 ( p 2 + q2 ) sin θ (d) the medians are in AP
(c) (d)
p 2 cos θ + q2 sin θ ( p cos θ + q sin θ)2 π π
11. In a ∆ABC, ∠B = and ∠C = . Let D divides BC
5. If the angles A, B and C of a triangle are in an 3 4
arithmetic progression and if a, b and c denote the sin ∠BAD
internally in the ratio 1 : 3, then is equal to
lengths of the sides opposite to A, B and C sin ∠CAD
respectively, then the value of the expression 1 1 (1995, 2M)
a c (a) (b)
sin 2 C + sin 2 A is (2010) 6 3
c a 1 2
1 3 (c) (d)
(a) (b) (c) 1 (d) 3 3 3
2 2
rankershalt.com

Properties of Triangles 499

Objective Questions II Fill in the Blanks


(One or more than one correct option) 17. In a ∆ABC, AD is the altitude from A. Given
1 abc
12. In a ∆ PQR , P is the largest angle and cos P = . b > c, ∠ C = 23° and AD = , then ∠B = … .
3 b2 − c2 (1994, 2M)
Further in circle of the triangle touches the sides 18. If in a ∆ABC,
PQ , QR and RP at N , L and M respectively, such 2 cos A cos B 2 cos C a b
that the lengths of PN , QL and RM are consecutive + + = +
a b c bc ca
even integers. Then, possible length(s) of the side(s)
of the triangle is (are) (2017 Main) Then, the value of the ∠A is …… degree. (1993, 2M)

(a) 16 (b) 18 (c) 24 (d) 22


Analytical & Descriptive Questions
π
13. Let ABC be a triangle such that ∠ACB = . If a, b 19. Let A1 , A2, …, An be the vertices of an n-sided regular
6 1 1 1
and c denote the lengths of the sides opposite to A, B polygon such that = + . Find the
A1 A2 A1 A3 A1 A4
and C respectively. Then, the value(s) of x for which (1994, 4M)
a = x 2 + x + 1, b = x 2 − 1 and c = 2x + 1 is (are) (2010) value of n.
(a) − (2 + 3 ) (b) 1 + 3 (c) 2 + 3 (d) 4 3 20. The sides of a triangle are three consecutive natural
14. In a ∆ABC with fixed base BC, the vertex A moves numbers and its largest angle is twice the smallest
A one. Determine the sides of the triangle. (1991, 4M)
such that cos B + cos C = 4 sin 2
. If a, b and c denote
2 21. In a ∆ABC, the median to the side BC is of length
the lengths of the sides of the triangle opposite to the 1
and it divides the ∠A into angles 30° and
angles A, B and C respectively, then (2009)
11 − 6 3
(a) b + c = 4a (b) b + c = 2a
(c) locus of point A is an ellipse
45°. Find the length of the side BC. (1985, 5M)
(d) locus of point A is a pair of straight line b+ c c+ a
22. With usual notation, if in a ∆ABC =
15. Internal bisector of ∠A of ∆ABC meets side BC at D. 11 12
a+b cos A cos B cos C
A line drawn through D perpendicular to AD = , then prove that = = .
intersects the side AC at E and side AB at F. If a, b, c 13 7 19 25
represent sides of ∆ ABC, then (2006, 5M) (1984, 4M)

(a) AE is HM of b and c (b) AD =


2bc
cos
A 23. ABC is a triangle. D is the middle point of BC. If AD
b+ c 2 is perpendicular to AC, then prove that
(c) EF =
4bc A
(d) ∆ AEF is isosceles 2(c2 − a 2)
sin cos A cos C = .
b+ c 2 3ac (1980, 3M)

16. There exists a ∆ABC satisfying the conditions 24. If in a triangle ABC, a = 1 + 3 cm, b = 2 cm and
π π (1986, 2M) ∠C = 60° , then find the other two angles and the
(a) b sin A = a, A < (b) b sin A > a, A >
2 2 third side. (1978, 3M)
π π
(c) b sin A > a , A < (d) b sin A < a, A < , b > a
2 2

Topic 2 Applications of Area, Napier’s Analogy


and Solution of a Triangle
Objective Questions I (Only one correct option) the triangle opposite to the angles at P , Q and R,
1. With the usual notation, in ∆ABC, if 2 sin P − sin 2P
respectively. Then, equals
∠A + ∠B = 120°, a = 3 + 1 and b = 3 − 1, then the 2 sin P + sin 2P (2012)
ratio ∠A : ∠B, is (2019 Main, 10 Jan II)
3 45  3 
2
 45 
5

(a) 7 : 1 (b) 3 : 1 (a) (b) (c)   (d)  


4∆ 4∆  4∆   4∆ 
(c) 9 : 7 (d) 5 : 3
7 3. In radius of a circle which is inscribed in a isosceles
2. If PQR is a triangle of area ∆ with a = 2, b = and triangle one of whose angle is 2π / 3, is 3, then area
2
5 of triangle (in sq units) is (2006, 2M)
c = , where a, b and c are the lengths of the sides of (a) 4 3 (b) 12 − 7 3
2
(c) 12 + 7 3 (d) None of these
rankershalt.com

500 Properties of Triangles

4. The sides of a triangle are in the ratio 1 : 3 : 2 , then (i) The sides a , b, c and area of triangle are rational.
the angles of the triangle are in the ratio (2004, 1M) B C
(ii) a , tan , tan are rational.
(a) 1 : 3 : 5 (b) 2 : 3 : 2 (c) 3 : 2 : 1 (d) 1 : 2 : 3 2 2
(iii) a , sin A , sin B, sin C are rational.
Fill in the Blank Prove that (i) ⇒ (ii) ⇒ (iii) ⇒ (i) (1994, 5M)
5. If the angle of a triangle are 30° and 45° and the 12. In a triangle of base a, the ratio of the other two sides
included side is ( 3 + 1) cm, then the area of the is r(< 1). Show that the altitude of the triangle is less
triangle is … . (1988, 2M)
ar
than or equal to .
1 − r2
6. The set of all real numbers a such that a 2 + 2a , 2a + 3 (1991, 4M)

and a 2 + 3a + 8 are the sides of a triangle is …… 13. If in a ∆ABC, cos A cos B + sin A sin B sin C = 1, then
(1985, 2M) show that a : b : c = 1 : 1 : 2. (1986, 5M)
3
Analytical & Descriptive Questions 14. For a ∆ABC, it is given that cos A + cos B + cos C = .
2
7. If ∆ is the area of a triangle with side lengths a , b, c, Prove that the triangle is equilateral. (1984, 4M)
then show that
15. If p1 , p2 , p3 are the altitudes of a triangle from the
1 vertices A, B, C and ∆ is the area of the triangle, then
∆≤ ( a + b + c) abc
4 prove that
Also, show that the equality occurs in the above 1 1 1 2ab C
+ − = cos 2
inequality if and only if a = b = c. (2001, 6M) p1 p2 p3 (a + b + c)∆ 2 (1978, 3M)
8. Prove that a ∆ABC is equilateral if and only if 16. If p1 , p2 , p3 are the perpendiculars from the vertices
tan A + tan B + tan C = 3 3. (1998, 8M) of a triangle to the opposite sides, then prove that
9. Show that for any triangle with sides a , b, c a 2b2c2
p1 p2p3 = (1978, 3M)
3( ab + bc + ca ) ≤ ( a + b + c) ≤ 4( ab + bc + ca ).
2
8R3
(1979, 3M)
π Integer Answer Type Question
10. Let A, B, C be three angles such that A = and
4
17. Let ABC and ABC′ be two non-congruent triangles
tan B, tan C = p. Find all positive values of p such
with sides AB = 4 , AC = AC ′ = 2 2 and angle
that A, B, C are the angles of triangle. (1997C, 5M)
B = 30° .The absolute value of the difference between
11. Consider the following statements concerning a the areas of these triangles is (2009)
∆ABC

Topic 3 Circumcircle, Incircle, Escribed, Orthocentre


and Centroid of a Triangle
Objective Questions I (Only one correct option) 3. In a triangle, the sum of two sides is x and the
product of the same two sides is y. If x 2 − c2 = y,
1. Two vertices of a triangle are (0, 2) and
(4, 3). If its orthocentre is at the origin, then its third where c is the third side of the triangle, then the
vertex lies in which quadrant? (2019 Main, 10 Jan II) ratio of the inradius to the circumradius of the
(a) Fourth (b) Third
triangle is (2014 Adv)

(c) Second (d) First 3y 3y 3y 3y


(a) (b) (c) (d)
2x (x + c) 2 c (x + c) 4x (x + c) 4 c (x + c)
2. Let the equations of two sides of a triangle be
3x − 2 y + 6 = 0 and 4x + 5 y − 20 = 0. If the 4. Which of the following pieces of data does not
orthocentre of this triangle is at (1, 1) then the uniquely determine an acute angled ∆ABC (R being
equation of its third side is (2019 Main, 9 Jan II) the radius of the circumcircle)? (2002, 1M)
(a) 122 y − 26x − 1675 = 0 (b) 26x − 122 y − 1675 = 0 (a) a , sin A , sin B (b) a , b, c
(c) 122 y + 26x + 1675 = 0 (d) 26x + 61y + 1675 = 0 (c) a , sin B , R (d) a , sin A , R
rankershalt.com

Properties of Triangles 501

5. In a ∆ABC, let ∠ C = π / 2. If r is the inradius and R is Fill in the Blanks


the circumradius of the triangle, then 2 (r + R ) is
equal to (2000, 2M) 12. In a ∆ABC, a : b : c = 4 : 5 : 6. The ratio of radius of the
(a) a + b (b) b + c (c) c + a (d) a + b + c
circumcircle to that of the incircle is… . (1996, 1M)
13. The sides of a triangle inscribed in a given
Passage Based Problems circle subtend angles α, β and γ at the centre. The
minimum value of the arithmetic mean of
Consider the circle x 2 + y 2 = 9 and the parabola  π  π  π
y 2 = 8x. They intersect at P and Q in the first and the cos α +  , cos β +  and cos  γ +  is ….
 2  2  2 (1987, 2M)
fourth quadrants, respectively. Tangents to the
circle at P and Q intersect the X-axis at R and 14. A polygon of nine sides, each of length 2, is inscribed
tangents to the parabola at P and Q intersect the in a circle. The radius of the circle is… . (1987, 2M)
X-axis at S. (2007, 8M) Analytical & Descriptive Questions
6. The radius of the incircle of ∆PQR is
8
15. Circle with radii 3, 4 and 5 touch each other
(a) 4 (b) 3 (c) (d) 2 externally, if P is the point of intersection of tangents
3
to these circles at their points of contact. Find the
7. The radius of the circumcircle of the ∆PRS is distance of P from the point of contact. (2005, 2M)
(a) 5 (b) 3 3 (c) 3 2 (d) 2 3
16. I n is the area of n sided regular polygon inscribed in
8. The ratio of the areas of ∆PQS and ∆PQR is a circle of unit radius and On be the area of the
(a) 1 : 2 (b) 1 : 2 (c) 1 : 4 (d) 1 : 8 polygon circumscribing the given circle, prove that
O 
2
Objective Questions II  2I 
I n = n 1 + 1 −  n   .
(One or more than one correct option) 2   n   (2003, 5M)

9. In a ∆ PQR, let ∠PQR = 30° and the sides PQ and QR 17. Let ABC be a triangle with incentre I and inradius r.
have lengths 10 3 and 10, respectively. Then, which Let D , E , F be the feet of the perpendiculars from I to
of the following statement(s) is (are) TRUE? (2018 Adv) the sides BC, CA and AB, respectively. If r1 , r2 and r3
(a) ∠QPR = 45° are the radii of circles inscribed in the quadrilaterals
(b) The area of the ∆ PQR is 25 3 and ∠QRP = 120° AFIE, BDIF and CEID respectively, then prove that
(c) The radius of the incircle of the ∆ PQR is 10 3 − 15 r1 r2 r3 r1r2r3
(d) The area of the circumcircle of the ∆ PQR is 100 π + + = .
r − r1 r − r2 r − r3 (r − r1 ) (r − r2) (r − r3 ) (2000 3M)
10. In a ∆XYZ, let x , y , z be the lengths of sides opposite
to the angles X , Y , Z respectively and 2s = x + y + z. 18. Let ABC be a triangle having O and I as its
s−x s− y s−z circumcentre and incentre, respectively. If R and r
If = = and area of incircle of the
4 3 2 are the circumradius and the inradius respectively,
8π then prove that ( IO )2 = R 2 − 2Rr. Further show that
∆XYZ is , then (2016 Adv.)
3 the ∆BIO is a right angled triangle if and only if b is
(a) area of the ∆XYZ is 6 6 the arithmetic mean of a and c. (1999, 10M)
35
(b) the radius of circumcircle of the ∆XYZ is 6 19. The exradii r1 , r2 , r3 of ∆ABC are in HP, show that its
6
X +Y 3 sides a , b, c are in AP. (1983, 3M)
(d) sin 2 
X Y Z 4
(c) sin sin sin =  =
2 2 2 35  2  5 Integer Answer Type Question
11. A straight line through the vertex P of a ∆PQR 20. Consider a ∆ABC and let a, b and c denote the
intersects the side QR at the point S and the lengths of the sides opposite to vertices A, B and C,
circumcircle of the ∆PQR at the point T . If S is not respectively. a = 6, b = 10 and the area of the
the centre of the circumcircle, then (2008, 4M) triangle is 15 3. If ∠ACB is obtuse and if r denotes
1 1 2 1 1 2
(a) + < (b) + > the radius of the incircle of the triangle, then r 2 is
PS ST QS × SR PS ST QS × SR
equal to……
1 1 4 1 1 4
(c) + < (d) + >
PS ST QR PS ST QR
rankershalt.com

502 Properties of Triangles

Answers
Topic 1 10. p ∈ ( − ∞, 0 ) ∪ [3 + 2 2, ∞ )
1. (c) 2. (d) 3. (b) 4. (a) 17. 4 sq units
5. (d) 6. (d) 7. (a) 8. (b)
9. (a) 10. (b) 11. (a) 12. (b, d) Topic 3
13. (b) 14. (b, c) 15. (a, b, c, d) 16. (a, d) 1. (c) 2. (b) 3. (b) 4. (d)
17. 113° 18. 90° 19. n = 7 20. 4, 5, 6 units 5. (a) 6. (d) 7. (b) 8. (c)
21. 2 24. c = 6 , ∠B = 45 ° and ∠A = 75 ° 9. (b,c,d) 10. (a, c, d) 11. (b, d) 12.
16
7
Topic 2 3
13. − 14. cosec 20° 15. 5 20. 3
1. (a) 2. (c) 3. (c) 4. (d) 2
1+ 3
5. sq cm 6. a > 5
2

Hints & Solutions


Topic 1 Applications of Sine, Cosine, b+ c c+ a a + b
2. Given, = = = λ (say)
Projection and Half Angle 11 12 13
Formulae A
1. It is given that angles of a ∆ABC are in AP.
So, ∠A + ∠B + ∠C = 180º
c b
⇒ ∠B − d + ∠B + ∠B + d = 180º
[if ∠A , ∠B and ∠C are in AP, then it taken as ∠B − d,
∠B, ∠B + d respectively, where d is common difference
of AP] B a C
⇒ 3∠B = 180º ⇒ ∠B = 60º …(i) b + c = 11λ ,c + a = 12λ and a + b = 13λ ...(i)
a 1 ⇒ 2(a + b + c) = 36λ
and = [given]
b 3 ⇒ a + b + c = 18λ ...(ii)
sin A 1
⇒ = From Eqs. (i) and (ii), we get
sin B 3 a = 7λ, b = 6λ, c = 5λ
 sin A sin B sin C 
by sine rule = = Now,
 a b c 
b2 + c2 − a 2 λ2[36 + 25 − 49] 12 1
sin A 1  3 cos A = = = =
⇒ = Q sin B = sin 60° = 2  2bc 60λ2 60 5
3 3   a 2 + c2 − b2 λ2[49 + 25 − 36] 19
2 cos B = = =
2ac 70λ2 35
1
⇒ sin A = ⇒ ∠A = 30º a 2 + b2 − c2 λ2[49 + 36 − 25]
2 and cos C = =
So, ∠C = 90º 2ab 84λ2
∴ From sine rule, 60 5
= =
a b c 84 7
= =
sin A sin B sin C 1 7
Thus, cos A = = ,
a b 4 5 35
⇒ = = [Q c = 4 cm]
1 3 1 19 25
cos B = , cos C =
2 2 35 35
⇒ a = 2 cm, b = 2 3 cm cos A cos B cos C 1
= = =
1 1 7 19 25 35
∴ Area of ∆ABC = ab sin C = × 2 × 2 3 × 1
2 2 ⇒ (α , β , γ ) = (7, 19, 25)
= 2 3 sq. cm
rankershalt.com

Properties of Triangles 503

a b c q
3. We know that, = = = 2R D C
sin A sin B sin C α
and given that, a + b = x, ab = y and x2 − c2 = y θ

p
2
A

+
q
2
p p

π−(θ+α) α
c b A B
O x−q q
M
R x
p
B a C In ∆DAM, tan( π − θ − α ) =
x−q
p
∴ ( a + b)2 − c2 = ab ⇒ tan(θ + α ) =
q−x
⇒ a 2 + b2 − c2 = − 2ab + ab
⇒ q − x = p cot(θ + α )
⇒ a 2 + b2 − c2 = − ab
⇒ x = q − p cot(θ + α )
a 2 + b2 − c2 − ab 1
⇒ = =−  cot θ cot α − 1  q
2ab 2ab 2 = q− p  Q cot α = p 
 cot α + cot θ 
1
∴ cosC = − ⇒ C = 120°
2  q 
 cot θ – 1
a 2 + b2 − c2 p  q cot θ − p 
[using cosine rule, cosC = ] = q− p  = q− p 
 q   q + p cot θ 
2ab  + cot θ 
c  p 
Now, = 2R
sin C  q cos θ − p sin θ 
1 c c 2 = q− p 
⇒ R= =  q sin θ + p cos θ 
2 sin(120° ) 2 3
q2 sin θ + pq cos θ − pq cos θ + p2 sin θ
c ⇒ x=
∴ R= p cos θ + q sin θ
3
( p2 + q2)sin θ
4. Applying sine rule in ∆ABD, ⇒ AB =
p cos θ + q sin θ
q C
D 5. Since, A, B, C are in AP.
θ ⇒ 2 B = A + C i.e. ∠ B = 60º
a c
p
2

∴ (2 sin C cos C ) + (2 sin A cos A )


+
q
2

P
c a
= 2 k (a cos C + c cos A )
π−(θ+α)  
A B using, a = b = c = 1 
 sin A sin B sin C k 
AB p2 + q 2 = 2 k (b)
=
sin θ sin { π − (θ + α )} = 2 sin B [using b = a cos C + c cos A ]
AB p2 + q 2 = 3
⇒ =
sin θ sin(θ + α ) 6. Let a , b, c are the sides of ∆ABC.
p + q sin θ
2 2  q  b + c k (sin B + sin C )
⇒ AB = Q cos α =  Now, = [by sine rule]
sin θ cos α + cos θ sin α  p2 + q2 
a k sin A
 B + C  B − C  B − C
( p2 + q2)sin θ 2 sin   cos   cos  
=  2   2  b+ c  2 
p cos θ + q sin θ = ⇒ =
A A a A
p 2 sin cos sin
and sin α = 2 2 2
p2 + q 2  B − C
sin  
Alternate Solution b−c  2 
Also, =
Let AB = x a A
cos
2
rankershalt.com

504 Properties of Triangles

7. Given, ratio of angles are 4 : 1: 1. tan (P / 2) + tan (Q / 2)


⇒ =1
⇒ 4 x + x + x = 180° 1 − tan (P / 2) tan (Q / 2)
⇒ x = 30° − b/a
⇒ =1
1 − c/a
∴ ∠ A = 120°, ∠B = ∠C = 30°
−b / a
C ⇒ =1
a−c
30° a
−b
a ⇒ =1
a−c
b
⇒ −b = a − c
120 °
30° ⇒ a+ b=c
A c B
10. By the law of sine rule,
a a b c
Thus, ratio of longest side to perimeter = = = =k [say]
a+ b+ c sin P sin Q sin R
Let b=c=x P
⇒ a 2 = b2 + c2 − 2bc cos A [by cosine rule]
⇒ a 2 = 2x2 − 2x2 cos A
= 2x2(1 − cos A ) c b

⇒ a 2 = 4x2 sin 2 A / 2 p2 p3
p1
⇒ a = 2x sin A / 2
Q a R
⇒ a = 2x sin 60° = 3x
1
Thus, required ratio Also, ap1 = ∆
a 2
= 2∆
a+ b+ c ⇒ = p1
a
3x 2∆
= ⇒ p1 =
x + x + 3x k sin P
=
3 2∆ 2∆
Similarly, p2 = and p3 =
2+ 3 k sin Q k sin R
= 3 :2 + 3 Since, sin P, sin Q and sin R are in AP, hence p1 , p2, p3
8. We know that, A + B + C = 180° are in HP.

⇒ A + C − B = 180 − 2B 11. In ∆ABD, applying sine rule, we get


1  A
Now, 2ac sin ( A − B + C ) = 2ac sin (90° − B)
2  β
α
2 ac ⋅ (a + c − b )
2 2 2
= 2ac cos B = [by cosine rule]
2ac
= a 2 + c2 − b2
π/3 π/4
9. It is given that, tan (P / 2) and tan (Q / 2) are the roots of B x D 3x C
the quadratic equation ax2 + bx + c = 0
AD x
and ∠R = π /2 =
sin π / 3 sin α
∴ tan (P / 2) + tan (Q / 2) = − b/a 3
and tan (P / 2) tan (Q / 2) = c / a ⇒ AD = x sin α …(i)
2
Since, P + Q + R = 180°
⇒ P + Q = 90° and in ∆ACD, applying sine rule, we get
P+Q AD 3x
⇒ = 45° =
2 sin π / 4 sin β
 P + Q
⇒ tan   = tan 45° ⇒ AD =
3
x sin β
 2  …(ii)
2
rankershalt.com

Properties of Triangles 505

3x 3x ⇒ x = − (2 + 3 )
From Eqs. (i) and (ii), =
2 sin α 2 sin β and x=1+ 3
sin α 1 But x = − (2 + 3 )
⇒ =
sin β 6 ⇒ c is negative.
12. PLAN Whenever cosine of angle and sides are given or to find out, we ∴ x = 1 + 3 is the only solution.
should always use Cosine law. A
b2 + c 2 − a2 a2 + c 2 − b2 14. Given, cos B + cos C = 4 sin 2
i.e. cos A = , cos B = 2
2 bc 2 ac
a2 + b2 − c 2 A
and cosC =
2 ab

P
c b
n n
c b

B a C
(n + 2) (n + 4)
Fixed base

Q R  B + C  B −C 2 A
(n + 2) (n + 4) ⇒ 2 cos   cos   = 4 sin
 2   2  2
a
A  B −C A
b2 + c2 − a 2 ⇒ 2 sin cos   − 2 sin  = 0
∴ cos P = 2   2  2
2bc
1 (2n + 4)2 + (2n + 2)2 − (2n + 6)2  B −C  B + C A
⇒ = ⇒ cos   − 2 cos   = 0 as sin ≠ 0
 2   2  2
3 2(2n + 4) (2n + 2)
B C B C
 1  ⇒ − cos cos + 3 sin sin = 0
Q cos p = , given 2 2 2 2
 3 
B C 1
4n 2 − 16 1 ⇒ tan tan =
⇒ = 2 2 3
8(n + 1) (n + 2) 3 (s − a ) (s − c) (s − b) (s − a ) 1
n2 − 4 1 ⇒ . =
⇒ = s (s − b) s (s − c) 3
2(n + 1) (n + 2) 3
s− a 1
(n − 2) 1 ⇒ = ⇒ 2s = 3a
⇒ = s 3
2(n + 1) 3
⇒ b + c = 2a
⇒ 3n − 6 = 2n + 2 ⇒ n =8
∴ Locus of A is an ellipse.
∴ Sides are (2n + 2), (2n + 4), (2n + 6), i.e. 18, 20, 22.
15. Since, ∆ABC = ∆ABD + ∆ACD
a 2 + b2 − c2
13. Using, cos C = 1 1 A 1 A
2ab ⇒ bc sin A = c AD sin + b AD sin
2 2 2 2 2
A A

A A
2x

c 2 2
1

b
x 2-

1=
b=

E
30º

C B C
a = x2 + x + 1 B D
a
3 (x2 + x + 1)2 + (x2 − 1)2 − (2x + 1)2
⇒ =
2 2 (x2 + x + 1) (x2 − 1)
⇒ (x + 2) (x + 1) (x − 1) x + (x2 − 1)2 = 3 (x2 + x + 1) (x2 − 1) F
⇒ x2 + 2x + (x2 − 1) = 3 (x2 + x + 1) 2bc A
⇒ AD = cos
⇒ (2 − 3 ) x2 + (2 − 3 ) x − ( 3 + 1) = 0 b+ c 2
rankershalt.com

506 Properties of Triangles

Again, AE = AD sec
A
=
2bc k2 sin 2 A
⇒ sin A =
2 b+ c k sin 2 B − k2 sin 2 C
2

⇒ AE is HM of b and c. sin 2 A
A ⇒ sin A =
EF = ED + DF = 2DE = 2 AD tan sin B − sin 2 C
2

2
sin 2 A
2bc A A 4bc A ⇒ sin A =
=2 cos tan = sin sin (B + C ) sin (B − C )
b+ c 2 2 b+ c 2
sin 2 A
Since, AD ⊥ EF and DE = DF and AD is bisector. ⇒ sin A =
sin A ⋅ sin (B − C )
⇒ ∆AEF is isosceles.
Hence, (a), (b), (c), (d) are correct answers. ⇒ sin (B − C ) = 1 [Q sin A ≠ 0]
⇒ sin (B − 23° ) = sin 90°
16. The sine formula is
a b ⇒ B − 23° = 90°
= ⇒ a sin B = b sin A
sin A sin B ∴ B = 113°
(a) b sin A = a ⇒ a sin B = a 2 cos A cos B 2 cos C a b
18. Given, + + = + …(i)
π a b c bc ca
⇒ B=
2 b2 + c2 − a 2
π We know that, cos A =
Since, ∠ A < , therefore the triangle is possible. 2bc
2
c2 + a 2 − b2
(b) and (c) b sin A > a cos B =
2ac
⇒ a sin B > a ⇒ sin B > 1
∴ ∆ ABC is not possible. a 2 + b2 − c2
and cos C =
(d) b sin A < a 2ab
⇒ a sin B < a ⇒ sin B < 1 ⇒ ∠B exists. On putting these values in Eq. (i), we get
Now, b > a ⇒ B > A 2 (b2 + c2 − a 2) c2 + a 2 − b2
π +
Since, A < 2abc 2abc
2
∴ The triangle is possible. 2 ( a 2 + b2 − c2 ) a b
+ = +
Hence, (a) and (d) are the correct answers. 2abc bc ca
AD 2 (b2 + c2 − a 2) + c2 + a 2 − b2 + 2 (a 2 + b2 − c2)
17. In ∆ADC , = sin 23° ⇒
b 2abc
A
a 2 + b2
=
abc
b
c ⇒ 3b2 + c2 + a 2 = 2a 2 + 2b2
⇒ b2 + c2 = a 2
23° Hence, the angle A is 90°.
D a C
B
19. Let O be the centre and r be the radius of the circle
⇒ AD = b sin 23° passing through the vertices A1 , A2, …, An.
abc 2π
But AD = 2 [given] Then, ∠A1OA2 =
b − c2 n
abc also OA1 = OA2 = r
⇒ = b sin 23°
b2 − c2 Again, by cos formula, we know that,
a sin 23°  2π  OA1 + OA2 − A1 A2
2 2 2
⇒ = …(i) cos   =
b2 − c2 c  n 2(OA1 )(OA2)
Again, in ∆ABC, O
sin A sin 23°
=
a c 2
sin A a n
r r
⇒ = 2 [from Eq. (i)]
a b − c2
a2 A1
⇒ sin A = A2
b − c2
2
rankershalt.com

Properties of Triangles 507

 2π  r + r − A1 A2
2 2 2
⇒ cos   = 20. Let ABC be the triangle such that the lengths of its
 n 2(r )(r ) sides CA , AB and BC are (x − 1), x and (x + 1)
respectively, where x ∈ N and x > 1. Let ∠B = α be the
 2π 
⇒ 2r 2 cos   = 2r 2 − A1 A22 smallest angle and ∠ A = 2α be the largest angle.
 n
A
 2π 
⇒ A1 A22 = 2r − 2r cos  
2 2
 n 2 (x−1)
  2π   x
⇒ A1 A22 = 2r 21 − cos   
  n 
 π
⇒ A1 A22 = 2r 2 ⋅ 2 sin 2   B
(x +1)
C
 n
 π Then, by sine rule, we have
⇒ A1 A22 = 4r 2 sin 2  
 n sin α sin 2 α
=
 π x−1 x+1
⇒ A1 A2 = 2r sin  
 n sin 2α x + 1
⇒ =
 2π  sin α x−1
Similarly, A1 A3 = 2r sin  
 n x+1
⇒ 2 cos α =
 3π  x−1
and A1 A4 = 2r sin  
 n x+1
∴ cos α = ...(i)
Since,
1
=
1
+
1
[given] 2 (x − 1)
A1 A2 A1 A3 A1 A4
x2 + (x + 1)2 − (x − 1)2
1 1 1 Also, cos α = [using cosine law]
⇒ = + 2x (x + 1)
2r sin (π / n ) 2r sin (2π / n ) 2r sin (3π / n )
x+4
1 1 1 ⇒ cos α = ...(ii)
⇒ = + 2 (x + 1)
sin (π / n ) sin (2π / n ) sin (3π / n )
From Eqs. (i) and (ii),
 3π   2π  x+1 x+4
sin   + sin   =
1  n  n
⇒ = 2 (x − 1) 2 (x + 1)
sin(π / n ) sin (2π / n ) sin (3π / n )
⇒ (x + 1)2 = (x + 4) (x − 1)
 2π   3π   π  3π 
⇒ sin   ⋅ sin   = sin   sin   ⇒ x2 + 2x + 1 = x2 + 3x − 4
 n  n  n  n
⇒ x=5
 π  2π 
+ sin   ⋅ sin  
 n  n Hence, the lengths of the sides of the triangle are 4, 5
and 6 units.
 2π    3π   π   π  3π 
⇒ sin   sin  n  − sin  n   = sin  n  ⋅ sin  n  21. Let AD be the median to the base BC = a of ∆ABC
 n  
and let ∠ ADC = θ, then
 2π    3π + π   3 π − π  
⇒ sin   2 cos   sin  
 n    2n   2n    a a a a
 +  cot θ = cot 30° − cot 45°
 2 2 2 2
 π  3π 
= sin   . sin   3 −1
 n  n ⇒ cot θ =
2
 2π   2π   π  π  3π 
⇒ 2 sin   ⋅ cos   ⋅ sin   = sin   sin   A
 n  n  n  n  n
 2π   2π   3π  30° 45°
⇒ 2 sin   cos   = sin  
 n  n  n
 4π   3π 
⇒ sin   = sin  
 n  n
4π 3π
⇒ =π− B C
n n a/2 D a/2
7π Applying sine rule in ∆ ADC, we get
⇒ =π
n AD DC
=
⇒ n=7 sin (π − θ − 45° ) sin 45°
rankershalt.com

508 Properties of Triangles

a Applying cosine formula in ∆ABC, we have


AD 2
⇒ = b2 + c2 − a 2
sin (θ + 45° ) 1 cos A =
2bc
2
a 2 + b2 − c2
a and cos C = ...(ii)
⇒ AD = (sin 45° cos θ + cos 45° sin θ ) 2ab
2
a  cos θ + sin θ  a From Eqs. (i) and (ii),
⇒ AD =   = (cos θ + sin θ ) a 2 + b2 − c2 2b
2  2  2 =
2ab a
1 a  3 −1 2 
⇒ =  +  ⇒ a 2 + b2 − c2 = 4b2
11 − 6 3 2  8 −2 3 8 − 2 3 

⇒ a 2 − c2 = 3b2 ...(iii)
2 8 −2 3 2 8 −2 3 b + c − a 2b
2 2 2
⇒ a= = Now, cos A cos C = ⋅
( 3 + 1) 11 − 6 3 ( 3 + 1) 2
11 − 6 3 2bc a
b2 + c2 − a 2 3b2 + 3 (c2 − a 2)
2 8 −2 3 = =
= ac 3ac
(4 + 2 3 ) (11 − 6 3 )
(a 2 − c2) + 3 (c2 − a 2) 2 (c2 − a 2)
= =
2 8 −2 3 3ac 3ac
=
44 − 24 3 + 22 3 − 36 24. Given that,
8 −2 3 a = 1 + 3 , b = 2 and ∠C = 60°
=2 =2
8 −2 3 We have, c2 = a 2 + b2 − 2ab cos C
⇒ c2 = (1 + 3 )2 + 4 − 2(1 + 3 ) ⋅ 2 cos 60°
b+ c c+ a a + b
22. Let = = =λ ⇒ c2 = 1 + 2 3 + 3 + 4 − 2 − 2 3
11 12 13
⇒ c2 = 6
⇒ (b + c) = 11λ , c + a = 12λ , a + b = 13λ …(i)
⇒ c= 6
⇒ 2 (a + b + c) = 36λ
Using sine rule,
⇒ a + b + c = 18λ …(ii)
a b c
On solving Eqs. (i) and (ii), we get = =
sin A sin B sin C
a = 7λ , b = 6λ and c = 5λ
1+ 3 2 6
b2 + c2 − a 2 36λ 2 + 25λ 2 − 49λ 2 1 ⇒ = =
cos A = = = sin A sin B sin 60°
2bc 2 (30) λ 2 5
3

a 2 + c2 − b2 49λ 2 + 25λ2 − 36λ 2 19 2 sin 60° 2 = 1
cos B = = = ∴ sin B = =
2ac 70λ 2 35 6 6 2
a 2 + b2 − c2 49λ 2 + 36λ 2 − 25λ 2 5 ⇒ B = 45°
cos C = = =
2ab 84λ 2 7 ∴ A = 180° − (60°+45° ) = 75°
1 19 5
∴ cos A : cos B : cos C = : : = 7 : 19 : 25
5 35 7 Topic 2 Applications of Area, Napier’s
23. In ∆ADC, we have Analogy and Solution of a Triangle
B
1. For a ∆ABC, it is given that a = 3 + 1 ,
b = 3 − 1 and ∠A + ∠B = 120º
a/2
D A
90°–A

c
a/2

C c b
A b

AC
cos C = B a C
CD
2b Clearly, ∠C = 60º [Q ∠A + ∠B + ∠C = 180º ]
cos C = …(i)
a
rankershalt.com

Properties of Triangles 509

Now, by tangent law, we have A


A−B a−b C
tan = cot D 60° 60°
2 a+b 2 a a
( 3 + 1) − ( 3 − 1) √3
 60º 
= cot 
( 3 + 1) + ( 3 − 1)  2  O
2
= cot (30º ) 15°
2 3 15°
1 B C
= × 3 =1
3  1 + tan 45° tan 30°
⇒ a =1 + 3  
 A − B  tan 45° − tan 30° 
⇒tan   = 1 = tan 45º
 2   3 + 1
A−B =1 + 3   =4 + 2 3
⇒ = 45º  3 − 1
2
∴ Area of a triangle
⇒ ∠A − ∠B = 90º
1  3
On solving ∠A − ∠B = 90º and ∠A + ∠B = 120º ,we get = (4 + 2 3 )2   = (12 + 7 3 ) sq units
2  2
∠A = 105º and ∠B = 15º
So, ∠A : ∠B = 7 : 1 4. Let a : b : c = 1 : 3 : 2 ⇒ c2 = a 2 + b2
2. PLAN If ∆ABC has sides a, b, c . ∴ Triangle is right angled at C.
A
B
c b

B C c
a a
( s − b) ( s − a)
Then, tan ( A / 2) =
s ( s − a)
a+ b+ c
where, s= C b A
2
7 5 or ∠C = 90°
2+ +
⇒ s= 2 2 =4 a 1
2 and =
b 3
2 sin P − sin 2 P 2 sin P (1 − cos P )
∴ = a 1
2 sin P + sin 2 P 2 sin P (1 + cos P ) In ∆BAC , tan A = =
b 3
2 sin 2 (P /2)
= = tan 2 (P /2) ⇒ A = 30°
2 cos 2 (P /2)
and B = 60° [Q A + B = 90°]
P ∴ Ratio of angles, A : B : C = 30° : 60° : 90° = 1 : 2 : 3
a b c
b = 7/2
5. By sine rule, = =
c = 5/2 sin A sin B sin C
3 +1 b
Q R ⇒ =
a=2 sin (105° ) sin 30°
(s − b) (s − c) (s − b) (s − c)
⇒ × A
s (s − a ) (s − b) (s − c)
2 2 c 105° b
 7  5
4 −  4 − 
[(s − b)2 (s − c)2]  2  2
2
 3 B
30° 45°
= = =  C
∆2 ∆2  4∆  a = ( 3+1)

3. Let AB = AC = a and ∠ A = 120° . ( 3 + 1) sin 30°


1 ⇒ b=
∴ Area of triangle = a 2 sin 120° sin 105°
2
∴ Area of triangle
where, a = AD + BD = 3 tan 30° + 3 cot 15°
3 1 1 ( 3 + 1)sin 30° sin 45°
=1 + = ab sin 45° = ( 3 + 1)
tan (45° − 15° ) 2 2 sin 105°
rankershalt.com

510 Properties of Triangles

1 ( 3 + 1 )2 1 1 ⇒ tan A + tan B + tan C = 3 tan 60° = 3 3


= ⋅ ⋅ ⋅
2 (sin 45° cos 60° + cos 45° sin 60° ) 2 2 Conversely assume that,
1 (3 + 1 + 2 3 ) (4 + 2 3 ) tan A + tan B + tan C = 3 3
= = ⋅2 2
4 2  1 1 1 3  4 2 (1 + 3 ) But in ∆ABC, A + B = 180° − C
 ⋅ + ⋅ 
 2 2 2 2 Taking tan on both sides, we get
(1 + 3 ) 2
1+ 3 tan ( A + B) = tan (180° − C )
= = sq cm tan A + tan B
2(1 + 3 ) 2 ⇒ = − tan C
1 − tan A tan B
6. Since, a + 2a , 2a + 3 and a + 3a + 8 form sides of a
2 2
⇒ tan A + tan B = − tan C+ tan A tan B tan C
triangle.
⇒ tan A + tan B + tan C = tan A tan B tan C = 3 3
Now, a 2 + 3a + 8 < (a 2 + 2a ) + (2a + 3)
⇒ None of the tan A, tan B, tan C can be negative
⇒ a 2 + 3a + 8 < a 2 + 4a + 3
So, ∆ABC cannot be obtuse angle triangle.
⇒ a >5 ...(i)
Also, AM ≥ GM
Also, (a 2 + 3a + 8) + (2a + 3) > a 2 + 2a
1
⇒ 3a > −11 ⇒ [tan A + tan B + tan C ] ≥ [tan A tan B tan C ]1/3
3
11
⇒ a>− ...(ii) 1
3 ⇒ (3 3 ) ≥ (3 3 )1/3 ⇒ 3 ≥ 3.
3
Again, (a 2 + 3a + 8) + (a 2 + 2a ) > 2a + 3
So, equality can hold if and only if
⇒ 2a 2 + 3a + 5 > 0
tan A = tan B = tan C
which is always true.
or A = B = C or when the triangle is equilateral.
∴ Triangle is formed, if a > 5
9. By using triangular inequality,
1
7. Given, ∆≤ (a + b + c) abc c<a+b
4
1 ⇒ c2 < ca + ab
⇒ (a + b + c) abc ≥ 1
4∆
(a + b + c) abc Similarly, a 2 < ab + ac and b2 < bc + ab
⇒ ≥1
16∆2 ∴ a 2 + b2 + c2 < 2ab + 2bc + 2ca
2s abc ⇒ (a + b + c ) + 2ab + 2bc + 2ca < 4(ab + bc + ca )
2 2 2
⇒ ≥1
16∆2
⇒ (a + b + c)2 < 4(ab + bc + ca ) ... (i)
sabc
⇒ ≥1 Now, using AM-GM inequality in a , b and c, we get
8 ⋅ s (s − a ) (s − b) (s − c)
abc a 2 + b2 b2 + c2 c2 + a 2
⇒ ≥1 ≥ ab, ≥ bc and ≥ ca
8 (s − a ) (s − b) (s − c) 2 2 2
abc ⇒ a 2 + b2 + c2 ≥ ab + bc + ca
⇒ ≥ (s − a ) (s − b) (s − c)
8 ⇒ a 2 + b2 + c2 + 2ab + 2bc + 2ca ≥ 3(ab + bc + ca )
Now, put s − a = x ≥ 0, s − b = y ≥ 0, s − c = z ≥ 0 ⇒ (a + b + c)2 ≥ 3(ab + bc + ca ) ... (ii)
s−a + s−b=x+ y From Eqs. (i) and (ii), we get
2s − a − b = x + y 3(ab + bc + ca ) ≤ (a + b + c)2 < 4(ab + bc + ca )
c=x+ y 10. Since, A+ B+C=π
Similarly, a = y + z, b = x + z ⇒ B + C = π − π / 4 = 3π / 4 …(i)
(x + y) ( y + z ) (x + z ) [Q A = π / 4, given]
⇒ ⋅ ⋅ ≥ xyz
2 2 2 ∴ 0 < B, C < 3π / 4
which it true. Also, given tan B ⋅ tan C = p
Now, equality will hold, if x = y = z sin B ⋅ sin C p
⇒ =
⇒ a=b=c cos B ⋅ cos C 1
⇒ Triangle is equilateral. sin B ⋅ sin C + cos B cos C p+1
⇒ =
sin B ⋅ sin C − cos B ⋅ cos C p − 1
8. If the triangle is equilateral, then
cos (B − C ) 1 + p
A = B = C = 60° ⇒ =
cos (B + C ) 1 − p
rankershalt.com

Properties of Triangles 511

(1 + p)  B + C 1
⇒ cos (B − C ) = − …(ii) = cot   =
2 (1 − p)  2   B C
tan  + 
2 2
[Q B + C = 3π / 4]
Since, B or C can vary from 0 to 3π / 4  B C
1 − tan   ⋅ tan  
∴ 0 ≤ B − C < 3π / 4  2  2
=
1  B C
⇒ − < cos (B − C ) ≤ 1 …(iii) tan   + tan  
2  2  2
1 1+ p
From Eqs. (ii) and (iii), − < ≤1 Since, tan (B / 2) and tan (C / 2) are rational, hence
2 2 ( p − 1) tan ( A / 2) is a rational.
1 1+ p 1+ p 2 tan A / 2
⇒ − < and ≤1 Now, sin A = as tan ( A / 2) is a rational
2 2 ( p − 1) 2 ( p − 1) 1 + tan 2 A / 2
1+ p 1 + p − 2 p+ 2 number, sin A is a rational . Similarly, sin B and sin C
⇒ + 1 ≥ 0 and ≤0 are. Thus, a, sin A, sin B, sin C are rational, therefore
p−1 2 ( p − 1)
(ii) ⇒ (iii).
 1 + 2
(1 − 2 )  p −  Again, a, sin A, sin B, sin C are rational.
2p  1 − 2
⇒ ≥ 0 and ≤0 By the sine rule,
p−1 2 ( p − 1) a b c
= =
2p ( p − ( 2 + 1 )2 ) sin A sin B sin C
⇒ > 0 and ≥0
p−1 ( p − 1) a sin B a sin C
⇒ b= and c =
+ – + + – + sin A sin A
2
0 1 1 (√2+1) Since a, sin A, sin B and sin C are rational,
⇒ ( p < 0 or p > 1) Hence, b and c are also rational.
and ( p < 1 or p > ( 2 + 1)2) 1
Also, ∆ = bc sin A
On combining above expressions, we get 2

p < 0 or p ≥ ( 2 + 1)2 As b, c and sin A are rational, so triangle is rational


number. Therefore, a , b, c and triangle are rational.
i.e. p ∈ (−∞ , 0) ∪ [( 2 + 1) , ∞ ) 2
Therefore, (iii) ⇒ (i).
or p ∈ (−∞ , 0) ∪ [3 + 2 2 , ∞ )
12. Let ABC be a triangle with base BC = a and altitude
11. It is given that a , b, c and area of triangle are rational.
AD = p, then
B (s − c) (s − a )
We have, tan = A
2 s (s − b)
s (s − a )(s − b)(s − c)
= c b
s (s − b) p
a+ b+ c
Again, a , b, c are rational given, s = are
2
rational, Also, (s − b) is rational, since triangle is B C
D
rational, therefore we get a
 B ∆ 1
tan   = is rational. Area of ∆ABC = bc sin A
 2  s (s − b) 2
C ∆ 1
Similarly, tan = is rational. Also, area of ∆ABC = ap
2 s (s − c) 2
B C 1 1
Therefore a , tan , tan are rational. ∴ ap = bc sin A
2 2 2 2
which shows that, (i) ⇒ (ii). bc sin A
⇒ p=
a
Again, it is given that,
abc sin A
B C ⇒ p=
a , tan , tan are rational, then a2
2 2
A  π B + C abc sin A ⋅ (sin 2 B − sin 2 C )
tan = tan  −  ⇒ p=
2 2 2  a 2(sin 2 B − sin 2 C )
rankershalt.com

512 Properties of Triangles

abc sin A ⋅ sin (B + C ) sin (B − C ) ∴ Each term on the left of equation has positive
=
(b2 sin 2 A − c2 sin 2 A ) coefficient multiplied by perfect square, each term must
be separately zero.
 a b c 
using sine rule, sin A = sin B = sin C  ⇒ a=b=c
 
∴ Triangle is an equilateral.
abc sin 2 A ⋅ sin (B − C ) abc sin (B − C ) 1 1 a
= = 15. Since, ∆ = ap1 ⇒ =
(b2 − c2) ⋅ sin 2 A b2 − c2 2 p1 2∆
ab2r sin (B − C ) ar sin (B − C ) 1 b 1 c
= = Similarly, = , =
b2 − b2r 2 1 − r2 p2 2∆ p3 2∆
ar 1 1 1 1
⇒ p≤ [Q sin (B − C ) ≤ 1] ∴ + − = (a + b − c)
1 − r2 p1 p2 p3 2∆
2(s − c) s − c s(s − c) ab
13. Given, cos A cos B + sin A sin B sin C = 1 = = = ⋅
2∆ ∆ ab s∆
1 − cos A cos B
⇒ sin C = ...(i) ab C
sin A sin B = ⋅ cos 2
 a + b + c 2
1 − cos A cos B  ∆
⇒ ≤1 [Q sin C ≤ 1]  2 
sin A sin B
2ab C
= cos 2
⇒ 1 − cos A cos B ≤ sin A sin B (a + b + c)∆ 2
⇒ 1 ≤ cos ( A − B) 1
16. We know that, ∆ = ap1
⇒ cos ( A − B) ≥ 1 2
⇒ cos ( A − B) = 1 [Q as cos (θ ) ≤ 1] 2∆
⇒ p1 =
⇒ A − B =0 a
2∆ 2∆
On putting A = B in Eq. (i), we get Similarly, p2 = and p3 =
b c
1 − cos 2 A
sin C = 8∆3
sin 2 A Now, p1 p2 p3 =
abc
⇒ sin C = 1
abc
⇒ C = π /2 Since, ∆=
4R
Now, A+ B+C=π 8 (abc)3 (abc)2
π π  π ∴ p1 p2 p3 = ⋅ =
⇒ A+ B= ⇒ A= Q A = B and C = abc 64R3 8R3
2 4  2 
a 2 2 4
π π π 17. In ∆ ABC , by sine rule, = =
∴ sin A : sin B : sin C = sin : sin : sin sin A sin 30° sin C
4 4 2
⇒ C = 45° , C′ = 135°
1 1
⇒ a :b:c= : :1 When, C = 45° ⇒ A = 180° − (45°+30° ) = 105°
2 2
When, C′ = 135° ⇒ A = 180° − (135° + 30° ) = 15°
= 1 :1 : 2
A
14. Let a , b, c are the sides of a ∆ABC.
3 15°
Given, cos A + cos B + cos C =
2

2 4
2
2

2
b2 + c2 − a 2 a 2 + c2 − b2 a 2 + b2 − c2 3
⇒ + + = 45° 135° 30°
2bc 2ac 2ab 2 C
C′
B
⇒ ab2 + ac2 − a3 + ba 2 + bc2 − b3
1
+ ca 2 + cb2 − c3 = 3abc Area of ∆ABC = AB × AC sin A
2
⇒ a (b − c)2 + b (c − a )2 + c (a − b)2 1
(a + b + c) = × 4 × 2 2 sin (105° )
= [(a − b)2 + (b − c)2 + (c − a )2] 2
2
3 +1
⇒ (a + b − c) (a − b)2 + (b + c − a ) (b − c)2 =4 2 ×
2 2
+ ( c + a − b) ( c − a )2 = 0
= 2 ( 3 + 1) sq. units
[as we know, a + b − c > 0, b + c − a > 0, c + a − b > 0]
rankershalt.com

Properties of Triangles 513

Area of ∆ABC′ =
1
AB × AC sin A ⇒ b =3
2 3
and a=− [from Eq. (i)]
1 4
= × 4 × 2 2 sin (15° )
2 So, the third vertex
 3 
= 2 ( 3 − 1) sq. units (a , b) ≡  − , 3 , which lies in II quadrant.
 4 
Difference of areas of triangle
=|2 ( 3 + 1) − 2 ( 3 − 1)|= 4 sq units 2.
A
Alternate Solution
C′ E F
H
D
2 (1, 1)
2 2 2 C
2 2 C B
30° B 3x–2y+6=0 4x+5y–20=0
A
Let equation of AB be 4x + 5 y − 20 = 0 and AC be
3x − 2 y + 6 = 0
3
Clearly, slope of AC =
2
a
Here, AD = 2 , DC = 2 [Q slope of ax + by + c = 0 is −
]
b
Difference of areas of ∆ABC and ∆ABC′ ∴ Slope of altitude BH , which is perpendicular to
= Area of ∆ACC′ 2  1 
AC = − . Q mBH = − 
1 1 3  mAC 
= AD × CC′ = × 2 × 4 = 4 sq units
2 2 Equation of BH is given by y − y1 = m( x − x1 )
2
Here, m = − , x1 = 1 and y1 = 1
Topic 3 Circumcircle, Incircle, Escribed, 3
2
Orthocentre and Centroid of a ∴ y − 1 = − ( x − 1)
3
Triangle ⇒ 2x + 3 y − 5 = 0
1. Let ABC be a given triangle with vertices Now, equation of AB is 4x + 5 y − 20 = 0 and
B(0, 2), C (4, 3) and let third vertex be A (a , b) equation of BH is 2x + 3 y − 5 = 0
Solving these, we get point of intersection
A (a , b ) (i.e. coordinates of B).
4x + 5 y − 20 = 0
 ⇒ y = − 10
E 4x + 6 y − 10 = 0
F
(0,0)
On substituting y = − 10 in 2x + 3 y − 5 = 0, we get
35
x=
(0, 2) B C (4,3) 2
D  35 
∴ B , − 10
Also, let D , E and F are the foot of perpendiculars  2 
drawn from A , B and C respectively.
Solving 4x + 5 y − 20 = 0 and 3x − 2 y + 6 = 0, we get
b −0 3 −2
Then, AD⊥ BC ⇒ × = −1 coordinate of A.
a −0 4 −0
[if two lines having slopes m1 and m2, are
12x + 15 y − 60 = 0
12x − 8 y + 24 = 0 } ⇒ 23 y = 84
perpendicular then m1m2 = −1] 84 10
⇒ y= ⇒x=
⇒ b + 4a = 0 …(i) 23 23
and CF⊥ AB  10 84
∴ A , 
b −2 3 −0  23 23
⇒ × = −1
a −0 4 −0  84 
⇒ 3 b − 6 = −4 a  y2 − y1   23 − 1 61
Now, slope of AH =  = = .
⇒ 4a + 3b = 6 …(ii)  x2 − x1   10
− 1 − 13
From Eqs. (i) and (ii), we get  23 
−b + 3b = 6 ⇒ 2b = 6 Q BC is perpendicular to AH .
rankershalt.com

514 Properties of Triangles

13  1  But this could not determine the exact values of b


∴Slope of BC is Q mBC = − 
61  mAH  and c.
1
Now, equation of line BC is given by y − y1 = m(x − x1 ), 5. Here, R2 = MC 2 = (a 2 + b2) [by distance from origin]
4
where (x1 , y1 ) are coordinates of B.
1 2
13  35 = c [by Pythagoras theorem]
∴ y − (− 10) = x −  4
61  2
13 Y
⇒ y + 10 = ( 2x − 35)
61 × 2
⇒ 122 y + 1220 = 26x − 455 A
⇒ 26x − 122 y − 1675 = 0
a2 + b 2 − c 2 abc ∆
3. PLAN (i) cos C = (ii) R = ,r= M (a /2, b /2)
2 ab 4∆ s b

where, R, r , ∆ denote the circumradius, inradius and


area of triangle, respectively.
C X
a B
Let the sides of triangle be a , b and c.
Given, x=a + b c
⇒ R=
y = ab 2
x2 − c2 = y Next, r = (s − c) tan (C / 2) = (s − c) tan π / 4 = s − c
⇒ (a + b)2 − c2 = y ∴ 2(r + R) = 2r + 2R = 2s − 2c + c
⇒ a 2 + b2 + 2ab − c2 = ab = a + b + c−c
⇒ a + b − c = − ab
2 2 2 =a+b

a +b −c
2 2 2
1 6. Radius of incircle is, r =
⇒ = − = cos 120° s
2ab 2
2π Since, ∆ = 16 2
⇒ ∠C = 6 2 +6 2 +4 2
3 Now, s=
abc ∆ r 4∆2 2
Q R= ,r = ⇒ = =8 2
4∆ s R s (abc)
2 16 2
1  2π   ∴ r=
4  ab sin    8 2
 3 
= 
2
x+ c =2
⋅ y⋅ c
2 7. Equation of circumcircle of ∆PRS is
r 3y (x + 1) (x − 9) + y2 + λy = 0
∴ =
R 2c (x + c) It will pass through (1, 2 2 ), then
4. First solve each option separately. − 16 + 8 + λ ⋅ 2 2 = 0
(a) If a, sin A, sin B are given, then we can determine 8
a a ⇒ λ= =2 2
b= sin B, c = sin C. So, all the three 2 2
sin A sin A
∴ Equation of circumcircle is
sides are unique.
So, option (a) is incorrect. x2 + y2 − 8 x + 2 2 y − 9 = 0
(b) The three sides can uniquely make an acute angled Hence, its radius is 3 3 .
triangle. So, option (b) is incorrect. Alternate Solution
(c) If a, sin B, R are given, then we can determine 2 2
a sin B Let ∠ PSR = θ ⇒ sin θ =
b = 2R sin B, sin A = . So, sin C can be 2 3
b
determined. PR
∴ sin θ =
Hence, side c can also be uniquely determined. 2R
(d) If a, sin A, R are given, then ⇒ PR = 6 2 = 2R ⋅ sin θ
b c
= = 2R ⇒ R=3 3
sin B sin C
rankershalt.com

Properties of Triangles 515

8. Coordinates of P and Q are (1, 2 2 ) and (1, − 2 2 ). 10. Given a ∆XYZ, where 2s = x + y + z
s−x s− y s−z
Now, PQ = (4 2 )2 + 02 = 4 2 and = =
4 3 2
1
Area of ∆ PQR = ⋅ 4 2 ⋅ 8 = 16 2 sq units X
2
1
Area of ∆ PQS = ⋅ 4 2 ⋅ 2 = 4 2 sq units z y
2
Y
Y Z
x
P s−x s− y s−z
(1, 2 2) ∴ = =
4 3 2
(1,0) R 3s − (x + y + z ) s
X' S
X = =
(-3, 0) (-1, 0) O 4+3+2 9
(9, 0)
s−x s− y s−z s
Q or = = = = λ (let)
4 3 2 9
(1, -2 2)
⇒ s = 9λ, s = 4λ + x, s = 3λ + y
Y' and s = 2λ + z
Ratio of areas of ∆PQS and ∆PQR is 1 : 4. ∴ s = 9λ, x = 5λ, y = 6λ, z = 7λ
9. We have, Now, ∆ = s(s − x)(s − y)(s − z )
In ∆PQR P
[Heron’s formula]
∠PQR = 30° = 9λ ⋅ 4λ ⋅ 3λ ⋅ 2λ = 6 6λ2 …(i)
PQ = 10 3 8π
10 3 Also, πr 2 =
QR = 10 3
30° 8
⇒ r =
2
…(ii)
By cosine rule Q 10 R 3
PQ 2 + QR 2 − PR 2 xyz (5λ )(6λ )(7λ ) 35λ
cos 30° = and R= = = …(iii)
2PQ ⋅ QR 4∆ 4 ⋅ 6 6λ2 4 6
3 300 + 100 − PR 2 8 ∆2 216λ4
⇒ = Now, r2 = = =
2 200 3 3 S2 81λ2
⇒ 300 = 300 + 100 − PR 2 8 8 2
⇒ = λ [from Eq. (ii)]
⇒ PR = 10 3 3
Since, PR = QR = 10 ⇒ λ =1
∴ ∠QPR = 30° and ∠QRP = 120°
(a) ∆XYZ = 6 6λ2 = 6 6
1
Area of ∆PQR = PQ ⋅ QR ⋅ sin 30° ∴ Option (a) is correct.
2
1 1 35 35
= × 10 3 × 10 × = 25 3 (b) Radius of circumcircle (R) = λ=
2 2 4 6 4 6
Radius of incircle of ∴ Option (b) is incorrect.
Area of ∆ PQR
∆ PQR = (c) Since,
X Y
r = 4R sin ⋅ sin ⋅ sin
Z
Semi - perimetre of ∆ PQR 2 2 2
∆ 25 3 25 3
i.e. r= = = ⇒
2 2
= 4⋅
35 X Y
sin ⋅ sin ⋅ sin
Z
s 10 3 + 10 + 10 5( 3 + 2) 3 4 6 2 2 2
2
4 X Y Z
⇒ r = 5 3(2 − 3) ⇒ = sin ⋅ sin ⋅ sin
35 2 2 2
= 10 3 − 15
∴ Option (c) is correct.
abc 10 3 × 10 × 10 X +Y
and radius of circumcircle (R) = = = 10 2 Z 
4 × 25 3 (d) sin 2  = cos  
4∆  2   2
∴Area of circumcircle of X +Y Z s(s − z ) 9 × 2 3
∆PQR = πR 2 = 100 π as = 90° − = = =
2 2 xy 5 ×6 5
Hence, option (b), (c) and (d) are correct answer. ∴ Option (d) is correct.
rankershalt.com

516 Properties of Triangles

11. Let a straight line through the vertex P of a given 13. Since, sides of a triangle subtends α , β, γ at the centre.
∆ PQR intersects the side QR at the point S and the A
circumcircle of ∆ PQR at the point T.
Points P , Q , R, T are concyclic, then PS ⋅ ST = QS ⋅ SR
PS + ST
Now, > PS ⋅ ST [Q AM > GM]
2 γ β
α
P

B C

∴ α + β + γ = 2π ...(i)
Q Now, arithmetic mean
R
S π  π  π 
cos  + α  + cos  + β + cos  + γ
2  2  2 
T =
3
1 1 2 2 As we know that, AM ≥ GM, i.e.
and + > =
PS ST PS ⋅ ST QS ⋅ SR π π π
AM is minimum, when + α = + β = + γ
SQ + QR 2 2 2
Also, > SQ ⋅ SR
2 or α = β = γ = 120°
QR ∴ Minimum value of arithmetic mean
⇒ > SQ ⋅ SR
2
π  3
1 2 = cos  + α  = cos (210° ) = −
⇒ > 2  2
SQ ⋅ SR QR
360°
2 4 14. Here, central angle = = 40°
⇒ > 9
SQ ⋅ SR QR
1 1 2 4
∴ + > >
PS ST QS ⋅ SR QR
abc ∆ C
12. We have, R = and r = 20°
4∆ s
R abc s abc ⋅ s r
= ⋅ =
r 4∆ ∆ 4∆2 1 1
A1 M A2
abc
=
4 (s − a ) (s − b) (s − c) 1
In ∆ACM , = sin 20°
But a : b : c = 4 : 5 : 6 [given] r
a b c ⇒ r = cosec 20°
⇒ = = =k [let]
4 5 6 ∴ Radius of circle = cosec 20°
⇒ a = 4k, b = 5k, c = 6k 15. Since, the circles with radii 3, 4 and 5 touch each other
1 1 15k externally and P is the point of intersection of tangents.
Now, s = (a + b + c) = (4k + 5k + 6x) =
2 2 2 A
R (4k) (5k) (6k)
∴ =
r  15k   15k   15k 
4 − 4k  − 5k  − 6k
 2   2   2  C2
4 5
C1
30k3 30 ⋅ 8 16 4 P 5
= = =
3  15 − 8  15 − 10  15 − 12 7 ⋅5 ⋅3 7
k      3 3
 2   2  2 
C3
rankershalt.com

Properties of Triangles 517

⇒ P is incentre of ∆C1 C 2 C3 . Now, in right angled ∆IHJ,


Thus, distance of point P from the points of contact ∠ JIH = π / 2 − A / 2
= inradius (r) of ∆C1C 2 C3 [Q ∠ IEA = 90°, ∠ IAE = A /2 and ∠ JIH = ∠ AIE]
∆ s (s − a )(s − b)(s − c) In ∆JIH,
i.e. r= =
s s  π A r A r
tan  −  = 1 ⇒ cot = 1
where, 2s = 7 + 8 + 9 ⇒ s = 12  2 2  r − r1 2 r − r1
(12 − 7)(12 − 8)(12 − 9) 5 ⋅4 ⋅3 B
=
r2 C
and cot = 3
r
Hence, r = = = 5 Similarly, cot
12 12 2 r − r2 2 r − r3
n 2 2π On adding above results, we get
16. We know that, I n = r sin
2 n cot A / 2 + cot B / 2 + cot C / 2
[since, I n is area of regular polygon] = cot A / 2 cot B / 2 cot C / 2
2I n 2π r1 r2 r3 r1r2 r3
⇒ = sin [Q r = 1] …(i) ⇒ + + =
n n r − r1 r − r2 r − r3 (r − r1) (r − r2) (r − r3 )
π
and On = nr 2 tan 18. It is clear from the figure that, OA = R
2
A
[since, On is area of circumscribing polygon]
On π
= tan …(ii) A/2

90
n n B

°
F G
R
On dividing Eq. (i) by Eq. (ii), we get 90°
2π r
I
sin O
2I n n
=
On π
tan B C
n
2π IF
1 + cos AI =
In π n sin ( A / 2)
⇒ = cos 2 =
On n 2 r
Q ∆AIF is right angled triangle, so =
sin ( A / 2)
I n 1 + 1 − (2I n / n )
2
∴ = [from Eq. (i)]
On 2 But r = 4R sin ( A / 2) sin (B / 2) sin (C / 2)
On ∴ AI = 4R sin (B / 2) sin (C / 2)
⇒ In = (1 + (1 − (2I n / n )2 )
2 Again, ∠GOA = B ⇒ OAG = 90° − B
17. The quadrilateral HEKJ is a square, because all four Therefore, ∠IAO = ∠IAC − ∠OAC
angles are right angles and JK = JH . 1
= A / 2 − (90° − B) =
( A + 2B − 180° )
A 2
1 1
= ( A + 2B − A − B − C ) = (B − C )
2 2
A/2
A/2

In ∆OAI,OI 2 = OA 2 + AI 2 − 2(OA )( AI ) cos (∠IAO )


K = R2 + [4R sin (B / 2) sin (C / 2)]2
r1
 B − C
J 90° − 2R ⋅ [4R sin (B / 2) sin (C / 2)] cos  
 2 
F E
r1 90° r1 = [R2 + 16R2 sin 2(B / 2) sin 2(C / 2)
H  B − C 
− 8R2 sin (B / 2) sin (C / 2) cos  
 2  
I
r2 r3

= R2[1 + 16 sin 2(B / 2) sin 2(C / 2)


 B − C 
− 8 sin (B / 2) sin (C / 2) cos  
C
 2  
B
D
90° = R2[1 + 8 sin (B / 2) sin (C / 2)
  B − C  
HE = JK = r1 IE = r 2 sin (B / 2) sin (C / 2) − cos  
Therefore, and [given]
 2  
⇒ IH = r − r1
rankershalt.com

518 Properties of Triangles

= R2[1 + 8 sin (B / 2) sin (C / 2) 4∆2


⇒ b [b2 − (a − c)2] =
  B − C  B + C  B − C   s
cos   + cos   − cos  
 2   2   2   ⇒ b [b2 − (a − c)2] = 8(s − a )(s − b)(s − c)
⇒ b [{ b − (a − c)}{ b + (a − c)}]
  B + C 
= R21 − 8 sin (B / 2) sin (C / 2) cos   = 8(s − a )(s − b)(s − c)
  2  
⇒ b [(b + c − a )(b + a − c)] = 8(s − a )(s − b)(s − c)
  π A 
= R21 − 8 sin (B / 2) sin (C / 2) cos  −   ⇒ b [(2s − 2a )(2s − 2c)] = 8(s − a )(s − b)(s − c)
 2 2
⇒ b [2 ⋅ 2 (s − a )(s − c)] = 8(s − a )(s − b)(s − c)
 A B C π
Q + + = ⇒ b = 2s − 2b
 2 2 2 2 
⇒ 2b = a + c
= R2[1 − 8 sin ( A / 2) sin (B / 2) sin (C / 2)]
which shows that b is arithmetic mean between a
  r 
= R2 1 − 8    = R2 − 2Rr and c.
  4 R 
19. Since, r1 , r2 and r3 are exradii of ∆ABC are in HP.
Now, in right angled ∆BIO, 1 1 1
OB2 = BI 2 + IO 2 ∴ , , are in AP.
r1 r2 r3
⇒ R2 = BI 2 + R2 − 2Rr s−a s−b s−c
⇒ , , are in AP.
⇒ 2Rr = BI 2 ∆ ∆ ∆
⇒ 2 Rr = r 2 /sin 2(B / 2) ⇒ s − a , s − b, s − c are in AP.
⇒ 2R = r /sin 2(B / 2) ⇒ − a , − b, − c are in AP.
⇒ 2R sin 2 B / 2 = r ⇒ a , b, c are in AP.
⇒ R (1 − cos B) = r 3
abc ∆ 20. sin C = and C is given to be obtuse.
⇒ (1 − cos B) = 2
4∆ s 2π
⇒ C= = a 2 + b2 − 2ab cos C
4∆2 3
⇒ abc (1 − cos B) =
s 2π
= 62 + 102 − 2 × 6 × 10 × cos = 14
 a 2 + c2 − b2  4∆2 3
⇒ abc 1 − = s
2ac ∆ 225 × 3
  ∴ r= ⇒ r2 = =3
2
2ac − a 2 − c2 + b2  4∆2
s  6 + 10 + 14
⇒  
abc 
2ac = s  2 
 

Download Chapter Test


http://tinyurl.com/y4qv54y2 or
rankershalt.com

24
Vectors

Topic 1 Scalar Product of Two Vectors


Objective Questions I (Only one correct option) → $ , →b = $i − $j + k
7. Let a = $i + $j + k $ and → $ be
c = $i − $j − k
1. Let A( 3, 0, −1), B ( 2, 10, 6) and C(1, 2, 1) be the vertices → → →
three vectors. A vector v in the plane of a and b,
of a triangle and M be the mid-point of AC. If G → 1
whose projection on c is , is given by (2011)
divides BM in the ratio 2 : 1, then cos ( ∠GOA) (O 3
being the origin) is equal to (2019 Main, 10 April I)
(a) $i − 3$j + 3 k
$ (b) − 3 $i − 3$j − k$
1 1 1 1
(a) (b) (c) (d) $ $
(c) 3 i − j + 3 k$ $ $
(d) i + 3 j − 3 k $
15 2 15 30 6 10
π π 8. Two adjacent sides of a parallelogram ABCD are
2. If a unit vector a makes angles with $i, with $j and
3 4 → →
$ given by AB = 2$i + 10$j + 11k$ and AD = − $i + 2$j + 2k$ .
θ ∈( 0, π ) with k, then a value of θ is (2019 Main, 9 April II)
The side AD is rotated by an acute angle α in the
5π π 5π 2π
(a) (b) (c) (d) plane of the parallelogram so that AD becomes
6 4 12 3
AD’. If AD’ makes a right angle with the side AB,
3. Let A( 3, 0, −1), B ( 2, 10, 6) and C(1, 2, 1) be the vertices then the cosine of the angle α is given by (2010)
of a triangle and M be the mid-point of AC. 8 17 1 4 5
If G divides BM in the ratio 2 : 1, then cos ( ∠GOA) (a) (b) (c) (d)
9 9 9 9
(O being the origin) is equal to (2019 Main, 10 Jan I)
1 1 9. Let P, Q, R and S be the points on the plane with
(a) (b)
15 2 15 position vectors − 2$i − $j , 4$i , 3$i + 3$j and − 3$i + 2$j,
1 1 respectively. The quadrilateral PQRS must be a
(c) (d)
30 6 10 (2010)
$ , b = b i$ + b $j + 2 $ (a) parallelogram, which is neither a rhombus nor a
4. Let a = $i + $j + 2 k 1 2 k and rectangle
$
c = 5 $i + $j + 2 k be three vectors such that the (b) square
(c) rectangle, but not a square
projection vector of b on a is a. If a + b is
(d) rhombus, but not a square
perpendicular to c, then|b | is equal to
(2019 Main, 9 Jan II) 10. Let two non-collinear unit vectors a$ and b$ form an
(a) 6 (b) 4 (c) 22 (d) 32 acute angle. A point P moves, so that at any time t

5. If lines x = ay + b, z = cy + d and x = a ′ z + b′, the position vector OP (where, O is the origin) is
y = c′ z + d ′ are perpendicular, then(2019 Main, 9 Jan II) given by a$ cos t + b$ sin t. When P is farthest from

(a) ab′+ bc′+1 = 0 (b) bb′+ cc′+1 = 0 origin O, let M be the length of OP and u $ be the unit

(c) aa ′+ c + c′ = 0 (d) cc′+ a + a ′ = 0 vector along OP. Then, (2008, 3M)
6. Let O be the origin and let PQR be an arbitrary a$ + $b $ )1/ 2
(a) u$ = and M = (1 + a$ ⋅ b
triangle. The point S is such that | a$ + $b|
OP ⋅ OQ + OR ⋅ OS = OR ⋅ OP + OQ ⋅ OS a$ − $b $ )1/ 2
(b) u$ = and M = (1 + a$ ⋅ b
= OQ ⋅ OR + OP ⋅ OS | a$ − $b|
a$ + b$
Then the triangle PQR has S as its (c) u$ = and M = (1 + 2 a$ ⋅ $b)1/ 2
(2017 Adv.)
| a$ + $b|
(a) centroid (b) orthocentre a$ − $b
(d) u$ = and M = (1 + 2 a$ ⋅ $b)1/ 2
(c) incentre (d) circumcentre | a$ − $b|
rankershalt.com

520 Vectors

→ →
11. Let, a = $i + 2$j + k$ , b = $i − $j + k,
$ →c = $i + $j − k$ . A vector 18. The points with position vectors
→ → → 60$i + 3$j, 40i$ − 8$j, ai$ − 52$j are collinear, if (1983, 1M)
coplanar to a and b has a projection along c of
1 (a) a = − 40 (b) a = 40
magnitude , then the vector is (2006, 3M) (c) a = 20 (d) None of these
3
(a) 4 $i − $j + 4 k$ (b) 4 $i + $j − 4 k$ Objective Question II
(c) 2 $i + $j + k$ (d) None of these (One or more than one correct option)
→ → → → → →
12. If a , b , c are three non-zero, non-coplanar vectors 19. Let a = 2i$ − $j + k$ , b = i$ + 2$j − k$ and c = i$ + $j − 2 k$ be
→ → → →
→ →
→ → b⋅a→ → → b⋅a→ three vectors. A vector in the plane of b and c, whose
and b1 = b − a , b2 = b + a,
→2 → →
| a| | a|2 projection on a is of magnitude 2 / 3 , is (1993, 2M)
→ → → → → → → → (a) 2 $i + 3$j − 3 k$ (b) 2 $i + 3$j + 3 k$
→ → c⋅a→ c⋅b→ → → c⋅a→ c ⋅ b1 →
c1 = c − a − b , c2 = c − a− b1 , (c) − 2 $i − $j + 5 k$ (d) 2 $i + $j + 5 k$
→ → → → 2
| a|2 | b|2 | a |2 |b|
→ → → → → → Numerical Value
→ → c ⋅ a → c ⋅ b2 → → → c ⋅ a →
c3 = c− a− b 2 , c4 = a − a. 20. Let a and b be two unit vectors such that a ⋅ b = 0. For
→ 2 → 2 →
| a| | b2 | | a |2 some x , y ∈ R, let c = xa + yb + (a × b ). If |c| = 2 and
the vector c is inclined at the same angle α to both a
Then, which of the following is a set of mutually and b, then the value of 8 cos2 α is ......... . (2018 Adv.)
orthogonal vectors? (2005, 1M)
→ → → → → → → → → → → →
(a) {a , b1 , c1 } (b) {a , b1 , c2 } (c) {a , b2 , a3 } (d) {a , b2 , c4 } Fill in the Blanks
→ → → → →
13. If a and b1 are two unit vectors such that a + 2 b and 21. The components of a vector a along and perpendicular
→ → →
5 a − 4 b , are perpendicular to each other, then the to a non-zero vector b are ……and……respectively.
→ → (1988, 2M)
angle between a and b is (2002, 1M)
22. A, B, C and D, are four points in a plane with position
(a) 45° (b) 60° → → → →
1 2 vectors a , b , c and d respectively such that
(c) cos−1   (d) cos−1  
 3  7 → → → → → → → →
( a − d) ⋅ ( b − c ) = ( b − d) ⋅ ( c − a ) = 0
→ → →
14. If a,b and c are unit vectors, then The point D, then, is the… of the ∆ ABC. (1984, 2M)
→ →2 → →2 → →2
→ → →
|a − b| +|b − c| +| c − a| does not exceed (2001, 2M)
23. Let A, B, C be vectors of length 3, 4, 5 respectively.
(a) 4 (b) 9 → → → → → → →
(c) 8 (d) 6 Let A be perpendicular to B + C, B to C + A and C to
→ → → → →
→ → → → → → → A + B. Then, the length of vector A + B + C is ... .
15. Let u, v and w be vectors such that u + v + w = 0. If
→ → → → → → → → → (1981, 2M)
|u|= 3,|v|= 4 and|w|= 5, then u⋅ v + v⋅ w + w⋅ u is
(1995, 2M)
True/False
(a) 47 (b) – 25 (c) 0 (d) 25 → → → →
24. The points with position vectors a + b, a − b and
16. The number of vectors of unit length perpendicular → →
→ → a + k b are collinear for all real values of k. (1984, 1M)
to vectors a = (1, 1, 0) and b = ( 0, 1, 1) is (1987, 2M)

(a) one (b) two Analytical & Descriptive Questions


(c) three (d) infinite → → →

17. A vector a has components 2 p and 1 with respect to a 25. Find 3-dimensional vectors v1 , v 2 , v3 satisfying
→ → → → → →
rectangular cartesian system. This system is rotated v1 ⋅ v1 = 4, v1⋅ v2 = − 2, v1 ⋅ v3 = 6,
through a certain angle about the origin in the → → → → → →
counter clockwise sense. If, with respect to the new v2⋅ v2 = 2, v2⋅ v3 = − 5, v3 ⋅ v3 = 29 (2001, 5M)

system, a has components p + 1 and 1, then (1986, 2M) 26. Show, by vector methods, that the angular bisectors
1
(a) p = 0 (b) p = 1 or p = − of a triangle are concurrent and find an expression
3
1 for the position vector of the point of concurrency in
(c) p = − 1 or p = (d) p = 1 or p = − 1 terms of the position vectors of the vertices.
3
(2001, 5M)
rankershalt.com

Vectors 521

27. In a ∆ ABC, D and E are points on BC and AC → →


A ( t ) and B ( t ) are non-zero vectors for all t and
respectively, such that BD = 2DC and AE = 3EC. Let
→ → →
P be the point of intersection of AD and BE. Find A ( 0) = 2$i, A (1) = 6$i + 2$j, B ( 0) = 3$i + 2$j and
BP / PE using vector methods. (1993, 5M) → → →
B (1) = 2$j + 6$j. Then, show that A ( t ) and B ( t ) are
28. Determine the value of c, so that for all real x, the
parallel for some t.
vector cx $i − 6$j − 3k$ and x $i + 2$j + 2cx k$ make an
obtuse angle with each other. (1991, 4M)
Integer Answer Type Questions
29. In a ∆ OAB, E is the mid-point of BO and D is a point 32. Suppose that p , q and r are three non-coplanar
on AB such that AD : DB = 2 : 1. If OD and AE vectors in R3 . Let the components of a vector s along
intersect at P, determine the ratio OP : PD using p , q and r be 4, 3 and 5, respectively. If the
methods. (1989, 4M) components of this vector s along ( − p + q + r ),
30. Let OACB be a parallelogram with O at the origin ( p – q + r ) and ( − p – q + r ) are x , y and z respectively,
and OC a diagonal. Let D be the mid-point of OA. then the value of 2x + y + z is (2015 Adv.)
Using vector methods prove that BD and CO → → →
intersect in the same ratio. Determine this ratio. 33. If a, b and c are unit vectors satisfying
→ → 2 → → 2 → → 2
(1988, 3M) | a − b| +| b − c| +| c − a| = 9, then
→ → → → →
31. Let A ( t ) = f1( t )$i + f2( t )$j and B ( t ) = g( t )$i + g2( t )$j, | 2 a + 5 b + 5 c| is equal to (2012)

t ∈ [0, 1], f1 , f2 , g1g2 are continuous functions. If A ( t )

Topic 2 Vector Product of Two Vectors


Objective Questions I (Only one correct option) $ and b = $i − $j + k
5. Let a = 3$i + 2$j + xk $ , for some real x.
$ and b = $i + 2$j − 2k
1. Let a = 3$i + 2$j + 2k $ be two vectors. Then|a × b| = r is possible if (2019 Main, 8 April II)
3 3 3
If a vector perpendicular to both the vectors a + b and (a) 0 < r ≤ (b) <r≤3
a − b has the magnitude 12, then one such vector is 2 2 2
(2019 Main, 12 April II) 3 3 3
(c) 3 <r<5 (d) r ≥ 5
(a) 4 (2$i + 2$j + k$) (b) 4 (2$i − 2$j − k
$) 2 2 2
(c) 4 (2i$ + $ $
2 j − k) $ $
(d) 4 (− 2i − 2 j + k$) 6. A tetrahedron has vertices P(1, 2, 1),
Q( 2, 1, 3), R( − 1, 1, 2) and O( 0, 0, 0). The angle
2. Let α ∈ R and the three vectors between the faces OPQ and PQR is (2019 Main, 12 Jan I)
$ , b = 2i$ + $j − αk
a = αi$ + $j + 3k $ 7 9 19 17
(a) cos−1   (b) cos−1   (c) cos−1   (d) cos−1  
$ . Then, the set  31  35   35   31
and c = αi$ − 2$j + 3k
S = { α : a, b and c are coplanar} (2019 Main, 12 April II) 7. Let a = 2$i + $j − 2k,
$ b = i$ + $j and c be a vector such
(a) is singleton that |c − a| = 3, |( a × b ) × c| = 3 and the angle
(b) is empty between c and a × b is 30°. Then, a ⋅ c is equal to
(c) contains exactly two positive numbers (2017 Main)
(d) contains exactly two numbers only one of which is 25 1
(a) (b) 2 (c) 5 (d)
positive 8 8
→ → → →
3. If the length of the perpendicular from the point 8. If a and b are vectors such that |a + b| = 29 and
x y−1 z +1 3 →
(β, 0, β ) (β ≠ 0) to the line, = = is , $ ) = ( 2$i + 3$j + 4k
a × ( 2$i + 3$j + 4k $)× → b, then a possible
1 0 −1 2
→ → $ ) is
then β is equal to (2019 Main, 10 April +I) value of ( a + b ) ⋅ ( − 7$i + 2$j + 3k (2012)
(a) 2 (b) − 2 (a) 0 (b) 3 (c) 4 (d) 8
(c) − 1 (d) 1 → → → → → → →
r r r r r 9. Let a , b, c be unit vectors such that a + b + c = 0 .
$ If β = β − β , where
4. Let α = 3i$ + $j and β = 2i$ − $j + 3k.
r r 1 2
r r Which one of the following is correct? (2007, 3M)
β1 is parallel to α and β 2 is perpendicular to α, then → → → → → → →
r r (a) a × b = b × c = c × a = 0
β1 × β 2 is equal to (2019 Main, 9 April I) → → → → → → →
(b) a × b = b × c = c × a ≠ 0
1 1
(a) (3i$ − 9$j + 5k $) (b) (−3$i + 9$j + 5k$)
→ → → → → → →
2 2 (c) b × b = b × c = a × c = 0
(c) −3$i + 9$j + 5k
$ (d) 3i$ − 9$j − 5k
$) → → → → → →
(d) a × b, b × c , c × a are mutually perpendicular
rankershalt.com

522 Vectors

→ → →
10. If the vectors a , b and c from the sides Passage Based Problems
BC , CA and AB respectively of a ∆ ABC, then Let O be the origin and OX , OY, OZ be three unit
→ → → →
(a) a ⋅ b + b⋅ c + c ⋅ a = 0
→ → vectors in the directions of the sides QR , RP, PQ
(2000, 2M)
→ → → → → →
respectively, of a triangle PQR. (2017 Adv.)
(b) a × b = b × c = c × a
→ → → → → →
15. If the triangle PQR varies, then the minimum value of
(c) a ⋅ b = b ⋅ c = c ⋅ a cos ( P + Q ) + cos (Q + R ) + cos( R + P ) is
→ → → → → → → 3 3 5 5
(d) a × b+ b× c + c × a = 0 (a) − (b) (c) (d) −
2 2 3 3

Objective Questions II 16. |OX × OY| =


(a) sin(P + Q ) (b) sin(P + R )
(One or more than one correct option) (c) sin (Q + R ) (d) sin 2R
11. Let ∆PQR be a triangle. Let a = QR , b = RP and
c = PQ. If |a| = 12 ,| b| = 4 3 and b ⋅ c = 24, then Fill in the Blanks
which of the following is/are true? (2015 Adv.) → → →
17. If b and c are any two non-collinear unit vectors and a
|c|2 |c|2
(a) − |a| = 12 (b) + |a| = 30 is any vector, then
2 2
→ → →
(c)|a × b + c × a|= 48 3 (d) a ⋅ b = − 72 → → → → → → a ⋅(b × c) → →
(a ⋅ b) b + (a ⋅ c) c + ( b × c ) = ……
→ → →2 (1996, 2M)
12. Let A be vector parallel to line of intersection of | b × c|
planes P1 and P2 through origin. P1 is parallel to the
vectors 18. The unit vector perpendicular to the plane determined
by P (1,–1, 2), Q( 2, 0,–1) and R ( 0, 2, 1) is … . (1983, 2M)
2$j + 3k$ and 4$j − 3k$ and P2 is parallel to $j − k$ and
→ 19. The area of the triangle whose vertices are
3$i + 3$j , then the angle between vector A and
A(1,–1, 2), B( 2, 1,–1) C( 3,–1, 2) is … . (1983, 2M)
2i$ + $j − 2k$ is (2006, 5M)
π π π 3π True/False
(a) (b) (c) (d)
2 4 6 4
→ → → → → → →
→ → 20. Let A, B and C be unit vectors. If A⋅ B = A⋅ C = 0 and
13. Let a and b be two non-collinear unit vectors. If → →
→ → → → → → → → → that the angle between B and C is π / 6.
u = a − ( a ⋅ b ) b and v = a × b , then|v| is (1999, 3M)
→ → →
→ → → → Then, A = ± 2 ( B × C). (1981, 2M)
(a)|u| (b)|u|+|u ⋅ a|
→ → → → → → →
(c)|u|+|u ⋅ b| (d)|u|+ u ⋅ (a + b) Analytical & Descriptive Questions
→ → → →
Assertion and Reason 21. If a , b , c , d are four distinct vectors satisfying the
→ → → → → → → →
For the following question, choose the correct conditions a × b = c × d and a × c = b × d , then prove
answer from the codes (a), (b), (c) and (d) defined as → → → → → → → →
follows. that a⋅ b + c⋅ d ≠ a ⋅ c + b ⋅ d. (2004, 2M)
(a) Statement I is true, Statement II is also true; → →
Statement II is the correct explanation of 22. For any two vectors u and v, prove that
Statement I → → → → → →
(b) Statement I is true, Statement II is also true; (i)|u ⋅ v|2 +|u × v|2 =|u|2|v|2
Statement II is not the correct explanation of → → → → → → → →
(ii) (1 +|u|2 )(1 +|v|2 ) =|1 − u ⋅ v|2 +|u + v + ( u × v )|2
Statement I
(c) Statement I is true; Statement II is false (1998, 8M)
(d) Statement I is false; Statement II is true 23. If A, B, C , D are any four points in space, then prove
→ → → → → → → → → → → →
14. Let the vectors PQ, QR , RS , ST , TU and UP that|AB × CD + BC × AD + CA × BD|
represent the sides of a regular hexagon. = 4(area of ∆ ABC). (1987, 2M)
→ → → →
Statement I PQ × ( RS + ST ) ≠ 0 . 24. If A1 , A2 , …, An are the vertices of a regular plane
because polygon with n sides and O is its centre. Then, show
→ → → → → → that
Statement II PQ × RS = 0 and PQ × ST ≠ 0 . n −1
→ → → →
(2007, 3M)
∑ (OA i × OA i + 1) = (1 − n ) (OA 2 × OA 1).
i =1 (1982, 2M)
rankershalt.com

Vectors 523

Topic 3 Scalar Triple Product/Dot Product/Mixed Product


Objective Questions I (Only one correct option) → → →
9. If a , b and c are unit coplanar vectors, then the scalar
→ → → → → →
1. The sum of the distinct real values of µ, for which triple product [ 2 a − b 2 b − c 2c − a ] is (2000, 2M)
$ $i + µ$j + k
the vectors, µ$i + $j + k, $ , $i + $j + µk
$ are
(a) 0 (b) 1 (c) − 3 (d) 3
coplanar, is (2019 Main, 12 Jan I) → → →
(a) 2 (b) 0 (c) 1 (d) − 1 10. For three vectors u , v , w which of the following
$, $ expressions is not equal to any of the remaining
2. Let a = $i + 2$j + 4k b = i$ + λ$j + 4k and three?
$ $ 2 $
c = 2i + 4 j + ( λ − 1) k be coplanar vectors. Then, → → → → → →
(a) u ⋅ ( v × w ) (b) ( v × w ) ⋅ u (1998, 2M)
the non-zero vector a × c is (2019 Main, 11 Jan I) → → → → → →
(c) v ⋅ (u × w ) (d) (u × v ) ⋅ w
(a) − 10 $i + 5$j (b) − 10 $i − 5$j
→ → →
(c) − 14 $i − 5$j (d) − 14 $i + 5$j 11. If a = $i + $j + k$ , b = 4$i + 3$j + 4 k$ and c = $i + α $j + β k$
→ → → → →
3. If a , b , c and d are the unit vectors such that are linearly dependent vectors and| c| = 3 , then
(1998, 2M)
→ → → → → → 1 (a) α = 1, β = − 1 (b) α = 1, β = ± 1
( a × b ) ⋅ ( c × d ) = 1 and a ⋅ c = , then (2009)
(c) α = − 1, β = ± 1 (d) α = ± 1, β = 1
2
→ → → → → →
(a) a , b, c are non-coplanar 12. If a , b and c are three non-coplanar vectors, then
→ → → → → → → → → →
(b) a , b, d are non-coplanar ( a + b + c ) ⋅ [( a + b ) × ( a + c )] equals (1995, 2M)
→ →
(c) b, d are non-parallel →→→ →→→ →→→
(a) 0 (b) [ c b c ] (c) 2⋅ [ c b c ] (d) [a b c ]
→ → → →
(d) a , d are parallel and b, c are parallel → → → →
13. Let a = $i − $j , b = $j − k$ , c = k$ − $i . If d is a unit vector
4. The edges of a parallelopiped are of unit length and
→ → →→→ →
are parallel to non-coplanar unit vector a$ , $b, c$ such such that a⋅ d = 0 = [b c d ] , then d equals (1995, 2M)
1
that a$ ⋅ $b = $b ⋅ c$ = c$ ⋅ a$ = . Then, the volume of the $i + $j − 2 k$ $i + $j − k$
2 (a) ± (b) ±
6 3
parallelopiped is (2008, 3M) $i + $j + k$
(a)
1
cu unit (b)
1
cu unit (c) ± (d) ± k$
2 2 2 3
3 1 14. Let a , b, c be distinct non-negative numbers. If the
(c) cu unit (d) cu unit
2 3 vectors a $i + a $j + c k$ , $i + k$ and c $i + c $j + bk$ lie in a
5. The number of distinct real values of λ, for which plane, then c is (1993, 1M)
the (a) the arithmetic mean of a and b
vectors − λ i + $j + k$ , i$ − λ 2 $j + k$ and i$ + $j − λ 2 k$ are
2$ (b) the geometric mean of a and b
(c) the harmonic mean of a and b
coplanar, is (2007, 3M)
(d) equal to zero
(a) 0 (b) 1 (c) 2 (d) 3
→ →
6. The value of a, so that the volume of parallelopiped 15. Let a = a1i$ + a2$j + a3 k$ , a = b1$i + b2$j + b3 k$ and

formed by $i + a $j + k$ , $j + a k$ and a $i + k$ become a = c1$i + c2 $j + c3 k$ be three non-zero vectors such that
minimum, is (2003, 1M) → →
c is a unit vector perpendicular to both the vectors c
(a) − 3 (b) 3 (c) 1 / 3 (d) 3 → → → π
→ → → → → → → → and b. If the angle between a and b is , then
7. If V = 2 i + j − k and W = i + 3 k. If U is a unit 6
2
vector, then the maximum value of the scalar triple a1 a2 a3
→ → → b1 b2 b3 is equal to
product [U V W ] is (2002, 1M)
c1 c2 c3 (1986, 2M)
(a) −1 (b) 10 + 6
(c) 59 (d) 60 (a) 0
→ → (b) 1
8. If a = $i − k$ , b = x $i + $j + (1 − x ) k$ and 1
(c) (a12 + a22 + a32 ) (b12 + b22 + b32 )
→ → → → 4
c = y $i + x$j + (1 + x − y ) k$ . Then, [a b c ] depends on 3
(d) (a12 + a22 + a32 ) (b12 + b22 + b32 ) (c12 + c22 + c32 )
(a) only x (b) only y (2001, 2M) 4
(c) neither x nor y (d) both x and y
rankershalt.com

524 Vectors

16. The volume of the parallelopiped whose sides are → → →


→ → 23. If A , B , C are three non-coplanar vectors, then
given by OA = 2i$ − 3$j, OB = $i + $j − k$ , → → → → → →
→ A ⋅( B × C) B ⋅( A × C )
+ = ….
OC = 3$i − k$ , is (1983, 1M)
→ → → → → →
(1985, 2M)
4 ( C × A )⋅ B C ⋅( A × B )
(a) (b) 4
13
2  a a 2 1 + a3 
(c) (d) None of these 24. If  b2 1 + b3  = 0 and the vectors
7 bc c2 1 + c3 
→ → → → → →  
17. For non-zero vectors a , b , c|, ( a × b ) ⋅ c| → → →
→ → → A = (1, a , a 2 ), B = (1, b, b2 ), C (1, c, c2 )
=|a||b|| c| holds, if and only if (1982, 2M)
are non-coplanar, then the product abc = ...(1985,. 2M)
→ → → → → → → →
(a) a ⋅ b = 0, b⋅ c = 0 (b) b⋅ c = 0, c ⋅ a = 0
→ → → →
(c) c ⋅ a = 0, a ⋅ b = 0
→ → → → → →
(d) a ⋅ b = b⋅ c = c ⋅ a = 0
True/False
→ → →
→ → → → → → 25. For any three vectors a , b and c,
18. The scalar A ⋅ [( B + C ) × ( A + B + C )] equals → → → → → → → → →
(1981, 2M) ( a − b ) ⋅ { ( b − c ) × ( c − a )} = 2 a⋅ ( b × c ) (1983, 1M)

(a) 0 → → → → → →
→→→ →→→ 26. If X ⋅ A = 0, X ⋅ B = 0, X ⋅ C = 0 for some non-zero
(b) [ A B C ]+ [ B C A ]
→→→
→ →→→ (1983, 1M)
(c) [ A B C ] vector X , then [ A B C ] = 0.
(d) None of the above
Analytical & Descriptive Questions
Objective Questions II → → →
27. If u, v , w are three non-coplanar unit vectors and
(One or more than one correct option) → → → → →
α , β , γ are the angles between u and v , v and w , w
19. Let u = u1$i + u 2$j + u3 k$ be a unit vector in R3 and
→ → → →
1 $ $ $ ). Given that there exists a vector v and u respectively and x, y , z are unit vectors along
w= (i + j + 2k
6 the bisectors of the angles α , β , γ respectively. Prove
in R3 , such that|u + v| = 1 and w ⋅ (u + v) = 1.(2016 Adv.) that
Which of the following statement(s) is/are correct? → → → → → → 1 → → → 2 2α β γ
[x × y y × z z × x ] = [u v w] sec sec2 sec2
(a) There is exactly one choice for such v 16 2 2 2
(2003, 4M)
(b) There are infinitely many choices for such v
(c) If u$ lies in the XY-plane, then|u1| = |u2| 28. Let V be the volume of the parallelopiped formed by
(d) If u$ lies in the XY-plane, then 2|u1| = |u3| the vectors

20. Which of the following expressions are meaningfull $ →
a = a1$i + a2$j + a3 k, b = b1$i + b2$j + b3 k$
operations? (1998, 2M) →
→ → → → → → and c = c1$i + c2$j + c3 k$
(a) u⋅ ( v × w ) (b) (u ⋅ v )⋅ w
→ → → → → → If ar , br , cr , where r = 1, 2, 3 are non-negative real
(c) (u ⋅ v ) w (d) u × ( v ⋅ w ) 3
numbers and ∑ ( ar + br + cr ) = 3L. Show that V ≤ L3 .
r =1 (2002, 5M))
Fill in the Blanks
→ → →
→ → → → → → → 29. Let u and v be unit vectors. If w is a vector such
21. Let OA = a , OB = 10 a + 2 b and OC = b, where O, A → → → →
and C are non-collinear points. Let p denotes the that w + ( w × u) = v, then prove that
area of the quadrilateral OABC and let q denotes, → → → 1
|( u × v ) ⋅ w|≤ and that the equality holds if and
the area of the parallelogram with OA and OC as 2
adjacent sides. If p = kq, then k = …… . (1997, 2M) → →
(1999, 10M)
only if u is perpendicular to v.
22. If the vectors $ $ $ $ $ $
ai + j + k , i + bj + k and
30. The position vectors of the points A, B, C and D are
i$ + $j + ck$ ( a ≠ b ≠ c ≠ 1) are coplanar, then the value 3$i − 2$j − k$ , 2$i + 3$j − 4k$ , − $i + $j + 2k$ and 4$i + 5$j + λk$ ,
1 1 1
of + + = …… . (1987, 2M) respectively. If the points A, B, C and D lie on a
(1 − a ) (1 − b) (1 − c) plane, find the value of λ . 1
(1986, 2 M)
2
rankershalt.com

Vectors 525

Topic 4 Vector Triple Product


Objective Questions I (Only one correct option) → → →
8. Let a = 2$i + $j − 2k$ and b = $i + $j . If c is a vector such
$ and c be a vector such that
1. Let a = i$ − $j, b = $i + $j + k → → → → →
a × c + b = 0 and a ⋅ c = 4, then|c|2 is equal to that a ⋅ c =| c|,| c − a|= 2 2 and the angle between
(2019 Main, 9 Jan I) → → → → → →
19 17
( a × b ) and c is 30°, then|( a × b ) × c| is equal to
(a) 8 (b) (c) 9 (d) 2 3
2 2 (a) (b)
3 2 (1999, 2M)
2. Let a, b and c be three unit vectors, out of which (c) 2 (d) 3
vectors b and c are non-parallel. If α and β are the
→ → →
angles which vector a makes with vectors b and c 9. If a , b , c are non-coplanar unit vectors such that
1 → →
respectively and a × ( b × c) = b, then |α − β| is → → → (b + c) →
2 a × (b × c) = , then the angle between a and
equal to (2019 Main, 12 Jan II) 2
→ (1995, 2M)
(a) 30º (b) 45º (c) 90º (d) 60º b is
3π π π
$ b$ and c$ be three unit vectors such that
3. Let a, (a) (b) (c) (d) π
4 4 2
3 $
a$ × ( b$ × c$ ) =
( b + c$ ). If b$ is not parallel to c,
$ then
2 Objective Question II
the angle between a$ and b$ is (2016 Main)
3π π (One or more than one correct option)
(a) (b)
4 2 10. The vector(s) which is/are coplanar with vectors
(c)

(d)
5π $ and $i + 2$j + k
$i + $j + 2k $ , are perpendicular to the
3 6 $ is/are
vector $i + $j + k (2011)
→ → →
4. Let a, b and c be three non-zero vectors such that no (a) $j − k
$ (b) − $i + $j
→ → → 1 → → → $ $
two of them are collinear and ( a × b ) × c = |b|| c| a. (c) i − j (d) − $j + k$
3
→ →
If θ is the angle between vectors b and c , then a Numerical Value
value of sinθ is 11. Consider the cube in the first octant with sides OP,
2 2 – 2 2 –2 3 OQ and OR of length 1, along the X-axis, Y -axis and
(a) (b) (c) (d)
3 3 3 3 Z-axis, respectively, where O( 0, 0, 0) is the origin. Let
5. The unit vector which is orthogonal to the vector  1 1 1
S  , ,  be the centre of the cube and T be the
3$i + 2$j + 6 k$ and is coplanar with the vectors  2 2 2
2i$ + $j + k$ and i$ − $j + k$ is (2004, 1M) vertex of the cube opposite to the origin O such that S
lies on the diagonal OT . If p = SP , q = SQ , r = SR and
2 $i − 6 $j + k$ 2 $i − 3 $j
(a) (b) t = ST, then the value of|( p × q ) × ( r × t )| is ..... .
41 13
3 $j − k$ 4 $i + 3 $j − 3 k$ Fill in the Blank
(c) (d)
10 34 → → →
12. Let a , b and c be three vectors having magnitudes 1,
→ → → → → → → → → → →
6. If a = ( $i + $j + k$ ), a ⋅ b = 1 and a × b = $j − k,
$ then b is 1 and 2, respectively. If a × ( a × c ) + b = 0 , then the
→ →
equal to (2003, 1M) actue angle between a and c is …… . (1997, 2M)
(a) $i − $j + k$ (b) 2$j − k$
(c) $i (d) 2 $i
Analytical & Descriptive Questions
→ → → → → → → → →
7. Let the vectors a , b , c and d be such that 13. If A , B and C are vectors such that| B|=| C|. Prove
→ → → → → → → → →
→ → → →→ that [( A + B ) × ( A + C )] × ( B × C ) ⋅ ( B + C ) = 0 .
( a × b ) × ( c × d ) = 0 . If P1 and P2 are planes
(1997, 5M)
→ → → → → → →
determined by the pairs of vectors a , b, and c , d 14. If the vectors b , c , d are not coplanar, then prove
respectively, then the angle between P1 and P2 is → → → → → → → →
that the vector ( a × b ) × ( c × d ) + ( a × c ) × ( d × b )
(2000, 2M)
→ → → → →
(a) 0 (b) π / 4 + ( a × d ) × ( b × c ) is parallel to a. (1994, 4M)
(c) π / 3 (d) π / 2
rankershalt.com

526 Vectors

→ → components of A in the new coordinate system, in


15. (i) If C is a given non-zero scalar and A and B be given terms of A1 , A2, A3 . (1983, 2M)
→ →
non-zero vectors such that A ⊥ B , then find the
→ → →
Integer Answer Type Question
vector X which satisfies the equations A ⋅ X = c and $i − 2$j
→ → →
→ → → 16. If a and b are vectors in space given by a =
A × X = B. (1983, 2M)
5
→ → 2$i + $j + 3k$
(ii) A vector A has components A1, A2 , A3 in a and b = , then the value of
14
right-handed rectangular cartesian coordinate → → → → → →
system OXYZ. The coordinate system is rotated ( 2 a + b ) ⋅ [( a × b ) × ( a − 2 b )] is…. (2010)
π
about the X-axis through an angle . Find the
2

Topic 5 Solving Equations and Reciprocal of Vectors


Objective Questions I (Only one correct option) Fill in the Blanks

1. Let α = ( λ − 2) a + b and β = ( 4λ − 2) a + 3b be two 5. A non-zero vector a is parallel to the line of
given vectors where vectors a and b are non-collinear. intersection
The value of λ for which vectors α and β are collinear,
is (2019 Main, 10 Jan II) of the plane determined by the vectors $i, i$ + $j and
(a) 4 (b) −3 (c) 3 (d) −4 the plane determined by the vectors $i − $j, $i + k$ . The
2. Let a = 2$i + λ 1$j + 3k $ , b = 4$i + ( 3 − λ )$j + 6k
$ and →
2 angle between a and the vector $i − 2$j + 2k$ is…… .
c = 3$i + 6$j + ( λ 3 − 1)k $ be three vectors such that (1996, 2M)

b = 2a and a is perpendicular to c. Then a possible 6. A unit vector coplanar with $i + $j + 2k$ and $i + 2$j + k$
value of ( λ 1 , λ 2 , λ 3 ) is (2019 Main, 10 Jan I)
and perpendicular to $i + $j + k$ is …… . (1992, 2M)
(a) (1, 3, 1) (b) (1, 5, 1)
1 1 → →
(c)  − , 4, 0 (d)  , 4, − 2 7. Let b = 4$i + 3$j and c be two vectors perpendicular to
 2  2 
each other in the XY-plane. All vectors in the same
→ → →
3. Let a = 2$i + $j + k$ , b = $i + 2$j − k$ and a unit vector c be plane having projections 1 and 2 along b and c,
→ →

→ → → respectively are given by…. . (1987, 2M)


coplanar. If c is perpendicular to a, then c is equal to
(1999, 2M) → →
1 1 8. If A = (1, 1, 1), C = ( 0, 1, − 1) are given vectors, then
(a) (− $j + k$ ) (b) (− i$ − $j − k$ )
2 3 → → → →
1 $ 1 $ $ $ a vector B satisfying the equations A × B = C and
(c) (i − 2$j) (d) (i − j − k )
5 5 → →
A ⋅ B = 3 is …. (1985, 91, 2M)
→ → →
4. Let p, q , r be three mutually perpendicular vectors of
the same magnitude. If a vector x satisfies the
→ Analytical & Descriptive Questions
equation 9. Incident ray is along the unit vector v$ and the
→  → → → → → → → $ The normal
reflected ray is along the unit vector w.
p × (x − q ) × p + q × (x − r ) × q  is along unit vector a$ outwards. Express w
$ in terms
   
→ → → → → →
of a$ and v.
$ (2005, 4M)
+ r × (x − p) × r  = 0, then x is given by
  10. Let ABC and PQR be any two triangles in the same
(1997C, 2M)
plane. Assume that the perpendiculars from the
1→ → → 1→ → →
(a) ( p+ q − 2 r ) (b) ( p+ q + r ) points A, B, C to the sides QR , RP , PQ respectively
2 2
1 → → → 1 → → →
are concurrent. Using vector methods or otherwise,
(c) ( p + q + r ) (d) (2 p + q − r ) prove that the perpendiculars from P , Q , R to
3 3
BC , CA, AB respectively are also concurrent.
(2000, 4M)
rankershalt.com

Vectors 527

11. The position vectors of the vertices A, B and C of a → → →


tetrahedron ABCD are $i + $j + k$ , $i and 3$i, 14. If A = 2$i + k$ , B = $i + $j + k$ and C = 4$i − 3$j + 7k$ .
respectively. The altitude from vertex D to the → → → → →
Determine a vector R satisfyingR × B = C × B and
opposite face ABC meets the median line through A
→→
of the ∆ ABC at a point E. If the length of the side AD R ⋅ A = 0. (1990, 3M)
2 2
is 4 and the volume of the tetrahedron is , then → → →
3 15. If vectors a , b , c are coplanar, then show that
find the position vector of the point E for all its → → →
possible positions. (1996, 5M) a b c
→ → → → → → →
12. Prove, by vector methods or otherwise, that the point a⋅a a⋅ b a⋅c = 0
of intersection of the diagonals of a trapezium lies on → → → → → →
the line passing through the mid-points of the b⋅a b⋅ b b⋅ c
(1989, 2M)
parallel sides. (you may assume that the trapezium
is not a parallelogram). (1998, 8M) 16. Find all values of λ, such that x , y , z ≠ ( 0, 0, 0) and
→ → → ( i$ + $j + 3k$ )x + ( 3i$ − 3$j + k$ ) y
13. Let a , b and c be non-coplanar unit vectors, equally
+ ( − 4i$ + 5$j)z = λ ( i$x + $jy + k$ z ), where $i , $j, k$ are unit
inclined to one another at an angle θ. If
→ → → → → → → vectors along the coordinate axes. (1982, 2M)
a × b + b × c = p a + q b + r c , then find scalars p, q
and r in terms of θ. (1997C, 5M)

Answers
Topic 1 9. (a) 10. (c) 11. (d) 12. (d)
1. (a) 2. (d) 3. (a) 4. (b) 13. (a) 14. (b) 15. (c) 16. (b)
5. (c) 6. (b) 7. (c) 8. (b) 17. (d) 18. (a) 19. (b, c) 20. (a, c)
9. (a) 10. (a) 11. (a) 12. (b) 21. (6) 22. (1) 23. (0) 24. (–1)
13. (b) 14. (b) 15. (b) 16. (b) 146
25. True 26. True 30.
17. (b) 18. (a) 19. (a,c) 20. (3) 17
 → →   → →
 a ⋅ b →
21.  b and

a
 a⋅ b → Topic 4
→ 2 
−  → 2 b 1. (b) 2. (a) 3. (d) 4. (a)
 | b|   | b| 
5. (c) 6. (c) 7. (a) 8. (b)
22. Orthocentre 23. (5 2 ) 24. False π
9. (a) 10. (a, d) 11. (0.5) 12.
→ → → 6
25. v1 = 2 $i, v 2 = − $i + $j and v3 = 3 i$ ± 2 $j ± 4 k$
→ → →  →   
α a + β b+ γ c 8  4  →  c  →  1  → →
26. I = 27. 28. c ∈  − ,0 15. (i) X =  A −  → 2  ( A × B) (ii) ( A2$i − A1$j + A3k$ )
α +β+ γ 3  3  → 2
 | A|   | A| 
29. (3:2) 30. (2:1) 32. (9) 33. (3)
16. 5
Topic 2
1. (b) 2. (b) 3. (c) 4. (c) Topic 5
5. (d) 6. (c) 7. (b) 8. (c) 1. (d) 2. (c) 3. (a) 4. (b)
π 3π 1 $ $
9. (b) 10. (b) 11. (a, c, d) 12. (b, d) 5. or 6. ± ⋅ (− j + k ) 7. (2i$ − $j)
4 4 2
13. (a, c) 14. (c) 15. (a) 16. (a)
5 2 2 
→ $ $j+ k$ )
(2i+ 8.  $i , $j, k$  9. w$ = $v − 2 ( a$ ⋅ v)
$ a$
17. a 18. ± 3 3 3 
6
11. − $i + 3 $j + 3 k$ and 3 $i − $j − k$
19. 13 sq units 20. True
1 − 2 cos θ
13. p = r = , q=
Topic 3 1 + 2 cosθ 1 + 2 cos θ
1. (d) 2. (a) 3. (c) 4. (a) $ $ $
14. − i − 8 j + 2 k 16. 0,−1
5. (c) 6. (c) 7. (c) 8. (c)
rankershalt.com

528 Vectors

Hints & Solutions


Topic 1 Scalar Product of Two Vectors ⇒ OA = 9 + 1 = 10 and OG ⋅ OA = 6 − 2 = 4
4 1
1. Key Idea Use the angle between two non-zero vectors a and b ∴ cos (∠ GOA ) = =
a⋅ b 24 10 15
is given by cos θ = and coordinates of the centroid i.e.
| a | | b| 4. According to given information, we have the following
 x 1 + x 2 + x 3 , y 1 + y 2 + y 3 , z 1 + z 2 + z 3  of a triangle formed figure.
 
 3 3 3  a+b
with vertices; ( x 1, y 1, z 1), ( x 2 , y 2 , z 2) and ( x 3 , y 3 , z 3). b
c
Given vertices of a ∆ABC are A(3, 0, − 1), B(2, 10, 6) and
θ
C (1, 2, 1) and a point M is mid-point of AC. An another a
point G divides BM in ratio 2 : 1, so G is the centroid of
∆ABC. b⋅a
Clearly, projection of b on a =
| a|
 3 + 2 + 1 0 + 10 + 2 −1 + 6 + 1
∴ G
 3
,
3
,
3
 = (2, 4, 2).
 (b1$i + b2$j + 2k$ ) ($i + $j + 2k$ )
=
Now, cos (∠ GOA ) =
OG ⋅ OA
, where O is the origin. 1 2 + 1 2 + ( 2 )2
OG OA b1 + b2 + 2 b1 + b2 + 2
= =
Q OG =`2$i + 4$j + 2k $ ⇒ OG = 4 + 16 + 4 = 24 4 2
and OA = 3$i − k
$ ⇒ OA = 9 + 1 = 10 But projection of b on a =| a|
b + b2 + 2
∴ 1 = 1 2 + 1 2 + ( 2 )2
and OG ⋅ OA = 6 − 2 = 4 2
4 1 b + b2 + 2
∴ cos (∠ GOA ) = = ⇒ 1 = 2 ⇒ b1 + b2 = 2 ...(i)
24 10 15 2

2. Given unit vector a makes an angle


π π
with $i, with $j Now, a + b = ($i + $j + 2k$ ) + (b1$i + b2$j + 2k$ )
3 4 = (b1 + 1)$i + (b2 + 1)$j + 2 2k$
$
and θ ∈ (0, π ) with k.
Q (a + b ) ⊥ c, therefore (a + b ) ⋅ c = 0
Now, we know that cos 2 α + cos 2 β + cos 2 γ = 1 , where
α , β , γ are angles made by the vectors ⇒ {(b1 + 1)$i + (b2 + 1)$j + 2 2k} $ (5$i + $j + 2k$ ) = 0
with respectively $i, $j and k.
$ ⇒ 5(b1 + 1) + 1(b2 + 1) + 2 2 ( 2 ) = 0
2 π  2 π  2 ⇒ 5b1 + b2 = − 10 ...(ii)
∴ cos   + cos   + cos θ = 1
 3  4 From Eqs. (i) and (ii), b1 = − 3 and b2 = 5
1 1 1 1
⇒ + + cos θ = 1 ⇒ cos 2 θ = ⇒ cos θ = ±
2
⇒ b = − 3$i + 5$j + 2k$
4 2 4 2
 π  2π  π
⇒ cos θ = cos   or cos   ⇒ θ = or
2π ⇒ | b | = (− 3)2 + (5)2 + ( 2 )2 = 36 = 6
 3  3 3 3
2π 5 Let 1st line is x = ay + b, z = cy + d.
So, θ is , according to options.
3 x−b z−d x−b z−d
⇒ = y, = y⇒ = y=
a c a c
3. Key Idea Use the angle between two non-zero vectors a and b The direction vector of this line is b1 = a$i + $j + ck$ .
a⋅ b
is given by cos θ = and coordinates of the centroid i.e. Let 2nd line is x = a′ z + b′, y = c′ z + d′.
| a | | b|
 x 1 + x 2 + x 3 , y 1 + y 2 + y 3 , z 1 + z 2 + z 3  of a triangle formed x − b′ y − d′ x − b′ y − d′
  ⇒ = z, =z⇒ = =z
 3 3 3  a′ c′ a′ c′
with vertices; ( x 1, y 1, z 1), ( x 2 , y 2 , z 2) and ( x 3 , y 3 , z 3). The direction vector of this line is b 2 = a′ $i + c′ $j + k$ .
Given vertices of a ∆ABC are A(3, 0, − 1), B(2, 10, 6) and Q The two lines are perpendicular, therefore, b1 ⋅ b 2 = 0.
C (1, 2, 1) and a point M is mid-point of AC. An another ⇒ (a$i + $j + ck$ ) ⋅ (a′ $i + c′ $j + k$ ) = 0
point G divides BM in ratio 2 : 1, so G is the centroid of ⇒ aa′ + c′ + c = 0 ⇒ aa′ + c + c′ = 0
∆ABC.
 3 + 2 + 1 0 + 10 + 2 −1 + 6 + 1 6. OP ⋅ OQ + OR ⋅ OS = OR ⋅ OP + OQ ⋅ OS
∴ G , ,  = (2, 4, 2).
 3 3 3  ⇒ OP(OQ − OR) + OS(OR − OQ) = 0
OG ⋅ OA ⇒ (OP − OS)(OQ − OR) = 0
Now, cos (∠ GOA ) = , where O is the origin.
OG OA
⇒ SP ⋅ RQ = 0
Q OG =`2$i + 4$j + 2k $ ⇒ OG = 4 + 16 + 4 = 24
Similarly SR ⋅ PQ = 0 and SQ ⋅ PR = 0
and OA = 3$i − k
$
∴ S is orthocentre.
rankershalt.com

Vectors 529


7. Let v = a + λ b
→ → = $i (1 + t ) + $j (2 − t ) + k$ (1 + t )

→ → 1
v = (1 + λ ) i$ + (1 − λ ) $j (1 + λ ) k$ The projection of r on c = ⋅ [given]
3
→ → → →
→ 1 → v⋅ c 1 r⋅c 1
Projection of v on c = ⇒ = ⇒ =
3 →
| c| 3 → 3
| c|
(1 + λ ) − (1 − λ ) − (1 + λ ) 1 |1 ⋅ (1 + t ) + 1 ⋅ (2 − t ) − 1 ⋅ (1 + t )| 1
⇒ = ⇒ =
3 3 3 3
⇒ 1 + λ −1 + λ −1 − λ = 1 ⇒ (2 − t ) = ± 1 ⇒ t = 1 or 3
When, t = 1, we have r = 2$i + $j + 2k$
⇒ λ −1 = 1 ⇒ λ = 2 →

∴ v = 3 i − $j + 3k$
$ When, t = 3, we have r = 4$i − $j + 4k$

→ → → → → →
8. AB = 2 i$ + 10 $j + 11 k,
$ AD = − $i + 2 $j + 2 k$ →
12. Since, b1 = b −
→ b⋅a → → → b⋅a →
a , b1 = b + a
2 2
→ → |→
a| |→
a|
Angle ‘θ’ between AB and AD is
→ → → → → → → →
→ → → → c ⋅ a → c ⋅ b → → → c ⋅ a → c ⋅ b1 →
AB⋅ AD − 2 + 20 + 22 8 and c1 = c − a− b c2 = c − a − b1
cos(θ ) = = = |→ 2
|→ 2
|→ 2
|→ 2
→ →
(15) (3) 9 a| b| a| b|
| AB|| AD| → → → → → →
c ⋅ a → c ⋅ b2 → → → c ⋅ a →
→ →
17 c3 = c −
a− b2 , c 4 = a − a.
⇒ sin (θ ) = → 2
|a | → 2
|b | |→ |2
9 a
→ → → →2 → →
Since, α + θ = 90° which shows a ⋅ b1 = 0 = a ⋅ c2 = b1 ⋅ c2
17 → → →
∴ cos (α ) = cos (90° − θ ) = sin (θ ) = So, { a , b1 , c2} are mutually orthogonal vectors.
9
D C → → → →
13. Since, ( a + 2 b) ⋅ (5 a − 4 a ) = 0
→ → → →
α ⇒ 5|a|2 + 6 a ⋅ b − 8|b|2 = 0
θ → → → →
⇒ 6 a ⋅ b = 3 [ Q|a|=|b|= 1]
A B 1
1 1 ⇒ cos θ = ⇒ θ = 60°
9. mPQ = , mSR = , mRQ = − 3, mSP = − 3 2
6 6 → → → →2 → →
S (- 3, 2) R (3, 3)
14. Now, (a + b + c )2 = Σ a + 2 Σ a ⋅ b ≥0
→ → → → →
⇒ 2 Σ a⋅ b ≥ −3 [Q|a|=|b| =|c|= 1]
→ → → → →
Now, Σ|a − b|2 = 2 Σ a 2 − 2 Σ a ⋅ b ≤ 2 (3) + 3 = 9
→ → → → → → →
15. Since, u + v + w = 0 ⇒ |u + v + w|2 = 0
→ → → → → → → →
P (- 2, - 1) Q (4, 0) ⇒ |u|2 + |v|2 + |w|2 + 2 (u⋅ →
v + v⋅ w + w⋅ u ) = 0
→ → → → → →
⇒ Parallelogram, but neither PR = SQ nor PR ⊥ SQ. ⇒ 9 + 16 + 25 + 2 (u⋅ v + v⋅ w + w⋅ u ) = 0
→ → → → → →
∴ So, it is a parallelogram, which is neither a ⇒ u ⋅ v + v ⋅ w + w ⋅ u = − 25
rhombus nor a rectangle. → →
→ → → (a × b)
10. Given, OP = a$ cos t + $b sin t 16. A vector perpendicular to a and b is ± .
→ |→ →
a × b|
$ cos 2 t + ( $b⋅ $b) sin 2 t + 2a$ ⋅ $b sin t cos t
⇒ | OP| = (a$ ⋅ a) →
→ 17. Here, a = (2 p) $i + $j , when a system is rotated, the new
⇒ | OP| = 1 + a$ ⋅ $b sin 2t →
component of a are ( p + 1) and 1.
→ π
| OP|max = M = 1 + a$ ⋅ $b at sin 2t = 1 ⇒ t =
→ → →
⇒ i.e. b = ( p + 1) i$ + $j ⇒ |a|2 =|b|2
4
π → 1 or 4 p 2 + 1 = ( p + 1 )2 + 1 ⇒ 4 p 2 = p 2 + 2 p + 1
At t = , OP = $
(a$ + b) 2
4 2 ⇒ 3 p − 2 p − 1 = 0 ⇒ (3 p + 1) ( p − 1) = 0
→  π a$ + $b ⇒ p = 1, − 1 / 3
Unit vector along OP at  t =  =
 4  | a$ + $b| →
18. Three points A , B, C are collinear, if AB = − 20$i − 11$j
→ → →
11. Let vector r be coplanar to a and b. → → →
→ → →
and AC = (a − 60) $i − 55$j, then AB ||AC
∴ r =a+ tb a − 60 −55
→ ⇒ = ⇒ a = − 40
⇒ r = (i$ + 2$j + k$ ) + t (i$ − $j + k$ ) −20 −11
rankershalt.com

530 Vectors

→ $ →
19. Given vectors are a = 2$i − $j + k, b = $i + 2$j − k$ and 23. Given,
→ → →
| A | = 3,| B | = 4,| C | = 5
→ → → →
c = $i + $j − 2 k$ . Any vector r in the plane of b and c is → → → → → → → → →
Since, A ⋅ ( B + C ) = B⋅ ( C + A ) = C⋅ ( A + B ) = 0 ... (i)
→ → →
r = b + t( c ) = $i + 2$j − k$ + t ($i + $j − 2k)
$
→ → → → → →
∴ | A + B + C |2 = | A | + | B |2 + | C |2
= (1 + t )$i + (2 + t )$j − (1 + 2t )k$ …(i)
→→ → → →→
→ → 2 + 2 ( A ⋅ B + B ⋅ C + C⋅ A )
Since, projection of r on a is .
3
= 9 + 16 + 25 + 0
→ →
r⋅a 2 →→ →→ →→
∴ = [from Eq. (i), A ⋅ B + B⋅ C + C⋅ A = 0]

| a| 3
→ → →
2(1 + t ) − (2 + t ) − (1 + 2t ) 2 ∴ | A + B + C |2 = 50
⇒ =
6 3 → → →
⇒ | A + B + C |= 5 2
⇒ − (1 + t ) = 2 ⇒ t = 1 or −3
→ → → → → → →
On putting t = 1 , −3 in Eq. (i) respectively, we get 24. Let position vectors of points A, B and C be a + b, a − b
→ → →
r = 2$i + 3$j − 3k$ and a + k b , respectively.
→ → → → → → → →
→ (a − b) − ( a + b) = ( a + k b) − ( a − b)
or r = − 2$i − $j + 5k$ ∴
→ →
20. We have, ⇒ −2 b = (k + 1) b
→ → → → → → →
c = x a + y b + a × b and a ⋅ b = 0 ⇒ k + 1 = −2
→ → → ⇒ k = −3
| a | =| b | = 1 and| c | = 2
Hence, it is false statement.
→ → →
Also, given c is inclined on a and b with same angle α. → → →
→ → → 2 → → → → → 25. We have, |v1|= 2,|v2|= 2 and |v3 |= 29
∴ a ⋅ c = x| a| + y( a ⋅ b ) + a ⋅ ( a × b ) → →
→ → If θ is the angle between v1 and v2 , then
| a|| c | cos α = x + 0 + 0
2 2 cos θ = − 2
x = 2 cosα
1
Similarly, ⇒ cos θ = −
→ → 2
| b|| c |cos α = 0 + y + 0
⇒ θ = 135°
⇒ y = 2 cosα
→ → →
Since, any two vectors are always coplanar and data is
| c |2 = x 2 + y 2 +| a × b|2 not sufficient, so we can assume v1 and v2 in x − y
→ →

4 = 8 cos2 α +| a |2| b|2 sin2 90° plane.


4 = 8 cos2 α + 1 ⇒ 8 cos2 α = 3 →
v1 = 2$i [let]
→ →
21. Vector component of a along and perpendicular to b are →
v2 = − i$ + $j →
 → →  → → v2
 a⋅ b → →  a ⋅ b → →
 → 2 b and a − 
→ 2
b and v3 = α i$ + β $j + γ k$ →
v1
|b|   |b| 
→ →
→ → → → → → → →
Since, v3 ⋅ v1 = 6 = 2 α ⇒ α =3
22. As, (a − d ) ⋅ ( b − c ) = (b − d ) ⋅ ( c − a ) = 0 → →
Also, v3 ⋅ v2 = − 5 = − α ±β ⇒ β=±2
⇒ AD ⊥ BC and BD ⊥ CA
→ →
which clearly represents from figure that D is and v3 ⋅ v3 = 29 = α 2 + β 2 + γ 2 ⇒ γ = ± 4
orthocentre of ∆ABC. →
→ Hence, v3 = 3$i ± 2$j ± 4k$
A(a )
26. Let AD be the angular bisector of angle A. Let BC, AC
BD γ
and AB are α , β and γ, respectively. Then, = .

D(d ) DC β
→ →
B(b ) C(c )
rankershalt.com

Vectors 531

→ → Now, comparing the coefficients, we get


γ c + βb
Hence, position vector of D = . On AD, there 3l 2x
γ+β = …(i)
4 (l + m) 3 (x + y)
lies a point I which divides it in ratio γ + β : α .
→ → → l y
α a+ β b + γ c = , …(ii)
Now, position vector of I = 4 (l + m) x + y
α+β+γ
m x
→ → → and = …(iii)
which is symmetric in a , b, c , α , β and γ. l + m 3 (x + y)


On dividing Eq. (i) by Eq. (iii), we get
A (a)
3l 2x
4 (l + m) 3 (x + y)
=
F (γ) E (β) m x
l+m 3 (x + y)
I
3 l l 8 BP
⇒ ⋅ =2 ⇒ = =
→ → → 4 m m 3 PE
B (b) D (α) C(c)
→ → →
28. Let a = cx $i − 6$j + 3k$ and b = x $i + 2$j + 2cxk$ . Since, a and
Hence I lies on every angle bisector and angle →
bisectors are concurrent. b makes an obtuse angle.
→ →
→ → →
Here, α =|b − c|, β =|a − c|, γ =|a − b|.
→ → → ⇒ a ⋅ b < 0 ⇒ c x2 − 12 + 6 cx < 0
⇒ c <0 and discriminant < 0
27. Let the position vectors of A
→ → →
⇒ c <0 and 36c2 − 4 ⋅ (−12)c < 0
A , B and C are a , b and c 3 ⇒ c <0 and 12 c (3c + 4) < 0
respectively, since the point x
m E ⇒ c <0 and c > − 4 /3
D divides BC in the ratio of
l y 1 ∴ c ∈ (−4 / 3, 0)
2 : 1, the position vector of D P
will be B C
2 D 1 → → → →
29. Let O be origin and OA = a , OB = b
 → → →
 2 c + b → b →
D≡ OE = [since E being mid-point of OB]
3  2
 

and the point E divides AC in the ratio 3 : 1, B (b)
 → → 1
 3 c + a 1
therefore E ≡  . D
4  E
 
2
Now, let P divides BE in the ratio l : m and AD in the 1 P
ratio x : y. →
O A (a)
Hence, the position vector of P getting from BE and
AD must be the same. → →
→ a ⋅1 + b⋅2
Hence, we have OD =
1+2
 → →  → →
 3 c + a →  2 c + b →

l  + m b x   + ya
4 3 (since, D divides AB in the ratio of 2 : 1)
   
= → →
l+m x+ y → →  a + 2 b
⇒ Equation of OD is r = t 
→ → → → 3 
3l c l a → 2cx bx →
 
+ +m b + + ya
⇒ 4 4 = 3 3 → 
→ → →  b →
l+m x+ y and equation of AE is r = a + s  − a
2
 

3l → l → m b
⇒ c+ a+ → →
4 (l + m) 4 (l + m) l+m If OD and AE intersect at P, then there must be some
2x → x → y → →
= c+ b+ a r for which they are equal.
3 (x + y) 3 (x + y) (x + y)
rankershalt.com

532 Vectors

→ → →  From Eqs. (i) and (ii),


 a + 2 b →  b →
⇒ t = a + s − a → → → →
3  2  a+ b µ a+2a 1 µ 1 1
    = ⇒ = and =
λ + 1 2 ( µ + 1) λ + 1 2 ( µ + 1) λ+1 µ+1
t 2t s
⇒ = 1 − s and = On solving, we get µ = λ = 2
3 3 2
3 4 Thus, required ratio is 2 : 1.
⇒ t= and s =
5 5 → →
31. A (t ) is parallel to B(t ) for some t ∈[0, 1], if and only if
→ →
a+2b f1 (t ) f (t )
∴ Point P is . …(i) = 2 for some t ∈ [0, 1]
5 g1 (t ) g2(t )
→ or f1 (t ) ⋅ g2(t ) = f2(t ) ⋅ g1 (t ) for some t ∈ [0, 1]
Since, P divides OD in the ratio of λ : 1.
Let h (t ) = f1 (t ) ⋅ g2(t ) − f2(t ) g1 (t )
→ →
 a + 2 b ∴ h(0) = 2 × 2 − 3 × 3 = −5 < 0
λ + 1 ⋅0
3  → →
and h (1) = f1 (1) ⋅ g2(1) = f2(1) ⋅ g1 (1)
   a + 2 b
∴ = …(ii)
λ+1 5  = 6 × 6 − 2 × 2 = 32 > 0
 
Since, h is a continuous function and h (0) ⋅ h (1) < 0,
From Eqs. (i) and (ii), Therefore, here is some t ∈ [0, 1] for which h (t ) = 0,
→ →
λ 1 i.e. A (t ) and B(t ) are parallel vectors for this t.
=
3 (λ + 1) 5
32. Here, s = 4 p + 3q + 5r …(i)
⇒ 5λ = 3λ + 3
and s = (− p + q + r )x + ( p − q + r ) y + (− p − q + r ) z …(ii)
3
⇒ λ= ∴ 4 p + 3q + 5r = p (− x + y − z ) + q(x − y − z ) + r (x + y + z )
2
OP 3 On comparing both sides, we get
∴ =
PD 2 − x + y − z = 4, x − y − z = 3 and x + y + z = 5

30. OACB is a parallelogram with O as origin. Let On solving above equations, we get
→ → → → → → → 9 −7
OA = a, OB = b , OC = a + b x = 4, y = , z =
2 2
→ 9 7
B (b) ∴ 2x + y + z = 8 + − = 9
C 2 2
33. PLAN If a, b,c are any three vectors
→ → →
P Then | a + b + c| 2 ≥ 0
→ → → → → → → → →
1 ⇒ | a | 2 + | b | 2 + | c | 2 + 2 ( a ⋅ b + b⋅ c + c ⋅ a ) ≥ 0
1
→ → → → → → −1 → → →

→ ∴ a ⋅ b + b⋅ c + c ⋅ a ≥ (| a | 2 + | b | 2 + | c | 2 )
O A (a) 2
D (a /2)
→ → → → → →
→ Given, |a − b|2 + |b − c |2 + |c − a |2 = 9
→ a
and OD = → → → → → → → → → →
2 ⇒ |a |2 + | b|2 − 2 a ⋅ b + |b|2 + |c |2 − 2 b ⋅ c + |c |2 + |a |2
→ →
→ → −2 c⋅ a = 9
CO and BD meets at P.
→ → → → → → → → →
→ → ⇒ 6 − 2 (a ⋅ b+ b ⋅ c + c⋅ a ) = 9 [Q|a|=|b|=|c |= 1]
→ λ ⋅ 0 + 1 (a + b) →
∴ OP = [along OC] → → → → → → 3
λ+1 ⇒ a ⋅ b + b⋅ c + c ⋅ a = − …(i)
2
→ →
→ a+b → → → → → → −1 → 2 → 2 → 2
⇒ OP = …(i) Also, a ⋅ b + b⋅ c + c ⋅ a ≥ (|a | + |b| + |c | )
λ+1 2
 → 3
 a → ≥− …(ii)
µ   + 1 (b) 2
2
→   → → → →
Again, OP = [along BD] From Eqs. (i) and (ii), |a + b + c |= 0
µ+1
→ → → → → → → → →
→ → as a ⋅ b + b ⋅ c + c ⋅ a is minimum when|a + b + c |= 0
→ µ a+2b → → →
⇒ OP = …(ii) ⇒ a+ b+ c =0
2 (µ + 1)
∴ |2 a + 5b + 5 c|=|2 a + 5 (b + c)|=|2 a − 5 a|= 3
rankershalt.com

Vectors 533

$i $j $
Topic 2 Vector Product of Two Vectors k
1. Given vectors are So, PQ × PM = β −1 β + 1
1 0 −1
$ and b = $i + 2$j − 2k
a = 3i$ + 2$j + 2k $
= $i − $j(− β − β − 1) + k
$ = $i + (2 β + 1)$j + k
$
Now, vectors a + b = 4i$ + 4$j and a − b = 2$i + 4k
$
|PQ × PM| 1 + (2β + 1)2 + 1
∴ A vector which is perpendicular to both the vectors Now, = …(iii)
|PM| 2
a + b and a − b is
From Eqs. (ii) and (iii), we get
$i $j k
$
1 + (2β + 1)2 + 1 3 1 + (2β + 1)2 + 1 3
(a + b) × (a − b) = 4 4 0 = ⇒ =
2 2 2 2
2 0 4
[squaring both sides]
= $i (16) − $j(16) + k
$ (− 8) = 8(2$i − 2$j − k
$) ⇒ (2β + 1)2 = 1 ⇒ 2β + 1 = ±1
⇒ 2β + 1 = 1 or 2β + 1 = −1 ⇒ β = 0 or β = −1
Then, the required vector along (a + b) × (a − b) having
→ →
magnitude 12 is 4. Given vectors α = 3$i + $j and β = 2$i − $j + 3k$ and
8(2i$ − 2$j − k
$) → → → → → →
± 12 × = ± 4(2$i − 2$j − k
$) β = β1 − β 2 such that β1 is parallel to α and β 2 is
8× 4+4+1 perpendicular to α

2. Given three vectors are So, β = λα = λ (3$i + $j)
1
a = αi$ + $j + 3k
$
Now,
→ → →
β 2 = β1 − β = λ (3$i + $j) − (2$i − $j + 3k
$)
b = 2i$ + $j − αk$ $ $
= (3λ − 2)i + (λ + 1) j – 3k $
and c = α$i − 2$j + 3k
$ → →
Q β 2 is perpendicular to α, so β 2⋅ α = 0
α 1 3
[since if non-zero vectors a and b are perpendicular to
Clearly, [a b c] = 2 1 −α each other, then a ⋅ b = 0]
α −2 3 ∴ (3λ − 2)(3) + (λ + 1)(1) = 0
= α (3 − 2 α ) − 1 (6 + α 2) +3 (− 4 − α ) ⇒ 9λ − 6 + λ + 1 = 0
= − 3 α 2 − 18 = − 3 (α 2 + 6) ⇒ 10λ = 5 ⇒ λ =
1
Q There is no value of α for which − 3(α 2 + 6) becomes 2
α 1 3 → 3 1
So, β1 = i$ + $j
zero, so = 2 1 −α [a b c] ≠ 0 2 2
→ 3  $ 1  $ = − 1 $i + 3 $j − 3k
α −2 3 and β 2 =  − 2 i +  + 1 $j − 3k $
2  2  2 2
⇒ vectors a , b and c are not coplanar for any value
α ∈ R. $i $j $
So, the set S = {α : a , b and c are coplanar} is empty set. k
→ → 3 1  3   9  $  9 1
∴β 1 × β 2
= 0 = $i  − − 0 − $j − − 0 + k + 
3. Equation of given line is 2 2  2   2   4 4
x y−1 z + 1 −1 3 − 3
= = …(i)
1 0 −1 2 2
3 9 5$ 1
Now, one of the point on line is P(0, 1, − 1) and the given = − $i + $j + k = (−3$i + 9$j + 5k
$)
point is Q(β , 0, β ). 2 2 2 2
5. Given vectors are a = 3i$ + 2$j + xk$
Q(β,0, β)
and b = i$ − $j + k$

l  $i $j k $
 
∴ a × b = 3 2 x = $i (2 + x) − $j(3 − x) + k
$ (−3 − 2)

 

M x y–1 z+1 1 −1 1
P(0, 1, –1) = =  
1 0 –1
= (x + 2)i + (x − 3)$j − 5k
$ $
From the figure, the length of the perpendicular
3 ⇒ |a × b| = (x + 2)2 + (x − 3)2 + 25
QM = l = (given)
2
= 2x2 − 2x + 4 + 9 + 25
PQ × PM 3
⇒ = …(ii) 2
PM 2  1 1  1 75
= 2 x2 − x +  − + 38 = 2 x −  +
Q PQ = β$i − $j + (β + 1)k
$  4 2  2 2
and PM = a vector along given line (i) = $i − k
$
rankershalt.com

534 Vectors

2 → → → → → → →
 1 1  1 75 ⇒ a × b − a × c = 0 ⇒ a × ( b − c) = 0
= 2  x2 − x +  − + 38 = 2  x −  +
 4  2  2  2 → → → → → →
i.e. a || ( b − c) or b − c = λ a
75 1 → →
So, |a × b| ≥ [at x = , |a × b|is minimum] Here, a × ( 2i$ + 3$j + 4k$ ) = ( 2i$ + 3$j + 4k$ ) × b
2 2
→ →
3 ⇒ a × ( 2$i + 3$j + 4k$ ) − ( 2$i + 3$j + 4k$ ) × b = 0
⇒ r ≥5
2 → →
⇒ (a + b) × ( 2$i + 3$j + 4k$ ) = 0
6. The given vertices of tetrahedron PQRO are P(1, 2, 1), → →
Q (2, 1, 3), R (−1, 1, 2) and O (0, 0, 0). ⇒ a + b = λ ( 2$i + 3$j + 4k$ ) …(i)
The normal vector to the face OPQ → →
Since, | a + b| = 29
= OP × OQ = ($i + 2$i + k$ ) × (2$i + $j + 3k$ )
⇒ ± λ 4 + 9 + 16 = 29
$i $j k$ ⇒ λ=±1
= 1 2 1 = 5$i − $j − 3k$ ∴
→ →
a + b = ± ( 2$i + 3$j + 4k$ )
2 1 3 → →
Now, (a + b) ⋅ (− 7$i + 2$j + 3k$ ) = ± (− 14 + 6 + 12) = ± 4
and the normal vector to the face PQR
→ → → → → → →
= PQ × PR = ($i − $j + 2 k$ ) × (−2$i − $j + k$ ) 9. Since, a , b, c are unit vectors and a + b + c = 0, then
→ → →
$i $j k$ a , b, c represent an equilateral triangle.
= 1 −1 2 → → → → → → →
∴ a × b = b × c = c × a ≠ 0.
−2 −1 1
→ → → →
= $i (− 1 + 2) − $j (1 + 4) + k$ (− 1 − 2) = $i − 5$j − 3k$ 10. By triangle law, a + b + c = 0
Now, the angle between the faces OPQ and PQR is the A
angle between their normals
→ →
5+5+9  19 c b
= cos − 1 = cos − 1  
25 + 1 + 9 1 + 25 + 9  35
C
7. We have, a = 2i$ + $j − 2k$ B

a
⇒ |a | = 4 + 1 + 4 = 3
→ → →
and b = $i + $j Taking cross product by a , b , c respectively,
⇒ |b| = 1 + 1 = 2 → → → → → → →
a × (a + b + c ) = a × 0 = 0
2
Now, |c − a | = 3 ⇒|c − a | = 9 → → → → → → →
⇒ a × a+ a × b+ a × c=a
⇒ (c − a ) ⋅ (c − a ) = 9 → → → → → → →
⇒ a × b= c × a [Q a × a = 0 ]
⇒ |c |2 + |a |2 − 2 c ⋅ a = 9 …(i)
→ → → →
Again, |(a × b) × c| = 3 Similarly, a × b=b × a
⇒ |a × b|| c |sin 30° = 3 → → → → → →
∴ a × b=b × c= c × a
6
⇒ |c| =
|a × b| 11. Given, |a| = 12 ,| b| = 4 3 R
a + b+ c= 0
$i $j k$
⇒ a = − ( b + c)
But a × b = 2 1 − 2 = 2$i − 2$j + k$
We have, |a|2 = | b + c|2 b a
1 1 0 2 2 2
⇒ |a| = | b| + |c| + 2 b ⋅ c
6 ⇒ 144 = 48 + |c|2 + 48
∴ |c| = =2 …(ii) P c Q
4+4+1 ⇒ |c|2 = 48
From Eqs. (i) and (ii), we get ⇒ |c| = 4 3
(2)2 + (3)2 − 2c ⋅ a = 9 Also, |c|2 = |a|2 + | b|2 + 2 a ⋅ b
⇒ 4 + 9 − 2c ⋅ a = 9 ⇒ 48 = 144 + 48 + 2 a ⋅ b
⇒ c⋅ a = 2 ⇒ a ⋅ b = − 72
→ → → →
8. Plan If a × b = a × c ∴Option (d) is correct.
rankershalt.com

Vectors 535

Also, a × b= c× a → → → →
∴ |u|+ |u ⋅ a|= sin θ + sin 2 θ ≠ |v|
⇒ a × b + c × a = 2a × b → → → → → → →
u ⋅ b = [a − (a ⋅ b) b] ⋅ b
⇒ |a × b + c × a| = 2|a × b| = 2 |a|2| b|2 − (a ⋅ b)2
→ → → → → → → → → → →
= 2 (144) (48) − (− 72)2 = a ⋅ b − (a ⋅ b) (b ⋅ b) = a ⋅ b − a ⋅ b|b|2
→ → → →
= 2 (12) 48 − 36 = 48 3 = a⋅ b − a⋅ b =0 …(i)
∴ Option (c) is correct. → → → → → →
∴ |u|+ |u⋅ b|=|u|+ 0 =|u|=|v|
|c|2
Also, − |a| = 24 − 12 = 12 → → → → → → → → →
2 Also, u ⋅ (a + b) = u ⋅ a + u ⋅ b = u ⋅ a
∴ Option (a) is correct. → → → → → → → →
⇒ |u|+ u ⋅ (a + b) =|u|+ u ⋅ a ≠|v|
|c|2
and + |a| = 24 + 12 = 36 → →
2 14. Since, PQ is not parallel to TR.
T S
∴ Option (b) is not correct. → → →
Q TR is resultant of RS and ST
→ → → → →
12. Let vector AO be parallel to line of intersection of planes vectors. ⇒ PQ × (RS + ST) ≠ 0. U R
P1 and P2 through origin.
But for Statement II, we have
Normal to plane p1 is P Q
→ → →
→ PQ × RS = 0
n1 = [(2$j + 3k$ ) × (4$j − 3k$ )] = −18$i
→ →
Normal to plane p2 is which is not possible as PQ not parallel to RS.
→ Hence, Statement I is true and Statement II is false.
n 2 = ($j − k$ ) × (3 i + 3$j) = 3i$ − 3$j − 3k$
^

→ → →
15. cos(P + Q ) + cos(Q + R) + cos(R + P )
So, OA is parallel to ± (n1 × n 2) = 54 $j − 54k$ .
= − (cos R + cos P + cos Q )
∴ Angle between 54 ($j − k$ ) and (2$i + $j − 2k$ ) is 3
Max. of cos P + cos Q + cos R =
 54 + 108 1 2
cos θ = ±   =± 3
 3 ⋅ 54 ⋅ 2  2 Min. of cos(P + Q ) + cos(Q + R) + cos(R + P ) is = −
π 3π 2
∴ θ= ,
4 4 16. sin R = sin(P + Q )
Hence, (b) and (d) are correct answers. →
17. Let $i be a unit vector in the direction of b, $j in the
→ → → → → →
13. Let θ be the angle between a and b. Since, a and a are direction of c . Note that c = $j
non-collinear vectors, then θ ≠ 0 and θ ≠ π .
→ → → →
→ →
We have, a ⋅ b =|a||a| cos θ
→ →
and (b × c ) =|b||c|sin α k$ = sin α k$
→ → → →
= cos θ [Q|a|= 1,|b|= 1, given] where, k$ is a unit vector perpendicular to b and c.
→ → → → → → → → → → → →
Now, u = a − (a ⋅ b) b ⇒ |u|=|a − (a⋅ b) b| → → b× c
⇒ |b × c|= sin α ⇒ k$ =
→ → → → → → →
⇒ |u|2 =|a − (a ⋅ b) b|2 |b × c|

a = a1i$ + a 2$j + a3 k$
→ → →
⇒ |u|2 =|a − cos θ b|2 Let
→ → →
a ⋅ b = a ⋅ $i = $i ⋅ (a1$i + a 2$j + a3 k$ ) = a1
→ → → → →
⇒ |u|2 =| a|2 + cos 2 θ|b|2 − 2 cos θ (a⋅ b) Now,
→ → →
a ⋅ c = a ⋅ $j = $j ⋅ (a1$i + a 2$j + a3 k$ ) = a 2

⇒ |u|2 = 1 + cos 2 θ − 2 cos 2 θ and
→ → → →
⇒ |u|2 = 1 − cos 2 θ ⇒ |u|2 = sin 2 θ and

a⋅
b× c → $
= a ⋅ k = a3
→ → → → →
Also, v=a × b [given] |b × c|
→ → →
→ 2 → → 2 → 2 → 2 → 2 → → → → → → a⋅ (b × c ) → →
⇒ |v| =|a × b| ⇒ |v| =|a| |b| ⋅ sin 2 θ ∴ (a ⋅ b ) b + (a ⋅ c ) c + (b × c )
→ →2
→ → → |b × c|
⇒ |v|2 = sin 2 θ ∴ |u|2 =|v|2
→ →
→ → → → → → → → → → → → → → → (b × c ) →
Now, u ⋅ a = [a − (a ⋅ b) b] ⋅ a = a ⋅ a − (a ⋅ b) (b ⋅ a ) = a1 b + a 2 c + a3 = a1i$ + a 2$j + a3 k$ = a
→ →
→ |b × c|
= (a )2 − cos 2 θ = 1 − cos 2 θ = sin 2 θ
rankershalt.com

536 Vectors

→ → → → → →
18. A unit vector perpendicular to the plane determined by ⇒ a × (b − c ) = ( c − b) × d
→ →
(PQ) × (PR) → → → → → →
P ,Q, R = ± ⇒ a × (b − c ) − ( c − b) × d = 0
→ →
| PQ× PR| → → → → → →
⇒ a × (b − c ) − d × (b − c ) = 0
→ → → → → → → → → →
(PQ) × (PR) ⇒ (a − d ) × (b − c ) = 0 ⇒ (a − d )||(b − c )
∴ Unit vector = ±
→ → → → → →
| PQ × PR| ∴ (a − d ) ⋅ (b − c ) ≠ 0

PQ = $i + $j − 3k$
→ → → → → → → →
where, ⇒ a⋅ b + d⋅ c ≠ d ⋅ b + a ⋅ c
→ → → → →
and PR = − $i + 3$j − k$ 22. (i) Since, u ⋅ v =|u||v|cos θ
→ → → →
i$ $j k$ and u × v =|u||v|sin θ n$
→ → → →
∴ PQ × PR = 1 1 −3 where, θ is the angle between u and v and n$ is unit
−1 3 −1 → →
vector perpendicular to the plane of u and v.
= $i ( − 1 + 9) − $j(− 1 − 3) + k$ (3 + 1) Again,
→ → →
|u ⋅ v|2 =|u|2|v|2 cos 2 θ and

= 8i + 4j + 4k $ → → → → → →
|u × v|2 =|u|2|v|2 sin 2 θ =|u|2 |v|2 sin 2 θ
→ → → → → → → →
⇒ |PQ × PR|= 4 4 + 1 + 1 = 4 6 ∴ |u ⋅ v|2 + |u × v|2 =|u|2 |v|2 (cos 2 θ + sin 2 θ )
→ →
4(2i$ + $j + k$ )
→ →
PQ × PR =|u|2 |v|2 …(i)
∴ =±
→ → 4 6 → → → → 2
|PQ × PR| (ii) |u + v + ( u × v)|

(2$i + $j + k$ )
→ → → → → → → →
=± =|u + v|2 + |u × v|2 + 2 (u + v) ⋅ (u × v)
6 → → → → → →
=|u|2 + |v|2 + 2 u ⋅ v + |u × v|2 + 0
1 → →
19. Area of ∆ABC = |AB × AC| → → → →
2 [Q u × v is perpendicular to the plane of u and v]
→ → → → → → → →
AB = i$ + 2$j − 3k$ and AC = 2i$ + 2$j − 3k$ ∴ |u + v + (u × v)|2 + |1 − u ⋅ v|2
$i $j k$ → → → → → → → → → →
→ → =|u|2 + |v|2 + 2 u ⋅ v + |u × v|2 + 1 − 2 u ⋅ v + |u ⋅ v|2
∴ AB × AC = 1 2 −3 = 2(−3$j − 2k$ ) → → → →
2 0 0 =|u|2 + |v|2 + 1 + |u|2 |v|2 [from Eq. (i)]
→ 2 → 2 → 2 → 2 →
1 → → =|u| (1 + |v| ) + (1 + |v| ) = (1 + |v| ) (1 + |u|2 )
⇒ Area of triangle = |AB × AC|
2 23. Let the position vectors of points A, B, C, D be a , b, c and
→ → →

1 →
= ⋅ 2 ⋅ 9 + 4 = 13 sq units d, respectively.
2
→ → → → → → → → →
→ → → → Then, AB = b − a , BC = c − b, AD = d − a,
20. Given, A⋅ B = A⋅ C = 0
→ → → → → → → → → → → →
⇒ A is perpendicular to both B and C. BD = d − b, CA = a − c , CD = d − c
→ → → → → → → → →
⇒ A = λ (B × C) Now,|AB × CD + BC × AD + CA × BD|
→ → → → → → → → → → → → → → → → → →
|A|=| λ ||B × C|, where A , B, C are unit vectors. =|(b − a ) × (d − c ) + ( c − b) × (d − a ) + (a − c ) × (d − b)|
1 → → → → → → → → → → → → → →
⇒ |λ |= ⇒ |λ |= 2 ⇒ λ=±2 =|b × d − a × d − b × c + a × c + c × d − c × a − b × d
1 ⋅ sin 30°
→ → → → → → → → → → → → →
∴ A = ± 2 (B × C) + b × a + a × d − a × b − c × d + c × b|
→ → → → → →
Hence, given statement is true. = 2| a × b + b × c + c × a ) …(i)
→ → → → → → → →
21. Given, a × b = c × d and a × c=b ×d Also, area of ∆ ABC
→ → → → → → → →
⇒ a × b−a × c= c ×d −b ×d
rankershalt.com

Vectors 537

1 → → 1 → → → → ⇒ (λ − 2)(λ2 − 9) = 0
= |AB × AC| = |(b − a ) × ( c − a )|
2 2 ⇒ (λ − 2) (λ + 3) (λ − 3) = 0
1 → → → → → → → → ∴ λ = 2, 3 or − 3
= |b × c − b × a − a × c + a × a|
2 If λ = 2 , then
1 → → → → → → $i $j k$
= |a × a + b × c + c × a| …(ii)
2 a × c = 1 2 4 = $i (6 − 16) − $j(3 − 8) + k$ (4 − 4)
From Eqs. (i) and (ii), 2 4 3
→ → → → → → = − 10$i + 5$j
|AB × CD + BC × AD + CA × BD| 2 (2 area of ∆ ABC )
$i $j k$
= 4 (area of ∆ ABC)
If λ = ± 3, then a × c = 1 2 4 = 0
→ → →
24. Since, OA1 , OA2,... , OAn are all vectors of same 2 4 8
magnitude and angle between any two consecutive
vectors is same i.e. (2π / n ). (because last two rows are proportional).
→ → → → → →
→ → 2π 3. Let angle between a and b be θ1 , c and d be θ 2 and a × b
∴ OA1 × OA2 = a 2 ⋅ sin ⋅ $p …(i)
n → →
and b × d be θ.
where, $p is perpendicular to plane of polygon. → → → →
n −1 n −1 Since, (a × b) ⋅ ( c × d ) = 1
→ → 2 2π
Now, ∑ (OA i × OA i + 1 ) = ∑ a ⋅ sin n ⋅ p$ ⇒ sin θ1 ⋅ sinθ 2 ⋅ cosθ = 1
i =1 i =1

2π ⇒ θ1 = 90° , θ 2 = 90° , θ = 0°
= (n − 1) ⋅ a 2 ⋅ sin $
⋅p
n → → → → → → → →
⇒ a ⊥ b, c ⊥ d , (a × b)||( c × d )
→ → → → → → → → → →
= (n − 1) [ OA1 × OA2] So, a × b = k ( c × d ) and a × b = k ( c × d )
→ → → → → → → →
= (1 − n ) [OA2 × OA1 ] = RHS ⇒ (a × b) ⋅ c = k ( c × d ) ⋅ c
→ → → → → →
an d (a × b) ⋅ d = k ( c × d ) ⋅ d
Topic 3 Scalar Triple Product/Dot
→→→ →→→
Product/Mixed Product ⇒ [a b c ] = 0 and [a b d ] = 0
1. Given vectors, µ$i + $j + k$ , $i + µ$j + k$ , $i + $j + µk$ will be → → → → → →

coplanar, if ⇒ a , b, c and a , b, d are coplanar vectors, so


options (a) and (b) are incorrect.
µ 1 1
→ → → →
1 µ 1 =0 Let b||d ⇒ b = ± d
1 1 µ → → → → → → → →
As (a × b) ⋅ ( c × d ) = 1 ⇒ (a × b) ⋅ ( c × b) = ± 1
⇒ µ (µ 2 − 1) − 1 (µ − 1) + 1 (1 − µ ) = 0 → → →→ →→→ →
⇒ (µ − 1) [ µ (µ + 1) − 1 − 1] = 0 ⇒ [ a × b c b] = ± 1 ⇒ [ c b a × b] = ± 1
⇒ (µ − 1) [ µ 2 + µ − 2] = 0 → → → → → → → → →
⇒ c ⋅ [b × (a × b)] = ± 1 ⇒ c ⋅ [a − (b ⋅ a ) b] = ± 1
⇒ (µ − 1) [(µ + 2) (µ − 1)] = 0
→ → → →
⇒ µ = 1 or − 2 ⇒ c⋅a = ± 1 [Q a ⋅ b = 0]
So, sum of the distinct real values of which is a contradiction, so ←
b
µ = 1 − 2 = − 1. option (c) is correct.
2. We know that, if a , b , c are coplanar vectors, then Let option (d) is correct.
→ →
[a b c ] = 0 ⇒ d=±a α ←
a
1 2 4 60º
→ →
and c =± b
∴ 1 λ 4 =0
← ←
2 → → → → d c
2 4 λ −1 As (a × b) ⋅ ( c × d ) = 1
⇒ 1 { λ (λ2 − 1) − 16} − 2((λ2 − 1) − 8) + 4 (4 − 2λ ) = 0 → → → →
⇒ (a × b) ⋅ (b × a ) = ± 1
⇒ λ3 − λ − 16 − 2λ2 + 18 + 16 − 8λ = 0
⇒ λ3 − 2λ2 − 9λ + 18 = 0 which is a contradiction, so option (d) is incorrect.
Alternatively, options (c) and (d) may be observed
⇒ λ2(λ − 2) − 9 (λ − 2) = 0
from the given figure.
rankershalt.com

538 Vectors

4. The volume of the parallelopiped with coterminus edges Applying C3 → C1 + C3 ,


$ , c$ is given by [a$ $b $c] = a$ ⋅ (b
as a$ , b $ × c$ ) 1 0 0
Z x 1 1 =1
y x 1+ x
Therefore, it neither depends on x nor y.
← → → → → → → →
c 9. If a , b, c are coplanar vectors, then 2 a − b, 2 b − c and

b → →
Y 2 c − a are also coplanar vectors.

a → → → → → →
i.e. [2 a − b 2b− c 2 c − a] = 0
X →→ → → → → → →→ →→ →
$ a$ ⋅ c$
10. [u v w] =|v w u|= [w u v] = − [ v u w]
a$ ⋅ a$ a$ ⋅ b 1 1 /2 1 /2
$ $ $ 2 $ $ $ $ $ $ Therefore, (c) is the answer.
Now, [a b c] = b ⋅ a b ⋅ b b ⋅ c = 1 / 2 1 1 / 2
$ c$ ⋅ c$ → → →
c$ ⋅ a$ c$ ⋅ b 1 /2 1 /2 1 11. Since, a , b, c are linearly dependent vectors.
→ → →
 1 1  1 1 1  1 1 1 ⇒ [a b c] = 0
⇒ [a$ $b c$ ]2 = 1 1 −  −  −  +  −  =
 4 2  2 4 2  4 2 2 1 1 1
Thus, the required volume of the parallelopiped ⇒ 4 3 4 =0
1 1 α β
= cu unit
2 Applying C 2 → C 2 − C1 , C3 → C3 − C1,
5. Since, given vectors are coplanar 1 0 0
− λ2 1 1 4 −1 0 = 0 ⇒ − (β − 1) = 0 ⇒ β = 1
∴ 1 −λ 2
1 =0 1 α −1 β −1

1 1 − λ2 Also, |c|= 3 [given]
⇒ λ6 − 3λ2 − 2 = 0 ⇒ (1 + λ2)2 (λ2 − 2) = 0 ⇒ λ = ± 2 ⇒ 2
1 + α + β =3 2
[given, c = i + α$j + βk$ ]
$

6. We know that, volume of parallelopiped whose edges ⇒ 1 + α2 + 1 = 3 ⇒ α2 = 1⇒ α = ± 1


→ → → →→→ → → → → → → →
are a , b , c = [a b c ]. 12. ( a + b + c ) ⋅ [(a + b) × (a + c )]
1 a 1 → → → → → → → → → → →
→→→ = (a + b + c ) ⋅ [a × a + a × c + b × a + b × c ]
∴ [a b c ] = 0 1 a = 1 + a3 − a
→ → → → → → → → → → → →
a 0 1 = { a⋅ (a × c ) + a⋅ (b × a ) + a⋅ (b × c )} + { b⋅ (|a × c )
→ → → → → → → → → → → → → → →
Let f (a ) = a3 − a + 1 + b⋅ (b × a ) + b⋅ (b × c )} + { c⋅ (a × c ) + c⋅ (b × a ) + c⋅ (b × c )}
⇒ f ′ (a ) = 3a 2 − 1 →→→ →→→ →→→ →→→
= [a b c ] + [b a c ] + [ c b a ] = [a b c ]
⇒ f ′ ′ (a ) = 6a

For maximum or minimum, put f ′ (a ) = 0 13. Let d = x$i + y$j + zk$
1 1
⇒a =± , which shows f (a ) is minimum at a = where, x2 + y2 + z 2 = 1 …(i)
3 3 →
1 [Q d being unit vector]
and maximum at a = − . → →
3 Since, a⋅ d = 0
→ →
7. Given, V = 2i$ + $j − k$ and W = i$ + 3k$ ⇒ x− y=0 ⇒ x= y …(ii)
→→ → → →→→
[ U V W ] = U ⋅ [( 2$i + $j − k$ ) × ( $i + 3k$ )] Also, [b c d ] = 0
→ →  0 1 −1 
= U ⋅ (3$i − 7$j − k$ ) =| U||3$i − 7$j − k$ | cos θ  −1 0 1 = 0 ⇒
⇒ x+ y+ z =0
→  
which is maximum, if angle between U and 3i$ − 7$j − k$  x y z 
is 0 and maximum value ⇒ 2x + z = 0 [from Eq. (ii)] …(iii)
=|3i$ − 7$j − k$ | = 59 From Eqs. (i), (ii) and (iii),
1 0 −1 1
x2 + x2 + 4x2 = 1 ⇒ x = ±
→→→ 6
8. [a b c ] = x 1 1 − x
→ 1 $ $
y x 1+ x− y ∴ d=± (i + j − 2k$ )
6
rankershalt.com

Vectors 539

14. Since, three vectors are coplanar. P


a a c
1 0 1 =0 v
1
c c b
0 a c θ
O Q u
Applying C1 → C1 − C 2 , 1 0 1 = 0
0 c b → →
Clearly, there may be infinite vectors OP = v, such that
2 2
⇒ − 1 (ab − c ) = 0 ⇒ ab = c $
P is always 1 unit distance from u.
→ → → → π ∴ Option (b) is correct.
15. Since, (a × b) =|a||b|sin ⋅ n$ →
6
Again, let φ be the angle between ww and u × v.
→ → 1 → →
→ →
$ ⋅c
(a × b) ⋅ c = |a||b|⋅ n → →
2 ∴ w ⋅ (u × v) = 1 ⇒ | w|| u × v|cos φ = 1
→→→ 1 → →
[a b c ] = |a||b| ⋅ cos 0° ⇒ cos φ = 1 ⇒ φ = 0
2 →
→ → → Thus, w=u×v
Q n$ is perpendicular to both a, b and c is also a unit
→ →
vector perpendicular to both a and b. Now, if u$ lies in XY -plane, then
2 i $j k
 a1 a 2 a3  →
→→→ 1 → → u × v = u1 u2 0 or u = ui$ + u2$j
∴  b1 b2 b3  = [a b c ]2 = ⋅|a|2|b|2
  4 v1 v2 v3
 c1 c2 c3  1
= (a12 + a 22 + a32 ) (b12 + b22 + b32 )
4 ∴ w = (u2v3 ) $i − (u1v3 )$j + (u1v2 − v1u2)k
$
1 $ $ $)
 2 −3 0  = (i + j + 2k
→→→ 6
16. The volume of parallelopiped = [a b c ] =  1 1 −1 
  1 −1
 3 0 −1  ∴ u2v3 = , u1v3 =
6 6
= 2 (−1) + 3 (−1 + 3) = − 2 + 6 = 4
u2v3
→ → → → → → ⇒ = − 1 or | u1 | = | u2|
17. Given, |(a × b ) ⋅ c|=|a||b||c| u1v3
→ → → → → →
⇒ ||a||b|sin θ n$ ⋅ c|=|a||b||c| ∴ Option (c) is correct.


→ → →
|a||b||c||sin θ ⋅ cos α | = | a||b||c|
→ → → Now, if u$ lies in XZ-plane, then u = u1$i + u3 k
$

π $i $j k
$
⇒ |sin θ ||cos
⋅ α |= 1 ⇒ θ = and α =0 →
2 ∴ u × v = u1 0 u3
→ → →
∴ $
a ⊥ b and c||n v1 v2 v3
→ → → → →
i.e. a ⊥ b and c perpendicular to both a and b. ⇒ w = (− v2u3 ) i$ − (u1v3 − u3 v1 ) $j + (u1v2) k
$
→ → → → → → O
18. A ⋅ {(B + C) × (A + B + C)} A $ = 1 ($i + $j + 2k
⇒ k $)
6
[Q it is a scalar triple product of three
→ → → → → → 1 2
vectors of the form A, B + C, A + B + C] ⇒ − v2u3 = and u1v2 =
6 6
B C
→ → → → →
= A⋅ (B × A + B × B ∴ | u2| = 2| u3 |
→ → → → → → → →
+ B × C + C × A + C × B + C × C) ∴Option (d) is wrong.
→ → → → → → → → →
→ → →
= A⋅ (B × A) + A⋅ (B × C) + A⋅ (C × B) 20. (a) u ⋅ ( v × w) is a meaningful operation.
→→→ →→→
= [ A B A] − [ A B C] = 0 Therefore, (a) is the answer.
→ → → → →
→ (b) u⋅ ( v ⋅ w) is not meaningful, since v ⋅ w is a scalar
19. Let θ be the angle between u$ and v.
→ → quantity and for dot product both quantities should
∴ | u × v| = 1 ⇒ | u | v|sin θ = 1 be vector.
→ Therefore, (b) is not the answer.
∴ | v|sin θ = 1 [Q| u | = 1] …(i)
rankershalt.com

540 Vectors

→ → →
(c) (u⋅ v) w is meaningful, since it is a simple a a2 1 a a 2 a3
multiplication of vector and scalar quantity. ⇒ b b 1 + b b2 b3 = 0
2

Therefore, (c) is the answer. c c2 1 c c2 c3


→ → → → →
(d) u × ( v ⋅ w) is not meaningful, since v ⋅ w is a scalar 1 a a2
quantity and for cross product, both quantities
⇒ (1 + abc) 1 b b2 = 0
should be vector.
Therefore, (d) is not the answer. 1 c c2
→ → 1 a a2
21. Since, q = area of parallelogram with OA and OC as
→ → ⇒ Either (1 + abc) = 0 or 1 b b2 = 0
→ →
adjacent sides =|OA × OC|=| a × b| 1 c c2
and p = area of quadrilateral OABC
But (1, a , a 2), (1, b, b2), (1, c, c2) are non-coplanar.
1 → → 1 → →
= |OA × OB|+ |OB × OC| 1 a a2
2 2
⇒ 1 b b2 ≠ 0
1 → → → 1 → → →
= |a × (10 a + 2 b) + |(10 a + 2 b) × b| 1 c c2
2 2
→ → → → → → ∴ abc = − 1
=|a × b|+ 5|a × b|= 6|a × b|
→ → → → → → → → → → → → → →
∴ p = 6q ⇒ k =6 25. (a − b) ⋅ {(b − c ) × ( c − a )} = (a − b) ⋅ (b × c − b × a + c × a )
→ → → → → → →→→ →→→
22. Since, vectors are coplanar. = a⋅ (b × c ) − b⋅ ( c × a ) = [a b c ] + [a b c ]
a 1 1 →→→ → → →
∴ 1 b 1 =0 = 2 [a b c ] = 2 a⋅ (b × c )
1 1 c Hence, it is a true statement.
→ → → → → →
Applying R2 → R2 − R1, R3 → R3 − R1, 26. Since, X⋅ A = X⋅ B = X⋅ C = 0
a 1 1 → → → → →→→
⇒ X is perpendicular to A , B, C, therefore [A B C] = 0
1 − a b −1 0 =0
1−a 0 c−1 Hence, given statement is true.

a / (1 − a ) 1 / (1 − b) 1 / (1 − c)
→ → 
→ u+ v1 α → →
27. x= = sec (u + v) 
⇒ 1 −1 0 =0 → → 2 2 
| u + v| 
1 0 −1 → → 
→ w+ v 1 β → →
a 1 1 y= = sec ( v + w) …(i)
⇒ (1) − (−1) + (1) = 0 → → 2 2 
1−a 1−b 1−c | v + w|
→ →

a 1 1 → w+ u 1 γ → → 
⇒ + + =0 z= = sec (w + u ) 
1−a 1−b 1−c → → 2 2 
| w + u|
1 1 1
⇒ −1 + + + =0 → → → → → → →→→
1 − a 1 − b 1 −c Since, [x × y y × z z × x ] = [x y z ]2 [from Eq. (i)]
1 1 1 1 α β γ → →→ → → →
⇒ + + =1 = sec2 ⋅ sec2 ⋅ sec2 [u + v v + w w + u ]2 …(ii)
1−a 1−b 1−c 64 2 2 2
→ → → → → → →→ →
→ → → → → → and [u + v v + w w + u ] = 2 [u v w] …(iii)
A⋅ (B × C) B⋅ (A × C)
23. → → → + → → → → → → → → →
(C × A) ⋅ B C⋅ (A × B) ∴[x × y y × z z × x ]
→→→ →→→ →→→ →→→ 1 α β γ →→ →
[A B C] [B A C] [A B C] − [A B C] = sec2 ⋅ sec2 ⋅ sec2 ⋅ 4 [u v w]2
= →→→
+ →→→
= →→→
=0 64 2 2 2
[C A B] [C A B] [C A B] 1 →→ → 2 2α β γ
= ⋅ [u v w] sec ⋅ sec2 ⋅ sec2
a a 2 1 + a3 16 2 2 2
24. Since, b b2 1 + b3 = 0 28.
→ → →
V = |a ⋅ (b × c )|≤ a12 + a 22 + a32
c c2 1 + c3
b12 + b22 + b32 c12 + c22 + c32 …(i)
rankershalt.com

Vectors 541

(a1 + a 2 + a3 ) + (b1 + b2 + b3 ) + (c1 + c2 + c3 ) → → → 1 → → 1 → →


Now, L = ⇒ (u × v) ⋅ w = 1 − (u ⋅ v)2 ≤ [Q (u ⋅ v)2 ≥ 0]
3 2 2
≥ [(a1 + a 2 + a3 ) (b1 + b2 + b3 ) (c1 + c2 + c3 )]1/3
→ → →
[using AM ≥ GM] The equality holds if and only if u ⋅ v = 0 or iff u is

⇒ L3 ≥ [(a1 + a 2 + a3 ) (b1 + b2 + b3 ) (c1 + c2 + c3 )] …(ii) perpendicular to v.
Now, (a1 + a 2 + a3 )2 →
30. Here, AB = − i$ + 5$j − 3k$
= a12 + a 22 + a32 + 2a1a 2 + 2a1a3 + 2a 2a3 ≥ a12 + a 22 + a32
→ →
AC = − 4$i + 3$j + 3k$ and AD = i$ + 7$j + (λ + 1) k$
⇒ (a1 + a 2 + a3 ) ≥ a12 + a 22 + a32
→ → →
We know that, A , B, C , D lie in a plane if AB, AC, AD
Similarly, (b1 + b2 + b3 ) ≥ b12 + b22 + b32 are coplanar i.e.
and (c1 + c2 + c3 ) ≥ c12 + c22 + c32 −1 5 −3
→ → →
[AB AC AD] = 0 ⇒ −4 3 3 =0
∴ L3 ≥ [(a 12 + a 22 + a32 ) (b12 + b22 + b32 ) (c12 + c22 + c32 )]1/ 2
1 7 λ+1
⇒ L3 ≥ V [from Eq. (i)]
→ → → → ⇒ −1 (3λ + 3 − 21) − 5 (−4λ − 4 − 3) − 3 (−28 − 3) = 0
29. Given equation is w + (w × u ) = v …(i)
⇒ −17λ + 146 = 0

Taking cross product with u, we get 146
∴ λ=
→ → → → → → 17
u × [w + (w × u )] = u × v


→ → → →
u × w + u × (w × u ) = u × v
→ → →
Topic 4 Vector Triple Product
→ → → → → → → → → → 1. We have, (a × c ) + b = 0
⇒ u × w + (u ⋅ u ) w − (u ⋅ w) u = u × v
→ → → → → → → → ⇒ a × (a × c ) + a × b = 0
⇒ u × w + w − (u ⋅ w) u = u × v …(ii) (taking cross product with a on both sides)
→ $i $j k$
Now, taking dot product of Eq. (i) with u, we get
→ → → → → → → ⇒ (a ⋅ c )a − (a ⋅ a ) c + 1 −1 0 = 0
u ⋅ w + u⋅ (w × u ) = u ⋅ v
→ → → → → → →
1 1 1
⇒ u⋅ w = u ⋅ v [Q u ⋅ (w × u ) = 0] …(iii)
[Q a × (b × c ) = (a . c )b − (a ⋅ b )c]

Now, taking dot product of Eq. (i) with u, we get $ $ $ $
⇒4( i − j) − 2c + (− i − j + 2k$ ) = 0
→ → → → → → →
v ⋅ w + v⋅ (w × u ) = v ⋅ v [Q a ⋅ a = ($i − $j)($i − $j) = 1 + 1 = 2 and a ⋅ c = 4]


→ → →→ → → →
v ⋅ w + [ v w u ] = 1 ⇒ v⋅ w + [ v w u ] − 1 = 0
→→ →
⇒ 2c = 4i − 4$j − $i − $j + 2k$
→ → → → → 3$i − 5$j + 2k$ 9 + 25 + 4 19
⇒ − (u × v) ⋅ w − v ⋅ w + 1 = 0 ⇒ c= ⇒ |c|2 = =
2 4 2
→ → → → →
⇒ 1 − v ⋅ w = (u × v) ⋅ w …(iv) 1 1
2. Given, a × (b × c) = b ⇒ (a ⋅ c)b − (a ⋅ b)c = b
→ 2 2
Taking dot product of Eq. (ii) with w, we get
[Q a × (b × c) = (a ⋅ c)b − (a ⋅ b)c]
→ → → → → → → → → → → →
(u × w) ⋅ w + w ⋅ w − (u ⋅ w) (u ⋅ w) = (u × v) ⋅ w …(v) On comparing both sides, we get
1

→ 2 → → 2
0 + |w| − (u ⋅ w) = (u × v) ⋅ w
→ → → a⋅c = …(i)
2
→ → → → → →
⇒ (u × v) ⋅ w = |w|2 − (u ⋅ w)2 and a⋅b = 0 …(ii)

Q a , b and c are unit vectors, and angle between a and b
Taking dot product of Eq. (i) with w, we get is α and angle between a and c is β, so
→ → → → → → → 1
w ⋅ w + (w × u ) ⋅ w = v ⋅ w |a||c|cos β = [from Eq. (i)]
2
→ → →
⇒ |w|2 + 0 = v ⋅ w …(vi) ⇒ cos β =
1
[Q|a|= 1 =|c|]
→ 2 → → → 2
⇒ |w| = 1 − (u × v) ⋅ w [from Eq. (iv)]
π π 1
⇒ β= …(iii) Q cos = 
Again, from Eq. (v), we get 3  3 2 
→ → → → → → → → → → →
(u × v) ⋅ w + |w|2 − (u ⋅ w)2 = 1 − (u × v) ⋅ w − (u ⋅ w)2 and |a||b|cos α = 0 [from Eq. (ii)]
π
→ → → → → ⇒ α= …(iv)
⇒ 2 (u × v) ⋅ w = 1 − (u ⋅ v)2 [from Eq. (iii)] 2
rankershalt.com

542 Vectors

From Eqs. (iii) and (iv), we get 7. If θ is the angle between P1 and P2 , then normal to the
π π π planes are
|α − β| = − = = 30º → →
2 3 6 N1 = a × b,

3. Given,|a$ | = |b$ | = |c$ | = 1 N2
→ →
= c × d 
and $ × c$) = 3 ( b
a$ × ( b $ + c$)
2 ∴ N1 × N 2 = 0

Now, consider a$ × ( b$ × c$ ) = 3 ( b
$ + c$ ) Then,|N 1| × |N 2|sin θ = 0
2 ⇒ sin θ = 0 ⇒ θ = 0
$ − (a$ ⋅ b
⇒ (a$ ⋅ c$ ) b $ ) c$ = 3 b $ + 3 c$ →
2 2
8. NOTE In this question, vector c is not given, therefore, we
→ → →
cannot apply the formulae of a × b × c (vector triple
On comparing, we get
product).
$ = − 3 ⇒ |a$ ||b
a$ ⋅ b $ |cos θ = − 3 → → → → → →
2 2 Now,|(a × b) × c|=|a × b||c|sin 30° …(i)
3
⇒ cos θ = − $ |= 1 ]
[Q |a$ | = |b i$ $j k$
2 → →
 π 5π Again, |a × b|= 2 1 −2 = 2i$ − 2$j + k$
⇒ cos θ = cos  π −  ⇒ θ = 1 1 0
 6 6
→ → → 1 → → → → →
4. Given, (a × b) × c = b c  a ⇒ |a × b|= 22 + (−2)2 + 1 = 4 + 4 + 1 = 9 = 3
3
→ → → 1 → → → → →
⇒ − c × (a × b) = b c  a Since, |c − a|= 2 2 [given]
3
→ →

→ → → → → → 1 → → →
− ( c⋅ b) ⋅ a + ( c⋅ a ) b = bc a ⇒ |c − a|2 = 8
3 → → → →
⇒ (c − a ) ⋅ (c − a ) = 8
1 → → → →  → → →
⇒  3 bc+ ( c⋅ b) a = ( c⋅ a ) b → → → → → → → →
⇒ c⋅ c − c⋅ a − a⋅ c + a⋅ a =8
Since, a and b are not collinear. → → → →
⇒ |c|2 + |a|2 − 2 a ⋅ c = 8
→ → 1 → → → →
∴ c⋅ b + bc= 0 and c⋅ a = 0 → →
3 ⇒ |c|2 + 9 − 2|c|= 8
→ → 1 → → → →  1 → →
⇒ bc cosθ + bc= 0 ⇒ bc  cos θ +  = 0 ⇒ |c|2 − 2|c|+ 1 = 0
3  3
→ →
1 ⇒ (|c|− 1)2 = 0 ⇒|c|= 1
⇒ cos θ + = 0 [Q |b|≠ 0,|c|≠ 0]
3 → → →  1 3
1 8 2 2 From Eq. (i), |(a × b) × c| = (3) (1) ⋅   =
⇒ cos θ = − ⇒ sin θ = =  2 2
3 3 3
→ →
→ → → → b+ c→
5. As we know that, a vector coplanar to a , b and 9. Since, a × (b × c ) =
2
→ → → →
orthogonal to c is λ {( a × b) × c} . → → → → → → 1 → 1 →
⇒ (a⋅ c ) b − (a⋅ b) c = b+ c
∴ A vector coplanar to (2i$ + $j + k$ ), (i$ − $j + k$ ) and 2 2
orthogonal to 3$i + 2$j + 6k$ On equating the coefficient of c, we get

= λ [{(2i$ + $j + k$ ) × ($i − $j + k$ )} × (3i$ + 2$j + 6k$ )] → →


a⋅ b = −
1
= λ [(2i$ − $j − 3k$ ) × (3i$ + 2$j + 6k$ )] 2
→ → 1
= λ (21$j − 7k$ ) ⇒ |a||b|cos θ = −
2
(21$j − 7k$ ) (3$j − k$ ) 1 3π
∴ Unit vector = + =+ ∴ cos θ = − ⇒ θ=
(21)2 + (7)2 10 2 4
→ → → → → → → → → → →
6. We know that, a × (a × b) = (a ⋅ b) a − (a⋅ a ) b 10. Let a = $i + $j + 2k$ , b = $i + 2$j + k$
→ →
∴ ($i + $j + k$ ) × ($j − k$ ) = (i$ + $j + k$ ) − ( 3 )2 b and c = i$ + $j + k$
→ → → → →
⇒ − 2$i + $j + k$ = $i + $j + k$ − 3 b ⇒ 3 b = + 3$i ∴ A vector coplanar to a and b, and perpendicular to c
→ → → → → → → → →
→ = λ (a × b) × c = λ {(a ⋅ c ) v − (b ⋅ c ) a }
∴ b = $i
rankershalt.com

Vectors 543

= λ {(1 + 1 + 4) ($i + 2$j + k$ ) − (1 + 2 + 1) ($i + $j + 2k$ )} ⇒



( 2 cos θ a − c )2 = (− b)2
→ →

= λ {6 $i + 12 $j + 6 k$ − 6$i − 6$j − 12k$ } → → → → →


= λ {6 $j − 6 k$ } = 6λ { $j − k$ } ⇒ 4 cos 2 θ ⋅|a|2 + |c|2 − 2 ⋅ 2 cos θ a⋅ c =|b|2
1 ⇒ 4 cos 2 θ − 4 − 8 cos 2 θ = 1
For λ = ⇒ Option (a) is correct.
6 ⇒ 4 cos 2 θ = 3
1 3
and for λ = − ⇒ Option (d) is correct. ⇒ cos θ = ±
6 2
11. (0.5) Here, P(1, 0, 0), Q (0, 1, 0), R (0, 0, 1), T = (1, 1, 1) and 3 π
For θ to be acute, cos θ = ⇒θ=
 1 1 1 2 6
S= , , .
 2 2 2 → → → →
13. Now, (A + B) × (A + C)
Y
→ → → → → → → →
= A × A + B × A + A × C+ B × C
(0, 1, 0) Q
→ → → → → → → →
= B × A + A × C+ B × C [Q A × A = 0]
→ → → → → →
T (1, 1, 1)
∴ [(A + B) × (A + C)] × (B × C)
→ → → → → → → →
1 1 1
S 2 , 2 , 2 
 = [B × A + A × C + B × C] × (B × C)
X → → → → → → → →
O P = (B × A ) × (B × C) + (A × C) × (B × C)
(1, 0, 0)
→ → → → → → → →
= {(B × A ) ⋅ C} B − {(B × A ) ⋅ B } C
R (0, 0, 1) → → → → → → → →
Z + {(A × C) ⋅ C} B − {(A × C) ⋅ B} C
→ → → → → → → → → → → → →
[Q (a × b) × c = (a ⋅ c ) b − (b ⋅ c ) a ]
Now, p = SP = OP − OS
→→→ → →→→ →
1 1 1  1 = [B A C] B − [A C B] C
=  i$ − $j − k$ = ( i$ − $j − k$ )
2 2 2  2 →→→
[Q [a b c ] = 0, if any two of a , b, c are equal]
→ → →
→ → 1
q = SQ = ( − i$ + $j − k$ ) →→→ → →
2 = [A C B] (B − C)
→ → 1
r = SR = ( − i$ − $j + k$ ) → → → →
Now, [(A + B) × (A + C)] × (B × C) ⋅ (B + C)
→ → → →
2
→ → 1 →→→ → → → →
and t = ST = ( i$ + $j + k$ ) = ([A C B]{ B − C}) ⋅ (B + C)
2
→→→ → → → →
i$ $j k$ = [A C B]{(B − C) ⋅ (B + C)}
→ → 1 1
p× q = 1 −1 −1 = ( 2i$ + 2 $j ) →→→ → → → → →
4 4 = [A C B]{|B|2 −|C|2 } = 0 [Q |B|=|C|, given]
−1 1 −1
→ → → →
14. Considering first part (a × b) × ( c × d )
i$ $j k$
→ → 1 1 → → →
and r × t = −1 −1 1 = ( −2i$ + 2 $j ) Let c ×d=e
4 4 → → → → → → → → →
1 1 1 (a × b) × e = (a ⋅ e ) b − (b ⋅ e ) a
→ → → → → → → → →
i$ $j k$ [Q (a × b) × c = (a ⋅ c ) b − (b ⋅ c ) a ]
→ → → → 1 → → → → → → → →
Now, ( p × q ) × ( r × t ) = 2 2 0 = { a ⋅ ( c × d )} b− { b ⋅ ( c × d )} d
16
−2 2 0 →→→ → →→→ →
= [a c d ] b − [b c d ] a …(i)
1 1
= ( 8k$ ) = k$ Similarly,
16 2
→ → → → → → →→→ → →→→ →
1 $ 1 (a × c ) × (d × b) = [a d b] c − [ c d b] a
∴|( p × q) × ( r × t )| = k = = 0.5
2 2 →→→ → → →→ →
→ → → → →
= [a d b] c − [b c d ] a …(ii)
12. Given, a × (a × c ) + b = 0 → → → → → → → →
→ → → → → → → →
Also, (a × d ) × (b × c ) = − (b × c ) × (a × d )
⇒ (a⋅ c ) a − (a⋅ a ) c + b = 0 → → → → →→→ → →→→ →
= (b × c ) × (d × a ) = [b d a ] c − [ c d a ] b
→ → → →
⇒ ( 2 cos θ ) a − c + b = 0 →→→ → →→→ →
= [a d b] c − [a c d ] b …(iii)
rankershalt.com

544 Vectors

From Eqs. (i), (ii) and (iii), → → → → → →


Now, (2 a + b) ⋅ [(a × b) × (a − 2 b)]
→ → → → → → → → → → → → → → → → → → → → →
( a × b ) × ( c × d ) + (a × c ) × (d × b) + (a × d ) × (b × c ) = (2 a + b) ⋅ [a 2 b − (a ⋅ b) ⋅ a + 2 b2 ⋅ a −2 (b⋅ a ) ⋅ b]
→→→ → →→→ → →→→ → →→→ → →→→ →
= [a c d ] b − [b c d ] a + [a d b] c − [b c d ] a − [a d b] c → → → → → →
= [2 a + b] ⋅ [b + 2 a ] = 4 a 2+ b2
→→→ → →→→ →
− [a c d ] b = − 2 [b c d ] a
= 4 ⋅1 + 1 = 5 [as a ⋅ b = 0]

∴ Parallel to a.
→ →
15. (i) Given, A ⊥ B ⇒ A⋅ B = 0
→ →
…(i) Topic 5 Solving Equations and
→ → → → → → → Reciprocal of Vectors
and A × X = B ⇒ A⋅ B = 0 and X⋅ B = 0 …(ii)
→→→ → → → → →
1. Two vectors c and d are said to be collinear if we can
Now, [X A A × B] = X⋅ { A × (A × B)} write c = λb for some non-zero scalar λ.
→ → → → → → → Let the vectors α = (λ − 2) a + b
= X⋅ {(A⋅ B) A − (A⋅ A ) B}
and β = (4λ − 2)a + 3 b are
→ → → → → → → →
= ( A⋅ B)( X⋅ A ) − ( A⋅ A )( X⋅ B) = 0 collinear, where a and b are non-collinear.
→ → → → ∴ We can write
⇒ X , A , A × B are coplanar.
α = kβ, for some k ∈ R − {0}
→ →
So, X can be represented as a linear combination of A ⇒ (λ − 2)a + b = k[(4λ − 2)a + 3 b]
→ → → → → →
and A × B. Let us consider, X = l A + m (A × B) ⇒ [(λ − 2) − k (4λ − 2)] a+ (1 − 3k)b = 0
→ →
Now, as a and b are non-collinear, therefore they are
Since, A⋅X =c linearly independent and hence (λ − 2) − k (4λ − 2) = 0
→ → → →
and 1 − 3k= 0
∴ A⋅ { l A + m (A × B)} = c ⇒ λ − 2 = k(4λ − 2) and 3k = 1
→ 1 Q 3k = 1 ⇒ k = 1 
⇒ l|A|2 + 0 = c ⇒ λ − 2 = (4λ − 2)

3 3 
c
⇒ l= ⇒ 3λ − 6 = 4λ − 2
→ 2
|A |
⇒ λ = −4
→ → → → → → → →
Also, A × X = B ⇒ A × { l A + m (A × B)} = B 2. We have, a = 2i$ + λ 1$j + 3k$ ; b = 4i$ + (3 − λ 2)$j + 6k$
c = 3i$ + 6$j + (λ 3 − 1)k
$,
→ → → → → →
⇒ l (A × A ) + m { A × (A × B)} = B and
→ → → such that b = 2a
⇒ 0 − m|A|2 B = B
Now, b = 2a
1 ⇒ 4i$ + (3 − λ 2)$j + 6k$ = 2 (2i$ + λ $j + 3k
$)
⇒ m=− 1
→ 2
|A| $ $ $ $ $
⇒ 4i + (3 − λ 2) j + 6k = 4i + 2λ 1 j + 6k $
    ⇒ (3 − 2λ − λ )$j = 0
1 2
→ c  →  1  → →
∴ X =  A −  → 2 (A × B) ⇒ 3 − 2λ 1 − λ 2 = 0
→ 2
|A|  |A|  ⇒ 2λ 1 + λ 2 = 3 … (i)
→ Also, as a is perpendicular to c, therefore a . c = 0
(ii) Since, vector A has components A1 , A2, A3 in the
⇒ (2$i + λ 1$j + 3k$ ) ⋅ (3i$ + 6$j + (λ − 1)k
3
$)=0
coordinate system OXYZ.
→ ⇒ 6 + 6λ 1 + 3(λ 3 − 1) = 0
∴ A = A $i + A $j + A k$
1 2 3 ⇒ 6λ 1 + 3λ 3 + 3 = 0
When the given system is rotated about an angle of ⇒ 2λ 1 + λ 3 = − 1 … (ii)
π / 2 , the new X-axis is along old Y-axis and new
Y-axis is along the old negative X-axis , whereas Now, from Eq. (i), λ 2 = 3 − 2λ 1 and from Eq. (ii)
z remains same. λ 3 = − 2λ 1 − 1
Hence, the components of A in the new system are ∴ (λ 1 , λ 2, λ 3 ) ≡ (λ 1 , 3 − 2λ 1 , − 2λ 1 − 1)
( A2, − A1 , A3 ). 1
If λ 1 = − , then
→ 2
∴ A becomes ( A i$ − A $j + A k$ ) .
2 1 3
λ 2 = 4, and λ 3 = 0
16. From the given information, it is clear that  1 
$i − 2$j Thus, a possible value of (λ 1 , λ 2, λ 3 ) =  − , 4, 0
→ → → → →  2 
a= ⇒ |a|= 1, |b|= 1, a ⋅ b = 0
5
rankershalt.com

Vectors 545

→ → → → →
3. It is given that c is coplanar with a and b, we take Similarly, taking dot of Eq. (ii) with q and r
→ → → respectively, we get
c = pa+ qb …(i)
→ → |λ |2 → →
where, p and q are scalars. x⋅ q = = x⋅ r
2
→ → → →
Since, c ⊥ a ⇒ c⋅ a =0 ∴ Eq. (ii) becomes
→ 2
|λ | → → →
Taking dot product of a in Eq. (i), we get 3→x |λ |2 − (→
p+→q +→ r )|λ |2 = (p+ q + r)
→ → → → → → → → → 2
c ⋅ a = p a ⋅ a + q b ⋅ a ⇒ 0 = p|a|2 + q|b ⋅ a| 1 → → → → → →
⇒ 3→x = (p+ q + r ) + (p + q + r )
 → $ $ $  2
Q →a = 2i + j + k  → 1 → → →
 |a|= 22 + 1 + 1 = 6.  ⇒ x = (p + q + r )
 →→  2
$ $ $ $ $ $
 a⋅ b = (2i + j + k ) ⋅ (i + 2 j − k )
 =2 + 2 −1 =3  5. Equation of the plane containing $i and $i + $j is
→ $
⇒ 0 = p ⋅ 6 + q ⋅ 3 ⇒ q = − 2p [( r − i) i$ (i$ + $j) ] = 0
On putting in Eq. (i), we get →
→ → →
⇒ ( r − $i ) ⋅ [ $i × ($i + $j) ] = 0
c = p a + b (−2 p)
⇒ {(x $i + y $j + zk)
$ − $i } ⋅ [$i × $i + $i × $j] = 0
→ → → → → →
⇒ c = p a − 2 p b ⇒ c = p (a − 2 b) ⇒ {(x − 1) $i + y$j + zk$ } ⋅ [k$ ] = 0
(x − 1) $i ⋅ k$ + y$j ⋅ k$ + zk$ ⋅ k$ = 0

⇒ c = p[(2i$ + $j + k$ ) − 2 (i$ + 2$j − k$ )] ⇒
→ → ⇒ z =0 …(i)
⇒ c = p (−3$j + 3k$ ) ⇒ |c|= p (−3)2 + 32
$ $ $ $
Equation of the plane containing i − j and i + k is
→ →
⇒ |c|2 = p2 ( 18 )2 ⇒ |c|2 = p2 ⋅ 18 →
[( r − ($i − $j) ) ($i − $j) ($i + k)
$ ]=0

⇒ 1 = p2 ⋅ 18 [Q|c|= 1] →
⇒ ( r − $i + $j) ⋅ [($i − $j) × (i$ + k$ )] = 0
1 1
⇒ p2 = ⇒ p=±
18 3 2 ⇒ {(x$i + y$j + zk$ ) − ($i − $j)} ⋅ [i$ × i$ + i$ × k$ − $j × i$ − $j × k$ ] = 0

→ 1 (− $j + k$ ) ⇒ {(x − 1) $i + ( y + 1) $j + zk$ )} ⋅ [ − $j + k$ − $i ] = 0
∴ c=±
2 ⇒ − (x − 1) − ( y + 1) + z = 0 …(ii)

Let a = a1$i + a 2$j + a3 k$
→ → →
4. Since, p, q , r are mutually perpendicular vectors of
same magnitude, so let us consider →
Since, a is parallel to Eqs. (i) and (ii), we obtain
→ →  →
|p|=|q|=|r|= λ and  …(i) a3 = 0
→ → → → → → 
p⋅ q = q⋅ r = r⋅ p = 0  and a1 + a 2 − a3 = 0 ⇒ a1 = − a 2, a3 = 0

Thus, a vector in the direction a is $i − $j .
→ → → → → → → →
Given, p × {(x − q ) × p} + q × {(x − r ) × q }
→ → → → → →
+ r × {(x − p) × r } = 0 If θ is the angle between a and $i − 2$j + 2k$ , then
→ → → → → → → → → → → →
⇒ (p⋅ p) (x − q ) − { p⋅ (x − q ) } p + (q⋅ q ) (x − r ) (1) (1) + (− 1) (− 2) 3
cos θ = ± =±
→ → → → → → → → → → → → 1+1 1+4+4 2 ⋅3
− {q⋅ (x − r )} q + ( r⋅ r ) (x − p) − { r⋅ (x − p)} r = 0
1 π 3π
→ → → → → → → → → → ⇒ cos θ = ± ⇒ θ= or
⇒ x{(p ⋅ p) + (q⋅ q ) + ( r⋅ r )} − (p ⋅ p) q 2 4 4
→ → → → → → → → → → →
− (q⋅ q )r − ( r⋅ r ) p = (x⋅ p) p + (x⋅ q ) q + (x⋅ r ) r
→ → → 6. Any vector coplanar with $i + $j + 2k$ and $i + 2$j + k$ is
given by
⇒ 3→
x |λ |2 − (→
p+→
q +→
r )|λ |2 = (→
x⋅→
p)→
p →
a = x ($i + $j + 2k$ ) + y (i$ + 2$j + k$ )

+ (→
x⋅→
q )→
q + (→
x⋅→
r )→
r …(ii) = (x + y) $i + (x + 2 y)$j + (2x + y) k$
→ This vector is perpendicular to $i + $j + k$ , if
Taking dot of Eq. (ii) with p, we get
(x + y) 1 + (x + 2 y)1 + (2x + y) 1 = 0
→ → → → → → 1
3 (x⋅ p)|λ |2 −| λ |4 = (x⋅ p)|λ |2 ⇒ x⋅ p = |λ |2 ⇒ 4x + 4 y = 0 ⇒ −x = y
2
rankershalt.com

546 Vectors

→ → On squaring both sides, we get


∴ a = − x$j + x k$ = x (− $j + k$ ) ⇒ |a| = 2|x|
$ = λ2 ⇒ 2 − 2 cos 2θ = λ2
$ ⋅v
⇒1+1−w
1
Hence, the required unit vector is a$ = ± ⋅ (− $j + k$ ) ⇒ λ = 2 sin θ
2
→ where, 2 θ is the angle between v $ and w.
$
7. Let c = a ^i + b ^j Hence, w − v = 2 sin θ ⋅ a = 2 cos (90° − θ ) a$ = − (2 a$ ⋅ v
$ $ $ $ ) a$
→ →
Since, b and c are perpendiculars to each other. Then, ⇒ w$ =v$ − 2 (a$ ⋅ v
$ ) a$
→ → →

b ⋅c =0


^ ^ ^
(4 i + 3 j) ⋅ (a i + b j) = 0
^ 10. Let the position vectors of A , B, C be a, b and c
→ → →
respectively and that of P , Q , R be p, q and r,
⇒ 4a + 3b = 0 ⇒ a : b = 3 : − 4 →

respectively. Let h be the position vector of the
^ ^
∴ c = λ (3 i − 4 j), where λ is constant of ratio. orthocentre H of the ∆ PQR. We have, HP ⊥ QR.
→ Equation of straight line passing through A and
Let the required vectors be a = p $i + q$j → → →
perpendicular to QR i.e. parallel to HP = P − h is
→ →
→ → a⋅ b → → → →
Projection of a on b is r = a + t1 (p − h ) …(i)

| b|
where, t1is a parameter.
4 p + 3q
∴ 1= ⇒ 4 p + 3q = 5 ... (i) Similarly, equation of straight line through B and
5 → → → →
→ → perpendicular to RP is r = b + t2 (q − h ) …(ii)
→ → a⋅ c
Also, projection of a on c is .
→ Again, equation of straight line through C and
|c| → → → →
3 λp − 4 λq perpendicular to PQ is r = c + t3 ( r − h ) …(iii)
⇒ 2= ⇒ 3 p − 4q = 10
5λ If the lines (i), (ii) and (iii) are concurrent, then there

On solving above equations, we get p = 2, q = − 1 exists a point D with position vector d which lies on
all of them, that is for some values of t1 , t 2 and t3 ,
c = 2 $i − $j



which implies that
8. Let B = x$i + y$j + zk$ 1→ 1→ → →
d = a + p−h …(iv)
Given,
→ $ →
A = i$ + $j + k, C = $j − k$ t1 t1
→ → → 1→ 1→ → →
Also, given A × B = C d = b+q −h …(v)
t2 t2
^ ^ ^ ^ ^
⇒ (z − y) i − (z − x) j + ( y − x) k = j − k 1 → 1 → → →
d= c+ r −h …(vi)
⇒ z − y = 0, x − z = 1, y − x = − 1 t3 t3
→ →
Also, A⋅ B = 3 ⇒ x + y + z = 3 From Eqs. (iv) and (v),
On solving above equations, we get 1 1 → 1 → 1 → → →
 −  d = a − b + p −q …(vii)
5 2  t1 t 2  t1 t2
x= ,y=z=
3 3 and from Eqs. (v) and (vi),
→ 5 2 2 $
∴ B =  $i , $j, k 1 1 → 1 → 1 → → →
 − d= b− c+q −r …(viii)
3 3 3 
 t2 t3  t2 t3
9. Since, v$ is unit vector along the incident ray and w
$ is the →
unit vector along the reflected ray. Eliminating d from Eqs. (vii) and (viii), we get
Hence, a$ is a unit vector along the external bisector of 1 1   1 → 1 → → →
 −   a − b + p −q
$ and w.
v $  t2 t3   t1 t2 
$ −v
∴ w $ = λa$  1 1   1 → 1 → → →
= −   b− c + q − r
 t1 t2  t 2 t3 
→ → → →
^
90° − θ a ⇒ (t3 − t2) [t 2 a − t1 b + t1t 2 (p − q )]
^
v ^
w → → → →
= (t2 − t1 ) [t3 b − t2 c + t2t3 (q − r )]
θ
θ [multiplying both sides by t1 t2 t3 ]
Mirror
→ →
⇒ t2 (t3 − t2) a + t2 (t1 − t3 ) b
→ →
+ t2 (t2 − t1 ) c + t1 t2 (t3 − t2) p
→ → →
+ t22 (t1 − t3 ) q + t2 t3 (t2 − t1 ) r = 0
rankershalt.com

Vectors 547

Thus, lines (i), (ii), and (iii) are concurrent is equivalent 3λ2
Therefore, 12 =
to say that there exist scalars t1 , t2 and t3 such that (λ + 1)2
→ → → →
(t 2 − t3 ) a + (t3 − t1 ) b + (t1 − t2) c + t1 (t2 − t3 ) p ⇒ 4 (λ + 1)2 = λ2 ⇒ 4λ2 + 4 + 8λ = λ2
→ → → ⇒ 3λ2 + 8λ + 4 = 0 ⇒ 3λ2 + 6λ + 2λ + 4 = 0
+ t2 (t3 − t1 ) q + t3 (t1 − t2) r = 0
⇒ 3λ (λ + 2) + 2 (λ + 2) = 0
On dividing by t1 t2 t3 , we get
→ → →
⇒ (3λ + 2) (λ + 2) = 0 ⇒ λ = − 2 / 3, λ = − 2
(λ 2 − λ 3 ) p + (λ 3 − λ 1 ) q + (λ 1 − λ 2) r When λ = − 2 / 3, position vector of E is given by
→ → →
+ λ 1 (λ 2 − λ 3 ) a + λ 2 (λ 3 − λ 1 ) b + λ 3 (λ 1 − λ 2) c = 0  2λ + 1 $ 1 $ 1 $
 i+ j+ k
1  λ +1 λ+1 λ+1
where, λ i = for i = 1, 2, 3
ti 2 ⋅ (−2 / 3) + 1 $ 1 $j + 1
= i+ k$
So, this is the condition that the lines from P − 2 /3 + 1 −2 / 3 + 1 −2 / 3 + 1
perpendicular to BC, from Q, perpendicular to CA and −4 / 3 + 1 $ 1 $ 1
from R perpendicular to AB are concurrent = i+ j+ k$
−2 + 3 −2 + 3 −2 + 3
(by changing ABC and PQR simultaneously). 3 3 3
i$ + 3i$ −4 + 3
11. F is mid-point of BC i.e. F = = 2$i and AE ⊥ DE
2 = 3 $i + 1 $j + 1 k$ = − $i + 3$j + 3k$
[given] 1 /3 1 /3 1 /3
^ ^ ^
A(i + j + k) D and when λ = − 2 , position vector of E is given by,
2 × (−2 ) + 1 $ 1 $ 1 $ −4 + 1 $ $ $
i+ j+ k= i − j−k
E −2 + 1 −2 + 1 −2 + 1 −1
1 = 3$i − $j − k$
Therefore, − i$ + 3$j + 3k$ and +3i$ − $j − k$ are the answer.
^ ^ ^
B(i) F(2i ) C(3i ) 12. Let O be the origin of reference. Let the position vectors
→ →
Let E divides AF in λ :1 . The position vector of E is of A and B be a and b respectively.
given by


B(b) N C(b + αa)
2 λ$i + 1 ($i + $j + k$ )  2λ + 1 $ 1 $ 1 $
=  i+ j+ k
λ+1  λ +1 λ+1 λ+1
Now, volume of the tetrahedron P
1
= (area of the base) (height)
3
2 2 1 O
⇒ = (area of the ∆ ABC) (DE) M(a/2) A(a)
3 3
→ → →
1 → → Since, BC||OA , BC = α OA = α a for some constant α.
But area of the ∆ ABC = |BC × BA |
2 → → →
1 Equation of OC is r = t (b + α a ) and
= |2i$ × ($j + k$ )| = | i$ × $j + i$ × k$ | =|k$ − $j| = 2
2 → → → →
2 2 1 equation of AB is r = a + λ (b − a )
∴ = ( 2 ) (DE ) ⇒ DE = 2
3 3 Let P be the point of intersection of OC and AB. Then,
→ → → → →
Since, ∆ ADE is a right angle triangle, then at point P, t (b + α a ) = a + λ ( b − a ) for some values of t
AD 2 = AE 2 + DE 2 and λ .
⇒ (4)2 = AE 2 + (2)2 ⇒ AE 2 = 12 ⇒

(t α − 1 + λ ) a = (λ − t ) b

→ 2λ + 1 $ 1 $ 1 $
But AE = i+ j+ k − ($i + $j + k$ ) → →
Since, a and b are non-parallel vectors, we must have
λ+1 λ+1 λ+1
λ $ λ $ λ $ tα − 1 + λ = 0 and λ = t ⇒ t = 1 / (α + 1)
= i− j− k → 1 → →
λ+1 λ+1 λ+1 Thus, position vector of P is r1 = (b + α a )
α+1
→ 1 3λ2
⇒ |AE|2 = [λ2 + λ2 + λ2] = 1→
(λ + 1) 2
(λ + 1)2 → → 1 →
Equation of MN is r = a + k b + (α − 1) a  …(i)
2  2 
rankershalt.com

548 Vectors

→ →
For k = 1 / (α + 1) {which is the coefficient of b in r1}, we = (1 + 2 cos θ ) ⋅ (1 − cos θ )2
→→→
get ⇒ [a b c ] = ( 1 + 2 cos θ ) ⋅ (1 − cos θ )
→ 1→ 1 → 1 →
r= a+ b + (α − 1) a  1 −2 cos θ 1
 ∴p = , q= and r =
2 α +1 2  1 + 2 cos θ 1 + 2 cos θ 1 + 2 cos θ
1 → 1 1 → 1→ →
=
(α + 1)
b + (α − 1) ⋅
2 α+1
a+ a
2 14. Let R = x$i + y$j + z k$
→ → → →
1 → 1 → ∴ R × B= C × B
= b+ (α − 1 + α + 1) a
(α + 1) 2 (α + 1) i$ $j k$ $i $j k$
1 → → → ⇒ x y z = 4 −3 7
= (b + α a ) = r1 ⇒ P lies on MN.
(α + 1) 1 1 1 1 1 1
→ → → ⇒ ( y − z )i − (x − z ) j + (x − y) k$ = − 10$i − 3$j + 7k$
$ $
13. Since, a , b , c are non-coplanar vectors.
→→→ ⇒ y − z = − 10, z − x = − 3, x − y = 7
⇒ [a b c ] ≠ 0 → →
→ →→ → → → → and R⋅ A = 0 ⇒ 2x + z = 0
Also, a×b+ b × c = pa+ qb+ r c
On solving above equations, x = − 1, y = − 8 and z = 2
→ → →

Taking dot product with a , b and c respectively both ∴ R = − $i − 8$j + 2k$
sides, we get
→→→
→→→
p + q cos θ + r cos θ = [a b c ] … (i) 15. Given that, a b c are coplanar vectors.
p cos θ + q + r cos θ = 0 …(ii) ∴ There exists scalars x, y, z not all zero, such that
→→→ → → →
and p cos θ + q cos θ + r = [a b c ] …(iii) x a + y b + z c =0 …(i)
→ →
On adding above equations, Taking dot with a and b respectively, we get
→→→ → → → → → →
2 [a b c ] x (a⋅ a ) + y (a⋅ b) + z (a⋅ c ) = 0 …(ii)
p+ q+ r= …(iv)
2 cos θ + 1 → → → → → →
and x (a⋅ b) + y (b⋅ b) + z ( c⋅ b) = 0 …(iii)
On multiplying Eq. (iv) by cos θ and subtracting
Eq. (i), we get Since, Eqs. (i), (ii) and (iii) represent homogeneous
→→→ equations with (x, y, z) ≠ (0, 0, 0) .
2 [a b c ] cos θ → → →
p (cos θ − 1) = − [a b c ] ⇒ Non-trivial solutions
2 cos θ + 1
→ → →
→→→ a b c
[a b c ] → → → → → → →
⇒ p= ∴ ∆ =0 ⇒ a⋅ a a⋅ b a⋅ c = 0
(1 − cos θ ) (2 cos θ + 1)
→ → → → → →
→→→ b⋅b b⋅b b⋅c
−2 [a b c ] cos θ
Similarly, q=
(1 + 2 cos θ ) (1 − cos θ )
→→→
16. Since, ($i + $j + 3k$ ) x + (3$i − 3$j + k$ ) y + (−4$i + 5$j )z
[a b c ]
and r= = λ ($i x + $j y + k$ z )
(1 + 2 cos θ ) (1 − cos θ )
→ → → → → →
⇒ x + 3 y − 4z = λx, x − 3 y + 5z = λy, 3x + y + 0 z = λz
a⋅ a a⋅ b a⋅ c 1 cos θ cos θ ⇒ (1 − λ ) x + 3 y − 4z = 0, x − (3 + λ ) y + 5z = 0,
→→→ → → → → → →
Now, [a b c ]2 = b⋅ a b⋅ b b⋅ c = cos θ 1 cos θ 3x + y − λ z = 0
→ → → → → →
c⋅ a c⋅ b c⋅ c cos θ cos θ 1 Since, (x, y, z ) ≠ (0, 0, 0)
∴ Non-trivial solution.
Applying R1 → R1 + R2 + R3
1 1 1 ⇒ ∆ =0
1−λ 3 −4
= (1 + 2 cos θ ) cos θ 1 cos θ
⇒ 1 − (3 + λ ) 5 = 0
cos θ cos θ 1
3 1 −λ
Applying C 2 → C 2 − C1 , C3 → C3 − C1
1 0 0 ⇒ (1 − λ ) (3λ + λ2 − 5) − 3 (− λ − 15) − 4 (1 + 9 + 3λ ) = 0
= (1 + 2 cos θ ) ⋅ cos θ 1 − cos θ 0 ⇒ − λ3 − 2λ2 − λ = 0 ⇒ λ (λ + 1)2 = 0
cos θ 0 1 − cos θ ∴ λ = 0, − 1

Download Chapter Test


http://tinyurl.com/y3bv4ey9 or
rankershalt.com

25
3D Geometry

Topic 1 Direction Cosines and Direction Ratios of a Line


Objective Question I (Only one correct option) x−2 y−3 z −4 x−1 y−4 z −5
5. If the lines = = and = =
1 1 −k k 2 1
1. The angle between the lines whose direction cosines
satisfy the equations l + m + n = 0 and l2 = m2 + n 2, is are coplanar, then k can have (2012 Main)
(2014 Main) (a) any value (b) exactly one value
π π π π (c) exactly two values (d) exactly three values
(a) (b) (c) (d)
3 4 6 2 x−1 y+ 1 z −1 x−3 y− k z
6. If the lines = = and = =
2 3 4 1 2 1
intersect, then the value of k is
Topic 2 Straight Line in Space and 3 9 2
(2004, 1M)
3
(a) (b) (c) − (d) −
Shortest Distance 2 2 9 2

Objective Questions I (Only one correct option) Objective Questions II


1. The distance of the point having position vector (One or more than one correct option)
− $i + 2$j + 6k
$ from the straight line passing through the
7. From a point P(λ , λ , λ ), perpendiculars PQ and PR are
point (2, 3, − 4) and parallel to the vector, 6$i + 3$j − 4k
$ is
drawn respectively on the lines y = x, z = 1 and
(2019 Main, 10 April II) y = − x, z = −1. If P is such that ∠QPR is a right angle,
(a) 2 13 (b) 4 3 (c) 6 (d) 7 then the possible value(s) of λ is (are) (2014 Adv.)
2. The vertices B and C of a ∆ABC lie on the line, (a) 2 (b) 1 (c) −1 (d) − 2
x + 2 y −1 z
= = such that BC = 5 units. Then, the area 8. Two lines L1: x = 5,
y
=
z
and
3 0 4 3 − α −2
(in sq units) of this triangle, given that the point
A(1, − 1, 2) is y z
(2019 Main, 9 April II) L2 : x = α , = are coplanar. Then, α can take
(a) 34 (b) 2 34 −1 2 − α
(c) 5 17 (d) 6 value(s) (2013 Adv.)

3. Let 3 $i + $j, $i + 3 $j and β$i + (1 − β)$j respectively be the (a) 1 (b) 2 (c) 3 (d) 4
position vectors of the points A , B and C with respect to
the origin O. If the distance of C from the bisector of the 9. A line l passing through the origin is perpendicular to
3 the lines (2013 Adv.)
acute angle between OA and OB is , then the sum of
2 $,− ∞ < t< ∞
l1 : ( 3 + t ) i$ + ( −1 + 2 t )$j + ( 4 + 2 t )k
all possible values of β is (2019 Main, 11 Jan II)
(a) 1 (b) 3 l2 : (3 + 2s)i$ + (3 + 2s)$j + (2 + s) k
$,−∞ < s< ∞
(c) 4 (d) 2 Then, the coordinate(s) of the point(s) on l2 at a
x−3 y+ 2 z + 4 distance of 17 from the point of intersection of l and
4. If the line, = = lies in the plane,
2 −1 3 l1 is (are)
lx + my − z = 9, then l2 + m2 is equal to (2016 Main)
(a)  , , 
7 7 5
(b) (−1, − 1, 0)
(a) 26 (b) 18  3 3 3
(d)  , , 
(c) 5 (d) 2 7 7 8
(c) (1, 1, 1)
 9 9 9
rankershalt.com

550 3D Geometry

9. A plane passing through the points (0, − 1, 0) and (0, 0, 1)


Topic 3 Equation of a Plane π
and making an angle with the plane y − z + 5 = 0, also
Objective Questions I (Only one correct option) 4
passes through the point (2019 Main, 9 April I)
1. The distance of the point (1, − 5, 9) from the plane
x − y + z = 5 measured along the line x = y = z is (a) ( 2 , 1, 4) (b) (− 2 , 1, − 4)
(2016 Main) (c) (− 2 , − 1, − 4) (d) ( 2 , − 1, 4)
(a) 3 10 (b) 10 3 x−1 y+ 1 z −2
10 20 10. If the line = = meets the plane,
(c) (d) 2 3 4
3 3 x + 2 y + 3z = 15 at a point P, then the distance of P from
2. The length of the perpendicular drawn from the point the origin is (2019 Main, 9 April I)
(2, 1, 4) to the plane containing the lines (a) 7 / 2 (b) 9 / 2 (c) 5 /2 (d) 2 5
r = (i$ + $j) + λ( i$ + 2$j − k
$ ) and r = (i$ + $j) + µ (− i$ + $j − 2k
$)
11. The vector equation of the plane through the line of
is (2019 Main, 12 April II) intersection of the planes x + y + z = 1 and
(a) 3 (b)
1
(c) 3 (d)
1 2x + 3 y + 4z = 5, which is perpendicular to the plane
3 3 x − y + z = 0 is (2019 Main, 8 April II)

3. A plane which bisects the angle between the two given (a) r ⋅ ($i − k
$)− 2= 0 (b) r × ($i + k$)+ 2= 0

planes 2x − y + 2z − 4 = 0 and x + 2 y + 2z − 2 = 0, passes $


(c) r × (i − k $)+ 2= 0 $ $
(d) r ⋅ (i − k) + 2 = 0
through the point (2019 Main, 12 April II)
12. The length of the perpendicular from the point (2, − 1, 4)
(a) (1, − 4, 1) (b) (1, 4, − 1)
x+3 y−2 z
(c) (2, 4, 1) (d) (2, − 4, 1) on the straight line, = = is
10 −7 1 (2019 Main, 8 April I)
x−2 y+ 1 z −1
4. If the line = = intersects the plane (a) greater than 3 but less than 4
3 2 −1
(b) less than 2
2x + 3 y − z + 13 = 0 at a point P and the plane
(c) greater than 2 but less than 3
3x + y + 4z = 16 at a point Q, then PQ is equal to
(2019 Main, 12 April I) (d) greater than 4
(a) 14 (b) 14 (c) 2 7 (d) 2 14 13. The magnitude of the projection of the vector 2$i + 3$j + k$
5. A perpendicular is drawn from a point on the line on the vector perpendicular to the plane containing the
x−1 y+ 1 z vectors $i + $j + k
$ and $i + 2$j + 3k
$ , is
= = to the plane x + y + z = 3 such that the (2019 Main, 8 April I)
2 −1 1
3 3
foot of the perpendicular Q also lies on the plane (a) (b) 6 (c) 3 6 (d)
2 2
x − y + z = 3. Then, the coordinates of Q are
(2019 Main, 10 April II) 14. The equation of a plane containing the line of
(a) (− 1, 0, 4) (b) (4, 0, − 1) intersection of the planes 2x − y − 4 = 0 and y + 2z − 4 = 0
(c) (2, 0, 1) (d) (1, 0, 2) and passing through the point (1, 1, 0) is
(2019 Main, 8 April I)
1 2
6. If the plane 2x − y + 2z + 3 = 0 has the distances and (a) x − 3 y − 2z = − 2 (b) 2x − z = 2
3 3
units from the planes 4x − 2 y + 4z + λ = 0 and (c) x − y − z = 0 (d) x + 3 y + z = 4
2x − y + 2z + µ = 0, respectively, then the maximum x+ 1 y−2 z −3
15. If an angle between the line, = = and
value of λ + µ is equal to (2019 Main, 10 April II) 2 1 −2
(a) 13 (b) 15 the plane, (2019 Main, 12 Jan II)
(c) 5 (d) 9  2 2
7. If Q(0, − 1, − 3) is the image of the point P in the plane x − 2 y − kz = 3 is cos− 1   , then value of k is
5 3  3 3 5
3x − y + 4z = 2 and R is the point (3, − 1, − 2) , then the (a) (b) (c) − (d) −
area (in sq units) of ∆PQR is (2019 Main, 10 April I) 3 5 5 3

(a)
91
(b) 2 13 16. Let S be the set of all real values of λ such that a plane
2 passing through the points (− λ2, 1, 1), (1, − λ2, 1) and
91 65 (1, 1, − λ2) also passes through the point (− 1, − 1, 1).
(c) (d)
4 2 Then, S is equal to (2019 Main, 12 Jan II)
(a) { 3 , − 3 } (b) {3, − 3} (c) {1, − 1} (d) { 3 }
8. Let P be the plane, which contains the line of
intersection of the planes, x + y + z − 6 = 0 and 17. The perpendicular distance from the origin to the plane
2x + 3 y + z + 5 = 0 and it is perpendicular to the x+ 2 y−2 z + 5
containing the two lines, = =
XY -plane. Then, the distance of the point (0, 0, 256) 3 5 7
from P is equal to x −1 y − 4 z + 4
(2019 Main, 9 April II) and = = , is
(a) 63 5 (b) 205 5 1 4 7 (2019 Main, 12 Jan I)
11 17 11
(c) (d) (a) 11 6 (b) (c) 11 (d) 6 11
5 5 6
rankershalt.com

3D Geometry 551

x−3 y+ 1 z −6 x+ 5 y−2 z −3 27. The plane through the intersection of the planes
18. Two lines = = and = =
1 3 −1 7 −6 4 x + y + z = 1 and 2x + 3 y − z + 4 = 0 and parallel to
intersect at the point R. The reflection of R in the Y -axis also passes through the point (2019 Main, 9 Jan I)
xy-plane has coordinates (2019 Main, 11 Jan II) (a) (3, 3, −1) (b) (−3, 1, 1)
(a) (2, − 4, − 7) (b) (2, − 4, 7) (c) (3, 2, 1) (d) (−3, 0, −1)
(c) (− 2, 4, 7) (d) (2, 4, 7) 28. The equation of the line passing through
19. If the point (2, α , β) lies on the plane which passes (−4, 3, 1), parallel to the plane x + 2 y − z − 5 = 0 and
through the points (3, 4, 2) and intersecting the line (2019 Main, 9 Jan I)
(7, 0, 6) and is perpendicular to the plane 2x − 5 y = 15, x+1 y−3 z −2
= = is
then 2α − 3β is equal to (2019 Main, 11 Jan II) −3 2 −1
(a) 17 (b) 7 (c) 5 (d) 12 x+4 y− 3 z −1 x+4 y− 3 z −1
(a) = = (b) = =
x −3 y + 2 z − 1 3 −1 1 −1 1 1
20. The plane containing the line = = and x+4 y− 3 z −1 x −4 y + 3 z + 1
2 −1 3 (c) = = (d) = =
also containing its projection on the plane 1 1 3 2 1 4
2x + 3 y − z = 5, contains which one of the following 29. If L1 is the line of intersection of the planes
points? (2019 Main, 11 Jan I) 2x − 2 y + 3z − 2 = 0, x − y + z + 1 = 0 and L 2 is the line of
(a) (−2, 2, 2) (b) (2, 2, 0) (c) (2, 0, − 2) (d) (0, − 2, 2) intersection of the planes x + 2 y − z − 3 = 0,
21. The direction ratios of normal to the plane through the 3x − y + 2z − 1 = 0, then the distance of the origin from
points (0, − 1, 0) and (0, 0, 1) and making an angle π /4 the plane, containing the lines L1 and L 2 is (2018 Main)
with the plane y − z + 5 = 0 are (2019 Main, 11 Jan I)
1 1
(a) (b)
(a) 2, − 1, 1 (b) 2 , 1, − 1

You might also like